You are on page 1of 1173

REPUBLIC v.

SANDIGANBAYAN (Dannah) RA 3019 (Anti-Graft and Corrupt Practices Act) and RA 1379 (The
21 July 2003 | Carpio, J. | Bill of Rights Not Operative During Act for the Forfeiture of Unlawfully Acquired Property). And on
Interregnum August 1, 1987, PCGG filed a petition for forfeiture under RA 1379
against Ramas, in which Dimaano was also impleaded.
PETITIONER: Republic of the Philippines In Ramas’ answer, he stated that the house and lot in La Vista, which
RESPONDENTS: Sandiganbayan, Maj. Gen. Ramas & Elizabeth Dimaano was worth P700,000, was not out of proportion to his salary and
other legitimate income. He also denied ownership of the mansion in
SUMMARY: The AFP Anti-Graft Board investigated the “unexplained Cebu, and the cash, equipment and etc. found in Dimaano’s home.
wealth” of Major General Ramas, and a search warrant was issued for Dimaano claimed ownership of all that was taken from her house by the
the home of Elizabeth Dimaano, Ramas’ alleged mistress. Philippine Constabulary.
Communications equipment, jewelry, land titles, money and weapons & The case was delayed for up to four years, because petitioner kept
ammunition were seized. Ramas denied allegations and Dimaano asking for postponement of trial. Respondents filed motions to
assumed ownership of seized items. Ramas used the case of Republic v. dismiss based on the case of Republic v. Migrino. The
Migrino as a defense, as he cannot be classified as a subordinate under Sandiganbayan then rendered a resolution, dismissing the complaint
its definition in E.O. No. 1. for lack of merit. It also ordered the return of confiscated items to
Dimaano.
Petitioner claims that during the interregnum, the Bill of Rights was not Petitioner filed a motion for reconsideration, which was denied.
operative. The Court agreed with petitioner but held that the Covenant on
ISSUE/s:
Civil and Political Rights and Universal Declaration of Human Rights
were still effective, which accorded Filipinos almost the same rights W/N PCGG has jurisdiction to investigate respondents – NO.
available from the Bill of Rights of the 1973 Constitution. The Court then W/N the Sandiganbayan erred in dismissing the case before completion
stated that those items which were not included in the search warrant are of presentation of evidence – NO (petitioner’s fault)
not admissible as evidence and must be returned to Dimaano. W/N the properties confiscated were illegally seized and therefore
inadmissible as evidence – YES
DOCTRINE: During the interregnum, a person could not invoke any IMPORTANT! W/N the revolutionary government was bound by the
exclusionary right under a Bill of Rights because there was neither Bill of Rights of the 1973 Constitution during the interregnum –
a constitution nor a Bill of Rights then. Nevertheless, the Filipino NO
people continued to enjoy, under the Covenant and Declaration, almost W/N the protection accorded to individuals under the International
the same rights found in the Bill of Rights of the 1973 Constitution. Covenant on Civil and Political Rights (Covenant) and the
Universal Declaration of Human Rights remained in effect
during the interregnum (Declaration) – YES
FACTS:
Former President Cory Aquino issued EO 1 creating the Presidential RULING: The petition for certiorari is dismissed. Records are remanded to
Commission on Good Government (PCGG). The PCGG was tasked the Ombudsman for such appropriate action may warrant.
to recover all ill-gotten wealth of Former Pres. Marcos, his immediate
family, relatives, subordinates and close associates. RATIO:
The PCGG, through Chairman Salonga, created an AFP Anti-Graft Board The PCGG has no jurisdiction to investigate respondents. The PCGG,
tasked to investigate reports of unexplained wealth and corrupt through the AFP Board, can only investigate the unexplained wealth and
practices by AFP personnel, whether in the active service or retired. corrupt practices of AFP personnel who fall under either of the two
Said Board investigated Major General Ramas. A Resolution was issued categories
stating Ramas had a house and lot in La Vista, and a house and lot AFP personnel who have accumulated illgotten wealth during the
in Cebu. A search warrant was then issued for the house of Elizabeth admin of former Pres Marcos by being the latter’s immediate
Dimaano, Ramas’ alleged mistress. family, relative, subordinate or close associate, taking
Equipment/items and communication facilities, money in the amount of undue advantage of their public office or using their powers,
P2.87M and $50,000, land titles, weapons and ammunition, etc. influence
were sequestered from Dimaano’s home. The money confiscated AFP personnel involved in other cases of graft and corruption
was never declared in Dimaano’s SALN. It also seemed impossible provided the President assigns their cases to the PCGG.
for her, a mere secretary to own that much money. Applying ejusdem generis, the term “subordinate” refers to one who
The Board recommended that Ramas be prosecuted and tried for violation of enjoys a close association with Marcos, and/or his wife, similar to the
immediate family member, reltive, and close associate in EO No. 1,
and the close
relative, business associate, dummy, agent, or nominee in EO No. 2.
There must be a prima facie showing that the respondent unlawfully
accumulated wealth by virtue of his close association or
relation with former Pres Marcos and/or his wife. There was no
such evidence shown.
Although the PCGG sought to investigate and prosecute respondents
under EO Nos. 1, 2, 14 and 14A, the result yielded a finding of
violation of RA 3019 and 1379 without any relation to said EOs.
The AFP Board resolution does not contain a finding that Ramas
accumulated his wealth because of his close association with former
President Marcos.
On third issue: The search warrant was captioned “Illegal Possession of
Firearms and Ammunition.” But the raiding team also seized, aside
from one baby armalite rifle, 40 rounds of ammunition, and one
pistol, communications equipment, cash, jewelry and land titles.
The EDSA revolution was done in defiance of the provisions of the 1973
Constitution, which is why the resulting government was a
revolutionary government. It was not bound by any constitution or
legal limitations except treaty obligations the government assumed
under international law.
During the interregnum the directives and orders of the revolutionary
government were the supreme law—there was no municipal law
higher than these. Thus, during the interregnum, a person could not
invoke any exclusionary right under a Bill of Rights because
there was neither a constitution nor a Bill of Rights then.
However, even after said government, the Court ruled that the Freedom
Constitution and later the 1987 Constitution expressly recognized
the validity of sequestration orders. (FATHER BERNAS WAS
SUPER AGAINST THIS—FAMILIARIZE)
Nevertheless, the Filipino people continued to enjoy, under the Covenant and
Declaration, almost the same rights found in the Bill of Rights of the
1973 Constitution. (FAMILIARIZE YOURSELVES W/ THE
PROVISIONS)
The Court has interpreted the Declaration as part of the generally
accepted principles of international law and binding on the State.
Thus, the revolutionary gov’t was also obligatied under international
law to observe the rights of individuals under the Declaration.
AS the de jure gov’t, the revolutionary gov’t could not escape
responsibility for the State’s good faith compliance with its treaty
obligations under int’l law.
During the interregnum, directives and orders issued by gov’t officers
were valid so long as these officers did not exceed the authority
granted them, and these should not have violated the Covenand or
the Declaration.
The warrant is thus valid with respect to the items specifically
described in the warrant, which is why the raiding team had no
legal basis to seize items without showing that these items could be
the subject of warrantless search and seizure. The seizure of these
items was therefore void and mus be returned to the person from
whom the raiding team seized them.
METRO MANILA DEVELOPMENT AUTHORITY V. BEL-AIR RULING: The petition is DENIED. The decision and resolution of the CA is
VILLAGE ASSOCIATION, INC. (IYA) affirmed.
27 March 2000 | Puno, J. | Police Power
Petitioner/s: Metropolitan Manila Development Authority
Respondent/s: Bel-Air Village Association, Inc. (BAVA)

Summary: The MMDA in its hopes to decongest and regulate traffic


seeks for the opening of Neptune St. for public use. BAVA, owns the
private property and refuses to comply with the notice sent by MMDA.
The Court of Appeals granted BAVA a TRO and preliminary injunction
preventing the MMDA to open Neptune St. and demolish the perimeter
wall. The MMDA claims it has authority to act as such pursuant to the
police power granted to it.

Doctrine: Police power is an inherent attribute of sovereignty. primarily in


the National Legislature. It cannot be exercised by any other group or
body not possessing legislative power.

It is vested by the Constitution in the legislature to make, ordain, and


establish all manner of wholesome and reasonable laws, statutes and
ordinances… as they shall judge for the good and welfare of the
commonwealth and for the subjects o the same.

FACTS:
MMDA Chairman sent a notice to the respondent requesting them to
open Neptune Street to public vehicular traffic starting January 2,
1996.
The notice/request was based on the MMDA Law or RA 7924 which
requires the Authority to rationalize the use of roads for the safe and
convenient movement of persons.
BAVA filed before the RTC of Makati a civil case for injunction and prayed
for a TRO and preliminary injunction enjoining the opening of
Neptune St. and prohibiting the demolition of the perimeter wall.
RTC denied the issuance but the appellate court reversed the decision
saying MMDA has no authority to order such, therefore granting the
respondents requests.
MMDA questions the CA decision claiming that it is mandated to open
Neptune St. to public traffic because it is endowed with police power
in the delivery of basic services in Metro Manila.
The MMDA cites two Sangalang cases which allowed the opening of
private roads to the general public. (To be discussed further in the
ratio)

ISSUE/S: W/N MMDA has the mandate to open Neptune St. to public traffic
pursuant to its regulatory and police powers? – NO
RATIO:

Police power is an inherent attribute of sovereignty. It is vested by the


Constitution in the legislature to make, ordain, and establish all
manner of wholesome and reasonable laws, statutes and
ordinances… as they shall judge for the good and welfare of the
commonwealth and for the subjects o the same.

Police Power is primarily in the National Legislature. It cannot be


exercised by any other group or body not possessing
legislative power. It is up to the National Legislature as to whom
they will delegate such power.

The powers of MMDA are limited to the following acts: formulation,


coordination, regulation, implementation, preparation management,
monitoring, setting o policies, installation of a system and
administration.

There is no provision under RA 7294 that grands the MMDA to


enact ordinances or approve resolutions. All its functions are
administrative in nature as summed in the charter.

The two Sangalang cases do not apply to the case at bar as both involve
zoning ordinances passed by the municipal council of Makati and the
MMC. The notice of MMDA does not cite any ordinal for law as basis,
it simply relied on its authority under its charter to “rationalize the use
of roads for the safe and convenient movement of persons.”

Unlike the MMC, the MMDA is not vested with legislative powers. The
MMC is a central government of Mentro Manila created under PD
824.

There is no grant of authority to the MMDA to enact ordinances and


regulations for the general welfare of the inhabitants of the
metropolis.
Philippine Blooming Mills Employees Organization v.
Philippine Blooming Mills Co., Inc. (Eliel) Issue/s: WoN the Constitutional rights of PBMEO were violated.
5 June 1973 | Makasiar, J. | Hierarchy of Rights
WoN the period to file for a motion to reconsider has prescribed.
Petitioner/s: Philippine Blooming Mills Employees Organization
RULING: GRANTED.
Respondent/s: Philippine Blooming Mills Co., Inc.
RATIO:
Summary: The PBMEO informed the Company that they would be having a
mass demonstration against alleged abuses of the Pasig Police. The
The Bill of Rights is designed to preserve the ideals of liberty, equality
Company, however, warned PBMEO that this demonstration would affect
and security “against the assaults of opportunism, the expediency of
their operations and their presence would tantamount to their dismissal.
the passing hour, the erosion of small encroachments, and the scorn
PBMEO countered that this demonstration is not against the company but in
and derision of those who have no patience with general principles.”
exercise of their rights to free expression, free assembly and petition.
Property and property rights can be lost thru prescription; but human
Doctrine: The Primacy of human rights over property rights is recognized. rights are imprescriptible. If human rights are extinguished by the
Because these freedoms are “delicate and vulnerable, as well as supremely passage of time, then the Bill of Rights is a useless attempt to limit
precious in our society” and the “threat of sanctions may deter their exercise the power of government and ceases to be an efficacious shield
almost as potently as the actual application of sanctions,” they “need against the tyranny of officials, of majorities, of the influential and
powerful, and of oligarchs – political, economic or otherwise.
breathing space to service,” permitting government regulation only “with
The demonstration was not against their employer but was purely and
narrow specificity.”
completely an exercise of their freedom of expression in general and
In the hierarchy of civil liberties, the rights of free expression and of assembly of their right of assembly and of petition for redress of grievances in
particular before the appropriate governmental agency, the Chief
occupy a preferred position, as they are essential to the preservation and
Executive, against the police officers of the municipality of Pasig.
vitality of our civil and political institutions; and such priority “gives these
The refusal on the part of the Company to permit its employees to join
liberties the sanctity and the sanction not permitting dubious intrusions.”
the mass demonstration against alleged police abuses and the
subsequent separation, constituted an unconstitutional restraint on
their rights.
FACTS: NO. The procedural rule of the Court of Industrial Relations must yield to
Petitioners claim that on March 1, 1969, they decided to stage a mass the constitutional rights. Enforcement of a basic human freedom is a
demonstration at Malacanang on March 4, 1969, in protest against compelling reason to deny the Courts rule which impinges human
alleged abuses of the Pasig police, to be participated in by the rights.
workers in the first shift as well as those in the regular second and
third shifts; and that they informed the respondent Company of their
proposed demonstration.
Management informed PBMEO that the demonstration is an inalienable
right of the union guaranteed by the Constitution but emphasized,
however, that any demonstration for that matter should not unduly
prejudice the normal operation of the Company.
The Company warned PBMEO those workers, who shall fail to report for
work the following morning shall be dismissed, because such failure
is a violation of the existing CBA and, therefore, would be amounting
to an illegal strike.
After the demonstration proceeded, the Company filed charges against
petitioners for violating the existing CBA and 8 of PBMEO were
found directly responsible for perpetrating unfair labor practice and
considered to have lost their status as employees.
Duncan Association v. Glaxo Wellcome Philippines, Inc. (JP) When the parties submitted the matter for voluntary arbitration, Glaxo offered
17 September 2004 | Tinga, J. | Equal Protection Clause Tecson separation pay of 50,000 but he declined. The National
Conciliation
Petitioner/s: Duncan Association of Detailman-PTGWO (Duncan) &
Pedro A. Tecson (Tecson)
Respondent/s: Glaxo Wellcome Philippines, Inc. (Glaxo)

Summary: Petitioner Tecson from Glaxo marries Betsy, an employee of


his pharmaceutical company’s rival, in clear contravention of his contract
of employment and Glaxo’s Employee Code of Conduct which prohibits
potential conflict of interest. Under the argument of the equal protection
clause, he claims that the company is restricting its employees from
marrying a person of their choosing. It was held that the policy was a
valid exercise of management prerogatives as companies have the right
to adopt policies for their business’s benefit, in accordance with Article
13 Section 3 of the constitution.

Doctrine: It is a settled principle that the commands of the equal


protection clause are addressed only to the state or those acting under
its authority. Equal protection clause erects no shield against merely
private conduct however discriminatory or wrongful. The only exception
is if the state is involved in wrongful private conduct.

FACTS:
Petitioner Tecson, as a medical representative, signed a contract of
employment with Duncan which stipulates that to avoid conflict of
interest, employees must disclose to the management any existing
or future relationship by consanguinity or affinity with co-employees
or employees of competing drug companies. Should the
management find such relationship as conflicting, the employee must
resign.
The Employee Code of Conduct of Glaxo also includes the prohibition of the
relationships and provides the consequences of transfer to another
department or preparation for employment outside the company for six
months.
Love prevailed. Tecson entered into a romantic relationship with Betsy,
an employee of Astra Pharmaceuticals (Astra), Glaxo’s competitior.
They eventually married amidst the persistent warnings of Tecson’s
supervisors.
Glaxo then transferred Tecson to the Butuan City-Surigao City-Agusan
del Sur sales area from his Camarines Sur-Camarines Norte area.
Tecson brought the matter to Glaxo’s Grievance Committee but was
denied.
Tecson was paid his salary but was not issued samples of products that
are competing with Astra. He was also excluded from product
conferences and seminars.
and Mediation Board (NCMB) rendered a decision declaring as valid
Glaxo’s policy.
Tecson filed a petition for review with the Court of Appeals but the CA
declared that the NCMB did not err. The subsequent MR was also
denied. Hence, the petition.

ISSUE/S: WoN the CA erred in ruling that Glaxo’s policy against its
employees marrying employees from competitor companies violates the
equal protection clause of the constitution – No

WoN Tecson was constructively dismissed – No.

RULING: Petition is denied for lack of merit. Glaxo’s policy is valid.

RATIO:

Glaxo’s policy of prohibiting relationships with competing companies is a


valid exercise of management prerogative. Glaxo has a right to
guard its trade secrets from competitors, especially so that it and
Astra are rivals in a highly competitive pharmaceutical industry.
Section 3 Article 13 of the Constitution provides the right of
enterprises to adopt and enforce policies necessary for reasonable
returns and growth.

The challenged company policy does not violate the equal protection
clause. It is a settled principle that the commands of the equal
protection clause are addressed only to the state or those acting
under its authority. Equal protection clause erects no shield against
merely private conduct however discriminatory or wrongful. The only
exception is if the state is involved in wrongful private conduct.

Glaxo does not impose an absolute prohibition against relationships. Its


employees are free to cultivate relationships and marry persons of
their choosing. What the company seeks to avoid is a conflict of
interest to the detriment of their profits and business success.

Tecson is also estopped from assailing the questioned policy because he


signed the employment contract, knowing all the stipulations therein.

There is also no merit in petitioner’s contention that he was


constructively dismissed. Constructive dismissal is defined as a
quitting, an involuntary resignation with demotion in rank or pay.
None of these conditions are present in the instant case. Glaxo
properly exercised its management prerogatives in reassigning
Tecson to Butuan City. The proximity of the areas of responsibility of
Tecson and his wife in Bicol, renders learning by one’s spouse of the
other’s marketing strategies inevitable.
STAR PAPER CORPORATION v. SIMBOL (Henry) Estrella alleges that she had a relationship with Zuñiga who
April 12, 2006 | Puno, J. | Valid exercise of management prerogative misrepresented himself as a married but separated man, but she
later found out after getting
PETITIONER: Star Paper Corporation, Josephine Ongsitco, et al.
RESPONDENTS: Ronald D. Simbol, et al.

SUMMARY: Respondents Simbol, Comia, and Estrella are employees of


Star Paper Corporation, who later were engaged in intimate relationships
with their co-employees. In lieu of the company policy that prohibited
spouses and relatives up to the third degree of affinity/consanguinity to
work in the petitioner’s company, the respondents allegedly resigned
voluntarily. Contrary to the respondent’s claim that they were illegally
dismissed, the a complaint was filed which prospered through the LA and
the NLRC who both dismissed the complaint, leaving the respondents to
file a petition for the CA who then reversed the decision of the
Commission. Star Paper then filed an appeal to the SC but the Court
ruled in favor of the respondents.

DOCTRINE: Failure of petitioners to prove a legitimate business concern


in imposing the questioned policy cannot prejudice the employee’s right
to be free from arbitrary discrimination based on stereotypes of married
persons working together in one company.

FACTS:
Star Paper Corporation, a corporation engaged in trading of paper
products, employed had the following regular employees as
petitioners of this case: Ronaldo Simbol, Wilfreda Comia, and Lorna
Estrella. Josephine Ongsitco is its Manager of Personnel and
Administration.
Simbol was employed 1993, who met Alma Dayrit, a co-employee, and
eventually married. Prior to this marriage, Ongsitco advised that one of
them should resign should they decide to get married, pursuant to a
rd
company policy about not having relatives up to the 3 degree of
relationship employed by the company. Simbol resigned a week before
he married Dayrit.
Wilfreda was employed 1997, met Howard Comia, a co-employee, and
eventually married. Prior to this marriage, Ongsitco reminded that
pursuant to company policy, one must resign should they get
married. Wilfreda resigned after marrying Howard.
Estrella was employed 1994, met Luisito Zuñiga, a co-employee and a
married man, who got her pregnant. Star Paper allegedly could have
terminated her services but she opted to resign 1999.
Respondents all signed a Release and Confirmation Agreement
indicating that they have no money and property accountabilities in
the company.
Respondents offer a different version of their dismissal. Simbol and
Comia alleged that they did not resign voluntarily; but were
compelled to resign in view of an illegal company policy.
pregnant that he was still married, thus she severed her relationship
with him to avoid dismissal due to the company policy. She met an
accident on November 1999, upon returning to work the next month,
she was denied entry in the company premises. She was directed to
the personnel office where she was asked to sign a memoranduym
stating her dismissal for immoral conduct. She refused to sign
because she was not given a chance to explain. Management asked
her to provide it but dismissed her anyway. Because of financial
needs, she later submitted a resignation letter in exchange of her
13th month pay.
Respondents filed a complaint for unfair labor practice, constructive
dismissal, separation pay, and attorney’s fees. They also contended
that they were dismissed due to their union membership.
Labor Arbiter del Rosario dismissed the complaint for lack of merit, and
said that the company policy was decreed pursuat to what the
respondent corporation perceived as management prerogative—
which is broad and encompassing. Except for or limited by special
law, an employer is free to regulate, ccording to his own discretion
and judgment all aspects of employment.
NLRC affirmed the decision of the LA. MR filed was denied as well.
Respondents appealed to the CA who reversed the NLRC decision,
declared illegal the dismissal, ordering reinstatement, and payment
of attorney’s fees and the cost of suit.
Hence, this petition.

ISSUE/s:
Whether or not Star Paper Corp validly exercised their management
prerogative - NO
Whether or not the company policy towards marriage and the family of
employees is violative of the constitutional rights – NO

RULING: SC affirmed the CA decision.

RATIO:
Companies prohibit close relatives from working in the same company by
means of anti-nepotism employment policies. Companies adopt
these policies to prevent hiring ofunqualified persons based on their
status as a relative, rather than upon their ability.
There are two types of employment policies involving spouses:
No-spuse employment policies – banning only spouses from
working in the same company
Anti-nepotism employment policies – banning all immediate family
members, including spouses, from working in the same
company
In the US, complainants who challenge tha anti-nepotism employment
policies use two theories of employment discrimination:
Disparate treatment – the plaintiff must prove that an
employment policy is discriminatory on its face
Disparate impact – complainants must prove that a facially
neutral policy has a disproportionate effecet on a particular
class
The Court notes that since finding of a bona fide occupational
qualification justifies an employer’s no-spouse rule, the exception is
interpreted strictly and narrowly by state courts. There must be a
compeling business necessity for which no alternative exists other
thatn the discriminatory practice.
The reasonableness of the company policy being employed as
standard is parallel to the bona fide occupational qualification
requirement. Reasonableness must be observed under the
circumstances of the case to qualify as a valid exercise of
management prerogative. It must be clearly established to uphold
the questioned employment policy.
In the case at bar, the court did not find any reasonable business
necessity. The company’s sole contention that they just did not want
to have two or more employees related to each other is lame. The
petitioners failed to show that the relationships of the respondents
posed detrimental to their business operations.
Their company policy is based on a mere fear that employees
married to each other will be less efficient.
Failure of petitioners to prove a legitimate business concern in
imposing the questioned policy cannot prejudice the
employee’s right to be free from arbitrary discrimination based
on stereotypes of married persons working together in one
company.
Absence of a statute expressly prohibiting marital discrimination cannot
benefit petitioners. The legislature’s silence cannot be a source of
inferences on protection to labor. There should be undisputed proof
presented.
As to Estrella, the Court ruled that it is illogical for her to resign then file a
complaint for illegal dismissal. Given the lack of sufficient evidence
on the part of the petitioners, her dismissal is hence illegal.
ARMY AND NAVY CLUB v. CA (Dannah) Petitioner invoked and capitalized on the fact that the Army and Navy
April 8, 1997 | Kapunan, J. | Classification of Property into Historical Club has been declared a national historical landmark by the
Treasures or Landmarks National Historical Commission on June 29, 1992. He contends that
all parties are enjoined by law to preserve its existence and site.
PETITIONER: Army and Navy Club of Manila, Inc.
RESPONDENTS: Court of Appeals, etc.

SUMMARY: The City of Manila filed an ejectment suit against Army and
Navy Club of Manila, Inc. for violating the contract of lease. Petitioner did
not construct a multi-story hotel, did not pay rent and taxes as agreed
upon.

Petitioner believes the Court of Appeals erred in affirming the decisions of the
lower courts. Petitioner invokes the argument that said property has been
declared a national historical landmark by the National Historical
Commission. The Court held that if in case the procedure for declaration had
been followed, petitioner cannot run from his obligations as he is only a
lessee. The law merely merely states that is hall be the policy of the state to
preserve and protect the important cultural properties and National Cultural
Treasures of the nation and to safeguard their intrinsic value. Hence, the
Court affirmed the deicions of the CA.

DOCTRINE: When the classification of property into historical treasures


or landmarks will involve the imposition of limits on ownership, the Bill of
Rights demands that it be done with due process both substantive and
procedural.

FACTS:
On November 29, 1989, the City of Manila filed an action against
petitioner with the MTC for ejectment for violation of the contract of
lease.
Said Contract stated that the lessee shall construct, at its own expense,
a modern multi-storied hotel at a cost of not less thant P50M, which
shall automatically belong to lessor upon expiration and/or
termination of lease agreement. It also stated that the lessee shall
pay a rent of P250,000 per year, and that the lessee shall pay the
realty tax, fees and charges.
The unpaid rent amounted to P1,604,166.70 as of May 1989, and the tax
liability amounted to P3,818,913.81 as of December 1989.
The City of Manila filed a Motion for Summary Judgment on the ground
that there exists no genuine tri-able issue in the case The MTC
ordered the petitioner to vacate the premises, and to pay (with legal
interest) the amount due.
The Regional Trial Court also affirmed the decision of the MTC, and the
Court of Appealse denied the motion for reconsideration of petitioner.
ISSUE/s:
W/N the Court of Appeals erred in affirming the decision of the MTC –
NO.

RULING: The decision of the CA is affirmed.

RATIO:
The instant case is a simple ejectment suit. The petitioner failed to pay
rent for 7 consecutive years.
Article 1673 of the New Civil Code states that the lessor may judicially
eject the lessee for causes, including the violation of any of the
conditions agreed upon in the contract.
When the classification of property into historical treasures or landmarks
will involve the imposition of limits on ownership, the Bill of Rights
demands that it be done with due process both substantive and
procedural.
A certain procedure must be complied with for something to be classified
as historical and cultural propery, which was provided for in PD 374
(SEE
CASE FOR PROCEDURE)
In the case at hand, there is no showing that the procedure has been
complied with, and it seems like the declaration (which was conferred
only in 1992, three years after the action for ejectment was instituted)
was merely an afterthought, an attempt to thwart any legal action taken
against the petitioner.
Nonetheless, such certification does not give any authority to the petitioner
to lay claim of ownership, or any right over the subject property.
Nowhere in the law does it state that such recognition grants possessory
rights over the peoprty to the petitioner. The law merely states that is hall
be the policy of the state to preserve and protect the important cultural
properties and National Cultural Treasures of the nation and to
safeguard their intrinsic value.
Unfortunately, petitioner is merely a lessee of the proprety. By virtue of
the lease contract, petitioner had obligations to fulfill. Petitioner
cannot just hide behind some recognition bestowed upon it in order
to escape from its obligation or remain in possession.
The Court finds no reversible error in the summary judgment rendered
by the trial court. Moreover, there is clearly no substantial triable
issue.
The argument that it was declared a historical landmark, is not a
substantial issue of fact which does not, in any way, alter or affect the
merit of the ejectment suit.
Having admitted in the original answer that the City of Manila is the
registered owner of the property and that it leased the property from
it, petitioner cannot now deny such claim of ownership.
LORLENE A. GONZALES V. NATIONAL LABOR RELATIONS Both parties appealed to the NLRC, which decided that the dismissal was
COMMISSION AND ATENEO DE DAVAO UNIVERSITY (IYA) valid and legal.
26 August 1999 | Bellosillo, J. | Due Process Gonzales now seeks a reversal of the NLRC decision.

Issue/s: WON the NLRC committed grave abuse of discretion in sustaining


Petitioner/s: Lorlene A. Gonzales
as valid and legal the dismissal of petitioner.
Respondent/s: NLRC and Ateneo de Davao University
Ruling: Decision of NLRC is REVERSED and SET ASIDE.
Summary: Lorlene Gonzales is an elementary teacher accused of
inflicting corporal punishment on her students. The Investigating Ratio:
Committee of the Ateneo de Davao refused to allow her counsel to
participate in the proceedings. Despite not having participated in the
The NLRC failed to acknowledge the absence of due process. Upon
hearings, she was terminated violating her right to due process.
being notified of her termination, she has the right to demand
Doctrine: Due process requires for both procedural and substantial compliance with the basic requirements of due process. Both
aspects be met. For procedural due process, ample opportunity must be procedural and substantial due process must be met.
afforded the employee to defend herself, to know the nature of the
Ample opportunity must be afforded the employee to defend herself
offense, and to cross examine the witnesses against her. Substantial due
either personally and/or with assistance of a representative; to know
process requires that the decision must be based on established facts
the nature of her offense, and to cross examine and confront face to
and a sound legal foundation.
face the witnesses against her.

Due process requires that the decision must be based on established


Facts: facts and on a sound legal foundation.
Lorlene Gonzales is a school teacher in the Elementary Department of The refusal of the Investigative Committee to revise the rules resulted in
Ateneo de Davao.
her failure to confront and cross-examine her accusers.
Fr. Oscar Millar, Sj, Headmaster of Ateneo Grade School, sent a letter
dated 11 April 1991 informing petitioner of the complaints of 2
Ateneo likewise failed to prove by substantial evidence that petitioner had
parents for the alleged use of corporal punishment.
inflicted corporate punishment on her students. “Substantial evidence
Petitioner claims that she was not confronted by Ateneo not he issue in 1991
and was only in 1993 after the complaints were made that she is more than mere scintilla. It means such relevant evidence as a
discovered, through her students and their parents, that Ateneo was reasonable mind might accept as adequate to support a conclusion.”
soliciting complaints.
On 9 June 1993, she was informed of the composition of an investigative The decision relied solely on the witnesses’ affidavits with questionable
committee organized by the headmaster. She was furnished with the veracity.
rules of procedure and informed of the schedule of the hearings, and
given affidavits executed by the students.
She refused to participate in the investigation unless the rules of
procedure be revised contending that it was violative of her right to
due process. Ateneo refused to grant such revision.
A provision states that a counsel of Gonzales shall not directly participate
int he investigation but will merely advise her.
On November 1993, Ateneo served a Notice of Termination. She also
received a letter from the President of Ateneo demanding her
voluntary resignation.
She filed before the Labor Arbiter for illegal dismissal. The LA found her
dismissal illegal for lack of factual basis but recognized that she was
accorded due process.
TUPAS v. CA (Eliel) that it be treated as a petition for certiorari under Rule 65 of the Rules
February 6 1991 | Cruz, J. | Due Process of Court, which can be filed within a reasonable time. This remedy
cannot be employed as a substitute for a lost appeal.
PETITIONER/S: Francisco Lim Tupas and Ignacio Lim Tupas It follows that for having themselves forfeited the right to appeal, the
RESPONDENT/S: Court of Appeals and petitioners cannot now plaintively claim that they have been denied
due process.
SUMMARY: Tupas received a copy of the decision of the RTC and Rules of procedure are intended to ensure the orderly
thereafter filed a motion for reconsideration. The decision denying the administration of justice and the protection of substantive rights
MR was given to the counsel of Tupas. Instead of filing the petition for in judicial and extrajudicial proceedings. It is a mistake to
review on time, or at least filed a motion to extend the reglementary suppose that substantive law and adjective law are
period, they did only 14 days after it expired. The court then denied their contradictory to each other or, as has often been suggested,
petition for review because of its failure to follow rules of procedure. that enforcement of procedural rules should never be permitted
if it will result in prejudice to the substantive rights of the
DOCTRINE: Rules of procedure are intended to ensure the orderly litigants.
administration of justice and the protection of substantive rights in judicial The policy of the courts, however, is to give effect to both kinds of
and extrajudicial proceedings. It is a mistake to suppose that substantive law, as complementing each other, in the just and speedy
law and adjective law are contradictory to each other or, as has often resolution of the dispute between the parties.
been suggested, that enforcement of procedural rules should never be Observance of both substantive and procedural rights is equally
permitted if it will result in prejudice to the substantive rights of the guaranteed by due process, whatever the source of such rights,
litigants. be it the Constitution itself or only a statute or rule of court.
With regard to the lawyer, Tupas failed to show that their counsel was
exceptionally inept or motivated by bad faith or excusably misled by
FACTS: the facts. Their counsel’s failure to appeal on time should be
regarded as excusable neglect or honest error is not compatible with
Tupas received a copy of the decision of the RTC of Pasay City on April
his impressive credentials. “A graduate of one of the top law schools
3, 1989, and that the motion for reconsideration thereof was filed on
in the country, a bar examiner in Remedial Law, a law professor in
April 17, 1989, or fourteen days later.
Remedial Law and other law subjects, a former National Officer of
The order of May 3, 1989, denying the motion was received by the
the IBP and a seasoned practitioner for more that 30 years.
counsel of Tupas on May 9, 1989.
For all its conceded merits, equity is available only in the absence of law
Instead of filing the petition for review with the CA within the remainder of
and not as its replacement. Equity is described as justice outside
the 15-day reglementary period, that is, on May 10, 1989, Tupas did
legality, which simply means that it cannot supplant although it may
only on May 23, 1989, or 14 days later.
as often happens, supplement the law.
Hence this petition.
Tupas’ counsel did not file the petition for review within the remaining
period, which he should have known was only one day. Neither did
he move for an extension that would have been granted as a matter
of course.
(ADDITIONAL INFO)

ISSUE/S: WoN Tupas was denied due process. NO

WoN Tupas’ lawyer can be liable for the delay. NO


WoN the principle of equity can be applied. NO

RULING: Motion for reconsideration denied.

RATIO:

The petition for review being indisputably late, he could not thereafter ask
BANCO ESPANYOL-FILIPINO v. PALANCA (JP) 7 years after the sale, a motion was made by Vicente Palanca, as
administrator of the estate of the original defendant, wherein Vicente
March 26, 1918 | Street, J. | Due Process
requested the court to
PETITIONER: El Banco Español-Filipino
RESPONDENTS: Vicente Palanca

SUMMARY: El Banco instituted a foreclosure for the mortageged


properties of Palanca which he used for security for a loan. 7 years after
the foreclosure and sale of the property, Palanca’s administratix
requested to set aside the previous judgement on the ground that the
court had no jurisdiction because no notice was actually received by the
defendant who is a nonresident. Trial court denied the motion. The SC
affirmed saying that since the requirement publication in a newspaper
was met, the due process requirement of the defendant’s right to be
heard is already met. Receipt of notice is not essential. The possibility of
the clerk failing to deliver the letter is not fatal.

DOCTRINE: In a foreclosure case, some notification of the proceedings


to the nonresident owner is everywhere recognized as essential. Statutes
generally provide for publication, and usually in addition for mailing the
notice to the defendant, if his residence is known. However, among the
two, only publication is necessarily essential.

FACTS:
El Banco Español-Filipino (bank) instituted action against Engracio
Palanca to foreclose a mortgage upon various parcels of real
property in Manila City. The property was used as security for a debt
by Palanca with the bank.
After the execution of the instrument by the mortgagor, Palanca, he
returned to China, his native country. He then died there.
Since required pursuant to Section 399 of the Code of Civil Procedure,
the defendant being a nonresident at the time of the institution of the
present action, publication was made in due form in a newspaper of
the City of Manila. The court further directed that the clerk of the
court should deposit in the post office a copy of the summons and
complain directed to the defendant at his last place of residence, the
City of Amoy, in the Empire of China.
Whether the clerk complied with this order does not affirmatively appear.
There is however, an affidavit signed by an employee of the bank’s
attorneys showing that he (the employee) had deposited in the
Manila post-office a registered letter of the summons, affidavit and
the complaint.
The case proceeded in the usual course. The defendant not having
appeared, the judgment was taken against him by default, in favor of
the El Banco. Since the defendant naturally did not satisfy the
judgment, the mortgage property was sold in a public auction and
was bought by the bank.
set aside the previous order of default because the court had never seem highly unreasonable to hold that the failure to mail the notice
acquired jurisdiction over the defendant or over the subject of the was fatal.
In every situation of this character (property related cases) an appropriate
action.
remedy is always at hand. If the property has been taken without due
The lower court denied the application and from this, Vicente appealed. process,
ISSUE/s:
WoN the acquired the necessary jurisdiction to enable it to proceed with
the foreclosure of the mortgage – YES
WoN the irregularity in the proceedings (the notice not being
secured) was of such gravity as to amount to a denial of the due
process of law -
NO

RULING: Judgement in question is affirmed.

RATIO:
The third requirement of due process is that the defendant shall have an
opportunity to be heard. In a foreclosure case, some notification of
the proceedings to the nonresident owner is everywhere recognized
as essential.
Statutes generally provide for publication, and usually in addition for
mailing the notice to the defendant, if his residence is known.
It will be observed that this mode of notification does not inolve any absolute
assurance that the absent owner shall thereby receive actual notice. The
periodical containing the notice has a chance to not come in the
defendant’s hands. The provision also of our law relative to the mailing
of notice does not absolutely require the mailing of notice unconditionally
and in every event, but only in the case where the defendant’s residence
is known.
Hence, actual notice to the defendant in cases of this kind is not under the
law not be considered absolutely necessary. Reasoning of this rule:
Property is always assumed to be in the possession of the owner. It can
be assumed that he will know of any proceeding related to it for its
condemnation and sale.
Duty of the owner of the real estate who is a nonresident to take
measures that he is represented when his property is called into
requisition. If he fails to do this or get notice of the publiczation which
have usually been required, it is his misfortune.
If property of a nonresident cannot be reached by legal process upon
constructive notice, then our statues were passed in vain, for if the
person is not within the jurisdiction of the court, no judgment can
operate on the property and the courts would be powerless to assist
a citizen against a nonresident.
Notice was given by publication in a newspaper and this is the only form
of notice which the law unconditionally requires. This is in our (the
SC) opinion all that was absolutely necessary to sustain the
proceedings.
All that due process of law of law thereafter requires is an opportunity for
the defendant to be heard. As publication was duly made, it would
the law concedes due process to recover it. In other words, there must always
be a remedy for property.
CATACUTAN v. PEOPLE (HENRY) forms bearing the letterhead of SNSAT and not of the CHED Regional
August 31, 2011 | Del Castillo, J. | Procedural Due Process Office who made the appointments, another lapse was that he only
received duplicate
PETITIONER: Jose R. Catacutan
RESPONDENTS: People of the Philippines

SUMMARY: Posensano and Divinagracia of the SNSAT were appointed


by CHED-CAR to Vocational Instruction Supervisor III, which was duly
approved by the CSC. However, SNSAT’s IOC/Principal refused the
implementation of such appointments, which lead to the filing of a
complaint against him by the two appointees. In his defense he said that
there were procedural lapses that hindered him in implementing the
same. Trial Court ruled against him and held him liable for violation of
Sec 3(e) of RA 3019, and he appealed to the Sandiganbayan— who
affirmed the trial court’s decision. He tried to present the CA decision
(dismissing the admin case filed against him) to the Sandiganbayan but
to no avail, which caused him to appeal to the SC, that he was denied of
due process when the inclusion of the CA Decision was denied by the
lower court. SC affirmed the ruling of the Sandiganbayan, and stated that
Catacutan was not deprived of due process.

DOCTRINE: Where an opportunity to be heard either through oral


arguments or through pleadings is accorded, there is no denial of
procedural due process.

FACTS:
Georgito Posesano was an Instructor II (Salary Grade 13) while
Magdalena Divinagracia was an Education Program Specialist II
(Salary Grade 16), both at Surigao del Norte School of Arts and
Trades (SNSAT).
Commission on Higher Education, Caraga Administrative Region (CHED-
CAR) appointed both Posesano and Divinagracia as Vocational
Instruction Supervisor III (Salary Grade 18) at SNSAT, which were
duly approved as permanent by the CSC.
Being the IOC/Principal of SNSAT, the appointments were formally
transmitted to Catacutan with copies sent over to both Posesano and
Divinagracia.
Despite receipt of such letters, both were not able to assume their new
position as Catacutan strongly opposed the appointments and even
said that he would not implement them despite orders from CHED
and the CSC, Caraga Regional Office.
Hence a complaint was filed by both instructors, before the RTC of
Surigao, where Catacutan pleaded “not guilty”. Petitioner expressed
that he did not implement the appointments due to some procedural
lapses or infirmities attending the preparation of the appointment
papers.
In his defense it was included as well that the appointment papers were
prepared by the SNSAT Admin Officer, Crispin Noguera, using blank
copies instead of the original appointment letters, which did not even
specify the effectivity of the appointments.
After calling CHED’s attention who said that the appointments were
regular and valid, he was not satisfied and sought the intercession of
CHED Chariman Angel Alcala to settle the administrative problem—
who did not respond.
With his reason being the protection of the interest of the government by
following strict compliance in the preparation of appointment papers,
the RTC held that Catacutan was guilty of the violation of Sec 3(e) of
RA 3019 (Anti-Graft and Corrupt Practices Act) with the penalty of
imprisonment of 6 years and 1 month and perpetual disqualification
from public office, with costs. MR was also denied.
Sandiganbayan affirmed the trial court’s decision, being supported by the
evidence and firmly anchored in law and jurisprudence. Hence, this
petition.

ISSUE/s:
WoN there was a violation of the petitioner’s constitutional rights to due
process and equal protection of the law when he was denied the
opportunity to present in evidence the CA’s decision. – NO

RULING: Petition denied, decision of Sandiganbayan affirmed.

RATIO:
Petitioner contends that he was not ab le to controvert findings as he
was not able to present the CA decision (CA-G.R. SP No. 51795)
which denied the administrative case filed against him.
“Due process simply demands an opportunity to be heard”. “Due process
is satisfied when the parties are accorded a fair and reasonable
opportunity to explain the respective sides of the controversy”.
“Where an opportunity to be heard either through oral arguments or
through pleadings is accorded, there is no denial of procedural due
process”.
Guided by these established jurisprudential pronouncements, petitioner
can hardly claim denial of his fundamental right to due process.
Petitioner was able to confront and cross-examine the witnesses against
him, argue his case vigorously, and explain the merits of his defense.
There is no denial of due process when the trial court did not allow
petitioner to introduce as evidence the CA Decision.
It is well within the court’s discretion to reject presentation of evidence
which it believes irrelevant and impertinent to the proceeding on
hand. Especially true when the evidence precented in a criminal
proceeding concerns an administrative manner, as remarked by the
Sandiganbayan in its ruling, that such admin cases are not binding
upon a criminal case even if based on the same facts.
At any rate, even assuming that the trial court erroneously rejected the
CA Decision, petitioner is not left without legal recourse, he can avail
of the remedy under Sec 40 of the rules of court, which states that he
may attach it
or make it part of the record.
As observed by the CA, if the petitioner is keen on having the RTC admit
the CA’s decision, he could have included the same in his offer of
exhibits. If it is rejected, the party should ask the court’s permission
to have the exhibit attached to the record—this was not observed in
this case, the Decision does not form art of the record, hence has no
weight.
Sec 3(e) of RA 30198, “this shall constitute corrupt practices of any
public officer and are unlawful: causing injury to any party including
Government or giving any private party any unwarranted benefits,
advantage or preference in the discharge of his official,
administrative or judicial functions through manifest partiality, event
bad faith or gross inexcusable negligence. This provision shall apply
to officers and employees of offices or government corporations
charged with the grant of licenses or permits or other concessions”.
MORTEL v. KERR (DANNAH) The pre-trial was reset 4 times but on the 5th setting, both Mortel and Atty.
12 November 2012 | Kapunan, J. | Deprivation of Property Without Due Mas were not around when the case was called.
Process The RTC declared Mortel in default, and allowed Kerr to present
evidence.
On Dec 2000, Atty. Tumulak filed a notice of appearance on behalf of Mortel
PETITIONER: Dennis Q. Mortel
RESPONDENTS: Salvador Kerr

SUMMARY: Salvador Kerr instituted a complaint for foreclosure of


Mortgage against Mortel. The pre-trial was reset 4 times but on the 5 th
setting, Mortel and his counsel were not present. Hence, the trial court
set Mortel in default and eventually ruled in favor of Kerr. He was to pay
his obligation in the amount of P130,000, Atty’s fees of P20,000, and his
house (which was mortgaged) was to be auctioned and the proceeds
would go to the payment of obligations.

A series of events happened, urging Mortel to change counsel’s a few


times. His motions and appeals were denied by the RTC (for
prescription) and the CA, which urged him to file a petition for review
on certiorari assailing the CA’s dismissal of his petition for review on
certiorari.

The Court then reversed the decision of the lower courts, stating that
Mortel should not be deprived of his day in court and of his property
because of the negligence and fault of his counsel.

DOCTRINE: 1. When the incompetence, ignorance or inexperience of


counsel is so great and the result is so serious that the client, who otherwise
has a good cause, is prejudiced and denied his day in court, the client
deserves another chance to present his case; hence, the litigation may be
reopened for that purpose.

When an unsuccessful party has been prevented from fully and fairly
presenting his case because of his attorneyÊs professional delinquency
or infidelity the litigation may be reopened to allow the party to present
his side.

Where counsel is guilty of gross ignorance, negligence and dereliction of


duty, which resulted in the clientÊs being held liable for damages in a
damage suit, the client is deprived of his day in court and the judgment
may be set aside on such ground.

FACTS:
On July 19, 2000, respondent Kerr instituted a complaint for
foreclosure of mortgage against petitioner Mortel, who filed an
answer on August 2000 through Atty. Mas of the Public Attorney’s
Office (PAO).
but the RTC did not act on it. It rendered a judgment in favor of Kerr on W/N Mortel was deprived of his property without due process of law
Feb 2001, ordering Mortel to pay the sum of P130,000 with interest, and –
P20,000 in attorney’s fees. The house and lot which was mortgaged was YES
to be sold at a public auction and proceeds applied to the obligation and
cost of the suit. RULING: WHEREFORE, the Court REVERSES the resolution
On March 2001, Mortel, through Atty. Lacambra, filed a motion for new promulgated on September 5, 2002; ANNULS and SETS ASIDE the
trial. The next day, Atty. Mas filed his withdrawal of appearance. decision rendered in Civil
On April, RTC denied Mortel’s motion of new trial because of
prescription.
On May 2001, Mortel, this time through Atty. Tumulak, filed a verified
petition for relief from judgment under Rule 39 of the Rules of Court.
On August, this was again denied due to prescription (exceeded 4 days).
On November 2001, Mortel moved for reconsideration of the denial of his
petition for relief.
On December 2001, RTC granted Atty Mas and Atty Lacambra’s
withdrawal, and finally recognized Atty. Tumulak as only counsel.
On January 2002, the RTC treated MortelÊs motion for reconsideration
as a mere scrap of paper and ordered it stricken from the records for
failure of the counsel to serve a notice of hearing with the motion for
reconsideration.
Mortel filed an urgent motion for reconsideration which was again denied
for being moot and academic.
Subsequently, on June 20, 2002, RTC issued a writ of execution and
Kerr was placed in possession of the property.
On August 2002, Mortel, through Atty. Tumulak, filed in the CA a petition
for review on certiorari with prayer for the issuance of a restraining
order.
On September, the CA dismissed said petition for failing to state the
material dates showing that the petition had been filed w/in the
reglementary period, violationg Section 6(d) Rule 43 of the Rules of
Court.
On October, Mortel sought for reconsideration but was denied on
November for lack of merit, and for availing himself of the remedy of
petition for review when he should have filed a petition for certiorari
instead.
Instead of appealing via petition for review of certiorari in the SC, Mortel,
through Atty. Tumulak, filed in the CA on December an urgent motion
for extension of time to appeal to the SC.
On the same month, Mortel took the matters into his own hands and by
himself sought an extension of time to file a petition for review on
certiorari.
On January 2003, the Court granted said motion. A few days after,
Mortel, still by himself, filed his petition for review on certiorari
assailing the CA’s dismissal of the same petition.

ISSUE/s:
W/N the negligence of Mortel’s previos counsels should bind him – NO.
Case No. 279-0-2000 on February 28, 2001 by the Regional Trial Court, In Apex Mining Inc. v. CA, it was held that “when the
Branch 72, in Olongapo City; and RE-OPENS Civil Case No. 279-0- 2000 for incompetence, ignorance or inexperience of counsel is so
the reception of evidence for the petitioner as the defendant. great and the result is so serious

RATIO:
As a rule, a client is bound by his counsel’s conduct, negligence and
mistake in handling a case. But the rule admist of exceptions.
The Court then cited several cases (3, to be exact) wherein negligence of
the counsel is sos gross that the client was deprived his day in court,
also depriving the client of his property without due process of law.
The relevant question becomes, therefore, whether the negligence of
Mortel’s counsels was so gross and palpable as to deprive him
of his property without due process of law.
Mortel did not have his day in court because on the 5th setting, he was
unable to submit his evidence after the RTC declared him in default.
He explained that he was actually there but was only late, which the
Court believed as he was present on the previous settings.
In Leyte v. Cusi, the Court stated that “There are instances when a party
may be properly defaulted, but such instances should be the
exception rather than the rule and should be allowed only in clear
cases of a litigant’s obstinate refusal or ordinate neglect to comply with
the orders of the court.”
Without such a showing, the litigant must be given every reasonable
opportunity to present his side and to refute the evidence of the
adverse party in deference to due process of law.
The negligence that actually warrants the undoing of the RTC’s decision
was serial on the part of Atty. Mas, the RTC and Atty. Tumulak.
The reason of the RTC’s actions upon a matter as essential to the client
and to the administration of justice in the case as the substitution of
counsel is not easy to appreciate, especially because the RTC
tendered no good reason for it.
Atty. Tumulak’s further moves to rectify the situation, no matter how well
intentioned, were contrary to the pertinent rules of procedure and
worked against the client’s interest.
The negligence and mistakes committed by his several counsels were so
gross and palpable that they denied due process to Mortel and
could have cost him his valuable asset.
Mortel stated that he had already paid the principal of the loan and the
interest, submitting in support of his statement a receipt for
P200,000.00 that Kerr had allegedly signed. He also stated that he
had actually overpaid in view of his arrangement for Kerr to withdraw
P6,000.00 each month from Mortel’s bank account as payment of the
interest.
that the client, who otherwise has a good cause, is prejudiced and
denied his day in court, the client deserves another chance to present
his case; hence, the litigation may be reopened for that purpose. Also,
when an unsuccessful party has been prevented from fully and fairly
presenting his case because of his attorney’s professional delinquency
or infidelity the litigation may be reopened to allow the party to present
his side. Lastly, where counsel is guilty of gross ignorance, negligence
and dereliction of duty, which resulted in the client’s being held liable
for damages in a damage suit, the client is deprived of his day in court
and the judgment may be set aside on such ground.”
Court litigation is primarily a search for truth, and a liberal interpretation of the
rules that gives to both parties the fullest opportunity to adduce proof is the
best way to ferret out such truth.
To cling to the general rule of having the ignorance, negligence and dereliction of
duty of the counsel bind the client is only to condone rather than to rectify a
serious injustice to a party whose only fault was to repose his faith and
entrust his cause to his counsel.
GRAVIDES v. COMELEC (IYA) During the preliminary conference, Gravides moved for the dismissal of
Month, date, year | Ponencia, J. | Procedural Due Process the election protest for non-compliance with the above mentioned
Rules of
PETITIONER: Isabelita P. Gravides
RESPONDENTS: Commission On Elections and Pedro C. Borjal

SUMMARY: The issue of this case revolves around the election for
Barangay Officials in Brgy UP Campus, Diliman. Gravides and Borjal
both ran for the position of Punong Barangay. Gravides won by mere 2
points. With this Borjal filed an Election Protest, he questioned the
results saying that there were several irregularities and violations
committed. Gravida in her answer stated that Borjal violated the Rules of
Procedure on Election Contests.

The MeTC on the pre-trial conference dismissed the case. Brojal appealed
the order of dismissal to the COMELEC which was granted. Gravides filed
an MR but was denied by the COMELC en banc, hence the present petition
for certiorari.

The SC ruled in favor of Borjal reiterating the decision of the CA that the
present circumstances require a liberal application of the Rules since the
lead of the winning candidate was only 2 points.

DOCTRINE: It bears stressing that blind adherence to a technicality, with the


inevitable result of frustrating and nullifying the constitutionally guaranteed of
right of suffrage cannot be countenanced. The paramount interest of
determining the true will of the electorate thus justified a relaxation of
procedural rules.

FACTS:
Petitioner Gravides and Respondent Borjal both ran for the position of
Punong Barangay of Brgy. UP Campus in Diliman, Quezon City in
the year 2010.
The Barangay Board of Canvassers proclaimed Gravides as winner with
a total of 2,322 votes. Borjal only obtained 2,320 votes.
Borjal then filed an Election Protest. He asserted that there is a need for
revision, re-appreciation of ballots, judicial recount and thorough
scrutiny of the election returns and minutes of voting.
Gravides filed her Answer denying the allegations. She contends that
such are general and sweeping allegations which violate the
provisions of the Rules of Procedure on Election Contests Before the
Courts Involving Elective Municipal and Barangay Officials.
The MeTC issued a Notice of Pre-Trial Conference, which enumerated
the points to be discussed on the said date.
Procedure on Election Contests. The MeTC granted the motion.
Borjal appealed the order of dismissal to the COMELEC. He argues that
the Rules of Procedure should not have been applied and that he
was misinformed in the contents of the preliminary conference brief.
COMELEC’s First Division granted the appeal saying that
Although Borjal failed in complying with the rules, the omissions
do not warrant the outright dismissal of the protest.
It is undoubted that Borjal does not seek the examination,
verification or re-tabulation of the election returns. Therefore,
a statement for its procedure is not necessary.
The lead of Gravidez is only by two points. It is highly possible
for the outcome of the elections be different upon a re-count
of votes.
It bears stressing that blind adherence to a technicality, with
the inevitable result of frustrating and nullifying the
constitutionally guaranteed of right of suffrage cannot
be countenanced.
Gravides filed a motion for reconsideration but was denied by the
COMELEC En Banc because of failure to pay motion fees.
Hence, the present petition for certiorari.

ISSUE/s:
WON there is grave abuse of discretion in the decisions of COMELEC —
NO GRAVE ABUSE

RULING: WHEREFORE, premises considered, the petition for certiorari is


DISMISSED.

RATIO:
Petitioner maintains that the Rules should not be taken lightly.
Preliminary conference and its governing rules are not mere
technicalities which the parties may blithely ignore or trifle with.
They are tools meant to expedite the disposition of election
cases and must, perforce, be obeyed.
However, the SC finds no grave abuse of discretion by the COMELEC.
The circumstances warrant a more reasonable and liberal application of
the rules.
Borjal was misled by the Notice of Preliminary Conference issued by the
MeTC which applied the provision on pre-trial brief under the Rules
of Civil Procedure.
The paramount interest of determining the true will of the electorate
thus justified a relaxation of procedural rules. An election
protest is imbued with public interest so much so that the need
to dispel uncertainties which becloud the real choice of the
people is imperative.
TUA v. MANGROBANG (Eliel) The CA denied the petition because the petition assailing the TPO and
motion to quash should have been filed in the RTC. And that the
February 6 1991 | Peralta, J. | Due Process
issuance of the TPO is valid.
PETITIONER/S: Ralph Tua Hence this petition.
RESPONDENT/S: Hon. Cesar Mangrobang, Presideing Judge, Branch ISSUE/S: WoN the issuance of the TPO in RA 9262 violates due process.
22, RTC, Imus, Cavite, and Rossana Honrado-Tua NO
SUMMARY: Rosanna filed in behalf of her children a petition for
WoN there is an invalid delegation of legislative power to the court and to
issuance of TPO, pursuant to RA 9262 against her husband, after claims
of threats and harm done. The RTC issued the TPO, to which Ralph barangay officials to issue protection orders.
replied with a motion to lift the TPO because it is violative of his due
WoN CA erred in not finding RTC to commit grave abuse of discretion in
process. Without awaiting for the decision, Ralph then filed with the CA
an injunction to restrain all orders and decisions of the RTC. The CA issuing the TPO.
denied for lack of merit, and hence this petition.
RULING: Motion for reconsideration denied.
DOCTRINE: It is a constitutional commonplace that the ordinary
requirements of procedural due process must yield to the necessities of RATIO:
protecting vital public interests, among which is protection of women and
Ralph directs his constitutional attack on Section 15 of RA 9262
children from violence and threats to their personal safety and security.
contending that had there been no ex parte issuance of the TPO, he
would have been afforded due process of law and had properly
FACTS: presented his side on the matter; that the questioned provision
simply encourages arbitrary enforcement repulsive to basic
Rosanna and Ralph are married with three kids, Joshua Raphael, Jesse
constitutional rights which affects his life, liberty and property.
Ruth Lois, and Jezreel Abigail.
A protection order is an order issued to prevent further acts of violence
Rosanna filed with the RTC of Cavite a verified Petition for herself and in
behalf of her minor children, for the issuance of a protection order, against women and their children, their family or household
members, and to grant other necessary reliefs. Its purpose is to
pursuant to RA 9262 or the Anti-Violence Against Women and their
Children Act of 2004, against her husband, Ralph Tua. safeguard the offended parties from further harm, minimize any
Rosanna claimed that she and her children suffered from petitioner’s abusive disruption in their daily life and facilitate the opportunity and ability to
conduct; that petitioner had threatened to cause her and the children regain control of their life.
physical harm for the purpose of controlling her actions or decisions; that
Since “time is of the essence in cases of VAWC if further violence is
she was actually deprived of custody and access to her minor children; to be prevented,” the court is authorized to issue ex parte a TPO
and that she was threatened to be deprived of her and her childnre’s after raffle but before notice and hearing when the life, limb or
financial support.
property of the victim is in jeopardy and there is reasonable
ground to believe that the order is necessary to protect the
The RTC issued a Temporary Protection Order, after Ralph allegedly took
victim from the immediate and imminent danger of VAWC or to
the children.
prevent such violence, which is about to recur.
Ralph replied with a Petition with Urgent Motion to Lift TPO, and
The grant of a TPO ex parte cannot, therefore, be challenged as violative
contended that the issuance of the TPO is unconstitutional for being
of the right to due process.
violative of the due process clause of the Constitution.
It is a constitutional commonplace that the ordinary requirements of
Without awaiting for the resolution of his Comment on the petition and
procedural due process must yield to the necessities of
motion to lift TPO, Ralph filed with the CA a petition for certiorari with
protecting vital public interests, among which is protection of
prayer for the issuance of a writ of preliminary injunction and hold
women and children from violence and threats to their personal
departure order assailing the TPO issued by the RTC.
The CA issued a temporary restraining order to temporarily enjoin the parties safety and security.
Ralph filed an Urgent Motion for Issuance of a Writ of Preliminary Injunction It should be pointed out that when the TPO is issued ex parte, the court
with Manifestation, praying that the enforcement of all orders, decision to shall likewise order that notice be immediately given to the
be issued by the RTC and all the proceedings therein be restrained.
respondent directing him to file an opposition within 5 days from
service.
The essence of due process is to be found in the reasonable
opportunity to be heard and submit any evidence one may have
in support of one’s
defense. “To be heard” does not only mean verbal arguments in court;
one may be heard also through pleadings. Where opportunity to be
heard, either through oral arguments or pleadings is accorded, there is
no denial of due process.
The act of congress to entrust the issuance of protection orders to courts
and barangay officials is in pursuance of authority to settle justiciable
controversies or disputes involving rights that are enforceable and
demandable before the courts of justice or the redress of wrongs for
violations of such rights.
It is within the court’s discretion, based on the petition and the affidavit
attached thereto, to determine that the violent acts against women
and their children for the issuance of a TPO have been committed.
It is a settled doctrine that there is GAD when there is a capricious and
whimsical exercise of judgment as is equivalent to lack of jurisdiction;
such as where the power is exercised in an arbitrary or despotic
manner by reason of passion or personal hostility; it must be so
patent and gross so as to amount to an evasion of positive duty or to
a virtual refusal to perform the duty enjoined or to act at all in
contemplation of law.
PEOPLE v. ROXAS (JP) her jewelries consisting of bracelets, earrings, necklace and watch,
August 17, 2010 | Peralta, J. | Due Process and her shoes (30,000-40,000

PETITIONER: People of the Philippines


RESPONDENTS: Venancio Roxas y Arguelles

SUMMARY: Agnes was maliciously misled by Roxas, wearing a PNP


vest, that she was driving against a one way street. Upon getting to ride
into the vehicle, Roxas then held Agnes at gun point and the latter was
then held against her will by co-accused Gungon. Ultimately, she was led
to Batangas where she would be salvaged. She was forced to drink a
drugged softdrink and she lost consciousness. Upon waking up, she was
bloody and had holes in her cheeks. After due investigation, the culprits
were captured. Roxas appealed and asked for the inhibition of the judge
of the case because he believes he was being deprived of due process
through the partiality of the judge, influenced by the presence of a justice
and the media. Since no substantial proof was showed, the SC affirmed
the trial court and Roxas was convicted of Kidnapping and Serious Illegal
Detention with Frustrated Murder, Carpanning and Theft.

DOCTRINE: 1. The fact that the trial judge opted to believe the
prosecution’s evidence rather than that of the defense is not a sign of
bias; bias and impartiality being breaches of due process of law.
Before concluding that there is indeed prejudice against an appellant,
substantial proof must be given and that this has indeed influenced the
court’s decision.

FACTS:
Agnes Guirindola (Agnes) was driving along Quezon City on her Nissan
Sentra when she was suddenly flagged down by a mena wearing a
PNP reflectorized vest who later on turned out to be defendant
Venancio Roxas (Roxas). She was told that she was driving against
a one-way street. Roxas then asked to ride inside Agnes’ car and
asked her to make a U-turn. Thinking Roxas asked for a bribe, she
gave him 50 pesos.
Roxas then poked her with a gun and asked to take control of the car.
Co-accused Gungon then entered the vehicle and then held Agnes,
pulling her to the back seat.
They stopped by a sari-sari store and Roxas offered Agnes a softdrink.
Agnes declined because of the presence of bubbling tablets inside it.
She was however forced to drink it at some point in the ride and was
even asked to take two additional tablets which she acted like she
did take but she actually spat out.
After Agnes saw from a signboard that she was in Batangas, she felt
dizzy and fell asleep. When she woke up, she found herself lying at
the back seat with her legs on the lap of Gungon. She noticed that
pesos) were already gone. She also lost her wallet and cash inside it The fact that the trial judge opted to believe the prosecution’s evidence
(4,000 pesos). She also noticed that there was a third man in the rather than that of the defense is not a sign of bias; bias being a
front seat. breach of due process. The trial was far from impartial.
She then asked to relieve herself and was escorted by Gungon at a
grassy area. When she was about to return to the car, she saw white
sparks at her right side and then she fell down. When she opened
her eyes, she saw Roxas walking towards the car with a gun in his
hand. She then lost consciousness.
When she regained consciousness, she was all alone. She followed a
“sparkling light” that led to her to a small house. When she was
ignored by the people outside because of the blood oozing from her
face and the holes in her cheeks, she went into the living room and
lied down on the sofa where she was discovered by certain
concerned people and carried to the Batangas Regional Hospital
where she was treated. Her parents immediately arrived who
immediately reported the incident to the NBI.
Gungon was arrested in Dabao City and after Gungon was found guilty,
Roxas was also apprehended after being at large for a while,
masking himself as worker under the Office of the Mayor in
Batangas. They were positively identified by Agnes in police line-ups.
Roxas used an alibi that he was fixing the antenna of his house and
didn’t leave for the whole night of the incident. Regardless, the court
a quo found Roxas guilty of Kidnapping and Serious Illegal Detention
with frustrated murder, carpanning and theft.
Roxas asked for a motion for reconsideration and asked for the inhibition
of the Honorable Judge Demetrio Macapagal, Sr. He argued that the
presence of then Justice Secretary Hernando Perez and the media
showed the court’s predisposition to convict him of the offense
charged (in other words he claimed that the court already pre-
decided him to guilty). He contended he was deprived of his due
process because Judge Demetrio has lost the cold neutrality of an
impartial judge required.
RTC denied the motions for reconsideration and inhibition of Judge
Demetrio. The records of this case were referred to the Court of
Appeals for intermediate review. The CA affirmed in toto the decision
of the court a quo. Hence, this appeal.

ISSUE/s:
WoN the court a quo erred in rendering the case despite the fact that
the presiding judge of that court has lost the cold neutrality of an
impartial judge thereby violating the right of the accused-appellant
to due process
– NO
WoN he is guilty of the crimes charged - YES

RULING: Roxas’ appeal is denied. The decision is affirmed.

RATIO:
There is no sufficient basis to show that the presence of Secretary
Hernando Perez and the media (their presence or the pervasive
publicity fo the issue) unduly influenced the court’s judgment.
He must first show substantial proof, not merely cast suspicions. There
must be a showing that adverse publicity indeed influenced the
court’s decision. The Court finds no such proof in the case at bar. He
was not deprived due process.
With regards with the crimes charged, Roxas was absolutely guilty
beyond reasonable doubt as per the testimony of Agnes which the
trial court upon proper observance, believed. They were guilty of
kindapping and serious illegal detention because they simulated
public authority and detained a female. They are guilty of frustrated
murder as qualified by treachery and evident premeditation because
of the gunshot wound sustained by the victim. They are also guilty of
carnapping for unlawfully taking Agnes’ car by means of force by
poking her with a gun with intent to gain. And they are guilty of theft
only, not robbery, because no qualifying circumstance of force was
proved.
ANG TIBAY v. CIR (HENRY) opposition for. Hence, this motion.
February 27, 1940 | Laurel, J. | Procedural Due Process
ISSUE/s:
PETITIONER: Ang Tibay, represented by Toribio Teodoro, et al. WoN NLU was denied due process by the CIR. – YES
RESPONDENTS: The Court of Industrial Relations, et al.
RULING: Motion for new trial granted, case is remanded to the CIR with
SUMMARY: Ang Tibay allegedly ran low of leather supplies and are instruction that it reopen the case, receive all such evidence as may be
unable to commit to the Philippine Army enough leather products. This relevant, and otherwise proceed in accordance with the requirements set
lead to the owner, Teodoro, to decide to lay off some of its employees. All forth.
being part of NLU, they filed against Teodoro with unfair labor practice as
ground, they were alleging that NWB was being unjustly favored as RATIO:
nobody from them was laid off. The CIR ruled in favor of Teodoro, Cir is a special court whose functions are stated in CA No. 103. It is more
however the NWB appealed to the SC, who affirmed the decision. There of an administrative board than a part of the integrated judicial
was then a motion for a new trial filed by NLU with new and solid system. It not only exercises judicial or quasi-judicial functions in the
evidence that made the SC grant their motion in the light of procedural determination of disputes between employers and employees but its
due process. functions are far more comprehensive and extensive, as far as
issues are concerned with employers and laborers, landlords and
DOCTRINE: The Court of Industrial Relations may be said to be free tenants, among others.
from the rigidity of certain procedural requirements but it does not mean Its duty is to prevent or arbitrate disputes which are submitted to the
that it can, in justifiable cases coming before it, entirely ignore or Secretary of Labor which are to be dealt by the Court for the sake of
disregard the fundamental and essential requirements of due process in public interest, which is possible through reconciliation of parties
trials and investigations of an administrative case. and/or inducing them to settle by amicable agreement.
SC had the occasion to point out that CIR is not narrowly constrained by
technical rules of procedure, and CA No. 103 requires it to act
FACTS:
according to justice and equity and substantial merits of the case,
Petitioner Ang Tibay, a leather company, experienced shortage of leather without regard to technicalities or legal forms and shall not be bound
soles on 1938, making it necessary for Toribio Teodoro (owner) to by any technical rules of legal evidence but may inform its mind in
temporarily lay off some of its employees (who in this case, such manner as it may deem just and equitable.
apparently are members of the National Labor Union Inc.). However, this does not entail that CIR is free from the rigidity of certain
The NLU claims that Ang Tibay is guilty of unfair labor practices because procedural requirements most specially the fundamental and
Teodoro is discriminating against the NLU and is unjustly favoring the essential requirements of due process in trials and investigations of
National Workers’ Brotherhood (company/employer union dominated an administrative character. There are cardinal primary rights which
by him) because nobody from NWB was included in those laid off. must be respected even in proceedings:
Aggrieved, the employees filed a case against Ang Tibay to the CIR, Right to a hearing
alleging it of unfair labor practice, that Teodoro’s claim of shortage of Tribunal must consider evidence presented
leather is entirely falseand unsupported by records of the BOC and Decision must have something to support itself
the Books of Accounts, and was only in fact a scheme to discharge Evidence must be substantial
members of the NLU from work. CIR ruled in favor of NLU. Decision must be based on evidence presented at a hearing
By appeal to the SC, the ruling was reversed after the SC found that Tribunal or body or any of its judges must act n its own
there was no merit to NLU’s contention. independent consideration of the law and facts of the
NLU then prays for the vacation of the judgment rendered by the SC with controversy, and not simply accept the views of a
the following claims, among others: subordinate
That Teodoro’s claim of shortage of leather soles in ANG TIBAY The board or body should, in all controversial questions, render
is entirely false and unsupported to its decision in such manner that the parties to the
Shortage of leather materials was but a scheme to systematically proceeding can know the various issue involved, and the
discharge all members of NLU from work reason for the decision rendered
NWB is dominated by Teodoro Since there was a failure to grasp the fundamental issue involved due to
NLU hence filed for a Motion for New Trial, with which Ang Tibay filed an failure to receive all relevant evidence, the motion for a new trial was
granted.
BANCO ESPANYOL-FILIPINO v. PALANCA (JP) 7 years after the sale, a motion was made by Vicente Palanca, as
administrator of the estate of the original defendant, wherein Vicente
March 26, 1918 | Street, J. | Due Process
requested the court to
PETITIONER: El Banco Español-Filipino
RESPONDENTS: Vicente Palanca

SUMMARY: El Banco instituted a foreclosure for the mortageged


properties of Palanca which he used for security for a loan. 7 years after
the foreclosure and sale of the property, Palanca’s administratix
requested to set aside the previous judgement on the ground that the
court had no jurisdiction because no notice was actually received by the
defendant who is a nonresident. Trial court denied the motion. The SC
affirmed saying that since the requirement publication in a newspaper
was met, the due process requirement of the defendant’s right to be
heard is already met. Receipt of notice is not essential. The possibility of
the clerk failing to deliver the letter is not fatal.

DOCTRINE: In a foreclosure case, some notification of the proceedings


to the nonresident owner is everywhere recognized as essential. Statutes
generally provide for publication, and usually in addition for mailing the
notice to the defendant, if his residence is known. However, among the
two, only publication is necessarily essential.

FACTS:
El Banco Español-Filipino (bank) instituted action against Engracio
Palanca to foreclose a mortgage upon various parcels of real
property in Manila City. The property was used as security for a debt
by Palanca with the bank.
After the execution of the instrument by the mortgagor, Palanca, he
returned to China, his native country. He then died there.
Since required pursuant to Section 399 of the Code of Civil Procedure,
the defendant being a nonresident at the time of the institution of the
present action, publication was made in due form in a newspaper of
the City of Manila. The court further directed that the clerk of the
court should deposit in the post office a copy of the summons and
complain directed to the defendant at his last place of residence, the
City of Amoy, in the Empire of China.
Whether the clerk complied with this order does not affirmatively appear.
There is however, an affidavit signed by an employee of the bank’s
attorneys showing that he (the employee) had deposited in the
Manila post-office a registered letter of the summons, affidavit and
the complaint.
The case proceeded in the usual course. The defendant not having
appeared, the judgment was taken against him by default, in favor of
the El Banco. Since the defendant naturally did not satisfy the
judgment, the mortgage property was sold in a public auction and
was bought by the bank.
set aside the previous order of default because the court had never All that due process of law of law thereafter requires is an opportunity for
acquired jurisdiction over the defendant or over the subject of the the defendant to be heard. As publication was duly made, it would
seem highly unreasonable to hold that the failure to mail the notice
action.
was fatal.
14. The lower court denied the application and from this, Vicente In every situation of this character (property related cases) an appropriate
appealed. remedy is always at hand. If the property has been taken without due
ISSUE/s: process,
WoN the acquired the necessary jurisdiction to enable it to proceed with
the foreclosure of the mortgage – YES
WoN the irregularity in the proceedings (the notice not being
secured) was of such gravity as to amount to a denial of the due
process of law -
NO

RULING: Judgement in question is affirmed.

RATIO:
The third requirement of due process is that the defendant shall have an
opportunity to be heard. In a foreclosure case, some notification of
the proceedings to the nonresident owner is everywhere recognized
as essential.
Statutes generally provide for publication, and usually in addition for
mailing the notice to the defendant, if his residence is known.
It will be observed that this mode of notification does not inolve any absolute
assurance that the absent owner shall thereby receive actual notice. The
periodical containing the notice has a chance to not come in the
defendant’s hands. The provision also of our law relative to the mailing
of notice does not absolutely require the mailing of notice unconditionally
and in every event, but only in the case where the defendant’s residence
is known.
Hence, actual notice to the defendant in cases of this kind is not under the
law not be considered absolutely necessary. Reasoning of this rule:
Property is always assumed to be in the possession of the owner. It can
be assumed that he will know of any proceeding related to it for its
condemnation and sale.
Duty of the owner of the real estate who is a nonresident to take
measures that he is represented when his property is called into
requisition. If he fails to do this or get notice of the publiczation which
have usually been required, it is his misfortune.
If property of a nonresident cannot be reached by legal process upon
constructive notice, then our statues were passed in vain, for if the
person is not within the jurisdiction of the court, no judgment can
operate on the property and the courts would be powerless to assist
a citizen against a nonresident.
Notice was given by publication in a newspaper and this is the only form
of notice which the law unconditionally requires. This is in our (the
SC) opinion all that was absolutely necessary to sustain the
proceedings.
the law concedes due process to recover it. In other words, there must always
be a remedy for property.
In his defense it was included as well that the appointment papers were
CATACUTAN v. PEOPLE (HENRY) prepared by the SNSAT Admin Officer, Crispin Noguera, using blank
forms bearing the letterhead of SNSAT and not of the CHED Regional
August 31, 2011 | Del Castillo, J. | Procedural Due Process
Office who
PETITIONER: Jose R. Catacutan
RESPONDENTS: People of the Philippines

SUMMARY: Posensano and Divinagracia of the SNSAT were appointed


by CHED-CAR to Vocational Instruction Supervisor III, which was duly
approved by the CSC. However, SNSAT’s IOC/Principal refused the
implementation of such appointments, which lead to the filing of a
complaint against him by the two appointees. In his defense he said that
there were procedural lapses that hindered him in implementing the
same. Trial Court ruled against him and held him liable for violation of
Sec 3(e) of RA 3019, and he appealed to the Sandiganbayan— who
affirmed the trial court’s decision. He tried to present the CA decision
(dismissing the admin case filed against him) to the Sandiganbayan but
to no avail, which caused him to appeal to the SC, that he was denied of
due process when the inclusion of the CA Decision was denied by the
lower court. SC affirmed the ruling of the Sandiganbayan, and stated that
Catacutan was not deprived of due process.

DOCTRINE: Where an opportunity to be heard either through oral


arguments or through pleadings is accorded, there is no denial of
procedural due process.

FACTS:
Georgito Posesano was an Instructor II (Salary Grade 13) while
Magdalena Divinagracia was an Education Program Specialist II
(Salary Grade 16), both at Surigao del Norte School of Arts and
Trades (SNSAT).
Commission on Higher Education, Caraga Administrative Region (CHED-
CAR) appointed both Posesano and Divinagracia as Vocational
Instruction Supervisor III (Salary Grade 18) at SNSAT, which were
duly approved as permanent by the CSC.
Being the IOC/Principal of SNSAT, the appointments were formally
transmitted to Catacutan with copies sent over to both Posesano and
Divinagracia.
Despite receipt of such letters, both were not able to assume their new
position as Catacutan strongly opposed the appointments and even
said that he would not implement them despite orders from CHED
and the CSC, Caraga Regional Office.
Hence a complaint was filed by both instructors, before the RTC of
Surigao, where Catacutan pleaded “not guilty”. Petitioner expressed
that he did not implement the appointments due to some procedural
lapses or infirmities attending the preparation of the appointment
papers.
made the appointments, another lapse was that he only received
duplicate copies instead of the original appointment letters, which did
not even specify the effectivity of the appointments.
After calling CHED’s attention who said that the appointments were
regular and valid, he was not satisfied and sought the intercession of
CHED Chariman Angel Alcala to settle the administrative problem—
who did not respond.
With his reason being the protection of the interest of the government by
following strict compliance in the preparation of appointment papers,
the RTC held that Catacutan was guilty of the violation of Sec 3(e) of
RA 3019 (Anti-Graft and Corrupt Practices Act) with the penalty of
imprisonment of 6 years and 1 month and perpetual disqualification
from public office, with costs. MR was also denied.
Sandiganbayan affirmed the trial court’s decision, being supported by the
evidence and firmly anchored in law and jurisprudence. Hence, this
petition.

ISSUE/s:
WoN there was a violation of the petitioner’s constitutional rights to due
process and equal protection of the law when he was denied the
opportunity to present in evidence the CA’s decision. – NO

RULING: Petition denied, decision of Sandiganbayan affirmed.

RATIO:
Petitioner contends that he was not ab le to controvert findings as he
was not able to present the CA decision (CA-G.R. SP No. 51795)
which denied the administrative case filed against him.
“Due process simply demands an opportunity to be heard”. “Due process
is satisfied when the parties are accorded a fair and reasonable
opportunity to explain the respective sides of the controversy”.
“Where an opportunity to be heard either through oral arguments or
through pleadings is accorded, there is no denial of procedural due
process”.
Guided by these established jurisprudential pronouncements, petitioner
can hardly claim denial of his fundamental right to due process.
Petitioner was able to confront and cross-examine the witnesses against
him, argue his case vigorously, and explain the merits of his defense.
There is no denial of due process when the trial court did not allow
petitioner to introduce as evidence the CA Decision.
It is well within the court’s discretion to reject presentation of evidence
which it believes irrelevant and impertinent to the proceeding on
hand. Especially true when the evidence precented in a criminal
proceeding concerns an administrative manner, as remarked by the
Sandiganbayan in its ruling, that such admin cases are not binding
upon a criminal case even if based on the same facts.
At any rate, even assuming that the trial court erroneously rejected the
CA Decision, petitioner is not left without legal recourse, he can avail
of the
remedy under Sec 40 of the rules of court, which states that he may attach it
or make it part of the record.
As observed by the CA, if the petitioner is keen on having the RTC admit
the CA’s decision, he could have included the same in his offer of
exhibits. If it is rejected, the party should ask the court’s permission
to have the exhibit attached to the record—this was not observed in
this case, the Decision does not form art of the record, hence has no
weight.
Sec 3(e) of RA 30198, “this shall constitute corrupt practices of any
public officer and are unlawful: causing injury to any party including
Government or giving any private party any unwarranted benefits,
advantage or preference in the discharge of his official,
administrative or judicial functions through manifest partiality, event
bad faith or gross inexcusable negligence. This provision shall apply
to officers and employees of offices or government corporations
charged with the grant of licenses or permits or other concessions”.
Due process of law is not denied by the exclusion of irrelevant,
immaterial, or incompetent evidence, or testimony of an incompetent
witness. It is not an error to refuse evidence which although
admissible for certain purposes, is not admissible for the puorpose
which counsel states as the ground for offering it.
MORTEL v. KERR The pre-trial was reset 4 times but on the 5th setting, both Mortel and Atty.
12 November 2012 | Kapunan, J. | Deprivation of Property Without Due Mas were not around when the case was called.
Process The RTC declared Mortel in default, and allowed Kerr to present
evidence.
On Dec 2000, Atty. Tumulak filed a notice of appearance on behalf of Mortel
PETITIONER: Dennis Q. Mortel
RESPONDENTS: Salvador Kerr

SUMMARY: Salvador Kerr instituted a complaint for foreclosure of


Mortgage against Mortel. The pre-trial was reset 4 times but on the 5 th
setting, Mortel and his counsel were not present. Hence, the trial court
set Mortel in default and eventually ruled in favor of Kerr. He was to pay
his obligation in the amount of P130,000, Atty’s fees of P20,000, and his
house (which was mortgaged) was to be auctioned and the proceeds
would go to the payment of obligations.

A series of events happened, urging Mortel to change counsel’s a few


times. His motions and appeals were denied by the RTC (for
prescription) and the CA, which urged him to file a petition for review
on certiorari assailing the CA’s dismissal of his petition for review on
certiorari.

The Court then reversed the decision of the lower courts, stating that
Mortel should not be deprived of his day in court and of his property
because of the negligence and fault of his counsel.

DOCTRINE: 1. When the incompetence, ignorance or inexperience of


counsel is so great and the result is so serious that the client, who otherwise
has a good cause, is prejudiced and denied his day in court, the client
deserves another chance to present his case; hence, the litigation may be
reopened for that purpose.

When an unsuccessful party has been prevented from fully and fairly
presenting his case because of his attorneyÊs professional delinquency
or infidelity the litigation may be reopened to allow the party to present
his side.

Where counsel is guilty of gross ignorance, negligence and dereliction of


duty, which resulted in the clientÊs being held liable for damages in a
damage suit, the client is deprived of his day in court and the judgment
may be set aside on such ground.

FACTS:
On July 19, 2000, respondent Kerr instituted a complaint for
foreclosure of mortgage against petitioner Mortel, who filed an
answer on August 2000 through Atty. Mas of the Public Attorney’s
Office (PAO).
but the RTC did not act on it. It rendered a judgment in favor of Kerr on W/N Mortel was deprived of his property without due process of law
Feb 2001, ordering Mortel to pay the sum of P130,000 with interest, and –
P20,000 in attorney’s fees. The house and lot which was mortgaged was YES
to be sold at a public auction and proceeds applied to the obligation and
cost of the suit. RULING: WHEREFORE, the Court REVERSES the resolution
On March 2001, Mortel, through Atty. Lacambra, filed a motion for new promulgated on September 5, 2002; ANNULS and SETS ASIDE the
trial. The next day, Atty. Mas filed his withdrawal of appearance. decision rendered in Civil
On April, RTC denied Mortel’s motion of new trial because of
prescription.
On May 2001, Mortel, this time through Atty. Tumulak, filed a verified
petition for relief from judgment under Rule 39 of the Rules of Court.
On August, this was again denied due to prescription (exceeded 4 days).
On November 2001, Mortel moved for reconsideration of the denial of his
petition for relief.
On December 2001, RTC granted Atty Mas and Atty Lacambra’s
withdrawal, and finally recognized Atty. Tumulak as only counsel.
On January 2002, the RTC treated MortelÊs motion for reconsideration
as a mere scrap of paper and ordered it stricken from the records for
failure of the counsel to serve a notice of hearing with the motion for
reconsideration.
Mortel filed an urgent motion for reconsideration which was again denied
for being moot and academic.
Subsequently, on June 20, 2002, RTC issued a writ of execution and
Kerr was placed in possession of the property.
On August 2002, Mortel, through Atty. Tumulak, filed in the CA a petition
for review on certiorari with prayer for the issuance of a restraining
order.
On September, the CA dismissed said petition for failing to state the
material dates showing that the petition had been filed w/in the
reglementary period, violationg Section 6(d) Rule 43 of the Rules of
Court.
On October, Mortel sought for reconsideration but was denied on
November for lack of merit, and for availing himself of the remedy of
petition for review when he should have filed a petition for certiorari
instead.
Instead of appealing via petition for review of certiorari in the SC, Mortel,
through Atty. Tumulak, filed in the CA on December an urgent motion
for extension of time to appeal to the SC.
On the same month, Mortel took the matters into his own hands and by
himself sought an extension of time to file a petition for review on
certiorari.
On January 2003, the Court granted said motion. A few days after,
Mortel, still by himself, filed his petition for review on certiorari
assailing the CA’s dismissal of the same petition.

ISSUE/s:
W/N the negligence of Mortel’s previos counsels should bind him – NO.
Case No. 279-0-2000 on February 28, 2001 by the Regional Trial Court, In Apex Mining Inc. v. CA, it was held that “when the
Branch 72, in Olongapo City; and RE-OPENS Civil Case No. 279-0- 2000 for incompetence, ignorance or inexperience of counsel is so
the reception of evidence for the petitioner as the defendant. great and the result is so serious

RATIO:
As a rule, a client is bound by his counsel’s conduct, negligence and
mistake in handling a case. But the rule admist of exceptions.
The Court then cited several cases (3, to be exact) wherein negligence of
the counsel is sos gross that the client was deprived his day in court,
also depriving the client of his property without due process of law.
The relevant question becomes, therefore, whether the negligence of
Mortel’s counsels was so gross and palpable as to deprive him
of his property without due process of law.
Mortel did not have his day in court because on the 5th setting, he was
unable to submit his evidence after the RTC declared him in default.
He explained that he was actually there but was only late, which the
Court believed as he was present on the previous settings.
In Leyte v. Cusi, the Court stated that “There are instances when a party
may be properly defaulted, but such instances should be the
exception rather than the rule and should be allowed only in clear
cases of a litigant’s obstinate refusal or ordinate neglect to comply with
the orders of the court.”
Without such a showing, the litigant must be given every reasonable
opportunity to present his side and to refute the evidence of the
adverse party in deference to due process of law.
The negligence that actually warrants the undoing of the RTC’s decision
was serial on the part of Atty. Mas, the RTC and Atty. Tumulak.
The reason of the RTC’s actions upon a matter as essential to the client
and to the administration of justice in the case as the substitution of
counsel is not easy to appreciate, especially because the RTC
tendered no good reason for it.
Atty. Tumulak’s further moves to rectify the situation, no matter how well
intentioned, were contrary to the pertinent rules of procedure and
worked against the client’s interest.
The negligence and mistakes committed by his several counsels were so
gross and palpable that they denied due process to Mortel and
could have cost him his valuable asset.
Mortel stated that he had already paid the principal of the loan and the
interest, submitting in support of his statement a receipt for
P200,000.00 that Kerr had allegedly signed. He also stated that he
had actually overpaid in view of his arrangement for Kerr to withdraw
P6,000.00 each month from Mortel’s bank account as payment of the
interest.
that the client, who otherwise has a good cause, is prejudiced and
denied his day in court, the client deserves another chance to present
his case; hence, the litigation may be reopened for that purpose. Also,
when an unsuccessful party has been prevented from fully and fairly
presenting his case because of his attorney’s professional delinquency
or infidelity the litigation may be reopened to allow the party to present
his side. Lastly, where counsel is guilty of gross ignorance, negligence
and dereliction of duty, which resulted in the client’s being held liable
for damages in a damage suit, the client is deprived of his day in court
and the judgment may be set aside on such ground.”
Court litigation is primarily a search for truth, and a liberal interpretation of the
rules that gives to both parties the fullest opportunity to adduce proof is the
best way to ferret out such truth.
To cling to the general rule of having the ignorance, negligence and dereliction of
duty of the counsel bind the client is only to condone rather than to rectify a
serious injustice to a party whose only fault was to repose his faith and
entrust his cause to his counsel.
GRAVIDES v. COMELEC (IYA) During the preliminary conference, Gravides moved for the dismissal of
Month, date, year | Ponencia, J. | Topic the election protest for non-compliance with the above mentioned
Rules of
PETITIONER: Isabelita P. Gravides
RESPONDENTS: Commission On Elections and Pedro C. Borjal

SUMMARY: The issue of this case revolves around the election for
Barangay Officials in Brgy UP Campus, Diliman. Gravides and Borjal
both ran for the position of Punong Barangay. Gravides won by mere 2
points. With this Borjal filed an Election Protest, he questioned the
results saying that there were several irregularities and violations
committed. Gravida in her answer stated that Borjal violated the Rules of
Procedure on Election Contests.

The MeTC on the pre-trial conference dismissed the case. Brojal appealed
the order of dismissal to the COMELEC which was granted. Gravides filed
an MR but was denied by the COMELC en banc, hence the present petition
for certiorari.

The SC ruled in favor of Borjal reiterating the decision of the CA that the
present circumstances require a liberal application of the Rules since the
lead of the winning candidate was only 2 points.

DOCTRINE: It bears stressing that blind adherence to a technicality, with the


inevitable result of frustrating and nullifying the constitutionally guaranteed of
right of suffrage cannot be countenanced. The paramount interest of
determining the true will of the electorate thus justified a relaxation of
procedural rules.

FACTS:
Petitioner Gravides and Respondent Borjal both ran for the position of
Punong Barangay of Brgy. UP Campus in Diliman, Quezon City in
the year 2010.
The Barangay Board of Canvassers proclaimed Gravides as winner with
a total of 2,322 votes. Borjal only obtained 2,320 votes.
Borjal then filed an Election Protest. He asserted that there is a need for
revision, re-appreciation of ballots, judicial recount and thorough
scrutiny of the election returns and minutes of voting.
Gravides filed her Answer denying the allegations. She contends that
such are general and sweeping allegations which violate the
provisions of the Rules of Procedure on Election Contests Before the
Courts Involving Elective Municipal and Barangay Officials.
The MeTC issued a Notice of Pre-Trial Conference, which enumerated
the points to be discussed on the said date.
Procedure on Election Contests. The MeTC granted the motion.
Borjal appealed the order of dismissal to the COMELEC. He argues that
the Rules of Procedure should not have been applied and that he
was misinformed in the contents of the preliminary conference brief.
COMELEC’s First Division granted the appeal saying that
Although Borjal failed in complying with the rules, the omissions
do not warrant the outright dismissal of the protest.
It is undoubted that Borjal does not seek the examination,
verification or re-tabulation of the election returns. Therefore,
a statement for its procedure is not necessary.
The lead of Gravidez is only by two points. It is highly possible
for the outcome of the elections be different upon a re-count
of votes.
It bears stressing that blind adherence to a technicality, with
the inevitable result of frustrating and nullifying the
constitutionally guaranteed of right of suffrage cannot
be countenanced.
Gravides filed a motion for reconsideration but was denied by the
COMELEC En Banc because of failure to pay motion fees.
Hence, the present petition for certiorari.

ISSUE/s:
WON there is grave abuse of discretion in the decisions of COMELEC —
NO GRAVE ABUSE

RULING: WHEREFORE, premises considered, the petition for certiorari is


DISMISSED.

RATIO:
Petitioner maintains that the Rules should not be taken lightly.
Preliminary conference and its governing rules are not mere
technicalities which the parties may blithely ignore or trifle with.
They are tools meant to expedite the disposition of election
cases and must, perforce, be obeyed.
However, the SC finds no grave abuse of discretion by the COMELEC.
The circumstances warrant a more reasonable and liberal application of
the rules.
Borjal was misled by the Notice of Preliminary Conference issued by the
MeTC which applied the provision on pre-trial brief under the Rules
of Civil Procedure.
The paramount interest of determining the true will of the electorate
thus justified a relaxation of procedural rules. An election
protest is imbued with public interest so much so that the need
to dispel uncertainties which becloud the real choice of the
people is imperative.
TUA v. MANGROBANG (Eliel) The CA denied the petition because the petition assailing the TPO and
February 6 1991 | Peralta, J. | Due Process motion to quash should have been filed in the RTC. And that the
issuance of the TPO is valid.
PETITIONER/S: Ralph Tua Hence this petition.
RESPONDENT/S: Hon. Cesar Mangrobang, Presideing Judge, Branch
22, RTC, Imus, Cavite, and Rossana Honrado-Tua ISSUE/S: WoN the issuance of the TPO in RA 9262 violates due process.
NO
SUMMARY: Rosanna filed in behalf of her children a petition for
issuance of TPO, pursuant to RA 9262 against her husband, after claims WoN there is an invalid delegation of legislative power to the court and to
of threats and harm done. The RTC issued the TPO, to which Ralph barangay officials to issue protection orders.
replied with a motion to lift the TPO because it is violative of his due
process. Without awaiting for the decision, Ralph then filed with the CA WoN CA erred in not finding RTC to commit grave abuse of discretion in
an injunction to restrain all orders and decisions of the RTC. The CA issuing the TPO.
denied for lack of merit, and hence this petition.
RULING: Motion for reconsideration denied.
DOCTRINE: It is a constitutional commonplace that the ordinary
requirements of procedural due process must yield to the necessities of RATIO:
protecting vital public interests, among which is protection of women and
children from violence and threats to their personal safety and security. Ralph directs his constitutional attack on Section 15 of RA 9262
contending that had there been no ex parte issuance of the TPO, he
would have been afforded due process of law and had properly
FACTS: presented his side on the matter; that the questioned provision
Rosanna and Ralph are married with three kids, Joshua Raphael, Jesse simply encourages arbitrary enforcement repulsive to basic
Ruth Lois, and Jezreel Abigail. constitutional rights which affects his life, liberty and property.
Rosanna filed with the RTC of Cavite a verified Petition for herself and in A protection order is an order issued to prevent further acts of violence
behalf of her minor children, for the issuance of a protection order, against women and their children, their family or household
pursuant to RA 9262 or the Anti-Violence Against Women and their members, and to grant other necessary reliefs. Its purpose is to
Children Act of 2004, against her husband, Ralph Tua. safeguard the offended parties from further harm, minimize any
Rosanna claimed that she and her children suffered from petitioner’s abusive disruption in their daily life and facilitate the opportunity and ability to
conduct; that petitioner had threatened to cause her and the children regain control of their life.
physical harm for the purpose of controlling her actions or decisions; that Since “time is of the essence in cases of VAWC if further violence is
she was actually deprived of custody and access to her minor children; to be prevented,” the court is authorized to issue ex parte a TPO
and that she was threatened to be deprived of her and her childnre’s after raffle but before notice and hearing when the life, limb or
financial support. property of the victim is in jeopardy and there is reasonable
The RTC issued a Temporary Protection Order, after Ralph allegedly took ground to believe that the order is necessary to protect the
the children. victim from the immediate and imminent danger of VAWC or to
Ralph replied with a Petition with Urgent Motion to Lift TPO, and prevent such violence, which is about to recur.
contended that the issuance of the TPO is unconstitutional for being The grant of a TPO ex parte cannot, therefore, be challenged as violative
violative of the due process clause of the Constitution. of the right to due process.
Without awaiting for the resolution of his Comment on the petition and It is a constitutional commonplace that the ordinary requirements of
motion to lift TPO, Ralph filed with the CA a petition for certiorari with procedural due process must yield to the necessities of
prayer for the issuance of a writ of preliminary injunction and hold protecting vital public interests, among which is protection of
departure order assailing the TPO issued by the RTC. women and children from violence and threats to their personal
The CA issued a temporary restraining order to temporarily enjoin the parties safety and security.
Ralph filed an Urgent Motion for Issuance of a Writ of Preliminary Injunction It should be pointed out that when the TPO is issued ex parte, the court
with Manifestation, praying that the enforcement of all orders, decision to shall likewise order that notice be immediately given to the
be issued by the RTC and all the proceedings therein be restrained. respondent directing him to file an opposition within 5 days from
service.
The essence of due process is to be found in the reasonable
opportunity to be heard and submit any evidence one may have
in support of one’s
defense. “To be heard” does not only mean verbal arguments in court;
one may be heard also through pleadings. Where opportunity to be
heard, either through oral arguments or pleadings is accorded, there is
no denial of due process.
The act of congress to entrust the issuance of protection orders to courts
and barangay officials is in pursuance of authority to settle justiciable
controversies or disputes involving rights that are enforceable and
demandable before the courts of justice or the redress of wrongs for
violations of such rights.
It is within the court’s discretion, based on the petition and the affidavit
attached thereto, to determine that the violent acts against women
and their children for the issuance of a TPO have been committed.
It is a settled doctrine that there is GAD when there is a capricious and
whimsical exercise of judgment as is equivalent to lack of jurisdiction;
such as where the power is exercised in an arbitrary or despotic
manner by reason of passion or personal hostility; it must be so
patent and gross so as to amount to an evasion of positive duty or to
a virtual refusal to perform the duty enjoined or to act at all in
contemplation of law.
PEOPLE v. ROXAS (JP) her jewelries consisting of bracelets, earrings, necklace and watch,
August 17, 2010 | Peralta, J. | Due Process and her shoes (30,000-40,000

PETITIONER: People of the Philippines


RESPONDENTS: Venancio Roxas y Arguelles

SUMMARY: Agnes was maliciously misled by Roxas, wearing a PNP


vest, that she was driving against a one way street. Upon getting to ride
into the vehicle, Roxas then held Agnes at gun point and the latter was
then held against her will by co-accused Gungon. Ultimately, she was led
to Batangas where she would be salvaged. She was forced to drink a
drugged softdrink and she lost consciousness. Upon waking up, she was
bloody and had holes in her cheeks. After due investigation, the culprits
were captured. Roxas appealed and asked for the inhibition of the judge
of the case because he believes he was being deprived of due process
through the partiality of the judge, influenced by the presence of a justice
and the media. Since no substantial proof was showed, the SC affirmed
the trial court and Roxas was convicted of Kidnapping and Serious Illegal
Detention with Frustrated Murder, Carpanning and Theft.

DOCTRINE: 1. The fact that the trial judge opted to believe the
prosecution’s evidence rather than that of the defense is not a sign of
bias; bias and impartiality being breaches of due process of law.
Before concluding that there is indeed prejudice against an appellant,
substantial proof must be given and that this has indeed influenced the
court’s decision.

FACTS:
Agnes Guirindola (Agnes) was driving along Quezon City on her Nissan
Sentra when she was suddenly flagged down by a mena wearing a
PNP reflectorized vest who later on turned out to be defendant
Venancio Roxas (Roxas). She was told that she was driving against
a one-way street. Roxas then asked to ride inside Agnes’ car and
asked her to make a U-turn. Thinking Roxas asked for a bribe, she
gave him 50 pesos.
Roxas then poked her with a gun and asked to take control of the car.
Co-accused Gungon then entered the vehicle and then held Agnes,
pulling her to the back seat.
They stopped by a sari-sari store and Roxas offered Agnes a softdrink.
Agnes declined because of the presence of bubbling tablets inside it.
She was however forced to drink it at some point in the ride and was
even asked to take two additional tablets which she acted like she
did take but she actually spat out.
After Agnes saw from a signboard that she was in Batangas, she felt
dizzy and fell asleep. When she woke up, she found herself lying at
the back seat with her legs on the lap of Gungon. She noticed that
pesos) were already gone. She also lost her wallet and cash inside it The fact that the trial judge opted to believe the prosecution’s evidence
(4,000 pesos). She also noticed that there was a third man in the rather than that of the defense is not a sign of bias; bias being a
front seat. breach of due process. The trial was far from impartial.
She then asked to relieve herself and was escorted by Gungon at a
grassy area. When she was about to return to the car, she saw white
sparks at her right side and then she fell down. When she opened
her eyes, she saw Roxas walking towards the car with a gun in his
hand. She then lost consciousness.
When she regained consciousness, she was all alone. She followed a
“sparkling light” that led to her to a small house. When she was
ignored by the people outside because of the blood oozing from her
face and the holes in her cheeks, she went into the living room and
lied down on the sofa where she was discovered by certain
concerned people and carried to the Batangas Regional Hospital
where she was treated. Her parents immediately arrived who
immediately reported the incident to the NBI.
Gungon was arrested in Dabao City and after Gungon was found guilty,
Roxas was also apprehended after being at large for a while,
masking himself as worker under the Office of the Mayor in
Batangas. They were positively identified by Agnes in police line-ups.
Roxas used an alibi that he was fixing the antenna of his house and
didn’t leave for the whole night of the incident. Regardless, the court
a quo found Roxas guilty of Kidnapping and Serious Illegal Detention
with frustrated murder, carpanning and theft.
Roxas asked for a motion for reconsideration and asked for the inhibition
of the Honorable Judge Demetrio Macapagal, Sr. He argued that the
presence of then Justice Secretary Hernando Perez and the media
showed the court’s predisposition to convict him of the offense
charged (in other words he claimed that the court already pre-
decided him to guilty). He contended he was deprived of his due
process because Judge Demetrio has lost the cold neutrality of an
impartial judge required.
RTC denied the motions for reconsideration and inhibition of Judge
Demetrio. The records of this case were referred to the Court of
Appeals for intermediate review. The CA affirmed in toto the decision
of the court a quo. Hence, this appeal.

ISSUE/s:
WoN the court a quo erred in rendering the case despite the fact that
the presiding judge of that court has lost the cold neutrality of an
impartial judge thereby violating the right of the accused-appellant
to due process
– NO
WoN he is guilty of the crimes charged - YES

RULING: Roxas’ appeal is denied. The decision is affirmed.

RATIO:
There is no sufficient basis to show that the presence of Secretary
Hernando Perez and the media (their presence or the pervasive
publicity fo the issue) unduly influenced the court’s judgment.
He must first show substantial proof, not merely cast suspicions. There
must be a showing that adverse publicity indeed influenced the
court’s decision. The Court finds no such proof in the case at bar. He
was not deprived due process.
With regards with the crimes charged, Roxas was absolutely guilty
beyond reasonable doubt as per the testimony of Agnes which the
trial court upon proper observance, believed. They were guilty of
kindapping and serious illegal detention because they simulated
public authority and detained a female. They are guilty of frustrated
murder as qualified by treachery and evident premeditation because
of the gunshot wound sustained by the victim. They are also guilty of
carnapping for unlawfully taking Agnes’ car by means of force by
poking her with a gun with intent to gain. And they are guilty of theft
only, not robbery, because no qualifying circumstance of force was
proved.
ANG TIBAY v. CIR (HENRY) opposition for. Hence, this motion.
February 27, 1940 | Laurel, J. | Procedural Due Process
ISSUE/s:
PETITIONER: Ang Tibay, represented by Toribio Teodoro, et al. WoN NLU was denied due process by the CIR. – YES
RESPONDENTS: The Court of Industrial Relations, et al.
RULING: Motion for new trial granted, case is remanded to the CIR with
SUMMARY: Ang Tibay allegedly ran low of leather supplies and are instruction that it reopen the case, receive all such evidence as may be
unable to commit to the Philippine Army enough leather products. This relevant, and otherwise proceed in accordance with the requirements set
lead to the owner, Teodoro, to decide to lay off some of its employees. All forth.
being part of NLU, they filed against Teodoro with unfair labor practice as
ground, they were alleging that NWB was being unjustly favored as RATIO:
nobody from them was laid off. The CIR ruled in favor of Teodoro, Cir is a special court whose functions are stated in CA No. 103. It is more
however the NWB appealed to the SC, who affirmed the decision. There of an administrative board than a part of the integrated judicial
was then a motion for a new trial filed by NLU with new and solid system. It not only exercises judicial or quasi-judicial functions in the
evidence that made the SC grant their motion in the light of procedural determination of disputes between employers and employees but its
due process. functions are far more comprehensive and extensive, as far as
issues are concerned with employers and laborers, landlords and
DOCTRINE: The Court of Industrial Relations may be said to be free tenants, among others.
from the rigidity of certain procedural requirements but it does not mean Its duty is to prevent or arbitrate disputes which are submitted to the
that it can, in justifiable cases coming before it, entirely ignore or Secretary of Labor which are to be dealt by the Court for the sake of
disregard the fundamental and essential requirements of due process in public interest, which is possible through reconciliation of parties
trials and investigations of an administrative case. and/or inducing them to settle by amicable agreement.
SC had the occasion to point out that CIR is not narrowly constrained by
technical rules of procedure, and CA No. 103 requires it to act
FACTS:
according to justice and equity and substantial merits of the case,
Petitioner Ang Tibay, a leather company, experienced shortage of leather without regard to technicalities or legal forms and shall not be bound
soles on 1938, making it necessary for Toribio Teodoro (owner) to by any technical rules of legal evidence but may inform its mind in
temporarily lay off some of its employees (who in this case, such manner as it may deem just and equitable.
apparently are members of the National Labor Union Inc.). However, this does not entail that CIR is free from the rigidity of certain
The NLU claims that Ang Tibay is guilty of unfair labor practices because procedural requirements most specially the fundamental and
Teodoro is discriminating against the NLU and is unjustly favoring the essential requirements of due process in trials and investigations of
National Workers’ Brotherhood (company/employer union dominated an administrative character. There are cardinal primary rights which
by him) because nobody from NWB was included in those laid off. must be respected even in proceedings:
Aggrieved, the employees filed a case against Ang Tibay to the CIR, Right to a hearing
alleging it of unfair labor practice, that Teodoro’s claim of shortage of Tribunal must consider evidence presented
leather is entirely falseand unsupported by records of the BOC and Decision must have something to support itself
the Books of Accounts, and was only in fact a scheme to discharge Evidence must be substantial
members of the NLU from work. CIR ruled in favor of NLU. Decision must be based on evidence presented at a hearing
By appeal to the SC, the ruling was reversed after the SC found that Tribunal or body or any of its judges must act n its own
there was no merit to NLU’s contention. independent consideration of the law and facts of the
NLU then prays for the vacation of the judgment rendered by the SC with controversy, and not simply accept the views of a
the following claims, among others: subordinate
That Teodoro’s claim of shortage of leather soles in ANG TIBAY The board or body should, in all controversial questions, render
is entirely false and unsupported to its decision in such manner that the parties to the
Shortage of leather materials was but a scheme to systematically proceeding can know the various issue involved, and the
discharge all members of NLU from work reason for the decision rendered
NWB is dominated by Teodoro Since there was a failure to grasp the fundamental issue involved due to
NLU hence filed for a Motion for New Trial, with which Ang Tibay filed an failure to receive all relevant evidence, the motion for a new trial was
granted.
DAZON v. YAP (DANNAH) prosecutor filed a Motion to Withdraw Information with the RTC, in
January 15, 2010 | Del Castillo, J. | Procedural Due Process which the RTC granted.
(Jurisdiction) Hence, the present Petition for Review on Certiorari.

PETITIONER: Ma. Luisa Dazon


RESPONDENTS: Kenneth Yap and People of the Philippines

SUMMARY: Ma. Luisa purchased a condominium unit from Primetwon


Property, Inc. (Kiener Hills Mactan Condominium), of which Kenneth Yap
was the president. Primetown was unable to finish said condominium
which urged Malu to demand a refund of her payments. However,
Primetown failed to deliver the refund and Malu filed a criminal case
against Kenneth, pursuant to Section 23 of PD 957.

An Information was filed by the City Prosecutor in the RTC of Lapu Lapu.
Kenneth filed a petition for review with the DOJ. The DOJ ordered a
withdrawal of Information, because of his belief that it was the Housing
and Land Use Regulation Board which has jurisdiction over the case. The
Court however explained that HLURB does not have jurisdiction over
criminal cases.

DOCTRINE: Jurisdiction is conferred by law and determined by the


material averments in the complaint as well as the character of the relief
sought. The scope and limitation of the jurisdiction of [the HLURB] are
well-defined.

FACTS:
Kenneth Yap was the president of Primetown Property Group, Inc.
(Primetown), the developer of Kiener Hills Mactan Condominium.
Ma. Luisa Dazon (Malu) entered into a contract of with Primetown for
purchase of a condominium unit. Malu made a downpayment and
several installment payments amounting to P1,114,274.
Primetown, however, failed to finish the condominium project, and Malu
demanded a refund of her payments, pursuant to Section 23 of PD
No. 957 (1976) otherwise known as “The Subdivision and
Condominium Buyers’ Protective Decree”.
Primetown failed to refund Malu’s payments, which urged Malu to file a
criminal complaint with the Office of the City Prosecutor of Lapu-
Lapu against Kenneth as president of Primetown. This was for
violation of Section 23 in relation to Section 39 of PD 957.
Subsequently, after a finding of probable cause, Information was filed
with RTC of Lapu-Lapu docketed as Criminal Case No. 015331-L.
Kenneth, in connection with the resolution finding probable cause, filed a
petition for review with the Department of Justice (DOJ).
On June 14, 2002, DOJ rendered a Resolution ordering the trial
prosecutor to cause the withdrawal of the Information. Hence, the
Administrative agencies being tribunals of limited jurisdiction can only wield
ISSUE/s: such powers as are specifically granted to them by their enabling
WoN a regional trial court has jurisdiction over a criminal action arising statutes.
from violation of PD 957 – YES The grant of power to HLURB over imposition of fines clearly shows that
the power in relation to criminal liability is to be imposed upon
RULING: WHEREFORE, the petition is GRANTED. The assailed October 2,
conviction.
2002 and January 13, 2003 Orders of the Regional Trial Court of Lapu-Lapu Not having been specifically conferred with power to hear and decide
City, Branch 54, are REVERSED and SET ASIDE. The said Court is cases which are criminal in nature, as well as to impose penalties
DIRECTED to proceed with the arraignment of the respondent and to hear therefor, we find
the case with dispatch.

RATIO:
The DOJ Resolution, which ordered the withdrawal of the information
was based on the finding that the Housing and Land Use
Regulatory Board HLURB, and not the regular court, has
jurisdiction over the case.
Both the respondent and the OSG agree with Malu that the regular
courts and not the HLURB have jurisdiction over the criminal
aspects of PD 957.
The basis of the resolution was not that there was lack of probable
cause, but the finding that it is the HLURB that has jurisdiction over
the case.
Jurisdiction is conferred by law and determined by the material
averments in the complaint as well as the character of the relief
sought. The scope and limitation of the jurisdiction of the HLURB are
well-defined.
Its precursor, the National Housing Authority (NHA), was vested under PD
957 with exclusive jurisdiction to regulate the real estate and trade
business.
It has exclusive jurisdiction to hear and decide cases of the following
nature:
Unsound real estate business practices
Claims involving refund and any other claims filed by subdivision
lot or condominium unit buyer against the project owner,
developer, dealer, broker or salesman
Cases involving specific performance of contractual and
statutory obligations filed by the buyers of the subdivision lot
or condominium unit against the owner, developer, dealer,
broker or salesman.
It is a settled rule of statutory construction that the express mention of
one thing in the law means the exclusion of others not expressly
mentioned.
Noticeably, cases that are criminal in nature are not mentioned in the
enumeration quoted above. The primordial function of the HLURB,
after all, is the regulation of the real estate trade and business and
not the conviction and punishment of criminals.
that the HLURB has no jurisdiction over criminal actions arising from
violations of PD 957.
On the other hand, BP 129 states that RTCs “shall exercise exclusive
original jurisdiction in all criminal cases not within the exclusive
jurisdiction of any court, tribunal or body……”
Based on the provision, it is evident that it is the RTC which has
jurisdiction over criminal cases arising from violations of PD 957.
The Office of the City Prosecutor found probable cause for the filing of an
Information for the subject offense. The DOJ made no reversal of
such finding of probable cause. Instead, it directed the withdrawal of
the information on the erroneous premise that it is the HLURB which
has jurisdiction over the case.
However, as above- discussed, and contrary to the resolution of the
Secretary of Justice, it is not the HLURB but the RTC that has
jurisdiction to hear the said criminal action.
SHU v. DEE (IYA) complaint and were not required to file their answer or present
April 23, 2014 | Brion, J. | Due Process counter evidence.
Shu filed a motion for reconsideration, but was denied. Hence, this
petition.
PETITIONER: Ray Shu
RESPONDENTS: Jaime Dee, Enriqueto Magpantay, Ramon Miranda,
Larry Macillan, and Edwin So

SUMMARY: Shu is questioning two deeds of real estate mortgage, which he


claims that his signatures were forged by the respondents Dee, Magpanta,
and others. Shu, with the NBI, filed a complaint alleging the crime of forgery
and falsification of public documents. The respondents argue that they were
denied due process in the investigations by the NBI. The Supreme Court
ruled in favor of Shu, saying that there was no denial of due process as the
respondents were able to file a motion for reconsideration, giving them the
opportunity to be heard.

DOCTRINE: The essence of due process is simply the opportunity to be


heard. What the law prohibits is not he absence of previous notice but its
absolute absence and lack of opportunity to be heard.

FACTS:
Shu is the president of 3A Apparel Corporation.
Shu filed complained before the NBI charging Dee, Magpantay, et. al.
(hereinafter Dee) with falsification of two deeds of real estate
mortgage.
After investigation the NBI filed a complaint with the City Prosecutor charging
Dee with the crime of forgery and falsification of public documents as
supported by a report issued by their Questioned Documents Division.
Dees argue that they were denied their right to due process during the
NBI investigation because they were never required to submit the
standard sample signature of Shu in their record for comparison.
The city prosecutor dismissed the case for lack of merit, finding no
probable cause against the Dee. It found that undeniable similarities
in the sample signature presented by the Dee and the two deeds of
real estate mortgage.
The Secretary of Justice reversed the findings of the city prosecutor. It
ruled that the evidentiary value of the findings of the NBI must be
entitled full faith and credit i n the absence of proof of irregularity in
the performance of the experts’ duties. It also denied the MR filed.
On appeal, the CA annulled the resolution of the DOJ Secretary. The CA
held that Dee was denied due process in the proceedings before the
NBI and Secretary of Justice. Dee was not furnished a copy of the
ISSUE/s:
WON Dee, et. al. were denied due process - NO

RULING: GRANT the petition and REVERSE and SET ASIDE the decision
of the court of Appeals.

RATIO:
The essence of due process is simply the opportunity to be heard. What
the law prohibits is not he absence of previous notice but its absolute
absence and lack of opportunity to be heard.
Dee filed with the Secretary of Justice a motion for reconsideration,
therefore it cured any defect in due process as they were given
opportunity to question the case thrust upon them.
There is no denial of due process in the NBI proceedings as the findings
were only recommendatory. The investigation was still subject to the
prosecutor and Secretary of Justice’s decision.
FABELLA v. CA (ELIEL) Teachers filed a motion for reconsideration but was denied. Afterwards,
November 28, 1997 | Panganiban, J. | Procedural Due Process brought it up to the Supreme Court, declaring void the RTC’s order of
dismissal and reinstating the teachers action, pedning decision of
PETITIONER: Hon. Armand Fabella, et al. their case.
RESPONDENTS: Court of Appeals, et al. (Teachers of Mandaluyong
High School)

SUMMARY: Teachers of Mandalyuong High School engaged in walk-outs


and strikes to demand payment of 13th month differentials, clothing
allowances and passage of a debt -cap bill in Congress. Sec. Carino filed
administrative cases against them and ordered their preventive
suspension. The teachers filed an injunctive suit alleging that they were
denied due process to be heard, because the burden of proof was on
them and that the DECS investigating committee rendered a decision
ordering their dismissal was impartial. The RTC ruled in favor of the
teachers, and affirmed by the CA and the SC.

DOCTRINE: Due process of law requires notice and hearing. Hearing, on


the other hand, presupposes a competent and impartial tribunal. The
right to be heard and, ultimately, the right to due process of laws lose
meaning in the absence of an independent, competent and impartial
tribunal.

FACTS:
On September 17, 1990, then DECS Secretary Carino issued a return-to-
work oder to all public school teachers who had participated in walk-
outs and strikes on various dates during the period September 26,
1990 – October 18, 1990.
Secretary Carino filed administrative cases against the teachers of the
Mandaluyong High School. At the same time ordered them to be
placed under preventive suspension.
Counsel for the teachers objected to the procedure adopted by the
committee and demanded that he be furnished a copy of the
guidelines adopted by the committee for the investigation and
imposition of penalties.
The teachers filed an injunctive suit with the RTC, charging the
committee appointed by Secretary Carino with fraud and deceit and
praying that it be stopped from further investigating them and from
rendering any decision in the administrative case. DENIED.
The amended complaint for certiorari and mandamus, alleged that the
investigating committee was acting with grave abuse of discretion
because its guidelines for investigation place the burden of proof on
them by requiring them to prove their innocence instead of requiring
Secretary Carino and his staff to adduce evidence to prove the charges
against the teachers.
Meanwhile, the DECS investigating committee rendered a decision
finding the teachers guilty as charged and ordering their immediate
dismissal.
By agreement of the parties, the trial conference was set. However, Indeed, in any proceeding, the essence of procedural due process
Secretary Carino failed to appear in court. is embodied in the basic requirement of noteice and a real
The Solicitor General informed the trial court that Carino had ceased to opportunity to
be DECS Secretary and asked for his substitution. DENIED.
The hearing of the case was thereafter conducted ex parte with only the
teachers allowed to present their evidence. RTC rendered decision in
favor of teachers, and affirmed by the CA.
Hence this petition brought by DECS Secretary Carino.

ISSUE/s:
WoN the teachers were denied due process in the administrative
proceedings – YES
WoN Section 9 of RA 4670 was complied - NO

RULING: Petition denied. Assailed decision affirmed.

RATIO:
In administrative proceedings, due process has been recognized to
include the following: (1) the right to actual or constructive notice of
the institution of proceedings which may affect a respondent’s legal
rights; (2) a real opportunity to be heard personally or with the
assistance of counsel, to present witnesses and evidence in one’s
favor, and to defend one’s rights; (3) a tribunal vested with
competent jurisdiction and so constituted as to afford a person
charged administratively a reasonable guarantee of honesty as
well as impartiality; and (4) a finding by said tribunal which is
supported by substantial evidence submitted for consideration during
the hearing or contained in the records or made known to the parties
affected.
The legislature enacted a special law, RA 4670 known as the Magna
Carta for Public School Teachers, which specifically covers
administrative proceedings involving public school teachers.
Section 9 of said law expressly provides that the committee to
hear public school teachers’ administrative cases should be
composed of the school superintendent of the division as
chairman, a representative of the local or any existing provincial
or national teachers’ organization and a supervisor of the
division.
In the present case, the various committees formed by DECS to hear the
administrative charges against the teachers did not include “a
representative of the local or in its absence, any existing provincial or
national teachers’ organization” as required by Section 9 of RA 4670.
Accordingly, these committees were deemed to have no
competent jurisdiction.
The inclusion of a representative of a teachers’ organization in these
committee was indispensable to ensure an impartial tribunal. It was
this requirement that would have given substance and meaning to
the right to be heard.
be heard.
Mere membership of said teachers in their respective teachers’
organization does not ipso facto make them authorized
representatives of such organizations as contemplated by Section 9
of RA 4670.
Such right to designate cannot be usurped by the secretary of education
or the director of public schools or their underlings. In the instant
case, there is no dispute that none of the teachers appointed by the
DECS as members of its investigating committee was ever
designated or authorized by a teachers’ organization as its
representative in said committee.
Moreover, the suspension or dismissal meted on them is baseless.
RA 6975 (Department of Interiror and Local Government Act of
SUMMARY DISMISSAL BAORD v. TORCITA (JP) 1990).
April 6, 2000 | Gonzaga-Reyes, J. | Procedural Due Process

PETITIONER: Summary Dismissal Board and the Regional Appellate


Board of
PNP, Region 6, Iloilo City
RESPONDENTS: C/Insp. Lazaro Torcita

SUMMARY: Congressman Manuel Puey et al. filed twelve administrative


complaints against Major Torcita for conduct unbecoming of a police
officer, for allegedly abusing his authority and creating a commotion in
front of their hacienda after an unreasonable car chase. The Summary
Dismissal Board dismissed their complaints and believed the testimony of
Major Torcita more, that he only acted in his official duty. The board
however found him guilty of the light offense of Simple Irregularity in the
Performance of Duty for drinking. RTC declared null the aforementioned
decision for violating due process. CA affirned. SC likewise affirmed
saying that Torcita should have been given an opportunity to defend the
charge for which he was found guilty.

DOCTRINE: It is a requirement fo due process that the parties be


informed of how the litigation was decided with an explanation of the
factual and legal reasons that led to the conclusions of the Court.

FACTS:
After attending a birthday party, Congressman Manuel Puey and four
other passengers boarded a Mazda pick-up. While on the road to the
congressman’s hacienda, Hda. Aimee, they overtook a red Cortina
Ford. This Cortina Ford turns out to be driven by Major Lazaro
Torcita, with his aide, his wife, and two ladies as passengers.
Major Torcita signaled the Mazda pick-up to stop but the latter instead
accelerated. This resulted to a vehicular chase which ended up at
Hda. Aimee where Major Torcita was blocked by two guards at the
gate. Major Torcita, then sensing there were armed men around,
called for back-up and Cadiz police cars appeared on the scene.
Twelve administrative complaints filed by Congressman Manuel Puey,
Jesus Puey and Alex Edwin del Rosario were the subject of
administrative hearings before the Summary Dismissal Board of the
PNP. They allege that Torcita created a commotion in front of the
gates of the Hacienda. They allege that he was very drunk,
threatened people with a gun and urinated in front of them. They
charged him of violation of domicile and illegal search, in addition to
their main common charge of Conduct Unbecoming of a Police
Officer. The parties agreed to consolidate the cases into one “major
complaint” for “conduct unbecoming of a police officer” pursuant to
The Summary Dismissal Board gave more credit to the testimony of the offense for which he was eventually found guilty is not
C/Insp, Torcita that a vehicular incident almost took place due to proper observance of du reprocess.
reckless driving of the driver of the mazda pick-up because of the It is a requirement of due process that the parties be informed of how
suspicious acceleration of the Mazda pick-up after leaving Torcito’s the
car hit the shoulder of the road. None of the then-complainants also
presented evidence with what transpired with near the gate of the
compound. Only Torcita gave his testimony.
In his testimony he does not deny that he drank a shot of alcohol in the
social event. But he says it was not to the point of drunkenness. He
was also not in his official duty. He was in civilian clothes during the
party and it was outside his jurisdiction. He only made the road
chase because it was his duty as a police officer and a traffic
enforcer.
The Board dismissed all twelve complaints for Conduct Unbecoming of a
Police Officer but suspended him for twenty days for having
committed Simple Irregularity in the Performance of Duty based on
Circular No. 91-002 (having the odor or smell of alcohol while on
duty).
Torcita filed a petition for certiorari with the RTC of Iloilo questioning the
legality if the conviction for an offense for which he was not charged
violating his due process of law. RTC granted the petition and
annulled the dispositive portion of the Summary Dismissal Board’s
decision. CA affirmed. Hence, the appeal by public respondents.
ISSUE/s:
WoN respondent can be guilty of an offense for which he was not
charged, without violating due process of law - NO

RULING: CA is affirmed and the instant petition for review is dismissed.

RATIO:
The charge of conduct unbecoming of a police officer is laid down in RA.
6975. On the other hand, Simple Irregularity in the Performance of
Duty is a light offense defined in Memo Circular No. 91-002. Note: he
was not even on duty.
The SC cannot sustain the theory of the petitioners that the definition of
conduct unbecoming of a police officer is broad enough to include
simple irregularity in the performance of duty. No matter how light, an
offense which is not properly charged and tried cannot be
countenanced without violating the rudimentary requirements of due
process.
None of the 12 charges mentuioned the specific act of being drunk while in
the performance of duty. There were no indications at all in the summary
dismissal proceedings that Torcita was alsoe being charged with breach
of internal disciplinge consisting of taking alcoholic drinks while in the
performace of his duties. The omission is fatal to the validity of the
judgment.
Although he was given an opportunity to be heard on the multiple
other charges filed against him, the absence of specification of
litigation was decided with an explanation of the factual and legal
reasons that led to the conclusions of the Court.
The charge of breach of internal discipling by taking drinks should
have been substantiated by factual findings referring to this
particular offense.
JALOSJOS v. COMELEC (HENRY) 5 petitions were lodged before the COMELEC, praying for the denial of
June 18, 2013 | Perlas-Bernabe, J. | Procedural due process due course and/or cancellation of the petitioner’s CoC. Pending
resolution,
PETITIONER: Romeo G. Jalosjos
RESPONDENTS: The Commission on Elections, et al.

SUMMARY: Jalosjos was convicted by final judgment of 2 counts of


statutory rape and 6 counts of acts of lasciviousness, with corresponding
penalties. After serving sentence, he returned to Zamboanga and applied to
register as a voter but was denied. During the pendency of his petition for
inclusion, he filed his CoC as he had the intent to run for Mayor of
Zamboanga City. His petition to be included as a registered voter was
denied by the MTCC—this ruling was affirmed by the RTC as well. Petitions
were lodged before the COMELEC to cancel his CoC, which was granted in
a resolution of which he claims his right to procedural due process was
violated. Aggrieved, he appealed to the SC, who said that it is the
COMELEC’s duty to cancel CoCs of those who suffer perpetual special
disqualification by virtue of final judgment.

DOCTRINE: Procedural due process is not hampered when the


Constitutional Commissions, or any government body for that matter, are
acting within their duties as provided for by the Constitution, or by law.

FACTS:
The Court previously ruled on a case emntitled “People of the Philippines
v. Jalosjos”, convicting the petitioner by final judgment of 2 counts of
statutory rape and 6 counts of acts of lasciviousness. He was
consequently sentenced to suffer penalties of reclusion perpetua and
reclusion temporal for each count, respectively, which carried the
accessory penalty of perpetual absolute disqualification pursuant to
Art. 41 of the RPC.
April 30, 2007, PGMA issued an order commuting his prison term to 16
years, 3 months, and 3 days. After serving the same he was issued a
Certificate of Discharge of Prison on March 18, 2009.
Petitioner Jalosjos applied to register as a voter in Zamboanga City, but
was denied by the Acting City Election Officer of the Election
Registration Board (ERB), prompting him to file a Petition for
Inclusion in the Permanent List of Voters (Petition for Inclusion)
before the MTCC.
During the pendency of the resolution, he filed a CoC seeking to run as
mayor of Zamboanga City in the upcoming local elections (May 13,
2013), stating that he his eligible for the said office and is a
registered voter of Brgy. Tetuan, Zamboanga.
MTCC denied his Petition for Inclusion on account of his perpetual
absolute disqualification which in effect deprived him of the right to
vote in any election. This was affirmed by the RTC, which pursuant
to the Omnibus Election Code, was immediately final and executory.
COMELEC En Banc issued a resolution resolving to cancel and deny those suffering from perpetual special disqualification by virtuye of
due course the CoC filed by petitioner Jalosjos due to his perpetual final judgment”.
absolute disqualification as well as his failure to comply with the Applying these principles at bar, it is evident that COMELEC didn’t exercise
voter registration requirement. Hence, this petition.

ISSUE/s:
WoN COMELEC acted beyond its jurisdiction when it issued its
resolution, and in so doing, violated petitioner’s right to due process -
NO

RULING: Petition dismissed.

RATIO:
Article IX-C Sec 3 of the 1987 Constitution provided that the COMELEC
may sit en banc or in divisions and shall promulgate its rules of
procedure in order to expedide disposition of election cases,
including pre-proclamation controversies. All such election cases
shall be heard and decided in division, provided that motions for
reconsideration of decisions shall be decided by the Commission en
banc.
This provision requiring a motion for reconsideration before the en banc
is only applicable to cases where the COMELEC exercises quasi-
judicial power. No application in administrative functions of the
commission.
Villarosa v. COMELEC differentiated the two functions, that
administrative connotes or pertains to “administration, especially
management, as by managing or conducting, directing, or
superintending, the execution, application, or conduct of persons or
things” whereas a quasi-judicial function is a term which applies to
“the action, discretion, etc. of public administrative officers or bodies
who are required to investigate facts, or ascertain the existence of
facts, hold hearings, and draw conclusions form them, as a basis for
their official action and to exercise discretion of a judicial nature”.
In Jalosjos, Jr. v. Cardino, the court held that the COMELEC’s denial of
due course to and/or cancellation of a CoC in view of a candidate’s
disqualification to run for elective office based on a final conviction is
subsimed nder its mandate to enforce and administer all laws
relation got the conduct of elections. Consequently, in such
situations, it is the COMELEC’s duty to cancel the CoC,
notwithstanding the absence of any petition initiating a quasi-judicial
proceeding for the resolution of the same.
The court in this case even stated that “even without a petition x x x the
COMELEC is under a lergal duty to cancel the [CoC] of anyone
suffering from the accessory penalty of perpetual special
disqualification to run for public office by virtue f a final judgment of
conviction x x x The COMELEC will be grossly remiss in its
constitutional duty to “enforce and administer all laws” relating to the
conduct of elections if it does not bar from running for public office
its quasi-judicial functions when it issued the resolution. It merely
performed its duty to enforce and administer election laws in cancelling
petitioner’s CoC on basis of his perpetual absolute disqualification, the
fact of which had already been established by his final conviction.
Hence, COMELEC En Banc was exercising administrative functions,
dispensing the need for a motion for reconsideration of a division
ruling (which is only required in quasi-judicial proceedings).
There is then, no violation of procedural due process since the
COMELEC En Banc would be acting in a purely administrative
manner.
Brion, concurring: Justice Brion agrees with the ruling but has
reservations as far as the ground of perpetual absolute
disqualification cited by the COMELEC in exercise of its
administrative power as an independent ground for the cancellation
is concerned. He stated that this ground involves a question of fact
that requires the full application of due process and cannot, moto
proprio and in the exercise of administrative powers, be simply cited
as a ground for the cancellation of a CoC.
DOH v. Phil Pharmawealth (JP) corresponding reply. There is no indication when its reply would be
February 20, 2013 | Del Castillo, J. | Procedural Due Process submitted. Nor did it seek an extension of the 10-day period, which
had already expired 7 days ago.
PETITIONER: Deparment of Health, the Secretary of Health, Margarita
Galon
RESPONDENTS: Phil Pharmawealth Inc. (PPI)

SUMMARY: By virtue of Memo 171-C, the DOH Undersecretary called 24


accredited pharmaceutical companies include respondent to a meeting
where they were asked to file an explanation for the report alleging that
they are offering into the market products unfit for human consumption.
PPI, instead of filing an explanation, submitted a belated letter. Since the
letter was insufficient, PPI was suspended for two years. PPI alleges that
it was denied due process because it was not given the chance to be
heard. SC disagreed and dimissed the complaint of PPI. PPI was given
the chance to rebut before it was suspended throught the notice of the
meeting, and the explanation it was asked to submit.

DOCTRINE: The court has repeatedly stressed that parties who chose
not to avail themselves of the opportunity to answer charges against
them cannot complain of a denial of due process.

FACTS:
Administrative Order 66 provided that the two year accreditation period of
government suppliers of pharmaceutical producs may be suspended
or revoked after due deliberation, hearing, and notice by the DOH
Accreditation Committee Chairman. The power granted to the DOH
Accreditation Committee Chairman to suspend was granted by AO
10.
DOH issued Memorandum No. 171-C which provided for a list and
category of sanctions to be imposed on accredited government
suppliers of pharmaceutical products in case of adverse findings
regarding their products (e.g. substandard, fake, or misbranded) or
violations committed by them during their accreditation. PPI was
found to sell products which are not fit for human consumption.
By virtue of Memo 171-C, DOH, through former Undersecretary
Margarita Galon (Galon), issued Memorandum No. 209, inviting
representatives of 24 acccredited drug companies to a meeting.
During the meeting, Galon informed them that according to Bureau
of Food and Drugs (BFAD) there were violations or adverse findings
relative to these drug companies’ products (including the respondent
PPI). They were required to submit their written explanations on the
adverse findings.
Instead of submitting a written explanation, PPI belatedly sent a letter
addressed to Galon, informing her that PPI has referred the matter to
their lawyers with instructions that they should prepare for a
Undersecretary Galon found untenable PPI’s letter and suspended PPI
for two years pursuant to AO 10 and Memorandum No. 171-C.
PPI filed before the RTC Civil Case No. 68200 seeking to declare null
and void such suspension with prayer for damages and injunction
against DOH former Secretary Romualdez and DOH Undersecretary
Galon.
RTC dismissed the Civil Case to be one instituted against the state,
where the principle of state immunity is applicable. CA reversed and
remanded to the trial court the case for further proceedings because
it has a found a cause of action.

ISSUE/s:
WoN Civil Case NO. 68200 be dismissed for being a suit against the
state –
YES
Whether responents’ due process was violated for its suspension
without allegedly the benefit of notice and hearing - NO

RULING: Petition granted. Civil Case No. 68200 is dismissed.

RATIO:
As a general rule, a state may not be sued. Exception: express consent
and
implied consent of the state. Express consent: when a law so
provides.
Implied consent: when the state eneters into a contract or itself
commences litigation where it subjects itself to counterclaims.
DOH being an unincorporated agency of the government can validly
invoke the defense of immunity from suit because it has not
consented, either expressly or impliedly, to be sued.
The rule is that if judgment against such officials will require the state
itself to perform an affirmative act to satisfy the same, such as the
appropriation of the amount needed to pay the damages awarded
against them, the suit must be regarded as against the state.
Regardless of the merits of PPI’s case, this case deserved a dismissal
because of the state’s immunity from suit. The very foundation of
Civil Case No. 68200 has crumbled at this initial juncture.
It is undisputed that Undersecretary Galon called into a meeting the
accredited drug companies to submit their commend or
reactions to the Report on Violative Products within 10 days.
PPI, instead of submitting its explanation, submitted a letter
whicha aside from being late, did not mention when such
response to the adverse complaint would be forthcoming.
PPI was not denied due process. Prior to its suspension, PPI had
the chance to rebut, explain, or comment on the findings
contained in the report.
The court has repeatedly stressed that parties who chose not to
avail themselves of the opportunity to answer charges against
them cannot complain of a denial of due process.
GMA v. COMELEC (HENRY) WoN the new rule on aggregate airtime mentioned in Section 9(a) of
September 2, 2014 | Peralta, J. | Procedural Due Process Resolution No. 9615 is valid and did not violate the petitioner’s right
to due process despite COMELEC failing to conduct prior hearing
PETITIONER: GMA Network, Inc. before coming up with such Resolution – NO
RESPONDENTS: Commission on Elections
PETITIONER-INTERVENOR: Senator Alan Peter “Compañero” S.
Cayetano

SUMMARY: COMELEC issued a resolution implementing the Fair


Elections Act, mandating the airtime limitation of candidates on a per
station basis. However prior to the 2013 elections, they issued another
resolution modifying the previous one and changing the per station basis
to a total aggregate basis. Petitioner questioned such provision and filed
a petition to the SC. It has been ruled in this case that since COMELEC
failed to conduct a hearing prior to promulgating such resolution that
introduced a radical change in the process, the specific provision that
amended the previous rule is unconstitutional, hence void.

DOCTRINE: When an administrative rule is merely interpretative in


nature, its applicability needs nothing further than its bare issuance for it
gives no real consequence more than what the law itself has already
prescribed. When, upon the other hand, the administrative rule goes
beyond merely providing for the means that can facilitate or render least
cumbersome the implementation of the law but substantially adds to or
increases the burden of those governed, it behooves the agency to
accord at least to thse directly affected a chance to be heard, and
thereafter to be duly informed, before that new issuance is given the force
and effect of law.

FACTS:
COMELEC issued Resolutions last 2010, implementing and interpreting
RA 9006 (Fair Elections Act) Sec 6, that the airtime limitation of a
candidate is 120 minutes (television advertisement) and 180 minutes
(radio advertisement) per station.
For the May 2013 elections however, COMELEC issued Resolution No.
9615, changing the interpretation from a per station basis to a total
aggregate basis.
Petitioner, together with other owners/operators of radio and television
networks (ABS-CBN, ABC, MBC, NBN, and RMN) together with the
KBP sent letters to the COMELEC questioning the provisions of the
resolution.
The COMELEC held public hearings, then issued Resolution No. 9361,
amending the provisions of the Resolution in dispute, which the
petitioners still found as objectionable and oppressive, hence, the
present petitions.

ISSUE/s:
RULING: SC partially granted the petition. Section 9(a) of Resolution No.
9615 as amended by Resolution 9631 is declared unconstitutional.
Remaining provisions are upheld and remain in full force and effect.

RATIO:
The COMELEC promulgated Resolution No. 9615 on January 15, 2013
and then came up with a public hearing on January 31 to explain
what it had done, particularly on the aggregate-based airtime limits.
This circumstance also renders the new regulation, particularly on
the adoption of the aggregate-based airtime limit, questionable.
Being such a radical change, it is not enough that the COMELEC
publish or explain the rule after it is implemented or adopted.
CIR v. CA: When an administrative rule is merely interpretative in nature,
its applicability needs nothing further than its bare issuance for it
gives no real consequence more than what the law itself has already
prescribed. When, upon the other hand, the administrative rule goes
beyond merely providing for the means that can facilitate or render
least cumbersome the implementation of the law but substantially
adds to or increases the burden of those governed, it behooves the
agency to accord at least to thse directly affected a chance to be
heard, and thereafter to be duly informed, before that new issuance
is given the force and effect of law.
Hence, for failing to conduct prior hearing before coming up with
Resolution 9615, specifically in regard to the new rule on aggregate
airtime is declared defective and ineffectual.
Carpio, concurring: In addition to the ruling of the SC to strike down
Sec 9(a) of Resolution 9615, Justice Carpio mentioned Section 6.2
of the Fair Elections Act for similarly trenching on the freedoms of
speech and of expression of candidates and poliutical parties.
Brion, concurring: Justice Brion agrees with the ruling with much
stress, that such new rule has been substantively unreasonable or
that its procedures and processes are unduly harsh.
Leonen, concurring: Similarlly, with the rest of the concurring Justices,
J. Leonen emphasized that fundamental rights are very serious
matters. The core of their existence is not always threatened through
the crude brazen acts of tyrants. Rather, it can also be threatened by
policies that are well-intentioned but may not have the desired effect
in reality.
Mingson also assailed the declaration on the ground that its own mining
APO CEMENT CORP. v. MINGSON (DANNAH) claims (Yellow Eagle I to VII) overlapped with the subject mining
claims of APOCEMCO (Allied 1 and 2, Lapulapu 31 and 32).
November 12, 2014 | Perlas-Bernabe, J. | Procedural Due Process: Void
The DENR Office on March 1995 decreed an Order stating that portions
Decisions
of
PETITIONER: Apo Cement Corporation
RESPONDENTS: Mingson Mining Industries Corporation

SUMMARY: Apo Cement submitted a Mineral Production Sharing


Agreement (MPSA) proposal before the DENR because of lack of
development and productive use of some mineral properties found in an
area. The DENR declared said mining claims abandoned and open for
location to other interested parties. Luzvimin, the current holder, filed an
appeal. Mingson also filed an appeal, on the ground that APOCEMCO’s
mining claims (Allied 1 and 2, Lapulapu 31 and 32), overlapped with its
own mining claims (Yellow Eagle I to VII).

DENR, upon APOCEMCO’s MR, awarded to APOCEMCO said mining


claims upon the review and concurrence of the Mines and Geosciences
Bureau Region 7-Panel of Arbitrators (POA). The POA upheld DENR’s
order, which Mingson appealed because it did not follow procedural due
process—both sides were not heard. The CA also affirmed said decision
which is why APOCEMCO brought forth an appeal before the SC.

The SC affirmed the CA’s decision, stating that the POA’s decision is void
ab initio for not observing due process.

DOCTRINE: The cardinal precept is that where there is a violation of


basic constitutional rights, courts are ousted from their jurisdiction. The
violation of a party’s right to due process raises a serious jurisdictional
issue, which cannot be glossed over or disregarded at will. Where the
denial of the fundamental right of due process is apparent, a
decision rendered in disregard of that right is void for lack of
jurisdiction.

FACTS:
Because of the supposed failure of the old locators to develop and put to
productive use the mineral properties found in the area, Apo Cement
Corp (APOCEMCO) submitted a Mineral Production Sharing
Agreement (MPSA) proposal before the DENR, essentially seeking
to take over their current holder, Luvimin Cebu Mining Corp
(LUVIMIN).
The DENR on August 1992 and March 1993 declared the mining claims
abandoned and open for location to other interested parties,
prompting LUVIMIN to file an appeal.
the subject mining claims be awarded to Mingson, considering that It has been established that the POA proceeded to resolve the present
said claims have encroached its Yellow Eagle claims. mining dispute without affording either party any fair and reasonable
Upon APOCEMCO’s MR, however, the DENR Regional Office’s Legal opportunity to be heard in violation of the aforementioned provisions
Division issued a Resolution instead awarding subject mining claims
of DENR DAO 95-23.
to
APOCEMCO.
In an Order dated September 1995, the DENR Regional Director
affirmed the resolution, but subject to the review and concurrence of
the Mines and Geosciences Bureau Region 7-Panel of
Arbitrators (POA). The POA has been mandated to resolve
disputes involing rights to mining areas.
In a Decision dated May 1996, the POA upheld the September 1995
Order without, however, requiring the parties to file any pleading or
setting the matter for hearing.
Aggrieved, Mingson appealed the POA’s Decision before the DENR
MAB, averring that said Decision was not supported by facts and the
evidence on record, and that it was arbitrary and issued with grave
abuse of discretion.
The DENR MAB on July 2007 granted Mingson’s appeal and reversed
and set aside the POA’s Decision. It found that the POA merely
conducted a review of the case and Mingson, in particular, was not
given an opportunity to be heard, which is repugnant to due process.
Dissatisfied, APEMCO elevated the matter of the CA, which dismissed
said appeal and sustained the DENR MAB’s finding. It also filed an
MR which was denied, hence this petition.

ISSUE/s:
WoN the POA observed due process – NO

RULING: The petition is denied. The Decision of the CA is hereby affirmed.


So ordered.

RATIO:
The Implementing Rules of the Philippine Mining Act of 1995 clearly
require that the parties involved in mining disputes be given the
opportunity to be heard.
These rules—which were already in effect during the time the dispute
between the parties arose—flesh out the core requirement of due
process.
The cardinal precept is that where there is a violation of basic
constitutional rights, courts are ousted from their jurisdiction. The
violation of a party’s right to due process raises a serious
jurisdictional issue, which cannot be glossed over or disregarded at
will. Where the denial of the fundamental right of due process is
apparent, a decision rendered in disregard of that right is void
for lack of jurisdiction.
Thus, as correctly ruled by the DENR MAB and later affirmed by the CA,
Mingson’s due process rights were violated, thereby rendering the
POA’s Decision null and void.
However, it must be stressed that given the lack of any formal procedure
on appeals at that time, the DENR MAB cannot be faulted for
considering the letter and the issues raised therein as part of
Mingson’s appeal.
The DENR MAB is not a court of law but an administrative body; hence,
it is not bound by strict rules of procedure and evidence, and is
allowed to use all reasonable means to ascertain the facts of each
case speedily and objectively without resort to technical rules, as in
this case.
Besides, an apparent lack of due process may be raised by a party at
any time since due process is a jurisdictional requirsite that all
tribunals, whether administrative or judicial, are duty-bound to
observe.
In Salva v. Valle, the Court pronounced that “[a] decision rendered
without due process is void ab initio and may be attacked at anytime
directly or collaterally by means of a separate action, or by resisting
such decision in any action or proceeding where it is invoked.”
upheld the validity of the Order of Arrest.
GOVERNMENT OF HONG KONG v. OLALIA, JR. (IYA) The Government of HK Special Administrative Region filed with the RTC
April 19, 2007 | Sandoval-Gutierrez, J. | Procedural Due Process a petition for the extradition of Munoz. Munoz filed a petition for bail
which was opposed by Gov of HK.
RTC judge denied the petition for bail holding that there is no PH law
PETITIONER: Government of Hong Kong Special Administrative Region granting bail in extradition cases and that Munoz is considered a
RESPONDENTS: Hon. Felixbierto Olalia, Jr. and Juan Antonio Munoz high “flight risk”.
Note: petitioners are represented by the PH DOJ during the trial Munoz filed a MR, (the previous judge inhibited himself therefore was
assigned to a different judge) which was granted allowing him to post
SUMMARY: Munoz is facing criminal charges with the Government of bail.
Hong Kong. His arrest is requested by HK through the PH DOJ. When a Gov of HK filed an urgent motion to vacate the order allowing to post bail
petition for extradition was filed before the RTC, Munoz applied for bail. but was denied.
Munoz was granted bail by the RTC. HK filed a motion to dismiss the Hence, this instant petition.
grant but was denied. HK then raised the issue to the SC.
ISSUE/s:
The SC overturned its previous decision not allowing bail. Due to WON the RTC committed grave abuse of discretion amounting to lack or
international treaties and agreements, it is bound to give importance to excess of jurisdiction in admitting private respondent to bail - NO
the rights of the individual as extradition may entail a deprivation of
liberty. It set a standard of “clear and convincing evidence” for the courts RULING: The case is remanded to the trial court for further
to follow in granting bail for extradition cases. determination.

DOCTRINE: While extradition is not a criminal proceeding, it entails


RATIO:
deprivation of liberty and the means employed to attain the purpose of
Gov of HK maintains that nothing in the Constitution or statutory law
extradition is also “the machinery of criminal law”.
providing that a potential extraditee has a right to bail, the right being
The standard of due process in the granting of bail for extradition cases, limited solely to criminal proceedings.
Munoz claims that the right to bail is guaranteed under the Bill of Rights,
will not be the same as that of criminal proceedings. The quantum of
and it extends to prospective extraditees because extradition is a
evidence needed would be “clear and convincing evidence”
harsh process resulting in a prolonged deprivation of one’s liberty.
Previous jurisprudence (Gov of USA v Hon. Guillermo Purganan and
Mark Jimenez) held that the constitutional provision on bail does not
FACTS: apply to extradition proceedings.
Munoz was charged before the Hong Kong Court with 3 counts of the The SC however cannot ignore the following trend of international law
offense of “accepting an advantage was agent” in violation of the giving primacy on the worth of the individual person and the sanctity
Prevention of Bribery Ordinance of HK. of human rights. It is also the duty of the SC to balance the rights of
He also faced 7 counts of the offense of conspiracy to defaud penalized the individual under the Constitution and the law on extradition.
by the common law of HK. As a signatory of the UN Declaration on Human Rights, we are bound to
Warrants of arrest were issued against him. The DOJ after receiving from adhere to it as it is now recognized as customarily biding upon the
the HK DOJ a request for provisional arrest of Munoz forwarded it to members of the international community.
the NBI. The NBI in turn filed with the RTC an application for the Due to the international treaties, a reexamination of the previous ruling of
provisional arrest of Munoz. this court not grating bail to extradition proceedings is in order.
The RTC issued the Order and on the same day the NBI arrested and The PH has allowed bail on administrative cases involving deportation
detained him. and quarantine. There would be no justification why this should not
Munoz filed with the CA a petition questioning the validity of the Order of be allowed din extradition cases.
Arrest. To which the CA declared as void. While extradition is not a criminal proceeding, it entails deprivation of
The DOJ then field with the SC a petition for review on certiorari praying liberty and the means employed to attain the purpose of extradition is
that the decision of the CA be reversed. The SC granted the petition also “the machinery of criminal law”.
and
Records show that Munoz was arrested and remained incarcerated for 2
over 2 years without having been convicted of any crime. Such an
extended period
of detention is a serious deprivation of his fundamental right to liberty.
The standard of due process in the granting of bail for extradition cases,
will not be the same as that of criminal proceedings. The quantum of
evidence needed would be “clear and convincing evidence”
In the present case, there is no showing that Munoz is a flight risk. The
case is therefore remanded to the trial court to determine whether
Munoz may be granted bail on the basis of “clear and convincing
evidence.”
RCBC responded and subsequently, an Arbitration Tribunal was
RCBC v. BDO (ELIEL) constituted. Mr. Neil Kaplan was nominated by RCBC; Justice
Santiao M. Kapunan was
December 10, 2012 | Villaarama, Jr., J. | Procedural Due Process

PETITIONER: RCBC Capital Corporation, Inc.


RESPONDENTS: Banco De Oro Unibank, Inc.

SUMMARY: RCBC and BDO (EPCIB) entered into a Share Purchase


Agreement for the sale of subject shares. After the execution of sale, RCBC
claimed that there was an overpayment of the subject shares, which
prompted them to settle the dispute with the ICC- ICA. In order that the
arbitration is continued, the ICC-ICA required both parties to pay advance on
costs where RCBC complied but BDO refused to pay. This refusal became a
ground for the tribunal to give the first partial award to RCBC. BDO claimed
that they have been violated their due process because they were not given
the opportunity to be heard by an impartial tribunal because of Chairman
Barker’s partiality towards RCBC. Despite this, RCBC was awarded the
second and final awards. Hence this petition. The SC ruled that there was
evident partiality in the arbitration tribunal because Chariman Barker
exhibited strong inclination to grant such relief to RCBC.

DOCTRINE: When a claim of arbitrator’s evident partiality is made, “the


court must ascertain from such record as is available whether the
arbitrators’ conduct was so biased and prejudiced as to destroy
fundamental fairness.”

FACTS:
RCBC entered into a Share Purchase Agreement (SPA) with Equitable-
PCI Bank, Inc. (EPCIB), George L. Go and the individual
shareholders of Bank Inc. (Bankard) for the sale to RCBC of
226,460,000 shaes (Subject Shares) of Bankard, constituting 67% of
the latter’s capital stock.
The dispute between RCBC and EPCIB arose sometime in May 2003
when RCBC informed EPCIB and the other selling shareholders of
an overpayment of the subject shares, claiming there was an
overstatement of valuation of accounts amounting to P478 million
and that the seelers violated their warranty under Section 5(g) of the
SPA.
As no settlement was reached, RCBC commenced arbitration
proceedings with the ICC-ICA in accordance with Sec 10 of the SPA.
Alleging that EPCIB contravened their generally accepted principle,
which were far from fair and accurate.
EPCIB denied RCBC’s allegations contending that RCBC’s claim is one
for overpayment or price reduction under Sec 5(h) of the SPA, which
is already time-barred, the remedy rescission is unavailable.
Thereafter a counterclaim was initiated.
nominated by EPCIB; and Sir Ian Barker was appointed by the ICC-ICA as
Chairman.
The ICC-ICA informed the parties that they are required to pay US$350,000 as
advance on costs pursuant to Art 30(3) of the ICC Rules of Arbitration.
Respondents filed an Application for Separate Advances on Costs under Art
30(2) of the ICC Rules, praying that the ICC fix separate advnces on the cost
of the parties’ respective claims and counterclaims, instead of directing them
to share equally on the advance cost of RCBC’s claim.
ICC-ICA informed EPCIB that their application for separate advances on costs
was premature pending the execution of the Terms of Reference (TOR).
EPCIB then refused to pay their share in the advance cost fixed.
The ICC-ICA issued Procedural Order No. 3 for the adjournment of the
substantive hearings and granting EPCIB a 2 month extention within which to
submit their brief of evidence and witnesses.
RCBC objected to the cancellation of hearings, pointing out that EPCIB have
been given ample time and opportunity to submit eveidence and prepare for
hearings. Praying that ECPIB then be declared in default.
Upon increase of payment of costs, EPCIB again refused to pay. ICC-ICA then
declared them in default. Meanwhile BDO merged with EPCIB assuming all
liabilities under the SPA.
The Arbitration Tribunal rendered a Partial award in favor of RCBC. This
prompted them to file with the RTC a motion to confirm, while EPCIB (BDO)
filed motion to vacate. RTC confirmed the partial award. MR of EPCIB was
denied. Hence, this first petition being raised to the SC.
The Arbitration Tribunal set a timetable for the filing of submission by the parties
on whether it should issue a Second Partial Award in respect of EPCIB’s
refusal to pay an advance on costs to the ICC-ICA.
RCBC filed application for reimbursement of advance on costs paid. They
invoked the plain terms of Art 30(2) and (3).
EPCIB opposed stating that the Arbitration Tribunal has lost its objectivity in an
unnecessary litigation over the payment of EPCIB’s share in the advance
costs. They state that they have been violated due process and to be heard
by an impartial tribual.
RCBC refuted EPCIB’s allegation of partiality on the part of Chairman Barker and
reiterated the prayer in its application for reimbursement of advance on costs
paid to the ICC-ICA. RCBC contended that based on Mr. Secomb’s article,
whether the “contractual” or “provisional measures” approach is applied, the
Arbitration Tribual is vested with jurisdiction and authrotiy to render an award
with respect to said reimbursement of advances cost paid by RCBC.
The Arbitration Tribual, however, rendered the Second Partial Award. EPCIB filed
a motion to vacate, while RCBC filed a motion to confirm.
EPCIB takes issue with Chariman Barker’s interpretation of RCBC’s letter as an
application for a partial award for reimbursement of the substituted
payments. Such conduct of Chairman Barker is prejudicial and proves his
evident partiality in favor of RCBC.
After further exchange of pleadings, the RTC confirmed the Second phrasing of the scope of the agreed issues in the TOR is
Partial Award and denied EPCIB’s motion to vacate. EPCIB filed an broad enough to accommodate a
MR and for the voluntary inhibition of the Presideing Judge, which
were both denied. Then was brought up to the CA, assailing RTC’s
decision.
Meanwhile, the Arbitration Tribual issued the Final award. BDO filed in
the RTC a motion to vacate final award ad cautelam. While, RCBC
field a motion to confirm. BDO answered with a motion to dismiss
because of the initial motion to vacate.
RCBC filed an Omnibus Motion praying for the dismissal of BDO’s
petition to vacate final award. BDO answered, and the omnibus
motion of RCBC was denied. RCBC filed MR but was also denied.
However, the motion to confirm was granted to RCBC by Branch
148.
George Go, in his personal capacity and as attorney-in-fact of the other listed
shareholders of Bankard, Inc. in the SPA filed a petition in the CA
seeking to set aside the final award. CA ruled in their favor, RCBC’s MR
denied.
RCBC immediately filed an urgent motion for issuance of writ of execution.
BDO then filed with the CA seeking the reversal of the awards and any
writ of execution issued by the Arbirtration Tribunal.
Br. 148 continued the execution proceedings. BDO filed a very urgent
motion to lift levy. However, the CA denied BDO’s application for
preliminary injunction. Hence this petition.

ISSUE/s:
11. WoN there is evident partiality as ground of vacating an arbitral award– YES

RULING: Both petitons denied. CA decision affirmed.

RATIO:
Evident partiality in its common definition thus implies “the
existence of signs and indications that must lead to an
identification or inference” of partiality.
Evident partiality “continues to be the subject of somewhat conflicting
and inconsisten judicial interpretation when an arbitrator’s failure to
disclose prior dealings is at issue.
Arbitrators should not automatically be disqualified from an arbitration
proceeding because of a business relationship where both aprties
are aware of the relationship in advance, or where the parties are
unaware of the circumstatnces but the relationship is trivial.
However, in the vent thtat the arbitrator has a “substantial interest” in
the transaction at hand, such information must be disclosed.
The case at bar does not present a non-disclosure issue but conduct
allegedly showing an arbitrator’s partiality to one of the parties.
The RTC held that BDO failed to substantiate these allegations. On
appeal, the CA likewise found that the Arbitration Tribunal do
not go beyond the submission of the parties because the
finding on the liability and the repercussion of BDO’s failure to share in
the advances on costs.
Instead, the Court adopts the reasonable impression of partiality standard, which
requires a showing that a reasonable person would have to conclude that an
arbitrator was partial to the other party to the arbitration.
When a claim of arbitrator’s evident partiality is made, “the court must
ascertain from such record as is available whether the arbitrators’
conduct was so biased and prejudiced as to destroy fundamental
fairness.”
Chairman Barker exhibited strong inclination to grant such relief to RCBC
notwithstanding his categorical ruling that the Arbitration Tribunal “has
no power under the ICC Rules to order EPCIB to pay the advance on
costs sought by the ICC or to give the Claimant any relief against the
Respondent’s refusal to pay.”
By furnishing the parties with a copy of Mr. Secomb’s article, Chariman Barker
practically armed RCBC with supporting legal arguments under the
“contractual approached”.
NON v. JUDGE DAMES II (JP) physically coercing students not to attend their classes, thereby
May 20, 1990 | Cortes, J. | Procedural Due Process distrupting the scheduled classes and depriving a great majority of
students of their right to be present in their
PETITIONER: Ariel Non, Rex Magana and other students of Mabini
Colleges
RESPONDENTS: Hon. Sancho Dames II, Presiding Judge of 5 th RTC,
Daet and Mabini Colleges Inc.

SUMMARY: Petitioners are student in Mabini College where they were


refused re-enrollment for participating and leading student protests and
coercing other students to not go to class in the previous semester. The
RTC relied on the Alcuaz doctrine which says that the contract betweent
the student and college lasts only for one semester and the college is not
obliged to take the student back for reenrollment. The SC ruled that due
process was denied to the petitioners when the school made it appear
that they were refused readmission due to failing grades when in fact it
was due to their possible breaches in discipline.

DOCTRINE: 1. The right of an institution of higher learning to set


academic standards cannot be utilized to discriminate against students
who exercise their constitutional rights to free speech and assembly, for
otherwise there will be a violation of their right to equal protection.
The imposition of disciplinary sanctions requires observance of
procedural due process. There are minimum standards which must be
met to satisfy the demands of procedural due process. See Ration No. 4

FACTS:
Petitioners, students in private respondent Mabini Colleges inc. in Daet,
Camarines Norte, were not allowed to re-enroll by the school for
leading or participating in student mass actions against the school in
the preceding semester. The subject of the protests is not however
made clear in the pleadings.
Petitioner students filed a petition in the court a quo seeking their
readmission or re-enrollment to the school, but the trial court
dismissed the petition. They filed for an MR but it was likewise
denied.
The RTC explained that the applicable ruling is the doctrine in Alcuaz, et al.
v. Philippine School of Business Administration, et al. to the effect that a
college student, once admitted by the school, is considered enrolled only
for one semester and hence, may be refused readmission after the
semester is over as the contract between the student and the school is
deemed terminated.
It was also discussed in the court a quo that petitioner’s claim of lack of due
process cannot prosper in view of their failure to specifically deny
respondent’s affirmative defenses that “they were given all chances to
air their grievances during which they were represented by Atty. Lapak,
and that on the day of resumption of classes at Mabini College,
petitioners conitued thei rally picketing without any renewal permit,
classes. The respondents excused that of the 13 students who was refused
It being a mere privilege and not a legal right for a student to be enrolled readmission, 8 failed and that their non-readmission was justified.
or re-enrolled, respondent Mabini College is free to admit or not The right of an institution of higher learning to set academic standards
admit the petitioners in view of the academic freedom enjoyed by the cannot
school in accordnaces with the SC rulings in the cases of Garcia v.
Faculty and Tangonon v. Pano
The CA resolved to certify the case back to the SC considering that only
pure questions of law were raised.

ISSUE/s:
WoN there were a valid grounds for the refusal to readmit the erring
student
– NO
WoN there was a denial of due process when the students were
excluded from respondent school for their possible breaches of
discipline - YES
RULING: Petition granted. Respondent Mabini College is ordered to readmit
the petitioners.

RATIO:
This is a case that focuses on the right to speech and assembly as
exercised by students vis-à-vis the right of school officials to
discipline them.
Students should not be denied their constitutional and staturory right to
education and right to free speech and peaceful assembly but these
rights of students are not without limit.
Tinker v. Des Moines Community School: “but conduct by the student
which for any reason materially distrupts classwork or involves
substantial disorder or invasion of the rights of others is of course not
immunized by the constitutional guarantee of freedom of speech.”
But as stated in Guzman, the imposition of disciplinary sanctions
requires observance of procedural due process. There are minimum
standards which must be met to satisfy the demands of procedural
due process; and these are that (1) the students must be informed in
writing of the nature andcause of any accusation against them; (2)
they shall have the right to answer the charges against them, with
the assistance of counsel, if desired; (3) they shall be informed of the
evidence against them; (4) they shall have the right to adduce
evidence in their own behalf; and (5) the evidence must be duly
considered by the investigating committee or official designated by
the school authorities to hear and decide the case. Moreover, the
penalty imposed must be proportionate to the offense committed or
else an element of arbitrariness intrudes.
The termination of contrac theory of the respondent school does not
even find support in the Manual of Regulations for Private Schools.
On the contrary, the manual recognizes the right of the student to be
enrolled in his course for the entire period he is expected to complete
it.
be utilized to discriminate against students who exercise their constitutional
rights to free speech and assembly, for otherwise there will be a violation of
their right to equal protection.
Certainly, excluding students because of failing grades when cause for
the action taken against them was in fact undeniably related to
another reason, which is the possible breaches of discipline, not only
is a denial of due process but also violates the basic tenets of fair
play.
Petitioners were already effectively excluded from respondent school for
4 semesters. They have already been sufficiently penalized and
should not be subject to any more disciplinary proceedings. The
matter has therefore become moot and academic.
ADMU v. CAPULONG (HENRY) Respondent students through counsel requested investigation against them
May 27, 1993 | Romero, J. | Procedural Due Process be

PETITIONER: Ateneo the Manila University, Father Joaquin Bernas, et al.


RESPONDENTS: Hon. Ignacio Capulong (Presiding Judge of RTC Makati
Br.
134), et al.

SUMMARY: Lennie Villa, a neophyte of Aquila Legis Fraternity, died after


undergoing the initiation rights of the said fraternity. Another fresman student
was hospitalized due to serious injuries. The Dean of ALS created an
Investigating Committee to take over, who eventually ruled on the dismissal
of the respondent students who were the involved in the incident. They filed a
petition to the RTC who ruled in their favor, that enjoined ALS to dismiss
them. However, upon appeal to the SC, the ruling was reversed, contrary to
the defense of the respondent students (based from Ang Tibay case) that
their right to due process was violated. No rights were violated by ALS, as all
requirements of the Guzman case were met.

DOCTRINE: An administrative proceeding conducted to investigate the


students’ participation in a hazing activity need not be clothed with the
attributes of a judicial proceeding.

FACTS:
Aquila Legis, a fraternity organized in the Ateneo Law School, held its
initiation rights on Febrary 8-10, 1991 for students interested in
joining its ranks. Leonardo “Lennie Villa (freshman) died of serious
physical injuries at the Chinese General Hospital on Feb 10, 1991.
Another freshman victim Bienvenido Marquez was likewise hospitalized
at Capitol Medical Center for acute renal failure occasioned by
serious physical injuries inflicted upon him on the same occasion.
Dean del Castillo created a Joint Administration-Faculty-Student
Investigating Committee which was tasked to investigate and submt
a report within 72 hours on the circumstances surrounding the death
of Lennie through a notice on Feb 1991. The same notice asked for
respondents to submit written statements within 24 hours.
Respondnet students failed to file a reply, and were placed on preventive
suspension for the meantime. But eventually through a notice, the
Committee after receiving the written statements found a prima facie
case against respondent students for violation of Rule 3 of the Law
School Catalogue entitled “Discipline”.
February 20, 1991, petitioner Dean created a Disciplinary Board
composed of petitioners Judge Kapunan, J. Escolin, Atty. Herras,
Fiscal Albar, and Atty. Casis, to hear the charges against respondent
students.
A letter was sent the same day to the respondent students wo were
informed of their violation of Rule 3 of the Rules on Discipline
contained in the Law School Catalogue.
temporarily suspended pending request for copies of evidence. RULING: SC granted the petition.
The respondent students were informed that:
Proceedings will be summary in nature, in accordance to the
rules laid down in the case of Guzman v. NU
Petitioners have no right to cross-examnie neophytes
Hazing not defined in the school catalogue shall be defined in
accordance to the prosed bill of Sen. Lina (Senate Bill No.
3815)
Board will take into consideration the degree of participation of
the petitioners in the alleged hazing incident in imposing
penalty
Decision of the Board shall be appealable to the University
President (F. Bernas)
March 1991, the Board found the reposndent students guilty. They acted
as master auxiliaries (auxies) during the initiation rights of Aquila
Legis, and exercised “auxies privilege” which allows them to
participate in physical hazing. Imposition of penalty was left to the
University Administration, due to the unanimity among mebers of the
Board on the penalty of dismissal.
Fr. Bernas imposed the penalty of dismissal on all respondent students,
but excluded respondents Abas and Mendoza inasmuch as at the
time the latter resolution was promulgated, neither had as yet
submitted their case to the Board.
The respondent students filed with the RTC a petition. Said trial court
issued a TRO enjoining petitioners from dismissing the students and
stopping the former from conducting hearings relative to the incident.
Upon expiration, Dean del Castillo created a special board to
investigate charges of hazing against Abas and Mendoza.
Respondent students filed a Supplemental Petition of certiorari,
prohibition and mandamus with a prayer for a TRO to include the
members of the special board as additional respondents to the
original petition, which the petitioners moved to strike out as the
creation of such special board was totally not related to the original
petition which alleged lack of due process in the investigations by the
Disciplinary Board against respondfent students.
RTC Judge ordered petitioners to reinstate respondent students, and
ordered ALS to conduct special examinations in lieu of the final
exams which the students were allegedly not allowed to take.
May 1991 the special board investigating Abas and Mendoza concluded
their investigation and imposed the penalty of dismissal to the two.
Respondent RTC judge issued a TRO enjoining petitioners to remove
respondent students from their roll of students. Hence, this special
civil action of certiorari under rule 65 with the prayer of issuance of a
TRO questioning the order of respondent judge.

ISSUE/s:
WoN the respondent studnets’ rights to due process were violated when
Fr. Bernas imposed the penalty of dismissal - NO
RATIO:
Respondent studnets’ rights in a school disciplinary proceeding (per Guzman
v. NU, Alcuaz v. PSBA) have been meticulously respected by petitioners
in the various investigative proceedings held before they were expelled.
The respondents were using in their argument the Ang Tibay case, not
the Guzman case, which is applicable in the case at bar, which
specifically deals with the minimums tandard to be satisfied in the
imposition of disciplinary sanctions in academic institutions:
0 Students must be informed in writing of the nature and cause
of any accusation against them
1 That they shall have the right to answer the charges against
them with the assistance of counsel, if desired
2 They shall be informed of the evidence against them
3 They shall have the right to adduce evidence in their own behalf
4 Evidence must be duly considered by the investigating
committee or official designated by the school authorities to
hear and decide the case
The above requirements were met in this case. The respondents cannot
argue that their right to procedural due process was denied because
they are not allowed to cross examine the neophytes. An
administrative proceeding conducted to investigate the students’
participation in a hazing activity need not be clothed with the
attributes of a judicial proceeding.
They identified senior members of the frat, which included Kim Go, then a
GO v. SAN JUAN DE LETRAN (DANNAH) senior.
Gerardo Manipol, Letran’s security officer, prepared a report. In it he stated
October 10, 2012 |, J. | Procedural Due Process (Jurisdiction)
that the frat had violated its covenant with Letran by recruiting members
from
PETITIONER: Sps. Eugene and Angelita Go, Minor Kim Go
RESPONDENTS: Colegio de San Juan de Letran, et al.

SUMMARY: An investigation regarding alleged high school students


joining the frat Tau Gamma Phi was conducted by Letran. A medical
examination on a couple of students revealed that they underwent hazing
rites. These students pointed to senior members of the fraternity, one of
which was Kim Go.

Letran informed Kim’s parents regarding the matter and invited them to a
couple of conferences. Kim’s parents did not want to believe Letran. The
supposed sanction was dismissal from school but Letran’s President
reduced it to suspension for seniors, so that they could still graduate.
Upon letting the parents of involved students sign a pro forma agreement,
Sps. Go declined and instead filed a case with the RTC against them for
allegedly not observing due process in Kim’s dismissal.

The RTC ruled in favor of the spouses and awarded damages to them.
The CA reversed the ruling and further stated that there was no basis for
the awarding of damages. The SC ruled that due process was observed,
and laid down the doctrine for disciplinary sanctions from the Guzman v.
NU case.

DOCTRINE: Where a party was afforded an opportunity to participate in


the proceedings but failed to do so, he cannot [thereafter] complain of
deprivation of due process.

What matters for due process purpose is notice of what is to be


explained, not the form in which the notice is given.

FACTS:
In October 2001, Mr. Isleta, Letran’s Head of Auxiliary Services Department,
received info that certain fraternities were recruiting high school
students.
School authorities started an investigation and conducted medical
examinations on those alleged students involved. The school physician
reported that 6 students bore injuries on the posterior portions of their
thighs.
The Assistant Prefect for Discipline, Mr. Rosarda, asked for their
explanations in writing. Four students admitted that they were
neophytes of the Tau Gamma Fraternity and were present during the
hazing rites on October 3, 2001 in a certain “Dulce’s” house in Tondo
Manila.
its high school department. He spoke to one of the neophytes and RULING: For these reasons, we find no reversible error in the assailed CA
obtained a list of 18 members currently enrolled in the high school decision, and accordingly, DENY the present petition.
department. Kim’s name was again on the list.
At the Parent-Teacher Conference, Mr. Rosarda informed Kim’s mother
about his involvement. She expressed disbelief.
Mr. Rosarda then spoke to Kim and asked for an explanation. Kim
responded through a written statement dated December 2001 in
which he denied being part of the fraternity. His alibi was that at that
time, he went to Dulce’s house to pick up a gift and was not present
during the hazing.
Mr. Rosarda also requested Kim’s parents (by notice) to attend the
conference on January 2002 to address said issue. Both Kim’s parents
did not attend.
In time, Letran et al. found that 29 of their students, including Kim, were
fraternity members. The Letran found substantial basis in the
neophytes’ statements that Kim was a senior frat member.
Based on their disciplinary rules, the Father Prefect for Discipline (Rev.
Hernandez) recommended the members’ dismissal from the high
school department rolls. This sanction was stated in a letter to Sps.
Go.
Fr. Lao, Father Rector and President of Letran, rejected the
recommendation to allow the fourth year students to graduate from
Letran.
Mr. Rosarda conveyed to Mrs. Go and Kim in their conference on
January 15 the decision to suspend Kim. Incidentally, Mr. Go did not
attend.
Mrs. Go submitted a request for the deferment of Kim’s suspension so
he could take his exams. The request was granted.
On January 22, Letran et al. conferred with the parents of the sanctioned
seniors to discuss the extension classes they would need to
complete all academic requirements for graduation. They proposed
parents and children to sign a pro0forma agreement to signify their
conformity with the suspension.
Sps. Go refused to sign. They also refused to accept Letran et al’s
finding that Kim was a fraternity member. They likewise insisted that
due process had not been observed.
The RTC held that the respondents had failed to observe “the basic
requirement of due process” and that their evidence was “utterly
insufficient” to prove that Kim was a fraternity member.
It also declared that Letran had no authority to dismiss students for fraternity
membership. It awarded petitioners moral and exemplary damages.
The CA reversed the RTC’s ruling. It held that petitioners were not
denied due process and there was no bad faith, malice to justify the
award of damages.

ISSUE/s:
WoN due process was observed in Kim’s suspension – YES
applicable to every imaginary situation.
RATIO: What matters for due process purpose is notice of what is to be
It is noted that the sanction Letran et al imposed on Kim was actually a explained, not the form in which the notice is given.
suspension and not a “dismissal” as petitioners aver. The raison d’eter of the written notice rule is to inform the student of the
Also, the RTC erred in stating that Letran had no authority to impose a disciplinary charge against him and to enable him to suitably prepare
dismissal on students who violate its policy against fraternity a defense. The essence of due process, it bears repeating, is simply
membership. DECS Order No. 20, prohibiting fraternities and the opportunity to be heard.
sororities in public elementary and secondary schools and penalizing And Kim had been heard. His written explanation was received by
those with expulsion, applies to both public and private schools. Letran. Thus, he cannot claim that he was denied the right to adduce
Even assuming arguendo that the education department had not issued evidence in his behalf. In fact, petitioners were given further
such prohibition, private schools still have the authority to promulgate opportunity to produce additional evidence with the conference they
and enforce a similar prohibition pursuant to their right to establish did not attend.
disciplinary rules and regulations. In this case, we find that the respondents observed due process in Kim’s
The right to establish disciplinary rules is consistent with the mandate in disciplinary case, consistent with our pronouncements in Guzman.
the Constitution for schools to teach disciplne. The Court also agreed with the CA that there were no further bases to
In Guzman v. National University, the Court has laid down minimum hold the respondents liable for moral or exemplary damages.
standards in the imposition of disciplinary sanctions in academic
institutions, as follows:
The students must be informed in writing of the nature and cause
of any accusation against them
They shall have the right to answer the charges against them,
with the assistance of counsel, if desired
They shall be informed of the evidence against them
They shall have the right to adduce evidence in their own behalf
The evidence must be duly considered by the investigating
committee or official designated by the school authorities to
hear and decide the case
Since disciplinary proceedings may be summary, the insistence that a
“formal inquiry” on the accusation against Kim should have been
conducted lacks basis. Letran et al. gave Sps. Go 2 notices dated
December 19, 2001 and January 8, 2002 for conferences.
The notices stated, “Dear Mr/Mrs Go, we would like to seek your help in
correcting Kim’s problem on: Discipline & Conduct Offense:
Membership in Fraternity.” Thus, Letran et al. had given them ample
opportunity to assist their son in his disciplinary case.
They also failed to attend the conferences without any explanation, which
proves fatal to their case. Where a party was afforded an
opportunity to participate in the proceedings but failed to do so,
he cannot [thereafter] complain of deprivation of due process.
Mr. Rosarda also informed Mrs. Go that the charge stemmed from the
fraternity neophytes’ positive identification of Kim as a member; thus
the petitioners fully knew of the nature of the evidence that stood
against Kim.
Jurisprudence has clarified that administrative due process cannot be
fully equated with due process in the strict judicial sense. The very
nature of due process negates any concept of inflexible procedures
universally
LAO GI v. CA (IYA) On February 1982, Chia’s filed with the SC a petition for certiorari and
December 29, 1989 | Gancayco, J. | Procedural Due Process

PETITIONER: Lao Gi alias Filomeno Chia, Sr (and his family)


RESPONDENTS: Court of Appeals and Commission on Immigration and
Deportation

SUMMARY: Family of Chia is facing a deportation case with the CID on


grounds that they entered the PH fraudulently without going through
proper inspection by the immigration office. The Acting Commissioner
directed the family to register themselves as aliens. Chia contented this
order seeking to be allowed to present evidence on the matter. The SC
dismissed the order of the CID and directed it to continue with the
hearing and base decision on the presented evidences.

DOCTRINE: Although not a criminal procedure, it is a harsh and


extraordinary administrative proceeding affecting the liberty of the
person. Therefore, the constitutional right of the person to due process
should be denied. Only after hearing may the CID decide on the proper
action to be taken.

FACTS:
On September 1958, Opinion No. 191 was rendered, finding Filomeno
Chia, Jr. a Filipino Citizen being the son of Filomeno Chia Sr. who’s
legitimate parents are Inocencio Chia and (Filipino National) Maria
Layug.
On October of 1980, however, the Ministry of Justice rendered Opinion
147, cancelling Opinion No. 191, on the ground that the citizenship of
Filomeno Sr. was founded on fraud and misrepresentation.
On March 1981, a charge for deportation was filed with the Commission
on Immigration and Deportation (CID) against the family of Chia Sr.
An amended was filed alleging that the family refused to register as
aliens after having been required to do so. A third amendment was
filed charging the son Manuel Chia of acts of undesirability.
Manuel Chia was charged with falsification of public documents in the
Court of First Instance for alleging that he was a Filipino citizen in the
execution of a Deed of Absolute Sale of real property. He was,
however, acquitted based on Opinion 191 which was equated as res
judicata and that Opinion No. 147 cannot be considered as just fair
and reasonable.
Chia’s filed a motion to dismiss the charges on the ground that the CID
had no authority. The motion was opposed by a private prosecutor.
The CID special prosecutor also filed an opposition on the ground
that citizenship may be questioned out as the occasion may demand
and that due process was accorded to Chia’s. The motion was
denied by the CID, a subsequent MR was also denied.
prohibition. The SC en bans resolved to dismiss the petition for lack of
merit.
On September 1982, the CID set the deportation case against Chia’s for
hearing. Acting Commissioner Nituda directed the family to register
as aliens within 2 days from the receipt of notice.
Chia’s filed with the trial court a petition for certiorari and prohibition with a
prayer for inductive relief. The CFI rendered dismissing the petition for
lack of legal basis and for want of supervisory jurisdiction the subject
involved.
An appeal with the CA but was later dismissed. A MR was also filed but
again was denied.
Hence, the present petition for certiorari filed by Chia’s wherein they seek to
set aside the decision of the CA and ask that a new one be rendered
setting
aside the order of the CID to register themselves as aliens, and
directing the CID to proceed with the reception of the ecidence in
support of the charges against the petitioners.
ISSUE/s:
WON the CID has jurisdiction over the case and should therefore
proceed with the hearing- YES

RULING: Petition is GRANTED. The CID is directed to continue hearing the


deportation case.

RATIO:
Before any alien may be deported upon warrant of the CID, there should
be a proper determination of the Board of Commissioners of the
existence of the ground as charged against the alien.
In this case, the petitioners are charged with having entered the PH by
means of false and misleading statements.
After the filing, the parties must be duly informed of a hearing and only
after hearing may the alien be ordered deported.
The Chia’s question the Order of Acting Commissioner Nituada that they
register as aliens as required by the Immigration Act.
Although within the Commssioner’s authority, such requirement must be
grounded on a positive finding that the person is an alien
The power to deport an alien is an act of the State. It is a police measure
against undesirable aliens whose presence int he country is found to
be injurious to the public good and domestic tranquility of the people.
Although not a criminal procedure, it is a harsh and extraordinary
administrative proceeding affecting the liberty of the person.
Therefore, the constitutional right of the person to due process
should be denied.
The charge against an alien must specify the acts or omissions
complained of which must be stated in ordinary and concise
language to enable a person of common understanding to know on
what ground he is intended to be deported and enable the CID to
pronounce a proper judgment.
MACEDA v. ERB (ELIEL) He points out thtat this relaxed procedure resulted in the denial of
July 18, 1991 | Meialdea, J. | Procedural Due Process due process.
Maceda also claims that there is no substantial evidence on record to
PETITIONER: Ernesto Maceda support the provisional relief.
RESPONDENTS: Enery Regulatory Board, Caltex Inc., Pilipnas Shell
Petroleum
Corporation and Petro Corporation

SUMMARY: The three oil companies filed with the ERB applications for
oil increase following the outbreak of the Persian Gulf conflict. ERB
granted a provisional increase of P1.42/liter. Maceda filed for a prohibition
but was denied. However, hearings were set in order to give opportunity
to registered oppositors. During the hearings, the three oil companies
filed a motion to leave and applications to further increase the oil prices.
ERB granted such motion, this then deprived Maceda his due process
and failed to finish his cross-examination.

DOCTRINE: Such a relaxed procedure is especially true in administrative


bodies, such as the ERB, which in matters of rate or price fixing, is
considered as exercising a quasi-legislative, not quasi-judicial, function.
As such administrative aency, it is not bound by the strict or technical
rules of evidence governing court proceedings.

FACTS:
Upon the outbreak of the Persian Gulf conflict on August 2, 1990, Caltex,
Petron, and Shell filed with the ERB their respective applications on
oil price increases.
The ERB issued an order granting a provisional increase of P1.42 per
liter.
Maceda field a petition for Prohibition seeking to nullify the provisional
increased. This was dimissed.
The ERB set applications for hearing with due notice to all interested parties.
Maceda failed to appear at said hearing as well as on the second
hearing.
The ERB set the continuation to hearing to afford registered oppositors
the opportunity to cross-examine the witnesses.
The three oil companies field their respective motions for leave to file or
admit amended/supplemental applications to further increase the
prices of petroleum products.
The ERB admitted the respective petitons and required them to publish
in two newspapers of general circulation.
Maceda maintains that this order of proof deprived him of his right to
finish his cross-examination of Petron’s witnesses and denied him
his right to cross-examine each of the witnesses of Caltext and Shell.
ISSUE/s:
WoN Maceda was denied due process because of the relaxation of
procedure–
NO

RULING: Dimissed.

RATIO:
The order of testimony both with respect to the examination of the
particular witness and to the general course of the trial is within the
discretion of the court and the exercise of this discretion in
permitting to be introduced out of the order prescribed by the rules
is not improper.
Such a relaxed procedure is especially true in administrative
bodies, such as the ERB, which in matters of rate or price
fixing, is considered as exercising a quasi-legislative, not quasi-
judicial, function. As such administrative aency, it is not bound
by the strict or technical rules of evidence governing court
proceedings.
In the broader interest of justice, the Board may, in any particular
matter, except itself from these rules and apply such suitable
procedure as shall promote the objectives of the Order.
The Court has already ruled “the Board Order authorizing the proceeds
generated by the increase to be deposited to the OPSF is not an act
of taxation but is authorized by PD 1956, as amended by EO 137.
Paras, J. Dissent: The stubborn refusal of the ERB to effecitively
rollback oil prices is a continuing bestial insult ot eh intelligence of
our countrymen, and a gross abandonment of the people in their
hour of economic misery.
Padilla, J. Dissent: In the matter of price increases of oil products, which
vitally affects the people, especially those in the middle and low income
goups, any increase, provisional or otherwise, should be allowed only
after the ERB shall have fully determined, through bona fide and full-
dress hearings, that it is absolutely necessary and by how much it shall
be effeted The people, represented by reputable oppositors, deserve to
be given full opportunity to be heard in their opposition to any increase in
the prices of fuel
Sarmiento, J. Separate: Oil pricing “is a question best judged by the
political leadership” and oil prices are, political, rather than economic,
decisions.
CORONA v. UNITED HARBOR PILOTS ASSOCIATION (JP) public official petitioners have acted in excess of jurisdiction in
December 12, 1997 | Romero, J. | Procedural Due Process promulgating PPA-

PETITIONER: Hon. Renato C. Corona, Assitant Secretary for Legal


Affairs, Hon. Jesus Garcia, Acting Secretary of DOTC and Rogelio
Dayan, General Manager of Philippine Ports Authority
RESPONDENTS: United Harbor Pilots Association of the Philippines and
Manila Pilots Association

SUMMARY : Philippine Ports Authority (PPI) issed PPA-AO No. 04-92


which subjected the licenses or existing appointments of pilots who have
already passed 5 examinations into an annual evaluation which can
subject them to the cancellation of their licenses. The pilot associations
averred that their due process was infringed since there were no prior
public hearings before the order. The SC held that procedurally there was
no denial of due process because only quasi-judicial functions of
administrative bodies require public hearings. Substnatively however,
petitioner public officials erred for unduly restricting the vested rights of
pilots.

DOCTRINE: When one speaks of due process of law, a distinction must


be made between matters of procedure and matters of substance.
Procedural due process “refers to the method or many by which the law
is enforced,” while substantive due process “requires the law itself, not
merely procedurs by which the law would be enforced, is fair, reasonable,
and just.”

FACTS:
Philippine Ports Authority was created by PD No. 505, vesting it with the
power of control, regulation and supervision of pilots and the pilotage
profession.
PPA General Manager Rogelio Dayan issued PPA-AO NO. 04-92. Under its
terms “all existing regular appointments which have been previously
issued by the Bureau of Customs or the PPA shall remain valid up to
Dec. 31, 1992 only and all apointments to harbor pilot associations shall
be only for a term of one year from the date of effectivity subject to
renewal or cancellation by the Authority after a conduct of a rigid
evaluation of performance.”
Upon complaint by respondent associations, the Office of the President
issued an order directing PPA to hold the implementation of PPA-AO
NO. 04-92. PPA however, countered that it was in the exercise of its
administrative control and superivision over harbor pilots that they
issued the PPA-AO. The OP then resolved to dimiss the appeal and
lifted the restraining order issued earlier.
Respondents filed a petition for certiorari, prohibition and injunction with a
prayer for a TRO and damages before the RTC. The RTC ruled that the
AO No. 04-92 and declares such null and void. There was denial of
due process when no public hearings were held prior to the issuance
of the order and respondents allegedly only knew about it after its
publication in the newspapers. From this decision, petitioner public
officials elevated their case to the SC on certiorari.

ISSUE/s:
WoN PPA-AO No. 04-92 denied respondents of procedural due process

NO
WoN PPA-AO No. 04-92 denied respondents of substantive due process
-
YES

RULING: Instant petition is dismissed. Assailed decision of the court a quo is


affirmed.

RATIO:
In the procedural aspect, respondent associations allege that PPA
violated due processs because no hearing ws conducted whereby
relevalt government agencies and pilto themselves could ventilate
their views. However, it should be noted that notice and hearing are
essential only when administrative bodies exercise their quasi-
judicial functions. Because PPA exercised its executive or legislative
function, such as in issuing rules and regulations, an administrative
body need not comply with the requirements of notice and hearing.
In the substantive aspect, respondents aver that said right to the
exercise of harbor pilotage by pilots has become a vested right and
can only be withdrawn or shortened by observing due process of law.
It is here that the court sides with the respondents because indeed,
substantive due process was not met in the case at bar.
It is readily apparent that PPA-AO No. 04-92 unduly restricts the right of
harbor pilots to enjoy their profession before their compulsory
retirement at the age of 70. In the past, they enjoyed a measure of
security knowing that after passive five examintions and undergoing
years of on the job training, they would have a license until
retirement, unless sooner revoked for mental or physical unfitness.
With the new issuance however they have to contend with an annual
cancellation of their license which can be temporary or permanent
depending on the outcome of their performance evaluation.
It is this pre-evaluation performace which primarily makes PPA-AO No.
04-92 unreasonable and constitutionally infirm. It is a deprivation of
property without due process of law.
PAGCOR v. CA (HENRY) A notice of preventive suspension signed by the Senior Branch Manager of
December 13, 2011 | Reyes, J. | Procedural Due Process CF-Pavilion was received by respondent Manahan. She was instructed
to report in the office and was questioned regarding the incident to the
PETITIONER: PAGCOR Corporate
RESPONDENTS: CA and Mia Manahan

SUMMARY: Respondent Manahan, a Treasury Officer of PAGCOR, was


being dismissed by the said company due to mishandling of funds. A
huge amount of money (4.2M to be exact) was being claimed by
Manahan to be transferred to CF-Laoag, who denied that the transaction
happened, despite the fact that she handed it to a certain Fuentabella
who even showed an ID for proof of identity. Manahan filed a petition to
the CSC who granted it, enjoining PAGCOR to dismiss her. The petitioner
company then appealed to the CA, which was denied, then to the SC,
affirming the same, ruling that Manahan was indeed denied of her right to
due process.

DOCTRINE: While a liberal consruction of administrative rules of


procedure is allowed and applied in some cases, this is resorted to when
it can promote their objective and aid the parties in reaching a just and
speedy and inexpensive determinatination of their respective claims and
defenses.

FACTS:
Respondent Manahan was a Treasury Officer of petitioner PAGCOR
assigned in Casino Filipino-Manila Pavilion (CF-Pavilion). Among her
functions was the handling of fund transfer requests received by CF-
Pavilion and the supervision of the office’s Vault-in-Charge and Senior
Cashier.
April 2004 1:30pm, Manahan received from fax machine of CF-Paviion’s
SVIP-Treasury a document that appeared to be a Request for Fund
Transfer from Casino Filipino-Laoag (CF-Laoag), amounting to 4.2M,
to be released to Arnulfo or David Fuentabella.
30 minutes in, a person who represented himself to be David Fuentabella
claimed the amount, he even presented an SSS ID for proof of
identity, which was accepted by the respondent who was the
Treasury Officer on duty. 2M was released in cash, 2.2M in the form
of chips.
Jennifer Bagtas (then officer on duty) informed CF-Laoag through hone
that the amount was already released to Mr. Fuentabella. However,
CF-Laoag’s Vault-in-Charg Norman Santiago and Treasury Head
Ricafort denied such transfer had been made.
Close to midnight of the same day, Manahan called PAGCOR Assisting
Chief Security Officer asking her to report immediately to CF-
Pavilion, where she was informed of CF-Laoag’s claim that no
transfer was made.
Manahan was interrogated by PAGCOR’s Casino Operations Manager
and Senior Chief Security Officer regarding the incident.
Investigation Unit (CIU) of PAGCOR. She failed to appear. Instead, Manaha, because these rights already form part of a procedural due
she filed a Written Statement narrating the events that transpired. process.
(because he wasn’t allowed to be assisted by counsel)
Eventually, Manahan received from the HR Department a letter informing her
of the PAGCOR Board of Directors’ decision to dismiss her from service.
Manahan filed an MR, with one of the claims being that she was
deprived of her constitutional right to due process of the law when
the BOD outrightly dismissed her from service without informing her
of the formal charges and apprising her of the documentary evidence
against her. This motion as denied by the PAGCOR BOD for lack of
merit.
She filed an appeal to the CSC, who granted Manahan’s appeal, after a
finding of violation of Manahan’s right to due process. The case was
remanded to PAGCOR for issuance of a formal charge, then a formal
investigation pursuant to the Uniform Rules on Adminisgtrative
Cases in the Civil Service. CSC likewise noted that the order of
dismissal served upon Manahan was a mere notice issued by the
HR Dept informing her of PAGCOR BOD’s decision to dismiss her,
instead of a copy of the BOD Resolution on the order of the
dismissal.
PAGCOR filed an MR which was denied, so they appealed to the CA,
who affirmed the decision of the CSC. Hence, this petition.

ISSUE/s:
WoN respondent Manahan was denied of her constitutional right to due
process - YES

RULING: SC dismissed the petition.

RATIO:
It is worthy to note that the respondent signified her desire to be
represented by counsel during the proceedings before PAGCOR and
even requested to be furnished with documents during the
investigations then being conducted by the petitioner. Her requests
were evidenced by her counsel’s letter to PAGCOR’s Head of
Investigation Unit.
Instead of allowing these requests of Manahan’s counsel, PAGCOR
replied that her requests deserved scant consideration, and were
even premature, that having a counsel was contrary with the internal
rules and/or customary practice of PAGCOR.
This stance of PAGCOr was in clear disregard of the respondent’s rights
protected under CSC Resolution No. 99-1936. While due process in
an agency investigation may be limited as compared to due process
in criminal proceedings, where however a staute specifically provides
for a procedure and grants particular rights to a party under
investigations uch as in the investigations of persons covered by the
Civl Service Rules, these rights shall not be utterly disregarded,
especially so when invoked by the party under investigation, as was
While a liberal consruction of administrative rules of procedure is allowed
and applied in some cases, this is resorted to when it can promote
their objective and aid the parties in reaching a just and speedy and
inexpensive determinatination of their respective claims and
defenses.
Without proper investigation and, a decision that clearly indicated
the facts constituting the offense imputed upon the respondent
and the company rules she supposedly violated, the respondent
did not get the chance to sufficiently defend herself; and more
importantly, the petitioner, the CSC, and the courts culd not
have had the chance to reasonably ascertain the truth which the
CSC rules aim to accomplish.
On December 1984, Salaw was requested by the bank’s manager, Rollie
SALAW v. NLRC (DANNAH) Tuazon, to appear before the bank’s Personnel Discipline and
Investigation Committee (PDIC) which would be meeting the
September 27, 1991 | Sarmiento, J. | Procedural Due Process: Right to
following day, in connection with the Worldwide case.
Counsel

PETITIONER: Espero Santos Salaw


RESPONDENTS: National Labor Relations Commission, Jose Tengco,
Rollie
Tuazon

SUMMARY: Salaw was employed by Associated Bank as a credit


investigator-appraiser. The Criminal Investigation Service of the
Philippine Constabulary was able to get a hold of a Sworn Statement by
Salaw, in which it appeared that salaw sold 20 esewing machines and
electric generators from the bank’s assets.

Salaw was subsequently terminated, which prompted him to file a


complaint for illegal dismissal with the NLRC. The Labor Arbiter ruled in
his favor. However, upon respondent’s (bank) appeal, the decision was
overturned. Salaw filed an MR which was denied, hence the case at
hand.

The SC ruled that Salaw was illegally dismissed, as there was nothing
else to prove the crime he committed, because said Sworn Statement
was obtained without the assistance of counsel.

DOCTRINE: It is true that administrative and quasi-judicial bodies are not


bound by the technical rules of procedure in the adjudication of cases.
However, the right to counsel, a very basic requirement of substantive
due process, has to be observed.

A decision with absolutely nothing to support it is a nullity. The law is


clear that even in the disposition of labor cases, due process must not be
subordinated to expediency or dispatch. Otherwise, the dismissal of the
employee will be tainted with illegality.

FACTS:
Espero Santos Salaw was employed by respondent bank (Associated
Bank) in September 1967 as a credit investigator-appraiser.
On November 21, 1984, The Criminal Investigation Service of the
Philippine Constabulary extracted from Salaw—without assistance of
counsel—a Sworn Statement which made it appear that Salaw, in
cahoots with a co-employee (Madrigal), a supervisor in charge of the
acquired assets of said bank, sold 20 sewing machines and electric
generators which had been foreclosed by the respondent bank from
Worldwide Garment and LP Money Garment for P60,000 and divided
the shares between the two.
On April 1985, Salaw was terminated from employment for alleged It is true that administrative and quasi-judicial bodies are not bound by
serious misconduct or willful disobedience and fraud or willful breach the technical rules of procedure in the adjudication of cases.
of the trust reposed on him. However, the right to counsel, a very basic requirement of
Salaw filed with the NLRC on April 1985 a complaint for illegal dismissal
substantive due process,
against herein respondents. Labor Arbiter Villarante endered a
decision in favor of Salaw.
Respondents filed an appeal to the NLRC which rendered a decision in
their favor. Salaw filed an MR which was denied, hence this petition.

ISSUE/s:
WoN the dismissal of Salaw was legally justified – NO

RULING: WHEREFORE, premises considered, judgment is hereby rendered


SETTING ASIDE the appealed decision of the NLRC and REINSTATING the
decision of the labor arbiter.

RATIO:
Under the Labor Code, the requirements for the lawful dismissal of an
employee by his employer are two-fold: the substantive and the
procedural.
The rudimentary requirements of due process—notice and hearing—
must also be observed before an employee may be dismissed.
The inviolability of notice and hearing for a valid dismissal of an
employee cannot be over-emphasized. Thos twin requirements
constitute essential elements of due process in cases of employeed
ismissal.
The requirement of notice is intended to inform the employee
concerned of the employer's intent to dismiss him and the reason for
the proposed dismissal; on the other hand, the requirement of
hearing affords the employee the opportunity to answer his
employer's charges against him and accordingly to defend himself
therefrom before dismissal is effected.
Neither one of these two requirements can be dispensed with without
running afoul of the due process of the Constitution.
The Cour agrees with the labor arbiter that Salaw was terminated without
the benefit of due process of law. His dismissal was therefore, illegal.
As the records clearly show, Salaw was denied that constitutional right
when his subsequent request to refute the allegations against him
was granted and a hearing was set "without counsel or
representative."
Section 5 of the Implementing Rules and Regulations of the Labor Code
states that "the employer shall afford the worker ample opportunity to
be heard and to defend himself with the assistance of his
representative, if he so desires."
has to be observed.
Indeed, the rights to counsel and to due process of law are two of the
fundamental rights guaranteed by the 1987 Constitution to any
person under investigation, be the proceeding administrative, civil, or
criminal.
Section 12(1) Article III states that "Any person under investigation for the
commission of an offense shall have the right to x x x have
competent and independent counsel preferably of his own choice. If
the person cannot afford the service of counsel, he must be provided
with one. These rights cannot be waived except in writing and in the
presence of counsel."
The respondent (bank) premised their action on the supposed admission
of the offense imputed to Salaw by the Criminal Investigation Service
in its interrogation in November 1984. The admission was contained
in a three-page Sworn Statement signed by Salaw. Aside from this,
no other evidence was presented.
The Court reiterated the 7 cardinal primary rights contained in Ang Tibay
(please refer to Ang Tibay case)
A decision with absolutely nothing to support it is a nullity. The law
is clear that even in the disposition of labor cases, due process
must not be subordinated to expediency or dispatch. Otherwise,
the dismissal of the employee will be tainted with illegality.
Those illegally dismissed are entitled to resintatement without seniority
rights and payment of backwages. Salaw is entitled to no less.
The Labor Arbiter held that the dismissal was justified. On appeal the
LOPEZ v. ALTURAS GROUP OF COMPANIES (IYA) NLRC set aside the decision of the Labor Arbiter finding that the
evidence of the company did not suffice to warrant termination.
April 11, 2011 | Carpio-Morales, J. | Procedural Due Process

PETITIONER: Quirico Lopez


RESPONDENTS: Alturas Group of Companies and/or Marlito Uy

SUMMARY: Lopez was a truck driver of the Alturas Group of


Companies. he was caught smuggling out iron scraps. He was asked by
the company to Show Cause as to the event, where he wrote an answer
denying the allegations. He was later terminated by the company on
ground of loss of trust and confidence and violation of company rules
and regulations.

Lopez then filed a case for illegal dismissal. The LA held the dismissal
was justified. The NLRC reversed this. The CA found just cause for
termination,b ut said that Lopez was not afforded due process because
no hearing was conducted and that Lopez was not assigned counsel.
The SC ruled that there exists valid ground for his termination and that
he was accorded due process contrary to the findings of the CA.

DOCTRINE: The employee can be fully afforded a chance to respond to


the charges against him, adduce his evidence or rebut the evidence
against him through a wide array of methods, verbal or written.

The right to counsel and the assistance of one in investigations involving


termination cases is neither indispensable nor mandatory, except when
the employee himself requests.

FACTS:
Lopez was hired by Alturas Group of Companies as truck driver in 1997.
In 2007, Lopez was allegedly caught by one of their security guards int
he act of attempting to smuggle out 60 kilos of scrap iron. Lopez
allegedly admitted to the guard that he was taking out the scrap
irons.
A Show Cause Notice was issued by the HR Department of the company
to which Lopez denied the allegations through a handwritten
explanation.
Finding the explanation unsatisfactory, the company terminated his
employment on grounds of loss of trust and confidence and violation
of company rules and regulations through a Notice of Termination.
The company also filed a criminal case for qualified theft before the RTC.
Lopez then filed a complaint against the company for illegal dismissal
and underpayment of wages.
Company filed a MR but was denied. They then filed an appeal with the CA.
The CA reversed the ruling of the NLRC saying that the act of smuggling
is sufficient ground which may lead the loss of trust and confidence.
The CA, however, held that due process was not observed when the
company failed to give him chance to defend his side in a proper
hearing.
Hence, the present petition for review on certiorari.

ISSUE/s:
WON due process was accorded to Lopez - YES

RULING: THE CA ruling is upheld.

RATIO:
As to substantive due process, the Court finds that respondent
company’s loss of trust and confidence arising from petitioner’s
smuggling out of the crap iron constituted just cause for terminating
his services.
Procedural due process has been defined as giving an opportunity to be
heard before judgement is rendered.
In Perez v PT&T it was held that: The employee can be fully afforded a
chance to respond to the charges against him, adduce his evidence
or rebut the evidence against him through a wide array of methods,
verbal or written.
Petitioner was given opportunity to explain his side when he was
informed of the charge against him and required to submit his written
explanation.
There is no violation of due process even if no hearing was conducted,
where the party was given a change to explain his side of the
controversy.
The right to counsel and the assistance of one in investigations involving
termination cases is neither indispensable nor mandatory, except
when the employee himself requests.
There is no showing that Lopez requested for a formal hearing to be
conducted or that he be assisted by counsel.
CARABEO v. CA (ELIEL) Secretary Teves issued Special Order N. 4-05 designating Assistant City
July 18, 1991 | Carpio, J. | Procedural Due Process Treasure as OIC. Carebeo filed another petition with the CA citing
Sec. Teves for contempt of court.
PETITIONER: Liberato M. CArabeo
RESPONDENTS: Court of Appeals et al.

SUMMARY: DOF-RIPS filed a complaint with the Ombudsman against


OIC Carebeo. He allegedly violated RA 3019 and RA 6713. His SALN
failed to delare the properties and vehicles he is in ownership of, and the
income he is earning does not match his businesses. The Ombudsman
directed the preventive suspension of Carabeo pending the investigation.
Carabeo filed a TRO with the CA, which granted the TRO. However, the
Sec. Teves designated the Assistant City Treasurer as OIC, which made
Carabeo file with the CA. This was then denied. Hence this petition.

DOCTRINE: While their right to due process may be relied upon by


public officials to protect their security of tenure which, in a limited sense,
is analogous to property, such fundamental right to security of tenure
cannot be invoked against a preventive suspension order which is a
preventive measure, not imposed as a penalty.

FACTS:
The Department of Finance-Revenue Integrity Protection Service (DOF-
RIPS) filed a complaint with the Office of the Ombudsman against
Carabeo, Officer-in-Charge (OIC) of the Office of the Treasurer of
Paranaque City.
The Net Worth based on his SALN ballooned from P114,000 in 1982 to
P7.5 million in 2004.
Equally noticeable as the drastic increase in his net worth is the steady
accumulation of various expensive properties by Carabeo and his
spouse ranging from real properties ot vehicles to club shares
ownership.
Carabeo did not declare in SALN most of his vehicles, but declared only
one vehicle. The LTO certification belies this declaration.
Carabeo also acquired a 1,000 sq.m. Tagaytay property, which was also
not declared in the SALN.
Carabeo’s failure to disclose his and his spouse’s ownership amounts to
a violation of Sec 7 of RA 3019 and Sec 8(A) of RA 6713.
The Office of the Ombudsman’s Preliminary Investigation and
Administrative Adjudication Bureau placed Carabeo under preventive
suspension for a period not to exceed six months without pay.
Carabeo filed a petition for ceritoari alleging grave abuse of discretion. CA
issued TRO enjoining the enformcement of Carabeo’s preventive
suspension.
CA granted Carabeo’s request for the petitions to be consolidated.
However, the CA dismissed these petitions. Carabeo filed a MR,
denied.
Hence this petition.

ISSUE/s:
WoN therew as grave abuse in failure to provide implementing rules of
EO 259 does not render the same unenforceable– NO
WoN the suspension order violates his due process - NO
WoN there was grave abuse in not considering the complaint against
Carabeo a violation of Sec 10 of RA6713 which entitles Carabeo to
be informed beforehand and to take the necessary corrective action -
NO

RULING: Dimissed.

RATIO:
The Court finds that EO 259 is basically internal in nature needing no
implementing rules and regulations in order to be enforceable.
The Creation of an internal body in the DOF (RIPS), through EO 259, is
but an essential component in the organized and effective collection
of evidence agasint corrupt DOF officials and employees.
Settled is the rule that prior notice and hearing are not required in
the issuance of a preventive suspension order, such
suspension not being a penalty but only a preliminary step in an
administrative investigation.
While a preventive suspension order may originate from a complaint, the
Ombudsman is not required to furnish the respondent with a copy of
the complaint prior to ordering a preventive suspension.
Two requisites: Evidence of guilt is strong and (a) offense involve dishonesty,
oppression grave misconduct or neglect in performance of duty; (b)
warrant removal from the service; or (c) continued stay may prejudice
the case.
Carabeo cannot claim any right against, or damage or injury that he
is bound to suffer from the issuance of the preventive
suspension order, since there is no vested right to a public
office, or even an absolute right to hold it.
While their right to due process may be relied upon by public
officials to protect their security of tenure which, in a limited
sense, is analogous to property, such fundamental right to
security of tenure cannot be invoked against a preventive
suspension order which is a preventive measure, not imposed
as a penalty.
Grave abuse of discretion implies such capricious and whimsical
exercise of judgment as is equivalent to lack of jurisdiction.
He likewise maintains that they are vague insofar as they reckon the date of
PEOPLE v. NAZARIO (JP)
payment: Whereas Ordinance No. 4 provides that parties shall
August 31, 1988 | Sarmiento, J. | Procedural Due Process

PETITIONER: People of the Philippines


RESPONDENTS: Eusebio Nazario

SUMMARY: Accused Nazario was a lessee of a fishpond in


Pinagbayanan. He was accused of violating Ordinance No. 4 for not
paying taxes with regard to his fishpond. He defends by saying that the
ordinance at bar is ultra vires for being outside the power of the municipal
council and being vague and ambigious. The SC ruled that the ordinance
is far from vague. Accused was clearly included in the “managers” of the
fishpond under the law and the dates to pay are clearly set forth in the
future amendments of the ordinance.

DOCTRINE: Vagueness is repugnant to the Constitution in two respects:


(1) it violates due process for failure to accord persons, especially the
parties targetted by it, fair notice of the conduct to avoid; and (2) it leaves
law enforcers unbridled discretion in carrying out its provisions and
becomes an arbitrary flexing of the Government muscle.

FACTS:
Accused Nazario was charged with violaion of Ordinance No. 4 for being
the then owner and operator of a fishpond situated in Pinagbayanan
of the Municipality of Pagbilao but unlawfully refusing and failing to
pay the municipal taxes of 362.62 pesos as required of him being
fishpond operator.
Section 1: Any owner or manager of fishponds in places within the
territorial limits of Pagbilao, Quezon, shall pay a municipal tax in the
amount of P3.00 per hectare of fishpond.
Evidence presented showed that accused is a lessee of a parcel of land,
with an area of 27.1998 hectares, for fishpond pursposes, under
Fishpond Lease Agreement No. 1066, entered into by the accused
and the government.
Evidence further showed that he actually converted this land into the
actual fishpond and used as such.
The defense by their evidence tried to show however that as lessee of a
forest land to be converted into a fishpond, Nazario is not covered by
said municipal ordinances as there is no fishpond yet as of the date the
tax was sought.
The defense also claims that the ordinance in question is ultra vires
as it is outside the power of the municipal council to enact and
that the odirnance in question is ambiguous and uncertain.
The petitioner contends that being a mere lessee of the fishpond, he is not
covered since the said ordinances speak of owner or manager.
commence payment after the lapse of three (3) years starting
from the date said fishpond is approved by the Bureau of
Fisheries. Ordinance No. 12 states that liability for the tax
accrues beginning and taking effect from the year 1964 if the
fishpond started operating before the year 1964.
RTC found defendant guilty of the crime of violation of Ordinance No. 4,
series of 1955, as amended by Ordinance 15, series of 1965, and as
further amended by Ordinance No. 12, series of 1966 of the
Municipal Council of Pagbialo, Quezon. Hence this appeal.

ISSUE/s:
WoN the lower court erred in not declaring that Ordinances No. 4, 15,
and 12 of the Municipality of Pagbilao are null and void for being
ambiguous and uncertain - NO

RULING: Apppeal is dismissed.

RATIO:
As a rule, a statute or act may be said to be vague when it lacks
comprehensible standards that men of common intelligence must
necessarily guess at its meaning and differ as to its application.
Vagueness is repugnant to the Constitution in two respects: (1) it violates
due process for failure to accord persons, especially the parties
targetted by it, fair notice of the conduct to avoid; and (2) it leaves
law enforcers unbridled discretion in carrying out its provisions and
becomes an arbitrary flexing of the Government muscle.
In no way may the ordinances at bar be said to be tainted with the vice of
vagueness. It is unmistakable from their very provisions that the
appellant falls within its coverage. As the actual operator of the
fishponds, he comes within the term “manager.” He does not deny
the fact that he financed the construction of the fishponds, introduced
fish fries into the fishponds, and had employed laborers to maintain
them
Whilt it appears that it is the Natinal Government which owns them, the
Government never shared in the profits they had generated. It is
therefore only logical that he shoulders the burden of tax under the
said ordinances.
Ordinance No. 15, in making the tax payable “after the lapse of three (3)
years starting from the date said fishpond is approved by the Bureau
of Fisheries,” is unequivocal about the date of payment, and its
amendment by Ordinance No. 12, reckoning liability there-under
“beginning and taking effect from the year 1964 if the fishpond
started operating before the year 1964,” give rise to any ambiguity.
To the Court, the ordinances in question set forth enough standards that
clarify imagined ambiguities. While such standards are not apparent
from the face thereof, they are visible from the intent of the said
ordinances.
ESTRADA v. SANDIGANBAYAN (HENRY) on ground that the facts did not constitute an indictiable offense since
November 19, 2001 | Peralta, J. | Procedural Due Process the law on which it was based was unconstitutional for vagueness.
June 21, 2001 the Government filed its Opposition to the Motion to
PETITIONER: Joseph Ejercito Estrada Quash, which was denied by the Sandiganbayan. Hence, this
RESPONDENTS: Sandiganbayan (Third Division) and People of the PH petition.

SUMMARY: Petitioner Estrada, then President of the PH, is being ISSUE/s:


prosecuted of Plunder, and is assailing the same, as he claims it is WoN RA 7080 (Plunder Law), as amended by RA 7659 is
unconstitutional due to its vagueness—specifically Secs. 1 par (d), 2, and unconstitutional for being vague and thus violating the right to due
4. The Office of the Ombudsman filed before the Sandiganbayan 8 process of the petitioner -
Informations, which the petitioner petitioned to be remanded to the NO
Ombudsman for preliminary investigation due to failure to specify under
part “d” the charges. The Sandiganbayan then ofund probable cause to RULING: SC ruled that RA 7080 (Plunder Law), as amended by RA 7659 is
charge the President of Plunder, which eventually justifies the issuance of Constitutional
the warrant of arrest of Estrada—he filed an MR which was denied. He
filed a Motion to Quash the Information, which was countered by the RATIO:
petition of the Government, which was again denied. Estrada then filed a Preliminarily, the whole gamut of legal concepts pertaining to the validity
petition before the SC, who ruled in favor of the respondents, and that the of legislation is predicated on the basic principle that a legislative
Plunder Law is Constitutional as it does not contain vagueness. measure is presumed to be in harmony with the Constitution—which
the court gives value. It is assumed that the legislature is ever
DOCTRINE: As long as the law affords some comprehensible guide or conscious of the borders and edges of its plenary powers, and has
rule that would inform those who are subject to it what conduct would passed the law with full knowledge of the facts and for the purpose of
render them liable to its penalties, its validity will be sustained. promoting what is right and advancing the welfare of the majority.
This is why, in La Union Credit Cooperative v. Yaranon, the SC ruled that as
long as there is some basis for the decision of the court, the
constitutionality of the challenged law will not be touched and the case
FACTS: will be decided on other available grounds. Hence, the onerous task of
Petitioner Estrada, the highest-ranking official to be prosecuted under RA rebutting the presumption weighs heavily on the party challenging the
7080 (Plunder), as amended by RA 7659, assailing it due to one of validity of the statute.
its “defects” of being vague. Petitioner in this case has miserably failed to discharge his burden and
The provisions of the Plunder Law claimed by the petitioner to have overcome the presumption of constitutionality of the Plunder Law.
transgressed constitutional boundaries are Secs. 1, par. (d), 2 and 4. The Plunder Law contains ascertainable standards and well-defined
April 4, 2001, Office of the Ombudsman filed before Sandiganbayan 8 parameters which would enable the accused to determine the nature
Informations. of his violation.
April 11, 2001, petitioner Estrada filed an Omnibus Motion for the remand As long as the law affords some comprehensible guide or rule that would
of the case to the Ombudsman for preliminary investigation with inform those who are subject to it what conduct would render them
respect to specification “d” of the charges in the Information in one of liable to its penalties, its validity will be sustained. It’s quite obvious
the cases, and for reconsideration/investigation of the offences to that the assailed statute punishes the act of a public officer in
give the accused (Estrada) an opportunity to file counter-affidavits amassing or accumulating ill-gotten wealth of at least 50M through a
and other documents necessary to prove lack of probable cause. series or combination of acts enumerated in Sec. 1 par (d) of the
The grounds raised were only lack of preliminary investigation, Plunder Law.
reconsideration/reinvestigation of offenses, and opportunity to prove The factual assertions indicated in the Information clearly show that the
lack of probable cause. Ambiguity and vagueness of the law were elements of crime are easily understood and provide adequate
never raised in that Omnibus Notion. contrast between the innocent and the prohibited acts.
April 25, 2001, Sandiganbayan (3 rd division) issued a Resolution finding Petitioner however, bewails the failure of the law to provide for the
probable cause for plunder to justify for the issuance of warrants for statutory definition of the terms “combination” and “series” in the key
arrest of Estrada. Estrada filed an MR which was denied. phrase “a combination or series of overt or criminal acts” found in
June 14, 2001 petitioner Estrada moved to quash the Information in the Sec. 1 par. (d) and Sec. 2, and the word “pattern” in Sec. 4.
case
Despite the above submissions of the petitioner, the court ruled that a
statute is nto rendered uncertain and void merely because general
terms are used
therein, or because of the employment of terms without defining them; much
less do we have to define every word we use. After all, there’s no positive
constitutional or statutory command requiring the legislature to define each
and every word in an enactment. As long as legislative will is clear, the word
in a statute will not result in the vagueness or ambiguity of the law.
It cannot plausibly be contended that the law does not give a fair warning
and sufficient notice of what it seeks to penalized.
According to the court, a statute/ act may be said to be vague when it
lacks comprehensible standards that men of common intelligence
must necessarily guess at its meaning and differ in its application.
But this doctrine does not apply as against legislations that are merely
couched in imprecise language but which nonetheless specify a
standard though defectively phrased; or those that are apparently
ambiguous yet fairly applicable to certain types of activities. The first
may be saved by proper construction while no challenge may be
mounted as against the second whenenver directed against such
activities.
An act will not be held invalid merely because it might have been more
explicit in its wordingsor detailed in its provisions, especially where
because of the nature of the act, it would be impossible to provide all
the details in advance as in all other statutes.
It is evident that the ambiguity that petitioner is claiming is more imagined
than real. Ambiguity where none exists cannot be created by
dissecting parts and words in the statute to furnish support to critics
who cavil at the want of scientific precision in the law.
The SC once held in Gallego v. Sandiganbayan, that courts are loathed
to declare a statute void for uncertainty/vagueness unless the law
itself is so imperfect and deficient in its details, and is susceptible of
no reasonable construction that will support and give it effect.
FEEDER v. CA (DANNAH) decrees of the CTA are within the exclusive appellate jurisdiction of
May 31, 1992 | Regalado, J. | Procedural Due Process: Right to the CA. The CA again affirmed the decisions of the lower courts,
Counsel hence this appeal.

PETITIONER: Feeder Int’l Line


RESPONDENTS: Court of Tax Appeals, Commissioner of Customs

SUMMARY: M/T Ulu Wai, operated by Feeder Int’l Line of Singapore,


illegally docked in Iloilo without informing local customs officers. Upon
investigation, it was seen that it did not contain necessary ship and
shipping papers, except a clearance for Zamboanga.

A Warrant of Seizure and Detention was issued, which Feeder appealed


to the Commissioner of Customs, Court of Tax Appeals, Court of Appeals
and the SC. The SC affirmed the lower courts’ decisions stating that it
was accorded due process, and it was evident that Feeder had the intent
to unload said cargo, amounting to illegal importation.

DOCTRINE: In other proceedings, however, the need for the assistance


of counsel is not as urgent nor is it deemed essential to their validity.
There is nothing in the Constitution that says a party in a non-criminal
proceeding is entitled to be represented by counsel. The assistance of
lawyers, while desirable, is not indispensible.

FACTS:
The M/T Ulu Wai is a foreign vessel of Honduran registry, owned and
operated by Feeder Int’l Shipping lines of Singapore. It left Singapore on
May 1986 carrying 1,100 metric tons of gas oil and 1,000 metric tons of
fuel oil consigned to Far East Synergy Corporation of Zamboanga,
Philippines.
On May 14, the vessel anchored in Iloilo without notifying customs
authorities. An Information by a civilian informer was the only reason
why they checked it out.
The Acting District Collector of Iloilo dispatched a Customs team on May 19
to verify the report. The team found that the vessel did not have the
required ship and shipping documents, except for a clearance for
Zambaonga.
The vessel and cargowere held and a Warrant of Seizure and Detention
over the same was issued after due investigation.
Feeder then filed its Motion to Dismiss and to Quash the Warrant which
the District Collector denied in his Order on December 1986. Feeder
also appealed to the Commissioner of Customs who affirmed said
decision.
On June 1987, Feeder filed a petition for review of the decisions of the
Collector and the Commissioner of Customs with the Court of Tax
Appeals, which yet again affirmed assailed decisions.
Feeder filed a petition for review of the CTA’s decision with this Court,
which the Court remanded to the CA because final judgments or
proceeding is entitled to be represented by counsel. The assistance
ISSUE/s: of lawyers, while desirable, is not indispensible.
WoN Feeder was deprived of property without due process of law – NO

RULING: WHEREFORE, the instant petition is DENIED for lack of merit and
the judgment appealed from is hereby AFFIRMED in toto.

RATIO:
Forfeiture proceedings under tariff and customs law are not penal in
nature, as they do not result in the conviction of the offender nor in
the imposition of penalties. These are purely civil and administrative
in character,
Considering, therefore, that these are not criminal in nature, proof
beyond reasonable doubt is not required in order to justify forfeiture
of goods.
In this case, the degree of proof required is merely substantial
evidence, which means such relevant evidence as a reasonable
mind might accept as adequate to support a conclusion.
Importation begins when the carrying vessel or aircraft enters the
jurisdiction of the Philippines with intention to unload therein. It is
clear from the provision of law that mere intent to unload is
sufficient to commence an importation.
Intent being a sate of mind must ordinarily be inferred from the facts, and
therefore can only be proved by unguarded expressions, conduct
and circumstances generally.
In the case at hand, Feeder is guilty of illegal importation, there having
been an intent to unload, is amply supported by substantial evidence
as clearly demonstrated.
The alibi that the reason for docking in Iloilo was for repairs is untenable.
Zamboanga is closer compared to Iloilo so it would be illogical to go
farther for repairs. Moreover, it did not nofity local customs authority
of its arrival.
The Court therefore finds no compelling reason to deviate from the
elementary principle that findings of fact of the Court of Appeals, and
of the administrative and quasi-judicial bodies for that matter, are
entitled to great weight and are conclusive and binding upon this
Court absent a showing of a grave abuse of discretion amounting to
lack of jurisdiction.
The right to assistance of counsel is not indespensable to due process
unless required by the Constitution or a law. Exception is made in the
charter only during the custodial investigation of a person suspected of a
crime, who may not waive his right to counsel except in writing and in
the presence of counsel.
In other proceedings, however, the need for the assistance of counsel is
not as urgent nor is it deemed essential to their validity. There is
nothing in the Constitution that says a party in a non- criminal
CB v. CA (IYA) CB and Tiaoqui elevated the orders of the RTC to the CA on a petition for
March 30, 1993 | Bellosillo, J. | Procedural Due Process certiorari and prohibition. CA upheld the RTC ruling.

PETITIONER: The Central Bank of the Philippines and Ramon V. Tiaoqui


RESPONDENTS: Court of Appeals and Triumph Savings Bank

SUMMARY: Triumph Savings Bank was placed by the CB, through a


Monetary Board Resolution, under receivership and ordered its
operations to be put to a stop. It was found that TSB has become
insolvent and that continued operations would cause injury to its debtors,
creditors, and stockholders.

TSB questions the MB Resolution saying it is tainted with arbitrariness


and bad faith because they were not sent a notice nor was there a
hearing conducted. The SC however ruled that the MB is granted
authority by its enacting statue.

DOCTRINE: Lack of notice and hearing cannot be deemed acts of


arbitrariness and bad faith in cases of bank closures.

The “close now and hear later” scheme is grounded on practical and
legal considerations to prevent unwarranted dissolution of the banks
assets and as valid exercise of police power to protect the depositors,
creditors, stockholders and in general the public.

FACTS:
Based on reports by the Supervision and Examination Section of the
Central Bank, Triumph Savings Bank (TSB) was found insolvent and
if it continues in business it would involve probable loss to its
depositors and creditors.
The monetary Board issued a resolution ordering the closure of TSB and
placing it under receivership of Ramon Tiaoqui.
TSB filed a complaint with the RTC seeking to annul the Monetary Board
resolution challenging the constitutionality of Sec 29 of the Central
Bank Act insofar as it authorizes the CB to take over banking
institution even if it is not charged with violation of law or regulation.
RTC temporarily restrained the MB resolution. The CB moved for
quashal saying TSB failed to show convincing proof that there exists
arbitrariness and band faith on the part of CB. The trial court granted
the quashal.
CB and Ramon Tiaoqui filed a motion to dismiss the complaint before the
RTC for failure to state cause of action.
TSB filed an urgent motion in the RTC to direct receiver Tiaoqui to
restore TSB to its private management.
The RTC dismissed the CB’s motion and ordered Tiaoqui to restore the
management of TSB to its elected board of directors.
Hence, the present petition of CB and Tiaoqui praying that the CA
decision be dismissed.
TSB contends that CB violated the rule on administrative due process
which requires that prior notice and hearing be afforded to all parties
in administrative proceedings.
Petitioners counter this by saying that prior notice and hearing in cases
involving bank closures should not be required since it would give
opportunity for the bank to destroy evidences or irregularities in the
banks operations.
ISSUE/s:
WON absence of proper notice and hearing may be considered acts of
arbitrariness and bad faith sufficient to annul a MB resolution - NO

RULING: The case is remanded to the RTC for further proceedings to determine
whether the issuance of the MB Resolution was tainted with arbitrariness and
bad faith.

RATIO:
Section 29 of the Central Board Act vests the MB with exclusive authority
to assess, evaluate and determine the condition of any bank. Finding
such condition to be one of insolvency, or that its continuance in
business would involve probable loss to its depositors or creditors,
forbid the bank or non-bank financial institution to do business in the
PH and shall designate an official of the CB as receiver to
immediately take charge of its assets and liabilities.
Sec 29 does not contemplate prior notice and hearing before a bank may
be directed to stop operations and placed under receivership.
The “close now and hear later” scheme is grounded on practical and
legal considerations to prevent unwarranted dissolution of the banks
assets and as valid exercise of police power to protect the
depositors, creditors, stockholders and in general the public.
Lack of notice and hearing cannot be deemed acts of arbitrariness and
bad faith.
PEREZ v. MADRONA (ELIEL) Perez thereafter filed a petition for certiorari before the CA assailing the
March 21, 2012 | Villarama, J. | Procedural Due Process default order. CA dismissed the petition, even MR.
The RTC issued an order dismissing the injunction complaint without any
PETITIONER: Jaime S. Perez prejudice (Refer to number 4). Held that respondents “have not
RESPONDENTS: Sps. Fortunito Madrona and Yoland B. Pante instituted any action before the Court showing that they are still
interest in further prosecuting the case” and “in accordance with Sec
SUMMARY: Perez sent a demolition notice to Sps. Madrona for allegedly 3, Rule 17 of the Rules of Court, the Court is constrained to dismiss
illegaly occupied/constructed improvement within the road right-of-way. the complaint for failure to prosecute their complaint for an
Sps. Respndend that the accusation was false, and upon another letter, unreasonable length of time.”
prompted them to file an injunction with the RTC aganist Perez and all However, upon motion Sps. Madora, the dismissal order was set aside
persons acting under him from doing any act of demolition. RTC ruled in and the complaint was reinstated by Order.
their favor, Perez appealed to the CA, denied. Hence this petition. RTC rendered decision in favor of Sps. Madrona. Perez appealed to CA,
which affirmed RTC decision.
Injunction was proper, because the demolition would have violated the
Hence this petition.
rights of Sps. Madrona.
ISSUE/s:
DOCTRINE: For injunction to issue, two requisites: First, there must be a
right to be protected and second, the acts against which the injunction is WoN the trial court err in reinstating the complaint of respondents– NO
WoN the requisites for the issuance of a writ of injunction are present – YES
to be directed are violative of said right.
WoN petitioner is liable to pay attorney’s fees and costs of suit - YES

RULING: Dimissed.

FACTS: RATIO:
Sps. Madrona are registered owners of a residential property in Marikina It did not appear that respondent lost interest in prosecuting their case nor
City. In 1989, they builty their house thereon and enclosed it with a was their counsel negligent in handling it. Accordingly, there was no
concrete fence and steel gate. basis for the dismissal order and reinstatement of respondent’s
Sps. Madrona received a letter from Jame Perez, Chief of the Marikina complaint was justified.
Demolition Office. For injunction to issue, two requisites: First, there must be a right to
Sps. Madrona responded that the accusation was libelous, as it is be protected and second, the acts against which the injunction
condemnin him and his property without due process; no basis and is to be directed are violative of said right.
authority since there is no court order authorizing him to demolish In the case it is present, respondents right over their concrete fence
their structure; no express authority to do so; and contained a fals which cannot be removed without due process; and the act, the
accusation since their fence did not in fact extend to the sidewalk. summary demolition of the concrete fence, against which the
Perez, required them to provide the relocation survey on the subject injunction is directed, would violate said right.
property. But did not oblige. Respondents, for their part, counter that the presumption of regularity
More than a year later, Perez sent another letter with the same contents has been negated by the fact that despite ther reply to the first notice,
as receipt thereof to remove the structure allegedly protruding to the which put petitioner on notice that what he was doing was ultra vires,
sidewalk. This prompted the Sps. Madrona to file a complaint for eh still reiterated his earlier demand and threat of demolition.
injunction before the RTC. As respondents were forced to file a case against petitioner to enjoin the
Sps. Madrona likewise sought the issuance ofa TRO and a writ of impending demolition of their property, the award of attorney’s fees
preliminary injunction to enjoin Perez and all persons acting under and costs of suit is justified.
him from doing any act of demolition on their property and that after And when petitioner did not get the “proof” he was requiring from
trial, the injunction be made permanent. respondents, he again sent a notice with a threat of summary
RTC issued a TRO against Perez. Perez filed an Urgent Ex Parte, demolition. This gave respondents no other choice but to file an
nevertheless failed to respond on motion of respondents, declaring him injunction complaint against petitioner to protect their rights.
in default.
Perez filed a motion to lift order of default because of voluminous work
load as lone lawyer. The RTC issued an order denying the motion to
lfie the order of default. Perez filed MR but was denied.
DPWH v. SPOUSES TECSON (JP) The respondent spouses then initiated this present motion for reconsideration
April 21, 2015 | Peralta, J. | Topic of the above SC decision praying for the “justness of the miserable
amount of compensation being awarded to the herein respondents.”
PETITIONER: Secretary of DPWH and Dsitrict Engineer Celestino R.
Contreras
RESPONDENTS: Spouses Heracleo and Ramona Tecson

SUMMARY: The present case is a motion for reconsideration of a


previous SC decision which awarded only P0.70 per square meter for the
properties of spouses Tecson for the their propery expropriated by the
Government. They demanded the fair market value of P1,500 per square
meter. The SC however, affirmed their previous decision and explained
that it is in the interest of just compensation and also for public good that
they use the fair market value at the time of taking of the property in
1940, as supported by several jurisprudence.

DOCTRINE: The Government’s failure, to initiate the necessary


expropriation proceedings prior to actual taking cannot simply invalidate
the State’s exercise of its eminent domain power. To hastily nullify said
expropriation in the guise of lack of due process would certainly diminish
or weaken one of the State’s inherent powers, the ultimate objective of
which is to serve the greater good.

FACTS:
In 1940, the Department of Public Works and Highways (DPWH) took
respondents-movants subject property without the benefit of
expropriation proceedings for the construction of the MacArthur
Highway.
Celestino R. Contreras (Contreras), then District Engineer of the First
Bulacan Engineering District of the DPWH, offered to pay for the
subject land at the rate of Seventy Centavos (P0.70) per square
meter, per Resolution of the Provincial Appraisal Committee (PAC) of
Bulacan.
Respondents-movant spouses demanded the return of their property, or
the payment of compensation at the current fair market value.
Hence, the complaint for recovery of possession with damages filed
by respondents-movants.
Respondents-movants were able to obtain favorable decisions in the
Regional Trial Court (RTC) and the Court of Appeals (CA), with the
subject property valued at P1,500 per square meter, with interest at
6% per annum.
Spouses then elevated the matter to the SC for certiorari. The SC then
however, with contrasting opinions, did not agree with both courts
and ruled instead that just compensation should be based on the
value of the property at the time of taking in 1940, which is P0.70
per square meter with 6% interest.
ISSUE/s: Velasco, dissenting: The government’s exercise of eminent domain is
WoN the amount received by respondent spouses in the assailed decision not absolute. It is subject, first and foremost, to constitutional
was just compensation for the taking of their property by the restrictions
government? – YES
WoN the respondent spouses due process is violated - NO

RULING: The motion for reconsideration is denied.

RATIO:
In this regard, the SC reiterates the doctrines laid down in the cases of
Forfom v. PNR, Eusebio v. Luis, MIAA v. Rodriguez, and Republic v.
Sarabia. In these cases, the Court has uniformly ruled that the fair
market value of the property at the time of taking is controlling for
purposes of computing just compensation.
As in the aforementioned cases, just compensation due respondents-
movants in this case should, therefore, be fixed not as of the time of
payment but at the time of taking in 1940 which is P0.70 and not
P1,500 per square meter, as valued by the RTC and CA.
While disparity in the above amounts is obvious and may appear
inequitable to respondents-movants as they would be receiving such
outdated valuation after a very long period, it should be noted that
the purpose of just compensation is not to reward the owner for the
property taken but to compensate him for the loss thereof.
The award of interest on the value of the land at the time of taking in
1940 until full payment is adequate compensation to respondents-
movants for the deprivation of their property without the benefit of
expropriation proceedings. Also, adding the interest computed to the
market value of the property at the time of taking signifies the real,
substantial, full and ample value of the property in order to place the
owner in a position as good as (but not better than) the position he
was in before the taking occurred.
Respondents-movants were deprived of beneficial ownership over their
property for more than seventy (70) years without the benefit of a
timely expropriation proceedings, and to serve as a deterrent to the
State from failing to institute such proceedings within the prescribed
period under the law, a grant of exemplary damages in the amount of
P1,000,000 is fair and reasonable. Moreover, an award for attorney’s
fees in the amount of P200,000 in favor of respondents-movants is in
order.
The Government’s failure, to initiate the necessary expropriation
proceedings prior to actual taking cannot simply invalidate the
State’s exercise of its eminent domain power, given that the
property subject of expropriation is indubitably devoted for public
use, and public policy imposes upon the public utility the
obligation to continue its services to the public. To hastily nullify
said expropriation in the guise of lack of due process would
certainly diminish or weaken one of the State’s inherent powers,
the ultimate objective of which is to serve the greater good.
enshrined in the Bill of Rights, viz.: Section 1. No person shall be deprived
of life, liberty, or property without due process of law, nor shall any person
be denied the equal protection of the laws. x x x x Section 9. Private property
shall not be taken for public use without just compensation. Respondents
were deprived of due process of law. The government failed to discharge
its burden of initiating condemnation proceedings prior to taking private
property. Since there was no deposit also, which serves as the assurance
that the landowner would pe paid, the taking of private property should
be illegal for lack of just compensation, in violation of due process.
ESTRADA v. OMBUDSMAN (HENRY) ISSUE/s:
January 21, 2015 | Carpio, J. | Procedural Due Process WoN the Ombudsman violated the constitutional right of petitioner
Estrada to due process when it issued the Order charging him of
PETITIONER: Sen. Jinggoy Estrada Plunder - NO
RESPONDENTS: Office of the Ombudsman, Field Investigation Office, et
al.

SUMMARY: Ombudsman has served petitioner Estrada with a copy of


complaints filed by NBI and Atty. Baligod, concerning criminal charges of
plunder against him. Petitioner filed a counter but he was still served.
Claiming that his constitutional right to due process was violated by the
Office of the Ombudsman when he was not provided with the counter-
affidavits of the respondnents, and charging him with Plunder. The SC
ruled that since the petitioner failed to actually specifu a rule that
mandates such requirement, the law is valid and no violation to due
process was committed.

DOCTRINE: The right to due process in administrative cases as


prescribed Ang Tibay, as amplified in GSIS, are granted by the
Constitution; hence these rights cannot be taken away by mere
legislation. On the other hand, as repeatedly reiterated by this Court, the
right to a preliminary investigation is merely a statutory right, not part of
the “fundamental and essential requirements” of due process as
prescribed in Ang Tibay and amplified in GSIS.

FACTS:
Ombudsman served petitioner Estrada a copy of the complaint filed by
the NBI and Atty. Baligod, which prayed that criminal proceedings fro
Plunder be conducted against him.
Petitioner Estrada filed his counter affidavit but it did not stop the
Ombudsman to begin with the criminal proceedings for Plunder.
Petitioner again filed a counter affidavit.
18 of Estrada’s corespondents in the two complaints filed their counter-
affidavits. Further, petitioner Estrada filed his Request to be
Furnished with Copies of the Counter-Affidavits of the Other
Respondents, Affidavits of New Witnesses, and Other Filings
(Request), and asked for the affidavits of the respondents—all
pursuant to his right to examine evidence submitted by the
complaint, and his right to have access on the evidence. His request
was denied.
The Ombudsman eventually issued a Joint Resolution which found
probable cause to indict petitioner and his correspondents with 1
count of plunder and 11 counts of violation of Sec 3(e) of RA 3019.
To no avail Estrada in an MR prayed for an issuance of a new
resolution dismissing him of his charges, but to no avail. Hence, this
petition.
and essential requirements” of due process as prescribed in Ang
RULING: SC dismissed the case. Tibay and amplified in GSIS.
RATIO:
The Ombudsman’s denial in its March 27 Order of petitioner Estrada’s
request did not constitute grave abuse of discretion.
There is no law or rule which requires the Ombudsman to furnish a
petitioner with copies of the counter-affidavits of his corespondents.
Petitioner Estrada fails to specify a law or rule which states that it is a
compulsory requirement of due process in a preliminary investigation
that the Ombudsman furnish a respondent with the counter-affidavits
of his correspondents.
Neither Section 3(b), Rule 112 of the Revised Rules of Criminal
Procedure nor Section 4(c), Rule II of the Rules of Procedure of the
Office of the Ombudsman support Sen. Estrada’s claim.
What the Rules of Procedure of the Office of the Ombudsman require is
the Ombudsman to furnish the respondent with a copu of the
complaint and the supporting affidavits and documents at the time
the order to submit the counter-affidavit is issued to the respondent.
Clearly, Sec 4(b) refers to affidavits of the complainant and his
witnesses, not the affidavits of the respondents.
What can be drawn from this is that, no grave abuse of discretion is
attributed to the Ombudsman for the issuance of the Order.Under Rule
112 of the Revised Rules of Criminal Procedure or under Rule II of the
Ombudsman’s Rules of Procedure, there is no requirement whatsoever
that the affidavits executed by the corespondents should be furnished to
a respondent.
Given that it is the Ang Tibay v. CIR case that shows the due process
standards at the very least that should be conducted in a preliminary
investigation, it does not apply to preliminary investigations in
criminal cases. Its application will have absurd and disastrous
consequences.
The Ang Tibay case enumerated the constitutional requirements of due
process. These requirements are fundamental and essential
because without these, there is no due process as mandated by the
Constitution. These fundamental and essential requirements canot
be taken away by legislation because they are part of constitutional
due process.
Another is the case of GSIS v. CA, which affirms the non-applicability of
the Ang Tibay guidelines to preliminary investigations in criminal
cases: the investigating officer will never be the impartial tribunal
required in Ang Tibay, as amplified by the GSIS case. The purpose of
the Office of the Ombudsman in conducting a preliminary
investigation is to determine probable cause for filingan information.
The right to due process in administrative cases as prescribed in Ang
Tibay, as amplified in GSIS, are granted by the Constitution; hence
these rights cannot be taken away by mere legislation. On the other
hand, as repeatedly reiterated by this Court, the right to a preliminary
investigation is merely a statutory right, not part of the “fundamental
AMERICAN INTER-FASHION CORP. v. OFFICE OF THE AIFC now alleges that the GTEB decision is res judicata and that
PRESIDENT (DANNAH) Glorious
May 23, 1991 | Gutierrez Jr., J. | Procedural Due Process: Findings of
Administrative Agencies

PETITIONER: American Inter-Fashion Corp.


RESPONDENTS: Office of the President, Garments and Textile Export
Board,
Glorious Sun Fashion

SUMMARY: Glorious Sun’s export quotas were cancelled after the


Garments and Textile Export Board found out that Glorious was allegedly
guilty of dollar salting. Glorious filed an appeal in the SC but
subsequently withdrew it, because of the alleged pressure from Mr.
Ongpin. The export quotas were then given to American Inter-Fashion
and De Soleil—alleged Marcos-crony operated corporations.

Two years later, Glorious filed an appeal with the Office of the President
which reversed the decision of GTEB and remanded it to them because
Glorious’ constitutional right to due process was violated. AIFC intervened
with Glorious’ appeal, claiming that the case should be barred because of
res judicata, and that Glorious was accorded due process.

The Court ruled in favor of Glorious, stating that it was not barred by res
judicata because its dismissal was not a judgment on the merits, and the
GTEB proceedings violated Glorious’ right to due process.

DOCTRINE: Findings of administrative agencies are accorded respect


and finality, and generally should not be disturbed by the court. However,
such factual findings may be disregarded when they “are not supported
by evidence; where the findings are initiated by fraud, imposition or
collusion; where the procedures which lead to the factual findings are
irregular; when palpable errors are committed; or when grave abuse of
discretion is manifest”.

FACTS:
The Garments and Textile Export Board (GTEB) found Glorious Sun gulty
of misdeclaration of imported raw materials resulting in dollar
salting abroad and therefore, its export quotas should be cancelled.
Its quotas were given to 2 newly-formed corps, De Soleil Apparel and
American Inter-Fashion (AIFC).
These two corps were joint ventures of the Hong Kong investors and
majority stockholders of Glorious Sun on one hand, and allegedly a
member of the family and a crony of Former Pres Marcos on the
other.
The Office of the President set aside the GTEB decision and remanded
the case for genuine hearings where due process would be accorded
to both parties.
Sun was given every opportunity to be heard by the Board. The dismissal of said case cannot be categorized as a judgment on the
Glorious Sun has never been sequestered. The records also show that merits.
AIFC’s sequestration has been lifted and apparently only De Soleil
remains sequestered.
The background of the case is as follows:
After said decision of the GTEB, Glorious filed a petition for certiorari with
the Court contending that its right to due process was violated, and
that said decision was not supported by substantial evidence. This
was in June 1984.
The Court issued a resolution ordering GTEB to conduct further
proceedings. However, one month after, Glorious filed a
manifestation of its intention to withdraw the petition.
Another month after, the Court granted Glorious’ motion for withdrawal.
Two years later, Glorious filed a petition for the restitution of its
export quota allocation. Moreover, it alleged that the cancelling of its
export quotas was rendered as a result of duress, threats,
intimidation and undue influence exercised by former Minister
Ongpin in order to transfer Glorious export quotas to Marcos crony-
owner corps.
Glorious also further alleged that it was coerced by Mr. Ongpin to
withdraw its petition.
On September 1987, the GTEB denied the petition of Glorious. An
appeal was taken to the Office of the President, which ruled in favor
of Glorious, finding the proceedings before the GTEB in 1984
irregular.
AIFC sought to intervene and is now assailing the appeal should be
barred on the ground of res judicata, and because they think
Glorious was accorded due process.

ISSUE/s:
WoN the previous GTEB decision constituted res judicata – NO
WoN respondent Glorious was accorded due process in the GTEB
proceedings of 1984 – NO

RULING:

RATIO:
For a judgment to be a bar to a subsequent case, the following requisites
must concur
It must be a final judgment
The court which resolved it has jurisdiction over the subject
matter and the parties
It must be a judgment on the merits
There must be identity between the two cases, as to the parties,
subject matter and cause of action
“A judgment on the merits is one rendered after a determination of which
party is right, as distinguished from a judgment rendered upon
preliminary or final or merely technical point.”
The dismissal was based solely on the notice of withdrawal by Glorious, and
not on the merits of the petition.
Moving to the issue on due process, the GTEB in the 1984 hearings
failed to disclose to Glorious vital evidence they used in arriving at
their conclusion that Glroious was guilty of dollar-salting.
The subsequent disclosure in 1987 of the GTEB to Glorious of the
documents did not cure the defect of non-disclosure of evidence.
The glaring fact is that Glorious was denied due process when the GTEB
failed to disclose evidence used by it in rendering its resolution (see
GTEB ratio in original case)
Moreover, just because other members would not agree that there was
pressure from Minister Ongpin to cancel the export quotas of
Glorious does not mean that Mr. Puno (who assailed said pressure
from Ongpin) was not telling the truth.
Findings of administrative agencies are accorded respect and finality,
and generally should not be disturbed by the court. However, such
factual findings may be disregarded when they “are not supported by
evidence; where the findings are initiated by fraud, imposition or
collusion; where the procedures which lead to the factual findings are
irregular; when palpable errors are committed; or when grave abuse
of discretion is manifest”.
Glorious’ constitutional right to due process was violated because
Glorious was never given the chance to present its side before its
export quota allocations were revoked and its officers suspended.
**In case it is asked: the case does not fall under the jurisdiction of
Sandiganbayan.
US v. TORIBIO (IYA) theft and to make easy the recovery and return of such cattle to their
January 26, 1910 | Carson, J. | Substantial Due Process proper owners, when lost, strayed, or stolen.
That the law in question "is not a taking of the property for public use, within
PETITIONER: The United States the meaning of the constitution, but isa just and legitimate exercise of the
power of the legislature to regulate and restrain such particular use of
RESPONDENTS: Luis Toribio
the property as would be inconsistent with or injurious to the rights of the
public.”
SUMMARY: Toribio caused the slaughter of a carabao without first
Rights of properly are subject to such reasonable limitations in their
securing a permit form the municipal treasurer. Act No. 1147 (sections 30
enjoyment as shall prevent them from being injurious, and to such
and 33) prohibit the slaughter of large cattle in a municipal
reasonable restraints and regulations established by law, as the
slaughterhouse without prior permit from the treasurer. Toribio contends
legislature, under the governing and controlling power vested in them
the proper construction of these provisions as he claims that he should
by the constitution, may think necessary and expedient.
not be liable since there is no municipal slaughterhouse in their area,
The restraint placed by the law on the slaughter for human consumption
therefore the law does not apply.
of carabaos fit for agricultural work and draft purposes is not an
The SC interpreted the provisions to mean to apply to all slaughters, appropriation of property interests to a "public use," and is not,
therefore, within the principles of the exercise by the State of the right
wherever they may be committed. This is in line with the intent of the law
of eminent domain.
to prevent slaughter of cows owned by other persons and to ensure that
It is in fact a mere restriction or limitation upon a private use, which the
no unhealthy or virus infected cow will be sold for human consumption.
legislature deemed to be detrimental to the public welfare.
DOCTRINE: The State may interfere in the property rights of persons The State may interfere wherever the public interests demand it, and in
wherever the public interests demand it, and in this particular a large this particular a large discretion is necessarily vested in the
legislature to determine, not only what the interests of the public
discretion is necessarily vested in the legislature to determine, not only
require, but what measures are necessary for the protection such
what the interests of the public require, but what measures are
interests.
necessary for the protection such interests.

FACTS:
Toribio slaughtered or caused to be slaughtered for human consumption,
a carabao without a permit from the treasurer of the municipality.
The act is in violation of provisions of Section 30 and 33 of Act No. 1147
or an Act regulating the registration, branding, and slaughter of large
cattle.
In the town of Carmen, Bohol, there is no municipal slaughterhouse. Toribio
contends that under such circumstances the provisions of Act No. 1147
does not apply to his case as the requirement of permit before slaughter
applies only where the act is to be done in a municipal slaughterhouse.

ISSUE/s:
WON Sections 30 and 33 is valid as it does not impair the right of person
to property - YES

RULING: The judgement of conviction and sentence imposed by the trial


court is affirmed.

RATIO:
The SC is in the opinion that the proper construction of the provisions in
question is that it should apply the slaughters of large cattle
wherever it may be done.
The Act primarily seeks to protect the “large cattle” of the Philippines against
YNOT v. IAC (ELIEL) station commander, were returned to the petitioner only after he had
March 20, 1987 | Cruz, J. | Substantial Due Process filed a complaint for recovery and given a supersedeas bond of
P12,000., which was ordered confiscated upon his failure to produce
PETITIONER: Restituto Ynot the carabaos when ordered by the trial court.
The executive order defined the prohibition, convieted the petitioner and
RESPONDENTS: Intermediate Appellate Court, Integrated Nation Polic,
and the Regional Director, Bureau of Animal Industry immediately imposed punishment, which were carried out forthright.
The measures struck at once and pounce upon the petitioner without
SUMMARY: President Marcos issued EO 626-A prohibiting the movement giving him a chance to be heard, thus denying him the centures-old
and slaughtering of carabaos. Ynot was transporting six carabaos in a pump guaranty of elementary fair play.
boat from Masbate to Iloilo when they were confiscated by the police station Due process is violated because the owner of the property confiscated is
commander. Ynot sued for recovery but were unable to be produced. RTC denied the right to be heard in his defense and is immediately
sustained the confiscation, and appealed to the IAC, upholding the same. condemned and punished.
Hence this petition.

DOCTRINE: Due process is violated because the owner of the property


confiscated is denied the right to be heard in his defense and is
immediately condemned and punished.

FACTS:
EO 626-A prohibiting interprovincial movement of carabaos and the
slaughtering of carabaos.
Ynot transported six carabaos in a pump boat from Masbate to Iloilo,
when they were confiscated by the police station commander of
Barotac Nuevo, Iloilo, for violation of EO 626-A
Ynot sued for recovery, and the RTC issued a writ of replevin upon filing
of bond. RTC sustained confiscation, since they could not longer be
produced, order the confiscation of the bond.
Ynot appealed the decision to the IAC, which upheld the RTC decision.
Hence this petition. Alleging that the EO is unconstitutional as it violates
the due process.

ISSUE/s:
WoN EO 626-A violates due process– YES

RULING: EO 626-A is declared unconstitutional.

RATIO:
The minimum requirements of due process are notice and hearing, which
generally speaking, may not be dispensed with because they are
intended as a safeguard agasint official arbitrariness.
There are instances, previous judicial hearing may be omitted without
violation of due process in view of the nature of the property involved
or the urgency of the need to protect the general welfare from a clear
and present danger.
In the instant case, the carabaos were arbitrarily confiscated by the police
CHURCHILL v. RAFFERTY (JP) are unconstitutional because (a) they attempt to deprive aggrieved
December 21, 1915 | Trent, J. | Substantive Due Process taxpayers of all substantial remedy for the protection of their property,

PETITIONER: Francis Churchill and Stewart Tait


RESPONDENTS: James Rafferty, Collector of Internal Revenue

SUMMARY: A judgment was rendered in favor of plaintiffs (Churchill and


Tait) where the respondent Collector of Internal Revenue was (1)
restrained from collecting the plaintiff’s property for the annual tax
described in Section 11 of Act No. 2339 and likewise (2) restrained from
destroying or removing signboards and billboards which are the property
of plaintiffs for being offensive to sight. The SC ruled that such judgment
should be reversed because the courts can in fact order the execution of
propertis summarily because it is upon taxation that government chiefly
relies for means. The SC also ruled that the power to remove signboards
and billboards which are a nuisance is validly under the police power of
the state because it is associated with the general welfare of the public,
not merely for aesthetic reasons.

DOCTRINE: "Due process of law" does not always require, in respect to


the Government, the same process that is required between citizens.

It is upon taxation that the Government chiefly relies to obtain the means
to carry on its operations, and it is of the utmost importance that the
modes adopted to enforce the collection of the taxes levied should be
summary and interfered with as little as possible.

The regulation of billboards and their restriction is not so much a


regulation of private property as it is a regulation of the use of the streets
and other public thoroughfares.

FACTS:
A judgment was rendered which perpetually restrains and prohibits the
defendant (CIR Collector Rafferty) and his deputies from collecting
and enforcing against plaintiffs Churchill and Tait their property for
the annual tax due to them. It also restrains and prohibits them from
destroying or removing any sign, or billboard, the property of the
plaintiffs, for the sole reason that such sign, or billboard is or maybe
offensive to sight.
As for the first issue: The sections of Act No. 2339, which bear directly
upon the subject, are 139 and 140. The firt (139) expressly forbids
the use of an injunction (as a remedy) to stay the collection of any
internal revenue tax; the second (140) provides a remedy for any
wrong in connection with such taxes, and this remedy was intended
to be exclusive, thereby not including the remedy by injunction.
Counsel for plaintiffs (Churchill and Tait) urge that the two sections (139
&
thereby, in effect, depriving them of their property without due A comparison of these two sections show that they are essentially
process of law (because they deprived of filing for an injunction, a the same. Both expressly prohibit the restraining of taxes by
supposed to be constitutional remedy), and (b) they attempt to injunction. If
diminish the jurisdiction of the courts.
For the second issue: counsel for the plaintiffs contend that that portion
of section 100 of Act No. 2339, empowering the Collector of Internal
Revenue to remove billboards as nuisances, if objectionable to the
sight, is unconstitutional, as constituting a deprivation of property
without due process of law.
ISSUE/s:
WoN the court has power to restrain by injunction the collection of the tax
complained of – NO
WoN the provisions of subsection (b) of section 100 of Act No. 2339,
conferring power upon the Collector of Internal Revenue to remove
any sign, signboard, or billboard upon the ground that the same is
offensive to the sight or is otherwise a nuisance are valid - YES

RULING: The judgment appealed from is hereby reversed and the action
dismissed upon the merits (refer to Fact No. 1 for the judgment).

RATIO:
It is also the settled law in the United States that "due process of law"
does not always require, in respect to the Government, the same
process that is required between citizens, though it generally implies
and includes regular allegations, opportunity to answer, and a trial
according to some well settled course of judicial proceedings.
A citizen's property, both real and personal, may be taken, and
usually is taken, by the government in payment of its taxes
without any judicial proceedings whatever. In this country, as
well as in the United States, the officer charged with the
collection of taxes is authorized to seize and sell the property of
delinquent taxpayers without applying to the courts for
assistance, and the constitutionality of the law authorizing this
procedure never has been seriously questioned.
This must necessarily be the course, because it is upon taxation
that the Government chiefly relies to obtain the means to carry
on its operations, and it is of the utmost importance that the
modes adopted to enforce the collection of the taxes levied
should be summary and interfered with as little as possible. No
government could exist if every litigious man were permitted to
delay the collection of its taxes. This principle of public policy
must be constantly borne in mind in determining cases such as
the one under consideration.
The provision assailed, Section 139, is found in substance in the
Constitution of the United States and in that of every state in the
Union, specifically Section 3224 of the Revised Statutes of the
United States.
the Supreme Court of the United States has clearly and definitely held
that the provisions of section 3224 do not violate the "due process of law"
and "equal protection of the law" clauses in the Constitution, we would
be going too far to hold that section 139 violates those same provisions in
the Philippine Bill. That the Supreme Court of the United States has so
held, cannot be doubted.
It can be said judicially that signs, signboards, and billboards, which are
admittedly offensive to the sight, are with the category of things
which interfere with the public safety, welfare, and comfort, and
therefore within the reach of the police power of the Philippine
Government.
The sense of sight is the primary essential to advertising success.
Billboard advertising is conducted out of doors and along the arteries
of travel and compels attention by the strategic location of the
boards, which obstruct the range of vision at points where traveler
are most likely to direct their eyes. It is quite natural for people to
protest against indiscriminate and wholesale use of the landscape by
advertisers.
The success of billboard advertising depends not so much upon the use
of private property as it does upon the use of the channels of travel
used by general public. The real and sole value of the billboard is its
proximity to the public thoroughfares. Hence, the SC conceives
that the regulation of billboards and their restriction is not so
much a regulation of private property as it is a regulation of the
use of the streets and other public thoroughfares. (They should
not be treated as private property and therefore be claimed as
deprivation of private property and hence deprive of due
process.)
The unsightly advertisements or signs, signboards, or billboards which
are offensive to the sight, are not disassociated from the general
welfare of the public. This is not establishing a new principle, but
carrying a well-recognized principle to further application. Therefore,
their regulation is constitutional.
PEOPLE v. FAJARDO (HENRY)
August 29, 1958 | Reyes J.B.L., J. | Procedural Due Process ISSUE/s:
WoN the Municipal Ordinance No. 7 is constitutional that it does not valid
PETITIONER: People of the Philippines rights to due process – NO
RESPONDENTS: Juan Fajardo, et al.

SUMMARY: When Fajardo was mayor of the municipality of Baao, he


passed Ordinance No. 7, requiring people to obtain a permit from the
Municipal Mayor building permits should they decide to construct a
project. After his term as mayor expired, he and his son -in-law sent a
letter to the mayor who apparently denied their request, due to its
obstruction of the view of the public plaza. This request was reiterated
once more only to be turned down for the second time. However, despite
the denial of the request Fajardo pursued the construction, for reasons
that they needed a house since their old one was destroyed by a
typhoon. Aggrieved, they appealed to the CA who forwarded their appeal
to the SC, who then ruled against the constitutionality of the questioned
ordinance, due to the fact that it gives too much power and discretion to
the mayor as to when he will issue or deny permits.

DOCTRINE: Such undefined and unlimited delegation of power to allow


or prevent an activity, per se lawful, is invalid.

FACTS:
During the incumbency of the Fajardo as mayor of the municipality of
Baao, camarines Sur, the municipal council passed the Ordinance
No. 7 (series of 1950) which provided a penalty/fee to those who will
constructs or repairs a building without obtaining a permit from the
Municipal Mayor
4 years later, after the term of Fajardo expired, he and his son-in-law filed
a written request with the incumbent municipal mayor for a permit to
construct a building adjacent to their gasoline station on a parcel of
land registered in Fajardo’s name, located along the national
highway. Request denied, as it was said to destroy the beautiful view
of the public plaza.
Fajardo reiterated their request for the permit but was once again turned
down. Apparently they proceeded with the construction even without
the permit, as they needed a place for residence very bad, since
their former house was destroyed by a typhone and they were living
on leased property.
Fajardo et al. were charged before and convicted by the justice of the peace
court of Baao for violation of the ordinance in question. They appealed in
the CFI, which affirmed the conviction and sentenced them to a fine of
Php 35.00 each and costs, as well as to demolish the building as it
destroys the view.
The accused appealed to the CA who forwarded the records to the SC
since the appeal attachs the constitutionality fo the ordinance in
question.
RULING: SC ruled that Municipal Ordinance No. 7 was beyond the authority
of said municipality to enact, and is therefore null and void.

RATIO:
This ordinance is giving the mayor unlimited or absolute discretion to
issue or deny a permit. The ordinance thus confers upon the mayor
arbitrary and unrestricted power to grant or deny the issuance of
building permits, and it is a settled rule that such undefined and
unlimited delegation of power to allow or prevent an activity, per se
lawful, is invalid.
This ordinance is clothed with the uncontrolled power to capriciously
grant the privilege to send and deny it to others; to refuse the
application of one landowner or lessee and to grant that of another,
when for all material purposes, the two applying for precisely the
same priveleges under the same circumstances.
Even though the refusal of the issuance of the permit was due to the sole
reason provided for in Sec. 3 of the ordinance (ruining of the view of
the public plaza), it is still unreasonable and oppressive, in that it
operates to permanently deprive appellants of the right to use their
own property; hence, it oversteps the bounds of police power, and
amounts to a taking of Fajardo’s property without just compensation.
The SC does not overlook that the modern tendency is to regard the
beautification of neighborhoods as conducive to the comfort and
happiness of residents. But while property may be regulated in the
interest of the general welfare, and in its pursuit, the State may
prohibit structures offensive to the sight (Churchill and Tait v.
Rafferty).
As the case now stands, every structure that may be erected regardless
of its own beauty, stands condemned under the ordinance in
question because it would interfere with the view of the public plaza
from the highway.
BALACUIT v. CFI OF AGUSAN DEL NORTE (DANNAH) hereby rendered declaring Ordinance No. 640 unconstitutional and,
June 30, 1988 | Gutierrez Jr., J. | Substantive Due Process: Police therefore, null and void. This decision is immediately executory.
Power

PETITIONER: Carlos Balacuit, Lamberto Tan and Sergio Yu


RESPONDENTS: Court of First Instance of Agustan Del Norte, et al

SUMMARY: The Municipal Board of Butuan passed Ordinance 640,


penalizing those persons/entities who will not sell tickets to children aged
7-12 for half the price. Petitioners are managers of Theaters and they
assail the constitutionality of said Ordinance for being violative of their
rights.

A TPO was issued by the CFI of Agusan, but its final decision declared
said Ordinance valid and constitutional. Petitioners appealed to the SC
which reversed the ruling, stating that it was an invalid exercise of police
power, because it encroaches upon petitioners’ property rights, without
valid reasons.

DOCTRINE: The exercise of police power by the local government is


valid unless it contravenes the fundamental law of the land, or an act of
the legislature, or unless it is against public policy or is unreasonable,
oppressive, partial, discriminating or in derogation of a common right.

FACTS:
The Municipal Board of Butuan City passed Ordinance 640, which states
that any person, group of persons, entity or corporation engaged in
the business of selling admission tickets…. should charge only one
half of said ticket price to children between 7 and 12 years old.
Those violating will be punished by a fine not less than P200 but not
more than P600 or imprisonment of not less than 2 mos but not more
than 6 mos.
Petitioners Balacuit, Tan and Yu are managers of the Maya and Dalisay
Theaters, the Crown Theater and Diamon Theater, respectively.
They filed a complaint before the CFI of Agusan praying, interalia, that
said ordinance be declared unconstitutional and therefore, void and
unenforceable.
A temporary restraining order was issued by the CFI. However, after both
parties filed their answers, the CFI declared said ordinance valid and
constitutional. Hence, this petition.

ISSUE/s:
WoN Ordinance 640 is a valid exercise of police power – NO

RULING: WHEREFORE, the decision of the trial court in Special Civil Case
No. 237 is hereby REVERSED and SET ASIDE and a new judgment is
RATIO: or conditional, as the case may be, according to the terms of the original
Applying the ruling in Kwong Sing v. City of Manila, where the word contract of sale. This
"regulate" was interpreted to include the power to control, to govern
and to restrain, it would seem that under its power to regulate places
of exhibitions and amusement, the Municipal Board of the City of
Butuan could make proper police regulations as to the mode in which
the business shall be exercised.
The City of Butuan, realizing that it has no authority to enact said
ordinance invokes the police power as delegated to it under the
general welfare clause to justify enactment of said ordinance.
To invoke the exercise of police power, not only must it appear that
the interest of the public generally requires an interference with
private rights, but the means adopted must be reasonably
necessary for the accomplishment of the purpose and not
unduly oppressive upon individuals.
The determination as to what is a proper exercise of police power is not
final or conclusive, but is subject to the supervision of the courts.
Individual rights may be adversely affected by the exercise of police
power to the extent only—and only to the extent—that may be fairly
required by the legitimate demands of public interest or public
welfare.
However, a review of the intent of encacting said ordinance shows that
the reason behind it is the complaints of parents that tickets are too
financially burdensome.
The ordinance is not justified by any necessity for the public interest. The
police power legislation must be firmly grounded on public interest
and welfare, and a reasonable relation must exist between purposes
and means.
A reduction in the price of admission would mean corresponding savings
for the parents; however, the petitioners are the ones made to bear
the cost of these savings. The ordinance does not only make the
petitioners suffer the loss of earnings but it likewise penalizes them
for failure to comply with it.
We can see that the ordinance is clearly unreasonable if not unduly
oppressive upon the business of petitioners.
There is nothing immoral or injurious in charging the same price for both
children and adults. In fact, no person is under compulsion to
purchase a ticket. It is a totally voluntary act on the part of the
purchaser if he buys a ticket to such performances.
A theater ticket has been described to be either a mere license, revocable at
the will of the proprietor of the theater or it may be evidence of a contract
whereby, for a valuable consideration, the purchaser has acquired the
right to enter the theater and observe the performance on condition that
he behaves properly. Such ticket, therefore, represents a right, positive
right is clearly a right of property.
In no sense could these businesses be considered public utilities. The
State has not found it appropriate as a national policy to interfere
with the admission prices to these performances.
A police measure for the regulation of the conduct, control and operation
of a business should not encroach upon the legitimate and lawful
exercise by the citizens of their property rights.
The right of the owner to fix a price at which his property shall be sold or
used is an inherent attribute of the property itself and, as such, within
the protection of the due process clause.
Hence, the proprietors of a theater have a right to manage their property
in their own way, to fix what prices of admission they think most for
their own advantage, and that any person who did not approve could
stay away.
The exercise of police power by the local government is valid unless it
contravenes the fundamental law of the land, or an act of the
legislature, or unless it is against public policy or is unreasonable,
oppressive, partial, discriminating or in derogation of a common right.
AGUSTIN v. EDU (IYA)
February 2, 1979 | Fernando, J. | Substantial Due Process RULING: The petition is dismissed.

RATIO:
PETITIONER: Leovillo C. Agustin
The assailed Letter was a valid exercise of the police power and the
RESPONDENTS: Hon. Romeo F. Edu, Land Transportation implementing rules and regulation of Edu is not an unlawful
Commissioner, et al. delegation of legislative power.
Police power is the authority to enact legislation that may interfere with
SUMMARY: Marcos enacted an administrative order (Letter of
personal liberty or property in order to promote the general welfare.
Instruction 299) directing motorists to have in their vehicles early warning
Person and property could be subject to all kinds of restraints and
devices. Edu, a motorist, questions this order as it is unduly oppressive
borders in order to secure the general comfort, health, and prosperity
since most vehicles are equipped with blinking lights which could serve
of the state.
as EWD. The Letter of Instruction by Marcos is to conform to the 1968 Vienna
Convention on Road Signs and Signals which was ratified by the PH
The SC holds that motorists need not purchase new EWDs, the law
under PD No. 207. It was promulgated to ensure safety in the road.
merely seeks to ensure the safety of people in the road. The law is
Motorists are not required by the law to purchase new EWDs, they may
therefore a valid exercise of police power.
even make their own devices so long as it fits the standards set by
DOCTRINE: Police power is the authority to enact legislation that may the law.
Teehankee, J., dissenting: Teehankee believes that the Letter of
interfere with personal liberty or property in order to promote the general
Instruction is oppressive, unreasonable, arbitrary, consolatory, and
welfare. Person and property could be subject to all kinds of restraints and
unconstitutional not he following considerations:
borders in order to secure the general comfort, health, and prosperity of the
It require owners of motor vehicles with built-in and more
state.
effective and efficient EWDs to purchase that specified in the
administrative order whose effectivity and utility have yet to
be demonstrated.
FACTS: Public necessity for the order has yet to be shown.
The present case questions the validity of a Letter of Instruction providing It adds financial burden on all motorists.
for an early warning device for motor vehicles is assumed in this No real effort has been made to show that there can be practical
prohibition proceeding as violative of the constitutional guarantee of and less burdensome alternative road safety devices.
due process and for transgressing the fundamental principle of non- No imperative need for imposing such blanket requirement on all
delegation of legislative power. vehicles
Marcos issued the assailed Letter of Instruction NO. 229 which directs all
owners and users of motor vehicles have at all times in their vehicles
at least one pair of early warning device. The letter also grants with
the Land Transportation Office the power to promulgate rules and
regulations appropriate to implement the said order.
Petitioner, Edu, owner of a Volkswagen Beetle Car, questions the said
Letter. Saying that his vehicle is already equipped with blinking lights
which would serve well as an early warning device.
For Edu the law is “oppressive, unreasonable, arbitrary, confiscatory, and
unconstitutional.” Edu maintains that it is the manufacturers and
dealers of EWDs that will benefit and would only add burden and
unnecessary expense to motorists.
He prayed for a judgement declaring the Letters of Instruction void and
unconstitutional.

ISSUE/s:
WON the Letter of Instruction is void and unconstitutional - NO
MAGTAGAS v. PRYCE PROPERTIES (ELIEL) CDO and Magtagas filed a petition for review under Rule 45 of the Rules
March 20, 1987 | Cruz, J. | Substantial Due Process of Court. Hence this petition.

PETITIONER: Mayor Pablo Magtagas & The City of CDO


RESPONDENTS: Pryce Properties Corporation, Inc. & PAGCOR

SUMMARY: PAGCOR decided to expand its operations in CDO, by


leasing the property owned by Pryce Corp. CDO and its mayor enacted
ordinances prohibiting the establishment of games of chances. Pryce
assailed the ordinances, to which the CA rule in their favor. When the MR
was denied, CDO and its mayor petitioned in the SC. Denying their
petition because it violates the substantive requirements of an ordinance:
(5) because it contravenes public policy and PD 1869.

DOCTRINE: An ordinance must conform ot the following substantive


requirments: (1) It must not contravene the constitution or any statute; (2)
it must not be unfair or oprressive; (3) it must not be partial or
discriminatory; (4) it must not prohibit but may regulate trade; (5) it must
be general and consistent with public policy; (6) it must not be
unreasonable.

FACTS:
PAGCOR is a corporation created directly by PD 1869 to help centralize
and regulate all games of chance, including casinos on land and sea
within the territorial jurisdiction of the Philippines.
CDO like other local political subdivision is empowered to enact
ordinacnes for the purposes indicated in the LGC. It is expressly
vested with the police power under what is known as the General
Welfare Clause.
PAGCOR decided to expand its operations to CDO.
To this end, it leased a portion of a building belonging to Pryce Properites
Corporation, Inc., one of the herein private respondents, renovated
and equipped the same, and prepared to inaugurate its casino there
during the Christmas season.
The reaction of the Sangguniang Panlungsod of CDO was swift and
hostile. It enacted Ordinance No. 3353, prohibiting any establishment
allowing gambliong or games of chances.
Pryce assailed the ordinances before the CA, where it was joined by
PAGCOR as intervenor and supplemental petitioner. Their challenge
succeeded. The CA declared the ordinances invalid and issued the
writ to prohibit their enforcement. Magtagas’ MR was denied.
The petitioners argue that by virtue of these provisions, the
Sangguninang Panglungsod may prohibit the operation of casinos
because they involve games of chance, which are detrimental to the
people.
ISSUE/s: reconciled, the ordinances should be construed as not applying to
14. WoN the ordinances conform to the substantive requirements– NO PAGCOR.

RULING: Petition Denied.

RATIO:
The morality of gambling is not a justiciable issue. Gambling is not illegal
per se. While it is generally considered inimical to the interests of the
people, there is nothing in the Constitution categoraically proscribing
or penalizing gambling or, for that matter, even mentioning it at all.
An ordinance must conform ot the following substantive
requirments:
(1) It must not contravene the constitution or any statute; (2) it
must not be unfair or oprressive; (3) it must not be partial or
discriminatory; (4) it must not prohibit but may regulate trade;
(5) it must be general and consistent with public policy; (6) it
must not be unreasonable.
The apparent falw in the ordinances in question is that they
contravene PD 1869 and the public policy embodied therein
insofar as they prevent PAGCOR from exercising the power
conferred on it to operate a casino in CDO.
There is no sufficient indication of an implied repeal of PD 1869.
PAGCOR is mentioned as the source of funding in two later
enactmetns of Congress, to wit, RA 7309 creating a Board of
Claims under the DOJ for the benefit of victims of unjust
punishment or detention or of violent crimes, and RA 7648,
providing for measures for the solution of the power crisis.
This would show that the PAGCOR charter has not been repealed by the
LGC but has in fact been improved as it were to make the entity
more responsive to the fiscal problems of the government.
The ordinances violate PD 1869, which has the character and force of a
statute, as well as the public policy expressed in the decree allowing
the playing of certain games of chance despite the prohibition of
gambling in general.
Padilla, J. Concurring: I concur with the majority holding that the city
ordinances in question cannot modify much less repeal PAGCOR’s
general authority to establish and maintain gambling casinos
anywhere in the PH under PD 1869. It is worth remembering in this
regard that, (1) what is legal is not always moral an (2) the ends do
not always justify the means.
Davide, J. Concurring: The nullification by the CA of the challenged
ordinances as unconstitutional primarily because it is in
contravention to PD 1869 is unwarranted. A contravention is not
necessarily a contravention of the constitution. In any case, the
ordinances can still stand even if hey be conceded as offending PD
1869. They can be reconciled, which is not impossible to do. So
BENNIS v. MICHIGAN (JP) prevented from abating petitioner’s interest absent proof that he knew to
March 4, 1996 | Rehnquist, C. J. | Substantive Due Process what

PETITIONER: Tina B. Bennis


RESPONDENTS: Michigan

SUMMARY: Petitioner’s husband was caught engaging in sexual activity


with a prostitute inside a car which she co-owns. The lower court
declared the car abated by virtue of the husband’s conviction of gross
indecency, without any offset for petitioner’s interest. She claims that she
did not be made liable because she had no knowledge of the husband’s
actions and that her due process was violated with the forgeiture, her
being an innocent co-owner. SC declares that based on firmly fixed
punitive and remedial jurisprudence, the government does need the
owner’s consent to abate property.

DOCTRINE: The government may not be required to compensate an


owner for property which it has already lawfully acquired under the
exercise of governmental authority other than the power of eminent
domain. The State here sought to deter illegal activity that contributes to
neighborhood deterioration and unsafe streets.

FACTS:
Petitioner Tina was a joint owner, with her husband, of an 11-year old
Pontiac Sedan.
Petitioner’s husband, John, was caught engaging in sexual activity with a
prostitute inside the aforementioned automobile while it was parked
along Detroit City street.
John Bennis was convicted of gross indecency. The state then sued both
John and his wife, Tina, to have the car declared a public nuisance
and abated as such under Michigan’s compiled laws.
The Wayne County Circuit Court ordered the automobile forfeited and
abated as a public nuisance, with no offset for petitioner’s interest,
even if she claims lack of knowledge of her husband’s activity.
In reaching this decision, the trial court judge recognized the remedial
discretion he had under Michigan’s case law. He took into account
the couple’s ownership of another automobile so they would not be
left without transportation. He also declared his authority to order the
payment of one half of the sale proceeds, after deduction of costs to
the innocent co title holder (Tina). He however delined to order such
division of sale proceeds in this case because of the age and value
of the car (11-year old car bought for $600). He commented that
“there’s practically nothing left.”
The Michigan Court of Appeals reversed and decided that based on a
Michigan Supreme Court precedent interpreting such law, the state is
end the car woud be used. It also ruled that the conduct in question
did not qualify as a public uisnace because only one occurrence was
showen and there was no evidence of payment for the sexual act.
The Michigan Supreme Court reversed the Court of Appeals and
reinstated the abatement in its entirety. It concluded as a matter of
state law that the episode in the Bennis vehicle was an abatable
nuisance. Rejecting the Court of Appeals' interpretation of
§600.3815(2), the court then announced that, in order to abate an
owner's interest in a vehicle, Michigan does not need to prove that
the owner knew or agreed that her vehicle would be used in a
manner proscribed by §600.3801.
Petitioner then filed a writ of certiorari to the Supreme Court of Michigan.

ISSUE/s:
WoN Michigan's abatement scheme has deprived petitioner of her
interest in the forfeited car without due process, in violation of the
Fourteenth Amendment, or has taken her interest for public use
without compensation, in violation of the Fifth Amendment as
incorporated by the Fourteenth Amendment - NO

RULING: Both the trial court and the Michigan Supreme Court followed our
longstanding practice, and the judgment of the Supreme Court of Michigan is
therefore affirmed.

RATIO:
The gravamen of petitioner's due process claim is not that she was denied
notice or an opportunity to contest the abatement of her car; she was
accorded both. Rather, she claims she was entitled to contest the
abatement by showing she did not know her husband would use it to
violate Michigan's indecency law. But a long and unbroken line of cases
holds that an owner's interest in property may be forfeited by reason
of the use to which the property is put even though the owner did
not know that it was to be put to such use.
Our earliest opinion to this effect is Justice Story's opinion for the Court
in The Palmyra: "The thing is here primarily considered as the
offender, or rather the offence is attached primarily to the thing."
Justice Story wrote for the Court that in in rem admiralty proceedings
"the acts of the master and crew . . . bind the interest of the owner of
the ship, whether he be innocent or guilty; and he impliedly submits
to whatever the law denounces as a forfeiture attached to the ship by
reason of their unlawful or wanton wrongs.
"It has long been settled that statutory forfeitures of property entrusted by
the innocent owner or lienor to another who uses it in violation of the
revenue laws of the United States is not a violation of the due
process clause of the Fifth Amendment."
In Calero Toledo v. Pearson Yacht Leasing Co., the most recent decision on
point, the Court reviewed the same cases discussed above, and
concluded that
"the innocence of the owner of property subject to forfeiture has almost
uniformly been rejected as a defense." Petitioner is in the same position as
the various owners involved in the forfeiture cases beginning with The
Palmyra in 1827. She did not know that her car would be used in an illegal
activity that would subject it to forfeiture. But under these cases the Due
Process Clause of the Fourteenth Amendment does not protect her
interest against forfeiture by the government.
Forfeiture also serves a deterrent purpose distinct from any
punitive purpose. Forfeiture of property prevents illegal uses
"both by preventing further illicit use of the [property] and by
imposing an economic penalty, thereby rendering illegal
behavior unprofitable." The State here sought to deter illegal
activity that contributes to neighborhood deterioration and
unsafe streets.
Petitioner also claims that the forfeiture in this case was a taking of
private property for public use in violation of the Takings Clause of
the Fifth Amendment, made applicable to the States by the
Fourteenth Amendment. But if the forfeiture proceeding here in
question did not violate the Fourteenth Amendment, the property in
the automobile was transferred by virtue of that proceeding from
petitioner to the State. The government may not be required to
compensate an owner for property which it has already lawfully
acquired under the exercise of governmental authority other
than the power of eminent domain.
At bottom, petitioner's claims depend on an argument that the Michigan
forfeiture statute is unfair because it relieves prosecutors from the
burden of separating co owners who are complicit in the wrongful
use of property from innocent co owners. This argument, in the
abstract, has considerable appeal, as we acknowledged in
Goldsmith Grant. Its force is reduced in the instant case, however, by
the Michigan Supreme Court's confirmation of the trial court's
remedial discretion, and petitioner's recognition that Michigan may
forfeit her and her husband's car whether or not she is entitled to an
offset for her interest in it.
OPLE v. TORRES (HENRY) and the heads of the government agencies, who as members of the
July 23, 1998 | Puno, J. | Substantive Due Process Inter-Agency Coordinating

PETITIONER: Sen. Blas F. Ople


RESPONDENTS: Ruben D. Torres, et al.

SUMMARY: President Ramos issued AO No. 308, with the goal of easing
government processes and hastening internal procedures by putting all
citizens’ information in an online database. The same issuance is being
assailed for being violative of substantive due process—as it violates the
right to privacy. Despite dissents, the SC ruled that it is indeed violative of
the right to privacy, as it was vague with regard to its processes,
specifically the security and safekeeping of the data of each and every
citizen.

DOCTRINE: The right to privacy as such is accorded to recognition


independently of its identification with liberty; in itself, it is fully deserving
of constitutional protection. Hence if a statute violates such rights, it shall
be unconstitutional.

FACTS:
President Ramos issued AO No. 308, entitled “Adoption of A Natioinal
Computerized Identification Reference System”. AO 308 basically by
its name is a computerized database that addresses the the citizens’
need to conveniently transact business with basic service and social
security providers and other government instrumentalities. (below is
a summary, but feel free to check the AO sa origs J )
With the InterAgency Coordinating Committee (IACC) as the
body that oversees the implementation, this is chaired by the
Executive Secretary.
To provide administrative and technical support, it designates the
National Computer Center (NCC) as secretariat.
Population Reference Number (PRN) generated by the NSO
shall serve as the common referece number to establish a
linkage among concerned agencies.
The government bodies in charge for the information dissemination
are as follows: Office of the Press Secretary, in coordination
with the NSO, GSIS, and SSS as well as other concerned
agencies.
Sen. Ople filed a petition to the SC with the hopes of preventing the
shrinking of the right to privacy. He prays that the SC invalidate AO
No. 308 on two important constitutional grounds:
It is a usurpation of the power of Congress to legislate
It impermissibly intrudes on our citizenry’s protected zone of
privacy
The AO in question was published in 4 newspapers of general
circulation. Respondents: then Executive Secretary Ruben Torres,
Committee, are charged with the implementation of AO No. 308. what particular biometrics technology shall be used to identify who
The SC ordered a TRO enjoining the implementation of the AO in will seek its coverage.
question.

ISSUE/s:
WoN the AO No. 308 is unconstitutional for being violative of
(substantial) due process - YES

RULING: SC granted the petition, Administrative Order No. 308 is declared


null and void for being unconstitutional.

RATIO:
AO No. 308 cannot pass constitutional muster as an administrative
legislation because facially it violates the right to privacy, which
has of its essence the “right to be alone”. The Griswold v.
Connecticut ruling stated that this right has a constitutional
foundation—it can be found within the penumbras of the First,
Third, Fourth, Fifth, and Ninth Amendments.
This was likewise applied in the SC ruling in the case of Morfe v. Mutuc,
it was emphasized that the right to privacy as such is accorded to
recognition independently of its identification with liberty; in itself, it is
fully deserving of constitutional protection.
Moreover, the court pointed out that the right of privacy is recognized
and enshrined in several provisions of our constitution.
Art. 3, Sec. 3(1): The privacy of communication and correspondence
shall be inviolable except upon lawful order of the court, or
when pybkuc safety or order requires otherwise as prescribed
by law
Art. 3, Sec. 2
Art. 3, Sec. 6
Art. 3, Sec. 8
Art. 3, Sec. 17
Other authorities that protect our right to privacy: Civil Code, Revised
Penal
Code, Special Laws (ex: Anti-Wiretapping Law), The Rules of Court.
The SC prescinds the dissenters from the premise that the right to
privacy is a fundamental riht guaranteed by the Constitution, hence,
it is the burden of government to show that AO No. 308 is justified by
some compelling state interest and that it is narrowly drawn.
The court challenged the objectives and the rationale behind the
issuance of such AO against its reasonableness. Its broadness,
vagueness, and overbreath which if implemented will put our
people’s right to privacy in clear and present danger.
The heart of AO No. 308 lies in Sec 4, which provides a PRN that shall
serve as a common reference number to establish linkage among
concerned agencies through Biometrics Technology—and the SC
highlighted that through the progress of technology through time, AO
No. 308 does not state what specific biological characteristics and
Also, the questioned AO does not state whether encoding of data is
limited to biological information alone for identification purposes. The
OSG claims that the adoption of the Identification Reference System
will contribute to the generation of population data for development
planning—the PRN will not be used solely for identification but for
the generation of other data.
The indefiniteness of AO No. 308 can give the government the roving
authority to store and retrieve information for a purpose other than
the identification of the individual through his PRN.
Important to add as well, that the AO does not provide who shall control
and access the data, under what circumstances and for what
purpose. These factors are essential to safeguard the privacy and
guaranty the integrity of the information. Computer linkage gives
other government agencies access to the information—yet, there are
no controls to guard against its leakage.
AO No. 308 falls short of assuring that personal information which will be
gathered about our people will only be processed for unequivocally
specified purposes. Lack of proper safeguards in this regard of
AO No. 308 ay interfere with the individual’s liberty of abode and
travel by enabling authorities to track down his movement; it
may also enable unscrupulous persons to access confidential
information and circumvent the right against self-incrimination;
it may pave the way for “fishing expiditions” by government
authorities and evade the right against unreasonable searches
and seizures.
The reasonableness of a person’s expectation of privacy depends on a
two-part test:
Whether by his conduct, the individual has exhibited an
expectation of privacy
Whether this expectation is one that society recognizes as reasonable
The threat to the right to privacy comes from the executive branch of the
government which by issuing AO No. 308 pressures the people to
surrender their privacy by giving information about themselves on the
pretext that it will facilitate delivery of basic services.
Kapunan, dissenting: The new identification system would
tremendously improve and uplift public service in our country to the
benefit of Filipino citizens and resident aliens. It would promote,
facilitate, and speed up legitimate transactions with government
offices as well as with private and business entities. Experience tells
us of the constant delays and inconveniences the public has to suffer
in availing of basic public services and social security benefits
because of inefficient and not too reliable means of identification of
the beneficiaries.
ACEBEDO OPTICAL v. CA (DANNAH) The City Mayor sent to Acebedo a Notice of Resolution and Cancellation
March 31, 2000 | Purisima, J. | Substantive Due Process: Police of
Power

PETITIONER: Acebedo Optical Company, Inc.


RESPONDENTS: Court of Appeals, Samahang Optometrist, Camilo Cabili
(Mayor of Iligan)

SUMMARY: The Mayor of Iligan, Mayor Cabili, issued a business license


to Acebedo Optical. It took into consideration the opposition of
Samahang Optometrist, and laid down conditions. Samahang
Optometrist filed a complaint before the Office of the City Mayor alleging
that Acebedo violated all the conditions. Officer Cahanap was ordered to
conduct an investigation and submitted a report stating Acebedo indeed
violated the conditions.

The City Mayor sent a Notice of Resolution and Cancellation to Acebedo.


Acebedo filed petitions in the RTC and the CA, both of which ruled in
favor of the Mayor. The Supreme Court reversed the rulings, stating that
the conditions set forth were not a valid exercise of police power.

DOCTRINE: A business permit is issued primarily to regulate the conduct


of business and the Mayor cannot, through its issuance, regulate the
practice of a profession, like that of optometry.

FACTS:
Acebedo Optical applied for a business permit with the Office of the City
Mayor of Iligan. The City Mayor issued Business Permit No. 5342,
after considering the opposition of Samahang Optometrist, subject to
the ff conditions:
Acebedo cannot put up an optical clinic but only a commercial store
Acebedo cannot examine/prescribe glasses because these are
functions of optical clinics
Acebedo cannot sell glasses without a prescription having been
made by an independent optometrist
Acebedo cannot advertise optical lenses and glasses
Acebedo is allowed to grind lenses but only upon prescription of
an independent optometrist.
On December 1998, private respondent Samahan ng Optometrist sa
Pilipinas (SOPI), Iligan Chapter, lodged a complaint against Acebedo
before the Office of the City Mayor. They alleged that Acebedo has
violated the conditions of the business permit, and they requested
the cancellation and/or revocation of said permit.
Mayor Cabili designated City Legal Officer Cahanap to conduct an
investigation on the matter. Officer Cahanap submitted a report
finding Acebedo guilty of violating ALL the conditions of its permit and
recommending the disqualification of Acebedo from operating its
business.
Business Permit and gave Acebedo 3 months to wind up its affairs. In the case at hand, the business permit granted was burdened with
On October 1989, Acebedo brought a petition for certiorari, prohibition several conditions. Acebedo agrees with the CA that the Mayor acted
and mandamus with a prayer for restraining order/prelminiary beyond his authority in imposing the conditions as they have no basis
injunction against the respondents (Mayor, Samahan and Officer)
in the law or
before the RTC of Iligan.
The trial court dismissed the petition for failure to exhaust
administrative remedies. Acebedo then filed an appeal with the CA
seeking to set aside the Order of Dismissal. The 9th Division of the
CA dismissed the petition for lack of merit. The MR was also
dismissed.
Hence, this present petition.

ISSUE/s:
WoN the conditions of Acebedo’s business permit are a valid exercise of
police power – NO

RULING: WHEREFORE, the petition is GRANTED; the Decision of the Court


of Appeals is REVERSED; and the respondent City Mayor is hereby ordered
to reissue petitioner’s business permit in accordance with law and with this
disposition.

RATIO:
Police power as an inherent attribute of sovereignty is the power to
prescribe regulations to promote the health, morals, peace,
education, good order or safety and general welfare of the
people.
The State, through legislature, has delegated the exercise of this power to
LGUs as agencies of the state. This is included in the General Welfare
Clause of the Local Government Code (please see Sec. 16 LGC in the
original case)
The scope of police power has been held to be so comprehensive as to
encompass almost all matters affecting the health, safety, peace,
order, morals, comfort and convenience of the community. It is
regulatory in nature and the power to issue licenses or grant
business permits, if exercised for a regulatory and not revenue-
raising purpose, is within the ambit of this power.
Mayors are authorized to issue or grant licenses and business permits,
as stated in Sec 171 of the LGC.
However, the power to grant or issue licenses or business permits must
always be exercised in accordance with law, with utmost observance
of the rights of all concerned to due process and equal protection.
The exercise of police power is valid unless it contravenes the
fundamental law of the land or an act of the legislature, or unless it is
against public policy or is unreasonable, oppressive, partial,
discriminating or in derogation of a common right.
ordinance. On the issue of the conditions being ultra vires: A license or permit is
The power to issue licenses and permits necessarily includes the not in the nature of a contract but a special privilege. Acebedo
corollary power to revoke, withdraw or cancel the same. In Austin formerly accepting
Hardware v. CA, it was held that the power to license carries with it
the authority to provide reasonable terms and conditions.
The power or authority of the Mayor to impose conditions or restrictions
in the business permit is indisputable.
Distinction must be made between the grant of a license or permit to do
business and the issuance of a license to engage in the practice of a
particular profession.
A business permit authorizes the person to engage in business
or some form of commercial activity.
A professional license is the grant of authority to engage in the
practice or exercise of his or her profession.
In the case at hand, what is sought by Acebedo from the Mayor is a
permit to engage in the business of running an optical shop.
In the case of Samahan ng Optometrists v. Acebedo, Justice Hermosisima
held that “the fact that Acebedo hires optometrists who practice their
profession does not translate into a practice of optometry by Acebedo
itself.”
In the old and new Optometry law, there is no prohibition against the
hiring by corporations of optometrists.
A business permit is issued primarily to regulate the conduct of business
and the Mayor cannot, through its issuance, regulate the practice of
a profession, like that of optometry.
During the deliberations on the Optometry Law, the Bicameral Committee
failed to reach a consensus as to the prohibition on indirect practice
of optometry by corporations. Congress left the resolution of such
issue for judicial determination.
The employment of a qualified optometrist by a corporation is not against
public policy. The manufacturing, selling, trading and bartering of
eyeglasses do not constitute the practice of optometry.
There is not public policy forbidding the commercialization of optometry,
as in law and medicine.
In analogy, it is like private hospitals being maintained by corporations
incorporated for the purpose of furnishing medical and surgical
treatment. These corporations employ doctors, in the same way that
optical shops hire licensed optometrists. There is indeed no valid
basis for treating corporations engaged in the business of running
optical shops differently.
the conditions of the license does not mean it is barred from questioning
said conditions.
Kapunan, Concurring: The corporate practice of any profession, including
optometry, must never be sanctioned. The test of valid exercise of police
power is classic reasonableness and propriety of the measures or the
means in the promotion of the end sought to be accomplished. In the
case at hand, the purported ends sought to be achieved go no deeper
than a recital of the general welfare clause, without establishing how
those goals could be reasonably achieved by imposing such conditions
in the permit.
Vitug, Dissenting: Acebedo does not dispute its having violated the
conditions stated in the business permit. The questioned conditionalities
are activities that cannot be performed by a corporation without such
engagement being translated into an unauthorized practice of optometry.
The City Mayor has merely restated what the Optometry Law mandates.
LAWRENCE v. TEXAS (IYA)
June 26, 2003 | Kennedy, J. | Subtantive Due Process ISSUE/s:

PETITIONER: John Geddes Lawrence, et al.


RESPONDENTS: Texas

SUMMARY: Lawrence and Garner, two males, were engaging the


sexual act when Harris County Police entered Lawrence’s home and
arrested them. They were convicted for having violated Tex. Penal Code
Ann. Section 21.06(a) (2003). Lawrence challenges the ruling and the
law as violation of the Equal Protection Clause of the Fourteenth
Amendment.

The CA rejected the arguments. On Certiorari the Supreme Court


decided to overturn a long standing precedent: Bowers v Hardwick which
penalized acts of sodomy. The Court now recognizes the freedom of
parties to engage in sexual acts in their private capacity.

DOCTRINE: The right to liberty under the Due Process Clause gives the
full right to engage in their conduct without intervention of the
government. “It is a promise of the Constitution that there is a realm of
personal liberty which the government ay not enter.”

FACTS:
Officers of the Harris County Police Department were dispatched to a
private residence in response to a reported weapons disturbance.
When the officers arrived, their entrance to the house of Lawrence did
not seem to be questioned.
The officers saw Lawrence and another man, Tyron Garner, engaging in
the sexual act. The two were arrested, held in custody, and charged
and convicted before a Justice of Peace.
The crime charged was described as ”deviate sexual intercourse, namely
anal sex, with a member of the same sex.” This is deemed in
violation of state law namely Tex. Penal Code Ann. section 21.06(a)
(2003).
“A person commits an offense if he engages in deviate sexual
intercourse with another individual of the same sex.”
Lawrence challenged the statue a violation of the Equal Protection
Clause of the Fourteenth Amendment. These contentions were
rejected.
The CA for the Texas Fourteenth District considered Lawrence’s case.
After hearing, the CA rejected the arguments and affirmed the
convictions.
In the CA Ruling the majority opinion was based greatly in a previous ruling
Bowers v Hardwick, which they considered as authoritative and proper.
Hence, this petition on certiorari.
Whether the petitioners were free as adults to engage in the private
conduct in the exercise of their liberty under the Due Process Clause
of the 14th Amendment to the Constitution. - YES

RULING: The Judgement of the CA for the Texas Fourteenth District is


reversed and the case is remanded for further proceedings.
RATIO:
The Court deems in necessary to revisit previous jurisprudence,
especially that of Bowers which will be discussed later.
In Griswold v Connecticut: the court invalidated a state law prohibiting
the use of drugs or devices of contraception and counseling or aiding
and abetting the use of contraceptives. The Court describe the
protected interest as a right to privacy and placed emphasis on the
marriage relation and protected space of the marital bedroom.
In Eisenstadt v. Baird: the Court invalidated a law prohibiting the
distribution of contraceptives to unmarried persons. The case was
decided under the Equal Protection Clause saying that the law
impaired the exercise of their personal rights.
In Roe v Wade: It recognized the right of a woman to make certain
fundamental decisions affecting her destiny and confined once more
that the protection of liberty under the Due Process Clause has a
substantive dimension of fundamental significance in defining the
rights of the person.
The facts in Bowers v Hardwick has similarities to the present case. Two
adult males were in sexual conduct and was deemed in violation of a
Georgia statute which makes it a criminal offense to engage in
sodomy.
The difference is that the Georgia statute prohibited the conduct whether
or not the participants were of the same sex, while the Texas statute
applies only to participants of the same sex.
The statues seek to control a personal relationship that, whether or not
entitled to formal recognition in the law, is within the liberty of
persons to choose without being punished as criminals.
The Court acknowledges that adults may choose to enter upon
relationships int he confines of their and their own private lives and
still retain their dignity as free persons.
History depicts that sodomy laws were enacted to prohibit non-
procreative sexual activity and not to be directed at homosexuals.
Laws prohibiting sodomy do not seem to have been forced against
consenting adults acting in private.
The Court stated that Bowers was not correct when it was decided and it
is to correct today. It ought not to remain biding precedent. This case
now overrules Bowers v Hardwick.
The right to liberty under the Due Process Clause gives the full right to
engage in their conduct without intervention of the government. “It is
a promise of the Constitution that there is a realm of personal liberty
which the government ay not enter.”
CRUZAN v. DIRECTOR, MISSOURI DOH withdrawal of “death prolonging procedures.” App. To Pet. For Cert.
June 25, 1990 | Rehnquist, J. | Substantive Due Process A99.
The court also found that Nancy’s “expressed thoughts at age 25 in
Petitioner/s: Nancy Beth Cruzen, by her parents and co guardians somewhat serious conversation with a housemate friend that if sick or
Respondent/s: Director, Missouri Department of Health injured she would not wish to continue her life unless she could live at
least halfway normally
Summary: Nancy Cruzen was involved in a car accident, which left her in a
vegetative state. Her parents realizing that she had no chance of regaining her
mental faculties, decided to take off her life support. The employees of the
hospital refused to do so without a court order. Upon getting the authorization,
the SC reversed the decision stating that the testimony provided by the
roommate did not establish a clear and convincing evidence of Nancy’s
wishes. Hence this petition. The SC ruled that the clear and convincing proof
should be established, because only Nancy has the right on whether she
wants to live or not. No other person can dictate, unless it was expressly
written by her through will or other instruments.

Doctrine: This Court has mandated an intermediate standard of proof,


clear and convincing evidence, when the individual interests at stake in a
state proceeding are both particularly important and more substantial than
mere loss of money.

Facts:
Nancy Cruzan lost control of her car as she traveled down Elm Road in
Jasper County, Missouri. The vehicle overturned, and Cruzan was
discovered lying dace down in a ditch without detectable respiratory or
cardiac function.
An attending neurosurgeon diagnosed her as having sustained probably
cerebral contusions compounded by significant anoxia (lack of
oxygen).
The Missouri trial court in this case found that permanent brain damage
generally results after 6 minutes in an anoxic state; it was estimated
that Cruzan was deprived of oxygen from 12 to 14 minutes.
She now lies in Missouri state hospital in what is commonly referred to as
a persistent vegetative state: generally, a condition in which a person
exhibits motor reflexes but evinces no indications of significant
cognitive function.
After it had become apparent that Nancy Cruzan had virtually no chance
of regaining her mental faculties, her parents asked hospital
employees to terminate the artificial nutrition and hydration
procedures. The employees refused to honor the request without
court approval.
The parents then sought and received authorization from the state trail
court for termination.
The court found that a person in Nancy’s condition had a fundamental
right under the State and Federal Constitutions to refuse or direct the
suggest that given her present condition she would not wish to This Court has mandated an intermediate standard of proof, clear
continue on with her nutrition and hydration. and convincing evidence, when the individual interests at stake
The SC reversed by a divided vote. The court found that Cruzan’s in a state proceeding are both particularly important and more
statements to her roommate regarding her desire to live or die under substantial than mere loss of money.
certain conditions were “unreliable for the purpose of determining her
intent, and thus insufficient to support the co-guardians claim to
exercise substituted judgment on Nancy’s behalf.”
It rejected the argument that Cruzan’s parents were entitled to order the
termination of her medical treatment, concluding that “no person can
assume that choice for an incompetent in the absence of the
formalities required under Missouri’s Living Will statues or the clear
and convincing, inherently reliable evidence absent here.”
Hence this petition.

Issue/s:

WoN Cruzan a right under the US constitution, which would require the
hospital to withdraw life-sustaining treatment under these
circumstances.
YES

Ruling: SC decision affirmed. Not allowed to take her life.

Ratio:

Such a “right” must be exercised for her, if at all, by some sort of


surrogate. Here, Missouri has in effect recognized that under certain
circumstances a surrogate may act for the patient in electing to have
hydration and nutrition withdrawn in such a ways as to cause death, but
has established a procedural safeguard to assure that the action of the
surrogate conforms best it may to the wishes expressed by the patient
while competent.
Missouri requires that evidence of the incompetent’s wishes as to
the withdrawal of treatment be proved by clear and convincing
evidence.
It cannot be disputed that the Due Process Clause protects an interest in
life as well as an interest in refusing life-sustaining medical
treatment.
A State may properly decline to make judgments about the “quality” of
life that a particular individual may enjoy, and simply assert an
unqualified interest in the preservation of human life to be weighed
against the constitutionally protected interests of the individual.
The function of a standard of proof, as that concept is embodied in the
Due Process Clause and in the realm of fact finding, is to instruct the
fact finder concerning the degree of confidence our society thinks he
should have in the correctness of factual conclusions for a particular
type of adjudication.
This level of proof, or an even higher one, has traditionally been imposed
in cases involving allegations of civil fraud, and in a variety of other
kinds of civil cases involving such issues as lost wills, oral contracts
to make bequests, and the like.
But not only does the standard of proof reflect the importance of a
particular adjudication, it also serves as “a societal judgment about
how the risk of error should be distributed between the litigants.”
The more stringent the burden of proof a party must bear, the more that
party bears the risk of an erroneous decision.
It is also worth noting that most, if not all, States simply forbid oral
testimony entirely in determining the wishes of parties in transactions
which, while important, simply do not have the consequences that a
decision to terminate a person’s life does.
BELTRAN v. SECRETARY OF HEALTH (JP) showed that many Filipino doctors are not yet fully trained on the
Novermber 5, 2005 | Azcuna, J. | Substantive Due Process specific indications for bloody component transfusion.
With this, RA 7719 or the National Blood Services Act of 1994 was
PETITIONER: Rodolfo S. Beltran, doing business under the name Our enacted
Lady of Fatima Blood Bank, et al. (comprising the majority of the Board of
Directors of the Philipine Association of Blood Banks) RESPONDENTS:
The Secretary of Health

SUMMARY: By virtue of a study of the Philippine blood banking system


which showed that commercial blood banks with poor donors are more
likely to spread blood transfusion diseases, Congress enacted RA 7719
wherein Sec. 7 and AO 9 orders the phasing out of commercial blood
banks. Petitioners, being commercial bloody banks raised the issue of
equal protection and deprivation of liberty. SC declared that since the
requirements for a valid classification are met, such law does not violate
equal protection. SC also declared that such law is a valid exercise of the
police power of the state in order to promote public welfare. Hence,
personal liberty may be interfered. The validity of the RA is upheld.

DOCTRINE: The classification (re: equal protection) to be reasonable: (a)


must be based on substantial distinctions which make real differences;
(b) must be germane to the purpose of the law; (c) must not be limited to
existing conditions only; and, (d) must apply equally to each member of
the class

The State, in order to promote the general welfare, may interfere with
personal liberty, with property, and with business and occupations.

FACTS:
The New Tropical Medicine Foundation in Alabang, with the assistance of
the US Agency for International Development (USAID) released its
final report of a study on the Philippine blood banking system. It was
revealed that 64.4% of the blood units collected in 1992 were
supplied by commercial blood banks. With only 24 registered
commercial blood banks, each commercial blood bank produces five
times more blood than the Red Cross and fifteen times more than
government-run blood banks.
The study likewise revealed that blood sold by persons to blood
commercial banks are three times more likely to have any of the four
tested infections or bloody transfusion transimissible diseases
namely malaria, syphilis, Hepatitis B and Acquired Immune
Deficiency Syndrom (AIDS) than those donated by PNRC. Many of
these donors are poor and often they are students who need cash
immediately. Since they need the money, these donors are not
usually honest about their medical or social history. The study also
into law on April 2, 1994. The Act seeks to provide an adequate authority to enact legislation that may interfere with personal liberty
supply of safe blood by promoting voluntary bloody donation and by or property in order to promote the general welfare.
regulating blood banks in the country. It was approved by President Petitioners are of the opinion that the Act is unconstitutional and void
Fidel Ramos. because it infringes on the freedom of choice of an individual in
Respondent Secretary of Health issued the implementing rules and connection to what
regulations of said law (Administrative Order No. 9).
Petitioners are assailing Section 7 of RA 7719: “all commercial blood
banks shall be phased out over a period of two years after the
effectivity of this act, extendable to a maximum of two years” and
Section 23 of AO No. 9 which effects Section 7 of RA 7719 “based on
the result of careful study and review of bloody supply and demand
and public safety.”

ISSUE/s:
WoN Sec. 7 and its IRR constitutes undue delegation of legislative power
-
NO
WoN Sec. 7 of RA 7719 and its IRR violate the equal protection
clause –
NO
WoN Sec. 7 of RA 7719 and its IRR constitute deprivation of
personal liberty and property - NO

RULING: The Court upholds the validity of Section 7 of RA 7719 and AO No.
9.

RATIO:
The SC deems the classification to be valid and reasonable for the
following reasons:
It was based on substantial distinctions. The former operates for
purely humanitarian reasons and as a medical service while
the latter is motivated by profit.
The classification, and the consequent phase out of commercial
blood banks is germane to the purpose of the law, that is, to
provide the nation with an adequate supply of safe blood by
promoting voluntary blood donation and treating blood
transfusion as a humanitarian or medical service rather than
a commodity.
The Legislature intended for the general application of the law.
Its enactment was not solely to address the peculiar
circumstances of the situation nor was it intended to apply
only to the existing conditions.
The law applies equally to all commercial blood banks without
exception.
The Court finds that the National Blood Services Act is a valid exercise of
the State's police power. Therefore, the Legislature, under the
circumstances, adopted a course of action that is both necessary
and reasonable for the common good. Police power is the State
he wants to do with his blood which should be outside the domain of State
intervention. Petitioners likewise claim that the phase out of the commercial
blood banks will be disadvantageous to them as it will affect their businesses
and existing contracts with hospitals and other health institutions, hence
Section 7 of the Act should be struck down because it violates the non-
impairment clause provided by the Constitution.
As answer, the SC staets that the State, in order to promote the general
welfare, may interfere with personal liberty, with property, and with
business and occupations. Thus, persons may be subjected to
certain kinds of restraints and burdens in order to secure the general
welfare of the State and to this fundamental aim of government, the
rights of the individual may be subordinated.
ONG v. SANDIGANBAYAN (HENRY) Another Order was issued by the Ombudsman in lieu of the documents
September 16, 2005 | Tinga, J. | Substantive Due Process he submitted, stating that no justification was made through such
documents as Allied Bank and SGV (via subpoenas duces tecum)
PETITIONER: Jose U. Ong and Nelly M. Ong were unable to provide supporting documents to aid Ong’s claims.
RESPONDENTS: Sandiganbayan (Third Division) and Office of the He was then ordered to provide the Ombudsman with relevant
Ombudsman documents to support his claims.
Instead of compliance, Ong field a Motion claiming that he was not
SUMMARY: Ong was alleged to have amassed properties notified of the subpoenas issued to SGV and Allied bank requiring
disproportionately more than his lawful income. Contrary to his SALN them to substantiate Ong’s claims. He was alleging that his right to
declaration, Ong has apparently acquired properties in the prime real due process was violates.
estate locations. The Ombudsman found probable cause and through an Consequently, the Ombudsman issued a Resolution, indicating Ong’s
order directed the Republic to file a petition to recover the ill-gotten failure to substantiate his claims of the sources financing his tons of
wealth of petitioner Ong. The petitioner in this case is basically property acquisitions. The same Resolution directed the filing of the
challenging the Ombudsman’s conduct as it allegedly violated his right to Ombudsman in collaboration with the OSG of a Petition for recovery
due process as it charged him of RA 1379 violation, which was according of ill-gotten/unexplained wealth under RA 1379, in relation to RAs
to him, vague. Also that, he was not informed of the subpoena duces 3019 and 6770, against the petitioners Ong.
tecum given to SGV and Allied Bank to provide supporting documents The Office of the Special Prosecutor concurred with the findings and
regarding his case. Moreover, that his wife Nelly, was not accorded with recommendations of the Ombudsman. This was followed by the filing
an opportunity to participate in the preliminary investigation. The SC ruled of a petition for forfeiture of unlawfully acquired property before the
in favor of the respondents, noting that RA 1379 has no constitutional Sandiganbayan by the Republic, via the Special Prosecutor and
defect and that Ong was not at all deprived of his right to due process. Deputy Ombudsman for Luzon.
In the said petition, it was alleged that the total value of the questioned
DOCTRINE: Although right to preliminary investigation is not a right assets is Php 21,474,585.00, which is grossly disproportionate to
vested by the Constitution, rather a statutory privilege, it is nonetheless Ong’s lawful income from his public employment and other sources
considered a component part of due process in criminal justice. amounting to Php 1,060,412.50, considering that Nelly (the wife) has
no visible means of income.
Sandiganbayan issued an order directing the issuance of a writ of
FACTS:
preliminary attachment against the properties of the petitioners.
Congressman Bonifacio H. Gillego executed a complaint, claiming that Petitioners filed an Answer, denying the allegations and that the Special
petitioner Jose Ong, then Commissioner of the BIR, has amassed Prosecutor as well as the Ombudsman intentionally failed to consider
properties worth disproportionately more than his lawful income. the retirement and separation pay Ong received from SGV and other
Petitioner Ong declared in his SALN (as of Dec 1989) that he had Php lawful sources of funds used in the acquisition of the questioned
750,000.00 on hand and in banks. Shortly, he was able to acquire properties. Along with this, they also challenged the failure of the
properties mostly in the millionaires’ choice areas in Alabang, Ombudsman to state a cause of action because RA 1379 is
Muntinglupa, and Metro Manila. Here’s a list fo the properties: unconstitutional due to vagueness in defining ill-gotten wealth.
House and lot in Alabang (Php 5,500,000.00) Sandiganbayan then issued a Resolution, that the petition for forfeiture is
Another lot in Alabang (Php 5,700,000.00) an action in rem, civil in character, that the Ombudsman’s powers to
Another lot in Alabang (Php 4,675,000.00) investigate and prosecute unexplained wealth cases is found on RAs
Another lot in Alabang (Php 5,055,000.00) 1379, 3019, and 6770. And that, contrary to the petitioner’s claims,
Lot in Makati (Php 832,000.00) the Petition of the Ombudsman sufficiently states a cause of action.
The Director of the Fact-Finding and Intelligence Bureau of the Office of Petitioners filed a Motion for Reconsideration, reasoning that although
the Ombudsman ordered the conduct of a pre-charge investigation forfeiture is a civil action, in substance it is a criminal proceeding as it is
on the matter, which recommended that forfeiture proceedings be a penalty for violation of RA 1379—and hence Nelly Ong was entitled to
instituted against Ong’s properties. a preliminary investigation. Proceeding against her conjugal share
Ong was directed to submit his counter-affidavit and other controverting without giving her opportunity to present her side violates her right to due
evidence in the Order of the Ombudsman. In the said document, he process. Also, they emphasized that they were not notified of the
stated his SALN, ITR, Bank Certificates, and other documents Resolution directing the filing of the petition for forfeiture and were
explaining sourcesof funds with which he acquired the assets. consequently deprived of their
right to file an MR under RA 6770 and pertinent rules. Ong was not served a copy of the resolution directing the
Sandiganbayan issued a Resolution directing Ombudsman to furnish filing of a petition for forfeiture depriving him of his right to file
Sps. Ong with a copy of the Resolution to file the forfeiture case and for a motion for reconsideration.
giving them 5 days to file an MR. Instead of waiting for the
Ombudsman’s compliance, the petitioners filed the present Petition
for Certiorari.
The OSG in its comment stated that Nelly Ong was only a formal party
as all acquisitions were made through Jose Ong’s efforts only.
Furthermore, that RA 1379 is constitutional as it defines legitimately
acquired properties and specifies that the acquisition of propertu out
of proportion to legitimate income is proscribed, and that the
Ombudsman did not exhibit bias and partiality against Petitioner
Ong.

ISSUE/s:
WoN the Sandiganbayan gravely abused its discretion in ruling that Nelly
Ong is not entitled to preliminary investigation – NO
WoN RA 1370 is unconstitutional – NO
WoN the petitioners’ right to due process was violated – NO

RULING: SC dismissed the petition.

RATIO:
In Republic v. Sandiganbayan, the SC ruled that forfeiture proceedings
under RA 1379 are civil and nature and not penal or criminal in
character, as they do not terminate in the imposition of a penalty but
merely in the forfeiture of the properties illegally acquired in favor of
the State.
In Almeda v. Perez, the SC ruled that forfeiture proceedings are civil in
nature in the sence that it applies purely to the procedural aspects of
such proceedigns and has no bearing on the substantial rights of the
respondents. This was reiterated in Katigback v. Solicitor General.
Although the right to a preliminary investigation is not a fundamental right
guaranteed by the Constitution but a mere statutory privilege, it is
nonetheless considered a component part of due process in criminal
justice.
The SC ruled that despite the silence of RA 1379 to the preliminary
investigation of Nelly Ong, the fact that what was involved are
properties in her conjugal partnership, entitles her to a prelinary
investigation even if she is just a formal party. However, since the
properties were acquired by his husband alone, such inquiry would
be, according to the SC, an “empty ceremony”.
Although Ong was given the opportunity to present documents in his
possession relevant to the investigation, the SC noted that Ong
should have been notified of the subpoenas duces tecum ad
testificandum issued to SGV, Allied Bank, and the BIR. The fact that
he was not so notified is a deniel of fundamental fairness which
taints the preliminary investigation. The same goes for the fact that
However, it is very important to note, that instead of awaiting the Ombudsman’s
compliance with the said Resolution, petitioners directly saught recourse with
the SC, waiving their right to avail of the remedy afforded by the second
Resolution.
Still, the SC found this time perfect to reprimand the Ombudsman to be more
careful in its conduct of preliminary investigation to the end that participants
are accorded the full measure of their rights under the Constitution and our
laws.
Finally, the attacks against the constitutionality of RA 1379 due to its vagueness,
among others, is unmeritorious. It is not vague as it defines with sufficient
particularity unlawfully acquired property of a public officer or employee as
that “which manifestly out of proportion to his salary as such public officer or
employee and to his other lawful income and the income from legitimately
acquired property”.
The law does not offend the basic concept of fairness and the due process
clause of the Constitution, as the public is given fair notice of what acts are
proscribed.
MANALO v. PNP CHIEF CALDERON (DANNAH) The SPO2s contend that the memorandum on their custody is ellegal and
October 15, 2007 | Reyes, J. | Substantive Due Process: Habeas not sanctioned by any existing provision of our constitution.
Corpus They also posit that what is only sanctioned is preventive suspension
under
PETITIONER: SPO2 Geronimo Manalo et al.
RESPONDENTS: PNP Chief Calderon et al.

SUMMARY: The 6 police officers (petitioners) were tagged by eye


witnesses as those who burnt a school which was converted into a
polling area. Two died while nine were injured and as a result, said police
officers were placed under restrictive custody.

Petitioners aver that this respective custody deprives them of liberty, and
filed a petition for habeas corpus. However, prior to the Court deciding on
the case, a memorandum was issued by the Chief of PNP recalling the
assailed restrictive custody.

Although moot and academic, the Court still proceeded with deciding the
case. They ruled that the restrictive custody was beyond the ambit of
habeas corpus, as it was at best, a nominal restraint.

DOCTRINE: The main thrust of the special proceeding of habeas corpus


is to inquire into the legality of one’s detention. It’s vital purpose is to
obtain immediate relief from illegal confinement, to liberate those who
may be imprisoned without sufficient cause and to deliver them from
unlawful custody.

Restrictive custody is not a form of illegal detention. The PNP has the
prerogative to impose administrative functions on its members.

FACTS:
On May 2007 at 3am, 5 unidentified malefactors with high-powered firearms
appeared at the Bgy. Pinagbayanan Elementary School in Taysan,
Batangas.
Earlier, the entire school was converted into a polling area for the 2007
national and local elections.
The 5 armed men entered Precinct 76-A and poured gasoline over a
ballot box, after which they fired several rounds of ammunition
setting the premises ablaze.
The fire caused the death of Banaag (a teacher) and Ramos, an election
supervisor and a poll watched. Nine others were also injured as a
result.
Several eye witnesses identified some of the petitioners (SPO2s) as the
perpertrators of the school burning. The investigation also yielded
that said police officers failed to timely respond to the incident.
Acting on the report, the PNP hierarchy issued 3 memoranda, putting the
police officers on restrictive custody.
which they can enjoy liberty and go home to their families pending Only if the Court is satisfied that a person is unlawfully restrained of his
administrative investigation. Hence, they urge, this practice should liberty will a petition for habeas corpus be granted, and the person
be put to stop. detained released from confinement.
In support of their argument, they cited Moncupa v. Enrile et al where it Sombong v. CA teaches us that for the write of habeas corpus to issue,
was held that the writ of habeas corpus applies to all cases of the
illegal confinement or detention by which any person is
deprived of his liberty.
They thus pray that writ of habeas corpus be issued.
In lieu of a comment, the OSG manifested that through a Memorandum
(on August 30, 2007) by Director Radovan of the PNP, the assailed
restrictive custody was recalled.

ISSUE/s:
WoN the petitioners (SPO2s) were unlawfully detained or restrained
of their liberty under their restrictive custody status – NO
WoN the Court should dismiss the petition on the ground of mootness –
NO

RULING: WHEREFORE, the petition is DENIED DUE COURSE and


DISMISSED.

RATIO:
Procedural Issue (Issue #2): Although the restrictive custody was
recalled, the Court may still proceed with the case. The moot and
academic principle is not a magical formula that can automatically
dissuade the courts in resolving the case. There are exceptions, to
wit:
There is a grave violation of the Constitution
The exceptional character of the situation and the paramount
public interest is involved
When constitutional issue raised requires formulation of
controlling principles to guide the bench, bar and public
When the case is capable of repetition yet evading review
The matter is capable of repetiion or susceptible of recurrence. It better
be resolved now for the education and guidance of all concerned.
On the issue on habeas corpus: The high prerogative writ of habeas
corpus was devised and exists as a speedy and effectual remedy
to relieve persons from unlawful restraint and as the best and
efficient defense of personal freedom.
The main thrust of the special proceeding of habeas corpus is to inquire
into the legality of one’s detention. It’s vital purpose is to obtain
immediate relief from illegal confinement, to liberate those who may
be imprisoned without sufficient cause and to deliver them from
unlawful custody.
restraint of liberty must be in the nature of an illegal and involuntary
deprivation of freedom and action. It must be actual and effective, not
merely nominal or moral.
To the mind of the Court, petitioners are NOT illegally and involuntarily
deprived of their freedom of action.
First, the memoranda cannot be considered a form of curtailment of their
freedom guaranteed under the Constitution.
What was ordered by the PNP is that their movements, inside and
outside camp be monitored
All their movements within camp should be monitored
When situation warrants their movement outside camp, they
should be properly escorted on a one-on-one basis
A logbook should be maintained
Even petitioners (SPO2s) admit that they are not actually detained or
imprisoned.
Secondly, the “restrictive custody” is at best, nominal restraint which is
beyond the ambit of habeas corpus. It is a precautionary measure to
assure the PNP authorities that the police officers concerned are
always accounted for.
Thirdly, petitioners’ reliance on Moncupa is misplaced. In that case,
petitioner was ordered release but his release was saddled with
restrictions. He needed prior approval for any travel outside of Manila
or change of residence. His freedom of speech was also muffled.
In the case at hand, no restrictions in the nature of those imposed in
Moncupa exist. They are not required to secure approval before they
can move out of the camp, only that each of them be accompanied
by an escort and their time of departure and arrival noted.
Fourthly, RA 6975 (DILG Act of 1990) clearly provides that members of
the police force are the subject to the administrative disciplinary
machinery of the PNP.
Evidently, the PNP is well within its authority to relieve petitioners
(SPO2s) from their former positions and place them under tight
watch, at least until the termination of the said investigation.
Lastly, although the PNP is civilian in character, its members are subject
to the disciplinary authority of the Chief of PNP.
In sum, etitioners are unable to discharge their burden of showing that
they are entitled to the issuance of the writ prayed for. The petition
fails to show on its face that they are unlawfully deprived of their
liberties guaranteed and enshrined in the Constitution. No unlawful
restraint is foisted on them by the PNP authorities under the
questioned memoranda.
LUCENA GRAND CENTRAL v. JAC LINER (IYA) ISSUE/s:
February 23, 2005| Carpio-Morales, J. | Substantive Due Process WON the City of Lucena properly exercised its police power when it
enacted the assailed ordinances - NO
PETITIONER: Lucena Grand Central Terminal, Inc.
RULING: Petition is DENIED.
RESPONDENTS: Jac Liner, Inc.
SUMMARY: Jac Liner questions Oridnance No. 1631 and 1778 as it grants
exclusive franchise of bus operation and terminals to Lucena Grand Central. RATIO:
The purpose of the ordinances is said to be to regulate the flow of traffic in As with the State, the local government may be considered as having
the city. properly exercised its police power only if the following conditions are
met:
The RTC ruled that the grant of exclusive franchise is oppressive and Interest of the general public require the interference of the State
unreasonable, that the city ordinances are not within the ambit of police The means employed are reasonably necessary for the
power. The CA agrees with the RTC ruling. On petition for review the SC attainment of the object sought to be accomplished and not
upholds the aforementioned ruling, stating that the ordinances does not unduly oppressive upon individuals
fulfill the condition of lawful method. There must be concurrence of lawful subject and lawful method.
The questioned ordinances were enacted with the objective of relieving
DOCTRINE: The local government may be considered as having traffic congestion in the City of Lucena, they involve public interest
properly exercised its police power only if there is a concurrence of warranting interference of the State. The condition of lawful subject is
lawful subject and lawful method. met.
The condition of lawful method, however, is lacking. Bus terminals do not
impede or help impede the flow of traffic. The grant of an exclusive
franchise has not been proven to be the only solution to the problem.
FACTS: The grant of exclusivity likewise deprives other entities which might be
Jac Liner, Inc. assailed City Ordinance Nos. 1631 and 1778 as able to provide better facilities.
unconstitutional not he ground that they constitute an invalid exercise The petitioners claim that ordinances have been proven effective in
of police power. easing traffic congestion. Whether an ordinance is effective is an
The ordinances grant exclusive franchise to Lucena Grand Central for issue different form whether it is reasonably necessary. It is
the construction and operation of one common bus and jeepney reasonableness, not effectiveness, which bears upon its
terminal facility in Lucena City. constitutionality.
The purpose of which according to the Whereas Clause of Ordinance
No. 1778 is to alleviate the traffic congestion caused by the existence
of various bus and jeepney terminals within the city.
RTC rendered a decision declaring:
Ordinance No. 1631 as valid having been issued int he exercise
of the police power of the City Government of Lucena insofar
as to grant franchise of construction to Lucena Grand.
However the provisions that no grant of concession to
operate a busses to other third party companies is
invalidated.
Ordinance No. 1778 as null and void, being an ultra vires act of
the City Government of Lucena raising from an invalid,
oppressive and unreasonable exercise of police power.
Lucena Grand filed a MR but was denied. It elevated via petition for
review under Rule 45 before he SC. The SC referred the petition to
the CA. The CA dismissed th petition and affirmed the RTC ruling. A
subsequent MR was filed but was denied.
Hence, petitioner comes again to the SC via petition for review.
ERMITA-MALATE v. MANILA (ELIEL) The lower court issued a writ of preliminary injunction ordering
July 31, 1967 | Fernandez, J. | Substantive Due Process respondent Mayor to refrain from enforcing said Ordinance No. 4760.
The Mayor prayed for its dissolution and the dismissal of the petition on
PETITIONER: Ermita-Malate Hotel and Motel Operators, Association, the
Inc., Hotel Del Mar, Inc. and Go Chiu
RESPONDENTS: Hon. City of Manila, Victor Alabanza

SUMMARY: The Municipal Board of the City of Manila enacted Ordinance


No. 4670 regulating the establishments of hotels, motels, and taverns.
Ermita-Malate hotel filed a petition for prohibition against the enacted
Ordinance for being unreasonable, oppressive, vague, and violative of due
process. Because Secs 1, 2 and 4 of which imposes them to pay license
fees, to refrain form entertaining guests without filling up registration form, not
allowing children below 18 y/o unless accompanied, and the penalty of
automatic cancellation. The lower court ruled in their favor and declared the
ordinance null and void. Hence this appeal. The SC reversed the decision
and ruled that (1) there is no fact evidence to offset the presumption of
constitutionality of an ordinance; (2) it was a valid exercise of police power,
because the City of Manila aimed to minimize ceratin clandestine practices
hurtful to public morals; and (3) it was not vague rather specific and detailed
because what makes a statute susceptible is when a man of common
intelligence must necessarily guess at its meaning and differ as to its
application.

DOCTRINE: Persons and property may be subjected to all kinds of restrains


and burdens, in order to secure the general comfort, health, and preosperity
of the state.

FACTS:
The Municipal Board of the City of Manila enacted Ordinance No. 4760,
which was approved on June 14, 1963, by Vice-Mayor Herminio
Astorga, then the acting City Mayor of Manila, in the absence of the
respondent regular City Mayor
A petition for prohibition against Ordinance No. 4760 was filed on July 5,
1963 alleging that it is unreasonable and violative of due process.
Section 1 of which, imposes P6,000 fee per annum for 1 st class motels
and P4,500 for 2nd class motels; requiring the management to refrain
from entertaining or accepting any guest or custormer or letting any
room or other quarter to any person or persons without his filling up
the prescribed form in a lobby open ot public view at all times and in
his presence.
It is also assailed that it is void for being arbitrary and vague. Sec 2 of which,
classifies motesl into 2 classes and requiring the maintenance of certain
minimum facilities. And that it prohibits a person less than 18 y/o from
being accepted in such establishments unless accompanied by parents
or a lawful guardian. Also to lease any room twice every 24 hours. And it
assailed Sec 4 providing for the penalty of automatic cancellation of the
license.
ground that the petition failed to state a cause of action and that the ‘Liberty” as understood in democracies, is not license; it is ‘liberty
challenged ordinance bears a reasonable relation, toa proper regulated by law’
purpose, which is to curb immorality, a valid and proper exercise of
the police power.
Instead of giving evidence, both parties gave a stipulation of facts.
Memorandas were issued, however, since there is no evidence
submitted by both parties, the lower court decided to make
permanent the injunction.
Hence this appeal.

ISSUE/s:
16. WoN Ordinance No. 4670 is violative of the due process clause– NO

RULING: Judgment of lower court reversed.

RATIO:
Primarily what calls for a reversal of such a decision is the absence of
any evidence to offset the presumption of validity that attaches to a
challenged statute or ordinance.
The Judiciary should not lightly set aside legislative action when there is
not a clear invasion of personal or property rights under the guise of
police regulation.
No such factual foundation being laid in the present case, the lower
court deciding the matter on the pleadings and the stipulation
of facts, the presumption of validity must prevail and the
judgment against the ordinance set aside.
There is no question but that the challenged ordinance was
precisely enacted to minimize certain practices hurtful to public
morals. The explanatory note included as annex to the
stipulation of facts, speaks of the alarmin increase in the rate of
prostitution, adultery and fornication in Manila traceable in
great part ot the existence of motels, which “provide a
necessary atmosphere for clandestine entry, presence and exit”
and thsus become the “ideal haven for prostitutes and thrill-
seekers.”
The challenge ordinance then “proposes to check the clandestine
harboring of transeients and guests of these establisments by
filling up registration forms and the increase in the licensed
fees was intended to discourage from operating other –than
legal and to increase income of the city government.”
Persons licensed to pursue occupations which may in the public
need and interest be affected by the exercise of the police
power embark in these occupations subject to the
disadvantages which may result from the legal exercise of that
power.
Petitioners cannot be unaware that every regulation of conduct amounts
to curtailment of liberty which as pointed out by Justice Malcom
cannot be absolute.
The right of the individual is necessarly subject ot reasonable
restraint by general law for the common good xxx The liberty of
the citizen may be restrained in the interest of the public health,
or of the public order and safety, or otherwise within the proper
scope of the police power.
Persons and property may be subjected to all kinds of restrains and
burdens, in order to secure the general comfort, health, and
preosperity of the state.
To this fundamental aim of our Government the rights of the individual
are subordinated.
On the attempt to impugn the ordinance on vagueness or
uncertainty: what makes a statute susceptible to such a charge
is an enactment either forbidding or requiring the doing of an
act that men of common intelligence must necessarily guess at
its meaning and differ as to its application.
On the legislative organs of the government, whether national or
local, primarily rest the exercise of the police power, which, it
cannot be too often emphasized, is the power to prescribe
regulations to promote the health, morals, peace, good order,
safety and general welfare of the people.
Due process has been correctly identified as freedom from
arbitrariness. It is the embodiment of the sporting idea of fair play.
It exacts fealty “to those striving for justice” and judges the act of
officialdom for whatever branch “in the light of reason drawn from
considerations of fairness that reflect [democratic] traditions of
legal and political thought.”
CITY OF MANILA v. LAGUIO (JP) Sec. 3 Owners and/or operator of establishments engaged
April 12, 2005 | Tinga, J. | Substantive Due Process in, or devoted to, the businesses enumerated in Section 1
hereof are hereby given three (3) months from the date of
PETITIONER: City of Manila, Hon. Alfredo Lim (Mayor of Manila City), approval of this ordinance within which to wind up business
Hon. Joselito Atienza (Vice-Mayor) and Councilors operations or to transfer to any place outside of the Ermita-
RESPONDENTS: Hon. Perfecto Laguio, Jr. (Presiding Judge, RTC, Malate area or convert said businesses to other kinds of
Manila) and Malate Tourist Development Corporation business allowable within the area such as: restaurants,
coffeeshops, flower shops, x x x
SUMMARY: MTDC is engaged in the business of motels, hostels and Sec. 4 x x x Provided further, that in case of subsequent
lodging houses. It is assailing the validity of Odrinance No. 7783 by the violation and conviction, the premises of the erring
City of Manila which prohibited the establishment and the operation of establishment shall be closed and padlocked permanently.
certain businesses which it considers as detrimental to the morality of the Respondent Judge Laguio issued an ex-parte TRO against the
people in the Ermita-Malate Area. RTC granted the petition. SC affirmed enforcement of the Ordinance. Judge Laguio then rendered the
saying that the Ordinance invades fundamental personal and property assailed decision enjoining the petitioners from implementing the
rights and impairs personal privileges. It is constitutionally infirm because Ordinance declaring it null and void. The present petition is then filed.
it violates due process, especially the equal protection clause for ISSUE/s:
including motels and inns in its enumeration of immoral establishments
given that these businesses are not immoral per se. WoN the lower court erred in declaring the Ordinance void and
unconstitutional – YES
DOCTRINE: Substantive requirements for an ordinance to be valid: (1) WoN the Ordinance infringes the due process clause - YES
must not contravene the Constitution or any statute; (2) must not be
unfair or oppressive; (3) must not be partial or discriminatory; (4) must RULING: The lower court did not err in declaring the Ordinance, as it did,
not prohibit but may regulate trade; (5) must be general and consistent ultra vires and therefore null and void.
with public policy; and (6) must not be unreasonable.
RATIO:
The Ordinance is so replete with constitutional infirmities that almost
every sentence thereof violates a constitutional provision. The
FACTS:
prohibitions and sanctions therein transgress the cardinal rights of
Private respondent Malate Tourist Development Corporation (MTDC) is
persons enshrined by the Constitution. The Court is called upon to
operating hotels, motels, hostels, and lodging houses. It built and
shelter these rights from attempts at rendering them worthless.
opened Victoria Court in Malate (licensed motel but duly accredited
For an ordinance to be valid, it must not only be within the corporate
in DoT as a hotel).
powers of the local government unit to enact and must be passed
MTDC filed a Petition for Declaratory Relief with Prayer for a Writ of according to the procedure prescribed by law, it must also conform to
Preliminary Injunction and/or Temporary Restraining Order (RTC the following substantive requirements: (1) must not contravene
Petition) praying that the Ordinance No. 7783 (Ordinance) of the City the Constitution or any statute; (2) must not be unfair or
of Manila be declared as invalid and unconstitutional for including oppressive; (3) must not be partial or discriminatory; (4) must
motels and inns as among its prohibited establishments. not prohibit but may regulate trade; (5) must be general and
Ordinance No. 7783: An Ordinance Prohibiting The Establishment Or consistent with public policy; and (6) must not be unreasonable.
Operation Of Businesses Providing Certain Forms Of Amusement, The Ordinance must satisfy two requirements: it must pass muster under
Entertainment, Services Andfacilities In The Ermita-Malate Area the test of constitutionality and the test of consistency with the
Sec. 1 No person, partnership, corporation or entity shall, in the prevailing laws. Lacking a concurrence of these two requisites, the
Ermita-Malate area be allowed or authorized to engage in any police measure shall be struck down as an arbitrary intrusion into
business providing certain forms of amusement, entertainment, private rights ·a violation of the due process clause.
services and facilities where women are used as tools in The object of the Ordinance was, accordingly, the promotion and protection
entertainment and which tend to disturb the community, annoy of the social and moral values of the community. Granting for the sake of
the inhabitants, and adversely affect the social and moral argument that the objectives of the Ordinance are within the scope of the
welfare of the community, such as but not limited to: Sauna
City
Parlors, Massage Parlors, Karaoke bars, beerhouses, night
clubs, motels, inns x x x
Council’s police powers, the means employed for the cases of subsequent violations found in Section 4 of the Ordinance is
accomplishment thereof were unreasonable and unduly also equivalent to a taking of private property.

oppressive.
The closing down and transfer of businesses or their conversion into
businesses allowed under the Ordinance have no reasonable
relation to the accomplishment of its purposes. Prohibition of the
enumerated establishments will not per se protect and promote the
social and moral welfare of the community; it will not in itself
eradicate the alluded social ills of prostitution, adultery, fornication
nor will it arrest the spread of sexual disease in Manila.
It is insupportable to bring within that classification sauna parlors, massage
parlors, karaoke bars, night clubs, day clubs, super clubs, discotheques,
cabarets, dance halls, motels and inns. This is not warranted under the
accepted definitions of these terms. The enumerated establishments
are lawful pursuits which are not per se offensive to the moral
welfare of the community. The acrid truth is that sexual immorality,
being a human frailty, may take place in the most innocent of places that
it may even take place in the substitute establishments enumerated
under Section 3 of the Ordinance.
In the instant case, there is a clear invasion of personal or property
rights, personal in the case of those individuals desirous of owning,
operating and patronizing those motels and property in terms of the
investments made and the salaries to be paid to those therein
employed.
If the City of Manila so desires to put an end to prostitution, fornication
and other social ills, it can instead impose reasonable regulations
such as daily inspections of the establishments for any violation of
the conditions of their licenses or permits; it may exercise its
authority to suspend or revoke their licenses for these violations.
It is part of the rights of the citizen to be free to use his faculties in all
lawful ways; to live and work where he will; to earn his livelihood by
any lawful calling; and to pursue any avocation are all deemed
embraced in the concept of liberty.
In another case, it also confirmed that liberty protected by the due
process clause includes personal decisions relating to
marriage, procreation, contraception, family relationships, child
rearing, and education.
Motel patrons who are single and unmarried may invoke this right
to autonomy to consummate their bonds in intimate sexual
conduct within the motel premises. Be it stressed that their
consensual sexual behavior does not contravene any
fundamental state policy as contained in the Constitution.
The directive to “wind up business operations” in 3 months amounts to a
closure of the establishment, a permanent deprivation of property, and is
practically confiscatory. The second and third options·to transfer to any
place outside of the Ermita-Malate area or to convert into allowed
businesses·are confiscatory as well. The penalty of permanent closure in
The conversion into allowed enterprises is just as ridiculous. How may the
respondent convert a motel into a restaurant or a coffee shop, art gallery or
music lounge without essentially destroying its property? This is a taking of
private property without due process of law, nay, even without compensation.
Distinction should be made between destruction from necessity and eminent
domain. It needs restating that the property taken in the exercise of
police power is destroyed because it is noxious or intended for a noxious
purpose while the property taken under the power of eminent domain is
intended for a public use or purpose and is therefore “wholesome.”
It is a violation of the equal protection clause. In the Court’s view, there are
no substantial distinctions between motels, inns, pension houses,
hotels, lodging houses or other similar establishments. By definition,
all are commercial establishments providing lodging and usually meals
and other services for the public. No reason exists for prohibiting
motels and inns but not pension houses, hotels, lodging houses or
other similar establishments. The classification in the instant case is
invalid as similar subjects are not similarly treated, both as to rights
conferred and obligations imposed. It is arbitrary as it does not rest on
substantial distinctions bearing a just and fair relation to the purpose
of the Ordinance.
The standard “where women are used as tools for entertainment” is also
discriminatory as prostitution·one of the hinted ills the Ordinance aims to
banish is not a profession exclusive to women. Both men and women
have an equal propensity to engage in prostitution.
WHITE LIGHT v. CITY OF MANILA (HENRY) Respondents of the present case filed an appeal before the CA, asserting
January 20, 2009 | Tinga, J. | Substantive Due Process that t was a valid exercise of their police power pursuant to the Local
Government
PETITIONER: White Light Corporation, et al.
RESPONDENTS: City of Manila, represented by Mayor Alfredo S. Lim

SUMMARY: Mayor Lim signed into law Ordinance No. 7774, prohibiting
short tiem admission in hotels, motels, and similar establishments.
Aggrieved, the petitioners went to the RTC to file a complaint with prayer
for TRO, which was granted, together with the ruling that the said
ordinance was unconstitutional. Respondents went to the CA who ruled
on their favor, and reversed the RTC ruling. Petitioners then sought
recourse with the SC, who granted their petition and reinstated the RTC
ruling that the ordinance is unconstitutional for being violative of the
rights provided for by the 1987 Constitution.

DOCTRINE: Substantive due process completes the protection


envisioned by the due process clause. It inquires whether the
government has sufficient justification for depriving a person of life,
liberty, or property.

FACTS:
December 1992, Mayor Lim signed into law Ordinance No. 7774,
prohibiting short time admission in hotels, motels, lodging houses,
pension houses, and similar establishments in the City of Manila.
Short time admission – admittance and charging of room rate for
less than 12 hours at any given time / renting out rooms
more than twice a day or any other term that may be
concocted by owners or managers of said establishments
Penalty: Php 5,000.00 / imprisonment (not exceeding 1 year) or both
The same month, Malate Tourist and Development Corporation (MTDC)
filed a complaint with prayer for a TRO with the RTC, assailing the
constitutionality of the said ordinance. MTDC claimed that as owner
and operator of Victoria Court in Malate, it was authorized by PD 259
to admit customers on a short time basis as well as to charge
customers wash up rates for stays of only 3 hours.
Petitioners eventually filed a motion to intervene and to admit attached
complaint-in-intervention on the ground that the ordinance directly
affects their business intersts as operators of drive-in-hotels and
motels in Manila.
RTC granted the motion to intervene, and notified the Solicitor General of
the proceedings. The said trial court then granted MTDC’s motion
and issued a TRO, and eventually, ruled in favor of the petitioners,
declaring the ordinance null and void.
RTC noted that the ordinance “strikes at the personal liberty of the
individual guaranteed and jealously guarded by the Constitution”. It
likened the law to the ordinance annulled in Ynot v. IAC.
Code (LGC) and the Revised Manila Chapter. there is a discrimination against a discrete and insular minority or
CA reversed the RTC decision and held that it did not violate the right to infringement of a fundamental riht.
privacy or freedom of movement, as it only penalizes owners or
operators of estasblishments that admit individuals for short time
stays, and the that the virtually limitless reach of police power is only
constrained by having a lawful object obtained through a lawful
method. CA also said that the adverse effect on the establishments is
justified by the well-being of its constituents in general, and that
liberty is regulated by law, as ruled in Ermita-Malate Hotel v. City
Mayor of Manila.
Aggrieved, the petitioners came to the SC, and hence, this petition for
review on certiorari.

ISSUE/s:
WoN Ordinance No. 7774 is unconstitutional - YES

RULING: SC granted the petition, reversed the CA decision, and reinstated


the RTC decision.

RATIO:
1. SC noted the long line of decisions including City of Manila that for an
ordinance to be valid, it must not only be within the corporate powers of
the local government unit to enact and pass according to the procedure
prescribed by law, it must also conform to the following substantive
requirements:
a. must not contravene the Constitution or any
statute b. must not be unfair or oppressive
c. must not be partial or discriminatory
d. must not prohibit but may regulate trade
e. must be general and consistent with public
policy f. must not be unreasonable
2. Although the court acknowledged that the ordinance falls within the
ambit of the police power of the State, it ruled that the desirability of
the ends of the ordinance do ntot sanctify any and all means for their
achievement. Those means used in the ordinance must align with
the Constitution
3. SC in this case put emphasis on the first article of the Bill of Rights,
and explained the two kinds of due process, both procedural and
substantive. Then, SC said that if due process were confined solely
to its procedural aspects, there would arise absurd situationsof
arbitrary government action, provided the proper formalities are
followed. SC emphasized that substantive due process completes
the protection envisioned by the due process clause. It inquires
whether the government has sufficient justification for depriving a
person of life, liberty, or property.
4. The test of validity of an ordinance on substantive process grounds is
best tested when assessed with the evolved footnote 4 test laid
down by the US SC in US v. Carolene Products, which
acknowledged that the judiciary would refer to the legislature unless
2 snandards of judicial review were established: strict scrutiny for laws
dealing with freedom of the mind or restricting the political process,
and the rational basis standard of review for economic legislation.
Although the SC recognized the ordinance’s goal the curtailment of
sexual behavior (prostitution, adultery, illegal drugs, etc), it cannot be
denied that there are other legitimate activities which the ordinance
would proscribe or impair:
Families are known to choose to pass time in a motel/hotel whilst
power is momentarily out in their homes
Transit passengers who wish to wash up and rest between trips
have a legitimate purpose for abbreviated stays
Any person/groups in need of comfortable spaces for a span of
few hours with purposes other than having sex / using illegal
drugs
The SC laid down the requisites for the legitimacy of the ordinance as a
police power measure:
Interests of the public generally, as distinguished from those of a
particular class, require an interference with private rights
and the means must be reasonably necessary for the
accomplishment of the purpose and not unduly oppressive of
private rights
No other alternative for the accomplishment of the purpose less
intrusive of private rihts can work
Reasonable relation must exist between purposes of the
measure and the means employed for its accomplishment
Lacking the said requisites, the ordinance shall be struck down as an
arbitrary intrusion of private rights.
Since the ordinance does not make any distinction between places
frequented by patrons engaged in illicit activities and patrons
engaged in legitimate actions it prevents legitimate use of the place
where illicit activiites are rare or even unheard of.
Plain reading of the ordinance shows that it makes no classification of
places of lodging, thus deems them all susceptible to illicit patronage
and subject them without exception to the unjustified prohibition.
KMU v. NEDA (DANNAH) Marital status
April 19, 2006 | Carpio, J. | Substantive Due Process: Right to Privacy Names of parents
Height
PETITIONER: Kilusang Mayo Uno et al. Weight
RESPONDENTS: Director-General, National Economic Dev’t Authority Two index fingers and two thumbmarks
Any prominent distinguishing features like moles and others
SUMMARY: Former Pres. GMA issued EO 420, a uniform data collection Tax Identification Number (TIN)
system for government entities under the Executive department such as
ISSUE/s:
the SSS, GSIS, LTO and etc. Petitioners assail its constitutionality on the
grounds of it being a usurpation of legislative power, and violative of the WoN this constitutes a usurpation of legislative power – NO
right to privacy. WoN EO 420 infringes on the citizens right to privacy – NO

The SC ruled in favor of its constitutionality, as EO 420 falls under RULING:.


Section 17 Article VII. Moreover, it does not violate one’s right to privacy
as the data required is the normal data that said gov’t entities already RATIO:
collect from the public (if not less), and there are safeguards created for On Issue #1: Even prior to EO 420, government entities (such as GSIS,
the protection of said data. SSS, Philhealth, Mayors Office, LTO, PRC) have already been
issuing ID cards. EO 420 merely requires said entities to adopt a
DOCTRINE: The right to privacy does not bar the adoption of uniform data collection and format for their IDs.
reasonable ID systems by government entities. The limited specific data are usual data required for personal
identification by said entities. Anyone who applies for or renews a
drivers license provides to the LTO all these 14 specific data.
FACTS: The IDs that the SC actually issues contain 15 specific data. In contrast,
Former Pres. GMA issued EO 420, which directs all government the uniform ID format under EO 420 requires only the first five
agencies and government-owned and controlled corporations to
items listed in
adopt a uniform data collection and format for their existing
identification ID systems. Here the Director-General of NEDA was Section 3, plus the fingerprint, agancy number and common
authorized to streamline and harmonize all gov’t ID systems. reference number, or only eight specific data.
Petitioners contend that this is unconstitutional as it constitutes A unified ID system for all said entities can be achieved in two ways
usurpation of legislative functions by the executive branch of the The heads of these entities can enter into a memorandum of
government and that it infringes on the citizen’s right to privacy. agreement making their systems uniform
The objectives of EO 420 are: The President may by executive or administrative order direct
To reduce costs
said entities under the Executive department to adopt a
To ensure greater convenience
To facilitate private businesses uniform ID data collection and format
To enhance the integrity and reliability of government-issued ID Section 17, Article VII provides that the President shall have control of all
cards executive departments, bureaus and offices. The same section also
To facilitate access to and delivery of quality and effective mandates the President to ensure that the laws be faithfully
government service. executed.
The data to be collected and recorded by the participating agencies shall There are sevseral laws mandating government entities to reduce costs,
be limited to the following (this is Section 3 of said EO):
increase efficiency and in general, improve public services. Thus, in
Name
Home address issuing EO 420, the President is simply performing the constitutional
Sex duty to ensure that laws are faithfully executed.
Picture Clearly, it is well within the constitutional power of the president to
Signature promulgate. In issuing EO 420, the President did not make, alter or
Date of birth repeal any law but merely implemented and executed existing laws.
Place of birth
Moreover, EO 420 (1) does not require special appropriation, (2) is not On US Justice Dept: The law authorized the DOJ to collect and
compulsary on all branches of government, and (3) does not require preserve fingerprints and criminal identification records
additional data beyond what is routinely or usually required. IT DOES nationwide. The US Court ruled that these were confidential and
NOT the Freedom of Information Act
ESTABLISH A NATIONAL ID CARD SYSTEM.
On Issue #2: There have been no complaints in the past from citizens
that the ID cards of said entities violate their right to privacy. In fact,
petitioners in this case do not claim that the ID systems of gov’t
entities prior to EO 420 violate their right to privacy.
The data collection, recording and ID card system under EO 420 will
even require less data collected, stored and revealed than under the
disparate systems prior to EO 420.
Under EO 420, government entities can collect and record only the 14
specific data mentioned in Section 3 of EO 420. In addition,
government entities can show in their ID cards only eight of these
specific data, seven less data than what the Supreme Courts ID
shows.
Also, prior to EO 420, there was no executive issuance to government
entities prescribing safeguards on the collection, recording, and
disclosure of personal identification data to protect the right to
privacy.
Now, under Section 5, the ff safeguards are instituted:
The data shall be limited to those specified in Section 3
In no case shall the collection/compilation of other data in
violation of a person’s right to privacy be allowed
Stringent systems of access to control data shall be instituted
Data collected and stored for this purpose shall be kept and
treated as strictly confidential (a personal/written
authorization of the owner shall be required for access and
disclosure)
The ID card shall be protected by advanced security features
and cryptographic technology
A written request by the Owner shall be required for any
correction or revision
On its face, EO 420 shows no constitutional infirmity because it even
narrowly limits the data that can be collected, recorded and shown
compared to the existing ID systems of government entities. EO 420
further provides strict safeguards to protect the confidentiality of the data
collected, in contrast to the prior ID systems which are bereft of strict
administrative safeguards.
More than 100 countries have compulsary national ID systems. Even
with EO 420, we still wouldn’t have one.
exempts release of some information, including these. clause. The scope of its usage was staggering and all-encompassing.
With the exepction of the 8 data shown, other data collected are treated as With its
not only strictly confidential but also personal matters. Thus, the ruling in
the aforementioned case does not collide but support EO 420.
On Whalen v. Roe, the Court upheld the validity of a New York law tha
requires doctors to furnish the government reports identifying patiens
who received prescription drugs that have a potential for abuse.
Compared to this, the data required under EO 420 are far less sensitive and
far less personal.
On Planned Parenthood v. Danforth, the Court upheld the validity of a law
that required doctors performing abortions to fill up forms, maintain
records for seven years, and allow the inspection of such records by
public health officials.
Also in the same case, the U.S. Supreme Court upheld a law that required
doctors performing an abortion to file a report to the government that
included the doctors name, the womans age, the number of prior
pregnancies and abortions that the woman had, the medical complications
from the abortion, the weight of the fetus, and the marital status of the
woman.
The disclosure requirements of EO 420 are far more benign and do not
constitue a violation of the right to privacy. The data cannot possibly
embarras or humiliate anyone.
On Ople v. Torres, our SC ruled against the assailed issuance, because it
was devoid of safeguards and was annulled solely on the ground that the
subject matter required legislation. Panaganiban stated “The voting is
decisive only on the need for appropriate legislation, and it is only on this
ground that the petition is granted by this Court.”
The assailed executive issuance in Ople v. Torres sought to establish a
National Computerized Identification Reference System, a national
ID system that did not exist prior to the assailed executive issuance.
Obviously, a national ID card system requires legislation because it
creates a new national data collection and card issuance system where
none existed before.
Azcuna and Ynares-Santiago, Dissenting: Notwithstanding respondents'
avowal that EO 420 was merely an internal regulation to promote efficiency
in government operations and greater convenienc for those transacting
business with the government, the unrestricted and unrestrained impact of a
unified multipurpose ID system divested itself of the pretensions of being an
internal management issuance. As the term denoted, the multipurpose ID
card system could be utilized in any and all conceivable situations involving
governmental or even private transactions, as stated by the “Whereas”
ubiquitous application, its legal and practical repercussions would not be
confined solely to the corridors of the executive departments, but would
overflow even beyond.
MMDA v. VIRON (IYA) saying that the EO was a valid exercise of police power of the State
August 15, 2007 | Carpio-Morales, J. | Substantive Due Process air satisfied the two tests of lawful subject and lawful means.
On MR, the RTC reversed its decision saying the EO is an unreasonable
exercise of police power and that MMDA does not have authority to
PETITIONER: Metropolitan Manila Development Authority
close terminals of the petitioners.
RESPONDENTS: Viron Transportation Co., Inc. MMDA filed this petition which faults the RTC for its ruling.
SUMMARY: GMA issued EO 179 which directs the MMDA to implement
ISSUE/s:
a transport system. The MMDA issued orders requiring bus corporations
to abandon their terminals. Viron and Mencorp, transport providers, WON the EO is valid exercise of police power - NO
question the authority of the MMDA and the validity of the EO.
RULING: EO No. 179 is declared NULL and VOID for being ultra vires.
The RTC first ruled in favor of constitutionality, but later reversed its
decision. On petition to the SC by MMDA, EO 179 was declared null and RATIO:
void for being ultra vires. The MMDA does not posses police power and Police power is the plenary power vested in the legislature to make,
the means to be employed provided by the EO is not reasonably ordain, and establish wholesome and reasonable laws, statutes and
necessary. ordinances, not repugnant to the Constitution, for the good and
welfare of the people.This power to prescribe regulations to promote
DOCTRINE: This Court fails to see how the prohibition against the the health, morals, education, good order or safety, and general
existence of respondents’ terminals can be considered a reasonable welfare of the people flows from the recognition that salus populi est
necessity to ease traffic congestion in the metropolis. suprema lex—the welfare of the people is the supreme law.
The MMDA is devoid of authority to implement the Project as envisioned
by the EO. It is the DOTC and not the MMDA which is authorized to
establish and implement a project similar to this case. MMDA does
FACTS: not posses police power.
Specifically challenged are two Orders issued by Judge Pampilo of the By designating the MMDA as the implementing agency of the Project, the
RTC of Manila. President clearly overstepped the limits of the authority conferred by
First Order: Declared EO No. 179 as unconstitutional as it law, rendering E.O. No. 179 ultra vires.
constitutes an unreasonable exercise of police power. Even assuming that the MMDA is vested with police power, the exercise
Second: That which denied the petitioners Motion for of such power does not satisfy the two tests of a valid police power
Reconsideration. measure:
President GMA issued EO 179: Providing for the Establishment of the interest of the public generally, as distinguished from that of a
Greater Manila Mass Transport System particular class, requires its exercise; and
The EO noted that the primary cause of traffic congestion in Metro Manila (2) the means employed are reasonably necessary for the
has been numerous buses plying hte streets and insufficient accomplishment of the purpose and not unduly oppressive
connectivity of the different transport modes. upon individuals.
The EO designated MMDA as the implementing agency for the Greater Notably, the parties herein concede that traffic congestion is a public concern
Manila Mass Transport System Project. that needs to be addressed immediately. Indeed, the E.O. was issued
Viron Transport, a domestic corporation engaged in the business of due to the felt need to address the worsening traffic congestion in Metro
public transport, filed a petition for declaratory relief before the RTC Manila which, the MMDA so determined, is caused by the increasing
of Manila. volume of buses plying the major thoroughfares and the inefficient
He alleges that the MMDA does not have authority to direct provincial connectivity of existing transport systems. It is thus beyond cavil that the
bus operators to abandon their existing bus terminals and thus motivating force behind the issuance of the E.O. is the interest of the
deprive them of use of their property. public in general.
Mencorp Transportation System filed a similar petition. It asked the court As in Lucena, this Court fails to see how the prohibition against the existence
to declare the EO unconstitutional and illegal for transgressing the of respondents’ terminals can be considered a reasonable necessity to
possessory rights of owners and operators of public land ease traffic congestion in the metropolis. On the contrary, the elimination
transportation units over their respective terminals. of respondents’ bus terminals brings forth the distinct possibility and the
The two cases were consolidated. The RTC ruled in favor of constitutionality
equally harrowing reality of traffic congestion in the common parking
areas, a case
of transference from one site to another.
Less intrusive measures such as curbing the proliferation of “colorum”
buses, vans and taxis entering Metro Manila and using the streets for
parking and passenger pick-up points, as respondents suggest,
might even be more effective in easing the traffic situation.
MMDA cannot order the closure of respondents’ terminals not only
because no authority to implement the Project has been granted nor
legislative or police power been delegated to it, but also because the
elimination of the terminals does not satisfy the standards of a valid
police power measure.
SEC v. INTERPORT (ELIEL) A member of the Westmont Group of Companies in Malaysia,
October 6, 2008 | Chico Nazario, J. | Substantive Due Process shall extend or arrange a loan required to pay for the
proposed acquisition by IRC and PRCI.
PETITIONER: Securities and Exchange Comission
RESPONDENTS: Interport Resources Corporation

SUMMARY: IRC entered into a Memorandum of Agreement (MOA) with


GHB. The SEC alleged that the IRC failed to make a timely disclosure of
the said MOA and that some of its directors heavily traded using this
undisclosed material information. The SEC required the IRC to present
the negotiations of their agreement. Upon doing so, the SEC then found
that IRC violated the Rules on Disclosure of Material Facts, and that
some of its directors are in contravention of Secs. 30 and 36 of the
Revised Securities Act. IRC claimed that SEC violated their right to due
process when it ordered them to appear before the SEC. SEC issued an
Omnibus Motion for a special investigatory panel, which was assailed by
IRC. When it was appealed to the CA, it ruled in favor of the IRC for the
reason that there were no implementing rules and regulations regarding
disclosure and insider trading, which IRC allegedly violated. The SEC
brought it up to the SC, who reversed the judgment of the CA. SC ruled
that (1) there is presumption of validity and that the absence of
implementing rules and regulations do not make a specific provision of
law invalid nor is it ambiguous; (2) Secs 30 and 36 were enacted to
promote full disclosure in the public market to avoid unfair competiton
using insider trading; (3) Lack of implementing rules does not bar the
SEC from investigating and looking for probable cause, since the
proceeding itself is summary in nature. All that is necessary is a
reasonable opportunity to be heard; and (4) the preliminary investigation
was an interruption to the prescription period.

DOCTRINE: In order to comply with the requirements of due process, what is


required, among other things, is that every litigant be given reasonable
opportunity to appear and defend his right and to introduce relevant evidence
in his favor.

FACTS:
The Board of Directors of Interport Resources Corporation (IRC)
approved a Memorandum of Agreement with Ganda Holdings
Berhad (GHB).
IRC acquired 100% or the entire capital stock of Ganda Energy
Holdings Inc. (GEHI), which would own and operate a 102
megawatt (MW) gas turbine power-generating barge.
GEHI would assume a 5-year power purchase contract with
NAPOCOR.
In exchange, IRC will issue to GHB 55% of the expanded capital
stock of IRC amounting to 40.88 B shares, which had a total
par value of P488.44 M.
IRC alleged that a press release announcing the approval of the It is well settled that every law has in its favor the presumption of
agreement was sent through facsimile transmission to the Philippine validity. Unless and until a specific provision of the law is declared
Stock Exchange and the SEC, but that the facsimile machine of the invalid and unconstitutional, the same is valid and binding for all
SEC could not receive it. intents
The SEC averred that it received reports that IRC failed to make timely
public disclosures of its negotiations with GHB and that some of its
directors, heavily traded IRC shares utilizing this material insider
information.
The SEC Chairman issued a directive requiring IRC to submit to the SEC
a copy of its aforesaid Memorandum of Agreemenet with GHB.
Further directed all principal officers of IRC to appear at a hearing
before the Brokers and Exchanges Department of the SEC to explain
IRC’s failure to immediately disclose the information as required by
the Rules on Disclosure of Material Facts.
In compliance with the SEC Chairman’s directive, the IRC sent a letter to
the SEC, attaching thereto copies of the MOA.
The SEC Chairman issued an Order finding that IRC violated the Rules
on Disclosure of Material Facts, in connection with the Old Securities
Act of 1936, when it failed to make timely disclosure of its
negotiations with GHB.
In addition, the SEC pronounced that some of the officers and directors
of IRC entered into transactions involving IRC shares in violation of
Sec 30, in relation to Sec 36 of the Revised Securities Act.
IRC filed an Omnibus Motion alleging that the SEC had no authority to
investigate the subject matter, since the jurisdiction was conferred
upon the Prosecution and Enforcement Department (PED under PD
1758).
IRC claims that SEC violate their right to due process when it ordered
that they appear before the SEC and “show cause why no
administrative, civil, or criminal sanctions should be imposed on
them,” and thus shifted the burden of proof to them.
SEC issued an Omnibus Order creating a special investigating panel to
hear the case and the denied the Motion for Continuance. IRC
assailed and filed for reconsideration but also denied.
IRC appealed to the CA who granted their motion and issued a writ of
preliminary injunction. Stating that there were no implementing rules
and regulations regarding disclosure, insider trading, or any of the
provisions of the Revised Securities Acts, which IRC allegedly
violated.
The SEC filed a Motion for Leave to Quash SEC Omnibus Orders so that
the case may be investigated but was denied. Hence this petition.

ISSUE/s:
19. WoN SEC violated IRC rights of due process clause– NO

RULING: Judgment of CA reversed. Petition Granted.

RATIO:
and purposes. The mere absence of implementing rules cannot It is not necessary for the rules to require affiants to appear and
effecitively invalidate provisions of law, where a reasonable testify and to be cross-examined by the counsel of the
construction that will support the law may be given. adverse party. To require otherwise would negate the
It is well-established that administrative authorities have the power summary nature of the administrative or quasi-judicial
to promulgate rules and regulations to implement a given proceedings.
statute and to effectuate its policies, provided such rules and In order to comply with the requirements of due process, what is
regulations conform to the terms and standards prescribed by
the statute as well as purport to carry into effect its general
policies.
This Court does not discern any vagueness or ambiguity in Sec 30 and
36 of the Revised Securities Act, such that the acts proscribed
and/or required would not be understood by a person of ordinanry
intelligence.
The provision explains in simple terms that the insider’s misuse of
nonpublic and undisclosed information is the gravamen of illegal
conduct.
IRC furthere arvers that under Sec 30 of the Revised Securities Act, the
SEC still needed to define the ff terms: “material fact”, “reasonable
persons”, “nature and reliability”, and “generally available.”
Under the law, what is required to be disclosed is a fact of “special
significance” which may be (a) a material fact which would be likely,
on being made generally available, to affect the market price of a
security to a significant extent, or (b) one which a reasonable person
would consider especially important in determining his course of
action with regard to the shares of stock.
IRC failed to allege that the negotiations of their agreement with GHB
were made known to the public through any form of media for there
to be a proper appreciation of the issue presented.
Secs 30 and 36 of the Revised Securities Act were enacted to
promote full disclosure in the securities market and prevent
unscrupulous individuals, who by their positions obtain non-
public information, from taking advantage of an uninformed
public. No individual would invest in a market which can be
manipulated by a limited number of corporate insiders.
Among the words or phrases that the Court upheld as valid standards
were “simplicity and dignity,” “public interest,” and “interests of law
and order.”
The lack of implementing rules cannot suspend the effectivity of
these provisions. Thus, the Court cannot find any cogent
reason to prevent the SEC from exercising its authority to
investigate IRC for violation of Section 8 of the Revised
Securities Act.
Firstly, Sec 4, Rule 1 of the PED Rules of Practice and Procedure,
provides the proceedings before the PED are summary in nature.
Secondly, the law creating PED empowers it to investigate violations
of the rules and regulations promulgated by the SEC and to file and
prosecute such cases.
required, among other things, is that every litigant be given
reasonable opportunity to appear and defend his right and to
introduce relevant evidence in his favor.
In the present case, a criminal case may still be filed against IRC despite the
repeal, since Secs 8, 12, 26, 27 and 23 of the Securities Regulations
Code impose duties that are substantially similar to Secs 8, 30 and 36 of
the repealed Revised Securities Act.
The enactment of the Securities Regulations Code did not result in the
dismissal of the cases; rather, this Court ordered the transfer of one
case to the proper regional trial court and the SEC to continue with
the investigation of the other case.
It is an established doctrine that a preliminary investigation interrups
the prescription period. A preliminary investigation is essentially a
determination whether an offense has been committed, and whether
there is probably cause for the accused to have committed an
offense.
It should be noted that the SEC started investigative proceedings against
IRC as early as 1994. This investigation effectively interrupted the
prescription period.
The CA, thus, reasoned that if the quashal of the orders was granted,
then it would be deprived of the opportunity to determine the
validity of the aforementioned rules and statutory provisions. In
addition, the SEC would merely pursue the same case without the
CA having determined whether or not it may do so in accordance
with due process requirements.
Tinga, J. Concurring: By no means is the Congress impervious to the
concern that certain statutory provisions are best enforced only after an
administrative regulation implementing the same is promulgated. In such
cases, the legislature is solicitous enough to specifically condition the
enforcement of the statue upon the promulgation of the relevant
administrative rules.
Caripo, J. Dissenting: Only the institution of judicial proceedings can
interrupt the running of the prescriptive period.
Respondent filed a petition for the issuance of a writ of habeas data against
MERALCO v. LIM (JP) petitioners (Deyto) and a grant of TRO before the Regional Trial Court
October 5, 2010 | Carpio-Morales, J. | Substantive Due Process – Writ of (RTC)
Habeas Data

PETITIONER: Manila Electric Company


RESPONDENTS: Rosario Gopez Lim

SUMMARY: An anonymous letter denouncing respondent Lim was circulating


in the Plaridel branch of MERALCO. Petitioner Deyto, Head of Human
Resource then sought it best to transfer respondent Lim for her safety.
Respondent appealed her transfer as violative of her right of privacy and
denial of due process through filing for a writ of habeas data. RTC granted
the writ. SC reverses however, explaining that the writ of habeas data does
not apply to property rights such as employment. The NLRC has the propert
jurisdiction on the matter.

DOCTRINE: The writs of amparo and habeas data will not issue to
protect purely property or commercial concerns nor when the grounds
invoked in support of the petitions therefor are vague or doubtful.
Employment constitutes a property right under the context of the due
process clause of the Constitution.

FACTS:
Respondent Rosario Lim, also known as Cherry Lim, is an administrative
clerk at the Manila Electric Company (MERALCO).
An anonymous letter was posted at the door of the Metering Office of the
Administration building of MERALCO Plaridel, Bulacan Sector, at
which respondent is assigned, denouncing respondent.
The letter reads: “Cherry Lim: matapos mong lamunin lahat ng biyaya
ng Meralco, ngayon naman ay gusto mong palamon ang buong
kumpanya sa mga buwaya ng gobyerno. Kapal ng mukha mo,
lumayas ka rito, walang utang na loob.”
Petitioner Alexander Deyto, Head of MERALCO’s Human Resource Staffing,
directed the transfer of respondent to MERALCO’s Alabang Sector in
Muntinlupa as “OTMS Clerk” in light of the receipt of “reports that there
were accusations and threats directed against [her] from unknown
individuals and which could possibly compromise [her] safety and
security.”
Respondent by letter appealed her transfer and requested for a dialogue
so she could voice her concerns and misgivings on the matter,
claiming that the “punitive” nature of the transfer amounted to a
denial of due process. She cited the grueling travel from her
residence in Pampanga to Alabang and back entails, and violation of
the provisions on job security of their Collective Bargaining
Agreement (CBA).
of Bulacan. context of the due process clause of the Constitution. It is
Respondent prayed for the issuance of the writ commanding the evident that respondent’s reservations on the
petitioners to file a written return containing the following:
A full disclosure of the data or info about the report
The measures taken by petitioners to ensure confidentiality
The currency and accuracy of such data
The trial court granted the prayers of respondent including the issuance
of a writ of preliminary injunction directing petitioners to desist from
implementing respondents transfer until such time that petitioners
comply with the disclosures required. Hence the present petition for
review.

ISSUE/s:
WoN the issuance of the writ is outside the parameters expressly set
forth in the Rule on the Writ of Habeas Data – YES

RULING: Petition is granted. RTC decision is reversed.

RATIO:
Respondent’s plea that she be spared from complying with MERALCOÊs
Memorandum directing her reassignment to the Alabang Sector,
under the guise of a quest for information or data allegedly in
possession of petitioners, does not fall within the province of a writ of
habeas data.
Section 1 of the Rule on the Writ of Habeas Data provides: the writ of
habeas data is a remedy available to any person whose right to
privacy in life, liberty or security is violated or threatened by an
unlawful act or omission of a public official or employee or of a
private individual or entity engaged in the gathering, collecting or
storing of data or information regarding the person, family, home
and correspondence of the aggrieved party.
The habeas data rule, in general, is designed to protect by means of
judicial complaint the image, privacy, honor, information, and
freedom of information of an individual. It is meant to provide a forum
to enforce one’s right to the truth and to informational privacy against
abuse in this age of information and technology.
It bears reiteration that like the writ of amparo, habeas data was
conceived as a response, given the lack of effective and available
remedies, to address the extraordinary rise in the number of killings
and enforced disappearances. Its intent is to address violations of or
threats to the rights to life, liberty or security as a remedy
independently from those provided under prevailing Rules.
The writs of amparo and habeas data will not issue to protect purely
property or commercial concerns nor when the grounds
invoked in support of the petitions therefor are vague or
doubtful. Employment constitutes a property right under the
real reasons for her transfer·a legitimate concern respecting the terms
and conditions of one’s employment·are what prompted her to adopt the
extraordinary remedy of habeas data. Jurisdiction over such concerns is
inarguably lodged by law with the NLRC and the Labor Arbiters.
There is no showing from the facts presented that petitioners committed
any unjustifiable or unlawful violation of respondent’s right to privacy
vis-à- vis the right to life, liberty or security. To argue that petitioner’s
refusal to disclose the contents of reports allegedly received on the
threats to respondent’s safety amounts to a violation of her right to
privacy is at best speculative.
POLLO v. CONSTANTINO (HENRY) Back up files in (17) diskettes were examined by the CSC Office
October 18, 2011 | Villarama, Jr., J. | Substantive Due Process for Legal Affairs (OLA), and found that it contained mostly of
pleadings (for and on
PETITIONER: Briccio “Ricky” A. Pollo
RESPONDENTS: Chairperson Karina Constantino-David, et al.

SUMMARY: There was an anonymous letter of complaint sent to the


CSC Chairerson, alleging that petitioner Pollo has been offering legal
assistance to people with administrative cases with the CSC. Chairman
David’s recourse was then to form an IT team to investigate Pollo’s
computer and files and make a back up of them for further examining—
which was what Pollo found violative of his constitutional rights to privacy
and due process because of lack of reasonableness of the search. After
being found guilty of the complaint alleged, he was dismissed by the
CSC. He went to the CA who dismissed his petition, then appealed to the
SC who affirmed the CA’s decision, holding that no constitutional violation
as made by the said government agency.

DOCTRINE: Some government offices may be so open to fellow


employees or the public that no expectation of privacy is reasonable.
Given the great variety of work environments in the public sector, the
question of whether an employee has a reasonable expectation of
privacy must be addressed on a case-by-case basis.

FACTS:
Petitioner Pollo is a former Supervising Personnel Specialist of the CSC
Regional Office No. IV and also the OIC of the Public Assistance and
Liaison Division (PALD) under the “Mamamayan Muna Hindi
Mamaya Na” program of the CSC.
January 2007, there was an unsigned letter-complaint addressed to
respondent CSC Chairperson Constantino-David marked
“Confidential” sent through LBC from a certain Alan San Pascual of
Caloocan. This letter was received by the Integrated Records
Management Office (IRMO) which was marked “Confidential” and left
unopened. The letter alleged that Petitioner Pollo has been offering
legal assistance government employees who have pending cases in
the CSC.
Chairperson David immediately formed a team of 4 personnel with IT
background, issued a memo directing them to investigate and to
back up all the files in computers found in the Mamamayan
Muna PALD and Legal Divisions.
The respondents (specifically Director Unite), being the officials as to where
the said computers were located (also the witnesses of the investigation
and backing up by the IT guys), texted petitioner regarding the memo, to
which he replied and said he’ll leave the matter to Director Unite and will
get a lawyer. The investigation culminated with the sealing of the
computers the next day, for the purpose of preserving all the files
restored therein.
behalf of parties facing charges as respondents), or letters in connection to
admin cases in the CSC and other tribunals.
Based on the finding, Chairperson David issued a Show-Cause Order requiring
petitioner, who had gone on extended leave, to submit explanation within 5
days (order also indicated the findings of the investigation by the CSC OLA).
Petitioner filed a Comment denying the allegations, as he is not a lawyer and
neither is he “lawyering” people with cases in the CSC. He accused CSC
officials of conducting an “fishing expedition” when they gained access to his
computer without his consent as he was on leave, which violated his right
against self-incrimination. That they were not authorized in sealing, copying,
duplicating, and printing such files as such would violate his constitutional
right to privacy and protection against self-incrimination and
warrantless search and seizure.
Moreover, it was being argued that the anonymous letter is not actionable as it
failed to comply with the requirements of a formal complaint under the
Uniform Rules on Administrative Cases in the Civil Service (URACC). In view
of the illegal search, the files/documents copied from his computer without
his consent is thus inadmissible as evidence, being “fruits of a poisonous
tree”. luh
CSC issued Resolution No. 070382, charging petitioner with Dishonesty, Grave
Misconduct, Conduct Prejudicial to the Best Interest of the Service, and
Violation of RA 6713 (Code of Conduct and Ethical Standards for Public
Officials and Employees), he was directed to file an answer and is placed
under preventive suspension effective immediately.
Petitioner filed an Omnibus Motion for Reconsideration to Dismiss and/or to
Defer, assailing that the formal charge was baseless and tha the never aided
anyone with pending cases at the CSC. The CSC resolved to treat the said
motion as the petitioner’s answer.
Petitioner then filed an Urgent Petition assailing both Show-Cause Order and the
Resolution issued by CSC, for having been issued withg grave abuse of
discretion amounting to excess/ total absence of jurisdiction.
Prior the Urgent Petition, petitioner lodged an administrative/ criminal complaint
against respodnents before the Office of the Ombudsman, and a separate
complaint for disbarment against Director Buensalida.
Petitioner received a notice of hearing from CSC, and then filed in the CA an
Urgent Motion for the issuance of TRO and preliminary injunction. Having
failed to attend the pre-hearing with CSC, the latter reset it to another date,
with warting that failure to attend will entitle prosecution to proceed with
formal investigation ex-parte.
Petitioner moved to reset the pre-hearing conference, claiming that the
investigation proceedings must be put into halt pending the resolution of his
petition to the CA. CSC denied his request and scheduled another pre-
hearing conference, which lead for petitioner to file another motion in the CA
citing them in indirect contempt.
CSC issued Resolution No. 071134, denying petitioner’s motion to set aside
the denial of his motion to defer proceedings, and went ahead with the reasonable expectation of privacy must be addressed on
formal investigation, taking the petitioner’s absence as his waiver to his a case-by-case basis.
right.
CSC issued Resolution NO. 071420, finding petitioner Pollo guilty, and
ordering him with a penalty of dismissal. With regard to the legality of
the search conducted, CSC quoted relevant rulings of the US SC:
O’Connor v. Ortega: govt agencies as employers could valildly
conduct search and seizure in the governmental workplace
without meeting probable cause on warrant requirement for
search and seizure.
US v. Simons: federal agency’s comuter use policy foreclosed
any inference of reasonable expectation of privacy on the
part of its employees. Government employers are entitled to
conduct a warrantless search pursuant to an investigation of
work-related misconduct provided that the search is
reasonable in its inception and scope.
CSC was of the view that the search of petitioner’s computer
successfully passed the test of reasonableness for warrantless
searches in the workplace as enunciated in the above cases.
Petitioner added in his petition to the CA the recent resolution of the CSC
ruling on is dismissal, which was dismissed, with the CA finding no
grave abuse of discretion. Aggrieved, petitioner went to the SC.
Hence, this petition.

ISSUE/s:
WoN petitioner’s constitutional rights to due process was violated by the
CSC’s issuance of Resolution No. 071420 – NO
WoN the search authorized by the CSC Chair, the copying of the
contents in the hard drive on petitioner’s computer reasonable in its
inception and scope was unreasonable – NO

RULING: SC denied the petition and affirmed the CA resolution.

RATIO:
The court emphasized that the constitutional guarantee is not a prohibition of
all searches and seizures but only of “unreasonable” searches and
seizures.
A case cited by the SC for reference is the 1987 case of O’Connor v.
Ortega, where a physician was employed by a state hospital, who
claimed for a violation of his Fourth Amendment rights when hospital
officials were investigating charges of mismanagement of the
psychiatric residency program, among others, and earched his office
and seized personal items from his desk and filing cabinets. In this
case, the court ruled that some government offices may be so open
to fellow employees or the public that no expectation of privacy is
reasonable. x x x Given the great variety of work environments in
the public sector, the question of whether an employee has a
As for the reasonableness, the court noted the relevant surrounding
circumstances to consider:
Employee’s relationship to the item seized
Whether the item was in the immediate control of the employee when it
was seized
Whether the employee took actions to maintain his privacy in the item
In accordance with the rules, the court held that, the petitioner failed to prove that
he had actual (subjective) expectation of privacy either in his office or
government-issued computer. He did not allege that he had a separate
enclosed office which he did not share with anyone, and in fact described his
office as “full of people, his friends, unknown people”
Under the facts obtaining, the search conducted on petitioner’s computer was
justified at its inception and scope. Hence, his claim of violation of
constitutional rights must fail.
Carpio, concurring: A government employee cannot expect any privacy when
he uses a government-owned computer because he knows he cannot use
the computer for any private purpose. The constitutional guarantee applies
only to devices privately owned, and not with government property.
Bersamin, concurring and dissenting: The fact that employees may be given
individual accounts and password protection is not deemed to create any
expectation of privacy. Also, unlike the majority, J. Bersamin did not
absolutely waive his right to privacy; OM No. 10 gave the petitioner privileged
access to the Internet and allowed him to use the computer after office hours.
That he used it after office hours entitled him to a reasonable
expectation of privacy vis-à-vis his communications made during such
time.
STO. TOMAS v. PANEDA (DANNAH) order 10 and prohibit DOLE, POEA and TESDA from implementing
November 13, 2012 | Abad, J. | Substantive Due Process: Police Power the same, and enjoy said agencies to comply with the policy of
deregulation.
PETITIONER: Hon. Patricia Sto. Tomas et al. The QC RTC granted Salac et al’s petition. The agencies filed the
RESPONDENTS: Rey Salac et al. (GR 152642), Jose G. Paneda et al. present petition with the Court and Philippine Association of Service
(GR 152710) Exporters intervened in the case as the RTC decision paralyzed the
deployment abroad of OFWs and performance artists.
SUMMARY: These are consolidated cases assailing the constitutionality The Court issued a TRO enjoining QC RTC from enforcing its decision.
of the Migrant Workers Act. In a parallel case, respondents Asian Recruitment Council filed a petition
for TRO against the DOLE Secretary, petitioners et al.
First case: Regarding the constitutionality of Sections 29 and 30 of RA QC RTC also granted the petition but on December 2008, the Republic
9042 (Migrant Worker’s Act) regarding the deregulating of DOLE and informed the Court that on April 2007 former PGMA expressly
regulatory functions of POEA regarding migration of OFWs, the case was repealed the secionts in dispute and adopted the poligyc of close
rendered moot and academic by PGMA’s RA 9422. government regulation. RA 9422 provides that
The POEA shall regulate private sector participation in the
Second case: PASEI filed a TRO against RA 9042 with the RTC of recruitment and overseas placement of workers.
Manila. RTC of Manila held Sections 6, 7 and 9 unconstitutional because It shall also inform migrant workers not only of their rights as
of vagueness of the term illegal recruitment, because of no distinction in workers but also of their rights as human beings.
seriousness of acts and because of jurisdiction matters, respectively. The And it shall deploy only to contries where the PH has concluded
Court reversed this decision and held said acts valid and constitutional. bilateral labor agreements or arrangements
The respondents Salac et al agreed that the repeal of Sections 29 and
Third case (most relevant): Spouses Cuaresma filed for death insurance 30 renders the issues they raised moot and academic.
benefits for the passing of their daughter Jasmin in Saudi Arabia. The GR 167590: Respondent Philippine Association of Service Exporters
NLRC granted their petition and awarded them $113k. The CA held (PASEI) filed a petition for declaratory relief and issuance of TRO
Becmen and White Falcon jointly and severally liable with their Saudi with the MNL RTC seeking to annul Sections 6, 7 and 9 of RA 8042
Arabian employer for actual damages. The Court deleted the award for for being unconstitutional.
actual damages but upheld the liability of Becmen’s corporate officers Section 6 defines the crime of illegal recruitment
under Section 10 of RA 8422. It also upheld the constitutionality of said Section 7 provides penalties for prohibited acts
section, as evidence is required to prove said liability. Section 9 allowed the filing of criminal actions arising from illegal
recruitment before the RTC of province/city where the
DOCTRINE: In the absence of a clear and unmistakable case that the offense was committed or where offended party resides
MNL RTC declared Section 6 unconstitutional, as its definition of “illegal
statute is unconstitutional, the Court must uphold its validity.
recruitment” is vague because it fails to distinguish between licensed
and non-licensed recruiters.
SC: illegal recruitment in Section 6 is clear and unambiguous
FACTS: It makes a distinction between licensed and non-licensed
These consolidated cases pertain to the constitutionality of RA 8042 or recruiters
the Migrant Workers and Overseas Filipinos Act of 1995. Persons who engage in enlisting, contracting without the
On June 7, 1995, Congress enacted RA 8042. This sets the appropriate government license are guilty of illegal
Government’s policies on overseas employment and establishes a recruitment whether or not they commit the wrongful acts
higher standard of protection and promotion of the welfare of OFWs, enumerated
their familes and migrant workers. Recruiters with said license are guilty only if they commit any of
GR 152642 and GR 152710: Sections 29 and 30 commanded the DOLE said acts
to begin deregulating within 1 year the business of handling the It also declared Section 7 unconsitutional because it failed to make
recruitment and migration of OFWs and phase out within 5 years the distinctions as to the seriousness of acts committed
regulatory functions of the POEA. SC: in fixing uniform penalties, Congress was within its prerogative
Respondents Salac et al filed a petition for certiorari, prohibition and to determine what individual acts are equally reprehensible
mandamus against petitioners Sto. Tomas et al to nullify DOLE
department
It also invalidated Section 9 regarding jurisdiction because offended parties filing
the case in their place of residence would negate the general rule on
venue of criminal cases. Venue, said the RTC, is jurisdictional in In the PASEI case, the QC RTC held as unconstitutional the last
penal laws and allowing this violates the right to due process sentence of the 2nd paragraph of Section 10. It pointed out that
SC: there is nothing unconstitutional in Congress fixing an absent sufficient proof that the corporate officers and directors had
alternative venue for violations knowledge of and allowed the illegal recruitment, making them
GR 167590, 182978-79, 184298-99: Respondent spouses Cuaresma
automatically liable would violate their right to due process.
filed a claim for death and insurance benefits and damages against
petitioners Becmen and White Falcon for the death of their daughter But the Court has already held that the liability of corporate directors
Jasmine while working as staff nurse in Saudi and officers is not automatic. To make them liable, there must be a
The Labor Arbiter dismissed the claim since Cuaresmas had already finding that they were remiss in directing the affairs of that company,
received insurance benefits from the Overseas Welfare such as sponsoring or tolerating the conduct of illegal activities.
Administration Act While there is evidence that these companies were at fault in not
On appeal, the NLRC found Becmen and White Falcon jointly and investigating the cause of Jasmin’s death, there is no mention of any
severally liable for Jasmin’s death and ordered them to pay $113k
evidence in the case against them that intervenors Gumabay et al,
Becmen and White Falcon appealed and the CA held both jointly and
severally liable with their Saudi Arabian employer for actual Becmen’s corporate officers and directors, were personally involved
damages. in their company’s particular actions/omissions in Jasmine’s case.
The Court however found Jasmin’s death not work-related since her rape RA 8042 is a police power measure intended to regulate the recruitment and
and death did not occur while she was on duty. It then deleted the deployment of OFWs. It aims to curb the injustices and abuses OFWs
award of actual damages but ruled that Becmen’s corporate directors suffer.
and officers are solidarily liable. The rule is settled that every statute has in its favor the presumption of
It then held that foreign employer Rajab and Silsilah, White Falcon and
constitutionality.
Becmen and the latter’s corporate directors and officers jointly and
severally liable to the Cuaresmas for P2.5M as moral damages, In the absence of a clear and unmistakable case that the statute is
2.5M exemplary damages with attorney’s fees and cost of suit. unconstitutional, the Court must uphold its validity.

ISSUE/s:
(LAST CASE: GUMABAY CASE) WoN Section 10 which holds that corporate
directors, officers and partners of recruitment/placement agencies jointly
and solidarily liable for money claims and damages is unconstitutional
– NO

RULING: WHEREFORE, in G.R. 152642 and 152710, the Court DISMISSES


the petitions for having become moot and academic.

In G.R. 167590, the Court SETS ASIDE the Decision of the Regional Trial
Court of Manila dated December 8, 2004 valid and constitutional.

In the last case, the Court HOLDS the last sentence of the second paragraph
of Section 10 of Republic Act 8042 valid and constitutional. The Court,
however, RECONSIDERS and SETS ASIDE the portion of its Decision in
G.R. 182978-79 and G.R. 184298-99 that held intervenors Gumabay et al
liable with respondent Becmen Services Exporter and Promotion, Inc. to
spouses Cuaresma for lack of a finding in those cases that such intervenors
had a part in the act or omission imputed to their corporation.

RATIO:
DE LIMA v. GATDULA (IYA) Hence, the present petition for review on certiorari by the party of De
February 19, 2013 | Leonen, J. | Substantive Due Process: Writ of Lima, et al. to this court.
Amparo

PETITIONER: Secretary Leila M. De Lima, Director Nonnatus R Rojas


and Deputy Director Reynaldo O. Esmeralda RESPONDENTS:
Magtanggol B. Gatdula

SUMMARY: Gatdula filed before the RTC a petition for the Issuance of a
Writ of Amparo to have the party of De Lima from continuing
investigations against him. Judge Pampilo was assigned the case. Judge
Pampilo ordered for the filing of an Answer by De Lima, which was not
submitted because they believed it was a Return required not an Answer.
Judge Pampilo proceeded with the main case without any Answer or
Return from the part of De Lima and rendered a judgment in favor of
Gatdula.

De Lima, et al. filed before the SC a petition for review on certiorari. The
SC granted that such petition is barred in cases of Amparo. However,
since the appeal sought for is based on irregularities by the RTC Judge,
the court decided to remand back to the lower court for proper hearing
the case.

DOCTRINE: The procedural irregularities in the RTC affected the mode


of appeal of the part of De Lima. Procedural rules are meant to assist the
parties and courts. When it is the judge himself who disregards the rules
of procedure, delay and confusion result.

FACTS:
Gatdula filed a Petition for Issuance of a Writ of Amparo in the RTC of
Manila. It was raffled to the sala of Judge Silbino T. Pampilo Jr.
The Amparo was directed against De Lima, et al. of the NBI. Gatdula
wanted De Lima, et al. to cease and desist from framing him up for
the fake ambush incident by filing bogus charges of Frustrated
Murder.
Instead of deciding whether to issue a Writ of Amparo, Judge Pampilo
issued summons and ordered De Lima, et al. to file an Answer.
Counsel for De Lima, et al. manifested that a Return not an Answer is
appropriate for Amparo Cases.
Judge Pampilo ruled that the Revised Rules of Summary Procedure
should be applied. Thus, an Answer and not a Return is proper.
Judge Pampilo proceeded with the main case without a Return or
Answer form the party of De Lima.
RTC rendered the assailed Decision granting the issuance of the Writ of
Amparo. RTC also granted interim reliefs pray for such as TPO, and
production and inspection orders.
A Motion for Reconsideration was filed by De Lima, et al. but was denied.
ISSUE/s: Returns should be done prior to the hearing, not after as believed by
WON the RTC “Decision” is proper? - NO

RULING: SC will rendered to nullify all orders by Judge Pampilo and directs
him to determine within 48 hours whether the issuance of the Writ of Amapro
is proper.

RATIO:
The SC believes that the petition filed by De Lima, et al. is improper for
an Amparo case. Before furthering with this discussion, the nature of
the remedy of Amparo is to be discussed.
The remedy of the Writ of Amparo is an equitable and extraordinary
remedy to safeguard the right of the people to life, liberty and
security as enshrined in the 1987 Constitution. The Rule on the Writ
of Amparo was issued as an exercise of the Supreme Court’s power
to promulgate rules concerning the protection and enforcement of
constitutional rights. It aims to address concerns such as, among
others, extrajudicial killings and enforced disappearances. It is
conducted in the form of a special proceeding.
The judge or justice makes an immediate evaluation of the facts. After
evaluation, the judge has the option to issue the Writ of Amparo or
immediately dismiss the case.
Dismissal is proper if the petition and affidavits do not show that the
petitioner’s right to life, liberty, or security is under threat or the acts
complained of are not unlawful.
The Issuance of the writ itself sets in motion presumptive judicial
protection for petitioner. The court requires the respondent to file a
Return after the issuance of the writ through the clerk of court.
The Return has the purpose of identifying the issues in the case and to
detail the actions the respondents have taken to determine the fate
of the aggrieved party.
A summary hearing will only ensue after the Return is filed. If the Return
is not filed, the hearing will be done expert. After hearing, the court
will render judgment.
A judgement will only be fully satisfied when the threats to the life, liberty
or security of the petitioner ceases. Until fulfilled, the extraordinary
remedy of Amparo allows vigilant judicial monitoring.
The Decision assured in this case is not appealable for it is deemed an
interlocutory order as suggested by the fact that temporary
protection, production and inspection orders were given together with
the decision.
The confusion of the parties arose due to the procedural irregularities in
the RTC, which are as follows:
Insistence on filing an Answer was inappropriate. It is the Return
that serves as the responsive pleading for petitions for the
issuenac elf Writs of Amparo.
Holding of a hearing on the main case prior to the issuance of
the writ and filing of a Return is irregular.
the Judge.
Body of the decision of the RTC itself is irregular. The privilege of
the Writ of Amparo must be distinguished from the actual
order called the Writ of Amparo. The privilege includes
availment of the entire procedure in outlined in the AM on
Rule on the Writ of Amparo. A judgment which simply grants
the privilege without following the rules is tantamount to
failure.
The procedural irregularities in the RTC affected the mode of appeal of
the part of De Lima. Procedural rules are meant to assist the parties
and courts. When it is the judge himself who disregards the rules of
procedure, delay and confusion result.
A Petition for Certiorari is prohibited for this case. However, given the
circumstances the SC will render a decision to nullify all orders by
Judge Pampilo and directs him to determine within 48 hours whether
the issuance of the Writ of Amapro is proper.
it 80% see-thru, and at the same time, to move it back about 6
FERNANDO v. ST. SCHO (ELIEL) meters to provide parking space for vehicles to park.
March 12, 2013 | Mendoza, J. | Substantive Due Process

PETITIONER: Hon. Ma. Lourdes C. Fernando, City Mayor of Marikina


RESPONDENTS: St. Scholastica’s College and Academy

SUMMARY: Sangunniang Panglungsod of Marikina enacted Ordinance


No. 192, regulating the construction of fences and walls. Following the
requirements provided therein, Sec 3(1) prescribes the height and the
80% open fence type, and Sec 5 provides for the setback requirement in
parking areas. The City Government of Marikina implemented said
ordinance and ordered St. Scho to demolish and replace their fences. St.
Scho requested for an extension of time, but Fernandez (rep. City Gov of
Marikina) enforced the implementation, which opted St. Scho to file
action, assailing that Fernandez acted beyond jurisdiction in ordering
them to comply with Secs. 3.1 and 5. RTC and CA ruled in favor of St.
Scho. Hence this appeal. SC affirmed lower courts decision, and
invalidated Secs. 3.1 and 5 for being unreasonable and oppressive.
There was no justification provided that an open fence type would be
more beneficial, and that the parking space tantamount to illegal taking of
proprety. Secs. 3.1 and 5 failed to pass the rational relationship test
because it was not reasonably necessary to accomplish the city’s
purpose of ensuring public safety.

DOCTRINE: To successfully invoke the exercise of police power as the


rationale for the enactment of an ordinance and to free it from the
imputation of constitutional infirmity, two tests have been used by the
Court:
a. Rational Relationship Test – laws or ordinances are upheld if they
rationally further a legitimate governmental interest
b. Strict Scrutiny Test – the focus is on the presence of compelling, rather
than substantial, governmental interest and on the absence of less
restrictive means for achieving that interest

FACTS:
The Sangguniang Panglungsod of Marikina City enacted Ordinance No.
192 entitled “Regulating the Construction of Fences and Walls in the
Municipality of Marikina”
Sec 3(1) requires the prescribed height of 1 meter and that at least 80%
of it should be open type (see thru). Sec 5 requires that no walls and
fences be built within the 5 meter parking area allowance.
The City Government of Marikina sent a letter to St. Scho ordering them
to demolish and replace the fence of their Marikina property to make
St. Scho requested for an extension of time to comply with the directive. public, considering that it would no longer be for the exclusive
Fernando, however, insited on the enforcement. Hence, St. Scho use of the respondents as it would also be available for use by
filed for a writ of preliminary injunction before the RTC, stating that the general public. This would tantamount to a taking of a total of
Fernandez was acting in excess of jurisdiction in enforcing 3,762.36 sqm of the respondents’ private propert for public use
Ordinance No. 192. without just compensation, in contravention to the constitution.
St. Scho asserted that the implementation of the ordinance on their Therefore, Sec. 5 is invalid.
property would be tantamount to an appropriation of property without
due process of law. Demolishing their fence and constructing 6
meters back would result in the loss of at least 1,808.34 sq.m. worth
P9,041,700.
They also pointed out that the goal of the provisions to deter lawless
elements and criminality did not exist as the solid concrete walls of
the scholl had served as sufficient protection for may years.
Fernandez answered it was valid exercise of police power. RTC ruled in
their favor. Brought it up to the CA, who affirmed the RTC decision,
reasoned out that the objectives stated in Ordinance No. 192 did not
justify the exercise of polic power, as it did not only seek to regulate,
but also involved the taking of the respondents’ property without due
process
Hence this appeal.

ISSUE/s:
WoN Ordinance No. 192 (Secs. 3.1 and 5) is violative of the due process
clause– Yes

RULING: Judgment of lower court affirmed. Petition Denied.

RATIO:
Police power is the plenary power vested in the legislature to make
statutes and ordinances to promote the health, morals, peace,
education, good order or safety and general welfare of the
people.
To successfully invoke the exercise of police power as the rationale
for the enactment of an ordinance and to free it from the
imputation of constitutional infirmity, two tests have been used
by the Court:
Rational Relationship Test – laws or ordinances are upheld
if they rationally further a legitimate governmental
interest
Strict Scrutiny Test – the focus is on the presence of
compelling, rather than substantial, governmental interest
and on the absence of less restrictive means for achieving
that interest.
Even without going to a discussion of the strict scrutiny test, Ordinance
No. 192 must be struck down for not being reasonably necessary to
accomplish the City’s purpose. More importantly, it is oppressive of
private rights.
The Court joins the CA in finding that the real intent of the setback
requirement was to make the parking space free for use by the
Ordinance No. 303 did not cure Ordinance No. 192, because a party
cannot change the legal theory of the case under which the
controversy was heard and decided in the trial court. Much
more, Ordinance 192 regulates fences; Ordinance 303 classifies the
city into specific land uses.
As to Sec 3.1: The ultimate goal of this objective is clearly the prevention
of crime to ensure public safety and security. The means employed
by the petitioners is not reasonably necessary for the
accomplishment of this purpose and is unduly oppressive to
private rights.
The petitioners have not adequately shown, and it does not appear
obvious to the Court, than an 80% see-thru fence would provide
better protection and a higher level of security, or serve as a
more satisfactory criminal deterrent, than a tall solid concrete
wall.
Compelling the respondents to construct their fence in accordance witht
the assailed ordinance is, thus, a clear encroachment on their right to
property, which necessarily includes their right to decide how best to
protect their property.
Petitioners argue that Ordinance No. 192 is a curative statute as it aims
to correct or cure a defect in the National Building Code, namely, its
failure to provide for adequate guidelines for the construction of
fences.
At any rate, there appears to be no insuffieciency in the National
Building Code with respect to parking provisions in relation to
the issue of the respondents. As found, respondents have more
than complied with the law.
Nonetheless, “the general rule is that where part of a statute is void
as repugnant to the Constitution, while another part is valid, the
valid portion, if susceptible to being separated form the invalid,
may stand and be enforced.”
LIGOT v. REPUBLIC (JP) The Ombudsman’s investigation also looked into Mrs. Ligot’s younger
March 6, 2013 | Brion, J. | Substantive Due Process – Freeze Order brother, Edgardo Tecson Yambao. Edgardo was employed in the
private sector. Despite Yamabao’s lack of susbstantial income
PETITIONER: Ret. Lt. Gen. Jacinto Ligot, his wife and his four children hwoever, the records show that he has real properties and vehicles
RESPONDENTS: Republic of the Philippines represented by the Anti- registered in his name amounting to P8,763,550. The Ombudsman
Money Laundering Council concluded that Yambao acted as a dummy of the Ligot Spouses and
all properties registered in Yambao’s name actually belong to the
SUMMARY: Retired Lt. Gen. Ligot, his wife and his four children were Ligot Family.
found by the Ombudsman to have amassed wealth which are grossly The Ombudsman for the Military and Other Law Enforcement Officers
disproportionate to their family’s main income which is Lt. Gen. Ligot’s issued a resolution holding that probable cause exists that Lt.
salary as an AFP officer. After finding probable cause, the CA issued a Gen. Ligot violated Section 8 of RA No. 6713 (Code of Ethical
freeze order of Lt. Ligot and his younger brother, Yambao’s properties Standards for Public Officials) as well as Article 183 of the RPC
and vehicles until all proceedings against them will be terminated. (untruthful statements and perjury).
Meanwhile, a new Rule in Civil Forfeiture proceedings was passed which Republic of the Philippines (Republic), represented by the Anti-Money
limited the period for freeze orders to six months. Ligot filed for a motion Laundering Council (AMLC), filed an Urgent Ex-Parte Application for
for reconsideration then. The SC ruled that the freeze order should be the issuance of a freeze order with the CA against certain monetary
lifted because it is violative of due process and the right of property of a instruments and properties of the petitioners, pursuant to Section 104
person to lose access to his property for more than the six months. The of Republic Act (RA) No. 9160, as amended (otherwise known as the
Ligots have already lost access to their properties for six years. Anti-Money Laundering Act of 2001) based on the Ombudsman’s
letter to it.
DOCTRINE: A freeze order is intended solely as an interim relief. The Accordingly, the CA issued a freeze order against the Ligots and
unsubstantiated extension of such borders on inflicting a punishmen, in Yambao’s various bank accounts, web accounts and vehicles,
violation of the constitutionally protected right to be presumed innocent, valid for 20 days from the date of issuance.
because the unreasonable denial of property comes before the final The Republic filed an Urgent Motion for Extension of Effectivity of Freeze
conviction. Order, arguing that if the bank accounts, web accounts and vehicles
were not continuously frozen, they could be placed beyond the reach
of law enforcement authorities and the government’s efforts to
recover the proceeds of the Ligots’ unlawful activities would be
FACTS: frustrated.
In support of its motion, it informed the CA that the Ombudsman was
Lt. Gen. Ligot served in the Armed Forces of the Philippines (AFP) for 33
presently investigating the following cases involving the Ligots:
years and 2 months as a cadet until his retirement. He and Mrs. Ligot plunder, perjurt, violation of RA 3019, forfeiture proceedings,
have four children. malicious mischief, violation of TA No. 7856, etc.
Lt. Gen. Ligot declared in his SALN that as of December 31, 2003 he had CA granted the motion and extended the freeze order until after all appropriate
assets worth P3,848,003. In contrast, his declared assers in his 1982 proceedings and/or investigations have been terminated.
SALN was just P105,000. Meanwhile, on November 15, 2005 Republic Act No. 9160, as amended
The Ombudmsan revealed that Lt. Gen. Ligot and his family had other (Rule in Civil Forfeiture Cases) took effect. Under this rule, a freeze
properties and bank accounts, not declared in his SALN amounting order could be extended for a maximum period of six months.
to at least P54,001,217. Ligots filed a motion for reconsideration of the CA’s resolution taking into
Bearing in mind that Lt. Gen. Ligot’s main source of income was his account the new rule. CA denied this motion. The Ligots then filed
salary as an officer of the AFP, and given his wife and children’s lack the present petition.
of any other substantial sources of income, the Ombudsman ISSUE/s:
declared the assets registered in Lt. Gen. Ligot’s name, as well WoN the CA, in extending the freeze order, unduly deprived him and his
as those in his wife’s and children’s names, to be illegally family of their property, in violation of due process, and penalized
obtained and unexplained wealth, pursuant to the provisions of them before they had been convicted of the crimes they stand
RA No. 1379 (An Act Declaring Forfeiture in Favor of the State Any accused of – YES
Property Found to Have Been Unlawfully Acquired by Any Public
Officer or Employee and Providing for the Proceedings Therefor). RULING: SC grants the petition and lifts the freeze order issued by the
Court of Appeals without prejudice to, and shall not affect, the preservation
orders that the lower courts have ordered on the same properties in the
cases pending before them.
The CA is hereby ordered to remand the case to the RTC of Manila. protected right to be presumed innocent, because the
unreasonable denial of their property comes before
RATIO:
1. From the ex parte application and the Ombudsman’s complaint, we
glean that Lt. Gen. Ligot himself admitted that his income came from
his salary as an officer of the AFP. Yet, the Ombudsman’s
investigation revealed that the bank accounts, investments and
properties in the name of Lt. Gen. Ligot and his family amount to
more than Fifty-Four Million Pesos (P54,000,000.00). Since these
assets are grossly disproportionate to Lt. Gen. Ligot’s income, as
well as the lack of any evidence that the Ligots have other sources of
income, the CA properly found that probable cause exists that these
funds have been illegally acquired.
2. A freeze order is an extraordinary and interim relief issued by the CA
to prevent the dissipation, removal, or disposal of properties that are
suspected to be the proceeds of, or related to, unlawful activities.
3. The Court issued A.M. No. 05-11-04-SC (Rule in Civil Forfeiture
Cases), limiting the effectivity of an extended freeze order to six
months because to otherwise leave the grant of the extension to
the sole discretion of the CA, which may extend a freeze order
indefinitely or to an unreasonable amount of time, carries
serious implications on an individual’s substantive right to due
process. This right demands that no person be denied his right
to property or be subjected to any governmental action that
amounts to a denial. The right to due process, under these
terms, requires a limitation or at least an inquiry on whether
sufficient justification for the governmental action.
4. The Ligots’ case perfectly illustrates the inequity that would
result from giving the CA the power to extend freeze orders
without limitations. As narrated above, the CA, via its
September 20, 2005 resolution, extended the freeze order over
the Ligots various bank accounts and personal properties “until
after all the appropriate proceedings and/or investigations
being conducted are terminated.”
5. By its very terms, the CA resolution effectively bars the Ligots
from using any of the property covered by the freeze order until
after an eventual civil forfeiture proceeding is concluded in their
favor and after they shall have been adjudged not guilty of the
crimes they are suspected of committing.
6. These period sof extension are way beyond the intent and
purposes of a freeze order which is intended solely as an interim
relief. The term of the CA’s extension too borders on inflicting a
punishment to the Ligots, in violation of their constitutionally
the final conviction.
The freeze order over the Ligots’ properties has been in effect since 2005, while
the civil forfeiture case―per the RepublicÊs manifestation―was filed only in
2011 and the forfeiture case under RA No. 1379―per the petitioners’
manifestation―was filed only in 2012. This means that the Ligots have not
been able to access the properties subject of the freeze order for six
years or so simply on the basis of the existence of probable cause to
issue a freeze order, which was intended mainly as an interim
preemptive remedy.
The evils caused by the law’s silence on the freeze order’s period of
effectivity compelled this Court to issue the Rule in Civil Forfeiture
Cases. Specifically, the Court fixed the maximum allowable extension
on the freeze order’s effectivity at six months. In doing so, the Court
sought to balance the State’s interest in going after suspected money
launderers with an individual’s constitutionally-protected right not to be
deprived of his property without due process of law, as well as ot be
presumed innocent until proven guilty.
As a rule however, the effectivity of a freeze order may be extended by the CA for
a period not exceeding six months. Before or upon the lapse of this period,
ideally, the Republic should have already filed a case for civil forfeiture
against the property owner with the proper courts. Otherwise the property
owner should already be able to fully enjoy his property without any legal
process affecting it.
However, should it become completely necessary for the Republic to further
extend the duration of the freeze order, it should file the necessary motion
before the expiration of the six-month period and explain the reason or
reasons for its failure to file an appropriate case and justify the period of
extension sought. In the present case, the Republic has not offered any
explanation why it took six years before a civil forfeiture case was filed
in court despite the clear tenor of the Rule in Civil Forfeiture Cases
allowing the extension of a freeze order for only a period of six months.
REPUBLIC v. ROQUE (HENRY) already been upheld by the SC in the Southern Hemisphere cases,
September 24, 2013 | Perlas-Bernabe, J. | Substantive Due Process which was denied by the RTC, who held that the SC did not pass
upon the constitutionality of RA 9372 and that respodnent’s petition
PETITIONER: Republic of the Philippines, et al. for declaratory relief was properly filed.
RESPONDENTS: Herminio Harry Roque, et al. Petitioners moved for reconsideration, which was denied by the RTC.
Hence, this petition.
SUMMARY: Respondents filed before the RTC a petition for declaratory
ISSUE/s:
relief assailing the constitutionality of RA 9372. The petitioners moved to
suspend the proceedings, claiming that the same has been pending in WoN there is a valid and existing judicial controversy to rule that RA 9372
the SC (Southern Hemisphere case). RTC granted motion to suspend. is violative of the constitutional right to due process - NO

Supreme Court then dismissed the case of Southern Hemisphere, which RULING: SC granted the petition and set aside the RTC decision and
then lead to the petitioners filing for dismissal of the present case in the dismissed the respondent’s petition for declaratory relief.
RTC, claiming that SC did not pass upon RA 9732’s constitutionality—
RATIO:
RTC denied their motion, and hence they went to the SC.
It is clear that the SC, in Southern Hemisphere, did not make any
SC granted the petition, ruling that since the constitutionality of RA 9732 definitive ruling on the constitutionality of RA 9372. The petitions
wasn’t discussed in the Southern Hemisphere decision and that it was were only dismissed solely based on procedural grounds.
dismissed due to procedural grounds (that, there was no judicial The requisites for the action for declaratory relief are as follows (used by
controversy because no real events are shown to prove threat), then the SC to dismiss Southern Hemisphere case):
present case in the RTC should be dismissed as well. Subject matter of the controversy must be a deed, will, contract,
or other written instrument, statute, executive order, or
DOCTRINE: Allegations of abuse must be anchored on real events regulation, or ordinance
before the courts may step in to settle actual controversies involving Terms of said documents and validity are doubtful and require
rights which are legally demandable and enforceable. judicial construction
There must have been nob reach of the documents in question
There must be an actual justiciable controversy or the
FACTS:
“ripening seeds” of one between persons whose interests
Respondents filed a petition for declaratory relief before the RTC are adverse
assailing the constitutionality of RA 9372 (An Act to Secure the State Issue must be ripe for judicial determination
and Protect our People from Terrorism): Adequate relief is not available through other means or other
Section 3 – vagueness forms of action or proceeding
Section 7 – violating right to privacy of communication and due In the case at bar, no actual judiciable controversy is present. Petitioners
process and the privileged nature of priest-penitent only assert general interests as citizens, and taxpayers and infractions
relationships which the government could prospectively commit if the enforcement of
Section 18 – violating due process, the prohibition against ex the law would remain untrammeled. No particular, real, or imminent
post facto laws or bills of attainder, UDHR, and ICCPR as threat was present.
well as for contradicting Art. 125 of the RPC Citing Southern Hemisphere, the court emphasized that allegations of
Section 26 – violating right to travel abuse must be anchored on real events before the courts may
Section 27 – violating prohibition against unreasonable searches step in to settle actual controversies involving rights which are
and seizures legally demandable and enforceable.
Petitioners moved to suspend the proceedings in the RTC, claiming that That the SC dismissed petitions in Southern Hemisphere, the RTC
certain petitions (in the SC) have already been raised concering RA should have dismissed the respondents’ petition for declaratory relief
9372 (Southern Hemisphere Engagement Network, Inc. v. Anti all the same.
Terrorism Council cases). Their motion to suspend was granted.
SC then dismissed the SC petition re: Southern Hemisphere cases.
Petitioners filed a motion to dismiss in the RTC, with a claim that
respondents failed to satisfy the requisites for declaratory relief and that
RA 9372 had
Presuming that the possession/transportation of cash in excess
BAP v. COMELEC (DANNAH) of P500,000 from May 8-13 was for the purpose of vote-
buying when the same was without tenable justification.
November 27, 2013 | Brion, J. | Substantive Due Process: Police
Petitioners (Bankers Association) assailed its constitutionality and
Power
invoked the Court’s power of judicial review to strike down the Money
Ban
PETITIONER: Bankers Association of the Philippines
RESPONDENTS: Commission on Elections

SUMMARY: For the May 2013 elections, COMELEC promulgated


Resolution No. 9688, the short title of which is the Money Ban
Resolution. It prohibits the withdrawal of amounts more than P100,000
and transportation of cash more than P500,000 without valid
justification/usage during May 8-13. It states that the Anti-Money
Laundering Council will look into transactions that seem dubious, and the
presumption will be that said monies will be used for vote-buying.

The Court on May 10 issued a Status Quo Ante Order. It also released its
decision after the 2013 elections, in which it held that the issue was moot
and academic and it did not fall under the exceptions in which the Court
should proceed in hearing the case. It called on the Congress to look into
the matter and to possibly legislate on the issue for future elections.

DOCTRINE: When issues have become moot and academic and


the four exceptions are absent, the Court will no longer decide on
a case.

FACTS:
COMELEC promulgated Resolution No. 9688 on May 7, 2013 entitled “In
the Matter of Implementing a Money Ban to Deter and Prevent Vote-
Buying in Connection with the May 13, 2013 National and Local
Elections”
To prohibit the withdrawal of cash, encashment of checks and
conversion from May 8-13 exceeding P100,000
To prohibit the possession, transportation and/or carrying of cash
exceeding P500,000
All of that violates the first two sections shall be presumed to be
for the purpose of accumulating funds for vote-buying and
election fraud and shall be treated as a “suspicious
transaction” under RA 9160 or the “Anti-Money Laundering
Act of 2001”. The AMLC is hereby deputized to monitor and
initiate investigations.
COMELEC’s Resolution No. 9688-A amended the aforesaid Resolution
Exempting withdrawals that are routine, regulare under the
“Know Your Client/Customer” policy of the BSP
Resolution. On May 10, the Court issued a Status Quo Ante Order, RULING:. WHEREFORE, we hereby DISMISS the petition for having become
enjoining the parties to maintain the status quo prevailing before the moot and academic. The Status Quo Ante Order issued by the Court on May 10,
issuance of said Money Ban Resolution.
2013,
Bankers Associaation believes that the Money Ban Resolution was
issued without jurisdiction since COMELEC’s power to supervise and
regulate the enjoyment/utilization of franchises or permits under
Section 4 Article IX-C does not extend to the BSP, which is not a
holder of any special privilege from the government.
COMELEC claims that its power to regulate covers banks and
other finance companies, since these entities operate under
an “authority” granted by the BSP. It is of the same nature as
“grants, special priveleges or concessions” under Section 4,
Art IX-C.
BSP has power to regulate and supervise banking operations because
of RA 8791 (The General Banking Law of 2000).
Moreover, they believe that COMELEC’s power to deputize extends only
to law enforcement agencies and only if the President concurs. They
argue that BSP and Anti Money Laundering Council are not law
enforcement agencies unlike NBI and PNP.
COMELEC claims that its power extends to instrumentalies of
the government, and that the Constitutional intent is to give
COMELEC unrestricted access to the full machinery of the
State to ensure free, orderly, honest, peaceful and credible
elections.
Also that presidential concurrence is required only when it
involves agencies under the Executive. Moreover, the
concurrence was given through Memorandum Order No. 52.
They also claim that it violates Section 1 Article III that no person shall be
deprived of life, liberty and property without due process of law, as it
unduly and unreasonably restricts and prohibits the withdrawal,
possession and transporatation of cash.
The choice of the measures that the COMELEC may undertake
is a policy beyond the scope of judicial review.
According to them, although the intent is laudible (to curb vote-buying
and selling), the means employed is not reasonably necessary and
oppressive.
It is not unduly oppressive as it merely limits transactions
involving cash. Only the medium or instrument is affected;
the transaction may proceed using non-cash medium or
instrument.
They also believe that it violates the constitutional presumption of
innocence because it declares that “all cash being transported
exceeding P500k shall be presumed for the purpose of vote-buying
and electoral fraud in violation of the money ban.”

ISSUE/s:
WoN said Resolution is violative of substantive due process – NO
having been rendered functus oficio by the May 13, 2013 elections, is hereby
formally LIFTED.

RATIO:
The Court issued a Status Quo Ante Order on May 10, thus the Money
Ban Resolution was not in force during the most critical period of
elections—from May 10 to actual election day.
With the May 13 elections over, the Money Ban Resolution no longer
finds any application so that the issues raised have become moot
and academic.
The power of judicial review is limited to actual cases or controversies.
If the May 2013 elections had come and gone without any need for the
measures the assailed Resolution put in place and if no such
measure was necessary in the elections that immediately followed,
we believe that it is now premature for the Court to assume that a
similar Money Ban Resolution would be issued in succeeding
elections.
Nothing in the facts indicate the presence of the fourth requirement
(exception) for the Courts to decide on moot cases (that the case is
capable of repetiion yet evading review)
Grave violation of the Constitution
Situation is of exepctional character and paramount public
interest is involved
The constitutional issue raised requires formulation of controlling
principles to guide the bench, bar and public
The case is capable of repetition yet evading review.
We consider it significant that the BSP and Monetary Board continue to
possess full and sufficient authority to address the COMELEC’s
concerns and to limit banking transactions to legitimate purposes
without need for any formal COMELEC resolution if and when the
need arises.
Congress too, at this point, should have taken note of this case and
through its lawmaking powers, it may address the circumstances and
evils the Money Ban Resolution seeks to address.
Petitioners in this case, Manila Memorial and La Funeraria, feeling aggrieved
MANILA MEMORIAL v. DSWD (IYA) by the tax deduction scheme, filed the present case praying that Section
4 of RA 7432 as amended by RA 9257 and the various RRs be declared
December 3, 2013 | Del Castillo, J. | Substantive Due Process:

PETITIONER: Manila Memorial Park, Inc. and La Funeraria Paz-Sucat,


Inc.
RESPONDENTS: Secretaries of Department of Social Welfare and
Development and Department of Finance

SUMMARY: Petitioners in this case assail the Revenue Regulations


implementing the Senior Citizens Act. The Senior Citizens Act grants
senior citizens 20% discount from specific services and provides for a
tax deduction for those providing the services. Petitioners maintain that
the tax deduction is not a just compensation for the discount they provide
for the citizens.

The Supreme Court rules that the law in question and its implementing
rules is promulgated pursuant to police power, thus no just compensation
is needed.

DOCTRINE: While the Constitution protects property rights, petitioners


must accept that the realities of business and the State, in the exercise
of police power, can intervene in the operations of a business which may
result in the impairment of property rights.

FACTS:
RA 7432 was passed into law granting seniors citizens privileges such as
20% discounts in several services.
A Revenue Regulation (No. 02-94) was issued to implement said RA.
Section 2(i) provides fro the definition of Tax Credit. Section 4
provides for the procedure required for private respondents in
complying with the law.
In CIR v Central Luzon Drug Corporation, the Court declared Section 2(i)
of the Revenue Regulation as erroneous as it contravenes RA 7432.
The RA allows private establishment to claim as tax credit the
amount of discounts they grant. The Regulation, however, in
providing the procedure for availment limits/denies such
credit.
The definition of Tax Credit is also found erroneous. The option
to avail of the tax credit benefit depends upon the existence
of a tax liability, but to limit the benefit to a sales discount
does not define it at all and serves no useful purpose.
The law cannot be amended by a mere regulation.
On February 26, 2004, RA 9257 amended certain provisions of RA 7432.
To implement the tax provisions of RA 9257, the Secretary of
Finance issued Revenue Regulation nO. 4-2006. DSWD likewise
issued its own Rules and Regulations.
unconstitutional insofar as these allow business establishments
claim the 20% discount given to senior citizens as tax deduction.
ISSUE/s:
WON the tax deduction scheme prescribed are constitutional? - YES
BECAUSE IT IS AN EXERCISE OF POLICE POWER

RULING: Petitions are hereby denied for lack of merit.

RATIO:
Petitioners posit that the tax deduction scheme contravenes Article III
Section 9 of the Constitution which provides that private property
shall not be taken for public use without just compensation.
Petitioners seek the reversal of the ruling in Carlos Superdrug
Corporation that the tax deduction scheme adopted but he
government is justified by police power.
They assert that although both police power and the power of eminent
domain have the general welfare for their object, there are still
traditional distinctions between the two and that eminent domain
cannot be made less supreme than police power.
Petitioners question whether the deduction scheme is made pursuant to
police power or eminent domain.
That if the 20% discount is an exercise of police power no just
compensation is needed.
However if it is eminent domain, the tax deductions scheme is
unconstitutional because it is not a peso for peso reimbursement of
the 20% discount given to senior citizens. It constitutes a taking of
private property without payment of just compensation.
The Court believes that the State in promoting health and welfare of a
special group of citizens can impose upon private establishments the
burden of partly subsidizing a government program.
The law is legitimate exercise of police power, which has general welfare
for its object.
When the conditions so demand as determined by the legislature,
property rights use bow to the primacy of police power because
property rights, though sheltered by due process, must yield to
general welfare.
It is incorrect for petitioners to insist that the grant of the senior citizen
discount is unduly oppressive to their business, because they have
not taken time to calculate correctly and come up with a financial
report to show that the tax deduction scheme is to their
disadvantage.
While the Constitution protects property rights, petitioners must accept
that the realities of business and the State, in the exercise of police
power, can intervene in the operations of a business which may
result in the impairment of property rights.
Police power is the inherent power of the State to regulate or to restrain
the use of liberty and property for public welfare. The only limitation
is that the restriction imposed should be reasonable, not oppressive.
Eminent domain, on the other hand, is the inherent power of the State to Bersamin, J. concurring opinion: I find that the imposition of the
take or appropriate private property for public use. The Constitution, discount does not emanate from the exercise of the power of
however, requires that private property shall not be taken without due eminent domain, but from
process of law and the payment of just compensation.
In the exercise of the power of eminent domain, property interests are
appropriated and applied to some public purpose which necessitates
the payment of just compensation therefor.
The 20% discount is intended to improve the welfare of senior citizens
who, at their age, are less likely to be gainfully employed, more
prone to illnesses and other disabilities, and, thus, in need of subsidy
in purchasing basic commodities.
The subject regulation affects the pricing, and, hence, the profitability of a
private establishment. However,
the discount does not prevent the establishments from adjusting
the level of prices of their goods and services, and
the discount does not apply to all customers of a given
establishment but only to the class of senior citizens.
Private establishments still have the capacity to revise their pricing
strategy so that whatever reduction in profits that they may sustain
because of the sales to senior citizens, can be recouped through
higher markups or from other products not subject to discounts.
The individual, who enjoys the rights, benefits and privileges of living in a
democratic polity, must bear his share in supporting measures
intended for the common good.
Carpio, J. dissenting opinion: The main points of Justice Carpio’s
Dissent may be summarized as follows:
the discussion on eminent domain in Central Luzon Drug
Corporation is not obiter dicta;
allowable taking, in police power, is limited to property that is
destroyed or placed outside the commerce of man for public
welfare;
the amount of mandatory discount is private property within the
ambit of Article III, Section 9 of the Constitution; and
the permanent reduction in a private establishment’s total
revenue, arising from the mandatory discount, is a taking of
private property for public use or benefit, hence, an exercise
of the power of eminent domain requiring the payment of just
compensation.
Velasco, J. concurring opinion:
RA 9257 is no more than a regulation of the right to profits of
certain taxpayers in order to benefit a significant sector of
society. It is, thus, a valid exercise of the police power.
This Court has recognized the fundamental police power of the
State to regulate the exercise of various rights holding that
“equally fundamental with the private right is that of the
public to regulate it in the common interest”
the exercise of police power.
Lenonen, J. concurring and dissenting opinion:
The imposition of a discount for senior citizens affects the price. It is
thus an inherently regulatory function. However, nothing in the
law controls the prices of the goods subject to such discount.
Legislation interferes with the autonomy of contractual
arrangements in that it imposes a two-tiered pricing system.
There will be two prices for every good or service: one is the
regular price for everyone except for senior citizens who get a
twenty percent (20%) discount. Businesses’ discretion to fix the
regular price or improve the costs of the goods or the service
that they offer to the public — and therefore determine their profit
— is not affected by the law.
The case is premised on the inevitable loss to be suffered by the
petitioners. There is no factual basis for that kind of certainty. We do
not decide constitutional issues on the basis of inchoate losses and
uncertain burdens. Furthermore, income and profits are not vested
rights. They are the results of good or bad business judgments
occasioned by the proper response to their economic environment.
The Constitution provides for limitations on the power of taxation.
First, “the rule of taxation shall be uniform and equitable.” This
requirement for uniformity and equality means that “all taxable
articles or kinds of property of the same class shall be taxed at
the same rate.” The tax deduction scheme for the 20% discount
applies equally and uniformly to all the private establishments
covered by the law. Thus, it complies with this limitation.
owner of the use thereof at the sole determination of any traffic enforcer
LEGASPI v. CITY OF CEBU (ELIEL) or regular PNP
December 10, 2013 | Bersamin, J. | Substantive Due Process

PETITIONER: Valentino L. Legaspi


RESPONDENTS: City of Cebu, Tito Sayson and Ricardo Hapitan

SUMMARY: Sangunniang Panglungsod of Cebu enacted Ordinance No.


1644 authorizing traffic enforecers to immobilize by clamping any vehicle in
violation of the Traffic Code. The Jabans and Legaspi, who both complained
as to the clamping of their vehicles, assailed the ordinance for being
unconstitutional and violative of due process. They complain that there was
no notice or hearing and that it deprives them of their property. The RTC
ruled in their favor. However, the CA overturned the RTC ruling and stated
that Ordinance No. 1644 is a valid exercise of police power. Hence this
appeal. The SC ruled and affirmed the CA decision validating the said
ordinance. It passed the tests of valid ordinances and complied with the
Constitution limitations. And as much as the exercise of police power, it’s
objective is to serve public interest and advance general welfare.

DOCTRINE: Tests of valid ordinance: (a) must not contravene the


Constitution or any statute; (b) must not be unfair or oppressive; (c) must
not be partial or discriminatory; (d) must not prohibit but regulate trade;
(e) must be general and consistent with public policy; (f) must not be
unreasonable

FACTS:
The Sangguniang Panglungsod of City of Cebu enacted Ordinance No.
1664 to authorize the traffic enforcers of Cebu City to immobilize any
motor vehicle violating the parking restrictions and prohibitions
defined in Ordinance No. 801.
Essentially, Ordinance No. 1664 regulates any vehicle violating existing
ordinances to be clamped on the tired with the use of a Denver boot.
Atty. Jaban brought suit in the RTC against the City of Cebu, seeking the
declaration of Ordinance No. 1644 as unconstitutional for being in
violation of due process and for being contrary to law and damages.
He complained that after properly parking for less than 10 minutes,
his car was immobilized by a steel clamp.
Valentino Legaspi likewise sued in the RTC, demanding the delivery of
personal property, declaration of nullity of the Traffic Code of Cebu
City, and damages. Averred that a portion of his car was parked on
the sidewalk, and while waiting for the anay exterminator to finish
unloading, unknown persons had clamed the front wheel of his car.
The cases were consolidated before the RTC and declared the Ordinance
null and void. The violating vehicle is immobilized, thus, depriving its
personnel or Cebu City Traffic Law Enforcement Personnel. Firstly, Ord 1664 is far from oppressive and arbitrary. Any driver or
Otherwise stated, the owner is deprived of his right to the use of vehicle owner whose vehicle was immobilized by clamping
his/her vehicle and penalized without a hearing by a person who is could protest such action of a traffic enforcer or PNP personnel
not legally or duly vested with such rights, power or authority. The enforcing the
City of Cebu appealed to the CA.
The CA overturned the RTC and reasoned that the passage of
Ordinance 1664 is in accordance with the police powers exercised
by the City of Cebu through the Sangunniang Panglungsod and
granted by RA 7160, otherwise known as the LGC.
Upon denial of motion for reconsideration, the Jabans and Legaspic
came to to the Court.

ISSUE/s:
WoN Ordinance No. 1664 is violative of the due process clause– NO

RULING: Judgment of Court of Appeals affirmed. Petition Denied.

RATIO:
Tests of valid ordinance: (a) must not contravene the Constitution
or any statute; (b) must not be unfair or oppressive; (c) must
not be partial or discriminatory; (d) must not prohibit but
regulate trade; (e) must be general and consistent with public
policy; (f) must not be unreasonable
Police power is regarded as “the most essential, insistent and the least
limitable of powers, extending as it does ‘to all the great public needs.”
Indeed, the LGUs would be in the best position to creaft their traffic
codes because of their familiarity with the conditions peculiar to their
communities.
It is in compliance with the guaranty embedded in Art 3. Sec 1. of the
Constitution that “No personal shall be deprived of life, liberty or
property without due process of law, nor shall any person be denied
of the equal protection of the laws.”
In City of Manila, Procedural due process refers to the procedures that
the government must follow before it deprives a person of life, liberty
or property; Substantive due process asks whether the
government has an adequate reason for taking away a person’s
life, liberty, or property.
Even under the stricty scrutiny review, Ordinance 1664 met the
substantive tests of validity and constitutionality by its conformity with
the limitations under the Constitution and the statutes, as well as the
requirements of fairness and reason, and its consistency with public
policy.
The plain objective of Ordinance 1664 was to serve the public
interest and advance the general welfare in the City of Cebu.
There could be no confusion on the meaning and coverage of the
ordinance. But should there be any vagueness and ambiguity in the
provisions, which the OSG does not concede, there was nothing that
a proper application of the basic rules of statutory construction could
not justly rectify.
ordinance.
Secondly, the immobilization of a vehicle by clamping pursuant to
the ordinance was not necessary if the driver or vehicle owner
was around at the time of the apprehension for illegal parking or
obstruction.
And lastly, the towing away of the immobilized vehicle was not
equivalent to a summary impounding, but designed to prevent
the immobilized vehicle from obstructing traffic in the vicinity of
the apprehension and thereby ensure the smooth flow of traffic.
Notice and hearing are the essential requirements of procedural
due process. Yet, there are many instances under our laws in
which the absence of one or both of such requirements is not
necessary a denial or deprivation of due process.
By virtue of the new law, real estate developers will now be compelled to
REMMAN ENTERPRISES v. PRB (JP) hire the services of one licensed real estate broker for every twenty
February 4, 2014 | Villarama, Jr., J. | Substantive Due Process – Police salespersons to guide and supervise the coterie of salespersons
Power under the employ of the real estate developers.

PETITIONER: Remman Enterprises, Inc. and Chamber of Real Estate


and Builders’ Association
RESPONDENTS: Professional Regulatory Board of Real Estate Service
and Profession Regulation Commission (PRC)

SUMMARY: RA No. 9646 was enacted. The RA required that real estate
brokers must hire real estate brokers which should have passed a license
examination. Petitioners Remman Enterprises aver that such ordinance is a
violation due process constituting a deprivation of property due to the added
burden and costs for them being the real estate brokers. RTC denied the
petition. SC upheld the RTC saying it’s a valid exercise of police power to
regulate the proliferation of erring real estate brokers to the detriment of
condominium, lot and house buyers.

DOCTRINE: No right is absolute, and the proper regulation of a


profession, calling, business or trade has always been upheld as a
legitimate subject of a valid exercise of the police power of the State
particularly when their conduct affects the execution of legitimate
governmental functions, the preservation of the State, public health and
welfare and public morals.

FACTS:
R.A. No. 9646, otherwise known as the “Real Estate Service Act of the
Philippines” was signed into law by President Gloria Macapagal-
Arroyo. It aims to professionalize the real estate service sector under
a regulatory scheme of licensing, registration and supervision of real
estate service practitioners.
Petitioners Remman Enterprises and Chamber of Real Estate and
Builder’s Association (CREBA) instituted a civil case wherein they
sought to declare as void and unconstitutional provisions of RA No.
9646.
SEC. 32 (just one of the provisions they sought to declare as
void under the Act). Corporate Practice of the Real Estate
Service.·(a) No partnership or corporation shall engage in
the business of real estate service unless it is duly registered
with the Securities and Exchange Commission (SEC), and
the persons authorized to act for the partnership or
corporation are all duly registered and licensed real
estate brokers, appraisers or consultants
According to petitioners, the new law is constitutionally infirm because: No right is absolute, and the proper regulation of a profession, calling,
It violates Article VI, Section 26 (1) of the 1987 Philippine
Constitution which mandates that “[e]very bill passed by Congress
shall embrace only one subject which shall be expressed in the title
thereof”;
It is in direct conflict with Executive Order (E.O.) No. 648 which
transferred the exclusive jurisdiction of the National Housing
Authority (NHA) to regulate the real estate trade and business to the
now Housing and Land Use Regulatory Board (HLURB), which
authority includes the issuance of license to sell of subdivision
owners and developers pursuant to Presidential Decree (P.D.) No.
957;
It violates the due process clause as it impinges on the real
estate developers’ most basic ownership rights, the right to use
and dispose property, which is enshrined in Article 428 of the
Civil Code;
Section 28(a) of R.A. No. 9646 violates the equal protection
clause as no substantial distinctions exist between real estate
developers and the exempted group mentioned since both are
property owners dealing with their own property.
On due process: Petitions claim that real estate developers
(petitioner being one of them) are now burdened by law to
employ licensed real estate brokers to sell, market and dispose
of their properties. Despite having invested a lot of money, time
and resources in their projects, petitioners aver that real estate
developers will still have less control in managing their
business and will be burdened with additional expenses.
The RTC denied the issuance of the writ of preliminary injunction. Hence
this appeal.

ISSUE/s:
WoN Sections 28(a), 29, and 32 of [R.A. No. 9646], insofar as they affect
the rights of real estate developers, are unconstitutional for violating
substantive due process - NO

RULING: Petition is denied. RTC decision is upheld.

RATIO:
There is no deprivation of property as no restriction on their use and
enjoyment of property is caused by the implementation of R.A. No.
9646. If petitioners as property owners feel burdened by the new
requirement of engaging the services of only licensed real estate
professionals in the sale and marketing of their properties, such is an
unavoidable consequence of a reasonable regulatory measure.
business or trade has always been upheld as a legitimate subject of a valid
exercise of the police power of the State particularly when their conduct
affects the execution of legitimate governmental functions, the preservation
of the State, public health and welfare and public morals.
To pretend that licensing or accreditation requirements violate the due
process clause is to ignore the settled practice, under the mantle
of police power, of regulating entry to the practice of various
trades or professions.
The legislature recognized the importance of professionalizing the ranks
of real estate practitioners by increasing their competence and
raising ethical standards as real property transactions are
“susceptible to manipulation and corruption, especially if they
are in the hands of unqualified persons working under an
ineffective regulatory system.”
RA No. 9646 is thus a valid exercise of the State’s police power. Property
rights must bow to the primacy of police power because property
rights, though sheltered by due process, must yield to general
welfare.
R.A. No. 9646 was intended to provide institutionalized government
support for the development of “a corps of highly respected,
technically competent, and disciplined real estate service
practitioners, knowledgeable of internationally accepted standards
and practice of the profession.” Real estate developers at present
constitute a sector that hires or employs the largest number of
brokers, salespersons, appraisers and consultants due to the sheer
number of products (lots, houses and condominium units) they
advertise and sell nationwide.
As early as in the 70s, there has been a proliferation of errant
developers, operators or sellers who have reneged on their
representation and obligations to comply with government
regulations such as the provision and maintenance of subdivision
roads, drainage, sewerage, water system and other basic
requirements.
The foregoing shows that substantial distinctions do exist between
ordinary property owners exempted under Section 28(a) and real
estate developers like petitioners, and the classification enshrined in
R.A. No. 9646 is reasonable and relevant to its legitimate purpose.
There is no violation of the equal protection clause. The Court thus
rules that R.A. No. 9646 is valid and constitutional.
UE v. PEPANIO (HENRY) at the PNU grad school but no evidence that she finished. Pepanio
January 23, 2013 | Abad, J. | Substantive Due Process earned 27 units in her grad studies at Gregorio Araneta University
Foundation but could no longer be
PETITIONER: University of the East, Dean Eleanor Javier, Ronnie
Gillego, and Dr. Jose C. Benedicto
RESPONDENTS: Analiza F. Pepanio and Mariti D. Bueno

SUMMARY: DECS issued a Revised Manual of Regulations for private


schools, which required professors to have a master’s degree as a
minimum educational qualification for acquiring regular status.
Respondents, unqualified, have been teaching in UE for more than 3
years, and yet they haven’t completed their postgraduate studies yet. UE
offered to extend their probationary period, but the respondents did not
accept the offer. Both filed for a complaint on illegal dismissal to the LA,
who ruled in favor of them. UE appealed to the NLRC, who reversed the
ruling. Respondents filed an appeal to the CA, who granted it and thus
lead the petitioners to submit this petition to the SC, which was granted,
on the ground that ample opportunity was provided to the respondents
but they did not avail of their postgraduate degrees despite the chance
given. The Regulation was then ruled to be reasonable, as it protects not
only the studnets, but also the public, from ill-prepared teachers who lack
knowledge.

DOCTRINE: The government has a right to ensure that only qualified


persons, in possession of sufficient academic knowledge and teaching
skills, are allowed to teach in such institutions.

FACTS:
1992, Department of Education, Culture and Sports (DECS) issued the
Revised Manual of Regulations for Private Schools, Art IX, Sec 44,
paragraph 1(a), of which requires college faculty members to have a
master’s decree as a minimum educational qualification for acquiring
regular status.
1994, Petitioner UE and the UE Faculty Association entered into a 5-year
Collective Bargaining Agreement (CBA) which provided that UE shall
extend only semester-to-semester appointments to college faculty
staffs who did not possess the minimum qualifications. Those
qualified shall be given probationary appointments and performance
on a full-time or full-load basis shall be reviewed for 4 semesters.
Feb 1996, several concerned government agencies issued DECS-
CHED-TESDA-DOLE Joint Order 1 which reiterated the policy in the
Manual of Regulations (refer to fact #1). Consequently, UE President
issued a University Policy stating that beginning SY 1996-1997, it
would hire professors w/o postgraduate degrees only on a semester-
to-semester basis.
UE hired respondent Bueno (1997), and Pepanio (2000) on a semester-to-
semester basis. Both do not qualify for probationary or regular status
because they lacked post graduate degrees. Bueno enrolled 6 subjects
credited as she failed to continue her studies within 5 years.
2001, UE and the UE Faculty Assoc entered into a new CBA, indicating
that UE will extend probationary full-time appointements to full-time
faculty members who have no postgraduate degree provided that the
latter comply with such requirement within their probationary period.
Pursuant to the new CBA, UE extended probationary appointments to
respondents.
2003, UE Dean sent notices to probationary faculty members, reminding
them of the expiration of the allowed period for them to complete
their postgraduate qualification. Pepanio replied and said she was
enrolled at PNU grad school, while Bueno said that she was no
longer interested as she will return to the province.
UE Dean subsequently issued a memorandum expressing
recommendation of extension of probationary appointees for 2 more
semesters for those who want, as directed by UE President. Pepanio
requested for a 3-semester extension but was denied.
UE extended probationary period for the respondents, but neither
reported for work.
Bueno later wrote to UE demanding to be considered a regular employee
based on 6 ½ year service on a full-load basis. Pepanio made the
same demand citing her 3 ½ year service on a full-load basis. Both
requests were denied by UE, hence both filed cases of illegal
dismissal before the Labor Arbiter’s (LA) Office.
LA ruled in favor of the respondents, and stated tha the same were
bound by their old CBA (regular employees, taught at UE for at least
4 semesters), and hence directed UE to reinstate respondents with
backwages. UE appealed to the NLRC.
NLRC set aside the LA decision, ruling that the respondents did not
automatically confer permanent status because of the old CBA,
because they still had to meet standards for permanent employment
provided under the Manual of Regulations (refer to fact#1).
Respodnents sought recourse with the CA.
CA reinstated the LA’s decision by reason of technicality, and held that
the 10-day period for appela already lapsed when UE filed it in the
NLRC. Aggrieved, UE filed this petition to the SC.

ISSUE/s:
WoN Revised Manual of Regulations for Private Schools, Art IX, Sec 44,
paragraph 1(a) violates constitutional right to due process - NO

RULING: SC granted the petition and reversed the CA decision. NLRC


decision reinstated.

RATIO:
Acknowledging the CBA (1994) being used by respondents as defense,
however the postgraduate degree requirement was provided 1992.
This new requirement is not unreasonable. The operation of educational
institutions involves public interest. The government has a right to
ensure that only qualified persons, in possession of sufficient
academic knowledge and teaching skills, are allowed to teach in
such institutions. Government regulation in this field of human
activity is desirable for protecting, not only the students, but the
public as well from ill-prepared teachers who are lacking in the
required scientific or technical knowledge.
In this case, UE gave the respondents more than ample opportunities to
finish their postgraduate degree that was required, but they did not take
advantage of it. Justice, fairness, and due process demand that an
employer should not be penalized for situations where it had little or no
participation or control.
GARCIA v. DRILON (DANNAH) Threats were made regarding halting financial support because Rosalie
June 25, 2013 | Perlas-Bernabe, J. | Substantive Due Process intended to file a case against her husband’s paramour. He beat her
up and her daughter, and left them.
PETITIONER: Jesus Garcia Finding reasonable ground to believe that an imminent danger of
violence against Rosalie and her children exists or is about to recur,
RESPONDENTS: Hon. Drilon, Rosalie Garcia the RTC of
SUMMARY: Rosalie and Jesus got married in 2002, and their relationship
bore
children (the other child Jo-Ann’s was Rosalie’s but Jesus adopted her).
Their relationship turned sour when Rosalie found out Jesus was having
an affair with the manager of one Robinson’s Bank of Bacolod. Jesus
started beating Rosalie and Jo-Ann up, and threatened to cut off financial
support.

Rosalie filed a case against her husband Jesus under RA 9262, in which
the RTC of Bacolod issued several TPOs, amending these and extending
the same. Jesus questioned said TPOs and the constitutionality of RA
9262 for being violative of the due process clause. The CA first issued a
TRO on the TPOs but then dismissed the petition, which prompted Jesus
to file an appeal with the Court.

The Court ruled that RA 9262 is NOT violative of the due process clause,
as the two requirements for substantive due process are present, namely
the interest being curbing domestic violence against women and children,
and the means RA 9262 which includes the issuance of TPOs.

DOCTRINE: It is a constitutional commonplace that the ordinary


requirements of procedural due process must yield to the necessities of
protecting vital public interests, among which is protection of women
and children from violence and threats to their personal safety and
security.

FACTS:
On March 2004, Congress enacted RA 9262 entitled “An Act Defining
Violence Against Women and Their Children, Providing for Protective
Measures for Victims, Prescribing Penalties Therefor, and for Other
Purposes”.
Rosalie Jaype-Garcia filed in her and her minor children’s behalf a TPO
against her husband, Jesus pursuant to RA 9262. She claimed to be
a victim of physical, emotional, psychological abuse as a result of
marital infidelity.
Jesus was very strict, even stopped Rosalie from pursuing her career as
a lawyer. Things turned for the worse when Jesus started having an
affair with a bank manager of Robinson’s bank, Bacolod.
Bacolod issues a TPO.
The TPO included things like prohibiting him from being within 1000 meters
from them and their help, drivers and conjugal home., giving monthly
support, letting the family use 2 cars, giving up all his firearms and etc.
A series of amended TPOs and extensions were granted because Jesus
would not comply with those indicated, he continued to harass them.
There were incidents such as attempted kidnapping on the two youngest
boys, and physical abuse and threats on Jo-Ann, the eldest. She
subsequently filed a criminal complaint against her father for violation
of RA 7610, or the Special Protection of Children Against Child
Abuse Exploitation and Discrimination Act.
Jesus filed before the CA a petition for prohibition with prayer for
injunction and temporary restraining order, challenging (1) the
constitutionality of RA 9262 for being violative of due process and
equal protection, and (2) the validity of the modified TPO issued in
the civil case for being “an unwanted product of an invalid law”.
The CA then issued a 60-day TRO against said TPOs, but dismissed the
petition for failure of Jesus to raise the constitutional issue in his
pleadings before the trial court in the civil case, which is clothed with
jurisdiction to resolve the same.

ISSUE/s:
WoN RA 9262 is violative of substantive due process – NO
WoN the family courts have jurisdiction to rule on constitutionality – YES

RULING:.

RATIO:
On jurisdiction: family courts are on the same level as RTCs, and RTCs
are clothed with jurisdiction to decide on constitutionality of laws. The
issue of constitutionality of RA 9262 should have been raised at the
earliest opportunity.
The intent of Congress in enacting RA 9262 was to curb domestic
violence against women and children, as statistics show that they are
those usually victimized in families.
RA 9262 contains sections giving the courts authority to issue Temporary
Protection Orders, to prevent further acts of violence against women
and their children. Its purpose is to safeguard the offended parties
from further harm, minimize any disruption in their daily life and
facilitate the opportunity and ability to regain control of their life.
Since “time is of the essence in cases of VAWC if further violence is to
be prevented”, courts can issue ex parte a TPO after raffle but before
notice and hearing when the life, limb or property of the victim is in
jeopardy, or when the violence is likely to recur.
The removal and exclusion of Jesus in the VAWC case from the
residence of Rosalie, regardless of ownership of the residence, is not
unconstitutional. Jesus may be removed and excluded from the
residence, but only temporarily for the purpose of protecting
Rosalie.
Such removal and exclusion may be permanent only when no property
rights are violated.
From the foregoing, it can be seen that the two requisites for substantial
due process are present: national interest which is curbing of
domestic violence against women and children, and the law RA 9262
which includes the issuance of TPOs, to protect the victims of this
kind of abuse.
NMSM v. MSSP-VFO (IYA) Bicutan. A Memorandum implementing this was issued on the same
June 05, 2013 | Sereno, CJ. | Substantive Due Process: Publication day.
Informal settles in the area increased and occupied some areas of the Fort
Bonifacio and portions of the LNMB. The Brigadier General Bautiesta
PETITIONER: Nagkakaisang Maralita ng Sitio Masigasig, Inc.
issued
RESPONDENTS: Military Shrine Services- Philippine Veterans Affairs
Office

SUMMARY: Marcos issued a Proclamation which excluded certain


portions of land from the military reservation. The questioned
proclamation contains an added handwritten note at the end. NMSM and
WBLOI filed before the COSLAP a petition to have portions of land they
occupy as alienable and disposable pursuant to the handwritten
addendum.

COSLAP granted their petition. MSS-PVAO, respondents of tis case


appealed to the CA which reversed the decision. On appeal to the SC, it
was found that the addendum is invalid as it was not published in the
Official Gazette.

DOCTRINE: All statutes, including those of local application and private


laws, shall be published a a condition for their effectivity. Mysterious
pronouncements and rumored rules cannot be recognized as biding
unless their existent and contents are confirmed by a valid publication
intended to make full disclosure and give proper notice to the people.

FACTS:
On July 12, 1957, Proclamation No. 423 was issued by President Carlos
P. Garcia. It reserved parcels of land in the Municipalities of Pasig,
Taguig, Parañaque, Rizal, and Pasay for a military reservation now
known as Fort Bonifacio.
On May 28 1967, Ferdinand Marcos issued Proclamation No. 208
amending the aforementioned proclamation. It excluded a certain
area of Fort Bonifacio and reserved it for a national shrine now
known as the LNMB undertake administration of the respondents of
this case.
On. January 7, 1986, Marcos issued another Proclamation (No. 2476)
further amending the proclamation of Garcia. It excluded brangays
Lower Bicutan, Upper Bicutan and Signal Village from its operation
and declared it open for disposition.
At the bottom of is a handwritten addendum which reads: (this is the
center of the controversy)
a. “PS— This includes Western Bicutan (SGD.) Ferdinand
Marcos”
Years later, Cory Aquino issued Proclamation 172 which reiterated the
Proclamation of Marcos, but excluded Lots 1 and 2 of Western
a General Order creating Task Force Bantay to prevent further
unauthorized occupation and cause the demolition of illegal
structures in the Fort.
Nagkakaisang Maralita ng Sitio Masigasig (NMSM), petitioner, filed with
the Commission on Settlement of Land Problems (COSLAP).
Western Bicutan Lot Owners Association, Inc. (WBLOI) intervened.
They pray to have the portions of land alienable and disposable.
COSLAP issued a resolution granting the petition. It ruled that the
handwritten Addendum in an integral part of the Proclamation and
therefore controlling. It also ruled that the Proclamation of Aquino
could not have superseded the Marcos Proclamation as the latter
was exercised with legislative powers and is therefore law. A law
cannot be amended by an executive enactment.
MSS-PVAO filed a petition with the CA to reverse the COSLAP
Resolution. CA granted the petition.
NMSMI and WBLOI appealed with the SC.
ISSUE/s:
WON the handwritten addendum of President Marcos is valid - NO

RULING: Petitions are hereby denied for lack of merit.

RATIO:
SC denies the petitions for lack of merit.
The handwritten addendum was not included when Proclamation NO.
2478 was published din the Official Gazette.
The requirement of publication is indispensable to give effect other law,
unless the law itself has proceeded otherwise.
Publication is indispensable in every case, but the legislature may in its
discretion provide the usual fifteen-day period shall be shortened or
extended.
The reason that such omission would offend due process in so far as it
would deny the public knowledge of the laws that are supposed to
govern it.
It is not unlikely that persons not aware of the law would be prejudiced as
a result; and they would be so not because of failure to comply with
the law but simply because they did not know of its existence.
All statutes, including those of local application and private laws, shall be
published a a condition for their effectivity.
Covered by this rule are presidential decrees and executive orders
promulgated by the President in the exercise of legislative powers.
Mysterious pronouncements and rumored rules cannot be recognized as
biding unless their existent and contents are confirmed by a valid
publication intended to make full disclosure and give proper notice to the
people.
HERMANO OIL MANUFACTURING v. TRB (ELIEL) Injunction. RTC granted stating that it is a suit against the state and
November 26, 2014 | Bersamin, J. | Substantive Due Process beyond its jurisdiction, under PD 1818.

PETITIONER: Hermano Oil Manufacturing & Sugar Corporation


RESPONDENTS: Toll Regulatory Board, Engr. Dumlao, Philippine
National Construction Corporation, Dept. of Public Works and Highways

SUMMARY: Hermano Oil owned parcel of land located at the right side
of the Sta. Rita Exit of the NLEX, which was bounded by an access
fence. Hermano Oil sent a letter to the TRB requesting an easement of
right-of-way. But the TRB denied such request because it would have
detrimental effects. Thereafter, Hermano Oil filed in the RTC to demand
specific performance and a writ of preliminary injunction. The RTC denied
such petition, and the CA affirmed RTC decision dismissing the complaint
because (1) it was a suit against the state; and
it was beyond the jurisdiction of the RTC to issue injunctions against
infrastructure projects. Hence the appeal. The SC affirmed lower courts
decision and stated that (1) it is indeed against the State, and that TRB is
an unincorporated agency of the government except for PNCC; (2)
according to PD 1818 only the SC can issue writ of injunctions in
infrastructure projects; and (3) since NLEX is a special kind of road, it
meets the standard of public interest and safety, therefore it was a valid
exercise of police power.

DOCTRINE: Moreover, the putting up of the access fence on Hermano


Oil’s property was in the valid exercise of police power, assailable only
upon proof that such putting up unduly violated constitutional limitations
like due process and equal protection of the law

FACTS:
Hermano Oil owned a parcel of land located at the right side of the Sta.
Rita Exit of the NLEX. The parcel of land was bounded by an access
fence along the NLEX.
In its letter, Hermano Oil requested that Toll Regulatory Board (TRB)
grant an easement of right of way, contending that it had been totally
deprived of the enjoyment and possession of its property by the
access fence that had barred its entry into and exit from the NLEX.
TRB denied because allowing easement of right-of-way may have
detrimental/adverse effect on the scheduled rehabilitation and
improvent of the NLEX interchanges.
Hermano Oil sued the TRB and ENgr. Dumalo, in the RTC demanding
specific performance, the grant of the easement of right of way and
damages. Stating that the access fecne had totally deprived it of the
use and enjoyment of its property by preventing ingress and egress
to its property.
In reply, the OSG filed a Motion to Dismiss with Opposition to the
Application for the Issuance of TRO and/or Writ of Preliminary
Hermano Oil brought it up to the CA who affirmed the RTC’s dismissal of motories using the NLEX, like gasoline service stationsand food
the complaint. CA stated that NLEX was already existent even before stores.
Hermano Oil had the property and that there were other adequate The limited acess imposed on Hermano Oil’s property did not partake of
access. a compensable taking due to the exercise of the power of eminent
Hence this appeal. domain.

ISSUE/s:
20. WoN the access fence deprived Hermano Oil of its access to
property – NO

RULING: Judgment of lower court affirmed. Petition Denied.

RATIO:
The TRB and the DEPWH performed purely or essentially government
or public functions. As such, they were invested with the inherent
power of sovereignty. Being unincorporated agencies or entities of
the National Government, they could not be sued as such. On his
part, Dumlao was acting as the agent of the TRB in respect of the
matter concerned.
The reliefs being sought by Hermano Oil were beyond the jurisdiction of
the RTC because no court except the SC could issue an injunction
against an infrastructure project of the Government.
PD 1818, issued on January 16, 1982, prohibited judges from
issuing restraining orders against government infrastructure
projects, stating in its sole provision: “No court in the
Philippines shall have jurisdiction to issue any restraining
order, preliminary injunction or preliminary order, preliminary
mandatory injunction in any case, dispute or controversy
involving an infrastructure project.”
It is relevant to mention that the access fence was put up pursusant
to RA 2000 (Limited Access Highway Act), the enforcement of
which was under the authority of the DOTC.
Moreover, the putting up of the access fence on Hermano Oil’s
property was in the valid exercise of police power, assailable
only upon proof that such putting up unduly violated
constitutional limitations like due process and equal protection
of the law.
A toll way is not an ordinary road. As a facility designed to promote
the fastest access to certain destinations, its use, operation and
maintenance require close regulation. Public interest and safty
require the imposition of certain restrictions on toll ways that
do not apply to ordinary roads. As a special kind of road, it is
but reaosonable that not all forms of transport could use it.
The access fence was a reasonable restriction on Hermano Oil’s
property given the location thereof at the right side of Sta. Rita Exit of
the NLEX.
Although some adjacent properties were accorded unrestricted access to
the expressway, there was a valid and reasonable classification for
doing so because their owners provided ancillary services to
There is no question that the property was not taken and devoted for public
use. Instead, the property was subjected to a certain restraint and valid
exercise of police power, Hermano Oil was certainly not entitled to any just
compensation.
Petitioner Francisco I. Chavez, a licensed gun owner to whom a PTCFOR
CHAVEZ v. ROMULO (JP) has been issued, requested the Department of Interior and Local
June 9, 2004 | Sandoval-Gutierrez, J. | Substantive Due Process: Police Government
Power

PETITIONER: Francisco I. Chavez


RESPONDENTS: Hon. Alberto G. Romulo as Executive Secretary,
Director General Hermogenes E. Ebdane, Jr. as Chief of the PNP, et al.

SUMMARY: President Arroyo delivered a speech wherein she stressed


the need for nationwide gun ban in public places. Respondent Ebdane
then issued the assailed guidelines. Petitioner, a licensed gun owner with
a valid permit to carry firearms assailed the guidelines as violative of due
process by infringing on his vested property right of owning a gun. The
DILG did not honor his reqest for reconsideration. The SC held that the
Guidelines is a valid police measure. They are intended to stop the
proliferation of crimes, especially by the NPA and the measure of not
entirely prohibiting guns but only through new valid permits is reasonable.

DOCTRINE: The test to determine the validity of a police measure:


The interests of the public generally, as distinguished from those of a
particular class, require the exercise of the police power; and
The means employed are reasonably necessary for the accomplishment
of the purpose and not unduly oppressive upon individuals.

FACTS:
President Gloria Macapagal-Arroyo delivered a speech before the
members of the PNP stressing the need for a nationwide gun ban in
all public places to avert the rising crime incidents (recent issue is
the death of an NPA leader). She directed the then PNP Chief,
respondent Ebdane, to suspend the issuance of Permits to Carry
Firearms Outside of Residence (PTCFOR)
Acting on President Arroyo's directive, respondent Ebdane issued the
assailed Guidelines. Specific Instructions on the Ban on the Carrying of
Firearms:
All PTCFOR are hereby revoked. Authorized holders of
licensed firearms covered with valid PTCFOR may re-apply
for a new PTCFOR in accordance with the conditions
hereinafter prescribed.
All holders of licensed or government firearms are hereby
prohibited from carrying their firearms outside their residence
except those covered with mission/letter orders and duty
detail orders issued by competent authority.
(DILG) to reconsider the implementation of the assailed invokes this provision, asserting that the revocation of his PTCFOR
Guidelines. pursuant to the assailed Guidelines deprived him of his "vested property
However, his request was denied. Thus, he filed the present petition. right" without due process of law and in violation of the equal protection
of law.
ISSUE/s:
WoN respondent Ebdane is authorized to issue the assailed guidelines –
YES
WoN whether the citizens’ right to bear arms is a constitutional
right –
NO
WoN the revocation of petitioner’s PTCFOR pursuant to assailed
Guidelines is a violation of his right to property - NO
WoN the issuance of the assailed Guidelines is a valid exercise of
police power - YES

RULING: Petition is dismissed. The Guidelines are valid.

RATIO:
It must be emphasized that President Arroyo's speech was just an
expression of her policy and a directive to her subordinate. It cannot,
therefore, be argued that President Arroyo enacted a law through a
mere speech.
However, whenever a specific function is entrusted by law or regulation
to her subordinate, she may act directly or merely direct the
performance of a duty. Thus, when President Arroyo directed
respondent Ebdane to suspend the issuance of PTCFOR, she was
just directing a subordinate to perform an assigned duty. Such act is
well within the prerogative of her office.
Possession of firearms by the citizens in the Philippines is the exception,
not the rule. The right to bear arms is a mere statutory privilege, not
a constitutional right. It is a mere statutory creation.
The first real firearm law is Act No. 1780 enacted by the Philippine
Commission on October 12, 1907. It was passed to regulate the
importation, acquisition, possession, use and transfer of firearms.
The foregoing provision was restated in Section 887 of Act No. 2711
that integrated the firearm laws. Thereafter, President Ferdinand E.
Marcos issued P.D. No. 1866. It codified the laws on illegal
possession, manufacture, dealing in, acquisition of firearms,
ammunitions or explosives and imposed stiffer penalties for their
violation. R.A. No. 8294 amended some of the provisions of P.D. No.
1866 by reducing the imposable penalties.
Being a mere statutory creation, the right to bear arms cannot be
considered an inalienable or absolute right.
Section 1, Article III of the Constitution provides that "no person shall be
deprived of life, liberty or property without due process of law." Petitioner
Where state law gives the issuing authority broad discretion to grant or
deny license application in a closely regulated field, initial applicants
do not have a property right in such licenses protected by the
Fourteenth Amendment. These cases enunciated that the test
whether the statute creates a property right or interest depends
largely on the extent of discretion granted to the issuing authority.
In our jurisdiction, the PNP Chief is granted broad discretion in the
issuance of PTCFOR. This is evident from the tenor of the
Implementing Rules and Regulations of P.D. No. 1866 which
state that "the Chief of Constabulary may, in meritorious cases
as determined by him and under such conditions as he may
impose, authorize lawful holders of firearms to carry them
outside of residence." Following the American doctrine, it is
indeed logical to say that a PTCFOR does not constitute a
property right protected under our Constitution.
Consequently, a PTCFOR, just like ordinary licenses in other regulated
fields, may be revoked any time. It does not confer an absolute right,
but only a personal privilege to be exercised under existing
restrictions, and such as may thereafter be reasonably imposed.
It is apparent from the assailed Guidelines that the basis for its
issuance was the need for peace and order in the society.
Owing to the proliferation of crimes, particularly those
committed by the New People's Army (NPA), which tends to
disturb the peace of the community, President Arroyo deemed it
best to impose a nationwide gun ban. Undeniably, the
motivating factor in the issuance of the assailed Guidelines is
the interest of the public in general.
The only question that can then arise is whether the means
employed are appropriate and reasonably necessary for the
accomplishment of the purpose and are not unduly oppressive.
In the instant case, the assailed Guidelines do not entirely
prohibit possession of firearms. What they proscribe is merely
the carrying of firearms outside of residence. However, those
who wish to carry their firearms outside of their residences may
re-apply for a new PTCFOR. This we believe is a reasonable
regulation.
With the revocation of all PTCFOR, it would be difficult for criminals to
roam around with their guns. On the other hand, it would be easier
for the PNP to apprehend them.
Notably, laws regulating the acquisition or possession of guns have
frequently been upheld as reasonable exercise of the police power.
GSIS v. MONTESCARLOS (HENRY) WoN there was a violation of Milagro’s constitutional right to due process by
July 14, 2004 | Carpio, J. | Substantive Due Process

PETITIONER: Government Service Insurance System, Cebu City Branch


RESPONDENTS: Milagros O. Montescarlos

SUMMARY: Milagros, after the death of her husband, filed for


survivorship benefits from GSIS. GSIS denied the claim, saying that per
Sec 18 of PD 1146, which stated a person has no right to survivorship
pension if the surviving spouse contracted the marriage with the
pensioner within 3 years before the petitioner qualified for the pension.
Milagros lodged a complaint with the RTC who ruled in her favor, which
led GSIS to file an appeal to the CA, who affirmed the same. Aggrieved,
they went to the SC, who affirmed the appellate court’s ruling, as the
provision in PD 1146 is indeed violative of the constitutional right to due
process as it immediately does not allow benefits to be given, providing
no opportunity to be heard by the claimant.

DOCTRINE: The heart of due process is the opportunity to be heard, and


by outright confiscation of the benefits due, without providing an
opportunity to be heard, is already a violation of the constitutional right to
due process on its face.

FACTS:
Sangguniang Bayan member Nicolas (72-year old widower) married
Milagros (43 years old) on July 10, 1983.
1985, Nicolas filed with the GSIS an application for retirement benefits
under PD No. 1146 (Revised Government Service Insurance Act of
1977), with Milagros as sole beneficiary.
Nicolas’ last day of actual service was on Feb 17, 1985.
Jan 1986, GSIS approved Nicolas’ application, effective Feb 17, 1984,
granting a lump sum payment of annity for the first 5 years and a
monthly annuity thereafter.
Nicolas died April 22, 1992, which lead Milagros to file with GSIS a claim
for survivorship pension under PD 1146.
GSIS denied the claim, stating Section 18 of PD 1146, which provides
that a surviving spouse has no right to survivorship pension if the
surviving spouse contracted the marriage with the pensioner within 3
years before the pensioner qualified for the pension. Per GSIS,
Nicolas and Milagros’ wedding is less than 1 year from his date of
retirement (refer to fact #3).
Milagros filed with the RTC a special civil action for declaratory relief,
assailing the validity of Sec 18, PD 1146, disqualifying her form
receiving survivorship pension.
RTC ruled in favor of Milagros, which led to GSIS filing an appeal with
the CA, who affirmed the trial court’s decision. Hence, this petition.

ISSUE/s:
reason of PD 1146 - YES

RULING: SC denied the petition. Sec 18 of PD 1146 is void for being


violative of the constitutional guarantees of due process and equal protection
of the law.

RATIO:
The SC ruled that the proviso, which was the sole basis for the rejection
of Milagros’ claim, is contrary to Art 3 Sec 1 of the Constitution.
It is unduly oppressive in outrightly denying a dependent spouse’s claim
for survivorship pension if the dependent spouse contracted
marriage to the pensioner within a 3-year prohibited period. There is
outright confiscation of benefits due the surviving spouse without
giving the surviving spouse an opportunity to be heard.
Sec 18 undermines the purpose of PD 1146, which is to assure
omprehensive social security and insurance benefits to government
employees and their dependents in the event of sickness, disability,
death, and retirement of the government employees.
PD 1146 has the ff purposes:
To preserve at all times, the actuarial solvency of the funds
administered by the System
To guarantee to the government employee all benefits due him
To expand, increase, and improve social security and insurance
benefits made available to him and his dependents
The law extends survivorship benefits to the surviving and qualified
beneficiaries of the deceased member or pensioner to cushion the
beneficiaries against the adverse economic effects resulting from the
death of the wage earner or pensioner.
CHAVEZ v. COMELEC (DANNAH) and all advertisements on print, radio or television showing
August 31, 2004 | Azcuna, J. | Substantive Due Process: Police the image or mentioning a person’s name, who after the
Power placement becomes a

PETITIONER: Francisco Chavez


RESPONDENTS: COMELEC, Chairman Abalos et al.

SUMMARY: Chavez entered into endorsement agreements with 96 North,


Konka International and G-Box. Due to these agreements, billboards in the
Balintawak Interchange of NLEX were put up, with his name and image as
endorser. A few months after, Chavez filed his certificate of candidacy as
Senator.

On January 2004, COMELEC issued Resolution No. 6520. Section 32 states that
propaganda materials mentioning a person’s name or containing his image, who,
after said placement, becomes a candidate for public office shall immediately
removed said materials. Chavez assailed the constitutionality of said provision,
on the issues of being an invalid exercise of police power and invalid due to
overbreadth, to name a few.

The Court ruled in favor of said provision as the means employed were
necessary to equalize the situation between rich and poor candidates. On the
issue of overbreadth, the Court stated that the provision is limited in its
operation in both time and scope.

DOCTRINE: Police power, is the power to prescribe regulations to promote


the health, morals, peace, education, good order, or safety, and the general
welfare of the people. It is valid if there is a necessity and if the means
employed are reasonably necessary.

A provision is not classified as overbreadth if it is limited in its operation


in both time and scope.

FACTS:
Francisco Chavez entered into formal agreements with certain brands to
endorse their products. These were 96 North (a clothing company),
Konka International Plastics Manufacturing Corporation and G-Box
(amusement and videogames).
Pursuant to these, three billboards were set up along the Balintawak
Interchange of NLEX. These showed him promoting said brands.
On December 2003, Chavez filed his certificate of candidacy for Senator
under Alyansa ng Pag-Asa.
On January 2004, COMELEC issued Resolution No. 6520, which
contained Section 32, the assailed provision herein which states:
All propaganda materials such as posters, streamers, stickers…..
and other materials showing the picture, name of a paerson,
candidate for public office shall be immediately removed by To equalize the situation between popular and rich candidates on
said candidate… within 3 days. Otherwise, he and the radio one hand, and lesser-known or poorer candidates on
station or print media shall be presumed to have conducted another, by preventing the former from enjoying undue
premature campaigning in violation of Section 80 of the
advantage in exposure
Omnibus Election Code
In the same month, COMELEC directed Chavez to comply with the said
provision.
Chavez replied by requesting the COMELEC that he be informed as to
how he may have violated said provision. He also sent a letter asking
to be exempted from the application of Section 32, considering that
the billboards are mere product endorsements and cannot be
construed as paraphernalia for premature campaigning under the
rules.
COMELEC answered by ordering him to remove or cause the removal of the
billboards, or to cover them from public view pending approval of his
request.
Chavez, feeling agrrieved, asked the Court that COMELEC be enjoined
from enforcing the assailed provision and to declare the same as
unconstitutional because it is allegedly:
Gross violation of the non-impairment clause
An invalid exercise of police power
In the nature of an ex-post facto law
Contrary to the Fair Elections Act
Invalid due to overbreadth.

ISSUE/s:
WoN Section 32 is an invalid exercise of police power – NO

RULING:.

RATIO:
Police power, is the power to prescribe regulations to promote the health,
morals, peace, education, good order, or safety, and the general
welfare of the people.
To determine the validity of a police measure, two questions must be
asked:
Does the interest of the public in general, as distinguished from
those of a particular class, require the exercise of police
power?
Are the means employed reasonably necessary for the
accomplishment of the purpose and not unduly oppressive
upon individuals?
Its primary objectives are to:
Prohibit premature campaigning and to level the playing field for
candidates of public office
and publicity premature campaigning and to equalize the situation of all
The latter is a valid reason for the exercise of police power. The obvious candidates. Under
inetnion of this provision is to equalize, as far as practicable, the
situations of rich and poor candidates by preventing the former
from enjoying the undue advantage offered by huge campaign
“war chests”.
Under the Omnibus Election Code, “election campaign” or “partisan
political activity” is defined as an act designed to promote the
election or defeat of a particular candidate to a public office.
Included under this is directly or indirectly soliciting votes, pledges
or support for or against a candidate.
It is true that when Chavez entered into the agreements he acted a s a
private individual. However, when he filed his COC for Senator, the
billboards assumed partisan political character because the same
indirectly promoted his candidacy.
An individual intending to run for public office could pay private
corporations to use him as their image model, which would be a
circumvention of the rule against premature campaigning (Sec 80
OEC)
Under Article IX-C(4) of the Constitution, the COMELEC is expressly
authorized to supervise or regulate the enjoyment/utilization of all
media communication or information to ensure equal opportunity,
time, and space. All these are aimed at the holding of free, orderly,
honest, peaceful, and credible elections.
On violation of the non-impairment clause: Contracts affecting public
interest contain an implied reservation of the police power as a
postulate of the existing legal order. This power can be activated at
anytime to change the provisions of the contract, or even abrogate it
entirely, for the promotion or protection of the general welfare. Such
an act will not militate against the impairment clause, which is
subject to and limited by the paramount police power.
Moreover, the contracts stated that Chavez’ photograph and image shall
be utilized in whatever form, mode and manner in keeping with
norms of decency, reasonableness, morals and law and in whatever
form, mode and manner not contrary to law and norms of decency.
On the issue of it being an ex-post facto law: It should be noted that
the offense is not the putting up of propaganda materials but it is the
non-removal of said materials three days after the effectivity of
COMELEC Resolution No. 6520. Hence, it is not applied
retroactively.
On the alleged violation of the Fair Elections Act: The assailed provision
only regulates the use of billboards (and does not prohibit) to prevent
said act, COMELEC has the power to regulate all propaganda materials.
On the issue of overbreadth: A statute or regulation is considered void for
overbreadth when it offends the constitutional principle that a
governmental purpose to control or prevent activities constitutionally
subject to State regulations may not be achieved by means that sweep
unnecessarily broadly and thereby invade the area of protected
freedoms.
The provision in question is limited in its operation both in time and scope. It
only disallows the continued display of a person’s propaganda materials
and advertisements after he has filed a certificate of candidacy and
before the start of the campaign period. Said materials and
advertisements must also show his name and image.
BAYAN v. ERMITA (IYA) of executive secretary Ermita, to the second group of petitioners
April 25, 2006 | Azcuna, J. | Substantive Due Process which:
Instructed the PNP as well as the LGUs to strictly enforce a “no
permit, no rally” policy,
PETITIONER: Bayan, Katapatan, Kilusang Magbubukid Ng Pilipinas Disperse groups that run afoul to this standard, and
(KMP), Gabriela, et al. Arrest all persons violating the laws of the land as well as
RESPONDENTS: Eduardo Ermita as Executive Secretary of Manila City ordinances on the proper conduct of mass actions and
Mayor Lito Atienza, et. al. demonstrations.
It is also stated that “the rule of calibrated preemptive response
SUMMARY: Groups of petitioners assails the validity of BP No. 880 and is in lieu of maximum tolerance”
the implementing regulation “Calibrated Preemptive Response”. The Petitioners in Bayan, et al contend that the BP is a
main contention of the petitioners is that the BP curtails their right to violation of the Constitution, the ICCPR. and other human rights
peaceably assembly by imposing a permit requirement. The CPR treaties.
likewise imposes a different standard other than that provided in the BP the BP requires a permit before one can stage a public assembly
and is therefore void. regardless of the presence or absence of a clear and present
danger.
The SC ruled that the right to peaceably assembly does maintain primacy in It also curtails the voice of venue and is repugnant to the
the realm of constitutional protection. However it is subject to limitations by freedom of expression clause.
the State should public order and safety be involved. The CPR is declared It is not content-neutral as it does not apply to mass actions in
null and void if it means anything other than the maximum standard support of the government. And that it cannot pass the strict
tolerance standard in the law. scrutiny test.
26 individual petitioners argue that the BP
DOCTRINE: The right to peacably assemble and petition for redress is unconstitutional as it is a curtailment of the right to peacefully
grievances is a right that enjoys primacy in the realm of constitutional assemble and petition for redress of grievances because it
protection. These rights constitute the very basis of a functional and puts a condition or the valid exercise of that right
democratic polity. The right while sacrosanct, is not absolute. It may be it delegates powers to the Mayor without providing clear
so regulated that it shall not be injurious to the equal enjoyment of others standards
having equal rights, nor injurious to he rights of the community or society. In re CPR: it is an ultra vires act that alters the standard of of
maximum tolerance set forth in BP No. 880, and that it is void
FACTS: for being vague and for lack of publication
Present case is consolidated from three groups of petitioners assailing BP No. Petitioners KMU, et al. argue that the Constitutions sets no limits on the
880 and the “Calibrated Preemptive Response” (CPR): right to assembly and therefore BP No. 880 cannot put the prior
First: Bayan, et al. in GR No. 169838 filing their capacity as requirement of securing a permit.
citizens and tax payers and allege that their rights as Assuming that legislature can set limits, the limits provided are
organizations and individuals were violated when the rally unreasonable because:
their participated in was violently dispersed by policemen 0 allowing the Mayor to deny the permit on clear and
Second: 26 individual petitioners who allege that they were convincing evidence of a clear and present danger is
injured, arrested and detained when peaceful mass action too comprehensive
they held was preempted and violently dispersed by police 1 five-day requirement to apply for a permit is too long
Third: Kilusang Mayo Uno (KMU) et al. allege that they conduct a certain events require instead public assembly.
peaceful mass actions and that their rights as organizations In re CPR: they argue that it is preemptive, that the government
and those of their individual members as citizens are takes action even before the rallyists can perform their act. It
affected by the said Bp and the policy of “Calibrated also contravenes the maximum tolerance policy of the BP
Preemptive Response” being followed to implement said BP. and the Constitution as it causes a chilling effect on the
BP No. 880 is also known as The Public Assembly Act of 1985. It is an exercise by the people to the right to peaceably assembly.
act ensuring the free exercise by the people of their right peaceably Respondents argue that:
to assemble and petition the government and for other purposes. Petitioners have not presented evidence that they had been “injured,
The CPR is a policy set forth in a press release by Malacañang, via statement arrested, or detained because of the CPR” and that “those
arrested stand to be charged with violating BP No. 880 and other
offenses.
BP No. 880 nor CPR is void. The BP is content neutral and is
narrowly tailored to serve a significant governmental issue. The Court extends its decision further by directing all to take
Nothing in the BP authorizes the denial of a permit on the basis necessary steps for the immediate compliance with
of a rally program’s content or statements of speakers Section 15 of the BP No. 88- through the
except under the constitutional precent of the “clear and
present danger test”

ISSUE/s:
WON BP No. 880 is void - NO
WON the CPR is void - NULL AND VOID IF IT MEANS ANYTHING
OTHER THAN MAXIMUM TOLERANCE

RULING: Validity of BP. 880 is upheld.

RATIO:
The right to peacably assemble and petition for redress grievances is a
right that enjoys primacy in the realm of constitutional protection.
These rights constitute the very basis of a functional and democratic
polity.
The right while sacrosanct, is not absolute. It may be so regulated that it
shall not be injurious to the equal enjoyment of others having equal
rights, nor injurious to he rights of the community or society.
The power to regulate the exercise of such and other constitutional rights
is termed as the sovereign police power.
What is guaranteed peaceable assembly. The Constitution frowns on
disorder or tumult attending a rally or assembly.
The authority of a municipality to impose regulations in order to assure
the safety and convenience of the people have never been regarded
as inconsistent with civil liberties but rather as one of the means of
safeguarding the good order upon which they ultimately depend.
The law is very clear and nowhere vague it its provisions. Neither is the
law overboard. It regulates the exercise of the right to peaceful
assembly and petition only to extent needed to avoid a clear and
present danger.
On the CPR: The Solicitor general has conceded that the use of the term
should be discontinued, since it does not mean anything other than
the maximum tolerance policy set forth in BP NO. 880
The Court rules that in view of the maximum tolerance mandated by B.P.
No. 880, CPR serves no valid purpose if it means the same thing as
maximum tolerance and is illegal if it means something else.
Accordingly, what is to be followed is and should be that mandated
by the law itself, namely, maximum tolerance.
BP No. 880 cannot be condemned as unconstitutional, it does not curtail
or unduly restrict freedoms, it merely regulates the use of public
places as to the time, place and manner of assemblies. The
“maximum tolerance” is for the benefit of the rallyists and not the
government. The delegated power to mayors to grant permits is valid
because it is subject to the “clear and present danger” standard.
establishment or designation of at least one suitable freedom part or plaza in every
city and municipality of the country within 30 days from the finality of the decision.
This is so that rallyists may express themselves peacefully without permit at any
time.
ST. LUKES v. NLRC (ELIEL) The Director of the Institute of Radiology (Director) issued a final notice
March 7, 2007 | Azcuna, J. | Substantive Due Process to Maribel requiring her to comply wwith RA 7431 by taking and
passing the fortcoming examination scheduled in June 1997.
PETITIONER: St. Luke’s Medical Center Employee’s Association – AFW The Director issued a memorandum to Maribel directing her to submit her
and Maribel Santos PRC Registration form/Examiniation permit per Memorandum.
RESPONDENTS: National Labor Relations Commission (NLRC) and St. The Director issued another memorandum to Maribel adivising her that
Luke’s Medical Center, Inc. only a license can assure her of her continued employment at the
Institute of Radiology of SLMC and giving her the last chance to take
SUMMARY: Congress enacted RA 7431 requring practing Radiology and pass the examination; otherwise SLMC shall be constrained to
Technologist to obtain a proper certification form the Board of Radiologic take action which may include her separation from employment.
Technology. SLMC issued a memorandum and notice to Santos to apply The Director issued a notice to Maribel informing her that SLMC has
for the license required by the law; otherwise, SLMC will be forced to approved her retirement in lieu of separation pay. And that she does
take measures to either remove her from the job or transfer her to a not fit with any of the present vacant positions in the hospital.
vacant position for which she qualifies. Given ample time to take the The Personnel Manager of SLMC issued a “Notice of Separation from
required exam to get her practicing license as a radiology technologist, the Company” to Maribel effective Feb 5 1999 after she failed to
Santos failed to do so and this forced SLMC to retire her. Santos went to present/submit her appeal for rechecking to the PRC of the recent
the LA, who ordered SLMC to pay her separation pay. Santos was board examination which she took and failed.
dissatisfied, and brought it to the NLRC, who affirmed the decision. Maribel filed a complaint agasint SLMC for illegal dismissal and non-
Hence this appeal. The SC ruled that RA 7931 and RA 4226 were valid payment of salaries, allowances and other monetary benefits.
exercise of police power because it involves public health. Security of In the meantime, AFW addressed a letter to Rita Marasigan, HR SLMC,
tenure may be regulated in order to ensure that those practicing a certain to assign Maribel to the vacant position arising form the death of an
profession will be competent. In this case, radiology technologist requires employee. To which Rita denied the assignment.
scientific and technological knowledge; thus, regulation in this fiels is a The Labor Arbiter odered SLMC to pay separation pay. Maribel, dissatisfied,
reasonable method of protecting public health and safety. appealed to NLRC, who affirmed LA decision. Hence this appeal.

DOCTRINE: While the right of workers to security of tenure is guaranteed ISSUE/s:


by the Constitution, its exercise may be reasonably regulated pursuant to WoN RA 7431 violated Maribel’s right to due process – NO
the police power of the State to safeguard health, morals, peace,
RULING: Petition Denied.
education, order, safety and the general welfare of the people
RATIO:
While the right of workers to security of tenure is guaranteed by the
Constitution, its exercise may be reasonably regulated pursuant
FACTS:
to the police power of the State to safeguard health, morals,
Maribel S. Santos was hired as X-Ray Technician in the Radiology peace, education, order, safety and the general welfare of the
deparment of St. Luke’s Medical Center, Inc. (SLMC). She is a people.
graduate of Associate in Radiologic Technology from The Family Consequently, person who desire to engage in the learned
Clinic Incorporated School of Radiologic Technology. professions requiring scientific or technical knowledge may be
Congress passed and enacted RA 7431, “Radiologic Technology Act of required to take an examination as a prerequisite to engaging in
1992.” Indicates that no personal shall practice or offer to practice as their chosen careers.
a radiology and/or x-ray technologist in the Philippines without It has long been recognized that the regulation of this field is a
having obtained the proper certificate of registration from the Board reasonable method of protecting the health and safety of the
of Radiologic Technology. public to protect the public from the potentially deadly effects of
The Assistnat Executive and HR Director of SLMC issued a final notice to incompetence and ignorance among those who would practice
all practitioners of Radiologic Technology to compy with the medicine.
requirement of RA 7431 by December 31, 1995; otherwise, the No malice or ill-will can be imputed upon SLMC as the separation of
unlicensed employee will be transferred to an area which does not Maribel was undertaken by it comformably to an existing
reauire a license to practice if a slot is available. statute. It is undeniable that her continued employment without
the required Board certification exposed the hospital to
possible sanctions and even to a
revocation of its license to operate
This notwithstanding, the records bear out the fact that Maribel was given
ample opportunity to qualify for the position and was sufficiently warned
that her failure to do so would result in her separation form work in the
event there were no other vacant positions to which she could be
transferred.
While our laws endeavor to give life to the constitutional policy on social
justice and the protection of labor, it does not mean that every labor
dispute will be decided in favor of the workers.
SLMC is free to determine, using its own discretion and business
judgment, all elements of employment, “from hiring to firing” except
in cases of unlawful discrimination or those which may be provided
by law.
The fact that another employee, who likewise failed to pass the required
exam, was allowed by SLMC to apply for and transfer to another
position with the hospital does not consitutte unlawful discrimination.
This was a valid exercise of management prerogative, Maribel not
having alleged nor proven that the reassigned employee did not
qualify for the position where she was transferred.
SEC OF DND v. MANALO (JP) water and got some of his body parts burnt with burning wood. He was
October 7, 2008 | Puno, J. | Substantive Due Process: Writ of Amparo threatened that

PETITIONER: Secretary of National Defense, The Chief of Staff, Armed


Forces of the Philippines
RESPONDENTS: Raymond Manalo and Reynaldo Manalo

SUMMARY: Suspecting that petitioners are NPA sympathizers, CAFGU


soldiers forcibly captured and detained them in different camps where
they were tortured. Upon their escape, they filed petitions for injunctions
and TROs. However, upon the coming into effect of the writ of amparo,
petitioners requested to petition under it instead. RTC and CA granted the
writ of amparo. SC likewise granted the writ of amparo explaining that the
threat to the life, liberty and security of the respondents is still apparent
since they can still be abducted again, especially after reporting such
incident.

DOCTRINE: The writ of amparo remedy provides rapid judicial relief as it


partakes of a summary proceeding that requires only substantial
evidence to make the appropriate reliefs available to the petitioner; it is
not an action to determine criminal guilt requiring proof beyond
reasonable doubt, or liability for damages requiring preponderance of
evidence, or administrative responsibility requiring substantial evidence
that will require full and exhaustive proceedings.

The swiftness required to resolve a petition for a writ of amparo through


summary proceedings and the availability of appropriate interim and
permanent reliefs under the Amparo Rule, this hybrid writ of the common
law and civil law traditions - borne out of the Latin American and
Philippine experience of human rights abuses - offers a better remedy to
extralegal killings and enforced disappearances and threats thereof.

FACTS:
Petitioner Rarymond Manalo was sleeping in their house in Buhol na
Mangga, Bulacan when several armed CAFGU soldiers entered their
house and roused him. The armed soldiers were looking for Bestre,
his brother (also known as Ka-Bestre of the NPA). He was then
forcibly taken outside his house.
Raymond was forced into a van blindfolded together with other captured
people. When they stopped, he was forced into a room where he
was tortured. He saw his brother being tortured in another room. The
soldiers kepd on asking him if he was a member of the NPA and
everytime he said no, he was hit with the butt of their guns.
Raymond was fed only at night usually with left-over and rotten food. He was
once beat up when he was sleeping, got doused with urine and hot
when they would come bak the next day, they would kill him.
When he attempted escaping once, he was captured by some soldiers
and he was again beat up and got poured with gasouline. Then a so-
called “Mam” suddenly called saying that she wanted to see
Raymond before he was killed. Torture ceased and he was just
detained inside Fort Magsaysay where Reynaldo was detained.
Raymond was detained in a room enclosed by steel bars. He stayed all
the time in that small room measuring 1 x 2 meters, and did
everything there, including urinating, removing his bowels, bathing,
eating and sleeping. He counted that eighteen people had been
detained in that bartolina, including his brother Reynaldo and
himself.
In a sudden turn of events, they were then treated by a doctor and some
nurses. They were given medicine.
They were transferred into a big unfinished house inside the compounf of
a certain “Kapitan” for about three months. They were brought to sit
on a basketball court one night where Gen. Palparan offered them a
deal where they would live if they tell their parents not to go to rallies,
hearings about human rights. They agreed and they were returned to
their parents’ houses and did as so. Their parents were threatened
that if they do no accede, they would not see their children again.
The Manalos were brought back to the detention site.
Reynaldo was then brought to Camp Tecson where they were chained at
night but unchanined in daytime to clean the barracks. They were
threatened that if they escaped, their families would be all killed.
Respondents initially filed an action for "Prohibition, Injunction, and
Temporary Restraining Order” to stop petitioners and/or their officers
and agents from depriving the respondents of their right to liberty and
other basic rights, prior to the promulgation of the Amparo Rule. They
also sought ancillary remedies including Protective Custody Orders,
Appointment of Commissioner, Inspection and Access Orders and
other legal and equitable remedies under Article VIII, Section 5(5) of
the 1987 Constitution and Rule 135, Section 6 of the Rules of Court.
However, when the Amparo Rule came into effect on October 24, 2007,
they moved to have their petition treated as an amparo petition as it
would be more effective and suitable to the circumstances of the
Manalo brothers' enforced disappearance. The Court granted their
motion.
The RTC ruled to treat the petition as a petition under the Amparo rule
and issued the writ of Amparo requiring the respondents to file with
the CA verified return within 5 days from the issuance. CA affirmed
the decision of the RTC and granted the privilege of the writ of
Ampraro. Petitioners here assail this decision.

ISSUE/s:
WoN the writ of Amparo should be issued in the present case - YES However, with the swiftness required to resolve a petition for a
writ of
RULING: Petition is dismissed. CA decision is affirmed. The writ of amparo is
granted, requiring the Sec. of DND and the AFP Chief of staff to:
Furnish all investigations taken related to this case
Confirm in writing the present places of official assignment of involved
officials
Produce all medial reports given to the petitioners and all medical
personal who attended them

RATIO:
The adoption of the Amparo Rule surfaced as a recurring proposition in
the recommendations that resulted from a two-day National
Consultative Summit on Extrajudicial Killings and Enforced
Disappearances.
The writ of amparo the spread throughout the Western Hemisphere,
gradually evolving into different forms. It later evolved for several
purposes: 1. Amapro libertad for the protection of personal freedom,
equivalent to the writ of habeas corpus; 2. Amparo contra leyes for
judicial review of the constitutionality of statues; 3. Amparo casacion
for the judicial review of administrative actions; 4. Amparo
administrativo for the judicial review of administrative actions; 5.
Amparo agrario for the protection of peasants’ rights derived from the
agrarian reform process.
The Court promulgated the Amparo Rule "in light of the prevalence of
extralegal killing and enforced disappearances." It was an exercise for
the first time of the Court's expanded power to promulgate rules to
protect our people's constitutional rights, which made its maiden
appearance in the 1987 Constitution in response to the Filipino
experience of the martial law regime.
"Extralegal killings" are "killings committed without due process of law,
i.e., without legal safeguards or judicial proceedings." On the other
hand, "enforced disappearances" are "attended by the following
characteristics: an arrest, detention or abduction of a person by a
government official or organized groups or private individuals acting
with the direct or indirect acquiescence of the government; the
refusal of the State to disclose the fate or whereabouts of the person
concerned or a refusal to acknowledge the deprivation of liberty
which places such persons outside the protection of law."
While constitutional rights can be protected under the Grave Abuse
Clause through remedies of injunction or prohibition under Rule 65 of
the Rules of Court and a petition for habeas corpus under Rule 102,
these remedies may not be adequate to address the pestering
problem of extralegal killings and enforced disappearances.
amparo through summary proceedings and the availability of
appropriate interim and permanent reliefs under the Amparo Rule, this
hybrid writ of the common law and civil law traditions - borne out of the
Latin American and Philippine experience of human rights abuses -
offers a better remedy to extralegal killings and enforced
disappearances and threats thereof.
The remedy provides rapid judicial relief as it partakes of a summary
proceeding that requires only substantial evidence to make the
appropriate reliefs available to the petitioner; it is not an action to
determine criminal guilt requiring proof beyond reasonable doubt, or
liability for damages requiring preponderance of evidence, or
administrative responsibility requiring substantial evidence that will
require full and exhaustive proceedings.
The writ of amparo serves both preventive and curative roles in addressing the
problem of extralegal killings and enforced disappearances. It is preventive in
that it breaks the expectation of impunity in the commission of these
offenses; it is curative in that it facilitates the subsequent punishment of
perpetrators as it will inevitably yield leads to subsequent investigation and
action. In the long run, the goal of both the preventive and curative roles
is to deter the further commission of extralegal killings and enforced
disappearances.
The possibility of respondents being executed stared them in the eye while they
were in detention. With their escape, this continuing threat to their life is
apparent, moreso now that they have surfaced and implicated specific
officers in the military not only in their own abduction and torture, but also in
those of other persons known to have disappeared such as Sherlyn
Cadapan, Karen Empeño, and Manuel Merino, among others.
The circumstances of respondents' abduction, detention, torture and
escape reasonably support a conclusion that there is an apparent
threat that they will again be abducted, tortured, and this time, even
executed. These constitute threats to their liberty, security, and life,
actionable through a petition for a writ of amparo.
To this day, however, almost a year after the policy directive was issued by
petitioner Secretary of National Defense on October 31, 2007, respondents
have not been furnished the results of the investigation which they now seek
through the instant petition for a writ of amparo.
Under these circumstances, there is substantial evidence to warrant the
conclusion that there is a violation of respondents' right to security as
a guarantee of protection by the government. In sum, we conclude that
respondents' right to security as "freedom from threat" is violated by
the apparent threat to their life, liberty and security of person. Their
right to security as a guarantee of protection by the government is
likewise violated by the ineffective investigation and protection on the
part of the military.
ROXAS v. GMA (HENRY) RC but out of fear that she was being monitored, she threw the
September 7, 2010 | Perez, J. | Substantive Due Process phone.

PETITIONER: Melissa C. Roxas


RESPONDENTS: Gloria Macapagal-Arroyo, et al.

SUMMARY: Petitioner Roxas was abducted, during her immersion trip


under one of the programs organized by BAYAN-USA. During the 5 days
that she was held, she claims to have been tortured and interrogated,
and being alleged as a member of the NPA. She was eventualy released,
and soon after, saught remedies with the SC by filing a petition for the
writs of amparo and habeas data, to which the SC referred to the CA. The
appellate court granted the writ of habeas data, but did not rule in favor of
the petitioner with regard to the respondents being responsible of her
abduction due to insufficient evidence. She filed for a petition to the SC,
who against the issuance of her writs, plainly because as far as the rule
of amparo is concerned, no extraordinary diligence was performed in the
investigation to substantiate the allegations made by the petitioner
against the respondents.

DOCTRINE: An indispensable requirement before the privilege of the writ


may be extended is the showing, at least by substantial evidence, of an
actual or threatened violation of the right to privacy in life, liberty, or security
of the victim.

FACTS:
Petitioner Roxas is an American citizen of Filipino descent. She enrolled
in an exposure program to the Philippines with the group Bagong
Alyansang Makabayan-United States of America (BAYAN-USA).
During her immersion, they toured various towns of central Luzon and
she volunteered to join members of BAYAN-Tarlac in conducting an
initial health survey in La Paz for a future medical mission.
After doing the survey work they decided to rest in the house of one Mr.
Paolo, where they were abducted by 15 heavily armed men, for the
purpose of allegedly being a member of the Communist Party of the
Philippines-New People’s Army (CPP-NPA). She was taken to the
military camp of Fort Magsaysay in Nueva Ecija.
She was interrogated for 5 days straight, with the thrust that she
abandon her communist beliefs. She was blindfolded the entire time,
except when she was bathed by a certain Rose. Her interrogators
were Dex, James, and RC, the latter told her that those who tortured
her came from the “Special Operations Group”.
She was finally released and returned to her uncle’s house in QC by her
abductors, who provided her with a phone and sim card, and an
email with a password, among others (with the condition that she
never reports her abduction lest something bad will happen to her
and her family). Post-release period, she was receiving calls from
She filed in the SC a petition for the writs of Amparo and Habeas data, simply grant or dismiss the petition for the writ, and hence, it must be
and impleaded public officials with high ranks in the military and remanded to
police hierarchy as respondents, together with Rose, Dex, and RC.
SC issued the desired writs and referred the case to the CA for hearing,
reception of evidence, and appropriate action. Respondents were
directed to file a written return.
Respondents aver that the abuction is only staged, thus denying such
allegations, and pointing out that contrary to petitioner’s claims of
being tortured, her medical certificate did not show that she acquired
the injuries.
CA granted the privilege of the writ of habeas data, mandating the public
respondents to refrain from distributing to the public any records, in
whatever form, relative to the petitioner’s alleged ties with the CPP-
NPA or pertinently related to her abduction and torture. CA was
convinced that petitioner was telling the truth.
However, CA was not convinced that the military or the police had
anything to do with the abduction. It likewise dismissed the petition
as against GMA, in lieu of her immunity from suit. Roxas’ petition for
the return of her personal belongings was likewise denied. Hence,
this appeal.

ISSUE/s:
WoN the decision the CA to grant the petition for the privilege of the writ
of habeas data and amparo is valid - NO

RULING: SC remanded the case to the CA for monitoring of compliance of


the SC’s directives.

RATIO:
The SC ruled that the use by the petitioner of the doctrine of command
responsibility as the justification in impleading the respondents in
their maparo petition is legally inaccurate, if not incorrect.
Doctrine of command responsibility – rule of substantive
law that establishes liability and, by this account, cannot
be a proper legal basis to implead a party-respondnet in an
amparo petition. This is more aptly invoked in a full-blown
criminal or administrative case rather than in a summary
amparo proceeding.
Having this said, commanders or the respondents may be impleaded not
on the basis of command responsibility, but on their responsibility, or
at least, accountibilty.
Responsibility – extent of actors in participation by substantial
evidence by act or ommission
Accountability – measure of remedies addressed to those who
exhibited involvement
Mentioning that throughout the investigation after the allegation of petitioner
Roxas of the people behind her abduction there was no extraordinary
diligence as required by the Amparo rule, the SC sees it not right to
the CA in order for a more thorough investigation.
Likewise, it must be noted that the grant of the privilege of habeas data is
hence reversed, since there has been no legitimate evidence to back
up the allegations made by petitioner Roxas as far as information
about her (videos and pictures during her training with NPA) came
from the police or the military.
The SC directed the CHR as the appropriate body to pursue the
investigation when the case is directed to the CA, as the CHR enjoys
the trust and confidence of the petitioner.
CARAM v. SEGUI (DANNAH) On November 26, Marcelino suffered a heart attack and died.
August 5, 2014 | Perlas-Bernabe, J. | Substantive Due Process: Writ of Thereafter, Christina disclosed to his family that she and
Amparo Marcelino had a son and that she had to give him up for
adoption due to financial distress and initial embarrassment.
PETITIONER: Infant Julian Caram, represented by his mother, Ma. Christina
Caram
RESPONDENTS: Atty. Marijoy Segui et al.

SUMMARY: Christina got impregnated by Marcelino without the benefit of


marriage. She led Marcelino to believe that she had an abortion but kept Baby
Julian and had him adopted through Sun and Moon Home for Children. A few
weeks after giving birth, Christina voluntarily surrendered Julian to the DSWD by
a way of a Deed of Voluntary Commitment. In the same year, Marcelino had a
heart attack and died. Christina disclosed to his family that they had a baby
because of financial difficulty and initial embarrassment. They promised to help
her recover Julian.

She filed a petition for the Writ of Amparo with the RTC which the RTC did
not grant. She then elevated the case to the Supreme Court. The Court
ruled that it was the improper remedy, as the third and fourth elements
required are missing, as she knew his whereabouts and even got to see him
in one hearing conducted by the RTC. What she was clearly asserting was
her parental authority and custody over Julian; she was not looking for a lost
child.

DOCTRINE: To reiterate, the privilege of the writ of amparo is a remedy


available to victims of extrajudicial killings and enforced disappearances or
threats of a similar nature, regardless of whether the perpetrator of the
unlawful act or omission is a public official or employee or a private
individual. It is envisioned basically to protect and guarantee the right to life,
liberty and security of persons, free from fears and threats that vitiate the
quality of life.

The four elements (see Ratio 3b) must also be present for a valid claim of the
writ.

FACTS:
Petitioner Christina had an amorous relationship with Marcelino Constantino
III, and eventually became pregnant with his child without marriage.
After getting pregnant, Christina made Marcelino believe she had
abortion. During this time, she intended to have the child adopted
through Sun and Moon Home for Children in Paranaque.
On July 26, 2009, Christina gve birth to Julian at Amang Rodriguez Memorial
Medical Center, Marikina. On August 13, Christina voluntarily
surrendered Baby Julian by a way of a Deed of Voluntary Commitment
to the DSWD.
Marcelino’s family vowed to help her recover and raise the baby.
On November 27, the DSWD issued a certificate declaring baby Julian as
“Legally Available for Adoption.” A local matchmaking conference was held
on January 2010 and on February, Julian was “matched” with the Medina
Spouses of the Kaisahang Bahay Foundation. Supervised trial custody then
commenced.
On May 2010, Christina wrote a letter to the DSWD asking for the suspension of
Baby Julian’s adoption proceedings. She also said she wanted her family
back together.
The DSWD, through respondent Atty. Marijoy, sent a Memorandum to DSWD
Assistant Secretary Cabrera informing her that the certificate declaring Baby
Julian legally available for adoption had attained finality on November 13,
2009
This was three months after Christina signed the Deed of Voluntary
Commitment which terminated her parental authority and made Baby Julian
a ward of the State.
On July 2010, Marcelino’s brother Noel sent a letter to Atty. Escutin informing her
that DNA testing was scheduled on July 16, 2010 at the DNA Analysis Lab at
the University of the Philippines.
Assistant Secretary Cabrera informed Noel that the DSWD was no longer in the
position to stop the adoption process. She further stated that should
Christina wish to reacquire her parental authority, she may bring the matter to
regular courts.
This was due to the fact that the reglementary period for her to regain her
parental rights had already lapsed under Section 7 of RA 9523.
Christina then filed a petition before the RTC of Quezon City for the issuance of a
writ of amparo, seeking to obtain custody of Baby Julian from Atty. Segui et
al.
In her petition, Christina accused respondents of “blackmailing” her into
surrendering custody of Julian to the DSWD utilizing what she claims as an
invalid certificate of availability of adoption which they allegedly used as
basis to misrepresent that all legal requisites for adoption of Julian had been
complied with.
She further argued that by making these representation, respondents acted
beyond the scope of their legal authority, thereby causing the enforced
disappearance of Julian and depriving her of custodial rights and parental
authority.
The RTC Branch of QC through Judge Quimpo-Sale issued a Write of Amparo,
commanding the four respondents (Atty. Segui et al) to produce the body of
Baby Julian at a hearing scheduled on August 4.
The respondents complied with the writ and filed their Return on August 2. On
August 4, they appeared before the RTC but failed to bring Julian, stating
that threats of kidnapping were made to him and his caregivers.
Judge Sale in her order acknowledged that the child was brought before the
court and Christina was allowed to see him and take photographs of him.
On August 17, the RTC dismissed the petition for issuance of the writ stating
that Christina availed of the wrong remedy to regain custody of her child. 3 The intention for such refusal is to remove
The RTC also stated that Christina should have filed a civil case for subject person
custody. If there is extreme urgency to secure custody of a minor
who has been illegally detained by another, a petition for the
issuance of a writ of habeas corpus may be availed of, either as a
principal or ancillary remedy, pursuant to the Rule on Custody of
Minors and Writ of Habeas Corpus in Relation to Custody of Minors.
The RTC also denied Christina’s motion for reconsideration, hence this
present petition via petition for review on certiorari under Rule 45.

ISSUE/s:
WoN a petition for a Writ of Amparo is the proper recourse for obtaining
parental authority and custody of a minor child – NO

RULING:. Petition is denied. Ruling of the lower court is affirmed without


prejudice to petitioner’s right to avail of proper legal remedies afforded
to her by law and related rules.

RATIO:
Section 1 of the Rule on the Writ of Amparo provides as follows:
The petition for a writ of amparo is a remedy available to any
person whose right to life, liberty and security is violated
or threatened with violation by an unlawful act or omission of
a public official or employee, or of a private individual or
entity.
The writ shall cover extralegal killings and enforced
disappearances or threats thereof.
In Secretary of Nat’l Defense et al v. Manalo et al, the Court held that:
The Amparo Rule was intended to address the intractable
problem of “extralegal killings” and “enforced
disappearances,” its coverage, in its present form, is
confined to these two instances or to threats thereof.
In Lozada Jr. v. Macapagal-Arroyo:
The writ is confined only to cases of EJK and enforced
disappearances, or to threats thereof.
Elements:
0 There be an arrest, detention, abduction or any form
of deprivation of liberty
1 It be carried out by or with the authorization, support
or acquiescence of the State or a political
organization
2 It be followed by the State or political organization’s
refusal to acknowledge or give information on the
fate or whereabouts of the person subject of the
amparo petition
from the protection of the law for a prolonged period of time
In the case at hand, Christina alleged that the respondent DSWD officers caused
her “enforced separation” from Julian and that their action amounted to an
“enforced disappearance” within the context of the Amparo rule.
However, the DSWD officers never concealed Julian’s whereabouts. In fact,
Christina was given a copy of a Memorandum stating that Julian was in the
custody of the Medina spouses when she filed her RTC petition.
She even admitted that she got to see him in a hearing. There is therefore no
“enforced disappearance” as used in the context of the Amparo rule as the
third and fourth elements are missing.
Her assertions clearly indicate that she is not searching for a lost child but
asserting her parental authority over the child and contesting custody over
him.
To reiterate, the privilege of the writ of amparo is a remedy available to victims of
extrajudicial killings and enforced disappearances or threats of a similar
nature, regardless of whether the perpetrator of the unlawful act or omission
is a public official or employee or a private individual.
It is envisioned basically to protect and guarantee the right to life, liberty and security
of persons, free from fears and threats that vitiate the quality of life.
MISON v. GALLEGOS (IYA) The SC issued a Resolution issuing a TRO enjoin gin the enforcement of
June 23, 2015| Perez, J. | Substantive Due Process: Writ of Amparo the above said orders and directing the BI to retain custody of Ku and
requiring Ku to comment on the petition.
PETITIONER: fChairperson Siegred B. Mison in his capacity as
chairperson of the Bureau of Immigration nd Deportation
RESPONDENTS: Hon. Paulino Q. Gallegos in his capacity as presiding
jude fo the RTC-Manila and Ja Hoon Ku

SUMMARY: Inerpol of Korea sent a notice to the Interpol of Manila


requesting assistance for the arrest of Ja Hoon Ku. The Embassy of
Korea wrote to Mison for the arrest and detainment of Ku for being an
undesirable alien. Ku was later on arrested and detained in the BI.

Ku filed a petition for the issuance of a writ of amparo. Judge Gallegos was
assigned the case. Several resolutions were made in contravention with SC
orders until it resulted to the granting of the privilege of the writ by the RTC
judge.

The SC, in finality, denied the privilege of the writ for the present case
does not fall under “extralegal killings and enforced disappearances”.

DOCTRINE:As the Amparo Rule was intended to address the intractable


problem of “extralegal killings” and “enforced disappearances,” its
coverage, in its present form, is confined to these two instances or to
threats thereof.

FACTS:
On 23 December 2013, the International Criminal Police Organization
(Interpol) of Seoul, Republic of Korea sent a notice to Interpol Manila
requesting assistance in the location and deportation of Ja Hoon Ku for
arbitrarily spending money allowed as reserve fund of Phildip Korea Co.,
Ltd.
The Embassy of Korea wrote a letter to Hon. Siegfred Mison for the
immediate arrest and deportation of Ku to Korea for being an
undesirable alien.
Bureau of Immigration officers arrested Ku and detained at the BI
detention center.
On 17 January 2014, Ku filed a Petition for the Issuance of a Writ of Amapro.
Finding the petition to be sufficient in forma nd subtance, Judge Gallegos
issued a Writ of Amparo. Ku filed a Motion for the Issuanc elf a
Temporary Protection Order.
On 28 January 2014, Judge Gallegos issued the first assailed order
granting the TPO. Mison challenges this order before the SC via a
petition for certiorari.
Basis of this resolution is the possibility of misuse by Ku of the government officials who will likely take advantage of their positions
writ of amaro given that he was validly arrested and placed and use the power of the government at their command.
under the jurisdiction and custody of the BI.
His case cannot be categorizes as one of extralegal killing or
enforced disappearance.
By virtue of the SC resolution, Mison verbally moved for the dismissal of the
amparo petition in the trial court. Judge Gallegos on February 18
however issued another order denying the motion to dismiss for lack of
merit.
Mison appealed this to the SC via Petition for Certiorari and Prohibition.
Ku then filed an appeal memorandum on his deportation order
addressed to the office of the president.
Judge Gallegos issued a Resolution granting the privilege of the writ of
amaro on March 14.
Meanwhile, the Court issued a TRO enjoining the RTC from enforcing
the Order dated February 18 and from further proceeding with the
case.
The Office of the President granted ku provisional liberty only until 31
August or until his appeal was resolved whichever came first.
Due to the complexities involved, Mison filed the Petition for Review on
Certiorari assailing the Resolution dated March 14.

ISSUE/s:
WON the RTC Judge issued the assailed orders validly - NO

RULING: Petition is granted. SC DENIES the privilege of the writ of amparo.


Administrative charges against Gallegos shall be filed.

RATIO:
The Court promulgated the Amparo Rule “in light of the prevalence of
extralegal killings and enforced disappearances.” It was an exercise
for the first time of the Court’s expanded power to promulgate rules
to protect our people’s constitutional rights.
As the Amparo Rule was intended to address the intractable problem of
“extralegal killings” and “enforced disappearances,” its coverage, in
its present form, is confined to these two instances or to threats
thereof.
Ku’s circumstances does not come under the statutory definition of an
enforced or involuntary disappearance.
Ku was arrested by agents of the BI, but there was no refusal on the part
of the BI to acknowledge such arrest not was there any refusal give
information on the whereabouts of Ku.
Ku claims that he fears for his life and feels the serious danger of being
detained for a long period of time without any cause, and that he
fears that the BI will fabricate criminal cases against him to hold him
under detention.
According to Ku, what he seeks to obtain in filing an amparo petition is
the protection it will give to his person against the actions of some
However, Ku was not able to present evidence that he was exposed to
“life-threatening situations” while confined at the BI Detention Center.
On the contrary, the records show that he si afforded visitorial rights
and that he has access to his counsel.
As such, it can readily be discerned that the RTC’s grant of the privilege
of the writ of amparo was improper in this case as Ku and his
whereabouts were never concealed, and as the alleged threats to his
life, liberty and security were unfounded and unsubstantiated. It is to
be emphasized that the fundamental function of the writ of amparo is
to cause the disclosure of details concerning the extrajudicial killing
or the enforced disappearance of an aggrieved party.
The Court observes that Judge Gallegos knowingly disregarded the
Court’s directives as regards this case.
It is well to note that a resolution of the Supreme Court should not be
construed as a mere request, and should be complied with promptly
and completely. Such failure to comply accordingly betrays not only a
recalcitrant streak in character but also disrespect for the Courts
lawful order and directive.
ZARATE v. AQUINO (ELIEL) About 1,300 Manobos allegedly evacuated to Davao City to escape the
November 10, 2015 | En Banc | Substantive Due Process effects of said military operations. These evacuees returned to their
communities in May 2014.
PETITIONER: Zarate (Bayan Muna), De Jesus (Gabriela), Mariano
(Anakpawis), Casino (Bayan Muna), et al. RESPONDENTS: H.E.
Benigno Simeon C. Aquin III

SUMMARY: During the rise of counterinsurgency in Davao, many


Manobos evacuated the area. After such issue declined, some of them
returned, and sought refuge in the United Church of Christ in the
Philippines (UCCP) Haran. However, certain Manobos claimed that
instead of being a refugee center, the UCCP Haran deprived them of
their liberty, and forced them to listen to rallies, and were only allowed to
go home after they found a body, hanging lifeless on a tree. The CIDG
forwarded a complaint to the Office of the Prosecutor, and to determine
the defendants, the complainants were shown “lists”. The “lists” indicated
the petitioners, who then applied for a writ of amparo and habeas data.
The SC En Banc dismissed the petition and did not grant the writs of
amparo and habeas data, because the petitioners failed to show
substantial evidence that there was (1) actual threats that translates to
extralegal killings and forced disappearances [amparo]; that (2) mere
membership does not grant them issuance of the writs; (3) it failed to
show that their right to privacy was violated, since what was shown in the
“lists” are readily and accessible data to the public.

DOCTRINE: The writ of amparo is a “remedy available to any person


whose right to life, liberty and security is violated or threatened with
violation by an unlawful act or omission of a public official or employee, or
of a private individual or entity.”

The writ of habeas data is a “remedy available to any person whose right
to privacy in life, liberty or security is violated or threatened by an
unlawful act or omission of a public official or employee, or of a private
individual or entity engaged in the gathering, collecting, or storing of data
or information regarding the person, family, home and correspondence of
the aggrieved party.”

FACTS:
Petitioners aver that they are members of various progressive partylists
and/or national and religious organizations, and that these
organizations have been wrongfully tagged by the military and the
police as “communist front organizations.”
The petitioners herein filed for a Petition for a Writ of Amparo and a Writ
of Habeas Data.
Government commenced intensified military offensives in Davao del
Norte under the rubric of counterinsurgency.
Beginning Jan 2015, however, some of the Manobos started going back Secs. 17 & 17 of the Rule on the Writ of Amparo requires substantial
to Davao City. Approximately 700 Manobos were at the United evidence. “Substantial evidence is that amount of relevant
Church of Christ in the Philippines (UCCP) Haran. evidence which a reasonable mind might accept as adequate to
Petitioners claimed that these Manobos sought refuge at UCCP Haran support a
due to the persisting militarization of their communities and their
forcible recruitment to the paramilitary group, Alamara.
Certain Manobos claimed, on the other hand, that they were deceived
into going to Davao City; that upon reaching UCCP Haran, they were
deprived of their freedom of locomotion and were held against their
will; that they were forced to listen to rallies; and they were only
allowed to go home after a body was found.
The Criminal and Detection Group (CIDG) forwarded to the Office of the
City Prosecuter a complaint for violation of Art. 267 of the RPC
(Kidnapping and Serious Illegal Detention); RA 9208 (Anti
Trafficking).
To determine who would be charged in the complaint, the complainants
were shown “lists” from which they purportedly identified the
defendants.
Petitioners now aver that the inclusion of their names and photographs in the
“lists” indicates that they are and have been the subject of State
surveillance.
Coupled with instances of harassment in the past, attempts to
incriminate them in fabricated criminal charges, and insinuations of
their links with the NPA, petitioners argue that their inclusion in the
“lists” are threats to their life, liberty and security, warranting the
protection of the writ of amparo.
Petitioners claim that as there is absolutely no basis for the inclusion of
their names and photographs in the “lists,” then respondents should
be compelled via the writ of habeas data to disclose and to provide
petitioners with copies of all information and evidence pertaining to
them which respondents have in their file or records, and for such
information to be destroyed.

ISSUE/s:
WoN petitioners may avail writs of amparo and habeas data – NO

RULING: Petition Dismissed.

RATIO:
The writ of amparo is a “remedy available to any person whose
right to life, liberty and security is violated or threatened with
violation by an unlawful act or omission of a public official or
employee, or of a private individual or entity.”
Rule on the Writ of Amparo requires every petition to state “the
right to life, liberty and security of the aggrieved party violated
or threatened with violation by an unlawful act or omission of
the respondent, and how such threat or violation is committed
with the attendant circumstances detailed in supporting
affidavits.”
conclusion. It is more than a mere imputation of wrongdoing or violation
that would warrant a finding of liability against the person charged.
“only actual threats, as may be established from all the facts and
circumstaces of the case, can qualify as a violation that may be
addressed under the Rule on the Writ of Amparo.”
Having these guidelines in mind, petitioners failed to substantially
prove that their life, liberty and security are threatened with
violation.
Mere membership in these organizations or sectors cannot equate
to an actual threat that would warrant the issuance of a writ of
amparo.
Moreover, as the writ of amparo is sought individually and granted
individually, then we should assess the situation of the
petitioners individually. A perusal of their individual
circumstances negates the conclusion that they are each
entitling to a writ of amparo.
The filing of cases, however, cannot be characterized as an unlawful act
or omission in the context of the Amparo Rule. This rule was
intended to address the intractable problem of “extralegal killings”
and “enforced disappearances.”
Clearly then, having failed to show how the legal cases against them
translates to threats of extralegal killings and enforced
disappearances, petioners are not entitled to the writ of amparo.
The writ of habeas data is a “remedy available to any person whose
right to privacy in life, liberty or security is violated or
threatened by an unlawful act or omission of a public official or
employee, or of a private individual or entity engaged in the
gathering, collecting, or storing of data or information regarding
the person, family, home and correspondence of the aggrieved
party.”
The extraordinary writ of habeas data “provides a judicial remedy to
protect a person’s right to control information regarding
oneself, particularly in instance where such information is being
collected through unlawful means in order to achieve unlawful
ends.”
Rule on the Writ of Habeas Data provides that the petition should
aver “the manner the right to privacy is violated or threatened
and how it affects the right to life, liberty or security of the
aggrieved party.” The Habeas Data Rule likewise requires
substantial evidence.
In the present petition, petitioners fail to show how their right to privacy is
violated given that the information contained in the “lists” are only
their names, their positions in their respective organizations, and
their photographs. All these data are of public knowledge and are
readily accessible even to civilians, especially since petitioners are
known personalities who are often feature in news reports.
Although the petition for a writ of habeas data may be filed by family
member, or even relatives, on behalf of the aggrieved party, the
Habeas Data Rule presupposes that the aggrieved party is still alive
as Sec 6 of the said Rule requires the petitioner to show how the
violation of the aggrieved party’s right to privacy or threats of such
violation affect the aggrieved party’s right to life, liberty or security.
PEOPLE v. CAYAT (JP)
May 5, 1939 | , J. | Equal Protection Clause RATIO:
Counsel for the appellant holds out his brief as the "brief for the non-
PETITIONER: People of the Philippines Christian tribes.
RESPONDENTS: Cayat Throughout the Spanish regime, it had been regarded by the Spanish
Government as a sacred "duty to conscience and humanity" to
SUMMARY: Cayat was found guilty by the lower court of violating Sec 2 civilize these less fortunate people living "in the obscurity of
and 3 of Act No. 1639 for having in his possession A-1-1 gin. Act No. ignorance" and to accord them the "moral and material advantages"
1639 punishes the possession and use of ardents and spirits that are
of community life and the "protection and vigilance afforded them by
other than the so-called native wines and liquors. Accused appealed and
the SC affirmed the lower court decision explaining that the Act was a the same laws."
valid police measure and it meets all the requirements of a reasonable 102. To complement this policy of attraction and assimilation, the
classification. The Act was meant to meet the peculiar conditions existing Legislature has passed Act No. 1639 undoubtedly to secure for them
in non-Christian tribes who are classified as such not based on religion the blessings of peace and harmony; to facilitate, and not to mar,
but by geographic area and more directly to natives with low grade of their rapid and steady march to civilization and culture. It is,
civilization. therefore, in this light that the Act must be understood and applied.
DOCTRINE: The guarantee of the equal protection of the laws is not 103. Act No. 1639 satisfies the reasonable classification
requirements.
violated by reasonable classification which (1) must rest on substantial
104. The classification rests on real or substantial, not merely
distinctions; (2) must be germane to the purposes of the law; (3) must not
imaginary or whimsical, distinctions. It is not based upon
be limited to existing conditions only; and (4) must apply equally to all
"accident of birth or parentage," as counsel for the appellant
members of the same class. asserts, but upon the degree of civilization and culture.
105. "The term 'non-Christian tribes' refers, not to religious belief,
but, in a way, to the geographical area, and, more directly, to
natives of the Philippine Islands of a low grade of civilization,
FACTS:
usually living in tribal relationship apart from settled
Section 2 of Act No. 1639: it shall be unlawful for any native of the
Philippine Islands who is a member of a non-Christian tribe to buy, communities."
receive, have in his possession, or drink ardent spirits, ale, beer, This distinction is unquestionably reasonable, for the Act was
wine, or intoxicating liquors, other than the so-called native wines intended to meet the peculiar conditions existing in the non-
and liquors which the members of such tribes have been Christian tribes.
accustomed themselves to make prior to the passage of this Act. That it is germane to the purposes of law cannot be doubted. The
Section 3 talks about the fines and imprisonment. prohibition to possess non-native liquor is unquestionable
Accused Cayat, a native of Baguio, a member of a non-Christian tribe,
desined to insure peace and order in and among the non-
who was sentenced by the justice of the peace court of Baguio to
violations of Secs 2 and 3 of the Act 1639 for illegally having in his Christian tribes.
possession one bottle of A-1-1 gin, an intoxicating liquor, not usually It has been the sad experience of the past, as the observations of the
accustomed to his tribe or part of native wines. lower court disclose, that the free use of highly intoxicating liquors
The trial court found him guilty of the crime. Hence this appeal, by the non-Christian tribes have often resulted in lawlessness and
questioning the constitutionality of the Act. crimes, thereby
ISSUE/s:
hampering the efforts of the government to raise their standard
WoN Act No. 1639 is discriminatory and denies the equal protection
of the laws – NO of life and civilization.
WoN Act No. 1639 is violative of due process – NO 109. The law is not limited in its application to conditions existing at
WoN it is an improper exercise of police power - NO the time of its enactment. It is intended to apply for all times as
long as those conditions exist. The Act was not predicated, as
RULING: Judgement of the lower court is affirmed.
counsel for appellant asserts, upon the assumption that the
non-Christians are "impermeable to any civilizing influence.
Finally, that the Act applies equally to all members of the class is
evident from a perusal thereof. That it may be unfair in its operation
against a certain number of nonChristians by reason of their
degree of culture, is
not an argument against the equality of its application.
Neither is the Act an improper exercise of the police power of the state. Act
No. 1639, as above stated, is designed to promote peace and order in
the non-Christian tribes so as to remove all obstacles to their moral and
intellectual-growth and, eventually, to hasten their equalization and
unification with the rest of their Christian brothers. Its ultimate purpose
can be no other than to unify the Filipino people with a view to a greater
Philippines.
VILLEGAS v. HIU CHIONG TSAI PAO HO (HENRY) property, and therefore violates due process and equal protection clause
of the Constitution.
November 10, 1978 | Fernandez, J. | Equal Protection of Law
CFI granted the petition, which led for Mayor Villegas to file the present
petition before the court.
PETITIONER: Mayor Antonio J. Villegas
RESPONDENTS: Hiu Chiong Tsai Pao Ho and Judge Francisco Arca

SUMMARY: Ordinance No. 6537, an ordinance mandating aliens to


secure first a permit from the Mayor before engaging in trade, business,
or employment (with a fee of 50php) was signed by petitioner Villegas.
Respondent Ho, took this into the CFI, assailing it due to its violation of
equal protection, because of section 1, which mandates that aliens get
their permits first and pay a fee before they are allowed to actually work.
CFI ruled in respondent’s favor, which lead for petitioner Villegas to
appeal to the SC. The SC affirmed the lower court’s decision, ruling that
indeed it is violative of equal protecetion.

DOCTRINE: In equal protection, it is imperative that the classification


should be based on real and substantial differences having a reasonable
relation to the subject of the particular legislation.

FACTS:
Ordinance No. 6537 (entitled, “An ordinance making it unlawful for any
person not a citizen of the Philippines to be employed in any place of
employment or be engaged in any kind of trade, business or
occupation within the City of Manila without firs securing an
employment permit from the Mayor of Manila; and for other
purposes”) was passed by the Municipal Board of Manila, signed by
petitioner Mayor Villegas.
Section 1 – prohibits aliens from being employed or to engage/
participate in any position of occupation or business
enumerated therein, whether permament, temporary or
casual without first securing a permit from the Mayor, with a
fee of Php 50.00 except those employed in diplomatic or
consular missions of foreign countries and those working in
their respective households and members of religious orders
or congregations, sect or denomination, who are not paid
monetarily or in kind
Penalty – imprisonment (3 to 6 months / fine 100-200 php or
both)
Hiu Chiong Tsai Pao Ho filed for the issuance of the writ of preliminary
injuction and TRO for the said ordinance, and that it be declared null
and void.
In his petition assailing the ordinance, one of his claims was that, the
ordinance was arbitrary, oppressive, and unreasonable, being applied
only to aliens, who are thus, deprived of their rights to life, liberty, and
ISSUE/s:
WoN Ordinance No. 6537 is violative of the equal protection clause of
the Constitution - YES

RULING: SC affirmed the decision of the CFI.

RATIO:
The Php 50.00 fee is unreasonable not only because it’s excessive but
also because it fails to consider valid substantial differences in
situation among individual aliens who are required to pay it.
In equal protection, it is imperative that the classification should be
based on real and substantial differences having a reasonable
relation to the subject of the particular legislation.
Where an ordinance of a municipality fails to state any policy or set up
any standard or guide to limit the mayor’s action, expresses no
purpose to be attained by requiring a permit, enumerates
noconditions for its grant or refusal, and entirely lacks standard, thus
conferring upon the Mayor arbitrary and unrestricted power to grant
or deny the issuance of building permits, such ordinance is invalid,
being an undefined and unlimited delegation of power to allow or
prevent an activity per se lawful.
Requiring a person to secure a permit from a Mayor who may withhold or
refuse it at will is tantamount to denying him the basic right of people
in the Philippines to engage in a means of livelihood.
While it is true that the PH as a state is not obliged to admit aliens, once
an alien is admitted, he cannot be deprived of life without due
process of law.
Teehankee, concurring: The national policy on admitting aliens
recognize the right of aliens to obtain gainful employment in the
country. Such national policies cannot be stepped over by local
government policies.
DUMLAO v. COMELEC (DANNAH) WoN Section 4 of BP 52 violates the equal protection clause – NO
January 22, 1980 | Melencio-Herrera, J. | Equal Protection Clause
RULING:. WHEREFORE, 1) the first paragraph of section 4 of Batas
PETITIONER: Patricio Dumlao, Romeo Igot and Alfredo Salapantan Jr. Pambansa Bilang 52 is hereby declared valid. 2) That portion of the second
RESPONDENTS: Commission on Elections paragraph of section 4 of Batas Pambansa Bilang 52 providing that „x x x the
filing of charges for the commission of such crimes before a civil court or
SUMMARY: Dumlao, retired Governor of Nueva Vizcaya questions the first military tribunal after preliminary investigation shall be prima facie evidence of
paragraph of Sec 4 BP 52 which prohibits the running of retired elective such fact, is hereby declared null and void, for being violative of the
official from running the same office. Igot and Alfredo question another
constitutional presumption of innocence guaranteed to an accused.
paragraph which states that those charged (even not yet convicted) of crimes
such as sedition, rebellion etc. are also disqualified from running office.
RATIO:
The SC ruled that for the first paragraph, there is a rational basis for Procedural aspect: Dumlao, Igot and Alfred have different complaints,
classification hence making it valid. For the latter one, it is invalid because it and Dumlao joined the suit in his individual capacity while the two
doesn’t differentiate between those convicted and just charged. others entered in the nature of a taxpayer’s suit.
Dumlao has not been adversely affected by the application of said
DOCTRINE: The constitutional guarantee of equal protection of the laws is provision. No petition seeking his diqualification has been filed before
subject to rational classification. If groupings are based on reasonable and the COMELEC. His is a question posed in the abstract. Moreover, his
real differentiations, one class can be treated and regulated differently from case is clearly within the primary jurisdiction of respondent
another class. COMELEC as provided for in Art. XII-C Section 2.
In the case of Igot and Andres, it was only during the hearing, not their
Petition that Igot was said to be a candidate for Councilor. Moreover,
FACTS: neither one has been convicted nor charged with acts of disloyalty to
Patricio Dumlao was a former Governor of Nueva Vizcaya, who has filed the State, nor disqualified from being candidates for local elective
his COC for said position of Governor in the forthcoming January positions.
1980 elections. Theirs is a generalized grievance. They have no personal nor substantial
Romeo Igot is a taxpayer and a qualified member of the Bar. Alfredo is
also a taxpayer, a qualified voter and resident of San Miguel, Iloilo. interest at stake; they can claim no locus standi.
Dumlao questions the constitutionality of Sec 4 of BP 52 as Also, the provisions in question do not directly involve the disbursement
discriminatory and contrary to the equal protection and due process of public funds.
guarantees of the Constitution. However, the Court will rule on the challenged provision because of the
Sec. 4: …any retired elective provincial, city or municipal official paramount public interest involved and proximity of the elections
who has received payment of the retirement benefits to which is only a few days away.
which he is entitled under the law and who shall have been The constitutional guarantee of equal protection of the laws is subject to
65 years of age at the commencement of the term of office to
which he seeks to be elected, shall not be qualified to run for rational classification. If groupings are based on reasonable and
the same elective office which he has retired. real differentiations, one class can be treated and regulated
He alleges that this is directly insidiously against him, and the differently from another class.
classification is based on “purely arbitrary grounds and therefore, For the purpose of public service, employees 65 years of age have been
class legislation” validly classified differently from younger ones. Employees attaining
On the other hand, Igot and Alfredo question another part of said section that age are subject to compulsary retirement.
which states that the filing of charges for the commission of crimes
Retirement from government service may or may not be a reasonable
such as subversion, insurrection, rebellion or other similar crimes will
be prima facie evidence of being convicted of said crimes. These disqualification for elective local officias. It may neither be reasonable
people, whether convicted or not, shall be disqualified to run. to disqualify retirees, aged 65, for a 65 y/o retiree could be a good
local official just like one, aged 65, whos is not a retiree.
ISSUE/s:
BUT in the case of a 65 y/o elective local official, who has retired from a A person disqualified to run on the ground that charges have been
provincial, city or municipal office, there is reason to disqualify him filed against him is virtually placed in the same category as a
from running for the same office from which he had retired, as person already convicted of a crime with the penalty of arresto.
provided for in the challenged provision.
The tiredness of the retiree for government work is present, and what is
emphatically significant is that the retired employee has already
declared himself tired and unavailable for the same government
work, but, which, by virtue of a change of mind, he would like to
assume again.
It is for this very reaosn that inequality will neither result from the
application of the challenged provision.
It bears reiteration that the equal protection clause does not forbid all
legal classification. What is proscribed is a classification which is
arbitrary and unreasonable. The consitutional guarantee is not
violated by a reasonable classification based upon substantial
distinctions, where the classification is germane to the purpose of the
law and applies to all those belonging to the same class.
The purpose of the law is to allow the emergence of younger blood in
local governments. The classification being pursuant to that purpose,
it cannot be considered as invalid “even if at times, it may be
susceptible to the objection that it is marred by theoretical
inconsistencies.”
Moreover, to justify the nullification of a law, there must be a clear and
unequivocal breach of the Constitution, not a doubtful and equivocal
breach.
It is within the competence of the legislature to prescribe qualifications for
one who desires to become a candidate for office provided they are
reasonable, as in this case.
On the case of Igot and Alfred: Explicit is the constitutional provision that
in all criminal prosecutions, the accused shall be PRESUMED
INNOCENT until the contrary is proved, and shall enjoy the right to
be heard by himself and counsel.
The challenged proviso contravenes the constitutional presumption of
innocence, as a candidate is disqualified from running for public
office on the ground alone that charges have been filed against
him before a civil or military tribunal. It condemnts before one is
fully heard.
In ultimate effect, except as to the degree of proof, no distinction is made
between a person convicted of acts of disloyalty and one against
whom charges have been filed for such acts, as both of them would
be ineligible to run for public office.
A legislative/administrative detrimination of guilt should not be allowed to be
substituted for a judicial determination.
Fernando, Concurring: For the first paragraph, the classification cannot be
stigmatized as lacking in rationality. It is germane to the subject. Age, as
well as the fact of retirement and the receipt of retirement benefits are
factors that can enter into any legislative determination of what
disqualifications to impose.
It suffices then that the laws operate equally and uniformly on all persons
under similar circumstances or that all persons must be treated in the
same manner, the conditions not being different, both in the privileges
conferred and the liabilities imposed. Favoritism and undue preference
cannot be allowed. For the principle is that equal protection and security
shall be given to every person under circumstances, which if not
identical, are analogous. If law be looked upon in terms of burden or
charges, those that fall within a class should be treated in the same
fashion, whatever restrictions cast on some in the group equally binding
on the rest.
Teehankee, Dissenting: The first paragraph is arbitrary, oppressive and
unreasonable. Persons similarly situated are not similarly treated, eg.g. a
retired vice-governor, mayor or councilor of 65 is entitled to run for
governor, but the petitioner is barred from doing so. Both are 65 and are
retirees, yet one is barred from running for the office of governor.
Is there not here, therefore, a gross denial of the cardinal constitutional
guarantee that equal protection and security shall be given under the law
to every person, under analogous if not identical circumstances?
COMELEC’s claim, as accepted by the majority is that the purpose of the
special disqualification is “to infuse new blood in local governments” but
the classification (that would bar 65-year old retirees from running for the
same elective local office) is not rationan or reasonable.
It is not germane nor relevant to the alleged purpose of “infusing new blood”
because such “old blood” retirees may continue in local governments
since they are not disqualified at all to run for any other local elective
office.
Furthermore, other 65-year olds who have likewise retired from the judiciary
and other branches are not in any manner disqualified to run.
If they have not received such retirement benefits, they are not disqualified.
Certainly, their disqualification or non-disqualification and consequent
classification as “old blood” or “new blood” cannot hinge on such an
irrelevant question of whether or not they have received their retirement
benefits.
GOESAERT v. CLEARY (IYA) While Michigan may deny to all women opportunities for
Novembre 19, 1948 | Frankurter, J. | Equal Protection Clause bartending, Michigan cannot play favorites among women
without rhyme or reasons.
PETITIONER: Goesaert
RESPONDENTS: Cleary

SUMMARY: The present case questions the state law which prohibits
females from being bartenders except those who are wives or daughters
of male liquor establishments.

The SC upheld the validity of the law saying that the Michigan
legislature’s wisdom cannot be questioned, the law may be the
legislature’s way of promoting safety and security amongst its people.

DOCTRINE:The Constitution in enjoining the equal protection of the laws


upon States precludes irrational discrimination as between persons or
groups of persons in the incidence of a law. However, the Court annot cross-
examine either actually or argumentative the mind of Michigan legislators
nor question their motives.

FACTS:
Section 19 of Act 133 of the Public Acts of Michigan 1945 controls the
sale of liquor. It requires bartenders to be license in all cities having a
population of 50,000 or more, but no female may be so licensed
unless she be ‘the wife or daughter of the male owner’ of a licensed
liquor establishment.
The claim in the present case is that Michigan cannot forbid females
generally for being barmaids and at the same time make an
exception in favor of the wives and daughters of male owners of
liquor establishments.

ISSUE/s:
WON the Equal Protection of the Laws Clause of the Fourteenth
Amendment barred Michigan form making the classification the State
has made between wives and daughters of owners of liquor places
and wives and daughters of non-owners - NO

RULING: Validity of the law is upheld.

RATIO:
The Fourteenth Amendment did not tear history up by the roots, and the
regulation of the liquor traffic is one of the oldest and most
untrammeled of legislative powers.
The Constitution in enjoining the equal protection of the laws upon States
precludes irrational discrimination as between persons or groups of persons
in the incidence of a law.
Michigan evidently believes that the oversight assured through ownership of a
bar by a barmaid’s husband or father minimizes the hazards that may
confront a barmaid without such protecting oversight.
Michigan has not violated its duty to afford equal protection of its laws.
We cannot cross-examine either actually or argumentative the mind of Michigan
legislators nor question their motives.
Rutledge, Douglas, and Murphy, dissenting: The statute arbitrarily
discriminates between male and female owners of liquor establishments. A
male owner may imply his wife or daughter, while a female owner may not
work as barmaid herself nor employ her daughter int hat position. Since there
could be no other conceivable justification for such discrimination against
women owners of liquor establishments, the statue should be held invalid as
a denial of equal protection.
ORMOC SUGAR COMPANY v. ORMOC CITY (ELIEL) CFI rendered a decision that upheld the constitutionality of the ordinance.
February 17, 1968 | Bengzon, J.P., J | Equal Protection Clause Hence this appeal.

PETITIONER: Ormoc Sugar Company, Inc.


RESPONDENTS: Treasurer of Ormoc City, The Municipal Board of Ormoc

SUMMARY: The Municipal Board of Ormoc City passed Ordinance No. 4


imposing a tax equivalent of 1% per export sale to foreign countries on all
productions of centrifugal sugar milled at Ormoc Sugar Company, Inc.
After paying said municipal taxes, Ormoc Sugar filed a complaint with the
CFI declaring that the enacted ordinance violates the equal protection
clause and that the City had no authority to pass such tax ordinance. The
CFI, however, upheld that the City is well within its authority to pass tax
ordinances under RA 2264, and that the ordinance was valid. On appeal
to the SC, the Court reversed the decision of the CFI and declared such
ordinance unconstitutional and violative of the equal protection clause,
because it only taxes Ormoc Sugar and does not apply to future
conditions. In effect, it does not tax any subsequently established sugar
mills who belong to the same class.

DOCTRINE: Equal protection clause applies only to persons or things


identically situated and does not bar a reasonable classification of the
subject of legislation, and a classification is reasonable where (1) it is
based on substantial distinctions which make real differences; (2) these
are germane to the purpose of the law; (3) the classification applies not
only to present conditions but also to future conditions which are
substantially identical to those of the present; (4) the classification
applies only to those who belong to the same class.

FACTS:
The Municipal Board of Ormoc City passed Ordinance No. 4, imposing
“on any and all productions of centrifugal sugar milled at the Ormoc
Sugar Company, Inc., a municipal tax equivalent to 1% per export
sale to the USA and other foreign countries.”
Payments for said tax were made by Ormoc Sugar Company, Inc. for a
total of P12,087.50
Ormoc Sugar Company, Inc. filed before the CFI a complaint against the City
of Ormoc alleging that the ordinance is unconstitutional for being
violative of the equal protection clause and the rule of uniformity of
taxation.
It further alleged that the tax is neither a production nor a license tax
which Ormoc City under Sec 2 RA 2264 is authorized to impose; and
that the tax amounts to a customs duty, fee or charge in violation of
paragraph 1 Sec 2 of RA 2264 because the tax is both the sale and
export of sugar.
City Ormoc asserted that the tax ordinance was within the city’s power to
enace under the Local Autonomy Act and that the same did not
violate the constitutional limitations.
ISSUE/s:
WoN City of Ormoc can impose tax ordinances - YES
WoN Ordinance No. 4 violates the equal protection clause – YES

RULING: Reversed. Ordinance is unconstitutional.

RATIO:
The production of sugar alone is not taxable; the only time the tax
applies is when the sugar produced is exported.
Contrary to Sec 2287 not allowing municipal councils to impose tax, Sec
2 of RA 2264 gave chartered citites, municipalities and municipal
districts authority to levy for public purposes just and unofrm taxes,
licenses or fees.
Equal protection clause applies only to persons or things
identically situated and does not bar a reasonable classification
of the subject of legislation, and a classification is reasonable
where (1) it is based on substantial distinctions which make real
differences; (2) these are germane to the purpose of the law; (3)
the classification applies not only to present conditions but also
to future conditions which are substantially identical to those of
the present; (4) the classification applies only to those who
belong to the same class.
A perusal of the requisites instantly shows that the questioned
ordinance does not meet them, for it taxes only centrifugal sugar
produced and exported by the Ormoc Sugar Company, Inc. and
none other.
At the time of the taxing ordinance’s enactment, Ormoc Sugar
Company, Inc., it is true, was the only sugar central in the city of
Ormoc. Still, the classification, to be reasonable, should be in
terms applicable to future conditions as well.
The taxing ordinance should not be singular and exclusive as to
exclude any subsequently established sugar central, of the
same class as Ormoc Sugar, for the coverage of the tax.
HIMAGAN v. PEOPLE (JP) be limited to ninety days and, also, on the ruling in Deloso v.
October 7, 1994 | Kapunan, J. | Equal Protection Clause Sandiganbayan,

PETITIONER: Ishmael Himagan


RESPONDENTS: People of the Philippines, Hon. Judge Hilario Mapayo

SUMMARY: Petitioner policeman Himagan was implicated in the killing of


Bejamin Machiar, Jr. and the attempted murder of Bernabe Machitar. The
trial court preventively suspended the petitioner according to RA 6975.
Petitioner claims that the second sentence of RA 6975, the Civil Service
Law and jurisprudence support that he should only be suspended for a
maximum of 90 days. SC held that there is nothing ambiguous in RA
6975. According to the RA, members of the PNP shall be preventively
suspended from their position when a complain of sufficient substance is
filed for a grave felony with a penalty of 6 years and more. Since murder
and attempted murder falls squarely in the law, petitioner should be
suspended. The law for preventive suspension is not violative of equal
protection because policemen carry the badge of the law and they can
use them to harass or intimidate normal citizens.

DOCTRINE: The reason why members of the PNP are treated differently
from the other classes of persons charged criminally or administratively
insofar as the application of the rule on preventive suspension is concerned
is that policemen carry weapons and the badge of the law which can be used
to harass or intimidate witnesses against them, as succinctly brought out in
the legislative discussions.

FACTS:
Petitioner Himagan, a policeman assigned with the medical company of
the PNP Regional Headquarters in Davao, was implicated in the
killing of Benjamin Machiar, Jr. and the attempted murder of Bernabe
Machitar. (Note: the details of the murder are not explained in the
case)
The trial court issued an order suspending petitioner Himagan until the
termination of the case. This is in accordance with Section 47 of
RA 6975 otherwise known as Department of Interior and Local
Government Act of
1990 which provides:
SEC. 47. Preventive Suspension Pending Criminal
Case.·Upon the filing of a complaint or information sufficient
in form and substance against a member of the PNP for
grave felonies where the penalty imposed by law is six (6)
years and one (1) day or more, the court shall immediately
suspend the accused from office until the case is terminated.
Petitioner filed a motion to lift the order for his suspension, relying on Section
42 of P.D. 807 of the Civil Service Decree, that his suspension should
and Layno v. Sandiganbayan. qualify or limit the first sentence. The two can stand independently
Respondent judge denied the motion reiterating that according to Sec. of each other.
47 of RA 6975, the accused shall be suspended until his case is The first refers to the period of suspension. The second deals with
terminated. the time frame within which the trial should be finished.
Hence the petition for certiorari and mandamus to command the judge to
lift petitioner’s preventive suspension.

ISSUE/s:
WoN Sec. 47 limits the period of suspension of erring police officers to
90 days - NO

RULING: Petition is hereby dismissed.

RATIO:
There is no question that the case of petitioner who is charged with
murder and attempted murder under the Revised Penal Code falls
squarely under Sec. 47 of RA 6975 which specifically applies to
member of the PNP.
In dispute however, is whether the provision limits the period of
suspension to 90 days, considering that:
While the first sentence of Sec. 47 provides that the accused
who is charged with grave felonies where the penalty
imposed is six (6) years and one (1) day shall be suspended
from office “until the case is terminated”
The second sentence on the other hand of the same section
mandates that the case, which shall be subject to continuous
trial, shall be terminated within 90 days from the
arraignment of the accused.
Petitioner also posits that as a member of the PNP, he is covered by the
Civil Service Law, particularly Sec. 42 of PD 807 of the Civil Service
decree which likewise limits the maximum period of suspension to 90
days.
He claims that an imposition of preventive suspension of over 90 days is
contrary to the Civil Service Law and would be a violation of his
constitutional right to equal protection of laws.
The court disagrees with the petitioner. The language of the first
sentence of Sec. 47 of R.A. 6975 is clear, plain and free from
ambiguity. It gives no other meaning than that the suspension
from office of the member of the PNP charged with grave
offense where the penalty is six years and one day or more
shall last until the termination of the case. The suspension
cannot be lifted before the termination of the case.
The second sentence of the same Section providing that the trial must
be terminated within ninety (90) days from arraignment does not
There is nothing in R.A. 6975 that suggests that the preventive
suspension of the accused will be lifted if the trial is not terminated
within that period.
Petitioner misapplies Sec. 42 of PD 807. A meticulous reading of the
section clearly shows that it refers to the lifting of preventive
suspension in pending administrative investigation, not in criminal
cases, as here.
With regard to the Civil Service Law argument, its IRR are applicable to
the members of the PNP insofar as the provisions are not
inconsistent with RA 6975. Certainly, Section 42 of the Civil Service
Decree which limits the preventive suspension to ninety (90) days
cannot apply to members of the PNP because Sec. 47 of R.A. 6975
provides differently, that is, the suspension where the penalty
imposed by law exceeds six (6) years shall continue until the case is
terminated.
Petitioner’s reliance on Layno and Deloso is misplaced. These cases all
stemmed from charges in violation of R.A. 3019 (1060), otherwise
known as the Anti- Graft and Corrupt Practices Act which, unlike R.A.
6975, is silent on the duration of the preventive suspension.
In the case of Layno, the duly elected mayor of Lianga, he had been
suspended for four months. The SC held that his indefinite
preventive suspension violated the equal protection clause and
shortened his term of office. The case of Deloso had the same
doctrine.
On equal protection: The reason why members of the PNP are
treated differently from the other classes of persons charged
criminally or administratively insofar as the application of the
rule on preventive suspension is concerned is that policemen
carry weapons and the badge of the law which can be used to
harass or intimidate witnesses against them, as succinctly
brought out in the legislative discussions.
If a suspended policeman criminally charged with a serious offense
is reinstated to his post while his case is pending, his victim
and the witnesses against him are obviously exposed to
constant threat and thus easily cowed to silence by the mere
fact that the accused is in uniform and armed.
The imposition of preventive suspension for over 90 days under
Section 47 of R.A. 6975 does not violate the suspended
policemanÊs constitutional right to equal protection of the laws.
Recognizing the existence of real differences among men, the equal
protection clause does not demand absolute equality. It merely
requires that all persons shall be treated alike, under like
circumstances and conditions both as to the privileges
conferred and liabilities enforced.
SISON v. ANCHETA (HENRY) WoN BP Blg 135 Sec. 1 is violative of the equal protection clause of the
Constitution - NO
July 25, 1984 | Fernando, C.J. | Equal Protection of Law

PETITIONER: Antero M. Sison


RESPONDENTS: Ruben B. Ancheta, Actiong Commissioner, BIR, et al.

SUMMARY: Petitioner assails the constitutionality of BP Blg. 135 Sec 1


in the SC, that it violates equal protection, as it imposes higher tax rates
upon his income. The SC ruled in favor of the respondents, that indeed,
assailed BP is constitutional, as equal protection does not necessarily
mean putting everyone in equal positions, as each shall be classified
according to class.

DOCTRINE: The Constitution does not require things which are different in
fact or opinion to be treated in law as though they were the same” Hence the
constant reiteration of the view that classification if rational in character is
allowable.

FACTS:
What is being assailed in this petition for review is BP Blg. 135, Sec. 1,
which further amends Sec 21 of the National Internal Revenue Code
of 1977 (NIRC) which provides the rates of tax on citizens or
residents on:
Taxable compensation income
Taxable net income
Royalties, prizes, and other winnings
Interest from bank deposits and yield or any other monetary
benefit from deposit substitutes and from trust fund and
similar arrangements
Dividends and share of individual partner in the net profits of
taxable partnership
Adjusted goss income
Petitioner alleges in the SC that he would be unduly discriminated
against by the imposition of higher rates of tax upon his income
arising from the exercise of his position vis-à-vis those which are
imposed uponfixed income or salaried individual taxpayers.
He claims that the BP is arbitrary amounting to calss legislation,
oppressive and capricious in character. There is hence a
transgression of both equal protection and due process clauses of
the Constitution as well as the rule of requiring uniformity in taxation.

ISSUE/s:
RULING: SC dismissed the petition with costs against the petitioner.

RATIO:
The applicable standard to avoid the charge that there is a denial of this
constitutional mandate whether the assailed act is in the exercise fo
the police power or the power of eminent domain is to demonstrate
“that the governmental act assailed, far from being inspired by the
attainment of the common weal as prompted by the spirit of histolity,
or at the very least, discrimination that finds no support in reason.
Laws operate equally and uniformly on all persons under similar
circumstances or that all persons must be treated in the same
manner, the conditions not being different, both in the privileges
conferred and the liabilities imposed.
Equal protection and security shall be given to every person under
circumstances, which if not identical are analogous.
The equal protection clause is, of course, inspired by the noble concept
of approximating the ideal of the law’s benefits being available to all
and the affairs of men being governed by that serene and impartial
uniformity, which is of the very essence of the idea of law.
However, as Justice Frankfurter said, “the Constitution does not
require things which are different in fact or opinion to be treated
in law as though they were the same” Hence the constant
reiteration of the view that classification if rational in character is
allowable.
Equality and uniformity in taxation means that all taxable articles or kinds
of property of the same class shall be taxed at the same rate. The
taxing power has the authority to make reasonable and natural
classifications for purposes of taxation.
The SC hence held that there is ample justification then for the Batasang
Pambansa to adopt the gross system of income taxation to
compensation income, while continuing the system or net income
taxation as regards professional and business income.
In sum, the petition is without merit because of:
Lack of factual foundation to show arbitrary character of the
assailed provision
Force of controlling doctrines on due process, equal protection,
and uniformity in taxation
Reasonableness of distinction between compensation and
taxable net income of professionals and businessmen
PHILIPPINE JUDGES ASSOCIATION v. PRADO (DANNAH) WoN the title of RA 7354 embraces more than one subject – NO
November 11, 1993 | J. | Equal Protection Clause WoN it passed the required readings in both Houses of Congress – YES
WoN it is discriminatory and encroaches on the independence of
PETITIONER: The Philippine Judges Association et al. the Judiciary – YES
RESPONDENTS: Hon. Pete Prado as Secretary of DOTC et al.
RULING:. ACCORDINGLY, the petition is partially GRANTED and Section 35
SUMMARY: Petitioners who are members of lower courts assail the of R.A. No. 7354 is declared UNCONSTITUTIONAL. Circular No. 92-28 is
repealing clause (or Section 35) of RA 7354 which is entitled “An Act SET ASIDE insofar as it withdraws the franking privilege from Judiciary, and
Creating the Philippine Postal Corporation, Defining its Powers, Functions the National Land Registration Authority and its Registers of Deeds to all of
and Responsibilities, Providing for Regulation of the Industry and for Other which offices the said privilege shall be RESTORED. The temporary
Purposes Connected Therewith.” This clause repealed the franking privilege restraining order dated June 2, 1992, is made permanent.
given to the Judiciary, National Land Registration Authority and etc. while
still keeping the privilege for the President, Vice President, members of
Congress and etc.
RATIO:
Respondents claim that it is not discriminatory and the privilege was also On issue #1: The purposes of Article VI Sec. 26 stating that “Every bill
withdrawn from other agencies. The Court ruled otherwise, stating that there passed by the Congress shall embrace only one subject which shall
is no valid basis for the withdrawal of the franking privilege from the be expressed in the title thereof” are:
Judiciary, especially since it is the Judiciary that needs it the most. To prevent hodge-podge or “log-rolling legislation”
To prevent surprise or fraud upon the legislature
DOCTRINE: Equal protection simply requires that all persons or things
To fairly apprise the people.
similarly situated should be treated alike aboth as to rights conferred and The title of the bill is not required to be an index to the body of the act, or
responsibilities imposed. to be comprehensive as to cover every single detail of the measure.
Similar subjects, in other words, should not be treated differently, so as It has been held that if the title fairly indicates the general subject and
reasonably covers all the provisions of the act, and is not calculated
to give undue favor to some and unjustly discriminte against others.
to mislead the legislature or the people, there is sufficient
compliance with the constitutional requirement.
FACTS: Where a statute repeals a former law, such repeal is the effect and not
The Philippine Postal Corporation through Circular No. 92-28 the subject of the statute.
implemented section 35 of RA 7354. These measures withdrew the The Court is convinced that the withdrawal of the franking privilege from
franking privilege1 from the Supreme Court, the Court of Appeals, some agencies is germane to the accomplishment of the principal
RTCs, MTCs, etc. objective of the statute.
Petitioners are members of lower courts who feel that their official functions On issue #2: While it is true that a conference committee is the
as judges will be prejudiced because of these measures. The National
Land Registration Authority took a common cause with them as well. mechanism for compromising differences between the Senate and
RA 7354 is entitled “An Act Creating the Philippine Postal Corporation, the House, it is not limited in its jurisdiction to this question.
Defining its Powers, Functions and Responsibilities, Providing for It is a matter of record that the Conference Committee Report on the bill in
Regulation of the Industry and for Other Purposes Connected question was returned to and duly approved by both the Senate and the
Therewith.” HoR. Thereafter, the bill was enrolled with certification of the Senate
Respondents believe that it is not discriminatory because the withdrawal President and Speaker of the House, and was approved by President
is based on a valid classification in accordance with the equal Cory Aquino.
protection clause, because the privilege has also been withdrawn
Under the doctrine of separation of powers, the Court may not inquire
from other agencies. beyond
ISSUE/s:
Franking privilege means sending certain matter through the public mails without
payment of postage in pursuance of a personal or official privilege.
the certification of the approval of a bill from the presiding officers of The Government supplied 55% of the capital (P10B) of the Philippine
Congress. Postal Service. It enjoys tax exemption and substantial revenues.
Casco Philippine Chemical v. Gimenez laid down the rule that the It is not likely that the Judiciary will cripple the Corporation.
enrolled bill is conclusive upon the Judiciary.
On issue #3: It is alleged that the statute is discriminatory because while
withdrawing the privilege from the Judiciary, it retains the same for
the President, VP, Senators, Members of the House and etc.
The equal protection of the laws is embraced in the concept of due
process, as every unfair discrimination offends the requirements of
justice and fairplay.
Arbitrariness in general may be challenged in the basis of the due
process clause.
But if the particular act assailed partakes of an unwarranted partiality or
prejudice, the sharper weapon to cut it down is the equal protection
clause.
Equal protection simply requires that all persons or things similarly
situated should be treated alike aboth as to rights conferred and
responsibilities imposed.
Similar subjects, in other words, should not be treated differently,
so as to give undue favor to some and unjustly discriminte
against others.
What it requires is equality among equals as determined according to a valid
classification (or the grouping of persons or things similar to each other
in certain particulars and different from all others in these same
particulars).
In the Court’s view, the only acceptable reason for the grant of the
franking privilege was the perceived need of the grantee for the
accomodation, which would justify a waiver of substantial revenue by
the Corporation in the interest of providing for a smoother flow of
communication between the government and the people.
Assuming that basis, it is questinable why it is the Judiciary that has
been denied the franking privilege.
Prado et al. point out that data from the Postal Service Office show that from
1988 to 1992, frank mails from the Judiciary amounted to P86M out of
P90M.
They conclude that because of this considerable volume, the franking
privilege must be withdrawn from the Judiciary.
The argument is self defeating. They are in effect saying that the franking
privilege should be extended only to those who do not need it very
much, if at all, (like the widows of former Presidents) but not to those
who need it badly (especially the courts of justice).
The Judiciary is being faulted for the delay of administration of justice and
this will only deepen the serious problem.
The Judiciary also has the lowest appropriation in the national budget. This
provision will make it even more difficult for the Judiciary.
Its repealing clause is a discriminatory provision that denies the Judiciary the
equal protection of the laws guaranteed for all persons or things similarly
situated.
TIU v. CA (IYA) area” and not to include the entire Olongapo City and other areas
January 20, 1999 | Panganiban, J. | Equal Protection Clause mentioned in Section 12 of the law.
The CA further justified the limited application of the tax incentives as
being within the prerogative of the legislature. EO 97-A merely
PETITIONER: Conrado L. Tiu, Juan T. Montelibano, Jr. and Isagani M.
implements the legislative purpose of RA 7227.
Jungco RESPONDENTS: Court of Appeals, Hon. Teofisto T. Guingona,
Jr. Bases Conversion nd Development Authority, et al. ISSUE/s:
WON EO 97-A violates the equal protection clause of the Constitution
SUMMARY: Petitioners in this case question the limitation of EO 97-A to
-NO
the “Secured Area” in the former Subic Military Base. They claim that the Specific Issue: WON the provisions of EO 97-A confining the
special privileges such as the tax and duty-free importation of goods application of RA 7227 within the secured area and excluding
should apply also to those outside the said area. the residents of the zone outside of the secured area is
discriminatory or not
The SC held that the EO is valid because it merely implements the intent
of RA 7227. RULING: The validity of EO 97-A is upheld.

DOCTRINE : The fundamental right of equal protection of the laws is not RATIO:
absolute, but is subject to reasonable classification. Classification to be
Petitioners contend that the SSEZ encompasses the City of Olongapo
valid must:
and the Municipality of Subic in Zambales and the area formerly
rest on substantial distinctions occupied by the Subic Naval Base.
germane to the purpose of the law However EO 97-A, according to them, narrow down the area within which
not limited to existing conditions only the special privileges granted to the entire zone would apply to the
apply equally to al members of the same class present “fenced-in former Subic Naval Base” only.
The Solicitor General defends that the validity of the EO 97-A by arguing
that Section 12 of RA 7227 clearly vests in the President the authority
FACTS:
to delineate the metes and bounds of the SSEZ.
Congress passed into law RA 7277 entitled An Act Accelerating the The SC rules favor of the constitutionality and validity of the assailed EO.
Conversion of Military Reservations Into Other productive Uses, It is not violative of the equal protection clause, neither is id
Creating the Bases Conversion and Development Authority for this discriminatory.
Purpose, Providing Funds Therefor and for Other Purposes. The fundamental right of equal protection of the laws is not absolute, but
Section 12 created the Subic Special Economic Zone and granted is subject to reasonable classification. Classification to be valid must:
thereto special privileges of tax and duty-free among goods within rest on substantial distinctions
the area. germane to the purpose of the law
President Fidel V. Ramos issued EO No. 97 clarifying the application of not limited to existing conditions only
the tax and duty incentives. Nine days after the President issued EO apply equally to al members of the same class
No. 97-A specifying the area within which the tax-and-duty-free In Ichong v. Hernandez: The equal protection clause does not demand
privilege was operative. absolute equality among residents; it merely requires that all persons
Petitioners challenge before the SC the constitutionality of EO 97-A for shall be treated alike, under like circumstances and conditions both
allegedly being violative of their right to equal protection of the laws. as to privileges conferred and liabilities enforced.
The SC referred the matter to the CA. Purpose of the law: RA 7277 aims primarily to accelerate the conversion
The CA held that there is no substantial difference between the of military reservations into productive uses. It is reasonable for the
provisions of EO 97-A and Section 12 of 7227. In both, the ‘Secured President to have delimited the application of some incentives to he
Area’ is precise and well defined as the lands occupied by the Subic confines of the former Subic military base.
Naval Base and its contiguous extensions. It is biased to big inventions because they are the ones who can
The CA concluded that the petitioners could not claim that EO 97-A is pour huge investments to spur economic growth. The
unconstitutional, while at the same time maintaining the validity of classification, therefore, is germane to the purposes of the
RA7227. law.
The Congress’ intent was to confine the coverage of the SSEZ to the
“secured
The equal protection guarantee doe snot require territorial uniformity of laws.
As long as there are actual and material differences between territories,
there
i son violation of the constitutional clause.
The classification applied equally to all the resident individuals and
businesses within the secured are. The residents being in like
circumstances or contributing directly to the achievement of the end
purpose of the law, are not categorized further.
They are all similar treated, both in privileges granted and in obligations
required.
The School grants foreign-hires certain benefits not accorded local-hires.
INTERNATIONAL SCHOOL v. QUISUMBING (ELIEL) These include housing, transportation, shipping costs, taxes and
home leave
June 1, 2000 |Kapunan, J | Equal Protection Clause

PETITIONER: International School Alliance of Educators


RESPONDENTS: Hon. Leonardo Quisumbing; Internation School, Inc.

SUMMARY: The School under PD 732 employs its own teaching and
managemtn personell elected either locally or abroad. These employees
are classified as foreign-hires and local-hires. Such classification
provides that foreign-hires are given 25% more salary than local-hires,
and in addition are provided housing, transporation, and shipping costs.
The School justified this on the “dislocation factor” and “limited tenure” of
such foreign-hires. The Alliance went on a strike, and since the NCMB
could not settle the issue the DOLE took jurisdiction and ruled in favor of
the School. Hence this appeal, whether there is a reasonable distinction
for the salary rates. The SC ruled that the classification of foreign-hires
and local-hires in terms of salary is not justified based on the “dislocation
factor” and “limited tenure”. And that there is no evidence proving that
foreign-hires perform 25% more effectively or efficiently as compared to
local-hires. The presumption is that they perform the same under equal
qualifications and rank. However, the SC appreciated that the housing,
transportation, and other benefits are deemed adequate to compensate
the foreign-hires.

DOCTRINE: The State, therefore, has the right and duty to regulate the
relations between labor and capital. These relations are not merely
contractual but are so impressed with public interest that labor contracts,
collective bargaining agreements included, must yield to the common
good

FACTS:
International School, Inc. (The School), pursuant ot PD 732, is a
domestic educational institution established primarily for dependents
of foreign diplomatic personnel and other temporary residents.
To enable the School to continue carrying outs its education program and
improve it standard of instruction, Sec 2(c) of the same decree
authorizes the School to employ its own teaching and management
personnel selected by it either locally or abroad, from PH or other
nationalities, such personnel being exempt from otherwise applicable
laws and regulations attending their employment, except laws that
have been or will be enacted for the protection of employees.
The School hires both foreign and local teachers as members of its
faculty, classifying the same into two: (1) foreign-hires and (2) local-
hires.
Tests to determine whether foreign or local: Domicile, home economy,
economic allegiance, and purpose of being hired.
travel allowance. Foreign-hires are also paid a salary rate 25% more similar functions and responsibilities, wchi they perform under
than local-hires. The School justifies the difference on two “significant similar working conditions.
economic disadvantages” foreign-hires have to endure, namely: (a)
the “dislocation factor” and (b) limited tenure”.
Internation School Alliance of Educators (Alliance), “ a legitimate labor union
and the collective bargaining rep of all faculty members of the School,
contested the difference in salary rates between foreign and local-hires.
Alliance filed a notice of strike. The failure of the National Conciliation
and Mediation Board to bring the parties to a compromise prompted
the DOLE to assume jurisdiction over the dispute.
DOLE Acting Sec Trajano, issued an order resolving the parity and
representation issues in favor of the School.
DOLE Sec Quisumbing subsequently denied Alliance’s MR. Hence this
appeal.

ISSUE/s:
WoN the classification of hire and salaries violates equal protection -
YES

RULING: Party Granted. DOLE decision reversed.

RATIO:
International law, which springs from general principles of law, likewise
proscribes discrimination. UHRD, ICESCR, ICEFRD, CDE, all
embody the general principles against discrimination, the very
antitheses of fairness and justice. The Philippines, through its
Constitution, has incorporation this principle as part of its national
laws.
The Constitution specifically provides that labor is entitled to “humane
conditions of work” These conditions are not restricted to the physical
workplace but include as well the manner by which employers treat
their employees.
The Constitution also directs the State to promote “equality of
employment opportunities for all.” Similarly, the Labor Code provides
that the State shall “ensure equal work opportunities regardless of
sex, race or creed.”
Persons who work with substantially equal qualifications, skill,
effort and responsibility, under similar conditions, should be
paid similar salaries. This rule applies to the School, its
“international character” notwithstanding.
If an employer accords employees the same position and rank, the
presumption is that these employees perform equal work. This
presumption is borne by logic and human experience.
The employer has discriminated against that employee; it is for the
employer to explain why the employee is treated unfairly.
The employer in this case has failed to discharge this burden. There
is no evidence here that foreign-hires perform 25% more
efficiently or effectively than the local-hires. Both groups have
The local-hires perform the same services as foreign-hires and they
ought to be paid the same salaries as the latter.
For the same reason, the “dislocation factor” and the foreign-hires’
limited tenure also cannot serve as valid bases for the
distinction in salary rates.The discloation factor and limited
tenure affecting foreign-hires are adequately compensated by
certain benefits accorded them which are not enjoyed by local-
hires, such as housing, transportation, shipping costs, taxes
and home leave travel allowances.
The Constitution enjoins the State to “protect the rights of works and
promote their welfare,” “to afford labor full protection.”
The State, therefore, has the right and duty to regulate the relations
between labor and capital. These relations are not merely
contractual but are so impressed with public interest that labor
contracts, collective bargaining agreements included, must
yield to the common good.
The point-of-hire classification employed by the School to justify
the distinction in the salary rates of foreign-hires and local-hires
is an invalid classification. There is no reasonable distinction
between the services rendered by foreign-hires and local-hires.
The practice of the School of according higher salaries to
foreign-hires contravenes public policy and, certainly, does not
deserve the sympathy of this Court.
DE GUZMAN v. COMELEC (JP) Section 44 of RA 8189 enjoys the presumption of validity, and the
July 19, 2000 | Purisima, J. | Equal Protection Clause Court

PETITIONER: Agripino A. De Guzman Jr. et al.


RESPONDENTS: Commission on Elections

SUMMARY: Petitioners are City or Municipal Election Officers who were


assigned by the COMELEC to different stations by virtue of Sec. 44 of RA
8189 (Voter’s Registration Act of 1996). They impugne Sec. 44 for being
violative of equal protection for there being no substantial distinction
between them and other COMELEC officials. The SC held that the law
had the purpose of eliminating corruption and ensuring impartiality of the
highest officials of the COMELEC in a city or municipality. The
constitutionality of Sec. 44 is upheld.

DOCTRINE: The singling out of election officers in order to “ensure the


impartiality of election officials by preventing them from developing
familiarity with the people of their place of assignment” does not violate
the equal protection clause of the Constitution.

FACTS:
RA 8189 or otherwise known as “The Voter’s Registration Act of 1996” was
enacted by President Fidel Ramos. Section 44 provides: No Election
Officer shall hold office in a particular city or municipality for more than
four (4) years. Any election officer who, either at the time of the
approval of this
Act or subsequent thereto, has served for at least four (4) years
in a particular city or municipality shall automatically be
reassigned by the Commission to a new station outside the
original congressional district.
By virtue of such provision of law, COMELEC promulgated Resolution
Nos. 97-0002 and 97-010 for the implementation thereof.
Thereafter, the COMELEC issued several directives reassigning the
petitioners, who are either City or Municipal Election Officers, to
different stations.
Aggrieved, petitioners found their way to the SC via the present petition
assailing the validity of Section 44 of RA 8189.

ISSUE/s:
WoN RA 8189 violates the equal protection clause enshrined in the
constitution - NO

RULING: Petition is dismissed. The constitutionality of Sec. 44 of RA 8189 is upheld.

RATIO:
discerns no ground to invalidate it.
etitioners theorize that Section 44 of RA 8189 is violative of the equal protection
clause of the 1987 Constitution because it singles out the City and Municipal
Election Officers of the COMELEC as prohibited from holding office in the
same city or municipality for more than four (4) years. They maintain that
there is no substantial distinction between them and other COMELEC
officials, and therefore, there is no valid classification to justify the objective
of the provision of law under attack.
The equal protection clause of the 1987 Constitution permits a valid classification
under the following conditions:
The classification must rest on substantial distinctions;
The classification must be germane to the purpose of the law;
The classification must not be limited to 2 existing conditions only; and
The classification must 4 apply equally to all members of the same class
After a careful study, the ineluctable conclusion is that the classification under
Section 44 of RA 8189 satisfies the aforestated requirements.
The singling out of election officers in order to “ensure the impartiality of
election officials by preventing them from developing familiarity with
the people of their place of assignment” does not violate the equal
protection clause of the Constitution.
In this case, it can be discerned that the legislature thought the noble purpose of the
law would be sufficiently served by breaking an important link in the chain of
corruption than by breaking up each and every link thereof.
Verily, under Section 3(n) of RA 8189, election officers are the highest officials or
authorized representatives of the COMELEC in a city or municipality. It is
safe to say that without the complicity of such officials, large scale anomalies
in the registration of voters can hardly be carried out.
FARINAS v. EXECUTIVE SECRETARY (HENRY) Elective officials – occupy office by virtue of their mandate based
December 10, 2003 | Callejo, Sr., J. | Equal Protection of Law upon the popular will
Appointive officials – not elected by popular will
PETITIONER: Rodolfo C. Fariñas, et al. Respondnets further argue that equal protection simply requires that all
RESPONDENTS: The Executive Secretary, COMELEC, et al. persons or things similarly situated are treated alike, both as to rights
conferred and responsibilities imposed.
SUMMARY: Petitioners assail RA 9006 Sec 14’s constitutionality, as it ISSUE/s:
amends Sec 67 of the Omnibus Election Code, and does not amend Sec WoN RA 9006 Sec 14 is violative of the eqiual protection clause of the
66, thus violates equal protection clause. They claim that, amending 67, Constitution - NO
which refers to elective officials, shall also be applicable to appointive
RULING: SC dismissed the petition.
officials. Respondents on the other hand aver by saying that equal
protection is not violated because there is a substantial distinction
RATIO:
between the two. The SC ruled that indeed there is a distinction between
the two types of officials, and that no violation is made by enacting the Equal protection of the law clause in the Constitution is not absolute, but
said RA. is subject to reasonable classification. If the groupings are
characterized by substantial distinctions that make real differences,
DOCTRINE: Equal protection of the law clause in the Constitution is not one class may be treated and regulated differently from the other.
absolute, but is subject to reasonable classification. If the groupings are Equal protection of the law clause is against undue favor and individual
characterized by substantial distinctions that make real differences, one or class privilege, as well as hostile discrimination or the oppression
class may be treated and regulated differently from the other. of inequality. It is not intended to prohibit legislation which is limited
either in the object to which it is directed or by territory within which it
is to operate. It does not demand absolute equality, it merely requires
FACTS: that all persons shall be treated alike under like circumstances and
What is being assailed in this case is RA 9006 (An act allowing the use of conditions both as to privileges conferred and liabilities enforced.
mass media for election propaganda, amending for purpose BP Blg Substantial distinctions clearly exist between elective officials and
881 otherwise known as the Omnibus Election Code [OEC]) Sec 14, appointive officials. The former occupy their office by virtue of
insofar as it expressly repeals BP Blg. 881 Sec 67, which provides: mandate of the electorate. They are elected to an office for a definite
Sec. 67: any elective official (national/ local) running for any office term and may be removed therefrom only upon stringent conditions.
other than the one he is holding in a permanent capacity, except On the other hand, appointive officials hold their office by virtue of
for President and VP, shall be considered ipso facto resigned their designation thereto by an appointing authority.
from his office upon the filing of his certificate of candidacy. Another substantial distinction between the two is that under Sec 55
Petition for certiorari and prohibition was filed by petitioners, who are hapter 8 Title 1 Subsection A Civil Service Commission, Bok V of the
members of the minority bloc in the HoR. Administrative Code,
Among the claims of the petitioners was that the said RA violates the Administrative officials – as officers and emoyees in the civil
equal protection clause of the Constitution insofar as it repeals service, are strictly prohibited from engaging in any partisan
Secetion 67 only of the OEC, leaving intact Sec 66 intact, which political activity or take part I any elections except to vote.
imposes a similar limitation: Elective officials – as officers and employees holding political
Sec. 66: any person holding a pblic appointive office/ position, offices are allowed to take part in political and electoral
including active members of the AFP, and officers and activities
employees in GCCs, shall be considered ipso fato resigned There being such existence of a substantial distinction between the two,
from his office upon the filing of his certificate of candidacy. equal protection is hence not infringed.
That Sec 14 discriminates against appointive officials. That by repeal of
Sec 67, an elective official who runs for office other than one which
he is holding is no longer considered ipso facto resigned therefrom
upon filinghis certificate of candidacy. Sec 66 being retained, thus the
limitations on appointive officials remain.
Respondents aver by saying that Sec 67 pertains to elective officials
while 66 refers to appointive officials. A substantive distinction exists
between these two sets of officials:
CBEA v. BSP (DANNAH) Standardization Law (SSL) or the exempt class and the rank-and-
December 15, 2004 | Puno, J. | Equal Protection Clause file (SG 19 and below) or those

PETITIONER: Central Bank (now Bangko Sentral ng Pilipinas) Employees


Association
RESPONDENTS: Bangko Sentral ng Pilipinas & Executive Secretary

SUMMARY: The CBEA assails the constitutionality of the last proviso in


Section 15(c) Article II of RA 7653 because of the alleged cut between those
with Salary Grade 20 or higher (exempt) and those with Salary Grade 19
and below (non- exempt). The others are exempted from the application of
the Salary Standardization Law and the others are not.

The Court ruled that there is a rational basis for the classification of exempt
and non-exempt. However, the Court declared said proviso unconstitutional
because of the theory of relative constitutionality. This theory provides that
although some statutes were not ruled on as constitutional, circumstances
which affect the law may render it oppressive. The promulgation of certain
statutes exempting rank-and-file employees of other Government Financial
Institutions (GSIS, SSS, Land Bank etc) made the said proviso violative of
the equal protection clause. There is no substantial or reasonable
classification between those rank-and-file employees of other GFIs, and the
rank-and-file employees of the BSP.

DOCTRINE: The classification must not only be reasonable, but must also apply
equally to all members of the class. The proviso may be fair on its face and
impartial in appearance but it cannot be grossly discriminatory in its operation,
so as practically to make unjust distinctions between persons who are without
differences.

FACTS:
On July 1993, RA 7653 (the New Central Bank Act) took effect. It
abolished the old Central Bank and created a new BSP.
On June 2001, almost eight years after the effectivity of said RA,
petitioner CBEA filed a petition for prohibition against BSP and the
Executive Secretary to restrain them from further implementing the
last proviso in Section 15(c), Article II of RA 7653
It provided that the Monetary Board will make its own system
conform as closely as possible with the principles provided
for under RA 6758 (Salary Standardization Act)
Provided, however, that compensation and wage structure of
employees whose positions fall under salary grade 19 and
below shall be in accordance with the rates prescribed
under RA 6758
CBEA assails that the proviso makes an unconstitutional cut between
BSP officers exempted from the coverage of the Salary
not exempted from coverage of SSL or the non-exempt class. observe a vigil of at least 12 years before they could be paid
CBEA also claims that it is not germane to the purposes of RA 7654 back.
which is to establish professionalism and excellence at all levels in Applicability of the equal protection clause:
the BSP. Atlantic v. Coast Line: SC of Florida ruled against continued
ISSUE/s:
WoN the last paragraph of Section 15(c) in violation of the due process
clause
– NO

RULING:. IN VIEW WHEREOF, we hold that the continued operation and


implementation of the last proviso of Section 15(c), Article II of Republic Act
No. 7653 is unconstitutional.

RATIO:
The “equal protection” clause does not prevent the Legislature from
establsihing classes of individuals or objects upon which different
rules shall operate so long as the classification is not unreasonable.
If groupings are characterized by substantial distinctions that make real
differences, one class may be treated and regulated differently from
another.
In the case at hand, it is clear in the legislative deliberations that the
exemption of officers from the SSL was intended to address the
BSP’s lack of competitiveness in terms of attracting competend
officers and executives.
The discrimination or distinction has a rational basis and is not palpably,
purely and entirely arbitrary in the legislative sense.
However, while RA 7652 started as a valid measure, the enactment of
subsequen laws exempting all rank-and-file employees of other
Government Financial Institutions (GFIs) leeched all validity out of
the challenged proviso.
Concept of relative constitutionality: A statute valid at one time may
become void at another time because of altered circumstances.
Vernon Park v. Mount Vernon: declared as unreasonable and
arbitrary a zoning ordinance because amendments to the
ordinance prohibited the use of a property except for parking
and storage of automobiles. An ordinance valid when
adopted will nevertheless be stricken down as invalid
when, at a later time, its operation under changed
conditions proves confiscatory.
Rutter v. Esteban: upheld the constitutionality of a moratorium
law but also ruled that its continued enforcement would be
unreasonable or oppressive. Creditors would have had to
application of statutes authorizing the recovery of double damages It fails to appreciate that what is at issue in the second level of scrutiny
+ attorney fees against railroad companies. The Court ruled is not the declared policy of each law per se, but the oppressive
that the statues became invalid as denying “equal protection results of Congress’ inconsistent and unequal policy towards the
of the law” BSP rank-and-file and those
Louisville v. Faulkner: CA of Kentucky declared the statute
which imposed a duty upon a railroad company of proving it
was free from negligence in the killing or injury of cattle by its
engine or cars as unconstitutional. This, notwithstanding that
the constitutionality of a statute enacted in 1893 had been
previously sustained. A statute nondiscriminatory on its
face may be grossly discriminatory in its operation.
Enactment of Ras 7907 + 8282 + 8289 + 8291 + 8523 + 8763 + 9302 =
conseqential unconstitutionality of challenged proviso.
CBEA also alleged that the last proviso of RA 7653 is also
violative of the EQP because after it was enacted, the
charters of GSIS, LBP, DBP and SSS were also amended,
but the personnel of said GFIs were all exempted from the
coverage of SSL.
Thus, within the class of rank-and-file personnel of said GFIs, the BSP
rank-and-file are also discriminated upon.
The subsequent charters of the seven other GFIs (Land Bank, SSS,
SBGFC, GSIS, DBP, HGC, PDIC) share this common proviso: a
blanket exemption of all their employees from the coverage of the
SSL.
Eleven years after the amendment of the BSP charter, the rank-and-file
of seven other GFIs were granted the exemption that was specifically
denied to the rank-and-file of the BSP. Even the SEC was granted
this in 2002.
The above-mentioned enactments constitute significant changes in
circumstance that considerably alter the reasonability of the
continued operatin of the last proviso of Section 15(c), Art II of RA
7652, thereby exposing the proviso to more serious scrutiny.
The classification must not only be reasonable, but must also apply
equally to all members of the class. The proviso may be fair on its
face and impartial in appearance but it cannot be grossly
discriminatory in its operation, so as practically to make unjust
distinctions between persons who are without differences.
The “policy determination” argument may suppor the inequality of
treatment between the rank-and-file and the officers of BSP, but it
cannot justify the inequality of treatment between BSP rank-and-file
and other GFIs who are similarly situated.
other GFIs.
The Compensation and Position Classification System of the SSL was to be
governed by the following principles
Just and equitable wages, with the ratio of compensation between
pay distinctions maintained at equitable levels
Basic compensation generally comparable with the private sector in
accordance with prevailing laws on minimum wages
The justification of Congress to the grant or withdrawal of exemption from the
SSL were
To offer competitive salary vs. private sector
The GFIs were experiencing difficulty filling up plantilla positions
The fragility of this argument is manifest. First, the BSP is the central
monitary authority. Second, it is ertainly misleading to say that “the need
for the scope of exemption necessarily varies with the particular
circumstances of each institution”
Nowhere in the deliberations is there a cogent basis for the exclusion of the
BSP rank-and-file from the exemption which was appointed to the rank-
and-file of other GFIs and SEC.
The violation to the equal protection clause becomes even more pronounced
when we are faced with this undeniable truth: that if Congress had
enacted a law for the sole purpose of exempting the eight GFIs from the
coverage of the SSL, the exclusion of the BSP rank-and-file employees
would have been devoid of any substantial or material basis.
In the case at bar, it is precisely the fact that as regards the exemption
from the SSL, there are no characteristics peculiar only to the seven
GFIs or their rank-and-file so as to justify the exemption which BSP
rank-and-file employees were denied.
While the granting of a privilege per se is a matter of policy exclusively within
the domain and prerogative of Congress, the validity or legality of the
exercise of this prerogative is subject to judicial review.
Again, it must be emphasized that the equal protection clause does not demand
absolute equality but it requires that all persons shall be treated alike, under
like circumstances and conditions both as to privileges conferred and
liabilities enforced. Favoritism and undue preference cannot be allowed.
In light of the lack of real and substantial distinctions that would justify the
unequal treatment between the rank-and-file of BSP from the seven other
GFIs, it is clear that the enactment of the seven subsequent charters has
rendered the continued application of the challenged proviso anathema to
the equal protection of the law, and the same should be declared as an
outlaw.
Equal Protection in International Law
Equal Protection in the US: The intensive review associated with Strict Scrutiny Test: For equal protection analysis,
the new equal protection imposed two demans—a demand financial need alone does not identify a suspect
not only as to means but also one as to ends. class. A law does not become invalid because os
Equal protection in Europe: Prohibits discrimination on grounds simple inequality. Financial or otherwise.
such as “ex, race, color, language, religion…” Discrimination Employment in the government is not a fundamental
on the basis of these is regarded as grouns that require strict right and government employees below salary grade
scrutiny. 20 are not a suspect class; the government is not
Equality under International Law: A classification may be struck required to present a compelling objective to justify a
down if it has the purpose or effect of violating the right to possible infringement under the strict scrutiny test.
equal protection. International law recognizes that The Intensified Means Test: The Court should accept the
discrimination may occur indirectly. legislative end, but should closely scrutinize its
Thus, the two-tier analysis made in the case at hand of the challenged relationship to the classification made. Non-exempt
provision, and its conclusion of unconstitutionality by subsequent government employees may be a sensitive but not a
operation, are in cadence and in consonance with the progressive suspect class, and their employment status may be
trend of other jurisdictions and in international law. important although not fundamental. The Court is
Generally we use the “rational basis” test, but if the challenge to the without power to disturb a legislative judgment
statute is premised on the denial of a fundamental right, or the unless there is not fair reason for the law that would
perpetuation of prejudice against persons favored by the Constitution not require with equal force its extension to others
with special protection, judicial scrutiny ought to be more strict. whom it leaves untouched.
TWO-TIER ANALYSIS Carpio, Dissenting: The BSP is a regulatory agency exercising
Rational Relationship/Rational Basis Test sovereign functions, the others are non-regulatory corporations
0 Based on substantial distinctions that make for real exercising purely commercial functions.
differences Carpio-Morales, Dissenting: The central purpose of the Equal
1 Germane to the purposes of the law Protection Clause was to eliminate racial discrimination from official
2 Not limited to existing conditions only sources in the States. I fail to see the justification for the use of the
3 Equally applicable to all members of the same class two-tier approach in determining the constitutionality of questioned
Strict Scrutinty Test provision. Why a “deferential test” for the comparison between the
0 The limitation on fundamental rights executives and rank-and-file of the BSP and a “strict test” for the
1 The implication of suspect classes other. Strict scrutiny cannot apply in the case because the petitioner
Panganiban, Dissenting: does not allege that said proviso burdens a fundamental right of its
The theory of relative constitutionality is not applicable. The members.
promulgation of several RAs regarding the exemption of the
SSL to other employees of other GFIs cannot be classified
as altered circumstances or changed conditions. This theory
finds relevance only when the factual situation covered by an
assailed law changes, not when another law is passed
pertaining to subjects not directly covered by the former.
Said proviso passed the three tests
0 Rational basis: The retention of the best and
brightest officials in an independent central monetary
authority is a valid governmental objective.
PT&T v. NLRC (IYA) The Labor arbiter decided that de Guzman already gained he statues of
May 23, 1997 | Regalado, J. | Equal Protection Clause a regular employee and was illegally dismissed by PT&T.
On appeal, the NLRC upheld the decision of the LA saying that de
Guzman was subjected to unjust and unlawful discrimination by her
PETITIONER: Philippine Telegraph and Telephone Company
employer.
RESPONDENTS: National Labor Relations Commission and Grace de MR was denied by the NLRC. Hence, this special civil action assailing
Guzman the
aforestated decisions.
SUMMARY: de Guzman was employed by PT&T as reliever several time ISSUE/s:
and as a probationary employee. In her application she concealed her WON policy of not hiring married women is valid - NO
civil status as a married woman. Upon knowledge by the company of
ther true civil status, she was dismissed from her position. PT&T has a
RULING: The petition of PT&T is dismissed for lack of merit.
policy against employing married women. She filed with the Labor Arbiter
that she was illegally dismissed, contesting the said company policy.
RATIO:
The LA and NLRC ruled in favor of de Guzman. On appeal to the SC, the The Constitution recognizes the disparity in rights between men and
policy is declared as unlawful and discriminatory as it is directed to women in almost all phases of social and political life. It therefore
married women only. contains several provisions which cater specifically to the protection
of womens’ rights.
DOCTRINE: PT&T’s policy of not accepting or considering as Section 14, Article II recognizing the role do women in nation
disqualified from work any woman worker who contracts marriage runs building
afoul of the test of and the right against discrimination afforded all Section 3, Article XIII requires the State to afford full protection to
women workers by the labor laws and the Constitution. labor and equality of employment opportunities for all
Section 14, Article XIII mandates that the State shall protect
working women
FACTS: Several Repulic Acts were also enacted to promote gender equality in
Grace de Guzman was initially hired by PT&T as a reliever for a fixed labor.
period in substitution for CF Tenorio who went on maternity leave. RA 6727 which prohibits discrimination against women with
Under the Reliever Agreement she signed, her employment was o be respect to terms and conditions of employment
immediately terminated upon expiration of the agreed period. RA 6955 which bands the mail order bride practice
After the lapse, she was again hired as reliever this time in replacement RA 7192 or the Women Development and Nation Building Act
of Erlinda Dizon who went on leave. which affords equal opportunities with men to act and to
de Guzman was once more asked to work for PT&T, this time however, enter into contracts.
as probationary employee to cover 150 days. RA 7322 increasing the maternity benefits granted to women
In the job application form she indicated that her civil status was single RA 7877 which outlaws and punishes sexual harassment in the
despite the fact that she contracted marriage a few months earlier. workplace
When PT&T knew about the false information, the branch supervisor, The Labor code specifically Article 136 explicitly prohibits discrimination
Delia Oficial, sent to de Guzman a memorandum requiring her to merely by reason of the marriage of female employees.
explain the discrepancy and reminded her that the company has a An employer is free to regulate, according to his discretion and best
policy of not accepting married women for employment. business judgment, all aspect so employment except in cases of
de Guzman stated that sh was not aware of the policy regarding married unlawful discrimination or those which may be provided by law.
women at the time and that all along she had not deliberately hidden PT&T’s policy of not accepting or considering as disqualified from work
her true civil status. any woman worker who contracts marriage runs afoul of the test of
She was dismissed from the company, which resulted in the filing of a and the right against discrimination afforded all women workers by
complaint for illegal dismissal before the Labor Arbiter. the labor laws and the Constitution.
At the preliminary conference, she volunteered the information that she de Guzman’s act of concealing the true nature of her status from PT&T
had failed to remit the amount of P2,380.75 of her collections. To could not be properly characterizes a willful or in bad faith as she
which she executed a promissory note for the amount. was moved to act the way she did mainly because he wanted to
retain a permanent job in ta stable company.
The collateral insistence of PT&T on the admission of de Guzman that
she misappropriated company funds as an additional ground to
dismiss her is
insincere. Their main reason for dismissing her was her marriage status.
PT&T’s rule is directed towards married women but not against married
men. The variable is sex and the discrimination is unlawful.
MIRASOL v. DPWH (ELIEL) ISSUE/s:
June 8, 2006 | Carpio, J | Equal Protection Clause WoN RTC’s decision is barred by Res Judicata – NO

PETITIONER: James Mirasol, Richard Santiago, Luzon Motorcyclists Fed.


RESPONDENTS: DPWH and TRB

SUMMARY: The DPWH issued several Orders and Rules on Limited Access
Facilities (toll ways) and issued AO 1, which prohibits motorcycles on such
roads. Mirasol et al filed a complaint seeking to nullify the orders and applied
for a writ of injunction because it is against RA 2000 and it violates the right
their constitutional rights. The trial court granted the injunction, however,
another DO was issued by DPWH but the trail court (different judge)
dismissed Mirasol et als petition. Hence this appeal. The SC ruled out that (1)
the preliminary injunction did not tantamount to res adjudicate because there
was no final injunction; (2) the DPWH has no authority under EO 546 to
regulate, restrict, and prohibit rules and regulations on limited access
facilities, and that such authority is vested with the DOTC [DO 74, 215, 123,
TRB regulation]; and that (3) AO 1 is valid because not only does it benefit
from the presumption of constitutionality, it is also a valid exercise of police
power. The toll ways are a special road, which is grounded on public interest
and safety, therefore, government may regulate. There is a reasonable
classification for motorcyles and 4 wheeled vehicles, because the latter are
more stable and safer. Right of travel is not infringed because what is
important is that there are alternative access roads. And that the license only
permits a person to drive a certain vehicle and not a permit to drive
anywhere.

DOCTRINE: A classification can only be assailed if it is deemed


invidious, that is, it is not based on real or substantial differences.

FACTS:
Mirasol et. al filed before the trail court a Petition for Declaratory Judgment
with Application for TRO and Injunction to nullify the following issuances
for being inconstitent with RA 2000, “Limited Access Highway Act”: (a)
AO 1; DO 74; and Art II, Sec 3a of Revised Rules on Limited Access
Facilities
Pursuant to its mandate under RA 2000, DPWH issued DO 215 declaring
the Manila-Cavite Toll Expressway as limited access facilities.
Mirasol filed an amendate petition seeking to nullify aforesaid issuances.
Trial Court, after due hearing, issued an ordering granting Mirasol’s
application for preliminary injunction.
The DPWH acting thru the TRB, issued DO 123 allowing motorcycles
with engine displacement of 400 cubic cm inside limited access
facilities (toll ways)
Trial court issued the assailed decision dismissing the petition but
declaring invalid DO 123. Mirasol filed MR, but denied. Hence this
appeal.
WoN DO 74, DO 215 and the TRB regulations contravene RA 2000 – AO 1 does not impose unreasonable restrictions. It merely outlines
YES several precautionary measures, to which toll way users must
WoN A01 and DO 123 are unconstitutional – NO/YES adhere. These rules were deisgned to ensure public csafety and
the uninhibited
RULING: Party Granted. DOLE decision reversed.

RATIO:
A preliminary injunction does not serve as a final determination of the
issues. It is a provisional remedy, which merely serves to preserve
the staus qup until the court could hear the merits of the case.
Rules on Civil Proceudre requires the issuance of a final injunction to
confirm the preliminary injunction should the court during trial
determine that the acts complaine of deserve to be permanently
enjoined.
DPWH issued DO 74 declaring Balintawak to Tabang and Nichols to
Alabang as Limited Access Facilities. DO 215 declaring Seaside
drive to Zapote, Zapote to Noveleta as Limited Access Facilities. And
TRB regulations, which ban motorcycles’ entry or access to the
limited access facilities.
Under Sec 1 of EO 546, the Ministry of Public Works (now DPWH) assumed
the public works functions of the Ministry of Public Works, Transporation
and Communications. On the other hand, among the functions of the
Ministry of Transportation and Communications (now DOTC) were (1) to
formulate and recommend national policies and guidelines for the
preparation and implementation of an integrated and comprehensive
transporation and communications systems at the national, regional and
local levels; and (2) regulate, whenever necessary, activities relative to
transporation and communications and prescribe and collect fees in the
exercise of such power.
Clearly, under EO 546, it is the DOTC, not the DPWH, which has
authority to regulate, restrict, or prohibit access to limited access
facilities.
Thus, DO 74 and DO 215 are void because the DPWH has no authority
to declare certain expressways as limited access facilities. Under the
law, it is the DOTC which is authorized to administer and enforce all
laws, rules and regulations in the field of tranposration and to
regulate related activities.
DPWH issued DO 123 Revising the rules on limited access facilities, but
as stated above, DPWH has no authority to regulate such.
On the constitutionality of AO 1 (prohibits motorcylcles on limited
access highways, administrative issuances have the force and
effect of law. They benefit from the same presumption of validity
and constitutionality enjoyed by statutes.
The use of public highways by motor vehicles is subject to regulation
as an exercise of the police power of the state. The polaching ins
cope and is the “most essential, insistent and illimitable” of all
government powers. The tendency is to extend rather than to
restrict the use of police power. The sole standard in measuring its
exercise is reasonableness.
flow of traffic within limited access facilities. evolved from its predecessors, the BPH, DPH, MPH and MPWH, I
The purpose of these rules and the logic behind them are quite submit, supports my view that it is the DPWH, and not the DOTC,
evident. A toll way is not an ordinary road. The special purpose which has inherited the functions previously exercised by the BPH,
for which a toll way is constructed necessitates the imposition including those granted by the Limited Access Highway Act.
of guidleines in the manner of its use and operation. Inevitably,
such rules will restrict certain rights. But the mere fact that
certain rights are restricted does not invalidate the rules.
The exercise of police power involves restriction, restriction being
implicit in the power itself. Thus, the test of constitutionality of
a police power measure is limited to an inquiry on whether the
restriction imposed on constitutional rights is reasonable, and
not whether it imposes a restriction on those rights.
Mirasol and others are not being deprived of their right to use the
limited access facility. There are merely being required, just like
the rest of the public, to adhere to the rules on how to use the
facility.
A police power measure may be assailed upon proof the it unduly
violates constitutional limitations like due process and equal
protection of the law.
A classification can only be assailed if it is deemed invidious, that
is, it is not based on real or substantial differences.
The SC finds that it is neither warranted nor reasonable for Mirasol
et al to say that the only justifiable classification among modes
of transport is the motorized against the non-motorized. Not all
motorized vehicles are created equal.
The SC finds that real and substantial differences exist between a
motorcycle and other forms of transport sufficient to justify its
classification among those prohibited from plying the toll ways.
The most obvious and troublic difference would be that a two-
wheeled vehicle is less stable and more easily overturned than
a four-wheeled vehicle.
A classification based on practical convenience and common
knowledge is not unconstitutional simple because it may lack
purely theoretical or scientific uniformity.
A toll way is not an ordinary road. As as facility designed to promote the
fastest access to certain destinations, its use, operation, and
maintenance require close regulation.
Public interest and safety require the imposition of certain restrictions on
toll ways that do not apply to ordinary roads. As a special kind of
road, it is but reasonable that not all forms of transport could use it.
The right to travel does not mean the right to choose any vehicle in
traversing a toll way. The right to travel refers to the right to move
from one place to another.
A driver’s license issued by the LTO merely allows one to drive a
particular mode of transport. It is not a license to drive or operate any
form of transportation on any type of road.
Tinga, J. Dissenting: The foregoing history of the DPWH, which has
PARRENO v. COA (JP) 101. Petitioner claims COA should have authority because his monthly
June 7, 2007 | Carpio, J. | Equal Protection Clause pension

PETITIONER: 2nd Lt. Salvador Parreno


RESPONDENTS: Commission on Audit and Chief of Staff, Armed Forces
of the Philippines

SUMMARY: Petitioner Parreno is a retird 2nd lieutenant of the AFP. The


AFP however stopped petitioner’s monthly pension in accordance with
Sec. 27 of PD 1638 because petitioner migrated to Hawaii and became a
naturalized American citizen. PD 1638 terminates the retirement benefits
of retired military personnel who loses their Filipino citizenship. COA
denied the petitioner’s request for the continuation of his monthly pension
due lack of jurisdiction since the disupute involves the constitutionality of
the law. SC affirms and explains that the law does not violate equal
protection because there is substantial distinction between those retired
AFP military personnel who lost their citizenship and those who remained
Filipinos. Those who renounced their citizenship cannot anymore be
compelled by the state to render compulsory military service.

DOCTRINE: If the groupings are characterized by substantial distinctions


that make real differences, one class may be treated and regulated
differently from another.

FACTS:
Salvador Parreño (petitioner) served in the Armed Forces of the
Philippines for 32 years. Petitioner retired from the Philippine
Constabulary with the rank of 2nd Lieutenant.
Petitioner availed and received payment, of a lump sum pension
equivalent to three years pay. The monthly pension amounted to
P13,680.
Petitioner migrated to Hawaii and became a naturalized American citizen.
The AFP stopped petitoner’s monthly pension in accordance with Section
27 of PD 1638 as amended by PD 1650.
PD 1638: Section 27. Military personnel retired under Sections 4, 5, 10,
11 and 12 shall be carried in the retired list of the Armed Forces of
the Philippines. The name of a retiree who loses his Filipino
citizenship shall be removed from the retired list and his retirement
benefits terminated upon such loss.
Petitioner requested for reconsideration but the Judge Advocate General
of the AFP denied the request. Petitioner filed a claim before the
COA for the
continuation of his monthly pension.
100. COA denied the petitioner’s claim for lack of jurisdiction, ruling that
the issue hinges upon the validity of Sec. 27 of PD 1638. Pursuant to
the constitution, whenever disputes involve the validity of laws, the
courts must have the inherent authority.
involves government funds. COA denied the motion. Hence, the their Filipino citizenship. If the groupings are characterized by
petition before this court. substantial distinctions that make real differences, one class may be
treated and regulated differently from another. (Note: this is the only
ISSUE/s:
extent the case discussed the
WoN Sec. 27 of PD 1638 is constitutional and not violative of the equal
protection clause - YES

RULING: Petition is dismissed. COA resolution is affirmed.

RATIO:
The OSG agrees with petitioner that Section 27 of PD 1638, as
amended, is unconstitutional. The OSG argues that the obligation
imposed on petitioner to retain his Filipino citizenship as a condition
for him to remain in the AFP retired list and receive his retirement
benefit is contrary to public policy and welfare, oppressive,
discriminatory, and violative of the due process clause of the
Constitution.
The OSG argues that the retirement law is in the nature of a contract
between the government and its employees. The OSG further argues
that Section 27 of PD 1638, as amended, discriminates against AFP
retirees who have changed their nationality.
The SC disagrees with the OSG.
115. The constitutional right to equal protection of the laws is not
absolute but is subject to reasonable classification.[18] To be
reasonable, the classification (a) must be based on substantial
distinctions which make real differences; (b) must be germane
to the purpose of the law; (c) must not be limited to existing
conditions only; and (d) must apply equally to each member of
the class
There is compliance with all these conditions. There is a substantial
difference between retirees who are citizens of the Philippines and
retirees
who lost their Filipino citizenship by naturalization in another country,
such as petitioner in the case before us.
117. The constitutional right of the state to require all citizens to
render personal and military service necessarily includes not
only private citizens but also citizens who have retired from
military service.
118. A retiree who had lost his Filipino citizenship already
renounced his allegiance to the state. Thus, he may no longer
be compelled by the state to render compulsory military service
when the need arises.
Petitioner's loss of Filipino citizenship constitutes a substantial
distinction that distinguishes him from other retirees who retain
Equal Protection Clause. The other elements were not discussed.)
Section 11 of RA 7077 provides that citizen soldiers or reservists include
ex-servicemen and retired officers (such as the petitioner) of the AFP.
Hence, even when a retiree is no longer in the active service, he is
still a part of the Citizen Armed Forces. Thus, we do not find the
requirement imposed by Section 27 of PD 1638, as amended,
oppressive, discriminatory, or contrary to public policy.
The state has the right to impose a reasonable condition that is
necessary for national defense. To rule otherwise would be
detrimental to the interest of the state.
Petitioner will be entitled to receive his monthly pension should he
reacquire his Filipino citizenship since he will again be entitled to the
benefits and privileges of Filipino citizenship reckoned from the time
of his reacquisition of Filipino citizenship. There is no legal obstacle
to the resumption of his retirement benefits from the time he
complies again with the condition of the law, that is, he can receive
his retirement benefits provided he is a Filipino citizen.
YRASUEGUI v. PAL (HENRY) promised as well to personally report to PAL on his weight check
October 17, 2008 | Reyes, R.T., J. | Equal Protection Clause sessions.

PETITIONER: Armando G. Yrasuegui


RESPONDENTS: Philippine Airlines, Inc.

SUMMARY: Petitioner Yrasegui was a former international flight steward


of PAL, who remained consistently overweight, despite PAL being lenient
to him for over 5 years (composed of frequent weight checks). Due to his
continuous noncomopliance then of PAL’s company policy on weight
standards, they dismissed Yrasegui, who then appealed to the LA. LA
ruled in his favor, ruling that he was illegally dismissed. PAL appealed to
NLRC, who affirmed the same. PAL appealed to the CA, who reversed
the decision, which prompted petitioner to appeal to SC, who affirmed the
CA decision, ruling that the company policy was not violative of his
constitutional right to due process in lieu of the absence of government
interference, and hence he was legally dismissed.

DOCTRINE: In the absence of governmental inference, the liberties


guaranteed by the Constitution cannot be invoked.

FACTS:
Petitioner Armando Yrasegui was a former international flight steward of
PAL. He stands 5’8” with a large body frame. The proper weight for a
man of his height and body structure is from 147 to 166 lbs (ideal
weight: 166 lbs), as mandated by the Cabin Crew Administration
Manual of PAL.
The weight problem of petitioner dates back to 1984, when PAL advised
him to go to an extended Vacation Leave (VL) to address his weight
concerns, which he failed to meet—prompting another Leave
Without Pay (LWOP) from March-November 1985.
After meeting the required weight, he was allowed to return to work, but
the weight problem reoccurred, which led him to go for another
LWOP from October 1988 to February 1989.
April 1989, petitioner weighted 209 lbs. In line with company policy, he
was removed from flight duty effective May 1989 to July 3 1989. He
was formally requested to trim down to ideal weight and report for
weight checks on several dates. He was offered to avail of the
services of the company physician.
February 1989 he went on weight check which had a whooping result of
215 lbs, 49 lbs beyond the limit. His off-duty status was retained.
October 1989, PAL Line Administrator personally visited petitioner’s
residence to check on him, and discovered that we weighed 217 lbs
(2lbs more than previous weight checks).
He then made a written commitment that he will lose weight from 217-
200 lbs until Dec 31, 1989, and that he will continue reducing at a
reasonable percentage until his ideal weight is achieved. He
Despite the above commitment, petitioner remained overweight. January
1990 PAL issued a decision for him to remain grounded until he
complies with weight standards (he was then directed to report every
2 weeks for the weight checks—which he failed to comply, even with
PAL’s extension of another month for the compliance).
He was seen to submit his passport for processing at PAL Staff Service
Division despite not going to PAL’s doctor for weight checks.
April 1990, petitioner was formally warned that repeated refusal to report
will be dealt with accordingly. He was given another set of weight
check dates, to which he ignored. July 1990 (212 lbs), August 1992
(219 lbs), November 1992 (205 lbs).
November 1992, petitioner was finally served by PAL of a Notice of
Administrative Charge for violation of company standards on weight
requirements, to which he was given 10 days to respond.
As defense petitioner said he was being treated unfairly as to other
employees who had the same situation. Eventually he was
dismissed, effective immediately, due to failure to comply with the
policy even if he was given a period of leniency for almost 5 years,
on June 1993. The MR he filed was denied.
He went to the LA, who ruled that he was illegally dismissed, that despite
the weight standards being reasonable, it must not be complied with
under pain of dismissal since his weight did not hamper his
performance.
PAL appealed to NLRC, which affirmed the LA’s decision, ruled that
obesity being a disease, shows unintentional defiance or serious
misconduct by the petitioner.
PAL appealed to the CA, who reversed the NLRC decision, ruling that
the relevant question in this case is the reasonableness of PAL’s
standard and hence, petitioner was legally dismissed.
Petitioner appealed to the CA, but was denied. Hence, this petition.

ISSUE/s:
WoN PAL’s company policy was violative of the petitioner’s constitutional
right to equal protection – NO
WoN petitioner was illegally dismissed - NO

RULING: SC affirmed the CA decision, but modified in such a manner that


entitled petitioner to a separation pay in an amount equivalent to ½’s pay for
every year of service, which should include his regular allowances.

RATIO:
Petitioner calims that PAL is using passenger safety as a convenient
excse to discriminate against him, to which the SC held otherwise.
SC upheld the CA decision, that “the element of discrimination came
into play in this case as a secondary position for the private
respondent in order to escape the consequence of dismissal that
being overweight entailed. It is a confession-and-avoidance position
that impliedly admitted the cause of dismissal,
including the reasonableness of the applicable standard and the private
respondent’s failure to comply.”
There is nothing in the records which could support the finding of
discriminatory treatment. Petitioner cannot establish discrimination
by simply naming the supposed cabin attendants who are allegedly
similarly situated with him.
Substantial proof mst be shown as to how they are similarly situated and
the differential treatment petitioner got from PAL despite the similarly
of his situation with other employees.
In the petitioner’s pointing of those “overweight”, he failed to indicate their
respective weights, period allowed to fly despite such situation,
particular flight, discriminating treatment, and other relevant data.
Although LA and NLRC are experts within their field (which is, labor), this
principle is not a hard and fast rule, as it is only applicable when
supported by substantial evidence. And since in this case both have
misappreciated evidence, their findings must be annulled.
In the claims with regard to violation of equal protection clause of the
Constitution, the SC ruled that in the absence of governmental
inference, the liberties guaranteed by the Constitution cannot
be invoked.
The Bill of Rights is not menat to be invoked against acts of private
individuals. US SC in interpreting the 14th amendment is consistent
in saying that EQP erects no shield against private conduct however
discriminatory or wrongful, nomatter how egregious, cannot violate
the equal protection guarantee.
SERRANO v. GALLANT (DANNAH) Serrano was hired by respondent Gallant and Marlow under a POEA-
March 24, 2009 | Austria-Martinez, J. | Equal Protection Clause approved Contract of Employment. It provided the following:

PETITIONER: Antonio Serrano


RESPONDENTS: Gallant Maritime Services Inc. & Marlow Navigation

SUMMARY: Serrano was hired by respondent companies, in which his


contract of employment stated that his basic monthly salary would be
$1,400. Before his departure, however, he was constrained to accept a
downgrade to a monthly salary of $1k, but he was promised he would be
promoted to Chief Officer by April 1998.

Respondent companies did not deliver, which prompted Serrano to refuse


staying. He was repatriated to the Philippines. His contract was for a period
of 12 months but he only got to serve 2. The Labor Arbiter awarded
monetary benefits in the amount of $8k representing Serrano’s salary for 3
months of the remaining portion. Both parties appealed, and the NLRC
changed the award to $4k.

Serrano raied the question of constitutionality when he appealed to the CA


which the CA did not rule on. Hence, he filed a petition with the SC. The SC
ruled that the portion of RA 8042 which states that those who have a
contract of 1 year or more, in case of illegal dismissal, are entitled to only 3
months for every year of the unexpired term. They struck it down for being
violative of the equal protection clause, because there is no substantial
distinction between them, other OFWs and local workers. It was ruled to be
a suspect classification.

DOCTRINE: When laws contain a suspect classification, the Court subjects


the classification to a strict judicial scrutiny, and determines whether it
serves a compelling state interest through the least restrictive means.

FACTS:
The last clause of 5th paragraph of Section 10, RA 8042 claims
Money claims—In cae of termination of overseas employment without
just, valid or authorized cause as defined by law or contract, the
workers shall be entitled to the full reimbursement of his placement
fee with interest of 12% per annum, plus his salaries for the
unexpired portion of his employment contract or for 3 months for
every year of the unexpired term, whichever is less.
Antonio Serrano, a Filipino seafarer, believes that this provision is
violative of the OFWs constitutional rights in that it impairs the terms
of their contract, deprives them of equal protection and denies them
due process.
By way of Petition for Review under Rule 45, Serrano assails The 2004
Decision and 2005 Resolution of the Court of Appeals, which applied
said provision, entreating the Court to declare it unconstitutional.
0 Duration of contract – 12 months
1 Position – Chief Officer
2 Basic monthly salary - $1,400
3 Hours of work – 48/week
4 Overtime - $700
5 Vacation leave with pay – 7 days/month
On March 1998, the date of his departure, petitioner was constrained to
accept a downgraded employment contract for the position of
Second Officer with a monthly salary of $1k, upon the assurance that
he would be made Chief Officer by end of April 1998.
Respondent companies did not deliver on their promise to make
Serrrano Chief Officer, hence he refused to stay on as Second
Officer and was repatriated to the Philippines
Serrano’s contract was for a period of 12 months but at the time of his
repatriation, he had served only 2 months and 7 days of his contract,
leaving an expired portion of 9 months and 23 days.
Serrano filed with the Labor Arbiter (LA) a Complaint against respondent
companies for constructive dismissal and for payment of his money
claims in the total amount of $26k, as well as damages.
The LA rendered a Decision dated July 1999 declaring the dismissal of
petitioner illegal and awarding him monetary benefits in amount of
$8,770 representing Serrano’s salary for 3 months of the unexpired
portion of the contract.
In the award, the LA based his computation on the salary period of three
months only—rather than the whole unexpired portion of 9 months+.
Respondent companies appealed to the NLRC on finding that the LA
was illegally dismissed.
Serrano also appealed on the ground that in the ruling of the Court in
Triple v. NLRC, in cases of illegal dismissal, OFWs are entitled to
their salaries for the unexpired portion of their contracts.
The NLRC rendered a decision awarding the amount of $4,669. It
corrected the LA’s computation of lump-sum salary because RA 8042
“does not provide for the award of overtime pay….”
Serrano filed a Motion for Partial Reconsideration, this time questioning
constitutionality of said clause but the NLRC denied the motion. After
dismissing the petition on technicality, the CA eventually granted the
petition for certiorari.
The CA affirmed the NLRC ruling on the reduction of salary, but skirted
the constitutional issue raised by Serrano. The MR having been
denied, Serrano now brings this case to the Court.
On February 2008 however, Serrano withdrew his petition as he was
already old and sickly, and wanted to make use of the monetary
award for his medical treatment.

ISSUE/s:
WoN the 5th paragraph of Section 10, RA 8042 is unconstitutional for
being
violative of the due process clause – YES Our present Constitution has gone further in guaranteeing vital
WoN it’s unconstitutional for being violative of the non-impairment clause? social and economic rights to marginalized groups of society,
– NO
including labor. Under
WoN the question of constitutionality should not be sustained because it
was not raised at the earliest opportunity – NO

RULING:. The subject clause is declared UNCONSTITUTIONAL. The


resolution of the CA is modified. Award is salaries for the entire unexpired
portion; 9 months and 23 days computed at $1,400/mo.

RATIO:
Issue #3: There exists an actual controversy. The constitutional
challenge is timely. It is deemed seasonably raised because it is not
the NLRC but the CA which has the competence to resolve the
constitutional issue.
Issue #2: “No law impairing the obligation of the contracts shall be passed.”
The non-impairment clause is limited in application to laws about to be
enacted that would in any way derogate from existing acts or contracts
by enlarging, abridging or in any manner changing the intention of the
parties.
The enactment of RA 8042 in 1995 preceded the excution of the
employment contract between Serrano and repsondent companies.
It cannot be argued that said RA impaired the employment contract
of the parties.
Rather, when the parties executed their 1998 employment contract, they
were deemed to have incorporated into it all the provisions of said
statute.
Moreover, it was enacted in the exercise of police power of the State.
Issue #1: There are three levels of scrutiny at which the court reviews
the constitutionality of a classification embodied in a law
Deferential or rational basis scrutiny – The challenged
classification needs only be shown to be rationally related to
serving a legitimate state interest
Middle-tier or intermediate scrutiny – The government must
show that the challenged classification serves an important
state interest and the clasisfication is at least substantially
related to serving that interest
Strict judicial scrutiny – A legislative classification which
imperssibly interferes with the exercise of a fundamental
right, or operates to the peculiar disadvantage of a suspect
class is presumed unconstitutional, and the burden is upon
the government to prove that the classification is necessary
to achieve a compelling state interest and that it is the
least restrictive means to protect such interest
the policy of social justice, the law bends over backward to accommodate
the interests of the working class on the humane justification that those with
less privilege in life should have more in law.
The Court in the present case, imbued with the same sense of “obligation to
afford protection to labor” employs the standard of strict judicial scrutiny
(such as in the CBEA case), for it perceives in the subject clause a
suspect classification prejudicial to OFWs.
In the case of Marsaman v. NLRC, Section 10(5) of RA 804 states that the
choice of which amount to award an illegally dismissed overseas contract
worker comes into play only when the employment contract concerned has a
term of at least one year or more.
This is evident from the words “for every year of the unexpired term” which
follows the words “salaries … for three months”
In said case, the OFW involved was illegally dismissed 2 months into his
10month contract but was awarded his salaries for the remaining 8 months.
In the trend of previous jurisprudence, it can be seen that the subject clause
classifies OFWs into two categories
First category – OFWs with fixed-period employment contracts of less
than one year; in case of illegal dismissal they are entitled to their
salaries for the entire unexpired portion of their contract
Second category – OFWs with fixed-period employment contracts of
one year or more; in case of illegal dismissal they are entitled to
monetary award equivalent to only 3 months of the unexpired portion
of their contracts
The disparity of the treatment of the two groups cannot be discounted. It
becomes more aggravating when the Court takes into account jurisprudence
that, prior to the effectivity of RA 8042, illegally dismissed OFWs, no matter
how long the period of employment of contracts, were entitled to their
salaries for the entire unexpired portions of their contracts.
The clause creates a sublayer of discrcimination among OFWs whose contract
periods are for more than one year: those who are illegally dismissed with less
than one year left in their contracts shall be entitled to their salaries for the entire
unexpired portion, while those who are illegally dismissed with one year or more
remaining in their contracts shall be covered by the subject clause, and monetary
beneftis are limited to their salaries for 3 months only.
Prior to RA 8042, OFWs and local workers with fixed-term employment who were
illegally discharged were treated alike in computation of their money claims:
they were uniformly entitled to their salaries for the entire unexpired
portions of their contracts.
But with the enactment of said RA, those who belong to the second category
have been treated differently in that their money claims are subject to a 3-
month cap, whereas no such limitation is imposed on local workers.
The subject clause contains a suspect classification in that, in the
computation of the monetary benefits of fixed-term employees who
are illegally discharged, it imposes a 3-month cap on their claim, but
none on the claims of other OFWs or local workers with fixed-term
employment.
The subject clause singles out one classification of OFWs and burdens it
with a peculiar disadvantage.
There being a suspect classification, the Court now subjects the
classification to a strict judicial scrutiny, and determins whether it
serves a compelling state interest through the least restrictive
means.
What constitutes tate interest is measured by the scale of rights
and powers arrayed in the Constitution and calibrated by
history.
In the present case, there is no compelling state interest that the subject
clause may possibly serve.
The Government has failed to discharge its burden of proving the
existence of a compelling state interest that would justify the
perpetuation of the discrimination against OFWs under the subject
clause.
There can never be a justification for any form of government action that
alleviates the burden of one sector, but imposes the same on
another, especially when the favored sector is composed of private
businesses such as placement agancies, while the disadvantaged
sector is composed of OFWs whose protection no less than the
Constitution commands.
Thus, the subject clause is violative of the right of petitioner and other
OFWs to equal protection.
Moreover, it violates Serrano’s right to substantive due process, for it
deprives him of property, consisting of monetary benefits, without
existing avalid governmental purpose.
Carpio, Concurring: The clause is unconstitutional on a different ground. It
violates the prohibition against deprivation of property without due
process of law. It is an invalid exercise of police power. It is unduly
oppressive, unreasonable and repugnant to the Constitution. It
undermines the mandate to protect the rights of overseas workers and to
promote their welfare.
Brion, Concurring: The provision should be struck down for violations of
the constitutional provisions in favor of labor and of the substantive
aspect of the due process clause, the issue on equal protection (or
bringing it up) was not necessary.
BRITISH AMERICAN TOBACCO v. CAMACHO (IYA) Revenue Memorandum order No. 6-2003 was issued prescribing the
August 20, 2006 | Ynares-Santiago, J. | Equal Protection Clause guidelines and procedures in establishing current net retail prices of
new
PETITIONER: British American Tobacco
RESPONDENTS: Jose Isidro Camacho, in his capacity as Secretary of
Department of Finance and Guillermo Parayno Jr, in his capacity as
Commissioner of the Bureau of Internal Revenue

SUMMARY: DEPUTA x1000. Ok so gabito kasi.


British American Tobacco, producer of Lucky Strike cigarettes, assails
Section 145 of the NIRC and its implementing regulations. The law and
regulations impose different excise tax rates on older brands and newer
brands. To British American, this is violative of the equal protection
clause and uniformity of taxation clause.

SC rules upholding the constitutionality of the laws. It made mention that


British American was not able to prove or show how it has affected the
overall competition in the market.

DOCTRINE: The legislative classification under the classification freeze


provision, after having been shown to be rationally related to achieve
certain legitimate state interests and done in good faith, must, perforce,
end the inquiry of the SC.

FACTS:
Present case assails the validity of the ff as violative of the equal
protection clause of the Constitution:
Section 145 of the National Internal Revenue Code as rectified
by RA 8424
RA 9334 which further amended Section 145 of the NIRC
Revenue Regulations Nos. 1-97, 9-2003 and 22-2003
Revenue memorandum Order No. 6-2003
Paragraph (c) of Section 145 provides for four tiers of tax rates based on
the net retail price pr pack of cigarettes.
RR No. 1-97 classified new brands shall be initially assessed at their
suggested retail price until such time that the appropriate survey to
determine their current net retail price is conducted.
British American Tobacco introduced into the market Lucky Strike Filter,
Lucky Strike Lights and Lucky Strike Menthol Lights cigarettes.
These brands were initially assessed the excise tax at P8.96 per
pack.
RR No. 9-2003 amended RR 1-97 by providing a periodic review every
two years or earlier of the current net retail price of new brands and
variants thereof for the purpose of establishing and updating their tax
classification.
brands of cigarettes and alcohol products. Section 4(B)(e)(c), 2nd paragraph of Revenue Regulations No. 1-97, as
RR No. 22-2003 was later issued. It was to implement the revised tax amended by Section 2 of Revenue Regulations 9-2003, and Sections
classification of certain new brands based on survey of their current II(1)(b), II(4)(b), II(6), II(7), III (Large Tax Payers Assistance Division II)
net retail price. II(b) of
The survey recommended that the applicable tax rate for Lucky Stike, et
al. is P13.44 per pack.
British American Tobacco then filed before the RTC of Makati a petition
for induction with prayer for the issuance of a TRO. It sought to
enjoin the implementation of the above stated laws and regulations
on the ground that they discriminate against new brands of cigarettes
in violation of the equal protection and uniformity provisions of the
Constitution.
Commissioner of Internal Revenue file dan Opposition. RTC denied the
TRO holding that courts have no authority to restrain the collection of
taxes.
Secretary of Finance file da Motion to Dismiss contending that the
petition is premature for lack of an actual controversy or urgent
necessity.
RTC denied the motion to dismiss and issued a writ of preliminary
injunction to enjoin the implementation of the above stated laws and
revenue regulations.
RTC rendered a decision upholding the constitutionality of Section 145 of
the NIRC and the various regulations. Writ of Preliminary Injunction
was also lifted.
British American Tobacco brought the instant petition or review with the SC.
Pending the case in the SC, RA 9344 was enacted which increased the
excise tax due on Lucky Strike cigarettes to P25.00 per pack.
There exists a classification freeze provision where older brands’ excise
tax is not increased, while those of newer brands were subjected to
increase.
Philip Morris along with other cigarette brands filed a Motion for Leave to
Intervene with the case. According to intervenors, no inequality exists
and that nullification of said rules would bring loss of revenue to the
government.
Intervenor Fortune Tobacco contends that petitioner is estopped form
questioning he constitutionality of Section 145 and its implementing
rules and regulations because it entered into the cigarette industry
fully aware of the existing tax system and its consequences.

ISSUE/s:
WON the laws and regulations violate the equal protection clause of the
Constitution - NO

RULING: WHEREFORE, the petition is PARTIALLY GRANTED and the


decision of the Regional Trial Court of Makati, Branch 61, in Civil Case No.
03-1032, is AFFIRMED with MODIFICATION. As modified, this Court
declares that:
Section 145 of the NIRC, as amended by Republic Act No. 9334, is
CONSTITUTIONAL; and that
Revenue Memorandum Order No. 6-2003, insofar as pertinent to cigarettes
packed by machine, are INVALID insofar as they grant the BIR the power
to reclassify or update the classification of new brands every two years or
earlier.

RATIO:
The British American generalizes that the differential tax treatment arising
form the classification freeze provision adversely impacts the
fairness of the playing field in the industry, particularly between older
and newer brands.
British American did not clearly demonstrates the exact extend of such
impact. It has not been shown that the retail prices of the older
brands previously classified under this classification system have
already pierced their tax brackets, and, if so, how this has affected
the overall competition in the market.
The legislative classification under the classification freeze provision, after
having been shown to be rationally related to achieve certain legitimate
state interests and done in good faith, must, perforce, end the inquiry of
the SC.
Absent a clear showing of breach of constitutional limitations, Congress,
owing to its vast experience and expertise in the field of taxation,
must be given sufficient leeway to formulate and experiment with
different tax systems to address the complex issues and problems
related to tax administration.
The Revenue Regulations and Revenue Memorandum is ruled to have
unjustifiable emasculated the operation of Section 145 of the NIRC
because they authorize the Commissioner of Internal Revenue to
update the tax classification of new brands every two years or earlier
only to its issuance of the appropriate Res, when nowhere in Section
145 is such authority granted to the Bureau. It was not the intent of
the Congress to give the BIR the authority to reclassify them every
so often.
PEOPLE v. SITON (ELIEL) The MTC also noted that in the affidavit of the arresting police officer, it was
September 18, 2009 | Ynares-Santiago, J. | Equal Protection Clause stated that there was a prior surveillance conducted on the two accused
in an
PETITIONER: People of the Philippines
RESPONDENTS: Evangeline Siton and Krystal Sagarano

SUMMARY: Siton and Sagarano were charged with vagrancy after


wandering and lotering around San Pedro and Legaspi Streets, without
any visible means to support themselves. Siton and Sagarano filed their
Motions to Quash with the MTC assailing Art 202(2) anti-vagrancy law as
unconstitutional. The MTC denied their motions and stated that Art 202(2)
is valid exercise of police power. Siton and Sagarano brought it up with
the RTC, which ruled in their favor and declared Art 202(2)
unconstitutional for being vague and a violation of equal protection.
Hence this appeal. The SC ruled in favor of Petitioners (People of the PH)
on the ground that (1) it is not vague because before a person may be
arrested under the assailed law, there is a need for probably cause as is
evident in the case; (2) it is a valid exercise of police power because the
State aims to curb the immorality and to protect the streets as it is
becoming all the more dangerous form robbers and thieves; (3) it does
not violate the equal protection clause, because what is being punished is
conducting themselves at a certain place and time which orderly society
finds unusual, under such conditions that are repugnant and outrageous
to the common standards and norms of decency.

DOCTRINE: Offenders of public order laws are punished not for their
status, as for being poor or unemployed, but for conducting themselves
under such circumstances as to endanger the public peace or cause
alarm and apprehension in the community

FACTS:
Evangeline Siton and Krystal Sagarano were charged with vagrancy
pursuant to Art 202 (2) of the RPC in two separate informations.
Art 202 (2): Any person found loitering about public or semi-public
buildings or places or tramping or wandering about the country or the
streeets without visible means of support;
Instead of submitting counter-affidavits as directed, Siton and Sagarano
filed Motions to Quash on the ground that Art 202(2) is
unconstitutional for being vague and overbroad.
The MTC denied the motions and directed Siton to file their respective
counter-affidavits. The MTC likewise declared tht the law on
vagrancy was enacted pursuant to the State’s police power and
justified by the Latin maxim “salus populi est suprema lex,” which
calls for the subordination of individual benefit to the interest of the
greater number.
area reported to be frequented by vagrants and prostitutes who The grounds of suspicion are reasonable when, in the absence of actual
solicited sexual favors. belief of the arresting officers, the suspicion that the person to be
Siton filed petition for certiorari and prohibition with the RTC, challenging arrested is
the constitutionality of the Anti-vagrancy law, claiming that it is vague
and overbreadth, and that it violates the equal protection clause
because it discriminates against the poor and unemployed, thus
permitting an arbitrary and unreasonable classification.
RTC granted the petition. Holding that the law is vague and vilated the
equal protection clause. Hence this appeal.

ISSUE/s:
WoN Art 202(2) of the RPC is unconstitutional –

NO RULING: Petition Ganted.

RATIO:
The power to define crimes and prescribe their corresponding penalties
is legislative in nature and inherent in the sovereign power of the
state to maintain social order as an aspect of police power. The
legislature may even forbid and penalize acts formerly considered
innocent and lawful provided that no constitutional rights have been
abridged.
However, in exercising its power to declare what acts constitute a crime,
the legislature must inform the citizen with reasonable precision what
acts it intends to prohibit so that he may have a certain
understandable rule of conduct and know what acts it is his duty to
avoid – void for vagueness doctirine.
Void for vagueness a statute which either forbids or requires the doing
of an act int erms so vague that men of common intelligence must
necessarily guess at its meaning and differ as to its application,
violats the first essential due process of law.
Under the Constitution, the people are guaranteed the right to be secure
in their persons, houses, papers and effecs against unreasonable
searches and seizure of whatever nature and for any purpose, and
no search warrant or warrant of arrest shall issue except upon
probably cause to be deteremined personablly by the judge after the
examination under oath or affirmation of the complainant and the
witnesses he may produce, and particularly describing the place to
be searched and the persons or things to be seized.
The requirement of probably cause cannot be done away with arbitrarily
without pain of punishment, for, absent this requriement, the
authorities are necessarily guilty of abuse.
probably guilty of committing the offense, is based on actual facts supported
by circumstacnes sufficiently stron in themselves to create the probably cause
of guilt of the perosn to be arrested.
Applied to the instant case, it appears that the police authorities have
been conducting previous surveillance operations on Sitons’ prior to
their arrest. On the surface, this satisfies the probably cause
requirement under our Constitution.
The court has witness that the streets and parks become dangerous and
unsafe, a haven for beggars, harrassing “watch your car” boys, petty
theives and robbers, pickpokects, swindlers, gangs, prostitutes, and
individuals performing acts that go beyond decency and morality, if
not basic humanity. The streets and parks have become the training
ground for petty offenders who graduate into hardened and battle-
scarred criminals.
The streets must be protected. Our people should never dread having to
ply them each day, or else we can never say that we have performed
our task to our brothers and sisters. We must rid the streets of the
scourge of humanity, and restor order, peace, civility, decency and
morality in them.
This is exactly why we have public order laws, to which Art 202(2)
belongs. These laws were crafted to maintain minimum standards of
decency morality and civility in human society.
Art 202(2) does not violate the equal protection clause; neither does
it discriminate agaisnt the poor and the unemployed.
Offenders of public order laws are punished not for their status, as
for being poor or unemployed, but for conducting themselves
under such circumstances as to endanger the public peace or
cause alarm and apprehension in the community.
Vagrance is a public order crime which punsihes persons for
conducting themselves, at a certain place and time which orderly
society finds unusual, under such conditions that are repugnant
and outrageous to the common standards and norms of decency
and morality in a just, civilized and ordered society, as would
engender a justifiable concern for the safety and well-being of
members of the community.
When confronted with a constitutional question, it is elementary that
every court must approach it with grave care and considereable
caution bearing in mind that every statute is presumed valid and
every reasonable doubt should be resolved in favor of its
consitutionality.
QUINTO v. COMELEC (JP) prohibition and certiorari, seeking the declaration of the afore-quoted
December 12, 2009 | Nachura, J. | Equal Protection Clause Section 4(a) of

PETITIONER: Eleazar P. Quinto, Gerino A. Tolentino, Jr.


RESPONDENTS: Commission on Elections
Note: The syllabus cites the MR case, not this one.

SUMMARY: Petitioners hold appointive positions in the government.


Based on RA No. 9369 which amended RA 8436, COMELEC enacted
Resolution No. 8678 which provides that public appointive officials shall
be considered ipso facto resigned from their office upon filing of their
certificates for candidacy. Wanting to run for the next elections,
petitioners are assailing the constitutionality of the provsisions of the law.
The SC held that the law does not pass the standards under the equal
protection clause. The distinctions between elective and appointive
officials are not germane to the purpose of the law. The law unduly
discriminates against appointive officials. Other officials such as elected
ones are not ipost facto resigned upon filing for their COCs.

DOCTRINE: The classification, even if based on substantial distinctions,


will still be invalid if it is not germane to the purpose of the law.

FACTS:
Congress enacted RA No. 8436: An Act Auhtorizing the COMELEC to
Use an Automated Election System in Subsequent National and
Local Electoral Exercises. Almost a decade thereafter, Congress
amended the law by enacting R.A. No. 9369.
Pursuant to its constitutional mandate to enforce and administer election
laws, COMELEC issued Resolution No. 8678, the Guidelines on the
Filing of Certificates of Candidacy (CoC) and Nomination of Official
Candidates of Registered Political Parties in Connection with the
May 10, 2010 National and Local Elections.
Sections 4 of Resolution No. 8678 provide:
SEC. 4. Effects of Filing Certificates of Candidacy.-a) Any
person holding a public appointive office or position including
active members of the Armed Forces of the Philippines, and
other officers and employees in government-owned or
controlled corporations,
shall be considered ipso facto resigned from his office
upon the filing of his certificate of candidacy.
Alarmed that they will be deemed ipso facto resigned from their offices the
moment they file their CoCs, petitioners Eleazar P. Quinto and Gerino A.
Tolentino, Jr., who hold appointive positions in the government and who
intend to run in the coming elections, filed the instant petition for
Resolution No. 8678 as null and void. officials hold their office in a permanent capacity and are entitled to
Petitioners (Quinto, Tolentino) contend that the COMELEC gravely abused security of tenure while others serve
its discretion when it issued the assailed Resolution. Petitioners assert
they should not be deemed ipso facto resigned from their government
offices when they file their CoCs, because at such time they are not yet
treated by law as candidates. They only become candidates at the start
of the campaign period.
Petitioners further posit that the provision considering them as ipso facto
resigned from office upon the filing of their CoCs is discriminatory
and violates the equal protection clause in the Constitution.
The OSG maintains that the COMELEC did not gravely abuse its
discretion in phrasing Section 4(a) of Resolution No. 8678 for it
merely copied what is in Sec. 13 of RA 9369.

ISSUE/s:
WoN the second proviso in the third paragraph of Section 13 of R.A. No.
9369, a reproduction of Section 66 of the OEC, is violative of the
equal protection clause - YES

RULING: Petition is granted. The second proviso of the third paragraph of


Sec. 13 of RA 9369 and Section 4(a) of COMELEC Resolution No. 8678 are
declared as unconstitutional.

RATIO:
Section 67 of the Omninbus Election Code and Section 11 of R.A. No.
8436 contained a similar provision on automatic resignation of
elective officials upon the filing of their CoCs for any office other than
that which they hold in a permanent capacity or for President or Vice-
President. However, with the enactment of R.A. No. 9006, or the Fair
Election Act, in 2001, this provision was repealed by Section 14 of
the said act. There was, thus, created a situation of obvious
discrimination against appointive officials who were deemed ipso
facto resigned from their offices upon the filing of their CoCs, while
elective officials were not.
This situation was incididentally addressed by the court in Fariñas v.
Executive Sec when it ruled that Sec. 14 of RA No. 9006 is not
violative of the equal protection clause.
Substantial distinctions clearly exist between elective officials and
appointive officials. The former occupy their office by virtue of the
mandate of the electorate. They are elected to an office for a definite
term and may be removed therefrom only upon stringent conditions.
On the other hand, appointive officials hold their office by virtue of
their designation thereto by an appointing authority. Some appointive
at the pleasure of the appointing authority. appointive office or an elective one, the evils sought to be
However, it must be remembered that the Court, in Fariñas, was intently prevented by the measure remain. With the fact that they both
focused on the main issue of whether the repealing clause in the Fair head executive offices, there is no valid justification to treat
Election Act was a constitutionally proscribed rider, in that it unwittingly them differently when both file their CoCs for
failed to ascertain with stricter scrutiny the impact of the retention of the
provision on automatic resignation of persons holding appointive
positions (Section 66) in the OEC, vis-a-vis the equal protection clause.
The Court's afore-quoted declaration in Fariñas may then very well be
considered as an obiter dictum.
The SC however adopts a different view. Petitioners' interest in
running for public office, an interest protected by Sections 4
and 8 of Article III of the Constitution, is breached by the
proviso in Section 13 of R.A. No. 9369. It is the opportune time
for the Court to strike down the said proviso for being violative
of the equal protection clause and for being overbroad.
In considering persons holding appointive positions as ipso facto
resigned from their posts upon the filing of their CoCs, but not
considering as resigned all other civil servants, specifically the
elective ones, the law unduly discriminates against the first
class. The fact alone that there is substantial distinction
between those who hold appointive positions and those
occupying elective posts, does not justify such differential
treatment.
In order that there can be valid classification so that a discriminatory
governmental act may pass the constitutional norm of equal protection, it
is necessary that the four (4) requisites of valid classification be
complied with.
Applying the four requisites to the instant case, the Court finds that
the differential treatment of persons holding appointive offices
as opposed to those holding elective ones is not germane to
the purposes of the law.
The obvious reason for the challenged provision is to prevent the use of a
governmental position to promote one's candidacy, or even to wield a
dangerous or coercive influence on the electorate. The measure is
further aimed at promoting the efficiency, integrity, and discipline of the
public service by eliminating the danger that the discharge of official duty
would be motivated by political considerations rather than the welfare of
the public. It also eliminates the neglect or inefficiency in the
performance of duty because they would be attending to their campaign
rather than their office work.
Glaringly absent is the requisite that the classification must be
germane to the purposes of the law. Indeed, whether one holds an
the elections. Under the present state of our law, the Vice-President,
in the example, running this time, let us say, for President, retains
his position during the entire election period and can still use the
resources of his office to support his campaign.
There is thus no valid justification to treat appointive officials
differently from the elective ones. The classification simply fails to
meet the test that it should be germane to the purposes of the law.
The measure encapsulated in the second proviso of the third
paragraph of Section 13 of R.A. No. 9369 and in Section 66 of the
OEC violates the equal protection clause.
The challenged provision also suffers from the infirmity of being overbroad:
the provision pertains to all civil servants holding appointive posts without
distinction as to whether they occupy high positions in government or not.
the provision is directed to the activity of seeking any and all public
offices, whether they be partisan or nonpartisan in character, whether
they be in the national, municipal or barangay level. Congress has not
shown a compelling state interest to restrict the fundamental right
involved on such a sweeping scale.
It may not be amiss to state that the Americans, from whom we copied the
provision in question, had already stricken down a similar measure for
being unconstitutional.
QUINTO v. COMELEC MR (JP) The assailed provisions do not violate the equal protection
February 22, 2010 | Puno, C. J. | Equal Protection Clause clause when they accord differential treatment to elective and
appointive officials, because such differential treatment rests on
PETITIONER: Eleazar P. Quinto, Gerino A. Tolentino, Jr. material and substantial distinctions and is germane to the
RESPONDENTS: Commission on Elections purposes of the law;
The assailed provisions do not suffer from the infirmity of
SUMMARY: This case is an MR forwarded by the COMELEC assailing
overbreadth
the 2009 decision which declared Sec. 4(a) of COMELEC Resolution
8678 as unconstitutional for violating the equal protection clause and for (4) There is a compelling need to reverse the assailed
overbreadth. Refer to 2009 case digest for the facts of the previous case. Decision, as public safety and interest demand such
The SC here reverses its 2009 decision explaining that the Fariñas ruling reversal.
must be upheld and that there are valid substantial distinctions between
elective officials and appointive officials, the former being chosen by the ISSUE/s:
sovereign will of the electorate. Hence, equal protection is not violated by WoN the assailed provisions violate the equal protection clause – NO
the assailed provisions and appointive officials should be ipso facto WoN the previous decision should be overruled - YES
declared as resigned from their public positions upon filing of their COCs.
RULING: The Court grants the motion for reconsideration. The
DOCTRINE: The equal protection of the law clause in the Constitution is December 2009 decision is reversed and set aside. Sec. 4(a) of COMELEC
not absolute, but is subject to reasonable classification. If the groupings Resolution No. 8678 and the 2 nd proviso in the third paragraph of Sec. 13 of
are characterized by substantial distinctions that make real differences, RA 9369 is not unconstitutional.
one class may be treated and regulated differently from the other.
RATIO:
The assailed Decision struck down Section 4(a) of Resolution 8678, the
second proviso in the third paragraph of Section 13 of Republic Act
FACTS:
(RA) 9369, and Section 66 of the Omnibus Election Code, on the
The assailed SC Decision granted the Petition for Certiorari and
following grounds:
Prohibition filed by Eleazar P. Quinto and Gerino A. Tolentino, Jr. and
They violate the equal protection clause of the Constitution
declared as unconstitutional the second proviso in the third
because of the differential treatment of persons holding
paragraph of Section 13 of Republic Act No. 9369, Section 66 of the
appointive offices and those holding elective positions;
Omnibus Election Code and Section 4(a) of COMELEC Resolution
They are overbroad insofar as they prohibit the candidacy of all
No. 8678, mainly on the ground that they violate the equal protection
civil servants holding appointive posts: (a) without distinction as
clause of the Constitution and suffer from overbreadth. (Important:
to whether or not they occupy high/influential positions in the
Refer to the 2009 original case for the facts. The facts were not
government, and (b) they limit these civil servants' activity
reiterated in this MR case.)
regardless of whether they be partisan or nonpartisan in
The assailed Decision thus paved the way for public appointive officials
character, or whether they be in the national, municipal or
to continue discharging the powers, prerogatives and functions of
barangay level; and
their office notwithstanding their entry into the political arena.
In support of their respective motions for reconsideration, respondent Congress has not shown a compelling state interest to
COMELEC and movants-intervenors submit the following arguments restrict the fundamental right of these public appointive
(only no. 2 is relevant): officials.
The court hereby reverses its previous decision. Section 4(a) of
The assailed Decision is contrary to, and/or violative of, the
COMELEC Resolution 8678 is a faithful reflection of the present state
constitutional proscription against the participation of public
of the law and jurisprudence on the matter.
appointive officials and members of the military in partisan
Incumbent Appointive Official. - Under Section 13 of RA 9369 (the assailed
political activity;
provision in the previous decision), which reiterates Section 66 of the
Omnibus Election Code, any person holding a public appointive
office or position, including active members of the Armed Forces of the
Philippines, and officers and employees in government-owned or
-controlled corporations, shall be considered ipso facto resigned
from his office upon
the filing of his certificate of candidacy. Some appointive officials hold their office in a permanent
Incumbent Elected Official. - Upon the other hand, pursuant to Section capacity and are entitled to security of tenure while others
14 of RA 9006 or the Fair Election Act, which repealed Section 67 serve at the pleasure of the appointing authority.
of the Omnibus Election Code and rendered ineffective Section 11 of Another substantial distinction between the two sets of officials is
R.A. 8436 insofar as it considered an elected official as resigned only that
upon the start of the campaign period corresponding to the positions
for which they are running, an elected official is not deemed to
have resigned from his office upon the filing of his certificate of
candidacy for the same or any other elected office or position.
In fine, an elected official may run for another position without
forfeiting his seat.
These laws and regulations implement Section 2(4), Article IX-B of the
1987 Constitution, which prohibits civil service officers and
employees from engaging in any electioneering or partisan political
campaign.
The constitutional ba does not cover elected officials because public officials,
by the very nature of their office, engage in partisan political activities
almost all year round, even outside of the campaign period. Political
partisanship is the inevitable essence of a political office, elective
positions included.
Fariñas, et al. v. Executive Secretary, et al. is controlling (in the original
2009 case, the court considered it obiter dictum and did not follow it.)
Fariñas, et al. v. Executive Secretary, et al: Section 67 of the Omninbus
Election Code and Section 11 of R.A. No. 8436 contained a similar
provision (as with the assailed provisions on 2009 case) on
automatic resignation of elective officials upon the filing of their CoCs
for any office other than that which they hold in a permanent capacity
or for President or Vice-President. However, with the enactment of
R.A. No. 9006, or the Fair Election Act, in 2001, this provision was
repealed by Section 14 of the said act. There was, thus, created a
situation of obvious discrimination against appointive officials who
were deemed ipso facto resigned from their offices upon the filing of
their CoCs, while elective officials were not.
The petitioners in Fariñas thus brought an equal protection challenge
against Section 14, with the end in view of having the deemed-
resigned provisions "apply equally" to both elected and appointive
officials.
Substantial distinctions clearly exist between elective officials and
appointive officials. The former occupy their office by virtue of the
mandate of the electorate. They are elected to an office for a
definite term and may be removed therefrom only upon stringent
conditions. On the other hand, appointive officials hold their office
by virtue of their designation thereto by an appointing authority.
under Section 55, Chapter 8, Title I, Subsection A. Civil Service
Commission, Book V of the Administrative Code of 1987 (Executive
Order No. 292), appointive officials, as officers and employees in
the civil service, are strictly prohibited from engaging in any
partisan political activity or take (sic) part in any election except to
vote. Under the same provision, elective officials, or officers or
employees holding political offices, are obviously expressly
allowed to take part in political and electoral activities.
Since the classification justifying Section 14 of Rep. Act No. 9006, i.e.,
elected officials vis-a-vis appointive officials, is anchored upon
material and significant distinctions and all the persons belonging
under the same classification are similarly treated, the equal
protection clause of the Constitution is, thus, not infringed.
The previous decision declared the assailed provisions as violative of the
EPC because the differential treatment of appointive officials with elected
officials is not germane to the purpose of the law.
Sad to state, this conclusion conveniently ignores the long-standing rule that
to remedy an injustice, the Legislature need not address every
manifestation of the evil at once; it may proceed "one step at a time.
The SC may not strike down a law merely because the legislative aim would
have been more fully achieved by expanding the class. Stated differently,
the fact that a legislative classification, by itself, is underinclusive will not
render it unconstitutionally arbitrary or invidious. There is no
constitutional requirement that regulation must reach each and every
class to which it might be applied;
Thus, any person who poses an equal protection challenge must
convincingly show that the law creates a classification that is "palpably
arbitrary or capricious. In the case at bar, the petitioners failed and in fact
did not attempt to discharge this heavy burden.
There is a rational justification for excluding elected officials from the
operation of the deemed resigned provisions.
Considering that elected officials are put in office by their constituents for
a definite term, it may justifiably be said that they were excluded from
the ambit of the deemed resigned provisions in utmost respect for the
mandate of the sovereign will. In other words, complete deference is
accorded to the will of the electorate that they be served by such
officials until the end of the term for which they were elected. In
contrast, there is no such expectation insofar as appointed officials are
concerned.
The SC’s previous decision relied extensively on Mancuso v. Taft. It was a
decision of the First Circuit of the US Court of Appeals which struck down
as unconstitutional a sumular statutory provision.
The SC’s reliance on Mancuso is completely misplaced. The Mancuso
ruling was effectively overruled by the US Supreme Court three
months after its promulgation in the US CA.
Nachura, dissenting: In considering persons holding appointive
positions as ipso facto resigned from their posts upon the filing of
their certificates of candidacy (CoCs), but not considering as
resigned all other civil servants, specifically the elective ones, the law
unduly discriminates against the first class. (Note: Basically use the
ponencia’s arguments in the 2009 case.)
BIRAOGO v. PTC (HENRY) 1 They investigate patterns of abuse committed over a period
December 7, 2010 | Menodza, J. | Equal Protection Clause of time, as opposed to a particular event
2 They are temporary bodies that finish heir work with the
submission
PETITIONER: Louis “Barok” C. Biraogo
RESPONDENTS: The Philippine Truth Commission of 2010

SUMMARY: Biraogo assails in the SC the constitutionality of EO 1 which


created the PTC, an ad hoc committee assigned to invesitate the
members of the previous (Arroyo) administration with regard to graft and
corruption. Biraog contends that it is violative of the equal protection
clause of the constitution, as it specifically investigates only the previous
administration. The SC indeed ruled that it is violative and hence void, as
it fails to be applied to all members of the same class since it will only
investigate the Arroyo Admin, and not all the past administrations.

DOCTRINE: Not to include past administrations similarly situated


constitutes arbitrariness which the equal protection clause cannot
sanction. Such differentiation clearly reverberates to label the
commission as a vehicle for vindictiveness and selective retribution.
Hence, in for a classification to be reasonable as far as a class is
concerned, it must be applied to ALL members, and not to only a
particular one.

FACTS:
The genesis of the two cases (1st case yung kay Biraogo) at hand can be
traced to the events prior to the historic May 2010 elections, when
Senator Aquino III declared his staunch condemnation of graft and
corruption with his slogan, “Kung walang corrupt, walang mahirap”.
To transform his campaign slogan into reality, PNoy found a need for a
special body to investigate reported cases of graft and corruption
allegedly committed during the previous admin. Hence, his signing of
EO 1.
By virtue of EO 1, the Philippine Truth Commission (PTC) is created. This
is a mere ad hoc body formed under the office of the President with
the primary task to investigate reports of graft and corruption
committed by third-level public officers and employees, their co-
principals, accomplices and accessories during the previous
administration, and thereafter to submit its finding and
recommendations to the President, Congress, and the Ombudsman.
Basically, PTC’s function is to collect and assess evidence of graft and
corruption and make recommendations. It cannot determine from
such facts if probable cause exists as to warrant the filing of an
information. It cannot impose criminal, civil, or administrative
penalties or sanctions.
The characteristics of truth commissions, such as the PTC, are as
follows:
0 They examine only past events
of a report containing conclusions and recommendations is to investigate and find out the truth concerning the reported cases of
They are officially sanctioned, authorized, or empowered by the graft and corruption
state
After barely a month of the issuance of EO 1, the petitioners asked the
Court to declare it unconstitutional and enjoin PTC from performing
its functions. They are claiming that, EO 1 vioates the equal
protection clause because it selectively targets for investigation and
prosecution officials and personnel of the previous administration as
if corruption is their peculiar species even as it excludes those of the
other administrations, past and present, who may be indictable.
Hence, this petition.
Respondents, through OSG, on the other hand, in their consolidated
comment argue that it does nto violate equal protection clause
because it was validly created for laudable purposes.

ISSUE/s:
WoN EO 1 is violative of the equal protection clause - YES

RULING: SC granted the petition.

RATIO:
The equal protection of the laws is embraced in the concept of due
process, as every unfair discrimination offends the reuqirements of
justice and fair play. It has been embodied in a separate clause,
however, to provide for a more specific guaranty against any form of
undue favoritism or hostility from the government.
Arbitrariness in general may be challenged on the basis of the due
process clause. But if the particular act assailed partakes of an
unwarranted partiality or prejudice, the sharper weapon to cut down
is the equal protection clause.
Equal protection simply requires that all persons or things similarly
situated should be treated alike, both as to rights conferred and
responsibilities imposed. It requires public bodies and institutions to
treat similarly situated individuals in a similar manner.
The concept of equal justice under the law requires the state to govern
impartially, and it may not draw distinctions between individuals
solely on differences that are irrelevant to a legitimate governmental
objective.
Equal protectgion does not require universal application of the laws to all
persons or things without distinction. What it simply requires is
equality amonge equals as determined according to a valid
classification.
Such classification shall pass the reasonableness test:
Rests on substantial distinctions
Germane to the purpose of the law
Not limited to existing conditions
Applies equally to all members of the same class
SC ruled that, EO 1 should be struck down as violative of the equal
protection clause. The clear mandate of the envisioned truth commission
during the previous administration only. The intent to single out the Applies equally to all members of the same class
previous administration is plain, patent and manifest. Nachura, concurring and dissenting: In the area of equal protection
It must be borne in mind that the Arroyo administration is but just a
member of a class, that is, a class of past administrations. It is
nto a class of its own. Not to include past administrations
similarly situated constitutes arbitrariness which the equal
protection clause cannot sanction. Such differentiation clearly
reverberates to label the commission as a vehicle for
vindictiveness and selective retribution.
“Superficial differences do nto make for a valid classification” (Justice
Isagani Cruz)
It could be argued that considering that the PTC is an ad hoc body, its
scope is limited. The Court, however, is of the considered view that
although its focus is restricted, the constitutional guarantee of equal
protection under the laws should not in any way be circumvented.
Corona, C.J, separate: Given the indubitably clear mandate of EO 1,
does the identification of the Arroyo Admin as the subject of the Truth
Commsision’s investigation pass the jurisprudential test of
reasonableness? Stated differently, doe sthe mandate of EO 1
violate equal protection clause of the Constitution? Yes.
Carpio, dissenting: These are not only reasonable but also compelling
grounds for the Truth Commission to prioritize the investigation of the
Arroyo Admin. To prioritize based on reasonable and even
compelling grounds is not to discriminate, but to act sensibly and
responsibly. In any event, there is no violation of the equal protection
clause just because the authorities focus their investigation or
prosecution on one particular alleged law-breaker, for surely a
person accused of robbery cannot raise as a defense that other
robbers like him all over the country are not being prosecuted. By the
very nature of an investigation or prosecution, there must be a focus
on particular act or acts of a person or a group of persons. A claim of
selective prosecution that violates the equal protection clause be
raised only by the party adversely affected by the discriminatory act.
Carpio-Morales, dissenting: petitioners cannot properly assert the
equal protection caim of the previous admin. While legislators have
locus standi in certain cases, their legal standing as such is
recognized only insofar as assailed issuance affects their functions
as legislators. Breacgh of equal protection clause raised by
petitioner-legislators on behalf of the Executive Department of the
immediate past admin has nothing to do with the impairment of the
powers of Congress. Carpio-Morales finds nothing arbitrary or
unreasonable in the Truth Commission’s defined scope of
investigation. In issues involving the equal protection clause, the test
developed by jurisprudence is that of reasonableness which has 4
requisites:
Classification rests on substantial distinctions
Germane to the purpose of the law
Not limited to existing conditions only
analysis, a court objects the legislative/ executive action to one of the
three levels of scrutiny, depending on the class of persons and rights
affected by the action:
Rational basis scrutiny – presence of any plausible legitimate
objective for the classification, where the classification serves to
accomplish at objective to any degree, no matter how tiny, would
validate the classification. To be invalidated, the test requires:
0 it has absolutely no conceivable legitimate purpose
1 it is so uinconnected to any conceivable objective, that is
ubsurd, utterly arbitrary, whimsical, or erven perverse
Intermediate scrutiny
Strict scrutiny
Peralta, separate concurring: The distinctions cited by the OSG are not
substantial to separate the previous administration as a distinct class
from prior administrations as subject matter for investigation for the
purpose of ending graft and corruption. There is no substantial distinction
cited between public officers who may be involved in reported cases of
graft and corruption during the previous administration and public officers
who may be involved in reported cases of graft and corruption during
prior administrations in relation to the purpose of ending graft and
corruption.
Abad, separate dissenting: The idea behind the “equal protection clause” is
that public authorities should treat all persons or things equally in terms
of rights granted to and responsibilities imposed on them. As an element
of due process, the equal protection clause bars arbitrary discrimintation
in favor of or against a class whether in what the law provides and how it
is enforced.
Sereno, dissenting: The majority decision defeats the constitutional
mandate on public accountability; it effectively tolerates impunity for graft
and corruption. Its invocation of the constitutional clause on equal
protection of the laws is an unwarranted misuse of the same and is a
disservice to those classes of people for whom the constitutional
guarantee was created as a succor. The majority decision accomplished
this by completely disregarding the “reasonableness” and all its
jurisprudential history as constitutional justification for classification and
by replacing the analytical test of reasonableness with mere recitations of
general case doctrines to arrive at its forced conclusion.
IN THE MATTER OF THE BREWING CONTROVERSIES IN THE IBP President and EVP
ELECTION IN THE INTEGRATED BAR OF THE PHILIPPINES The appointment of a caretaker board to administer the affairs of
(DANNAH) the IBP pending the holding of special elections
April 11, 2013 | Mendoza, J. | Equal Protection Clause The Court also amended Section 47, Article VII of the IBP By Laws
Following the rotation following individuals representing different regions
of the IBP served as IBP President.
COMPLAINANTS: Attys. Marcial Magsino, Maramba et al.
On January 1999, in Velez v. de Vera, the Court considered the election
RESPONDENTS: Attys. Rogelio Vinluan, Estrada et la. of then EVP De Vera representing the Eastern Mindanao region as
one completing the first rotational cycle and affirmed the election
SUMMARY: There have been many controversies surrounding the IBP
of Salazar of Bicolandia region as the EVP
elections because of the amendment of the Court regarding the strict
For having misappropriated his clients’ funds and committing acts
rotational basis for regions. The effect of De Vera resigning and not finishing
inimical, De Vera was removed as governor of Eastern Mindanao
his term as EVP created chaos. and as EVP.
Thus, Salazar became IBP President for the 2005-07 term with Bautista
The Court decided to further amend the By Laws but instructed officials of
of Central Luzon as EVP.
the IBP to follow it strictly, so as to give all regions an equal opportunity to
The term of Salazar was the start of the second rotational cyrcle. Bautista
have their representative become EVP on the next rotational cycle. eventually succeeded to the IBP presidency with herein respondent
Atty. Vinluan as his EVP.
DOCTRINE: Courts may amend rules or By Laws in order to stay true to
In 2009, the national and regional IBP elections were again tained with
giving equal opportunity for everyone. numerous controversies and the Court resolved it.
The Current Controversy: On July 2012, IBP Southern Luzon filed its
Motion for Leave to Intervene and to Admit the Attached Petition in
FACTS: Intervention and the subject Petition In Intervention, seeking a
On November 1, 1974, the IBP By-Laws took effect, providing that the declaration that the post of EVP for the 2011-13 term be held open to
IBP President and EVP be chosen by the Board of Governors from all regions and that it be qualified to nominate a candidate for the
among 9 regional goevernors, as much as practicable, on a rotation position of EVP for the 2011-13 term.
basis. The subject Petition was opposed by Fortunato, who insisted that the
It was also provided that the President and EVP hold office for a term of of 2 Western Visayas was the only region that could vie for the position of
years from July 1 following their election until June 30 of their second EVP for said term.
year in office and until their successors shall have been duly chosen and In the 2012 Resolution, the Court deferred its action on the intervention
qualified. sought by the IBP Southern Luzon and required the IBP-BOG to
Several amendments in the IBP By Laws were introduced, among which submit its comment.
were the provisions relating to the election of its national officers. IBP BOG prayed that Southern Luzon be allowed to nominate a
In Bar Matter 287, the Court approved the recommendation allowing the candidate for EVP.
IBP President, the EVP and the officers of the House of Delegates to
be directly elaected by the House of Delegates. ISSUE/s:
However, this mode was marred with unethical politicking, electioneering WoN the motion for intervention of IBP-SL can be allowed and admitted –
and other distasteful practice. YES
The Court on 1989 in Bar Matter 491 ordered: WoN the first rotational cycle was completed with the election of Atty. De
The annulment of the just concluded national elections Vera – YES
The abolition of the system of election of national officers by Whether IBP-SL has already served in the current rotation –
direct action of the House of Delegates WoN IBP-WV has already served the current rotation –
The restoration of the former system of having the IBP President
and the EVP elected by the IBP-Board of Governors from RULING:.
among themselves, with right of succession by the EVP to
the presidency subject to the rule that “the position of EVP RATIO:
shall be rotated among the 9 IBP regions The Court is allowing the matter to be raised as an issue because it has
The holding of special elections for the election of the first set of not
yet been squarely settled. chapter shall serve consecutively. Every winner shall then be
On issue #2: The IBP By Laws was amended in December 2010 excluded after its term. Romblon then joins the succeeding elections
Resolution to read as follows after the first winner in the
The IBP shall have a President, an EVP and 9 regional
Governors. The EVP shall be elected on a strict rotation
basis by the BOG form among themselves, by the vote of at
least 5 Governors. The Governors shall be ex officio VP for
their respective regions…
From the above it is clear that the amendment was effected to
underscore the shift of the rotation from position of president to EVP.
The purpose was to ensure that all regions will have an equal
opportunity to serve as EVP then automatically succeed as
president.
The amendment to Section 47 was effected to underscore the shift of the
rotation from the position of president to that of EVP.
Even if Atty. De Vera was removed, the rotation had been completed
despite the non-assumption by Atty. De Vera to the IBP Presidency.
Only the governors of the Western Visayas and Eastern Mindanao
regions have not had their turn as EVP, while Central Luzon and
Bicolandia have had 2 terms already.
Apparently, the report of the Special Committee failed to take into
account the ruling in Velez that the service of then EVP De Vera
representing the Eastern Mindanao region, completed the first
rotational cycle.
Thus, two inaccuracies have been committed by the Special Committee
It erroneously reported that “only the governnors of the WV and
EM regions have not yet had their turn as EVP
It erroneously considered Central Luzon & Bicolandia as having
had 2 terms each in the First Rotational Cycle, when their
second services were for the Second Cycle.
With Eugene Tan’s election as IBP President, IBP-SL argues that with his
election and service as IBP-President from 1990-91, the IBP WV
should no longer be allowed to field a candidate.
As Velez declared that the election of EVP De Vera completed the first
rotational cyrcle, it could only mean that all regions had their respective
turns in the first cycle. Thus in the second, issues as to the nature of his
election and services as IBP President during the First cycle are
inconsequential.
Tan resigned which is why his post was only until 1991. And it was ruled
that this completed one term.
Thus, initially, all chapters shall have the equal opportunity to vie for the
position of Governor for the next cycle except Romblon, so as no
cycle.
To avoid the endless conflicts, the solution is to start another rotational round,
a new cycle, open to all regions.
Section 47 should be further amended.
Each region, shall have the opportunity to have its representative elected as
EVP, provided that, the election for the position of EVP shall be on a
strict rotation by exclusion basis.
At the end of the rotational cycle, all regions, except the region whose
representative has just served the immediately preceding term, may be
elected for another term as Executive Vice-President in the new
rotational cycle. The region whose representative served last in the
previous rotational cycle may be elected Executive Vice-President only
after the first term of the new rotational cycle ends, subject once more to
the rule on exclusion.
Velasco, Dissenting: It bears mentioning that with the election and service
of Atty. Vinluan of the IBP-SL as EVP-IBP for the 2007-09 term, the
purpose of the rotation sysem was to give equal opportunity to all regions
of the IBP has already been satisfied.
On the issue on intervention: Velasco cited Chavez v. PCGG
Movants Imelda Marcos et al claimed to have a legal interest in the
matter of litigation, or in the success of parties or an interest against
both as to warrant their intervention. They added that their exclusion
from the instant case resulted in a denial of their constitutional right
to due process and to equal protection of the laws.
Intervention should not be allowed after final judgment.
There is no jurisprudence allowing an intervention by a person who
has not shown any legal interest in the matter in litigation after
the decision has become final and executory.
AQUINO v. PHILIPPINE PORTS AUTHORITY (IYA) ISSUE/s:
April 17, 2013 | Ynares-Santiago, J. | Equal Protection Clause WON the officials are entitled to continued RATA under LOI No. 97 -

PETITIONER: Amelia Aquino, Rodolfo Tuggueg, Jr., Adelaida Hernandez,


Leopoldo Biscocho Jr,
RESPONDENTS: Philippine Ports Authority

SUMMARY: Officials of PPA request for the disallowance of their RATA


be reconsidered. They assail that the distinction between officials
employed before the implementation of the LOI and of new officials is
violative of the equal protection clause as it allows older officials to
receive RATA.

The SC ruled upholding the COA order denying the grant of RATA.

DOCTRINE: The Court takes note that the Constitution does not require
hat things which are different in fact be treated in law as though they
were the same.

FACTS:
Congress passed RA No. 6758 otherwise known as the Salary
Standardization Law. It is “An Act Prescribing a Revised
Compensation and Position Classification in the Government and for
Other Purposes”.
Marcos issued a Letter of Implementation NO. 97 authorizing the
implementation of standard compensation position classification
plans for the infrastructure/utilities group of GOCCs.
Philippine Ports Authority issued Memorandum Circular No. 57-87 which
granted to its officials holding managerial and supervisory positions
representation and transportation allowance (RATA) in an amount
equivalent to 40% of their basic salary.
PPA issued MC No. 36-89 which extended the RATA entitlement to its
Section Chiefs and senior personnel at the rate of 20% of their basic
pay.
Another MC (No. 46-90) adjusted the former RATA increasing the 20% to
40%.
Commission on Audit Corporate Auditor addressed PPA, which
disallowed the payment of the RATA differentials. It disallowed in
audit the grant of RATA to PPA Section Chiefs and senior personnel
occupying position with salary grades of 17 and above who were
appointed after the effectivity of RA 6758.
In view of the disallowances, PPA officials filed a petition before the SC
claiming their entitlement to the RATA provided under LOI No. 97.
RULING: WHEREFORE, the instant Petition for Review on Certiorari is
DENIED.

RATIO:
There are at present two categories of managers and supervisors at the
PPA.
First category is composed of PPA officials who were occupying their
positions and actually receiving the 40% RATA under LOI No. 97 as
of 1 July 1989 and who continue to receive such benefit.
The Second category officials receive a lesser RAT under the General
Appropriations Act although the hold the same rank, title, and may
have the same responsibilities as their counterparts in the first
category.
The Court takes note that the Constitution does not require hat things
which are different in fact be treated in law as though they were the
same.
The equal protection clause does not prohibit discrimination as to things
that are different. it does not prohibit legislation which is limited either
in object to which it is directed or by the territory within which it is to
operate.
All that is required of a valid classification is that it be reasonable, which
means that the classification should be based on substantial
distinctions which make for real differences, that it must be germane
to the purpose of the law; that it must not be limited to existing
conditions only; and that it must apply equally to each member of the
class.
If the incumbent resigns or is promoted to a higher position, his
successor is no longer entitled to his RATA privilege or the transition
allowance.
It does not infringe the quell protection clause as it is based on
reasonable classification intended to protect the rights of the
incumbents against diminution of their pay and benefits.
GARCIA v. DRILON (ELIEL) Threats were made regarding halting financial support because Rosalie
June 25, 2013 | Perlas-Bernabe, J. | Equal Protection Clause intended to file a case against her husband’s paramour. He beat her
up and her daughter, and left them.
PETITIONER: Jesus Garcia
RESPONDENTS: Hon. Drilon, Rosalie Garcia

SUMMARY: Rosalie and Jesus were married with two children. However,
their relationship took a different turn when Rosalie found out about the affair
of Jesus with the manager of Robinson’s Bank of Bacolod. Knowing about
Rosalie’s intent to sue his paramour, Jesus threatened to cut of financial
support of Rosalie, beat her up, and left. This prompted Rosalie to file with
the RTC a complaint under RA 9262. The RTC ruled in her favor and issued
a TPO. Jesus appealed to the CA questioning the TPO for being violative of
the equal protection clause. The CA dimissed his petition. Hence this appeal.
The SC denied Jesus’ petition and ruled that RA 9262 is not violative of the
equal protection clause. (1) It rests substantial distinctions because of the
unequal power relationship between men and women, gender bias, and
women often being victims of violence; (2) Is that it is germane to the purpose
of law as it aims to address the violence against women and children
– this in accordance with the fact that the PH is a signatory of CEDAW;
(3) It applies not only to the present condition but to future conditions
also, for as long as there is a threat or abuse to women and children; (4)
It applies to all women and children who are subject to violence and
abuse; moreover, it does not single out men for it also punishes women
who does the same thing to any person.

DOCTRINE: All that is required a valid classification is that it be


reasonable, which means that the classification should be based on
substantial distinctions which make for real differences; that it must
be germane to the purpose of the law; that it must not be limited to
existing conditions only; and that it must apply equally to each
member of the class.

FACTS:
On March 2004, Congress enacted RA 9262 entitled “An Act Defining
Violence Against Women and Their Children, Providing for Protective
Measures for Victims, Prescribing Penalties Therefor, and for Other
Purposes”.
Rosalie Jaype-Garcia filed in her and her minor children’s behalf a TPO
against her husband, Jesus pursuant to RA 9262. She claimed to be
a victim of physical, emotional, psychological abuse as a result of
marital infidelity.
Jesus was very strict, even stopped Rosalie from pursuing her career as
a lawyer. Things turned for the worse when Jesus started having an
affair with a bank manager of Robinson’s bank, Bacolod.
Finding reasonable ground to believe that an imminent danger of likely than men to be victims of violence; and the widespread gender
violence against Rosalie and her children exists or is about to recur, bias and
the RTC of Bacolod issues a TPO.
The TPO included things like prohibiting him from being within 1000
meters from them and their help, drivers and conjugal home; giving
monthly support, letting the family use 2 cars, giving up all his
firearms and etc.
A series of amended TPOs and extensions were granted because Jesus
would not comply with those indicated, he continued to harass them.
There were incidents such as attempted kidnapping on the two youngest
boys, and physical abuse and threats on Jo-Ann, the eldest. She
subsequently filed a criminal complaint against her father for violation
of RA 7610, or the Special Protection of Children Against Child
Abuse Exploitation and Discrimination Act.
Jesus filed before the CA a petition for prohibition with prayer for
injunction and temporary restraining order, challenging (1) the
constitutionality of RA 9262 for being violative of due process and
equal protection, and (2) the validity of the modified TPO issued in
the civil case for being “an unwanted product of an invalid law”.
The CA then issued a 60-day TRO against said TPOs, but dismissed the
petition for failure of Jesus to raise the constitutional issue in his
pleadings before the trial court in the civil case, which is clothed with
jurisdiction to resolve the same.

ISSUE/s:
WoN RA 9262 is violative of equal protection clause –

NO RULING: Petition Denied.

RATIO:
Equal protection simply requires that all persons or things similarly
situated should be treated alike, both as to rights conferred and
responsibilities imposed.
The equal protection of the laws clause of the Constitution allows
classification. Classification in law, as in the other departments of
nkowledge or practice, is the grouping of things in speculation or
practice because they agree with one another in certain particulars.
All that is required a valid classification is that it be reasonable, which
means that the classification should be based on substantial
distinctions which make for real differences; that it must be
germane to the purpose of the law; that it must not be limited to
existing conditions only; and that it must apply equally to each
member of the class.
RA 9262 rests on substantial distcintions. The unequal power
relationship between women and men; the fact that women are more
prejudice agaisnt women all make for real differences jsutifying the long as the safety and security of women and their children
classification under the law. are threatened by
Societal norms and traditions dictate people to think men are the leaders,
pursuers, providers, and take on dominant roles in society while
women are nurturers, men’s companions and supporters, and take
on subordinate roles in society.
This perception leads to men gaining more power over women. With
power comes the need to control to retain that power. And VAW is a
form of men’s expression of controlling women to retain power.
Women are the “usual” and “most likely” victimes of violence. There
st
are 1,091 DSWD cases out of a total number of 3,471 cases for the 1
semester of 2003. Femal violence comprised more than 90% of all forms
of abuse and violence and more than 90% of these reported cases were
committed by the women’s intimate partners such as their husbands and
live-in partners.
While there are, indeed, relatively few cases of violence and abuse
perpetrated against men in the Philippines, the same cannot render
RA 9262 invalid.
The mere fact that the legislative classification may result in actual
inequality is not violative of the right ot equal protection, for
every classification of persons or things for regulation by law
produces inequality in some degree, but the law is not thereby
rendered invalid.
Gender bias and prejudices. From the initial report to the police
through prosecution, trial, and sentencing, crimes against women are
often treated differently and less seriously than other crimes.
Sadly, our own courts, as well, have exhibited prejudices and biases
against our women.
The enactment of RA 9262 aims to address the discrimination
brought about by biases and prejudices against women.
The distinction between men and women is germane to the purpose
of RA 9262, which is to address violence committed against
women and children.
The Optional Protocal to the Committee on the Elimination of
Discrimination Against Women (CEDAW) was also ratified by the PH.
This Convention mandates that State parties shall accord to women
equality with men before the law and shall take all appropriate
measures to elimante discrimination agaisnt women in all matters
relating to marriage and family erlations on the basis of equality of
men and women.
The application of RA 9262 is not limited to the existing conditions
when it was promulgated, but to future conditions as well, for as
violence and abuse.
RA 9262 applies equally to all women and children who suffer violence
and abuse.
There is likewise no merit to the contention that RA 9262 singles out the
husband or father as the culprit. As defined above, VAWC may likewise
be committed “against a woman with whom the person has or had a
sexual or dating relationship.” Clearly, the use of the gender-neutral word
“person” who has or had a sexual or dating relationship with the woman
encompasses even lesbian relationships.
De Castro, J. Concurring: In my view, a suspect classification and the
accompanying strict scrutiny should depend on the circumstances of the
case, on the impact of the illegal differential treatment on the group
involved, on the needed protection and the impact of recognizing a
suspect classification on future classification. A suspect classification
label cannot solely and automatically be triggered by the circumstance
that women and children are accorded special prtoection by the
Constitution. In fact, there is no place for a strict level of scrutiny when
the Constitution itself has recognized the need for special protection;
where such recognition has been made, congressional action should
carry the presumption of validity.
Leonen, J. Concurring: By concurring with these statements I express a
hope: that the normative constitutional requirmements of human dignity
and fundamental equality can become descriptive reality. The scoailly
constructed distinctions between women and men that have afflicted us
and spawned discrimination and violence should be eradicated sooner.
Power and intimacy should not co-exist.
Abad, J. Concurring: CJ Puno’s thesis is that the right to equal protection
casts another shadow when the issue raised under it involves persons
protected by the social justice provision of the Constitution, specifically,
Sec 1, Art 13. The equal protection clause can no longer be interpreted
as only a guarantee of formal equality but of substantive equality. “It
ought ot be construed,” said the CJ, “in consonance with social justice as
‘the hear’ particularly of the 1987 Constitution – a transformative
covenant in which the Filipino people agreed to enshrine asymmetrical
equality to uplift disadvantaged groups and build a genuinely egalitarian
democracy.
ALMARIO v. EXECUTIVE SECRETARY (JP) decoration or award, for persons who have significantly
July 16, 2013 | Leonardo-De Castro, J. | Equal Protection Clause contributed to the development and promotion of Philippine
culture and arts.
PETITIONER: National Artist For Literature Virgilio Almario and other As both the CCP Board of Trustees and the NCCA have been
National Artists, University Professor Emeritus Gemino Abad, et al. mandated by
RESPONDENTS: Executive Secretary, Cultural Ceneter of the
Philippines, the National Commission on Culture and the Arts, et al.

SUMMARY: The Board of Trustees of the Cultural Center of the Philippines


and the National Commission for Culture and the Arts were given express
mandate to confer the National Artist Award. They were to conduct strict
deliberations and submit their judgments to the Committee of Honors of the
Office of the President. Petitioners here filed the present petition for certiorari
because the National Artist Awards were awarded to four artists who did not
undergo the strict deliberations of the CCP and the NCCA. They allege that
their was the President favored the four respondents as a class on their own.
The SC held that the proclamation of the awards shoud be invalid because
the preferential treatment for the respondets constitute violation of the equal
protection clause against those who had undergone the strict process of
deliberations.

DOCTRINE: To pass the equal protection clause, the classification


should be based on substantial distinctions which make for real
differences; that it must be germane to the purpose of the law; that it
must not be limited to existing conditions only; and that it must apply
equally to each member of the class.

FACTS:
Former President Ferdinand Marcos issued Proclamation No. 1001 and upon
the recommendation of the Board of Trustees of the Cultural Center
of the
Philippines (CCP), created the category of Award and Decoration of
National Artist to be awarded to Filipinos who have made distinct
contributions to arts and letters.
103. RA No. 7356 established the National Commission for Culture
and the Arts (NCCA) and gave it the mandate over the
development, promotion and preservation of the Filipino national
culture.
104. Among the specific mandates of the NCCA under Republic Act No.
7356 is to “extend recognition of artistic achievement through
awards, grants and services to artists and cultural groups which
contribute significantly to the Filipino’s cultural legacy.”
The NCCA is vested with the power to „advise the President on matters
pertaining to culture and the arts, including the creation of a special
law, the two bodies decided to team up and jointly administer the National
Artists Award.
Executive Order No. 236, s. 2003, entitled Establishing the Honors Code of the
Philippines to Create an Order of Precedence of Honors Conferred and for
Other Purposes, was issued. It further created a Committee on Honors to
“assist the President in evaluating nominations for recipients of Honors.”
The Committee on Honors shall, as a general rule, serve as a screening
committee to ensure that nominations received from the various awards
committees meet two tests: that there has not been an abuse of discretion in
making the nomination, and that the nominee is in good standing. Should a
nomination meet these criteria, a recommendation to the President for
conferment shall be made.
109. A joint meeting of the NCCA Board of Commissioners and the CCP Board of
Trustees was held to discuss, among others, the evaluation of the 2009
Order of National Artists and the convening of the National Artist Award
Secretariat.
The First Deliberation Panel met. A total of 87 nominees were considered during
the deliberation and a preliminary shortlist of 32 names was compiled. During
the final deliberations, from the 13 second shortlisted, a final list of four
names was agreed upon: 1. Manuel Conde 2. Ramon Santos 3. Lazaro
Francisco 4. Federico Aguilar-Alcuaz
The Office of the President then allegedly received nominations from various
sectors, cultural groups and individuals strongly endorsing private
respondents Cecile Guidote-Alvarez, Carlo Magno Jose Caparas, Francisco
Mañosa and Jose Moreno.
The Committee on Honors thereafter submitted a memorandum to then
President Gloria Macapagal-Arroyo recommending the conferment of the
Order of National Artists on the four recommendees of the NCCA and the
CCP Boards, as well as on private respondents Guidote- Alvarez, Caparas,
Mañosa and Moreno.
Acting on this recommendation, Proclamation No. 1823 declaring Manuel Conde
a National Artist was issue. Subsequently, Proclamation Nos. 1824 to 1829
were issued declaring Lazaro Francisco, Federico AguilarAlcuaz and private
respondents Guidote-Alvarez, Caparas, Mañosa and Moreno, respectively,
as National Artists.
Petitioners instituted this petition for prohibition, certiorari and injunction (with
prayer for restraining order) praying that the Order of National Artists be
conferred on Dr. Santos and that the conferment of the Order of National
Artists on respondents Guidote- Alvarez, Caparas, Mañosa and Moreno be
enjoined and declared to have been rendered in grave abuse of discretion.
Petitioners allege that in adding the names of respondents Caparas, Yet, the four of them (respondents) were treated differently and
Guidote-Alvarez, Mañosa and Moreno while dropping Dr. Santos considered favorably when they were exempted from the
from the list of conferees, the PresidentÊs own choices constituted rigorous screening process of
the majority of the awardees in utter disregard of the choices of the the NCCA and the CCP and conferred the Order of National
NCCA and the CCP and the arts and culture community which were Artists.
arrived at after a long and rigorous process of screening and 125. The Committee on Honors and the former President
deliberation. effectively treated
Petitioners further argue that the choice of respondent Guidote-Alvarez (as
one of the national artists) was illegal and unethical because, as the
then Executive Director of the NCCA and presidential adviser on
culture and arts, she was disqualified from even being nominated.
117. The President gave preferential treatment to respondent
Guidote-Alvarez by naming the latter a National Artist despite
her not having been nominated and, thus, not subjected to the
screening process provided by the rules for selection to the
Order of National Artists.
118. Her inclusion in the list by the President represented a clear and
manifest favor given by the President in that she was exempted
from the process that all other artists have to undergo. According
to petitioners, it may be said that the President used a different
procedure to qualify respondent Guidote-Alvarez. This was clearly
grave abuse of discretion for being manifest and undue bias
violative of the equal protection clause.

ISSUE/s:
WoN the conferment of the National Artists was in grave abuse of
discretion amounting to a violation of the due process clause - YES

RULING: Proclamation Nos. 1826 to 1829 dated July 6, 2009 proclaiming


respondents Cecile Guidote-Alvarez, Carlo Magno Jose Caparas, Francisco
Mañosa, and Jose Moreno, respectively, as National Artists are declared
INVALID and SET ASIDE for having been issued with grave abuse of
discretion.

RATIO:
123. Among the other petitioners, Prof. Gemino Abad presents a unique
valid personal and substantial interest. Like respondents Caparas,
Mañosa and Moreno, he was among the 87 nominees for the 2009
Order of National Artists. Like respondent Moreno, he made it to the
preliminary shortlist. As he did not make it to the second shortlist, he
was not considered by the Final Deliberation Panel, more so by the
former President.
respondents Guidote-Alvarez, Caparas, Mañosa and Moreno as a preferred
class. The special treatment accorded to respondents Guidote-Alvarez,
Caparas, Mañosa and Moreno fails to pass rational scrutiny.
The rational basis scrutiny is one of three tests used by the Court to test
compliance with the equal protection clause. It is the minimal level of scrutiny
which requires that the challenged classification is rationally related to
serving a legitimate State interest. It is used when the government action
is a type of discrimination that does not warrant the intermediate and strict
levels of scrutiny.
127. No real and substantial distinction between respondents and
petitioner Abad has been shown that would justify deviating from the
laws, guidelines and established procedures, and placing respondents
in an exceptional position.
128. The undue classification was not germane to the purpose of the law.
Instead, it contradicted the law and well-established guidelines, rules
and regulations meant to carry the law into effect. While petitioner Abad
cannot claim entitlement to the Order of National Artists, he is entitled
to be given an equal opportunity to vie for that honor.
In view of the foregoing, there was a violation of petitioner Abad’s right to equal
protection, an interest that is substantial enough to confer him standing in
this case.
There was a violation of the equal protection clause of the Constitution when the
former President gave preferential treatment to respondents Guidote-
Alvarez, Caparas, Mañosa and Moreno.
The former President’s constitutional duty to faithfully execute the laws and observe
the rules, guidelines and policies of the NCCA and the CCP as to the
selection of the nominees for conferment of the Order of National Artists
proscribed her from having a free and uninhibited hand in the conferment of
the said award.
132. The manifest disregard of the rules, guidelines and processes of the
NCCA and the CCP was an arbitrary act that unduly favored
respondents Guidote-Alvarez, Caparas, Mañosa and Moreno. The
conferment of the Order of National Artists on said respondents was
therefore made with grave abuse of discretion and should be set aside.
LTFRB v. STRONGHOLD INSURANCE COMPANY (HENRY) be donated to the government. In consideration, however, of
October 2, 2013 | Carpio, J. | Equal Protection Clause the initial

PETITIONER: Land Transportation Franchise and Regulatory Board,


Jaime Jacob, Chairman oof LTFRB, et al.
RESPONDENTS: Stronghold Insurance Company, Inc.

SUMMARY: LTFRB created the Passenger Accident Insurance Program,


where it will accredit 2 groups of insurance providers to provide such
services to PUV operators, covering passengers against accident-
related risks. UNITRANS is one of the two groups, where respondent
Stronghold is the lead insurer.

Shortly before the first MOA expired, LTRB opened the floor for bidding
for the accreditation of new insurance providers. While Stronghold
qualified at the 1st and 2nd References, it barely made itself to the 3 rd, as
it’s minimum capitalization is only 140M, contrary to the required 250M.

Stronghold now claims that there being only a limited number of


insurance companies with such amount as capital (18 to be exact),
LTFRB’s procedure violated their right to equal protection. The CA did not
pass upon this claim. Same goes for the SC. However, it was held that
LTFR did not commit grave abuse of discretion.

DOCTRINE: Reasonableness of the classification scheme does not imply


the violation of the constitutional right to equal protection of the laws, as
provided for by the Constitution.

FACTS:
LTFRB is the government agency charged with the regulation of
franchises of land-based public utility vehicles. To implement the law
requiring operators of passenger public utility vehicles to obtain
accident insurance policies, LTFRB created the Passenger Accident
Insurance Program (Program).
Under the Program, LTFRB will accredit 2 groups of insurance providers,
selected through open bidding, to provide insurance policies to public
utility vehicle operators, covering passengers against accident-
related risks.
Universal Transport Solutions, Inc. (UNITRANS) is one of the 2 groups of
insurance providers. Respondent Stronghold Inurance Company, Inc.
(Stronghold) was the lead insurer of UNITRANS.
LTFRB’s 5-year contract with UNITRANS embodied a MOA, with the
following clause:
“WHEREAS, after the expiration of the contract for accreditation,
all facilities used by the accredited management groups shall
investment and the assumption of initial risk, the two
management groups herein shall be given the right to match
the best bid/proposal in the event another management
group qualifies t the end of the term of this agreement.”
Shortly before the first MOA expired and after its term was extended,
LTFRB thrice opened bidding for the accreditation of new insurance
providers, the first two biddings were canceled by the DOTC.
In each round, LTFRB required minimum peso capitalization for the lead and
member insurers. Of which, the Third Reference reckoned complance
with the minimum capital requirement for the lead and member insurers
singly or on a “per insurer” basis. It required a minimum of 10 members
nd
for each group of insurers, the same number in the 2 Reference but
st
half of that in the 1 .
Stronghold participated in all three, but failed to qualify in the third as its
group only had 6 members and its minimum capitalization, as lead
insurer, was only Php 140M (below minimum, which is 250M).
Stronghold sought a writ of prohibition from the CA, to enjoin LTFRB
from opening the bid documents and to nullify bidding proceedings.
They claim the Third Reference to be a refusal of their right to equal
protection under the Constitution, due to the fact that only 18 out
of the 87 insurance companies in the PH have a minimum paid-
up capital of 250M.
CA required LTFRB to file a comment, then ruled in favor of Stronghold.
CA no longer passed upon Stronghold’s claim of denial of equal
protection.
The CA held, that LTFRB, as far as the 3 rd reference is concerned,
abused its discretion when it released the 3 rd reference without
considering the legal ramifications on the terms of the MOA.
Stronghold’s group had already acquired a property right which the
LTFRB cannot just set aside without due process.
Hence, this petition.

ISSUE/s:
WoN LTFRB’s Third Reference is a refusal to the right to equal
protection of the respondents - NO

RULING: SC granted the petition and set aside the CA ruling.

RATIO:
LTFRB did not commit grave abuse of discretion.
Stronghold’s petition before the CA did not claim that LTFRB lacked
jurisdiction to implement the Program or to issue References for
each round of bidding. Rather, it rested its case on the theory that
LTFRB acted with grave abuse of discretion amounting to lack or
excess of jurisdiction when LTFRB required in the 3 rd Reference a
minimum capital requirement on a “per insurer” basis.
The SC held that it is undisputed that Stronghold id not meet the
minimum capitalization required for a leadinsurer under the 3 rd
Reference, leaving LTFRB no choice but to disqualify it.
To find fault in its exclusion, Stronghold pins it to LTFRB, alleging that
they committed grave abuse in abandoning the aggregated mode to
reckon compliance with the minimum capitalization requirement
under the 1st and 2nd References and in adopting the new non-
aggregated, “per insurer” basis under the 3rd Reference.
Stronghold questions the change in the maner by which the minimum
capitalization of lead and member insurers is determined under the
3rd Reference.
The 3rd Reference, is simply te result of LTFRB’s proper exercise of its
power under its charter to “formulate, promulgate, administer,
implement, and enforce rules and regulations on land transportation
public utilities”.
SC found the scheme to be reasonable, as it ensures that the accredited
prviders are able to cover all potential claims arising out of the
insurance policies pursuant to the program, for the protection of the
general riding public.
As a measure partaking of the state’s power to promote public safety and
public welfare, the 3rd Reference need only be tested by its liberal
standard of reasonableness.
SPOUSES DACUDAO v. DOJ (DANNAH) Their right to the speedy disposition of cases
January 8, 2013 | Bersamin, J. | Equal Protection Clause They also charge as unconstitutional the issuance of a DOJ
Memorandum exempting from the coverage of DO 182 all the cases
PETITIONER: Spouses Augusto and Ofelia Dacudao for syndicated estafa already filed and pending in the OCP of
RESPONDENTS: Raul Gonzales, incumbent Secretary of Justice Cagayan de Oro.
They aver that said DOJ Memo violated their right to equal protection.
SUMMARY: Spouses Dacudao filed a number of charges for syndicated
estafa against Delos Angeles et al. for allegedly defrauding them through ISSUE/s:
the Legacy Group of Company’s “buy back agreement”. These were filed WoN petitioners properly brought their petition directly to the Court – NO
with the Office of the City Prosecutor in Davao. The Secretary of Justice WoN the Secretary of Justice committed grave abuse of discretion – NO
issued Department Order No. 182 directing all prosecutors to forward all WoN DO 182 and the DOJ Memorandum violate their constitutionally
cases already filed against Delos Angeles et al. to the Secretariat of DOJ. guaranteed rights? – NO
He filed another Memorandum exempting the Office of the City Prosecutor
RULING:. WHEREFORE, the Court DISMISSES the omnibus petition for
of CDO from following DO No. 182
certiorari, prohibition, and mandamus for lack of merit. Petitioners shall pay
Petitioners then filed a petition for certiorari, prohibition and mandamus the costs of suit.
directly to the Court. The Court ruled that this was improper, but proceeded
on ruling on the constitutionality of said DO and DOJ Memo. The Court held RATIO:
that these were not violative of the spouses’ constitutional right to the equal On issue #1: The spouses unduly disregarded the hierarchy of courts
protection of the law because the spouses failed to prove otherwise. Writ of certiorari – available only when any tribunal board or
officer exercising judicial or quasi-juidical functions has acted
DOCTRINE: Hence, the Court has affirmed that if a law neither burdens a
without or in excess of its/his jurisdiction, with grave abuse.
fundamental right nor targets a suspect class, the classification stands as
Writ of mandamus – remedy available only when “any tribunal,
long as it bears a rational relationship to some legitimate government end.
corporation, board, officer or person unlawfully neglects the
performance of an act which the law specifically enjoins….
On issue #2: DO 182 was issued pursuant to Department Order No. 84
FACTS:
that the Secretary of Justice had promulgated to govern the
Spouses Dacudao who are residents of Davao were among the investors
whom Celso Delos Angeles and his associates in the Legacy Group performance of the mandate of the DOJ to “administer the criminal
of Companies allegedly defrauded through the group’s “buy back justice system in accordance with the accepted processes thereof.”
agreement” that earned them check payments that were dishonored. On issue #3: Petitioners attack the exemption from the consolidation
After their written demands for the return of investments were unheeded, decreed in DO 182 of the cases filed or pending in the OP of CDO
they initiated a number of charges for syndicated estafa against claiming that the exemption traversed the constitutional guaranty in
Delos Angeles et al. in the Office of the City Prosecutor (OCP) of
their favor of the equal protection of law.
Davao.
On March 2009, the Secretary of Justice issued DO No. 182 directing all The exemption is covered by the assailed DOJ Memo of March 2009:
Prosecutors (Regional, Provincial and City) to forward all cases “It has come to the attention of the undersigned that cases for
already filed against Delos Angeles et al. to the Secretariat of the syndicated estafa were filed with your office against officers of the
DOJ Special Panel in Manila for appropriate action. Legacy Group of Companies. Considering the distance of the place
Pursuant to DO 182, the complaints of the spouses were forwarded by of complainants therein to Manila, your Office is hereby exempted
the OCP of Davao to the Secretariat of the Special Panel of the DOJ. from the directive previously issued by the undersigned requiring
Aggrieved by such turn of events, the spouses directly came to the Court prosectution offices to forward the records of all cases involving
via petition for certiorari, prohibition and mandamus ascribing Legacy Group of Companies to the Task Force.
grave abuse of discretion in the issuance of DO 182. Anent the foregoing, you are hereby directed to conduct preliminary
They claim that said DO violated: investigation of all cases involving the Legacy Group of Companies
Their right to due process filed in your office with dispatch and to file the corresponding
Their right to the equal protection of the laws informations if
evidence warrants and to prosecute the same in court.”
The equal protection clause does not require universal application of the
laws to all persons or things without distinction; what it requires is
simply equality among equals as determined according to a valid
classification.
Hence, the Court has affirmed that if a law neither burdens a
fundamental right nor targets a suspect class, the classification
stands as long as it bears a rational relationship to some legitimate
government end.
That is the situation at hand. In issuing the assailed DOJ Memo, the
Secretary of Justice took into account the relative distance
between CDO, where many complainants against the Legacy Group
resided, and Manila, where the preliminary investigations would be
conducted.
He also took into account that the cases had already been filed in the
City Prosecutor’s Office of Cagayan de Oro at the time he issued DO
No. 182.
Given the considerable number of complainants residing in CDO, the
Secretary was fully justified in exluding the cases commenced in
CDO from the ambit of DO 182.
The classification taken into consideration by the Secretary was really valid.
On the right to speedy disposition: the Court has clarified that
although the Constitution guarantees the right to the speedy
disposition of cases, such speedy disposition is a flexible concept.
On retroactivity: All procedural laws are retroactive in that sense and to
that extent. The retroactive application is not violative of any right of
a person who may feel adversely affected, for, verily, no vested right
generally attaches to or arises from procedural laws.
On obstruction of justice: Unsubstantiated.
The authority of the Secretary of Justice to assume jurisdiction over
matters involving the investigation of crimes and the prosecution of
offenders is fully sanctioned by law.
GOLDENWAY MERCHANDISING v. EQUITABLE PCI (IYA) trial. The attempt of GMC to redeem was already late and there was
March 13, 2013 | Villarama Jr., J. | Equal Protection Clause no valid redemption made because the counsel did not have
authority to transact on
PETITIONER: Goldenway Merchandising Corporation
RESPONDENTS: Equitable PCI Bank

SUMMARY: Goldenway Merchandising failed to pay its loan obligation to


Equitable PCI Bank, which lead to the foreclosure of his mortgaged
property. GMC wanted to redeem said property, but PCI refuses to grant
such requisition on the ground that the right to redemption has lapsed
because the property was already registered. GMC assails Section 47 of
RA 8791 as violative of the equal protection clause as it discriminates
juridical persons.

Lower courts and SC all agree that the questioned provision is a valid
exercise of police power.

DOCTRINE: The freedom to contract is not absolute; all contracts and all
rights are subject to the police power of the State and not only may
regulations which affect them be stablished by the State, but all such
regulations must be subject to change from time to time for the general well-
being of the community.

FACTS:
Goldenway Merchandising Corporation (GMC) executed a Real Estate
Mortgage in favor of Equitable PCI Bank over its real properties situated
in Bulacan. The Mortgage secured the P2M loan granted by PCI Bank to
GMC.
GMC failed to settle its loan obligation, which lead to PCI Bank
extrajudicially foreclosing the mortgaged property.
During public auction, the property was sold to PCI Bank for P3.5M. The
Certificate of Sale was registered and inscribed on February 16,
2001.
In March 8, 2001 the counsel of GMC wrote seeking to redeem the
foreclosed properties by tendering a check in the amount of P3.5M.
The same was reiterated to the counsel of PCI Bank in March 12.
Counsel of PCI Bank informed GMC that such redemption is no longer
possible because the certificate of sale had already been registered.
On December 7, 2001, PCI filed a complaint for specific performance
and damages against PCI asserting that it is the one-year period of
redemption under Act No. 3135 which should apply and not the
shorter redemption period provided in Section 47 of RA 8791.
GMC argues that applying Section 47 would result in the impairment of
obligation of contracts and violation of the equal protection clause.
RTC rendered a decision dismissing the complaint. It noted that the issue
of constitutionality was never raised by petitioner during pre-trial and
behalf of the company.
GMC appealed to the CA. The CA affirmed the RTC decision saying that
GMC failed to justify why Section 47 of RA 8791 should be declared
unconstitutional. The MR was also denied.
Hence, the present petition.

ISSUE/s:
WON Section 47 of RA 8791 is violative of the proscription against
impairment of obligations of contract and equal protection clause. -
NO

RULING: Petition is DENIED.

RATIO:
Under RA 8791, juridical persons are allowed to exercise the right of
redemption only until, but not after, the registration of the certificate
of foreclosure sale and in no case more than three months after
foreclosure, whichever comes first.
The purpose of the non-impairment clause of the Constitution is to safeguard
the integrity of contracts against unwarranted interference by the State.
Section 47 did not divest juridical persons of the right to redeem their
foreclosed properties but only modified the time for exercise of such
right by reducing the one year period originally provided in Act 3135.
The contention that Section 47 is violative of the equal protection clause
as it discriminated mortgagors/property owners who are juridical
persons does not have merit.
Equal protection permits reasonable classification. Classes may be
treated differently form another where groupings e based on
reasonable and real distinctions.
The legislature clearly intended to shorten the period of redemption for
juridical persons. The difference in treatment of juridical persons and
natural persons was based on the nature of the properties
foreclosed.
Residences are allowed the more liberal one year redemption
period
Industrial or commercial property is reduced to a shorter
redemption term to reduce the period of uncertainty in
ownership of property and enable mortgage-banks to
dispose sooner of the acquired assets. This is to ensure the
solvency and liquidity of banks as provided in the General
Banking Law of 2000.
The right of redemption being statutory must be exercised in the manner
prescribed by law.
The freedom to contract is not absolute; all contracts and all rights are
subject to the police power of the State and not only may regulations
which affect them be stablished by the State, but all such regulations
must be subject to change from time to time for the general well-being of
the community.
ESPINAS v. COA (ELIEL) According to the AOM, this violated the CoA Circular which states that the
April 1, 2014 | Perlas-Bernabe, J. | Equal Protection Clause “claim for reimbursement of such expenses shall be supported by
receips
PETITIONER: Arnaldo M. Espinas, Lillian N. Asprer, Eleanora De Jesus
RESPONDENTS: Commission on Audit

SUMMARY: Espinas et al, LWUA officials, were trying to claim


reimbursement by presenting “certifications.” The CoA auditor denied the
reimbursement and issued a notice of disallowance because it is a violation
of the CoA Circular, which requires that the disbursement should be
supported by a receipt or other similar supporting documents. Espinas
appealed the notice of disallowance, contending that under Sec 397
“certifications” are valid documents to present for disbursement and that the
Circular is violative of equal protection clause because NGAs are allowed to
claim reimbursement using such “certifications.” The SC ruled that (1) there is
a substantial distinction between “certifications” and “receipts”; the claimant
issues the former, and usually self-seriving while the latter is issued by a third
person. Moreover, the Sec 397 of GAAM applies to NGAs while the CoA
Circular applies to the GOCC. (2) there is no violation of equal protection
because there is a substantial difference between GOCC officials and NGAs.
EME of GOCC’s are usually allocated by the internal governing board; on the
other hand, EME of NGAs are duly enacted in law by Congress.

DOCTRINE: The CoA is endowed with enough latituted to determine,


prevent, and disallow irregular, unnecessary, excessive, extravagant or
nconscionable expenditures of government funds. It is tasked to be
vigilant and conscientious in safeguarding the proper use of the
government’s, and ultimately, the people’s property.

FACTS:
The Local Waater Utilities Administration (LWUA) is a government-owned
and controlled corporation (GOCC) created pursuant to PD 198.
Petitioners are deparment managers of the LWUA who, together with 28
other LWUA officials, sought reimbursement of their extraordinary
and miscellaneous expenses for the period Jan-Dec 2006.
The Office of the CoA Auditor, through Priscilla Cruz, the Supervising
Auditor assigned to the LWUA (SA Cruz), issued Audit Observation
Memorandum (AOM) No. AOM-2006-27, revealing that the 31 LWUA
officials were able to reimburse P16,900,705.69 in EME, including
expenses for official entertainment, service, awards, gifts and
plaques, membership fees, and seminars/conferences. Out of the
said amount, P13,110,998.26 was reimbursed only through an
attached certification attesting to their claimed incurrence
(“certification”).
and/other documents evidencing disbursements.” The Court also rejected Espinas’ invocation of the provision of Sec
LWUA management officials manifested that they were unawareof the 397 of the GAAM, since such rules are applicable only to NGAs,
existence of the Circular. and not to
SA Cruz issued Notice of Disallowance, disallowing EME reimbursement
claims of the 31 LWUA officials, P13,110,998.26, for the reason that they
“were not supported by receipts and/or documents evidencing
disbursements.
Espinas et al appealed the notice of disallowance to the CoA Cluster
Director, contending that the “certification” they attached in support of
their EME reimbursement claims was originally allowed under Sec
397 of the Government Accounting and Auditing Manual (GAAM).
Espinas also alleged that the Circular is violative of the equal protection
clause since officials of GOCCs, such as the LWUA officials, are,
among others, prohibited by virtue of the same issuance form
supporting their reimbursement claims with “certifications,” unlike
officials of the national government agencies (NGAs) who have been
so permitted.
The CoA Cluster Division denied petition stating that “certification” is not
of the same class as receipts evidencing disbursements.” Also that
no violation of equal protection clause since GOCCs and GFIs are
empowered to appropriate EME through board resolutions, while the
EME for NGAs must be provided in a law enacted by Congress.
Accordingly, there is a reasonable classification, which is germane to
the purpose of the Circular.
Espinas appealed to the CoA, which affirmed the decision of the Cluster
Division. Hence the appeal.

ISSUE/s:
WoN Certifications are evidences of disbursements – NO
WoN the CoA Circular is violative of equal protection - NO

RULING: Petition Ganted.

RATIO:
The Court concurs with the CoA’s conclusion that the “certification”
submitted by Espinas cannot be properly considered as a supporting
document within the purview of the Circular.
That said, it logically follows that Espinas’ “certification,” so as to fall under
the phrase “other documents” under the CoA Circular, must substantiate
the “paying out of an account payable,” or, in simple term, a
disbursement.
The signatory merely certifies that he/she has spent, within a
particular month, a certain amount of meetings, seminars,
conferences, official entertainment, public relations, and the
like, and that the certified amount is within the ceiling
authorized under the LWUA corporate budget.
GOCCs, GFIs and their subsidiaries which are specifically governed by
the CoA Circular.
The Court upholds the CoA’s finding that there exists a substantial
distinction between officials of NGAs and the officials of
GOCCs, GFIs and their subsidiaries which justify the peculiarity
in regulation.
Since the EME of GOCCs, GFIs and their subsidiaries, are, pursuant to
law, allocated by their own internal governing boards, as opposed
to the EME of NGAs which are appropriated in the annual GAA duly
enacted by Congress, there is a perceivable rational impetus for
the CoA to impose nuanced control measures to check if the EME
disbursements of GOCCs, GFIs, and their subsidiaries constitute
irregular, unnecessary, excessive, extravagant, or unconscionable
government expednitures.
Case in point is the LWUA Board of Trustees which, pursuant to Sec
69 of PD 198, as amened, is “authorized to approprate out of
any funds of the Administration, such amounts as it may deem
necessary for the operation and other expenses of the
Administration including the purchase of necessary equipment.
IMBONG v. OCHOA (JP) that promotes contraceptive use. The petitioners argue that, rather
April 8, 2014 | Mendoza, J. | Equal Protection Clause than promoting

PETITIONER: James M. Imbong and spouse Lovely-Ann Imbong, et al.


RESPONDENTS: Executive Secretary Hon. Paquito Ochoa

SUMMARY: Upon the enactment of the RH Law, several petitioners for


certiorari and prohibition were filed requesting to maintain the status quo.
Of the different rights that the petitioner claim that the RH Law violates,
they claim that RH Law violates the equal protection clause for
discriminating against the poor. Petitioners claim that the RH law seeks
to introduce contraceptives to the poor to reduce their number. SC
declares that the RH Law is not violative of the due process clause
because there is a distinct necessity in the constitution to address the
needs of the underprivileged. RH Law is constitutional, with exceptions.

DOCTRINE: According to a long line of decisions, equal protection simply


requires that all persons or things similarly situated should be treated
alike, both as to rights conferred and responsibilities imposed. It,
however, does not require the universal application of the laws to all
persons or things without distinction.

FACTS:
Republic Act (R.A.) No. 10354, otherwise known as the Responsible
Parenthood and Reproductive Health Act of 2012 (RH Law), was
enacted by Congress on December 21, 2012.
The RH Law was enacted to provide Filipinos, especially the poor and
the marginalized, access and information to the full range of modem
family planning methods, and to ensure that its objective to provide
for the peoples' right to reproductive health be achieved.
To make it more effective, the RH Law made it mandatory for health
providers to provide information on the full range of modem family
planning methods, supplies and services, and for schools to provide
reproductive health education.
To put teeth to it, the RH Law criminalizes certain acts ofrefusals to carry
out its mandates.
Shortly after the President signed the law, 14 petitions and 2 petitions-in-
intervention were filed. These petitions are mostly for certiorari and
prohibition.
Petitioners allege that the RH Law violates the right to equal protection
of the law. It is claimed that the RH Law discriminates against the
poor as it makes them the primary target of the government program
reproductive health among the poor, the RH Law seeks to introduce services available to all the people at affordable cost. There
contraceptives that would effectively reduce the number of the poor. shall be priority for the needs of
Note: Equal protection is only one of the many rights the RH Law
violated according to the petitioners. In the petitions, RH Law was
also alleged to violate: right to life of the unborn, right to halth, right
to religious freedom, involuntary servitude, due process, right to
privacy, non-delegation of legislative authority, local autonomy.
Meanwhile, the RH-IRR took effect.
After considering the issues, the Court issued a Status Quo Ante Order
(SQAO), enjoining the effects and implementation of the assailed
legislation for 120 days. After the lapse of time, the SQAO was
ordered extended. Petitioners claim the maintenance of the status
quo.
ISSUE/s:
WoN the RH Law violated the equal protection clause - NO

RULING: Petition is partly granted. The Court declares the RH Law as


constitutional except with the enumerated provisions (no relation to EPC).

RATIO:
The petitioners claim that the RH Law violates the equal protection
clause under the Constitution as it discriminates against the poor
because it makes them the primary target of the government
program that promotes contraceptive use.
They argue that, rather than promoting reproductive health among the
poor, the RH Law introduces contraceptives that would effectively
reduce the number of the poor.
They add that the exclusion of private educational institutions from the
mandatory reproductive health education program imposed by the
RH Law renders it unconstitutional.
According to a long line of decisions, equal protection simply requires that all
persons or things similarly situated should be treated alike, both as to
rights conferred and responsibilities imposed. It, however, does not
require the universal application of the laws to all persons or things
without distinction.
To provide that the poor are to be given priority in the government's
reproductive health care program is not a violation of the equal
protection clause. In fact, it is pursuant to Section 11, Article XIII
of the Constitution which recognizes the distinct necessity to
address the needs of the underprivileged by providing that they
be given priority in addressing the health development ofthe
people.
Section 11. The State shall adopt an integrated and
comprehensive approach to health development which shall
endeavor to make essential goods, health and other social
the underprivileged, sick, elderly, disabled, women, and distinction why they should not be considered exempt from the
children. The State shall endeavor to provide free mandates of the law.
medical care to paupers. The protection accorded to other conscientious objectors should equally
It should be noted that Section 7 of the RH Law prioritizes poor and
marginalized couples who are suffering from fertility issues and
desire to have children. There is, therefore, no merit to the
contention that the RH Law only seeks to target the poor to
reduce their number.
While the RH Law admits the use of contraceptives, it does not, as
elucidated above, sanction abortion. As Section 3(1) explains, the
"promotion and/or stabilization of the population growth rate is
incidental to the advancement of reproductive health."
While the petitioners surmise that the assailed law seeks to charge couples
with the duty to have children only if they would raise them in a truly
humane way, a deeper look into its provisions shows that what the law
seeks to do is to simply provide priority to the poor in the
implementation of government programs to promote basic
reproductive health care.
With respect to the exclusion of private educational institutions from the
mandatory reproductive health education program under Section 14,
suffice it to state that the mere fact that the children of those who are
less fortunate attend public educational institutions does not amount
to substantial distinction sufficient to annul the assailed provision.
On the other hand, substantial distinction rests between public
educational institutions and private educational institutions,
particularly because there is a need to recognize the academic
freedom of private educational institutions especially with
respect to religious instruction and to consider their sensitivity
towards the teaching o f reproductive health education.
Minor mention of the Equal Protection Clause is also mentioned in another
section (but not part of the actual EPC Header in the case): The last
paragraph of Section 5.24 of the RH-IRR (implementing rules of the RH
Law) reads:
Provided, That skilled health professional such as provincial,
city or municipal health officers, chiefs of hospital, head nurses,
supervising midwives, among others, who by virtue of their office
are specifically charged with the duty to implement the provisions
of the RPRH Act and these Rules, cannot be considered as
conscientious objectors.
This is discriminatory and violative ofthe equal protection clause. The
conscientious objection clause should be equally protective of the
religious belief of public health officers. There is no perceptible
apply to all medical practitioners without distinction whether they
belong to the public or private sector. After all, the freedom to believe
is intrinsic in every individual and the protective robe that guarantees its
free exercise is not taken off even if one acquires employment in the
government.
Del Catillo and Reyes, Concurring and Dissenting: I agree with the
ponencia that the RH Law does not violate the equal protection clause
insofar as it is claimed to single out the poor to reduce their numbers and
that the poor may be the subject of government subsidy for the programs
under the RH Law for reasons stated in ponencia.
DISINI v. SECRETARY OF JUSTICE (HENRY) Prevention Act.
February 18, 2014 | Abad, J. | Equal Protection Clause But, petitioners Disini et al claim that the means adopted by the
cybercrime law for regulating undersirable cyberspace violate their
PETITIONER: Jose Jesus M. Disini, et al. constitutional rights,
RESPONDENTS: The Secretary of Justice, et al.

SUMMARY: Cybercrime law is the government’s way to regulate the


access and use of the cyberspacec, which prohibits acts which may be
abused by ill-minded people with the agenda of destroying or destructing
or violating other people’s rights. Petitioners in this case assail the
provisions for their violation of the equal protection clause. In particular,
the provision on cyber squatting and the provision that states that those
who commit violations of the cybercrime law are punished with penalties
of one degree higher.

The SC ruled, that these provisions are valid, particularly because the
law is reasonable for punishing a person, whether or not he uses his real
name, when in bad faith takes the domain name to profit. Also, that the
law is valid because there is a substantial distinction between crime
scommitted in the light if information and technology (larger scope of
victims, identity may be concealed) and that of normal crimes, and
hence, a penalty of a higher degree is valid.

DOCTRINE: The law is reasonable in penalizing him for acquiring the


domain name in bad faith to profit, mislead, destroy reputation, or deprive
others who are not ill-motivated.

There exists a substantial distinction between crimes committed through


the use of information and communciations technology and similar crimes
using other means. The former allows a wider range of victims and often
the offender may evade identification. The distinction therefore, creates a
basis for higher penalties for cybercrimes

FACTS:
Cybercrime law aims to regulate access to and use of the cyberspace.
Using his laptop or computer, a person can connect to the internet, a
system that links him to other computers and enables him to do a lot
of things.
This is cyberspace, a system that accommodates millions and billions of
simultaneous and ongoing individual accesses to and uses of the
internet. The cyberspace is a boon to the need of the current
generation for greater information and facility of communication. But
all Is not well with the system since it oculd not filter out a number of
persons of ill will to who would want to use cyberspace technology
for mischiefs and crimes.
For this reason, government has a legitimate right to regulate the use of
cyberspace and contain and punish wrongdoings, hence, the
Cybercrime
specificially equal protection.
Petitioners challenge the constitutionality of the following provisions of
the cybercrime law that regard certain acts as crimes and impose
penalties for their commission as well as provisions that would
enable the government to track down and penalize violators:
Provisions on: Illegal Access, Data Interference, Cyber-
squatting, Identity Theft, Cybersex, Child Pornography,
Unsolicited Commercial Communications, Libel, Aiding or
abetting and attempt in the commission of cybercrimes,
penalty of one degree higher, Prosecution of both the RPC
and RA 10175, Penalties, Real-time Collection of Traffic
Data, preservation of Computer Data, Disclosure of
Computer Data, Search Seizure and Examinataion of
Computer Data, Destruction of Computer Data,
Restricting/Blocking Access to Computer Data, Obstruction
of Justice, Cybercrime Investigation and Coordinating Center
(CICC), and CICC’s powers and functions.
Hence, this petition.

ISSUE/s:
WoN the Cybercrime Prevention Act violates the constitutional rights on
equal protection - NO

RULING: SC

RATIO:
On Illegal Access: petitioners contend that this provision fails to meet
the strict scrutiny standard required of laws that interfere with the
fundamental rights of the people and should thus be struck down. SC
ruled that strict scrutiny is not applicable, as there is no fundamental
freedom, like speech, involved in punishing what is essentially a
condemnable act—accessing the computer system of another
without right.
On Data Interference: petitioners claim that this provision suffers
overbreath, that while it seeks to discourage data interference, it
intrudes into the area of protected speech and expression, creating a
chilling and deterrent effect on these guaranteed freedoms. SC ruled
that this provision does not encroach the area of protected freedoms,
which essentially what being overbreadth is, as it simply punishes
what is a form of vandalism. Definitely, there is no freedom to destroy
other people’s computer systems and private documents.
On Cyber-Squatting: petitioners claim this provision violative of the equal
protection clause, causing a user using his real name to suffer the same
fate as those who use aliases or take the name of another. They claim
that, considering the substantial distinction between the two, the law
should recognize the difference. The SC ruled that there is no difference
if someone uses a real or a fake name. The law is reasonable in
penalizing him for acquiring the domain name in bad faith to profit,
mislead, destroy reputation,
or deprive others who are not ill-motivated. may evade identification. The distinction therefore, creates a
On Identity Theft: petitioners claim that this provision violates due basis for higher penalties for cybercrimes.
process and right to privacy and correspondence, and transgresses On real-time collection of traffic data: petitioners assail the grant to
freedom of the press. SC ruled that the petitioners failed to show law enforcement agencies of the power to collect or record traffic
how government effort to curb computer-related identity theft violates data in real time
such rights.
On Cybersex: petitioners claim that this provision violates freedom of
expression. SC ruled that per Nogales v. People, the state can
regulate materials that serve no other purpose than satisfy the
market for violence, lust, or pornography. The SC weighed the
property rights of individuals against the public welfare. Private
roperty, if containing pornographic materials, may be forfeited and
destroyed.
On Child Pornography: petitioners claim that this provision expands the
scope of the Anti-Child Pornography Act of 2009 (ACPA) to cover
identical activities in cybrerspace, making the penalty higher by one
degree when the crime is committed in cyberspace. SC ruled that the
constitutionality of the provision is not successfully challenged.
On Unsolicited Commercial Communications: this provision, which
penalzies spammers, is being assailed to violate freedom of expression,
that there are no proven facts that accord spammers to reduce the
efficiency of computers. The SC ruled in the petitioner’s favor, indeed,
the State cannot rob him of his right without violating the constitutionally
guaranteed freedom of expression. Unsolicited advertisements are
legitimate forms of expression.
On libel: petitioners claim that libel violates the country’s obligations
under ICCPR. The SC ruled that libel is nto a constitutionally
protected speech and that the government has an obligation to
protect private individuals from defamation. Indeed, cyberlibel
isactually not a new crime since the RPC already punishes it. In
effect, the cybercrime law merely affirms that online defamation
constitutes similar means of committing libel.
On Aiding or abetting and attempt in the commission of
cybercrimes: petitioners assail the constitutionality of this provision
on the grounds of overbreadth, creating a chilling and deterrent effect
on protected expression. SC ruled that libel in the cyberspace can of
course stain a person’s image with just one click. It goes hand in
hand with cyberbullying, among others. Still, a governmental
purpose, which seeks to regulate the use of this cyberspace
communication technology to protect a person’s reputation and
peace of mind, cannot adopt means that will unnecessarily and
broadly sweep, invading the area of protected freedoms.
On the penalty being one degree higher: petitioners assail that this is
violative of the constitutional right to due processs. SC cited the Sol
Gen’s argument, that there exists a substantial distinction between
crimes committed through the use of information and
communciations technology and similar crimes using other means.
The former allows a wider range of victims and often the offender
as tending to curtail civil liberties or provide opportunities for official
abuse. SC ruled that the State has a compellinginterest in enacting the
cybercrime law for there is a need to put order to the tremendous
activities in cyberspace for public good.
On preservation of computer data: petitioners claim that this constitutes undue
deprivation of right to property. SC ruled that data on service providers
preserve on orders of law enforcement authorities are not made inaccessible
to users by reason of issuance of such orders. The process of preserving
data will not unduly hamper normal transmission or use of the same.
On disclosure of computer data: petitioner claim that this is similar to a
subpoena, which is a judicial function. SC ruled that executive agencies
have the power to issue subpoena as an adjuct of their investigatoruy
powers. The prescribed procedure for disclosure would not constitute an
unlawful search or seizure nor would it violate the privacy of
communications and correspondence.
On search, seizure, and examination of computer data: petitioners claim
that it will supplant established search and seizure procedures. SC ruled
that IT does not pose any threat on rights of theperson whom the
information or data were taken. It does not supersede existing search
and seizure rules but merely supplements them.
On destruction of computer data: petitioner claims tha that such
destruction of computer data subject of previous preservation or
examination violates the user’s right against deprivation of property
without due process. SC ruled that it is unclear that the user has a
demandable right to require the service provider to havethat copy fothe
data saved indefinitely for him in its storage system.
On Restricting/Blocking Access to Computer Data: petitioner contends
that this provision violates rights against unreasonable searches and
seizures. SC ruled that it is violative of the constitutional guarantees to
freedom of expression and against unreasonable searches and seizures,
hence shall be struck down.
On obstruction of justice: petitioners claim this t be a bill of attainder. Mere
failure to comply constitutes a legislative finding of guilt withut regard to
situations where non-compliance would be reasonable or valid. SC ruled
that this provision is valid.
On CICC: petitioners contend that it there is undue delegation when Congress
gave CICC power to formulate a national cybersecurity planwithout any
sufficient standards or parameters for it to follow. SC ruled that the
cybercrime law is complete in itself, and that, it provided sufficient standards
for the CICC to follow when it provided a definition of cybersecurity.
PEOPLE OF THE PHILIPPINES v. JUMAWAN (DANNAH) affairs. He also claimed that she fabricated the rape charges
April 21, 2014 | Reyes, J. | Equal Protection Clause because of revenge. Allegedly, when he took over the business, their
money dwindled from P3M to a few thousands.
PETITIONER: People of the Philippines Their driver Equia also corroborated the claims of Jumawan that he was
RESPONDENTS: Edgar Jumawan in Bukidnon peeling corn when these events occurred.
What further confirmed his hunch was when one of their children OOO
SUMMARY: KKK claims that her husband, Edgar Jumawan, raped her told him KKK was better off with her boyfriend who was young,
twice. He started being brutal to her in bed since 1997, and two Informations handsome and a businessman compared to him who smelled bad,
for rape were filed before the RTC. Jumawan’s defense was that KKK’s and was old.
stories were fabricated and she actually had paramours. He also thinks that The RTC ruled in favor of the prosecution because of the spontaneous
sexual community is a legal right between husbands and wives. and straightforward testimonies, and as it is not natural in our culture
for daughters to testify against their own father for a crim such as
The RTC and CA ruled in favor of KKK. The SC likewise affirmed the rulings, rape.
stating that in our jurisdiction, there is no longer a difference between The CA affirmed the RTC decision in toto, because all the elements of
nonmarital and marital rape. To allow distinction would be a violation of the rape were proven beyond reasonable doubt. The CA also held that
equal protection of laws afforded to each individual. physical showing of external injuries was not indispensable.
Moreover, it noted that the fact that KKK and Jumawan are spouses
DOCTRINE: The Constitutional right to equal protection of the laws ordains only reinforces the truthfulness of KKK’s accusations because no
that similar subjects should not be treated differently, so as to give undue wife in her right mind would accuse her husband of having raped her
if it were not true.
favor to some and unjustly discriminate against others; no person or class of
Hence, the present review.
persons shall be denied the same protection of laws, which is enjoyed, by
other persons or other classes in like circumstances. ISSUE/s:
WoN there is a substantial distinction between non-marital and marital
rape
FACTS: – NO
Jumawan and KK were married on October 1975, and have 4 children.
They put up several businesses over the years. RULING:. Ruling of the CA and RTC is affirmed. Jumawan is found guilty
On February 1999, KKK executed a Complaint-Affidavit alleging that beyond reasonable doubt of 2 counts of rape, to suffer the penalty of
Jumawan raped her at 3am of December 3, 1998 and that he boxed reclusion perpetua for each count.
her shoulder on December 12 for refusing to have sex with him.
Two Informations for rape were filed before the RTC. RATIO:
Version of the Prosecution: KKK claimed that Jumawan used to treat According to history, the evolution of rape stems from the fact that a
her well but sometime in 1997 he started being brutal in bed, and woman was the property of her father until she marries to become
even told her to stop handling the businesses and stay home instead
the property of her husband. If a man aducted an unmarried woman,
because wives should serve and be good in bed.
On October 16, 1998, Jumawan forced KKK to lay beside him and urged he had to pay the owner, and later buy her from the owner; buying
her to have sex with him. When she said she wasn’t feeling well, he and marrying a wife were synonymous.
forced her panties forcefully. Their daughter MMM heard the In the 17th Century, Sir Matthew Hale, a Chief Justice in England
commotion and went to comfort her mother. conceived the implied consent theory that would later on emerge
The following night, the aggression recurred. KKK wanted to sleep in as the marial exemption rule in rape. He stated that the husband
their daughter’s room but was forced by Jumawan. He even cannot be guilty of rape as through their mutual matrimonial consent
attempted to carry her out the room and when she defied, he the wife had given up herself unto her husband.
grabbed her short pants causing them to tear apart.
He claimed that he did not care that he was doing it in front of their children.
In the 1970s the rule was challenged by woen’s movements in the USA
Version of the Defense: Jumawan claimed that KKK had extra-marital demenading for its abolition for being violative of married women’s
right to be equally protected under the rape law.
In 1983, the marital exemption rule was abandoned in New York for
being unconstitutional. By 1993, marital rape was a crime in all 50
states.
The case at hand is the first case on marital rape in the Philippines.
The old provisions on rape adhered to Hale’s irrevocable implied consent rape his wife with impunity.
theory. Lastly, the human rights of women include their right to have control over
In 1981, the Philippines joined 180 contries in ratifying the United Nations and decide freely and responsibly on matters related to their
Convention on the Elimination of all Forms of Discrimination sexuality, including sexual and reproductive health, free of coercion,
Against Women (UN-CEDAW). discrimination and violence. Women do not divest themselves of
This was hailed as the first international women’s bill of rights. such right by contracting marriage for the simple reason that human
In compliance, the Philippines enshrined the principle of gender equality rights are inalienable.
in Sections 11 and 14 of the Constitution.
The Legislature then pursued the enactment of laws to propagate gender
equality.
In 1997, RA 8353 eradicated the stereotype concept of rape in Article 335.
The law reclassified rape as a crime against persons and
removed it from the ambit of crimes against chastity.
The paradigm shift on marital rape is further affirmed by RA 9262, which
regards rape within marriage as a form of sexual violence that may
be committed by a man against his wife.
Jumawan theorizes that the two incidents of sexual intercourse which
gave rise to the charges were consensual, obligatory even because
he and KKK were legally married and cohabiting couple. He also
believed that sexual community is a mutual right and obligation
between husband and wife.
Subsequent to the CEDAW, the UN Declaration on the Elimination of
Violence Against Woman was promulgated. It also identified marital
rape as a species of sexual violence.
To treat marital rape cases differently from nonmarital rape cases in
terms of the elements that constitute the crime and in the rules
for their proof, infringes on the equal protection clause.
The Constitutional right to equal protection of the laws ordains that
similar subjects should not be treated differently, so as to give undue
favor to some and unjustly discriminate against others; no person or
class of persons shall be denied the same protection of laws, which
is enjoyed, by other persons or other classes in like circumstances.
The definition of rape in Section 1 of RA 8353 pertains to
Rape, as traditionally known
Sexual assault
Marital rape or that where the victim is the perpetrator’s own spouse.
The single definition for all three forms of the crime shows that the law
does not distinguish between rape committed in wedlock and those
committed without a marriage. Hence, the law affords protection to
women raped by their husband and those raped by any other man
alike.
A marriage license should not be viewed as a license for a husband to forcibly
BARTOLOME v. SSS (IYA) ISSUE/s:
November 12, 2014 | Velasco Jr., J. | Equal Protection Clause WON Bernardina Bartolome is entitled to the death benefits claim in view
of John’s work related demise - YES
PETITIONER: Bernardina Bartolome
RESPONDENTS: Social Security System and Scanmar Maritime Services

SUMMARY: Bernardina Bartolome is the biological mother of John


Colcol. John met an accident while at work. She is seeking to claim, as
secondary parent beneficiary, death benefits from SSS. She was denied
by SSS La Unuon and Employees Compensation Commission by reason
that John has been adopted by Cornelio Colcol, therefore she does not
have parental authority over John divesting her of the claim of benefits.

The SC ruled that the ECC should not have limited the application of the
phrase “dependent beneficiaries” to legitimate parents only as it is
violative of the grant of equal protection of laws.

DOCTRINE: There is no compelling reasonable basis to discriminate


against illegitimate parents. The Court sees no pressing government
concern or interest that requires protection as to warrant balancing the
right of unmarried parents on one hand and the rationale behind the law
on the other.

FACTS:
John Colcol was an employee of Scanmar Maritime Services, Inc., on
board the vessel Maersk Danville since February 2008. He was
enrolled under the government’s Employees’ Compensation Program
(ECP).
On June 2, 2008 an accident occurred onboard the vessel where steel
plates fell on John which led to his death.
John was unmarried and had not children. Bernardina Bartolome,
petitioner and biological mother of John was his sole remaining
beneficiary. She claims to fall specifically as a secondary parent
benficiary.
Bartolome filed a claim for death benefits under PD626 with he SSS of
La Union.
SSS of La Union denied the claim saying she is not considered a parent
of John as he was adopted by Cornelio Colcol, great grandfather of
John.
She appealed to the Employees Compensation Commission (ECC)
which affirmed the ruling of SSS La Union. For ECC and SSS La
Union it is Cornelio who is qualified as John’s beneficiary. Her MR
was also denied.
Hence, the present petition.
RULING: Petition is GRANTED.

RATIO:
The ECC issued Amended Rules on Employees’ Compensation which
interpreted Art 167 of the Labor Code.
The Labor Code provided for a definition of who are beneficiaries
without classifying what a “dependent parent” is.
The Amended Rules limited the definition to “legitimate parents”.
The above stated is the basis of the ECC in denying the claim of
Bartolome saying she is no longer the legitimate parent of John.
ECC indulged in unauthorized administrative legislation. It read into Art.
67 of the Labor Code an interpretation not contemplated by the
provision.
The term parents in the phrase “dependent parents” in the labor Code is
used and ought to be taken in its general sense an cannot be unduly
limited to legitimate parents as what the ECC did.
It should include all parents, whether legitimate or illegitimate and
whether by nature or by adoption. When the law does not
distinguish, one should not distinguish.
To insist that the ECC validly interpreted the Labor Code provision is
violative of the guarantee of equal protection of laws.
There is no compelling reasonable basis to discriminate against
illegitimate parents. The Court sees no pressing government concern
or interest that requires protection as to warrant balancing the right of
unmarried parents on one hand and the rationale behind the law on
the other.
There is no justification for limiting secondary parent beneficiaries to
legitimate ones.
LEAGUE OF CITIES v. COMELEC (ELIEL) No. 1 and forwarded it to the Senate for approval. However, the
November 18, 2008 | Carpio, J. | Equal Protection Clause Senate failed to approve the Joint Resolution.
Following the advice of Sen Pimentel, 16 municipalities filed, through
PETITIONER: League of Cities of the Philippines their respective sponsors, individual cityhood bills. The 16 cityhood
RESPONDENTS: Commission on Elections bills contained a common provision exempting all the 16
municipalities from the P100M income requirement in RA 9009.
SUMMARY: Congress enacted a law converting 33 municipalites into cities; On Dec 22, 2006 the HoR approved the cityhood bills. The cityhood bills
however, 24 were only acted upon. Following the next congress, RA 9009 lapsed into law on various dates from March – July 2007 without the
was approved increasing the income requirement for conversion to a city President’s signature.
th
from P20M to P100M. A joint resolution was approved by the 13 Congress, The Cityhood Laws direct the COMELEC to hold plebiscites to determine
whether the voters in each respondent municipality approve of the
wherein 16 municipalities were exempted from the income requirement –
Cityhood Laws. This prompted the League of Cities to assail the validity of
conversion of their municipality into a city.
the Cityhood Laws approved because the exemption violates Secs. 6 and 10, League of Cities filed the present petitions to declare the Cityhood Laws
Art X of the Constitution and violative of the equal protection clause. The SC unconstitutional for violation of Sec 10, Art X of the Constitution, as
granted the petition and declared the Cityhood laws unconstitutional. As to well as for violation of the equal protection clause.
the violation of Secs. 6 and 10: Sec 6 is violated because national taxes are
ISSUE/s:
not fairly and justly distributed since the income requirement is not followed;
meanwhile for Sec 10, there is a strict requirement that the creation of a city WoN Cityhood Laws violate Sec 10, Art 10 of the Constitution– YES
should be in accordance with the written laws in the LGC, there is no WoN Cityhood Laws violate the equal protection clause- YES
exemption – thus violated. As for the equal protection clause: (1) there is no
substantial distinction between those municipalities with pending cityhood RULING: Petition Granted. Unconstitutional.
bills and those that do not have; (2) the pendency of a cityhood bill limits the
exemption to a specific condition that does not apply to future conditions; (3) RATIO:
and that this exemption is limited only to the 16 municipalities and does not RA 9009 increased the income requirement for conversion of a
apply to municipalities with the same circumstances. municipality into a city from P20M to P100M. Sec 450 of LGC, as
amended by RA 9009, does not provide any exemption from the
DOCTRINE: To be a valid classification it must (1) rest on substantial icnreased income requirement.
distinctions; (2) must be germane to the purpose of the law; (3) must Congress passed the Cityhood Laws long after the effectivity of RA 9009.
not be limited to existing conditions only; and (4) apply equally to all RA 9009 became effective on June 30 2001 or during the 11 th
members of the same class.
Congress. The 13th Congress passed in Dec 2006 the cityhood bills
which became law only in 2007. Thus League of Cities cannot invoke
the principle of non-retroactivity of laws.
FACTS: Sec 10, Art 10 requires that the creation of local government units
11th Congress: enacted into law 33 bills converting 33 municipalities into
must follow the criteria established in the LGC and not in any
cities. However, Congress did not act on bills converting 24 other
municipalities into cities. other law. There is only one LGC. The Constitution requires
th
12 Congress: enacted into law RA 9009, which took effect June 30, Congress to stipulate in the LGC all the criteria necessary for
2001. RA 9009 amended Sec 450 of the LGC by increasing the the creation of a city, including the conversion of a municpality
annual income requirement for conversion of a municipality into a into a city. Congress cannot write such criteria in any other law,
city from P20M to P100M. like the Cityhood laws.
After the effectivity of RA 9009, the HOR of the 12 th Congress adopted The clear intent of the Constitution is to insure that the creation of
Joint Resolution No. 29, exempting from the P100M income
requirement in RA 9009 the 24 municipalities whose cityhood bills cities and other political units must follow the same uniform,
were not approved in the 11 th Congress. However, the 12th Congress non-discriminator criteria found solely in the LGC. Any
ended without the Senate approving Joint Resolution No. 29. derogation or deviation from the criteria prescribed in the LGC
13th Congress: HoR re-adopted Joint Resolution No. 29 as Joint violates Sec 10, Art 10 of the Constitution.
Resolution
RA 9009 took effect on June 2001. Hence, from the moment the LGC distinguish one municipality from another for the purpose of
required that any municipality desiring to becoem a city must the income requirment.
satisfy the P100M income requirement. Sec 450 of the LGC, as
amended by RA 9009, does not contain any exemption form this
income requirement.
Such exemption clearly violates Sec 10, Art 10 and is thus patently
unconstitutional. To be valid, such exemption must be written in the
LGC and not in any other law, including the Cityhood laws.
Sec 6, Art X requires that fair and just distribution of the national
taxes to LGUs.
Since the Cityhood Laws do not follow the income criterion in Sec
450 of the LGC, they prevent the fair and just distribution of the
Internal Revenue Allotment in violation of Sec 6, Art X of the
Constitution.
Since the law is clear, plain and unambiguous that any municipality
desiring to convert into a city must meet the increased income
requirement, there is no reason to go beyond the letter of the law in
applying Sec 450 of the LGC, as amended by RA 9009.
Congress cannot create a city through a law that does not comply with
the criteria or exemption found in the LGC.
Congress is not a continuing body. The unapproved cityhood bills
filed during the 11th Conrgess became mere scraps of paper
upon the adjournment of the 11th Congress. All the hearings and
deliberations conducted during the 11 th Congress on
unapproved bills became worthless upon the adjournment of
the 11th Congress. These hearings and deliberations cannot be
used to interpret bills enacted into law in the 13 th or subsequent
Congresses.
Even if the Court takes into account the deliberations in the 11 th
Congress that municipalities with pending Cityhood bills should be
exempt from the P100M income requirement, there is still no valid
classification to satisfy the equal protection clause.
The exemption will be based solely on the fact that the 16 municipalities had
cityhood bills pending in the 11th Congress when RA 9009 was enacted.
To be a valid classification it must (1) rest on substantial
distinctions; (2) must be germane to the purpose of the law; (3)
must not be limited to existing conditions only; and (4) apply
equally to all members of the same class.
There is no substantial distinction between municipalities with
pending cityhood bills in the 11 th Congress and municipalities
that did not have pending bills. There mere pendency of a
cityhood bill in the 11th Congress is not a material difference to
The fact of a pendency of a cityhood bill in the 11 th Congress limits the
exemption to a specific condition existing at the time of passage of
RA 9009. That specific condition will never happen again. This
violates the requirement that a valid classification must not be
limited to existing conditions only.
In the same vein, the exemption provision in the Cityhood Laws gives
the 16 municipalities a unique advantage based on an abitrary date
– the filing of their cityhood bills before the end of the 11 th
Congress – as against all other municipalites that want to convert
into cities after the effectivity of RA 9009.
Furthermore, limiting the exemption only to the 16 municipality violates
the requirement that the classification must apply to all similarly
situated. Municipalities with the same income as the 16 respondent
municipalities cannot convert into cities, while the 16 respondent
municipalites can.
Clearly, as worded the exemption provision found in the Cityhod Laws,
even if it were written in Sec 450 of the LGC, would still be
unconstitutional for violation of the equal protection clause.
LEAGUE OF CITIES v. COMELEC MR (ELIEL) 13th Congress: HoR re-adopted Joint Resolution No. 29 as Joint
December 21, 2009| Velasco JR, J. | Equal Protection Clause Resolution

PETITIONER: League of Cities of the Philippines


RESPONDENTS: Commission on Elections

SUMMARY: Congress enacted a law converting 33 municipalites into


cities; however, 24 were only acted upon. Following the next congress,
RA 9009 was approved increasing the income requirement for
conversion to a city from P20M to P100M. A joint resolution was
approved by the 13th Congress, wherein 16 municipalities were
exempted from the income requirement – Cityhood Laws. This prompted
the League of Cities to assail the validity of the Cityhood Laws approved
because it is violative of the equal protection clause.

The SC granted the petition and declared the Cityhood laws Constitutional. It
is not in violation of the equal protection clause: (1) there is a substantial
distinction between those municipalities with pending cityhood bills and those
that do not have. These municipalities were already qualified prior to the
enactment of RA 9009; (2) it is germane to the purpose of the law, which is to
reduce injustice to these municipalities to be exempt from the P100M income
requirement; (3) it was not limited to existing conditions only, for as long as
the municipalities were able to submit their cityhood bill before the passage
of RA 9009 it applies; (3) and that this condition applies to those
municipalities during the 11th congress and the 16 municipalities herein, thus,
are under like circumstances.

DOCTRINE: To be a valid classification it must (1) rest on substantial


distinctions; (2) must be germane to the purpose of the law; (3) must
not be limited to existing conditions only; and (4) apply equally to all
members of the same class.

FACTS:
11th Congress: enacted into law 33 bills converting 33 municipalities into
cities. However, Congress did not act on bills converting 24 other
municipalities into cities.
th
12 Congress: enacted into law RA 9009, which took effect June 30,
2001. RA 9009 amended Sec 450 of the LGC by increasing the
annual income requirement for conversion of a municipality into a
city from P20M to P100M.
After the effectivity of RA 9009, the HOR of the 12 th Congress adopted
Joint Resolution No. 29, exempting from the P100M income
requirement in RA 9009 the 24 municipalities whose cityhood bills
were not approved in the 11 th Congress. However, the 12th Congress
ended without the Senate approving Joint Resolution No. 29.
No. 1 and forwarded it to the Senate for approval. However, the the 12 bills seeks exemption from the higher income requirement of
Senate failed to approve the Joint Resolution. RA 9009. The proponents are invoking the exemption on the basis of
Following the advice of Sen Pimentel, 16 municipalities filed, through justice and fairness.
their respective sponsors, individual cityhood bills. The 16 cityhood
bills contained a common provision exempting all the 16
municipalities from the
P100M income requirement in RA 9009.
124. On Dec 22, 2006 the HoR approved the cityhood bills. The cityhood
bills lapsed into law on various dates from March – July 2007 without
the President’s signature.
The Cityhood Laws direct the COMELEC to hold plebiscites to determine
whether the voters in each respondent municipality approve of the
conversion of their municipality into a city.
League of Cities filed the present petitions to declare the Cityhood Laws
unconstitutional for violation of Sec 10, Art X of the Constitution, as
well as for violation of the equal protection clause.

ISSUE/s:
WoN Cityhood Laws violate the equal protection clause-

NO RULING: Petition Granted. Constitutional.

RATIO:
The equal protection guarantee is embraced in the broader and elastic
concept of due process, every unfair discrimination being an offense
agaisnt the requirements of justice and fair play. It has nonetheless
come as a separate
clause in Sec. 1, Art 3 o the constitution to provide for a more specific
protection aainst any undue discrimination or antaganoism from
government.
134. As a matter of settled legal principle, the fundamental right of equal
protection does not require absolute equality. It is enough that all
person or things similarly situated should be treated alike, both as to
rights or privileges conferred and responsibilities or obligations
imposed.
The equal protection clause does not preclude the state from recognizing
and acting upon factual differences between individuals and classes.
It recognizes that inherent in the right to legislate is the right to
classify, necessarily implying that the equality guaranteed is not
violated by a legislation based on reasonable classification
Classification, to be reasonable, must (1) rest on substantial
distinctions;
(2) be germane to the purpose of the law; (3) not be limited to
existing conditions only; and (4) apply equally to all members of
the same class.
In essence, the Cityhood Bills now under consideration will have the
same effect as that of House Joint Resolution No. 1 because each of
Each of the 12 municipalities has all the requisites for conversion
into a component city based on the old requirements set forth
under Section 450 of the [LGC], prior to its amendment by RA
9009
In hindsight, the peculiar conditions, as depicted in Senator Lim’s
speech, which respondent LGUs found themselves in were
unsettling. They were qualified cityhood applicants before the
enactment of RA 9009. Because of events they had absolutely
nothing to do with, a spoiler in the form of RA 9009 supervened.
Now, then, to impose on them the much higher income requirement
after what they have gone through would appear to be indeed
"unfair," to borrow from Senator Lim
140. The classification is also germane to the purpose of the law. The
exemption of respondent LGUs/municipalities from the PhP 100 million
income requirement was meant to reduce the inequality occasioned by
the passage of the amendatory RA 9009.
From another perspective, the exemption was unquestionably designed to
insure that fairness and justice would be accorded respondent LGUs.
Let it be noted that what were then the cityhood bills covering
respondent LGUs were part and parcel of the original 57 conversion bills
filed in the 11th Congress, 33 of those became laws before the
adjournment of that Congress.
Respondent municipalities and the 33 other municipalities, which
had already been elevated to city status, were all found to be
qualified under the old Sec. 450 of the LGC of 1991 during the
11th Congress. As such, both respondent LGUs and the 33
other former municipalities are under like circumstances and
conditions.
Indeed, to deny respondent LGUs/municipalities the same rights
and privileges accorded to the 33 other municipalities when, at
the outset they were similarly situated, is tantamount to denying
the former the protective mantle of the equal protection clause.
In effect, petitioners and petitioners-in-intervention are creating
an absurd situation in which an alleged violation of the equal
protection clause of the Constitution is remedied by another
violation of the same clause.
Then too the non-retroactive effect of RA 9009 is not limited in application
only to conditions existing at the time of its enactment. It is intended to
apply
for all time, as long as the contemplated conditions obtain.
145. To be more precise, the legislative intent underlying the enactment of RA
9009 to exclude would-be-cities from the PhP 100 million criterion would
hold sway, as long as the corresponding cityhood bill has been filed before
the effectivity of RA 9009 and the concerned municipality qualifies for
conversion into a city under the original version of Sec. 450 of the LGC of
1991.
Lastly and in connection with the third requisite, the uniform
exemption clause would apply to municipalities that had
pending cityhood bills before the passage of RA 9009 and were
compliant with then Sec. 450 of the LGC of 1991, which
prescribed an income requirement of PhP 20 million.
VILLANUEVA v. JBC (JP) WoN the policy of JBC requiring five years of service as judges of first-
April 7, 2015 | Reyes, J. | Equal Protection Clause level courts before they can qualify as applicant to second-level
courts is constitutional – YES
PETITIONER: Ferdinand R. Villanueva, Presiding Judge, MCTC Bataan
RESPONDENTS: Judicial and Bar Council (JBC)

SUMMARY: Petitioner was a presiding judge of the MCTC of New Bataan.


He applied for the RTC courts of Tagum, Davao and Agusan del Sur but was
declined because the JBC held that judges, in order to be promoted to
second-level courts need to have served 5 years in a first- level court.
Petitioner alleges a violation of the equal protection of the laws. The SC held
that there is a valid classification between judges with 5-year experience and
those with not. As the JBC is mandated to promote competence in the
judiciary, it is vested with the power to impose rules such as the 5-year
requirement so as to assure that the judge being promoted to the second-tier
court is vested with sufficient professional skills.

DOCTRINE: If a law neither burdens a fundamental right nor targets a


suspect class, the classification stands as long as it bears a rational
relationship to some legitimate government end.

FACTS:
Petitioner Villanueva was appointed as Presiding Judge of the Municipal
Circuit Trial Court of Compostela-New Bataan, Compostela Valley
which is first level court.
Petitioner applied for the vacant position of Presiding Judge in the
following RTCs: Tagum City, Davao City, and Branch 6, Agusan Del
Sur.
In a letter, the JBC’s Office of Recruitment informed petitioner that he
was not included in the list of candidates for the said stations.
JBC En Banc upheld the decision not to include the petitioner due to the
JBC’s long- standing policy of opening the chance for promotion to
second-level courts to, among others, incumbent judges who have
served in their current position for at least five years, and since the
petitioner has been a judge only for more than a year, he was
excluded from the list. This caused the petitioner to take recourse to
this Court.
Petitioner argued that: (1) the Constitution already prescribed the
qualifications of an RTC judge, and the JBC could add no more; (2)
the
JBC’s five-year requirement violates the equal protection and
due process clauses of the Constitution

ISSUE/s:
RULING: Petition is dismissed. level court, is a direct adherence to the qualities prescribed by
the Constitution.
RATIO: Placing a premium on many years of judicial experience, the JBC is
The JBC is mandated to recommend appointees to the judiciary and only merely
those nominated by the JBC in a list officially transmitted to the
President may be appointed by the latter as justice or judge in the
judiciary.
In carrying out its main function, the JBC has the authority to set the
standards/criteria in choosing its nominees for every vacancy in the
judiciary, subject only to the minimum qualifications required by the
Constitution and law for every position.
The equal protection clause of the Constitution does not require the
universal application of the laws to all persons or things without
distinction; what it requires is simply equality among equals as
determined according to a valid classification. Hence, the Court has
affirmed that if a law neither burdens a fundamental right nor targets a
suspect class, the classification stands as long as it bears a rational
relationship to some legitimate government end.
The mere fact that the legislative classification may result in actual
inequality is not violative of the right to equal protection, for every
classification of persons or things for regulation by law produces
inequality in some degree, but the law is not thereby rendered
invalid.
In issuing the assailed policy, the JBC merely exercised its discretion in
accordance with the constitutional requirement and its rules that a
member of the Judiciary must be of proven competence, integrity,
probity and independence.
To ensure the fulfillment of these standards in every member of the
Judiciary, the JBC has been tasked to screen aspiring judges and
justices, among others, making certain that the nominees submitted
to the President are all qualified and suitably best for appointment.
In this way, the appointing process itself is shielded from the possibility of
extending judicial appointment to the undeserving and mediocre and,
more importantly, to the ineligible or disqualified.
Consideration of experience by JBC as one factor in choosing
recommended appointees does not constitute a violation of the
equal protection clause.
The number of years of service provides a relevant basis to
determine proven competence which may be measured by
experience, among other factors.
The assailed criterion or consideration for promotion to a second-
level court, which is five years experience as judge of a first-
applying one of the stringent constitutional standards requiring that a member
of the judiciary be of proven competence. In determining competence, the
JBC considers, among other qualifications, experience and performance.
Based on the JBC’s collective judgment, those who have been
judges of first-level courts for five (5) years are better qualified
for promotion to second-level courts. It deems length of
experience as a judge as indicative of conversance with the law
and court procedure.
Five years is considered as a sufficient span of time for one to
acquire professional skills for the next level court, declog the
dockets, put in place improved procedures and an efficient case
management system, adjust to the work environment, and gain
extensive experience in the judicial process.
A five-year stint in the Judiciary can also provide evidence of the
integrity, probity, and independence of judges seeking promotion.
It would be premature or difficult to assess their merit if they have had
less than one year of service on the bench.
At any rate, five years of service as a lower court judge is not the only
factor that determines the selection of candidates for RTC judge to
be appointed by the President.
Persons with this qualification are neither automatically selected nor do
they automatically become nominees. The applicants are chosen
based on an array of factors and are evaluated based on their
individual merits.
Clearly, the classification created by the challenged policy satisfies the
rational basis test.
The foregoing shows that substantial distinctions do exist between lower
court judges with five-year experience and those with less than five
years of experience, like the petitioner, and the classification
enshrined in the assailed policy is reasonable and relevant to its
legitimate purpose.
The Court, thus, rules that the questioned policy does not infringe on the
equal protection clause as it is based on reasonable classification
intended to gauge the proven competence of the applicants.
Brion, concurring: The application of this policy, according to the
Villanueva petition, violated the Constitution as it disregarded the
enumeration of qualifications of members of the judiciary under
Article VIII, Section 7; violated as well his due process and equal
protection rights.
Leonen, concurring: The five-year requirement imposed by the Judicial
and Bar Council for first-level court judges before they can be
considered for another tier is reasonable. This same requirement
cannot be imposed on applicants from the public service, private
practice, or the academe simply because they are not from a judicial
service.
FERRER v. BAUTISTA (HENRY) Petitioner Ferrer alleges that he is a registered co-owner of a 371-sqm
June 30, 2015 | Ponencia, J. | Equal Protection Clause residential property in QC, he paid his realty tax which included the
garbage
PETITIONER: Jose J. Ferrer, Jr.
RESPONDENTS: City Mayor Herbert Bautista, et al.

SUMMARY: QC Council enacted an ordinance on Socialized Housing


Tax, and on Garbage Fee collection annually, which is assailed by the
petitioner in the SC, with claims that it is violative of the equal protection
clause of the Constitution. The SC ruled, that the SHT is constitutional, it
following the measures on reasonable classification, and rendered
unconstitutional the garbage fee collection, as there should be no
substantial distinction between occupants of a house and lot, a socialized
housing project, and an apartment.

DOCTRINE: Equal protection requires that all persons or things similarly


situated should be treated alike, both as to rights conferred and
responsibilities imposed. The guarantee means that no person or class of
persons shall be denied the same protection of laws chich is enjoyed by
other persons or other classes in like circustances.

FACTS:
Quezon City council enacted Ordinance No. SP-2095, S-2011, or the
Socialized Housing Tax of Quezon City, with Sec 3 providing as
follows:
IMPOSITION – a special assessment equivalent to .5% on the
assessed value of land in excess of Php 100,000.00 shall be
collected by the City Treasurer which shall accrue to the
Socialized Housing Programs of the QC Government. The
special assessment shall accrue to the General Fund under
a special account to be established for the purpose.
Effective for 5 years, the Socialized Housing Tax (SHT hahahahah) shall
be utilized by the QC Government for the ff projects:
Land purchase/ land banking
Improvement of current/ existing socialized housing facilities
Land development
Construction of core houses, sanitary cores, medium-rise
buildings and other similar structures
Financing of public-private patnersheep of the WC Governemt
and the National Housing Authority (NHA) with the private
sector.
Ordinance No. SP-2235, S2013 was enacted December 2013, which
stated that proceeds collected from the garbage fees on residential
properties shall be deposited solely and exclusively in an earmarked
special account under the general fund to be utilized for garbage
collections. Section 1 of the said ordinance provided for the schedule
and manner for the collection of fees (refer to origs)
fee in sum of Php 100.00. The differentiation conforms to the practical dictates of justice and equity
Petitioner filed the instant petition in the SC on Jan 2014, and the SC and is not discriminatory within the meaning of the Constitution.
issued a TRO which enjoined the enforcement of the ordinances,
and required the respondents to comment on the petition. He is
assailing the constitutionality of the two ordinances.

ISSUE/s:
WoN SP-2235, S-2013 (annual garbage fee collection) is
unconstitutitonal -
YES
WoN SP-2095, S-2011 (Socialized Housing Tax of QC) is
unconstitutitonal
– NO

RULING: SC partially granted the petition.

RATIO:
147. On the Socialized Housing Tax: respondents emphasize that the
SHT is pursuant to the social justice principle. There is no way the
ordinance could violate the equal protection clause because real
property owners and informal settlers do not belong to the same
class.
148. On the contrary, petitioner claim that the collection of the SHT is a
kind of class legislation that violates the right to property owners to
equal protection of the laws since it favors informal settlers who
occupy property not their own and pay no taxes over law-abiding real
property owners who pay income realty taxes.
149. On the Garbage Fee: petitioners claim that this ordinance is
discriminatory as it collects garbage fee only from domestic households
and not from restaurants, food courts, fast food chains, and other
commercial dining places that spew garbage much more than residential
property owners.
The SC ruled that, the ordinance has measures it should confirm to for it
to be valid (refer to VIctorias Milling Co., Inc. v. Municipality of
Victoria’s supra note 49 for reference)
Equal protection requires that all persons or things similarly situated
should be treated alike, both as to rights conferred and
responsibilities imposed. The guarantee means that no person or
class of persons shall be denied the same protection of laws chich is
enjoyed by other persons or other classes in like circustances.
Similar subjects should not be treated differently as to give undue favor
to some and unjustly discriminate against others. The law therefore
may treat and regulate one class differently from another class
provided that there ar e real and substantial differences to distinguish
one class from another.
For the purpose of undertaking a comprehensive and continuing urban
development and housing program, the disparities between a real
property owner and an informal settler as two distinct classes are too
obvious and need not be discussed at length.
Notably, the public purpose of a tax may legally exist even if the motive
which impelled the legislature to impose the tax was to favor one
over another. Inequities which result from a singling out of one
particular class for taxation or exemption infringe no constitutional
limitation.
Hence, the SHT is valid.
For the garbage fee, the SC ruled that it is violative of the equal
protection clause of the Constitution and the provisions of the LGC,
that an ordinance must be equitable and based as far as practicable
on the taxpayer’s ability to pay, and not unjust, excessive,
oppressive, confiscatory.
For the purpose of garbage collection, there is no substantial distinction
between an occupant of a lot and that of a unit in a condominium, a
socialized housing project, or apartment. Most likely, garbage output
produced by these types of occupants is uniform and does not vary
to a large degree; thus a similar schedule of fee is both just and
equitable. (context, sa table kasi sa orig, on domestic huseholds, it is
much more expensive, as compared to condominium units)
1 UNITED TRANSPORT KOALISYON (1-UTAK) v. COMELEC PUV owners in the same position do in fact possess a franchise
(DANNAH) and/or certificate of pulic convenience and operate as a public
utility
April 14, 2015 | Reyes, J. | Equal Protection Clause
Having placed their property for used by the public, 1-UTAK and
other PUV and terminal owners cannot now complain that
PETITIONER: 1-United Transport Koalisyon
their property is subject to regulation by the state
RESPONDENTS: COMELEC They also aver that the exact purpose for placing political advertisements
SUMMARY: COMELEC, in relation to the Fair Elections Act promulgated on a PUV or in transport terminals is exactly because it is public
Resolution No. 9614. Sections of the Resolution prohibit the posting of and can be seen by all; and although it is true that private vehicles
election propaganda in PUVs and public transport terminals. 1-UTAK claims ply the same route as public vehicles, the exposure of a [PUV]
that this violates their right to free speech. COMELEC claims that this is a servicing the general, riding public is much more compared to private
valid Governmental restriction. vehicles. Categorizing PUVs and transport terminals as public
places under Section 7(f) of Reso. No. 9615 is therefore logical.
The SC ruled said sections invalid for infringing on people’s freedom of 1-UTAK claims that there is no substantial public interest threatened by
speech and equal protection. There is no valid distinction between owners
the posting of political advertisements on PUVs and transport
of private vehicles and owners of PUVs and transport terminals.
terminals to warrant the prohibition imposed by the COMELEC.
DOCTRINE: Superficial differences do not make for a valid classification. Also that the ownership of the PUVs per se, as well as the transport
terminals, remains private and, hence, the owners thereof could not
be prohibited by the COMELEC from expressing their political
FACTS: opinion lest their property rights be unduly intruded upon.
On February 2001, RA 9006 known as the Fair Elections Act was COMELEC on the other hand points out that PUVs and private transport
passed. COMELEC promulgated Resolution No. 9614 which terminals hold a captive audience · the commuters, who have no
provided for the rules implementing said RA in connection with the choice but be subjected to the blare of political propaganda.
May 2013 elections.
Section 7 thereof, which enumerates the prohibited forms of election ISSUE/s:
propaganda, pertinently provides:
WoN the sections of the regulation are valid – NO
7g. Prohibited Forms of Election Propaganda: It is unlawful to post,
display,… outside of authorized common poster areas, in public RULING:. Petition granted. Sections declared null and void.
places, or private properties without the consent of owners
thereof RATIO:
0 5. Public places includes PUVs such as buses, Section 7(g) items 5 and 6 are prior restraints of speech. These
jeeps, trains, taxi cabs, etc.
provisions unduly infringe on the fundamental right of the people to
1 6. Within the premises of public transport terminals
such as bus terminals, airports, seaports, etc. freedom of speech. Central to the prohibition is the freedom of
Violation of the abovementioned shall be a cause for the individuals, i.e., the owners of PUVs and private transport terminals,
revocation of the public utility franchise and will make the to express their preference.
owner and/or operator of the transport service and/or It states that the penalty of revocation of the public utility franchise and
terminal liable for an election offense… shall make the owner thereof liable for an election offense. The
1-UTAK through its president, Vargas sought clarification particularly prohibition constitutes a clear prior restraint on the right to free
Section 7(g) items and 5 and 6, from COMELEC in relation to
expression of the owners of PUVs and transport terminals.
Section 7(f) vis-à-vis privately-owned PUVs and transport terminals.
He explained that the prohibition impedes the right to free speech of In Adiong v. COMELEC, the Court struck down COMELEC’s prohibition
the private owners of PUVs and transport terminals. against the posting of decals and stickers on “mobile places” as it
COMELEC En Banc issued Minute Resolution No. 13-0214 which denied strikes at the freedom of an individual to express his preference and,
the UTAK’s request to reconsider the implementation of assaild by displaying it on his car, to convince others to agree with him.
provisions stating: It is also an invalid content-regulation. The requisites are:
The government regulation is within the constitutional power of class. However, the classification remains constitutionally
the Government impermissible since it is
It furthers an important substantial governmental interest
The governmental interest is unrelated to the supporession of
free expression
The incidental restriction on freedom of expression is no greater
than is essential to the furtherance of that interest.
They are content-neutral regulations since they merely control the place
where election campaign materials may be posted. However, the
prohibition is still repugnant to the free speech clause as it fails to
satisfy all of the requisites for a valid content-neutral regulation.
The prohibition under Section 7(g) items (5) and (6), in relation to Section
7(f), of Resolution No. 9615 is not within the COMELEC’s
constitutionally delegated power of supervision or regulation.
Such supervisory power does not extend to the very freedom of an
individual to express his preference of candidates in an election by
placing election campaign stickers on his vehicle.
There is a marked difference between the franchise or permit to operate
transportation for the use of the public and the ownership per se of
the vehicles used for public transport.
The right to operate a public utility may exist independently and
separately from the ownership of the facilities thereof. One can own
said facilities without operating them as a public utility, or conversely,
one may operate a public utility without owning the facilities used to
serve the public.
It is also not justified under the captive-audience doctrine. The captive-
audience doctrine states that when a listener cannot, as a practical
matter, escape from intrusive speech, the speech can be restricted.
The captive-audience doctrine recognizes that a listener has a right
not to be exposed to an unwanted message in circumstances in
which the communication cannot be avoided.
These provisions not only run afoul of the free speech clause, but also of
the equal protection clause.
“The equal protection clause is aimed at all official state actions,
not just those of the legislature. Its inhibitions cover all the
departments of the government including the political and
executive departments, and extend to all actions of a state
denying equal protection of the laws, through whatever agency
or whatever guise is taken.”
It is conceded that the classification under said provisions is not limited to
existing conditions and applies equally to the members of the purported
not based on substantial distinction and is not germane to the purpose of
the law.
A distinction exists between PUVs and transport terminals and private vehicles
and other properties in that the former, to be considered as such, needs to
secure from the government either a franchise or a permit to operate.
Nevertheless, as pointed out earlier, the prohibition imposed under Section
7(g) items (5) and (6) of Resolution No. 9615 regulates the ownership
per se of the PUV and transport terminals; the prohibition does not in any
manner affect the franchise or permit to operate of the PUV and transport
terminals.
As regards ownership, there is no substantial distinction between owners of
PUVs and transport terminals and owners of private vehicles and other
properties. As already explained, the ownership of PUVs and transport
terminals, though made available for use by the public, remains private.
If owners of private vehicles and other properties are allowed to express their
political ideas and opinion by posting election campaign materials on
their properties, there is no cogent reason to deny the same preferred
right to owners of PUVs and transport terminals. In terms of ownership,
the distinction between owners of PUVs and transport terminals
and owners of private vehicles and properties is merely superficial.
Superficial differences do not make for a valid classification.
The fact that PUVs and transport terminals are made available for use by the
public is likewise not substantial justification to set them apart from
private vehicles and other properties.
Admittedly, any election campaign material that would be posted on PUVs
and transport terminals would be seen by many people. However,
election campaign materials posted on private vehicles and other places
frequented by the public, e.g., commercial establishments, would also be
seen by many people. Thus, there is no reason to single out owners of
PUVs and transport terminals in the prohibition against posting of
election campaign materials.
Moreover, classifying owners of PUVs and transport terminals apart from
owners of private vehicles and other properties bears no relation to the
stated purpose of Section 7(g) items (5) and (6) of Resolution No. 9615,
i.e., to provide equal time, space and opportunity to candidates in
elections.
VALMONTE v. GENERAL DE VILLA (IYA) Checkpoints are one of such means.
1989 | Padilla, J. | When intrusion is a “search” If vehicles are stopped and extensively searched, it is because of
some

PETITIONER: Valmonte
RESPONDENTS: General de Villa

SUMMARY: Petitioner Valmonte questions the constitutionality of


checkpoints as it is tantamount to warrantless search and seizure, which
violates the constitutional guarantee against unreasonable searches and
seizures. The Supreme Court previously decided that checkpoints are
not illegal per se and are a valid exercise of the government’s police
power as it is expected to defend itself from its enemies. The present
case is a MR assailing the aforementioned decision. The SC upheld its
stance saying that when vehicles are stopped at a check point it is
because there is probable cause to believe that the motorist is an
offender or is in possession of articles related to an offense.

DOCTRINE: If vehicles are stopped and extensively searched, it is


because of some probable cause which justifies a reasonable belief of
the men at the checkpoints that either the motorist is a law offender or
the contents of the vehicle are or have been instruments of some
offense.

FACTS:
Valmonte filed a motion for reconsideration of the previous SC decision
dismissing the petition for prohibition seeking the declaration of the
checkpoints as unconstitutional.
Note: Nowhere in the questioned decision did the Court legalize ALL
checkpoints. What the court declared is that checkpoints are not
illegal per se.
Recent on going events have pointed to the continuing validity and need
for checkpoints manned by either military or police forces.
There has been a 6th attempt at a coup d’etat and the NPA has not
relented but instead accelerated its liquidation of armed forces and
police personnel.
Valmonte argues that checkpoints is considered as warrantless search
and seizure and, therefore, violative of the Constitution.

ISSUE/s:
WON checkpoints is in practice of warrantless search and seizure? - NO

RULING: Motion for Reconsideration is Denied

RATIO:
The government has the equal right, under its police power, to select the
reasonable means and methods for best defending itself from
enemies.
probable cause which justifies a reasonable belief of the men at the
checkpoints that either the motorist is a law offender or the contents of the
vehicle are or have been instruments of some offense.
Checkpoints are quite similar to searches and seizures accompanying
warrantless arrests during the commission of a crime or immediately
thereafter and of warrantless search of incoming and outgoing passengers at
the arrival and departure areas of an international airport.
The Court’s decision on checkpoints does not, in any way, validate nor condone
abuses committed by the military manning the checkpoints.
Whether there is abuse in a particular situation is a different question to be raised
in a different case.
Those who abuse their authority act beyond the scope of their authority and are
liable criminally and civilly for their abusive acts.
PEOPLE v. ESCAO (ELIEL) contained a parcel wrapped in tape, which, upon eamination by NBI
January 28, 2000 | Davide Jr., CJ. | “Intrusion” is a search Forensic Chemist Rosaldos, was found positive for hashish weighing
3.3143 kilos.
137. A certification was issued by the Firearms and Explosive Office of the
PETITIONER: People of the Philippines
National Police Commission (NAPOLCOM) to the effect that Escao was
RESPONDENTS: Julian Escao, Virgilio Usana, Jerry Lopez
not a licensed/registered firearms holder of any kind and caliber. Neither
SUMMARY: During a COMELEC gun ban, some law enforcers of tha of the
Makati Police manned a checkpoint wherein Escao, Usana and Lopez
were stopped. Upon inspection, they were caught with possession of
firearms and brought to the police station. The trial court convicted them
with illegal possession of firearms and violation of the gun ban. Hence
the appeal.

They complain that the checkpoint was illegal and the search and seizure
was invalid because they had no warrant. The SC ruled that there are legal
checkpoints, although intrusive to the right of free passage, for as long as the
inspection is limited to a visual search or its occupants are not subjected to a
body search. Furthermore, there are exceptions to the requirement of a
warrant. In this case, Escao consented to the inspection. However, Usana
and Lopez should be acquitted because during the search of the police only
Escao was present.

DOCTRINE: For as long as the vehicle is neither searched nor its


occupants subjected to a body search, and the inspection of the vehicle
is limited to a visual search, said routine checks cannot be regarded as
violative of an individuals right against unreasonable search. In fact,
these routine checks, when conducted in a fixed area, are even less
intrusive.

FACTS:
During a COMELEC gun ban, some law enforcers of the Makati Police
were manning a checkpoint at the corner of Gil Puyat and SLEX.
They were checking the cars going to Pasay City, stopping those
they found suspicious, and imposing merely a running stop on the
others.
At about past midnight, they stopped a Kia Pride car with Plate No. TBH
493. One of the officers saw a long firearm on the lap of Usana. They
asked Escao, the driver, to open the door and the officer seized the
long firearm.
134. Upon order of the police parked along Gil Puyat Ave., the other
passengers were search for more weapons. Their search yielded a .
45 caliber firearm, which they seized from Escao.
The three passengers were thereafter brought to the police station Bolck
5 in the Kia Pride driven by another police officer.
Escao readily agreed and opened the trunk himself using his key. They
noticed a blue bag inside it, which they asked Escao to open. The
bag
two guns seized were licensed/registered with the NAPOLCOM.
The trial court found the prosecutions version more credible than that of
any one of the accused, and ruled that the evidence presented by
the prosecution was sufficient to convict the accused as charged.
Accused-appellants Usana and Lopez anchor their appeal.

ISSUE/s:
29. WoN the search and seizure was valid – YES

RULING: Usana and Lopez Acquitted. Escao convicted.

RATIO:
159. The Court ruled that not all checkpoints are illegal. Those
which are warranted by the exigencies of the public order and
are conducted in a way least intrusive to motorists are allowed.
For admittedly, routing checkpoints do intrude, to a certain extent,
on motorists right to “free passage without interruption,” but it
cannot be denied that, as a rule, it involves only a brief
detention of travelers during which the vehicles occupants are
required to answer a brief question or two.
For as long as the vehicle is neither searched nor its occupants
subjected to a body search, and the inspection of the vehicle is
limited to a visual search, said routine checks cannot be
regarded as violative of an individuals right against
unreasonable search.
The COMELEC would be hard put to implement the ban if its deputized
agents were limited to a visual search of pedestrians. It would also
defeat the purpose for which such ban was instituted. Those who
intend to bring a gun during said period would know that they only
need a car to be able to easily perpetrate their malicious designs.
The court sees no need for checkpoints to be announced, as the
accused have invoked. Not only would it be impractical, it would also
forewarn those who intend to violate the ban. Even so, badges of
legitimacy of checkpoints may still be inferred from their fixed
location and the regularized manner in which they are operated.
Exceptions to warrant requirement: (1) search incidental to arrest;
(2) search of moving vehicls; (3) evidence in plain view; (4)
customs searches; (5) consented warrantless search; and (6)
stop-and-frisk situations.
Even though there was ample opportunity to obtain a search warrant, we
cannot invalidate the search of the vehicle, for there are indications
that the search done on the car of Escao was consented to by him.
Despite the validity of the search, the Court cannot affirm the
conviction of Usana and Lopez for violation of RA 6425 as
amended. The following facts militate against a finding of
conviction: (1) the car belonged to Escao; (2) the trunk of the car
was not opened soon after it was stopped and after the accused
were searched for firearms; (3) the care was driven
by a policeman form the place where it was stopped until the police
station; (4) the cars trunk was opened, with the permission of Escao,
without the presence of Usana and Lopez; and (5) after arrival at the
police station and until the opening of the cars trunk, the car was in the
possession and control of the police authorities.
ALVAREZ v. CFI (JP)
ISSUE/s:
January 29, 1967 | Imperial, J. | Requisites of a Valid Search Warrant
30. WoN the issued search warrant passed the requirements of a valid
search
PETITIONER: Narciso Alvarez
RESPONDENTS: Court of First Innstance of Tayabas, Anti-Usurt Board

SUMMARY: Upon the affidavit of Agent Almeda, the court issued a search
warrant against petitioner Alvarez for alleged illegal activites as a money-
lender, charging usurious rate of interest in violation of the law. Petitioner
enumerates five grounds to declare such search and seizure void.

The SC declared such search and seizure void for the failure of the
complainant to have personal knowledge, for failing to present other
witnesses and for conducting the search to incriminate petitioner against
the Anti-Usury Board.

DOCTRINE: See Ratio No. 1, 4, 8, 10, 14.

FACTS:
Petitioner Alvarez askes that the warrant issued by the CFI of Tayabas,
ordering the search of his house and the seizure, at any time of the
day or night, if certain accounting books, documents and papers
belonging to him in his residence as well as the order of a later date,
authorizing the agents of the Anti-Usury Board to retain the articles
seized be celared illegal and set aside and prays that all the articles
in question be returned to.him.
The petitioner claims that the search warrant issued by the court is illegal
because it has been based upon the affidavit of agent Mariano G.
Almeda in whose oath he declared that he had no personal
knowledge of the facts which were to serve as a basis for the
issuance of the warrant but that he had knowledge thereof through
mere information secured from a person whom he considered
reliable.
First ground: Almeda was claiming that a person he considers liable
reported to him that in the said premises, books, documents,
receipts, lists and other papers are used by the accused Alvarez
in connection in his activies as a money-lender, charging
usurious rate of interest in violation of the law.
Second ground: Petitioner alleges that in asking that the search warrant
was not supported by other affidavits aside from that made by the
applicant.
Third ground: That the search was authorized its execution at night.
Fourth ground: That the warrant was issued illegally because of the lack
of an adequate description of the books and documents to be seized.
Last ground: that the articles were seized in order that the Anti-Usury
Board might provide itself with evidence to be used by it in the
criminal case or cases which might be filed against him for violation
of the Anti-Usury Law.
warrant - YES

RULING: The search warrant and seizure are declared illegal and are set
aside.

RATIO:
On the first ground: The oath required must refer to the truth of the facts within
the personal knowledge of the petitioner or his witnesses, because the
purpose thereof is to convince the committing magistrate, not the
individual making the affidavit and seeking the issuance of the warrant, of
the existence of probable cause.
It appears that the affidavit, which served as the exclusive basis of the
search warrant, is insufficient and fatally defective by reason of the
manner in which the oath was made, and therefore, it is hereby held
that the search warrant in question and the subsequent seizure of
the books, documents and other papers are illegal and do not in any
way warrant the deprivation to which the petitioner was subjected.
On the second ground: the purpose of both in requiring the
presentation of depositions is nothing more than to satisfy the
committing magistrate of the existence of probable cause.
Therefore, if the affidavit of the applicant or complainant is
sufficient, the judge may dispense with that of other witnesses.
Inasmuch as the affidavit of the agent in this case was insufficient
because his knowledge of the facts was not personal but merely
hearsay, it is the duty of the judge to require the affidavit of one or
more witnesses for the purpose of determining the existence of
probable cause to warrant the issuance of the search warrant.
When the affidavit of the applicant or complainant contains sufficient
facts within his personal and direct knowledge, it is sufficient if the
judge is satisfied that there exists probable cause; when the
applicant's knowledge of the facts is mere hearsay, the affidavit of
one or more witnesses having a personal knowledge of the facts is
necessary.
Therefore, that the warrant issued is likewise illegal because it was based
only on the affidavit of the agent who had no personal knowledge of the
facts.
On the third ground: Section 101 of General Orders, No. 58
authorizes that the search be made at night when it is positively
asserted in the affidavit that the property is on the person or in
the place ordered to be searched.
As we have declared the affidavit insufficient and the warrant issued
exclusively upon it illegal, our conclusion is that the contention is
equally well founded and that the search could not legally be made
at night.
On the fourth ground: Section 1, paragraph 3, of Article III of the Constitution,
and section 97 of General Orders, No. 58 provide that the affidavit to be
presented, which shall serve as the basis for determining whether probable
cause exists and whether the warrant should be issued, must contain a
particular description of the place to be searched and the
person or thing to be seized. These provisions are mandatory and must

be strictly complied with.


But where, by the nature of the goods to be seized, their description must
be rather general, it is not required that a technical description be
given, as this would mean that no warrant could issue.
Taking into consideration the nature of the articles so described, it is
clear that no other more adequate and detailed description could
have been given, particularly because it is difficult to give a particular
description of the contents thereof.
The description so made substantially complies with the legal provisions
because the officer of the law who executed the warrant was thereby
placed in a position enabling him to identify the articles, which he did.
Last ground: The seizure of books and documents by means of a search
warrant, for the purpose of using them as evidence in a criminal case
against the person in whose possession they were found, is
unconstitutional because it makes the warrant unreasonable, and it is
equivalent to a violation of the constitutional provision prohibiting the
compulsion of an accused to testify against himself.
Therefore, it appearing that at least nineteen of the documents in
question were seized for the purpose of using them as evidence
against the petitioner in the criminal proceeding or proceedings for
violation of the Anti-Usury Law, which it is attempted to institute
against him, we hold that the search warrant issued is illegal and that
the documents should be returned to him.
PEOPLE v. VELOSO (HENRY) In the MTC, the persons arrested in the raid were accused of gambling.
October 20. 1925 | Malcolm, J. | Search Warrant All were eventually acquitted in the CFI for lack of proof except for
Veloso, who was found guilty of maintaining a gambling house.
PETITIONER: People of the Philippines
RESPONDENTS: Jose M. Veloso

SUMMARY: Jose M. Veloso, a member of the HoR and the manager of


Parliamentary Club, was basically assailing that he had the right to
protest against the validity of the search warrant that permitted policemen
to raid his club, allegedly for violation of the Gambling Law. Primarily, his
contention was, the search warrant indicated “John Doe”, instead of
“Jose M. Veloso” and hence he shall not be apprehended. The RTC ruled
against his favor, which was affirmed by the CA.

The SC ruled that the search warrant was valid insofar as it described in
detail the premises, and the person, despite it having “John Doe” as its
name. Basing it in the previous cases that the SC decided, the validity of
the search warrant was upheld.

DOCTRINE: A warrant for apprehension of a person whose true name is


unknown, by the name of John or Richard Doe, is void, without other and
further descriptions to be apprehended, and such warrant will not justify
the officer in acting under it. Such a warrant must contain the BEST
description personae possible to be obtained of the person or persons to
be apprehended.

FACTS:
May 1923, the Building located at No. 124 Calle Arzobispo, Manila, was
used by the Parliamentary Club. Jose M. Veloso, a member of the
House of Representatives (HoR), was the manager of the said club.
Police had reliable information that the said club was more than a
gambling house. Indeed, J.F. Townsend, chief of the gambling squad,
had been to the club and verified this fact.
Consequently, Detective Geronimo of the secret service of Manila,
applied for, and obtained a search warrant from J. Garduno of the
MTC.
With the opportunity, the police attempted to raid the club a little after
3pm and found that the doors were closed and barred so the
policemen entered through the windows while the others broke in the
outer door.
Inside, over 50 were apprehended bythe police, one of them was
respondent Veloso, who assailed the validity of the search warrant
as it indicated “John Doe” instead of his name. Although JF
Townsend explained that he was considered as “Mr. Doe”.
The search continued, but Veloso was still anything else but supportive
of such action—he shouted offensive words, refused to obey, etc.
This case reached the CA, where he was finally sentenced to pay a fine seized, and that warrant in this case sufficiently described the place and
of Php 500.00. Hence this appeal. the gambling

ISSUE/s:
WoN the search warrant was valid despite it having indicated a “John
Doe” instead of “Jose M. Veloso” – YES

RULING: SC affirmed the lower court’s decision of Mr. Veloso’s guilt. The
search warrant’s validity was upheld.

RATIO:
The defense has been all about the fact that since the name of Veloso is
not found in the search warrant, then Veloso has a legal right to
resist the police by force.
A search warrant must conform strictly to the requirements of the
constitutional and statutory provisions under which it is issued.
Otherwise, it is void. The warrant will always be construed strictly
without however, going to the full length of requiring technical
accuracy.
Search warrants have been likened to a warrant of arrest. Although
apprehending that there are material differences, in view of the
paucity of authority to John Doe search warrants, the SC proposes to
take into consideration the authorities in previous cases as far as this
topic is concerned:
Name and description of the accused should be inserted in
the body of the warrant, and where the name is unknown
there must be such description f the person accused as
will enable the officer to identify him when found.
Warrant of apprehension of unnamed party, or containing a
wrong name for the party to be apprehended is void,
except in those cases where it contains a “description
personae” such as will enable the officer to identify the
accused
John Doe Warrants – a warrant for apprehension of a
person whose true name is unknown, by the name of
John or Richard Doe, is void, without other and further
descriptions to be apprehended, and such warrant will
not justify the officer in acting under it. Such a warrant
must contain the BEST description personae possible to
be obtained of the person or persons to beapprehended.
While the search warrant did not specify Veloso’s name, it indicated the
ff:
“John Doe has illegally in his possession in the building
occupied by him, and which is under his control namely, in
the building numbered 124 Calle Arzobispo, City of Manila,
Philippine Islands, certain devices and effects used in
violation of the Gambling Law.”
The Organic Act (pertaining to search warrants) requires a particular
description of theplace to be searched, and the person or things to be
apparatus, and in addition, contained a description of the person to be seized.
As the warrant stated that John Doe had gambling apparatus in his
possession in the building, and as this John Doe was Jose Veloso,
the manager, the police could identify John Doe as Veloso without
difficulty.
John Doe search warrants should be the excepetion and not the rule.
The police should particularly describe the place to be searched and
the person or things to be seized, wherever and whenever feasible.
The police should ot be hindered in the performance of their duties,
which are difficult enough of performance under the best of
conditions, buy superficial adherence to technicality or far fetched
judicial interference.
SOLIVEN v. MAKASIAR (DANNAH) 1. WoN judges should personally examine complainant and witness
November 14. 1988 | Per Curiam | Warrant of Arrest: Probable Cause before

PETITIONER: Maximo Soliven, Antonio Roces et al. and Luis Beltran


RESPONDENTS: Judge Makasiar of RTC Manila, et al.

SUMMARY: Herein Petitioners Beltran and Soliven were impleaded in a


libel case because of an article Beltran wrote about former President
Aquino. Soliven questions the manner in which the judge issued the warrant
of arrest. His understanding of Section 2 Article III was that judges should
personally examine the complainant and his witness.

The Court ruled that this was a wrong interpretation, as judges will be
burdened with examination and investigation if it were so. He may not be
able to discharge his other important functions if that were the case. In
satisfying himself of the existence of probable cause for the issuance of a
warrant of arrest, the judge is not required to personally examine the
complainant and his witnesses.

DOCTRINE: What the Constitution underscores is the exclusive and


personal responsibility of the issuing judge to satisfy himself of the existence
of probable cause. Following established doctrines and procedure, he shall:

Personally evaluate the report and the supporting documents


submimtted by the fiscal regarding the existence of probable cause
and, on the basis thereof, issue a warrant of arrest or
If on the basis thereof he finds no probable cause, he may disregard the
fiscal’s report and require the submission of supporting affidavits of
witnesses to aid him in arriving at a conclusion as to the existence
of probable cause.

FACTS:
Beltran and his publisher Soliven were impleaded in a libel case because
Beltran allegedly wrote that former President Aquino hid under her
bed during a coup d’etat attempt at the presidential palace in Manila.
Soliven now questions the procedure in which the warrants of arrest
were released.
He believes that the word “personally” after the word “determined” and
the deletion of the grant of authority of the 1973 Constitution to issue
warrants to “other responsible officers as may be authorized by law”
now requires the judge to personally examine the complainant and
his witness in determination of probable cause for the issuance of
warrants of arrest.

ISSUE/s:
determination of probable cause for the issuance of warrants of
arrest – NO

RULING: Petition dismissed. Status quo contained in the previous


Resolution is lifted.

RATIO:
Article III Section 2 states:
…no search warrant or warrant of arrest shall issue except upon
probable cause to be determined personally by the judge
after examination under oath or affirmation of the
complainant and the witness he may produce, and
particularly describing the place to be searched and the
persons or things to be seized.
What the Constitution underscores is the exclusive and personal
responsibility of the issuing judge to satisfy himself of the existence
of probable cause.
In satisfying himself of the existence of probable cause for the issuance
of a warrant of arrest, the judge is not required to personally examine
the complainant and his witnesses.
Following established doctrines and procedure, he shall:
Personally evaluate the report and the supporting documents
submimtted by the fiscal regarding the existence of probable
cause and, on the basis thereof, issue a warrant of arrest or
If on the basis thereof he finds no probable cause, he may
disregard the fiscal’s report and require the submission of
supporting affidavits of witnesses to aid him in arriving at a
conclusion as to the existence of probable cause.
Sound policy dictates this procedure, otherwise judges will be unduly
laden with the preliminary examination and investigation of criminal
complaints instead on hearing and deciding cases filed before their
courts.
On June 1987, the SC unanimously adopted Circular No. 12, setting
down guidelines for the issuance of warrants of arrest.
The procedure therein provided is reiterated and clarified in this
resolution.
It has not been shown that respondent judge has deviated from the
prescribed procedure. There is no grave abuse of discretion.
On Presidential Immunity: Only the President can invoke this. There is
also nothing stipulated in the Constitution saying she is not allowed
to waive this right.
PLACER v. VILLANUEVA (IYA) He may disregard the fiscal’s certification and
1983 | Escolin, J. | Issance of a warrant require the submission of the affidavits of
witnesses to aid him in arriving at a conclusion
as to the existence
PETITIONER: Placer
RESPONDENTS: Villanueva

SUMMARY: Note: There was no discussion of facts in the Case Book


Under PD 911 fiscals or state prosecutors are granted the authority to
determine whether there is probable cause for the issuance of a warrant
of arrest. The SC ruled that this certification is not binding upon judges
although it may be given weight. The judge must conduct his own
examination for probable cause.

DOCTRINE: He may disregard the fiscal’s certification and require the


submission of the affidavits of witnesses to aid him in arriving at a
conclusion as to the existence of a probable cause.

FACTS:
The legal question raised in this petition is wether the certification of the
investigating fiscal in the information as to the existence if probable
cause obligates respondent city Judge to issue a warrant of arrest.
The primary requirement for the issuance of a warrant of arrest is the
existence of probable cause.
PD No. 911 authorizes the fiscal or state prosecutor to determine the
existence of probable cause.
There is no dispute that the judge may rely upon the fiscal’s certification
for the existence of probable cause and on the basis thereof, issue a
warrant of arrest.

ISSUE/s:
WON a certification by the fiscal bind the judge to come out with the
warrant? - NO

RULING: Motion for Reconsideration is Denied

RATIO:
The issuance of a warrant is not a mere ministerial function.
It calls for the judicial discretion on the part of the issuing magistrate.
Under Section 6 Rule 112 of the Rules of Court the judge must satisfy
himself of the existence of probable cause before issuing a warrant or
order of arrest.
of a probable cause.
STONEHILL v. DIOKNO (ELIEL)
June 19, 1967 | Concepcion, CJ. | Search and Seizures RATIO:

PETITIONER: Harry Stonehill, Robert Brooks, John Brooks, Karl Beck


RESPONDENTS: Jose Diokno, NBI, Prosectors

SUMMARY: Officers of the government issued 42 search warrants


against Stonehill et al and their corporations. The application alleged that
the things to be seized are books, records, vouchers, all adverted to be in
relation to the violation of the Central Bank Laws, RPC, Internal Revenuc
Code, and Tariff and Customs Laws. Hence this appeal.

The SC ruled that the issued warrants were general and does not particularly
describe the things to be seized, much more the crime for which they were
alleged. The SC further exclaimed that it was possible for the judge, upon
probable cause, the issued warrants because of there was no particular
crime alleged.

DOCTRINE: Two points must be stressed in connection with this


constitutional mandate, namely: (1) that no warrant shall issue but upon
probable cause, to be determined by the judge in the manner set forth ins
aid provision; and (2) that the warrant shall particularly describe the
things to be seized.

FACTS:
Officers of the government issued, on different dates, a total of 42 search
warrants against Stonehill et al and/or the corporations of which they
were officers, directed to any peace officer.
To search the persons Stonehill et al and/or the premises of their offices,
warehouses and/or residences, and to seize and take possession of
the following personal property as “the subject of the offense; stolen
or embezzled and proceeds or fruits of the offense,” or “used or
intended to be used as the means of committing the offense,” which
is described in the applications adverted to above as “violation of
Central Bank Laws, Tariff and Customs Laws, Internal Revenue
(Code) and the RPC.
Books of accounts, financial records, vouchrs, correspondence,
receipts, ledgers, journals, portfolios, credit journals,
typewriters, and other documents and/or papers showing all
business transactions including disbursements receipts,
balance sheets and profit and loss statements and Bobbins
(cigarette wrappers)
Hence this appeal.

ISSUE/s:
WoN the search and seizure was valid – NO

RULING: Warrants are null and void, search and seizure is illegal.
The documents, papers, and things seized under the alleged authority of the
warrants in question may be split into two (2) major groups, namely: (a)
those found and seized in the offices of the aforementioned corporations,
and (b) those found and seized in the residences of Stonehill et al.
As regards the first group, Stonehill et al have no cause of action to assail the
legality of the contested warrants and of the seizures made in pursuance
thereof, for the simple reason that said corporations have their respective
personalities, separate and distinct from their personality, regardless of the
amount of shares of stock or of the interest of each of them in said
corporations, and whatever the offices they hold therein may be.
It is well settled that the legality of a seizure can be contested only by the
party whose rights have been impaired thereby, and that the objection
to an unlawful search and seizure is purely personal and cannot be
availed of by third parties.
With respect to the documents, papers and things seized in Stonehill et al
residences, the Court restrained the prosecution from using it against them
because of the writ of preliminary injunction issued.
“The right of the people to be secure in their persons, houses, papers, and
effects against unreasonable searches and seizures shall not be
violated, and no warrants shall issue but upon probable cause, to be
determined by the judge after examination under oath or affirmation of
the complainant and the witnesses he may produce, and particularly
describing the place to be searched, and the persons or things to be
seized.”
Two points must be stressed in connection with this constitutional
mandate, namely: (1) that no warrant shall issue but upon probable
cause, to be determined by the judge in the manner set forth ins aid
provision; and (2) that the warrant shall particularly describe the thigns
to be seized.
None of these requirements has been complied with in the contested warrants. In
other words, no specific offense had been alleged in said applications. The
averments thereof with respect to the offense committed were abstract.
As a consequence, it was impossible for the judges who issued the
warrants to have found the existence of probable cuase, for the same
presupposes the introduction of competent proof that the party against
whom it is sought has performed particular acts, or committed specific
omissions, violating a given provision of our criminal laws.
The applications involved in this case do not allege any specific acts performed by
Stonehill et al. It would be the legal heresy, of the highest order, to convict
anybody of a “violation of Central Bank Laws, Tariff and Customs Laws, Internal
Revenue (Code) and RPC,” as alleged in the aforementioned applications,
without reference to any determinate provision of said laws
To uphold the validity of the warrats in question would be to wipe out completely
one of the most fundamental rights guaranteed in our Constitution, for it
would place the sanctity of the domicile and the privacy of communication
and correspondence at the mercy of the whims caprice or
passion of peace officers.
“a search warrant shall not issue but upon probable cause in
connection with one specific offense.”
“No search warrant shall issue for more than one specific offense.”
Thus, the warrants authorized the search for and seizure of records
pertaining to all business transactions of Stonehill et al,
regardless of whether the transactions were legal or illegal. The
warrants sanctioned the seizure of all records of Stonehill and
the aforementioned corporations, whatever their nature, thus
openly contravening the explicit command of our Bill of Rights
– that the things to be seized be particularly described – as well
as tending to defeat its major objective: the elimination of
general warrants.
To be sure, if the applicant for a search warrant has competent evidence
to establish probable cause of the commission of a given crime by
the party against whome the warrant is intended, then there is no
reason why the applicant should not comply with the requirements of
the fundamental law.
If he has no such competent evidence, then it is not possible for the
Judge to find that there is probable cause, and hence, no justification
for the issuance of the warrant.
Castro, J. Concurring and Dissenting: All the search warrants, without
exception, in this case are admittedly general, blanket and roving
warrants and are therefore admittedly and indisputedly outlawed by
the Constitution; and the searches and seizures made were
therefore unlawful.
Prescinding from the foregoing, this Court, at all events, should order the
return to the petitions all personal and private papers and effects
seized, no matter where these were seized, whether form their
residences or corporate offices or any other place or places.
CENTRAL BANK v. MORFE (JP) business of a savings or mortgage bank without having first complied
June 30, 1967 | Concepcion, C. J. | Valid Search Warrant with the provisions of RA 337.
The search warrant lists articles, papers, or effects which includes
PETITIONER: Central Bank of the Philippines General
RESPONDENTS: Honorary Judge Jesus Morfel, presiding judge of he
CFI of Manila

SUMMARY: The First Mutual Savings and Loan Organizations, a loan


association, was reported to engage in illegal bank activities violating RA
337. The municipal judge granted such. Respondent judge however,
upon motion of the Organization, granted an injunction against the
previous order on the reasoning that the warrant is roving commission for
not specifcying specif transactions of specific persons.

The SC ruled that the injunction must be lifted because it does not
necessarily mean that there is no personal knowledge by the witness if
specific names were not enumerated in the complaint. There is no law
also which limits search warrants to things which cannot be secured
otherwise.

DOCTRINE: The failure of the witness to mention particular individuals


does nto necessarily prove that he had no personal knowledge of specific
illegal transaction.

There is also not any rule that limits the use of search warrants to papers
or effects which cannot be secured otherwise.

FACTS:
The First Mutual Savings and Loan Organization, Inc. (Organization) is a
non-stock corporation who extends financial assistance in the forms of
loans.
The legal department of the Central Bank of the Philippines (Bank)
rendered an opinion to the effect that the Organiztion and others of
similar nature are banking institutions, falling within the purview of
the Central Bank Act (RA No. 337).
The bank caused to be published in newspapers: “The Central Bank of the
Phillippines wishes to announce that all savings and loan associations
… have never been authorized by the monetary board of the Central
Bank to accept deposit of funds from the public nor to engage in the
banking business. Sunc institutions violate Section 2 of the General
Banking Act, RA 337.”
The intelligencedivision of the Bank filed with the Municipal Court of Manila a
verified application for a search warrant alleging that after close
observation of the Organization’s building’s offices, the latter is being
used unlawfully because said Organization is illegally engaged in
banking activites, by receiving deposits of money and transacting the
Journal, General Ledger, financial statemetns, etc.
Hon. Cancino, issued the warrant.
The Organization commenced a civil action for certiorari and injunction
against said municipal court, to annul the search warrant explaining
that:
Such search warrant is a roving commission, general in its terms
The use of the word “others” permits unreasonable search of
documents which have no relation to any criminal act
No court in the Philippines has any jurisdiction to try a criminal
case against a corporation.
Respondent judge issued the assailed resolution, granting the injunction,
ordering Central Bank to return to the petitioner Organization all the
books, documents, and papers so far seized.
Respondent reasoned out that since the complainant (name not in Case
book) knows that the bank is engaged in illegal activities, the warrant
should have covered only the specific purportedly illegal transactions
of the Organization with specific persons. To authorize and seize all
the records listed would be to harass the Organization, and would be
a roving commission or fishing expedition.
Central Bank appealed.

ISSUE/s:
WoN respondent Judge acted in grave abuse of discretion in issuing the
assailed order - NO

RULING: The order of respondent judge is annulled.

RATIO:
The failure of the witness to mention particular individuals does nto
necessarily prove that he had no personal knowledge of specific
illegal transactions of the Organization, for the wintess might be
acquainted with such specific transactions even if the names of such
individuals concerned were unknown to him.
An illegal banking transaction complaint must not always connote the
existence of a victim.
The law requiring compliance with certin requirements before any body
can engage in banking obviously seeks to protect the public against
actual as well as potential injury.
There is also not any rule that limits the use of search warrants to papers
or effects which cannot be secured otherwise.
The line of reasoning of respondent Judge might perhaps be justified if
the acts imputed to the Organization are isolated transcations.
However, such is not the case at bar. The records suggest clearly
that the transactiosn objected to by the Bank constitute the general
patern of the business of the Organization.
The principal issue raised by the Organizaion is predicated upon the
theory that the aforementioned transactions of the Organizations do
not amount to
“banking”, as the term is used in RA 337.
The SC is satisfied however that the Municipal Judge did not commit
grave abuse of discretion in finding that there was probable cause
that the Organization violated RA 337. The warrant is also valid in
line with Alvarez v. CA (not expounded in casebook).
BACHE & CO v. RUIZ (HENRY) determine the existence or non-existence of a probable cause, pursuant
February 27, 1971 | Villamor, J. | Search Warrant to Art. 3, Sec. 1, Par. 3 of the Constitution, and Sec. 3, Rule 126 of the
RRC, both of which prohibit the issuance of warrants EXCEPT upon
PETITIONER: BACHE & CO. (Phil.), Inc. and Frederick E. Seggerman probable cause.
RESPONDENTS: Hon. Judge Vivencio M. Ruiz, Misael P. Vera, in his The determination of whether or not a probable cause exists calls for the
capacity as Commissioner of Internal Revenue, Arturo Logronio, Rodolfo exercise of judgment after a judicial appraisal of facts and should not
de Leon, Gavino Velasquez, Mimir Dellosa, Nicanor Alcordo, et al. be allowed to be delegated in the absence of any rule to the contrary.
In the present case, NO personal examination at all was conducted by
SUMMARY: The case commenced when respondent Vera wrote a letter respondent Judge of the complainant and his witness.
to respondent Judge, requesting for the issuance of a search warrant While it is true that the complaint’s application for search warrant and the
against the petitioners. Now the petitioners are assailing such warrant’s witness’ printed-form deposition were subscribed and sworn to before
validity, ultimately because the judge did not personally examine the respondent Judge, the latter did not ask either of the two any
complainant and his witness. While the lower courts ruled of it’s validity, question the answer to which could possibly be the basis for
the SC ruled otherwise. determining whether or not there was probable cause.
Moreover, ia warrant may be said to particularly describe the things to be
Per SC, it is non negotiable that the judge shall personally examine the seized when the description therein is as specific as the
complainant as well as his witness, as this will aid him in determining a circumstances will ordinarily allow, or when the things described are
probable cause—which is detrimental to the issuance of a warrant. limited to those whichb ear direct relation to the defense for which the
warrant is being issued.
DOCTRINE: determine the existence or non-existence of a probable In the case at hand, the warrant does not conform to any of the foregoing
cause, pursuant to Art. 3, Sec. 1, Par. 3 of the Constitution, and Sec. 3, tests. If the articles desired to be seized have any direct relation to an
Rule 126 of the RRC, both of which prohibit the issuance of warrants offense committed, the applicant must necessarily have some
EXCEPT upon probable cause. evidence, to prove the said offence.

FACTS:
February 1970, respondent Vera (Commissioner of Internal Revenue),
wrote a letter addressed to respondent Judge Ruiz, requesting the
issuance of a search warrant No. 2-M-70 against petitioners.
The same warrant is being assailed because:
The Judge failed to personally examine the complainant as well
as his witness
The search warrant was issued for more than one specific offence
The search warrant does not particularly describe the things to
be seized

ISSUE/s:
WoN the search warrant is valid despite the irregularities being claimed
by respondents - NO

RULING: SC granted the petition, and declared the Search Warrant No. 2-M-70 void.

RATIO:
Th examination of the complaint and the witnesses he may produce,
required by Art. 3 Sec 1 par 3 of the Constitution, and by Secs 3 and
4, Rule 126 of the Revised Rules of Court (RRC), should be
conducted by the judge imself and not the others.
Such personal examination by the judge is necessary to enable him to
COLUMBIA PICTURES v. CA (DANNAH) duly observed, and that the questioned search warrants were not
October 6, 1994 | Per Curiam | Warrant of Arrest: Requisites general in character.

PETITIONER: Columbia Pictures


RESPONDENTS: Court of Appeals, Tube Video, Edward Cham et al.

SUMMARY: NBI Agent Reyes filed with the RTC of Pasig applications for
search warrant, alleging that private respondents Tube Video, Edward
Cham et al. had in their possession pirated video tapes, and equipment
used in recording and dubbing films. Judge Austria personally examined
Reyes and his witnesses, and finding just and probable cause, she issued
the corresponding Search Warrants.

On a motion to quash filed by Tube Video et al., Judge Austria reversed her
ruling based on the fact that (1) Columbia was uncertain of their ownership
to the titles, (2) They failed to present the master tapes, and (3) They did not
deposit and properly register their applications. Columbia appealed to this
Order. CA Justice Montoya ruled that the first and third assignment of errors
are to be sustained, but the second is not.

The SC affirmed this ruling. Jurisprudence provides that in cases regarding


PD 49 or Intellectual Property Rights, the linkage between copyrighted films
to pirated films must be established.

DOCTRINE: A basic requirement for the validity of search warrants, in cases


of this nature, is the presentation of the master tapes of the copyrighted
films from which pirated films are supposed to have been copied.

FACTS:
On April 1988, the NBI through Agent Reyes filed with the RTC of Pasig
three applications for search warrant against private respondents:
Tube Video, Edward Cham, the Blooming Rose Tape Center and Ma.
Jajorie Uy, and the Video Channel and Lydia Nabong.
Said respondents were charged with violation of PD 49, Section 56 otherwise
known as the Decree on the Protection of Intellectual Property.
The NBI claimed that said Tube Video et al. had with them pirated video
tapes, posters, leaflets, television sets, dubbing machines, laser disc
recorders, rewinders and etc. Things that are used for pirating films.
The RTC Judge Austria conducted a joint hearing wherein she made a
personal examination of the Agent Reyes and his witnesses. Finding
just and probable cause, she issued the corresponding Search
Warrants.
Tube Video et al. filed motions to quash said warrants.
In a Joint Order, Judge Austria ruled that the NBI had the authority to
apply for the search warrants; that in the issuance due process was
However, she reversed her former stand find and ordered the quashal on to the question of whether there was unauthorized transfer of a
these grounds: sound recording or motion picture.
Columbia Pictures et al. were uncertain of their ownership of the titles
subject of the seized video tapes
Columbia Pictures et al. did not comply with the requirement that the
master tapes should be presented during the application for
search warrants
Columbia Pictures et al. cannot seek the protection of Philippine laws as
they failed to comply with the deposit and registration requirements.
Judge Austria thus ordered the return of the items seized by virtue of the
warrants.
Columbia Pictures et al. appealed said Order and assigned the following
errors:
Court erred in ruling that they were uncertain of their ownership of said
titles
Court erred in ordering the quashal on the ground that the requirement
of producing said tapes was applicable to the facts of the instant
case
Court erred in ruling they do not have a protectable copyright under
Philippine laws for failure to deposit and register
On October 1990, the CA through Justice Montoya rendered a decision
sustaining the first and third (a & c) assignment of errors but rejected
the second.
Hence, this appeal.

ISSUE/s:
WoN CA erred in ruling that the requirement of producing master tapes in
cases of this nature is required for a search warrant to be issued –
NO

RULING:. Petitions denied for lack of merit.

RATIO:
In 20th Century Fox v. CA, the Court has already laid down the rule that a
basic requirement for the validity of search warrants, in cases
of this nature, is the presentation of the master tapes of the
copyrighted films from which pirated films are supposed to
have been copied.
The essence of a copyright infringement is the similarity or at least
substantial similarity. Hence, the applicant must present to the court
the copyrighted films to compare. This linkage of the copyrighted
films to the pirated films must be established to satisfy the
requirements of probable cause. Mere allegations as to the existence
of the copyrighted films cannot serve as basis for the issuance of a
search warrant.
There is a need to establish probable cause that the tapes being sold,
leased or distributed are pirated tapes, hence the issue reverts back
COLUMBIA PICTURES v. CA (IYA) Judge Pelayo granted the motion to quash and ordered the return of all
September 20, 1996 | Romero, J. | Validity of a Search Warrant seized articles.

PETITIONER: Columbia Pictures Entertainment, Inc., MGM


Entertainment Co., Orion Pictures Corporation, Paramount Pictures
Corp., Universal City Studios, Inc. The Walt Disney Company, and
Warner Brothers, Inc.
RESPONDENTS: Court of Appeals and Jose B. Jingco of Showtime
Enterprises Inc.

SUMMARY: Petitioners are media and entertainment corporations who


question the cancellation of the search warrant against a certain Jingco
who was found in possession of pirated videotapes, etc. The lower court
in lifting the search warrant based in decision in 20th Century Fox v CA
which requires the presentation of the master tape as proof of probable
cause. The SC in the present case ruled that the aforementioned case
should not apply since there has been strict compliance of the judge
issuing the warrant in the requirements of a valid search warrant as it
personally conducted examination on probable cause.

DOCTRINE: A search warrant may be deemed to satisfy the legal


requirements of specificity when:
description is specific as the circumstances will allow
description expresses a conclusion of fact not of law by which
the warrant officer may be guided in making search and
seizure
things described are limited to those which bear direct
relation to the offense

FACTS:
Alfredo Ramos, intelligence office of the Videogram Regulatory Board
(VRB), received information that Jingco had in his possession pirated
videotapes, posters, advertising materials, and other items used for
the purpose of sale, lease, distribution, etc. of said pirated
videotapes.
Ramos ascertained the information to be true and filed a verified
Application for Search Warrant with a prayer for the seizure of the
properties described in the search warrant.
Judge Flor of RTC of Pasig issued Search Warrant No. 23
Jingco filed a Motion to Quash Search Warrant No. 23 on the ground that
the warrant did no state a specific offense and assuming that it did, it
covered more that one specific offense. VRB opposed this motion.
RTC denied the motion to quash.
Jingco filed an Urgent Motion to Lift the Search Warrant and For the
Return of the Seized Articles
Petitioner media/entertainment corporations appealed the above
decision. CA appealed the RTC decision in toto. Hence, this petition.

ISSUE/s:
WON the grant of Search Warrant 23 was valid - YES

RULING: Motion for Reconsideration is Denied

RATIO:
The RTC relied ona Court’s ruling in 20th Century Fox Film Corporation v
CA:
Search warrants issued by the lower court were lifted (by the
lower court also) not he ground of lack of probable cause
because the master tapes of the alleged pirated tapes were
never shown to the lower court.
The SC affirmed the lifting of the search warrants holding that
the presentation of master tapes was necessary for the
validity of the search warrants against those who have the
pirated films in their possession.
Petitioners now question the retroactive application father 20th Century
Fox decision which had not yet been promulgated in 1986 when the
search warrant was issued and that said warrant was specific and
complete.
The SC ruled that the 20th Century Ruling is not applicable to the
present case.
In the issuance of Search Warrant 23 there has been satisfactory
compliance with the then prevailing standards under the law for
determination of probable cause.
The court could not possibly have expected more evidence from
petitioners int heir application for a search warrant other than what
the law and jurisprudence, then existing and judicially accepted,
required with respect to the finding for probable cause.
A search warrant may be deemed to satisfy the legal requirements of
specificity when:
description is specific as the circumstances will allow
description expresses a conclusion of fact not of law by which
the warrant officer may be guided in making search and
seizure
things described are limited to those which bear direct relation to
the offense
Presentation of master tapes in such cases is not an absolute
requirement for a search warrant to issue. The 20th Century case
needs revisiting and clarification.
BURGOSv. CHIEF OF STAFF (ELIEL) RULING: Warrants are null and void, search and seizure is illegal.
December 26, 1984 | Escolin, J. | Search and Seizures

PETITIONER: Jose Burgos, Sr., Jose Burgos, Jr., Bayani Soriance and
Burgos Media Servics, Inc.
RESPONDENTS: Chief of Staff, AFP, Philippine Constabulary

SUMMARY: Col. Abadillo filed a complaint that the “Metropolitan Mail”


and “We Forum” and the equipments therein owned by Burgos, were
being used and continuously used for subversion. As such, a search
warrant was issued and several items (printers, vehicles, newspapers)
were seized. Hence this appeal.

The SC ruled that the issued warrants were invalid because: (1) they were
general in nature – they did not state as to what particular items are used in
furtherance of subversion, but only indicated equipments; and (2) that the
allegation by Abadillo did not provide sufficient evidence to establish probable
cause. Thus, it should’ve stated with particularity and with supporting
affidavits to establish probable cause that the materials are being used or
intended to be used are for subversion

DOCTRINE: Probable cause for a search is defined as such facts and


circumstances which would lead a reasonably discreet and prudent man
to believe that an offense has been committed and that the objects
sought in connection with the offense are in the place sought to be
searched.

FACTS:
Assailed in this petition is the validity of the two search warrants issued by
Judge Cruz-Pano of the QC RTC, under which the premises known as
No. 19, Road 3, Project 6, QC, and 784 Units C and D, RMS Building,
QC business addresses of the “Metropolitan Mail” and “We Forum”
newspapers, respectively, were searched, and office and printing
machines, equipment, paraphernalia, motor vehicles and other aticles
used in the printing, publication and distribution of the said newspapers,
as well as numerous papers, documents, books and other written
literature alleged to be in the possession and control of Burgos of the
“We Forum” newspaper, were seized.
Burgos prays that a writ of preliminary mandatory and prohibitory injunction
be issued for the return of the seized articles, and that Chief of Staff et al
be enjoined from using the articles seized as evidence against them.
The Sol Gen manifested, however, that the seized articles would not be
used as evidences.

ISSUE/s:
WoN the search and seizure was valid – NO
RATIO: the witnesses he may produce, and particularly describing the
The Court takes cognizance of the petition by Burgos in view of the place to be searched and the persons or things to be seized.
seriousness and urgency of the constitutional issues raised not to Probable cause for a search is defined as such facts and
mention the public interest generated by the search of the “We circumstances
Forum” offices, which was televised in Channel 7 and widely
publicized in all metropolitan dailies. The existence of this special
circumstance justifies the Court to exercise its inherent power to
suspend its rules.
In the determination of whether a search warrant describes the
premises to be searched with sufficient particularity, it has been
held “that the executing officer’s prior knowledge as to the
place intended in the warrant is relevant.
This would seem to be especially true where the executing officer is
the affiant on whose affidavit the warrant had issued, and when
he knows that the judge who issued the warrant intended the
building described in the affidavit. And it has also been said that
the executing officer may look to the affidavit in the official
court file to resolve an ambiguity in the warrant as to the place
to be searched.
A search warrant may be issued for the search and seizure of the
following personal property:
Property subject of the offense;
Property stolen or embezzled and other proceeds or fruits of the
offense; and
Property used or intended to be used as the means of
committing an offense.
Ownership is of no consequence, and it is sufficient that the person
against whome the warrant is directed has control or possession of
the property sought to be seized, as Burgos was alleged to have in
relation to the articles and property seized under the warrants.
The Court ruled that machinery which is movable by nature becomes
immobilized when placed by the owner of the tenement, property or
plant, but not so when placed by a tenant, usufructuary, or any other
person having only a temporary right, unless such person acted as
the agent of the owner.
In the case at bar, Burgos does not claim to be the owners of the land
and/or building on which the machineries were placed. This being
the case, the machineries in question, while in fact bolted to the
ground remain movable property susceptible to seizure under a
search warrant.
It is contended by Burgos, however, that the abovementioned documents
could not have provided sufficient basis for the finding of a probable
cause upon which a warrant may validly issue in accordance with the
constitution.
Sec 2: No search warrant or warrant of arrest shall issue except
upon probable cause to be determined by the judge, or such
other responsible officer as may be authorized by law after
examination under oath or affirmation of the complainant and
which would lead a reasonably discreet and prudent man to believe that
an offense has been committed and that the objects sought in connection
with the offense are in the place sought to be searched.
And when the search warrant applied for is directed against a
newspaper publisher or editor in connection with the publication of
subversive materials, as in the case at bar, the application and/or
its supporting affidavits must contain a specification, stating with
particularity the alleged subversive material he has published or is
intending to publish.
Thus, the broad statement in the application that Burgos “is in
possession or has in his control printing equipment and other
paraphernalia, news publications and other documents which
were used and are all continuously being used as a means of
committing the offense of subversion punishable under PD 885
is a mere conclusion of law and does not satisfy the
requirements of probable cuase.
Allegation cannot serve as basis for the issuance of a search
warrant and it was a grave error for respondent judge to have
done so.
The Constitution requires no less than personal knowledge by the
complainant or his witnesses of the facts upon which the
issuance of a search may be justified. The oath required must
refer to the truth of the facts within the personal knowledge of
petitioner or his witnesses because the purpose thereof is to
convince the committing magistrate, not the individual making
the affidavit and seeking the issuance of the warrant, of the
existence of probable cause.
Another factor which makes the search warrants under
consideration constitutionally objectionable is that they are in
the nature of general warrants.
The premises searched were the business and printing offices of the
“Metropolitan Mail” and the “We Forum” newspapers. As a
consequence of the search and seizure, these premises were
padlocked and sealed, with the further result that the printing and
publication of said newspapers were discontinued.
Such closure is in the nature of previous restraint or censorship
abhorrent toteh freedom of the press guaranteed under the
fundamental law and constitutes a virtual denial of petitioners’
freedom to express themselves in print.
Abad Santos, J. Concurring: Any search warrant s conducted in
disregard of the points mentioned above will result in wiping “out
completely one of the most fundamental rights guaranteed in our
Constitution, for it would place the sanctity of the domicile and the
privacy of communication and correspondence at the mercy of the
whims caprice or passion of peace officers.”
PEOPLE v. TEE (JP) requirements of a valid search warrant - YES
January 20, 2003 | Quisumbing, J. | Validity of a Search Warrant
RULING: SC affirmed the lower court’s decision with modification of the
PETITIONER: People of the Philippines penalty.
RESPONDENTS: Modesto Tee
RATIO:
SUMMARY: Appellant Tee is a Chinese businessman who smuggled The constitutional requirement of reasonable particulairt of description of
marijuana in the pretext of cigarettes. He avers that the search warrant the things seized is primarily meant to enable the law enforcers
issued to him was too general as to make the evidence searched serving the warrant to:
(marijuana) inadmissible. The RTC ruled that they are admissible. SC Readily identify the properties to be seized and thus prevent
agreed explaing that the purpose of the requirement was still met with the them from seizing the wrong items
language of the warrant. It is not necessary to be technical and specific if Leave said peace officers with no discretion regarding the
the nature of the object itself (narcotics in this case) permits so. articles to be seized and thus prevent unreasonable
searches and seizures
DOCTRINE: Tt is not required that technical precision of description be What the constitution seeks to avoid are search warrants of broad
required, particularly, where by the nature of the goods to be seized, their or general characterization or sweeping descriptions, which will
description be required, particularly, where by the nature of the goods to authorize police officers to undertake a fishing expedition to seize
be seized, their description must be rather general, since the requirement and confiscate any and all kinds fo evidence or articles relating to an
of a technical description woud mean that no warrant could issue. offense.
However, it is not required that technical precision of description be
required, particularly, where by the nature of the goods to be seized,
their description be required, particularly, where by the nature of the
FACTS: goods to be seized, their description must be rather general, since
Appellant Modesto Tee is a Chinese businessman. Under the pretext that the requirement of a technical description woud mean that no warrant
he would be storing smuggled cigarettes, a certain Ballesteros could issue.
agreed to store it in house. The term “narcotics” paraphernalia” is not so wanting in particularity as to
After learning about the Marijuana stored in the boxes, Ballesteros asked create a general warrant.
witness Danilo, a taxi driver, to store it elsewhere. Nor is the description “any and all narcotics” and “all implements,
Upon being tricked by Tee to load more Marijuana in his car, Danilo paraphernalia, articles, papers and records pertaining to the use,
reported disclosed the matter to the NBI. possession, or sale of narcotics or dangerous drugs so broad as to
Eventually, the NBI officers were able to seize several boxes of be unconstitutional.
marijuana in Modesto’s residence (Second Case). He was then A search warrant commanding officers to seize “a quantitu of loose
arrested for illegal possession of marijuana. heroin” has been held sufficiently particular.
Appellant Tee avers that the phrase “an undetermined amount of
marijuana” as used in the search warrant fails to satisfy the
requirement of Article 3, Section 2 of the Constitution that things to
be seized must be particularly described. (NOTE: This is the only fact
in the case book. Yes, only fact no. 3. All other facts here are from
the orig.)
The trial court ruled that the marijuana seized in the first case was
inadmissible, being products of illegal search. Hence, he was
acquitted. However, for the second case, the trial court
convicted him of illegal possession of marijuana.

ISSUE/s:
1. WoN the phrase “undetermined amount of marijuana” satisfied
the
PEOPLE v. DEL NORTE (HENRY) obtaining the search warrant and subsequently implementing it. They
March 31, 2004 | Puno, J. | Search Warrant had personal knowledge of the identity of the persons and the place
to be searched although they did not specifically know the names of
PETITIONER: People of the Philippines the accused.
RESPONDENTS: Priscilla del Norte In the present case, the irregularity of the search warrant is
unacceptable.
SUMMARY: Respondent has been assailing the in the lower courts the It is evident that the authorities did not have personal knowledge of the
validity of the search warrant used as basis of the arrest of Priscilla, it circumstances surrounding the search. No surveillance was done
baring the name of her sister, Ising, instead of hers. The lower courts prior to the search warrant being obtained.
ruling on her favor led for the petitioners to file a motion in the SC. It was only when the warrant was implemented that they coordinated with
the barangay officials.
The SC ruled in favor of Priscilla. While the SC acknowledged the fact
that they ruled in some cases the validity of such warrants even if the
names were incorrect, it was emphasized that such warrants underwent
prior investigation by the authorities through surveillance—contrary to
what happened in this case, where no surveillance was conducted.

DOCTRINE: Search warrants with defects with regard to the name of


theaccused are to be rendered valid if it is shown that authorities
conducted surveillance and a test-buy operation before obtaining the
search warrant and subsequently implementing it.

FACTS:
Priscilla is assailing the admissibility of the marijuana seized as a result
of the search conducted by the authorities, as the warrant contained
the name Ising Gutierrez Diwa (her sister) and not hers.
During her arrest, she claimed she saw Ising nearby and pointed her to
the authorities, but her efforts were useless, and the arrest was
successful.
Her contention was rendered invalid by the lower curt, hence the
petitioner’s current motion to the SC.

ISSUE/s:
WoN the warrant of arrest is valid despite it being named after Priscilla’s
sister - NO

RULING: SC denied the petition.

RATIO:
The Constitution requires search warrants to particularly describe not
only the place to be searched, but also the persons to be arrested.
In only a few cases was it ruled that mistakes in the name of the person
subject of the search warrant, provided that the place to be searched
is properly described.
In People v. Tiu Won Chua, the court upheld the validity of the search
warrant despite the mistake in the name of the persons to be searched.
In the cited case, the authorities conducted surveillance and a test-buy
operation before
LIM v. FELIX (DANNAH) Susana Lim et al. of the crime of multiple murder and frustrated
February 19, 1991 | Gutierrez, Jr., J. | Warrant of Arrest: Probable murder in connection with said incident.
Cause After conducting a preliminary investigation, the court issued an order on

PETITIONER: Vicente Lim, Sr. and Mayor Susana Lim


RESPONDENTS: Hon. Felix and Hon. Alfane

SUMMARY: Congressman Espinosa Sr. and his security escorts were


assassinated in the Masbate Domestic Airport. One of his escorts survived
but was also wounded. Investigator Tantiado filed with the MTC of Masbate
a complaint accusing the respondents Lim et al. of multiple murder and
frustrated murder. After conducting preliminary investigation, the court
issued an order stating that probable cause has been established for the
issuance of a warrant of arrest.

Fiscal Alfane a few months after issued a Resolution affirming the finding of
a prima facie case against respondents Lim et al. but ruled that the crime
must be one count of murder for each and physical injuries for the survivor.
4 separate informations of murder were filed against respondents Lim with
the Masbate RTC. Respondents Lim filed a petition for change of venue with
the SC and was granted.

It was filed to the Sala of Judge Felix, who ruled that the court finds it proper
to rely on Fiscal Alfane’s certification in each information. Respondents Lim
et al. then filed consolidated petitions questioning said Order, and the SC
issued a TRO against the issuance of the warrants without bail.

If a Judge relies solely on the certification of the Prosecutor as in this case


where all the records of the investigation are in Masbate, he or she has not
personally determined probable cause. The determination is made by the
Provincial Prosecutor. Because of this, the Court ruled that constitutional
requirement has not been satisfied.

DOCTRINE: The Court reiterates that in making the required personal


determination, a Judge is not precluded from relying on the evidence earlier
gathered by responsible officers. The extent of the reliance depends on the
circumstances of each case and is subject to the Judge’s sound discretion.
However, the Judge abuses that discretion when having no evidence before
him, he issues a warrant of arrest.

FACTS:
On the morning of March 17, 1989, Cong. Espinosa Sr. and his security
escorts were assassinated at the airport road of the Masbate
Domestic Airport. One of his escorts survived but was also wounded.
Investigator Tantiado of the PC Criminal Investigation Service filed a
complaint with the MTC of Masbate accusing Vicente Lim Sr., Mayor
July 1989 stating that:
After weighing the affidavits and answers given by the witnesses
for the prosecution during the preliminary investigation, a
probable cause has been established for the issuance of a
warrant of arrest
In the same Order, the court ordered the arrest of Vicente Lim et al. and
recommended a bail of P200k. Others appealed and bail was
lowered to P150k for them. All paid bail except for Cabarles.
On September 1989, Fiscal Alfane issued a Resolution affirming the
finding of a prima facie case against Lim et al. but ruled that the
crime must be one count of murder for each of the killing of the four
victims and physical injuries for the one who survived.
A motion to reconsider was filed by the Lims but was denied.
On October 1989, Fiscal Alfane filed with the RTC of Masbate 4 separate
informations of murder against the 12 accused with the
recommendation of no bail.
On November 1989, Lim et al. filed a petition for change of venue which
was granted. The cases were raffled to Branch 56 presided by
respondent Judge Felix.
The Lims filed with the court several motions, which prayed for the
transmittal of initial records of the preliminary inquiry or investigation.
This was opposed by the prosecution.
On June 1990, the respondent court issued an order denying the motion
for lack of merit and issued the warrants against Lim et al.
Judge Felix stated:
Considering that both the competent officers to whom such duty was
entrusted by law have declared the existence of probable cause,
each information is complete in form and substance, and there is
no visible defect on its face, this Court finds it just and proper to
rely on the prosecutor’s certification in each information.
Lim et. Al then filed consolidated petitions questioning the Order, and the
SC filed a TRO against the implementation of the warrants of arrest.

ISSUE/s:
WoN a Judge may issue a warrant of arrest without bail by simply relying
on the prosecution’s certification and recommendation that a
probable cause exists – NO

RULING:. Petitions granted. Order of Judge Felix declared Null and Void and
set aside. TROs and Injunctions issued in the Petitions are made permanent.

RATIO:
It was ruled in Placer v. Villanueva that a judge may rely upon the fiscal’s
certification of the existence of probable cause and, on the basis
thereof, issue a warrant of arrest. However, the certification does not
bind the judge to come out with the warrant of arrest.
Section 6, Rule 112 of the Rules of Court states that: In People v. Delgado, the Court ruled that the judge may rely on the
Warrant of arrest, when issued: If the judge be satisfied from the resolution of the COMELEC to file the information, by the same
preliminary examination conducted by him or by the investigating
token that it may rely
officer that the offense complained of has been committed and that
there is reasonable ground to believe that the accused has
committed it, he must issue a warrant or order for his arrest.
Under this section, the judge must satisfy himself of the existence of
probable cause before issuing a warrant of arrest.
If on the face of the information the judge finds no probable cause, he
may disregard the fiscal’s certification and require the submission of
the affidavits of witnesses to aid him in arriving at a conclusion as to
the existence of a probable cause.
Reiterating Soliven v. Makasiar, the addition of the word “personally” after
“determined” in Section 2 Article III means that it is the exclusive
and personal responsibility of the issuing judge to satisfy
himself of the existence of probable cause.
People v. Honorable Inting ruled the following:
First, the determination of probable cause is a function of the Judge. It is not
for the Provincial Fiscal or Prosecutor nor for the Election Supervisor to
ascertain. Only the Judge and the Judge alone makes this
determination.
Second, the preliminary inquiry made by a Prosecutor does not bind the
Judge. It merely assists him to make the determination of probable
cause. By itself, the Prosecutor’s certification of probable cause is
ineffectual. It is the report, the affidavits, the transcripts of
stenographic notes (if any), and all other supporting documents
behind the Prosecutor’s certification which are material in assisting
the Judge to make his determination.
Third, Judges and Prosecutors alike should distinguish the preliminary
inquiry which determines probable cause for the issuance of a
warrant of arrest from the preliminary investigation proper which
ascertains whether the offender should be held for trial or released.
The determination of probable cause for the warrant of arrest is made by
the Judge. The preliminary investigation proper—whether or not
there is reasonable ground to believe that the accused is guilty of the
offense charged and, therefore, whether or not he should be subjected
to the expense, rigors and embarrassment of trial—is the function of the
Prosecutor.
It must be clear that while an RTC Judge may no longer conduct
preliminary investigations to ascertain whether there is sufficient
ground for the filing of a criminal complaint or information, he retains
the authority, when such a pleading is filed with his court, to
determine whether there is probable cause justifying the issuance of
a warrant of arrest.
the certification made by the prosecutor who conducted the preliminary
investigation in the issuance of the warrant of arrest.
It was reiterated, however that “the court may require that the record of the
preliminary investigation be submitted to it to satisfy itself that there is
probable cause…”
If a Judge relies solely on the certification of the Prosecutor as in this case
where all the records of the investigation are in Masbate, he or she has
not personally determined probable cause. The determination is made by
the Provincial Prosecutor. The constitutional requirement has not been
satisfied. The Judge commits a grave abuse of discretion.
The records of the preliminary investigation were still in Masbate when Judge
Felix issued said warrants. There was no basis for him to make his
own personal determination regardin the existence of probable cause.
He could not possibly have known what transpired in Masbate as he had
nothing but the certification.
It is also worthy to note that petitioners Lim presented a recantation of
witnesses whose testimonies were used to establish a prima facie case.
Although the general rule is that recantations are not given much weight,
Jude Felix should, at the very least, have gone over the records
conducted.
The Court reiterates that in making the required personal determination, a
Judge is not precluded from relying on the evidence earlier gathered by
responsible officers. The extent of the reliance depends on the
circumstances of each case and is subject to the Judge’s sound
discretion. However, the Judge abuses that discretion when having no
evidence before him, he issues a warrant of arrest.
Judge Felix committed a grave error when he relied solely on the
Prosecutor’s certification and issued the questioned Order dated July 5,
1990 without having before him any other basis for his personal
determination of the existence of a probable cause.
PEOPLE v. FRANCISCO (IYA) WON the search and seizure conducted is in accordance with the
August 22, 2002 | Ynares-Santiago, J. | Validity of a Search Warrant warrant -

PETITIONER: People of the Philippines


RESPONDENTS: Annabelle Francisco y David

SUMMARY: SPO2 Teneros together with police officers conducted a


search and seizure operation in apartment no. 120 of Annabelle
Francisco. There they confiscated several drug paraphernalia which was
used as evidence in the prosecution of Annabelle. The validity of the
search is being questioned as it conducted the search in a different place
rather than that written in the warrant (Apartment no. 122).

The SC ruled that the search and seizure conducted was invalid as the
officers, although familiar with the place to be searched, acted outside
the authority granted by the warrant.

DOCTRINE: The place to be searched, as set in the warrant, cannot be


amplified or modified by the officers own personal knowledge of the
premises. The particularization of the description f the place to be
searched may properly be done only by the judge and only in the warrant
itself. It cannot be led tot he discretion of the police officers conducting
the search

FACTS:
Frederico Verona and live in girlfriend Annabelle Francisco were under
surveillance after the police confirmed, through a test-buy operation,
that they were engaged in selling shabu.
SPO2 Teneros sought the assistance fo the arrest of Verona. Judge
Bayhon authorized the search of shabby and paraphernalia at No.
122 M. Hizon Street, Caloocan City.
Annabelle Francisco while at No. 120 M. Hizon Street, heard a loud bang
and saw that policemen suddenly entered and conducted a search.
The police team seized drugs and drug paraphernalia. They also
allegedly seized the amount of P180,000, a Fiat car, jewelry, set of
keys, an ATM card, bnank books, and car documents.
Annabelle pleaded not guilty and filed a motion to quash the search
warrant
RTC upheld the validity of the search warrant.
It is argued that the warrant was served at apartment no 120 and not the
stated apartment no. 122. RTC ruled that this will not by itself render
as illegal the search and seizure.

ISSUE/s:
WON the search warrant is valid - YES
NO

RULING: Accused is acquitted.

RATIO:
RTC argues that while it is true that the rationale behind the
constitutional and procedural requirements that the search warrant
must particularly describe the place to be searched is to the end that
no unreasonable search warrant and seizure may not be made and
abuses may not be committed. However, this is not without
exception.
A description of the place is sufficient if the officer with the warrant
can with reasonable effort ascertain and identify the place
intended.
The requisites for the issuance of a valid search warrant are:
probable cause is present
such presence is determined personally by the judge
the complainant and the witnesses he or she may produce are
personally examined by the judge
applicant and the witness testify on the facts personally known to
them
warrant specifically describes the place to be searched and
things to be seized
The absence of any of these requisites will cast the downright
nullification of the search warrants.
There is nothing wrong or lacking in the issued warrant. What seems to
be the controversy is the actions of the police officers.
It was neither fair not licit to allow police officers to search place different
form that stated in the warrant not he claim that the place actually
searched although not that specified in the warrant was exactly what
they had in view when they applied for the warrant.
The place to be searched, as set in the warrant, cannot be amplified or
modified by the officers own personal knowledge of the premises.
The particularization of the description f the place to be searched
may properly be done only by the judge and only in the warrant itself.
It cannot be led tot he discretion of the police officers conducting the
search.
All items seized during the illegal search are prohibited form being used
in evidence. Hence, the acquittal of the accused.
MARQUEZ v. DESIERTO (ELIEL) WoN Marquez may be cited for indirect contempt for failure to produce
June 27, 2001 | Pardo, J. | Bank Secrecy documents requested– NO

PETITIONER: A. Marquez
RESPONDENTS: Hon. Aniano Desierto

SUMMARY: The Ombudsman requested Union Bank of the Philippines


Branch Manager Marquez to produce several bank documents of Amado
Lagdameo et al., for purposes of inspection in violation of RA 3019. The
Ombudsman stated that it is within its power to conduct investigation and
require the production of records and documents, under RA 6770.
However, Marquez contends that there is a bank secrecy law and she is
not allowed to produce said documents except when (see below). Hence
the appeal.

The SC ruled that although the purpose of the inspection is grounded on RA


3019, it is settled that before an in camera inspection may be allowed, there
must be a pending case before a court of competent jurisdiction. In this case,
there was no pending case but only an investigation by the Ombudsman to
gather additional evidence to formally charge lagdameo. Thus, Marquez may
not be cited for indirect contempt and may not give the bank accounts to the
Ombudsman.

DOCTRINE: RA 1405, the secrecy of bank deposits law allows exceptions:


Where the depositor consents in writing;
Impeachment cases;
By court order in bribery or dereliction of duty cases against
public officials;
Deposit is subject of litigation;
Sec 8, RA 3019, in cases of unexplained wealth.

FACTS:
Marquez received an Order from the Ombudmsna Desierto to produce
several bank documents for purposes of inspection in camera
relative to various accounts maintained at Union Bank of the
Philippines, Julia Vargas Branch, where Marquez is the branch
manager.
It is worth mentioning that the power of the Ombudsman to investigate
and to require the production and inspection of records and
documens is sanctioned by the 1987 Philppine Constitution, RA
6770, otherwise known as the Ombudsman Act of 1989 and under
existing jurisprudence on the matter.
Clearly, the specific provision of RA 6770, a later legislation, modifies the
law on the Secrecy of Bank Deposits (RA1405) and places the office
of the Ombdusman in the same footing as the courts of law in this
regard.

ISSUE/s:
WoN the order of the Ombudsman is allowed as exception to the law on
RA 1405 - NO

RULING: Petition Granted. Ombudsman order is not anexception the law on RA


1405

RATIO:
RA 1405, the secrecy of bank deposits law allows exceptions:
Where the depositor consents in writing;
Impeachment cases;
By court order in bribery or dereliction of duty cases against
public officials;
Deposit is subject of litigation;
Sec 8, RA 3019, in cases of unexplained wealth.
We rule that before an in camera inspection may be allowed, there must
be a pending case before a court of competent jurisdiction.
Further, the account must be clearly identified, the inspection limited to the
subject matter of the pending case before the court of competent
jurisdiction.
The bank personnel and the account holder must be notified to be
present during the inspection, and such inspection may cover only
the account identified in the pending case.
In the case at bar, there is yet no pending litigation before any court of
competent authority.
What is existing is an investigation by the office of the Ombudsman. In
short, what the Office of the Ombdusman would wish to do is to fish
for additional evidence to formally charge Amado Lagdamaeo, e a;.,
with the Sandiganbayan.
Clearly, there was no pending case in court, which would warrant the
opening of the bank account for inspection.
Zones of privacy are recognized and protected in our laws.
It also holds a public officer or employee or any private individual liable
for damages for any violation of the rights and liberties of another
person, and recognizes the privacy of letters and other private
communications.
MHP GARMENTS INC. v. CA (JP) A criminal complaint for unfair competition was then filed against private
September 2, 1994 | Ponencia, J. | Unreasonable Searches and respondents. The Provincial Fiscal of Rizal dismissed the complaint
Seizures against

PETITIONER: MHP Garments, Inc., Larry De Guzman


RESPONDENTS: Honorable Court of Appeals, Agnes Villa Cruz, et al.

SUMMARY: Petitioner MHP Garments was awarded the exclusive franchise


to sell and distribute scout uniforms. By virtue of a memorandum agreement
and an LOI, petitioner de Guzman, employee of MHP Garments, raided
respondents in their stores at the Marikina Public Market for selling scout
uniforms. RTC held that such seizure was illegal and ordered the return of
the items. CA affirmed.

The SC ruled that the search and seizure was indeed invalid. Such
search and seizure did not fall under any of the exceptions (exception
No. 2 was specifically mentioned). There was ample time for petitioners
to secure a judicial warrant but they deliberately failed to do so. There
was also no probably cause and the arrest was against the mandate of
the LOI.

DOCTRINE: Under the Rules of Court, a warrantless search can only be


undertaken under the following circumstance: Sec. 12. Search incident
to a lawful arrest. A person lawfully arrested may be searched for
dangerous weapons or anything which may be used as proof of the
commission of an offense, without a search warrant.

FACTS:
Petitioner MHP Garments, Inc. was awarded by the Boy Scouts of the
Philippines the exclusive franchise to sell and distribute official Boy
Scouts uniforms, supplies, etc.
In their Memorandum Agreement and Letter of Instruction No. 1299,
petitioner corporation was given authority to undertake the
prosecution in court of all illegal sources of scout uniforms.
Respondents Agnes Villa et al. were selling Boy Scouts items and
paraphernalia without any authority.
Petitioner de Guzman, employee of petitioner corporation was tasked to
undertake necessary surveillance and make a report tot he
Philippine Constabulary.
Petitioner de Guzman, and three other constabulary men, of the
Reaction Force Battalion went to the stores of respondents at the
Marikina Public Market.
Without any warrant they seized the boy and girl scout pants, dresss,
and suits on display at respondents’ stalls. The seizure caused
commotion and embarrassed private respondents. The items were
then turned over to petitioner corporation.
all respondents. He also ordered the return of the seized items. Not
all seized items were returned. Some were returned in inferior
quality.
Private respondents (Agnes et al.) filed a civil case against petitioner
corporation for sums of money and damages. The trial court ruled for
the respondents. The CA affirmed.

ISSUE/s:
WoN the rights of respondents against unreasonable search and
seizures were violated - YES

RULING: CA decision is affirmed.

RATIO:
The seizure was made without any warrant. Under the Rules of
Court, a warrantless search can only be undertaken under the
following circumstance: Sec. 12. Search incident to a lawful
arrest. A person lawfully arrested may be searched for
dangerous weapons or anything which may be used as proof of
the commission of an offense, without a search warrant.
The SC holds that the evidence did not justify the warrantless search and
seizure of private respondents’ goods.
The progression of time between the receipt of the information and
the read of the stores of private respondents shows there was
sufficient time for petitioners and the PC raiding party to apply
for judicial warrant.
Despite the sufficient of time, they did not apply for a warrant and seized
the goods of private respondents.
In doing so, they took the risk of a suit for damages.
In the case at bench, the search and seizure were clearly illegal. There
was no probable cause for the seizure. After a preliminary
investigation, the Provincial Fiscal of Rizal even dismissed their
complaint for unfair competition and later ordered the return of the
seized goods.
Also, the Letter of Instruction No. 1299 was precisely crafted to
safeguard not only the privilege of franchise holder (petitioner) of
scouting items but also the citizen’s (respondents) constitutional
rights. According to the LOI, there is a need for strict proper
application by the Boy Scouts of the Philippines for warrant of
arrest and/or search warrant with a judge before apprehending
unauthorized manufacturers.
Since petitioners were indirectly involved in transgressing the right of
private respondents against unreasonable search and seizure, the
award for damages is justified.
PEOPLE v. ARUTA (HENRY) The following cases (of allowable warrantless search) are
April3, 1998 | Romero, J. | Searches and seizures specifically provided or allowed by law:

PETITIONER: People of the Philippines


RESPONDENTS: Rosa Aruta

SUMMARY: December 1988, respondent Aruta was arrested and


charged for transporting drugs. She was arrested right after she was
dropped of by a bus. Basically the respondent’s contention is that the
search is invalid, it being issued without probable cause and that there
was no warrant presented upon her arrest.

The SC ruled that the search and arrest was invalid. There was an
absence of probable cause, there was no warrant of arrest presented
(which consequently invalidated the search done hence the drugs
recovered are inadmissible.

DOCTRINE: The guidelines on warrantless searches and seizures


SHOULD NOT BECOME unbridled licenses for law enforcement officers
to trample upon the constitutionally guaranteed and more fundamental
right of persons against unreasonable search and seizures. The essential
requisite of probable cause MUST still be satisfied before a warrantless
search and seizure may be lawfully conducted.

FACTS:
December 1988, in Olongapo, respondent Aruta was arrested and
charged for transporting 8 kilos and 500 grams of dried marijuana
packed in plastic bags marked “Cash Katutak” placed in a travelling
bag.
Respondent plead “not guilty” in the arraignment.
The defense filed a “Demurrer to Evidence” alleging the illegality of the
search and seizure of the items thereby violating the respondent’s
constitutional right against unreasonable search and seizure as well
as the inadmissibility of the evidence.
In her defense, respondent Aruta claims that she just came from Choice
Theater where she watched the movie “Balweg” and that she was in
the middle of the road (alighting a bus) when the authorities asked
her to go with them to the NARCOM (police) office.
Defense was unsuccessful, the RTC was sentenced to life imprisonment
and a fine of Php 20,000.00 without subsidiary imprisonment in case
of
insolvency. Hence, this appeal.
ISSUE/s:
WoN the search warrant was valid - NO
RULING: SC reversed the RTC ruling and respondent is hereby acquitted,
unless she is being held for some other legal grounds.

RATIO:
Warrantless search incidental to a lawful arrest under Section 12, Rule
126 of the Rules of Court and by prevailing jurisprudence
Seizure of evidence in “plain view”, with the following elements:
0 A prior valid intrusion based o the valid warranatless arrest in
which the police are legally present in the pursuit of their
official duties
1 The evidence was inadvertently discovered by thepolice who
had the right to be where they are
2 The evidence must be immediately apparent
3 “plain view” justified mere seizure of evidence without further
search
Search of a moving vehicle. Highly regulated by the government, the
vehicle’s inherent mobility reduces expectation of privacy especially
when its transit in public thoroughfares furnishes a highly reasonable
suspicion amounting to probable casue that the occupant committed
a criminal activity
Consented warrantless search
Customs search
Stop and frisk
Extigent and emergency circumstances
The above exceptions SHOULD NOT BECOME unbridled licenses for law
enforcement officers to trample upon the constitutionally guaranteed and
more fundamental right of persons against unreasonable search and
seizures. The essential requisite of probable cause MUST still be satisfied
before a warrantless search and seizure may be lawfully conducted.
The right of a person to be secured against any unreasonable seizure of his body
and any deprivation of his liberty is a most basic and fundamental one.
In the present case, the policemen (NARCOM agents) were admittedly not armed
with a warrant of arrest. To legitimize the warrantless search and seizure of the
respondent’s bag, she should’ve been arrested validy under Sec 5 Rule 113,
which states that officers when in their presence, the person to be arrested has
committed, is actually committing, or is attempting to commit an offense. In this
case since the arrest was invalid, the search is likewise void.
In the absence of probable cause to effect a valid and legal warrantless arrest,
the search and seizure of the respondent’s bag would also not be justified as
seizure of evidence in “plain view” under the second exception. The
marijuana was obviously not immediately apparent as shown by the fact that
the policemen had to request the respondent to open her bag.
Neither will it be categorized as a search of a moving vehicle, as in the instant
case she was apprehended after going down the bus. She was literally in the
middle of the street.
NOLASCO v. PANO (DANNAH) searched.”
February 19, 1991 | Teehankee, J. (Concurring) | Warrant of Arrest / Search What must be considered is the balancing of the individual’s right to
and Seizure privacy and the public’s interest in the prevention of crime and the
apprehension of criminals.
This pronouncement had the support of a majority of 9 justices, three
PETITIONER: Cynthia Nolasco, Mila Aguilar-Roque et al. dissented while 2 took no part/dissented.
RESPONDENTS: Hon. Ernano Pasto, Hon. Santos, Hon. Apostol, Lt. Gen. The petitioners Mila et al. stressed in their motion for reconsideration that
Fidel Ramos et al. if this opinion becomes settled law, the constitutional protection
would become meaningless. The military or police would no longer
SUMMARY: Petitioner Mila was arrested aboard a public vehicle. In that case, apply for search warrants.
the majority opinion was that as an incident to her arrest, her dwelling could They would simply wait for a person to reach his house, then arrest him.
be searched even without a warrant for evidence of the charges of Even if the person arrested does not resist and has been taken away,
rebellion filed against her. They cited Section 12 Rule 126 of the rules of court under the majority ruling, the arresting party would still have the right
which states that a person charged with an offense may be searched for to search the house and cart away all his things and use them as
dangerous weapons or anything which may be used as proof of the commission evidence.
of the offense. They held that a person In such situation, what then happens to the stringent constitutional
requirement that “no search warrant… shall issue except upon
In that case, Justice Teehankee was one of the dissenters. In this case now, probable cause to be determined by the judge… and the
the majority upholds his dissent and uses his opinion as the ratio of the constitutional injunction that ‘any evidence obtained in violation of
case. He claims that a better application of said Rule should be to absolutely this… shall be inadmissible for any purpose in any proceeding’”.
limit a warrantless search of a person who is lawfully arrested to his or her
person at the time of and incident to his or her arrest. ISSUE/s:
WoN the majority ruling should be upheld – NO
DOCTRINE: The better and established rule is a strict application of the
exception provided for Sec. 12 Rule 126, and that is to absolutely limit a RULING:. Motion granted.
warrantless search of a person who is lawfully arrested to his or her person
at the time of and incident to his or her arrest and to “dangerous weapons or RATIO:
anything which may be used as proof of the commission of the offense.” The better and established rule is a strict application of the exception
Such warrantless search obviously cannot be made in a place other than provided for Sec. 12 Rule 126, and that is to absolutely limit a
the place of arrest. warrantless search of a person who is lawfully arrested to his or
her person at the time of and incident to his or her arrest and to
“dangerous weapons or anything which may be used as proof
FACTS: of the commission of the offense.”
Petitioner Mila Aguilar-Roque was arrested at 11:30am aboard a public Such warrantless search obviously cannot be made in a place other than
vehicle on the road (at Mayon and P. Margal Streets). the place of arrest.
The majority’s pronouncement at that time (in that case, Teehankee’s This constitutional mandate expressly adopting the exclusionary rule has
opinion was the dissent, now they are upholding his dissent), was proved by historical experience to be the practical means of
that as an incident to her arrest, her dwelling could be searched even
enforcing the constitutional injunction against unreasonable searches
without a warrant for evidence of the charges of rebellion filed
against her. and seizures by outlawing all evidence illegally seized and thereby
They cited Section 12 Rule 126 of the Rules of Court2 and held that: removing the incentive on the part of the military and police officers
A general rule that, as an incident of an arrest, the place or premises to disregard such basic rights.
where the arrest was made can also be searched without a search In this case, the arresting CSG group of the military themselves knew
warrant.
In this latter case, “the extend and reasonableness of the search must that they needed a search warrant but they obtained the void general
be decided on its own facts and circumstances, and it has been warrant in question. Necessarily, the seizure of documents and
stated that, in the application of general rules, there is some personal effects with
confusion in the decisions as to what constitutes the extend of the
place or premises which may be
Search without warrant of person arrested – A person charged with an offense may be searched
for dangerous weapons or anything which may be used as proof of the commission of the
offense.
such a void warrant could not be justified “as an incident of an arrest” outside
petitioner’s dwelling and the Constitution bars their admissibility as evidence
and ordains their return to petitioners.
PAPA v. MAGO (IYA) deputized in writing by the Commissioner of Customs, could, for the
February 28, 1968 | Zaldivar, J. | Warrantless Searches purposes of the enforcement of the customs and tariff laws, effect
searches, seizures and arrests.
Alagao had the legal duty to render assistance to Papa.
PETITIONER: Ricardo Papa, Juan Ponce Enrile, Martin Alagao The Tariff and Customs Code does not require warrant in the instant
RESPONDENTS: Mago case.
The Code authorizes persons having police authority under Section 2203
SUMMARY: Martin Alagao, head of the counter intelligence unit of of the Tariff and Customs Code to enter, pass through, or search any
Manila Police Department, received information of illegal goods to be land, inclosure; warehouse, store or building, not being a dwelling
released in the customs zone in the port of Manila. Acting on the house and also to inspect search and examine any vessel or aircraft
instructions of the Chief Police, his team intercepted two trucks and and any trunk, package, box or envelope or any person on board, or
seized the goods inside. Mago, respondents, claim that the seizure was stop and search and examine any vehicle, beast or person
invalid. suspected of holding conveying any dutiable or prohibited article
introduced into the Philippines contrary to law.
The SC ruled that the seizure was legal as Manila Police Department is There is a necessary difference between a search of a dwelling house or
given authority to conduct such search pursuant to the Tariff and other structure in respect of which a search warrant may readily be
Customs Code. obtained and a search of a ship, motorboat, wagon, or
automobile for contraband goods where it is not practicable to
DOCTRINE: There is a necessary difference between a search of a secure a warrant because the vehicle can be quickly moved out
dwelling house or other structure in respect of which a search warrant of the locality or jurisdiction in which the warrant must be
may readily be obtained and a search of a ship, motorboat, wagon, or sought.
automobile for contraband goods where it is not practicable to secure a
warrant because the vehicle can be quickly moved out of the locality or
jurisdiction in which the warrant must be sought.

FACTS:
Martin Alagao, head of the counter intelligence unit of the Manila Police
Department, acting upon a reliable information stating that a certain
shipment of misdeclared and undervalued goods would be released
at the customs zone of the port of Manila.
Following the orders of Ricardo Papa, Chief Police of Manila and ad duly
deputized agent of the Bureau of Customs, Alagao conducted
surveillance a the customs zone.
The intelligence unit went after two trucks in question and intercepted.
The goods inside the trucks were seized.
Mago contends that Alagao could not seize the goods in question ihtout a
search warrant.

ISSUE/s:
WON the warrantless search is valid - YES

RULING: The seizure by the members of the Manila Police Department of


the goods in question as in accordance with law.

RATIO:
1. The Chief of Manila Police Department, Rocardo Papa, having been
PEOPLE v. CFI (ELIEL) When asked where they were taking it, Medina replied that they were
November 17, 1980 | Guerrero, J. | Warrantless Searches bringing them to the Tropical Hut at EDSA.
Sabado rode with them and arriving at Tropical hut, waited for the man
PETITIONER: People of the Philippines who
RESPONDENTS: CFI Rizal, Jessie Hope and Monina Medina

SUMMARY: RASAC agents were informed of a shipment of highly dutiable


goods from Angeles City to Manila. When they spotted the car, the agents
stopped them for questioning and introduced themselves as RASAC agents.
They saw 4 boxes and asked Hope and Medina where they were taking
them, the agents followed and when the suspected man who was supposed
to receive them did not show up, Hope and Medina were brought to Camp
Aguinaldo. In their presence and of agents of BOC, BIR, and PC, a total of 11
boxes of highly dutiable watches and bracelets were retrieved and seized.
When a case was being filed against them in violation of Tariff and customs
laws, Hope and Medina contend that the evidence is inadmissible because
there was no warrant. Hence this appeal.

The SC ruled that there are certain cases wherein a warrant may be
indispensible. One of those is the case at bar, the authorized agents
were simply performing their duties under the Tariff and Customs laws.
Under the law on Tariff and customs, authorized egents may enforced the
law without need of warrant. Furtheremore, to rule whether a search is
reasonable or unreasonable is on the discretion of the judge.

DOCTRINE: “Except in the case of the search of a dwelling house,


persons exercising police authority under the customs law may effect
search and seizure without a search warrant in the enforcement of
customs laws.”

FACTS:
The Regional Anti-Smuggling Action Center (RASAC) was informed by
an undisclosed Informer that a shipment of highly dutiable goods
would be transported to Manila frm Angeles City on a blue Dodge
car.
Spurred by such lead, RASAC Agenes Manuel and Sabado, upon order
of the Chief of Intellgence and Operations, Col. Abad, stationed
themselves int eh vicinity of the toll gate of the North Diversion Road
at Balintawak, QC.
A light blue dodge car with Plate No. 21-87-73, driven by Sgt. Hope who
was accompanied by Medina approached the exit gate and after
giving the toll receipt swept away towards Manila.
At this point, the agent succeeded in blocking Sgt. Hope’s car and the
latter stopped. Manuel and Sabado who were in civilian clothes
showed their identification cards to Hope and introduced themselves
as RASAC agents.
The Agents saw 4 boxes on the back seat of the Dodge and upon inquiry
as to what those boxes were, Sgt. Hope answered, “I do not know.”
according to Medina was supposed to receive the boxes. As the man Clearly therefore, the agents acted not on the basis of a mere heresay
did not appear, they called off the mission and brought Hope to but on a confirmed information worthy of belief and probable cuase
Camp Aguinaldo. enough for them to adopt measures to freeze the fleeting event.
An inspection of Hope’s car at Camp Aguinaldo yielded eleven sealed
boxes, 4 on the rear sear and 7 more in the baggage compartment
which was opened on order of Col. Abad.
The boxes were opened beforer the presence of Hope and Medina,
representatives of BIR, BOC, PC, COSAC, and photographers of
DND.
The contents of the boxes revealed some 4,441 more or less writes
watches of assorted rands; 1,075 more or less watch bracelets of
assorted brands, supposedly untaxed.
After the requisite preliminary investigation, the Fiscal found the existene
of a prima facie case against Hope. When evidence was being
adduced, Hope contends that the evidence is inadmissible because
they were seized without warrant. CFI agrees with petitioners. Hence
this appeal.

ISSUE/s:
WoN search and seizure was unreasonable– NO

RULING: Ruling of CFI reversed and set aside.

RATIO:
The authority of persons duly commissioned to enforce tariff and
customs laws is quite exceptional when it pertains to the domain of
searches and seizures of goods suspected to have been introduced
in the country in violation of the customs laws.
If it shall appear that any breach or violation of the customs and tariff
laws of the PH has been committed, whereby or in consequence of
which such vessels or aircrafts, or the article, or any part thereof, on
board of or imported by such vessel or aircrafts, is liable to forfeiture
to make seizure of the same or any part thereof.
In Papa v. Mago, the Code does not mention the need of a search
warrant unlike Sec 2209, which explicitly provides that a “dwelling
house may be entered and searched only upon warrant issued by a
judge, upon sworn application showing probable cause and
particularly describing the place to be searched and person or thing
to be seized.”
“Except in the case of the search of a dwelling house, persons exercising
police authority under the customs law may effect search and seizure
without a search warrant in the enforcement of customs laws.”
Providentially, however, things turned out differently when the
undisclosed informer himself went along with the agents to the
rendezvous point where at the appointed time he positively identified
an approaching car as the one described by him a week earlier to be
the suspected carrier of untaxed merchandise.
The purpose of the constitutional guarantee against unreasonable
searches and seizures is to prevent violations of private security in
person and property and unlawful invasion of the sanctity of the
home by officers of the law acting under legislative or judicial
sanction and to give remedy against such usurpation when
attempted.
The right to privacy is an essential condition to the dignity and happiness
and to the peace and security of every individual, wheter it be of
home or of prsons and correspondence.
In the ordinary cases where warrant is indispensably necessary, the
mechanics prescribed by the Constitution and reiterated in the Rules
of Court must be followed and satisfied.
However, in the extraordinary events where warrant is not necessary to
effect a valid search or seizure, or when the latter cannot be
performed except without warrant, what constitutes a reaonsable or
unreasonable search or seizure becomes purely a judicial question,
determinable form the uniqueness of the circumstances involved.
Including the purpose of the search or seizure, the presence or absence
of probable cause, the manner in which the search and seizure was
made, the pace or thing searched and the character of the articles
procured.
The circumstances of the case at bar, undoubtedly fall squarely within
the privileged area where search and seizure may lawfully be
effected without the need of a warrant.
The authorized agents only did a faithful performance of a djuty
authorized under the Tariff and Customs Code directing them to
retrieve articles reasonably suspecte of having been possessed,
issued, or procured in violation of the tariff laws for which the
government has a direct interest.
PEOPLE v. LO HO WING (JP) The operatives disembarked from their car and inspected the baggage
January 21, 1991 | Gancayco, J. | Warrantless Search: Exception

PETITIONER: People of the Philippines


RESPONDENTS: Lo Ho Wing alis Peter Lo, Antonio Lim

SUMMARY: Tia was recruited as a deep penetration agent by the


Philippine Constabulary to bust his co-accused. He was tasked by his
co-suspect Lim with Peter Lo to embark to China. There, defendant
Peter Lo bought tin cans of tea bags from a local store. The tea bags
then contained meth. Upon their arrival in the Philippines, their taxi cabs
were searched by the officers without a valid warrant. The lower court
indicted them.

The SC affirmed the warrantless search explaining that the case at bar is
an exception to the general rule that searches must have a valid warrant.
The SC held that a search of a moving vehicle without a warrant is
permissible because the vehicle can quickly move out of the locality.

DOCTRINE: There are at least three well-recognized exceptions to valid


search warrant (instances of legal warrantless searches):
Search incidental to arrest
search of a moving vehicle
seizure of evidence in plain view

FACTS:
In a project of the the Philippine Constabulary codenamed Oplan Sharon
887, co-accused Reynaldo Tia (Tia) was recruited as a deep
penetration agent to bust his co-suspects for smuggling of illegal
drugs.
Tia was introduced by co-accused Lim Cheng Huat (Lim) to Peter Lo
(defendant herein). Peter Lo was the one assigned by Lim to go to
China in lieu of the the latter. Tia accompanied Peter Lo.
Tia and Peter Lo, upon arrival in Hong Kong, visited a local store where
Peter Lo purchased six tin cans of tea. Defendant Peter Lo then kept
the tea bags in his room.
Later that evening, Tia went to defendant’s room to talk to him but instead
saw two other men with defendant, burning a substance on a piece of
aluminum foil. They informed Tia that their actual cargo was Chinese
drugs.
On the expected date of arrival of the suspects from Hong Kong, upon
succeeding reports of Tia as a deep penetration agent, a team of
Narcotics Command formed in NAIA.
Lim met Peter Lo and Tia and boarded different cabs.
The car of the NARCOM operatives cut the path of the taxicab ridden by
defendant Peter Lo and Tia.
compartment. They found a tin can of tea from the red traveling bag
owned by defendant Lim. Some crystalline substance was found
inside the tea bags. A total of six coin cans were found. They were
later found positive of meth.
Lower court indicted the suspects.

ISSUE/s:
WoN the search on the appellant’s car without a search warrant is valid -
YES

RULING: The warrantless search is valid.

RATIO:
Defendant Lim contends that the warrantless search and seizure were
illegal for violating Section 2 Article 3.
According to Lim, the PC-CIS officers concerned should have had
procured a valid search warrant because they knew the date and
place of arrival of the appellants anyway, specifically, 2 days in
advance.
SC disagrees. There are at least three well-recognized exceptions to
valid search warrant (instances of legal warrantless searches):
Search incidental to arrest
search of a moving vehicle
seizure of evidence in plain view
The case at bar falls under the second exception. The search in question
was made as regards a moving vehicle. A valid warrant was not
necessary to effect the search on appellant and co-accused.
Warrantless searches are justified on the ground that it is not practicable
to secure a warrant because the vehicle can be quickly moved out of
the locality or jurisdiction in which the warrant must be sought.
Note: a fishing vessel found to be violating fishery laws may be seized
without a warrant on two grounds: (1) because they are usually
equipped with powerful motors that enable them to elude pursuit (2)
because the seizure would be incidental to the lawful arrest of the
crew.
CABALLES v. CA (HENRY) of the guidelines on warrantless searches and seizure.
October 20. 1925 | Puno, J. | Search Warrant
RATIO:
PETITIONER: Rudy Caballes The Constitution as well as the Rules of Court provides guidelines that
RESPONDENTS: Court of Appeals and People of the Philippines should be taken into consideration in cases where the warrant is
necessary. As for the exceptions of such warrant, what constitutes a
SUMMARY: Two policemen found a passenger jeep to be unusually reasonable or unreasonable search or seizure is purely a judicial
covered with kakawati leaves, leading them to inspect the said vehicle question, where several things are considered:
only to find smuggled goods. The petitioner here contends that such Uniqueness of the circumstances involved
evidence found in his jeep is inadmissible as the search was performed Purpose of search or seizure
without a warrant. And hence his constitutional right to unreasonable Presence or absence of probable cause
searches and seizures is violated. Manner which search and seizure was made
Place or thing searched and character of the articles procured
The SC ruled that the evidence is indeed inadmissible, as it violates the In the present case, the search was conducted with the absence of the
rules as far as warrantless searches and seizures are concerned. search warrant.
Highly regulated by the government, the vehicle’s inherent mobiulity
DOCTRINE: A violation of the exceptions on the rule pertaining to the reduces expectation of privacy especially when its transit in public
requisites of warrants of search and seizures will render the evidence thoroughfares furnishes a highly reasonable suspicion amounting to
obtained as inadmissible and will lead to acquittal of the accused. probable cause that the occupant committed a criminal activity.
One form of search of moving vehicles is the “stop and search” without
warrant at military or police checkpoints, which has been declared
FACTS: not to be illegal for as long as it is warranted by the exigencies of
9:15pm of June 1989, Sgt. Victorino Noceja and Pat. Alex de Castro public order and conducted in a way least intrusive to motorists. A
while on a routine patrol spotted a passenger jeep unusually covered checkpoint may either be a mere routine inspection or it may invove
with “kakawati”3 leaves. extensive search.
Suspecting that the jeep contained smuggled goods, the 2 police officers Routine inspections are not regarded as violative to the individual’s rights
flagged down the vehicle and asked what was loaded, but the driver against unreasonable search. The serarch normally permissible in
did not answer. this instance is limited to the ff:
Allegedly, the petitioner gave his consent when the policemen asked if Where the officer merely draws aside the courtain of a vacant
they could check the cargo—the two discovered bundles of vehicle which is parked on thepublic fair grounds
conductor wires owned by the National Power Corporation (NPC), Simply looks into a vehicle
weighing 700 kilos valued at Php 555,244.45. Flashes a light therein without opening the car’s doors
The jeep was vroight to the police station, where the policemen took Where the occupants are not subjected to a physical or body
pictures of the driver and the jeep with wires. Petitioner (driver) was search
incarcerated for 7 days in the Municipal Jail. Where the inspection of the vehicles is limited to a visual search
Petitioner is assailing the validity of the search and seizure performed by or visual inspection
the officers, as he was flagged down merely on suspicion that his Where the routine check is conducted in a fixed area
cargo might contain smuggled goods—hence no probable cause None of the above are observed by the policemen in the case at hand.
present. They had to reach inside the vehicle, lift the kakawati leaves and look
With the lower courts not ruling in his favor, the present petition is viled. inside the sacks before they are able to see the cable wires. It cannot
be considered as a simple routine check.
ISSUE/s: US v. Pierre held that physical intrusion of a part of the body of an agent
WoN the evidence procured in the search of smuggled goods in the into the vehicle goes beyond the area protected by the Fourth
jeepney is inadmissible against the petitioner - YES amendment.
Such search made by the policemen will only be permissible if the officers
RULING: SC granted the petition, finding no probable cause present and the
conducting the search have reasonable or probable cause to believe
defiance
before the
Kakawati leaves are shaped like oregano leaves. If you feel extra the day you get called for this,
tell ma’am this irrelevant fact lol
search, that either motorist is a law offender or they will find the
instrumentality or evidence pertaining to a crime in the vehicle to be searched
The vehicle in this case was flagged down just because of the kakawati
leaves covering it, which was found to be unusual. This defense of
the policeme is not tantamount to probable cause as such that would
justify the conduct of a search without a warrant.
OBRA v. CA (DANNAH) The trial court ruled in favor of the spouses, finding no investigation had been
October 28, 1999 | Mendoza, J. | Search and Seizure conducted by Obra or Dumpit to verify the complaint of Ms. Grybos. On

PETITIONER: Benjamin Obra, Brig. Gen. Tomas Dumpit


RESPONDENTS: Court of Appeals, Sps. James and June Brett

SUMMARY: Obra who was the Regional Director of the Bureau of Mins and
Geo Sciences (BMGS) in Baguio received a letter from Grybos, who was
complaining that private respondent Sps. Brett were conducting illegal
mining activities. Obra asked the help of Brig. Gen. Dumpit to help
apprehend a truck allegedly used by Sps. Brett in illegal mining in their area.

The truck was seized and Sps. Brett filed a complaint with the RTC alleging
that the truck had been seized without prior investigation to determine the
existence of probable cause. The defense of Obra and Dumpit was the
“moving vehicle” doctrine. The Court ruled that it could not be justified under
this doctrine because the truck was seized while it was entering the mining
area, not transporting minerals outside the area. Hence, there was no
finding of probable cause to warrant the unlawful seizure of the truck.

DOCTRINE: When a vehicle is stopped and subjected to an extensive


search, such a warrantless search has been held to be valid as long as the
officers conducting the search have reasonable or probable cause to
believe before the search that they will find the instrumentality or evidence
pertaining to a crime in the vehicle to be searched.

FACTS:
Petitioner Obra was the Regional Director of the Bureau of Mines and
Geo Sciences (BMGS) in Baguio.
On June 1985, he received a letter from Jeannette Grybos complaining
that private respondents Sps. Brett were conducting illegal mining
activities.
On the same day, Obra wrote to Brig. Gen. Dumpit, then the
Commanding General of the Philippine Constabulary, requesting
assistance in apprehending a truck allegedly used by Sps. Brett in
illegal mining in their area.
The following day, Obra wrote to Sps. Brett and Ms. Grybos informing them
that the BMGS was to conduct an ocular and field investigation in their
area.
Accordingly, those under Brig. Gen. Dumpit seized on July 1985 a truck
belonging to Sps. Brett as it was entering their mining area. The truck
was impounded and prevented from leaving except for military-
related missions.
Sps. Brett then filed a complaint with the RTC of Baguio and Benguet. They
allege that their truck was seized without determination of probable
cause.
appeal, the CA affirmed the RTC’s ruling. Hence, this petition.

ISSUE/s:
WoN there was determination of probable cause for seizing the truck –
NO

RULING:. Petition dismissed. Decision of CA affirmed.

RATIO:
…Nor indeeed could the seizure of the vehicle be justified under the
“moving vehicle” doctrine. The truck was seized while it was entering
the mining area; it was not transporting minerals outside of the area.
As held in People v. Bagista,
With regard to the search of moving vehicles, this had been
justified on the ground that the mobility of motor vehicles
makes it possible for the vehicle to be searched to move out
of the locality or jurisdiction in which the warrant must be
sought.
This in no way, however, gives the police officers unlimited
discretion to conduct warrantless searches of automobiles in
the absence of probable cause. When a vehicle is stopped
and subjected to an extensive search, such a warrantless
search has been held to be valid as long as the officers
conducting the search have reasonable or probable cause to
believe before the search that they will find the
instrumentality or evidence pertaining to a crime in the
vehicle to be searched.
There could not have been, therefore, any finding of probable cause that
the truck was being used for any illegal mining activities.
PEOPLE v. MALMSTEDT (IYA) Malmstedt was asked to alight the bus, in his travel bags was found two
June 19, 1991 | Padilla, J. | Warrantless Searches teddy bears. Inside the bears were also hashish (cute lol).
RTC found Malsmtedt guilty beyond reasonable doubt. He now seeks the
reversal fo the decision stating that the search of his personal effects
PETITIONER: People of the Philippines
was illegal because it was made without a search warrants and
RESPONDENTS: Mikael Malmstedt therefore cannot be admitted as evidence against him.

SUMMARY: Malmstedt is a foreign national who was found to be ISSUE/s:


transporting illegal drugs through a search and seizure conducted in a WON the warrantless search is valid - YES
checkpoint. He was found guilty by the RTC. He seeks the reversal of
the decision contending that the search and seizure is invalid for lack of RULING: Appealed judgement of conviction by the trial court is hereby
warrant. AFFIRMED.

The SC held that, as an exemption to the general rule, the warrantless RATIO:
search is legal. He was caught in flagrante delicto, which under Rule 112
A lawful arrest without a warrant may be made a peace officer or a
is exempt from the requirement of a warrant.
private person under the following circumstances:
The person to be arrested has committed, is actually committing,
DOCTRINE: A lawful arrest without a warrant may be made a peace or is attempting to commit an offense.
officer or a private person under the following circumstances: When an offense has just in fact been committed and the police
The person to be arrested has committed, is actually committing, or officer has personal knowledge of the facts indicating that the
is attempting to commit an offense. person to be arrested has committed it.
When an offense has just in fact been committed and the police Person to be arrested is a prisoner who has escaped…
officer has personal knowledge of the facts indicating that the Malmstedt was searched and arrested while transporting prohibited
person to be arrested has committed it. drugs. A crime was actually being committed by the accused and he
Person to be arrested is a prisoner who has escaped… was caught in flagrante delicto.
Under the circumstances of the case, there was sufficient probable cause
for said officers to believe that the accused was then and there
committing a crime.
FACTS:
Narvasa, concurring and dissenting: Narvasa cites several cases
Mikael Malmstedt, a swedish national, left for Baguio City in the evening
where the decision of the SC on warrantless searches are held
of May 7, 1989. The following day he took a bus to Sagada and
differently depending on the circumstances.
stayed in the place for two days (yess that thing called tadhana ang
In the five cases of Claudio, Tangliben, Posadas, Maspil, and
peg ni kuya)
LoHo Wing, facts existed which were found but he Court as
On May 11th, Captain Alan Vasco, Commanding Officer of the First
justifying warrantless arrests.
Regional Command of NARCOM, ordered his men to set up a
0 In Claudio: arresting officer had secretly ascertained
temporary checkpoint for the purpose of checking all vehicles coming
the woman he was resting was in fact in possession
form the Cordillera Region.
of marijuana, he had personally seen that her bag
The setting up of a checkpoint was because of information that vehicles
contained not only vegetables but also a package
coming from Sagada were transporting marihuana and other
emitting the odor of marijuana.
prohibited drugs. Information was also received that on the same
1 In Tangliben, the person arrested and searched was
morning, a Caucasian coming from Sagada had in his possession
acting suspiciously, and had been positively pointed
prohibited drugs.
to as carrying marijuana.
On the same day, Malmstedt was on his way back to Baguio City. 2 (the above cases, the accused were about to board
The bus he was on board was searched upon reaching the checkpoint.
passenger buses, making it urgent for the police to
Chief investigating officer Galutan asked for the presentation of
take quick and decisive action)
identification/passport but Malmstedt refused. He then noticed a
3 In Posadas, the person arrested and searched was
bulge on Malmstedt waist, which he asked to reveal. It contained
acting suspicious and when accosted had attempted
suspicious looking objects wrapped in brown packaging tape. It was
to flee form
later identified as hashish, a derivative of marijuana.
the police.
In Maspil and LoHo Wing, there was definite information of
the precise identity of the persons engaged in
transporting prohibited drugs at a particular time and
place
In People v Aminnudin, however, the Court took a different
stance. It appeared that the police officers were aware of
Aminnudin’s identity, his projected criminal enterprise and
the vessel on which he would be arriving, and equally as
importantly, had sufficient time and opportunity to obtain a
search warrant. The seized goods were declared as a result
of illegal search, therefore inadmissible.
In the present case, it seems as if the officers conducting the
search, were fishing for evidence. There was in this case no
confidential report from, or positive identification by an informer;
no attempt to flee; no bag or package emitting tell-tale odors; no
other reasonably persuasive indication that Malmstedt was at
the time in the process of perpetrating the office for which he
was subsequently prosecuted. The accused should be absolved
on reasonable doubt.
ROAN v. GONZALES (ELIEL) There must be a specific description of the place to be searched and the
November 25, 1986 | Cruz, J. | Warrantless Searches things

PETITIONER: Josefina Roan


RESPONDENTS: Hon. Romulo Gonzales, RTC

SUMMARY: Gonzales issued a search warrant following the application


of Quillosa. Two days later, Roan’s house was searched and none of
those listed articles were seized; except when they discovered a revolver.
Roan contends that the search warrant is invalid because there were no
depositions and the judge did not follow rule 26 of the Rules of Court
requiring personal examination.

The SC ruled that Gonzales did not conduct a personal examination on


Quillosa. He merely asked him questions and did not attach the oath and
affidavit on the search warrant. Much more is that he based his probable
cause on the testimony of the witness, which tantamounts to hearsay.
Seeing that there were defects in determination of probable cause, the
warrant is invalid. Following that, although the pistol is necessarily illegal,
a valid warrant is still required. Thus, the pistol and bullets were
confiscated illegally.

DOCTRINE: To be valid, a serach warrant must be supported by


probable cause to be determined by the judge or some other authorized
officer after examining the complainant and the witnesses he may
produce.

FACTS:
Quillosa personally filed an application for a search warrant along with
his two witnesses, before Gonzales.
Gonzales examined Quillosa and ascertained if he knew and understood
the application
Roan’s house was searched two days after the issued warrant but none
of the articles listed in the warrant was discovered.
However, the officers conducting the search discovered one Colt
Magnum revolver and eighteen live bullets, which they confiscated.
Hence this appeal.

ISSUE/s:
WoN search and seizure was unreasonable– NO

RULING: Ruling of CFI reversed and set aside.

RATIO:
To be valid, a serach warrant must be supported by probable cause to be
determined by the judge or some other authorized officer after
examining the complainant and the witnesses he may produce.
to be seized, to prevent arbitrary and indiscriminate use of the warrant.
Probable cause was described as such facts and circumstances which would
elad a reasonably discreet and prudent man to believe that an offense has
been committed and that the objects sought in connection with the iffense
are in the place sought to be searched. And it must refer to only one specific
offense.
Roan contends that no depositions were takne by Gonzales in accordance with
Rule 26 requring the personal examination on the oath or affirmation of the
complainant.
In this case, Gonzales did not ask his own searching questions. And limited
himself to the contents of the affidavit. He did not take the applciant’s
deposition in writing and attach them to the record together with the affidavit
presented to him.
It is axiomatic that the examination must be probing and exhaustive, not merely
routinary or pro-forma, if the claimed probable cause is to be established.
A study of the depositions taken from witnesses, shows that they were in the
main a mere restatement of their allegations in their affidavits, except that
they were made in the form of answers to the questions put to them by
Gonzales.
Significantly, the meaningful remark made by one witness is that they were
suspicious of Roan because he was a follower of the opposition candidate in
the forthcoming election. These defects have rendered the search warrant
invalid.
Possession of the revolver, was violated of PD 1866 and considered malum
prohibitum. Hence, illegal articles could be taken even without a warrant.
Prohibited articles may be seized but only as long as the serach is valid. In this
case, it was not because: (1) there was no valid search warrant; and (2)
absent such a warrant, the right thereto was not validly waived by Roan.
In short, the military officers who entered Roan’s premises had no right to be
there and therefore had no right to seize the pistol and bullets.
It is true that there are certain instances when a search may be validly made
without warrant and articles may be taken validly as a result of that search:
incidental to a lawful arrest; (2) inspected at borders; (3) Those that are
mobile and relative ease in fleeing [vessels and aircrafts]; (4) consent; and
visible.
Hence, the rule having been violated and no exception being applicable, the
conclusion is that Roan’s pistol and bullets were confiscated illegally.
PEOPLE v. TABAR (JP) ISSUE/s:
May 17, 1993 | Davide Jr., J. | Warrantless Search and Seizure WoN the warrantless search and seizure was valid - YES.

PETITIONER: People of the Philippines


RESPONDENTS: Carmelina Tabar

SUMMARY: A 17-year old boy named Rommel approached the officers’


informant in a buy-bust operation and handed the latter drug cigarettes.
Upon the arrest of Rommel, and inspection of the shanty where Rommel
allegedly got the cigarettes, a certain Carmelina appeared with rolled up
pants in her hand. Upon being asked, she revealed three pairs of
marijuana cigarettes. The lower court indicted Carmelina.

The SC ruled that Carmelina is indeed guilty because she voluntarily


surrendered to the search and seizure. Upon being asked, she handed the
cigarettes which upon Rules of Court, could be used as proof for her
commission of the crime.

DOCTRINE: Section 12 Rules of Court (instance of a warrantless


search): Search incident to a lawful arrest. A person lawfully arrested
may be searched for dangerous weapons or anything which may be
used as proof of the commission of an offense, without a search warrant.

FACTS:
The officers’ informant in the buy-bust operation for alleged marijuana
pushers in Punta Princesa, Cebu, stood in a shanty to wait for the
suspects.
A young boy of 17 years old, Rommel, Arriesgado approached their
informant and handed cigarettes to him who in turn handed the
marked money to the young boy. The young boy came from the
shanty.
Upon proceeding of the officers to the shanty they met Carmelina Tabar,
accused herein, suspicious looking and pale. She was holding white
pants.
The officers asked her to open the rolled up pants which revealed three
more cigarette packs containing marijuana. Rommel was arrested
first and then Carmelina.
Upon cross-examination, the boy admitted that the marijuana sticks are
from his aunt, the accused.
Rommel being a minor (17) was entitled to a suspended sentence by the
RTC. Carmelina was convicted of the Dangerous Drugs Act of 1972.
RULING: The warrantless search and seizure was valid. RTC ruling is
affirmed.
Carmelina is guilty.

RATIO:
Carmelina voluntarily spread out the rolled pair of pants which was then
carrying hurriedly, upon being asked.
The package containing the packs of marijuana sticks were thus
exposed in plain view to the members of the team.
A crime was thus committed in the presence of the policemen.
Pursuant to the Revised Rules of Court, she could be lawfully
searched because such search is an incident to a lawful arrest.
Section 12 Rules of Court (instance of a warrantless search):
Search incident to a lawful arrest. A person lawfully arrested
may be searched for dangerous weapons or anything which
may be used as proof of the commission of an offense, without
a search warrant.
Even assuming ex gratia argumento that the search and seizure
was without warrant, the appellant had effectively waived her
constitutional right relative thereto by voluntarily submitting to
the search and seizure. She voluntarily opened the roll of pants,
upon being asked.
Note: the constitutional right to be presumed innocent is not infringed by
the reliance of the trial court on the presumption of regularity of the
performance of the official functions on the part of the arresting
officers. The SC has relied on such presumption to determine if the
testimonies of the police who conducted the buy-buy operation
deserve full faith and credit.
POSADAS v. CA (HENRY)
August 2, 1990 | Gancayco, J. | Search Warrant RULING: SC denied the petition and affirmed the CA’s ruling.

PETITIONER: Romeo Posadas RATIO:


RESPONDENTS: CA and the People of the Philippines Sec 5, Rule 113, Rules on Criminal Procedure:
A peace officer or a private person may without a warrant arrest
SUMMARY: The petitioner was arrested for illegal possession of a person:
firearms. The two policemen who inspected his buri bag did not catch him 0 When in his presence, the person t be arrested has
in the act but only suspected that he was up to committing a crime. This committed, is actually committing, or is attempting to
was the plain reason why the petitioner also claims that his arrest was commit an offense
invalid, that it was not one of the rules mentioned in the warrantless 1 When an offense has in fact just been committed,
arrests. and he has personal knowledge of facts indicating
that the person to be arrested has committed it
The SC ruled that the arrest was valid, there being a probable cause as 2 When the person to be arrested is a prisoner who
far as his suspicious acts and bag was concerned. That even if the has escaped from a penal establishment or place
policemen did not catch him in the act of the commission of the crime, where he is serving final judgment or temporarily
probable cause is present when they checked his bag. confined while his case is pending, or has escaped
while being transferred from one confinement to
DOCTRINE: A peace officer or a private person may without a warrant another
arrest a person: (1) When in his presence, the person t be arrested has OSG argues that when two policemen approached the petitioner, he was
committed, is actually committing, or is attempting to commit an offense actually committing or had just committed the offense of illegal
possession of firearms and ammunitions.
The SC disagrees. The policemen did not know that he had committed
FACTS:
such crime, they just suspected that he was hiding something in the
October 1986, 10am, Pat. Ursicio Ungab and Pat. Umbra Umpar, buri bag. Hence, the circumstances did not justify an arrest without a
members of the Integrated National Police (INP) of the Davao warrant.
Metrodiscom assigned with the Intelligence Task Force, were A warrantless search and seizure conducred at military or police
conducting a surveillance along Magallanes, Davao. checkpoints and the search thereat in the case at bar, there is no
At the premises of Rizal Memorial Colleges, they spotted petitioner question that indeed latter is more reasonable considering that unlike
Posadas carrying a “buri” bag and they noticed him to be acting the former, it was effected on the basis of a probable cause.
suspiciously. The probable is that when the petitioner acted suspiciously and
The two pffocers approached petitioner Posadas, and identified attempted to flee, there was a probable cause that he was
themselves as members of the INP, which led for Posadas to flee but concealing something illegal in the bag and it was the right and duty
was eventually caught. of the police officers to do inspection.
The two officers then checked the “buri” bag where they found: It is too much to require the police to only search the bag after obtaining
1 caliber .38 revolver a warrant.
2 rounds of live ammunition for the said revolver
a smoke (tear gas) grenade
2 live ammunitions for a .22 caliber gun
The two offiers brought the petitioner to the police station for
investigation, and was asked to provide for the licences required to
possess firearms but Posadas was not able to show any.
Petitioner Posadas was eventually prosecuted for illegal possession of
firearms and ammunitions in the RTC with a plea of not guilty.
Petitioner unsatisfied with the RTC decision appealed to the CA which
affirmed the trial court’s decision, hence this appeal.

ISSUE/s:
WoN the warrantless arrest was lawful – YES
ANIAG v. COMELEC (DANNAH)
October 7, 1994 | Bellosillo, J. | Search and Seizure: Checkpoints ISSUE/s:
WoN the warrantless search on Aniag’s car was valid – NO
PETITIONER: Congressman Francisco Aniag
RULING: Petition granted. The warrantless search is declared illegal.
RESPONDENTS: COMELEC & DOJ Special Task Force
Firearms seized cannot be used as evidence in any proceeding case.
SUMMARY: House of Representatives Seargent-at-Arms Taccad wrote to
Cong. Aniag requesting the return of 2 firearms issued to him by the HoR, RATIO:
pursuant to the gun ban. Aniag then instructed his driver Arellano to deliver As a rule, a valid search must be authorized by a search warrant duly
said firearms to Congress. However, Arellano was apprehended and the car issued by an appropriate authority.
was searched in a PNP checkpoint which was 20 meters away from the However, this is not absolute. Some instances wherein warrantless
entrance of Batasan Complex. There the PNP found the firearms and searches have been upheld are:
detained Arellano. A search incident to a lawful arrest
Aniag protests against the manner by which the PNP conducted the search. In cases of moving vehicles
He claims that it was done without a warrant and therefore it is illegal. The Seizure of evidence is in plain view
Court explained that there are cases wherein warrantless searches may be Search conducted at police or military checkpoints
legal, including checkpoints. However it ruled that since there was no In case of checkpoints, these are not illegal per se and not violative of the
reasonable or probable cause to search the car, the search was illegal and constitution for as long as the vehicle is neither searched nor its
violative of the right against unlawful searches and seizures. Arellano did not occupants subjected to a body search, and the inspection of the
have the firearms tucked in his waist, nor did the package evince that it
vehicle is merely limited to a visual search.
contained firearms, and there was no unnatural reaction from Arellano when
they flagged the car down. Aniag’s contention that the guns were not tucked in Arellano’s waist nor
placed within his reach, and that they were neatly packed is with
DOCTRINE: An extensive search without warrant could only be resorted to if merit.
the officers conducting the search had reasonable or probable cause to The records do not show that the manner by which the package was
believe before the search that either the motorist was a law offender or that bundled led the PNP to suspect that it contained firearms.
they would find the instrumentality or evidence pertaining to the commission There was also no mention of any report regarding any nervous,
of a crime in the vehicle to be searched. suspicious or unnatural reaction from Arellano when the car was
stopped and searched.
Given these circumstances and relying on its visual observation, the PNP
FACTS: could not thoroughly search the car lawfully as well as the package
On January 1992, pursuant to the “Gun Ban”, Serapio Taccad, Seargent- without violating the constitutional injunction.
at-Arms of the House of Representatives wrote to Cong. Aniag of An extensive search without warrant could only be resorted to if the
Bulacan requesting the return of 2 firearms issued to him by the Hor.
officers conducting the search had reasonable or probable cause
Aniag instructed his driver, Arellano, to pick up the firearms from his Valle
Verde house and return them to Congress. to believe before the search that either the motorist was a law
Meanwhile, the PNP set up a checkpoint outside the Batasan Complex offender or that they would find the instrumentality or evidence
20 meters away from the entrance. pertaining to the commission of a crime in the vehicle to be
About 30 minutes later, the policemen manning the outpost flagged down searched.
Arellano’s car and searched it. They found the firearms neatly The existence of probable cause justifying the warrantless search is
packed in their gun cases and placed in a bag in the trunk of the car. determined by the facts of each case. Thus, the Court upheld the
Arellano was then apprehended and detained. He explained that he was
validity of the warrantless search in situations where the smell of
following Aniag’s orders.
Aniag strongly protests against the manner by which the PNP conducted marijuana emanated from a plastic bag, or where the accused was
the search. He claims that without a warrant and without informing acting suspiciously and attempted to flee.
Arellano of his fundamental rights, the policemen searched the car.
The Court also recognizes the stop-and-search without warrant
conducted by police officers on the basis of prior confidential
information which were
reasonably corroborated by other attendant matters such as: victim of an unlawful search without a warrant
Where a confidential report that a sizable volume of marijuana The facts yield that there was consent
was to be transported along a certain route where the search As held in People v. Excela, consent to a search may be given
was conducted and appellants were caught in flagrante expressly or impliedly
delicto transporting the drugs As early as People v. Malasugui, the settled rule is that a search
Where apart from intelligence information, there were reports by may be validly conducted without a warrant if the person
an undercover “deep penetration” agent thatappelants were searched consented thereto
bringing prohibited drugs into the country and etc… Prefers to sustain exoneration but with justifying circumstance of
In the case at hand, the Court finds that the check-point was set up 20 “acting in obedience to what he innocently believed to be a
meters from Batasan Complex to enforce the Gun Ban. lawful order of a superior”
There was no evidence to show that the policemen were impelled to do
so because of a confidential report.
Nor was there any indication from the package or behavior of Arellano
that could have triggered the suspicion of policemen.
Absent justifying circumstances specifically pointing to the culpability of
Aniag and Arellano, the search could not be valid.
The action then of the policemen unreasonably intruded into petitioner’s
privacy and the security of his poperty, in violation of Section 2 Art III.
The firearms obtained cannot be admitted for any prupose in any proceeding.
It may be argued that the seeming acquiescence of Arellano constitutes
an implied waiver, however, he did not have any choice but to submit
to the PNP’s scrutiny.
Cruz, Concurring:
Searches and seizures are peremptorily pronounced to be
reasonable even without proofof probable cause and much
less the required warrant.
The improbable excuse is that they are aimed at
0 Establishing an effective territorial defense
1 Maintaining peace and order
2 Providing an atmosphere conducive to the social,
economic and political development of the NCR
For these purposes, any indivudla may be stopped and searched at
random and at any time simply because he excites the
suspicion, caprice, hostility or malice of the officers manning the
checkpoints, on pain of arrest or worse, even being shot to
death if he resists
Cruz also hopes that other justices will reconsider the stand of
the Court on checkpoints and finally dismantle them
altogether as an affront to individual liberty.
Regalado, Concurring and Dissenting (OMITTED):
He dissents from the rationale submitted that Arellano was the
PEOPLE v. TUDTUD (IYA) were seized in violation of their right against unreasonable search
September 26, 2003 | Tinga, J. | Warrantless Arrest and seizures. (Walang sinabi na ruling ng CA)

PETITIONER: People of the Philippines


RESPONDENTS: Noel Tudtud

SUMMARY: Davao police received information that a certain Tudtud is in


possession of illegal drugs. Acting on the information, the police
conducted a surveillance in the area of Tudtud. As Tudtud alighted the
bus, the police approached him and introduced themselves as officials.
The police asked him to open the box he was carrying, which he obliged
to do. The police found therein marijuana leaves. He was then accused
and sentenced of the crime of illegal possession. On appeal he contends
the validity of the search and seizure being done without a warrant.

The SC held that the arrest, search and seizure was illegal. The officers
did not have sufficient personal knowledge as to the crime, basing their
actions solely on heresy. Tudtud also did not perform suspicious acts
which could be related to a crime during the arrest.

DOCTRINE: “Reliable information” alone is not sufficient to justify a


warrantless arrest. The rule requires that the accuse perform some over
act that would indicate that he has committed, is actually committing, or
is attempting to commit an offense.

FACTS:
Toril Police Station of Davao City received a report from a civilian asset
named Soldier about a certain Noel Tudtud.
Solier related that his neighbors have been complaining about Tudtud
who was allegedly responsible for the proliferation of marijuana in
their area.
PO1 Ronald Desierto, PO2 Ramil Floreta and their superior, SPO1
Villalonghan, conducted surveillance in soldier’s neighborhood.
Tudtud was not in the area when the surveillance was conducted,
only from the neighbors did they deduce that Tudtud is engaged in
the illegal act.
On the arrival of the bus Tudtud was aboard, the police officers
approached and his companion, Dindo Bulong.
The police introduced themselves and requested Tudtud to open the
boxes they were carrying. Found, wrapped in newspeper, were
marijuana leaves.
Tudtud and Bulong were then charged before the RTC for illegal
possession. The RTC found them guilty.
On appeal, Tudtud and Bolong assign, among other errors, the
admission in evidence of the marijuana leaves, which they claim
ISSUE/s:
WON the search and seizure is valid - NO

RULING: Accused is acquitted because no other evidence other than the


illegal confiscated marijuana can be produced as evidence.

RATIO:
The search in question preceded the arrest. Jurisprudence holds that the
arrest must precede the search; the process cannot be reversed for
warrantless searches.
Nevertheless, a search substantially contemporaneous with an arrest
can precede the arrest if the police have probable cause to make the
arrest at the outset of the search.
“Reliable information” alone is not sufficient to justify a warrantless
arrest. The rule requires that the accuse perform some over act that
would indicate that he has committed, is actually committing, or is
attempting to commit an offense.
There is, however, another set of jurisprudence that deems reliable
information sufficient to justify a search incident to a warrantless
arrest.
Tudtud was not performing any overt act or acting in a suspicious manner
that would hint that a crime has bee, was being, or was about to be
committed.
Also, in no sense can the knowledge of the officers that Tudtud was in
possession of marijuana be described as “personal” having learned
the same only from Solier. Such information is hearsay and not
personal knowledge.
On waiver of rights (NO DISCUSSION ON WHY THEY MENTIONED
THIS): There is an effectivee waiver of rights against unreasonable
searches and seizures if the following requisites are present:
Must appear that the right exist
The person involved had knowledge of the existence of such
right
Said person had ana ctual intention to relinquish the right.
There is no showing that the second and third requisites were existing.
Although Tudtud did not resist and opened the box, there was no
showing of his intent to waive his right.
Seeing that the seizure is invalid, the items cannot be used as evidence
against Tudtud.
VERNONIA SCHOOL DISTRICT v. ACTON (ELIEL) reasonabless of all government searches; and when a warrant is not
June 26, 1995 | Scalia, J. | Warrantless searches required, probable cuase is not invariable required either.
The Court found “special needs” to exist in the public school context.
PETITIONER: Vernonia School District There, the warrant requirement, “would unduly interfere with the
maintenance of the swift and informal disciplinary procedures [that
RESPONDENTS: Acton
are] needed,” and “strict adherence to the requirement that
SUMMARY: Motivated by the arising drug culture, especially lead by searches be based upon probable cause” would undercut “the
student athletes, the District imposed a policy for random drug testing. substantial need of teachers and administrators for freedom to
maintain order in the schools.”
Action, a football athlete was removed from the team because of his
refusal to subject himself to the drug test. Hence this appeal. The first factor to be considered is that nature of the privacy interest
upon which the search here at issue intrudes.
The SC ruled that there is a government interest that is reasonable in Central in our view, to the present case is the fact that the subjects
order to conduct a random drug testing among students, especially of the Policy are (1) children, who (2) have been committed to
student athletes. The SC further explained that (1) given that parents the temporary custody of the State as schoolmaster.
delegated the children under the care and guidance of the school/state, When parents place minor children in private schools for their education,
they may be subject to regulation because of less expectation of privacy. the teachers and administrators of those schools stand in loco
Furthermore, student athletes have less expectation of privacy in parentis over the children entrusted to them.
general, and may be subject to high regulations; and (2) although the T.L.O did not deny, but indeed emphasized, that the nature of that power
method is negligible, the urinalysis results will be kept confidential. is custodial and tutelary, permitting a degree of supervision and
control that could not be exercised over free adults.
DOCTRINE: The most significant element in this case is the first we The reasonableness inquiry cannot disregard the schools’ custodial
discussed: that the Policy was undertaken in furtherance of the and tutelary responsibility for children. For their own good and
government’s repsonsibilities, under a public school system, as guardian that of their classmates, public school children are routinely
and tutor of children entrusted to its care.. required to submit to various physical exainations and to be
vaccinated agasint various diseases.
Particularly with regard to medical examination and procedures,
therefore, students within the school environment have a lesser
FACTS: expectation of privacy than members of the population generally.
Motivated by the discovery that athletes were leaders in the student drug Legitimate privacy expectations are even less with regard to
culture and concern that drug use increases the risk of sports-relate student athletes. School sports are not for the bashful. They
injury, School District (District) adopted the Student Athlete Drug require suting up before each practice or event, and showering
Policy (Policy, which authorizes random urinalysis drug testing of and changing afterwards.
students who particupate in its athletics programs. There is an additional respect in which school athletes have a
Respondent Acton was denied participation in his school’s football reduce expectation of privacy. By choosing to go out for the
program when he and his parents refused to consent to the testing. team, they voluntarily subject themselves to a degree of
Action filed a declaratory relief because they claim that the Policy regulation even higher than that imposed on students generally.
violated their constitutional rights. In Vernonia’s public schools, they must submit to a pre-season physical
The District Court denied the claims, but the CA reversed, holding that exam, they must acquire adequate insurance coverage or sign an
the Policy invade the constitutional rights. Hence this appeal. insurance waiver, maintain a minimum grade point average, and
comply with any rules of conduct, dress, training hours and related
ISSUE/s:
matters as ma be established for each sport by the head coach and
31. WoN students may be subjected to random drug testing –YES athletic director with the principal’s approval.
Students who voluntarily participate in school athletics have reason to
RULING: Remanded for further proceedings. expect intrusions upon normal rights and privileges, including privacy.
The Court noted that the degree of intrusion depends upon the manner in
RATIO: which production of the urine sample is monitored. Under the Policy,
Warrants cannot be issued, of course, without the showing of probable male students produce samples at a urinal along a wall. They remain
cause, required by the Warrant Clause. But a warrant is not required to fully clothed and are only observed from behind if at all. Female students
establish the produce samples
in an enclosed stall, with a female monitor standing outside listening only for
sounds of tampering.
These conditions are nearly identical to those typically encountered
in public restrooms, which men, women, and especially school
children use daily. Under such conditions, the privacy interest
compromised by the process of obtaining the urine are in our
view negligible.
The drugs for which the samples are screened are standard, and do not
vary according to the identity of the student. And finally, the results of
the tests are disclosed only to a limited class of school personnel
who have a need to know; and they are not turned over to law
enforcement authoritites or used for any interenal disciplinary
function.
In both Skinner and Von Raab, we characterized the government
interest motivating the search as compelling. The District Court
held that because the District’s program also called for drug
testing in the absence of individualized suspicion, the District
must demonstrate a compelling need for the program.
Deterring drug use by our Nations’ schoolchildren is at least as important
as enhancing efficient enforcement of the Nation’s laws against the
imporation of drugs, which was the governmental concern in Von
Raab, or dettering drug use by engineers and trainmen, which was
the governmental concern in Skinner.
In the present case, the necessity for the State to act is magnified
by the fact that this evil is being visited not just upon
individuals at large, but upon children for whom it has
undertaken a special responsibility of care and direction.
The most significant element in this case is the first we discussed:
That the Policy was undertaken in furthernace of the
government’s responsibilities, under a public school system, as
guardian and tutor of children entrusted to ist care.
PEOPLE v. COMPACION (JP) A criminal complaint for violation of the Dangerous Drugs Act was filed
June 20, 2001 | Kapunan, J. | Warrantless Search and Seizure against appelland Compacion.
Appellant contends that his constitutional right against unreasonable
PETITIONER: People of the Philippines searches and seizures had been violated with the search.
RESPONDENTS: Armando Compacion
ISSUE/s:
SUMMARY: Upon a tip, the Narcotics Command agents decided to WoN the warrantless search was valid - NO
secure a search warrant for their raid to accused Compacion’s place
where he allegedly was cultivating marijuana plants. The judge of RULING: The search and seizure was unreasonable. The evidence should
Bacolod did not issue the search warrant however because of lack of be excluded.
jurisdiction. The offices of the judge of San Carlos were likewise closed.
RATIO:
Warrantless, the agents searched and seized the house of appellant.
Appellant alleged a violation of his constitutional guarantee. The search and seizure conducte by agents are without a search
warrant. It also does not appear that the situation falls under any of
The SC held that the warrantless search and seizure is invalid. The the exceptions. Appellant’s right against unreasonable search and
appellant’s silence during the raid does not mean that he waived his right seizure was clearly violated.
thereto. There was no intention on his part to waive his right. Also, the While the right to be secure from unreasonable search and seizure like
plain view doctrine cannot be invoked by the agents because they any
intentionally wanted to seize the evidence, not just see it in plain view. other right can be waived, such waiver must constitute a valid waiver.
169. The act of appellant in allowing the members of the military to
DOCTRINE: To constitute a valid waiver: (1) the right exists (2) person enter his premises and his consequent silence during the
has knowledge of the existence of the right (3) said person had actual search could not be construed as voluntary submission to the
intention to relinquish the right. warrantless search, especially so when members of the raiding
team were intimidatingly numerous and heavily armed.
Plain view doctrine elements: (1) prior valid intention (2) evidence was 170. To constitute a valid waiver: (1) the right exists (2) person has
inadvertently discovered who have the right to be there (3) evidence knowledge of the existence of the right (3) said person had actual
must be immediately apparent (4) “plain view” justified were seizure of intention to relinquish the right.
evidence without further search. Neither can the NARCOM agents invoke necessity and urgency of the
situation. The surveillance was conducted 4 days earlier. Surely the
raiding
FACTS:
team had the time to secure a valid search warrat.
Upon a confidential tip that accused Compacion is growing and 172. The plain view doctrine (exception) cannot also be used by the
cultivating marijuana plants, Narcotics Command (NARCOM) agents agrents because they forced their way into accused’s place. The
tried to secure search warrant from the Executive Judge of Bacolod NARCOM conducted the seach with the intention to seize the
City. evidence. They did not just come across the evidence “in plain
The Bacolod judge denied the same however explaining that he had no view.”
territorial jurisdiction on the matter. Plain view doctrine elements: (1) prior valid intention (2) evidence was
The NARCOM agents traveled to the judge of San Carlos City but the inadvertently discovered who have the right to be there (3) evidence
offices were closed since it was night time. Search-warrantless, they must be immediately apparent (4) “plain view” justified were seizure
then went to the place of the accused regardless. of evidence without further search.
Prosecution contends: the appellant opened the gate and permitted them
to come in. Appellant then admitted he planted marijuana for his wife
who is suffering from migraine.
Appellant has a different story: while he (appellant) and his family were
sleeping, he heard somebody knocing outside his house. He then
saw what he though were military men enter his premises and roam
around his house. He was called to the backyard where he was
confronted with the plants wherein he replied that he didn’t know
they were marijuana plants and just planted them for his wife.
ASUNCION v. CA (HENRY) In Vic’s denial of the charges against him, his version of the story was that, in
February 1, 1999 | Martinez, J. | Warrantless Search the evening of the incident he was abducted at gun point and was taken
out
PETITIONER: Jose Maria M. Asuncion
RESPONDENTS: CA and People of the Philippines

SUMMARY: etitioner Asuncion, with an alias Vic Vargas, is charged for


violating the Dangerous Drugs Act. His car was flagged down, where the
authorities found a plastic packet of shabu in his possession. He
consented as well to the search of the vehicle.

Vic is assailing the admissibility of the evidence as the search was made
without warrant. The RTC and the CA ruled against his favor which lead
him to appeal to the SC.

SC ruled that the shabu was admissible, for the reason that: he
consented to the search of his car, and that search of moving vehicles is
one of the exceptions that allow admissibility of evidence found through
warrantless searches.

DOCTRINE: Well-entrenched in this country is the rule that no arrest,


search, and seizure can be made without a valid warrant issue by
competent judicial authority. However, this rule is not absolute. One of the
exceptions provided by law is that of a search in a moving vehicle.

FACTS:
Petitioner Asuncion (aka Binggoy and/or Vic Vargas) is charged with
violation of Sec 16, Art 3 of RA 6425 (Dangerous Drugs Act), for
having in his possession a small plastic packet containing shabu.
Petitioner pleaded not guilty.
Evidence for the prosecution shows that in compliance with the
campaign against drugs by the Malabon Municipal Mayor, the Chief
of the Malabon Police Anti-Narcotics Unit was ordered to watch out
for a certain drug pusher named Vic Vargas.
The confidential informant of the aforementioned team informed them
that a gray Nissan car is always parked in a particular area to sell
shabu.
As the policemen flagged down the car upon sight, they discovered the
driver to be Vic Vargas himself, hence they asked to inspect the
vehicle. Upon searching, they found a plastic packet containing
shabu, which was denied by Vic, as according to him, he only
borrowed the car.
In the police station he was seen to get something from his underwear,
apparently a plastic packet of shabu, to which he finally admitted that
he needed for shooting (FYI, movie actor daw siya yes)
Vic was assailing that his arrest was invalid, as the authoritites did not
secure a search warrant.
for drug testing (although it was only his BP that was checked in
Pagamutang Bayan ng Malabon)
RTC found Vic guilty, to which he appealed to the CA, only to find out
that the appellate court affirmed the RTC’s decision. Hence, this
appeal.

ISSUE/s:
WoN the shabu recovered was inadmissible as evidence – NO

RULING: SC denied the petition filed and affirmed the CA decision.

RATIO:
Well-entrenched in this country is the rule that no arrest, search, and
seizure can be made without a valid warrant issue by competent
judicial authority. However, this rule is not absolute.
Among its exceptions is the search of a moving vehicle, which, in the
case at bar, is present, as the car of Vic even had to be “flagged
down” upon identification. This justifies the warrantless search made
by the police authorities, since the situation demanded immediate
action.
The SC cited people v. Lo Ho Wing, where they ruled that this exception
considers the impossibility of undergoing through the entire process
of securing a warrant from a judge as far as moving vehicles are
concerned, as it is not practical. Vehicles can be quickly moved out
of the locality or jurisdiction in which the warrant must be sought.
In this case, the apprehending officers even sought permission of Vic to
search the car, to which he agreed. As such, since the shabu was
discovered by virtue of a valid warrantless search and the petitioner
himself freely gave his consent to said search, the prohibited drugs
found as a result were admissible evidence.
For fear that the accused will escape and be lost, the operatives had to act
quickly. Since the Nissan car was already identified as that which sells
shabu, upon sight, the authorities do not have enough time to secure a
search warrant.
PEOPLE v. VALDEZ (DANNAH) The police uprooted the plants and took photos of Della Cruz standing
September 25, 2000 | Quisumbing, J. | Search and Seizure: Plain View beside them.
Doctrine

PETITIONER: People of the Philippines


RESPONDENTS: Abe Valdez y Dela Cruz

SUMMARY: Law enforcers received a tip from an unidentified informant that


a certain Dela Cruz had cannabis planted near his nipa hut. They proceeded
to make a reaction team. They caught Dela Cruz and took photos of him
with the marijuana plants. The RTC found that Dela Cruz was guilty beyond
reasonable doubt.

On appeal, however, the Supreme Court reversed this decision. The facts
stated by the prosecution differ; one claimed that they found the plants first
before apprehending Dela Cruz, one said they apprehended Dela Cruz
before finding the plants. It was also stated that the plants were seen in
plain view, but one witness actually said they had to look around the area
before finding the plants. The OSG contends the “plain view” doctrine
should apply, however the SC ruled that the elements for the doctrine to
apply are not present, because it was evident that the discovery of the
cannabis was inadvertent.

DOCTRINE: The seizure of evidence in “plain view” applies only where the
police officer is not searching for evidence against the accused, but
inadvertently comes across an incriminating object.

For the doctrine to apply, the following elements must be present:

A prior valid intrusion based on the valid warrantless arrest in which


the police are legally present in the pursuit of their official duties
The evidence was inadvertently discovered by the police who
have the right to be where they are
The evidence must be immediately apparent
Plain view justified mere seizure of evidence without further search

FACTS:
On September 25, 1996, Dela Cruz was allegedly caught in flagrante
delicto because of unlawfully cultivating and culturing 7 fully grown
marijuana plants or indian hemp.
On November 1996 he was arraigned and he pleaded not guilty.
The first witness for the prosecution, SPO3 Tipay, testified that he
received a tip from an informer about the presence of marijuana
plantation, which were allegedly planted close to Dela Cruz’ hut.
A reaction team was then formed and they found Dela Cruz alone in his nipa
hut, and proceeded to look around the area and saw the marijuana
plants.
As a sole witness for the defense, Dela Cruz claimed that he was The seizure of evidence in “plain view” applies only where the police officer
weeding his vegetable farm when he was called by an udentified
person, who took him to the place of the marijuana plants. They then
took his photos, and brought him to the police station.
On cross-examination, Dela Cruz attested that there were 10 other
houses around the vicinity and one house belonged to one Pascua,
an uncle of the barangay peace officer who had a grudge against
him.
The spot where the plants were found was located between their homes.
The lower court then held Dela Cruz guilty beyond reasonable doubt.
On appeal, Dela Cruz contended that there was unlawful search
because the records show that the law enforcers had ample time to
secure a search warrant.
The OSG however argues that since the marijuana was visibly seen, the
case must be treated as a warrantless lawful search under the “plain
view” doctrine.

ISSUE/s:
WoN the plain view doctrine is applicable at the case at hand – NO

RULING:. Decision reversed and set aside. Dela Cruz acquitted.

RATIO:
In the case at hand, there was no search warrant issued by a judge after
personal determination of the existence of probable cause.
From the declarations of the police officers themselves, it is clear that
they had at least one (1) day to obtain a warrant to search
appellant’s farm.
They could have convinced a judge that there was probable cause to
justify the issuance of a warrant. But they did not.
Instead, they uprooted the plants and apprehended the accused on the
excuse that the trip was a good six hours and inconvenient to them.
We find no reason to subscribe to Solicitor General’s contention that we
apply the “plain view” doctrine. For the doctrine to apply, the following
elements must be present:
A prior valid intrusion based on the valid warrantless arrest in which
the police are legally present in the pursuit of their official duties
The evidence was inadvertently discovered by the police who
have the right to be where they are
The evidence must be immediately apparent
Plain view justified mere seizure of evidence without further
search
In the said case, PO2 Balut testified that they first located the marijuana
pants before Dela Cruz was arrested without a warrant, hence there
was no valid warrantless arrest which preceded the search of Dela
Cruz’ premises.
is not searching for evidence against the accused, but inadvertently comes
across an incriminating object.
Clearly, their discovery of the cannabis plants was not inadvertend.
Also, SPO2 Tipay’s testimony was that upon arriving at the area, they
had to first “look around” before they could spot the illegal plants.
Patently, the seized marijuana plants were not “immediately apparent”
and a “further search” was needed. In sum, the marijuana plants in
question were not in “plain view” or “open to eye and hand.” The
“plain view” doctrine, thus, cannot be made to apply.
PEOPLE v. DE GRACIA (IYA)
July 6, 1994 | Regalado, J. | Warrantless Searcc ISSUE/s:
WON there was a valid search and seizure - YES
PETITIONER: People of the Philippines
RESPONDENTS: De Gracia RULING: (lol no ruling in the casebook, assume nalang ako) SC held the
warrantless search as valid.
SUMMARY: The incidents of the present case took place during a coup
d’etat. De Gracia was arrested during a raid conducted by military RATIO:
officers. During said raid, they found numerous ammunitions and It is admitted that the military operatives who raided the Eurocar Sales
explosives, which was confiscated. Although not put in issue, the SC Office were not armed with a search warrant at the time.
held it would be proper to discuss the validity of the warrantless search. The raid was caused by intelligence reports that the said office was being
used as the HQ of RAM.
It held that due to the circumstances surrounding the case, there being a Another cause for the raid was the surveillance where the military men
coup, a warrantless search is valid to protect the society, government were fired at by a group of men coming from the Eurocar building.
and its duly constituted authorities. Considering the simultaneous and intense firing within the vicinity and in
the nearby Camp Aguinaldo which was under attack by rebel forces,
DOCTRINE: There had been reasonable groups to believe that a crime courts in the surrounding areas were obviously closed.
was being committed. There was more than sufficient probable cause to The instant case falls under one of the exceptions to the prohibition
warrant their action. Under such urgency and exigency of the moment, a against warrantless search.
search warrant could lawfully be dispensed with. There had been reasonable groups to believe that a crime was being
committed. There was more than sufficient probable cause to warrant
their action.
Under such urgency and exigency of the moment, a search warrant
FACTS: could lawfully be dispensed with.
The incidents of the case took place at the height of the coup d’erat by In People v Malmsted the court define probable cause as:
ultra-rightist elements headed by the Reform the Armed Forces “such facts and circumstances which would lead a reasonable,
Movement-Soldiers of the Filipino People (RAM-SFP) discreet, and prudent man to believe that an offense has
In the morning of December 1, 1989, Maj. Soria, together with his team been committed, and that the objects sought in connection
of military men, was conducting a surveillance of the Eurocar Sales with the offense are in the place sought to be searched.”
Office located at EDSA, Quezon City. The case of Umil v Ramos was held applicable by analogy to the present
The surveillance was pursuant to an intelligence report received that said case:
establishment was being occupied by the RAM_SFP as a “the absence of a judicial warrant is no legal impediment to
communication command post. arresting or capturing persons committing overt acts of
A crowd was gathered near the Urocar office watching the on-going violence against government forces. The arrest or capture is
bombardment near Camp Aguinaldo. A group of five men separated impelled by the exigencies of the situation that involves the
from the crowd and was about to approach the vehicle of the very survival of society and its government and duly
surveillance team. constituted authorities.”
Seeing them approach, Maj. Soria directed his team to drive away, the
approaching group drew their guns and fired at them.
On December 5, 1989, a searching team headed by First Lt. Babao
raided the Eurocar Sales office.
They were able to find several ammunitions and explosives, which they
seized. The only person present was De Gracia, who they arrested
together with two janitors of the building.
No search warrant was secured by the raiding team. According to them
they were not able to procure a warrant because at the time there
was so much disorder aside from the fact that the courts were
consequently closed.
BOARD OF EDUCATION v. EARLS, ET AL. (ELIEL) of criminal investigations, respondents do not contend that the
June 27, 2002 | Thomas, J. | Warrantless searches School District requires probable cause before testing students for
drug use.
PETITIONER: Board of Education While schoolchildren do not shed their constitutional rights when they
RESPONDENTS: Earls, et al. enter the schoolhouse, Fourth Amendment righs are different in
public schools than elsewhere; the reasonableness inquiry cannot
SUMMARY: The School District of Tecumseh, Oklahoma set up a policy disregard the school’ custodial and tutelary responsibility for children.
requiring students to undergo a uniralysis (drug) test in order to Central is the fact that the subjects of the Policy are (1) children, who (2)
participate in any extracurricular activity. The students through their have been committed to the temporary custody of the State as
parents filed a compliant that this suspicionless drug test is violative of schoolmates.
their rights. Hence this appeal. The most significant element in this case is the first we discussed: that
the Policy was undertaken in furtherance of the government’s
The SC ruled that the drug testing is a reasonable means to prevent the repsonsibilities, under a public school system, as guardian and tutor
use and to detect drugs, given the nationwide epidemic. (1) is that there of children entrusted to its care.
is less expectation of privacy from students, because they submit A student’s privacy interest is limited in a public school environment
themselves under the protection of the state and that they already where the State is responsible for mainitaing discipline, health, and
undergo certain examinations. (2) is that the means of taking the samples safety. Schoolchildren are routinely required ot submit to physical
provides a less problematic method in colleting to prevent the tampering examinations and vaccinations against disease. Securing order in the
of speciments; and that the results are confidential. And that (3) there is school environment sometimes requires that students be subjected
an immediacy in addressing the governmental concern of preventing the to greater controls than those appropriate for adults.
use of drugs by schoolchildren. In any event, students who participate in competitive extracurricular
activities voluntarily subject themselves to many of the same
DOCTRINE: The most significant element in this case is the first we intrusions on their privacy as do athletes. Some of these clubs and
discussed: that the Policy was undertaken in furtherance of the activities require occasional of-campus travel and communal
government’s repsonsibilities, under a public school system, as guardian undress.
and tutor of children entrusted to its care.. This regulation of extracurricular activities further diminishes the
expectation of privacy among schoolchildren. We therefore
conclude that the students affected by this Policy have a limited
expectation of privacy.
FACTS:
Urination is an excretory function traditionally shielded by great privacy.
The School District in Oklahoma requires all middle and high school Bu thte degree of intrusion on one’s privacy caused by collecting a
students to consent to urinalysis testing for drugs in order to urine sample depends upon the manner in which production of the
particpate in any extracurricular activity. urine sample is monitored.
It concluded that before imposing a suspicionless drug-testing program a A faculty monitor waits outside the closed restroom stall for the student to
school must demonstrate some identifiable drug abuse problem produce a sample and must listen for the normal sounds of urination
among a sufficient number of those tested, such that testing that in order to guard against tampered specimens and to insure an
group will actually redress its drug problem. accurate chain of custody. The monitor then pours the sample into
The court then held that the School Distrct had failed to demonstrate two bottles that are sealed and placed into a mailing pouch along
such a problem among Tecumseh students participating in with a consent form signed by the student.
competitive extracurricular activites. Given that we considered the method of collection in Vernonia a
Hence this appeal. “negligible” intrusion, the method here is even less problematic.
In addition, the Policy clearly requires that the test results be kept in
ISSUE/s:
confidential files separate from a student’s other education records
WoN students may be conducted to suspicionless drug testing –YES and released to school personnel only on a “need to know” basis.
Given the minimally intrusive nature of the sample collection and
RULING: Policy for drug testing is considered constitutional the limited uses to which the test results are put, we conclude
that the invasion of students’ privacy is not significant.
RATIO:
Given that the School District’s Policy is not in any way related to the conduct
Finally, this Court must consider the nature and immediacy of the
govenrment’s concerns and the efficacy of the Policy in meeting them.
This Court has already articulated in detail the importance of the
governmental concern in preventing drug use by schoolchildren. The
drug abuse problem among our Nation’s youth has hardly abate since
Vernonia was decided in 1995.
Additionally, the School District in this case has presented specific
evidence of drug use at Tecumseh schools. Teachers testified
that they had seen students who appeared to be under the
influence of drugs and that they had heard students speaking
openly about using drugs.
Given the nationwide epidemic of durg use, and the evidence of
increase drug use in Tecumseh schools, it was entirely
reasonable for the School District to enact this particular drug
testing policy.
Finally, we find that testing students who participate in
extracurricular activities is a reasonably effective means of
addressing the School District’s legitimate concerns in
preventing, and detecting drug use.
SJS v. DDB (JP) WoN mandatory drug testing is legal for the following:
November 3, 2008 | Velasco, Jr., J. | Warrantless Search and
Seizures

PETITIONER: Aquilino Pimentel, Jr., Social Justice Society, Atty.


Manuel Laserna, Jr.
RESPONDENTS: Dangerous Drugs Board

SUMMARY: Three petitions emerged assailing the constitutionality of RA


9165 of otherwise known as the Comprehensive Drugs Act insofar as it
required the mandatory drug testing of different persons.

On candidates for public office, the SC held that it was unconstitutional


because it added qualifications for senators beyond the mandate of the
constitution. On students of secondary and tertiary schools, the SC held
that it was constitutional because students have a lower expectation of
privacy. On employees of public and private offices, valid because of
compelling state interest. On persons charged, the SC held that the
mandatory drug testing is unconstitutional as the law’s mandate of
randomness and suspicionless is not present for the accused.

DOCTRINE: As the warrantless clause of Sec. 2, Art 3 of the Constitution


is couched and as has been held, reasonabless is the touchstone of the
validity of a government search or intrusion.

FACTS:
These three petitioners question the constitutionality of Sec. 36 of RA
9165 or otherwise known as the Comprehensive Dangerous Drugs
Act of 2002 insofar as it require mandatory drug testing of:
Candidates for public office
Students of secondary and tertiary schools
Officers and employees of public and private offices
Persons charged before the prosecutor’s office
Pimentel petition: Pimentel, a senator, seeks to nullify Sec. 30 of RA
9165 as well as the COMELEC Resolution No. 6486 which
implemented it in the sense that they impose an addition qualification
for senators in addition to those enumerated in the constitution
SJS (Social Justice Society) Petition: seeks to prohibit the Dangerous
Drugs Board from enforcing RA 9165 in the sense that they
constitute undlue delegation of legislative epower, entrench on the
equal protection clause and breach a person’s right against
unreasonable searches and seizures.
Laserna petition: petition for certiorari on RA 9165 for infringing on the
constitutional right to privacy, right against unreasonable searches
and seizures and the right against self incrimination, and for being
contrary to the due process and equal protection of the laws.

ISSUE/s:
Candidates for public office - NO
Students of secondary and tertiary schools - YES
Officers and employees of public and private offices - YES
Persons charged before the prosecutor’s office - NO

RULING: Pimentel petition granted. SJS petition denied. Laserna petition


partially granted.

RATIO:
Pimentel petition:
Sec. 36(g) of RA 9165 should be declared as unconsituttional.
Sec. 36 effectively enrlages the qualification requirements
enumerated in the Sec. 3, Art. 6 of the Constitution.
Said Sec. 36 unmistakable requires a candidate for senator to be
certified illegal-drug clean, obviously as a precondition to the
validity of a certificate of candidacy.
The right of a citizen in the democratic process of election should
not be defeated by unwarranted impositions of requirement
not specified in the constitution.
SJS petition:
The provisions are valid and constitutional.
The objective of the law is to stamp out illegal drug and
safeguard in the process the well being of the citizenry,
particularly the youth who is most susceptible, from the
harmful effects of dangerous drugs.
Based on the US cases of Veronic and Board of Education,
mandatory drug testing on schools is valid because:
0 Schools and their administrators stand in loco
parentis to their students
1 Minor students have fewer rights than an adult
2 Schools have aduty to safueguard the health and
well-being of their students
3 Schools have the right to impose conditions on
applications for admission
Likewise, the provisions on manadatory drug testing for private
and public offices are justifiable because:
0 Reduced expectation of privacy on the part of the
employees (agreements with their employers)
1 Compelling state interest (health and well-being)
2 Well-defined limits set forth in the law to guide
auhtorities
Petitioner SJS assail that such subjecting to mandatory drug test
is a violation of the right to privacy and unreasonable
searches and seizures but fails to substantiate their claim.
Laserna Petition:
The court finds no valid justification for mandatory drug testing
for
pesons accused of crimes.
The operative cocnepts in the mandatory drug testing are
randomness and suspicionless. In the case of persons
charged with a crime before the prosecutor’s office, a
mandatory drug testing can never be random and
suspicionless.
They are singled out and are impleaded against their will.
To mpose a mandatory drug testing on them is a blatant attemplt
to harness a medical test as a tool for criminal prosecution
contrary to the stated objectives of RA 9165.
Drug testing in this casw would violate a person’s right to privacy.
Worse, the accused are forced to incriminate themselves.
PEOPLE v. CANTON (HENRY) examination where she was found to have packages of drugs hidden in
December 27, 2002 | Davide, Jr., C.J. | Warrantless arrest her genital area. After examination, the drug was found to be shabu.
The defense claims that, no investigation was made on susan, ever.
PETITIONER: People of the Philippines
RESPONDENTS: Susan Canton

SUMMARY: Petitioner Susan Canton was departing the country (extra:


she’s going to Saigon, Vietnam). Upon entry, the metal detector beeped,
which lead to the attending employee to conduct a search on Susan.
There they found a bulge which turned out to be shabu. She was taken to
the ladies’ room as well for further search and more packages were
found, some in her genital area. RTC found her guilty of violating the
Dangerous Drugs Act, to which she appealed to the SC, claiming that the
arrest was not valid due to it being warrantless, and that the search was
invalid as well since it was not incidental to an arrest.

SC ruled that, despite the authorities not having warrants with them, they
are duty bound to arrest since Susan is caught red handed. Her
possession of the drugs and her outright denial of it when she was asked
is already enough.

DOCTRINE: Police authorities are duty bound to arrest a person if they


have knowledge that he is actually committing a crime.

FACTS:
Petitioner Susan was charged before the RTC with the violation of sec 16
of the Dangerous Drugs Act, for possessing methamphetamine
hydrochloride, a regulated drug without the corresponding
prescription or license.
Susan pleaded not guilty upon arraignment.
At the trial, witnesses Julieta Flores (Forensic Chemist), Mylene
Cabunoc (lady frisker), and SPO4 Victorio de los Reyes (supervisor
of Mylene).
Evidence for prosecution established that on Feb 1998, Susan was at
NAIA, being a departing passenger bound for Saigon, Vietnam. The
metal detector booth beeped after she passed, which led for
Cabunoc, an employee of the National Action Committee on
Hijacking and Terrorism (NACHT) and the frisker on duty at that time
called her attention and asked if she could be searched.
When she was being searched, Mylene felt something bulging at her
abdominal area. Mylene inserted her hand under Susan’s skirt and
pinched the package several times and noticed that the package
contained what felt like rice granules. When Mylene passed her
hand, she felt similar packages in front of Susan’s genital area and
thighs.
Upon asking her to bring it out, Susan said it was only her money.
De los Reyes (Mylene’s supervisor) instructed her to call Customs Examiner
Lorna Jalac to bring Susan to a comfort room for a thorough physical
RTC found Susan guilty for violating the Dangerous Drugs Act. Her MR
was filed, but was likewise denied. Hence, this appeal.

ISSUE/s:
WoN Susan’s conviction is valid – YES

RULING: SC affirmed the RTC decision.

RATIO:
SC does not agree that the warrantless search and subsequent seizure
of the regulated drugs were violative of her constitutional rights.
What constitutes unreasonable search varies in each case. The
recognized exceptions established in case law are:
Search of moving vehicles
Seizure in plain view
Customs searches
Waiver or consented searches
Stop and frisk situations (aka Terry search)
Search incidental to a lawful arrest
SC ruled that, the search and seizure was not incidental to a lawful
arrest. Susan’s arrest did not precede the search. When the metal
detector alarmed, the frisker patted her down, and in the process the
bulge was discovered. While Susan was deprived of her will and
liberty it did not amount to an arrest. An arrest according to the
Revised Rules of Criminal Procedure is te taking of a person into
custody x x x.
The search made pursuant to airport security procedure is not that of the
Terry search, which is only limited to weapons, as it also is a search
for prohibited materials or substances. Nevertheless, the search is
justified.
The SC ruled as well that, Susan, having been caught flagrante delicto,
was lawfully arrested without a warrant. The search conducted on
Susan resulted in the discovery of the packages containing shabu.
Armed with the knowledge that she was committing a crime, the
airport security personnel and police authorities were duty bound to
arrest her.
DISINI v. SECRETARY OF JUSTICE (DANNAH) For this reason, government has a legitimate right to regulate the use of
February 18, 2014 | Abad, J. | Search and Seizure cyberspace and contain and punish wrongdoings, hence, the
Cybercrime
PETITIONER: Jose Jesus M. Disini, et al. Prevention Act.
But, petitioners Disini et al claim that the means adopted by the
RESPONDENTS: The Secretary of Justice, et al.
cybercrime
SUMMARY: Cybercrime law is the government’s way to regulate the access
and use of the cyberspacec, which prohibits acts which may be abused by
ill-minded people with the agenda of destroying or destructing or violating
other people’s rights. Petitioners in this case assail Sections 4(b)(3), 12, 14,
15, 19 as a violation of the Constitutional guarantee against searches and
seizures.

The Court ruled that Section 4b3 that penalizes identity theft is constitutional
for failure of Petitioners Disini et al. to show how this provision violates
citizen’s rights. Section 12 that authorizes collection or recording of traffic
data in real-time is unconstitutional because the authority it gives to law
enforcers is too sweeping and lacks restraint.
Section 14 that authorizes disclosure of computer data under a court-issued
warrant is constitutional because it follows the same process as subpoena,
and it envisions the enforcement of a duly-issued court warrant.
Section 15 that authorizes the search, seizure and examination of computer
data under a court-issued warrant is constitutional because it merely
supplements existing laws on searches and seizures.
Section 19 that authorizes the DOJ to restrict or block access to suspected
Computer Data is unconstitutional because the DOJ order cannot substitute
a court warrant.

DOCTRINE: The constitutional guarantee against searches and seizure will


be upheld against laws which lack proper safeguards, and which set aside
the proper rules and processes for Section 3 Article II of the Constitution.

FACTS:
Cybercrime law aims to regulate access to and use of the cyberspace.
Using his laptop or computer, a person can connect to the internet, a
system that links him to other computers and enables him to do a lot
of things.
This is cyberspace, a system that accommodates millions and billions of
simultaneous and ongoing individual accesses to and uses of the
internet. The cyberspace is a boon to the need of the current
generation for greater information and facility of communication. But
all Is not well with the system since it oculd not filter out a number of
persons of ill will to who would want to use cyberspace technology
for mischiefs and crimes.
law for regulating undersirable cyberspace violate their constitutional expectation has been violated by unreasonable
right against searches and seizures. government intrustion.
Petitioners challenge the constitutionality of the following provisions of
the cybercrime law that regard certain acts as crimes and impose
penalties for their commission as well as provisions that would
enable the government to track down and penalize violators:
Provisions on: Illegal Access, Data Interference, Cyber-
squatting, Identity Theft, Cybersex, Child Pornography,
Unsolicited Commercial Communications, Libel, Aiding or
abetting and attempt in the commission of cybercrimes,
penalty of one degree higher, Prosecution of both the RPC
and RA 10175, Penalties, Real-time Collection of Traffic
Data, preservation of Computer Data, Disclosure of
Computer Data, Search Seizure and Examinataion of
Computer Data, Destruction of Computer Data,
Restricting/Blocking Access to Computer Data, Obstruction
of Justice, Cybercrime Investigation and Coordinating Center
(CICC), and CICC’s powers and functions.
Hence, this petition.

ISSUE/s:
WoN Section 4(b)(3) is Constitutional – YES
WoN Section 12 is Constitutional – NO
WoN Section 14 is Constitutional – YES
WoN Section 15 is Constitutional – YES
WoN Section 19 is Constitutional – NO

RULING:. 4(b)(3), 14 and 15 are constitutional. 12 and 19 are struck down


for being unconstitutional.

RATIO:
On Section 4(b)(3) of the Cybercrime Law, which provides that
Computer-related Identity Theft, which is the intentional acquisition,
misuse, transfer, possession, alteration or deletion of identifying
information belonging to another, whether natural or juridical without
right…
Petitioners Disini et al. claim that this section violates the right to
privacy.
The right to privacy, or right to be left alone, is included in the
Constitutional guarantees:
0 The right against unreasonable searches
1 The right to privacy of communication and
correspondence
A court must determine whether a person has exhibited a
reasonable expectation of privacy and, if so, whether that
The usual identifying information regarding a person includes his from day to day. It is only real-time traffic
name, citizenship, residence address, contact number, data collection or recording
place and date of birth, name of his spouse if any,
occupation and etc
The law punishes those who acquire or use such information
without right, implicitly to cause damage.
Petitioners Disini et al. simply fail to show how this government
effort to curb computer-related identity theft violates the right
to privacy and correspondence….
On Section 12, which provides that Real-Time Collection of Traffic Data—
Law enforcement authorities, with due cause, shall be authorized to
collect or record by technical or electronic means traffic data in real-time
associated with specified communications transmitted by means of a
computer system
Traffic data refer only to the communication’s origin, destination,
route, time, date, size… but not content, nor identities.
All other data to be collected or seized or disclosed will require a
court warrant. Service providers are required to cooperate
and assist law enforcement authorities in the collection or
recording of the above-stated information.
The court warrant required shall only be issued or granted upon
written application and examination under oath or affirmation
of the applicant and the witnesses he may produce and the
showing
0 That there are reasonable grounds to believe that
any of the crimes enumerated has been committed,
or is being committed, or is about to be committed
1 That there are reasonable grounds to believe that
evidence that will be obtained is essential to the
conviction of any person for, or to the solution of, or
to the prevention of, any such crimes
2 That there are no other means readily available for
obtaining such evidence.
Disini et al. assail the grant to collect or record traffic data
provides opporunities for abuse.
They invoke the right of every individual to privacy and to be
protected from the government snooping into the messages
or information that they send to one another
The compelling state interest is to put order to the tremendous
activities in cyberspace for public good.
Those who commit cybercrimes could easily evade detection and
prosecution by moving the physical location of their gadgets
and a subsequent recourse to court-issued search and seizure warrant that can
succeed in ferreting them out.
Disini et al. claim that the provisions of Section 12 are too broad and do not provide
safeguards against crossing legal boundaries and invading people’s right to
privacy.
However, that right is not unqualified. In Whalen v. Roe, the US SC classified privacy
into
Decisional Privacy: right to independence in making certain important
decisions
Informational Privacy: the interest in avoiding disclosurre of personal
matters
They assail that it is informational privacy they want to protect. Its two aspects are:
The right to have private information disclosed
The right to live freely without surveillance and intrusion.
In determining whether or not a matter is entitled to the right to privacy, the Court laid
down a two-fold test:
Subjective test: One is claiming the right must have an actual or legitimate
expectation of privacy over a certain matter
Objective test: His or her expectation of privacy must be one society is
prepared to accept as objectively reasonable.
Information and Communications Technology (ICT) users know that they cannot
communicate or exchange data with one another over cyberspace except
through some service providers to whom they must submit certain traffic data.
Section 12 empowers law enforcement authorities “with due cause” to collect or
record data in real-time. The authority that this Section gives is too sweeping and
lacks restraint.
The OSG points out that Section 12 needs to authorize collection of traffic data “in
real time” because it is not possible to get a court warrant that would authorize
the search of what is akin to a “moving vehicle”.
But warrantless search is associated with a police officer’s determination of probable
cause that a crime has been committed, that there is no opportunity for getting a
warrant, and that unless the search is immediately carried out, the thing to be
searched stands to be removed.
These preconditions are not provided in Section 12. The grant of power to track
cyberspace communications in real time and
determine their sources and destinations must be narrowly without the necessity of a warrant
drawn to preclude abuses. Carpio, Concurring and Dissenting:
On Section 14 which provides that Law enforcement authorities, upon Section 12 is unconstitutional because collection in bulk of
securing a warrant, shall issue an order requiring any person…. To electronic data rises to the level of search and seizure within
disclose or submit subscriber’s information… the meaning of Search and Seizure Clause.
Same process as subpoena. It envisions the enforcement of a Brion, Concurring and Dissenting:
duly issued court warrant. No reasonable expectiation of privacy exists in traffic data.
The prescribed procedure for disclosure would not constitute an Proposes that special cybercrime courts be designated because
unlawful search or seizure. the procedure for search warrants might not be responsive
On Section 15 which provides that where a search and seizure warrant enough to effectively track down cybercriminals and obtain
is properly issued, authorities shall also have the powers and duties evidence of their crimes.
to conduct interception to secure computers, retain copy of data, etc. Leonen, Concurring and Dissenting:
Disini et al. challenge this provision on the assumption that it will Agree with unconstitutionality of Section 12 because these
supplant established search and seizure procedures. present unwarranted chilling effects on the guaranteed and
On its face, however, it merely enumerates the duties of law fundamental rights of expression.
enforcment authorites that would ensure proper collection, Mentioned instances of valid warrantless searches.
preservation... of computers and data that have been seized “But the internet has created other dangers to privacy which may
by virtue of a court warrant. not be present in the usual physical spaces that have been
This merely supplements existing search and seizure rules, not the subject of searches and seizures in the past. Commercial
supplant them. owners of servers and information technologies as well as
On Section 19, which empowers the DOJ to restrict or block access to some governments have collected data without the
computer data, when it is prima facie found to be in violation of the knowledge of the users of the internet. It may be that our
provisions of this Act Data Privacy Law may be sufficient”
Disini et al. contest that this violates the right against searches
and seizure. Even the OSG concedes that this provision may
be unconstitutional.
The Government, in effect, seizes and places computer data
under its control and disposition without a warrant. The DOJ
order cannot substitute for a judicial search warrant. This
provision must be struck down.
Serreno, Concurring and Dissenting:
The Court should not have struck down Section 12 completely.
Rather, it should impose a strict interpretation of Section 12
in the light of existing constitutional, jurisprudential and
statutory guarantees and safeguards.
Traffic data per se do not enjoy privacy protection, hence no
determination of probable cause is needed for the real-time
collection thereof.
The collection of these data do not constitute a search in the
constitutional sense. As such, the collection thereof may be
done
MATERIAL DISTRIBUTORS v. NATIVIDAD (IYA) In the supplemental motion it is alleged that there direct conflict between he
1949 | Perfecto, J. | Searches ad seizures “of whatever nature and for allegations of the complaint and amended complaint and those of the
whatever purpose” answer
PETITIONER: Material Distributors
RESPONDENTS: Natividad

SUMMARY: Material Distributors is contesting the orders issued by


Natividad (I AM ASSUMING HE IS THE ISSUING JUDGE), which
allowed for the production and inspection of books and documents by
said corporation. Material Distributors contend that the orders violate
their constitutional right against self incrimination and guarantee of
privacy of communication and correspondence.

The SC ruled that there has been no showing that production of the
documents would lead to self-incrimination and that courts, upon lawful
order, may request the disclosure of communications and
correspondence.

DOCTRINE: The constitutional guarantee of privacy of communication


and correspondence will not be violated, because the trial court has the
power and jurisdiction to issue the order for the production and
inspection of the books and documents in question in virtue of the
constitutional guarantee making an express exception in favor too the
disclosure communication and correspondence upon lawful order of a
court of justice.

FACTS:
The production and inspection of documents and books here in question
call for the interpretation and application of Section 1 of Rule 21:
Upon motion of any party showing good case therefor and upon
notice to all other parties:
0 Produce and permit the inspection and copying or
photographing of any documents, papers , books…
which constitute or contain evidence material to any
matter involved which are in his possession,
custody, or control
1 Permit entry upon designated land or other property..
for the purpose of inspecting… the property…
The order shall specify the time, place and manner of making the
inspection and taking copies and photographs , and may
prescribe such terms and conditions as are just.
Material Distributors contend that Sarreal has failed to show good cause
for the issuance of the requested order.
In the original motion it appears that the books and papers therein
mention “constitute or contain the evidence material tot he matters
involved in the case.”
and amended answer as to whether or not the names of Gil Puyat
and Raymond Lehmann appear in any part of the originals of
Annexes A and B of the complaint.
Sarreal wanted the production and inspection of said originals to show that
they did not contain the names of Puyat and Lehmann. And that if said
names appear int he Annexes it had been typed by direction of Harry
Lyons without the knowledge or consent of Sarreal and the changes
introduced are a forgery.
With these above allegations (Fact 5&6) in the original and supplemental
motions Sarreal has fulfilled the requirement of showing good cause
for the
production and inspection of the books and documents in question
under Rule 21.
ISSUE/s:
WON the orders issued by the court violate the constitutional right to
privacy of communication and correspondence - NO
RULING:

RATIO:
Material Distributors contend that the order of the trial judge violated their
constitutional right against self incrimination.
To answer the contention, the SC held that there is no showing that the
inspection of annexes A and B would lead to incrimination.
Second contention is that the orders trampled upon petitioners right to he
inviolability of the correspondence and communication as
guaranteed by the Constitution.
The SC held that the provisions of Rule 21 pertain to a civil procedure
that cannot be identified or confused with the unreasonable searches
prohibited by the Constitution.
Considering the erroneous hypothesis that the production and inspection
of the books and documents in question is tantamount to a search
warrant, the procedure in Rule 21 and followed by the judge place
them outside the realm of prohibited unreasonable searches.
The constitutional guarantee of privacy of communication and
correspondence will not be violated, because the trial court has the
power and jurisdiction to issue the order for the production and
inspection of the books and documents in question in virtue of the
constitutional guarantee making an express exception in favor of the
disclosure communication and correspondence upon lawful order of
a court of justice.
CAMARA v. MUNICIPAL COURT (ELIEL) presentation of prper credentials, have the right to enter, at
June 5, 1967 | White, J. | Searches and seizures of”whatever nature of reasonable times, any building, structure, or premises in the City
purpose” to perform any duty imposed upon

PETITIONER: Camara
RESPONDENTS: Municipal Court of San Francisco

SUMMARY: A housing inspector of the Dept. of Public health conducted


a routinary inspection of the building, wherein Camara was living. The
inspecter wanted to search the premises, because Camara’s residential
area is not allowed in the said premises. Camara refused to allow the
search by the inspector because of lack of warrant. After a few more
refusals, a complaint by the inspector was charged against for violation of
a lawful inspection. Camara contends that such search is unconstitutional
because it allows warrantless search of a private property without
probable cause. Hence this appeal.

The SC ruled that a warrantless search by the housing inspector is


unconstitutional. Because (1) it leaves the discretion to the inspector to
conduct the search without knowing whether there is proper authority, or
the lawful limist of the inspector’s power to search; and (2) that although
public safety may warrant it, it was not urged that the inspection
programs could not achieve their goal within the confines of a reasonable
search warrant requirement.

DOCTRINE: The SC says that the protections provided by the warrant


procedure are not needed in the context; broad statutory safeguards are
no substitute for individualized review, particularly when those safeguards
may only be invoked at the risk of a criminal penalty.

FACTS:
An inspector of the Division of Housing Inspection of the San Francisco
Department of Public Health entered an apartment building to make
a routine annual inspection for possible violations of the city’s
Housing Code.
Claiming that the building’s occupany permit did not allow residential use
of the ground floor, the inspector confronted Camara and demanded
that he prmit an inspection of the premises. But Camara refused
because of lack of warrant.
The inspector returned and again without a warrant, and Camara again
refused to allow an inspection. Then a mail was sent ordering him to
appear to the district attorney’s office.
When he failed to appear, they informed Camara that he was required by
law to permit an inspection under 503 of the Housing Code.
Sec 503 RIGHT TO ENTER BUILDING. Authorized employees of
the City departments or City agencis, so far as may be
necessary for the performance of their duties, shall, upon
them by the Municipal Code. The final justification suggested for warrantless administrative searches
Camara nevertheless refused to give inspectors access to his apartment is
without a search warrant.
ISSUE/s:
WoN housing inspector may search without warrant – NO

RULING: Judgment vacated. Remanded for further proceedings.

RATIO:
The basic purpose of the Fourth Amendment (Sec 2) is to safeguard the
privacy and security of individuals against arbitrary invasions by
governmental officials.
One governing principle, justified by history and by current experience,
has consistently been followed: except in certain carefully defined
classes of cases, a search of private property without proper consent
is “unreasonable” unless it has been authorized by a valid search
warrant.
To the Frank majority, municipal fire, health, and housing inspection
programs “touch at most upon the periphery of the important
interests safeguarded by the Fourteenth Amendment’s protection
against official intrusions,” because the inspections are merely to
determine whether physical conditions exist which do not comply
with minimum standards prescribed in local regulatory ordinances.
The Frank majority suggested two other justifications for permitting
administrative health and safety inspections without a warrant. First,
it is argued that these inspections are “designed to make the least
possible deamdn on the individual occupant.”
The ordinances authorizing inspections are hedge with safeguards, and
at any rate the inspector’s particular decision to enter must comply
with the constitutional standard of reasonableness even if he may
enter without a warrant. The decision to inspect an entire municipal
area is based upon legislative or administrative assessment of broad
factors such as the area’s age and condtion.
Under the present system, when the inspecteor demands entry, the
occupant has no way of knowing whether enforcement of the
municipal code involved requires inspection of his premises, no way
of knowing the lawful limits of the isnpector’s power to search, and
no way of knowing whether the inspector himself is acting under
proper authorization.
The practical effect of this system is to leave the occupant subject
to the discretion of the official in the field. This is precisely the
discretion to invade private property hich we have consistently
circumscribed by a requirement that a disinterested party
warrant the need to search.
The SC says that the protections provided by the warrant procedure
are not needed in the context; broad statutory safeguards are
no substitute for individualized review, particularly when those
safeguards may only be invoked at the risk of a criminal penalty.
that the public interest demands such a rule.
It is vigorously argued that the health and safety of entire urban
population is dependent upon enforcement of minimum fire, housing,
and sanitation standards, and that the only effective means of
enforcing such codes is by routine systematized inspection of all
physical structure.
It has nowhere been urged that fire, health, and housing code
inspection programs could not achieve their goals within the
confines of a reasonable search warrant requirement.
In summary, we hold that administrative searches of the kind at
issue here are significant intrusiosn upon the interests
protected by the Fourth Amendment, that searches when
authorized and conducted without a warrant procedure lack the
traditional safeguards which the Fourth Amendment guarantees
to the individual, and that the reasons put forth in Frank v.
Maryland and in other cases for upholding these warrantless
searches are insufficient to justify so substantial a weakening
of the Fourht Amendment’s protections.
UMIL v. RAMOS (JP) Petitioners herein assail the decision for disregarding the constitutional
October 3, 1991 | Per Curiam | Warrantless Arrest guarantee of the accused against a warrantless arrest and seeks to
abandon the Garcia v. Enrile doctrine.
Full case title: In The Matter Of The Petition For Habeas Corpus Of
Roberto Umil, Rolando Dural And Renato Villanueva, Manolita O. Umil
And Nicanor P. Dural, Felicitas V. Sese

PETITIONER: Roberto Umil et al.


RESPONDENTS: Fidel Ramos et al.

SUMMARY: Ronaldo Dural shot two policemen as part of his mission as


a sparrow agent of the NPA. Petitioners are filing for an MR for the
decision of the court which declared the warrantless arrest of Dural as
valid because it was a continuing offense.

The SC denied the MR explaining that because of the ideological content


of membership in the NPA, Dural did not cease to be less subversive
even upon hs confinement in St. Agnes Hospital. He can still be
subjected to arrest because of the continuing nature of subversion and
rebellion.

DOCTRINE: Given the ideological content of membership in the


CCP/NPA which includes armed struggle for the overthrow of organized
government, one does not cease to be, or become less of a subversive,
for purposes of arrest even under confinement.

The absence of a judicial warrant is of no legal impediment to arresting or


capturing persons committing overt acts of violence against government
forces because such arrest is justified by the exigencies of the situation.

FACTS:
Before the court are separate motions filed by the petitioners (Umali et
al) seeking reconsideration of the Court’s decision promulgated in
1990.
The previous court decision in 1990 said that the arrest of Rolando Dural
without a warrant is justified as it can be aid that within the
contemplation of Sec. 5a Rule 113, Dural was committing an offense
when arrested, for being a member of the New People’s Army and
for subversion.
Dural had shot two policemen in their patrol car as part of his mission as
a sparrow (NPA member).
The court in the previous decision used the doctrine of Garcia v. Enrile:
the crimes of insurrection, rebellion, subversion, etc. under
Presidential Proclamation No. 2045 are all in the nature of continuing
offenses, involving a massive conspiracy of nationwide magnitude.
ISSUE/s:
WoN Dural’s warrantless arrest was valid because it constituted a
continuing offense - YES

RULING: The motion for reconsideration is of no merit.

RATIO:
Given the ideological content of membership in the CCP/NPA which
includes armed struggle for the overthrow of organized
government, Dural did not cease to be, or became less of a
subversive, for purposes of arrest, even if he was, at the time of
arrest, confined in the St. Agnes Hospital.
Dural given another opportunity would’e have shot other policemen
anywhere as agents of the organized government.
It is in this sense that subversion like rebellion (or insurrection) is
perceived as a continuing offense.
Unlike other so-called common offenses such adultery, murder, arson,
etc. which generally end upon their commission, subversion and
rebellion are anchored on ideological base which compels
repetition of the same acrs of lawlessness and violence until the
overriding objective of overthrowing the government is
attained.
(Note: this is the extent of the discussion in the casebook. No specific
mention of Sec. 2.)
Additional doctrine of this case as mentioned by another case: The arrest
was for quelling rebellion than for the purpose of immediately
prosecuting them in court for a statury offense. The arrest therefore need
not follow the usual procedure in the prosecution of offenses which
requires the determination of the judge of probable cause before the
issuance of a judicial warrant.
The absence of a judicial warrant is of no legal impediment to
arresting or capturing persons committing overt acts of
violence against government forces.
The arrest or capture is thus impelled by the exigencies of the
situation that involves the very survival of society and its
government.
PEOPLE v. DELA CRUZ (HENRY) RULING: SC affirmed the RTC decision.
April 18, 1990 | Regalado, J. | Warrantless Arrests

PETITIONER: People of the Philippines (plaintiff appellee)


RESPONDENTS: Juan de la Cruz and Reynaldo Beltran

SUMMARY: Repsondents Dela Cruz et al are caught in flagrante delicto


as they are entrapped by the Narcotics team in a buy-bust operation for
violation of the Dangerous Drugs Act. Found guilty by the RTC, they
appeal to the SC, claiming that the evidence is inadmissible, as the drugs
were seized without a warrant.

The SC ruled that the evidence is admissible, it being seized incidental to


arrest, after being caught in flagrante delicto in a buy-bust operation.

DOCTRINE: SC concedes to the fact that buy-bust operations constitute


search and seizure of evidence without a warrant. But it was noted that a
warrant is no longer necessary, the search being incident to a lawful
arrest. This is so, because a police officer may, without a warrant, arrest a
person when, in his presence, the person to be arrested has committed,
is actually committing, or is attempting to commit an offence.

FACTS:
Prosecution alleged, that after receiving a confidential report from an
informant, a “buy-bust” operation was conducted by the Narcotics
team in Tondo around 2:30 in the afternoon to catch the pushers.
At the scene, dela cruz was caught in flagrante delicto, and thus the
Narcom agents effected the arrest of Dela Cruz et al.
This flow of facts was denied by the respondents, as they claim that Dela
Cruz was suffering from LBM and hence was on bedrest. Then all of
a sudden at around 2 in the afternoon, Narcom agents raided his
place without a search warrant or without first securing his previous
permission, and conducted a search. His body was even searched
too.
Beltran’s alibi was that, he was playing pool at Aling Ely’s place and that
he was taken because he was pointed at by the informant as one of
those selling marijuana.
Both respondents, allegedly, were brought to the NARCOM headquarters
for investigation.
Respondents Dela Cruz et al were then charged in the RTC for violation
of RA 6425, for illegal possession and distribution by sale of
dangerous drugs. RTC found both guilty.
Hence, this appeal.

ISSUE/s:
32. WoN the evidence is admissible despite the warrantless seizure –
YES
RATIO:
Respondents assail the buy-bust operation, it being conducted in order
to enforce the Dangerous Drugs Act. He claims it as no different from
a seizure of evidence without a warrant.
Despite acknowledging the risks involved as far as the conduction of
buy-bust operations are concerned, the SC stated that, the
proliferation of drug addiction and trafficking has already reached an
alarming level, and entrapment (through buy-bust operations) is one
of the proven effective means of apprehending drug dealers. Just like
how the respondents were
arrested.
176. SC cited the Sol Gen’s explanation, that a buy-bust operation is a
method employed by peace officers to trap and catch a malefactor in
flagrante delicto. It is essentially a form of entrapment since
thepeace officer neither instigates nor induces the accused to
commit a crime.
Entrapment is the employment of such ways and means for the purpose
of trapping or capturing a law breaker from whose mind the criminal
intent originated.
178. SC concedes to the fact that buy -bust operations constitute search
and seizure of evidence without a warrant. But it was noted that a
warrant is no longer necessary, the search being incident to a lawful
arrest. This is so, because a police officer may, without a warrant,
arrest a person when, in his presence, the person to be arrested has
committed, is actually committing, or is attempting to commit an
offence.
There being no violation of the constitutional right against unreasonable
search and seizure, the confiscated articles are admissible in
evidence.
PEOPLE v. AMINUDIN (DANNAH) ISSUE/s:
July 6, 1988 | Cruz, J. | Warrantless Arrest / Search and Seizure WoN the case falls under Rule 113 of the Rules of Court – NO

PETITIONER: People of the Philippines RULING: Decision of the trial court reversed.
RESPONDENTS: Idel Aminudin
RATIO:
SUMMARY: Philippine Constabulary officers received a tip that a certain In the case at hand, there was no warrant of arrest or search warrant
Idel Aminudin was on board the M/V Wilcon and was carrying with him issued by a judge after personal determination of probable cause.
marijuana. They then waited for him to disembark in Iloilo, accosted him and Contrary to the Government’s averments, Idel was not caught in
inspected his bag. After taking what looked like marijuana, they brought him flagrante delicto nor was a crime about to be committed or had
to their station for questioning. When the bundles they took were verified to just been committed to justify the warrantless arrest allowed under
be marijuana, an Information for the violation of the Dangerous Drugs Act
Rule 113 of the Rules of Court.
was filed against him. This initially included Farida, his companion, but she
was absolved after PC officers in a sworn statement attested that they Even expediency could not be invoked to dispense with the obtention of
thoroughly searched her. the warrant as in the case of Roldan v. Arca:
Here it was held that vessels and aircraft are subject to warrantless
The RTC convicted him and sentenced him to life imprisonment. The searched and seizures for violation of the customs law because
Supreme Court ruled however that his case did not fall under those these vehicles may be quickly moved out of the locality or
exceptions wherein a warrantless search was allowed. The PC officers jurisdiction before the warrant can be secured.
knew all the details about Idel: his name, the vessel he rode, when he was The present case presented no such urgency. From the conflicting
gonna arrive in Iloilo two days prior to its happening. However, they did not declatraions of the PC witnesses, it is clear that they had at least
do anything to act on it. They had time to procure a search warrant and they two days within which they could have obtained a warrant to
could have convinced the judge that there was probable cause but they
arrest and search Idel.
didn’t. This, to the Court, was a violation of Idel’s right.
His coming to Iloilo and his name was known. The vehicle was identified.
DOCTRINE: A warrantless search may be justified when one is caught in The date of arrival was certain. They could have persuaded the judge
flagrante delicto, or when a crime is about to be committed or has just been there was probable cause, and yet they did nothing.
committed,under Rule 113 of the Rules of Court. The Bill of Rights was ignored altogether because the PC head decided
that “a search warrant was not necessary”
There have been cases wherein warrantless arrests were upheld such in
FACTS: those which were buy-bust operations. This case is different. Idel was
Idel Aminudin, who claimed that his business was selling watches, was not commiting a crime, or was not about to, or it didn’t show that he
arrested on June 1984 shortly after disembarking the M/V Wilcon 9 has just committed one.
at about 8:30pm in Iloilo. He was only descending the gangplank, and to all appearances he was like
The PC officers who were waiting for him simply accosted him, inspected any of the other innocent passengers. It was only when the informer
his bag and finding what looked like marijuana took him to their
poointed to him that he suddenly became the suspect and subject to
headquarters for investigation.
When the bundles were verified as marijuana leaves, an Information for apprehension.
violation of the Dangerous Drugs Act was filed against him. The identification by the informer was the probable cause.
The information was amended to include Farida Hassen who was also Although this is not to say that Idel is innocent as it is obvious he is lying, that
arrested with him, but Farida was absolved after sworn statements of fact alone does not justify a finding that he is guilty. The constitutional
the arresting officers that hey thoroughly searched her. presumption is that he is innocent, and he will be so declared even his
According to the prosecution, the PC officers had received a tip that Idel defense is weak, as long as the prosecution is not strong enough to
was on board a vessel bound for Iloilo and was carrying marijuana.
convict him.
He was identified by name.
Without the evidence of the marijuana allegedly seized from him, the
case of the prosecution must fail. The evidence cannot be admitted,
and should never have been considered.
The search was not an icncident of a lawful arrest as there was no
warrant, and the warrantless arrest did not come under the
exceptions allowed by the Rules of Court.
HARVEY v. DEFENSOR-SANTIAGO (IYA) III of Section 2 is available to all persons, including aliens, water
1988 | Melencio-Herrera, J. | Warrant of Arrest: Deportation accused of crime or not.
It must be based upon probable cause. Probable sue has been defined
as referring to “such facts and circumstances antecedent to the
PETITIONER: Andrew Harvey, John Sherman, Adrian Van Dan Elshout
issuance of the warrant that in themselves are sufficient to induce a
RESPONDENTS: Miriam Defensor-Santiago (Commission on cautious man to rely on them and act in pursuance thereof”
Immigration and Deportation [CID]) Arrest of petitioners was based on probable cause determined after close
surveillance for three months during which period their activities are
SUMMARY: Harvey and Sherman contend the validity of the arrest and
monitored.
seizure made by agents of CID for they were done without warrant. The seizure of the photo negatives, and photographs without warrant
were seized as an incident to a lawful arrest, therefore admissible in
The SC ruled that the search and seizure performed need not follow the evidence.
stringent rules of criminal proceedings because what the case involves The restraint against their persons has become legal before of the formal
deportation proceedings which are administrative in nature. deportation charges have been filed against them as undesirable
aliens.
DOCTRINE: Deportation proceedings do not constitute a criminal action,
The requirement of probable cause, to be determined by a Judge, does
they are administrative in character, summary in nature, and need not be not extend to deportation proceedings.
conducted strictly in accordance with the ordinary court proceedings. What is essential is that there should be a specific charge against the
alien intended to be arrested and deported, that affair hearing be
conducted.”
FACTS:
Deportation proceedings do not constitute a criminal action, they are
Petitioners Harvey, Sherman, and Elshout are foreign nationals residing administrative in character, summary in nature, and need not be
in Pagsanjan, Laguna. conducted strictly in accordance with the ordinary court proceedings.
On 27 February 1988, they were apprehended in their respective residences What is essential is the warrant of arrest shall give the alien sufficient
by agents of the CID by virtue or Mission orders issued by Defensor
information about the charges against him, relating the facts relied
Santiago.
upon.
Harvey, et al. are currently detained.
Seized during the apprehension are rolls of photo negatives and photos
of the suspected child prostitutes shown in spacious poses.
On 4 March 1988, deportation proceedings were instituted against
petitioners for being undesirable aliens under Section 69 of the
Revised Administrative Code.
On 7 March 1988, Warrants of Arrests were issued by Defensor-
Santiago against Harvey and Sherman for violation of sections 37,
45, and 46 of the Immigration Act and Section 69 of the RAC.
Petitioners question the validity of their detention saying
Their right to unreasonable search and seizure was violated as the
agents of CID were not clothed with valid warrants of arrest,
search, and seizure as required by Section 2 of Article III of the
Constitution

ISSUE/s:
WON the arrest was valid - YES

RULING: SC affirmed the lower courts decision. Pwede rin wherefore.

RATIO:
The right against unreasonable searches and seizures guaranteed by Article
PEOPLE v. SUCRO (ELIEL) At that point, the team proceeded to the area and intercepted Sucor and
March 18, 1991 | Gutierrez, JR., J. | Warrantless Arrests Macabante.

PETITIONER: People of the Philippines


RESPONDENTS: Edison Sucro

SUMMARY: Fulgencio was assigned to monitor Sucro’s activities.


Fulgencio was 2 meters away surveying Sucro, who happened to be
selling marijuana in tea bags. After reporting the nefarious activities,
Fulgencio and the drug team chased and arrested Sucro to which they
found 4 tea bags and 19 sticks of marijuana. Sucro assails that there was
no warrant to arrest him. Hence this appeal.

The SC ruled that there was a lawful arrest without warrant. (1) An
offense has been committed when Sucro sold the marijuana during the
surveillance of Fulgencio, 2 meters away from the chapel.This
tantamounts to an offense committed in his presence. And that (2) during
the third sale, the team arresting had personal knowledge because of the
surveillance and that Macabante was caught throwing marijuana –
hence, an offense has just been committed.

DOCTRINE: Arrest without warrant, when lawful:


When in his presence, the person to be arrested has committed, is
actually committing, or is attempting to commit an offense;
When an offense has in fact just been committed, and he has
personal knowledge of facts indicating that the person to be
arrested has committed it;

FACTS:
Pat. Roy Fulgencio, a member of the INP was instructed by P/Lt. Seraspi
to monitor the activities of Edison Sucro, because of information
gathered by Seraspi that Sucro was selling Marijuan.
Pat. Fulgenco positioned himself under the house of a certain Regalado
at C. Quimo St. adjacent to the house of Regalado, about 2 meters
away, was a chapel.
Thereafter, Pat. Fulgencio saw Sucro enter the chapel, taking something
which turned out later to be marijuana from the compartment of a
cart found inside the chapel, and tehn return to the street where he
handed the same to a buyer.
After a while Sucro went back to the chapel and again came out with
marijuana which he gave to a group of persons. IT was at this instance
that Pat. Fulgencio radioed Seraspi, who instructed Fulgencio to keep
monitoring.
Fulgencio again called up Seraspi to report that a third buyer later
identited as Macabante, was transacting with Sucro.
Upon seeing the police, Macabante threw something to the ground which
turned out to be a tea bag of marijuana.
When confronted, Macabante readly admitted that he bought the same
from Sucro, who was later caught.
Sucro stresses that there was sufficient time for the officers to apply for a
search and arrest warrants. Hence this appeal.

ISSUE/s:
WoN the arrest without warrant is valid – YES
WoN evidence is admissible - YES

RULING: Edison is guilty for sale of prohibited drugs.

RATIO:
An offense is committed in the presence or within the view of an officer,
within the meaning of the rule authorizing an arrest without a
warrant, when the officer sees the offense, althout at a distance, or
hears the disturbances created thereby and proceeds at once to the
scene thereof.
The records show that Fulgencio went to Regalado’s house to monitor
the activities of the accused who was earlier reported to be selling
marijuana at a chapel 2 meters away from Regalado’s house.
Anent the second requirement, the fact that Macabante, when intercepted by
the police, was caught throwing the marijuana stick and when
confronted, readily admitted that the bought the same from accused
clearly indicates that Sucro had just sold the marijuana stick to
Macabante, and therefore, had just committed an illegal act of which the
police officers had personal knowledge, being members of the team
which monitored Sucro’s nefarious activity.
As for the time of warrant, because of reliable information given by some
informants that selling was going on everyday, he was constrained to
report the matter ot the Station Commander.
On the other hand, the failure of the police officers to secure a warrant
stems form the fact that their knowledge acquired from the
surveillance was insufficient to fulfill the requirments for the issuance
of a search warrant. What is paramount is that probable cause
existed.
As for the evidence being admissibe. That searches and seizures must
be supported by a valid warrant is not an absolute rule. Among the
excpetions granted by law is a search incidental to an arrest, which
provides that a person lawfully arrest may be searched for
dangersous weapons or anything which may be used as proof of the
commission of an offense, without a search warrant.
PEOPLE v. RODRIGUEZA (JP) ISSUE/s:
February 4, 1992 | Regalado, J. | Invalid Warrantless Arrest WoN the warrantless arrest was valid - NO

PETITIONER: People of the Philippines RULING: SC affirmed the lower courts decision. Pwede rin wherefore.
RESPONDENTS: Don Rodrigueza
RATIO:
SUMMARY: Through a confidential informant, a buy-bust operation was A buy-bust operation is a form of entrapemtn employed by peace officers
held. When the NARCOM agents arrived, a certain Samuel returned to to trap and catcha malefactor in flagrante delicto.
them with the suspect Don. Don rode a tricycle and returned to them with Flagrante delicto requires that a drug dealer must be caught redhanded
a plastic-wrapped object which later on turned out to be marijuana. The in the act of selling marjiana or any prohibited drug to a person acting
officers however returned to the station and only ordered another set of as a poseur-buyer.
agents to arrest the suspects without a warrant. In the instant case however, the procedure adopted by the NARCOM
agents failed to meet this qualification.
The SC ruled that the appellant should be acquitted because the conduct After allegedly consummating the sale of dried marijuana leaves,
of the officers does not match the purpose of a buy-bust operation. A buy- CIC Taduran (poseur-buyer) immediately released appellant
bust operation must catch the suspect in fragrante delicto. When Rodrigueza instead of arresting and taking him into his custody.
appellant Don gave the dried marijuana leaves, the NARCOM agents This act is contrary to the natural course of things and inconsistent with
should have arrested him then and there during the act. the aforestated prupose of a buy-bust operation.
Appellant should be acquitted.
DOCTRINE: In flagrante delicto (as a valid warrantless arrest) requires
that a drug dealer must be caught redhanded in the act of selling
marjiana or any prohibited drug to a person acting as a poseur-buyer.

FACTS:
Major Crisostomo Zeidem was approached by a confidential informant
reporting that there was an ongoing illegal traffic of prohibited drugs
in Tagas, Albay by a certain Don (petitioner).
CIC Ciriaco Taduran was to act as the poseur-buyer in the buy-bust
operation. On their way, Taduran met a certain Samuel Segovia.
Segovia left and when he returned, Don was with him.
When Taduran asked to buy marijuana, Don stopped a tricycle and
boarded such. When he returned, he gave a certain objct wrapped in
plastic which later on was identified as marijuana.
Thereafter, Taduran returned to the headquarters and made a report
regarding his said purchase.
Based on that information, Major Zeidem ordered a team to arrest the
suspects who did so later that day.
Thereafter the NARCOM agents conducted a raid in the house of
Jovencio Rodrigueza, father of appellant. During the raid they were
able to confiscate dried marijuana leaves and a plastic syringe. They
were not authorized by a search warrant.
Appellant contends that the 100 grams of marijuana leaves allegedly
bought from him were not properly identified and his constitutional
rights were violated.
PEOPLE v. BAGISTA (HENRY) their regular informant was describing.
November 18, 1997 | Nocon, J. | Warrantless search and seizure 157. RTC convicted appellant. Hence, this appeal, where respondnent
Elsie claims the search as invlaid, it being one without a valid
PETITIONER: People of the Philippines warrant.
RESPONDENTS: Elsie Bagista

SUMMARY: A regular informant tipped the NARCOM unit in Baguio of a


curly-haired girl that will transport drugs from up north. In response,
NARCOM agents set up a checkpoint in Benguet. Moments after, they
flagged down a bus, to which they introduced themselves as agents and
commenced the search. They found a girl matching the description given
by the informant, and found marijuana leaves in her bag. She was taken
for further investigation, and the leaves were tested positive. RTC
convicted her for illegal possession of drugs, but she appealed to the SC,
claiming that the evidence is inadmissible it being not obtained from a
search with warrant.

SC ruled that the warrantless search is valid, as it was done in a moving


vehicle, which is one of the exceptions to the rule as far as Art 3 Sec 2 is
concerned.

DOCTRINE: As a general rule, no search and seizure (and even arrest)


may be made without a valid search warrant. But the SC notes, that there
are exceptions to this rule, such as that of warrantless searches in
moving vehicles.

FACTS:
151. July 1988, NARCOM Baguio received a tip from their regular
informant of a girl with curly hair, 23 years old, who will be
transporting drugs from up north.
152. Upon knwoedge, the NARCOM agents went to BEnguet and set up
a checkpoint.
After about 4 and a half hours, they flagged down Dangwa Tranco Bus.
With which, after introducing themselves as NARCOM agents,
inspected the baggages of all passengers.
At the far back of the bus agents noticed a curly-haired girl, with a bag on
the side.
The agents introduce themselves as agents, and asked to inspect her
bag— they found 3 bundles of marijuana leaves covered in assorted
clothing. They confiscated this bag and brought her to NARCOM
office for further investigation. The girl is identified to be respondent
Elsie.
Respondent claims on the other hand that she is in fact engaged in buying
and selling vegetables (cabbage pa nga daw lol). That the baggage
(where the marijuana is) was taken from the luggage carrier above the
passenger seats and when nobody responded when agents asked who
owned it, they went up to Elsie because of the sole reason that she
matches the description of the girl
ISSUE/s:
33. WoN the warrantless search and seizure is valid – YES

RULING: SC affirmed the RTC ruling.

RATIO:
180. As a general rule, no search and seizure (and even arrest) may be
made without a valid search warrant. But the SC notes, that there
are exceptions to this rule, such as that of warrantless searches in
moving vehicles.
It is justified that the mobility of motor vehicles makes it impossible for
the vehicle to be searched to move out of the locality or jurisdiction in
which the warrant must be sought.
This does not give the police officers unlimited discretion to conduct
warrantless searches of automobiles in the absence of probable
cause. When a vehicle is stopped and subjected to extensive search,
such warrantless search is valid only as long as the officers
conducting have reasonable or probale cause to believe that they will
find the evidence pertaining to the crime.
The NARCOM officers in this case had probable cause to stop and
search all vehicles due to the confidential information they received
from their informant.
In the same manner, NARCOM officers had probable cause to search
Elsie’s belongings, reason being that she fits the description given by
the NARCOM regular informant.
Padilla, dissenting: Despite the similarity of this case to that of
Malstedt, J. Padilla said that there is no probable cause, insofar as
the warrantless search was done to all passengers, instead of just
doing it to those who fit the description given by the regular
informant. In the case, extensive search was indiscriminately made
on all baggages of the bus. This would then seem to be a fishing
expedition when all the other baggatges had to be searched. The
NARCOM officers had NO probable cause to reasonably believe that
the woman indeed have drugs in her bag, the informant only said
she will be transporting drugs. Marijuana obtained should be
inadmissible in evidence.
PEOPLE v. MENGOTE (DANNAH) One of them, who turned out to be accused-appellant Rogelio Mengote, was
June 22, 1992 | Cruz, J. | Warrantless Arrest / Search and Seizure found with a revolver with six live bullets. His companion, later identified
as
PETITIONER: People of the Philippines
RESPONDENTS: Rogelio Mengote

SUMMARY: An informant called the Western Police District about three


suspicious looking persons in an area in Tondo, Manila. A surveillance team
was dispatched and upon seeing two men who were “looking side to side”
and one man holding his abdomen, Patrolmen Mercado and Juan
approached them and identified themselves as policemen. They tried to
escape but because they were unable to, they were searched. The person
holding his abdomen turned out to be Mengote, in his possession a revolver
with live bullets was found. His companion, Morellos, also had a knife in one
of his pockets. Both were brought to the headquarters and an Information
for violation of PD 1866 (Illegal Possession of Firearms) was filed against
the two.

Mengote argued that the revolver was illegally seized, while the SolGen stated
that the case fell under one of a valid warrantless search under Rule 113
Section 5 of the Rules of Court. The Supreme Court ruled that it was not
justified. There was no crime being committed / about to be committed. Nor did
the policemen have personal knowledge of the fact that a crime had just been
committed. Moreover, on the argument of the SolGen that existence of an
offense was not necessary, Mengote’s action of “looking side to side” and
holding his abdomen could not have created reasonable suspicion to induce the
law enforcers that Mengote committed an offense.

DOCTRINE: A warrantless search cannot be justified if it does not fall under


Rule 113 Section 5 of the Rules of Court which states that a peace officer or
private person may, without a warrant, arrest a person:
When, in his presence, the person to be arrested has committed, is actually
committing or is attempting to commit an offense
When an offense has in fact just been committed and he has personal
knowledge of the facts indicating that the person to be arrested has
committed it
When the person to be arrested is a prisoner who has escaped from a penal
establishment…

FACTS:
The Western Police District received a telephone call from an informer
that there were three suspicious looking persons in an area in
Tondo, Manila.
A surveillance team was dispatched and Patrolmen Mercado and Juan saw
two men “looking from side to side,” one of whom was holding his
abdomen.
The Patrolmen approached the two and identified themselves as
policemen. The two attempted to run away but were unable to
escape, and then they were searched.
Morellos, had a fan knife hidden in his pocket. The caller did not explain why he though the men looked suspicious nor
The weapons were taken for them and they were brought to the did he elaborate on the impending crime.
headquarters. An Information for violation of PD 1866 (Illegal
Possession of Firearms) was filed against the two.
The weapon was the principal evidence that let to Mengote’s conviction.
He was sentenced to reclusion perpetua.
Mengote’s defense was that the revolver should not have been admitted
as evidence as it was illegally seized.
The Solicitor General however averred the arrest and search was lawful
under Rule 113 Section 5 of the Rules of Court, which states a peace
officer or private person may, without a warrant, arrest a person:
When, in his presence, the person to be arrested has committed, is
actually committing or is attempting to commit an offense
When an offense has in fact just been committed and he has personal
knowledge of the facts indicating that the person to be arrested
has committed it
When the person to be arrested is a prisoner who has escaped from a
penal establishment…

ISSUE/s:
WoN certain gestures like “looking side to side” or holding an abdomen can
satisfy the requirement of probable cause for warrantless arrests and
searches
– NO

RULING:. Appealed decision reversed and set aside. Rogelio acquitted.

RATIO:
It is evident that Paragraph (c) of Section 5 is inapplicable.
Paragraph (a) however requires that the person be arrested:
After he has committed or while he is actually committing or is at least
attempting to commit an offense
In the presence of an arresting officer
The requirements have not been established. Mengote at that time was
just “looking from side to side” and holding his abdomen. There was
no offense that was just committed or was being actually committed
or attempted.
The SolGen submits that the existence of an offense was not necessary
as long as Mengote’s acts “created a reasonable suspicion on the
part of the arresting officers and induced them in the belief that an
offense had been committed and Mengote had committed it”.
The question is, WHAT OFFENSE?
The policemen themselves testified that they were dispatched to that
place only because of the telephone call from the informer that there
were “suspicious looking” persons who were about to commit a
robbery.
In People v. Malmstedt:
The Court sustained the warrantless arrest of the accused because
there was a bulge in his waist that excited the suspicion of the
arresting officer
4
Upon inspection, it turned out to be a pouch containing hashish
In People v. Claudio:
The accused put the bag she was carryng behind the seat of the
arresting officer while she herself sat in the seat before him
His suspicion aroused and he examined the bag, which he found to
contain marijuana.
He then made the warrantless arrest and seizure and it was upheld on
the ground that the probable cause had been sufficiently
established
The case at hand is different because there was nothing to support
the arresting officers’ suspicion other than Mengote’s darting eyes
and his hand on his abdomen.
The case is simular to People v. Amminudin:
Idel was not acting suspiciously; he was just disembarking from the ship
There was no probable cause that, as the prosecution incorrectly
suggested, dispensed with the constitutional requirement of a
warrant
Par (b) is also inapplicable because its no less stringent requirements
have ot been met. The prosecution has not shown that at the time of
Mengote’s arrest an offense had in fact just been committed and the
arresting officer had personal knowledge of the facts.
All they had was hearsay information from the telephone caller.
It was only later, after Danganan had appeared at the headquarters that they
learned of the robbery in his house and of Mengote’s supposed
involvement.
As for the illegal possession of firearm found on Mengote’s person, the
policemen discovered this only after he had been searched and the
investigation conducted later revealed that he was not its owners nor
was he licensed to possess it.
An extract of the cannabis plant
PEOPLE v. BURGOS (IYA) unlawful as it is done without valid warrant, that the trial court erred in
September 4, 1986 | Gutierrez, J. | Warrantless Arrest holding the search warrant in his house for the firearm lawful.

ISSUE/s:
PETITIONER: People of the Philippines
WON the arrest and search is lawful -
RESPONDENTS: Ruben Burgos y Tito

SUMMARY: A certain Masalmok made testimonies that Burgos is a RULING: WHEREFORE, the judgment of conviction rendered by the trial
member of the NPA who is in possession of firearms. Police members, court is REVERSED and SET ASIDE. The accused-appellant is hereby
acting on the information, went tot he house of Burgos. They then ACQUITTED
arrested and seized the firearms found inside the house of Burgos.
RATIO:
Burgos was charged with illegal possession in furtherance of subversion Records disclose that when the police went to defendant’s house to
and was found guilty by the RTC. Burgos appealed the decisions saying arrest him upon the information given by Masamlok, they had neither
that the arrest and seizure was done without a valid warrant. search nor arrest warrant with them.
The Court held in Villanueva vs Querubin:
The SC ruled in favor of Burgos saying that at the time of the arrest no the state, however powerful, doesn’t have access to a man’s
crime was being committed, nor was there evidence that a crime has home, his haven of refuge where his individuality can assert
actually been committed. itself in his choice of welcome and in the kind of objects he
wants around him
DOCTRINE: It is not enough that there is reasonable ground to believe a man’s house, however humble, is his castle, and thus is
that the person to be arrested has committed a crime in a warrantless outlawed any unwarranted intrusion by the government
arrest. An essential precondition is that a crime must have been in fact or The trial court justified the warrantless arrest under Rule 113 Sec 6 of the
actually have been committed first; it isn’t enough to suspect a crime RoC:
may have been committed. When the person to be arrested has committed, is actually
committing, or is about to commit an offense in his presence;
When an offense has in fact been committed, and he has
FACTS:
reasonable ground to believe that the person to be arrested
Cesar Masamlok personally and voluntarily surrendered to the authorities has committed it;
stating that he was forcibly recruited by accused Ruben Burgos as When the person to be arrested is a prisoner who has escaped
member of the NPA. from a penal establishment or place where he is serving final
He was threatened with the use of firearm against his life, if he refused. judgment or temporarily confined while his case is pending or
Pursuant to this information, PC-INP members went to the house of the has escaped while being transferred from one confinement
Burgos and saw him plowing his field when they arrived. to another
One of the arresting offices called Burgos and asked him about the It justified the confiscation of the firearm under Rule 126, Sec 12:
firearm. At first, Burgos denied having any firearm, but later, Burgos's A person charged with an offense may be searched for
wife pointed to a place below their house where a gun was buried in dangerous weapons or anything which may be used as proof
the ground. of the commission of the offense.
After recovery of said firearm, Burgos pointed to a stock pile of cogon At the time of defendant’s arrest, he wasn’t in actual possession of any
where the officers recovered alleged subversive documents. Burgos firearm or subversive document, and was not committing any
further admitted that the firearm was issued to him by Nestor “subversive” act for he was plowing his field
Jimenez, team leader of sparrow unit. It is not enough that there is reasonable ground to believe that the person to
He was charged with illegal possession of firearm in furtherance of be arrested has committed a crime in a warrantless arrest. An essential
subversion. precondition is that a crime must have been in fact or actually have been
He denies being involved in any subversive activities and claims that he committed first; it isn’t enough to suspect a crime may have been
has been tortured in order to accept ownership of subject firearm and committed.
that his alleged extrajudicial statements have been made only under The Court fails to see why they failed to first go through the process of
fear, threat and intimidation on his person and his family. obtaining a warrant of arrest, if indeed they had reasonable ground to
The RTC of Davao del Sur found him guilty. He avers that his arrest is believe that the accused had truly committed a crime.
GO v. CA (ELIEL) The security guard of the bakeshop was shown a picture of Go and he
February 11, 1992 | Feliciano., J. | Warrantless Arrests positively identified him as the same person who had shot Maguan.
Having established that the assailant was probably Go, the police launched a
PETITIONER: Rolito Go
RESPONDENTS: Court of Appeals

SUMMARY: Go and Maguan nearly bumped each other in Wilson St.,


Manila, when the former entered a one way road. Go alighted from his
car, walked over to Maguan’s car and shot the latter inside his car, and
sped off. A security guard was able to get a photo of the plate, and
verified with the LTO that Elso Go owned the car. When a manhunt was
launched against Rolito Go, he submitted himself to the Police station 6
days later and was then arrested. Go contends that he was unlawfully
arrested without warrant. Hence this appeal.

The SC ruled that the arrest was unlawful because: (1) it does not fall
within the exception of Sec 5(a) because the alleged offense was not
committed in the presence of the arresting officer; (2) in relation with 5(b),
the offense does not fall within the purview of “just committed” and that
since the information was based on eyewitnesses, arresting officers had
no personal knowledge.

DOCTRINE: Sec 5, Rule 113:


When in his presence, the person to be arrested has committed, is
actually committing, or is attempting to commit an offense;
When an offense has in fact just been committed, and he has
personal knowledge of facts indicating that the person to be
arrested has committed it;
When the person to be arrested is a prisoner who has escaped form
a penal establishment or place where he is serving final
judgment or temporarily confined while his case is pending, or
has escaped while being transferred from one confinement to
another.

FACTS:
Eldon Maguan was driving his car along Wilson St., Metro Manila, heaing
towards P. Guevarra St. Rolito Go, entered Wlso St., where it was
one-way street and started travelling in the opposite or “wrong”
direction.
Go and Maguan’s cars nearly bumped each other, then Go alighted from
his car, walked over and shot Maguan inside his car.
A security guard at a nearby restaurant was able to take down Go’s car
plate number. Verification at the LTO showed that the car was
registered to one Elsa Ang Go.
The police obtained a facsimile or impression of the credit card used by
Go from the cashier of the bakeshop he had dined before the
shooting.
manhunt for Go.
Go presented himself before the San Juan Police Station to verify news
reports that he was being hunted by the police; he was accompanied
by 2 lawyers. The police forthwith detained him.
Go contends that he was not lawfully arrested without warrant because
he went to the police station 6 days after the shooting which he had
allegedly perpetrated. Thus, Go argues, the crime had not been “just
committed” at the time that he was arrested.

ISSUE/s:
WoN the arrest without warrant is valid – NO

RULING: Edison is guilty for sale of prohibited drugs.

RATIO:
Go’s arrest took place 6 days after the shooting of Maguan. The arresting
officers obviously were not present, within the meaning of Sec 5(a),
at the time Go allegedly shot Maguan.
Neither could the arrest effected six days after the shooting be
reasonably regarded as effected whe the shooting had in fact just
been committed within the meaning of Sec 5(b).
Moreover, none of the “arresting” officers had any personal knowledge of
facts indicating that Go was the gunman who had shot Maguan.
PEOPLE v. MANLULU (JP)
April 22, 1994 | Bellosillo, J. | Warrantless Arrest RATIO:
The killing took place at around 7 that morning. The arrest and the
PETITIONER: People of the Philippines consequent search and seizure came at around 7 that evening, some 19
hours later.
RESPONDENTS: Rolando Manlulu, Dante Samson
This instance cannot be in the purview of a valid warrantless arrest.
SUMMARY: Witness Manlapaz and the accused Manlulu and Samson The Rules of Criminal Procedure provides that the arresting officer must
were joined by NARCOM agent Alfaro in a drinking spree. Manlulu have personal knowledge of an offense which has “in fact just been
however suddenly stabbed Agent Alfaro. Samson likewise stabbed Alfaro committed.”
with an ice pick. Manlulu then finished brutally by shooting Agent Alfaro In the instant case, neither did Pat. Perez have “personal knowledge” nor
with his very own service pistol. Upon information by the witness, was the offense “in fact just been committed.”
Patrolman Perez arrested the accused some 19 hours later. While Pat. Perez may have personally gathered the information which led
to the arrest of Manlulu, that is not enough.
The SC ruled that the warrantless was indeed illegal because Patrolman The law required “personal knowledge” which is different from “personally
did not have personal knowledge to validate the warrantless arrest. He gathering of information.”
only personally gathered the facts. The rule also required that the arrest The rule requires that the arrest immediately follows the commission of
must immediately precede the act, not 19 hours later. However, since the the offense, not some 19 hours later.
eyewitness’ claim was valid, despite the illegal warrantless arrest, However, no matter how fatal this flaw of unlawful arrest may be, or
accused is still considered guilty. inadmissible the evidences obtained by Pat. Perez, the prosecution
was still able to prove the guilt of the accused beyond reasonable
DOCTRINE: The Rules of Criminal Procedure provides that the arresting doubt. The flaw becomes moot in view of the eyewitness account of
officer must have personal knowledge of an offense which has “in fact Manlapaz which the court finds credible. (Note: Not in casebook but I
just been committed.” think it’s relevant)
The illegality of the warrantless arrest cannot deprive ths tate of its right to
prosecute the guilty when all other facts on reconrd point to their
FACTS: culpability.
According to witness Manlapaz, he and Samson and Manlulu (both
accused herein) were having a drinking spree. They were later on
joined by NARCOM Agent Alfaro (in drinking).
When they transferred to the house of the witness to continue to drink,
Samson suddenly stabbed Agent Alfaro in the chest boasting “dapat
sa iyo manahimik na.”
Manlulu then followed suit and stabbed Alfaro several times in the
abdomen with an ice pick.
Samson then grabbed the service pistol of Agenet Alfaro and shot him in
the neck.
They initially fled but went back to steal Alfaro’s wristwatch and wallet.
Some 19 hours later, Patrolman Perez arrested Manlulu on the
information given by witness Manlapaz.
Perez admitted that he arrested Manlulu and seized from him the items
without a warrant nor did he inform the accused of his right to
counsel.
The accused alleged violation of his constitutional rights. (Note: No facts
in casebook at all, added from orig.)

ISSUE/s:
WoN the warrantless arrest was valid - NO

RULING: The arrest was invalid but accused is still guilty of Homicide.
MANALILI v. CA (HENRY) found on them, but the policemen let only the driver go. Manalili was
October 9, 1997 | Panganiban, J. | Searches and Seizures borught to the

PETITIONER: Alain Manalili


RESPONDENTS: CA and People of the PHilippines

SUMMARY: Petitioner Manalili was charged of illegal possession of


marijuana residue, a prhobited drug. RTC convicted him, which was
affirmed by the CA. Hence, a petition was filed by Manalili, assailing the
validity of the search and arrest.

SC ruled that the search was valid, as he was wandering with red eyes
and was swaying (he was high), hence there was probable cause for the
policemen to performa stop-and-frisk search.

DOCTRINE: In Philippine jurisprudence, the general rule is that a search


and seizure must be validated by a previously secured judicial warrant;
otherwise such search and seizure is unconstitutional and subject to
challenge. Any evidence obtained in violation of such is rendered as
inadmissible evidence, being a “fruit of the poisonous tree”. But note, that
this right is not absolute.
People v. Lacerna provided us with 5 exceptions: (#5) Stop-and-frisk
(ruled by the SC as one exception in Posadas v. CA)

FACTS:
Around 2pm of April 1988, policemen from the Anti-Narcotics Unit of the
Kalookan City Police Station were conducting a surveillance along
Mabini, Kalookan in fromt of the City Cemetery. This was because of
information that drug addicts were roaming around the vicinity.
Version of the Prosecution: Upon alighting from their vehicle, the
policemen chanced upon a male person who appeared high on
drugs, he was swaying. As this person tried to avoid the policemen,
they approached him and introduced themselves. When they asked
what he was holding, he tried to resist. But eventually, he showed his
wallet, where they found crushed marijuana residue.
This male person was brought to the Anti-Narcotics Unit of Kalookan,
and finally the male person was identified to be Alain Manalili.
Further examinations was made in the substance they got from his
wallet, and confirmed positive of marijuana.
Version of the defense: petitioner Alain was aboard a tricycle at A. Mabini
street near Kalookan City Cemetery on his way to his boarding house. 3
policemen ordered the driver to stop because the driver and his lone
passenger were under the influence of marijuana. Policemen brought
him and the driver of the trike inside their car and then a bodily search
was made on them both.
Manalili asked the policemen why he was being searched and they replied
that it was because he (Manalili) was carrying marijuana. Nothing was
police headquarters.
On their way, Manalili saw a neighbor and signaled the latter to follow
him. The neighbor thus followed until the police headquarters. Upon
arrival, he was asked to remove his pants in the presence of the
neighbor. Again, nothing was found (except from dirt and dust daw
haha). This prompted the companion of the neighbor to ask the
policemen to release the Manalili, who was later led to a jail cell.
Policemen eventually told Manalili that they found marijuana inside
his pockets.
RTC convicted Manalili for illegal possession of drugs. Manalili sought an
appeal with the CA, who affirmed the RTC ruling. Hence, this
petition.
ISSUE/s:
WoN the search and the arrest was valid -
YES RULING: SC denied the petition.

RATIO:
On the admissibility of the evidence: SC ruled that it was admissible
because the search was valid, being akin to a stop-and-frisk. In Terry
v. Ohio, a stop and frisk was defined as the vernacular designation of
the right of a police officer to stop a citizen on the street, interrogate
him, and pat him for weapons.
In Philippine jurisprudence, the general rule is that a search and seizure
must be validated by a previously secured judicial warrant; otherwise
such search and seizure is unconstitutional and subject to challenge.
Any evidence obtained in violation of such is rendered as
inadmissible evidence, being a “fruit of the poisonous tree”. But note,
that this right is not absolute.
People v. Lacerna provided us with 5 exceptions:
Incidental to lawful arrest
Search of moving vehicle
Plain view
Customs
Waiver
Stop-and-frisk (ruled by the SC as one exception in Posadas v.
CA)
In the present case, the policemen said that Manalili had red eyes and
was wobbling like a drunk man along Kalookan City Cemetery, which
according to the police information was a popular hangout of drug
addicts. (in short, high nga daw)
Besides him being high, the SC noted as well that Manalili essentially
waived the inadmissibility when he did not raise the violation of his
right to a valid search in the RTC.
A valid waiver of right has the ff requirements:
Right to be waived existed
Person waiving it had knowledge, actual or constructive, thereof
He or she had an actual intention to relinquish that right
In this case, petitioner is deemed to have waived such right for failure to
raise its violation before the RTC.
PEOPLE v. BOLASA (DANNAH) conversed with him for a while. She denied knowing the PO3s and the
June 22, 1992 | Belosillo, J. | Warrantless Arrest / Search and Seizure fact that they saw

PETITIONER: People of the Philippines


RESPONDENTS: Zenaida Bolasa, Roberto delos Reyes

SUMMARY: Upon information provided by an anonymous caller, a team of


officers proceeded to the house of Zenaida and Roberto. Upon arrival, they
proceeded to the house and peeped through the window. After seeing the
two packing suspected marijuana, they entered and introduced themselves
as police officers. They confiscated the marijuana and drug paraphernalia,
and upon confirmation that the tea bags did indeed contain marijuana, they
were charged with violation of The Dangerous Drugs Act of 1972.

The RTC sentenced the two to reclusion perpetua, but upon appeal, the Court
reversed the decision. They ruled that the warrantless search was not valid
because it did not fall under the exceptions. Moreover, they stated that since the
arresting officers knew of the names and the location of the house, they should
have first conducted surveillance and procured a warrant after determining
existence of probable cause.

DOCTRINE: Warrantless arrest and searches are valid if they fall under
Rule 113 Section 5 of the Rules of Court (please refer to Ratio 4), and if the
objects were seized in plain view. Other exceptions would be a search of a
moving vehicle, a consented warrantless search, a customs search, or a
stop and frisk or exegent and emergency circumstances.

FACTS:
An anonymous caller tipped of PO3 Dante, Salonga and Carizon that a
man and woman were repacking prohibited dtugs at a certain house
in Valenzuela, Metro Manila.
A team immediately proceeded to the house of the suspects and walked
towards the lair, accompanied by their informer.
When they reached the house, they “peeped through a small window…
and saw a man and woman repacking suspected marijuana.”
They entered the house and introduced themselves as police officers,
and confiscated the tea bags and drug paraphernalia.
Zenaida and Roberto were arrested, and after the confirmation that the
tea bags contained marijuana, they were charged with violation of
The Dangerous Drugs Act of 1972.
Both however denied the ownership of drugs and paraphernalia.
Roberto’s defense was that he and his wife were tenants of Zenaida and
when he learned that Zenaida was repacking marijuana, he
immediately asked her to leave, and it was at that moment that
police entered.
As to Zenaida, she narrated that she was about to leave for Caloocan where
she was working as a waitress when she met a certain “Rico” and
her repacking marijuana.
The trial court found the prosecution’s version more plausible and
sentenced the two to reclusion perpetua, hence this appeal.

ISSUE/s:
WoN the case constitutes a valid warrantless arrest and seizure – NO

RULING:. Zenaida and Roberto acquitted

RATIO:
This case clearly illustrates how constitutional guarantees against illegal
arrests and seizures can be violated by overzealous police officers in
the arrest of suspected drug offenders.
The Court strikes down the process adopted by the prosecution and
acquits Zenaida and Roberta fo insufficiency of evidence and
reasonable doubt.
The constitutional provision sheathes the private individual with an
impenetrable armor against unreasonable searches and seizures.
Their arrest is illegal because:
First, the arresting officers had no personal knowledge that at the time
of their arrest, Zenaida and Roberto had just committed, were
committing, or were about to commit a crime
Second, the arresting officers had no personal knowledge that a crime
was committed nor did they have any reasonable ground to believe
that Zenaida and Robert committed it
Third, both Zenaida and Roberto were not prisoners who have escaped
Neither can it be said that the objects were seized in plain view
First, there was no valid intrusion
Second, the evidence i.e. teabags was not inadvertently discovered.
The police officers intentionally peeped.
In like manner, the search cannot be categorized as a search of a
moving vehicle, a consented warrantless search, a customs search,
or a stop and frisk; it cannot even fall under exigent and emergency
circumstances. The evidence at hand is bereft of any such showing.
On the contrary, it indicates that the officers should have conducted first
a surveillance considering that the identities and address of the
culprits were already ascertained.
After conducting surveillance and determining existence of probable
cause, they should have secured a search warrant prior to effecting a
valid search and seizure.
The arrest being illegal ab initio, the accompanying search was likewise
illegal.
PEOPLE v. ESCORDIAL (IYA) the commission of the crime.
January 16, 2002 | Ponencia, J. | Warrantless Arrest Escordial however has waived his right to question the validity of the
arrest because he did not raise it at the earliest opportunity.
PETITIONER: People of the Philippines
RESPONDENTS: Escordial

SUMMARY: Escordial was arrested without warrant a week after the alleged
commission of offense. The question before the court is whether the
arresting officers had personal knowledge of facts and circumstances which
would lead them to believe that Escordial had committed a crime to validate
the arrest made.

The SC ruled that they did not have personal knowledge as they were not
present during the commission of the offense and the arrest was made days
after.

DOCTRINE: Deportation proceedings do not constitute a criminal action,


they are administrative in character, summary in nature, and need not be
conducted strictly in accordance with the ordinary court proceedings.

FACTS:
At the time of his arrest, Escordial was watching game in a basketball
court. He was not committing or attempting to commit a crime when
he was arrested by the police. Nor was he an escaped prisoner.
The question is whether his case falls under a valid warrantless search
because the police officers had personal knowledge of facts and
circumstances that would lead them to believe that Escordial had just
committed a crime.

ISSUE/s:
WON there was personal knowledge - NO

RULING:

RATIO:
Personal knowledge of facts in arrests without warrant must be based
upon probable cause which means an actual belief or reasonable
grounds of suspicion.
The grounds of suspicion are reasonable when the absence of actual
belief of arresting officers, the suspicion that the person to be
arrested is probably guilty of committing the offense.
In the present case, the crime alleged took place on December 27, 1996,
he was arrested only on January 3, 1997.
The arresting officers were not present when the crime was committed.
They could not have personal knowledge of the facts and circumstances of
PEOPLE v. SALVATIERRA (ELIEL) Marciano, upon being informed that Salvatierra was transferred to the WPD,
July 24, 1997 | Kapunan, J. | Warrantless arrests proceed to the station with Milagros who executed a sworn statement.
Thereafter, he was charged with murder. And signed the booking sheet
PETITIONER: People of the Philippines and arrest order
RESPONDENTS: David Salvatierra

SUMMARY: Charlie was a vendor of palamig when Salvatierra took a


lunge of a sharp instrument at him that caused his death. 3 months after
the killing of Charlie, Salvatierra was arrested by the officers for
misfeasance, which in turn transferred him to the Western Police District.
There, he was identified as the suspect of the killing of Charlie. After
which, the booking report and information was signed by Salvatierra.
However, he contests that he was illegally arrested.

The SC ruled that although he may be illegally arrested without a warrant,


he is estopped from questioning the same. Furthermore, the signing of
the booking report and information was deemed to be a voluntary
submission. Following which, he participated in the trial and even entered
his plea.

DOCTRINE: Verily, the illegal arrest of appellant is not a sufficient cause


for setting aside a valid judgment rendered upon a sufficient complaint
and where the trial was free from error.

FACTS:
Charlie Fernandez, a vendor of palamig was walking along M. De la
Fuente St., going towards the direction of Quiapo at the opposite
side of the street.
Suddenly, three persons met him, one of them was Salvatierra who
lunged a pointed instrument at Charlie.
Charlie still managed to walk home to tell his father about the incident but
suddenly collapsed. He was taken immediately to the hospital.
The assault was witnessed by Martinez, an ambulant vendor of fish and
salted eggs who stopped by to rest a thte rights side of J. Fajardo St.,
Manila.
Caharlie’s father, Marciano, reported the crim to the police station. The
next day, however, Carlie died.
Marciano went back to the police station to inform the authorities that his
son had died. He was advised to report the matter to the Homicide
Section of the Western Police District, where an advance information
was prepared indicating that four unidentified person perpetrated the
crime.
3 months later, Pthe police station received a complaint that Salvatierra
was creating a commotion along Sampaloc, Manila. He was thereby
taken into custody and later found out that he was a suspect in the
killing of Charlie.
ISSUE/s:
WoN his rights to warrantless arrests was violated– NO

RULING: David Salvatierra is convicted of Murder.

RATIO:
Indeed, Salvatierra’s arrest on suspicion that he was involved in the
killing of Charlie was made almost three months after the
commission of the crime and only after he had bene takien in police
custody for a minor offense.
As such, because no warrant had been obtained during the 3 month
intervening period between the commission of the crime and his
apprehension, his arrest would have ordinarily been rendered
unconstitutional and illegal inasmuch as even warrantless arrest mde
within shorter periods like 10 days are illegal.
It should eb stressed that Sec 5(b) of Rule 113 of the Rules of Court had
excluded situations under the old rule which allowed a warrantless
arrest provided that the offense “has in fact been committed.”
Salvatierra is estopped form questioning the legality of his arrest
considering that he never raised this before entering his plea.
Any objection involving a warrant of arrest or the procedure in the
acquisition of jurisdiction over the preson of an accused must be
made before he enters his plea, otherwise the objection is deemed
waived.
Consequently, any irregularty attendant to his arrest, if any, had been
cured by his voluntary submission to the jurisdiction of the trial court
when he entered his plea and participated during the trial.
Verily, the illegal arrest of appellant is not a sufficient cause for setting
aside a valid judgment rendered upon a sufficient complaint and
where the trial was free from error.
PEOPLE v. HERNANDEZ (JP) confession confessing that he was asked to join a group to kidnap
December 12, 1997 | Puno, J. | Warrantless Arrest Sharleen Tan. He confessed the

PETITIONER: People of the Philippines


RESPONDENTS: Efren Hernandez, Dionisio Jacob et al.

SUMMARY: Witness Eva as nanny, picked up Sharleen from school. When


they rode their car, she was forced in by an unidentified man. Another man
rode in the front seat and conspired with the driver to drive to a place where
they usually bring their kidnapped victims. Eva was allowed to alight from the
vehicle to inform her employers, the parents of the victim, for ransom. They
complied and their daughter was released a week later. Some of the accused
were arrested without a warrant and were convicted based on their
extrajudicial confessions.

The SC held that the warrantless arrest did not fall into any of the
exceptions. Hence, the warrantless arrest should have been invalid. But,
because the accused did not move to quash the information but
participated in the pleadings, they have effectively waived their right.

DOCTRINE: Jurisprudence is settled that an accused may be estopped


from assailing the illegality of his arrest if he fails to move for the
quashing of the Information against him before his arraignment.

FACTS:
The advent of the 90’s saw sharp increase in the incidence of kidnapping
cases in the country. The usual victims are opulent Chinese families
who readily pay ransom by the threat of death of their loved one.
Witness Eva, the victim’s nanny, testified that on this one school day,
when she and the driver picked up the Sharleen Tan (the 6-year old
victim) from school, an unidentified man pushed her into the car (Tan
family’s car) and slumped her head on the car floor.
Another man boarded the front seat. It turns out, the driver was in
conspiracy with the kidnappers. They rode to the “usual place” as
alleged by the man who just boarded.
Eva was allowed to alight from the vehicle so she could inform her employer
of Sharleen’s random. She phoned them and they immediately picked
her up.
The kidnappers repeatedly called to the Tan’s asking for ransom. The
first amount asked was P10M, then the Sharleen’s father haggled it
to P409k. The kidnappers agreed and asked him to leave the amount
in a trash can in Old Sta. Mesa.
The family complied. The kidnappers called a week later and informed
Jacinto that they had released Sharleen and left her at the Perpetual
Help Hospital.
An intensive manhunt was conducted and accused herein Efren
Harnandez was the first to be arrested. He gave an extrajudicial
participation of the other members. A series of extrajudicial
confessions were given by other accused who were arrested. They
were not arrested with a valid search arrant.
Appellants pleaded not guilty. RTC convicted them based on their
extrajudicial confessions however.

ISSUE/s:
WoN the warrantless arrests were valid - NO

RULING: Conviction affirmed but some kidnappers were acquitted due to


insufficiency of evidence.

RATIO:
Appellants were arrested under circumstances other than those justifying
a warrantless arrest.
Clearly, their warrantless arrest violated the constitution.
However, jurisprudence is settled that an accused may be estopped
from assailing the illegality of his arrest if he fails to move for
the quashing of the Information against him before his
arraignment.
In the case at bar, by entering a plea of non guilty and participating
in the trial, appellants waived their right to challenge the legality
of their warrantless arrests.
PEOPLE v. JAYSON (HENRY) hence conclusive of the fact that the policemen acted on
NOVEMBER 18, 1997 | Mendoza, J. | Warrantless Arrest basis of their personal knowledge of the victim’s death and
the responsibility of Jayson as far as the incident is
PETITIONER: People of the Philippines
RESPONDENTS: Wenceslao Jayson

SUMMARY: Wenceslao Jayson is being held as guilty of possession of


unlicensed firearms, and is held responsible of the shooting incident in the
vicinity of the nightclub where he works as a bouncer, in Ihaw-Ihaw, Davao.
Upon charge by the RTC he is contesting the validity of his arrest, it being
without a warrant.

SC held that with authorities possessing personal knowledge being in the


crime scene shortly after the incident happened, the warrantless arrest is
valid.

DOCTRINE: A peace officer or private person may, without a warrant,


arrest a person when: an offence has in fact just been committed, and he
has personal knowledge of the facts indicating that the person to be
arrested has committed it.

FACTS:
Petitioner Jayson works aas a bouncer at the Ihaw-Ihaw nightclub in Davao.
Around 10pm one evening, police received a radio message ordering
them to go to Ihaw-Ihaw for a shooting incident.
Upon arrival, they saw the victim, Nelson Jordan.
The bystanders pointed at Jayson for being responsible of the incident,
which prompted the police to arrest him.
RTC charged Jayson for vilation of PD 1866 for illegal possession of
firearms. On appeal to the CA, ruling in the trial court was affirmed,
and penalty increased. Hence, this appeal.

ISSUE/s:
WoN the warrantless arrest is valid - YES

RULING: SC affirmed the CA decision.

RATIO:
The Revised Rules of Criminal Procedure (RRCP) provides that, a peace
officer or private person may, without a warrant, arrest a person
when: an offence has in fact just been committed, and he has
personal knowledge of the facts indicating that the person to be
arrested has committed it.
In the case at bar, there was a shooting. The policemen were summoned
to the crime scene, where the bystanders pointed at the accused
Jayson as the culprit—he even tried to flee but to no avail. This is
concerned.
The SC in several cases upheld warrantless arrests with similar situations.
People v. Tonog – SC upheld the warrantless arrest, the accused being
someone found with blood-stained (blood matched victim’s) pants
and being pointed at by the informer.
People v. Gerente – SC upheld the warrantless arrest, it being made
3hrs after the incident happened, when, upon arrival of the police,
the neighbors who saw the entire thing ointed to the accused.
People v. Acol – SC upheld the warrantless arrest, where the victims of a
hold up scenario in a passenger jeepney pointed to the accused, the
same time they ran away from the authorities.
Subsequent search after arrest is likewise lawful, as provided by law.
People v. Lua is also one case where a warrantless arrest (performed after a
buy-bust operation) is upheld by the court, as the accused are caught in
flagrante delicto. This situation necessarily allows warrantless searches, too.
PEOPLE v. DORIA (DANNAH) Acts of persuasion, trickery or fraud carried out by law enforcement
June 22, 1999 | Puno, J. | Entrapment officers or the agents to induce a defendant to commit a crime
The origin of the criminal design in the minds of the government
PETITIONER: People of the Philippines officials rather than that of the innocent defendant, such that
RESPONDENTS: Florencio Doria, Violeta Gaddao the crime is the product of the creative activity of the law
enforcement officer
SUMMARY: An entrapment was organized upon an informant’s tip that a It is recognized that in every arrest, there is a certain amount of
certain “Jun” in Mandaluyong was selling marijuana. PO3 Manlangit who entrapment used to outwit the persons violating or about to violate
posed to buy one kilo of marijuana, caught Jun in flagrante delicto selling to the law. However, not every entrapment is forbidden.
them. The marked money, however, was in the house of Violeta (this is The type the law forbids is the inducing of another to violate the law, the
where he got the marijuana from after getting the money from Manlangit). “seduction” of an otherwise innocent person into a criminal career.
American federal and state courts use the “subjective” or “origin of intent”
The Court in the ratio of this case differentiated entrapment and instigation. test laid down in Sorrells v. US.
In our jurisdiction, it is instigation, not entrapment, that is a valid defense of The focus of the inquiry is on the accused’s predisposition to
the accused. Moreover, the Court stated that all the circumstances of the commit the offense charged, his state of mind and inclination
case must be looked at in entrapment cases, because this may be subject before his initial exposure to government agencies.
to abuse. There are cases in the past wherein the Court applied both the All relevant facts such as the accused’s mental and character
“objective” and “subjective” tests, in determining the guilt of the accused traits, past offenses, activities, etc are considered to assess
when the arrest was made via entrapment or “buy bust” operations. his state of mind before the crime.
Some states, however, have adopted the “objective test”.
DOCTRINE: Courts should look at all factors to determine the predisposition The inquiry is focused on the inducements used by government
of an accused to commit an offense insofar as they are relevant to agents, on police conduct, not on the accused and his
determine the validity of the defense of inducement. predisposition.
However, in Philippine jurisdiction, entrapment is not a defense available
to the accused. It is instigation that is a defense and is considered an
FACTS:
absolutory cause.
Version of the prosecution: November 1995, PNP received info that one To determine whether there is entrapment or instigation, our courts
“Jun” was engaged in illegal activity in Mandaluyong. have mainly examined the conduct of the apprehending officers, not
An entrapment was organized and when PO3 Manlangit posed to buy the predisposition of the accused to commit the crime.
one kilo of marijuana, Jun was caught selling in flagrante delicto. In People v. Boholst, we applied both tests by examining the conduct of
They frisked him but couldn’t find the marked bills on him, because the police officers in a buy-bust operation and admitting evidence of
he left it elsewhere while getting the marijuana. the accused’s membership with the notorious Sputnik Gang.
They went to the place where the marked bills were (Neneth’s house), We also considered accused’s previous convictions and held that his past
and PO3 Malangit noticed a box wherein more marijuana was and membership with the gang strengthened the state’s evidence
placed. against him.
They were able to recover the marked money and brought the money, Conversely, the evidence that the accused did not sell or smoke
marijuana found in the box, Jun (who is Doria) and Neneth (who is marijuana and did not have any criminal record was likewise
Violeta). admitted in People v. Yutuc thereby sustaining his defense that led to
his acquittal.
ISSUE/s: Though considered essential by the police in enforcing vice legislation,
WoN the buy-bust operation was legal and valid – YES the confidential informant system breeds abominable abuse.
He may himself be a drug addict, pickpocket, pimp or other petty
RULING:. RTC decision modified; Doria convicted, Violeta acquitted. criminal.
Like the informant, unscrupulous law enforcers’ motivations are legion—
RATIO: harassment, extortion, vengeance, blackmail or a desire to report an
Buy bust operation: Form of entrapment employed by peace officers as accomplishment to their superiors.
an effective way of apprehending a criminal in the act of the The Court has taken judicial notice of thie ugly reality where it observed
commission of the offense. that it is a common modus operandi of corrupt law enforcers to prey
In the American jurisdiction, entrapment has two elements: on weak and hapless persons.
We therefore stress that the “objective” test in buy-bust operations demands
that the details of the purported transaction must be clearly and adequately
shown.
This must start from the initial contact between the poseur-buer and the
pusher, the offer to purchase, the promise or payment of the
consideration until the consummation of the sale by the delivery of the
illegal drug subject.
The manner by which the initial contact was made, whether or not
through an informant, the offer to purchase, the payment, the
delivery, whether to the informant alone or the police officer, must be
the subject scrutiny by the courts to insure that law-abiding citizens
are not unlawfully induced to commit an offense.
If there is overwhelming evidence of habitual delinquency, recidivism or
plain criminal proclivity, then this must also be considered.
Courts should look at all factors to determine the predisposition of
an accused to commit an offense insofar as they are relevant to
determine the validity of the defense of inducement.
In the case at hand, the evidence shows that it was the confidential
informant who initially contacted Doria.
At the pre-arranged meeting, the informant was accompanied by PO3
Manlangit who posed as the buyer, handed the marked money and
received the marijuana. PO3 Manlangit testified, in a frank,
spontaneous, straightforward and categorical matter.
The non-presentation of the confidential informant is not fatal to the
prosecution. Informants are not usually presented in court because
of the need to hid their identity and preserve their invaluable service
to the police.
It is well settled that except when the appellant vehemently denies
selling prohibited drugs and there are material inconsistencies
in the testimonies of the arresting officers, or there are reasons
to believe that the arresting officers had motives to testify
falsely against the appellant, or that only the informant was the
poseurbuyer who actually witnessed the entire transaction, the
testimony of the informant may be dispensed with as it will merely be
corroborative of the apprehending officers’ eyewitness testimonies.
There is no need to present the informant in court where the sale was
actually witnessed and adequately proved by prosecution witnesses.
TERRY v. OHIO (IYA) between a “frisk” of the outer clothing for weapons and full blown
1968 | Ponencia, J. | Stop and Frisk search for evidence of crime.
Where a reasonably prudent officer is warranted in the circumstances of a
given case in believing that his safety or that of others is endangered, he
PETITIONER: People of the Philippines may make a reasonable search for weapons of the person believed by
RESPONDENTS: Terry and Chilton him to be armed and dangerous regardless of whether he has probable
cause to arrest that individual for crime or the absolute certainty that the
SUMMARY: Detective McFadden observed two individuals (Terry and individual is armed.
Chilton) acting suspiciously. After prolonged observation, he approached
the two and introduced himself as an officer. He then conducted a stop
and frisk on the two, only searching the outer portions of their garments.
He only searched the inside of the pockets when he felt a gun.

Being questioned is the validity of the search conducted. The Court held
that the conduct of McFadden was lawful and justified.

DOCTRINE: The officer, for his own protection, had the right to pat down
their outer clothing having reasonable cause to belief that they might be
armed.

FACTS:
Cleveland detective McFadden had been patrolling when he observed
Terry and Chilton repeatedly walking back and fort and observing a
store.
Suspecting the two men, the officer followed and then approached.
He identified himself as an officer and asked the men their names.
The men “mumbled something” where McFadden then patted down their
outside clothing.
He did not inspect the insides of the garments until he felt the guns.
The men were then taken to the police station and charged with carrying
concealed weapons.
Terry, et al. filed a motion so that the weapons confiscated wont be
admitted as evidence

ISSUE/s:
WON the search was valid - YES

RULING:

RATIO:
The guns were seized during a search incident to a lawful arrest.
McFadden had cause to believe that Terry and Chilton were acting
suspiciously, that their interrogations as warranted.
The officer, for his own protection, had the right to pat down their outer
clothing having reasonable cause to belief that they might be armed.
The court distinguished between and investigatory “stop” and an arrest, and
MALACAT v. CA (ELIEL) As the policemen gave chase, Yu caught up with and apprehended
December 12, 1997 | Davide, Jr., J. | Stop and Frisk Malacat. Upon searhing, Yu found a fragmentation grenade tucked
inside Malacat’s
PETITIONER: Sammy Malacat
RESPONDENTS: Court of Appeals

SUMMARY: According to Yu, they were on foot patrol after receiving


reports of bomb threats. They placed themselves in Plaza Miranda,
wherein they encountered two groups of Muslim Men acting suspiciously.
When they approached the group, they dispersed and eventually caught
Malacat, and upon searching found a fragmentation of a bomb. The trial
court ruled that it was a valid stop and frisk search. Hence the appeal.

The SC ruled that the stop and frisk was invalid. A stop and frisk is
defined as a limtited protective search of outer clothing for weapons. In
this case, (1) Yu’s credibility is diminished as he had to chase down
Malacat in order to search him and the testimonies leave serious doubts;
(2) there was nothing suspicious with Malacat’s conduct that would prove
a cause to search him, other than his eyes moving. The SC said that it
was dusk and highly improbable to see the eyes moving fast. And (3) the
search was not incidental to the arrest, because the grenade was not
visible. The arrest was not valid because there was no overt act
committed in his presence. The grenade was even discovered in the front
waist line.

DOCTRINE: A stop and firsk serves a two fold interest: (1) the general
interest of effective crime prevention and detection, which underlies the
recognition that a police officer may, under appropriate circumstances
and in an appropriate manner, approach a person for purposes of
investigating possible criminal behavior even without probable cause; (2)
the ore pressing interest of safety and self-preservation which permit the
police officer to take steps to assure himself that the person with whom
he deals is not armed with a deadly weapon that could nexpectedly and
fatally be used against the police officer

FACTS:
Rodolfo Yu of the Western Police District testified that in response to
bomb threats reported 1 week earlier, he was on foot patrol with
three other police officers (all of them in uniform) along Quezon
Blvd., Manila at Plaza Miranda.
They chanced upon two groups of Muslim-looking men, with each group,
comprised of 3-4 men posted at opposite sides of the corner of
Quzen Blvd. These men were acting suspiciously with “their eyes
moving very fast.”
The police officers tehn approached on group of men, who then fled in
different dirrections.
“front waist line.”
According to Malact, he was only catching fresh air at Plaza Miranda
when the policemen arrived and ordered all males to stand aside.
The policemen searched Malacat and tow other men, but found nothing
in their possession. However, he was arrested with two others,
brought to and detained at Precinct No. 3, where he was acused of
having shot a police officer.
The trial court ruled that the warrantless search and seizure of Malact
was akin to a “stop and frisk” where a “warrant and seizure can be
effected without necessarily being preceded by an arrest and whose
object is either to maintain the status quo momentarily while the
police officer sees to obtain more information.”
Malacat argued that the warrantless arrest was invalid due to absence of
any o the conditions provided in Sec 5, Rule 113.

ISSUE/s:
WoN stop and frisk was valid – NO

RULING: Trial court ruling reversed. Invalid stop and frisk.

RATIO:
In a search incidental to a lawful arrest, as the precedent arrest
determines the validity of the incidental search, the lagility of the
arrest is questioned in a large majority of these cases.
In this instance, the law requires that there first be a lawful arrest before
a search can be made – the process cannot be reversed.
At bottom, assuming a valid arrest, the arresting officer may search the
person of the arrestee and the area within which the latter may reach
for a weapon or for evidence to destroy, and seize any money or
property found which was used in the commission of the crime or the
fruit of the crime, or that which may be used as evidence, or which
might furnish the arrestee with the means of escaping or committing
violence.
Here, there could have been no vlaid in flagrante delcito or hot pursuit
arrest preceding the search in light of the lack of personal knowledge
on the part of Yu, the arresting officer, or an overt physical act, on the
part of Malacat, indicating that a crime had just been committed, was
being committed or was going to be committed. Thus not a search
incidental to an arrest.
Other notable points of Terry are that while probable cause is not
required to conduct a stop and frisk, it nevertheless holds that mere
suspicion or a hunch will not validate a stop and firsk.
A genius reason must exist, in light of the police office’s experience and
surrounding conditions to warrant the belief that the person detained
has weapons concealed about him.
A stop and firsk serves a two fold interest: (1) the general interest of effective
crime prevention and detection, which underlies the recognition that a
police officer may, under appropriate circumstances and in an
appropriate manner,
approach a person for purposes of investigating possible criminal behavior
even without probable cause; (2) the ore pressing interest of safety and self-
preservation which permit the police officer to take steps to assure himself
that the person with whom he deals is not armed with a deadly weapon that
could nexpectedly and fatally be used against the police officer
Three reasons why stop and frisk is invalid:
First, the credibility of Yu as a witness is impaired and this
likewise diminishes the probability that a genuine reason
existed so as to arrest and search petitioner.
Second, there was nothing in Malacat’s behavior or conduct,
which could have reasonably elicited even mere suspicion
other than that his eyes were moving very fast.
Third, none was visible to Yu, for as he admitted, the alleged
grenade was “discovered” inside the front waistline of
Malacat.
PEOPLE v. CHUA (JP) Appellant Chua alleges that the arrest was not conducted in a lawful or
February 4, 2003 | Ynares-Santiago, J. | Warrantless Arrest valid manner. RTC convicted Chua for illegal possession of drugs
and for illegal possession of Ammunitions.
PETITIONER: People of the Philippines
RESPONDENTS: Binad Sy Chua

SUMMARY: Based on a report by an informant, the police officers


headed to Thunder Inn Hotel where the accused was to allegedly deliver
Shabu. After parking and while the accused was walking with a sealed
Zest-O juice box, the officers immediately apprehended the suspect. The
RTC convicted the accused.

The SC held that misapprehended in flagrante delicto and stop-and-frisk.


Both are actually inapplicable in the case at bar. Mere reliable information
absent an overt act of the crime merits no probable cause hence in
flagrante delicto is unavailable and the conduct of the accused was not
suspicious as for the application of stop-and-frisk. Accused should be
acquitted.

DOCTRINE: For flagrante delicto two elements must occur: (1) the
person to be arrested must execute an overt act indicating that he has
just committed, is actually committing or is attempting to commit a crime
(2) such overt act is doen in the presence or within the view of the
arresting officer.

Stop-and-frisk rule is defined as the act of a police officer to stop a citizen


on the street, interrogate him and pat him for weapons or contraband.
The apprehending officer must have a genuine reason, in accordance
with experienceand surrounding conditions.

FACTS:
Police officers received a report from their confidential informant that
accused-appellant Chua was about to delivery drugs at Thunder Inn
Hotel in Angeles City. The informant further reported that accused-
appellant distributes illgel durgs in different karaoke bars in Angeles
City.
Based on this lead, they formed a team of operatives. When they went to
the area, their informant pointed a to a card driven by accussed
Chua which just arrived and just parked.
After accused Chua alighted from the car carrying a sealed Zest-O juice box,
the agents hurriedly went to him and introduced themselves as police
officers.
As accused pulled out his wallet, a small transparent plastic bag with a
crystalline substance protruded from his right back pocket.
The agents subjected him to a body search which yielded 20 pieces of
live firearm bullets. When they peeked into the contents of the Zest-
O box, they likewise saw crystalline substance and seized such.
They had no valid warrant.
ISSUE/s:
WoN warrantless search and seizure was valid - NO

RULING: Appellant Chua should be acquitted.

RATIO:
The trial court confused the two different concepts of a search incidental
to a lawful arrest (in flagrante delicto) and of a stop and frisk.
Regardless, both exceptions for warrantless arrest are inapplicable
in the case at bar.
For flagrante delicto two elements must occur: (1) the person to be
arrested must execute an overt act indicating that he has just
committed, is actually committing or is attempting to commit a crime
(2) such overt act is doen in the presence or within the view of the
arresting officer.
The two elements are lacking in the case at bar. In walking with a sealed
Zest-O juice box, appellant did nto act in a suspicious manner. There
was no over manifestation of a crime. Probably cause herein was more
imagined than real.
As applied to in flagrante delicto arrests, reliable information alone,
absent any overt act, is not sufficient to constitute probably cause
that would justify the arrest.
The police officers also cannot feign ignornarce of the alleged illegal
activities of the accused. His identity, address and activities were
already ascertained two years prior tot eh actual arrest. There was
no reason why the police officers could not have obtained a juidical
warrant before the arresnt.
Also, there was no stop and frisk in this case. The appellant was first
arrested before the search and seizure of the allegeld illegal items. The
apprehending officer failed to make any initial inquiry into accused-
appellant’s business in the vicinity or the contents of the Zest-O box he
was carrying.
The accused did not exhibit manifest unusual conduct and suspicious
conduct reasonable enough o dispense witht eh procedure required
by law.
POLLO v. CONSTANTINO (HENRY) Back up files in (17) diskettes were examined by the CSC Office
October 18, 2011 | Villarama, Jr., J. | Valid expectation of privacy for Legal Affairs (OLA), and found that it contained mostly of
pleadings (for and on
PETITIONER: Briccio “Ricky” A. Pollo
RESPONDENTS: Chairperson Karina Constantino-David, et al.

SUMMARY: There was an anonymous letter of complaint sent to the


CSC Chairerson, alleging that petitioner Pollo has been offering legal
assistance to people with administrative cases with the CSC. Chairman
David’s recourse was then to form an IT team to investigate Pollo’s
computer and files and make a back up of them for further examining—
which was what Pollo found violative of his constitutional rights to privacy
and due process because of lack of reasonableness of the search. After
being found guilty of the complaint alleged, he was dismissed by the
CSC. He went to the CA who dismissed his petition, then appealed to the
SC who affirmed the CA’s decision, holding that no constitutional violation
as made by the said government agency.

DOCTRINE: Some government offices may be so open to fellow


employees or the public that no expectation of privacy is reasonable.
Given the great variety of work environments in the public sector, the
question of whether an employee has a reasonable expectation of
privacy must be addressed on a case-by-case basis.

FACTS:
Petitioner Pollo is a former Supervising Personnel Specialist of the CSC
Regional Office No. IV and also the OIC of the Public Assistance and
Liaison Division (PALD) under the “Mamamayan Muna Hindi
Mamaya Na” program of the CSC.
January 2007, there was an unsigned letter-complaint addressed to
respondent CSC Chairperson Constantino-David marked
“Confidential” sent through LBC from a certain Alan San Pascual of
Caloocan. This letter was received by the Integrated Records
Management Office (IRMO) which was marked “Confidential” and left
unopened. The letter alleged that Petitioner Pollo has been offering
legal assistance government employees who have pending cases in
the CSC.
Chairperson David immediately formed a team of 4 personnel with IT
background, issued a memo directing them to investigate and to
back up all the files in computers found in the Mamamayan
Muna PALD and Legal Divisions.
The respondents (specifically Director Unite), being the officials as to where
the said computers were located (also the witnesses of the investigation
and backing up by the IT guys), texted petitioner regarding the memo, to
which he replied and said he’ll leave the matter to Director Unite and will
get a lawyer. The investigation culminated with the sealing of the
computers the next day, for the purpose of preserving all the files
restored therein.
behalf of parties facing charges as respondents), or letters in connection to
admin cases in the CSC and other tribunals.
Based on the finding, Chairperson David issued a Show-Cause Order requiring
petitioner, who had gone on extended leave, to submit explanation within 5
days (order also indicated the findings of the investigation by the CSC OLA).
Petitioner filed a Comment denying the allegations, as he is not a lawyer and
neither is he “lawyering” people with cases in the CSC. He accused CSC
officials of conducting an “fishing expedition” when they gained access to his
computer without his consent as he was on leave, which violated his right
against self-incrimination. That they were not authorized in sealing, copying,
duplicating, and printing such files as such would violate his constitutional
right to privacy and protection against self-incrimination and
warrantless search and seizure.
Moreover, it was being argued that the anonymous letter is not actionable as it
failed to comply with the requirements of a formal complaint under the
Uniform Rules on Administrative Cases in the Civil Service (URACC). In view
of the illegal search, the files/documents copied from his computer without
his consent is thus inadmissible as evidence, being “fruits of a poisonous
tree”. luh
CSC issued Resolution No. 070382, charging petitioner with Dishonesty, Grave
Misconduct, Conduct Prejudicial to the Best Interest of the Service, and
Violation of RA 6713 (Code of Conduct and Ethical Standards for Public
Officials and Employees), he was directed to file an answer and is placed
under preventive suspension effective immediately.
Petitioner filed an Omnibus Motion for Reconsideration to Dismiss and/or to
Defer, assailing that the formal charge was baseless and tha the never aided
anyone with pending cases at the CSC. The CSC resolved to treat the said
motion as the petitioner’s answer.
Petitioner then filed an Urgent Petition assailing both Show-Cause Order and the
Resolution issued by CSC, for having been issued withg grave abuse of
discretion amounting to excess/ total absence of jurisdiction.
Prior the Urgent Petition, petitioner lodged an administrative/ criminal complaint
against respodnents before the Office of the Ombudsman, and a separate
complaint for disbarment against Director Buensalida.
Petitioner received a notice of hearing from CSC, and then filed in the CA an
Urgent Motion for the issuance of TRO and preliminary injunction. Having
failed to attend the pre-hearing with CSC, the latter reset it to another date,
with warting that failure to attend will entitle prosecution to proceed with
formal investigation ex-parte.
Petitioner moved to reset the pre-hearing conference, claiming that the
investigation proceedings must be put into halt pending the resolution of his
petition to the CA. CSC denied his request and scheduled another pre-
hearing conference, which lead for petitioner to file another motion in the CA
citing them in indirect contempt.
CSC issued Resolution No. 071134, denying petitioner’s motion to set aside
the denial of his motion to defer proceedings, and went ahead with the Whether the item was in the immediate control of the
formal investigation, taking the petitioner’s absence as his waiver to his employee
right.
CSC issued Resolution NO. 071420, finding petitioner Pollo guilty, and
ordering him with a penalty of dismissal. With regard to the legality of
the search conducted, CSC quoted relevant rulings of the US SC:
O’Connor v. Ortega: govt agencies as employers could valildly
conduct search and seizure in the governmental workplace
without meeting probable cause on warrant requirement for
search and seizure.
US v. Simons: federal agency’s comuter use policy foreclosed
any inference of reasonable expectation of privacy on the
part of its employees. Government employers are entitled to
conduct a warrantless search pursuant to an investigation of
work-related misconduct provided that the search is
reasonable in its inception and scope.
CSC was of the view that the search of petitioner’s computer
successfully passed the test of reasonableness for warrantless
searches in the workplace as enunciated in the above cases.
Petitioner added in his petition to the CA the recent resolution of the CSC
ruling on is dismissal, which was dismissed, with the CA finding no
grave abuse of discretion. Aggrieved, petitioner went to the SC.
Hence, this petition.

ISSUE/s:
WoN warrantless search and seizure is valid - YES

RULING: SC denied the petition and affirmed the CA resolution.

RATIO:
The court emphasized that the constitutional guarantee is not a prohibition of
all searches and seizures but only of “unreasonable” searches and
seizures.
A case cited by the SC for reference is the 1987 case of O’Connor v.
Ortega, where a physician was employed by a state hospital, who
claimed for a violation of his Fourth Amendment rights when hospital
officials were investigating charges of mismanagement of the
psychiatric residency program, among others, and earched his office
and seized personal items from his desk and filing cabinets. In this
case, the court ruled that some government offices may be so open
to fellow employees or the public that no expectation of privacy is
reasonable. x x x Given the great variety of work environments in
the public sector, the question of whether an employee has a
reasonable expectation of privacy must be addressed on a
case-by-case basis.
As for the reasonableness, the court noted the relevant surrounding
circumstances to consider:
Employee’s relationship to the item seized
when it was seized
Whether the employee took actions to maintain his privacy in the item
In accordance with the rules, the court held that, the petitioner failed to prove that
he had actual (subjective) expectation of privacy either in his office or
government-issued computer. He did not allege that he had a separate
enclosed office which he did not share with anyone, and in fact described his
office as “full of people, his friends, unknown people”
Under the facts obtaining, the search conducted on petitioner’s computer was
justified at its inception and scope. Hence, his claim of violation of
constitutional rights must fail.
Carpio, concurring: A government employee cannot expect any privacy when
he uses a government-owned computer because he knows he cannot use
the computer for any private purpose. The constitutional guarantee applies
only to devices privately owned, and not with government property.
Bersamin, concurring and dissenting: The fact that employees may be given
individual accounts and password protection is not deemed to create any
expectation of privacy. Also, unlike the majority, J. Bersamin did not
absolutely waive his right to privacy; OM No. 10 gave the petitioner privileged
access to the Internet and allowed him to use the computer after office hours.
That he used it after office hours entitled him to a reasonable
expectation of privacy vis-à-vis his communications made during such
time.
SALCEDO ORTANEZ v. CA (DANNAH) under RA 4200.
August 4, 1994 | Padilla, J. | Privacy of Communication

PETITIONER: Teresita Salcedo-Ortanez


RESPONDENTS: Court of Appeals, Hon. Zamora, Rafael Ortanez

SUMMARY: Rafael filed an annulment case against Teresita on grounds of


lack of marriage license and/or psychological incapacity of Teresita. As
evidence he gave 3 casette tapes of alleged phone conversations between
Teresita and unidentified people.

Teresita objected to the entry of evidence. The RTC denied the objection,
and so did the CA. The SC ruled that the tapes were inadmissible as
evidence because of RA 4200 entitled “An Act to Prohibit and Penalize Wire
Tapping and Other Related Violations of the Privacy of Communication”.

DOCTRINE: Absent a clear showing that both parties to the telephone


allowed the recording of the same, the inadmissibility of the subject tapes is
mandatory under RA 4200.

FACTS:
On May 1990, private respondent Rafeal filed with the RTC a complaint
for annulment of marriage with damages against petitioner Teresita,
on grounds of lack of marriage license and/or psychological
incapacity of Teresita.
Rafael, after presenting his evidence, orally formally offered in evidence
exhibits A-M, which included 3 cassette tapes of alleged telephone
conversations between Teresita and unidentified persons.
Teresita submitted her Objection/Comment to Rafael’s oral offer of
evidence, but the RTC admitted all the evidence. An MR by Teresita
was likewise denied.
Teresita then filed a petition for certiorari in the CA. The CA dismissed the
case. Hence, this position.

ISSUE/s:
WoN the tapes are admissible as evidence – NO

RULING:. Decision of CA set aside. Casette tapes declared inasmissible in evidence.

RATIO:
RA 4200 entitled “An Act to Prohibit and Penalize Wire Tapping and
Other Related Violations of the Privacy of Communication” expressly
makes such tape recordings inadmissible in evidence.
Clearly, the RTC and CA failed to consider the said RA in admitting the
cassette tapes in question.
Absent a clear showing that both parties to the telephone allowed the
recording of the same, the inadmissibility of the subject tapes is
mandatory
ZULUETA v. CA (IYA) The intimacies between husband and wife do not justify anyone of
February 20, 1996 | Mendoza, J. | Privacy of Communication them in breaking the drawers and cabinets of the other and in
ransacking them for any

PETITIONER: Cecilia Zulueta


RESPONDENTS: Court of Appeals, Alfredo Martin

SUMMARY: Cecilia forcibly took several documents from the office of


her husband, Martin. The items she seized were to be used as evidence
in a case for legal separation and for the disqualification from practice of
medicine of Martin.

The RTC and CA ordered the return of the documents and papers taken by
Cecilia without Martin’s knowledge and consent. Cecilia questions these
decisions.

The SC upheld the lower court decisions. It ruled that the items are
inadmissible as evidence. Although they are husband and wife, their
marriage does mean that each party is no longer entitled to privacy.

DOCTRINE: A person, by contracting marriage, does not shed her/his


integrity or her/his right to privacy as an individual and the constitutional
protection is ever available to him or to her.

FACTS:
Cecilia Zulueta is the wife of Alfredo Martin who is a doctor of medicine.
On March 26, 1962, Cecilia entered the clinic of her husband with her
mother and their driver. In the clinic was the secretary of Martin.
She forcibly opened the drawers and cabinets of ther husband’s clinic
and took several documents consisting of letters of Martin with his
alleged paramours, greeting cards, diaries, photographs, and
passport.
The documents and papers seized was for use as evidence in a case for
legal separation and for disqualification from the practice of medicine
which Cecilia filed against Martin.
Martin contends before the Court that the items seized cannot be used
against him as they were illegally obtained.

ISSUE/s:
WON the items seized are admissible as evidence - NO

RULING: Petition is DENIED.

RATIO:
telltale evidence of marital infidelity.
A person, by contracting marriage, does not shed her/his integrity or her/his right
to privacy as an individual and the constitutional protection is ever available
to him or to her.
The Constitution respects the privacy of communications and correspondence of
each individual as inviolable.
The only exception to the prohibition in the constitution is if there is a "lawful
order from the court or which public safety or order require otherwise, as
prescribed by law."
Any violation of this provision renders the evidence obtained inadmissible "for
any purpose in any proceeding."
PEOPLE v. MARTI (ELIEL) Job Reyes, proprietor and husband of Anita, following standard operating
January 18, 1991 | Bidin, J. | Exclusionary Rule procedure, opened the boxes for final inspection. When he opened
Marti’s box, a peculiar odor emitted therefrom.
PETITIONER: People of the Philippines His curiosity aroused, he squeezed oe of the bundles allegedly
containing
RESPONDENTS: Andre Marti

SUMMARY: Marti sent 4 gift packages to Zurich through Manila Packing


and Export Forwarders. The proprieter, Job on his final search of the
things in accordance with the standard operation procedure noticed and
smelt something peculiar from Marti’s packages. When he searched the
package, he brought some to the NBI for examination. Upon which, the
Job showed the packages to the NBI in his office and the latter seized the
boxes. Marti contends that the NBI committed an illegal search and
seizure without warrant. Hence this petition.

The SC ruled that there was no illegal search and seizure in this case, thus
evidence is admissible. (1) It was not the NBI who searched the packages,
rather it was Job
– a private individual – who showed the articles to the NBI; (2) Mere
presence and observance does not constitute to a warrantless arrest
when the narcotics articles were simply shown to them in plain sight; and
(3) the constitutional proscription cannot be invoked to a private person,
but it is a guarantee against the State.

DOCTRINE: The constitutional proscription against unlawful searches


and seizures therefore applies as a restraint directed only against the
government and its agencies tasked with the enforcement of the law.
Thus, it could only be invoked against the State to whom the restraint
against arbitrary and unreasonable exercise of power is imposed.

FACTS:
Marti and his common law wife, Shirley Reyes, went to the booth of the
“Manila Packing and Export Forwarders” in the Pistang Pilipino
complex, carrying with them 4 gift wrapped packages.
Marti informed Anita Reyes, proprietess, that he was sending the
packages to a friend in Zurich, Switzerland.
Anita Reyes then asked Marti if she could examine and inspect the
packages. Marti, however, refused, assuring her that the packages
simply contained books, cigars, and gloves and were gifts to his
friend in Zurich.
4 packages were then placed inside a brown corrugated box on by 2 ft in
size. Styrofoam was placed at the bootom and on top of the packages
before the box was sealed with masking tape, thus making the box
ready for shipment.
glovs and felt dried leaves inside. Opening one of the bundles, he
pulled out a cellophane wrapper protruding from the opening of one
of the glvoes. He made an opening on one of the cellophane
wrappers and took several grams of the contents thereof.
Job Reyes forthwith prepared a letter reporting the shipment to the NBI
and requesting a laboratory examination of the samples he extracted
form the cellophane wrapper.
Job informed the NBI that the rest of the shipment was still in his office. He
then brought out the box in which Marti’s packages were placed and in
the presence of the NBI agents, opened the top flaps, removed the
styro-foam and took out the cellophane wrappers form insdie the gloves.
Dried marijuana leaves were found to have been contained inside the
cellophane wrappers.
Hence the petition. Marti contends that NBI agents made an illegal
search and seizure of the evidence used in prosecuting the case.

ISSUE/s:
WoN the search and seizure was invalid – NO

RULING: Guilty of the crime charged.

RATIO:
In the absence of governmental interference, the liberties guaranteed by
the Constitution cannot be invoked against the State.
The contraband in the case at bar having come into possession of the
Government without the latter transgressing Marti’s right agasint
unreasonable search and seizure, the Court sees no cogent reason
why the same should not be admitted agasint him in the prosecution
of the offense charged.
Records of the case clearly indicate that it was Job, the proprietor of the
forwarding agency, who made search/inspection of the packages.
Said inspection was reasonable and a standard operating procedure
on the part of Job as a precautionary measure before delivery of
packages to the BOC or Bureau of Posts.
Clearly, the NBI agaents made no search, much less an illegal one,
contrary to the postulate of Marti.
The mere presence of the NBI agents did not convert the reasonable
search effected by Job into a warrantless search and seizure
proscribed by the Constitution.
Merely to observe and look at that, which is in plain sight is not a search.
Having observed that which is open, where no trespass has been
committed in aid thereof, is not search. Where the contraband
articles are identified without a trespass on the part of the arresting
officer, there is not the search that is prohibited by the constitution.
That the Bill of Rights embodied in the Constitution is not meant to
be invoked against acts of private individuals finds support in
the deliberations of the Constitutional Commission.
“Protection against the state. The Bill of Rights governs the
relationship between the individual and the state. Its concern is
not the relation between individuals, between a private
individual and other individuals. What the Bill of Rights does is
to declare some forbidden zones in the private sphere
inaccessible to any power holder.”
The constitutional proscription against unlawful searches and
seizures therefore applies as a restraint directed only against
the government and its agencies tasked with the enforcement of
the law. Thus, it could only be invoked against the State to
whom the restraint against arbitrary and unreasonable exercise
of power is imposed.
The admissibility of the evidence procured by an individual effected
through private seizure equally applies, in pari, passu, to the
alleged violation, non-governmetntal as it is, of Marti’s
constitutional rights to privacy and communication.
RAMIREZ v. COURT OF APPEALS (JP) ISSUE/s:
September 28, 1995 | Kapunan, J. | Right to Privacy WoN petitioner Ramirez can be convicted of wire tapping - YES

PETITIONER: Socorro Ramirez


RESPONDENTS: Court of Appeals, Ester Garcia

SUMMARY: Petitioner Ramirez is claiming that Respondent Garcia


confronted her in her office and insulted and vexed her to her humiliation.
Ramirez recorded such confrontation and used its transcript as evidence
for her civil case for damages. Respondent in response a criminal case
against Ramirez for violating RA 4200 for petitioner Ramirez’s illegal wire
tapping. Petitioner Ramirez countered by saying the RA 4200 does not
apply to her because she was a party to the conversation. RTC granted
Ramirez’ motion to quash. CA reversed. Ramirez appealed.

SC ruled that there was no ambiguity to the law and that the law makes
no distinction whether one is a party to the conversation or not. What is
penalized is the violation of the privacy of one for wire tapping and
recording without his/her consent. Petitioner should be liable under RA
4200.

DOCTRINE : The law makes no distinction as to whether the party


sought to be penalized by the statute ought to be a party other than or
different from those involved in the private communication.

FACTS:
Petitioner Ramirez once filed a civil case for damages against
respondent Garcia. Ramirez is claiming that Garcia confronted her in
her (Ramirez’) office and allegedly vexed, insulted and humiliated
her in a hostile and furious mood and ina manner offensive to
petitioner’s dignity and personality.
To support her claim, petitioner Ramirez produced a verbatim transcript
of the event and sought for damages in the amount of P610,000. The
transcript was from a tape recording (by the petitioner) of the
confrontation.
Private respondent Garcai filed a criminal case against petitioner
Ramirez for the alleged illegal taping, in violation of RA 4200 or ‘An
act to prohibit and penalize wire tapping and other related violations
of private communication and other purposes.’
Petitioner Ramirez counters and argues that RA 4200 does not apply to
the case at bar because it was private conversation and the
recording was done by a party to that conversation (her, Ramirez);
that RA 4200 only unauhtorizes taping of a conversation by a party
other than those involved.
RTC granted the motion to quash. CA reversed. Hence, Ramirez
appealed.
RULING: CA decision is affirmed. The allegations sufficiently constitute an
offense under RA 4200.

RATIO:
Sec. 1 of RA 4200 states: It shall be unlawfull for any person, not being
authorized by all the parties to any private communication or spoken
word, to tap any wire or cable, or by using any other device or
arrangement, to secretly overhear, intercept, or record such
communication or spoken word by using a device commonly known
as a dictaphone or dictagraph or detectaphone or walkie-talkie or
tape recorder, or however otherwise described.
The aforestated provision clearly and unequivocally makes it illegal for
any person, not authorized by all the parties to any private
communication to secretly record such communication by means of
a tape recorder.
The law makes no distinction as to whether the party sought to be
penalized by the statute ought to be a party other than or different
from those involved in the private communication.
The statute's intent to penalize all persons unauthorized to make such
recording is underscored by the use of the qualifier "any."
Consequently, as respondent Court of Appeals correctly concluded,
"even a (person) privy to a communication who records his private
conversation with another without the knowledge of the latter (will)
qualify as a violator."
The unambiguity of the express words of the provision, taken together
with deliberations from the Congressional Record (which show that
they have intended likewise so), therefore plainly supports the view
held by the respondent court that the provision seeks to penalize
even those privy to the private communications. Where the law
makes no distinctions, one does not distinguish.
SILAHIS v. SOLUTA (HENRY) Once inside, a search was made, to which tey saw a plastic bag
February 20, 2006 | Carpio-Morales, J. | Exclusionary Rule containing marijuana.
(inadmissibility)

PETITIONER: Silahis International Hotel


RESPONDENTS: Rogelio Soluta, et al.

SUMMARY: Information reached the Rapier Enforcement Professional


Investigation and Security Agency, Inc (RAPISA) about drug trade,
prostitution, and dollar smuggling, happening at the union office of
petitioner company. Taking the matter at hand, the RAPISA raided the
office and searched the place, and found a box of drugs. The search,
being warrantless, rendered the evidence gathered to be inadmissible,
per RTC. This was affirmed by the CA, hence an appeal was filed.

The SC ruled that, there was sufficient time for the petitioners to issue a
warrant, and yet no warrant was issued. This constitutes inadmissibility of
evidence gathered, as it violated the constitutional right of the
respondents against unreasonable (warrantless) searches and seizures.

DOCTRINE: If there is sufficient time for the authorities to secure a warrant


and yet they pushed through with a warrantless search, the evidence is
inadmissible, it being a violation of the Constitutional right of unreasonable
searches and seizures.

FACTS:
This case is squarely about petitioner Silahis assailing the CA ruling
holding petitioners et al civilly liable to respondents or violation of
their constitutional right against unreasonable search of their office.
The petitioner’s version of the antecedents: Maniego, General
Manager of Rapier Enforcement Professional Investigation and
Security Agency, Inc. (RAPISA) contracted by the hotel for security,
received reports of drug sale, consumption, as well as dollar
smuggling and prostitution, taking place allegedly at the union office.
One morning, one Panlilio (VP for Finance of Silahis) including security,
entered the union office, with the permission of union officer Henry
Babay, who told the abut the suspected activities. They found a
plastic bag under the table, which contained dried marijuana leaves.
Respondents’ version of the antecedents: Loida, a laundrywoman of
the hotel, stayed overnight at the female locker room of the hotel
(near union office). At dawn, she heard poinding sounds outside.
Upon peeking, she saw 5 men in barong tagalog, not employees,
forcebly opening the union office door. She saw one hid something
behind his back.
The next morning, she reported what she saw to the Security Officer,
which lead for them to go inside the union office. Upon arrival to the
front door, they were unable to open the door—which made them
force themselves in.
RTC acquitted the respndents, ruling that the evidence is inadmissible.
This was likewise affirmed by the CA, and hence this appeal.

ISSUE/s:
WoN the evidence gathered in the warrantless earch and seizure is
admissible
– NO

RULING: SC denied the petition.

RATIO:
Petitioners claim that being private persons, they are not covered by the
standards on illegal searches and seizures as elaborated in People
v. Aruta.
The course taken by the petitioners is illegal, it not falling under any of
the exceptional instances when a warrantless search is allowed by
law.
Petitioners’ violation of individual respondents’ constitutional right against
unreasonable search thus furnishes the basis of award of damages.
MHP Garments v. CA was a case where the SC ruled that if there is
sufficient time for the authorities to secure a warrant and yet they
pushed through with a warrantless search, the evidence is
inadmissible, it being a violation of the Constitutional right of
unreasonable searches and seizures.
The petitioner’s defense that property rights justified the search is of no
merit. Respodnents, being lawful occupants, had the right to raise
the question of validity of the search and seizure.
Also the petitioner’s claim that Henry allowed them to enter the union
office is bereft of merit. Henry even said, that he was forced to allow
them to enter the office despite his protest of such search to be
conducted. He even asked them for a warrant—something they did
not possess.
PEOPLE v. DAMASO (DANNAH) The purpose of the law is to prevent violations of private security
August 12, 1992 | Medialdea, J. | Waiver of Rights Under Sections 2&3 in person and property, and unlawful invasions of the sanctity
of the home by officers
PETITIONER: People of the Philippines
RESPONDENTS: Basilio Damaso (Bernardo) & Bernie Mendoza (Ka Dado)

SUMMARY: PC Officers led by Lt. Quijardo were sent to verify presence of


CPP/NPA members in Dagupan. Some people they interviewed directed
them to the house of Damaso, where they found alleged subversive
materials. Damaso was convicted of violation of PD1866, in furtherance of,
or incident to, or in connection with, the crime of subversion.

The Court ruled that the warrantless search in his home was illegal. The
consent was given by a certain Luzviminda and Damaso’s maid, Luz and
not by Damaso. The PC Officers had no right to invade Damaso’s home,
especially since Damaso was not around when this occurred.

DOCTRINE: The constitutional immunity from unreasonable searches and


seizures, being a personal one, cannot be waived by anyone except the person
whose rights are invaded or one who is expressly authorized to do so in his or
her behalf.

FACTS:
On June 1988, Lt. Quijardo, a PC officer and some companions were
sent to verify the presence of CPP/NPA members in Dagupan.
When they apprehended certain people and interrogated them, the
persons apprehended revealed that there was an underground
safehouse in Gracia Village, Urdaneta, Pangasinan.
There they found subversive documents, a radio, 1x7 caliber, .45 firearm,
and others.
Damaso was singled out as the sole violator of PD 1866, in furtherance
of, or incident to, or in connection with the crime of subversion.

ISSUE/s:
WoN the consent of Luz and Luzviminda was a valid waiver of Damaso’s
right against unreasonable searches and seizures – NO

RULING:.

RATIO:
Even assuming that Damaso is the lessee of the house, the case against
him will still not prosper, the reason being that the law officers failed
to comply with the requirements of a valid search and seizure
proceedings.
The right against unreasonable searches and seizures is enshrined in
the constitution.
of the law acting under legislative or judicial sanction and to give remedy
against such usurpations when attempted.
However, such right is not absolute. There are instances when a warrantless
search and seizure becomes valid, namely:
Search incidental to an arrest
Search of a moving vehicle
Seizure of evidence in plain view
None of these exceptions is present in this case.
The SolGen argues however that:
The group of Lt. Quijardo entered Damaso’s house upon invitation of Luz
and Luzviminda, helper of Damaso.
That when Luz opened on of the rooms, they saw a copier machine,
computer, M14 rifle, bullets and ammos, radio set and more
subversive item.
That technically, there was no search as the group was voluntarily shown
the articles used
That besides, a search may be validly conducted without a search
warrant with the consent of the person searched in this case,
Damaso’s helper and Luz allowed them to enter and to look around
Damaso’s house
And that since the evidence seized was in plain view of the authorities,
the same may be seized without a warrant.
The Court is not persuaded. The constitutional immunity from unreasonable
searches and seizures, being a personal one, cannot be waived by
anyone except the person whose rights are invaded or one who is
expressly authorized to do so in his or her behalf.
In the case at hand, records show that Damaso was not in his house at that time
Luz and Luzviminda allowed the authorities to enter.
There is no evidence that would establish the fact that Luz was indeed Damaso’s
helper, or if it were true, that Damaso had given her the authority to open his
house in his absence.
SPOUSES VEROY v. LAYAGUE (IYA) Capt. Obrero, with Major Macasaet, then went to the house. They had to
June 18, 1992 | Paras, J. | Waiver of Rights under Sections 2 and 3 break open, with help of a locksmith, the doors to the rooms during
their search and found illegal weapons inside.
Spouses now contest the validity of the seizure saying that they only
PETITIONER: Spouses Very
RESPONDENTS: Layague

SUMMARY: The house of Spouses Veroy in Davao was reported to be in


use of rebel forces. The officers sought for the approval of the Veroy’s, who
were in Manila at the time, before conducting the search. The spouses
agreed to the inspection only if their family friend, Major Macasaet would be
present and only to the extent of identifying whether the house indeed was
being used by rebels.

In the search conducted, the officers had to employ a locksmith to open


the doors of the different rooms. There they found weapons and
ammunition among other things.

Spouses now question the admissibility of the seized items in a case


against them for illegal possession as it was obtained without proper
warrant.

The SC ruled that the officers acted outside of their authorization. As the
circumstances of the case is not tantamount to any of the exemptions of
a valid warrantless search, the items could not be used as evidence.

DOCTRINE: The officer, for his own protection, had the right to pat down
their outer clothing having reasonable cause to belief that they might be
armed.

FACTS:
Spouses Veroy owned and formerly resided at Skyline Village, Davao
City. They moved to East Kamias, Quezon City because of the job
promotion of Leopoldo Veroy.
They left their house at Davao with a caretaker. Only keys to the kitchen
were given to the caretaker, keys to the main house and rooms
inside remained with the Veroys.
Capt. Obrero of the Talomo Patrol Station received a directive saying that
the house of Veroys in Davao was being used as a hideout of rebel
soldiers.
They first sought for the approval of the Veroys through a telephone call
before entering the house.
Spouses Veroy would only allow the inspection if it were to be conducted
in the presence of Major Macasaet who was their family friend.
authorized the inspection of whether or not there were rebel soldiers
residing in their house and not the intensive search conduced by the
officers.

ISSUE/s:
WON the search was valid - NO

RULING:

RATIO:
The Constitution guarantees the right of the people to be secure in their
persons houses and effects against unreasonable searches and
seizures.
However, the rule that searches and seizures must be supported by a
valid warrant is not an absolute one. Among the recognized
exemptions are:
search incidental to an arrest
search of a moving vehicle
seizure of evidence in plain view
None of these exceptions pertain to the case at hand. The articles seized
were confiscated illegally and are therefore protected by the
exclusionary principle.
They cannot be used as evidence against the petitioners in the criminal
action against them for illegal possession of firearms.
PEOPLE v. EVARISTO (ELIEL) by the police officers and even subjected to physical and mental
December 11, 1992 | Padilla, J. | Waiver of Rights indignities.
The trial court found the guilty of illegal possession of firearms. Hence
PETITIONER: People of the Philippines this
RESPONDENTS: Santiago Evaristo and Noli Carillo

SUMMARY: During the routine patrol of Romeroso, they heard


successive gunshots to which they responded and saw Rosillo. When
they chased Rosillo, they chanced upon Evaristo and Carillo. When they
asked if they could search Evaristo’s home, the latter agreed and found a
number of firearms. As for Carillo, a noticeable bulge was evident then
admitted it was a .38 revolver. Hence the petition, contending the validity
of the search and seizure.

The SC ruled that it was valid. For Evaristo, he waived his right when he
granted permission to enter the house and incidentally and inadvertently
saw the firearms. As for Carillo, he false under Sec 5(b) Rule 113 of
warrantless arrests, when the possession of firearm has been committed
with personal knowledge of the officer after the successive gunshots, this
visual observation is in line with the earlier report of gunfire.

DOCTRINE: Yet, the rule that searches and seizures must be supported
by a valid search warrant is not an absolute and inflexible rule, for
jurisprudence has recognized several excpetions to the search warrant
requirement.

FACTS:
Sgt. Eladio Romeroso and CIC Edgardo Vallarta of the PC, indicates that
they were on routine patrol duty in Cavite, when successive bursts of
gunfire were heard in the vicinity.
Proceeding to the approximate source of the same, they came upon one
Barequiel Rosillo who was firing a gun into the air.
Seeing the patrol, Rosillo ran to the nearby house of Evaristo prompting
the lawmen to pursue him. Inquiring as to the whereabouts of
Rosillo, the police patrol members were told that he had already
escaped through a window of the house.
Sgt. Vallarta immediately observed a noticeable bulge around the waist
of Carillo who, upon being frisked, admitted the same to be a .38
revolver. The gun was confiscate and Carillo was invited for
questioning.
As the patrol was still in pursuit of Rosillo, Sgt. Romeroso sought
Evaristo’s persmission to scour through the house, which was
granted. In the sala, he found, not Rosillo, but a number of firearms
and paraphernalia supposedly used in the repair and manufacture of
firearms, all of which, thereafter, became the basis for the present
indictment against Evaristo.
For their part, Evaristo and Carillo dispute the above narration of the
events in question, alleging that they wre foricibly taken into custody
petition.

ISSUE/s:
WoN the search and seizure was invalid – NO

RULING: Guilty of the crime of illegal possession of firearms.

RATIO:
It is to be noted that what the constitutional provisions prohibit are
unreasonable searches and seizures. For a search to be
reasonable under the law, there must as a rule, be a serach
warrant validly issued by an appropriate judicial officer.
Yet, the rule that searches and seizures must be supported by a
valid search warrant is not an absolute and inflexible rule, for
jurisprudence has recognized several excpetions to the search
warrant requirement. Among these exceptions is the seizure of
evidence in plain view. Thus it is recognized that objects
inadvertently falling in the plain view of an officer who has the
right to be in the position to have that view, are subject to
seizure and may be introduced in evidence.
The records in this case show that Sgt. Reomerosa was granted permission
by Evaristo to enter his house. Therefore, it is clear that the search for
firearms was not Romerosa’s purpose in entering the house, thereby
rendering his discovery of the subject firearms as inadvertent and even
accidental.
With respect to the firearm seized from Carillo, the SC sustains the
validity of the firearm’s seizure and admissibility in evidence, based
on the rule on authorized warrantless arrests.
Sec 5 (b), Rule 113: When an offense has in fact just been
committed, and he has personal knowledge of facts
indicating that the person to be arrested has committed
it.
For as disclosed by the records, the peace officers, while on patrol heard
bursts of gunfire and thus proceed to investigate the matter.
An offense is committed in the presence or within the view of an
officer, within the meaning of the rule authorizing an arrest
without a warrant, when the officer sees the offense, although at
a distance, or hears the disturbances created thereby and
proceeds at once to the scene thereof.
As to the existence of personal knowledge, giving chase to Rosillo,
the peace offciers came upon Evaristo and Carillo who were
then asked concerning Rosillo’s whereabouts. This visual
observation along with the earlier report of gunfire, as well as
the peace officer’s profession sinticnts, are more than sufficient
to pass the test of the Rules.
Consequently, under the facts, the firearm taken form Carillo can be said
to have been seized incidental to a lawful and valid arrest.
VIVARES v. ST. THERESA’S COLLEGE (JP) Julia and Julienne drinking hard liquor and smoking
September 29, 2014 | Velasco, Jr., J. | Right to Privacy cigarettes inside a bar;

PETITIONER: Rhonda Ave. S. Vivares and Sps. Suzara


RESPONDENTS: St. Theresa’s College et al.

SUMMARY: Computer Teacher Escudero was informed by her students


that some seniours of St. Theresa’s College posted pictures online of
them only in brassieres. Since it was a violation of the student handbook,
the students were barred from their upcoming commencement exercises.
Petitioners herein, the parents of the minors who posted such, filed for
damages and eventually a writ of habeas data for the photos and
documents. RTC denied such.

The SC ruled that the writ cannot be granted because of the absence of
an actual violation of the privacy of the minors. The petitioners failed to
prove that the minors utilized the privacy settings given by Facebook as a
safeguard for them to prove their expectation of privacy.

DOCTRINE: The writ of habeas data can be availed of as an independent


remedy to enforce one’s right to privacy, more specifically the right to
informational privacy.

It is through the availability of said privacy tools that many OSN (online
social netork) users are said to have a subjective expectation that only
those to whom they grant access to their profile will view the information
they post or upload thereto.

FACTS:
Nenita Julia V. Daluz (Julia) and Julienne Vida Suzara (Julienne), both
minors, were, during the period material, graduating high school
students at St. Theresa’s College (STC), Cebu City.
Sometime in January 2012, while changing into their swimsuits for a
beach party they were about to attend, Julia and Julienne, along with
several others, took digital pictures of themselves clad only in their
undergarments.
These pictures were then uploaded by Angela Lindsay Tan (Angela) on
her Facebook profile.
Escudero, a computer teacher at STC’s high school department, learned
from her students that some seniors at STC posted pictures online,
depicting themselves from the waist up, dressed only in brassieres.
Escudero then asked her students if they knew who the girls in the
photos are.
The students logged their respective personal Facebook accoutns and
showed Escudero photos of the identified students which include:
Julia and Julienne along the streets of Cebu wearing articles Without these privacy settings, respondents’ contention that there is no
of clothing that show virtually the entirety of their black reasonable expectation of privacy in Facebook would, in context, be
brassieres. correct. However, such is not the case. It is through the availability of
What is more, Escudero’s students claimed that there were times when said privacy tools that many OSN (Online Social Network) users are
access to or the availability of the identified students’ photos was not said to have a
confined to the girls’ Facebook friends, but were, in fact, viewable by any
Facebook user.
Escudero reported the matter and through one of her student’s Facebook
page, showed the photos to Tigol, STC’s Discipline-in-charge.
Tigol found the students to have violated the Student Handbook. The
Assistant principal then informed their parents that the students were
barred from joining the coming commencement exercises.
A week before graduation, Angela’s mother filed a petition for injuction
and damages. The RTC issued the TRO but STC nevertheless
barred the students from participating in the graduation irghts.
Petitioners herein (Rhonda Vivares is the mother of Julia) then filed
before the RTC a petition for the Issuance of a Writ of Habeas Data
claiming that the rights of the minors especially their right to privacy
has been invaded. RTC dismissed such petition. Petitioners
appealed.

ISSUE/s:
WoN a writ of habeas data should be issued given the factual milieu –
NO
WoN there was indeed an actual or threatened violation of the right to
privacy in the life, liverty or security of the minors involved - NO

RULING: Lower court decision is affirmed. Petition for writ of habeas data is
denied

RATIO:
The writ of habeas data can be availed of as an independent remedy
to enforce one’s right to privacy, more specifically the right to
informational privacy. The remedies against the violation of such
right can include the updating, rectification, suppression or
destruction of the database or information or files in possession or in
control of respondents.
Habeas data is a protection against unlawful acts or omissions of public
officials and of private individuals or entities engaged in gathering,
collecting, or storing data about the aggrieved party and his or her
correspondences, or about his or her family. Such individual or entity
need not be in the business of collecting or storing data.
To address concerns about privacy, but without defeating its purpose,
Facebook was armed with different privacy tools designed to
regulate the accessibility of a user’s profile as well as information
uploaded by the user.
subjective expectation that only those to whom they grant access to their
profile will view the information they post or upload thereto.
However, before one can have an expectation of privacy in his or her
OSN activity, it is first necessary that said user, in this case the
children of petitioners, manifest the intention to keep certain
posts private, through the employment of measures to prevent
access thereto or to limit its visibility.
It is well to note that not one of petitioners disputed Escudero’s sworn
account that her students, who are the minors’ Facebook “friends,”
showed her the photos using their own Facebook accounts. This only
goes to show that no special means to be able to view the allegedly
private posts were ever resorted to by Escudero’s students, and that it is
reasonable to assume, therefore, that the photos were, in reality,
viewable either by (1) their Facebook friends, or
(2) by the public at large.
Even assuming that the photos in issue are visible only to the sanctioned
students’ Facebook friends, respondent STC can hardly be taken to
task for the perceived privacy invasion since it was the minors’
Facebook friends who showed the pictures to Tigol.
Clearly, the fault, if any, lies with the friends of the minors. Curiously
enough, however, neither the minors nor their parents imputed any
violation of privacy against the students who showed the images to
Escudero.
In sum, there can be no quibbling that the images in question, or to be
more precise, the photos of minor students scantily clad, are
personal in nature, likely to affect, if indiscriminately circulated, the
reputation of the minors enrolled in a conservative institution.
However, the records are bereft of any evidence, other than bare
assertions that they utilized Facebook’s privacy settings to make the
photos visible only to them or to a select few. Without proof that they
placed the photographs subject of this case within the ambit of their
protected zone of privacy, they cannot now insist that they have an
expectation of privacy with respect to the photographs in question.
The writ of habeas data cannot be granted without an actual case of
violation of the right to privacy.
NEAR v. MINNESOTA (HENRY) Trial Court ruled against the issuance of a temporary injuction, but did not
June 1, 1931 | Hughes, J. | Prior restraint allow petitioners to publish, circulate, or have in their possession any
editions
PETITIONER: JM Near, et al
RESPONDENTS: Minnesota

SUMMARY: The Minessota Gag Law is being assailed by the petitioners


for allowing the abatement of malicious, scandalous, and defamatory
articles published in newspapers, magazines or periodicals, as there was
a complaint that seeks enjoinment of their periodical, The Saturday
Press.

The trial court was in their favor, but the Minessotta SC ruled in the
contrary, saying that newspapers are capable of destroying public peace.
Upon appeal to the US SC, it is ruled that such prior restraint is
unconstitutional, and the government officials could not be trusted with
the responsibility of regulating speech before it even reaches the public.

DOCTRINE: Prior restraints are generally unconstitutional, such as when


they forbid the publication of malicious, scandalous, and defamatory
content.

FACTS:
Chapter 285 of the Session Laws of Minnesota for the year 1925 (aka
Minnesota Gag Law) provides for the abatement, as a public
nuisance, of a “malicious, scandalous, and defamatory
newspaper, magazine, or other periodical”. [Section 1(b)]
The County Attorney of Hennepin County brought about an action to
enjoin The Saturday Press (a periodical), published by petitioner
Near et al in the City of Minneapolis. It was alleged in the complaint
that the published and circulated editions of that periodical which
were “largely devoted to malicious, scandalous, and defamatory
articles”
The articles were being found to be such a public nuisance because of a
eries of viciously anti-semitic articles, to which in substance charged
that, a Jewish gangster was in control of gambling, bootlegging, and
racketeering in Minneapolis, and that law enforcement agencies
were not energetically performing their duties.
Most of the charges in the article were directed against the ff:
Chief of Police – for gross neglect of duty, illicit relations with
gangsters, and with participation in graft
County Attorney – with knowing the existing conditions and with
failure to take adequate measures to remedy them
Mayor – inefficiency and dereliction
One member of the grand jury – in sympathy with gangsters
Special grand jury and special prosecutor – demanded by The
Saturday Press to deal with the situation in general
of the periodical from Sept 1927 to Nov 1927.
163. Upon appeal to the Minessotta SC, the injunction was ruled to be
appropriate because a newspaper is capable of disturbing the public
peace and causing violence, which the state could prevet by exercising
its police powers.
Aggrieved, petitioners appealed to the same State SC, which once again
affirmed their previous ruling, for the order not being to broad as it
does not prohibit the defendants from running another newspaper
with a proper structure that supports public welfare. The assailed law
is deemed constitutional. Hence an appeal was made before the US
Supreme Court

ISSUE/s:
WoN the statute authorizing the abatement of The Saturday Press was
constitutional – NO

RULING: SC granted the petition and overturned the Minessotta SC ruling.

RATIO:
The perception and effect of the statute in substance is that public
authorities may bring the owner or publisher of a newspaper or
periodical before a judge upon a charge of cinductung a business of
publishing scandalous and defamatory matter.
Unless the owner or publisher ius able and disposed to bring competent
evidence to satisfy the judge that the charges are true and are
published with good motives and for justifiable ends, his news apper
or periodical is suppressed and further publication is made
punishable as a contempt. Such is the essence of censorship
In determining the extent of the constitutional protection, it has been
generally, if not universally, considered that it is the chief purpose of
the guaranty to prevent previous restraints of publication.
In the present case, US SC issued a strong prohibition against prior
restraints
or government censorship. This is basing on the law’s general
application, that government officials could not be trusted with the
responsibility of regulating speech before it even reaches the public.
190. A ban though, on prior restraints, was not an unbending rule,
exceptions are present: when a nation is at war, many things that
might be said in time of peace are such a hindrance to its effort that
their utterance will not be endured so long as men fight and that no
court could regard them as protected by any constitutional right.
191. No one would question but that government might prevent actual
obstruction to its recruiting serice or the publication of the sailing
dates of transports or the number and location of troops.
192. On similar grounds, the primary requirements of decency may be
enforced against obscene publications. The security of the
community life may be protected against incitements to acts of
violence and the overthrow by force of orderly government.
NEW YORK TIMES v. US (DANNAH) Black, Concurring: In seeking injunctions against these
June 30, 1971 | Per Curiam | Prior Restraint of Speech newspapers, the Executive Branch seems to have forgotten
the essential purpose of the First
PETITIONER: New York Times, Co.
RESPONDENTS: United States

SUMMARY: The New York Times and Washington Post, after months of
studying 47 volumes that came unauthorized in its possession, was
enjoined by the United States from publishing the contents of its
classified study entitled “History of US Decision-Making Process on
Vietnam Policy.

The Court of Appeals of the Columbia Circuit decided that it was


unconstitutional as it entailed a prior restraint of speech. The Supreme
Court, in a split decision, upheld the lower court’s decision, as seen in the
Concurring and Dissenting opinions. The most important part of the
Concurring opinion is that the Courts should not heed to the wants of the
Executive under the guise of the excuse of “national security”. If it
decides otherwise, then the purpose of the Bill of Rights will be curtailed.

DOCTRINE: Any system of prior restraint of expression bears a heavy


presumption against its constitutional validity. The Government thus
carries a heavy burden of showing justification for the imposition of such
restraint.

FACTS:
The United States sought to enjoin the New York Times and Washington
Post from publishing the contents of a classified study entitled
“History of US Decision-Making Process on Vietnam Policy”

ISSUE/s:
WoN United States may employ prior restraint of speech in the case at hand
– NO

RULING: Judgment of CA for the Columbia Circuit affirmed. Order of the CA


for the Second Circuit reversed. Case remanded to the District Court for the
Southern District of New York.

RATIO:
Any system of prior restraint of expression bears a heavy presumption
against its constitutional validity. The Government thus carries a
heavy burden of showing justification for the imposition of such
restraint.
The District Court for the Southern Distrcit of New York, and for the
District of Columbia, and the CA for the Columbia Circuit, all held that
the Govrnment had not met that burden. The Court agrees.
Amendment. The Bill of Rights was created to safeguard certain basic
freedoms in order to prevent the government from interpreting that they are
permitted to curtail freedom of religion, press, assembly and speech.
The people shall not be deprived or abridged of their right to speak, to write or
publish their sentiments, and the freedom of the press, as one of the great
bulwarks of liberty, shall be inviolable.
Both the history and language of the First Amendment support the view that the
press must be left free to publish news, whatever the source, without
censorship, injunctions or prior restraints.
Only a free and unrestrained press can effectively expose deception in
government.
In its brief, the government argues that in spite of the First Amendment, the
authority of the Executive Department to protect the nation against
publication of information whose disclosure would endanger the national
security stems from two interrelated sources:
The constitutional power of the President over the conduct of foreign
affairs
His authority as Commander-in-Chief
It’s as if the Courts are asked to hold that, despite the First Amendment's
emphatic command, the Executive Branch, the Congress, and the Judiciary
can make laws enjoining publication of current news and abridging freedom
of the press in the name of "national security."
To find that the President has "inherent power" to halt the publication of news by
resort to the courts would wipe out the First Amendment and destroy the
fundamental liberty and security of the very people the Government hopes to
make "secure."
Douglas, Concurring: There is no statute barring the publication by the press of
the material which the Times and the Post seek to use.
In Organization for a Better Austin v. Keefe it was held that any prior restraint
on expression comes to this Court with a “heavy presumption” against its
constitutional validity.
My understanding: The provisions on espionage do not apply to publications and
the press. The suggested provision making it applicable to the press was
struckt down—the Bill of Rights prevailed.
The Government says that it has inherent powers to go into court and obtain an
injunction to protect the national interest, which, in this case, is alleged to be
national security.
However, Near v. Minnesota repudiated that expansive doctrine. It is common
knowledge that that the First Amendment was adopted against the
widespread use of the common law of seditious libel to punish the
dissemination of material that is embarrassing to the powers-that-be.
Brennan, Concurring: The entire thrust of the Government's claim throughout
these cases has been that publication of the material sought to be enjoined
"could," or "might," or "may" prejudice the national interest in various ways.
But the First Amendment tolerates absolutely no prior judicial restraints of
the press predicated upon surmise or conjecture that untoward During all of this time, the Times, presumably in its capacity as
consequences may result. trustee of the public's "right to know," has held up publication for
The First Amendment’s ban on prior judicial restraint may be overridden, purposes it considered proper, and thus public knowledge was
but such cases may arise only when the Nation “is at war,” during delayed.
which times no one would question but that a government might But why should the United States Government, from whom this
prevent actual obstruction to its recruiting service or the publication information was illegally acquired by someone, along with all the
of the sailing dates of transports or the number and location of counsel, trial judges,
troops.
However, in neither of these actions has the Government presented or
even alleged that publication of items from or based upon the
material at issue would cause the happening of an event of that
nature.
The chief purpose of the First Amendemnt’s guarany is to prevent
previous restraints upon publication. Thus, only governmental
allegation and proof that publication must be inevitably, directly and
immediately cause the occurrence of an event kindred to imperiling
the safety of a transport already at sea can support even the
issuance of an interim restraining order.
Stewart, Concurring: The Executive is endowed with enormous power
in the two related areas of national defense and international
relations.
In the absence of the governmental checks and balances present in
other areas of our national life, the only effective restraint upon
executive policy and power in the areas of national defense and
international affairs may lie in an enlightened citizenry -- in an
informed and critical public opinion which alone can here protect the
values of democratic government.
Without an informed and free press, there cannot be an
enlightented people.
In the case at hand, the Court is asked to perform a function the
Constitution gave to the Executive, not to the Judiciary. The Court is
asked, quite simply, to prevent the publication by two newspapers of
material that the Executive Branch insists should not, in the national
interest, be published.
I am convinced that the Executive is correct with respect to some of the
documents involved. But I cannot say that disclosure of any of them
will surely result in direct, immediate, and irreparable damage to our
Nation or its people.
Burger, Dissent: The newspapers make a derivative claim under the
First Amendment; they denominate this right as the public "right to
know"; by implication, the Times asserts a sole trusteeship of that
right by virtue of its journalistic "scoop." The right is asserted as an
absolute, but of course, the First Amendment right itself is not
an absolute.
It is not disputed that the Times has had unauthorized possession of the
documents for three to four months, during which it has had its
expert analysts studying them, presumably digesting them and
preparing the material for publication.
and appellate judges be placed under needless pressure? After these
months of deferral, the alleged "right to know" has somehow and
suddenly become a right that must be vindicated instanter.
It is hardly believable that a newspaper long regarded as a great institution in
American life would fail to perform one of the basic and simple duties of
every citizen with respect to the discovery or possession of stolen
property or secret government documents. That duty, I had thought --
perhaps naively -- was to report forthwith, to responsible public officers.
Harlan, Dissent: The scope of the judicial function in passing upon the
activities of the Executive Branch of the Government in the field of
foreign affairs is very narrowly restricted. This view is, I think, dictated by
the concept of separation of powers upon which our constitutional
system rests.
The nature of foreign negotiations requires caution, and their success must
often depend on secrecy; and even when brought to a conclusion, a full
disclosure of all the measures, demands, or eventual concessions which
may have been proposed or contemplated would be extremely impolitic;
for this might have a pernicious influence on future negotiations, or
produce immediate inconveniences, perhaps danger and mischief, in
relation to other powers.
In the performance of its duty to protect the values of the First Amendment
against political pressures, the judiciary must review the initial Executive
determination to the point of satisfying itself that the subject matter of the
dispute does lie within the proper compass of the President's foreign
relations power.
Moreover, the judiciary may properly insist that the determination that disclosure
of the subject matter would irreparably impair the national security be made
by the head of the Executive Department concerned. However, the very
nature of executive decisions as to foreign policy is political, not judicial.
Blackmun, Dissent: The New York Times clandestinely devoted a period of
three months to examining the 47 volumes that came into its
unauthorized possession.
Once it had begun publication, the case before us emerged. Seemingly, once
publication started, the material could not be made public fast enough.
Seemingly, from then on, every deferral or delay, by restraint or
otherwise, was abhorrent, and was to be deemed violative of the First
Amendment and of the public's "right immediately to know."
FREEDMAN v. MARYLAND (IYA) Under the statuette he exhibitor is required to submit the film to the
November 19, 1964 | Brennan, J. | Freedom of Speech Board for examination, but no time limit is imposed for
completion of Board action.
PETITIONER: Freedman
RESPONDENTS: State of Maryland

SUMMARY: Freedman was convicted of violating the Maryland Statue


on Censorship which requires the submission of the picture for approval
by a Board of Censors before it may be exhibited. He questions the
constitutionality of 2 of the said statute as it limits the protected right to
freedom of expression.

The SC ruled that 2 of the statue is unconstitutional as it grants to the


board unlimited discretion on the period or time upon which they may
decide on a submission before them.

DOCTRINE: The Maryland scheme fails to provide adequate safeguards


against undue inhibition of protected expression, and this renders the 2
requirement of prior submission of films to the Board an invalid previous
restraint.

FACTS:
Freedman exhibited the film “Revenge at Daybreak” at his theatre in
Baltimore without first submitting the picture to the State Board of
Censors as required by the censorship statute of the State of
Maryland.
The lower courts convicted him of violation thereof.
He now challenges the constitutionality of the Maryland Motion Picture
Censorship Statute, specifically the requirements under 2 (section
ata ‘to, pero 2 lang talaga nakasulat sa original)

ISSUE/s:
WON the Maryland Statute is unconstitutional for unduly suppressing
protected expression. - YES

RULING: Decision is REVERSED. 2 of the Maryland Motion Picture


Censorship Statue is UNCONSTITUTIONAL.

RATIO:
The reliance on Times Film by the lower court is misplaced. In Times
Film v City of Chicago, the court upheld the requirement of
submission of motion pictures.
Unlike the petitioner in Times Film, Freedman argues that 2 constitutes
an invalid prior restraint because, in the context of the remainder of
the statute, it presents a danger of unduly suppressing protected
expression.
There is no statutory provision for judicial participation in the procedure which
bars a film, nor even assurance of prompt judicial review. Risk of delay is
built into the Maryland procedure.
In the area of freedom of expression it is well established that one has standing
to challenge a statute on the ground that it delegates overly broad
licensing discretion to an administrative office, whether or not his
conduct could be proscribed by a properly drawn statute, and whether or not
he applied for a license.
Because the statute operates in a manner which judicial review may be too little
and too late, the Maryland statue lacks sufficient safeguards for confining the
censor’s action to judicially determined constitutional limits, and therefore
contains a vice as a statute delegating excessive administrative discretion.
Therefore, the procedure must also assure a prompt final judicial decision, to
minimize the deterrent effect of an interim and possibly erroneous denial of a
license.
The Court holds, therefore, that Freedman's conviction must be reversed. The
Maryland scheme fails to provide adequate safeguards against undue
inhibition of protected expression, and this renders the 2 requirement
of prior submission of films to the Board an invalid previous restraint.
The Court does not mean to lay down rigid time limits or procedures, but to
suggest considerations in drafting legislation to accord with local exhibition
practices, and in doing so to avoid the potentially chilling effect of the
Maryland statute on protected expression.
AYER PRODUCTIONS v. CAPULONG (DANNAH) film, no reference whatsoever (whether written, verbal or visual)
April 29, 1988 | Feliciano, J. | Freedom of Expression / Right to Privacy should not be made to [him] or any member of his family, much less
to any matter purely personal to them.”
McElroy acceded to this demand and Enrile was deleted from the movie
PETITIONER: Ayer Productions & McElroy Film Productions, Hal McElroy
script; they also proceeded to film said picture.
RESPONDENTS: Hon. Capulong & Juan Ponce Enrile Enrile then filed a complaint with application of TRO with the RTC of
Makati to enjoin McElroy from producing the movie. The complaint
SUMMARY: McElroy wanted to make a docu-drama 6 hour mini series
alleged that the production without his consent and over his objection
entitled “The Four Day Revolution”, which would show the struggle of the constitutes a violation of his right to privacy. The court then issued
Filipinos at EDSA. A local producer adviced him to seek the consent of the Order.
FVR and Enrile, as they played major roles in said Revolution. Ramos McElroy filed a motion to dismiss contending that the film would not
gave his consent, but Enrile declined. Hence, McElroy continued to film involve the private life of Enrile nor his family, and that the injunction
the picture but they deleted Enrile from the script. would amount to a prior restraint on their right of free expression.
Moreover, they averred a lack of cause of action because the film had
Enrile filed a TRO and injunction with the RTC of Makati, claiming that the
not yet been completed.
production was without his consent and even over his objection. The RTC
Petitioners McElroy et al. then came to the Court by Petition for Certiorari
granted said TRO and injunction, which led McElroy to file for a Petition for
with an urgent prayer for Preliminary Injunction. The Court granted a
Certiorari with the Court. Before deciding, the Court issued a TRO on the
limited TRO partially enjoining the implementation of the lower court’s
Order allowing McElroy to continue filming. Then, it decided on the case
Order, and allowed McElroy to resume producing and filming those
stating that Enrile, being a public figure, has a narrower right to privacy than portions which do not include Enrile or his family.
an ordinary citizen. Moreover, since the film was of national (even
international) interest, and must be regarded as having passed into the public ISSUE/s:
domain and as an appropriate subject for speech and expression and WoN the film entails a valid exercise of freedom of speech and of expression
coverage by any form of mass media. – YES
WoN the film would constitute an unlawful intrusion into the privacy of
DOCTRINE: The right of privacy, or “the right to be let alone,” like the
Enrile – NO
right of free expression, is not an absolute right. A limited intrusion into a
person’s privacy has long been regarded as permissible where that RULING: Writ of Preliminary Injunction set aside. TRO modified and made
person is a public figure and the information sought to be elicited from
permanent.
him or to be published about him constitute matters of a public character.
RATIO:
Freedom of speech and expression includes the freedom to film and
FACTS:
produce motion pictures and to exhibit such motion pictures in
Petitioner Hal McElroy, an Australian filmmaker, and his production
theaters or to diffuse them through television. Motion pictures
company petitioner Ayer Productions, wanted to make a docu-drama
mini series entitled “The Four Day Revolution”. constitute a principal medium of mass communication for information,
This was endorsed by the MTRCB and would revolve the historical education and entertainment.
peaceful struggle of the Filipinos at EDSA. Local movie producer The circumstance that the production of motion picture films is a
Juban suggested they consult with the appropriate government commercial activity expected to yield monetary profit is not a
agencies and with Fidel V. Ramos and Enrile, who had played major disqualification of availing of freedom of speech and of expression.
roles in the events proposed to be filmed. To exclude commercially owned and operated media from the exercise of
Gen. Ramos gave his approval, but Juan Ponce Enrile declined. He
constitutionally protected freedom of speech and of expression can only
stated that:
“He would not and will not approve the use, appropriation, reproduction result in the drastic contraction of such constitutional liberties in our
and/or exhibition of his name, or picture, or that of any member of his country.
family in any cinema or television production, film or other medium for The counter-balancing claim of Enrile is to a right of privacy. It was
advertising or commercial exploitation and further advised petitioners
that in the production, airing, showing, distribution or exhibition of said
demonstrated by Dean Cortes that our law, constitutional and
or similar
statutory, does include a right of privacy. It is left to caselaw,
however, to mark the precise scope and content of this right.
The right of privacy, or “the right to be let alone,” like the right of be imposed on a person claiming to exercise such Constitutional
free expression, is not an absolute right. A limited intrusion into freedoms.
a person’s privacy has long been regarded as permissible
where that person is a public figure and the information sought
to be elicited from him or to be published about him constitute
matters of a public character.
The right of privacy cannot be invoked to resist publication and
dissemination of matters of public interest. The interest sought to be
protected by the right of privacy is the right to be free from
“unwarranted publicity, from the wrongful publicizing of the private
affairs and activities of an individual which are outside the realm of
legitimate public concern."
Lagunzad v. Vda de Gonzales, in which Enrile relies on heavily:
Involved a suit for enforcing a licensing agreement between a producer
and the widow and family of late Moises Padilla as licensors.
They wanted to portray the life of Padilla, a mayoralty candidate.
In affirming the judgment of the lower court enforcing the licensing
agreement, the Court said that although petitioner had purchased
the rights to the book entitled The Moises Padilla Story, that did not
dispense with the need for prior consent of the deceased heirs.
Being a public figure ipso facto does not automatically destroy in toto a
person’s right to privacy. The right to invade a person’s privacy to
disseminate public information does not extend to a fictional or
novelized representation of a person, no matter how public a figure
he or she may be.
The Court also stated that “The right of freedom of expression, indeed,
occupies a preferred position in the ‘hierarchy of civil liberties’. It is
not, however, without limitations”
Two tests:
The Clear and Present Danger Rule
The Balancing of Interests Test
The production and filming by petitioners of the projected motion picture
does not, in the circumstances of this case, constitute an unlawful
intrusion upon Enrile’s right to privacy.
What is involved is a prior and direct restraint on the part of Judge
Capulong upon the exercise of speech and expression of McElroy.
He has restrained them from filming and producing. It is important to note
that in Lagunzad, there was no prior restrain of any kind imposed
upon the movie producer who in fact completed and exhibited the
film biography of Moises Padilla.
Because of the preferred character of the constitutional rights of freedom
of speech and of expression, a weighty presumption of invalidity
vitiates measures of prior restraint upon the exercise of such
freedoms. However, this does not mean that no liability may lawfully
The motion was not yet complete and has not yet been exhibited to any
audience. Neither Judge Capulong nor Enrile knew what the completed
film would look like.
There was, in other words, no “clear and present danger” of any
violation of any right to privacy that Enrile could lawfully assert.
Moreover, the subject matter of the film is one of public interest and concern.
Indeed, it is even of international interest. It relates to a highly cricial
stage in the history of our country and must be regarded as having
passed into the public domain and as an appropriate subject for speech
and expression and coverage by any form of mass media.
Moreover, the subject matter does not relate to the individual life and
certainly not to the private life of private respondent Ponce Enrile; unlike
in the case of Lagunzad which concerned the life of Padilla and his
family.
The extent of the intrustion upon Enrile’s life is liimited in character. It may be
generally described as such intrusion as is reasonably necessary to keep
that film a truthful historical account.
Enrile is what Prosser and Keeton have referred to as a public figure
A person who, by his accomplishments, fame, mode of living, or by adopting
a profession or calling which gives the public a legitimate interest in his
doings, affairs, characters, has become a public personage.
It includes public officers, war heroes, even ordinary soldiers.
In short, it incluedes anyone who has arrived at a position where public
attention is focused upon him as a person.
Such public figures were held to have lost, to some extent at least, their right
to privacy.
Three reasons:
0 They had sought publicity and had consented to it; they could
not complain
1 Their personalities and affairs had already become public;
could no longer be regarded as their own private business
2 The press had a privilege, under the Constitution, to inform the
public about those who have becom legitimate matters of
public interest
The right of privacy of a “public figure” is necessarily narrower than that of an
ordinary citizen. Enrile has not retired into the seclusion of simple private
citizenship. He continutes to be a public figure because he is an
incumbent senator.
Honasan also filed a complaint which was similar to Enrile’s, the Court also
dismissed said complaint. He is a fugitive and deemed to have forfeited
any right he might have had to protect his privacy through court
processes.
EASTERN BROADCASTING CORPORATION v. DANS, JR. 1. The cardinal primary requirements in
(ARIELLE) administrative
July 19, 1985 | Gutierrez, Jr., J. | Freedom of Speech

PETITIONER: Eastern Broadcasting Corportation


RESPONDENTS: Dans, Jr.

SUMMARY: Petitioner’s radio station was summarily


closed on grounds of national security. Petitioner claims
that the summary closure of the radio station violated the
right to free speech.
Records show that respondents’ general charge of “inciting
people to commit acts of sedition” arose from the
petitioner’s shift towards what it stated was the coverage of
public events and the airing of programs geared towards
public affairs.

DOCTRINE: All forms of media, whether print or broadcast,


are entitled to the broad protection of the freedom of
speech and expression clause.

FACTS:
The petition was filed to compel the respondents to allow the
re-opening of Radio Station DYRE which had been
summarily closed on grounds of national security.
Petitioner claims that the summary closure of the radio
station violated the right to free speech. Records show
that respondents’ general charge of “inciting people to
commit acts of sedition” arose from the petitioner’s shift
towards what it stated was the coverage of public events
and the airing of programs geared towards public affairs.
Before the Court could render a decision, the petitioner suddenly filed
a motion to withdraw or dismiss the petition because it already
sold its radio broadcasting station in favor of Manuel B.
Pastrana as well as its rights in the radio station DYRE
including its right to operate and its equipment.

RULING: The case is declared moot and academic, but for the
guidance of inferior courts and administrative tribunals, the Court
issued some guidelines.

RATIO:
proceedings laid down in Ang Tibay should be followed before a
broadcast station may be closed. All forms of media, whether
print or broadcast, are entitled to the broad protection of the
freedom of speech and expression clause.
The test for limitations on freedom of expression continues to be
the clear and present danger rule - that words are used in such
circumstances and are of such a nature as to create a clear and
present danger that they will bring about the substantive evils that the
lawmaker has a right to prevent.
The clear and present danger test, however, does not lend itself
to a simplistic and all-embracing interpretation applicable to all
utterances in all forums.
Broadcasting has to be licensed. A broadcast corporation cannot
simply appropriate a certain frequency without regard for government
regulation or for the rights of others.
All forms of communication are entitled to the broad
protection of the freedom of expression clause. However,
the freedom of television and radio broadcasting is somewhat
lesser in scope than the freedom given to newspaper and print
media. The American Court in Federal Communications
Commission v. Pacifica Foundation explained why radio
broadcasting receives the most limited protection from the free
expression clause:

a. Broadcast media have established a uniquely pervasive


presence in the lives of all citizens. Material presented
confronts the citizen, not only in public, but in the privacy of
his home.
b. Broadcasting is uniquely accessible to children. Books and
movies may be prohibited from making certain material
available to children, but not to radio or television, where the
listener or viewer is constantly tuning in and out.

b. The broadcast media have also established a uniquely


pervasive presence in the lives of all Filipinos. Newspapers and
current books are found only in metropolitan areas and in the
poblaciones of municipalities accessible to fast and regular
transportation. The transistor radio is found everywhere. The
television set is also becoming universal.
“If there is any principle of the Constitution that more
The clear and present danger test, therefore, must take imperatively calls for attachment than any other it
the particular circumstances of broadcast media into is the
account. The supervision of radio stations whether by
government or through self-regulation by the industry
itself calls for thoughtful, intelligent and sophisticated
handling.
The government has a right to be protected against
broadcasts which incite the listeners to violently
overthrow it. Radio and television may not be used to
organize a rebellion or to signal the start of widespread
uprising. At the same time, the people have a right to be
informed.
The freedom to comment on public affairs is essential to the
vitality of a representative democracy. Broadcast stations
deserve the special protection given to all forms of media
by the due process and freedom of expression clauses of
the Constitution.
Fernando, Concurring
Agreed that even if the case was declared moot and
academic, a statement of the governing principle is
still appropriate in the resolution of dismissal for the
guidance not only of the parties but of others similarly
situated.
Teehankee, Concurring
a. While withdrawal of the petition for loss of interest on
petitioner’s part may be granted, still the Court should
unequivocally set forth the guiding and controlling
precepts or doctrines in pursuance of its symbolic
function of educating bench and bar.
Teehankee, Concurring (before the petitioner filed a motion
to withdraw the case)
The basic standard for restricting or punishing the
exercise of these preferred freedoms of speech is
the clear and present danger test - danger of a
serious and imminent evil sought to be prevented.
that the summary closure in October, 1980 of
petitioner’s radio station attended by complete
absence of any hearing before or after violated its
constitutional rights and must therefore be declared
null and void.
principle of free thought - not free thought for those who
agree with us but freedom for the thought that we hate”
and that freedom of expression is a “preferred right” and
therefore stands on a higher level than substantive
economic or other liberties. This is because freedom of
thought and speech is the indispensable condition of
nearly every other form of freedom.
Political discussion even among those opposed to the
present administration is within the protective clause
of freedom of speech and expression. The same
cannot be construed as subversive activities per se or as
evidence of membership in a subversive organization.
Abad Santos, Concurring
The closure of the petitioner’s radio station on grounds of
national security without elaboration of the grounds and
without hearing deserves to be condemned in no uncertain
terms for it is manifest that due process was not observed.
ALEXANDER v. UNITED STATES (IYA) RULING: RICO forfeiture provisions DO NOT constitute prior restraint on
June 28, 1993 | Rehnquist, CJ. | Freedom of Speech speech.
Case is REMANDED back for imposition of penalty.
PETITIONER: Alexander
RESPONDENTS: United States

SUMMARY: Alexander is engaged in the business of dealing in sexually


explicit materials. Such was declared violative of obscenity laws and the
Racketeer Influenced and Corrupt Organizations Act. The District Court
apart from imposing a prison term and fine, also ordered for the forfeiture
of the businesses. The CA affirmed such order.

Alexander questions the validity of the forfeiture provisions of RICO as


they constitute prior restraint on speech and are overbroad.

The SC rules that the assets in question were ordered forfeited not
because they were believed to be obscene, but because they were
directly related to petitioner’s past violations.

DOCTRINE: The Maryland scheme fails to provide adequate safeguards


against undue inhibition of protected expression, and this renders the 2
requirement of prior submission of films to the Board an invalid previous
restraint.

FACTS:
After a full criminal trial, Alexander, the owner of numerous businesses
dealing in sexually explicit materials, was convicted of, inter alia,
violating federal obscenity laws and the Racketeer Influenced and
Corrupt Organizations Act (RICO).
The obscenity convictions, based on a finding that seven items sold at
several stores were obscene, were the predicates for his RICO
convictions.
In addition to imposing a prison term and fine, the District Court ordered
petitioner, as punishment for the RICO violations, to forfeit his
businesses and almost $9 million acquired through racketeering
activity.
In affirming the forfeiture order, the Court of Appeals rejected petitioner’s
arguments that RICO’s forfeiture provisions constitute a prior
restraint on speech and are overbroad.
The court also held that the forfeiture did not violate the Eighth Amendment,
concluding that proportionality review is not required of any sentence
less than life imprisonment without the possibility of parole. It did not
consider
whether the forfeiture was disproportionate or “excessive.”
ISSUE/s:
WON RICO’s forfeiture provisions constituted prior restraint on speech - NO
RATIO:
To accept petitioner’s argument would obliterate the distinction between
prior restraints and subsequent punishments.
Prior restraint is used “to describe administrative and judicial orders
forbidding certain communications when issued in advance of the
time that such communications are to occur.”
RICO forfeiture order in this case does not forbid petitioner to
engage in any expressive activities in the future, nor does it
require him to obtain prior approval for any expressive
activities.
It only deprives him of specific assets that were found to be related to his
previous violations.
The assets in question were ordered forfeited not because they
were believed to be obscene, but because they were directly
related to petitioner’s past violations.
Nor were the assets in question ordered forfeited without the requisite of
procedural safeguards. The seizure was not premature, because the
Government established beyond reasonable doubt the basis for the
forfeiture which was the full criminal trial conducted.
The RICO forfeiture order was not a prior restraint on speech, but a
punishment for past criminal conduct.
Alexander’s real complaint is not that the RICO statute is overbroad, but
that applying RICO’s forfeiture provisions to businesses dealing in
expressive materials may have an improper “chilling” effect on free
expression by deterring others from engaging in protected speech.
Although the forfeiture provision is not violative of the freedom of speech
or expression, the CA should have considered the penalty imposed
to be excessive.
The case is remanded back to the CA for further proceedings on the
penalty.
NEWSOUNDS BROADCASTING v. DY (ELIEL) City, Isabela. The property is owned by CBS Development
April 2, 2009 | Tinga, J. | Freedom of Speech and Expression Corporation (CDC), an affiliate

PETITIONER: Newsounds Broadcasting Network, Inc. and Consolidated


Broadcasting System, Inc.
RESPONDENTS: Hon. Ceasar G. Dy, Felicisimo Meer, Bagnos Maimo,
Racma Fernandez-Garcia and the City of Cauayan

SUMMARY: Newsounds relocated its property in Cauayan City, Isabela.


Upon their relocation the land on which the property situated was
classified as commercial. In the years 1997-2001, Newsounds have been
operating and paying what is due. However, beginning 2002 when
Newsounds was applying for business permits, they were required to
submit DAR certifications that initially were not required. This lasted until
2004; because they had no mayor’s permit, they were not able to
operate. This resulted to the closure of their property. Newsounds filed
with the COMELEC a preliminary injunction of the closure order and
invoked the OEC, which prohibits the closing of radio stations during
election period. Mayor Dy countered and issued instead a close order.
Hence the petition.

Newsounds invoked that their constitutional rights to free speech and free
press were violated. The SC held that the act of the mayor did in fact
infring on their constitutional rights. (1) Although the freedom of speech
and press may be regulated, there should be a clear and present danger
of substantive evil; none in this case; (2) the act of requiring a DAR
certification was not one of those initial requirements to secure a
business permits, in such a way they went beyond the law and its
capacity; (3) the land has already been established even prior to 2002
that it is commercial, no DAR certification was ever required; and (4)
clearly, there was circumstantial evidence to associate the interest of Dy
when Newsounds admitted that they were actively aggressive in
exposing widespread election irregularities.

DOCTRINE: There is to be no previous restraint on the communication of


views or subsequent liability whether in libel suits, prosecution for
sedition, or action for damages or contempt proceedings unless there be
a clear and present danger of substantive evil that Congress has a right
to prevent.

FACTS:
Bombo Radyo Philippines operates several radio stations under the AM
and FM band throughout the Philippines. These stations are
operated by corporations organized and incorporated by Bombo
Radyo, particularly Newsounds Broadcasting Network, Inc. and
Consolidated Broadcasting System, Inc.
Newsounds commenced relocation of tis broadcasting stations,
management office and transmitters on property located in Cauayan
corporation under the Bombo Radyo network which holds titte over
the prorpeties ued by Bombo Radyo sations throughout the country.
The Housing and Land Use Regulatory Board (HLURB) issued a Zoning
Decision certifying the property as commericla. That same day, the
Office of the Municipal Planning and Development Coordinator
(OMPDC) of Cauayan affirmed that the commercial structure to be
consturcte dby CDC conformed to local zoning regulations, noting as
well that the location “is classified as a Commercial are.”
A building was consequently erected on the property and therefrom,
DNZC and Star FM operated as radio stations. Both stations
successfully secured all necessary operating documents, including
mayor’s permits from 1997 to 2001. During that period, CDC paid
real property taxes on the property based on the classification of the
land as commercial.
All that changed beginning in 2002. When Newsounds applied for the
renewal of the mayor’s permit, they were required to submit “either
an approved land conversion papers from the Department of
Agrarian Reform (DAR) showing that the property was converted
from prime agricultural land ot commercial land, or an approved
resolution from the Sangguniang Bayan or Sangguniang
Panglungsod authorizing the re-classification of the proerptuy from
agricultural to commercial land.”
Due to this refusal by Maximo – City Zoning Administrator – Newsounds
were unable to secure a mayor’s permit. Newsounds applied for a
Petition for Mandamus with the RTC, but was denied for being moot
and academic.
In the meantime, Newsounds sought to obtain from the DAR a formal
recognition of the conversion of the CDC property form agricultural to
commercial.
When they applied for the 2003 mayor’s permit, attaching the DAR order,
Acting Administrator Meer claimed that such order is spurious. Thus,
they were not granted the permit.
Newsounds again applied for the 2004 mayor’s prmit, with the DAR
Order attached to the same. Meer claimed that no record existed.
After which they arrived at the property and closed the radio stations.
Newsounds filed a petition with the COMELEC seeking reinforcement of
the Omnibus Election Code, which prohibited the closure of radio
statins during the then-pendency of the election period.
Within hours, Mayor Dy issued a Closure Order stating therein that since
Newsounds did not have the requisite permits, the status quo mean
that the stations were not in fact allowed to operate.
Hence this petition.

ISSUE/s:
WoN the application for Mayor’s permit violated their free speech – YES

RULING: WHEREFORE, the petitions are GRANTED. Immediately issuing


to Newsounds zoning clerances and mayor’s permits for 2004.
RATIO: There is a long-standing tradition of special judicial solicitude for free
Free speech and free press may be identified with the liberty to speech, meaning that governmental action directed at exression
discuss publicly and truthfully any matter of public interest must satisfy a greater
without censorship and punishment.
There is to be no previous restraint on the communication of views
or subsequent liability whether in libel suits, prosecution for
sedition, or action for damages or contempt proceedings unless
there be a clear and present danger of substantive evil that
Congress has a right to prevent.
They admit that in 2001, Bombo Radyo “was aggressive in exposing the
widespread election irregularities in Isabela that appear to have
favored Dy and other members of the Dy political dynasty.”
The partisan component of this dispute will no doubt sway man
observers towards one opinion or the other, but not us. The comfort
offered by the constitutional shelter of free expression is netural as to
personality, affinity, ideology and popularity. The judges tasked to
enforce constitutional order are expected to rule accordingly from the
comfort of that neutral shelter.
The individual discomforts to particular people or enterprises
engendered by the exercise of the right, for which at times
remedies may be due, do not diminish the indispensable nature
of free expression to the democratic way of life.
Beginning in 2002, respondents in their official capacities have
taken actions, whatever may be the motive, that hae impeded
the ability of Newsounds to freely broadcast, if not broadcast at
all.
These actions have ranged from withholding permits to operate to
the physical closure of those stations under color of legal
authority.
While once Newsounds were able to broadcast freely, the weight of
government has since bore down upon them to silence their voices
on the airwaves.
Without taking into account any extenuating circumstances that may
favor the respondents, we can identify the bare acts of closing the
radio stations or preventing their operations as an act of prior
restraint against speech, expression or of the press.
Prior restraint refers to official governmental restrictions on the
press or other forms of expression in advance of actual
publication or dissemination.
While any system of prior restraint comes to court bearing a heavy
burden against its constitutionality, not all prior restraints on
speech are invalid.
And there is the fact that the mode of expression restrained in these
cases – broadcast – is not one which Newsounds are physically able
to accomplish without the regulatory arm of the government.
Expression in media such as print or the Internet is not burdened b such
requirements as congression franchises or administrative licenses
which bear upon broadcast media.
burden of justification than governmental action directed at most other forms
of behavior.
Content based law are generally treated as more suspect than content
neutral laws because of judicial concern with discrimination in the
regulation of expression.
Ostensibly, the act of an LGU requiring a business of proof that the property from
which it operates has been zoned for commercial use can be argued, when
applied to a radio station, as content-neutral since such a regulation would
presumably apply to any other radio station or business enterprise within the
LGU. However, the circumstances dictate that it is content-based.
Prior to 2002, Newsounds had not been frustrated in securing the various local
government requirements for the operation of their stations. It was only in the
beginning of 2002, after the election of Ceasar Dy as mayor of Cauayan, that the
local government started to impose these new requirements substantitation the
conversion of CDC’s property for commercial use.
Newsounds admit that during the 2001 elections, Bombo Radyo “was aggressive
in exposing the widespread election irregularities in Isabela that appear to
have favored Dy an other members of the Dy political dynasty.”
It also bears notice that the requiremnets required of Newsounds by the
Cauayan City government are frankly beyond the pale and not conventionally
adopted by local gvoernements throughout the Philippines.
All those circumstances lead to us to believe that the steps employed by
repsondents to ultimately shut down Newsounds radio station were ultimately
content-based.
A municipal license is essentially a governmental restriction upon private
rights and is valid only if absed upon an exercise by the municipality of
its police or taxing powers.
Nothin in national law exempts media entitites that also operate as
businesses such as newspapers and broadcast stations such as
Newsounds from being required to obtain permits or licenses from
local governments in the same manner as other businesses are
expected to do so.
While this may lead to some concern that requiring media entities to secure
licenses or permits from local government units infringes on the constitutional
right to a free press, we see no concern so long as such requirement has
been duly ordained through local legislation and content-neutral in character.
The power of the mayor to issue license and permits and suspend or revoke the
same must be exercised pursuan to law or ordinance.
INC v. CA (JP) TV program.
July 26 1996 | Puno, J. | Freedom of Speech The Second paragraph of the same decision stated however that INC is
also directed tor efrain from offending and attacking other existing
PETITIONER: Iglesia ni Cristo (INC) religions in showing the TV program.
RESPONDENTS: Honorable Court of Appeals, Board of Review for Petitioner moved for a reconsideration praying to remove the second
Moving Pictures and Television (The Board) paragraph of the decision and to perpetually enjoin the Board from
requiring petitioner to submit for review the tapes of its program.
SUMMARY: Petitioner Iglesia ni Cristo (INC) submitted its TV show “Ang The RTC granted such MR. The second paragraph of the decision was
Iglesia ni Cristo” to respondent Board. Respondent Board ruled such with deleted.
an X-rating not fit for public viewing as it is reported to criticize other The CA reversed the RTC. The respondent Board did not act with grave
religions. RTC ruled upon a motion for reconsideration that the show abuse of discretion when it denied permit for the exhibition on TV of
should be aired. CA reversed. the three series of “Ang Iglesia ni Cristo” on the ground that the
materials constitute an attack against another religion. It also found
SC reversed the CA and ruled that there is insufficient evidence the series “indecent, contrary to law and contrary to good customs.”
presented to render the show as contrary to law and morals. There is no INC appealed.
clear and present danger. Also, since the videotapes were not presented
in court, the declaration of such to be contrary to law interferes with the ISSUE/s:
right of speech and exercise of religion. WoN the Board gravely abused its discretion when it prohibited the airing
of petitioner’s religious program, series, for the reason that they
DOCTRINE: Any act that restrains speech is hobbled by the presumption constitute an attack against other religions and that they are
of invalidity and should be greeted with furrowed brows. indecent, contrary to law and good customs. - YES
Considering evidence not presented in court as contrary to law and RULING: CA decision is reversed as to its X-rating of the INC TV program.
morals suppresses petitioner’s freedom of speech and interferes with its
right to free exercise of religion. RATIO:
Freedom of religion has been accorded a preferred status by the framers
FACTS: of our fundamental laws, past and present.
Petitioner Iglesia ni Cristo (INC) is a duly organized religious organization The right to religious profession and worship has a two-fold aspect, viz.,
and has a television program entitled “Ang Iglesia ni Cristo” which freedom to believe and freedom to act on one’s beliefs. The first is
aired on Channel 2 every Saturday and on Channel 13 every
absolute as long as the belief is confined within the realm of thought.
Sunday.
The program presents and propagates petitioner’s religious beliefs, doctrines The second is subject to regulation where the belief is translated into
and practices oftentimes in comparative studies with other religions. external acts that affect the public welfare.
Petitioner INC submitted to the respondent Board (see Respondent) the Freedom to believe: The individual is free to believe (or disbelieve) as he
VTR tapes of its TV program. pleases concerning the hereafter. However absurd his beliefs may be
The Board classified the series as “X” or not for public viewing on the to others, even if they be hostile and heretical to the majority, he has
ground that they offend and constitute an attack against other full freedom to believe as he pleases.
religions which is expressly prohibited by law.
Freedom to act on one’s beliefs: But where the individual
They were reported to show criticisms against other religions. INC were
said to have presented literal interpretations of the Bible. They said externalizes his beliefs in acts or omissions that affect the
that the Catholic veneration of the Virgin Mary is not to be condoned public, his freedom to do so becomes subject to the authority of
as it is nowhere found in the Bible. the State. As great as this liberty may be, religious freedom, like
Petitioner INC alleged that the respondent Board acted with grave abuse all other rights guaranteed in the Constitution, can be enjoyed
of discretion in requiring petitioner to submit the VTR tapes of its TV only with a proper regard for the rights of others. The inherent
program nad in x-rating them.
The RTC issued the requested writ of preliminary injunction and ordered the police power can be exercised to prevent religious practices
Board to grant petititoner INC the necessary permit for all the series of inimical to society.
the
Accordingly, while one has full freedom to believe in Satan, he may not Such is void for it runs smack against the hoary doctrine that
offer the object of his piety a human sacrifice, as this would be administrative rules and regulations cannot expand the letter and
murder. spirit of the law they seek to enforce.
The SC thus reject petitioner’s postulate that its religious program is per Fourth. In x-rating the TV program of the petitioner, the respondents
se beyond review by the respondent Board. Its public broadcast on
TV of its religious program brings it out of the bosom of internal
belief.
Television is a medium that reaches even the eyes and ears of children.
The Court iterates the rule that the exercise of religious freedom can
be regulated by the State when it will bring about the clear and
present danger of some substantive evil which the State is duty
bound to prevent, i.e., serious detriment to the more overriding
interest of public health, public morals, or public welfare.
The SC reversed the ruling of the RTC.
First. Deeply ensconced in our fundamental law is its hostility
against all prior restraints on speech, including religious
speech. Hence, any act that restrains speech is hobbled by the
presumption of invalidity and should be greeted with furrowed
brows.
It is the burden of the respondent Board to overthrow this
presumption. If it fails to discharge this burden, its act of
censorship will be struck down. It failed in the case at bar.
Second, the videotapes were not viewed by the respondent court as
they were not presented as evidence. Yet they were considered
by the respondent court as indecent, contrary to law and good
customs, hence, can be prohibited from public viewing under
Section 3(c) of PD 1986. This ruling clearly suppresses
petitioner’s freedom of speech and interferes with its right to
free exercise of religion.
Under our constitutional scheme, it is not the task of the State to favor
any religion by protecting it against an attack by another religion.
In fine, respondent board cannot squelch the speech of petitioner
Iglesia ni Cristo simply because it attacks other religions, even
if said religion happens to be the most numerous church in our
country.
Third. The respondents cannot also rely on the ground “attacks against
another religion” in x-rating the religious program of petitioner. Even
a sideglance at Section 3 of PD No. 1986 will reveal that it is not
among the grounds to justify an order prohibiting the broadcast of
petitioner’s television program. The ground “attack against another
religion” was merely added by the respondent Board in its Rules
failed to apply the clear and present danger rule.
There is no showing whatsoever of the type of harm the tapes will bring
about especially the gravity and imminence of the threatened harm.
Prior restraint on speech, including religious speech, cannot be
justified by hypothetical fears but only by the showing of a
substantive and imminent evil which has taken the life of a reality
already on ground.
Given the specific content of the speech, it is not unreasonable to assume
that the respondent Board, with its expertise, can determine whether its
sulphur will bring about the substantive evil feared by the law.
Padilla, Concurring and Dissenting Opinion:
Kapunan, Concurring and Dissenting Opinon: The majority’s ruling
constitutes a threat to constitutionally protected speech and expression
and supplants a judicial standard for determining constitutionally
protected speech and expression with the censors standard.
The heavy burden on the imposition of prior restraints is shifted away from
the state by imposing upon the exhibitor the obligation of proving that the
religious programs fall within the realm of protected expression.
This leaves the exhibitor with only two unwanted options: either 1) he himself
deletes the portions which he anticipates the Board might possibly object
to prior to submission to that body and thereby obtains the censor’s nod,
or 2) submits the Video tapes in their entirety and risks disapproval or
deletion, in which case he may go to court and show that the Video tapes
contain constitutionally protected speech and expression.
In the first situation, the message loses its essence and substance. The
second scenario may entail tremendous amount of money, time and
effort in a prolonged litigation. Either case constitutes grievous
assault on the freedom of speech and religion.
Panganiban, Separate Opinion: In no way can this Court could authorize a
suppression of a species of the freedom of speech on the say-so of
anyone· not even of the MTRCB. Paraphrasing People vs. Fernando, the
disputable presumption (which is of statutory origin) that official duties
have been regularly performed must yield to the constitutionally
enshrined freedoms of expression and of religion.
DAVID v. ARROYO (HENRY) said event. A bomb was found and detonated at the PMA parade
May 3, 2006 | Sandoval-Gutierrez, J. | Right to peaceably assemble ground.

PETITIONER: Prof. Randolf S. David, et al.


RESPONDENTS: Gloria Macapagal-Arroyo

SUMMARY: PP 1017 was promulgated by PGMA, declaring a state of


national emergency. This lead to warrantless arrests all over the country
by policemen, in order to prevent lawless violence and suppress the acts
of terrorism in the country. Petitioner David et al were arrested in the
name of PP 1017 when they were arrested during the dispersal of the
rallyists.

They are assailing the validity of such enactment of PGMA, and they
claim that it violates their constitutional guarantee of freedom of
assembly. The SC ruled in their favor, that assembly is a crime itself, and
there must be clear and present danger first before an arrest be made in
such assembly.

DOCTRINE: As in the case of freedom of expression, this right is not to


be limited, much less denied, except on a showing of a clear and present
danger of a substantive evil that Congress has a right to prevent. In other
words, like other rights embraced in the freedom of expression, the right
to assemble is not subject to previous restraint or censorship.

FACTS:
As the nation celebrated the 20th Anniv of the EDSA 1, PGMA issued PP
1017, declaring a state of national emergency.
As basis, she cited the fact that over the past months, elements in the
political opposition have conspired with authoritarians of the extreme
Left and the extreme Right, who are now in a tactical alliance and
engaged in a concerted and systematic conspiracy, over a broad
front, to bring down the PGMA Administration.
On the same day, PGMA Issued G.O. No. 5, implementing PP 1017,
where she called the AFP and the PNP to prevent and suppress acts
of terrorism and lawless violernce in the country.
A week after, PGMA lifted PP 1017 by issuing Proclamation No. 1021.
2006, members of the Magdalo Group indicted in the Oakwood mutiny,
escaped their detention cell in Fort Bonifacio. And in a public statement
they called out upon people to show and proclaim their displeasure at
the sam regime. They urged the people to demonstraite their disgust,
not only by going to the streets in protest, but also by wearing red bands
on their left arms.
February of that same year, the authorities got hold of a document
entitled “Oplan Hackle I” which detailed plans for bomings and
attacks during the PMA Alumni Homecoming. The plot was to
assassinate select targets including the cabinet members and PGMA
herself. Upon advice of her security, PGMA decided not to attend the
That same month Lt. San Juan was recaptured (one of those who It may not be conditioned upon the prior issuance of a permit or authorization
escaped the cell in Fort) and found in his possession are two from the government authorities, except, of course, if the assembly is
diskettes containing minutes of meetings between NPA members, intended to be held in a public place, a permit for the use of such place,
among others. Prior to his arrest, Lt. San Juan announced through and
DZRH that the “Magdalo’s D-Day would be on Feb 24, 2006, the 20 th
Anniversary of EDSA I”.
Insofar as the petitioners are concerned, the OP cancelled all programs
and activities related to the 20th anniversary of EDSA 1, and revoked
permits to hold rallies issued earlier.
Justice Secretary Gonzales stated that, political rallies, which to the
President’s mind were organized for purposes of destabilization, are
cancelled.
Presidential Chief of Staff Defensor announced that warrantless arrests
and take-over of facilities including media can be implemented.
Undeterred by the announcements that rallies and public assemblies
would not be allowed, groups of protesters marched from various
parts of Metro Manila with the intention of converging at EDSA
shrine.
The policemen cited PP 1017 as the ground for the dispersal of their
assemblies. During dispersal of the rallyists, police arrested
petitioner David, a UP professor and newspaper columnist, together
wth Ronald Llamas of Akbayan.
February 25, PNP raided the Daily Tribune and confiscated news stories
by reporters, documents, among others. The raid is meant to show a
strong presence to tell media outlets not to connive or do anything
that would help the rebels in bringing down this government.
An appeal was henceforth filed with the SC.

ISSUE/s:
WoN PP 1017 is violative of the constitutional guarantee of freedom of
the press, of speech, and of assembly - YES

RULING: SC partly granted the petition.

RATIO:
What made it worse for petitioner David et al is not only was their right
against warrantless arrest violated, but also their right to paeacaby
assemble.
“Assembly” means a right on the part of the citizens to meet peaceably
for consultation in respect to public affairs. It is a necessary
consequence of our republican institution and complements the right
of speech.
As in the case of freedom of expression, this right is not to be limited,
much less denied, except on a showing of a clear and present
danger of a substantive evil that Congress has a right to prevent. In
other words, like other rights embraced in the freedom of expression,
the right to assemble is not subject to previous restraint or
censorship.
not for the assembly itself, may be validly required.
Petitioner David et al were arrested while they are exercising their right to
peacable assembly. They were not committing any crime, neither
was there a showing of a clear and present danger that warranted
the limitation of that right.
It was ruled in De Jonge v. Oregon that peacable assembly cannot be
made a crime. If persons assembling committed crimes elsewhere,
they may be prosecuted for their conspiracy or other violations. But it
is a different matter when the state seizes upon mere participation in
a peacable assembly and a lawful public discussion as the basis for
a criminal charge.
Freedom of assembly is not to be limited, much less denied, except on a
showing of a clear and present danger of a substantive evil that the
State has a right to prevent. Tolerance is the rule and limitation is the
exception.
Respondents failed to show or convince the Court that the rallyists
committed acts amounting to lawless violence, invasion, or rebellion.
With the blanket revocation of the permits, the distinction between
protected and unprotected assemblies was eliminated.
Moreover, BP 880 covers the authority of the government to issue and
revoke permits after due notice and hearing. This case showed an
absence of such notice and hearing. The first time the petitioners
learned of it was at the time of the dispersal, which is fatal.
CHAVEZ v. GONZALES (DANNAH) Sec. of Justice Gonzales warned reporters that those who had copies of
February 15, 2008 | Puno, J | the CD and those publishing its content could be held liable under
the Anti-
PETITIONER: Francisco Chavez
RESPONDENTS: Raul Gonzales (SOJ), National Telecomm.
Commission (NTC)

SUMMARY: Malacanang issued a statement that the opposition was


trying to destabilize the current administration. They released 2 tapes
with PGMA speaking with a COMELEC official, alleging that one was
altered and one was complete. Atty. Paguia, former counsel of Estrada
released an alleged authentic tape recording of the wiretap. Included
here were purported conversations of First Gentleman Arroyo, PGMA,
Garcillano et al.

The NTC and Secretary of Justice (KBP joined afterwards) issued


warnings that those media stations using their medium to spread false
informations may have their licenses revoked. Hence, petitioner Chaves
via Rule 65 filed a Petition for Certiorari with the Court. Although without
locus standing, the Court allowed the petition as it entailed
“transcendental importance”. Moreover, the Court ruled that the
statements issued by the SOJ and NTC took the form of a content-based
regulation and was a prior restraint. They also held that the great evil
which government wants to prevent is the airing of a tape recording in
alleged violation of the anti-wiretapping law. The records of the case at
bar, however, are confused and confusing, and respondents’ evidence
falls short of satisfying the clear and present danger test.

DOCTRINE: The need to prevent violation of laws cannot per se trump


the exercise of free speech and free press, a preferred right whose
breach can lead to greater evils.

FACTS:
On June 2005, Press Secretary Bunye told reporters that the opposition
was planning to destabilize the administration by releasing an
audiotape of a mobile conversation allegedly between PGMA and a
high-ranking official of the COMELEC. Conversation was audiotaped
allegedly through wire-tapping.
Later, Secretary Bunye produced 2 versions of the tape, one supposedly
the complete version, and the other which was “doctored” or altered.
It suggested that the President had instructed the COMELEC official
to manipulate election results.
Same month, former counsil of Estrada, Atty. Paguia, subsequently
released an alleged authentic tape recording of the wiretap. Included
here were purported conversations of the President, First Gentleman
Arroyo, Commissioner Garcillano and Sen. Barbers.
Wiretapping Act. He also ordered the NBI to go after media Dangerous Tendency Doctrine – permits limitations on speech once a
organizations found to have cause the spread of said tapes, starting rational connection has been established between the speech
with Inq7.net. restrained and the danger contemplated
The NTC also issued a press release giving fair warning to radio and TV
owners/operators stating that they shouldn’t use said mediums for
broadcasting or telecasting false information, lest their licenses be
revoked.
The NTC and Kapisanan ng mga Brodkaster sa Pilipinas (KBP) issued a
Joint Press Statement which includes the fact that the Circular they
issued did not constitute a restraint of press freedom, and that it was
only saying that the exercise of press freedom be done responsibly.
Petitioner Chavez filed the petition under Rule 65 against Sec. Gonzales
and NTC because their acts constitute violations of freedom on
expression and of the press, and the right of the people to
information on matters of public concern.
NTC and Sec. Gonzales on the other hand denied that their acts
transgress the Constitution, and attack the legal standing of Chavez.
They also claim that broadcast media enjoy lesser constitutional
guarantees compared to print media.

ISSUE/s:
WoN Chavez has legal standing – NO, but issue is on transcendental
importance
WoN the acts of NTC and Sec. Gonzales constitute prior restraint – YES

RULING: Petition granted. Prohibition issued nullifying the statements made,


for constituting unconstitutional prior restraint.

RATIO:
The freedom of expression is a preferred right that stands on a higher
level than substantive economic freedom or other liberties.
As early as 1920s, the trend as reflected in Philippine and American
decisions was to recognize the broadest scope and assure the
widest latitude for this constitutional guarantee.
The scope of freedom of expression is broad—it extends protection to
nearly all forms of communication including speech, print and
assembly.
In DYRE v. Dans, the Court held that all forms of media, whether print or
broadcast, are entitled to the broad protection of the clause on
freedom of speech and expression.
However, not all speech are treated the same. Some may be subjected
to regulation under police power. (For example, libel, obscene
speech and fighting words are not entitled constitutional protection
and are penalized)
Courts have developed 3 tests and use either or a combination of:
Balancing of Interests Tests – used as a standard when courts need content-based restraint will be struck down
to balance conflicting social values and individual interests 4 Must show the type of harm the speech sought to be
Clear and Present Danger Rule – Rests on the premise that speech restrained would bring about—gravity and imminence
may be restrained because there is a substantial danger that the 5 Cannot be justified by hypothetical fears; only be showing
speech will likely lead to an evil the government has the right to a substantive and imminent evil
prevent. The evil consequences must be substantive, “extremely It is clear that the challenged acts need to be subjected to the clear and
serious and the degree of imminence extremely high” present danger rule, as they are content-based regulations. The acts are
More recently, the Courts have adhered to the clear and present danger test. focused solely on one object—fixed as these were on the alleged taped
There are four aspects of freedom of the press, namely: conversations.
Freedom from prior restraint Although in the US radio and television have been held to have limited
Freedom from punishment subsequent to publication protection and excluded broadcast media from the application of the
Freedom of access to information
strict scrutiny standard, it can be seen that we have deviated from
Freedom of circulation
their conception of free speech.
Chavez had argued that the press statements constitute a form of
Again, in DYRES v. Dans, the Court adopted US jurisprudence to justify
impermissible prior restraint, hence a closer scrutiny of this principle
differentiation of treatment, but only after categorically declaring that
is in order, as well as its sub-specie of content-based regulations.
“the test for limitations on freedom of expression continues to be the
Prior restraint refers to official governmental restrictions on the press or
clear and present danger rule,” for all forms of media, whether print
other forms of expression in advance of actual publication or
or broadcast.
dissemination
Thus, when the Court declared in Dans that the freedom given to
It precludes governmental acts that required approval of a proposal to
broadcast media was “somewhat lesser in scope than the freedom
publish; licensing or permits as prerequisites to publication;
accorded to newspaper and print media,” it was not as to what test
injunctions
should be applied, but the context by which requirements of
Any law or official that requires some form of permission to be had before
licensing, allocation of airwaves, and application of norms to
publication can be made, commits an infringement of the
unprotected speech.
constitutional right, and remedy can be had at the courts.
Also in Gonzales v. Katigbak, the test to determine free expression
However, not all prior restraints are invalid. Certain previous restraints
challenges was the clear and present danger, without distinguishing
may be permitted by the Constitution, but determined only upon a
the media.
careful evaluation of the challenged act as against the appropriate
However, in Osmena v. COMELEC, the Court refused to apply the clear
test by which it should be measured against.
and present danger rule to a COMELEC regulation of time and
A distinction has to be made whether the restraint is:
manner of advertising of political advertisements because the
Content Neutral Regulation
0 Merely concerned with the incidents of the speech, or one challenged restriction was contentneutral, which shows that the rule
that merely controls the time, place or manner, under well doest not apply to all cases that involve broadcast media.
defined standards. The procedural map to follow in cases like the one at hand is:
1 Only a substantial governmental interest is required for its validity The test
2 Subject to an intermediate approach The presumption
Content Based Restraint The burden of proof
0 The restriction is based on the subject matter of the The party to discharge the burden
utterance or speech. The quantum of evidence necessary
1 Given the strictest scrutiny It appears that the great evil which government wants to prevent is the
2 Only when the challenged act has overcome the clear airing of a tape recording in alleged violation of the anti-wiretapping
and present danger rule will it pass constitutional muster, law. The records of the case at bar, however, are confused and
with the government having the burden of overcoming the confusing, and respondents’ evidence falls short of satisfying the
presumed unconstitutionality
clear and present danger test.
3 Unless the government can overthrow this presumption, the
The various statements of the Press Secretary obfuscate the identity
of the voices in the tape recording The public airing of the Garci tape is a protected expression because it
The integrity of the taped conversation is suspect. The Press does not fall under any of the four existing categories of
Secretary showed to the public 2 versions: one supposedly unprotected expression
“complete” and the other, “altered”
The evidence on the who’s and how’s of the wiretapping act is
ambivalent. The identity of the wire-tappers, manner of
commission and other related proofs are invisible
Given all unsettled factes of the tape, it is even arguable whether
its airing would violate the anti-wiretapping law.
The need to prevent violation of laws cannot per se trump the exercise of
free speech and free press, a preferred right whose breach can lead to
greater evils.
Any act done, such as a speech uttered, for and on behalf of the
government in an official capacity is covered by the rule on prior
restraint.
There is enough evidence of the chilling effect of the complained acts.
The warnings came from no less than NTC, and the Secretary of
Justice. After these warnings, the KBP inexplicably joing the NTC.
After everything, petitioner Chavez was left alone to fight the battle for
freedom of speech and of the press. This silence on the sidelines on
the part of some media practitioners is too deafening to be the
subject of misinterpretation.
Sandoval-Gutierrez, Concurring: The threat of suspension, revocation,
cancellation of licenses against radio and TV stations should they air
the Garci Tape is definitely a form of restraint. NTC’s warning that it
will not hesitate to apply with full force the provisions of the Circulars
and their accompanying sanction is a threat of a subsequent
punishment, an equally abhorred form of censorship. Moreover, it
must be stressed that it was a government official who initially
released the Garci tapes, not the media.
Carpio, Concurring:
The government action may chill into silence those to whom the action
is directed. Freedom of expression, being fundamental, is of
transcendental importance that must be defended. Any concerned
citizen has the standing to raise an issue of transcendental
importance.
The exceptions when expression may be subject to prior restraint,
apply only to: pornography, false or misleading advertisement,
advocacy of imminent lawless action and danger to national
security. All other expression is not subject to prior restraint.
Expression that may be subject to prior restraint is unprotected
expression or low-value expression
While there can be no prior restraint on protected expression, such
expression may be subject to subsequent punishment
recognized in this jurisdiction
The rule, which recognizes no exception, is that there can be no content-
based prior restaint on protected expression. On this ground alone, the
NTC press release is unconstitutional.
Azcuna, Concurring: The NTC and DOJ warnings violate Sec. 10, Art XVI
of the Constitution.
Tinga, Dissenting and Concurring:
Prior restraint, by contrast and definition, has an immediate and irreversible
action. The assailed act of the NTC, hardly constitutes and immediate
and irreversible sanction
What it says is that only “if it has been subsequently established that the
said tapes are false and/or fraudulent after a prosecution or appropriate
investigation” that the stations could be subjected to possible
suspension.
It is evident that the issuance does not prohibit the airing of the Garci tapes
or require that the broadcast stations obtain permission from the
government or the NTC to air such tapes.
The threat of restraint, as opposed to actual restraint itself, may deter the
exercise of the right to free expression almost as potently as the actual
application of sanctions.
While the act/issuance itself may evince the impression of a chilling effect,
there still must be factual evidence to support the conclusion that a
particular act of government actually engendered a chilling effect.
Velasco, Dissenting and Concurring:
The press release constitutes a clear instance of prior restraint
The fact that they disclaimed that they did not issue any Circular or oderer
constituting a restraint of press freedom/censorship tends to prove,
rather than disprove, the threatening/chilling tone of its press release.
Nachura, Dissenting:
Prior restraint should not be confused with subsequent punishment
To hold that the forfeiture order in this case constituted a prior restraint
would have the exact opposite effect. It would blur the line separating
prior restraints from subsequent punishments to such a degree that it
would be impossible to determine with any certainty whether a particular
measure is a prior restraint or not.
What we have is a press release, with the caveat that should its falsity be
subsequently established, the act could lead to the revocation or
cancellation of their licenses, after appropriate investigation.
The warnings on possible license revocation and criminal prosecution are
simply what they are, mere warnings. They have no compulsive effect,
as they do not impose a limit on speech or other forms of expression
nor do they prevent the expression of a message
GMA v. COMELEC (ARIELLE) candidate is entitled to the aforementioned number
2 September 2014 | Peralta, J. | Freedom of Speech and the of minutes “per station.”
Press For the 2013 elections, Comelec promulgated Resolution
No.
PETITIONER/S: GMA Network
RESPONDENT/S: Commission on Elections

SUMMARY: Comelec issued Resolution No. 9165 which changed


its former interpretation of Section 6 of RA 9006 of airtime
limitation of political candidates from a “per station” basis to a
“total aggregate” basis. With the new Resolution, candidates now
have a total of 120 minutes of television advertisement and 180
minutes of radio advertisement for all stations. Petitioners, various
media outfits, contend that this Resolution violate the
constitutionally protected right to freedom of expression, of
speech, and of the press. The Comelec claimed that this
regulation is within its “power to amplify” and is pursuant to its
constitutional mandate to regulate all election-related matters. The

Supreme Court held that it is not within the power of the Comelec to
issue such Resolution for it did not provide adequate justification for
limiting the allowable airtime for candidates. Moreover, Section 9(a)
with its adoption of the aggregate-based airtime limits unreasonably
restricts the guaranteed freedom of speech and of the press. Political
speech is one of the most important expressions protected by the
Fundamental Law.

DOCTRINE: The guaranty of freedom to speak is useless without


the ability to communicate and disseminate what is said. At the
same time, the right to speak and to reach out would not be
meaningful if it is just a token ability to be heard by a few.

FACTS:
Section 6 of RA 9006 otherwise known as the Fair Election
Act, provides that all registered parties and bona fide
candidates shall have equal access to media time and
space, with 120 minutes of television advertisement and
180 minutes of radio advertisement whether by purchase
or donation.
For the 2007 and 2010 elections, Comelec issued
Resolutions implementing and interpreting Section 6 of
the said Act regarding airtime limitations to mean that a
9615, changing its interpretation on the airtime limitation from a
“per station” basis, to a “total aggregate” basis.
Petitioners sent respective letters to the COMELEC, questioning
the provisions of the Resolution, which prompted COMELEC to
conduct public hearings. Comelec thereafter amended the
provisions of the Resolution, but petitioners still found the same
objectionable and oppressive.
Petitioners assail the following provisions:
Section 9(a) which provides for an “aggregate total” airtime
instead of the previous “per station” airtime for political
campaigns.
Section 14 which provides for a candidate’s or political
party’s “right to reply” as it constitutes prior restraint and
infringing petitioners’ freedom of expression, speech and
the press; and for being violative of the equal protection
guarantee.
Petitioners claim that Section 9(a), which imposes a notice
requirement, is vague and infringes on the constitutionally
protected freedom of speech, of the press and of expression,
and on the right of people to be informed on matters of public
concern.
Respondent contends that the per candidate rule or total aggregate
airtime limit is in accordance with RA No. 9006 as this would
truly give life to the constitutional objective to equalize access to
media during elections. It sees this as a more effective way of
levelling the playing field between candidates/political parties
with enormous resources and those without much.
The respondent also claims that the issuance of the Resolution is
pursuant to Section 4, Article IX(C) of the Constitution which
vests the COMELEC the power to supervise and regulate during
election periods, transportation and other public utilities, as well
as mass media.
Moreover, respondent avers that there is no prior restraint in the
provisions requiring notice to the COMELEC for appearances or
guestings of candidates in news broadcasts. It points out the fact
that the notice may be given 24 hours after first broadcast only
proves that the mechanism is for monitoring purposes only, not for
censorship. Further, petitioners are only required to
inform the COMELEC of candidates’/parties’ guestings, airtime limitations on political advertisements. This
but there is no regulation as to the content of the news or essentially consists
the expressions in news interviews or news
documentaries.
Respondent insists that the right to reply cannot be
considered a prior restraint on the freedoms of
expression, speech and the press, as it does not in any
way restrict the airing of bona fide news broadcasts.
Media entities are free to report any news event, even if it
should turn out to be unfavorable to a candidate or party.
The Resolution merely gives the candidate or party the
right to reply to such charges published or aired against
them in news broadcasts.
Respondent emphasized that the Supreme Court has held
that freedom of speech and the press may be limited in
light of the duty of the COMELEC to ensure equal access
to opportunities for public service.
The COMELEC also argued that there is no impairment of
the people’s right to information on matters of public
concern, because in this case, the COMELEC is not
withholding any access to any public record.
The Court issued a TRO in view of the urgency involved and
to prevent irreparable injury that may be caused to the
petitioners if respondent COMELEC is not enjoined from
implementing Resolution No. 9615.

ISSUE/S:
Whether or not Section 9(a) on airtime limits is
unconstitutional for violating freedom of expression, of
speech, and of the press - YES

Whether or not Section 14, which provides for the “right to


reply” is unconstitutional for violating freedom of
expression, of speech, and of the press - NO

RULING: Petition is partly meritorious.

RATIO:
The Comelec Resolution No. 9615 introduced a radical
change from the previous Comelec resolutions regarding
in computing the airtime on an aggregate basis involving all the
media of broadcast communications compared to the past
where it was done on a per station basis. Thus, it becomes
immediately obvious that there was effected a drastic reduction
of the allowable minutes within which candidates and political
parties would be able to campaign through the air.
It is not within the power of the COMELEC to promulgate such
resolution as the COMELEC did not have any other basis for
coming up with a new manner of determining allowable time
limits except its own idea as to what should be the maximum
number of minutes based on its exercise of discretion as to how
to level the playing field. Although the COMELEC is
constitutionally and statutorily authorized to enforce election
laws, it cannot exercise its powers without limitations or
reasonable basis.
The Comelec gave its own understanding of RA 9006 and said that
the Congress intended to impose the per candidate or political
party aggregate total airtime limits as evidenced by the dropping
of the “per day per station” language embodied in both versions
of the House and Senate bills in favor of the “each candidate”
and “not more than” limitations now found in Section 6 RA 9006.
The guaranty of freedom to speak is useless without the ability to
communicate and disseminate what is said. At the same time,
the right to speak and to reach out would not be meaningful if it
is just a token ability to be heard by a few. Section 9(a) with its
adoption of the aggregate-based airtime limits unreasonably
restricts the guaranteed freedom of speech and of the press.
Political speech is one of the most important expressions
protected by the Fundamental Law. Accordingly, the same must
remain unfettered unless otherwise justified by a compelling
state interest. [American case: A restriction on the amount of
money a person or group can spend on political communication
necessarily reduces the quantity of expression by restricting the
number of issues discussed, the depth of their exploration, and
the size of the audience reached.]
The Court agrees that the assailed rule on “aggregate-based”
airtime limits is unreasonable and arbitrary as it unduly restricts
and constrains the ability of candidates and political parties to
reach out and communicate with the people. The reason recognized, such is not applicable in this particular
for imposing the limit - leveling the playing field - does not matter because additional weight should be accorded
constitute a compelling state interest which would justify on the constitutional directive to afford a
such a substantial restriction. We must also consider that
the Philippines is composed of so many islands and there
are a lot of languages and dialects spoken by the citizens
across the country. When the press is silenced, or
muffled in its task of acting as a sounding board, the
people ultimately would be the victims.
The legal duty of monitoring lies with the Comelec.
Broadcast stations are merely required to submit certain
documents to aid the Comelec in ensuring that
candidates are not sold airtime in excess of the allowed
limits. The broadcast stations are not asked to do
monitoring.
Section 9(a) also provides that for purposes of monitoring by
the Comelec and ensuring that parties and candidates
were afforded equal opportunities to promote their
candidacy, the media entity shall give prior notice to the
Comelec. Such requirement is a reasonable means
adopted by the Comelec to ensure that parties and
candidates are afforded equal opportunities to promote
their respective candidacies. The directive to give prior
notice is not unreasonable, much less could it be
characterized as prior restraint since there is no
restriction on dissemination of information before
broadcast.
Section 14, or the “right to reply” provision is reasonable and
consistent with the constitutional mandate. It provides
that all bona fide candidates shall have the right to reply
to charges published by the media outfit which first
printed or aired the charges. The attack on the validity of
the “right to reply” is primarily anchored on the alleged
ground of prior restraint, specifically insofar as such a
requirement may have a chilling effect on speech or of
the freedom of the press.
The Constitution itself provides as part of the means to
ensure free, orderly, honest, fair and credible elections, a
task was given to Comelec to provide for a right to reply.
Although the traditional notions of preferring speech and
the press over so many other values of society is
right to reply. If there was no such mandate, then the
submission of petitioners may more easily commend
themselves for this Court’s acceptance.
In fine, when it comes to election and the exercise of freedom of
speech, of expression, and of the press, the latter must be
properly viewed in context as being necessarily made to
accommodate the imperatives of fairness by giving teeth and
substance to the right to reply requirement.
Carpio, Separate Concurring:
Comelec grounds its issuance of the Resolution not only on
RA 9006 but also on two provisions of the Constitution,
namely Section 2(7) and Section 5, both of Article IX-C.
Section 2(7) concerns the power of the Comelec to
recommend to the Congress effective measures to
minimize election spending. On the other hand, Section 4
authorizes the Comelec to supervise the enjoyment and
utilization of all franchises for the operation of media of
communication or information.
In capping the broadcast advertising time of candidates,
neither the Congress nor the Comelec supervised or
regulated the enjoyment and utilization of franchises of
media outfits. Media firms continue to operate under their
franchises free of restrictions despite the imposition of
these airtime caps.
Section 9(a) of the Resolution is content-neutral time
regulations which do not reach the content of campaign
speech but merely limit its cumulative broadcast time during
the campaign period. Such content-neutral regulations are
subjected to the intermediate level of scrutiny.
However, the capping of campaign airtime by Section 9(a)
advances the state interest of minimizing election
spending arbitrarily and the incidental restriction on the
freedoms of speech and expression these provisions
impose is greater than is essential to the furtherance of
such state interest.
Brion, Concurring
The Comelec failed to come up with a reasonable basis and
explanation for the change of airtime limits. The Comelec
explained that the new interpretation was prompted by the
need to level the playing field among the candidates.
The Resolution infringes on the people’s right to be duly Prior restraint is defined as the “official
informed about the candidates and the issues. governmental restrictions on the press or
It violates the candidates’ freedom of speech because it other forms of expression in
restricts their ability to reach out to a larger audience.
The change in interpretation was only changed for the 2013
elections under the assailed Resolution. The only
reasonable explanation provided by the Comelec
Chairman was that the Comelec has the “prerogative to
amplify” under RA 9006 and on the blanket invocation of
the need to level the playing field among candidates. The
failure to sufficiently explain the basis for the change of
interpretation is beyond legal limits. Basic fairness
demands that the Comelec provide a reasonable
justification, considering the Comelec’s own knowledge of
the dynamics of campaign strategy and influence of the
radio and television as medium of communication.
The quasi-judicial power of the Comelec embraces the
power to resolve controversies arising from the
enforcement of election laws and to be the sole judge of
all pre-proclamation controversies and of all contests
relating to the elections, returns, and qualifications.
In the exercise of quasi-judicial power, the Comelec must
necessarily ascertain the existence of facts, hold
hearings, weigh the presented evidence, and draw
conclusions from them as basis for its action and
exercise of discretion.
The Comelec’s quasi-legislative power is its power to issue
rules and regulations to implement these election laws.
Legislative rules are in the nature of subordinate
legislation and are designed to implement a law by
providing the details of the law. Interpretative rules are
intended to interpret, clarify or explain existing statutory
regulations under which the administrative body
operates.
The revenues that petitioners may potentially lose under the
Comelec’s restrictive interpretation have adverse effects on
the petitioners’ operations. On the other hand, substantially
limiting the allowable airtime advertisements of candidates
would have serious repurcussions on their campaign
activities and strategies.
Leonen, Concurring
advance of actual publication or dissemination.” Prior restraints
of speech are generally presumptively unconstitutional. The only
instances when this is not the case are in pornography, false
and misleading advertisement, advocacy of imminent lawless
action, and danger to national security.
Section 6 of the Fair Election Act is a form of prior restraint. While it
does not totally prohibit speech, it has the effect of limitations in
terms of the candidates’ and political parties’ desired time
duration and frequency. When an act of government is in prior
restraint of speech, government carries a heavy burden of
unconstitutionality.
Reducing the airtime to extremely low levels reduces information to
slogans and sound bites which may impoverish public dialogue.
Lacking the enlightenment that comes with information and
analysis makes the electorate’s role to exact accountability from
elected public officers a sham. More information requires more
space and airtime equally available to all candidates.
PEOPLE v. PEREZ (IYA) The words of the accused did not so much tend to defame, abuse, or
December 22, 1923 | Malcom, J. | Freedom of Speech insult, a person in authority as they did to raise a disturbance in the
community,
The provisions of the Treason and Sedition Law must not be interpreted
PETITIONER: People of the Philippine Islands
as to abridge the freedom of speech and the right of the people
RESPONDENTS: Isaac Perez peaceably to assemble and petition the government for redress of
grievances.
SUMMARY: Perez uttered words directed to the then Governor-General
Criticism, no matter how severe, on the Executive, the Legislature, and
which were believed to be libelous. He was charged and convicted with the Judiciary, is within the range of liberty of speech, unless the
violating Article 256 of the Penal Code. On appeal, the SC ruled that the intention and effect be seditious.
penal provision does not apply but instead he is penalized under the When the intention and effect of the act is seditious, the
Treason and Sedition Law. constitutional guaranties of freedom of speech and press and of
assembly and petition must yield to punitive measures designed
DOCTRINE: When the intention and effect of the act is seditious, the
to maintain the prestige of constituted authority, the supremacy
constitutional guaranties of freedom of speech and press and of
of the constitution and the laws, and the existence of the State.
assembly and petition must yield to punitive measures designed to
Perez has uttered seditious words. He has made a statement and done
maintain the prestige of constituted authority, the supremacy of the an act which suggested and incited rebellious conspiracies.
constitution and the laws, and the existence of the State. The penalty imposed by the trial court falls within the limits provided by
the Treason and Sedition Law, which the Court believes is applicable
to the accused.
FACTS:
Perez and Lodovice were engaged in a discussion regarding the
administration of Governor-General Wood.
Perez in the heat of the discussion, shouted a number of times “ The
Filipinos, like myself, must use bolos for cutting off Wood's head for
having recommended a bad thing for the Filipinos, for he has killed
our independence.”
The CFI of Sorosogon charged and convicted Perez of violating Article
256 of the Penal Code on contempt of ministers of the Crown or
persons in authority.
Perez testified that the discussion was held in a peaceful manner, that
what he wished to say was that the Governor-General should e
removed and substituted by another.
Trial court found that it has been proved beyond reasonable doubt guilty
of violating Art. 256 of the Penal Code.
Hence, the present case.

ISSUE/s:
WON Perez should be convicted of the said penal provision - NO

RULING: Perez is punishable under the Treason and Sedition Law, the
penalty imposed is the same.

RATIO:
The law infringed in this instance is not the said article but rather a
portion of the Treason and Sedition Law.
DENNIS v. US (ELIEL) ISSUE/s:
June 4, 1951 | Vinson, J. | Freedom of Speech and Expression WoN Dennis’ free speech or press has been violated - NO

PETITIONER: Dennis
RESPONDENTS: United States

SUMMARY: Dennis a leader of the Communist party was convicted


under the Smith Act for willfully and knowingly conspiring to overthrow
the government. Evidence against Dennis is regarding his speech. In this
regard, Dennis questions his right to free speech and expression for
being violated because intent should first be proven, and that he was
only exercising his rights. Hence this petition.

The US SC held that speech is not an absolute right. The State has a duty to
protect itself by regulating certain rights in furtherance of justice and public
order. However, the standard maintained in order to guarantee justice and
fairness in the freedom of speech and expression is clear and present
danger. This is present when the substantive evil is imminent, then the
Congress has a right. In the case at bar, Dennis was inciting to overthrow the
government by force and violence through conspiring with people with the
same purpose and intent. This is a substantive evil aimed to be sought by the
government, and thus, has the every right to regulate it. Therefore, there is
no violation against Dennis when he was convicted.

DOCTRINE: The rule we deduce from tehse cases is that, where an


offense is specified by a statute in nonspeech or nonpress terms, a
conviction relying upon speech or press as evidence of violation may be
sustained only when the speech or publication created a “clear and
present danger” of attempting or accomplishing the prohibited crime.

FACTS:
Dennis, leader of the Communist Party in the US, were indicted in a
federal district court under Sec 3 of the Smith Act for willfully and
knowingly conspiring (1) to organize as the Communist Party a group
of persons to teach and advocate the overthrow and destruction fo
the Government of the US by force and violence, and (2) knowingly
and willfully to advocate and teach the duty and necessity of
overthrowing and destroying the Governmetn of the US by force and
violence.
The trial judge instructed the jury that they could not convict unless they
found that Dennis intended to overthrow the Government “as
speedily as circumstances would permit,” but that, if they so found,
then, as a matter of law, there was sufficient danger of a substantive
evil that Congress has a right to prevent to justify application of the
statute under the 1st amendment.
Dennis was convicted, and the convictions were sustained by the CA.
Hence this petition.
conviction relying upon speech or press as evidence of
RULING: Convictions affirmed. violation may be sustained only when the

RATIO:
The Statute requires an essential element of the crime proof of the intent
of those who are charged with its violation to overthrow the
Government by force and violence.
The existence of a mens rea is the rule of, rather than the exception to,
the principles of Anglo-American criminal jurisprudence.
It has bee suggested that the presence of intent makes a difference in
the law when an “act otherwise excusable or carrying mior penalties”
is accompanied by such an evil intent.
The obvious purpose of the statute is to protect exsiting Government not
form cange by peaceable, lawful and constitutional means, but from
change by violence, revolution and terrorism.
No one cold conceive that it is nto within the power of Congress to
prohibit acts intended to overthrow the Government by force and
violence. The question with which we are concerned her is not
whether ongress has such power, but whether the means it has
employed conflict with the 1st and 5th amendments to the
Constitution.
The very language of the Smith Act negates the interpretation which
Dennis would have us impose on that Act. It is directed at advocacy,
not discussion. Thus, the trial judge properly charged the jury that
they could not convict if they found that Dennis did “no more than
pursue peaceful studies and discussions or teaching and advocacy
in the realm of ideas.”
Congress did not intend to eradicate the free discussion of political
theories, to destroy the traditional rights of Americans to discuss and
evaluate ideas without fear of governmental sanction. Rather
Congress was concerned with the very kind of activity in which the
evidence showed Dennis engaged.
An analysis of the leading cases in the Court which would have
involved direct limitations on speech, however, will demonstrate
that both the majority of the Court and the dissenters in
particular cases have recognized that this is not an unlimited,
unqualified right, but that the societal value of speech must, on
occasion, be subordinate to other values and considerations.
Justice Holmes, “question in every case is whether the words used
are used in such circumstances and are of such nature as to
create a clear and present danger that they will bring about the
substantive evils that Congress has a right to prevent.”
The fact is inescapable, too, that the phrase bore no connotation that the
danger was to be any threat to the safety of the Republic. The
charge was causing and attempting to cause insubordination in the
military forces and obstruct recruiting.
The rule we deduce from these cases is that, where an offense is
specified by a statute in nonspeech or nonpress terms, a
speech or publication created a “clear and present danger” of attempting
or accomplishing the prohibited crime.
The legislature had found that a certain kind of speech was, itself,
harmful, and unlawful.The constitutionality of such a state
statute had to be adjudged by this Court just as it determined
the constitutionality of any state statute, namely, whether the
statute was “reasonable.” Since it was entirely reaosonable for
a state to attempt to protect itself from violent overthrow, the
statute was perforce reasonable.
Whenever speech was the evidence of the violation, it was necessary to
show that the speech created the “clear and present danger” of the
substantive evil which the legislature had the right to prevent.
Speech is not an absolute, above and beyond control by the
legislature when its judgment, subject ot review here, is that
certain kinds of speech are so undesirable as to warrant
criminal sanction.
Nothing is more certain in modern society than the principle that tere are
no absolutes, that a name, a phrase, a standard has meaning only
whe associated with the considerations which gave birth to the
nomenclature.
Overthrow of the Government by force and violence is certainly a
substantial enough interest for the Government to limit speech.
Indeed, this is the ultimate value of any society, for if a society cannot
protect its very structure from armed internal attack, it must follow
that no subordinate value can be protected.
If, then, this interest may be protected, the literal problem which is
presented is what has been meant by the use of the phrase
“clear and present danger” of the utterances bringing about the
evil wthin the power of Congress to punish.
The doctrine that there must be a clear and present danger of a
substantive evil that Congress has a right to prevent is a
judicial rule to be applied as a matter of law by the courts.
GONZALES v. COMELEC (JP) political committee, to in an election campaign or partisan political
April 18, 1969 | Fernando, J. | Freedom of Speech activity except during the period of 120 days immediately preceding
an election involving a public office voted for at large and 90 days
PETITIONER: Arsenio Gonzales, Felicisimo Cabigao immediately preceding an election for any other elective office.
RESPONDENTS: Commission on Elections According to petitioners, the enactment of Republic Act [No.] 4880 under
the guise of regulation is but a dear and simple abridgment of the
SUMMARY: Petitioners were assailing the validity of RA 4880 and the
constitutional rights of freedom of speech, freedom of assembly and
additional provisions of the Revised Election Code as unconstitutional for
violating their right to speech, assembly and association. The first the right to form associations and societies for purposes not contrary
provision was on the prohibition of too early nominations of candidates to law.
and the second provision was on the limitation on the period of election There was the further allegation that the nomination of a candidate and the
campaigns and partisal political activity. fixing of period of election campaign are matters of political expediency
and convenience which only political parties can regulate or curtail by
The SC held as to the first provision that such is valid for such and among
curtailment is not unduly narrowed. As for the second provision, the
themselves through self-restraint.
provisio does sound initially vague but with the addition of the
enumeration of such acts which constitute “election campaign” and ISSUE/s:
“partisan political activity” and the saving clause of the law which WoN RA 4880 violates the right to speech, assembly and associtations–
excplicitly stated that such law shall not violate said rights, fixed such NO
vagueness. In addition, there is justification of the police power because
RULING: Petition is dismissed.
of the present dishonesty and corruption during the election period.

DOCTRINE: The clear and present danger doctrine rightly viewed RATIO:
requires that not only should there be an occasion for the imposition of On the prohibition of too early nomination of candidates, the SC sustains
such restrictions but also that they be limited in scope. its validity unanimously.
Political parties have less freedom as to the time during which they may
It is a well-settled principle that stricter standards of permissible statutory nominate candidates; the curtailment is not such, however, as to
vagueness may be applied to a statute having inhibiting effect on speech. render meaningless such a basic right. Their scope of legitimate
activities, save this one, is not unduly narrowed.
FACTS: On the limitation of election campaigns and partisan political activity: On
Petitioner Cabigao was an incumbent council of thr 4 th District Manila one hand, if that is all there is to that provision, it suffers from the
and candidate for Vice-Mayor of Manila. Petitioner Gonzales is a fatal constitutional infirmity of vagueness and may be stricken down.
private individual and a political leader of his co-petitioner. It is a well-settled principle that stricter standards of permissible
Petitioners challenged the validity of two new sections now included in statutory vagueness may be applied to a statute having inhibiting
the Revised Election Code, under Republic Act No. 4880 prohibiting effect on speech.
the too early nomination of candidates and limiting the period of
But what removes the sting from constitutional objection of vagueness is
election campaign or partisan political activity .
the enumeration of the acts deemed included in the terms election
The prohibition of too early nomination of candidates: It shall be
campaign or partisan political activity. As thus limited, the objection
unlawful for any political party, political committee, or political group
that may be raised as to vagueness has been minimized, if not totally
to nominate candidates for any elective public office voted for at
set at rest.
large earlier than 150 days immediately preceding an election, and
On the other hand, it cannot be denied either that evils substantial in
for any other elective public office earlier than ninety days
character taint the purity of the electoral process. There can be under
immediately preceding an election.
the circumstances then no outright condemnation of the statute.
The limitation on the period of election campaign or partisan political
The clear and present danger doctrine rightly viewed requires that not
activity: It is unlawful for any person whether or not a voter or
only should there be an occasion for the imposition of such
candidate, or for any group or association of persons, whether or not a restrictions but also that they be limited in scope.
political party or
Such provisions were deemed by the legislative body to be part and parcel
of the necessary and appropriate response not merely to a clear and
present danger but to the actual existence of a grave and substantive
evil of excessive
partisanship, dishonesty and corruption as well as violence that of late has
invariably marred election campaigns and partisan political activities in this
country.
Under the police power then, with its concern for the general welfare and
with the commendable aim of safeguarding the right of suffrage, the
legislative body must have felt impelled to impose the foregoing
restrictions.
Amicus Curae Sen. Tañada srressed that the provisions (below),
precisely placed in the statute as a manifestation of the undeniable
legislative determination not to transgress the preferred freedom of
speech, of press, of assembly and of association. If properly
implemented, the barrier to free expression becomes minimal .
“That simple expressions or opinion and thoughts concerning the
election shall not be considered as part of an election
campaign [and that nothing in the Act] shall be understood to
prevent any person from expressing his views on cur rent
political problems or issues, or from mentioning the names of
the candidates for public office whom he supports.”
It is its opinion that it would be premature, to say the least, for a judgment
of nullity of any provision found in Republic Act No. 4880. The need
for adjudication arises only if in the implementation of the Act, there
is in fact an unconstitutional application of its provisions.
US v. O’BRIEN (HENRY)
May 27, 1968 | Warren, J. | Symbolic Speech RULING: SC vacated the CA decision.

PETITIONER: United States


RESPONDENTS: David Paul O’Brien

SUMMARY: US issued the Selective Service Act which required


Americans to register upon reaching 18. ‘Brian and his 3 companions
burned in front of an audience their Selective Service Registration, which
lead for their conviction.

US CA vacated O’Brien from his sentence upon his appeal, and


remanded the case. The provision of the Selective Service Act banning
burning of draft cards was held unconstitutional. Upon appeal to the SC,
ruled that such law is constitutional.

DOCTRINE: When “speech” and “nonspeech” elements are combined in the


same course of conduct, a sufficiently important governmental interest in
regulating the nonspeech element can justify incidental limitations on free
speech.

FACTS:
US issued a peace-time draft with the Universal Military Training and
Serice Act (aka Selective Service Act) which required all male
American citizens to register with a local draft board upon reaching
18.
1965, Congress amended the act to prohibit the willful distruction of “draft
cards” or registration certificates.
Respondent O’Brien and his 3 companions burned their Selective
Service Registration Certificates on the steps of the South Boston
Courthouse, to which a sizeable crowd, including agents of the FBI,
witnessed the event.
O’Brien was indicted, tried, convicted, and sentenced in the United
States District Court of Massachusetts.
According to him, he did it in order for other people to reevaluate their
positions with Selective Service, with the armed forces and
reevaluate their place in the culture of today.
Upon appeal to the First Circuit Court of Appeals, the 1965 amendment
ran afoul the First Amendment because it signled out “persons
engaging in protest for special treatment”. His sentence was vacated
and his case was remanded to adjust the sentence in accordance
with a conviction under the Selective Service Act. Hence, this petition

ISSUE/s:
WoN provision of the Selective Service Act on banning the burning of the
draft cards is constitutional - YES
RATIO:
O’Brian first argues that the 1965 Amendment is unconstitutional as
applied to him because his act of burning his registration certificate
was protected by “symbolic speech” within the First Amendment. His
argument is that the freedom of expression which is the first
amendment guarantees and includes all modes of communication of
ideas by conduct, and that this conduct is within this definition
because he did it in demonstration against the war and against the
draft.
The SC cannot accept the view that an apparently limitless variety of
conduct be labeled as “speech”, whenever the person engaging in
the conduct intends thereby to express an idea.
SC held that when “speech” and “nonspeech” elements are combined in
the same course of conduct, a sufficiently important governmental
interest in regulating the nonspeech element can justify incidental
limitations on free speech.
A governmental regulation is sufficiently justified if:
It is within the constitutional power of the Government
Furthers an important or substantial governmental interest
unrelated to the suppression of free expression
If the incidental restriction on alleged freedom is no greated than
than is essential to that interest.
SC ruled that the Selective Service Act meets this test. Hence, the said
law is constitutional.
Harlan, concurring: In an extension to the majority’s standard, Harlan
argued that the 1st amendment should cover situations when a law
entirely prevented a speaker from reaching a significant audience by
denying the speaker any lawful means of communication. This was
clearly not O’Brien’s situation, so it was not relevant to the outcome
in this case.
Douglas, dissenting: Accepting the standard laid out by the Majority led
Douglas to opine that the government did not have an important
interest in draft cards unless war had been declared by Congress.
Vietnam War never had been officially declared, so he did not see it
as a “war” within the proper definitionof the term. Douglas argued
that a draft might not be appropriate in “peacetime”.
BLUE RIBBON COMMITTEE v. MAJADUCON (DANNAH) During the hearings, it appeared that the AFP-RSBS purchased a lot in
July 29, 2003 | Ynares-Santiago, J. | Freedom of the Press GenSan for P10.5k/sqm from private respondent Atty. Flaviano.
However, the records from the Register of Deeds indicated that the
PETITIONER: Senate Blue Ribbon Committee represented by Chairman purchase price was only P3k/sqm.
Sen. Pimentel
RESPONDENTS: Hon. Jose Majaducon, Atty. Flaviano

SUMMARY: The Senate Blue Ribbon Committee initiated proceedings


for legislative inquiry regarding alleged fund irregularities in the AFP.
During the hearings, it appeared that the AFP-RSBS purchased a lot
from Atty. Flaviano for P10.5k/sqm, but the Register of Deeds showed
that the price is only P3k/sqm. Hence, the Committee served a subpoena
to Atty. Flaviano. Instead of testifying, however, he filed a petition for
preliminary injunction and a TRO with the RTC of GenSan. Judge
Majaducon granted said TRO.

On the other hand, a news report by Philippine Star was published,


alleging that Judge Majaducon was guilty of gross ignorance when he
issued said Order. Hence, Judge Majaducon initiated a charge for indirect
contempt of court against Sen. Pimentel, and the publishers and editors
of PhilStar.

The Court ruled that 1) Judge Majaducon was indeed committed grave
abuse of discretion in issuing said Order. The court had no authority
prohibiting Atty. Flaviano from appearing because the hearing was an
inquiry in aid of legislation. Moreover, the Bengzon case found no
application. 2) Sen. Pimentel cannot be held guilty of indirect contempt
because he had no authority to prohibit the press from publishing the
news reports they want to publish. He had no right to choose which news
would be printed.

DOCTRINE: Newspaper publishers are allowed “the widest latitude of


choice as to what items should see the light of day so long as they are
relevant to a matter of public interest, pursuant to its right of press
freedom.”

FACTS:
The case at hand is two consolidated petitions.
On August 1998, Senator Ople filed a Senate Resolution directing the
Committee on National Defense and Security to conduct an inquiry,
in aid of legislation, into the charges of then Defense Sec. Mercado
that a group of active and retired military officers were planning a
coup.
The coup was to prevent Estrada from probing alleged fund irregularities
in the AFP.
Senator Sotto also referred the matter to the Blue Ribbon Committee and
the Committee on National Defense and Security.
The Committee then served a subpoena to Atty. Flaviano, directing him
to appear and testify. He refused and instead filed a petition for
prohibition and preliminary injunction with temporary restraining
order with the RTC of GenSan.
Said court issued the TRO directing the Committee to cease and desist
from proceeding with the inquiry.
The Committee on the other hand filed a motion to dismiss, but Hon.
Majaducon denid this and granted the writ of preliminary injunction.
Hence, this petition.
On the other hand, on January 1999, The Philippine Star published a
news report on the filing by the Committee with the SC of the petition
for certiorari of the aforementioned case.
The news report quoted portions of the petition filed, alleging that Judge
Majaducon was guilty of gross ignorance when he issued said Order,
and that under the principle of separation of powers, courts cannot
interfere with the exercise by the legislature of its authority to
conduct investigations in aid of legislation.
Reacting to the news report, judge Majaducon initiated a charge for
indirect contempt of court against Sen. Pimentel, news reporter
Echeminada, PhilStar publisher Soliven, et al.
Judge Majaducon averred that said report created in the minds of the
reader the impression that he violated the separation of powers
clause of the Constitution and that he was guilty of gross ignorance
of the rules and procedures.

ISSUE/s:
WoN Judge Majaducon committed grave abuse of discretion – YES
WoN Judge Majaducon erred in convicting Sen. Pimentel of indirect
contempt of court – YES

RULING: Both petitions granted. Writ of Preliminary Injunction set aside.


Resolution declaring Pimentel of indirect contempt reversed and set aside.

RATIO:
On first issue: There is grave abuse of discretion when the judge acts in
a capricious, whimsical, arbitrary or despotic manner in the exercise
of his judgment, as when the assaild order is bereft of any factual
and legal justification. Judge Majaducon’s Order was issued without
legal basis.
The Senate conducted the inquiry pursuant to its authority to conduct
inquiries in aid of legislation under Article VI Section 21 of the
Constitution. Hence, the RTC or any court had no authority to
prohibit the Committee from requiring Atty. Flaviano’s appearance.
Compared to the Bengzon case:
There, no intended legislation was involved. Here, there was, and the
subject matter was to enact appropriate legislation to protect the
rights and interests of the officers and members of the AFP.
There, the case was pending with the Sandiganbayan when the Senate
decided to conduct its investigation—the issue had already been
pre-empted by the court. Here, the complaint was still pending
before the Ombudsman. No court had acquired jurisdiction yet;
there was no encroachment by the legislature.
On the second issue: Sen. Pimentel claims that the phrase “gross
ignorance of the rules of law and procedure” which the Committee
used in the petition is not depreciatory, but merely a description of
normal usage in petitions where the acts of lower courts are
challenged before higher judicial bodies.
Also, the Committee did not use said phrase without malice. It was only
to stress the unfamiliarity of or disregard by the Judge of a basic rule
of procedure.
Pimentel also contends that he had no participation in the publication in
the Philippine Star of exceprts from the Committee’s petition for
certiorari.
Assuming arguendo that it was within his control, he pointed out that he
couldn’t have prevented the editors and writers of the newspaper
from publishing the same, lest he violate their constitutional right of
free expression.
Indeed, the report by the Philippine Star of the filing of the petition and
the reproduction of its contents was a legitimate exercise of press
freedom.
Judge Majaducon’s arguments why Pimentel was guilty of contempt:
For causing the publication, not withstanding the same was subjudice
For making derogatory remarks affecting his honor and integrity
For making it appear that an administrative case was filed against him
for gross ignorance of the law
This, he said, constituted malicious and false report which obstructed the
administration of justice.
It does not appear that Sen. Pimentel caused the publication of the
article. He had no right to choose which news would be printed;
rather, it is the publisher which decides which will be reported in the
broadsheet.
In doing so, it is allowed “the widest latitude of choice as to what items
should see the light of day so long as they are relevant to a matter of
public interest, pursuant to its right of press freedom.”
There is no showing that petitioner Pimentel, as representative of the
Committee, used the phrase to malign the trial court. Rather, it was
used to express what he believed as a violation of the basic principle
of separation of powers.
ROXAS v. ZUZUARREGUI (ARIELLE) The letter reads in part:
July 12, 2007 | Freedom of Speech “As an officer of the court, I am shocked beyond my
senses to realize that such a wrongful and unjust
PETITIONER: Romeo G. Roxas decision has been rendered with you no less as the
RESPONDENTS: Antonio de Zuzuarregui ponente.
We cannot fathom how such a decision could have
SUMMARY: Due to an unfavorable decision rendered by the been arrived at except through considerations other
Supreme Court, petitioner Atty. Roxas sent a letter to Assoc. than the pure merits of the case. Every law student
Justice Nazario, claiming that the decision was unjust and not reading through the case can see clearly how a brother
based on the merits of the case. The letter contained lawyer in the profession had been so short-changed by,
contemptuous remarks against the Supreme Court. For this, the ironically, the most sacred and highest institution in the
Supreme Court ordered Atty. Roxas to explain in writing why he administration and dispensation of justice.”
should not be held in contempt and be subjected to disciplinary In a Resolution, the Court En Banc ordered Atty. Romeo G.
action. For his part, Atty. Roxas argued that he was merely Roxas to explain in writing why he should not be held in
exercising his right to express a legitimate grievance and fair contempt and subjected to disciplinary action.
criticism of the Court’s ruling. Subsequently, Atty. Roxas submitted his written
explanation, which read:
SC held that the petitioner cannot escape liability by taking refuge “ With all due respect to this Honorable Court, and beyond
in the right to free speech, when such is used to spew my personal grievances, I submit that the ruling in the
contemptuous remarks against the Supreme Court when such subject consolidated cases may not have met the
attacks are warrantless and degrading. standards or adhered to the basic characteristics of fair
and just decision, such as objectivity, neutrality and
DOCTRINE: The making of contemptuous remarks conformity to the laws and the constitution. Aside from the
directed against the Court is not an exercise of free fact that the aforesaid ruling appears to be seriously
speech. Rather, it is an abuse of such right. flawed, it also casts grave aspersions on my personal and
professional integrity and honor as a lawyer, officer of the
FACTS: court and advocate of justice.”
In Atty. Roxas’ letter of explanation, the latter extended
In a previous decision rendered by the Supreme Court, apologies to Justice Nazario, to the other members of the
Atty. Roxas was ordered to return to Antonio High Court and said that he was merely exercising his right
to express a legitimate grievance and fair criticism of the
Zuzuarregui P17,073,224. He filed for motions for
Court’s ruling.
reconsideration but they were denied a few times for
lack of merit. ISSUE/s:
Because of this, Atty. Roxas was prompted to air his
grievances as to what he perceived as an unjust WoN petitioner can invoke his right to free speech and expression
judgment by Justice Nazario. in this case – NO.
Atty. Roxas sent a letter addressed to Assoc. Justice
Minita V. Chico-Nazario, with copies thereof furnished RULING: Atty. Roxas is guilty of contempt.
the Chief Justice and all the other Associate Justices,
RATIO:
intimated that Justice Nazario decided two cases on
considerations other than the pure merits of the case,
and called the SC a “dispenser of injustice.”
Atty. Roxas’ insistence that said decision did not meet the
standards or adhered to the basic characteristics of fair
and just decision, such as objectivity, neutrality and
conformity to the
laws and the Constitution, is simply without basis. The fact that
the decision was not in his favor does not mean that the same
was contrary to our laws and was not rendered in a fair and
impartial manner.
Atty. Roxas faulted the Supreme Court when two Motions for
Reconsiderations were unceremoniously denied via
Minute Resolutions without addressing at all the merits of
our very solid arguments. It is settled that the Court is not
duty-bound to render signed Decisions all the time. It has
ample discretion to formulate Decisions and/or minute
Resolutions, provided a legal basis is given.
To prevent liability, Atty. Roxas invokes his rights to free
speech. The invocation of this right will not free him
from liability. The making of contemptuous remarks
directed against the Court is not an exercise of free
speech. Rather, it is an abuse of such right.
Unwarranted attacks on the dignity of the courts cannot
be disguised as free speech, for the exercise of said
right cannot be used to impair the independence and
efficiency of courts.
Free expression must not be used as a vehicle to satisfy
one’s irrational obsession to demean, ridicule, degrade
and even destroy this Court and its magistrates.
The Court does not curtail the right of a lawyer, or any
person for that matter, to be critical of courts and
judges as long as they are made in proper and
respectful terms.
The Court held in Almacen that every citizen has the right to
comment upon and criticize the actuations of public officers.
This right is not diminished by the fact that criticism is aimed
at a judicial authority or that it is expressed by a lawyer. But
it is the cardinal condition of all such criticism that it shall be
bona fide and shall not spill over the walls of decency and
propriety.
The statements made by Atty. Roxas exceeded the
boundaries of decency and propriety. By his unfair and
unfounded accusation against Justice Nazario, and his
mocking of the Court for allegedly being part of a
wrongdoing and being a dispenser of injustice, he
abused his liberty of speech.
FORTUN v. QUINSAYAS (IYA) Court and its investigators to outside influence and public interference
February 13, 2013 | Carpio, J. | Freedom of Speech Fortun files a case against several persons, natural and juridical, which
engaged in acts contributory to the publication and spread of his
disbarment case whose defenses will be discussed in the ratio of this
PETITIONER: Philip Sigfrid A. Fortun
digest.
RESPONDENTS: Prima Jesusa B. Quinsayas, et al. ISSUE/s:
WON the respondents in this case violated the confidentiality rule in
SUMMARY: (homaygash my pamilee) disbarment proceedings, warranting a finding of guilt for indirect
Fortun is the lawyer of the Ampatuan’s who are the principal accused in contempt of court. - YES WITH RESERVATIONS (hehe)
the Maguindanao Massacre case. Several disbarment cases were filed
against Fortun for having used his knowledge of the technicalities of the RULING: Only Atty. Quinsayas is held responsible and guilty of contempt.
law to cause delay in the promulgation of justice in the Maguindanao
Massacre case.
RATIO:
Disbarment cases are confidential in nature, however, Atty. Quinsayas The court distinguishes criminal contempt from civil contempt.
caused the publication of the disbarment complaint through media CRIMINAL CIVIL
publications. Fortun filed CONTEMPT CONTEMPT
Directed against the dignity and authority of the It consists in the failing to do something
contempt against media groups and personalities involved.
court or a judge, it is na act obstructing the ordered to be done by a court in a civil action
administration of justice which brings the court for the benefit of the opposing party.
The SC absolved the media groups and media personalities on the basis into disrepute or disrespect.
that the case is of public interest as it is related to a sensitive and It is held to be an offense against public Proceedings to punish civil contempt are
scandalous matter. They were acting within their expected duty to deliver justice which raises an issue between he remedial and for the purpose of preservation
news to the public. Quinsayas was however held liable for contempt for public and the accused, and the of the right of private persons.
proceedings to punish it are punitive
causing the dissemination of the disbarment complaint.
The public circulation of the disbarment complaint against him exposed
DOCTRINE: In the absence of a legitimate public interest in a this
disbarment complaint, members of the media must preserve the
confidentiality of disbarment proceedings during its pendency.

FACTS:
Present case stems from the scandalous Maguindanao Massacre.
Petitioner in the present case, Fortun, is the counsel for Datu Ampatuan
who is the principal accused in the murder of 57 persons, 30 of
which were journalists.
In November of 2010, Atty. Quinsayas et. al. filed a disbarment complaint
against Fortun before the SC. The disbarment case is pending.
On the 22 November GMA News website posted an article on the
disbarment of Fortun which accuses him of “engaging in every
conceivable chi chancery or artifice to unduly delay the proceedings
by using and abusing legal remedies available.”
On 23 November, Channel 23 aired a program entitled “ANC Presents:
Crying for Justice: the Maguindanao Massacre” which allowed Atty.
Quinsayas to discuss the disbarment case against Fortun.
Fortun alleges that Quinsayas actively disseminated the details of the
disbarment complaint against him in violation of Rule 139-B of the
Rules Court on the confidential nature of disbarment proceedings.
Intent is a necessary element in criminal contempt. Intent is immaterial, good faith or absence of
intent
to violate the court’s order is not a defense in civil
contempt.

Defenses and arguments raised by the respondents:


GMA Network Inc.: It has no newspaper or any publication where
the article could be printed; it did not broadcast the disbarment
complaint in its televisions station; and that the publication was
already completed when Atty. Quinsayas distributed copies of
the disbarment complaint to the media. The Court ruled
however that online posting is already publication
considering it was done on GMA Network’s online news
website.
Philippine Daily Inquirer, Inc.: Inquirer.net has a separate
personality from it there being only syndication between PDI
and the website. The Court said that the website itself
recognizes that it is the official news website of PDI.
ABS-CBN Corporation: It alleges that SNN, which controls the
lineup of ANC, is a subsidiary of ABS and therefore has a
separate juridical personality. The Court agrees saying that
SNN is distinct from its parent company and that any suit
against the latter does not bind the former and vice-versa.
Maria Ressa: She alleges that she was on terminal leave and had
no hand in its production. This was not disputed by Fortun.
Dedace, Torres, Oreña-Drilon, and Punay: The disbarment
complaint was published wihhout any comment, in good faith
and
without malice. That the petitioner is a public figure; the
Maguindanao Massace is a matter of public interest.
For the above mentioned media groups and personalities, the Court
recognizes that “publications which are privileged for reasons of
public policy are protected by the constitutional guaranty of
freedom of speech.”
In this case, however, the filing of a disbarment complaint against
petitioner is itself a matter of public concern considering that it
arose from the Maguindanao Massacre case.
The interest of the public is not on petitioner himself but primarily on his
involvement and participation as defense counsel in the
Maguindanao Massacre case.
Since the disbarment complaint is a matter of public interest, legitimate
media had a right to publish such fact under freedom of the
press.
The Court also recognizes that respondent media groups and
personalities merely acted on a news lead they received when they
reported the filing of the disbarment complaint.
The distribution by Atty. Quinsayas to the media of the disbarment
complaint, by itself, is not sufficient to absolve the media from
responsibility for violating the confidentiality rule. However, since
petitioner is a public figure or has become a public figure because he
is representing a matter of public concern, the media has the right to
report the filing of the disbarment case as legitimate news.
In the absence of a legitimate public interest in a disbarment
complaint, members of the media must preserve the
confidentiality of disbarment proceedings during its pendency.
Other defenses of remaining respondents:
Mangdadatu, Ayon, Oquendo: They admitted that they were
some of the complainants in the disbarment complaint
against Fortun, however they allege that there was no proof
that they were the ones who disseminated the same. Fortun
failed to substantiate his claim against them.
De Jesus, Hulog, Batario, Mangahas, and Quinsayas: They allege
that Fortun was not able to establish the posting and publication
of the articles about disbarment and that assuming the same
had been established, he still failed to establish that they
disseminated the details. The Court ruled that, Atty.
Quinsayas alone, had a hand int the disseminationa nd
publication of the disbarment complaint and is responsible
for the distribution of the copies to the media. As a lawyer
and officer of the Court, he is expected to be familiar with
the confidential nature of disbarment proceedings. Instead
of preserving its confidentiality, he disseminated copies
which constitutes an act of contempt.
SANIDAD v. COMELEC (ELIEL) constitutional guarantees of the freedom of expression and of the
March 31, 1998 | Medialdea, J. | Freedom of Speech and Expression press enshrined in the Constitution.
Hence this petition.
PETITIONER: Pabilto V. Sanidad
RESPONDENTS: COMELEC

SUMMARY: An Oganic Act was enacted into law providing for a plebiscite for
the ratification of the Cordillera Autonomous Region. Comelec issued
Resolution 2167 on how the plebiscite shall be governed. Sec 19 of which,
prohibits columnists, and mass media participants during said periods.
Hence this appeal.

The SC ruled that although the Comelec may regulate the operations of
the media in terms of their free speech and press, they may only do so
under franchises, and other rights that may cause undue favor to
candidates. However, the regulation does not extend to plebiscites.
Because, a plebiscite is of public concern and importance. Much more is
that the people in the plebiscite are not candidates but people who have
the right to be informed. Although, Comelec spaces and airtime may be
used, it is not suffice since its scope is only limited to certain newspapers
and radio stations.

DOCTRINE: Plebiscite issues are matters of public concern and


importance. The people’s right to be informed and to be able to freely and
intelligently make a decision would be better served by access to an
unabridged discussion of the issues, including the forum.

FACTS:
RA 6766, entitled “AN ACT PROVDING FOR AN ORGANIC ACT FOR
THE CORDILLERA AUTONOMOUS REGION” was enacted into law.
Pursuant to said law, the City of Baguio and the Cordilleras which consist
of the provinces of Benguet, Mountain Province, Ifugao, Abra and
Kalinga-Aayao, all comprising the ordillera Autonomous Region, shall
take part in a plebiscite for the ratification of said Organic Act
originally scheduled last December 27,1989 which was, however,
rest to January 30, 1990 by virtue of Comelec Resolution No. 2226
dated December 27, 1989.
The Comelec promulgated Resolution No. 2167, to govern the conduct of the
plebiscite on the said Organic Act for the Cordillera Autonomous Region.
Sanidad, claims to be a newspaper columnist of the “OVERVIEW” for the
Baguio Midland Courier, a weekly newspaper circulated in the City of
Baguo and the Cordilleras assailed the constitutionality of Sec 19 of
Comelect Resolution No. 2167.
Said resolution prohibits columnists, commentators, or announcers
during the plebiscite campaign period. Alleging then that said
provision is void and unconstitutional because it violates the
ISSUE/s:
WoN Sec 19 of Comelec Resolution 2167 violates free speech and press

YES

RULING: Sec 19 is null and void.

RATIO:
The Comelec admits that it is a valid implementation of their power to
supervise and regulate media during election or plebiscite periods as
enunciate in Art IX-C, Sec 4 of the 1987 Constitution.
It is stated further by Comelec that Resolution 2167 does not absolutely
bar Sanidad from expressing his views and/or from campaigning for
or against the Organic Act. He may still express his views or
campaign for or against the act rhough the Comelec space and
airtime. This is provided under Secs. 90 and 92 of BP 881.
However, it is clear from Art IX-C that what was granted to the Comelec
was the power to supervise and regulate the use and enjoyment of
franchises, permits or other grants issued for the operation of
transportation or other public utilities, media of communication or
information to the end that equal opportunity, time and space, and
the right to reply, including reasonable, equal rates therefor, for
public information campaigns and forums among candidates are
ensured.
The evil sought to be prevented by this provision is the possibility
that a franchises holder may favor or give any undue advantage
to a candidate in terms of advertising space or radio or
television time.
However, neither Art IX-C nor Sec 11(b) of RA 6646 can be construed to
mean that the Comelec has alos been granted the right to supervise
and regulate the exercise by media practitioners themselves of their
right to expression during plebiscite periods.
Media practitioners exercising their freedom of expression during
plebiscite periods are neither the franchise holders nor the
candidates. In fact, there are no candidates involved in a plebiscite.
Therefore, Sec 19 of Resolution 2167 has no statutory basis.
In a plebiscite, votes are taken in an area on some special political
matter unlike in an election where votes are cast in favor of
specific persons for some office. In other words, the electorate
is asked to vote for or against issues, not candidates in a
plebiscite.
Comelec spaces and Comelec radio time may provide a forum for
expression but they do no guarantee full dissemination of
information to the public concerned because they are limited to
either specific portions in newspapers or to specifc radio or
television times.
NPC v. COMELEC (JP) Sec. 90 & 92 of the Omnibus Election Code require the COMELEC to
March 5, 1992 | Feliciano, J. | Freedom of Speech procure “Comelec space” in newspapers of general circulation
in every
PETITIONER: Naitonal Press Club
RESPONDENTS: Commission on Elections

SUMMARY: The three consolidated petitions by mass media


representatives assail as violative of the freedom of expression Sec. 11b
of RA 6646 (Electoral Reforms Law). Sec 11b prohibits the sale or
donation of print space and airtime for campaign purposes.

The SC held that Sec. 11b is constitutional because (1) it is limited in its
duration to the election period (2) it only covers paid political
advertisements not free opinions of media practitioners (3) COMELEC is
still mandated to distribute equitably the COMELEC space it is required to
buy (4) there are other options for politicians to advertise aside from
mass media.

DOCTRINE: The rights of free speech and free press are not unlimited
rights for they are not the only important and relevant values even in
the most democratic of polities. In our own society, equality of
opportunity to proffer oneself for public office, without regard to the
level of financial resources that one may have at one’s disposal, is
clearly an important value.

FACTS:
Three consolidated petitions are filed assailing the constitutionality of
Sec. 11(b) of RA 6646 as violative of the constitutional guarantee of
freedom of expression.
Petitioners in these cases consist of representatives of the mass media
(newspapers, radio, TV), two candidates for office and voters.
Section 11 (b) of Republic Act No. 6646, known as the Electoral Reforms
Law of 1987: it shall be unlawful; xxx b) for any newspapers, radio
broadcasting or television station, other mass media, or any person
making use of the mass media to sell or to give free of charge print
space or air time for campaign or other political purposes except to
the Commission as provided under Sections 90 and 92 of Batas
Pambansa Blg. 881 xxx
Section 11 (b) prohibits the sale or donation of print space and air
time for campaign or other political purposes except to the
COMELEC.
Section 11 (b) of Republic Act No. 6646 should be taken together with
Sections 90 and 92 of B.P. Blg. 881, known as the Omnibus Election
Code of the Philippines.
province or city and “Comelec time” on radio and television The Court considers that Section 11 (b) has not gone outside the
stations. Further, the Comelec is statutorily commanded to permissible bounds of supervision or regulation of media
allocate “Comelec space” and “Comelec time” on a free of operations during election periods.
charge, equal and impartial basis among all candidates within The restrictive impact upon freedom of speech and freedom of the press
the area served by the newspaper or radio and television station of
involved.
They assert that the prohibition is in derogation of media’s role and duty
to provide adequate channels of public information and public
opinion relevant to election issues.
Further, petitioners contend that Section 11 (b) abridges the freedom of
speech of candidates, and that the suppression of media-based
campaign or political propaganda would bring about a substantial
reduction in the quantity or volume of information concerning candidates
and issues in the election thereby curtailing and limiting the right of
voters to information and opinion.

ISSUE/s:
WoN Sec. 11(b) of RA 6646 invades and violates the constitutional
guarantees compromising freedom of expression - NO

RULING: Petitions should be dismissed.

RATIO:
The rights of free speech and free press are not unlimited rights for they
are not the only important and relevant values even in the most
democratic of polities.
In our own society, equality of opportunity to proffer oneself for public
office, without regard to the level of financial resources that one may
have at one’s disposal, is clearly an important value.
One of the basic state policies given constitutional rank by Article
II, Section 26 of the Constitution is the egalitarian demand that
the State shall guarantee equal access to opportunities for
public service and prohibit political dynasties as may be
defined by law.
Such supervision of COMELEC may result in some limitation of the rights
of free speech and free press. For supervision of operations of media
enterprises is scarecely conceivable without such accompanying
limitation.
Thus, the applicable rule is the general, time-honored one·that a statute
is presumed to be constitutional and that the party asserting its
unconstitutionality must discharge the burden of clearly and
convincingly proving that assertion.
Section 11 (b) is circumscribed by certain important limitations. There is here no censorship, whether disguised or otherwise. What
Firstly, Section 11 (b) is limited in the duration of its applicability Section
and enforceability. By virtue of the operation of Article IX (C) (4)
of the Constitution, Section 11 (b) is limited in its applicability in
time to election periods.
Secondly, and more importantly, Sec. 11(b) is limited in its purchase
and sale disguised as a donation, of print space and air time for
“campaign or other political purposes.” Section 11 (b) is
designed to cover only paid political advertisements of
particular candidates.
Section 11 (b) does not purport in any way to restrict the reporting by
newspapers or radio or television stations of news or news-worthy
events relating to candidates, their qualifications, political parties and
programs of government.
Moreover, Section 11 (b) does not reach commentaries and expressions
of belief or opinion by reporters or broadcasters or editors or
commentators or columnists in respect of candidates, their
qualifications, and programs and so forth, so long at least as such
comments, opinions and beliefs are not in fact advertisements for
particular candidates covertly paid for.
There is a third limitation upon the scope of application of Section
11 (b). Section 11 (b) exempts from its prohibition the purchase
by or donation to the COMELEC of print space or air time, which
space and time COMELEC is then affirmatively required to
allocate on a fair and equal basis, free of charge, among the
individual candidates for elective public offices in the province
or city served by the newspaper or radio or television station.
Some of the petitioners are apparently apprehensive that Comelec might
not allocate “Comelec time” or “Comelec space” on a fair and equal
basis among the several candidates. Should such apprehensions
materialize, candidates who are in fact prejudiced by unequal or
unfair allocations effected by Comelec will have appropriate judicial
remedies available, so long at least as this Court sits.
Until such time, however, the Comelec is entitled to the benefit of
the presumption that official duty will be or is being regularly
carried out.
The points that may appropriately be underscored are that Section 11 (b)
does not cut off the flow of media reporting, opinion or commentary
about candidates, their qualifications and platforms and promises.
Newspaper, radio broadcasting and television stations remain quite free
to carry out their regular and normal information and communication
operations.
11 (b), viewed in context, in fact does is to limit paid partisan political
advertisements to fora other than modern mass media, and to “Comelec
time” and “Comelec space” in such mass media.
Section 11 (b) does, of course, limit the right of free speech and of access
to mass media of the candidates themselves. The limitation, however,
bears a clear and reasonable connection with the constitutional
objective set out in Article IX (C) (4) and Article II (26) of the
Constitution.
For it is precisely in the unlimited purchase of print space and radio
and television time that the resources of the financially affluent
candidates are likely to make a crucial difference.
The objective which animates Sec. 11(b) is the equalizing of the
situations of the rich and poor candidates by preventing the former
from enjoying undue advantage offered by huge campaign war
chests.
Finally, the nature and characteristics of modern mass media,
especially electronic media, cannot be totally disregarded.
Realistically, the only limitation upon the free speech of candidates imposed
is on the right of candidates to bombard the helpless electorate with paid
advertisements commonly repeated in the mass media ad nauseam.
Gutierrez, Jr., Dissenting: Section 11 (b) of R.A. No. 6646 will certainly
achieve one result·keep the voters ignorant of who the candidates are
and what they stand for.
Cruz, Dissenting: The most important objection to Section 11 (b) is that it
constitutes prior restraint on the dissemination of ideas. It constitutes
censorship. All the channels of communication should be kept open to
ensure the widest dissemination of information bearing on the forthcoming
elections.
ADIONG v. COMELEC (HENRY) authorized posting areas that it fixes - NO
March 31, 1992 | Gutierrez, Jr., J. | Freedom of Speech
RULING: SC granted the petition. COMELEC Resolution 2347 is null and
PETITIONER: Blo Umpar Adiong void on constitutional grounds.
RESPONDENTS: COMELEC
RATIO:
SUMMARY: COMELEC issued Reso 2347, prohibiting the posting of The prohibition infringes on the citizen’s fundamental right of free speech
decals and stickers on “mobile” places, public or private, and limit their in Sec 4, Bill of Rights (1987 Consti). There is no public interest
location or publication to the authorized posting areas that it fixes. This is substantial enough to warrant the kind of restriction involved in this
being assailed by a neophyte senatorial candidate, petitioner Adiong, for case.
being violative of his freedom of speech. All of the protections expressed in the Bill of Rights are important but we
have accorded to free speech the status of a preferred freedom.
SC ruled that such Resolution is indeed unconstitutional. Posting of This qualitative significance of freedom of expression arises from the fact
decals and stickers in mobile places do not endanger any substantial that it is the matrix, the indispensable condition of nearly other
government interest. No public interest is being threatened. freedom.
The SC recognized that the Constitution gives the COMELEC during the
DOCTRINE: Under the clear and present danger rule, not only must the election period regulatory powers.
danger be patently celar and pressingly present but the evil sought to be The freedom of the citizen and the State’s power to regulate are not
avoided must be so substantive as to justify a clamo over one’s mouth or antagonistic. There can be no free and honest elections if in the
a writing instrument to be stilled. efforts to maintain them, the freedom to speak and the right to know
are unduly curtailed.
Regulation of election campaign activity may not pass the test of validity
FACTS:
if it is too general in its terms or not limited in time and scope in its
1992, COMELEC promulgated Resolution No. 2347, which Sec 21(f) is application, if it restricts one’s expression of belief in a candidate or
stated as follows: one’s opinion of his or her qualifications, if it cuts off the flow of media
Sec. 21. Prohibited forms of election propaganda – to draw, reporting, and if the regulatory measure bears no clear and
paint, inscfribe, post, display, or publicly exhibit any election reasonable nexus with the constitutionality sanctioned objective.
propaganda in any place, whether public or private, mobile A government regulation is sufficiently justified if it s within the
or stationary, except in the COMELEC common posted constitutional power of the Government, if it furthers an important or
areas and/or billboards, at the campaign headquarters of the substantial governmental interest; if the governmental interest is
candidate or political party, organization or coalition, or at the unrelated to the suppression of free expression; and if the incidental
candidate’s own residential house or one of his residential restriction on alleged First Amendment Freedoms is no greeater than
houses, if he has more than one: prvided that such posters is essential to the furtherance of that interest.
or election propaganda shall note exceed 2ft x 3ft in size. Posting of decals and stickers in mobile places like cars and other
Petitioner Adiong, a senatorial candidate in the 1992 elections assail the moving vehicles does not endanger any substantial government
COMELEC’s Resolution insofar as it prohibits the posting of decals interest. There is no clear public interest threatened by such activity.
and stickers in “mobile” places like cars and other moving vehicles. Under the clear and present danger rule, not only must the danger be
Petitioner believes that with the ban on radio, television and print political patently celar and pressingly present but the evil sought to be
advertisments, he, being a neophyte in politics stands to suffer a avoided must be so substantive as to justify a clamp over one’s
grave and irreparable injury with this prohibition. mouth or a writing instrument to be stilled.
Moreso, as of Feb 22 of the same year, he has not received any notice The regulations trikes the freedom of an individual to express his
from any of the Election Registrars in the entire country as to the preference and, by displaying it on his car, to convince others to
location of the supposed “Comelec Poster Areas”. agree with him.
Cruz, concurring: Instead of limiting the dissemination of information on
ISSUE/s:
election issues and the qualifications of those vying for public office,
WoN COMELEC may prohibit the posting of decals and stickers on “mobile” what the COMELEC should concentrate on is the education of the
places, public or private, and limit their location or publication to the voters on the proper exercise of their suffrages. The real threat in the
present election is the influx of the unqualified professional
entertainers whose only asset is the support of their drooling fans.
ABS-CBN v. COMELEC (DANNAH) issuance thereof was “pursuant to its constitutional and statutory
January 28, 2000 | Panganiban | Freedom of Speech and of the powers to promote a clean, honest, orderly and credible May 11,
Press 1998 elections”; and “to protect, preserve and maintain the secrecy
and sanctity of the ballot.”
PETITIONER: ABS-CBN Broadcasting Corp. It also wanted to avoid confusing and influencing voters, because the
RESPONDENTS: Commission on Elections surveys were designed “to condition the minds of people and cause
confusion as to who are the winners and the [losers] in the election”
SUMMARY: COMELEC issued a Resolution prohibiting ABS-CBN and
other groups from conducting exit surveys. ABS-CBN then filed a Petition which in turn may result in “violence and anarchy.”
for Certiorari with the SC, and the SC issued the TRO prayed for. Moreover, they claim that exit surveys indirectly violate the constitutional
COMELEC believes that the secrecy and sanctity of the ballot will be principle to preserve the sanctity of ballots as the voters are lured to
compromised because of said exit polls. ABS-CBN however argues that reveal the contenst of the ballots violationg Section 2 Article V of the
the holding of said polls entails a valid exercise of freedom of speech and Constitution.
of the press. The SolGen adds that the exit polls pose a “clear and present danger of
The Court ruled in favor of ABS-CBN, stating that since there was prior destroying the credibility and integrity of the electoral process”
restraint, the burden to show that the national interest outweights the considering that they are not supervised by any government agency
freedom of speech is with the COMELEC. Applying the clear and present and can in general be manipulated easily.
danger test, the evil that COMELEC seeks to curtail is at best,
speculative. There hasn’t been proof to show that exit polls will indeed ISSUE/s:
affect the sanctity of the ballots. WoN COMELEC may, in the exercise of its powers, totally ban exit polls

DOCTRINE: The Court adheres to the clear and present danger test. The NO
question in every case is whether the words used are used in such
circumstances and are of such a nature as to create a clear and present RULING: Petition granted. TRO issued by the Court made permanent
danger that they will bring about the substantive evils that Congress has
a right to prevent. It is a question of proximity and degree. RATIO:
Although the election has already been held with, the petition is not moot
The freedoms of speech and of the press should all the more be upheld because it has implications on people’s fundamental freedom of
when what is sought to be curtailed is the dissemination of information expression.
meant to add meaning to the equally vital right of suffrage. The Court has the duty to formulate guiding and controlling constitutional
principles, precepts, doctrines, or rules. It has the symbolic function
FACTS: of educating bench and bar on the extent of protection given by
COMELEC issued a Resolution approving the issuance of a restraining constitutional guarantees.
order to stop ABS-CBN (ABS) or other groups from conducting exit The requirement on exhausting available remedies may be laxed to
surveys. ABS then filed a Petition for Certiorari with the SC.
prevent miscarriage of justice. In the case at hand, ABS had no
The Resolution was issued allegedly upon “information from a reliable
source that ABS has prepared a project with PR groups to conduct opportunity to move for a reconsideration and obtain a swift
radio-TV coverage of elections and to make an exist survey of the resolution in time for the May 1998 elections. Also, it involves
votes which would be broadcasted immediately” transcendental issues.
COMELEC believed that the project would conflict with the official An exit poll is a species of electoral survey conducted by qualified
COMELEC count. individuals or groups of individuals for the purpose of determining the
On May 9, the court issued a TRO prayed for by ABS. The Court also
probable result of an election by confidentially asking randomly
directed COMELEC to cease and desist from implementing the
Resolution or the restraining order. selected voters whom they have voted for, immediately after they
ABS argues that the holding of exit polls and the reporting of results are have officially cast their ballots.
valid exercises of the freedoms of speech and of the press. The results are announced to the public through mass media to give an
COMELEC denies that it gravely abused its discretion. It insists that the advance overview of how, in the opinion of the polling individuals or
organizations, the electorate voted. In our electoral history, exit polls
had not been resorted to until the recent May 11, 1998 elections.
Freedom of expression is a preferred right, and stands on a higher level
than
substantive economic or other liberties. Our Constitution mandates because exit polls generate important research data which may
that no law shall be passed abridging the freedom of speech or of be used to study influencing factors and trends in voting
the press. behavior.
In Gonzales v. COMELEC, the Court enunciated that at the very least, COMELEC’s arguments are purely speculative and clearly untenable
free speech and a free press consist of the liberty to discuss publicly because
and truthfully any matter of public interest without prior restraint.
In Cabansag v. Fernandez, the Court issued two theoretical tests:
Clear and present danger rule – The evil consequence of the
comment or utterance must be extremely serious and the degree
of imminence extremely high before the utterance can be
punished. The danger to be guarded against is the “substantive
evil” sought to be prevented
Dangerous tendency rule – If the words uttered create a dangerous
tendency which the state has a right to prevent, then such words
are punishable. It is sufficient if the natural tendency and probable
effect of the utterance be to bring about the substantive evil which
the legislative body seeks to prevent
The Court adheres to the clear and present danger test. The question in
every case is whether the words used are used in such
circumstances and are of such a nature as to create a clear and
present danger that they will bring about the substantive evils
that Congress has a right to prevent. It is a question of
proximity and degree.
A limitation on the freedom of expression may be justified only by a
danger of such substantive character that the state has a right to
prevent. Unlike in the dangerous tendency doctrine, the danger must
not only be clear but also present.
The power to exercise prior restraint is not to be presumed; rather the
presumption is against its validity. And it is COMELEC’s burden to
overthrow such presumption.
To justify a restriction, the promotion of a substantial government interest
must be clearly shown.
The freedoms of speech and of the press should all the more be upheld
when what is sought to be curtailed is the dissemination of
information meant to add meaning to the equally vital right of
suffrage.
There can be no free and honest elections if, in the efforts to maintain
them, the freedom to speak and the right to know are unduly
curtailed.
In order to justify a restriction of the people’s freedoms of speech and of
the press, the state’s responsibility of ensuring orderly voting must
far outweigh them. These freedoms have additional importance,
By the very nature of a survey, the interviewees are selected at random
The survey result is not meant to replace or be at par with the official
COMELEC count; it consists merely of the opinion of the polling group
Not at stake here are the credibility/integrity of elections, which are exercises
that are separate and independent from exit polls
In Daily Herald Co. v. Munro, the US SC held that a statute which
prevented the broadcasting of early returns was unconstitutional. The
general interest of the State in insulating voters from outside influences is
insufficient to justify speech regulation. Hence, regulating speech via an
exit poll restriction is evidently impersmissible.
ABS-CBNs methodology is as follows:
Communities are randomly selected in each province
Residences are also chosen at random
Only individuals who have voted are interviewed
The interviewers use no cameras
Poll results are released only on the day after the elections
The interest of the state in reducing disruption is outweighed by the drastic
abridgment of the constitutionally guaranteed rights of the media and the
electorate.
Kapunan, Dissenting: COMELEC has not declared exit polls to be illegal.
The rule that the restrictions to freedom of expression would be
burdened with a presumption of invalidity does not apply where, as in this
case, COMELEC exercised its Constitutional functions of securing the
secrecy and sancity of ballots (citing NPC v. COMELEC). It holds that if
the right to free speech collides with a norm of constitutional stature, the
rule on heavy presumption of invalidity does not apply. Hence, if no
presumption of invalidity arises, there is no occasion for the application of
the clear and present danger test.
Vitug, Separate: The right to information and free speech is not illimitable
and immune from the valid exercise of an ever demanding and pervasive
police power.
SWS v. COMELEC (ARIELLE) “evils” sought to be prevented; and 3) the impairment of freedom of
May 5, 2001 | Mendoza, J. | Freedom of Speech expression is minimal - limited both in duration (15 days and 7 days)
and scope (not prohibiting election surveys, but only requires
PETITIONER: Social Weather Stations timeliness).
RESPONDENTS: COMELEC
ISSUE/s:
SUMMARY: Petitioner seeks to enjoin the COMELEC from enforcing WoN Section 5.4 of RA 9006 is unconstitutional for violating the right
Section 5.4 of RA 9006 which prohibits conducting election surveys to freedom of speech and of the press – YES
for national candidates 15 days before and election and surveys for
local candidates 7 days before an election. Petitioner claims that RULING:
such provision is violative of the constitutionally guaranteed right to Section 5.4 of RA No. 9006 constitutes an unconstitutional abridgement of
free speech and press. COMELEC justified such restriction on the freedom of speech, expression, and the press.
ground that it is necessary to prevent the manipulation of the
electoral process by unscrupulous and erroneous surveys just before RATIO:
the election, as it creates pressure on the voters, resulting to a Section 5.4 lays a prior restraint on freedom of speech by prohibiting
bandwagon mentality. the publication of election survey results within the prescribed
periods. Because of the preferred status of the constitutional rights of
DOCTRINE: The constitutional guarantee of freedom of expression speech, expression, and the press, such a measure is vitiated by a
means that “the government has no power to restrict expression weighty presumption of invalidity.
because of its message, its ideas, its subject matter, or its content.” The grant of power to the COMELEC under Art. IX-C, Section 4, is limited
Section 5.4 aims to prevent last-minute pressure on voters, creating to ensuring “equal opportunity, time, space, and the right to reply.”
the bandwagon effect, but these cannot be attained at the sacrifice Justice Kapunan disagrees. He rejects the clear and present danger test
of the fundamental right of expression, when such aim can be as inapplicable for determining the validity of Section 5.4. While it
pursued by punishing unlawful acts, rather than speech. may be useful for determining the validity of laws dealing with inciting
to sedition or incendiary speech, it may not be adequate for the
regulation in question. Instead, Justice Kapunan is more inclined to a
form of balancing by “weighing and balancing the circumstances to
FACTS: determine whether public interest is served by the regulation of the
Petitioner Social Weather Station, is a social research institution free enjoyment of rights.”
conducting surveys in various fields. Petitioners seek to enjoin However, the Court believed that the test used by Justice Kapunan was
the COMELEC from enforcing Section 5.4 of RA 9006, which at best a “balancing at the margin.”
provides that surveys affecting national candidates shall not be The O’Brien test may be used - a government regulation is justified if 1)
published 15 days before an election and surveys affecting local it is within the constitutional power of the Government; 2) it
candidates shall not be published 7 days before an election. furthers an important or substantial governmental interest; 3)
Election surveys refer to the measurement of opinions and perceptions of the governmental interest is unrelated to the suppression of free
the voters as regards a candidate’s popularity, qualifications, platforms or expression; and 4) the incidental restriction on alleged First
a matter of public discussion. Amendment freedoms is not greater than it is essential to the
Petitioner wants to conduct an election survey throughout the period furtherance of that interest.
of the elections both at the national and local levels. Petitioners Under this test, even if a law furthers an important or substantial
argue that the restriction on the publication of election survey governmental interest, it should be invalidated if such
results constitutes a prior restraint on the exercise of freedom of governmental interest is “not unrelated to the suppression of
speech without any clear and present danger to justify such free expression.” Moreover, even if the purpose is unrelated to the
restraint. suppression of free speech, the law should still be invalidated if the
COMELEC justified the restrictions imposed as necessary to prevent the restriction on freedom of expression is greater than is necessary to
manipulation and corruption of the electoral process by achieve the governmental purpose in question.
unscrupulous and erroneous surveys just before the election. It Section 5.4 fails to meet the third requisite of the O’Brien test
contends that 1) the prohibition during the period prescribed by law because the connection of expression to the asserted governmental
bears a rational connection to the objective of the law; 2) it narrowly interest is not unrelated to the suppression of free expression.
tailored to meet the
Section 5.4 actually suppresses a whole class of expression, while
allowing the expression of opinion on the same subject by
columnists, radio commentators, etc.
The constitutional guarantee of freedom of expression means that “the
government has no power to restrict expression because of its There is no compelling or justifiable reason for the prohibition made by
message, its ideas, its subject matter, or its content.” Congress under the assailed law. The Comelec also utterly fails to
Contrary to the claim of the Solicitor General, the prohibition cannot be convince
justified on the ground that it is only for a limited period and is 9. only
incidental. The prohibition may be limited, but the curtailment of the
right of expression is direct, absolute and substantial.
Even if the governmental interest sought to be promoted is unrelated to
the suppression of speech and the resulting restriction of free
expression is only incidental, Sec. 5.4 nonetheless fails to meet
criterion of the O’Brien test, namely, that the restriction be not greater
than is necessary to further the governmental interest. Section 5.4
aims to prevent last-minute pressure on voters, creating the
bandwagon effect, but these cannot be attained at the sacrifice of
the fundamental right of expression, when such aim can be
pursued by punishing unlawful acts, rather than speech.

Melo, Concurring
Freedom of speech not only includes the right to express one’s views,
but also other cognate rights relevant to the free communication of
ideas, including the right to be informed on matters of public concern.
However, despite the primacy of free expression in the hierarchy of civil
liberties, the same is not absolute. It can be regulated, but regulation
must be reasonable. It must be shown that the interest of the public
requires such regulation. It must also appear that the means used
are necessary for its accomplishment.
The provision in dispute constitutes prior restraint. Respondent has fallen
short of the required effort to overthrow the presumption of validity, it
having
failed to show that the means used by Section 5.4 are reasonably
necessary to accomplish the purpose.
Puno, Concurring
Freedom of speech and of the press enjoys a preferred status in our
hierarchy of rights because the preservation of other rights depends
on how well we protect our freedom of speech and of the press.
The provision is unconstitutional because it constitutes a clear prior
restraint on petitioners’ freedom of speech and of the press. Stress
must be given to the prohibition against prior restraint because 1) a
historical study of human rights will show that it is prior restraint that
gave rise to freedom of speech and of the press; and 2) there is a
growing tendency for governments to manipulate the free market of
ideas in the guise of merely regulating the manner of exercising free
speech and press.
The criminal sanction for the violation of the ban is pure and simple prior
restraint on the communication and free flow of ideas which should be
made available to voters before they exercise their right of suffrage.
Prior restraint
can be justified only on the narrowest ground like national security.
Panganiban, Concurring
that a substantive danger, lies lurking and threatening to explode
if ignited by the conduct and the dissemination of the prohibited
surveys.
Kapunan, Dissenting
Although among our most cherished rights, the freedoms of speech
and of the press are not absolute or unlimited.
The dangerous tendency rule provided that the State has the power
to proscribe and punish speech which creates a dangerous
tendency which the State has a right to prevent.
The clear and present danger rule postulates that the question in
every case is whether the words are used in such
circumstances and are of such nature as to create a clear and
present danger that they will bring about the substantive evils
that Congress has the right to prevent.
It is ultimately this Court’s function and duty to undertake the
delicate and difficult task of weighing and balancing the
circumstances to determine whether public interest is served by
the regulation of the free enjoyment of the rights.
Section 5.4 is a mere restriction, not an absolute prohibition, on
the publication of election surveys. It is limited in duration; it
applies only during the period when the voters are presumably
contemplating whom they should elect.

*Dagdag-bawas, a phenomenon peculiar to Philippine elections,


takes place when votes cast in favor of one candidate are deducted
then credited to another.
SWS v. COMELEC (IYA) Because of non-compliance, after having been send a Notice, the
April 7, 2015 | Leonen, J. | Freedom of Expression COMELEC issued a Subpoena for violation of the Omnibus Election
Code in relation to the Fair Elections Act.
PETITIONER: Social Weather Stations, Inc. and Pulse Asia, Inc.
RESPONDENTS: Commission on Elections

SUMMARY: COMELEC is requiring SWS and Pulse Asia to disclose the


commissioners of their published surveys by virtue of a Resolution.
COMELEC asserts that the disclosure is in accordance with he Fair
Elections Act because it affects the voting public’s decision making.

SWS and Pulse allege that such is ultra vires and that it limits their
freedom of speech.

The SC ruled that the freedom to publish surveys remain, it merely


regulates the manner of publication.

DOCTRINE: The freedom to publish election surveys remains. All


Resolution No. 9674 does is articulate a regulation as regards the
manner of publication, that is, that the disclosure of those who
commissioned and/or paid for, including those subscribed to, published
election surveys must be made.

FACTS:
SWS and Pulse Asia are social research and public polling firms who
conduct preelection surveys.
They conducted a reelection survey on voters’ preferences for senatorial
candidates and published their findings.
Representative Tobias Tiangco, Secretary-General of the UNA Partylist,
wrote to COMELEC to compel SWS to comply with the directive in
the Fair elections Act and COMELEC Resolution No. 9615 and give
the names or identities of the subscribers who paid for the survey
conducted.
Tiangco wrote to SWS to comply with the same. To which SWS replied
but did not identify the persons who commissioned or subscribed to
the survey.
Acting on the letter of Tiangco’s letter COMELEC issued an Oder setting
the matter for hearing. It later on issued Resolution No. 9674, which
directs SWS, Pluse Asia, and other survey firms to submit the names
of all commissioners and payors.
In issuing the same COMELEC cited the Constitution and the Fair
Elections Act.
SWS and Pulse Asia alleges that they had not been furnished copies of
the said Resolution and articulated their view that the Resolution was
tainted with irregularities for having been issued ultra vires.
SWS and Pulse maintains that before the subpoena they were never
informed of the criminal case field against them.
Hence, the present petition by SWS and Pulse Asia assailing the
Resolution for being ultra vires.
They posit that the Resolution requiring the submission of information is
in excess of wha the Fair Election Act requires (ultra vires).
They pray for the issuance of a temporary restraining order/or writ of
preliminary injunction in the interim.
ISSUE/s:
WON the Resolution is invalid in that it requires the disclosure of the
names of subscribers of election surveys - NO
WON the rights of petitioners to free speech will be curtailed by the
requirement to submit the names of their subscribers - NO

RULING: Petition is PARTLY GRANTED. COMELEC is enjoined from


prosecuting SWS and Pulse because it violated due process.

RATIO:
The Court sustains the validity of Resolution No. 9674. The names of
those who commission or pay for election surveys must be disclosed
pursuant ot the Fair Election Act.
The requirement is a valid exercise of police power and effects the
constitutional policy of guaranteeing equal access to opportunities for
public service.
However, it is evident that the Resolution was promulgated in violation of
the period set by the Fair Elections Act. SWD and Pulse were
likewise not served a copy of the Resolution with which it was asked
to comply. Therefore, violating their right to due process.
The Resolution is not ultra vires or exceeds the authorization given by
the Fair Elections Act.
The inclusion of the election surveys in the list of items regulated
by the Fair Elections Act is a recognition that election
surveys are not a mere descriptive aggregation of data.
Publishing surveys are a means to shape the preference of
voters.
Election surveys amplify the notion of na election as a “horse race”. The
following are the effects on voter behavior:
Bandwagon effect - electors would follow the tide of whoever is
leading in the polls.
Underdog effect - sympathy can shift to the candidate trailing
Motivating effect - an individual not intending to vote may be
pursuaded to
Demotivating effect - individuals may opt to abstain
Strategic Voting - voting is influenced by the chances of winning
Free-will effect - voters cast their ballots to prove the polls wrong.
The Fair Elections Act must be appreciated for what it is: a mechanism
for ensuring equality:
Diocese of Bacolod v COMELEC: In other words, abstract
guarantees of fundamental rights like freedom ofexpression
may become meaningless if not taken in a real context.
The nature of the speech involved, as well as the Fair Elections Act’s
purpose of ensuring political equality, calls into operation the equality-
based approach to weighing liberty to express vis-à-vis equality of
opportunities.
Diocese: “politically disadvantaged speech prevails over regulation,
but regulation promoting political equality prevails over speech.”
Osmeña v COMELEC and National Press Club v COMELEC:
“Thus clearly, regulation of speech in the context of electoral
campaigns made by candidates or the members of their
political parties or their political parties may be regulated as
to time, place, and manner.”
Published election surveys partake the nature of election propaganda.
Resolution No. 9674 is grounded in the Fair Election Act’s requirement of
disclosure of commissioners, there is therefore a valid basis on a
statute.
Not only is there substantial state interest but a compelling on which
requires such Resolution.
The Resolution is likewise narrowly tailored to meet the objective of
enhancing the opportunity of all candidates to be heard and
considering the primacy of the guarantee of free expression and is
the least restrictive means to achieve that object.
The freedom to publish election surveys remains. All Resolution No.
9674 does is articulate a regulation as regards the manner of
publication, that is, that the disclosure of those who commissioned
and/or paid for, including those subscribed to, published election
surveys must be made.
Peititoners claim that the requirement constitutes a prior restraint.
Prior restraint refers to official governmental restrictions on the
press or other forms of expression in advance of actual
publication or dissemination.
Freedom from prior restraint is largely freedom from government
censorship of publications, whatever the form of censorship,
and regardless of whether it is wielded by the executive,
legislative or judicial branch of the government.
The very definition provided above negates the claims of SWS. There is
no prohibition or censorship aimed at election surveys.
OSMENA v. COMELEC (ELIEL) The term political “ad ban,” when used to describe Sec 11(b) of RA
March 31, 1998 | Mendoza, J. | Freedom of Speech and Expression 6646, is misleading, for even as Sec 11(b) prohibits the sale or
donation of print space and air time to political candidates, it
PETITIONER: Emilio Osmena and Pablo Garcia mandates the COMELEC to procure
RESPONDENTS: COMELEC

SUMMARY: Osmena and Garcia are candidates vying for the respective
positions of President and Governer of Cebu. They assail the validity of
Sec 11(b) of RA 6646 as violative of their rights to free speech and press,
because it prohibits mass media from selling or giving free of charge print
space or air time for campaign and political purposes.

The SC held that there is no total ban on political campaigning; rather it is


only a regulatory policy to ensure equality, honesty, orderly, and peaceful
elections. If there is any restriction as to the freedom of speech and
press, it is merely incidental. For as long as the regulation is reasonable,
it is a valid exercise of police power. Moreover, the state and the
COMELEC has the power to intiate the regulation. Any irregularity, the
Congress is vested with authority to provide a remedy. Absent such, the
fundamental law should be respected.

DOCTRINE: The State can prohibit campaigning outside a certain period


as well as campaigning within a certain place. For unlimited expenditure
for political advertising in the mass media skews the political process and
subverts democratic self-government. What is bad is if the law prohibits
campaigning by certain candidates because of the views expressed in
the ad. Content regulation cannot be done in the absence of any
compelling reason.

FACTS:
This is a petition for prohibition, seeking a reexamination of the validity of
Sec 11(b) of RA No. 6646, the Electoral Reforms Law of 1987, which
prohibits mass media from selling or giving free of charge print space
or air time for campaign or other political purposes, except to the
COMELEC.
Osmena and Garcia are candidates for public office in the forthcoming
elections. They contend that events after the ruling in NPC v.
COMELEC “having called into question the validit of the premises of
that decision.

ISSUE/s:
WoN Sec 11(b) of RA No. 6646 violates free speech and press – NO

RULING: Dismissed.

RATIO:
and itself allocate to the candidates space and time in the media. There is no
suppression of political ads but only a regulation of the time and manner of
advertising.
The law’s concern is not with the message or content of the ad but with
ensuring media equality between candidates with “deep pockets,” and
those with less resources.
There is a differene in kind and in severity between restrictions such as those
imposed by the election law provisions in question in this case and those
found to be unconstitutional in the cases cited.
Here, on the other hadn, there is no total ban on political ads, much less restriction
on the content of the speech. Given the fact that print space and airtime can be
controlled or dominated by rich candidates to the disadvantage of poor
candidates, there is a substantial or legitimate governmental interest justifying
exercise of the regulatory power of the COMELEC.
The provisions in question involve no suppression of political ads. They only
prohibit the sale or donation of print space and dair time o candidates but
require the COMELEC instead to procure space and time in the ass media
for allocation, free of charge, to the candidates. In effect, during the election
period,, the COMELEC takes over the advertising page of newspapers or the
commercial time of radio and TV stations and allocates these to the
candidates.
Instead of leaving candidates to advertise freely in the mass media, the law
provides for allocation, by the COMELEC, of print spae and air time to give
all candidates equal time and space for the purpose of ensuring “free,
orderly, honest, peaceful, and credible elections.”
The State can prohibit campaigning outside a certain period as well as
campaigning within a certain place. For unlimited expenditure for political
advertising in the mass media skews the political process and subverts
democratic self-government. What is bad is if the law prohibits campaigning
by certain candidates because of the views expressed in the ad. Content
regulation cannot be done in the absence of any compelling reason.
The main purpose of Sec 11(b) is regulatory. Any restriction on speech is
only incidental, and it is no more than is necessary to achieve its
purpose of promoting equality of opportunity in the use of mass media
for political advertising. The restriction on speech, as pointed out in
NPC, is limited both as to time and as to scope.
What makes the regulation reasonable is precisely that it applies only to the
election period.
The notion of the government may restrict the speech of some in order to
enhance the relative voice of others may be foreign to the American
Constitution. It is not to the Philippine Constitution, being inf act an animating
principle of that document.
Well-settled is the rule that the choice of remedies for an admitted social
malady requiring government action belongs to Congress. The remedy
prescribed by it, unless clearly shown to be repugnant to fundamental law,
must be respected. As shown in this case, Sec 11(b) of RA 6646 is a
permissible restriction on the freedom of speech, of expression and of the
press.
Adiong v. COMELEC, A government regulation is sufficiently justified if it
is within the constitutional power of the Government, if it further an
important or substantial governmental interest; if the governmental
interest is unrelated ot the suppression of ree expression; and if the
incident restriction ofn alleged 1st amendment freedoms is no greater
than is essential to the furtherance of that interest.
Sec 11(b) of RA 6646 is a valid exercise of the power of the State to
regulate media of communication or information for the
purpose of ensuring equal opportunity, time and space for
political campaigns; that the regulation is unrelated t the
suppression of speech; that any restriction on freedom of
expression is only incidental and no more than is necessary to
achieve the purpose of promoting equality.
Panganiban, J. Dissenting: He invokes the clear and present danger
test and argues that “media ads do not partake of the ‘real
substantive evil’ that the state has a right to prevent and that justifies
the curtailment of the people’s cardinal right to coose their means of
expression and of access to information.” The clear and present
danger test is not, however, a sovereign remdy for all free speech
problems.
DIOCESE OF BACOLOD v. COMELEC (JP) Those who voted for the passing of the law were classified by
January 21, 2015 | Leonen, J. | Freedom of Speech petitioners as comprising “Team Patay,” while those who voted
against it form “Team Buhay.”
PETITIONER: The Diocese of Bacolod, Rev. Bishop Vicente M. Navarra Respondents conded that the tarpaulin was neither sponsored nor paid for by
RESPONDENTS: COMELEC, Atty. Mavil Majarucon

SUMMARY: Petitioner Diocese of Bacolod posted two large tarpaulins on


the front walls of the Bacolod cathedral within public view. The second
tarpaulin tabulated electoral candidates according to their votes in the RH
Law. The COMELEC law department asked them to take down said
tarpaulins mainly for violating the size requirements of electoral ads.
Petitioner asked for a TRO.

The SC granted such petition and held that regulation of speech in the
context of electoral campaigns made by persons who are not candidates
during elections, is unconstitutional (with exceptions as stated in Doctrine
part). Taken as a whole, the principal advocacies of such speech are
social issues, not to garner votes.

DOCTRINE: Regulation of election paraphernalia by persons who are not


candidates or who do not speak as members of a political party will be
constitutionally valid, only if what is regulated is declarative speech that,
taken as a whole, has for its principal object the endorsement of a
candidate only.

The regulation (a) should be provided by law, (b) reasonable, (c) narrowly
tailored to meet the objective of enhancing the opportunity of all
candidates to be heard and considering the primacy of the guarantee of
free expression, and (d) demonstrably the least restrictive means to
achieve that object.

FACTS:
Petitioners posted two tarpaulins within a private compound housing the
San Sebastian Cathedral of Bacolod.
Each tarpaulin was approximately six feet by ten feet in size. They were
posted on the front walls of the cathedral within public view.
The first tarpaulin contains the message “IBASURA RH Law” referring to
the Reproductive Health Law of 2012 or Republic Act No. 10354. The
second tarpaulin is the subject of the present case.
This tarpaulin contains the heading “Conscience Vote” and lists
candidates as either “(Anti-RH) Team Buhay” with a check mark,
or “(Pro- RH) Team Patay” with an “X” mark.
The electoral candidates were classified according to their vote on
the adoption of Republic Act No. 10354, otherwise known as the
RH Law.
any candidate.
Atty. Mavil V. Majarucon, in her capacity as Election Officer of Bacolod
City, issued a Notice to Remove Campaign Materials addressed to
petitioner Most Rev. Bishop Vicente M. Navarra.
The election officer ordered the tarpaulins removal within three (3) days
from receipt for being oversized. COMELEC Resolution No. 9615
provides for the size requirement of two feet by three feet.
COMELEC Law Department issued a letter ordering the immediate
removal of the tarpaulin; otherwise, it will be constrained to file an
election offense against petitioners.
Concerned about the imminent threat of prosecution for their exercise of
free speech, petitioners initiated this case through this petition for
certiorari and prohibition with application for preliminary injunction
and temporary restraining order.

ISSUE/s:
WoN the COMELEC has the competence to limit expressions made by
citizens who are not candidates during elections – NO
WoN the tarpaulins are a form of expression (protected speech), or
election propaganda/political advertisement – PROTECTED
SPEECH
WoN this form of expression may be regulated - NO

RULING: Instant Petition is granted. TRO is hereby made permanent. The


COMELEC in issuing the assailed notice is declared unconstitutional.
RATIO:
Labelling all expressions of private parties that tend to have an effect on
the debate in the elections as election paraphernalia would be too
broad a remedy that can stifle genuine speech like in this case.
Instead, to address this evil, better and more effective enforcement will
be the least restrictive means to the fundamental freedom.
The message of petitioners in this case will certainly not be what
candidates and political parties will carry in their election
posters or media ads. The message of petitioner, taken as a
whole, is an advocacy of a social issue that it deeply believes.
Through rhetorical devices, it communicates the desire of Diocese that
the positions of those who run for a political position on this social
issue be determinative of how the public will vote. It primarily
advocates a stand on a social issue; only secondarily, even
almost incidentally, will cause the election or nonelection of a
candidate.
The twin tarpaulins consist of satire of political parties. Satire is a “literary
form that employs such devices as sarcasm, irony and ridicule to
deride prevailing vices or follies.”
Satire frequently uses exaggeration, analogy, and other rhetorical Regulation of speech in the context of electoral campaigns made by
devices for political and social criticism. candidates or the members of their political parties or their
The tarpaulins exaggerate. Surely, “Team Patay” does not refer to a list of political parties may be regulated as to time, place, and manner.
dead individuals nor could the Archbishop of the Diocese of Bacolod This is the effect of our
have intended it to mean that the entire plan of the candidates in his
list was to cause death intentionally.
The list of “Team Patay” is juxtaposed with the list of “Team Buhay”
that further emphasizes the theme of its author: Reproductive
health is an important marker for the church of petitioners to
endorse.
The messages in the tarpaulins are different from the usual messages of
candidates.
Election paraphernalia from candidates and political parties are
more declarative and descriptive and contain no sophisticated
literary allusion to any social objective.
Thus, they usually simply exhort the public to vote for a person with a
brief description of the attributes of the candidate. For example “Vote
for [x], Sipag at Tiyaga,” “Vote for [y], Mr. Palengke,” or “Vote for [z],
Iba kami sa Makati.”
Also, considerations of equality of opportunity or equality in the ability
of citizens as speakers should not have a bearing in free speech
doctrine.
Members of the public are trusted to make their own individual
evaluations of speech, and government is forbidden to
intervene for paternalistic or redistributive reasons . . . [thus,]
ideas are best left to a freely competitive ideological market.
The textual basis of this view is that the constitutional provision uses
negative rather than affirmative language. It uses ‘speech’ as its
subject and not ‘speakers.’
Consequently, the Constitution protects free speech per se,
indifferent to the types, status, or associations of its speakers.
Pursuant to this, government must leave speakers and listeners
in the private order to their own devices in sorting out the
relative influence of speech.
Freedom of speech includes “not only the right to express one’s views,
but also other cognate rights relevant to the free communication [of]
ideas, not excluding the right to be informed on matters of public
concern.”
The traditional view has been to tolerate the viewpoint of the speaker and the
content of his or her expression. This view, thus, restricts laws or
regulation that allows public officials to make judgments of the value of
such viewpoint or message content. This should still be the principal
approach.
rulings in Osmeña v. COMELEC and National Press Club v. COMELEC.
Regulation of speech in the context of electoral campaigns made by
persons who are not candidates or who do not speak as members
of a political party which are, taken as a whole, principally
advocacies of a social issue that the public must consider during
elections is unconstitutional.
Such regulation is inconsistent with the guarantee of according the fullest
possible range of opinions coming from the electorate including those
that can catalyze candid, unihinbited and robust debate in the criteria for
the choice of a candidate.
This does not mean that there cannot be a specie of speech by a private
citizen which will not amount to an election paraphernalia to be validly
regulated by law.
Regulation of election paraphernalia by persons who are not
candidates or who do not speak as members of a political party will
be constitutionally valid, only if what is regulated is declarative
speech that, taken as a whole, has for its principal object the
endorsement of a candidate only.
The above regulation:
should be provided by law,
reasonable
narrowly tailored to meet the objective of enhancing the
opportunity of all candidates to be heard and considering the
primacy of the guarantee of free expression
demonstrably the least restrictive means to achieve that object.
The regulation must only be with respect to the time, place, and manner
of the rendition of the message. In no situation may the speech be
prohibited or censored on the basis of its content. For this purpose,
it will not matter whether the speech is made with or on private
property.
This is not the situation, however, in this case for two reasons.
First, as discussed, the principal message in the twin tarpaulins of
petitioners consists of a social advocacy.
Second, as pointed out in the concurring opinion of Justice Antonio
Carpio, the present law, Section 3.3 of Republic Act No. 9006 and
Section 6(c) of COMELEC Resolution No. 9615, if applied to this
case, will not pass the test of reasonability.
A fixed size for election posters or tarpaulins without any relation to the
distance from the intended average audience will be arbitrary. At certain
distances, posters measuring 2 by 3 feet could no longer be read by the
general public and, hence, would render speech meaningless. It will
amount to the abridgement of speech with political consequences.
EJERCITO v. COMELEC (HENRY) Petitioner claims that the advertising contracts between ABSCBN
November 25, 2014 | Peralta, J. | Freedom of speech and Scenema Concept Inc we re executed and identified
supporter without his
PETITIONER: Emilio Ramon “E.R.” Ejercito
RESPONDENTS: COMELEC and Edgar “Egay” San Luis

SUMMARY: A petition for disqualification was filed against petitioner


Ejercito for distributing orange cards amongst the constituents of Laguna,
which, allegedly, may be used in the hospital for medical needs.

COMELEC First division ruled in favor of his disqualification, which was


affirmed by the COMELEC En Banc. Upon appeal of the same to the SC,
the Court affirmed the COMELEC First division, that he is indeed to be
disqualified. As the the rights on free speech enshrined in the
constitution, in connection with election laws, entail protection as well to
candidates without deep pockets, in pursuance of the achieving of the
the substantial governmental onterest of promoting equality of
opportunity in political advertising.

DOCTRINE: Any restriction on speech or expression is only incidental


and is no more than necessary to achieve the substantial governmental
interest of promoting equality of opportunity in political advertising.

FACTS:
3 days prior to the 2013 National and Local Elections, a petition for
disqualification was filed by San Luise against Ejercito, who was a
fellow gubernatorial candidate and, the incumbent Governor of
Laguna. With the following allegations:
Ejercito distributed “orange cards” to influence, induce, or corrupt
the voters in voting for his favor.
Such card may be used in any pyblic hospital in Laguna for
medical needs.
Ejercito eventually won, and countered the petition against him. He
claims that the allegations were baseless, unfounded, and totally
speculative.
167. COMELEC First division granted the petition and disqualified
Ejercito, to which the provisional gov spot was declared vacant.
Ejercito appealed to COMELEC En Banc, who affirmed the First Division
ruling. To which, he filed the present petition.

ISSUE/s:
36. WoN ER Ejercito’s constitutional right to free speech is violated – NO

RULING: SC denied the petition. COMELEC 1st division ruling is affirmed.

RATIO:
knowledge and as in fact, his signature was forged. Assuming that suh
contract benefited hi, ejercito alleges that he should not be penalized for the
conduct of 3rd parties who acted on their own without his consent.
Ejercito argues that every voter has the right to support a particular candidate
according to their free exercise of her rights to speech and of expression, as
provided for by Art 4 and.
195. An advertising contract paid for by a third party without the candidate’s
knowledge and consent must be considered as a form of political speech that
must prevail against the law ssuppressing it, wherther any design or
inadvertence.
196. SC ruled that the advertising contracts between ABSCBN and SCenema
Concept Inc were not executed without Ejercito’s knowledge and consent.
197. SC noted RA 9006, which provided that broadcast advertisments donated to
the candidate shall not be broadcasted without the written acceptance of the
candidate, which shall be attached to the advertising contract and shall be
submitted to the COMELEC. In any case, it shall be signed by the donor.
Ejercito’s signature in the contracts was not a forger, as such issue is a belated
claim.
The inclusion of the amount contributed by a donor to the candidate’s allowable
limit of election expenses does not trample upon the free exercise of votger’s
rights of speech and of expression. As a content-neutral regulation, not to
curtail the message or content of the advertisement promoting a particular
candidate, but to ensure equality between and among aspirants with deep
pockets and those with less financial resources. Any restriction is only
incidental and is no more than necessary to achieve the substantial
governmental onterest of promoting equality of opportunity in political
advertising.
Indeed, to rule otherwise would practically result in an unlimited expenditure for
political advertising, which skews the political process and subverts the
essence of a truly democratic form of government.
RAPPLER v. BAUTISTA (DANNAH) TV 5, PhilStar and Businessworld the second phase in Visayas
April 5, 2016 | Carpio (and Leonen, Concurring) | Freedom of Speech / on March 2016
Press ABS-CBN and Manila Bulletin for the third leg in Luzon on April

PETITIONER: Rappler Inc.


RESPONDENTS: Andres Bautista

SUMMARY: COMELEC held meetings regarding the PiliPinas 2016


Debates. In the meetings, it was agreed upon that Rappler and CNN
would cover the VP debate for April 2016. Andres Bautista, COMELEC
Chair, urged Rappler to sign the MOA. Rappler, on the other hand,
expressed concerns regarding some provisions (i.e. maximum limit of two
minutes of debate excerpts for news reporting). Moreover, Rappler
wanted an equal right to live stream the debates. Bautista told Rappler
that the concerns would be addressed but the MOA needed to be signed
because time was of the essence.

Rappler communicated several times with COMELEC regarding its


concerns but received no response. Hence it filed a petition with the
Court. COMELEC’s defense was based solely on procedural defects,
which the Court ruled that the matter presented was of transcendental
importance hence procedure can be laxed. Moreover, it ruled that the
effect of COMELEC’s mandate empowering lead networks from excluding
other media is a prior restraint, albeit indirectly.

DOCTRINE: Freedom of speech is affected when government grants


benefits to some media outles (i.e. lead networks) while unreasonably
denying the same privileges to others. This has the effect of stifling
speech especially when the actions of a government agency (such as
COMELEC) have the effect of endowing a monopoly in the market of free
speech.

FACTS:
On September 2015, respondent Bautista called for a meeting with
various media outlets to discuss the “PiliPinas 2016 Debates” for
presidential and vice-presidential candidates.
Rappler sent a proposal draft for broadcast pool guidelines to COMELEC
and the KBP. A broadcast pool has a common audio and video feed
for the debates, and the cost will be apportioned among those
needing access to the same.
On October 2015, another meeting was held to discuss a draft MOA on
the debates. In the draft, Rappler and Google’s participation were
dropped in favor of the online outlets owned by the Lead Networks.
After the meeting, the respresentatives of said networks drew lots to
determine who will host each leg
GMA and PDI sponsored the first presidential debate in
Mindanao on February 2016
2016
The lone VP debate sponsored by CNN, Business Mirror and
Rappler on April 2016
On January 2016, Rappler was informed that the MOA signing would be
scheduled the following day. Rappler communicated its concerns
regarding online streaming and the imposition of a maximum limit of
two minutes of debate excerpts for news reporting.
It had to sign the MOA even with concerns because time was of the
essence. However, after several communications with Bautista,
Rappler received no response. Hence, this petition.
***PLEASE SEE ORIGINAL CASE FOR THE PROVISIONS OF THE
MOA.

ISSUE/s:
WoN Bautista should implement Part VI(C) Par. 19 of the MOA which
allows the debates to be shown or live-streamed unaltered in
Rappler’s and other websites subject to the copyright condition that
the source is clearly indicated – YES

RULING: WHEREFORE, we PARTIALLY GRANT the petition. Respondent


Andres D. Bautista, as Chairman of the COMELEC, is directed to implement
Part VI (C), paragraph 19 of the MOA, which allows the debates to be shown
or live streamed unaltered on petitioner's and other websites subject to the
copyright condition that the source is clearly indicated. Due to the time
constraint, this Resolution is immediately executory.

RATIO:
Procedural: Rappler did not avail itself the wrong remedy in raising
before the Court a controversy involving the fundamental right to free
speech.
Substantive: The Constitution mandates that “No law shall be passed
abridging the freedom of speech, of expression, or of the press…”
In ABS-CBN v. COMELEC, the Court nullified the COMELEC Resolution
prohibiting ABS-CBN from conducting exit polls.
The evil sought to be prevented in the protection of free speech is especially
grave during elections. In Osmena v. COMELEC, the Court mentioned
how “discussion of public issues and debate on the qualifications of
candidates in an election are essential to the proper functioning of the
government.
In Adiong v. COMELEC, the Court explained the importance of
protecting free speech that contributes to the web of information
ensuring the meaningful exercise of our right of suffrage.
Freedom of speech is affected when government grants benefits to some
media outles (i.e. lead networks) while unreasonably denying the same
privileges to others. This has the effect of stifling speech especially when
the
actions of a government agency (such as COMELEC) have the effect so that there is a lesser risk of excluding ideas for a public
of endowing a monopoly in the market of free speech. dialogue. The scrutiny for regulations which restrict speech during
Skewed distribution of resources as well as the cultural hegemony of the elections should be greater considering that these exercises
majority may have the effect of drowning out the speech and the substantiate the important right to suffrage.
messages of those in the minority. In a sense, social inequality does ***Rappler’s demand to exercise the right to live stream the debates
have its effect on the exercise and effect of the guarantee of free is a contractual right under the MOA. Under Part VI (C) Par. 19,
speech. the Lead Networks are expressly mandated to “allow the
Those who have more will have better access to media that reaches a debates they have produced to be shown or streamed on other
websites” but “subject to copyright conditions OR separate
wider audience than those who have less. Those who espouse the
negotiations with the Lead Networks.” The use of the word OR
more popular ideas will have better reception that the subversive and means that compliance with copyright conditions is sufficient.
the dissenters of society. To be really heard and understood, the
marginalized view normally undergoes its own degree of struggle.
Bautista avers that entering into the MOA does not trigger Article IX-C
Section 4 as this provision involves the COMELEC’s coercive power,
while the MOA was consensual.
Speech restraint regulation may also be:
Content-based: based on the subject matter of the utterance or
speech
Content-neutral: Merely concerned with the incidents of the
speech, or one that merely controls the time, place or
manner, and under well-defined standards
The effect of COMELEC’s mandate empowering lead networks from
excluding other media is a prior restraint, albeit indirectly. The evil
of prior restraint is not made less effective when a private corporation
exercises it on behalf of government.
In GMA v. COMELEC, the Court declared as unconstitutional Section
9(a) of Resolution 9165 that interpreted the 120 and 180-minute
airtime allocation under Section 6 of the Fair Elections Act.
A Concurring Opinion discussed free speech scrutiny against any kind of
prior restraint:
…Not only must the COMELEc have the competence, it must
also be cognizant of our doctrines in relation to any kind of
prior restraint.
…The industry structure consists of network giants with tremendous
bargaining powers that dwarf local community networks.
…Reducing the airtime simply results in a reduction of speech
and not a reduction of expenses
Election regulations are not always content-neutral regulations, and even
if they were, they do not necessarily carry a mantle of immunity from
free speech scrutiny. The question always is whether the
regulations are narrowly tailored so as to meet a significant
governmental interest and
1-UNITED v. COMELEC utility vehicles such as buses, jeepneys, trains, taxi
April 14, 2015 | Reyes, J. | Freedom of Speech cabs, ferries,

PETITIONER: 1-UNITED
RESPONDENTS: COMELEC

SUMMARY: COMELEC issued Resolution No. 9615, with Section


7(g) providing that posting election campaign materials in public
places outside of authorized common areas, including PUVs and
transport terminals, is prohibited. Petitioner contended that such
regulation violates the right of the owners of the PUVs and
transport terminals to free speech as it curtails their ideas of who
should be voted by the public. COMELEC claims that the
prohibition furthers two important and substantial governmental
interest – equalizing opportunity for all candidates and putting an
end the excessive campaign spending. The exact purpose for
placing political advertisements on a PUV or in a transport terminal
is exactly bcause it is public. Therefore, categorizing PUVs and
transport terminals as “public places” is therefore logical.

DOCTRINE: It is deeply embedded in our jurisprudence that


freedom of speech and of the press enjoys a preferred status in
our hierarchy of rights. The rationale is that the preservation of
other rights depends on how well we protect our freedom of
speech and of the press. Although the Resolution furthers an
important and substantial governmental interest, which is ensuring
equal opportunity among candidates, the governmental interest in
imposing the said prohibition is unrelated to the suppression of
free expression.

FACTS:
RA 9006, otherwise known as the Fair Elections Act was
passed. Section 9 provides that the COMELEC may
authorize political parties and party-list groups to erect
common poster areas for their candidates in not more
than ten public places such as plazas, markets, barangay
centers, etc. wherein candidates can display election
propaganda.
COMELEC promulgated Resolution No. 9615, Section 7
thereof provided prohibited forms of election propaganda
as follows:
To post election propaganda material outside of
authorized common poster areas, including public
pedicabs, tricycles, and within the premises of ISSUE/s:
public transport terminals. WoN Resolution No. 9615 violates the right to free speech of the
Petitioner sought clarification from COMELEC regarding the owners of PUVs and transport terminals – YES
application of Section 7 of the Resolution, in relation to
privately-owned pubic utility vehicles (PUVs) and
transport terminals.
Petitioner maintains that Section 7 violates the right to free
speech of the owners of PUVs and transport terminals as
it prohibits their ideas of who should be voted by the
public.
In a Minute Resolution, COMELEC postulated that the primary
fact to consider is whether 1-UTAK or any other PUV
owners in the same position possess a franchise and/or
certificate of public convenience and operate as a public
utility. If it does, then its operations will be placed directly
under the supervision and regulation of the COMELEC for
the duration of the election period to ensure equality of
opportunity for all candidates.
The prohibition furthers two important and substantial
governmental interests - equalizing opportunity for all
candidates and putting to a stop excessive campaign
spending. The infringement of freedom is merely incidental
and limited as to time. COMELEC has not taken away all
avenues of expression available to PUV and transport
terminal owners as they may express their political
preferences elsewhere.
The exact purpose for placing political advertisements on a
PUV or in a transport terminal is exactly because it is
public and can be seen by all. Categorizing PUVs and
transport terminal as “public places” under Section 7 is
therefore logical.
PUVs and private transport terminals hold a captive
audience - the commuters. Thus, it is within its
constitutional authority to prevent privately-owned PUVs
and transport terminals from concurrently serving
campaign materials to the captive audience that they
transport.
Resolution 9615 is a valid content-neutral regulation and
does not impinge on the constitutional right to freedom of
speech.
RULING: Resolution No. 9615 Section 7(g) items (5) and (6) are governmental interest, which is ensuring equal
declared unconstitutional. opportunity
RATIO:
Free speech may be identified with the liberty to discuss publicly
and truthfully any matter of public concern without prior
restraint or censorship. Prior restraint refers to official
governmental restrictions on the press or other forms of
expression in advance of actual publication or dissemination.
Section 7(g) items (5) and (6) unduly infringe on the
fundamental right of the people to freedom of speech.
Central to the prohibition is the freedom of individuals to
express their preference, through the posting of election
campaign material in their property.
Pursuant to the assailed provision, posting an election campaign
material during the election period in PUVs and transport
terminals carries with it the penalty of revocation of the
franchise. The prohibition constitutes a clear prior restraint on
the right to free expression of the owners of PUVs and
transport terminals.
It is deeply embedded in our jurisprudence that freedom of speech
and of the press enjoys a preferred status in our hierarchy of
rights. The rationale is that the preservation of other rights
depends on how well we protect our freedom of speech and of
the press.
A content-neutral regulation, which is merely concerned with
the incidents of the speech, or one that merely controls
the time, place or manner, and under well-defined
standards, is constitutionally permissible, even if it
restricts the right to free speech, provided that these
requisites concur: 1) the government regulation is
within the constitutional power of the Government; 2)
it furthers an important or substantial governmental
interest; 3) the governmental interest is unrelated to
the suppression of free expression; 4) the incidental
restriction on freedom of expression is no greater
than is essential to the furtherance of that interest.
Section 7 is a content-neutral regulation since it merely controls the
place where election campaign materials may be posted, but the
prohibition is still repugnant to the free speech clause as it
fails to satisfy all of the requisites for a valid content-neutral
regulation.
Although the Resolution furthers an important and substantial
among candidates, the governmental interest in imposing the
said prohibition is unrelated to the suppression of free
expression. Also, there is absolutely no necessity to restrict the
right to free speech of the owners of PUVs and transport
terminals.
While it is not disputed that the COMELEC has the power to
supervise or regulate the enjoyment of all franchises or permits
for the operation of transportation utilities during an election
period, such grant of supervisory and regulatory powers has its
limits.
The COMELEC’s constitutionally delegated powers of supervision
and regulation do not extend to the ownership per se of PUVs
and transport terminals, but only to the franchise or permit to
operate the same.
A restriction on the franchise or permit to operate transportation
utilities is necessarily a limitation on ownership, but a limitation on
the rights of ownership over the PUV is not necessarily a
regulation on the franchise or permit to operate the same.
The expression of ideas or opinion of an owner of a PUV through
the posting of election campaign materials on the vehicle does
not affect considerations pertinent to the operation of the PUV.
The COMELEC does not have the constitutional power to
regulate public transport terminals owned by private persons.
The ownership of such terminals, even if made available for use
by the public, still remains private.
A regulation of a public transport terminal based on external
circumstances, such as prohibiting the posting of election
campaign materials, amounts to regulating the ownership of the
transport terminal and not merely the permit to operate the same.
Section 7(g) failed to satisfy the fourth requisite of a valid
content-neutral regulation - the incidental restriction on
freedom of expression is no greater than is essential to the
furtherance of that interest. There is no necessity to restrict
the right of the owners to free speech to further the
governmental interest. The lofty aim of ensuring equality of
time may be achieved without any intrusion on the fundamental
right of expression.
A strict implementation of existing laws would suffice to achieve the
governmental interest of ensuring equal time, space
and opportunity for candidates. Thus, there is no necessity of
curtailing the right to free speech of the owners of PUVs and
transport terminals by prohibiting them from posting election
campaign materials on their properties.
COMELEC points out that PUVs and transport terminals hold a
“captive audience” - commuters who have no choice but
be subjected to the blare of political propaganda. The
captive-audience doctrine states that when a listener cannot
escape from intrusive speech, the speech can be restricted.
A regulation based on the captive-audience doctrine is in
the guise of censorship.
A government regulation based on the captive-audience
doctrine may not be justified if the supposed
“captive audience” may avoid exposure to the
otherwise intrusive speech. The prohibition in Section
7 is not justified under the captive-audience doctrine; the
commuters are not forced to read the election campaign
materials posted on PUVs and transport terminals.

RA 9165 curtail the choice of the owners of PUVs and transport


terminals on the advertisements that may be posted on their properties.
DCWD v. ARJANUEZ (IYA) They were found guilty as charged with penalties raging from suspension
June 16, 2015 | Perez, J. | Freedom of Expression to dismissal from service to which Gamboa executed Orders.
On appeal before the CSC, Aranjuz, et al. pray to Suspend the
Immediate Execution of Orders. They also bring up the violation of
PETITIONER: Davao City Water District represented by GM Rodora their constitutional rights to assemble and petition for redress of
Gamboa grievances.
RESPONDENTS: Rodrigo Aranjuez, Gregorio Cagula, et al.

SUMMARY: Members and Officers of NAMANDACWAD were


suspended and/or dismissed from employment because of wearing
shirts and posting their grievances during the anniversary celebration of
DCWD.

The CSC acting on appeal of the employees, ruled that the decision
violated their constitutional right to assemble and petition for redress of
grievances. The CA and SC affirmed such.

Although under public employment, this does not tantamount to waiving


ones right to express grievances.

DOCTRINE: The freedom to publish election surveys remains. All


Resolution No. 9674 does is articulate a regulation as regards the
manner of publication, that is, that the disclosure of those who
commissioned and/or paid for, including those subscribed to, published
election surveys must be made.

FACTS:
Davao City Water District (DCWD) is a GOCC represented by its General
Manager Gamboa.
Private Respondents are members of Nagkahiusang Mamumuo sa
Davao City Water Disctrict (NAMANDACWAD)
NAMANDACWAD was charged with several administrative cases due to
acts committed during the anniversary celebration of DCWD such as
wearing of t-shirt with inscriptions bearing grievances of the
employees and posting of complaints outside designated places.
Said anniversary celebration was a fun run. The employees were
required by Gamboa to attend such instructing them to wear sports
attire.
NAMADACWAD officers and members attended the event as required
wearing statement shirts for the only instruction was to wear sports
attire.
Cagula, however, attached inscriptions and posters of employee’s
grievances to a post in an area not among the officially designated
places for posting of grievances as prescribed by DCWD’s Office
Memorandum.
Gamboa field formal charges against the members and officers of
NAMADACWAD.
CSC partly granted the appeal. It held that the collective act in wearings-
shirts with grievance inscriptions is not within the ambit prohibited
mass action punishable as there was no intent to work stoppage.
The act can be considered as punishable under the Violation of
Reasonable Office Rules and Regulations. Cagula’s posting was
found violative of the DCWD Memorandum, but it is not serious in
nature due to the lack of any abusive, vulgar, defamatory or libelous
language.
Aggrieved, DCWD filed the a petition for Review with the CA. The CA
affirmed in toto the CSC resolution.
Hence, the present case.
ISSUE/s:
WON the concerted or mass action committed by its employees was not
permissible - NO

RULING: Petition is DENIED.

RATIO:
A prohibited concerted mass action is defined in Resolution No. 021316
but in Sec. 5 thereof.
The operative phrases are “any collective activity” and “work stoppage or
service disruption.”
Without the intent at work stoppage or service disruption, the concerted
activity is not prohibited. Whether done within government hours, a
concerted activity is allowed if it is without any intent at work
stoppage.
It is clear that the collective activity of joining the fun run in tshirts with
inscriptions on CNA incentives was not to effect work stoppage or
disrupt the service. As pointed out by the respondents, they followed
the advice of GM Gamboa “to be there” at the fun run.
It is correct to conclude that those who enter government service
are subjected to a different degree of limitation on their freedom
to speak their mind; however, it is not tantamount to the
relinquishment of their constitutional right of expression
otherwise enjoyed by citizens just by reason of their
employment.
Unarguably, a citizen who accepts public employment “must accept
certain limitations on his or her freedom.”
There are some rights and freedoms so fundamental to liberty that they
cannot be bargained away in a contract for public employment.
It is the Court’s responsibility to ensure that citizens are not
deprived of these fundamental rights by virtue of working for
the government. A regulation of the freedom of expression is
not removal of the constitutional right.
CENTRAL HUDSON v. PUBLIC SERVICE COMMISSION (ELIEL) given 24-hour period. The agency offered to review “specific
June 20, 1980 | Powell, J. | Freedom of Speech proposals by the companies for

PETITIONER: Central Hudson Gas & Electrical Corporation


RESPONDENTS: Public Service Commission of New York

SUMMARY: The Public Service Commission of New York ordered


electric utilities in New York, including Central Hudson to cease all
advertising that promotes the use of electricity. The reason behind is the
shortage of the government to provide. When the shortage eased, the
Public Service Commission decided to further the ban on advertising any
thing that promotes the use of electricity. This prompted Central Hudson
to file an action as a restriction to its constitutional right to commercial
speech.

The SC held that in order for a commercial speech to be protected, it must


concern lawful activity and not be misleading; and it must be determined
whether the asserted governmental interest ot be served by the restriction on
commercial speech is substantial. In the case at bar, (1) there is no report of
unlawful activity;
there is clearly a substantial governmental interest to prevent inequities in
the rates of electricities and fuel based on the successful promotion of
consumption in “off-peak” periods; however, (3) there is no link between
the restriction of the advertising and the government’s interest to prevent
inequity from Central Hudson’s rates.

DOCTRINE: The State must assert a substantial interest to be achieved


by restrictions on commercial speech. Moreoever, the regulatory
technique must be in proportion to that interest. The limitation on
expression must be designed carefully to achieve the State’s goal.

FACTS:
The Commission ordered electric utilities in New York State to cease all
advertising that “promotes the use of electricity.” The order was
based on the Commission’s finding that “the interconnected utility
sstem in New York State does not have sufficient fuel stocks or
sources of supply to continue furnishing all customer demands for
the 1973-1974 winter.”
Three years later, when the fuel shortage had eased, the Commission
requested comments from the public on its proposal to continue the
ban on promotional advertising. After reviewing the public comments,
the Commission extended the prohibition in a Policy Statement
issued on February 25, 1977.
The Commission’s order explicitly permitted “informational” advertising
designed ot encourage “shifts of consumption” from peak demand
times to periods of low electricity demand.
Information advertising would not seek to increase aggregate
consumption, but would invite a leveling of demand throughout any
specifically described [advertising] programs that meet these First, the restriction must directly advance the state interest involved; the
criteria.” regulation may not be sustained if it provides only ineffective or
The agency observed that additional electricity probably would be more remote support for the governmetn’s purpose.
expensive to produce than existing output. Because electricity rates Second, if the governmental interest could be served as well by a more
in New York were not then based on marginal cost, the limited restriction on commercial speech, the excessive restrictions
Commmission feared that additional power would be priced below cannot surive.
the actual cost of generation. The regulatory technique may extend only as far as the interest it serves.
The additional electricity would be subsidized by all consumers through The State cannot regulate speech that poses no danger to the asserted
generally higher rates. The state agency also thought that state interest,
promotional advertising would give “misleading signals” to the public
by appearing to encourage energy consumption at a time when
conservation is needed.

ISSUE/s:
WoN the Commission had restrained commercial speech in violation of
1st and 14th Amendments – YES
RULING: The record before us fails to show that the total ban on promotional
advertising meets the requirement. Reversed.

RATIO:
The Commission’s order restricts only commercial speech, that is expression
related solely to the economic interests of the speaker and its
audiences.
The 1st Amendment, as applied to the States through the 14th
Amendment, protects commercial speech from unwarranted
governmental regulation.
Commercial expression not only serves the economic interest of the
speaker, but also assists consumers and furthers the societal interest
in the fullest possible dissemination of information. In applying the 1 st
Amendment to this area, we have rejected the “highly paternalistic”
view that government has complete power to suppress or regulate
commercial speech.
The Constitution therefore accords a lesser protection to commercial
speech than to other constitutionally guaranteed expression. The
protection available for particular commercial expression turns on the
nature both of the expression and of the governmental interests
served by its regulation.
Consequently, there can be no constitutional objection to the
suppression of commercial messages that do not accurately inform
the public cabout lawful activity. The government may ban forms of
communication more likely to deceive the public than to inform it,
commercial related to illegal activity.
The State must assert a substantial interest to be achieved by
restrictions on commercial speech. Moreoever, the regulatory
technique must be in proportion to that interest. The limitation on
expression must be designed carefully to achieve the State’s goal.
nor can it completely suppress information when narrower restrictions on
expression would serve its interest as well.
For commercial speech to come within that provision, it at least must
concern lawful activity and not be misleading. Next, we ask whether
the asserted governmental interest is substantial. If both inquiries
yield posititve answers, we must determine whether the regulation
directly advances the governmental interest asserted, and whether it
is not more extensive than is necessary to serve that interest.
The court stated that advertising in a “noncompetitive market” could not
improve the decision making of consumers. The court saw no
constitutional problem with barring commercial speech that it viewed
as onveying little useful information.
This reasoning falls short of establishing that appellant’s advertising is
not commercial speech protected by the 1st Amendment. Even in
monopoly markets, the suppression of advertsing reduces the
information available for consumer decisions, and thereby defeats
the purpose of the 1st Amendment.
Without promotional advertising, the Commission stated, this inequitable
turn of events would be less likely to occur. The choice among rate
structures involves difficult and important questions of economic
supply and distributional fairness. The State’s concern that rates be
fair and efficient represents a clear and substantial governmental
interest.
The Commission’s laudable concern over the equity and efficiency of
Central Hudson’s rates does not provide a constitutionally adequate
reason for restricting protected speech. Central Hudson would not
contest the advertising ban unless it believed that promotion would
increase its sales. Thus, we find a direct link between the state
interest in conservation and the Commission’s order.
The Commission’s order reaches all promotional advertising, regardless
of the impact of the touted service on overall energy use. Bu te
energy conversation rationale, as important as it is, cannot justify
suppressing information about electric devices or services that would
cause no net increase in total energy use.
In addition, no showing has been made that a more limited restriction on
the content of promotional advertising would not serve adequately
the State’s interest.
The Commission also has not demonstrated that its interest in
conservation cannot be protected adequately by more limited
regulation of Cenral Hudson’s commercial expression. To further its
policy of conservation, the Commission could attempt to restrict the
format and content of Centrual Hudon’s advertising.
Philippines ratified the International Convention on the Rights of the
PHARMACEUTICAL v. DUQUE (JP) Child. Art. 24 of said instrument provites that all segments of scoiety
October 9, 2007 | Austria-Martinez, J. | Freedom of Speech should be

PETITIONER: Pharmaceutical and Health Care Association of the


Philippines
RESPONDENTS: Health Secretary Francisco Duque III

SUMMARY: Petitioners represent breastmilk substitute manufacturers.


They are assailing the RIRR of EO 51 or the Milk Code. They are
assailing that the RIRR are inconsistent with the Milk Code especially on
the absolute prohibition on the advertising of breastmilk substitutes.

C.J. Puno’s opinion labeled such prohibition as a restraint on commercial


speech. Applying the four-part test of a valid commercial speech, Puno
states that the RIRR passes the first three parts. They are related to the
law which is the Milk Code, likewise adheres to its purpose of ensuring
infant mortality. It however fails the fourth part of the test. The complete
suppression of the advertisement and promotion of breast-milk
substitutes is more than necessary for the purpose it seeks to attain.
DOCTRINE: Four-part analysis for evaluating the validity of regulations of
commercial speech: First, the commercial speech must “concern lawful activity
and not be misleading” if it is to be protected under the First Amendment. Next, the
asserted governmental interest must be substantial. If both of these requirements
are met, it must next be determined whether the state regulation directly advances
the governmental interest asserted, and whether it is not more extensive than is
necessary to serve that interest.

FACTS:
The present petition is a petition for certiorari seeking to nullify A.O. No.
2006-0012 entitled, Revised IRR of EO No. 51 otherwise known as
“The Milk Code.”
Executive Order No. 51 (Milk Code) was issued by President Corazon
Aquino by virtue of the legislative powers granted to the president
under the Freedom Constitution.
One of the preambular clauses of the Milk Code states that the law seeks
to give effect to Article 11 of the International Code of Marketing of
Breastmilk Substitutes (ICMBS), a code adopted by the World Health
Assembly (WHA) in 1981.
The WHA adopted several resolutions to the effect that breastfeeding
should be supported, promoted and protected, hence, it should be
ensured that nutrition and health claims are not permitted for breastmilk
substitutes.
informed of the advantages of breastfeeding. Thus, unlike what has been done with the ICMBS whereby the
DOH issued herein assailed RIRR. legislature enacted most of the provisions into law which is the Milk
Petitioners, representing its members that manufacturers of breastmilk
subsitutes, are saying that the RIRR is not valid as it containes
provisions that are not constitutional and go beyond the law it is
supposed to implement such as the absolute prohibition on the
advertising of breastmilk substitutes.

ISSUE/s:
WoN the respondents officers of the DOH acted without or in excess of
jurisdiction in violation of the Constitution in promulgating the RIRR -
YES

RULING: Petition is partially granted. Sections 4(f), 11 and 46 of


Administrative Order No. 2006- 0012 are declared void for being ultra vires.
The Department of Health and respondents are prohibited from implementing
said provisions.

RATIO:
(Note: The Main Decision Ratio does not seem to connect to mention or
discuss the freedom of speech or expression at all. Skip to Puno’s
Concurring.)
The international instruments that do have specific provisions regarding
breastmilk substitutes are the ICMBS and various WHA Resolutions.
Under the 1987 Constitution, international law can become part of the
sphere of domestic law either by transformation or incorporation.
The transformation method requires that an international law be
transformed into a domestic law through a constitutional mechanism
such as local legislation. The incorporation method applies when
by mere constitutional declaration, international law is deemed to
have the force of domestic law.
As for the ICMBS: The ICMBS which was adopted by the WHA in 1981
had been transformed into domestic law through local
legislation, the Milk Code. Consequently, it is the Milk Code that
has the force and effect of law in this jurisdiction and not the ICMBS
per se.
The Milk Code is almost a verbatim reproduction of the ICMBS, but it is well to
emphasize at this point that the Milk Code did not adopt the provision in the
ICMBS which absolutely prohibits the advertising or other forms of
promotion to the general public of products within the scope of the ICMBS.
Instead, the Milk Code expressly provides that advertising, promotion, or
other markeeting materials may be allowed if such materials are duly
authorized and apporved by the Inter-Agency Committee (IAC).
As for the WHA: The former Senior Legal Officer of WHO, Sami Shubber,
stated that WHA recommendations are generally not binding, but
they carry moral and political weight, as they constitute the judgment
on a health issue of the collective membership of the highest
international body in the field of health.
Code, the subsequent WHA Resolutions which specifically provides for Central Hudson v. Public Service Commission provides a four-part analysis for
evaluating the validity of regulations of commercial speech. To begin
(1) exclusive breastfeeding from 0-6 months, (2) continued
breastfeeding up to 24 months, and (3) absolutely prohibiting
advertisements and promotions of breastmilk substitutes, have
not been adopted as a domestic law.
It is propounded that WHA Resolutions may constitute “soft law” or non-
binding norms, principles and practices that influence state behavior
Soft law does not fall into any of the categories of international law. It is
however, an expression of non0binding norms, principles and
practices that influence state behavior. For ex. UN Declaration fo
Human Rights.
Consequently, legislation is necessary to transform the provisions of the
WHA Resolutions into domestic law. The provisions of the WHA
Resolutions cannot be considered as part of the law of the land
that can be implemented by executive agencies without the
need of a law enacted by the legislature.
The national policy of protection, promotion and support of breastfeeding
cannot automatically be equated with a total ban on advertising for
breastmilk substitutes.
In view of the enactment of the Milk Code which does not contain a total
ban on the advertising and promotion of breastmilk substitutes, but
instead, specifically creates an IAC which will regulate said
advertising and promotion, it follows that a total ban policy could be
implemented only pursuant to a law amending the Milk Code
passed by the constitutionally authorized branch of government, the
legislature.
Thus, only the provisions of the Milk Code, but not those of subsequent
WHA Resolutions, can be validly implemented by the DOH through
the subject RIRR.
Puno, Concurring and Separate Opinion: Another reason why the
absolute ban on the advertising and promotion of breastmlik substitutes
found under Sec. 4(f) and 11 of AO No. 2006-2012 of the RIRR should
be struck down is that such advertising and promotion of breastmlik
substitutes properly falls within the ambit fo the term commercial
speech.
Commercial speech is that speech that proproses an economic transaction.
This is a separate category of speech which is not accorded the same
level of protection as that given to other constitutionally guaranteed
forms of expression but is nonetheless entitled to protection.
In Virginia Pharmacy Board v. Virginia Citizens Consumer Council, the
U.S. Supreme Court struck down a law prohibiting the advertising of
prices for priscription drugs. It held that price information was
important to consumers, and that the First Amendment protects the
“right to receive information” as well as the right to speak.
Consumers have a strong interest in the free flow of information about
goods and services available in the marketplace and that any state
regulation must support a substantial interest.
with, the commercial speech must “concern lawful activity and not be
misleading” if it is to be protected under the First Amendment. Next, the asserted
governmental interest must be substantial. If both of these requirements are met, it
must next be determined whether the state regulation directly advances the
governmental interest asserted, and whether it is not more extensive than is
necessary to serve that interest.
Applying this four-part test to the case at bar: First, it is not claimed that the
advertisement at issue is an unlawful activity or is inaccurate. In fact,
both the International Code and the Milk Code recognize and concede
that there are instances when breastmilk substitutes may be necessary.
Second, there is no doubt that the governmental interest in providing safe
and adequate nutrition to infants and young children is substantial. This
interest is expressed as a national policy in no less than the fundamental
law of our land and is also embodied in various international agreements
where we are a party.
To be sure, the interest of the state in preserving and promoting the health of
its citizens is inextricably linked to its own existence.
Third, there is an undeniable causal relationship between the interest of
government and the advertising ban. Unquestionably, breastfeeding is
the tested and proven method of providing optimal nutrition to infants and
young children.
The rationale of the absolute ban is to prevent mothers from succumbing to
suggestive and misleading marketing and propaganda which may be
contained in advertisements of breastmilk substitutes.
Fourth and finally, prescinding from these predicates, we now come to the
critical inquiry: whether the complete suppression of the
advertisement and promotion of breast-milk substitutes is no more than
necessary to further the interest of the state in the protection and
promotion of the right to health of infants and young children.
C.J. Puno proffers the humble view that the absolute ban on advertising
prescribed under Sections 4(f) and 11 of the RIRR is unduly restrictive
and is more than necessary to further the avowed governmental
interest of promoting the health of infants and young children. It ought to
be self-evident, for instance, that the dvertisement of such products
which are strictly informative cuts too deep on free speech.
LADUE v. GILLEO (HENRY) “Residential identification signs” no longer than 1 sq ft. are allowed, as
June 13, 1994 | Stevens, J. | Freedom of Expression are

PETITIONER: City of Ladue et al.


RESPONDENTS: Margaret P. Gilleo

SUMMARY: Respondent Gilleo placed a sign in her front lawn, which


disappeared. She then put up another one but was knocked on the
ground. When she reported the incident to the police, she was told that
such sign was prohibited the City of Ladue.

She assailed the constitutionality of such ordinance for being violative of


her right insofar as the fist amendment, concerning the freedom of
expression, is concerned. The District Court held that the ordinance is
unconstitutional, to which the CA affirmed.

A petition was forwarded to the SC, to which ruled against the validity of
the statute, and held that it is indeed violative of the constitutional right to
freedom of expression.

DOCTRINE : Signs that react to a local happening or express a view on a


controversial issue both reflect and animate change in the life of a
community. Often placed on lawns or in windows, residential signs play
ain important part inpolitical campaigns, during which they are displayed
to signal the resident’s support for particular candidates, parties, or
causes. They may not afford the same opportunities for conveying
complex ideas as do other media, but residential signs have long been an
important and distinct medium of expression.

FACTS:
Respondent Gilleo owns 57 single family homes in the Willow Hill
subdivision, Ladue. On 1990, she placed on her front lawn a 24” by 36”
sign printed with words, “Say No War in the Persian Gulf, Call
Congress Now”
After that sign disappeared, Gilleo put up another but was knocked on
the ground. Gilleo reported these incidents to the police, who
adviced that such signs were prohibited in Ladue.
She filed a petition for a variance but was denied by the City Council.
This led her to file the present action against the City Mayor, and
members of the City Council, alleging that Ladue’s sign ordinance
violated her First Amendment right of free speech.
The district court issued a preliminary injuction against enforcement of
the ordinance, to which Gillio then placed a nother sign in the 2 nd
floor of her house (window) stating, “For Peace in the Gulf”.
Ladue City Council repealed its ordinance and enacted a replacement, which
likewise prohibited “signs” and defined that term broadly. The ordinance
prohibited signs except those that fall within one of the 10 exceptions.
signs advertising “that the property is for sale, lease, or exchange”.
Also exempted are signs for “churches, religions institutions, and
schools”, “commercial signs in commercially or industrial zoned
districts”, and on site signs advertising “gasoline filling stations.”
Unlike the previous ordinance, the new one acknowledged the
proliferation of an unlimited nuber of signs in private, residential,
industrial, and public areas of the City of Ladue would create
ugliness, visual blight and clutter, tarnish the natural beauty of the
landscape as well as the residential and commercial architecture,
impair property values, substantially impinge on the privacy of
special ambience of the community, and may cause safety and traffic
hazards to motorists, pedestrians, and children.
Gilleo amended her complaint to challenge the new ordinance, which
explicitly prohibits window signs like hers. The District Court held that
it was unconstitutional, to which the CA affirmed, and held that the
ordinance was invalid as a “content based” regulation because the
City treated commercial speech more favorably than noncommercial
speech and favored some kinds of noncommercial speech over
others.
Hence, an appeal by the City fo Ladue.

ISSUE/s:
WoN the ordinance is unconstitutional for violating the right to free speech –
YES

RULING: SC affirmed the CA decision.

RATIO:
While signs are a form of expression protected by the Free Speech
Clause, they pose distinctive problems that are subject to the
municipalities’ police powers, BUT measures regulating them
inevitably affect communication itself.
Such a regulation may be challenged on the ground that it restricts too
little speech because its exemptions discriminate on the basis of
sings’ messages, or on the ground that it prohibits too much
protected speech. Insofar as the case is concerned, the validity of
Ladue’s claim that the ordinance’s exemptions are free of
impermissible content or viewpoint discrimination is assumed.
Although the petitioner concededly has a valid interest in minimizing
visual clutter, it has almost completely foreclosed an important and
distinct medium of expression to political, religious, or personal
messages. Prohibitions foreclosing entire media may be completely
free of content or viewpoint discrimination, but such measures can
suppress too much speech by eliminating a commonmeans of
speaking.
The petitioner’s attempt to justify the ordinance as a “time, place, or
manner” restriction fails because alternatives such as handbills and
newspaper advertisements are inadequate substitutes for the
important medium that Ladue has closed off.
Displaying a sign from one’s own residence carries a message quite
distinct from placing the same sign someplace else, or conveying the
same text or picture by other means, for it provides information about
the speaker’s identity, an important component of many attempts to
persuade. Residential signs are also unusually cheap and
convenient form of communication. Furthermore, the audience
intended to be reached by a residential sign— neighbors—could not
be reached nearly as well by other means.
SC in their ruling likewise emphasized that their decision that the
petitioner’s ban on all residential signs violates the first amendment
by no means leaves the city powerless to address the ills that may
be associated with residential signs.
It bears mentioning that individual residents themselves have strong
incentives to keep their own property values up and prevent visual
clutter in their own yards and neighborhoods—incentives marked
different from htsoe of persons who erect signs on others’ land, in
thers’ neighborhoods, or on public property.
Resident’s sef interest diminishes the danger of the unlimited proliferation
of residential signs that concerns the City of Ladue.
RUBIN v. COORS BREWING CO. (DANNAH) a “reasonable fit” existed between the ban and the goal of avoiding
April 19, 1995 | Thomas, J | Protection of Commercial Speech strength

PETITIONER: Rubin, Secretary of the Treasury


RESPONDENTS: Coors Brewing Co.

SUMMARY: Coors applied to the Bureau of Alcohol, Tobacco and


Firearms for approval of its proposed labels. This was denied, because it
violated the Federal Alcohol Administration’s Act (FAAA) prohibition of
disclosing alcohol content on beer labels or advertisements. Coors then
filed a suit for relief.

The District Court, and the Court of Appeals (after remanding the case to
the District Court) invalidated said label ban. The Supreme Court affirmed
this, stating that it did not pass the Central Hudson test—although there
was substantial interest (suppressing the threat of “strength wars”), the
additional asserted interest in “facilitating” state efforts to regulate alcohol
under the Twenty First Amendment is not sufficiently substantial to meet
Central Hudson’s requirement. The Government has offered nothing to
suggest that States are in need of federal assistance in this regard.
Moreover, the ban cannot be said to advance the governmental interest
in suppressing strength wars because other provisions of the FAAA and
implementing regulations prevent said provision from furthering that
interest in a direct and material fashion.

DOCTRINE: The Central Hudson Test: In scrutinizing a regulation of


commercial speech that concerns lawful activity and is not misleading, a
court must consider whether the governmental interest asserted to
support the regulation is “substantial”. If that is the case, the court must
also determine whether the regulation directly advances the asserted
interest and is no more extensive than is necessary to serve that interest.

FACTS:
Coors applied to the Bureau of Alcohol, Tobacco and Firearms (BATF),
an agency of the Department of the Treasury for approval of its
proposed labels. The approval was rejected because it violated the
Federal Alcohol Administration’s Act (FAAA) prohibition of disclosing
the alcohol content on beer labels or advertisements.
This prompted Coors to file a suit for relief arguing the regulation violated
the First Amendment’s protection of commercial speech.
The Government argued that such labeling ban was necessary to suppress
the threat of “strength wars” among brewers, who, without the regulation,
would seek to compete in the marketplace based on the potency of
their beer.
The District Court invalidated said labeling ban, the Court of Appeals first
reversed, and remanded for further proceedings to ascertain whether
wars. strength may lead to greater alcoholism and its attendant social
The District Court upheld the ban on the disclosure of alcohol content in costs.
advertising but invalidated the ban as it applied to labels. The CA However, the additional asserted interest in “facilitating” state efforts to
then affirmed the District Court.
Although the CA found that the Government’s interest in suppressing the
“strength wars” was “substantial” under the test set out in Central
Hudson Gas & Electric Corp. v. Public Serv. Commission of New
York (Central Hudson), the ban violates the First Amendment
because it fails to advance that interest in a direct and material way.
The court found that there was no evidence of any relationship between
the publication of factual information regarding alcohol content and
competition on the basis of such content.

ISSUE/s:
WoN Section 5(e)(2) of the Federal Alcohol Administration Act violates
the First Amendment’s protection of commercial speech – YES

RULING: We conclude that the ban infringes respondent's freedom of


speech, and we therefore affirm.

RATIO:
Both parties agree that the information on beer labels constitutes
commercial speech.
Section 5(e)(2) prohibits any producer, importer, wholesaler, or bottler of
alcoholic beverages from selling, shipping, or delivering in interstate
or foreign commerce any malt beverages, distilled spirits, or wines in
bottles
“unless such products are bottled, packaged, and labeled in conformity
with such regulations, to be prescribed by the Secretary of the
Treasury, with respect to packaging, marking, branding, and
labeling and size and fill of container ... as will provide the
consumer with adequate information as to the identity and quality of
the products, the alcoholic content thereof (except that statements
of, or statements likely to be considered as statements of, alcoholic
content of malt beverages are prohibited unless required by State
law…”
In scrutinizing a regulation of commercial speech that concerns lawful
activity and is not misleading, a court must consider whether the
governmental interest asserted to support the regulation is
“substantial”.
If that is the case, the court must also determine whether the regulation
directly advances the asserted interest and is no more
extensive than is necessary to serve that interest.
The interest in curbing “strength wars” is sufficiently “substantial” to
satisfy Central Hudson. Brewers competing on the basis of alcohol
regulate alcohol under the Twenty First Amendment is not sufficiently
substantial to meet Central Hudson’s requirement. The Government has
offered nothing to suggest that States are in need of federal assistance in this
regard.
Said provision fails Central Hudson’s requirement that the measure
directly advance the asserted government interest. The ban cannot
be said to do so in suppressing strength wars because other
provisions of the FAAA and implementing regulations prevent said
provision from furthering that interest in a direct and material
fashion.
The provision is more extensive than necessary, as there are other
alternatives to the labeling ban—including directly limiting the alcohol
content of beers, prohibiting marketing efforts emphasizing alcohol
strength, and limiting the ban to malt liquors 5, the segment of the
beer market that allegedly is threatened with a strength war--would
prove less intrusive to the First Amendment.

It is the term used to designate those malt beverages with the highest alcohol content
DISINI v. SECRETARY OF JUSTICE (ARIELLE) DOCTRINE: SEE RATIO PLS ANG DAMI.
February 18, 2014| Abad, J. | Freedom of Speech

PETITIONER: Jose Jesus M. Disini, et al


RESPONDENTS: The Secretary of Justice

SUMMARY: RA 10175 or the Cybercrime Prevention Act of 2012 was


encted to regulate access to and use of the cyberspace. Pertitioners seek
to declare several provisions of the Act unconstitutional and void on the
ground that it violates the constitutional guarantee of freedom of
expression.

The SC held that:


The Court finds that Section 4(a)(1) does not violate any
fundamental freedom, like speech, because what is involved in punishing
the act of accessing the computer system of another without right as it is
a universally condemned conduct.

Section 4(a)(3) simply punishes what essentially is a form of


vandalism, the act of wilfully destroying without right the things that
belong to others.

Section 4(b)(3) is constitutional because the theft of information


must be intended for an illegitimate purpose. Acquiring and disseminating
information made public by the user cannot be regarded as a form of
theft.

Section 4(c)(1) deliberations of the Bicam show a lack of intent to


penalize a “private showing between and among two private persons”
although that may be a form of obscenity to some.

Section 4(c)(3) is constitutional because unsolicited


advertisements are legitimate forms of expression.

Section 4(c)(4) is constitutional because libel is not a


constitutionally protected speech and that the government has an
obligation to protect private individuals from defamation.

Section 5 is unconstitutional because a governmental purpose


which seeks to regulate the use of cyberspace to protect a person’s
reputation cannot adopt means that will unnecessarily and broadly
sweep, invading the area of protected freedoms because self-inhibition
borne of fear of punishment await internet users will suppress otherwise
robust discussion of public issues.

Section 19 is unconstitutional because the content of computer


data can also constitute speech. In such a case, Section 19 operates as a
restriction on the freedom of expression over cyberspace.
FACTS: the people. The Court held that the strict scrutiny standard was used to
Republic Act 10175, otherwise known as the Cybercrime Prevention assess the validity of
Act of 2012 was enacted to regulate access to and use of the
cyberspace. A person using his laptop or computer can connect to
the internet, which links him to other computers.

Linking with the internet opens up a user to communications from


others, and the ill-motivated can use the cyberspace for committing
theft by hacking into or surreptitiously accessing another’s bank
account to defraud others through false representation. It can also
be used to illicit trafficking in sex or for exposing to pornography
children who have access to the internet.
Petitioners claim that the means adopted by the cybercrime law for
regulating undesirable cyberspace activities violate certain
constitutional rights. Petitioners challenge the following provisions
as unconstitutional:

Section 4(a)(1) on Illegal Access; No


Section 4(a)(3) on Data Interference; No
Section 4(b)(3) on Identity Theft; No
Section 4(c)(1) on Cybersex; No
Section 4(c)(3) on Unsolicited Commercial Communications
No
Section 4(c)(4) on Libel; Partial invalidation – creates criminal
liability on the part of the persons who receive a libelous
post and merely react to it
Section 5 on Abetting in the Commission of Cybercrime – Yes
Section 19 on Restricting or Blocking Access to Computer
Data; Yes

ISSUE/s:
WoN Section 4(a)(1) on Illegal Access is unconstitutional; No
WoN Section 4(a)(3) on Data Interference is unconstitutional; No
WoN Section 4(b)(3) on Identity Theft is unconstitutional; No
WoN Section 4(c)(1) on Cybersex is unconstitutional ; No
WoN Section 4(c)(3) on Unsolicited Commercial Communications is
unconstitutional; No
WoN Section 4(c)(4) on Libel is unconstitutional; Partial invalidation –
creates criminal liability on the part of the persons who receive a
libelous post and merely react to it
Section 5 on Abetting in the Commission of Cybercrime – Yes
Section 19 on Restricting or Blocking Access to Computer Data; Yes
RULING: The Cybercrime Prevention Act is partly constitutional.

RATIO:
Section 4(a)(1) – Illegal Access
Petitioners contend that this provision fails to meet the strict scrutiny
standard required of laws that interfere with the fundamental rights of
laws dealing with the regulation of speech, gender, or race as well as The Court will not declare Section 4(c)(1) unconstitutional where it
other fundamental rights. stands a construction that makes it apply only to persons
The Court finds nothing in Section 4(a)(1) that calls for the engaged in the usiness of
application of the strict scrutiny standard since no fundamental
freedom, like speech, is involved in punishing the act of accessing
the computer system of another without right as it is a universally
condemned conduct.
Section 4(a)(3) – Data Interference
Data interference is the intentional or reckless alteration of computer
data without right. Petitioners claim that this section suffers from
overbreadth insofar as it seeks to discourage data interference, and
intrudes into the area of protected speech and expression, creating a
chilling and deterrent effect on these guaranteed freedoms.
Overbreadth doctrine: a proper governmental purpose may not be
achieved by means that unnecessarily sweep its subject broadly,
thereby invading the area of protected freedoms. However, Section
4(a)(3) does not encroach on these freedoms at all. It simply
punishes what essentially is a form of vandalism, the act of
wilfully destroying without right the things that belong to others.
All penal laws, like the cybercrime law, have an inherent chilling effect, an
in terrorem effect, or the fear of possible prosecution. Here, the
chilling effect that results in paralysis is an illusion since Sec
4(a)(3) clearly describes the evil that it seeks to punish.
Moreover, the overbreadth challenge places on petitioners the heavy
burden of proving that under no set of circumstances will Section
4(a)(3) be valid.
Section 4(b)(3) – Computer-related Identity Theft
Petitioners claim that this provision transgresses the freedom of the
press because it allegedly hinders journalists from accessing the
unrestricted user account of a person in the news to secure
information about him that could be published.
However, this is not the essence of identity theft that the law seeks
to punish. The theft of information must be intended for an
illegitimate purpose. Acquiring and disseminating information made
public by the user cannot be regarded as a form of theft.
Section 4(c)(1) – Cybersex
Petitioners contend that the provision violates the freedom of
expression. They express fear that private communications of sexual
character between husband and wife or consenting adults would now
be regard as crimes when done “for favor” in cyberspace.
The deliberations of the Bicam show a lack of intent to penalize a “private
showing between and among two private persons” although that may
be a form of obscenity to some. In cybercrime law, the element of
“engaging in a business” is necessary to constitute the illegal
cybersex. The Act seeks to punish cybercrime prostitution, white
slave trade, and pornography for favor and consideration.
maintaining, controlling, or operating the lascivious exhibition of sexual
organs or sexual activity with the aid of a computer system.
Section 4(c)(3) – Unsolicited Commercial Communications
The provision penalizes the transmission of unsolicited commercial
communications, also known as “spam.” The government points out that
spam is a nuisance that wastes the storage and network capacities of
internet service providers and interferes with the owner’s peaceful
enjoyment of his property. The OSG also contends that commercial
speech enjoys less protection in law.
People before the arrival of computers have already been receiving
unsolicited ads by mail and these have never been outlawed as nuisance
because people might have interest in them. What matters is that the
recipient has the option of not opening or reading these mail ads.
To prohibit the transmission of these ads would deny a person the right to
read his emails.
Commercial speech is a separate category of speech which is not
accorded the same level of protection as that given to other
constitutionally guaranteed forms of expression but is still entitled
to protection. Unsolicited advertisements are legitimate forms of
expression.
Section 4(c)(4) – Libel
Libel is a public and malicious imputation of a crime, real or imaginary
tending to cause the dishonor of a natural or juridical person, or to
blacken the memory of one who is dead (Art. 353, RPC). The provision
of libel in the cybercrime law merely incorporates to form part of it the
provisions of the RPC on libel.
Petitioners claim that the libel provisions of the cybercrime law carry with
them the requirement of “presumed malice” even when the latest
jurisprudence replaced it with the higher standard of “actual malice” as
basis for conviction.
The petitioners argue that “presumed malice” from the accused’s defamatory
statement infringes on his constitutionally guaranteed freedom of expression.
There is “actual malice” when the offender makes the defamatory statement
with the knowledge of its falsity with reckless disregard. There is a
relatively wide leeway given to utterances against public figures
when the Court modified the penalty of imprisonment to just a fine of
P6,000.00. But if the party is a private individual, the prosecution need
not prove the presence of malice. The law presumes its existence from
the character of the assailed statement.
Petitioners further claim that the provision on libel violate the country’s
obligations under the ICCPR. Indeed, the ICCPR states that although
everyone should enjoy freedom of expression, its exercise carries
with it special duties and responsibilities. Free speech is not
absolute.
The Court agrees that libel is not a constitutionally protected speech and
that the government has an obligation to protect private individuals from
defamation.
Section 5 – Abetting in the Commission of Cybercrime
Petitioners claim that this provision suffers from overbreadth, clear and present danger rule. Section 19, merely requires that
creating a chilling and deterrent effect on protected expression. the data to be blocked be found prima facie in violation of any
OSG contends that current jurisprudence and laws on aiding and provision of the cybercrime law. It does not take into
abetting sufficiently protects the freedom of expression of “netizens.” consideration any of the three tests.
He points out that existing laws delineate the meaning of “aiding or Serreno, Concurring and Dissenting
abetting” a crime as to protect the innocent.
Since the social media platforms today provide for “Likes” and
“Comments” and “Sharing,” the question is: are these considered as
“aiding or abetting?” If A posts the statement, “Armand is a thief!” he
could certainly be charged with libel. If B, upon seeing the post,
writes on it, “I like this!” that could not be libel since he did not author
the poster.
A governmental purpose which seeks to regulate the use of cyberspace
to protect a person’s reputation cannot adopt means that will
unnecessarily and broadly sweep, invading the area of protected
freedoms because self-inhibition borne of fear of punishment await
internet users will suppress otherwise robust discussion of public
issues.
The terms “aiding or abetting” constitute broad sweep that
genereates chilling effect on those who express themselves
through cyberposts, comments, and other messages.
When a penal statute encroaches upon the freedom of speech, a facial
challenge grounded on the void-for-vagueness doctrine is
acceptable. The overbreadth and vagueness doctrines are
inapplicable to penal statutes, but can be appropriate insofar as
these doctrines are used to mount “facial” challenges to penal
statutes not involving free speech.
The “as applied” challenge, the petitioner who claims a violation of his
constitutional right can raise any constitutional ground. Here, one can
challenge the constitutionality of a statute only if he asserts a violation of
his own rights. However, a petitioner may mount a “facial” challenge
even if he claims no violation of his own rights under the assailed statute
where it involes free speech on the grounds of overbreadth or
vagueness of the statute.
Section 19 – Restricting or Blocking Access to Computer Data
Petitioners assert that this provision stifles freedom of expression
because it allows the DOJ to restrict or block access to a computer
data when a computer data is prima facie found to be in violation of
the provisions of the cybercrime act. OSG contends that this
provision may be unconstitutional.
The content of computer data can also constitute speech. In such a case,
Section 19 operates as a restriction on the freedom of expression
over cyberspace. Not all forms of speech are protected. But for an
executive officer to seize content alleged to be unprotected without
any warrant, it is not enough for him to be of the opinion that such
content violates some law.
Restraints on free speech are generally evaluated using three tests: the
dangerous tendency doctrine, the balancing of interest test, and the
A facial challenge is a deviation from the general rule that Courts should
only decide the invalidit of a law “as applied” to the actual
circumstances before it. Because of its effect as a total nullification,
the facial invalidation of laws is deemed to be a “manifestly strong
medicine” that must be used sparingly and only as a last resort.
In relation to Section 6 (imposing a penalty one degree higher for
libel): Yes, penal statutes cannot be invalidated on the ground
that they produce a “chilling effect” since they are intended to
have an in terrorem effect. However, the Congress does not
have carte blance authority to indiscriminately impose and
increase penalties. When laws and penalties affect free speech,
it is beyond question that the Court may exercise its power of
judicial review.
When Congress enacts a penal law affecting free speech and
accordingly imposes a penalty that is so discouraging that it
effectively creates an invidious chilling effect, then there is
ground to invalidate the law.
Facial challenges have been entertained when the possibility that the
freedom of speech may be muted. In the present case, it is not
difficult to see how the increase of the penalty under Section 6
mutes freedom of speech.
Carpio, Concurring and Dissenting
There is a repugnancy between the malice rule in Art. 354 of the
RPC and the Free Speech Clause insofar as Art. 354 states that
malice is presumed even in factually false defamatory
statemetns.
Section 4(c)(1) fails the strict scrutiny standard. As a content-based
regulation, the provision triggers the most stringent standard of
review for speech restrictive laws to test its validity. The
government failed to discharge its burden. The state interests
appears to advance the bases for Section 4(c)(1) are:
0 The protection of children as cybersex operations
1 The cleansing of cyber traffic by penalizing the online
publication of porn
Even if these are legitimate or substantial, these interests fail to rise
to the level of compelling interests because the provision is
overinclusive in its reach of the persons to be exploited, and
because it is underinclusive in its mode of commission.
Section 4(c)(3) would be constitutional if it allowed the free
transmission of truthful and non-misleading commercial speech.
There is no legitimate government interest in criminalizing per se
the transmission in cyberspace of truthful and non-misleading
commercial speech.
Penalizing the transmission of commercial speech is devoid of any
legitimate government interest and thus violates the Free
Speech Clause. The free flow of truthful and non-misleading
commercial speech online should remain unhampered to assure
freedom of expression of protected speech.
Commercial speech may not enjoy the same protection as The constitution requires that the libel as
political speech but any regulation of truthful and non- presently contained in the RPC and as
misleading commercial speech must still have a legitimate reenacted in the Cybercrime law be
government purpose. Free flow of truthful commercial struck down as infringing upon the
speech should be encouraged for the enlightenment of the guarantee of freedom of expression.
consuming oublic. Online transmission of commercial
message is more economical.
Although censorship or prior restraint is permitted on speech
which is pornographic, commercially misleading or
dangerous to national security, only pornographic speech is
covered by RA 10175 on online child pornography.
Moreover, a court order is required to censor or effect prior
restraint on protected speech. By allowing the government to
search without warrant and administratively censor all
categories of speech which is non-pornogtaphic, not
commercially misleading and not a danger to national
security, Section 19 is undeniably unconstitutional.
Brion, Separate Concurring
Call to declare Art 354 of the RPC unconstitutional when applied to
libelous statements committed against public officers and
figures.
The Cybercrime Law makes the consequences of cyber-liber far
graver than libelous speech in the real world. Thus, the law,
through Section 5, opts to penalize the acts of aiding,
abetting, and attempting to commit a cybercrime.
The application of these provisions unduly increase the
prohibitive effect of libel law on online speech, and can have
the effect of imposing self-censorship in the Internet and of
curtailing an otherwise robust avenue for debate and
discussion on public issues. In other words, Section 5,6, and
7 should not apply to cyber-libel as they open the door to
application and overreach into matter other than libelous and
can thus prevent protected speech from being uttered.
There is no sufficient distinction between libelous speech committed
online and speech uttered in real life to warrant increasing the
prohibitive impact of penal law in cyberspace communications.
Penalizing libelous speech committed through the Internet with
graver penalties because it allegedly reaches a wider
audience creates an unreasonable classification between
communicaitnos made through the Internet and I nthe real
world. Art. 355 of the RPC does not distinguish among the
means of communications by which libel is published.
Malice is in fact necessary for libel committed against public
officers and figures to prosper; it must be proven that the
offender made the defamatory statement with the knowledge
that it is false.
Leonen, Dissenting and Concurring
The only instance when a facial review is permissible is when there is a clear
showing that the provisions are too broad under any reasonable reading that it
imminently threatens expression.
When there is overbreadth, an invalidation of the statute “on its face” rather
than “as applied.”
General rule: penal statutes cannot be subjected to facial attacks. Except when a
provision in a statute can be struck down as unconstitutional when there is a
clear showing that there is an imminent possibility that its broad language will
allow ordinary law enforcement to cause prior restraints of speech.
What we learn from others bears on what we think as well as what and how we
express. For the quality of our own expression, it is as important to tolerate the
expression of others.
A facial challenge can only be raised on the basis of overbreadth, not vagueness.
Vagueness relates to a violation of the rights of due process.
The provisions that provide for higher penalties for these as well as for dual
prosecutions should likewise be declared unconstitutional because they magnify
the “chilling effect: that stifles protected expression.
Section 19 is prior restraint because it allows the DOJ to restrict or block access to
computer data when it is prima facie found to be in violation of the provisions of
the Act. Section 19 provides an arbitrary standard by which the DOJ may
exercise its power to restrict or block access. A prima facie finding is sui generis
and cannot be accepted as basis to stop speech even before it is made.
The threat of being prosecuted for libel stifles the dynamism of the conversations that
take place in cyberspace.
Libel law now is used not so much to prosecute but to deter speech. What is charged
as criminal libel may contrain precious protected speech. It is time that we now
go further and declare libel, as provided in the RPC and in the Cybercrime law as
unconstitutional. The remedy to be availed of against unwarranted attacks will be
civil in nature and granted in the provisions such as the Chapter on Human
Relations in the Civil Code.
Section 4(c)(3) [commercial speech] has no chilling effect on speech. Commercial
speech is “low value” speech to which the clear and present danger test is not
applicable. Since commercial speech is valuable only to the extent of its ability to
inform, advertising is not at par with other forms of expression.
RE: REQUEST RADIO TV COVERAGE OF THE TRIAL IN THE
SANDIGANBAYAN OF THE PLUDER CASES AGAINST THE RATIO:
FORMER PRESDIENT JOSEPH E ESTRADA (IYA) An accused has a right to a public trial but it is a right that belongs
June 29, 2001 | Vitug, J. | Freedom of Press to him, more than anyone else, where his life or liberty can be
held critically in balance.
A public trial is not synonymous with publicized trial; it only implies that
SUMMARY: A letter of request was sent to the SC to allow the media to
the court doors must be open to those who wish to come, sit in the
record during the proceedings of the plunder case of former president available seats, conduct themselves with decorum and observe the
Estrada before the Sandmanbayan. trial process.
The courts recognize the constitutionally embodied freedom of the press
A balancing of two constitutional rights is before the Supreme Court.
and the right to public information.
Initially, the Court ruled that the right of the accused to a fair and It also approves of media’s exalted power to provide the most accurate
impartial trial must prevail over the right of freedom of press. The and comprehensive means of conveying the proceedings to the
rationale behind such ruling is that when court proceedings are public and in acquainting the public with the judicial process in action.
subjected to media exposure, the courts may be influenced by the Nevertheless, within the courthouse, the overriding consideration is
commentaries of the viewing public. Instead of applying the rule of law, still the paramount right of the accused to due process which
the justices may be swayed to rule in favor of public opinion. must never be allowed to suffer diminution in its constitutional
proportions.
Upon Motion for Reconsideration, the court allowed the documentation
Justice Clark thusly pronounced, “while a maximum freedom must be
of the proceedings justifying that present case is of significance to the allowed the press in carrying out the important function of
history of the country and there is importance of preserving records. In informing the public in a democratic society, its exercise must
allowing the press to be present during trial, they must follow guidelines necessarily be subject to the maintenance of absolute fairness
to be imposed by the Sandiganbayan. in the judicial process.”
A trial is not a free trade of ideas. Nor is a competing market of thoughts
FACTS: the known test of truth in a courtroom.
Kapisanan ng mag Brodkaster ng Pilipinas (KBP) sent a letter requesting the Puno, dissent: The 1991 resolution of this Court absolutely banning live
SC to allow live media coverage of the anticipated trial of the plunder radio and television coverage of criminal proceedings should be re-
case filed against former president Joseph Estrada before the examined to re-adjust the balance between a free press and a fair
Sandiganbayan. trial in the light of the continuing progress in communications
Purpose was to assure the public of full transparency in the proceedings technology and to expand the right of access of the press and the
of an unprecedented case in Philippine history. public to information without, however, impairing the right of an
Secretary of Justice Perez formally filed the instant petition averring the accused to due process.
right of the public to vital information affecting the nation. Panganiban, dissent: It is technologically possible to uphold the right of
The petition in effect seeks a re-examination of the former resolution of the people to public information without violating the right of the
the SC in the liber case filed by Cory Aquino where Rule 53 of the accused to due process and without impeding the orderly
Federal Rules of Criminal Procedure was upheld, said Rule forbids administration of justice. It is now feasible to satisfy the peoples right
the taking of photographs during the progress of judicial proceedings to information by less distracting, degrading and prejudicial means.
or radio broadcasting of such proceedings from the courtroom.
The propriety of granting or denying the instant petition involve the MOTION FOR RECONSIDERATION
weighing out of the constitutional guarantees of freedom of the press
and the right to public information, on the one hand, and the
ISSUE/s:
fundamental rights of the accused, on the other hand, along with the
constitutional power of a court to control its proceedings in ensuring WON the media may be allowed during the trial of Estrada - YES with
a fair and impartial trial. reservations.

ISSUE/s: RULING: WHEREFORE, an audio-visual recording of the trial of former


WON the media may be allowed during the trial of Estrada - NO. President Estrada before the Sandiganbayan is hereby ordered to be made.

RULING: WHEREFORE, the petition is DENIED. RATIO:


What follows is the opinion of the majority. Considering the significance
of the trial before the Sandiganbayan of former President Estrada
and the
importance of preserving the records thereof, the Court believes that there
should be an audio-visual recording of the proceedings.
The recordings will not be for live or real time broadcast but for
documentary purposes.
Only later will they be available for public showing, after the
Sandiganbayan shall have promulgated its decision in every case to
which the recording pertains.
The master film shall be deposited in the National Museum and Archives
Office for historical preservation and exhibition pursuant to law. the
hearings are of historic significance. They are an affirmation of our
commitment to the rule that “the King is under no man, but he is under
God and the law.”
Estrada cases involve matters of vital concern to our people who
have a fundamental right to know how their government is
conducted. This right can be enhanced by audio-visual
presentation.
Audio-visual presentation is essential for the education and civic training
of the people.
There is the need to keep audio-visual records of the hearings for
documentary purposes. The recordings will be useful in
preserving the essence of the proceedings in a way that the
cold print cannot quite do because it cannot capture the sights
and sounds of events.
In the documentary during the Estrada trial the following must be considered:
The trial shall be recorded in its entirety, excepting such
portions thereof as the Sandiganbayan may determine
should not be held public under Rule 119, §21 of the Rules
of Criminal Procedure;
Cameras shall be installed inconspicuously inside the
courtroom and the movement of TV crews shall be
regulated consistent with the dignity and solemnity of the
proceedings;
The audio-visual recordings shall be made for documentary
purposes only and shall be made without comment except
such annotations of scenes depicted therein as may be
necessary to explain them;
The live broadcast of the recordings before the
Sandiganbayan shall have rendered its decision in all
the cases against the former President shall be
prohibited under pain of contempt of court and other
sanctions in case of violations of the prohibition;
To ensure that the conditions are observed, the audio-visual
recording of the proceedings shall be made under the
supervision and control of the Sandiganbayan or its
Division concerned and shall be made pursuant to rules
promulgated by it; and simultaneously with the release of
the audio-visual recordings for public broadcast, the original
thereof shall be deposited in the National Museum and the
Records Management and Archives Office for preservation
and exhibition in accordance with law.
that live television and radio coverage of the trail in these criminal cases
RE: PETITION FOR RADIO AND TELEVISION COVERAGE OF THE be allowed,
MULTIPLE MURDER CASES AGAINST MAGUINDANAO GOVERNOR
ZALDY AMPATUAN, ET AL. (ELIEL)
June 14, 2011 | Carpio Morales, J. | Freedom of Speech

PETITIONER: Radio and Television Stations, Public


RESPONDENTS: Gov. Zaldy Ampatuan

SUMMARY: Several media entities and journalists invoke their freedom of


the press, information and petition the government for redress of grievances
to lift prohibition on the total ban of live broadcasting court proceedings.
Petitioners here are contending that the trail on the Maguindanao Massacre
case, which killed 57 people, is of public interest and that they have the right
to be informed of the proceedings through media. The trial court justified its
outright prohition on the rationale that the media might influence the
impartiality and dignity of the court.

The SC ruled that there is no empirical evidence or authoritative study to


show how the media influences the decision of the judges. The SC held
that it is now time to look for a situation wherein there is no need to
sacrifice the rights of the people and the rights in criminal administration
of justice. The SC granted the media broadcasting, but subject to the
guidelines it has laid down. Moreover, it is practical to allow the media
since the court must have a reasonable number of public. This case
however has a lot of interested parties, therefore, technology can be used
at its advantage.

DOCTRINE: People v. Teehankee, Jr., “totality of circumstances”, that the


right of an accused to a fair trial is no incompatible to a free press, that
pervasive publicity is not per se prejudicial to the right of an accused to a
fair trial, and that there must be allegation and proof of the impaired
capacity of a judge to render a bias-free decision. Mere fear of possible
undue influence is not tantamount to actual prejudice resulting in the
deprivation of the right to a fair trail.

FACTS:
On November 23, 2009, 57 people including 31 journalists and media
practitioners were killed while on their way to Shariff Aguak in
Maguindanao.
Touted as the worse election-related violence and the most brutal killing
of journalists in recent history, the tragic incident which came to be
known as the “Maguindanao Masscre.”
Almost a year later, the National Union of Journalists of the Philippines
(NUJP), ABS-CBN Broadcasting Corporation, GMA Network, Inc.,
relatives of the victims, individual journalists from various media entitites,
and members of the academe filed a petition before this Court praying
recording devices be permitted inside the courtroom to assist the On the media coverage’s influence on judges, counsels and witnesses,
working journalists, and reasonable guidleines be formulated to petitioners point out that Aquino and Estrada, lack empirical evidence
govern the broadcast coverage and the use of devices. to
President Benigno S. Aquino III, by letter addressed to CJ Renato
Corona, came out “in support of those who have petitioned [this
Court] to permit television and radio broadcast of the trial.”
Petitioners seek to lift the absolute ban on live television and radio
coverage of court proceedings. They contend that rulings in Aquino
libel case and Estrada plunder cases violate the doctrine that
proposed restrictions on constitutional rights are to be narrowly
construed and outright prohibition cannot stand when regulation is a
viable alternative.
Petitioners state that the trial of the Maguindanao Massacre cases has
attracted intense media coverage due to the gruesomeness of the
crime, prominence of the accused, and the number of media
personnel killed.
Petitioners invoke the exercise of the freedom of the press, right to
information, right to a fair and public trial, right to assembly and to
petition the government for redress of grievances, right of free
access to courts, and freedom of association, subject to regulations
to be issued by the Court.

ISSUE/s:
WoN live broadcasting of court proceedings is allowed – YES

RULING: Granted. Subject to the guidelines outlined.

RATIO:
The rationale for an outright total prohibition was shrouded, as it is now,
inside the comfortable cocoon of a feared speculation which no
scientific study in the Philippine setting confirms, and which fear, if
any, may be dealt with by safeguards and safety nets under existing
rules and exacting regulations.
Aquino, “[a] trial of any kind or in any court is a matter of serious
importance to all concerned and should not be treated as a
means of entertainment[s t]o so treat it deprives the court of the
dignity which pertains to it and departs from the orderly and
serious quest for truth for which our judicial proceedings are
formulated.”
“[m]assive intrusion of representatives of the news media into the
trial itself can so alter and destroy the constitutionally
necessary atmosphere and decorum”
Estrada, “[t]he propriety of granting or denying the instant petition
involve[s] the weighing out of the constitutional guarantess of
freedom of the press and the right ot public information, on the
one hand, and the fundamental rights of the accused, on the
other hand, along with the constitutional power of a court to
control its proceedings in ensuring a fiar and impartial trial.”
support the sustained conclusion. They point out errors of generalization
where the conclusion has been mostly supported by studies on American
attitueds, as there has been no authoritative study on the aprticular matter
dealing with Filipinos.
People v. Teehankee, Jr., “totality of circumstances”, that the right
of an accused to a fair trial is no incompatible to a free press,
that pervasive publicity is not per se prejudicial to the right of
an accused to a fair trial, and that there must be allegation and
proof of the impaired capacity of a judge to render a bias-free
decision. Mere fear of possible undue influence is not
tantamount to actual prejudice resulting in the deprivation of
the right to a fair trail.
Moreover, an aggrieve party has ample legal remedies. He may
challenge the validity of an adverse judgment arising from a
proceeding that transgressed a constitutional right.
One apparent circumstance that sets the Maguindanao Massacre cases
apart from the earlier cases is the impossibility of accommodating
even the parties to the cases.
“a courtroom should have enough facilities for a reasonable number of
the public to observe the proceedings, not too small as to render the
openness negligible and not too large as to distract the trial
participants from their proper functions,”
Technology tends to provide the only solution to break the inherent
limitations of the courtroom, to satisfy the imperative of a
transparent, open and public trial.
Guidelines must be laid down toward addressing the concernts of live
broadcasting:
Audio-visual recording for documentary and broadcasting purposes
Media entites must file with trial court a letter of application
Single fixed compact camera shall be installed inconspicuously
inside the courtroom
Transmittal of recording shall be conducted in the least physical
disturbance
Broadcasting for a particular day must be continuous and in its
entirety
No commercial break or any other gap
No voice overs
No repeat airing
Recording deposited in National Museum and the Records
Management and Archives Office
Recording of proceedins under supervision of the trial court
Court create special committee regardin the matter
And all other court directives shall be observed
Technology per se has always been neutral. It is the use and
regulation thereof that need fine-tuning. Law and technology
can work to the advantage and furtherance of the various rights
herein involved, within the contours of defined guidelines.
IN RE: PETITION TO ANNUL 98-7-02 SC (JP) allow the smooth flow of vehicular and pedestrian traffic.
July 7, 1998 | En banc | Freedom of Speech The ingress and engress in and to the premises of the courts
must never be obstructed.
Re: Guidelines on the Conduct of Demonstrations, Pickets, Rallies
and Other Similar Gatherings in the Vicinity of the Supreme Court and
All Other Courts

SUMMARY: The Supreme Court en banc resolved to develop policy and


procedural guidelines regarding the conduct of demonstrations and
rallies in the vicinity of the Supreme Court and lower courts in the name
of the administration of justice. It was assiled to have transgressed
freedom of expression.

98-7-02 SC prohibits demonstrators and rallyists from holding any activity


within a 200m radius from any hall of justice. It likewise prohibits all
obstruction in traffic within the premises of the courts. Lastly, it prohibits
all forms of camping and makeshift structures within the same radius.
The court reasoned out that free speech and peaceable assembly are
not absolute. They must be balanced with the need for impartial and
orderly judicial proceedings.

DOCTRINE: The right of a citizen to use the streets for communication of


views on national questions must be balanced with the need of our courts
for an atmosphere that will enable them to dispense justice free from bias
and unnecessary pressure.

FACTS:
(Note: This is not a case. 98-7-02-SC just imposed guidelines.)
In light of its inherent power to administration of justice and
complementing the resolution of the court in the case of Nestle Phil v.
Sanchez, the Court resolves to adopt formally the following policy
and procedural guidelines, regarding the conduct of
demonstrations, pickets, rallies and other similar gatherings in
the vicinity of the grounds and adjacent areas of the Supreme
Court and all other courts:
Demonstrators, picketers, rallyists and all other similar persons
are enjoined from holding any activity on the sidewalks
and streets adjacent to, in front of, or within a radius of
two hundred (200) meters from, the outer boundary of the
Supreme Court Building, any Hall of Justice, and any other
building that houses at least one
(1) court sala.
Demonstrators, picketers, rallyists and their sympathizers must
keep all public thoroughfares free and unimpeded to
Demonstrators, picketers, rallyists and their sympathizers are toward courts of justice when engaged in demonstrations, pickets,
prohibited from camping out on the streets, sidewalks or rallies and similar activities.
pavement adjacent to, in front of, or within a radius of
two hundred (200) meters from, the outer boundary of the
Supreme Court Building, any Hall of Justice, and any other
building that houses at least one (1) court sala. No
provisional shelters and kitchens, pickets' quarters, and
other similar makeshift structures shall be established in
said areas.
Any violation of this resolution shall be treated as contempt of court.
Members of the Bar violating this resolution may, in addition, be
subject to the administrative sanctions of fine, imprisonment,
suspension from the practice of law or disbarment as
circumstances may warrant.
The validity of the resolution was challenged on at least two grounds.
First, that it was an arrogation of legislative power thereby violating
the separation of powers. Second, that it transgressed freedom of
expression.

ISSUE/s:
WoN the resolution transgressed freedom of expression - NO

RULING: N/A

RATIO:
The Court characterized the first argument based on separation of
powers as “low watts” asserting against its “high wattage” right to
promulgate rules regulating conduct of demonstrations in the
vicinity of courts to assure our people of an impartial and
orderly administration of justice.
Free speech and peaceable assembly, along with the other intellectual
freedoms, are highly ranked in our scheme of constitutional values.
These freedoms, however, are not absolute.
The right of a citizen to use the streets for communication of views on
national questions must be balanced with the need of our courts for
an atmosphere that will enable them to dispense justice free from
bias and unnecessary pressure.
The courts should be preserved from all forms of distracting, degrading
and prejudicial influences that threaten the fair and orderly
administration of justice.
Lawyers of parties with cases pending in courts have a duty to properly
apprise their clients on matters of decorum and proper attitude
As officers of the court, they must help to preserve the dignity of the
courts and to insulate the courts from all forms of influence that may
adversely affect judicial impartiality and violate a party's right to due
process.
US v. GRACE (HENRY) attorney regarding the legality of his activities who informed him that
April 20, 1983 | White, J. | Freedom of Expression what was

PETITIONER: United States


RESPONDENTS: Mary Grace

SUMMARY: Zywicki distributed leaflets, handbills, and pamphlets in


separate occasions along the SC sidewalk, and in all instances he was
approached by a SC Police Officer, who said that he may be arrested of
violating USC Title 40, that prohibits leaflet distribution on SC grounds.
The same happened with respondent Grace, when she stood in the SC
sidewalk with a board that contained the text of the first amendment.

Grace and Zywicki, filed a suit in the District Court, assailing the statute’s
constitutionality, which was dismissed. Upon appeal to the CA, it was
affirmed, the statute was ruled to be unconstitutional.

This was affirmed in the SC ruling, which stated that such statute is
indeed violative of the rights enshrined in the first Amendment. That, such
rule, which totally bans the specified communicative activity on the public
sidewalks around the Court Grounds, cannot be justified as a reasonable
place restriction, primarily because it has an insufficient nexus with any of
the public interests that may be thought of to secure the objectives of
such statute.

DOCTRINE: A total ban on carrying a flag, banner, or device on the


public sidewalks does not substantially serve the purposes of the USC
Title 40—to provide for the maintenance of law and order on the Court
Grounds.

FACTS:
USc Title 40 was a law that for its purpose provides protection of the
building and grounds and of persons and property therein, as well as
the maintenance of proper order and decorum.
1978, Thaddeus Zywicki, standing on the sidewalk in front of the SC
building, distributed leaflets to passersby that contained reprints of a
letter to the editor of the Washington Post from a US Senator
concerning the removal of unfit judges from the bench.
A SC police officer approached him and said that Title 40 of the US Code
prohibited distribution of leaflets on SC grounds, which included the
sidewalk—Zwicki left.
1980, Zywicki dropped by the sidewalk again to distribute pamphlets
containing info on the forthcoming meertings and events on “the
oppressed peoples of Central America.” A SC police officer
approached him again and said that such act is prohibited by law—
Zywicki left.
Zywicki reappeared the following month in the sidewalk and distributed
handbills concerning oppression in Guatemala. He consulted with an
prohibited was the conduct done with specific intent to influence,
impede, or obstruct the administration of justice.
A SC police officer approached him and said that he will be arrested if he
persisted his leafleting. He then complained that he was denied a
right that others were granted, referring to the newspaper vending
machines located on the sidewalk. Nonetheless, Zywicki left.
The month after that, Mary Grace entered the sidewalk and began to
display a 4ft by 2.5ft sign that inscribed the text of the First
Amendment. A SC police officer approached her and said that she
would have to go across the street if she wished to display the sign.
Grace was informed of Title 40 of the US Code that prohibited her
conduct, and that if she did not comply, she will be arrested—Grace
left.
Zywicki and Grace filed the present suit in the US District Court, claiming
that Title 40 of the USC was unconstitutional. This was dismissed,
and upon appeal, the CA reversed the said ruling, to which the
Government appealed to the SC.

ISSUE/s:
WoN Title 40 of the USC is unconstitutional and violative of the freedom
of expression clause as enshrined in the First Amendment – YES

RULING: SC affirmed the CA ruling.

RATIO:
As a general matter, peaceful picketing and leafleting are expressive
activities involving “speech” protected by the First Amendment.
“Public places”, such as streets, sidewalks, and parks, historically
associated with the free exercise of expressive activiites, are
considered, without more, to be “public forums.” In such places, the
Government may enforce reasonable time, place, and manner
regulations, but additional restrictions, such as an absolute
prohibition of a particular type of expression, will be upheld only if
narrowly drawn to accomplish a compelling governmental interest.
The SC grounds are not transformed into “public forum” property merely
because the public is permitted to freely enter and leave the grounds
at practically all times and is admitted to the building during specified
hours. But where the sidewalks forming the perimeter of the grounds
are indistinguishable from any other sidewalks in Washington, they
should not be treated any differently, and thus are public forums for
First Amendment purposes.
Insofar as it totally bans specified communicative activity on the public
sidewalks around Court grounds, the assailed law cannot be justified
as a reasonable place restriction. A total ban on carrying a flag,
banner, or device on the public sidewalks does not substantially
serve the purposes of the USC Title 40—to provide for the
maintenance of law and order on the Court Grounds.
Nor does it serve the averred purpose of protecting the Court from
outside
influence or preventing it from appearing to the public that the Court is
subject to such influence or that picketing or marching is an acceptable way
of influencing the Court, where, as noted, the public sidewalks surrounding
th Court grounds are no different than other public sidewalks in the city.
Marshall, concurring and dissenting: the statute in no way
distinguishes sidewalks from the rest of the premises, and excising
the sidewalks from its purview does not bring it into conformity with
the First Amendment. Visitors to this Court do not lose their First
Amendment rights at the edge of the sidewalks any more than
“students or teachers shed their constitutional rights to freedom of
speech or expression at the schoolhouse gate” The statute is
unconstitutional on it face.
Stevens, concurring and dissenting: J. Stevens sees no reason to
stretch the language of the statute to encompass the activities of
either Zywicki or Grace. As a matter of statutory interpretation, the
Court should not infer that Congress intended to abridge free
expression in circumstances not plainly covered by the language of
the statute. As a matter of judicial restraint, the SC should avoid
unnecessary adjudication of constitutional questions.
Because neither of the appellees violated the statute, J. Stevens affirms
the CA ruling that requires appellants to be restrained from causing
grace and Zywicki’s arrest for engaging in the activities disclosed by
this record.
POLICARPIO v. MANILA TIMES (DANNAH) After the motion to dismiss of Manila Times et al. was denied by the CFI
May 30, 1962 | Concepcion, J | Libel: Requisites for Publication to Enjoy of Manila, they filed a join answer admitting the formal allegations but
Immunity denying the other allegations, and also filed for a counterclaim.
The court rendered decision upon the ground that Policarpio had not
PETITIONER: Lumen Policarpio proven
RESPONDENTS: The Manila Times Pub.co Inc., et al.

SUMMARY: Policarpio was the executive secretary of UNESCO. During her


occupancy, she initiated charges against her subordinate, Reyes which led
to Reyes’ separation from UNESCO. Reyes, in turn, initiated counter-
charges which were referred to Col. Alba, a Special Investigator of the Office
of the President. Reyes also filed with the Office of the City Fiscal of Manila
a complaint against Policarpio for alleged malversation and estafa through
falsification of public documents.

Saturday Mirror then published an article entitled “Woman Official Sued.


PCAC Raps L. Policarpio on Frauds…” (PCAC stands for Presidential
Complaints and Action Commission pero PCAC nakasulat sa title). It also
contained facts such as that PCAC was the one who initiated the filing of the
information after investigation by Col. Alba, that Policarpio stole stencils, and
that she reimbursed expenses which were made-up. Policarpio averred that
the article made her predicament seem worse, that PCAC was not the one
who filed the information but Reyes, and that the number of stencils were
minimal. These, she claimed, were defamatory.

The CFI ruled in favor of Manila Times, as Policarpio was not able to prove
the bad faith of Manila Times. However, the SC reversed this decision,
stating the article contained derogatory information and presented Policarpio
in a worse predicament, hence it did not enjoy immunity of publication.

DOCTRINE: Newspapers may publish news items relative to judicial, legislative


or other official proceedings, which are not of confidential nature, because the
public is entitled to know the truth with respect to such proceedings, which,
being official and
non-confidential, are open to public consumption. But to enjoy immunity, a
publication containing derogatory information must be not only true,
but also fair, and it must be made in good faith and without any
comments or remarks.

FACTS:
169. Lumen Policarpio seeks to recover damages because of the
publication in the Saturday Mirror of August 11, 1956 and in the Daily
Mirror of August 13, 1956 of 2 articles or news items, which are
claimed to be per se defamatory, libelous and false (MANILA TIMES
YUNG MAY ARI NUNG PUBLICATIONS)
that Manila Times et al. had acted maliciously in the publication,
although some portions were inaccurate or false.
Policarpio, a lawyer, was executive secretary of the local UNESCO
National Commission. As an officer, she initiated charges against
Herminia Reyes, one of her subordinates, and caused her to be
separated from service.
Reyes, in turn, preferred counter-charges which were referred to Col.
Alba, a Special Investigator in the Office of the President.
Pending investigation, Reyes filed with the Office of the City Fiscal of
Manila a complaint against Policarpio for alleged malversation and
estafa through falsification of public documents.
Meanwhile, an article appeared on the Saturday Mirror entitled “Woman
Official Sued. PCAC Raps L. Policaprio on Frauds…” and another
one on Daily Mirror entitled “Palace Opens Investigation Of Raps
Against Policarpio. Alba Probes Administrative Phase of Fraud
Charges Against UNESCO Woman…”
The important facts contained in both articles are:
Policarpio was charged with malversation and estafa filed with the city’s
fiscal office iled by the Presidential Complaints and Action
Commission (PCAC)
It was initiated by Col. Alba on charges filed by Herminia Reyes
Policarpio took stencils from the UNESCO storeroom and used these to
her personal advancements
Policarpio collected expenses for supposed trips like in Baler (she
reimbursed car expenses but she rode on a plane)
And expenses for going to a UNESCO Pangasinan project although she
was absent therein
***Daily Mirror corrected some details in the subsequent article…
The title of the August 11 article, WOMAN OFFICIALLY SUED, was
given prominence and its subtitle, PCAC RAPS POLICARPIO ON
FRAUD is not true.
Policarpio contends that the effect of these false statements was to give
the general impression that said investigation by Col. Alba had
shown Policarpio as guilty of the crimes, and as a consequence,
PCAC filed the corresponding complaints.
She also alleges that the articles did not mention the fact that the number
of stencils involved was only 18 or 20, and the sum allegedly
misappropriated by her was only P54.
180. By omitting these details, she avers that the article of August 11 had
the effect of conveying the idea that the offenses imputed to her were
more serious than they really were.

ISSUE/s:
WoN Manila Times is guilty of having published libelous/defamatory articles
– YES

RULING: Damages awarded to Policarpio.


RATIO: 210. How could they have acted in good faith in falsely stating
It is obvious that the filing of criminal complaints with the city fiscal's that the complainst had been filed by the PCAC? Either they
office by another agency of the Government, like the PCAC, imparts knew the truth about it or they didn’t know.
the ideal that the probability of guilt on the part of the accused is
greater than when the complaints are filed by a private individual.
This is especially when the latter is a former subordinate of the alleged
offender, who was responsible for the dismissal of the complainant
from her employment.
It is only too apparent that the article published on August 11, 1956,
presented the plaintiff in a more unfavorable light than she actually
was.
It goes without saying that newspapers must enjoy a certain degree of
discretion in determining the manner in which a given event should
be presented to the public, and the importance to be attached
thereto, as a news item, and that its presentation in a sensational
manner is not per se illegal.
Newspapers may publish news items relative to judicial, legislative or
other official proceedings, which are not of confidential nature,
because the public is entitled to know the truth with respect to such
proceedings, which, being official and non-confidential, are open to
public consumption.
But, to enjoy immunity, a publication containing derogatory
information must be not only true, but also fair, and it must be
made in good faith and without any comments or remarks.
Article 354 of the RPC provides that “Every defamatory imputation is
presumed to be malicious, even if it be true, if no good intention and
justifiable motive for making it is shown, except in the following
cases:
A private communication made by any person to another in the
performance of any legal, moral or social duty
A fair and true report, made in good faith, without any comments or
remarks, of any judicial, legislative or other official proceedings
which are not of confidential nature, or of any statement, report or
speech delivered in said proceedings, or of any other act performed
by public officers in the exericse of other functions.
In the case at hand, aside from containing information derogatory to the
plaintiff, the article published on August 11, 1956, presented her in a
worse
predicament than that in which she, in fact, was.
209. What is more, the subtitle of the article "PCAC RAPS L.
POLICARPIO ON FRAUD" is a comment or remark, besides being
false. Accordingly, the defamatory imputations contained in said
article are "presumed to be malicious".
If they did, then the publication would be malicious. If not, they acted under a
misapprehension of facts and they were guilty of negligence in making said
statement.
Although the August 13 article rectified a major inaccuracy, it does not wipe out
the responsibility arising from the first article.
LOPEZ v. CA (ARIELLE) The Philippines defense establishment rushed to the island with a
July 31, 1970 | Fernando, J. | Freedom of the Press; Libel platoon of scout rangers led by Maj. Encarnacion. Upon arriving at
the island, Maj. Encarnacion and his men found instead of alleged
PETITIONER: Eugenio Lopez, publisher and owner of the MANILA killers, Fidel Cruz who merely wanted transporation back to Manila.
Maj. Encarnacion branded this as a “hoax,” which was the term used
CHRONICLE and Juan T. Gatbonton
by other newspapers when referring to the incident.
RESPONDENTS: Court of Appeals and Fidel G. Cruz
This Week Magazine of the Manila Chronicle mentioned that while the
SUMMARY: Fidel Cruz, a sanity inspector in Babuyan Islands, sent a Fidel Cruz story turned out to be false, it brought to light the misery of
distress signal to the passing US Airforce, who in turn relayed the the people in that place, with almost everybody sick only two
message to Manila. Cruz was given a sustenance kit, containing a two- individuals able to read and write, food and clothing being scarce.
way radio. Cruz used this to transmit the message that there has been In the This Week Magazine’s Special Year End Quiz issue, reference was
killings in the island and that people are in grave danger. The Philippines made to a health inspector who suddenly felt lonely in his isolate post,
cooked up a story about a murderer running loose on the island, so that
establishment defense acted on such message and sent a platoon of
he could be ferried back to civilization. He was given the label “Hoax of
scout rangers to the island. Upon reaching the island, the officers
the Year.”
discovered that there was actually no killers in the island, but Cruz only
The magazine carried photographs of the person purporting to be Fidel
made such message so he can be transported back to Manila.
Cruz, but the pictures published were that of private respondent Fidel
This Week Magazine inadvertently published a picture of private G. Cruz, a businessman contractor from Bulacan. The switch was
respondent Fidel G. Cruz, instead of Fidel Cruz who caused the hoax. inadvertently made in the course of the preparation of the quiz
Due to this, private respondent filed a recovery for damages against format.
petitioners alleging the defamatory character of such publication. The CFI As soon as the error was brought to the attention of petitioners, the
awarded P5,000 as actual damages, another P5,000.00 for moral correction was immediately published in This Week Magaizne.
Private respondent Fidel G. Cruz sued petitioners in the CFI for the
damages, and P1,000 for attorney’s fees.
recovery of damages alleging the defamatory character of the
The SC held that the correction promptly made by petitioners calls for a publication of his picture. After trial, he was awarded P5,000.00 as
reduction in the damages awarded. There was no proof of any actual actual damages, another P5,000.00 for moral damages, and
pecuniary loss arising from the above publication. Moreover, in a civil P1,000.00 for attorney’s fees. Judgment was affirmed on appeal to
action for libel, so long as it is done in good faith, newspapers have the the CA. Hence, this petition.
legal right to have and express opinions on legal questions. To deny them
ISSUE/s:
that right would infringe upon the freedom of the press.
WoN petitioners should be liable for damages for the inadvertent
DOCTRINE: In preparation of stories, press reporters and edition usually publication of private respondent’s picture – YES; but it should be
have to race with their deadlines and with good faith and reasonable reduced.
care, they should not be held to account, to a point of suppression,
RULING:
for honest mistakes or imperfections in the choice of words.
The decision of the CA affirming the lower court decision is hereby
MODIFIED. Petitioners Eugenio Lopez and Juan Gatbonton being ordered to
pay jointly and severally the sum of P500.00 as moral damages and the
additional amount of P500.00 for attorney’s fees.
FACTS: RATIO:
In January 1956, there appeared on the front page of The Manila Petitioners anchor their petition on the freedom of the press to
Chronicle a news story of a sanity inspector assigned to the Babuyan demonstrate that no action for libel would lie arising from the
Islands, Fidel Cruz, sending a distress signal to a passing US publication of the picture. No liability will arise if it could be
Airforce plane which relayed the message to Manila. demonstrated that it comes within the all-embracing scope of
Fidel Cruz was not ignored, and an American Army dropped an freedom of the press.
emergency sustenance kit containing, among other things, a two- Included here is the widest latitude of choice as to what items should see
way radio set. He utilized it to inform authorities that the people in the light of day so long as they are relevant to a matter of public
Babuyan Islands were living in terror due to a series of killings. interest.
Libel was defined in the old Libel Law as a “malicious defamation,
expressed etiher in writing, printing, or by signs or pictures or the like,
tending to black th memory of one who is dead or to impeach the
honesty, virtue, or reputation,
or publish the alleged or natural defects of one who is alive, and For liability to arise without offending press freedom, the test is
thereby expose him to public hatred, contempt, or ridicule.” to determine if the statement was made with actual malice –
According to the standard treatise of Newell on Slander and Libel: that is, with knowledge that it was false or with reckless disregard
Publication of person’s photograph in connection with an article of whether it was false or not.
libelous of a third person, is a libel on the person whose picture is No inroads on press freedom should be allowed in the guise of punitive
published. action
Why libel has both criminal and civil aspect: Libeling a person results
in depriving him of his good reputation. Reputation is a thing of
value, an impairment of it is a personal wrong. On the other hand,
publication of defamatory statements tends strongly to induce breach
of the peace by the person defamed. From this perspective, libel is
a crime and as such, subjects the offender to a fine or
imprisonment.
In the case of Peck v. Tribune Co., the plaintiff complained of her picture
being published in an advertisement in defendant’s newspaper.
Justice Holmes pointed out: There was some suggestion that
defendant published the picture by mistake. But that fact was no
excuse.
A libel is harmful on its face. If a man sees fit to publish manifestly
hurtful statements without other justification, the usual principles of
tort will make him liable if the statements are false, or are true only of
someone else.
Many things that are defamatory may be said with impunity through speech.
But what gives the sting to the writing is its permanence of form.
The spoken word dissolves, but the written one abide and perpetuates
the scandal. Writing extends to symbols – pictures, hierogylphics,
shorthand notes, etc.
There would be curtailment of press freedom if an action for libel were
not rigorously scrutinized. In the leading case of United States v.
Bustos, J. Malcom said: “The interest of society and the maintenance
of good government demand a full discussion of public affairs.”
A public officer must not be too thin-skinned with reference to
comment upon his official acts. Of course, criticism does not
authorize defamation. The criticism must be for the common good.
In a civil action for libel, so long as it is done in good faith, newspapers
have the legal right to have and express opinions on legal questions.
To deny them that right would infringe upon the freedom of the press.
Newspapers should be given such leeway and tolerance as to enable
them to courageously and effectively perform their important role in
our democracy. In preparation of stories, press reporters and edition
usually have to race with their deadlines and with good faith and
reasonable care, they should not be held to account, to a point of
suppression, for honest mistakes or imperfections in the choice
of words.
Debate on public issues should be uninhibited and that it may well
include vehement and sometimes unpleasant sharp attacks on
government and public officials.
visited on what otherwise could be characterized as libel whether in the
form of printed words or a defamatory imputation resulting from the
publication of respondent’s picture with the caption as in the case here.
The correction promptly made by petitioners calls for a reduction in the
damages awarded. There was no proof of any actual pecuniary loss
arising from the above publication.

Dizon, Dissenting
The facts of the present case show no criminal liability for libel. It is
clear upon the record that there is no evidence of actual malice –
there is no evidence that petitioners knew that the imputation
made to respondent Cruz was false.
The alleged “hoax” if considered without passion and in the right
perspective, ascribes to him nothing immoral or involving moral
turpitude.
Whatever negligence there might have been on the part of
petitioners would amount only to what might be considered as
“excusable negligence” – thus eliminating any idea of malice or
intention to cause injury on their part.
VASQUEZ v. CA (IYA)
SEPTEMBER 15, 1999 | Mendoza, J. | Freedom of Expression: Libel RULING: WHEREFORE, petitioner is ACQUITTED.

PETITIONER: Rodolfo R. Vasquez


RESPONDENTS: Court of Appeals, RTC, and People of the Philippines
(wow naman kinalaban na niya lahat)

SUMMARY: Vasquez was charged with libel for the statements he made
to newspaper reporters which was published. The statements pertained
to Barangay Chairman Olmedo who was alleged to be involved in land
grabbing and other illegal activities within Tondo Foreshore Area. The
RTC and CA convicted Vasquez of libel.

The SC reversed the said decisions and acquitted Vasquez on ground


that he was able to prove that such statements made had basis as
corroborated by the NHA resolution he submitted.

DOCTRINE: A rule placing on the accused the burden of showing the


truth of allegations of official misconduct and/or good motives and
justifiable ends for making such allegations would not only be contrary to
Art. 361 of the Revised Penal Code but it would, above all, infringe on
the constitutionally citizens from performing their duties as members of a
self governing community.

FACTS:
Rodolfo Vasquez is a resident of Tondo Foreshore Area.
He and some 37 families from the area went the national Housing
Authority regarding their complaint against Baranggay Chairman
Jamie Olmedo.
After meeting with the NHA officials, they were met and interviewed by
newspaper reporters.
The next day, a news article appeared in the newspaper Ang Tinig ng
Masa
The article contended that Olmedo is engaged in land grabbing and
illegal gambling and stealing of chickens at the Tondo Foreshore
Area.
Olmedo filed a complaint for libel against petitioner alleging that the
statements cast attacked his integrity and damaged his reputation.
Vasquez entered a plea of not guilty.
The RTC rendered a judgment finding petitioner guilty of liber. On appeal,
the CA affirmed in toto.
Hence, the present petition before the SC.

ISSUE/s:
WON Vasquez is guilty of libel? - NO
RATIO:
Petitioner contends that what he said was true and was made with good
motives and for justifiable ends.
To be liable for Libel under Art. 353 of the RPC the following elements
must concurr:
The allegation of a discreditable act or condition concerning another
Publication of the charge
Identity of the person defamed
existence of malice.
An allegation is considered defamatory if it ascribes to person the
commission of a crime, the possession of a vice or defect, real or
imaginary, or any act, omission, condition, status or circumstance
which tend to dishonor or discredit or put him in contempt, or which
tends to blacken the memory of one who is dead.
There is no doubt that the first three elements are present. Nor is there
any doubt that the defamatory remarks referred to complainant and
were published.
The remaining question is whether from the fact that the statements were
defamatory, malice can be presumed so that it was incumbent upon
petitioner to overcome such presumption.
Art. 361: If the defamatory statement is made against a public
official with respect to the discharge of his official duties and
functions and the truth of the allegation is shown, the
accused will be entitled to an acquittal even though he does
not prove that the imputation was published with good
motives and for justifiable ends.
In this case, petitioner was able to prove the truth of his charges against
the barangay official.
Together with the recommendation of NHA General Manager Gaudencio
Tobias for the filing of administrative charges against NHA officials
responsible for the alleged irregular consolidation of lots in Tondo to
Jaime and Victoria Olmedo, Vasquez was able to prove his
statements were true.
Even if the defamatory statement is false, no liability can attach if it
relates to official conduct, unless the public official concerned proves
that the statement was made with actual malice that is, with
knowledge that it was false or with reckless disregard of whether it
was false or not.
A rule placing on the accused the burden of showing the truth of
allegations of official misconduct and/or good motives and
justifiable ends for making such allegations would not only be
contrary to Art. 361 of the Revised Penal Code but it would,
above all, infringe on the constitutionally citizens from
performing their duties as members of a self governing
community.
TIME v. FIRESTONE (ELIEL) schoolteacher; on grounds of extreme cruelty and adultery; after 6
March 2, 1976 | Rehnquist, J. | Freedom of the Press years of marriage, one son; in West Palm Beach, Fla. The 17 month
intermittent trail
PETITIONER: Time, Inc.
RESPONDENTS: Mary Alice Firestone

SUMMARY: Time published in its Milestones section the divorce of Alice


and Russell Firestone. It indicated the reason for which being extreme
cruelty and adultery. Alice demanded that it be retraced from its
publication, but Time’s refused. Time contends that it had the right to
publish such because (1) Alice was a public figure; and (2) there should
be actual malice.

The SC ruled that Alice is not a public figure as defined in jurisprudence


because she had no special prominence in the resolution of public
questions. Moreoever, there is no blanket provision that all judicial
proceedings are of public interest and may be published by Times. When
it comes to defamatory and inaccurate reports, there is no protection by
the constitution. And that Time’s had knowledge of the order by the court
and it could not escape civil liability.

DOCTRINE: It is argued that information concerning proceedings in our


nation’s courts may have such importance to all citizens as to justify
extending special 1st Amendment protection to the press when reporting
on such events. We have recently accepted a significantly more confined
version of this argument by holding that the Constitution precludes States
form imposing civil liability based upon the publication of truthful
information contained in official court records open to public inspection.

FACTS:
Mary Alice Firestone, married Russell Firestone, the scion of one of
America’s wealthier industrial families, in 1961. In 1964, they
separated and Alice filed a complaint for separate maintenance in
the Circuit Court of Palm Beach Country, Fla. Her husband
counterclaimed for divorce on grounds of extreme cruelty and
adultery. After a lengthy trial the Circuit Court issued a judgment
granting the divorce requested by Alice’s husband.
Time’s editorial staff, headquartered in New York, was alerted by a wire
service report and an account in a New York newspaper to the fact
that a judgement had been rendered in the Firestone divorce
proceeding. The staff subsequently received further information
regarding the Florida decision from Time’s Miami bureau chief and
froma “stringer” working ona special assignment basis in the Palm
Beach area. On the basis of these four sources, Time’s staff
composed the following item, which appeared in the magazine’s
“Milestones” section the following week.
“DIVORCED. By Rusell A. Firestone Jr., 31, heir to the tire fortune: Mary
Alice Sullivan Firestone, 32, his third wife; a ontime Palm Beach
produced enough testimony of extramarital adventures on both privilege is simply too broad. Imposing upon the law of private
sides, said the judge, ‘to make Dr. Freud’s hair curl.” defamation the rather
Within a few weeks of the publication of this article Alice demanded in
writing a retraction from Time, alleging that a portion of the article
was “false, malicious and defamatory.” Hence this petition.
Time initially contends that it cannot be liable for publishing any
falsehood defaming Alice unless it is established that the publication
was made “with actual malice,” as that term is defined in New York
Times Co. v. Sullivan.
Time advances two arguments in support of this contention: that Alice is
a “public figure” within this Court’s decisions extending New York
Times to defamation suits brought by such individuals; and that the
Time item constituted a report of a judicial proceeding, a class of
subject matter which Time claims deserves the protection of the
“actual malice” standard even if the story is proved to be defamatory,
false or inaccurate.

ISSUE/s:
WoN Time exercised freedom of the press – NO

RULING: Remanded for further proceedings.

RATIO:
Public Figure: “For the most part those who attain this status have
assume roles of special prominence in the affairs of society. Some
occupy positions of such persuasive power and influence that the
are deemed public figures for all purposes. More commonly, those
classed as public figures have thrust themselves to the forefront of
particular public controversies in order to influence the resolution
of the issues involved.”
Dissolution of a marriage through judicial proceedings is not the sort of
“public controversy” referred to in Gertz, even though the marital
difficulties of extremely wealthy individuals may be of interest to
some porition of the reading public.
Alice assumed no “special prominence in the resolution of public
questions.” We hold Alice was not a “public figure” for the
purpose of determining the constitutional protection afforded
Time’s report of the factual and legal basis for her divorce.
It is argued that information concerning proceedings in our nation’s
courts may have such importance to all citizens as to justify
st
extending special 1 Amendment protection to the press when
reporting on such events. We have recently accepted a
significantly more confined version of this argument by holding
that the Constitution precludes States from imposing civil liability
based upon the publication of truthful information contained in
official court records open to public inspection.
It may be argued that there is still room for application of the New York
Times protections to more narrowly focused reports of what actually
transpires in the courtroom. But even so narrowed, the suggested
drastic limitations worked by New York Times cannot be justified by
generalized references to the public interest in reports of judicial proceedings.
The public interest in accurate reports of judicial proceedings is
substantially protected by Cox Broadcasting Co., supra. As to
inaccurate and defamatory reports of facts, matters deserving
no 1st Amenedment protection, we think Gertz provides an
adequate safeguard for the constitutionally protected interests
of the press and affords it a tolerable margin for error by
requring some type of fault.
The Supreme Court of Florida on appeal concluded that the ground
actually relied upon by the divorce court was “lack of domestication
of the parties,” a ground not theretofore recognized by Florida law.
The Supreme Court nonetheless affirmed the judgment dissolving
the bonds of matrimony because the record contained sufficient
evidence to establish the ground of extreme cruelty.
Time must be able to establish not merely that the item reported
was a conceivable or plausible interpretation of the decree, but
that the item was factually correct. We believe there is ample
support for the jury’s conclusion, affirmed by the SC of Florida,
that this was not the case.
There was, therefore, sufficient basis for imposing liability upon
Time if the constitutional limitations we announce in Gertz have
been satisified. These are a prohibition against imposing
liability without fault and the requirement that compensatory
awards “be supported by competent evidence concerning the
injury.”
The failure to submit the question of fault to the jury does not of itself
establish noncompliance with the constitutional requriements
established in Gertz, however. The 1st and 14th Amendments do
not impose upon the
States any limitations as to how, within their own judicial
systems, factfinding tasks shall be allocated. If we were satisfied
that one of the Florida courts which considered this case had
supportably ascertained Time was at fault we would be required to
affirm the judgment below.
A careful examiniation of the final decree prior to publication would
have clearly demonstrated that the divorce had been granted o
the grounds of extreme cruelty, and thus the wife would have
been saved the humiliation of being accused of adultery in a
nationwide magazine. This is a flagrant example of ‘journalistic
negligence.’”
Nothing in that decision or in the 1 st or 14th Amendment requires that in a
libel action an appellate court treat in detail by written opinion all
contentions of the parties, and if the jury or trial judge had found fault in
fact, we would be quite willing to read the quoted passage as affirming
that conclusion.
Brennan, J. Dissenting: The First Amendment insulates from
defamation liability a margin for error sufficient to ensure the
avoidance of crippling press self-censorship in the field of reporting
public judicial affairs. To be adequate, that margin must be both of
sufficient readth and predictable in its application. In my view,
therefore, the actual-malic standard of New York Times must be met
in order to justify the imposition of liability in these circumstances.
HUSTLER MAGAZINE v. FALWELL (JP) In small print at the bottom of the page, the ad contains the disclaimer,
February 24, 1988 Rehnquist, C. J. | Freedom of Speech "ad parody -- not to be taken seriously."

PETITIONER: Hustler Magazine, Inc.


RESPONDENTS: Jerry Falwell

SUMMARY: Petitioner Hustler Magazine published a parody fo


respondent Falwell, a commentator on politics. Falwell in the parody was
portrayed to be a hypocritical preacher. He was drawn to be in a drunken
incestuous rendezvous with his mother. Respondent Falwell filed for
damages upon publication of such material. Lower court and CA favored
respondent on the damages.

SC however, reversed. According to the SC, the actual malice rule should
be applied in the case (see doctrine). Because there is an absence of
showing that the publication contains any false statement of fact, such
expression shall be protected by the First Amendment. Respondent is a
public official and it would produce a chilling effect on publishers’
expression if the rules where any stricter.

DOCTRINE: Actual Malice Rule: Public figures and public officials may
not recover for the tort of intentional infliction of emotional distress by
reason of publications such as the one here at issue without showing, in
addition, that the publication contains a false statement of fact which was
made with "actual malice," i.e., with knowledge that the statement was
false or with reckless disregard as to whether or not it was true.

FACTS:
Petitioner Hustler Magazine, Inc., is a magazine of nationwide circulation.
Respondent Jerry Falwell, a nationally known minister who has been
active as a commentator on politics and public affairs, sued petitioner
and its publisher, petitioner Larry Flynt, to recover damages for
invasion of privacy, libel, and intentional infliction of emotional
distress.
The inside front cover of the November, 1983, issue of Hustler Magazine
featured a "parody" of an advertisement for Campari Liqueur that
contained the name and picture of respondent and was entitled
"Jerry Falwell talks about his first time."
This parody was modeled after actual Campari ads that included
interviews with various celebrities about their "first times."
Copying the form and layout of these Campari ads, Hustler's editors
chose respondent Falwell as the featured celebrity and drafted an
alleged "interview" with him in which he states that his "first time"
was during a drunken incestuous rendezvous with his mother in an
outhouse.
The Hustler parody portrays respondent Falwell and his mother as drunk
and immoral, and suggests that respondent is a hypocrite who
preaches only when he is drunk.
Soon after the November issue of Hustler became available to the public, the petitioner (rather than the truth or falsity of the claim as in the Anti
offended respondent Fawlell brought this diversity action. malice
The District Court jury decided in favor of petitioner magazine on the
invasion of privacy claim and the libel. They explained that the ad
parody could not reasonably be understood as describing actual
facts about respondent Falwell.
However, the jury found in favor of respondent Falwell on the intentional
infliction of emotional distress claim. CA affirmed. Hence this appeal.

ISSUE/s:
WoN a public figure may recover damages for emotional harm caused by
the publication of an ad parody offensive to him, and doubtless gross
and repugnant in the eyes of most – NO

RULING: The judgment of the CA is reversed. Respondent’s claim cannot


consistently for a basis for the award of damages.

RATIO:
According to respondent Falwell, the State's interest in protecting
public figures from emotional distress is sufficient to deny First
Amendment protection to speech that is patently offensive and
is intended to inflict emotional injury, even when that speech could
not reasonably have been interpreted as stating actual facts about
the public figure involved.
The SC does not support such contention.
The SC has consistently ruled that a public figure may hold a speaker
liable for the damage to reputation caused by publication of a
defamatory falsehood, but only if the statement was made "with
knowledge that it was false or with reckless disregard of
whether it was false or not (Actual Malice Rule).
False statements of fact are particularly valueless; they interfere with the
truthseeking function of the marketplace of ideas, and they cause
damage to an individual's reputation that cannot easily be repaired
by counterspeech, however persuasive or effective.
But even though falsehoods have little value in and of themselves,
they are “nevertheless inevitable in free debate.”
A rule that would impose strict liability on a publisher for false
factual assertions would have an undoubted "chilling" effect on
speech relating to public figures that does have constitutional
value.”
Freedom of expression require “breathing space.” This breathing
space is provided by a constitutional rule that allows public
figures to recover for libel or defamation only when they can
prove both that the statement was false and that the statement
was made with the requisite level of culpability.
Respondent Falwell is saying that the actual malice rule should not apply
but instead look into the intent to inflict serious emotional distress of
rule). The Court rejects such contention.
Were the Court were to hold otherwise, there can be little doubt that
political cartoonists and satirists would be subjected to damages
awards without any showing that their work falsely defamed its
subject.
Webster's defines a caricature as "the deliberately distorted picturing or
imitating of a person, literary style, etc. by exaggerating features or
mannerisms for satirical effect."
The political cartoon is a weapon of attack, of scorn and ridicule and satire; it
is least effective when it tries to pat some politician on the back. It is
usually as welcome as a bee sting, and is always controversial in some
quarters.
The Court has long recognized that not all speech is of equal First
Amendment importance. But the sort of expression involved in this
case does not seem to us to be governed by any exception to the
general First Amendment principles (vulgar words, inciting words,
etc.).
JAL v. SIMANGAN (HENRY) his movements when the aircraft stops over Narita. This was ignored,
April 22, 2008 | Reyes, R.T., J. | Freedom of Speech and of the Press and he was bumped off the flight.
Respondent went to JAL’s ground office and waited for 3 hours. The plane
PETITIONER: Japan Airlines took off and he was left in the PH. Afterwards, he was informed that his
travel
RESPONDENTS: Jesus Simangan

SUMMARY: Simingan was about to undergo kidney transplant in UCLA


for his ailing cousin, Loreto. In his flight to US via Narita, he was halted
and bumped off the plane by the cabin crew, for the allegation that his
travel documents were fake. Upon verifying that his documents were
legit, he filed a suit claiming damages in the RTC that they breached their
contract of carriage, to which he publicized in newspapers. The RTC and
CA ruled in his favor. Upon appeal, JAL claimed that they suffered
damages too because of the attack on their brand made by respondent
when he caused the publication in the newspapers.

SC ruled that indeed JAL breached their contract of carriage. They


likewise ruled that respondent is not laible for causing the publication of
the issue in the newspapers, as it is covered by the constitutional right to
freedom of speech and of the press, specifically fair commentaries on
matters of public interest.

DOCTRINE: The rule on privileged commentaries on matters of public


interest applies not only to public officials but extends to a great variety of
subjects, and includes matters of public concern, public men, and
candidates for office.

FACTS:
Respondent Jesus Simingan decided to donate a kidney to his ailing
cousin, Loreto in UCLA School of Medicine. Upon request of UCLA,
respondent underwent lab tests at the National Kidney Institute (NKI)
in QC. Fortunately, his blood and tissue type were well-matched with
Loreto’s.
Respondent needed to go to the US to complete his preliminary work-up
and donation surgery. UCLA wrote a letter to the American Consulate
in Manila to arrange his visa, to which he was issued an emergency
US Vica by the US Embassy.
Respondent purchased a round trip ticket from petitioner JAL (USD
1,485.00) and was issued with a boarding pass. He was scheduled
to a flight bound for LA, via Narita, Japan.
While inside the plane, JAL’s airline crew suspected respondent of
carrying a falsified travel document and imputed that he would only
use the trip to the US as a pretext to stay and work in Japan. The
crew asked respondent of his travel docs, to which he was ordered
to stand up and leave the plane.
Respondent protested, claiming that he was issued a US visa just to
allow him to board the plane, he pleaded with JAL to closely monitor
documents were, indeed, in order. He was refunded the cost of his or concern is a legitimate topic of a public comment that may be
ticket plane less the sum of USD 500 deducted by JAL. His US visa validly published.
was subsequently cancelled.
Respondent filed an action fro damages vs. JAL with the RTC, and
calimed that he was not albe to donate his kidney to his cousin and
suffered terrible embarrassment and mental anguish, to which JAL
denied, and as defense said that a need for the travel documents to
be authenticated by the US embassy was needed because nobody
from JAL’s airport staff encountered a parole visa before. They said
that respondent may book the flight the following day and alleged
that he even agreed to be rebooked.
RTC ruled in favor of the respondent, that JAL breached its contract of
carriage; the mere fact that respondent already had a seat number
and everything proved of th authenticity of his travel documents.
Important note: Respondent Simangan caused publications of his
complaint agaisngt JAL in the newspaper, and JAL is claiming that
they suffered damages because its reputation was tarnished
because of the said issue.
JAL appealed to the SC, which affirmed the ruling of the RTC but
lowered the damages against JAL. Hence, this petition.

ISSUE/s:
WoN JAL is guilty of breach of the contract of carriage – YES
WoN JAL may claim damages on account that respondent caused the
publishing of his complaint against the petitioner – NO

RULING: SC denied the petition, and affirmed with modification, insofar as


damages are concerned, the CA ruling.

RATIO:
Lawful acts give rise to no injury. Walang perhuwisyong maaaring idulot
ang paggamit sa sariling karapatan.
During trial, JAL presented a witness who testified that JAL suffered
further damages. Respondent caused publications of his subject
complaint against JAL in the newspaper for which JAL suffered
damages.
Although these additional damages allegedly suffered by JAL were not
incorporated in its Answer as they arose subsequent to its filing,
JAL’s witness was able to testify on the same before the RTC.
Hence, although these issues were not raised by the pleadjings, they
shall be treated in all respects as if they had been.
However, with the above premise JAL’s counterclaim cannot be granted.
JAL is a common carrier, with business mainly with the travelling public.
It invites people to avail themselves of the comforts and advantages
it offers. Since JAL deals with the public, its bumping off of
respondent without a valid reason naturally drew public attention and
generated a public issue.
The publications involved matters about which the public has the right to
be informed because they relate to a public sisue. This public issue
Assuming hat respondent indeed caused the publication of his complaint,
he may not be held liable for damages for it. The constitutional
guarantee of freedom of the speech and of the press includes fair
commentaries on matters of public interest, as explained in Borjal v.
CA.
Even though JAL is nto a public official, the rule on privileged commentiaries
on matters of public interest applies to it. The privilege applies not only
to public officials but extends to a great variety of subjects, and includes
matters of public concern, public men, and candidates for office.
Hence, pursuant to the Borjal case, there must be an actual malice in order
that a discreditable imputation to a public person in his public capacity or
to a public official may be actionable. To be considered malicious, the
libelous statements must be shown to have been written or published
with knowledge that they are false or in reckless disregard of whetehr
they are false or not.
Considering that the published articles involve matters of public interest
and that its expressed opinion is not malicious but based on
established facts, the imputations against JAL are not actionable.
JAL may not claim damages for them.
ARAFILES v. PHILIPPINE JOURNALISTS, INC. (DANNAH) jeepney going home. He warned her that if she tells authorities, he
March 25, 2004 | Carpio-Morales, J | Freedom of the Press will kill her and her family, or remover her from her job.
Another similar incident occurred on April 13. Director Arafiles again saw
PETITIONER: Catalino Arafiles her while she was waiting for her cousin to buy medicine in UN Ave.
RESPONDENTS: Phil. Journalists, Inc., Romy Morales, Max Buan Jr, et al. corner Taft, forced her to ride in his car (this time without chloroform).
He brought her to Flamingo Hotel again and when he was about to rape
SUMMARY: Emelita, who also worked in National Institute of Atmospheric
her, the bellboy of the hotel interrupted by pressing the buzzer
Sciences (NIAS) was raped and abducted by (petitioner Arafiles) the director
of NIAS on March 14, and was again abducted (and almost raped) on April because people were looking for Emelita downstairs.. Arafiles went
13. Respondent Morales, a reporter of respondent Philippine Journalists Inc. out, handed the boy money, and they exited through the fire exit.
(PJI), was there when Emelita narrated the events to Patrolman Chio from After Patrolman Chino made the entry in the Police Blotter (which was
the Western Police District (WPD) headquarters on April 13. The next day, perused by Morales), Morales interviewed Emelita. After this, he
his article entitled “GOV’T EXEC RAPES COED” appeared on People’s attempted to get Arafiles’ side and called the NIAS office but it was
Journal Tonight, and included the details of said abuse. already closed.
One year after, Arafiles filed a Complaint with the RTC, as his image and On April 14, Morales’ report appeared as headline on People’s Journal
reputation was gravely affected by said article. He claimed that the article Tonight, entitled “GOV’T EXEC RAPES COED” and contains details
was grossly malicious and overly sensationalized by Morales. The RTC of Emelita’s narration of events.
ruled in favor of Arafiles and awarded damages. This was brought up to the A year after the publication, or April 13, 1988, Arafiles instituted a
CA who reversed the ruling of the lower court, stating that it was neither a complaint before the RTC, claiming that the report was grossly
false allegation of fact or a comment based on false supposition. The SC malicious and overly sensationalized by Morales, which was allowed
affirmed the CA’s decision ruling that the presentation of the news item may for publication by respondent Villareal as president of Phil Journalists
have been in a sensational manner, but it is not per se illegal. Moreover, it
Inc (PJI) et al.
had factual basis, as the article stemmed from the police blotter of Emelita.
His reputation as director of the NIAS was injured (he didn’t get the
DOCTRINE: In determining the manner in which a given event should be promotion he wanted), he became the object of public contempt and
presented as a news item and the importance to be attached thereto, ridicule and was depicted as a sex-crazed stalker and serial rapist.
newspapers must enjoy a certain degree of discretion. In their Answer, respondents PJI prayed for the dismissal of the
Complaint as the news item was sourced from the Police Blotter,
which is an official public document.
FACTS: The RTC ruled in favor of Arafiles and awarded damages. The CA
About 2 a.m. on April 14, 1987, while respondent Morales, a reporter of reversed, stating that “when the discreditable imputation is directed
People’s Journal Tonight was at the Western Police District along against a public person in his public capacity, it is not necessarily
United Nations Avenue, Emelita Despuig (Emelita), an employee of actionable. In order that it can be actionable, it must either be a false
the National Institute of Atmospheric Sciences (NIAS), lodged a allegation of fact or a comment based on a false supposition…”
complaint against petitioner Arafiles, a NIAS director. Hence the present petition.
The complaint stemmed from two incidents. Emelita executed a sworn
statement narrating the events surrounding the rape which occurred ISSUE/s:
on March 14, 1987. WoN the CA erred in holding that the publication of the news item was
She stated that on March 14, she was waiting for a ride hom in front of Plaza not attended with malice – NO
Miranda when petitoner Arafiles pulled her into his car, made her smell
RULING: Petition denied. This Court finds that case against respondents has
tissue which was color yellow (chloroform), and had a knife to her neck.
He brought her to Flamingo Hotel where he took her virginity. He even not been sufficiently established by preponderance of evidence.
slipped P55.00 in her bag after and brought her to a place where she RATIO:
rode a The complaint Arafiles instituted was for damages based on Article 33 of
the NCC.
A civil action for libel under this article shall be instituted and prosecuted
to final judgment and proved by preponderance of evidence
separately from and entirely independent of the institution, pendency
or result of the criminal action because it is governed by the
provisions of the New Civil Code and not by the Revised Penal Code
governing the criminal offense charged and the civil liability arising
therefrom.
In actions for libel, it is axiomatic that the published work alleged to
contain libelous material must be examined and viewed as a whole.
A publication claimed to be defamatory must be read and construed in
the sense in which the readers to whom it is addressed would
ordinarily understand it.
Arafiles brands the article as “malicious sensationalization”. He claims
that the police blotter which was the sole basis for the news item
plainly shows that there was only one count of abduction and rape
reported by Emelita.
However, the report showed that there was one abduction with rape
incident (March 14) and an abduction incident (April 13).
The presentation of the news item may have been in a sensational
manner, but it is not per se illegal.
PJI should have been more circumspect in their choice of words as the
article made it seem like a certain director of NIAS actually committed
the crime.
The succeeding paragraphs in which Arafiles and Emelita were
eventually identified sufficiently convey to the readers, however, that
the narration of events was only an account of what Emelita had
reported.
In determining the manner in which a given event should be presented
as a news item and the importance to be attached thereto,
newspapers must enjoy a certain degree of discretion.
The newspapers should be given such leeway and tolerance as to
enable them to courageously and effectively perform their important
role in our democracy.
In the preparation of stories, press reporters and [editors] usually have to
race with their deadlines; and consistently with good faith and
reasonable care, they should not be held to account, to a point of
suppression, for honest mistakes or imperfection in the choice of
words.
BAGUIO MIDLAND COURIER v. COURT OF APPEALS (ARIELLE) Baguio City and other provinces within the Cordillera region.
November 25, 2004 | Chico-Nazario, J. | Freedom of the Press Petitioner Cecille Afable (Afable) was Baguio Midland Courier’s
editor-in-chief and one of its columnists who ran
PETITIONER: Baguio Midland Courier and Cecille Afable
RESPONDENTS: Court of Appeals

SUMMARY: Ramon Labo was a mayoralty candidate and as part of his


campaign propaganda, political ads appeared in the various issues of Baguio
Midland Courier and campaign paraphernalia were printed by Baguio
Printing. Cecille Afable, Baguio Midland’s editor-in-chief, wrote in her column
a series of articles dealing with the candidates for the various elective
positions in Baguio. A portion of the column allegedly referred to Labo as
“dumpty in the egg” and described him as the candidate with the most
imponderables about him. Moreover, the article also revealed his
indebtedness to Baguio Midland for unpaid campaign materials.

Labo claimed that Afable’s column was tainted with malice and filed a
libel case against the petitioners. Petitioners Baguio Midland and
Hamada denied that petitioner Afable’s articles were libelous. They also
claimed that according to their company records, Labo still owed them a
certain sum of money for political ads printed by Bagiuo Printing and
Publishing during Labo’s 1984 campaign. Afable denied that her articles
were tainted with malice and at most, contained valid and timely doubts.
She also contended that the contents of her column were protected by
the constitutional guarantees of freedom of speech and of the press and
that the same were privileged as they dealt with a public figure.

The trial court dismissed the complaint for lack of merit, as the article in
question was privileged and constituted fair comment on matters of public
interest. The CA reversed the ruling of the trial court, and charged the
petitioners guilty of libel. SC reversed the CA’s ruling and held that Afable’s
article constitutes a fair comment on a matter of public interest as it dealt with
the character of Labo who was running for the top elective post in Baguio.
Afable’s column with respect to Labo’s debts provided the public with
information as regards his financial status, which, in all probability, was still
unknown to them at that time.

DOCTRINE: Occasional injury to the reputations of individuals must yield


to the public welfare, although at times such injury may be great.
However, this rule only applies to fair comment on matters of public
interest, fair comment being that which is true, or which if false, express
the real opinion of the author based upon reasonable degree of care and
reasonable grounds.

FACTS:
Petitioner Oseo C. Hamada (Hamada) was the president and general
manager of the Baguio Printing and Publisihing Co., Inc., which
publishes the Baguio Midland Courier, a weekly newspaper published in
the column “In and Out of Baguio.”
Private respondent Ramon Labo was among the mayoralty candidates in Baguio
for the 1988 local elections.
As part of the campaign propaganda for Labo, political ads appeared in the
various issues of Baguio Midland Courier and campaign paraphernalia were
printed by Baguio Printing and Publishing Co., Inc., on his behalf.
Before the local elections, Afable wrote in her column a series of articles dealing
wth the candidates for the various elective positions in Baguio. A portion of
the column referred to Labo as having the most imponderables about him,
with people asking, “Can he read and write?” and “Is he really a Japanese
Senator or a barrio kapitan?” Afable also referred to Labo as “Dumpty in the
egg.”
Labo claimed that portions of Afable’s column were tainted with malice, so he
filed a separate criminal and civil actions for libel against the petitioners. DOJ
dismissed the criminal case due to insufficiency of evidence, while the civil
suit was raffled to the RTC.
Labo alleged that Afable made it appear that he could not comply with his
financial obligations; that the subject articles were written solely for the
purpose of destroying his reputation, integrity, and personality as well as that
of Ms. Narukawa Labo; and that the articles were false, untrue, libelous and
published with evil intent.
Petitioners Hamada and Afable filed a motion to dismiss due to lack of cause of
action, and invoked PD 1508, or the Katarungang Pambarangay Law, which
required the referral of certain disputes to the barangay conciliation process
before they are filed in court. The RTC denied the motions to dismiss and
held that the PD was not applicable to them as one of the parties to the case
was a corporation.
Petitioners Baguio Midland and Hamada denied that petitioner Afable’s articles
were libelous. They also claimed that according to their company records,
Labo still owed them a certain sum of money for political ads printed by
Bagiuo Printing and Publishing during Labo’s 1984 campaign.
They also asserted that Afable’s write-ups were fair comments on facts and
reports that were of public interest as Labo was a mayoralty candidate at that
time.
Afable denied that her articles were tainted with malice and at most, contained
valid and timely doubts. She also contended that the contents of her
column were protected by the constitutional guarantees of freedom of
speech and of the press and that the same were privileged as they dealt
with a public figure. Afable also said that the phrase “dumpty in the egg”
referred to Horato Aquino, another candidate.
In the trial, Labo testified that he felt that the phrase “dumpty in the egg” referred
to him, interpreting it to mean someone who is a failure in his business
undertakings. Labo presented Dr. Pedro Rovillos who testified that he
understood “dumpty in the egg” to mean “a zero or a big lie.”
Petitioners presented Lambino, Baguio Printing’s bookkeeper as their witness.
According to Lambino, Baguio Printing sent several statements of
accounts and demand letters to Labo pertaining to his unpaid Sedano, the Court recognized the public’s right to be
obligations amounting to P27,415. informed on the mental, moral, and physical fitness of
The trial court dismissed the complaint for lack of merit, as the article in candidates for public office.
question was privileged and constituted fair comment on matters of
public interest. The CA reversed the ruling of the trial court, and charged
the petitioners guilty of libel. CA held that when the article was
published, Labo was not a public official but a private citizen seeking an
elective office.

ISSUE/s: WoN Afable’s article was libelous – NO.

RULING: The petition is GRANTED. The Decision of the CA denying


reconsideration are REVERSED and SET ASIDE.

RATIO:
Contrary to the findings of the CA that Labo was the only candidate
named in the article, said article actually dealt with other named
candidates and it would be grave error to impute malice on the article
based upon a finding that Labo was unduly singled out by Afable in
her column.
It is clear that the CA assumed that since the name of Labo and the
phrase “dumpty in the egg” appeared in the same paragraph, the
epithet referred onlt to Labo. A perusal of the paragraph easily
reveals that the person alluded to by Afable was someone who was
campaigning for a certain Atty. Cortes – one of the mayoralty
candidates in Baguio City.
It is a basic precept that in cases involving claims for damages
arising out of alleged defamatory articles, it is essential that the
alleged victim be identifiable although it is not necessary that
he be named.
It is enough that the publication contains matters of descriptions or
reference to facts and circumstances from which others reading the
article may know the plaintiff was intended.
In the Borjal v. CA case, the Court declared that it is not sufficient that
the offended party recognized himself as the person attacked or
defamed, but it must be shown that at least a third person could
identify him as the object of the libelous publication.
Labo has the duty to present evidence that a third person could easily
identify him as the person libeled. Labos has utterly failed to dispose
of this responsibility.
Labo’s lone witness was able to offer his own understanding of what the
phrase “dumpty in the egg” meant. However, during cross-
examination, he failed to sufficiently explain before the court how he
arrived at such conclusion that the term referred to Labo.
Labo was not yet a public official at the time the article was
published. However, this fact does not remove said article from
the mantle of protection guaranteed by the freedom of
expression provision of the Constitution. In the case of US v.
In the leading case of New York Times Co. v. Sullivan, the US Supreme Court
held that it is of utmost consequence that the people should discuss the
character and qualifications of candidates for their suffrages. The
importance to the state and to society of such discussions is so vast, and the
advantages derived are so great, that they more than counterbalance the
inconvenience of private persons whose conduct may be involved.
Occasional injury to the reputations of individuals must yield to the
public welfare, although at times such injury may be great.
The rule enunciated above only applies to fair comment on matters of public
interest, fair comment being that which is true, or which if false, express
the real opinion of the author based upon reasonable degree of care
and reasonabl grounds.
This principle does not grant an absolute license to authors or writers to destroy
the persons of candidates by providing the general public with publications
tainted with express or actual malice. While the law creates the presumption
that every defamatory imputation is malicious, the privileged character of a
communication destroys said presumption. The burden of proving actual
malice shall then rest on the plaintiff. In this case, on Labo.
In the present case, Labo was unable to prove that Afable’s column was tainted
with actual malice. The evidence showed that indeed, Labo incurred an
obligation which had remained unpaid until the time the questioned article
was published.
As held by the Court in the Borjal case: Even assuming that the contents of the
articles are false, mere error or even falsity alone does not prove actual
malice. Errors or misstatements are inevitable in any scheme of truly free
expression and debate. The press should not be held to account, to a
point of suppression, for honest mistakes or imperfections in the
choice of language. There must be some room for misstatement of fact
as well as for misjudgment.
Afable’s article constitutes a fair comment on a matter of public interest as it dealt
with the character of Labo who was running for the top elective post in
Baguio. Afable’s column with respect to Labo’s debts provided the public with
information as regards his financial status, which, in all probability, was still
unknown to them at that time.
Indeed, the information might have dissuaded some members of the electorate
from voting for Labo but such is the inevitable result of the application of the
law. The effect would have been advrse to Labo but public interest in this
case far outweighs the interest of Labo.
The concept of privileged communicaitons is implicit in the freedom of the
press.
MILLER v. CALIFORNIA (IYA)
June 21, 1973 | Burger, CJ. | Obscenity-Pornography RULING: The judgment of the Appellate Department of the Superior Court,
Orange County, California, is vacated and the case remanded to that court
for further proceedings not inconsistent with the First Amendment standards
PETITIONER: Miller
established by this opinion.
RESPONDENTS: California

SUMMARY: The jury and Appeals Department of California convicted RATIO:


Miller of having violated the California Penal Code on distributing In Roth v. United States (1957), the Court sustained a conviction under a
federal statute punishing the mailing of "obscene, lewd, lascivious or
obscene matter.
filthy . . .”
The Supreme Court is tasked to reexamine the standards for obscenity- In Chaplinsy v New Hampshire: “implicit in the history of the First
pornography cases. The ruling in Memoirs which provides for a test is Amendment is the rejection of obscenity as utterly without redeeming
social importance. . . .”
vacated as it is unworkable in application. The rule in Roth is reinstated,
In Roth: ”. . . There are certain well defined and narrowly limited
which declares that lewd and obscene expressions may be regulated by
classes of speech, the prevention and punishment of which
the State. Basic guidelines have also been enunciated by the Court.
have never been thought to raise any Constitutional problem.
DOCTRINE: To equate the free and robust exchange of ideas and These include the lewd and obscene. . . . It has been well
political debate with commercial exploitation of obscene material observed that such utterances are no essential part of any
demeans the grand conception of the First Amendment and its high exposition of ideas, and are of such slight social value as a step
purposes in the historic struggle for freedom to truth that any benefit that may be derived from them is clearly
outweighed by the social interest in order and morality. . . .”
In Memoirs v. Massachusetts(1966), the Court veered sharply away from
FACTS: the Roth concept and, with only three Justices in the plurality opinion,
The present case is a reexamination of standards for obsecnity- articulated a new test of obscenity. It must be established that
pornography cases. the dominant theme of the material, taken as a whole, appeals to
The facts attendant to the reexamination is as follows: a prurient interest in sex;
Miller conducted a mass mailing campaign to advertise the sale of the material is patently offensive because it affronts
illustrated books of “adult” in nature. contemporary community standards relating to the
His conviction was specifically based on his conduct in causing 5 description or representation of sexual matters; and
unsolicited advertising brochures sent through the mail in an the material is utterly without redeeming social value.
envelope addressed to a restaurant in Newport Beach, California. But now the Memoirs test has been abandoned as unworkable by its
The envelop was opened and its content advertise four books and a film author, and no Member of the Court today supports the Memoirs
of obscene character. formulation.
The brochure also contained photographs of men and women egaging in The basic guidelines for the trier of fact must be:
a variety of sexual activities with genitals prominently displayed. whether "the average person, applying contemporary
The jury trial convicted him of violating California Penal Code § 311.2(a), community standards" would find that the work, taken
a misdemeanor, by knowingly distributing obscene matter. The as a whole, appeals to the prurient interest,
Appellate Department affirmed the judgment. whether the work depicts or describes, in a patently
Hence, the present case before the Supreme Court which involves the offensive way, sexual conduct specifically defined by the
application of a State’s criminal obscenity statute to a situation which applicable state law; and
sexually explicit materials have been thrust by aggressive sales whether the work, taken as a whole, lacks serious literary,
action upon unwilling recipients who had in no way indicated any artistic, political, or scientific value.
desire to receive such materials. To equate the free and robust exchange of ideas and political
debate with commercial exploitation of obscene material
ISSUE/s: demeans the grand conception of the First Amendment and its
WON Miller is liable under the California Statue on obscenity - NOT high purposes in the historic struggle for freedom.
SURE YET. STANDARDS WERE SET FOR LOWER COURT. It is a "misuse of the great guarantees of free speech and free press. . . .”
In sum, The Supreme Court:
reaffirm the Roth holding that obscene material is not protected
by the First Amendment;
hold that such material can be regulated by the States, subject to
the specific safeguards enunciated above.
GONZALES v. KATIGBAK (ELIEL) Withheld the issuance of the Permit to exhibit until these deficiencies
July 22, 1985 | Fernando, CJ. | Freedom of Expression are supplied. Hence this petition.
The main objection was the classification of the film as “For Adults Only.”
PETITIONER: Jose Antonio Gonzalez
RESPONDENTS: Chairman Maria Kalaw Katigbak, The Board of Review
for Motion Pictures and Television

SUMMARY: A permit to exhibit Kapit sa Patalim was granted by the


Board, however it was classified as “For Adults Only.” Gonzales filed for a
motion for reconsideration stating that therew was no legal and factual
basis to provide the classification, and that it was an exercise of his
artistic right of expression. All of the parts is vitally essential to the movie,
including those that are being censored or deleted by the Board. Hence
this petition.

The SC ruled that there is a basis for such classification. (1) Free speech
and expression is not absolute; (2) the State must act as parens patriae
and regulate such medium; (3) a film may be regulated on the standard
of the clear and present danger rule. In this case, the State aims to
protect the people from obscenity and those without redeeming social
interest; (4) EO 876 was enacte as a standard in guiding the Board to
regulate and classify films by applying contemporary Filipino cultural
values.

DOCTRINE: It is the opinion of this Court, therefore, that to avoid an


unconstitutional taint on its creation, the power of respondent Board is
limited to the classification of films. It can, to safeguard other
constitutional objections, determine what motion pictures are for general
patronage and what may required either parental guidance or be limited
to adults only. That is to abide by the principle that freedom of expression
is the rule and restrictions the exemption. The power to exercise prior
restraint is not to be presumed, rather the presumption is against its
validity.

FACTS:
In a resolution of a sub-committee of respondent Board of Review for
Motion Pictures and Television (BRMPT), a permit to exhibit the film
Kapit sa Patalim under the classification “For Adults Only,” with
certan changes and deletions enumerated was granted.
A motion for reconsideration was filed by Gonzales stating that the
classification of the film “For Adults Only” was without basis.
BRMPT released its decision: “Acting on the application’s Motion for
Reconsideration dated 29 October 1984, the Board, after a review of
the resolution of the sub-committee and an examination of the film,
Resolves to affirm in toto the ruling of the sub-committee.
Considering, however, certain vital deficiencies in the application, the
Board further resolves to direct the Chairman of the Board to
For Gonzales, such classification “is without legal and factual basis connection must be evident. Also, there must be reasonable
and is exercised as impermissible restraint of artistic expression. The apprehension about its imminence. The time
film is an integral whole and all its portions, including those to which
the Board now offers belated objection, are essential for the integrity
of the film. Viewed as a whole, there is no basis even for the vague
speculations advanced by the Board as basis for its classification.

ISSUE/s:
WoN the freedom of expression of Gonzales was violated – NO

RULING: Petition dismissed because lack of votes that there was a grave
abuse of discretion.

RATIO:
Even if such were the case, there is justification for an inquiry into the
controlling standard to warrant the classification of “For Adults Only.”
This is especially so, when obscenity is the basis for any alleged
invasion of the right to the freedom of artstic and literary expression
embraced in the free speech and free press guarantees in the
Constitution.
Motion pictures are important both as a medium for the communication
of ideas and the expression of the artistic impulse. Their effects on
the perception by our people of issues and public officials or public
figures as well as the prevailing cultural traits is considerable.
There is no clear dividing line between what involves knowledge and
what affords pleasure. If such a distinction were sustained, there is a
diminution of the basic right to free expression.
Press freedom, as stated in the opinion of the Court (Reyes v.
Bagatsing), “may be identified with the liberty to discuss
publicly and truthfully and matter of public concern without
censorship or punishment.” This is not to say that such
reedom, as is the freedom of speech, absolute. It can be limite if
“there be a ‘clear and present danger of a substantive evil that
[the State] has a right to prevent.’”
It is the opinion of this Court, therefore, that to avoid an
unconstitutional taint on its creation, the power of respondent
Board is limited to the classification of films. It can, to safeguard
other constitutional objections, determine what motion pictures are
for general patronage and what may required either parental
guidance or be limited to adults only. That is to abide by the
principle that freedom of expression is the rule and restrictions the
exemption. The power to exercise prior restraint is not to be
presumed, rather the presumption is against its validity.
The test, to repeat, to determine whether freedom of expression may be
limited is the clear and present danger of an evil of a substantive
character that the State has a right to prevent. Such danger must not
only be clear but also present. There should be no doubt that what is
feared may be traced to the expression complained of. The causal
element cannot be ignored. Nor does it suffice if such danger be only
probable. There is the requirement of its being well-nigh inevitable.
“All ideas having even the slightest redeeming social importance –
unorthodox ideas, controversial ideas, even ideas hateful to the
prevailing climate of opinion – have the full protection of the
guaranties, unless excludable because they encroach upon the
limited area of more important interests. But implicit in the
history of the 1st Amendment is the rejection of obscenity as
utterly without redeeming social importance.”
“Obscene material is material which deals with sex in a manner
appealing to prurient interest. The portrayal of sex, e.g., in art,
literature and scientific works, is not itself sufficient reason to deny
material the constitutional protection of freedom of speech and
press. Sex, a great and mysterious motive force in human life, has
indisputably been a subject of absorbing interest to mankind through
the ages; it is one of the vital problems of human interest and public
concern.”
In the applicable law, EO 876, reference was made to respondent
Board “applying contemporary Filipino cultural values as a
standard,” words which can be construed in an analogous
manner. Moreoever, as far as the question of sex and obscenity
are concerned, it cannot be stressed strongly that the arts and
letters “shall be under the patronage of the State.”
What is seen or percieived by an artist is entitled to respect, unless there
is a showing that the product of his talent rightfully may be
considered obscene
On the question of obscentity, therefore, and in the light of the facts of
this case, such standard set forth in EO 876 is to be construed in
such a fashion to avoid any taint of unconstitutionality.
It is the consensus of this Court that where television is concerned, a
less liberal approach calls for observance. This is so because unlike
motion pictures where the patrons have to pay their way, television
reaches every home where there is a set. Children then will likely be
among the avide viewers of the programs therein shown.
SORIANO v. LAGUARDIA (JP) Respondents forward that Michael Sandoval (also one of the
April 29, 2009 | Velasco, Jr. J. | Freedom of Speech respondents) was the person directly alluded to in the petitioner’s
remark. He was then a
PETITIONER: Eliseo F. Soriano
RESPONDENTS: Ma. Consoliza P. Laguardia, Chairman of the Movie
and Television Review and Classification Board

SUMMARY: Petitioner Soriano is the host of Ang Dating Daan. On one


screening, he blatantly degraded respondent Michael Sandoval who has
a host at Ang Tamang Daan by comparing him to a putang babae.
Respondent INC members filed for a complaint under the MTRCB. The
latter then ruled in favor of respondents. Petitioner Soriano uses his right
to religious freedom and freedom of speech as defense.

The SC ruled that (1) on religious freedom: foul-mouthing cannot in


anyway be construed as religious speech even if aired on a bible
exposition television program (2) on freedom of speech: such statements
are considered obscene to a possible viewing child, in line with the G
rating of the program; hence, part of unprotected speech. The court used
the balancing of interests test and ruled that the compelling state interest
of being parents patrie to the children far outweighs the right of the
petitioner to continue airing his program.

DOCTRINE: It has been established in this jurisdiction that unprotected


speech or low-value expression refers to libelous statements, obscenity
or pornography, false or misleading advertisement, insulting or “fighting
words.”

A patently offensive utterance would come within the pale of the term
obscenity should it appeal to the prurient interest of an average listener
applying contemporary standards.

FACTS:
Petitioner Soriano as host of the TV program Ang Dating Daan, aired on
UNTV 37, made the following remarks: Lehitimong anak ng
demonyo; sinungaling; Gago ka talaga Michael, masahol ka pa sa
putang babae o di ba. Yung putang babae ang gumagana lang doon
yung ibaba, [dito] kay Michael ang gumagana ang itaas, o di ba! O,
masahol pa sa putang babae yan. Sabi ng lola ko masahol pa sa
putang babae yan. Sobra ang kasinungalingan ng mga demonyong
ito.
Two days after, respondents, who are all members of the Iglesia ni Cristo
filed complaints before the MTRCB against petitioner for the above
broadcast.
minister of the INC and a regular host of the TV program ang Tamang The Court finds that petitioner’s statement can be treated as
Daan. obscene, at least with respect to the average child. Hence, it is,
The MTRCB preventively suspended the showing of Ang Dating Daan in that context, unprotected speech.
program for 20 days initially in pursuance to PD 1986 (creating the There is no perfect definition of obscenity but the latest word is that of Miller
MTRCB), Sec. 3(d) and the IRR of such PD. Then MTRCB finally
issued a decision wherein the TV program was to be suspended for
three months.
Petitioner Soriano filed this petition for certiorari.

ISSUE/s:
WoN section 3(c) of PD 1986 and the IRR, as applied to petitioner
Soriano, unduly infringes on the constitutional guarantee of freedom
of religion, speech, and expression as it partakes of the nature of a
subsequent punishment curtailing the same– NO

RULING: The decision of the MTRCB is affirmed.

RATIO:
On religious freedom: Petitioner Soriano is submitting that what he
uttered was religious speech, adding that words like putang babae
were said in exercise of his religious freedom.
The argument has no merit.
The Court is at a loss to understand how petitioner’s utterances in
question can come within the pale of Sec. 5, Article III of the 1987
Constitution on religious freedom.
The fact that he came out with his statements in a televised bible
exposition program does not automatically accord them the
character of a religious discourse. Plain and simple insults directed
at another person cannot be elevated to the status of religious
speech.
Petitioner Soriano had descended to the level of name-calling and foul-
language discourse. Petitioner could have chosen to contradict and
disprove his detractors, but opted for the low road.
On freedom of speech and expression: Petitioner asserts that his
utterance in question is a protected form of speech.
The Court rules otherwise.
It has been established in this jurisdiction that unprotected speech or
low-value expression refers to libelous statements, obscenity or
pornography, false or misleading advertisement, insulting or “fighting
words,” i.e., those which by their very utterance inflict injury or tend
to incite an immediate breach of peace and expression endangering
national security.
v. California which established basic guidelines, to wit: meaning that the “material for television x x x in the
whether to the average person, applying contemporary judgment of the BOARD, does not contain anything
standards would find the work, taken as a whole, appeals to unsuitable for children and
the prurient interest;
whether the work depicts or describes, in a patently offensive
way, sexual conduct specifically defined by the applicable
state law;
whether the work, taken as a whole, lacks serious literary,
artistic, political, or scientific value.
A cursory examination of the utterances complained of and the
circumstances of the case reveal that to an average adult, the
utterances may not constitute obscene but merely indecent
utterances. They can be viewed as figures of speech or merely a
play on words.
In the context they were used, they may not appeal to the prurient
interests of an adult.
The problem with the challenged statements is that they were
uttered in a TV program that is rated G or for general
viewership, and in a time slot that would likely reach even the
eyes and ears of children.
Young minds, without the guidance of an adult, may from their end, view
this kind of indecent speech as obscene, if they take these words
literally and use them in their own speech of form their own ideas on
the matter.
Undeniably the subject speech is very suggestive of a female
sexual organ and its function as such. In this sense, we find
petitioner’s utterances obscene and not entitled to protection
under the umbrella of freedom of speech.
The suspension MTRCB imposed under the premises was, in one
perspective, permissible restriction. We make this disposition against
the backdrop of the following interplaying factors:
First, the indecent speech was made via television, a
pervasive medium that, to borrow from where there is a
set [and where] [c]hildren will likely be among the avid
viewers of the programs therein shown;
Second, the broadcast was aired at the time of the day when
there was a reasonable risk that children might be in the
audience; and
Third, petitioner uttered his speech on a “G” or for “general
patronage” rated program.
Under Sec. 2(A) of Chapter IV of the IRR of the MTRCB, a
show for general patronage is “[s]uitable for all ages,”
minors, and may be viewed without adult guidance or
supervision.” The words petitioner used were, by any
civilized norm, clearly not suitable for children.
Where a language is categorized as indecent, as in petitioner’s
utterances on a general-patronage rated TV program, it may
be readily proscribed as unprotected speech.
A view has been advanced that unprotected speech refers only to
pornography, false or misleading advertisement, advocacy of imminent
lawless action, and expression endangering national security. But this list
is not, as some members of the Court would submit, exclusive or carved
in stone.
This particular case constitutes yet another exception, another instance
of unprotected speech, created by the necessity of protecting the
welfare of our children. As unprotected speech, petitioner’s
utterances can be subjected to restraint or regulation.
Petitioner’s invocation of the clear and present danger doctrine, arguably the
most permissive of speech tests, would not avail him any relief, for the
application of said test is uncalled for under the premises.
Generally, said clear and present doctrine is applied to cases involving the
overthrow of the government and even other evils which do not clearly
undermine national security.
Where the legislation under constitutional attack interferes with the freedom
of speech and assembly in a more generalized way and where the effect
of the speech and assembly in terms of the probability of realization of a
specific danger is not susceptible even of impressionistic calculation,
then the balancing of interests test can be applied.
After a careful examination of the factual milieu and the arguments raised by
petitioner in support of his claim to free speech, the Court rules that the
government’s interest to protect and promote the interests and
welfare of the children adequately buttresses the reasonable
curtailment and valid restraint on petitioner’s prayer to continue as
program host of Ang Dating Daan during the suspension period.
The welfare of children and the State’s mandate to protect and care for them,
as parens patriae, constitute a substantial and compelling government
interest in regulating petitioner’s utterances in TV broadcast as provided
in PD 1986.
FCC case explains the duty of the government to act as parens patriae to
protect the children who, because of age or interest capacity, are
susceptible of being corrupted or prejudiced by offensive language.
On prior restraint: Petitioner theorizes that the three (3)-month suspension is
either prior restraint or subsequent punishment that, however, includes prior
restraint, albeit indirectly.
After a review of the facts, the Court finds that what MTRCB imposed on
petitioner is an administrative sanction or subsequent punishment for
his offensive and obscene language in Ang Dating Daan.
To clarify, statutes imposing prior restraints on speech are generally
illegal and presumed unconstitutional breaches of the freedom of
speech. The exceptions to prior restraint are movies, television, and
radio broadcast censorship in view of its access to numerous people,
including the young who must be insulated from the prejudicial
effects of unprotected speech.
PITA v. CA (HENRY) sellers of obscene publications (they cited Sec 1, Art 201 of the
October 5, 1989 | Sarmiento, J. | Freedom of Speech of the Press: RPC).
Obscenity Aggrieved, Pita filed the present petition in the SC.

PETITIONER: Leo Pita, under the name and style of Pinoy Playboy
RESPONDENTS: CA, Ramon Bagatsing, and Narciso Cabrera

SUMMARY: Manila Mayor initiated an Anti-Smut campaign to which a


wide seizure and destruction (by way of fire) was made on magazines,
and other reading materials that were found to be pornographic, obscene,
and indecent. One of the affected magazines was that of the petitioner,
“Pinoy Playboy”.

Petitioner then assailed the constitutionality of the act of the Mayor, and
claimed that his right to freedom of speech and of the press was violated.
RTC ruled against him, to which the CA affirmed.

Upon appeal to the SC, the tables turned. SC ruled that such seizure was
invalid and that the right to freedom of speech and of the press of
petitioner was violated, because there was lack of proof that justified the
legality of the act of the respondents.

DOCTRINE: SC stated that, “immoral” lore or literature comes within the


ambit of free expression, although not its protection. In free expression
cases, the SC consistently has been in the side of the exercise of such
right, barring a “clear and present danger” that would warrant State
interference and action.

FACTS:
December 1983, pursuant to an Anti-Smut Campaign initiated by the Manila
Mayor, elements of the Anti-Narctics Group seized and onfiscated from
dealers, distributors, news-stand owners and peddlers along Manila
sidewalks magazines, publications, and other reading materials believed
to be obscene, pornographic, and indecent and later burned and the
seized materials in public atht eh University belt along CM Recto, in the
presence of Mayor Bagatsing, and several officers and members of
various student organizatioins.
Among those seized and burned was “Pinoy Playboy”, published and co-
edited by petitioner Pita.
Pita field a case for injuction against respondent Mayor, with a claim that
the magazine is a decent, artistic, and educational magazine which
is not per se obscene, and that the publication is protected by the
constitutional guarantees of freedom of speech and of the press.
RTC dismissed the case for lack of merit, to which the CA affirmed. The
appellate court likewise stated in its ruling that freedom of the press
is not without restraint, as the stte has the right to protect society
from pornographic literature that is offensive to public morals, as
indeed the country have laws punishing the author, publishers, and
The lack of uniformity in American jurisprudence as to what constitutes
ISSUE/s: obscenity has been attributed to the reluctance of the courts to
WoN the Pinoy Playboy is an obscene magazine and thus covered by recognize the
the law punishing such publications under the RPC – NO
WoN Pita’s constitutional right to freedom of the press and of speech is
violated – YES

RULING: SC granted the petition.

RATIO:
SC in a previous case (People v. Kottinger) provided for the test to
determine existence of obscenity:
Whether the tendency of the matter charged as obscene, is to
deprave or corrupt those whose minds are open to such
immoral influences and to whose hands a publication or
other article charged as being obscene may fall
That which shocks the ordinary and common sense of men as
indecency
Another case put in another test, that “whether a picture is obscene or
indecent must depend upon the circumstances of the case” and that
ultimately the question to be decided by the “judgment of the
aggregate sense of the community reached by it”
Due to the complications given by the doctrine in Kottinger, the Court
promulgated its decision on People v. Go Pin, a prosecution under
Article 201 of the RPC, to whicht eh court ruled that, if the pictures
in question were used not exactly for art’s sake but rather for
commercial purposes, the pictures are not entitled to any
constitutional protection.
In a much later decision, in an attempt to square the haziness of
obscenety, the Court followed the trends of the US in Gonzales v.
Kalaw Katigbak, to which they adopted the following test: whether to
the average person, applying contemporary standards, the dominant
theme of the material taken as a whole appeal to prurient interest.
(prurient means nakakalibog)
This case undertook moreover to take determination of obscenity
essentially a judicial question and as a consequence, to temper the
wide discretion Kottinger had given to law enforcers
The latest US case also established basic guidelines on obscenetiy
(Miller v. California):
Whether the average person, applying contemporary standards
would find the work, taken as a whole, appeals to the
prurient interest
Whether the work depicts or describes in a patently offensive
way, sexual conduct and specifically defined by the
applicable state law
Whether the work taken as a whole, lacks serious literary,
artistic, political, or scientific value
constitutional dimension of the problem. US courts have assumed obscenity
not included in the guaranty o free speech, an assumption that, as the PH SC
averred, has allowed a climate of opinions among magistrates predicated
upon arbitrary, if vague theories of what is acceptance to society.
In the present case, there is no challenge on the right of the State, in the
legitimate exercise of police power, to suppress smut—provided that
it is smut. For obvous reasons, smut is not smut simply because one
insists it is smut. So it is equally evident that individual tastes
develop, adapt to wide-ranging influences, and keep in stpe with the
rapid advance of civilization.
SC stated that, “immoral” lore or literature comes within the ambit of free
expression, although not its protection. In free expression cases, the
SC consistently has been in the side of the exercise of such right,
barring a “clear and present danger” that would warrant State
interference and action.
The SC referred to the Bagatsing case, that involved the delivery of a
political speech, and said that the presumption is that the speech may
validly be said. The burden is on the state to demonstrate the clear and
present danger.
Government must allow the speech. It has no choice. However, if it acts
notwithstanding that (absence of evidence of a clear and present
danger), it must come to terms with, and be held accountable for,
due process.
The SC is not convinced that respondents showed enough proof to justify
the ban and to warrant confiscation of the magazines. They were not
possessed with a lawful court order:
Finding the said materials to be pornography, and
Authorizing them to carry out a search and seizure, by way of a
search warrant
CA said that it has no quarrel that freedom of the press is not without
restraint as the state has the right to protect society from
pornographic literature that is offensive to public morals. SC agrees
to this.
So the question that is left is, were the literature confiscated
pornographic? That we have laws punishing the author, publisher,
and sellers of such obscene publications (Art 201, RPC) but then
again, has the petitioner been found guilty? No.
BARNES v. GLEN THEATER (DANNAH) believes she would make more money doing so
June 21, 1991 | Rehnquist, C.J. | Expressive Conduct Glen Theater:
Supplies so-called adult entertainment through (written/printed)
PETITIONER: Barnes Movie showings, live entertainment
RESPONDENTS: Kitty Kat Lounge, Glen Theater et al. Enclosed Bookstore = nude and semi-nude performances and showings
of female body through glass panels where customers insert coins
SUMMARY: The State on Indiana passed a statute called as the public and they watch the dancers for a period of time
indecency law. It states that those who dance nude in public must wear Gayle Sutro, one of their dancers has been in the profession for more
psties and G-strings. Respondent establishments and dancers contend that than 15 years
said prohibition violates the First Amendment as it protects expression such Respondent establishments sued to enjoin the enforecement of the
as non-obscene nude dancing. Indiana public indecency statute, as its prohibition against complete
nudity in public places violated the First Amendment.
The District Court granted the complaint of respondent establishments
This was initially granted by the District Court. The CA reversed and
enjoining Indiana to to enforce the public indecency law. The CA initially
reversed, but affirmed the ruling stating that non-obscene nude dancing remanded for hearing, but eventually affirmed the lower court’s
performed for entertainment is protected expression, and that the statute decision ruling that non-obscene nude dancing performed for
was an improper infringement of that activity because its purpose was to entertainment is protected expression, and that the statute was an
prevent the message of eroticism and sexuality conveyed by the dancers. improper infringement of that activity because its purpose was to
prevent the message of eroticism and sexuality conveyed by the
The SC, however, reversed. Citing O’Brien, it ruled that not all forms of non-
dancers.
speech expression is protected by the First Amendment. Moreover, the
statute passed the O’Brien test. There is substantial government interest—
that is, to protect morals and public order. Moreover, it ruled that the statute ISSUE/s:
did not prohibit non-obscene nude dancing per se, but just added a WoN the Indiana statute is an impermissible infringement of protected
requirement that the dancers must wear pasties and G-strings. activity – NO

DOCTRINE: O’Brien test: A government regulation is sufficiently justified if RULING: The judgment of the Court of Appeals accordingly is reversed.
It is within the constitutional power of the Government Indiana’s public indecency law is not violative of the First Amendment
It furthers an important or substantial governmental interest
The governmental interest is unrelated to the suppression of free expression RATIO:
The incidental restriction on alleged (First Amendment) freedoms is no greater Indiana has not baned nude dancing but has proscribed public nudity
than is essential to the furtherance of that interest across the board.
Barnes contends that the restriction is a valid “time, place or manner”
FACTS: restriction.
Respondents are two establishments from Indiana that wish to provide The “time, place or manner” test was developed for evaluating
totally nude dancing as entertainment, and individual dancers who restrictions on expression taking place on public property which had
are employed at these establishments. been dedicated as a "public forum," and this test has been
They aver that the State of Indiana’s public indecency law to prevent this interpreted to embody much the same standards as those set forth in
form of dancing is violative of the First Amendment. United States v. O'Brien. (Will not explain the O’Brien case na but
Kitty Kat Lounge: this is the one where he burned his draft card)
Sells alcoholic beverages
There this Court has held that, when "speech" and "nonspeech"
Presents go-go dancing
elements are combined in the same course of conduct, a sufficiently
Desires to present totally nude dancing but the Indiana statute requires
the dancers to wear “pasties” and G0strings important governmental interest in regulating the nonspeech element
Dancers work on commission: 100% com for the first $60 drink sales can justify incidental limitations on First Amendment freedoms.
Darlene Miller, one of the respondents, wishes to dance nude because O’Brien test: A government regulation is sufficiently justified if
she a. It is within the constitutional power of the Government
It furthers an important or substantial governmental interest The fourth part of the test requires that the restriction be no greater than
The governmental interest is unrelated to the suppression of free expression is essential to the furtherance of the goevernmental interest. As
The incidental restriction on alleged (First Amendment) freedoms is no
indicated above,
greater than is essential to the furtherance of that interest
Applying the test, the Court finds that Indiana’s public indecency statute
is justified despite its incidental limitations on some expressive
activity.
The public indecency statute is clearly within the constitutional power of the
State, and furthers substantial governmental interests. The statute's
purpose of protecting societal order and morality is clear from its text
and history.
Public indecency statutes such as the one before us reflect moral
disapproval of people appearing in the nude among strangers in public
places.
As early as 1831, Indiana had a statute punishing "open and notorious
lewdness, or . . . any grossly scandalous and public indecency."
This and other public indecency statutes were designed to protect morals
and public order. The traditional police power of the States is defined
as the authority to provide for the public health, safety, and morals,
and we have upheld such a basis for legislation.
The public indecency statute furthers a substantial government interest in
protecting order and morality.
The interest is unrelated to the suppression of free expression. Some
may view restricting nudity on moral grounds as necessarily related
to expression. We disagree. It can be argued, of course, that almost
limitless types of conduct - including appearing in the nude in public -
are "expressive," and in one sense of the word this is true.
In O’Brien, the court rejected this expansive notion of “expresive
conduct”, saying:
We cannot accept the view that an apparently limitless variety of
conduct can be labelled "speech" whenever the person engaging in
the conduct intends thereby to express an idea
Respondent establishments contend that even though prohibiting nudity
in public generally may not be related to suppressing expression,
prohibiting the performance of nude dancing is related to expression
because the state seeks to prevent its erotic message. Hence, it fails
the thir part of the test which states that the governmental interest
must be unrelated to the suppression of free expression.
However, the requirement that the dancers don pasties and a G-string
does not deprive the dance of whatever erotic message it conveys; it
simply makes the message slightly less graphic. The perceived evil
that Indiana seeks to address is not erotic dancing, but public nudity.
governmental interest served by the text of the prohibition is societal
disapproval of nudity in public places and among strangers.
The statutory prohibition is not a means to some greater end, but an end in
itself. It is without cavil that the public indecency statute is "narrowly
tailored;" Indiana's requirement that the dancers wear at least pasties
and a G-string is modest, and the bare minimum necessary to achieve
the state's purpose.
FEDERAL COMMUNICATIONS COMMISSION v. PACIFICA driving with his young son complained to the Federal Communications
FOUNDATION (ARIELLE) Commission
JULY 3,1978 | Stevens, J. | Freedom of Speech; Censorship

PETITIONER: Federal Communications Commission


RESPONDENTS: Pacifica Foundation

SUMMARY:
One of Pacifica Foundation’s radio stations made a broadcast of a satiric
dialogue entitled “Filthy Words.” A father who heard the broadcast while
driving with his young son complained to the FCC. Pacifica argued that
the monologue had been played during a program about society’s
attitude toward language and might be regarded as offensive to some.
The FCC found that certain words in the monologue depicted sexual
activities in a particularly offensive manner, noting that they were
broadcast in the early afternoon when children are undoubtedly in the
audience, and concluded that the language was indecent and prohibited
by the First Amendment. The CA reversed, concluding that the FCC’s
action was invalid either on the ground that the order constituted
censorship, or on the ground that the FCC’s opinion was “overbroad.”

The SC held that the words of the monologue are unquestionably “speech”
within the meaning of the First Amendment. It is equally clear that the FCC’s
objections to the broadcast were based in part on its content. The rule that all
governmental regulation that depends on the content of speech must be
invalidated is not an absolute rule. The words used in the monologue offend
for the same reasons that obscenity offends, although they ordinarily lack
literary, political, or scientific value, they are not entirely outside the
protection of the First Amendment. Some uses of even the most offensive
words are unquestionably protected.

DOCTRINE:
It is a characteristic of speech such as this that both its capacity to offend
and its “social value” vary with the circumstances. The constitutional
protection accorded to a communication containing such patently
offensive sexual and excretory language need not be the same in every
context. Of all forms of communication, it is broadcasting that has
received the most limited First Amendment protection. A broadcaster may
be deprived of his license and his forum of the FCC decides that such an
action would serve the public interest.

FACTS:
A satiric humorist named George Carlin recorded a 12-minute
monologue entitled “Filthy Words” before a live audience in a theater,
The radio station Pacifica Foundation made a broadcast of the satiric
monologue. A few weeks later, a father who heard the broadcast while
(FCC), which issued a declaratory order granting the complaint. The prohibition against censorship denies the FCC any power to edit
In Pacifica’s response, it stated that the monologue had been played proposed broadcasts in advance and to excise material considered
during a program about contemporary society’s attitude toward
language and might be regarded as offensive to some. Pacifica
characterized Carlin as “a significant social satirist” and that Carlin is
not mouthing obscenities, he is merely using words to satirize as
harmless and essentially silly our attitudes towards those words.
While not imposing sanctions, the FCC stated that the order would be
associated with the station’s license and in the event subsequent
complaints are received, FCC will decide whether it should utilize
any of the available sanctions it has been granted by Congress.
In its memorandum, the FCC stated that it intends to clarify the
standards which will be utilized in considering the growing number of
complaints about indecent radio broadcasts, and advanced several
reasons for treating that type of speech differently from other forms
of expression.
FCC characterized the language of the monologue as “patently
offensive,” though not necessarily obscene, and expressed that it
should be regulated by principles analogous to the law of nuisance
where the law generally speaks to channeling behavior rather than
actually prohibiting it.
The FCC found that certain words in the monologue depicted sexual
activities in a particularly offensive manner, noting that they were
broadcast in the early afternoon when children are undoubtedly in
the audience, and concluded that the language was indecent and
prohibited by 1464 (I don’t know what 1464 is guys but this is what it
says in the case).
The FCC issued another opinion in which it pointed out that it never
intended to place an absolute prohibition on the broadcast of this
type of language, but rather sought to channel it to times of day
when children most likely would not be exposed to it.
The CA reversed, concluding that the FCC’s action was invalid either on
the ground that the order constituted censorship, or on the ground
that the FCC’s opinion was “overbroad.”

ISSUE/s:
Whether or not the FCC’s order was a form of censorship forbidden by
the First Amendment - NO.

RULING: We reverse the ruling of the CA.

RATIO:
Section 29 of the Radio Act of 1927 provides that nothing in the Act
should be understood to give the licensing authority the power of
censorship over the radio communications or signals transmitted by
any radio stations, and no regulation shall be promulgated by the
licensing authority which shall interfere with the right of free speech
by means of radio communication.
inappropriate for airwaves. Amendment. Some uses of even the most offensive words are
The prohibition, however, has never been construed to deny the unquestionably protected.
Commission the power to review the content of completed The constitutional protection accorded to a communication containing
broadcasts in the performance of its regulatory duties. The Court such
held that the provision of the Radio Act deprived the FCC of the
power to subject “broadcasting matter to scrutiny prior to its release,”
but they concluded that the FCC’s undoubted right to take note of
past program content when considering a licensee’s renewal
application is “not censorship.”
The subsequent review of program content is not the sort of censorship
at wich the statute was directed, as its history makes it perfectly clear
that it was not intended to limit the FCC’s power to regulate the
broadcast of obscene, indecent, or profane language.
FCC emphasized that its order was issued in a specific factual context,
and that the approach is appropriate for courts as well as the FCC
when regulation of indecency is at stake.
The approach is also consistent with the ruling in Red Lion Broadcasting
v. FCC, where the Court rejected an argument that the FCC’s
regulations defining the fairness doctrine were so vague that they
would inevitable abridge the broadcasters’ freedom of speech.
It is true that the FCC’s order may lead some broadcasters to censor
themselves. At most, however, the FCC’s definition of indecency will
deter only the broadcasting of patently offensive references to
excretory and sexual organs and activities. While some of these
references may be protected, they surely lie at the periphery of First
Amendment concern.
The words of the monologue are unquestionably “speech” within the
meaning of the First Amendment. It is equally clear that the FCC’s
objections to the broadcast were based in part on its content. The
rule that all governmental regulation that depends on the
content of speech must be invalidated is not an absolute rule.
Schenck v. US: The character of every act depends on the
circumstances in which it is done. The most stringent protection of
free speech would not protect a man in falsely shouting fire in a
theatre and causing panic.
The government may forbid speech calculated to provoke a fight. It may
treat libels against private citizens more severely than libels against
public officials.
The question in this case is whether a broadcast of patently
offensive words dealing with sex and excretion may be
regulated because of its content. Obscene materials have been
denied the protection of the First Amendment because their content
is so offensive to contemporary moral standards. But the fact that
society may find speech offensive is not a sufficient reason for
suppressing it.
The words used in the monologue offend for the same reasons that
obscenity offends, although they ordinarily lack literary, political, or
scientific value, they are not entirely outside the protection of the First
patently offensive sexual and excretory language need not be the same
in every context.
It is a characteristic of speech such as this that both its capacity to offend
and its “social value” vary with the circumstances.
It is undisputed that the content of the monologue was “vulgar,” “offensive,”
and “shocking.” Of all forms of communication, it is broadcasting
that has received the most limited First Amendment protection. A
broadcaster may be deprived of his license and his forum of the FCC
decides that such an action would serve the public interest.
Similarly, although the First Amendment protects newspaper publishers from
being required to print the replies of those whom they criticize, it affords
no such protection to broadcasters – they must give free time to the
victims of their criticism.
First, the broadcast media have established a uniquely pervasive presence
in the lives of all Americans. Patently offensive, indecent material
presented over the airwaves confronts the citizen, not only in public, but
also in the privacy of the home.
Because the broadcast audience is constantly tuning in and out, prior
warnings cannot completely protect the listener or viewer from
unexpected program content. One may hang up on an indecent phone
call, but that option does not give the caller a constitutional immunity or
avoid a harm that has already taken place.
Second, broadcasting is uniquely accessible to children, even those too
young to read. Pacifica’s broadcast could have enlarged a child’s
vocabulary in an instant. Other forms of offensive expression may be
withheld from the young without restricting the expression at its source.
The Court held in Ginsberg v. New York that the government’s interest in the
“well-being of its youth” and in supporting “parents’ claim to authority in
their own household” justified the regulation of otherwise protected
expression.
The FCC’s decision rested entirely on a nuisance rationale under which
context is all-important. The concept requires consideration of a host of
variables. The content of the program in which the language is used will
also affect the composition of the audience, and differences between
radio, television, and perhaps closed-circuit transmissions, may also be
relevant.
RENTON v. PLAYTIME THEATRES (IYA) The court found that the Renton ordinance did not substantially
February 25, 1986 | Rehnquist, J. | Freedom of Expression restrict First Amendment interests.
That Renton was not required to show specific adverse impact
on
PETITIONER: Renton, Washington
RESPONDENTS: Playtime Theatres

SUMMARY: Playtime Theatres questions the city ordinance that


prohibits adult motion picture theaters within 1,000 feet from any
residential zone, church, park, or school. Such is in the form of a zoning
ordinance.

The District Court upheld the validity of the ordinance. The CA reversed
holding that it restricted the Constitutional guarantee of freedom of
expression under the First Amendment.

The SC ruled the same as the District Court. There was valid
governmental interest in promulgating said ordinance and such is a
content neutral regulation. Any restriction as to expression is merely
incidental.

DOCTRINE: The ordinance is a valid governmental response to the


serious problems created by adult theaters and satisfies the dictates of
the First Amendment. Since the ordinance does not ban adult theaters
altogether, it is properly analyzed as a form of time, place, and manner
regulation.

FACTS:
Playtime Theatres purchased two theaters in Renton, Washington, with
the intention of exhibiting adult films.
At about the same time, filed suit in Federal District Court, seeking
injunctive relief and a declaratory judgment that the First and
Fourteenth Amendments were violated by a city ordinance that
prohibits adult motion picture theaters from locating within 1,000 feet
of any residential zone, single- or multiple-family dwelling, church,
park, or school.
The District Court on summary judgment, entered the preliminary
injunction, and Playtime began showing adult films in their two
theaters.
An amendment was added to the ordinance, adding a provision
explaining that its intention in adopting the ordinance had been "to
promote the City of Renton's great interest in protecting and
preserving the quality of its neighborhoods, commercial districts, and
the quality of urban life through effective land use planning."
Upon final decision by the District Court, they vacated the preliminary
injunction, denied Playtime Theaters' requested permanent
injunction, and entered summary judgment in favor of Renton.
Renton from the operation of adult theaters but could rely on on the content of the films shown there. The constitutionality of the
the experiences of other cities ordinance is therefore not correctly analyzed under standards applied to
That the purposes of the ordinance were unrelated to the content-neutral
suppression of speech.
That the restrictions on speech imposed by the ordinance were
no greater than necessary to further the governmental
interests involved.
The Court of Appeals reversed, holding that the ordinance constituted a
substantial restriction on First Amendment interests, and remanded
the case
for reconsideration as to whether the city had substantial
governmental interests to support the ordinance.
ISSUE/s:
Whether the Renton ordinance is designed to serve a substantial
governmental interest and allows for reasonable alternative avenues
of communication - YES

RULING: SC reversed CA decision.

RATIO:
The ordinance is a valid governmental response to the serious
problems created by adult theaters and satisfies the dictates of
the First Amendment
Since the ordinance does not ban adult theaters altogether, it is
properly analyzed as a form of time, place, and manner
regulation.
"Content-neutral" time, place, and manner regulations are
acceptable so long as they are designed to serve a
substantial governmental interest and do not unreasonably
limit alternative avenues of communication.
The District Court found that the Renton City Council's "predominate"
concerns were with the secondary effects of adult theaters on the
surrounding community, not with the content of adult films
themselves.
The city's pursuit of its zoning interests was unrelated to the suppression
of free expression, and thus the ordinance is a "content-neutral"
speech regulation.
The Renton ordinance is designed to serve a substantial governmental
interest while allowing for reasonable alternative avenues of
communication.
A city's interest in attempting to preserve the quality of urban life,
as here, must be accorded high respect.
There are no constitutional defect in the method chosen by
Renton to further its substantial interests. Cities may
regulate adult theaters by dispersing them, or by effectively
concentrating them, as in Renton.
Brennan and Marshall, dissent: Renton's zoning ordinance selectively
imposes limitations on the location of a movie theater based exclusively
time, place, and manner restrictions.
The amendment was added after the filing of the lawsuit. Prior to
the amendment, there was no indication that the ordinance
was designed to address any “secondary effects" a single
adult theater might create. In addition to the suspiciously
coincidental timing of the amendment, many of the City
Council's "findings" do not relate to legitimate land-use
concerns.
The circumstances here strongly suggest that the ordinance was
designed to suppress expression,
BETHEL v. FRASER (ELIEL) The morning after the assembly, the Assistant Principal called Fraser into her
July 7, 1986 | Burger, J. | Freedom of Speech office and notified him that the school considered his speech to have
been a
PETITIONER: Bethel School District
RESPONDENTS: Matthew Fraser

SUMMARY: In front of 600 students in Bethel High School, Matthew


Fraser delivered a speech nominating a fellow student for student
elective office. However, the speech connotes sexual innuendos that
clearly was obscene, lewd, inappropriate. The Assistant Principal
suspended Fraser. When he appealed, the District Court ruled in Fraser’s
favor that the school violated his right to freedom of speech and
expression. Hence this petition.

The SC reversed the decision and ruled in favor of the School. The SC held
that the school has a duty to protect its students from such offensive
language and prohibit those that are deemed to be inappropriate. The first
amendment does not allow obscene and lewd speech and is not absolute,
that is why it may be regulated. The SC ruled that it has to balance the
society’s interest and not just the individual speech of the person to assert if
the speech is well within the guarantee.

DOCTRINE: The undoubted freedom to advocate unpopular and


controversial views in schools and classrooms must be balanced against the
society’s countervailing interest in teaching students the boundaries of
socially appropriate behavior. Even the most heated political discourse in a
democratic society requires consideration for the personal sensibilities of the
other particpants and audiences.

FACTS:
Matthew Fraser, a student at Bethel High School in Pierce County,
Washinton, delivered a speech nominating a fellow student for
student elective office. During the entire speech, Fraser referred to
his candidate in terms of an elaborate, graphic, and explicit sexual
metaphor.
Two of Fraser’s teachers, with whom he discussed the contents of his
speech in advance, informed him that the speech was “inappropriate
and that he probably should not deliver it,” and that his delivery of the
speech might have “severe consequences.”
During Fraser’s delivery of the speech, a school counselor observed the
reaction of students to the speech. Some students hooted and
yelled; some by gestures graphically simulated the sexual activities
pointedly alluded to in Fraser’s speech. Other students appeared to
be bewildered and embarrassed by the speech.
A Bethel High School disciplinary rule prohibiting the use of obscene
language in the school provides: “Conduct which materially and
substantially interferes with the educational process is prohibited,
including the use of obscene, profane language or gestures.”
violation of this rule. Fraser was tehn informed that he would be But these “fundamental values” must also take into account consideration
suspended for three days, and that his name would be removed from of
the list of candidates for graduation speaker at the school’s
commencement exercises.
The hearing officer determined tahtthe speech given by Fraser was
“indecent, lewd, and offensive to the modesty and decency of many
of the students and faculty in attendance at the assembly.”
The examiner determined that the speech fell within the ordinary
meaning of “obscene,” as used in the disruptive-conduct rule, and
affirmed the discipline in its entirety. Fraser served two days of his
suspension and was allowed to return to school on the third day.
Fraser, by his father as guardian ad litem, then brough this action in the
US District Court for the Western District of Washington. Fraser
alleged a violation of his First Amendment right to freedom of speech
and sought both injunctive relief and monetary damages.
District Court and Court of Appeals ruled in favor of Fraser. Hence this
petition.

ISSUE/s:
WoN Bethel School violated the rights of Fraser – NO

RULING: Reversed.

RATIO:
This Court acknowledged in Tinker v. Des Moines Independent
Community School District that students do not “shed their
constitutional rights to freedom of speech or expression at the
schoolhouse gate.”
That court appears to have proceeded on the theory that the use of lewd
and obscene speech in order to make what the speaker considered
to be a point in a nominating speech for a fellow student was
essentially the same as the waring of an armband in Tinker as a form
of protest or the expression of a political position.
In upholding the student’s right to engage in a nondisruptive, passive
expression of a political viewpoint in Tinker, this Court was careful to
note that the case did “not concern speech or action that intrudes
upon the work of the schools or the right of other students.”
“Public education must prepare pupils for citizenship in the Republic… It
must inculcate the habits and manners of civility as values in
themselves conducive to happiness and as indispensable to the
practice of self-government in the community and the nation.”
We echoed the essence of this statement of the objectives of public
education as the “inculcation of fundamental values necessary to the
maintenance of a democratic political system.”
These fundamental values of “habits and manners of civility” essential ot
a democratic society must, of course, include tolerance of divergent
political and religious views, even when the views expressed may be
unpopular.
the sensibilities of others, and, in the case of a school, the they were broadast at a time when children were undoubtedly in
sensisbilites of fellow students. the aduinece.
The undoubted freedom to advocate unpopular and controversial “These words offend for the same reasons that obscenity offendes.
views in schools and classrooms must be balanced against the Their place in the hierarchy of First Amendment values was aptly
society’s countervailing interest in teaching students the sketched by Justice Murphy: ‘such utterances are no essential
boundaries of socially appropriate behavior. Even the most part of any exposition of
heated political discourse in a democratic society requires
consideration for the personal sensibilities of the other
particpants and audiences.
The first amendment guarantees wide freedom in matters of adult
public discourse. It does not follow, however, that simply
because the use of an offensive form of expression may not be
prohibited to adults making what the speaker considers a
political point, the same latitude msut be permitted to children
in a public school.
We reaffirmed that the constitutional rights of students in public school
are not automatically coextensive with the rights of adults in other
settings.
Indeed, the “fundamental values necessary to the maintenance of a
democratic political system “disfavor the use of terms of debate
highly offensive or highly threatening to others. Nothing in the
constitution prohibits the states from insisting that certain
modes of expression are inappropriate and subject to
sanctions.
The schools, as instruments of the state, may determine that the
essential lessons of civil, mature conduct cannot be conveyed in a
school that tolerates lewd, indecent, or offensive speech and conduct
such as that indulged in by this confused boy.
The pervasive sexual innuendo in Fraser’s speech was plainly offensive
to both teachers and students – indeed to any mature person. By
glorifying male sexuality, and in its verbal content, the speech was
acutely insulting to teenage girl students. The speech could well be
seriously damaging to its less mature audience, many of whom were
only 14 years old and on the threshold of awareness of human
sexuality. Some students were reported as bewildered by the speech
and reaction of mimicry it provoked.
The Court’s First Amendment jurisprudence has acknowledged
limitations on the otherwise absolute interest of the speaker in
reaching an unlimited audience where the speech is sexually
explicit and the audience may include children.
These cases recognize the obvious concern on the part of parents, and
school authorities acting in loco parentis, to protect children –
especially in a captive audience – from exposure to sexually explicit,
indecent, or lewd speech. We have also recognized an interest in
protecting minors frm exposure to vulgar and offensive spoken
language.
The Commission concluded that “certain words depicted sexual and
excretory activities in a patently offensive manner, and noted that
ideas, and are of such slight social value as a step to truth that any
benefit that may be derved from them is clearly outweight by the social
interest in order and morality.’”
The First Amendment does not prevent the school officials form
determining that to permit a vulgar and lewd speech such as
Fraser’s would undermine the school’s basic education mission.
Stevens, J. Dissenting: For three reasons, I think not. First, it seems highly
unlikely that he would have decided to deliver the speech if he had
known that it would result in his suspension and disqualification from
delivering the school commencement address. Second, I believe a
strong presumption in favor of free expression should apply whenever an
issue of this kind is arguable. Third, because the Court has adopted the
policy of applying contemporary community standards in evaluating
expression with sexual connotations, this Court should defer to the views
of the district and circuit judges who are in a much better position to
evaluate his speech than we are.
HAZELWOOD v. KUHLMEIER (JP) pregnancy; the other discussed the impact of divorce on students at
January 13, 1988 | White, J. | Freedom of Speech the school.
Reynolds was concerned that, although the pregnancy story used false
PETITIONER: Hazelwood School District names "to keep the identity of these girls a secret," the pregnant
RESPONDENTS: Kuhlmeier, Students of Hazelwood East students still might be identifiable from the text. He also believed that
the article's references to sexual activity and birth control were
SUMMARY: Petitioners are officials of the Hazelwood East School District. The
inappropriate for some of the younger students at the school.
respondents are three students who were staffs of Spectrum, the school
In addition, Reynolds was concerned that a student identified by name in
newspaper. The students are alleging that the deletion of the principal of two
the divorce story had complained that her father "wasn't spending
pages containing two articles, One on pregnance and one on divorce, in the
enough time with my mom, my sister and I" prior to the divorce, "was
recent newspaper that they decided to publish arrogated upon their First
always out of town on business or out late playing cards with the
Amendment rights. The District Court favored the school officials. The CA guys," and "always argued about everything" with her mother.
reversed. Reynolds believed that the student's parents should have been given
an opportunity to respond to these remarks, or to consent to their
SC reversed the CA and explained that Reynolds’ decision was reasonabele
publication.
because (1) the students who had written and edited these articles had not
Principal Reynolds believed that there was no time to make the
sufficiently mastered those portions of the Journalism II curriculum that
necessary changes in the stories before the scheduled press run,
pertained to the treatment of controversial issues and personal attacks, (2)
and that the newspaper would not appear before the end of the
the need to protect the privacy of individuals whose most intimate concerns
school year if printing were delayed to any significant extent.
are to be revealed in the newspaper, and (3) the legal, moral, and ethical
Reynolds concluded that his only options under the circumstances were
restrictions imposed upon
to publish a four-page newspaper instead of the planned six-page
journalists within a school community that includes adolescent subjects
newspaper, eliminating the two pages on which the offending stories
and readers. appeared, or to publish no newspaper at all.
Accordingly, he directed Emerson (Spectrums’ adviser) to withhold from
DOCTRINE: Students cannot be punished merely for expressing their
personal views on the school premises unless school authorities have publication the two pages containing the stories on pregnancy and
reason to believe that such expression will substantially interfere with the divorce.
work of the school or impinge upon the rights of other students. Respondents subsequently commenced this action in the United States
District Court for the Eastern District of Missouri, seeking a
FACTS: declaration that their First Amendment rights had been violated,
Petitioners are the Hazelwood School District in St. Louis County, injunctive relief, and monetary damages.
Missouri; various school officials; Robert Eugene Reynolds, the The district court approved saying that the principal’s decision has a
principal; and Howard Emerson, the newspaper adviser. substantial and reasonable basis for the betterment of the students.
Respondents are three former Hazelwood East students who were staff The CA reversed. Hence this appeal.
members of Spectrum, the school newspaper.
Spectrum was written and edited by the Journalism II class at Hazelwood ISSUE/s:
East. WoN the deletion of the two pages of a school publication derogates
The practice at Hazelwood East during the spring 1983 semester was for upon First Amendment rights of the publishers – NO
the journalism teacher to submit page proofs of each Spectrum issue
RULING: Judgment of the CA is reversed.
to Principal Reynolds for his review prior to publication.
RATIO:
On May 10, Emerson (Spectrum’s advisor) delivered the proofs of the
Students in the public schools do not "shed their constitutional rights to
May 13 edition to Reynolds (principal), who objected to two of the
freedom of speech or expression at the schoolhouse gate."
articles scheduled to appear in that edition.
One of the stories described three Hazelwood East students' experiences
with
Students cannot be punished merely for expressing their personal
views on the school premises unless school authorities have
reason to believe
that such expression will substantially interfere with the work of parents, teachers, and state and local school officials, and not
the school or impinge upon the rights of other students. of federal judges.
First Amendment rights of students in the public schools are not It is only when the decision to censor a school-sponsored
automatically coextensive with the rights of adults in other settings, and publication,
must be applied in light of the special characteristics of the school
environment.
School facilities may be deemed to be public forums only if school
authorities have "by policy or by practice" opened those
facilities "for indiscriminate use by the general public," or by
some segment of the public, such as student organizations.
If the facilities have instead been reserved for other intended purposes,
"communicative or otherwise," then no public forum has been
created, and school officials may impose reasonable restrictions
on the speech of students, teachers, and other members of the
school community
The policy of school officials toward Spectrum was reflected in
Hazelwood School Board Policy 348.51 and the Hazelwood East
Curriculum Guide:
"[s]chool sponsored publications are developed within the
adopted curriculum and its educational implications in
regular classroom activities."
School officials did not deviate in practice from their policy that
production of Spectrum was to be part of the educational curriculum,
and a "regular classroom activity.
The journalism teacher then has the final authority with respect to almost
every aspect of the production and publication of Spectrum.
Respondents' assertion that they had believed that they could
publish practically anything in Spectrum was therefore dismissed by
the District Court as simply not credible.
The petitioners (journalism students) fails to demonstrate the clear
intent to create a public forum.
School officials did not evince either by policy or by practice, any intent to
open the pages of Spectrum to indiscriminate use, by its student
reporters and editors, or by the student body generally. Instead, they
reserved the forum for its intended purpose, as a supervised learning
experience for journalism students.
The SC holds that educators do not offend the First Amendment by
exercising editorial control over the style and content of student
speech in school- sponsored expressive activities, so long as
their actions are reasonably related to legitimate pedagogical
concerns.
This standard is consistent with our oft-expressed view that the
education of the Nation's youth is primarily the responsibility of
theatrical production, or other vehicle of student expression has no
valid educational purpose that the First Amendment is so "directly
and sharply implicated," as to require judicial intervention to
protect students' constitutional rights.
The SC also concludes that Principal Reynolds acted reasonably in requiring
the deletion from the May 13 issue of Spectrum of the pregnancy article,
the divorce article, and the remaining articles that were to appear on the
same pages of the newspaper.
On the pregnancy article: the principal concluded that the students'
anonymity was not adequately protected, however, given the other
identifying information in the article and the small number of pregnant
students at the school.
The girls in the article, however, did comment concerning their sexual
histories and their use or nonuse of birth control. It was not unreasonable
for the principal to have concluded that such frank talk was inappropriate
in a school-sponsored publication distributed to 14-year-old freshmen.
On the divorce article: The student who was quoted by name in the version
of the divorce article seen by Principal Reynolds made comments
sharply critical of her father.
The principal could reasonably have concluded that an individual publicly
identified as an inattentive parent was entitled to an opportunity to defend
himself as a matter of journalistic fairness. These concerns were shared
by other Spectrum's faculty advisers.
Bernan (with Marshall, Blackmun), Dissenting: Spectrum was a forum.
The school principal, without prior consultation or explanation, excised
six articles comprising two full pages of the May 13, 1983, issue of
Spectrum. He did so not because any of the articles would "materially
and substantially interfere with the requirements of appropriate
discipline," but simply because he considered two of the six
"inappropriate, personal, sensitive, and unsuitable" for student
consumption.
The principal violated the First Amendment's prohibitions against censorship
of any student expression that neither disrupts classwork nor invades the
rights of others, and against any censorship that is not narrowly tailored
to serve its purpose.
Even assuming that the majority is correct that the principal could sensor the
material, the manner of censoring is brutal. He did not so much as
inquire into obvious alternatives, such as precise deletions or additions
(one of which had already been made), rearranging the layout, or
delaying publication. Such unthinking contempt for individual rights is
intolerable from any state official.
RENO v. AMERICAN CIVIL LIBERTIES UNION (HENRY) are almost always limited to materal lacking redeeming social value.
June 26, 1997 | Stevens, J. | Freedom of Expression Hence, this petition.

PETITIONER: Janet Reno


RESPONDENTS: American Civil Liberties Union, et al.

SUMMARY: 2 sections from the Communications Decency Act of 1996


(CDA) seek to protect minors from harmful material on the Internet. The
constitutionality of such provisions were challenged by the respondents by
filing against Janet Reno, Attorney General of the US, for being violative of
the freedom of speech.

3 District Court Judges convened and held that the provisions were
vague and overbroad, thus, unconstitutional, for violating First and Fifth
Amendments.

SC held that CDA is a content based blanket restriction of speech, thus,


the provisions are void for being unconstitutional.

DOCTRINE: A statute regulating the content of speech is void.

FACTS:
2 provisions of the Communications Decency Act of 1996 (CDA) seek to
protect minors from harmful material on the Internet.
Sec. 223(a)(1) criminalizes the “knowing” transmission of “obscene or
indecent” mesaages to any recipient under 18 years of age.
Sec. 223(d) prohibits the “knowing” sending or displaying to a person
under 18 of any message “that, in context, depicts or describes, in
terms patently offensive as measured by contemporary community
standards, secual or excretory activities or organs”
20 plaintiffs filed against the Attorney General of the US and the Department
of Justic challenging the constitutionality of Secs. 223(a)(1) and 223(d).
The Dirstrict Judge entered into a TRO against the enforcement of Sec.
223 insofar as it applies to indecent communications.
A second suit was filed by 27 additional plaintiffs, and the District Court
entered a preliminary injuction against enforcement of both the
challenged provisions.
The 3 District Court Judges convened and held on the consolidated
cases that the phrases “patently offensive display” and “indecent
transmission” were inherently vague and for being overbroad, that
the statute violates the Fifth and First Amendments, respectively.
The Government argues that the CDA is no more vague than the
obscenity standard established in Miller v. California; and on the
issue of CDA’s facial overbreadth, it contends that it leaves open
ample alternative channels of communication; that the plain meaning
of the CDA’s “konwledge and “specific person” requirement
significantly restricts its permissible applications; and the prohibitions
ISSUE/s:
WoN the CDA’s “indecent transmission” and “patently offensive display”
provisions violate the freedom of speech.– YES

RULING: SC affirmed the lower courts decision. Pwederin wherefore.

RATIO:
The vagueness of CDA violates the First Amendment as it is a content
based regulation of speech, has obvious chilling effect on free
speech and it is a criminal statute which threatens violators with
penalties.
The increased deterrent effect with the risk of discriminatory enforcement
of vague regulations poses greater First Amendment concerns.
The Miller case provides for the three-pronged test of obscenity: (a) whether
the average person, applying contemporary community standards would
find that the work, taken as a whole, appeals to the prurient interest; (b)
whether the work depicts or describes, in a patently offensive way,
sexual conduct specifically defined by the applicable state law; and (c)
whether the work, taken as a whole, lacks serious literary, artistic,
political, or scientific value.
The second test was omitted from the CDA – that the proscribed material
be “specifically defined by the applicable state law” – which reduces
the vagueness inherent in “patently offensive” and that CDA extends
to include “excretory activities” and “organs” of both sexual and
excretory nature.
However, with the requirements of societal value and prurient interest,
the phrases are still vague.
CDA also pursues that interest of protecting children by suppressing a
large amount of speech that adults have a constitutional right to send
and receive.
As to the contention that it leaves open alternative communications, the
same must also fail as CDA regulates speech on the basis of its
content; thus “a time, place, and manner” analysis is inapplicable.
As to the knowledge and specific person requirement, the fact remains
that most Internet fora are open to all comer. CDA would confer
broad powers of censorship, in the form of a “heckler’s vote,” upon
any opponent of indecent speech who mught simply log on and
inform that the would be discoursers that a specific person would be
present.
There is likewise no textual support as to its being limited to material
lacking redeeming social value.
ASHCROFT v. FREE SPEECH COALITION (DANNAH) Thus, Sec. 2256(8)(D) bans a range of sexually explicit images,
April 16, 2002 | Kennedy, J. | Art Expression sometimes called virtual child pornography, that appear to depict
minors but were produced by means other than using real children.
PETITIONER: Ashcroft, Atty. General et al. This can be done through
RESPONDENTS: Free Speech Coalition et al.

SUMMARY: Respondents, an adult-entertainment trade association, is


questioning the constitutionality of the Child Pornography Prevention Act of
1996 (CPPA). Sections 2256B which contains the words “appears to be”
and 2256D which contains the words “conveys the impression” appear to be
too sweeping.

The District Court voted in favor of constitutionality but the Court of Appeals
reversed. stating that the law was invalid on its face, finding it to be
substantially overbroad because it bans materials that are neither obscene
under Miller nor produced by the exploitation of real children as in Ferber.
The SC affirmed the CA’s decision, explaining that it is inconsistent with
what is not allowed or obscene under the Miller standard. Moreover, it
cannot find defense in Ferber because in that case, the speech had a
proximate link to the crime of child pornography. The Court also rejects
other arguments such as that the CPPA is necessary because pedophiles
may use virtual child pornography to seduce children. This runs afoul of the
principle that speech within the rights of adults to hear may not be silenced
completely in an attempt to shield children from it.

DOCTRINE: The mere tendency of speech to encourage unlawful acts is


not a sufficient reason for banning it, absent showing from conduct of a
direct connection
between the speech and imminent illegal conduct. The overbreadth
doctrine prohibits the Government from banning unprotected speech if
a substantial amount of protected speech is prohibited or chilled in the
process.

FACTS:
The Child Pornography Prevention Act of 1996 (CPPA) expands the
federal prohibition on child pornography to include not only
pornographic images using actual children, but also “any visual
depiction, including any photograph, film, video, picture, or computer
or computer-generated image or picture” that “is, or appears to be,
of a minor engaging in sexually explicit conduct” (Section 2256(8)
(B)), and any sexually explicit image that is “advertised, promoted,
presented, described or distributed in such a manner that conveys
the impression” it depicts “a minor engaging in sexually explicit
conduct” (Section 2256(8)(D)).
the use of youthful-looking adults or computer-imaging technology. statutory definition, their possessor would be subject to severe
Said section is aimed at preventing the production or distribution of punishment without inquiry into the literary value of the work.
pornographic material pandered as child pornography. This is inconsistent with an essential First Amendment rule:
Respondents, an adult-entertainment trade association among others,
feering that the CPPA threatened their activities, filed this suit
alleging that the “appears to be” and “conveys the impression”
provisions are overbroad and vague, chilling production of works
protected by the First Amendment.
The District Court disagreed but it was reversed by the CA stating that
pornography can be banned only if it is obscene under Miller v.
California, but pornography depicting actual children can be
proscribed whether or not the images are obscene because of the
State’s interest in protecting the children exploited by the production
process, and in prosecuting those who promote such exploitation.
The CA held the CPPA invalid on its face, finding it to be substantially
overbroad because it bans materials that are neither obscene under
Miller nor produced by the exploitation of real children as in Ferber.

ISSUE/s:
WoN the CA erred in invalidating the CPPA for being invalid on its face –
NO

RULING: The prohibitions of section B & D are overbroad and


unconstitutional.

RATIO:
Section B covers materials beyond the categories recognized in Ferber
and Miller.
It is inconsistent with Miller because it extends to images that are not
obscene under the Miller standard, which requires the Government
to:
Prove that the work in question, taken as a whole, appeals to the
prurient interest, is patently offensive in light of community
standards, and lacks serious literary, artistic, political, or scientific
value
In the CPPA materials need not appeal to the prurient interest. It is not
necessary, moreover that the image be patently offensive. Pictures of
what appear to be 17-year-olds engaging in sexually explicit activity
do not in every case contravene community standards.
A number of acclaimed movies, filmed without any child actors, explore
themes within the wide sweep of the statute’s prohibitions. If those
movies contain a single graphic depiction of sexual activity within the
A work’s artistic merit does not depend on the presence of a single The evil in question depends upon the actor’s unlawful conduct,
explicit scene. defined as criminal quite apart from any link to the speech in
Under Miller, redeeming value is judged by considering the work as a question, establishes that the speech ban is not narrowly drawn.
whole. Even though there is a scene in isolation which might be The mere tendency of speech to encourage unlawful acts is not a
offensive, if it is part of a narrative, the work does not automatically sufficient reason for banning it, absent showing from conduct of a
become obscene.
direct connection
The CPPA lacks the required link between its prohibitions and the affront
to community standards prohibited by the obscenity definition.
The CPPA finds no support in Ferber. The speech prohibited is virtually
distinguishable from material that may be banned under Ferber.
The case of Ferber upheld a prohibition on the distribution and sale of
child pornography, as well as its production, because these acts
were “intrinsically related” to the sexual abuse of children in two
ways:
As a permanent record of a child’s abuse, the continued circulation itself
would harm the child who had participated
Because the traffic in child pornography was an economic motive for its
production, the State had an interest in closing the distribution
network.
Under either rationale, the speech had a proximate link to the crime from
which it came.
Hence, in contrast to the speech in Ferber, speech that is itself the
record of sexual abuse, the CPPA prohibits speech that records no
crime and creates no victims by its production. Virtual child
pornography is not “intrinsically related” to the sexual abuse of
children.
The Government argues that Ferber acknowledged child pornography
rarely can be valuable speech. However,
The judgment was based upon how it was made, not on what it
communicated
The case did not hold that child pornography is by definition without
value. It recognized some works in this category might have
significant value, but relied on virtual images—the very images
prohibited by the CPPA which is an alternative and permissible
means of expression
As held in Ferber, “Where the speech is neither obscene nor the product
of sexual abuse, it does not fall outside the First Amendment
protection”.
The Court also rejects other arguments such as that the CPPA is
necessary because pedophiles may use virtual child pornography to
seduce children. This runs afoul of the principle that speech within
the rights of adults to hear may not be silenced completely in an
attempt to shield children from it.
between the speech and imminent illegal conduct.
The overbreadth doctrine prohibits the Government from banning
unprotected speech if a substantial amount of protected speech is
prohibited or chilled in the process.
The Government believes that the CPPA should not be read as a prohibition
on speech but as a measure to shift the burden to the accused to prove
that the speech is lawful.
However, even if an affirmative defense can save a statute from first
Amendment challenge, the defense is insufficient because it does not
apply to possession or to images ccreated by computer imaging, even
where the defendant could demonstrate no children were harmed in
producing the images.
Section D is also substantially overbroad.
The “conveys the impression” provision requires little judgment about the
image’s content; the work must be sexually explicit, but otherwise the
content is irrelevant.
Even if a film contains no sexually explicit scenes involving minors, t could be
treated as child pornography if the title and trailers convey the impression
that such scenes will be found in the movie.
Section D prohibits a substantial amount of speech that falls outside
Ginzburg’s rationale. Proscribed material is tainted and unlawful in the
hands of all who receive it, though they bear no responsibility for how it
was marketed, sold, or described.
US v. AMERICAN LIBRARY ASSOCIATION (ARIELLE) a. E-rate program – entitles qualifying libraries to buy Internet access at a
discount. à disabling is permitted “during use by an adult.”
June 23, 2003 | Rehnquist J. | Freedom of Speech

PETITIONER: United States


RESPONDENTS: American Library Association

SUMMARY:
Congress enacted the CIPA which provides that a public library may not
receive federal assistance to provide Internet access unless it installs a
software to block images that constitute obscenity or child pornography.
The American Library Association argue that the CIPA imposes an
unconstitutional condition on libraries that receive the E-rate and LSTA
subsidies by requiring them to surrender their First Amendment right to
provide the public with access to constitutionally protected speech. The
District Court ruled that the CIPA was facially unconstitutional and
enjoined the relevant agencies and officials from withholding federal
assistance for failure to comply with CIPA.

The SC held that A public library does not acquire Internet terminals in
order to create a public in order to create a public forum for Web
publishers to express themselves, any more than it collects books in
order to provide a public forum for the authors of books to speak. It
provides Internet access, not to “encourage a diversity of views,” but for
the same reasons it offers other library resources: to facilitate research,
learning, and recreational pursuits by furninshing materials of requisite
and appropriate quality. Moreover, , the CIPA does not “penalize” libraries
that choose not to install the software, but CIPA simply reflects the
Congress’ decision not to subsidize their doing so. To the extent that
libraries wish to offer unfiltered access, they are free to do so without
federal assistance.

DOCTRINE:
Most libraries already exclude pornography from their print collections
because they deem it inappropriate for inclusion. We do not subject these
decisions to heightened scrutiny; it would make little sense to treat
libraries’ judgments to block online pornography any differently, when
these judgments are made for just the same reason.

FACTS:
Congress enacted the Children’s Internet Protection Act (CIPA) which
provides that a public library may not receive federal assistance to
provide Internet access unless it installs software to block images
that constitute obscenity or child pornography.
The District Court held these provisions facially invalid on the ground that
they induce public libraries to violate the First Amendment.
To help public libraries provide their patrons with Internet access,
Congress offers two forms of federal assistance:
The grants of the Institute of Museum and Library Services – makes grants The Congress has wide latitude to attach conditions to the receipt of
to state library administrative agencies to “electronically link libraries federal assistance in order to further its policy objectives.
with educational, social, or information services.” à disabling is
permitted during use by any person.
These programs succeeded in bringing Internet access to public libraries
because by year 2005, 95% of the nation’s libraries provided public
internet access.
Internet provides patrons with a vast amount of valuable information, but
there is also an enormous amount of pornography on the Internet.
The accessibility of this material has created serious problems for
libraries, which have found that patrons of all ages, including minors,
regularly search for online pornography.
Upon discovering these problems, Congress became concerned that the
E-rate and LSTA were facilitating access to illegal and harmful
pornography.
Before Congress enacted CIPA, public libraries used softwares on at
their Internet terminals, which blocks categories of material such as
pornography. But a filter set to block obscene materials may
sometimes block other sites that present neither obscene nor
pornographic material. To minimize this problem, a library can set its
software bto prevent the blocking of material that falls into categories
like “Education,” “History,” and “Medical.”
Appellees, a group of library associations, sued the US and Government
agencies responsible for administering the E-rate and LSTA
programs, challenging the constitutionality of CIPA’s filtering
provisions. They argue that the CIPA imposes an unconstitutional
condition on libraries that receive the E-rate and LSTA subsidies by
requiring them to surrender their First Amendment right to provide
the public with access to constitutionally protected speech.
The District Court ruled that the CIPA was facially unconstitutional and
enjoined the relevant agencies and officials from withholding federal
assistance for failure to comply with CIPA.
According to the District Court, any public library that complies with
CIPA’s conditions will necessarily violate the First Amendment.
The District Court acknowledged that generally, the First Amendment
subjects libraries’ content-based decisions about which print
materials to acquire for their collections to only rational basis review.
The District Court also likened Internet access in libraries to
“traditional public fora” such as sidewalks and parks because it
promotes the First Amendment.

ISSUE/s:
WoN the CIPA, by not providing federal assistance to public libraries with
no filter software, violates the First Amendment – NO

RULING: We reverse the ruling of the District Court.

RATIO:
Public libraries pursue worthy missions of facilitating learning and cultural Internet collection, it could do so only at the cost of excluding an
enrichment. To fulfill their missions, public libraries must have broad enormous amount of valuable information that it lacks the capacity to
discretion to decide what material to provide their patrons. However, review.
their goal has never been to provide “universal coverage.” Due to the software’s limitations, many erroneously blocked web
Cases cited: pages contain content that is completely harmless for both
Arkansas Ed. Television v. Forbes: Public forum principles do not generally apply adults and minors.
to a public television station’s editorial judgments regarding the private Appellees urge the court to affirm the District Court’s judgment on the
speech it presents to viewers. alternative ground that CIPA imposes an unconstitutional condition
National Endowment for Arts v. Finley: Upheld the art funding program that on the receipt of federal assistance. Under this doctrine, the
required the National Endowment for bthe Arts to use content-based ctriteria
in making funding decisions.
government may not deny a benefit to a person on a basis that
The principles in Forbes and Finley apply to a public library’s exercise of infringes his constitutionally protected freedom of speech even if he
judgment in selecting the material it provides to its patrons. has no entitlement to that benefit.
The public forum principles on which the District Court replied on are out The Government counter’s appellees claims that the CIPA imposes an
of place in the context of this case. Internet access in public libraries unconstitutional condition on libraries on the ground that Government
is neither a “traditional” nor a “designated” public forum. entities do not have First Amendment rights. The First Amendment
Internet has not “immemorially been held in trust for the use of the protects the press from governmental interference; it confers no
public.” Doctrines surrounding traditional public forums may not be analogoues protection on the government.
extended to situations where such history is lacking. Even assuming that appellees may assert an “unconstitutional
Internet access in a public library does not satisfy our definition of a conditions,” claim, this claim would fail because when the
“designated public forum.” To create such forum, the government Government appropriates public funds to establish a program, it is
must make an affirmative choice to open up its property for use as a entitled to define the limits of that program.
public forum. The government does not create a public forum by The E-rate and LSTA programs were intended to help public libraries
inaction or by permitting limited discourse, but only by intentionally fulfill their traditional role of obtaining material of requisite and
opening a non-traditional forum for public discourse. appropriate quality for educational and informational purposes.
A public library does not acquire Internet terminals in order to create a public Congress may insist that these “public funds be spent for the
in order to create a public forum for Web publishers to express purposes for which they were authorized,” and Congress can
themselves, any more than it collects books in order to provide a public reasonably impose a parallel limitation on its Internet assistance
forum for the authors of books to speak. It provides Internet access, programs.
not to “encourage a diversity of views,” but for the same reasons it Justice Stevens asserts the premise that a federal statute penalizing a
offers other library resources: to facilitate research, learning, and library for failing to install filtering software on every one of its
recreational pursuits by furninshing materials of requisite and Internet-accessible computers would violate the First Amendment.
appropriate quality. However, the CIPA does not “penalize” libraries that choose not
The Internet is simply another method for making information available in to install the software, but CIPA simply reflects the Congress’
a school or library. It is no more than a technological extension of the decision not to subsidize their doing so. To the extent that
book stack. libraries wish to offer unfiltered access, they are free to do so
The District Court argues that a public library enjoys less discretion in without federal assistance.
deciding which Internet materials to make available than in making A refusal to fund protected activity cannot be equated with the imposition
book selections. We do not agree. A library’s failure to make quality- of a penalty on that activity. A legislature’s decision not to
based judgments about all the material it furnishes does not taint the subsidize the exercise of a fundamental right does not infringe
judgments it makes. the right.
Most libraries already exclude pornography from their print collections
because they deem it inappropriate for inclusion. We do not subject
these decisions to heightened scrutiny; it would make little sense to
treat libraries’ judgments to block online pornography any differently,
when these judgments are made for just the same reason.
Libraries cannot possibly segregate, item by item, all the Internet material
that is appropriate for inclusion from all that is not. While it can limit
its
FERNANDO v. CA (IYA) ISSUE/s:
December 6, 2006 | Quisimbing, J. | Freedom of Expression Whether the appellate court erred in affirming the conviction - NO

PETITIONER: Gaudencio Fernando and Rudy Estorninos


RESPONDENTS: Court of Appeals

SUMMARY: A search warrant was issued on the store Music Fair, owned
by Fernando and managed by Estorninos. The warrant also provided for
the seizure of obscene and phonographic materials found within the
store.

The PNP conducted the search and seizure. Fernando and Estorninos
was charged with violation of Art. 201 of the RPC. The RTC and CA
found the them guilty for the sale of obscene materials.

The SC ruled the same saying the State has authority to regulate the
sale of obscene materials as there is substantial governmental interest
that needs protecting.

DOCTRINE: As obscenity is an unprotected speech which the State has


the right to regulate, the State in pursuing its mandate to protect, as
parens patriae, the public from obscene, immoral and indecent materials
must justify the regulation or limitation.

FACTS:
Acting on reports of sale and distribution of pornographic materials,
officers of the Philippine National Police conducted police
surveillance on the store of Gaudencio E. Fernando Music Fair
(Music Fair).
Judge Perfecto Laguio of the RTC of Manila issued a search warrant for
violation of Article 201 of the RPC against Fernando and Tingchuy.
The warrant ordered the search of Gaudencio E. Fernando Music Fair at
564 Quezon Blvd., corner Zigay Street, Quiapo, Manila, and the
seizure of the specific items.
On the same day, police officers of the PNP-CIDG NCR served the
warrant on Rudy Estorninos, who introduced himself as the store
attendant of Music Fair.
The police searched the premises and confiscated twenty-five (25) VHS
tapes and ten (10) different magazines, which they deemed
pornographic.
Fernando and Estorininos were then charged of the felony under Art 201
of the RPC and was found guilty.
At the outset, we note that the trial court gave petitioners the opportunity
to adduce evidence to refute the prosecution’s evidence. Instead,
they waived their right to present evidence and submitted the case
for decision.
The CA affirmed in toto.
Hence the instant petition.
RULING: Decision of the CA is affirmed.

RATIO:
The ordinance is a valid governmental response to the serious problems
The Solicitor General counters that owners of establishments selling
obscene publications are expressly held liable under Article 201, and
petitioner Fernando’s ownership was sufficiently proven.
As the owner, according to the Solicitor General, Fernando was naturally
a seller of the prohibited materials.
As obscenity is an unprotected speech which the State has the
right to regulate, the State in pursuing its mandate to protect, as
parens patriae, the public from obscene, immoral and indecent
materials must justify the regulation or limitation.
There is no perfect definition of “obscenity” but the latest word is that of
Miller v. California which established basic guidelines, to wit:
whether to the average person, applying contemporary
standards would find the work taken as a whole, appeals to
the prurient interest;
whether the work depicts or describes, in a patently offensive
way, sexual conduct specifically defined by the applicable
state law; and
whether the work, taken as a whole, lacks serious literary,
artistic, political, or scientific value.
No one will be subject to prosecution for the sale or exposure of obscene
materials unless these materials depict or describe patently offensive
“hard core” sexual conduct.
The confiscated items were evidently obscene as adjudged by the RTC
and affirmed by the CA. Their opinion is given great respect.
Fernando, et al. did not adduce evidence to the contrary.
NAVARRO v. VILLEGAS (ELIEL) RATIO:
February 26, 1970 | Ponencia, J. | Right to Peacable Assembly That Mayor Villegas has not denied nor absolutely refused the permit
sought be Navarro.
PETITIONER: Nelson Navarro Mayor possess reasonable discretion ot determine or specify the streets
RESPONDENTS: Mayor Antonio Villegas or public places to be used for the assembly in order to secure
convenient use thereof by others and provide adequate and proper
SUMMARY: Navarro wrote a letter to Mayor Villegas requesting for a policing to minimize the risks of disorder and maintain public safety
permit to publicly assemble at Plaza Miranda. However, the Mayor and order;
refused to grant such request but instead gave Navarro an option to hold Mayor expressly stated his willingness to grant permits for peaceful
the assembly at Sunken Gardens. Navarro filed this present action assemblies at Plaza Miranda during Saturdays, Sundays and
asserting that his right to peacable assembly was infringed. holidays when they would not cause unnecessarily great disruption of
the normal activities of the community and has further offered
The SC held that the right to peaceable assembly is not absolute. It may Sunken Gardens as an alternative to Plaza Miranda as the site of the
be subject to regulation. The Mayor has discretion to grant or deny the demonstration sought to be held this afternoon.
permit because of the clear and imminent danger that it poses. There are A public rally at Plaza Miranda, as compared to one at the Sunken
aftermaths of bloodshed and breaches of peace. The Mayor did not Gardens as he suggested, poses a clearer and more imminent
outright refuse Navarro, but showed his willingness only subject to danger of public disorders, breaches of the peace, criminal act, and
certain conditions to prevent unnecessary great disruption. even bloodshed as an aftermath of such assemblies, and Navarro
has manifested that it has no means of preventing such disorders;
DOCTRINE: The right to freedom of assembly is not denied; but this right Civil rights and liberties can exist and be preserved only in an ordered
is neither unlimited nor absolute. It is not correct to say that the Mayor society;
has refused to grant the permit applied for; he offered an alternative Navarro failed to show a clear specific legal duty on the part of Mayor
which, in my opinion, is not unreasonable. There being no arbitrary Villegas to grant their application for permit unconditionally.
refusal to grant permit, Navarro is riot entitled to the writ. Villamor, J. Concurring: The right to freedom of assembly is not denied;
but this right is neither unlimited nor absolute. It is not correct to say
that the Mayor has refused to grant the permit applied for; he offered
FACTS:
an alternative which, in my opinion, is not unreasonable. There being
On February 24, 1970, Navarro, acting in behalf of the Movement of a no arbitrary refusal to grant permit, Navarro is riot entitled to the writ.
Democratic Philippines, wrote a letter to the respondent, the Mayor Castro and Fernando, J. Dissenting: The right to freedom of assembly
of the city of manila, applying to hold a rally at Plaza Miranda while not unlimited is entitled to be accorded the utmost deference
February 26, 1970, from 4-11 PM. and respect.
On the same day, Navarro wrote a reply, denying his request on the “For in deciding whether or not to withhold a permit, the members of the
grounds that, they have temporarily adopted the policy of not issuing Commission were to be guided only by their own ideas of ‘public
any permit for the use of Plaza Miranda for rallies or demonstration welfare, peace, safety, health, decency, good order, moreals or
during weekdays due to the events that happened from the past convenience.”
week. This ordinance as it was witten, therefore, fell squarely within the ambit of
On the same letter, Mayor Villegas gave Navarro an option to use the the many deisions of this Court over the last 30 years, holding that a
Sunken Garden near Intramuros for its rally, and for it to be held law subjecting the exercise of First Amendment freedoms to the prior
earlier for it to end before dark. restraint of a license, without narrow, objective, and definite
Navarro filed suit contesting the Mayor’s action on the ground that it standards to guide the licensing authority, is unconstitutional.
violates Navarro’s right to peaceable assembly and petition the
government for redress of grievances.

ISSUE/s:
41. WoN Mayor in denying permit violated the right of Navarro – NO

RULING: The right of peacable assembly is not absolute and subject to


regulation.
PBMEO v. PHILIPPINE BLOOMING MILLS (JP) Management informed PBMEO that the demonstration is an inalienable
June 5, 1973 | Makasiar, J. | Hierarchy of Rights right of the union guaranteed by the Constitution but emphasized,
however, that any demonstration for that matter should not unduly
prejudice the normal operation of the Company.
PETITIONER/S: Philippine Blooming Mills Employees Organization, et The Company warned PBMEO those workers, who shall fail to report for
al. RESPONDENT/S: Philippine Blooming Mills Co., Inc., Court of work the following morning shall be dismissed, because such failure
Industrial Relations is a violation of the existing Collective Bargaining Agreement (CBA)
and, therefore, would be amounting to an illegal strike. The CBA had
SUMMARY: The Phil. Blooming Mills Employees Organizationplanned a a No Lockout – No Strike policy.
mass demonstration at Malacañang against alleged abuses of the Pasig After the demonstration proceeded, the Company filed charges against
Police. The Company, however, warned PBMEO that this demonstration petitioners for violating the existing CBA and 8 of PBMEO were found
would affect their operations and a violation of their Collective Bargaining directly responsible for perpetrating unfair labor practice and
Agreement and the penalty would tantamount to their dismissal. PBMEO considered to have lost their status as employees.
countered that this demonstration is not against the company but in The RTC found petitioner PBMEO guilty of bargaining in bad faith and
exercise of their rights to free expression, free assembly and petition. dismissed the officers of such organization. PBMEO filed for an MR.
The CIR dismissed the case in favor of the corporation because the CIR dismissed the motion for reconsideration of PBMEO on the ground
petitioner PBMEO filed the petition two days late. of being pro forma for filing beyond the reglementary period (2-days
late).
The SC reverses the ruling of the CIR. Following the hierarchy of civil
ISSUE/S:
rights, the human rights of expression, assembly and petition for redress
of grievances by the employee organization must have primacy of the WoN the constitutional guarantees of free expression, of peaceful
unsupported property rights of the company. The mass demonstration of assembly and of petition of the employees are violated with the
PBMEO is a valid exercise of their constitutional rights. Respondent prohibition of the company to participate in the mass demonstration
company should be the one guilty of unfair labor practice according to RA – YES
875 for preventing their employees from exercising their rights.
RULING: CIR order is null and void. The dismissed employees are reinstated
DOCTRINE: The Primacy of human rights over property rights is with full back wages.
recognized. Because these freedoms are “delicate and vulnerable, as
well as supremely precious in our society” and the “threat of sanctions RATIO:
may deter their exercise almost as potently as the actual application of
sanctions,” they “need breathing space to service,” permitting While the Bill of Rights also protects property rights, the primacy of
government regulation only “with narrow specificity.” human rights over property rights is recognized.
Because these freedoms are "delicate and vulnerable, as well as
In the hierarchy of civil liberties, the rights of free expression and of supremely precious in our society" and the "threat of sanctions may
assembly occupy a preferred position, as they are essential to the deter their exercise almost as potently as the actual application of
preservation and vitality of our civil and political institutions; and such sanctions," they "need breathing space to survive," permitting
priority “gives these liberties the sanctity and the sanction not permitting government regulation only “within narrow specificities.”
dubious intrusions.” Property and property rights can be lost thru prescription; but human
rights are imprescriptible. If human rights are extinguished by the
passage of time, then the Bill of Rights is a useless attempt to limit
FACTS: the power of government and ceases to be an efficacious shield
Petitioners (Phil. Blooming Mills Employees Organization) claim that on against the tyranny of officials, of majorities, of the influential and
March 1, 1969, they decided to stage a mass demonstration at powerful, and of oligarchs – political, economic or otherwise.
Malacanang on March 4, 1969, in protest against alleged abuses of In the hierarchy of civil liberties, the rights of free expression and of
the Pasig police, to be participated in by the workers in the first shift assembly occupy a preferred position, as they are essential to
as well as those in the regular second and third shifts; and that they the preservation and vitality of our civil and political
informed the respondent Company of their proposed demonstration. institutions; and such priority “gives these liberties the sanctity
and the sanction not permitting dubious intrusions.”
A constitutional or valid infringement of human rights requires a more members as well as their total presence at the demonstration site in
stringent criterion, namely existence of a grave and immediate order to generate
danger of a substantive evil which the State has the right to prevent.
The freedoms of speech and of the press as well as of peaceful
assembly and of petition for redress of grievances are absolute when
directed against public officials or "when exercised in relation to our
right to choose the men and women by whom we shall be governed.
The demonstration was not against their employer but was purely and
completely an exercise of their freedom of expression in general and
of their right of assembly and of petition for redress of grievances in
particular before the appropriate governmental agency, the Chief
Executive, against the police officers of the municipality of Pasig.
The employees' pathetic situation was a stark reality: abused, harassed
and persecuted as they believed they were by the peace officers of
the municipality.
The company’s failure to defend its own employees all the more
weakened the position of its laborers vis-a-vis the alleged oppressive
police, who might have been all the more emboldened thereby to
subject its lowly employees to further indignities.
To regard the demonstration against police officers, not against the
employer, as evidence of bad faith in collective bargaining and hence
a violation of the collective bargaining agreement and a cause for the
dismissal from employment of the demonstrating employees,
stretches unduly the compass of the collective bargaining
agreement, is "a potent means of inhibiting speech" and
therefore inflicts a moral as well as mortal wound on the
constitutional guarantees of free expression, of peaceful
assembly and of petition.
The collective bargaining agreement which fixes the working shifts of the
employees, according to the respondent Court of Industrial
Relations, in effect imposes on the workers the "duty x x x to observe
regular working hours."
The strained construction of the Court of Industrial Relations that such
stipulated working shifts deny the workers the right to stage a mass
demonstration against police abuses during working hours,
constitutes a virtual tyranny over the mind and life of the workers and
deserves severe condemnation.
The mass demonstration staged by the employees on March 4, 1969
could not have been legally enjoined by any court, for such an
injunction would be trenching upon the freedom of expression
of the workers, even if it legally appears to be an illegal
picketing or strike.
The respondent firm claims that there was no need for all its employees
to participate in the demonstration in order that loss or damage to the
firm will be averted.
This stand failed to appreciate the sine qua non of an effective demonstration
especially by a labor union, namely the complete unity of the Union
the maximum sympathy for the validity of their cause but also immediate
action on the part of the corresponding government agencies.
The respondent company is the one guilty of unfair labor practice.
Because the refusal on the part of the respondent firm to permit all its
employees and workers to join the mass demonstration constituted an
unconstitutional restraint on their freedom of expression, freedom
of assembly and freedom to petition for redress of grievances, the
respondent firm committed an unfair labor practice defined in Section
4(a-1) in relation to Section 3 of Republic Act No. 875, otherwise
known as the Industrial Peace Act.
Section 3 of Republic Act No. 875 guarantees to the employees the right "to
engage in concerted activities for x x x mutual aid or protection"; while
Section 4(a-1) regards as an unfair labor practice for an employer "to
interfere with, restrain or coerce employees in the exercise of their rights
guaranteed in Section Three."
Court of Industrial Relations did not make any finding as to the fact of loss
actually sustained by the firm. This significant circumstance can only
mean that the firm did not sustain any loss or damage.
On the contrary, the company saved a sizable amount in the form of wages
for its hundreds of workers, cost of fuel, water and electric consumption
that day. Such savings could have amply compensated for unrealized
profits or damages it might have sustained by reason of the absence of
its workers for only one day.
There is no time limit to the exercise of these freedoms. The right to enjoy
them is not exhausted by the delivery of one speech, the printing of one
article or the staging of one demonstration.
It is a continuing immunity, to be invoked and exercised when exigent and
expedient whenever there are errors to be rectified, abuses to be
denounced, inhumanities to be condemned.
Management has shown not only lack of good-will or good intention, but a
complete lack of sympathetic understanding of the plight of its laborers
who claim that they are being subjected to indignities by the local police.
It was more expedient for the firm to conserve its income or profits than to
assist its employees in their fight for their freedoms and security against
alleged petty tyrannies of local police officers. This is sheer opportunism.
Barredo, Dissenting: The main opinion does not cite any constitutional
provision, law or rule or any judicial doctrine or principle
supporting its basic holding that infringement of constitutional
guarantees, other than denial of due process, divests courts of
jurisdiction to render valid judgments.
JBL REYES v. BAGATSING (HENRY) WoN the permit to rally shall be granted – YES
November 9, 1983 | Fernando, C.J. | Freedom of Speech and Peacable
Assembly

PETITIONER: Jose B.L. Reyes, on behalf of the Anti-Bases Coalition


(ABC)
RESPONDENTS: Ramon Bagatsing

SUMMARY: Petitioner JBL Reyes on behalf of the Anti-Bases Coalition


sought for a rally permit for their event infront of the US Embassy. This
permit was denied by the respondent City Mayor Bagatsing, to which
petitioner forwarded to the SC the present mandatory injunction.

SC ruled that since there is no clear and present danger that is eminent
to the case at bar, there is no reason to deny such permit. Hence, the
mandatory injunction is granted.

DOCTRINE: Freedom of assembly connotes the right people to meet


peaceably for consultation and discussion of matters of public concern. It is
entitled to be accorded the utmost deference and respect. It is not to be
limited, muc less denied, except on a showing as the case with freedom
of expression, of a clear and present danger of a substantive evil that
the state has a right to prevent.

FACTS:
Petitioner JBL Reyes, on behalf of the Anti-Bases Coalition (ABC) sought
a permit from the City of Manila to hold a peaceful march and rally on
Oct. 26, 1983 from 2-5pm in the afternoon, from Luneta to the US
Embassy. Once there, a short program wuld be held.
In the course of the oral argument, it was stated that after the delivery of
two brief speeches, a petition based on the resolution adopted on the
last day by the International Conference for General Disbarment,
World Peace and the Removal of All Foreign Military Bases held in
Manila, would be presented to a representative of the Embassy or
any of its personnel who may be there so that it may b delivered to
the US Ambassador.
Such permit was denied, and petitioner JBL Reyes was unaware of such
denial. The reason for refusing the permit was due to police
intelligence reports affirming the plans of subversive/criminal
elements to infilgrate and/or disrupt any assembly or congregations
where a large number of people is expected to attend.
Respondent Mayor suggested however, in accordance with the police
recommendation, a permit may be issued if the assembly is held at
the Rizal Coliseum or any other enclosed area where the safety of
the participants may be ensured.
Petitioner hence filed the present mandatory injunction to the SC.

ISSUE/s:
RULING: SC granted the mandatory injunctioin prayed for. The procedure for the securing of such permits for peacable assembly is
succinctly set forth in the opinion of the SC, with the injunction that
RATIO: “the presumption must be to incline the weight of the scales of justice
It is evident that the SC is called upon to protect the exercise of on the side of such rights, enjoying as they do, precedence and
thecognate rights to free speech and peaceful assembly, arising from primacy.” The exception
the denial of the permit. Free speech, like freepress, may be
identified with the liberty to discuss publicly and truthfully any matter
of public concern without censorship or punishment.
Freedom of assembly connotes the right people to meet peaceably for
consultation and discussion of matters of public concern. It is entitled
to be accorded the utmost deference and respect. It is not to be
limited, muc less denied, except on a showing as the case with
freedom of expression, of a clear and present danger of a
substantive evil that the state has a right to prevent.
The sole justification for a limitation on the exercise of this right, so
fundamental to the maintenance of democratic institutions, is the
danger, of a character both grave and imminent of a serious evil to
public safety, public morals, public health, or any other legitimate
public interest.
In the case of Municipality of Cavite v. Rojas, the SC affirmed that plazas
or parks and sreets are outside the commerce of man and thus
nullified a contract that a particular leased plaza is a promenade for
public use. There is hence no valid reason why a permit should not
be granted for the proposed march and rally starting from a public
park that is the Luneta.
Respondent Mayor posed the issue as well of the applicability of
Ordinance No. 7295, prohibiting the holding or staging of rallies or
demonstrations within a radious of 500 ft from any foreign mission or
chancery. However, this is not applicable in the present case as there
is no proof that the distance of the rally is withint the embassy’s 500ft
radius.
Not related to the topic pero baka itanong: Bagasting likewise pointed out
that it was his intention to provide protection to the US Embassy from
lawless elements pursuant to Art. 22 of the Vienna Convention on
Diplomatic Relations. Whereas the SC ruled that a treaty may NOT
supersede provisions of the Constitution. And hence, the Constitution
shall govern, insofar as it upholds freedom of expression and the
freedom to peaceably assemble.
Teehankee, concurring: In the case of Primicias v. Fugoso, “the right to
freedom of speech and peaceably assemble and petition the
government for redress of grievances are fundamental personal
rights of the people recognized and guaranteed by the constitutions
of democratic countries” and that the city or town mayors are not
conferred “the power to refuse to grant the permit, but only the
discretion, in issuing the permit, to determine or specify the streets or
public places where the pareade or procession may pass or the
meeting may be held.”
of the clcear and present danger rule, which alone would warrant a limitation
of these fundamental rights is stated in the SC’s opinion, that the sole
justification for a limitation on the exercise of this right, so fundamental to
the maintenance of democratic institutions, is the danger, of a character both
grave and imminent, of a serious evil to public safety, public morals, public
health, or any other legitimate public interest.
Makasiar, concurring: The 1987 Philippine Constitution should prevail
over the Vienna Convention.
Abad Santos, concurring: He just wanted to concur to state for the
record that he voted for the issuance ex-parte of a preliminary
mandatory injunction. Relevant talaga to yes wtf thanks abad santos
Plana, concurring: In J. Plana’s view, without saying that the Ordinance
is obnocious per se to the constitution, it cannot be validly invoked
whenever its application would collide with a constitutionally
guaranteed right such as freedom of assembly and/or expression, as
in the case at bar, regardless of whether the chancery of any foreign
embassy is beyond or within 500 ft from the situs of the rally or
demonstration
Aquino, dissenting: The holding of the rally in front of the US Embassy
violates Ordinance No. 7295 of the City of Manila.
MALABANAN v. RAMENTO (DANNAH) Petitioners were officers of the Supreme Student Council of the Gregorio
May 21, 1984 | Fernando, C.J. | Students’ Rights to Peaceable Araneta Univesrity Foundation (University). The principal
Assembly respondents are Ramento, Director of the NCR of the Ministry of
Education, Culture and
PETITIONER: Cris Malabanan, et al.
RESPONDENTS: Hon. Anastacio Ramento

SUMMARY: Petitioner students, who were officers of the Student Council of


the Gregorio Araneta Unversity Foundation (University), held a meeting of
which they have a permit. However, they held it the Veterinary Medicine and
Animal Science (VMAS) basketball court instead of the second floor lobby
(the place indicated in the permit). They manifested their opposition to the
propsed merger of the Institute of Animal Sciene with the Institute of
Agriculture. They marched towards the Life Science Building, disturbing
classes and non-academic employees.

They were asked to explain why they shouldn’t be liable for holding an illegal
assembly, and when they failed to do so, they were put on preventive
suspension. They brought the matter before the CFI of Rizal and Ministry of
Education, and respondent Ramento, the Director of the NCR of the Ministry of
Education and the University found them guilty of the charge having violated a
provision of the Manual for Private Schools more specifically their holding of an
illegal assembly which was characterized by the violation of the permit granted
resulting in the disturbance of classes and oral defamation. The penalty was
suspension for one academic year.

The SC issued a TRO from the Order of Ramento. Then, it reversed the
Order of Ramento stating that the penalty imposed was disproportional to
the violation committed. They have a right to freedom of expression and
peaceable assembly, even if they are students of a private institution.
Although they did not follow the venue in the permit, went beyond the hours
stated and caused disturbance, the Court seemed the proper penalty would
have been a one week suspension.

DOCTRINE: Justice Fortas in Tinker v. Des Moines stated that students do


not “shed their constitutional rights to freedom of speech or expression at
the schoolhouse gate”.

The rights to peaceable assembly and free speech are guaranteed students
of educational institutions. Necessarily, their exercise to discuss matters
affecting their welfare or involving public interest is not to be subjected to
previous restraint or subsequent punishment unless there be a showing of a
clear and present danger to a substantive evil that the state has a right to
present.

FACTS:
Sports and the University.
They sought and were granted a permit to hold a meeting from 8AM-
12PM on August 27, 1982. They then held a general assembly at the
Veterinary Medicine and Animal Science (VMAS) basketball court
instead of the second floor lobby (the place indicated in the permit).
Here they manifested in vehement and vigorous language their
opposition to the proposed merger of the Institute of Animal Science
with the Institute of Agriculture.
At 10:30 AM they marched toward the Life Science Building ang
continued their rally, disturbing classes and non-academic
employees within hearing distance.
They were asked why they should not be held liable for holding an illegal
assembly, then on September were informed that they were under
preventive suspension for not giving an explanation.
The students challenged the validity of the suspension before the CFI of
Rizal and before the Ministry of Education.
Ramento found them guilty of the charge having violated a provision of
the Manual for Private Schools more specifically their holding of an
illegal assembly which was characterized by the violation of the
permit granted resulting in the disturbance of classes and oral
defamation. The penalty was suspension for one academic year.
Hence, the present petition.

ISSUE/s:
WoN the facts as disclosed resulting in the disciplinary action and the
penalty imposed, there was an infringement of the right to peaceable
assembly and its cognate right of free speech – YES

RULING: Petition granted. Decision imposing one year suspension reversed


and set aside.

RATIO:
The Court issued a temporary restraining order enjoining Ramento et al.
from enforcing the Order of the Ministry of Education of Culture.
Although the suspension was enforced only briefly, and some of the
student petitioners finished their courses and were allowed to return
to school, the proceeding was considered moot and academic.
However, since the issue related to a violation of the constitutional rights
of freedom of peacable assembly and free speech, there is need to
pass squarely on the question raised.
The penalty imposed by Remanto was unduly severe. A lesser penalty is
more appropriate.
Reyes v. Bagatsing: They were convicted of sedition, but this was reversed by Justice
The invocation of the right to freedom of peaceable assembly carries with it Carson who stated that “if the prosecution be permitted to seize
the implication that the right to free speech has likewise been upon every instance of such disorderly conduct by individual
disregarded members of a crowd as an excuse
Here, a permit was sought to hold a peaceful march an rally from the
Luneta public park to the US Embassy
In an open space of public property, a short program was held
The Court held that streets and parks have immemorially been held in
trust for the use of the public (for purposes such as assemblies)
The situation in the case at hand is different. The assembly was held in a
private area, property of respondent University.
As stated in Reyes, “The applicants for a permit to hold an assembly
should inform the licensing authority of the date, the public place
where and the time when it will take place. If it were a private place,
only the consent of the owner or the one entitled to its legal
possession is required.”
Petitioner students did seek such consent, and it was indeed granted.
Petitioner students invoke their rights to peaceable assembly and
free speech, and they are entitled to do so.
Justice Fortas in Tinker v. Des Moines stated that students do not “shed
their constitutional rights to freedom of speech or expression at the
schoolhouse gate”.
A student’s rights do not embrace merely the classroom hour. He may
express his opinions on controversial subjects if he does so without
‘materially and substantially interfer[ing] with the requirements of
appropriate discipline in the operation of the school’ and without colliding
with the rights of others.
Although there was a vigorous presentation of views opposed to the
propsed merger, it was to be expected because they believed the
merger would result in increase of tuition fees, and it was a serious
problem.
Student leaders are hardly the timid, diffident types. They are likely to be
assertive and dogmatic. They would be ineffective if during a rally
they speak in the guarded and judicious language of the academe.
They may give the speakers the benefit of their applause, but with the
activity taking place in the school premises and during the daytime,
no clear and present danger of public disorder is discernible.
US v. Apurado:
500 residents of the municipality of San Carlos, Occidental Negros
assembled near the municipal building and upon the opening of the
session, barged into the council chamber
They demanded that officials be dismissed
They were wholly unarmed except for a few canes
to characterize the assembly as a seditious and tumultuous rising
against the authorities, then the right to assemble and to petition for
redress of grievances would become a delusion…”
The line must be drawn between disorderly and seditious conduct and
between an essentially peaceable assembly and a tumultuous uprising
However, the students should not be completely absolved because they did
indeed conduct the meeting in a place not specified in the permit, was
longer than the period allowed and caused disturbance.
If the concept of proportionality between the offense committed and the
sanction imposed is not followed, an element of arbitrariness intrudes.
Hence, a one-week suspension would have been proper.
The rights to peaceable assembly and free speech are guaranteed students
of educational institutions. Necessarily, their exercise to discuss matters
affecting their welfare or involving public interest is not to be subjected to
previous restraint or subsequent punishment unless there be a showing
of a clear and present danger to a substantive evil that the state has a
right to present.
BAYAN v. ERMITA (ARIELLE) choice of venue and is thus repugnant to the freedom of expression
April 25, 2006 | Azcuna, J. | Freedom of Speech; Peaceful Assembly; Petition clause. They also assert that the law is not content-neutral as it
the Government for redress of grievances

PETITIONER: Bayan, Karapatan, Kilusang Magbubukid ng Pilipinas


RESPONDENTS: Eduardo Ermita in his capacity as Executive Secretary

SUMMARY:
BP 880 or the Public Assembly Act was promulgated which provided for
guidelines for holding rallies, public meetings, processions, etc. It
required that a written permit shall be required to hold a public assembly
in a public place. Moreover, the police officers are given authority to
disperse such assemblies should they become violent. Petitioners assail
the constitutionality of the law, insofar as it curtails their freedom of
expression, the right to peaceably assemble, and the right to petition the
government for redress of grievances.

The SC ruled that such law is content-neutral, as it does not regulate the
speech, but only the time, manner and place. There is likewise no prior
restraint since the content of the speech is not relevant to the regulation.

DOCTRINE : It regulates the exercise of the right to peaceful assembly


and petition only to the extent needed to avoid a clear and present
danger of the substantive evils Congress has the right to prevent.

FACTS:
BP 880 or the Public Assembly Act of 1985 was enacted, providing that
the processions, rallies, parades, demonstrations, public meetings
and assemblages for religious purposes, shall be governed by local
ordinances. It also provides that a written permit shall be required for
any person or persons to organize and hold a public assembly in a
public place. Moreover, no public assembly with a permit shall be
dispersed except when the assembly becomes violent.
Statement of Exec. Secretary Ermita: In view of intelligence reports
pointing to credible plans of anti-government groups to inflame the
political situation, we have instructed the PNP as well as the LGUs to
strictly enforce a “no permit, no rally” policy, disperse groups that run
afoul of this standard, and arrest all persons violating the laws of the
land.
Bayan, et. al – allege that their rights as organizations and individuals
were violated when the rally dispersed by policemen implementing
BP No. 880. They assail BP 880 as a clear violation of the
Constitution and the ICCPR and other human rights treaties which
the PH is a signatory. They also argue that BP 880 requires a permit
beore one can stage a public assembly regardless of the presence
or absence of a clear and present danger, and that it curtails the
does not apply to mass actions in support of the government. fundamental personal rights of the people. But the exercise of these
Jess del Prado, et. al – allege that they were injured, arrested, and rights
detained when a peaceful mass action they held was preempted and
violently dispersed by the police. They argue that the law is
unconstitutional because it is a curtailment of the right to peacefully
assemble and petition for redress of grievances because it puts a
condition for the valid exercise of that right. Thus, its provisions are
not mere regulations but are actually prohibitions.
Kilusang Mayo Uno – allege that they conducted peaceful mass actions
and that their rights as organizations and those of their individual
members as citizens, specifically the right to peaceful assembly, are
affected by BP 880. They argue that the Constitution sets no limits
on the right to assembly and therefore BP 880 canto put the prior
requirement of securing a permit. Also, it contravenes the maximum
tolerance policy of BP 880 and violates the Constitution as it causes
a chilling effect on the exercise by the people of the right to
peaceably assemble.
Respondents’ arguments:
BP 880 is not void on its face because BP 880 is content-neutral, as
it has no reference to content of regulated speech; it is narrowly
tailored to serve a significant governmental interest; leaves open
alternative channels for communication of the information.
BP 880 is content-neutral as seen from the text of the law. Sec 5
requires the statement of the public assembly’s time, place and
manner of conduct. Nothing in BP 880 authorizes the denial of a
permit on the basis of a rally’s program content or the statements of
the speakers, except under the constitutional precept of the “clear
and present danger test.”
Adiong v. COMELEC held that BP 880 is content-neutral regulation
as it only regulates the manner od holding public assemblies.

ISSUE/s:
WoN BP 880 is a content-neutral regulation – NO
WoN BP 880 constitutes prior restraint – NO

RULING: The petitions are DISMISSED and the CONSTITUTIONALITY of


BP 880 is SUSTAINED.

RATIO:
The right to peaceably assemble and petition for redress of
grievances is, together with freedom of speech, of expression,
and of the press, is a right that enjoys primacy in the realm of
constitutional protection.
US v. Apurado: It is expected that more or less, disorder will mark the
public assembly of the people to protest against grievances
whether real or imaginary, because on such occasions feeling is
always wrought to a high pitch of excitement.
Primicias v. Fugoso: The right to freedom of speech and to peacefully
assemble and petition the government for redress of grievances are
is not absolute for it may be regulated that it shall not be injurious keeping authorities shall observe during
to the equal enjoyment of others having equal rights. a public assembly or in the dispersal of
Reyes v. Bagatsing: The Court is called upon to protect the exercise the same.
of the cognate rights to free speech and peaceful assembly, BP 880 cannot be condemned as
arising from the denial of a permit. Free speech may be identified unconstitutional; it does not curtail or
with the liberty to discuss publicly and truthfully any matter of unduly restrict freedoms; it merely
public concern without censorship or punishment. There must regulates the use of public places as
be no previous restraint on the communication of views or
subsequent liability unless there be a clear and present
danger of a substantive evil that the State has a right to
prevent.
The provisions of BP 880 codified the ruling in Reyes v. Bagatsing. It
is clear therefore, that BP 880 is not an absolute ban of public
assemblies but a restriction that simply regulates the time,
place and manner of the assemblies.
A fair and impartial reading of BP 880 readily shows that it refers to
all kinds of public assemblies that would use public places.
The reference to “lawful cause” does not make it content-
based because assemblies really have to be for lawful
causes.
The permit can only be denied on the ground of clear and present
danger to public order, public safety, public convenience, public
morals, or public health. This is a recognized exception to the
exercise of the right even under the UDHR and ICCPR.
BP 880 is not overbread. It regulates the exercise of the right to peaceful
assembly and petition only to the extent needed to avoid a clear and
present danger of the substantive evils Congress has the right to
prevent.
There is likewise no prior restraint since the content of the
speech is not relevant to the regulation.
Section 15 of the law provides for an alternative forum through the
creation of freedom parks where no prior permit is needed for
peaceful assembly and petition at any time. Freedom parks
shall be provided in every city and municipality.
Considering that the existence of such freedom parks is an essential part
of the law’s system of regulation of the people’s exercise of their
right to peacefully assemble, the Court is constrained to rule that
after 30 days from the finality of this Decision, no prior permit
may be required for the exercise of such right in any public
park or plaza until that city or municipality shall have complied
with Section 15 of the law.
On calibrated preemptive response (CPR): In view of the
maximum tolerance mandated by BP 880, CPR serves no valid
purpose if it means the same thing as maximum tolerance. What
is to be followed is and should be that mandated by law, which is
the maximum tolerance. Maximum tolerance means the highest
degree of restraint that the military, police and other peace
to the time, place and manner of assemblies.
AGLIPAY v. RUIZ (IYA) Constitution by Ruiz in issuing and selling postage stamps
March 13, 1937 | Laurel, J. | Freedom of Religion commemorative of the 33rd International Eucharistic Congress.
ISSUE/s:
WON Ruiz violated the Constitutional mandate of freedom of religion -
PETITIONER: Gregorio Aglipay NO
RESPONDENTS: Juan Ruiz
RULING: The petition for a writ of prohibition is hereby denied.
SUMMARY: Aglipay seeks to inhibit Ruiz, Director of Posts, form issuing
and selling stamps in commemoration of the 33rd International
Eucharistic Celebration. He argues that the issuance would be violative RATIO:
of the Constitutional mandate of freedom of religion, for the government, It is alleged that this action of the respondent is violative of the provisions
in a way, gives favor to the Roman Catholic Religion. He posits that the of section 13, subsection 3, Article VI, of the Constitution of the
stamps seems to advertise the Catholic faith. Philippines, which provides as follows:
"No public money or property shall ever be appropriated,
The Supreme Court denied the petition of Aglipay ruling that the Director applied, or used, directly or indirectly, for the use, benefit, or
of Posts, in his issuance, was not moved by any sectarian feeling to support of any sect, church, denomination, sectarian
favor a particular church or religious denomination. They merely made institution, or system of religion, or for the use, benefit, or support
use of the international event to earn revenue for the government, what of any priest, preacher, minister, or other religious teacher or
the stamps was promoting was the city of Manila. dignitary as such, except when such priest, preacher, minister, or
dignitary is assigned to the armed forces or to any penal
DOCTRINE: All the officers of the Government, in taking their oath to institution, orphanage, or leprosarium.”
support and defend the Constitution, bind themselves to recognize and The prohibition is a direct corollary of the principle of separation of
respect the constitutional guarantee of religious freedom, with its church and state.
inherent limitations and recognized implications. It should be stated that History of acknowledgement of the principle:
what is guaranteed by our Constitution is religious liberty, not mere Malolos Constitution first recognized in the early stages of the
religious toleration. constitutional development of the Philippines
The same was inserted in the Treaty of Paris between the US
and Spain.
It was reaffirmed int he Philippine Bill of 1902 and in the
Autonomy Act of August 29, 1916.
FACTS: It was finally embodied int he Constitution of the Philippines
Mons. Gregorio Aglipay, Supreme Head of the Philippine Independent All the officers of the Government, in taking their oath to support
Church, seeks the issuance of a writ of prohibition to prevent the and defend the Constitution, bind themselves to recognize and
Director of Posts from issuing and selling postage stamps respect the constitutional guarantee of religious freedom, with
commemorative f the 33rd Eucharistic Congress. its inherent limitations and recognized implications.
Aglipay requested Atty. Sotto to denounce the matter to the President of It should be stated that what is guaranteed by our Constitution is
the Philippines. Despite the protest made by Sotto, Ruiz publicly religious liberty, not mere religious toleration.
announce having sent to the US the designs of the postage stamps In the case at hand, it appears that the Director of Posts issued the
for printing. postage stamps in question under the provisions of Act No.
Several stamps were issued and sold, although majority remains unsold. 4052 of the Philippine Legislature.
The Solicitor General contends that the writ of prohibition is not the It will be seen that the Act appropriates the sum of 60k for the cost of
proper legal remedy. plates and printing of postage stamps with new designs and other
The SC ruled regarding the immediately preceding contentions that the expenses incident thereto, and authorizes the Director of Posts to
writ of prohibition is not confined exclusively to courts or tribunals to dispose of the amount appropriated in the manner indicated and "as
keep them within the limits of their own jurisdiction and to prevent often as may be deemed advantageous to the Government”.
them from encroaching upon the jurisdiction of other tribunals, but Act No. 4052 contemplates no religious purpose in view.
will issue, in appropriate cases, to an officer or person whose acts What it gives the Director of Posts is the discretionary power to
are without or in excess of his authority. determine when the issuance of special postage stamps would be
The more important question raised refers tot he alleged violation of the "advantageous to the Government.”
The issuance of the postage stamps in question by the Director of Posts
was not inspired by any sectarian feeling to favor a particular
church or religious denomination.
The stamps were not issued and sold for the benefit of the Roman
Catholic Church.
The only purpose in issuing and selling the stamps was "to advertise the
Philippines and attract more tourists to this country."
The officials concerned merely took advantage of an event considered of
international importance "to give publicity to the Philippines and its
people"
saint’s feast day. Resolutions Nos. 5 and 6 were submitted to a
GARCES v. ESTENZO (ELIEL) plebiscite and were
May 25, 1981 | Aquino, J. | Freedom of Religion

PETITIONER: Andres Garces, Fr. Sergio Osmena, Nicetas Dagar and


Jesus
Edullantes
RESPONDENTS: Hon. Numeriano Estenzo, Manuel Veloso

SUMMARY: The barangay council passed resolutions 5 and 6 wherein


they revived the tradition of the feast of day of San Vicente Ferrer. They
were to acquire an image of the Saint and build a waiting shed;
furthermore, assign to the chairman of the barangay to take care of the
statute. The controversy arose when Fr. Osmena, did not want to return
the statue to the barangay council and claimed that it was the property of
the church. When the barangay council claimed it in a replevin suit, Fr.
Osmena filed the present petition.

The SC held that the state is the owner of the statue. (1) The absence of
Manago during the resolution sessions does not make them void;
because he is an ex officio member and that there was quorum. (2) The
resolution was not in contravention of the constitution beacause it did not
favor any religion. The funds in order to acquire the image and construct
the waiting sheds were private funds and not public funds. Nevertheless,
since the state owns the statue, it can choose on who can have custody.

DOCTRINE: Not every governmental activity which involves the


expenditure of public funds and which has some religious tint is violative
of the constitutilan provisions regarding separation of church and state,
freedom of worship and banning the use of public money or property.

FACTS:
The Barangay Council of Valencia adopted Resolution No. 5, “reviving
the traditional socio-religious celebration” every fifth day of April “of
the feast day of Senor San Vicente Ferrer, the patron saint of
Valencia”.
It provided for (1) the acquisition of the image of San Vicente Ferrer and
(2) the construction of a waiting shed as the barangay’s projects.
Funds for the two projects would be obtained through the “selling of
tickets and cash donations.”
The Barangay council passed Resolution No. 6 which specified that, in
accordance with the practice in Eastern Leyte, Councilamn Tomas
Cabatingan, the Chairman or hermanyo mayor of the fiesta, would
be the caretaker of the image of San Vicente Ferrer and that the
image would remain in his residence for one year and until the
election of his successor as chairman of the next feast day.
It was further provided in the resolution that the image would be made
available to the Catholic parish church during the celebration of the
duly ratified by the barangay general assembly. The wooden image was purchased in connection with the celebration of the
Funds were raised by means of solicitations and cash donations of the barrio fiesta honoring the patron saint, San Vicente Ferrer, and not for
barangay residents and those of the neighboring places of Valencia. the urpose of favoring any religion nor interfering with religious matters
With those funds, the waiting shed was constructed and the wooden or the religious beliefs of the barrio residents. One of the highlights of the
image of San Vicente Ferrer was acquired in Cebu City by the fiest was
barangay council for four
hundred pesos.
186. A controversy arose after the mass when the parish priest, Fr. Osmena,
refused to return that image to the barangay council on the pretext that it
was the property of the church because church funds were used for its
acquisition.
187. And several days after the fiesta, Fr. Osmena allegedly uttered
defamatory remarks against the barangay captain, Manuel Veloso,
apparently in connection with the disputed image. That incident
provoked Veloso to file against Fr. Osmena in the city court of Ormoc
City a charge for grave oral defamation.
Meanwhile, the barangay council passed Resolution No. 12 appointing
Veloso as its representativein the replevin case. After the barangay
council posed a cash bond of P800, Fr. Osmena turned over the image
to the council.
Fr. Osmena challenges the constitutionality of the resolutions because
(1) the baranagay council was not duly constituted because Manago,
the chairman, was not allowed to participate in its sessions and (2)
that it contravenes the cosntittuional provisions on religion. Hence
this petition.

ISSUE/s:
42. WoN the resolutions are constitutional – YES

RULING: Affirmed lower courts decision dismissing the complaint.

RATIO:
Sec 3 of PD 684, which took effect on April 15, 1975, provides that the
“baranagay youth chairman shall be an ex-officio member of the
baranagay council”, having the same powers and functions as a
baranagay councilman.
In this case, Manago the barangay youth chairman, was notified of the
sessions of the barangay council to be held but he was not able to
attend those sessions because he was woring with a constructions
company.
Manago’s absence from the sessions of the barangay council did
not render the said resolutions void. There was a quorum when
the said resolutions were passed.
The questioned resolutions do not directly or indirectly establish any
religion, nor abridge religious liberty, nor appropriate public money or
property for the benefit of any sect, priest or clergyman. The image
was purchased with private funds, not with tax money. The
construction of a waiting shed is entirely a secular matter.
the mass. Consequently, the image of the patron saint had to be placed in the
church when the mass was celebrated.
222. As noted in the first resolution, the barrio fiesta is a socioreligious affair.
Its celebration is an ingrained tradition in rural communities. The fiesta
relieves the monotony and drudgery of the lives of the masses.
223. The barangay council designated a layman as the custodian of the
wooden image in order to forestall any suspicion that it is favoring the
Catholic church. A more practical reason for that arrangement would
be that the image, if placed in a layman’s custody, could easily be made
available to any family desiring to borrow the image in connection with
prayers and novenas.
There can be no question that the image in question belongs to the
barangay council. Fr. Osmena’s claim that it belongs to his
church is wrong. The barangay council, as owner of the image,
has the right to determine who should have custody thereof.
If it chooses to change its mind and decides to give the iage to the
Catholic church, that action would not violate the constitution
because the image was acquired with private funds and is its private
property.
Not every governmental activity which involves the expenditure of
public funds and which has some religious tint is violative of the
constitutilan provisions regarding separation of church and state,
freedom of worship
and banning the use of public money or property.
227. Abad Santos, J. Concurring: There can be no doubt that the statute was
bought with private funds raised by the barangay council which also decided
who should have custody of it. How the cura parroco got it into his head that
he should have custody of the statue defies logic. It is not, therefore,
surprising to hear statements that religion has no relevance to current
problems.
SCHOOL DISTRICT v. SCHEMPP (JP) parent or guardian.
June 17, 1963 | Clark, J. | Freedom of Religion The Schempp family is of the Unitarian faith and are members of the
Unitarian Church in Germantown, Philadelphia, Pennsylvania.
PETITIONER: School District of Abington Township They sought to enjoin the petitioner school district, wherein the Schempp
RESPONDENTS: Schempp family children attend school, and its officers and the Superintendent of
Public Instruction of the Commonwealth from continuing to conduct
SUMMARY: Case No. 142: The Schempp family os of the Unitarian faith.
such readings and recitation of the Lord's Prayer in the public
They are claiming that their children’s constitutional rights are violated
with the requirement of the Commonwealth of Pennsylvania that public schools of the district pursuant to the statute.
schools require their students to recite at least 10 verses of the Holy Bible On each school day at the petitioner Abington Senior High School
and the Lord’s prayer daily. The trial court declared the law between 8:15 and 8:30 a.m., while the pupils are attending their
unconstitutional. Case No. 119: The Murray family are atheists. They are home rooms or advisory sections, opening exercises are conducted
likewise claiming that the requirement in the Annotated Code of Maryland pursuant to the statute.
of the same practice violates their freedom of religion and separation of The exercises are broadcast into each room in the school building
the Church and State. The trial court sustained the validity of the law.
through an intercommunications system.
The SC declared both laws unconstitutional. The laws failed to pass the Selected students read 10 verses of the Holy Bible, broadcast to each
Establishment clause as they were never proven to be sectarian (non- room in the building.
religious related). Since the Holy Bible cannot be denied to be a religious This is followed by the recitation of the Lord's Prayer. The students in the
instrument, the reading of which, without interpretation is no doubt various classrooms are asked to stand and join in repeating the
religious in favor of the Christian religion. This is prohibited under the First prayer in unison.
and Fourth Amendments as states should not pass laws which are of a The student reading the verses from the Bible may select the passages
religious character.
and read from any version he chooses, although the only copies
DOCTRINE: The (Establishment Clause) test may be stated as follows: furnished by the school are the King James version, copies of which
what are the purpose and the primary effect of the enactment? If either were circulated to each teacher by the school district.
is the advancement or inhibition of religion, then the enactment There are no prefatory statements, no questions asked or solicited, no
exceeds the scope of legislative power as circumscribed by the comments or explanations made, and no interpretations given at or
Constitution. That is to say that, to withstand the strictures of the during the exercises.
Establishment Clause, there must be a secular legislative purpose and The students and parents are advised that the student may absent himself
from the classroom or, should he elect to remain, not participate in the
a primary effect that neither advances nor inhibits religion
exercises.
As for the Free Exercise Clause, its purpose is to secure religious At the first trial, Edward Schempp (father) and the children testified
liberty in the individual by prohibiting any invasions thereof by civil as to specific religious doctrines purveyed by a literal reading of
authority. Hence, it is necessary in a free exercise case (Free Exercise the Bible "which were contrary to the religious beliefs which
Test) for one to show the coercive effect of the enactment as it they held, and to their familial teaching."
operates against him in the practice of his religion. Expert testimony was introduced. Dr. Solomon Grayzel testified that there
were marked differences between the Jewish Holy Scriptures and the
Christian Holy Bible, the most obvious of which was the absence of
FACTS: the New Testament in the Jewish Holy Scriptures.
On Case No. 142: The Commonwealth of Pennsylvania requires Dr. Grayzel testified that portions of the New Testament were offensive to
that: At least ten verses from the Holy Bible shall be Jewish tradition, and that, from the standpoint of Jewish faith,
read, without comment, at the opening of each public the concept of Jesus Christ as the Son of God was "practically
school on each school day. Any child shall be excused blasphemous."
from such Bible reading, or attending such Bible Dr. Grayzel gave as his expert opinion that such material from the New
reading, upon the written request of his
Testament could be explained to Jewish children in such a way as to that official support of the State or Federal Government would
do no harm to them. But if portions of the New Testament were be placed behind the tenets of one
read without explanation, they could be, and, in his specific
experience with children, Dr. Grayzel observed, had been,
psychologically harmful to the child.
District Court for the Eastern District of Pennsylvania held that the statute
is violative of the Establishment Clause of the First Amendment.
The trial court, in striking down the practices and the statute requiring
them, said that the children's attendance at Abington Senior High
School is compulsory, and that the practice of reading 10 verses
from the Bible is also compelled by law.
On Case No. 119: the Board of School Commissioners of Baltimore City
adopted a rule pursuant to Art. 77 of the Annotated Code of
Maryland.
The rule provided for the holding of opening exercises in the schools of
the city, consisting primarily of the "reading, without comment, of a
chapter in the Holy Bible and/or the use of the Lord's Prayer."
The petitioners, Mrs. Madalyn Murray and her son, William J. Murray
III, are both professed atheists.
It was the practice under the rule to have a reading on each school
morning from the King James version of the Bible.
At petitioners' insistence, the rule was amended to permit children to be
excused from the exercise on request of the parent, and that William
had been excused pursuant thereto.
Nevertheless the rule as amended was alleged to be in violation of
the petitioners' rights "to freedom of religion under the First and
Fourteenth Amendments" and in violation of the principle of
separation between church and state.
The trial court sustained the validity of the statue. The Maryland CA
affirmed. Hence this appeal.

ISSUE/s:
WoN the statutes requiring public schools to require the Bible be read or that
the Lord’s Prayer be recited are violative of the Establishment Clause –
YES

RULING: Judgment is affirmed for No. 142 and is reversed for No. 119. The
cause is remanded to the Maryland Court of Appeals.

RATIO:
The wholesome "neutrality" of which this Court's cases speak thus stems
from a recognition of the teachings of history that powerful sects or
groups might bring about a fusion of governmental and religious
functions or a concert or dependency of one upon the other to the end
or of all orthodoxies. This what the Establishment Clause prohibits.
A further reason for neutrality is found in the Free Exercise Clause, which
recognizes the value of religious training, teaching and observance and,
more particularly, the right of every person to freely choose his own course
with reference thereto, free of any compulsion from the state. This what the
Free Exercise Clause guarantees.
The two clauses may overlap.
The (Establishment Clause) test may be stated as follows: what are the
purpose and the primary effect of the enactment? If either is the
advancement or inhibition of religion, then the enactment exceeds the
scope of legislative power as circumscribed by the Constitution. That is
to say that, to withstand the strictures of the Establishment Clause,
there must be a secular legislative purpose and a primary effect that
neither advances nor inhibits religion
As for the Free Exercise Clause, its purpose is to secure religious liberty in
the individual by prohibiting any invasions thereof by civil authority.
Hence, it is necessary in a free exercise case (Free Exercise Test) for
one to show the coercive effect of the enactment as it operates against
him in the practice of his religion.
The distinction between the two clauses is apparent -- a violation of the Free
Exercise Clause is predicated on coercion, while the Establishment Clause
violation need not be so attended.
Applying the Establishment Clause principles to the cases at bar, the SC finds
that the States are requiring the selection and reading at the opening of the
school day of verses from the Holy Bible and the recitation of the Lord's
Prayer by the students in unison.
The trial court in No. 142 has found that such an opening exercise is a
religious ceremony, and was intended by the State to be so. We agree
with the trial court's finding as to the religious character of the
exercises. Given that finding, the exercises and the law requiring them
are in violation of the Establishment Clause.
There is no such specific finding as to the religious character of the exercises in
No. 119, and the State contends (as does the State in No. 142) that the
program is an effort to extend its benefits to all public school children
without regard to their religious belief. Included within its secular
purposes, it says, are the promotion of moral values, the contradiction
to the materialistic trends of our times, the perpetuation of our
institutions and the teaching of literature.
However, the religious character of the exercise was admitted by the State.
But even if its purpose is not strictly religious, it is sought to be
accomplished through readings, without comment, from the Bible.
Surely the place of the Bible as an instrument of religion cannot sustained on constitutional grounds as necessary
be gainsaid, and the State's recognition of the pervading religious to secure to the members of the Armed Forces and
character of the ceremony is evident from the rule's specific prisoners those rights of worship guaranteed under
permission of the alternative use of the Catholic Douay version, as the Free
well as the recent amendment permitting nonattendance at the
exercises.
The conclusion follows that, in both cases, the laws require religious
exercises, and such exercises are being conducted in direct violation
of the rights of the appellees and petitioners.
Nor are these required exercises mitigated by the fact that individual
students may absent themselves upon parental request, for that fact
furnishes no defense to a claim of unconstitutionality under the
Establishment Clause.
Nothing we have said here indicates that such study of the Bible or
of religion, when presented objectively as part of a secular
program of education, may not be effected consistently with the
First Amendment.
But the exercises here do not fall into those categories. They are
religious exercises, required by the States in violation of the
command of the First Amendment that the Government
maintain strict neutrality, neither aiding nor opposing religion.
Brennan, Concurring: The States may not employ religious means to
reach a secular goal unless secular means are wholly unavailing.
The invalidation of the exercises at bar permits this Court no alternative
but to declare unconstitutional every vestige, however slight, of
cooperation or accommodation between religion and government. I
cannot accept that contention.
While it is not, of course, appropriate for this Court to decide questions
not presently before it, I venture to suggest that religious exercises in
the public schools present a unique problem. For not every
involvement of religion in public life violates the Establishment
Clause. Our decision in these cases does not clearly forecast
anything about the constitutionality of other types of interdependence
between religious and other public institutions.
There are certain practices, conceivably violative of the Establishment
Clause, the striking down of which might seriously interfere with
certain religious liberties also protected by the First Amendment.
Provisions for churches and chaplains at military establishments for
those in the armed services may afford one such example.
The like provision by state and federal governments for chaplains in penal
institutions may afford another example. It is argued that such provisions
may be assumed to contravene the Establishment Clause, yet be
Exercise Clause.
BOARD OF EDUCATION v. ALLEN (HENRY) assailed law was not in violation of either the State or the Federal
June 10, 1968 | White, J. | Freedom of Religion Constitution. NY CA ruled that the law’s purpose was to benefit all
school children, regardless of the type of school they attended, and
PETITIONER: Board of Education of Central School District No. 1 that only textbooks approved by public school authorities could be
RESPONDENTS: James Allen loaned.
Sec 701 was ruled to be neutral with respect to religion, merely making
SUMMARY: The Education Law of New York, Section 701 requires public available secular textbooks at the request of the individual student,
school authorities to lend textbooks free of charge to all students in grade and asking no question about what school he attends.
7-12, including those in private schools. Petitioners filed a suit in NY NY CA concluded that Sec 701 is not a law which “establishes a religion
Court against respondent, and claimed that Sec 701 is a violation of the or constitutes the use of public funds to aid religious schools”, hence
free exercise of religion. They claimed that if respondents failed to lend a petition laid down in the SC.
books to parochial school students, Allen would remove them from office.
ISSUE/s:
Trial Court held that the law was unconstitutional, which was reversed by WoN the law is prohibiting the right to the free exercise of religion – NO
the appellate court. Such ruling was affirmed by the NY CA, and further
RULING: SC affirmed the NY CA ruling, and said that Sec 701 does not
by the SC. It is rueld that Sec 701 is valid and not a violation of the
constitutional right to freedom of religion, for failure to prove and violate the Constitutional right to free exercise of religion.
substantiate such claims.
RATIO:
DOCTRINE: it cannot be assumed that school authorities are unable to With reference to Everson v. Board of Education, where the assailed law
distinguish between secular and religious books, or that they will not si said to merely make available to all children the benefits of a
honestly discharge their duties to approve only secular books. general program to lend school books free of charge, and the
financial benefit is to parents and children, not to schools.
There is no evidence that the religious books have been loaned, and
FACTS: it cannot be assumed that school authorities are unable to
Education Law of NY, Sec 701 authorized public school boards to distinguish between secular and religious books, or that they
designate textbooks for use in the public schools to purchase such will not honestly discharge their duties to approve only secular
books with public funds, and to rent or sell the books to public school books.
students. This was amended, insead of renting or selling, books are Parochial schools, in addition to their sectarian function, perform the task
actually lent without charge (open to public/private school students). of secular education, and, on the basis of this meager record, the
Petitioner brought suit in the NY courts against respondent Allen, and court cannot agree with the petitioners that all that is taught in a
alleged that Sec 701 violated both the State and Federal sectarian school is religious or that the intertwining of secular and
Constitutions; that if respondents, in reliance on their interpretation of religious training is such that secular textbooks furnished to students
the Constitution, failed to lend books to parochial school students are, in fact, instrumental in teaching religion.
within their counties, respondent Allen would remove petitioners from In the absence of specific evidence, and based solely on judicial notice, it
office, and that, to prevent this, petitioenrs were complying with the cannot be concluded that the statute results in unconstitutional state
law and submitting to their constitutents a school budget including involvement with religious instruction.
funds for books to be lent to parochial school pupils. Since petitioners failed to show how the law coerces them in any way in
Petitioners sought a declaration that Sec 701 was invalid, and moved for the practice of religion, there is no violation fo the Free Exercise (of
an order barring respondent Allen from removing appellants from religion) clause.
office for failing to comply with it, and another order restraining him A test was used in previous jurisprudence, to distinguish forbidden
from appropriating state funds to school districts for the purchase of involvement of the state with religion and those which is permitted:
textbooks to be lent to parochial school students. What is the purpose and the primary effect of the enactment? If
Trial court held the law unconstitutional under the First and Fourteenth either is the advancement or inhibition of religion, then the
aendments and entered judgment in favor of petitioners. enactment exceeds the scope of legislative power as
Appellate Division reversed and dismissed the case on the ground that circumscribed by the Constitution.
petitioners had no legal standing to attack the validity of the statute. In the present case, the express purpose of Sec 701 was to further the
On appeal, NY CA concluded that petitioners had standing, but that the educational opportunities available to the young. It merely makes
available to children the benefits of a general program to lend school
books free of
charged. faith. Parochial school textbook may contain many more seeds of
Harlan, concurring: the attitude of the government towards religion creed and dogma than a prayer.
must be neutral. Neutrality, however, is a coat of many colors; it Fortas, dissenting: The program made in compliance to the Sec 701 is
requires that government neither engage in nor compel religious a
practices, that it effect no favoritism among sects or between religion
and nonreligion, and that it work deterrence of no religious belief.
Sec 701 does not employ religion as its standard for action or
inaction.
Black, dissenting: Neither a state nor the Federal Government can set
up a church, nor pass laws which aid one religion, aid all religions, or
prefer one religion over another. Neither can force nor influence a
person to go or to remain away from church against his will or force
him to profess a belief or disbelief in any religion. No person can be
punished for entertaining or professing religious beliefs or disbeliefs,
for such attendance or non-attendance.
Although Sec 701 does not formally adopt or establish a state religion, it
akes a great stride in that direction. With reference to Douglas’
dissent, upholding a State’s power to pay bus or streetcar fares for
school children cannot provide support for the validity of a state law
using tax raised funds to buy school books for a religious school.
First Amendment’s bar to establishment of religion must preclude a
State from using funds levied from all its citizens to purchase books
for use by sectarian schools, which, although secular, realistically will
in some way inevitably tend to propagate religious views of the
favored sec.
Tax raised funds cannot constitutionally be used to support religious schools,
buy their school books, erect their buildings, pay their teachers, or pay
any other of their maintenance expenses, even to the extent of one
penny. The First amendment’s prohibition against governmental
establishment of religion as written on the assumption that state aid to
religion and religious schools generates discord, harmony, hatred, and
strife among our people, and that any government that supplies such
aids, is a tyranny.
Douglas, dissenting: Sec 701 on its face empowers each parochial
school to determine for itself which textbooks will be legible for loans
to its students, for the act providest that the only text which the state
may provide is “a book which a pupil is rewuired to use as a text for a
semester or more in a particular class in the school he legally
attends” (mentioned in my footnote). This initial selection made by
parochial schools are made, in the case of Roman Catholic Schools,
by priests or nuns.
Basically, Sec 701 provides that the parochial school will ask for the
books that it wants. If the board of education supinely submits by
approving and supplying the sectarian or sectarian-oriented
textbooks, the struggle to keep church and state separate will be
lost.
A textbook goes to the very heart of education in a parochial school. It is
the chief instrument for propagating a particular religious creed or
specific program to use state funds to buy books prescribed by sectarian
schools, which in NY, are primarily Catholic, Jewish, and Lutheran
sponsored schools. This program is not one in which all children are
treated alike, regardless of where they go to school.
This program in its unconstitutional features, is hand-tailored to satisfu needs of
sectarian schools. Children attending such schools are given special books.
The case is not within the principle of Everson v. Board of Education. The
present statute does not call for extending to children attending sectarian
schools the same service or facility extended to children in public
schools. It calls for furnishing special, separate, and particular books,
specially, separately, and particularly chosen by religious sects or their
representatives for use in their sectarian schools. This feature makes it
impossible to reach any conclusion other than this statute is
unconstitutional for the use of public funds to support an
establishment of religion.
LEMON v. KURTZMAN (DANNAH) The Act authorizes state officials to supplement the salaries of
June 27, 2002 | Rehnquist, C.J. | Establishment/Free Exercise Clause teachers of secular subjects in nonpublic elementary
schools by paying directly to a teacher an amount not in
PETITIONER: Alton Lemon et al. excess of 15% of current annual salary
In order to be eligible, the recipient must teach in a nonpublic school
RESPONDENTS: David Kurtzman et al. which

SUMMARY: Both states of Rhode Island and Pennsylvania enacted


statutes providing financial aid to teachers and schools regarding secular
subjects. However, it was found that majority of those who benefitted
from the programs are teachers of religious schools. The petitioners in
this case sought to have the statutes struck down for being
unconstitutional as these are violative of the Establishment and Free
Exercise Clauses.

In the case of Rhode Island, the District Court declared it unconstitutional,


which the SC affirmed. This is because the statute fostered excessive
entanglement between government and religion. The substantial religious
character of said schools gives rise to entangling church-state
relationships which the Religion Clauses sought to avoid. Moreover, the
government would have to closely surveil the teachers, as conditions are
set forth regarding their eligibility in the program. This, the Court saw as
too much of proximity between the State and religion, as most schools
involved are Catholic Schools.

In the case of Pennsylvania, the District Court dismissed the case.


Lemon, a taxpayer and whose child goes to a public school in the state,
appealed the decision. The Court overturned, stating that a direct money
subsidy would be pregnant with involvement of the state in religious
institutions.

DOCTRINE: A given law might not establish a state religion, but


nevertheless be one "respecting" that end in the sense of being a step
that could lead to such establishment, and hence offend the First
Amendment.

FACTS:
These are two consolidated cases regarding Pennsylvania and Rhode
Island statutes which provide state aid to church-related elementary
and secondary schools. These are challenged as violative of the
Establishment and Free Exercise Clauses (also Due Process pero
not relevant)
The Rhode Island Statute:
Was enacted in 1969 because the quality of education available in
nonpublic elementary schools has been jeopardized by the rapidly
rising salaries needed to attract competent and dedicated teachers
the average per-pupil expenditure on secular education is less than the
average in the State’s public schools
Must be examined in order to assess how much of the expenditure is attributable
to secular education and to religious activity
Also requires that teachers eligible must teach only those subjects that are
offered in the State’s public schools, must use only teaching materials which
are used in public schools, and must agree in writing not to teach a course in
religion
Appellees, citizens and taxpayers of Rhode Island, brought the suit to have the
statute declared unconstitutional. Appellants are state officials charged with
administration of the Act, teachers eligible for salary supplements, and
parents of children in church-related schools
Statistics show that Rhode Island’s nonpublic elementary schools
accommodated 25% of State’s pupils, and about 05% of these attended
schools affiliated with the Roman catholic Church.
Around 250 teachers applied for benefits under the Act and all of them are
employed by Roman Catholic schools.
The District Court found that the Act violated the Establishment clause, as it
fostered “excessive entanglement” between government and religion.
The Pennsylvania Statute:
Adopted a program that has some of the features of the Rhode Island program.
Passed in 1968 in response to a crisis that the Pennsylvania Legislature
found existed in the State’s nonpublic schools due to rapidly rising costs.
Authorizes Superintendent of Public Instruction to “purchase” specified “secular
educational services” from nunpublic schools
Under the contracts, the State directly reimburses nonpublic schools solely for
their actual expenditures for teachers’ salaries, textbooks and intstructional
materials
A school seeking reimbursement must maintain accounting procedures that
identify the separate the separate cose of the secular educational service,
which are subject to state audit
This is financed by a portion of the state tax on cigarettes (horse racing tax
before). Reimbursement is limited to courses presented in the curricula of
public schools
Limited solely to courses in the ff secular subjects: math, foreign language,
science and physical education
Instructional materials must be approved by the Superintendent of Public
Intruction
Prohibits reimbursement for any cours that contains any subject matter
expressing religious teaching
Around $5M annually has been expended under the Act, and contracts have
been entered with some 1,181 nonpublic schools. More than 96% of the
pupils attend church-related schools, and most are affiliated with the The cumulative impact of the entire relationship arising under these statutes
Roman Catholic church involves excessive entanglement between government and religion.
Appellants brought the action to the District Court. They are associations Total separation is not possible in an absolute sense. Some relationship
of persons declaring belief in the separation of church and state, and
are citizens and taxpayers.
Appellant Lemon is a parent of a child attending public school in
Pennsylvania. He also alleges he purchased a race track ticket, and
paid the specific tax that supports this Act.
The District Court granted the States’ motion to dismiss the complaint,
and held that the Act violated neither the Establishment nor Free
Exercise clauses. The SC reverses.
The language of the Religion Clauses of the First Amendment
commanded that there should be "no law respecting an
establishment of religion." A law may be one "respecting" the
forbidden objective while falling short of its total realization.
A law "respecting" the proscribed result, that is, the establishment of
religion, is not always easily identifiable as one violative of the
Clause. A given law might not establish a state religion, but
nevertheless be one "respecting" that end in the sense of being a
step that could lead to such establishment, and hence offend the
First Amendment.
The three main evils against which the Establishment Clause was
intended to afford protection: sponsorship, financial support, and
active involvement of the sovereign in religious activity. The three
tests:
The statute must have a secular legislative purpose
Its principal or primary effect must be one that neither advances nor
inhibits religion
The statute must not foster “an excessive government entanglement
with religion”
It is evident that the statutes afford no basis for a conclusion that the
intent was to advance religion. It is actually clear that they are
intended to enhance the quality of the secular education in all
schools covered by the compulsory attendance laws.
However, the statutes have also recognized that church-related
elementary and secondary schools have a significant religious
mission, and that a substantial portion of their activities is religiously
oriented.
They have therefore sought to create statutory restrictions designed to
guarantee the separation between secular and religious
educational functions, and to ensure that State financial aid
supports only the former.
between government ang religious organizations is inevitable.. In Allen, the Court refused to make assumptions about the religious
In order to determine whether the government entanglement with content of the textbooks that the State would be asked to provide.
religion is excessive, the character and purposes of the However, teachers have a substantially different ideological
institutions that are benefited, the nature of the aid that the State character from books. A textbook’s content is ascertainable, but a
provides, and the resulting relationship between the government teacher’s handling of a subject is not.
and the religious authority must be examined.
Here the Court finds that both statutes foster an impermissible
degree of entanglement.

ISSUE/s:
WoN these statutes provide entanglement between government and
religion
– YES

RULING: Judgment of Rhode Island case affirmed. Judgment of


Pennsylvania case reversed, case is remanded for further proceedings.

RATIO:
The Rhode Island Program:
The Disctrict Court made findings on the grave potential for
excessive entanglement that inheres in the religious character
and purpose of the Roman Catholic elementary schools of
Rhode Island, to date the sole beneficiaries of the statute
The church schools involved are located close to parish churches.
There is convenient access for religious exercises. The buildings
contain identifying religious symbols. There are religiously oriented
extracurricular activities. Approximately 2/3 of the teachers in these
schools are nuns.
It was concluded that the parochial schools constituted an integral
part of the religious mission of the Catholic Church
The substantial religious character of said schools gives rise to
entangling church-state relationships which the Religion
Clauses sought to avoid
Although the religious values do not necessarily intrude into the
content of secular subjects, the activites of these schools led
the legislature to provide for careful governmental controls and
surveillance by state authorities in order to ensure that state
aid supports only secular education.
The dangers and entanglements are enhance by the particular form
of aid that this statute provides
The decisions in Everson and Allen permitted the States to provide
church-related schools with secular materials. Bus
transportation, lunches etc were not though to offend the
Establishment Clause.
The dangers are present to a substantial degree. Religious authority On political division: The division along religious lines was one of the
necessarily pervades the shool system. evils at which the First Amendment aimed, and in these programs,
The Handbook used advises teachers to stimulate interest in religious where successive and probably permanent annual appropriations
vocations. The teachers testified, however, that they did not inject that benefit relatively few religious groups are involved, political
religion into their secular classes. However, the teacher is employed fragmentation and divisiveness on religious lines are likely to be
by a religious organization. intensified.
A dedicated religious person, teaching in a school affiliated with his or her Moreover, unlike the tax exemption upheld in Walz v. Tax Commission,
faith and operated to inculcate its tenets, will inevitably experience these new programs have self- perpetuating and self-expanding
great difficulty in remaining religiously neutral. propensities which provide a warning signal against entanglement
Another area of entanglement is seen because the statutes excludes between government and religion.
teachers employed by nonpublic schools whose average per-pupil Brennan, Statement: What the Framers meant to foreclose, and what
expenditures on secular education equal or exceed the figures for our decisions under the Establishment Clause have forbidden, are
public schools. those involvements of religious with secular institutions which (a)
The program requires the government to examine the school’s record. serve the essentially religious activities of religious institutions; (b)
This kind of state inspection and evaluation of the religious content of employ the organs of government for essentially religious purposes;
a religious organization is fraught with the sort of entanglement that or (c) use essentially religious means to serve governmental ends,
the Constitution forbids. It is a relationship pregnant with danges of where secular means would suffice.
excessive government direction of church schools, and of churches. One of the teacchers stopped praying with his classes because the
The Pennsylvania Program: Rhode Island statute imposed that teachers should not inject
Provides state aid to church-related schools for teachers’ salaries. religious teaching into their secular courses. The picture of state
The church-related schools are controlled by religious inspectors prowling the halls of parochial schools and auditing
organizations, have the purpose of propagating and classroom instruction surely raises more than an imagined specter of
promoting a particular religious faith, and conduct their governmental "secularization of a creed."
operations to fulfill that purpose. Where a sectarian institution accepts state financial aid, it becomes obligated
Reimbursement is not only limited to courses offered in the not to discriminate in admission policies and faculty selection. Applying
public schools and materials approved by state officials, but these standards to parochial schools might well restrict their ability to
the statute excludes "any subject matter expressing religious discriminate in admissions policies and in hiring and firing teachers. At
teaching, or the morals or forms of worship of any sect.” some point, the school becomes "public" for more purposes than the
Has the defect of providing state financial aid directly to the Church could wish. At that point, the Church may justifiably feel that its
church-related school. victory on the Establishment Clause has meant abandonment of the
In Everson and Allen, the Court was careful to point out that the state aid Free Exercise Clause.
was provided to students and parents—not to the church-related Douglas, Concurring: There is an entanglement because the
schools. Here, the Court warned of the dangers of direct payments to surveillance of the States needed to police grants involved puts a
religious orgs. public investigator into every classroom and entails a pervasive
Obviously a direct money subsidy would be a relationship pregnant with monitoring of these church agencies by the secular authorities.
involvement. Yet if that surveillance or supervision does not occur, the zeal of religious
The government cash grants in the statute provide no basis for predicting proselytizers promises to carry the day and make a shambles of the
that measures of surveillance and controls will not follow. Establishment Clause. Moreover, when taxpayers of many faiths are
In particular, the government's post-audit power to inspect and evaluate a required to contribute money for the propagation of one faith, the
church-related school's financial records and to determine which Free Exercise Clause is infringed.
expenditures are religious and which are secular creates an intimate White, Concurring and Dissenting: While the decision of the Court is
and continuing relationship between church and state. legitimate, it is surely quite wrong in overturning the Pennsylvania and
Rhode Island statutes on the ground that they amount to an establishment of
religion
forbidden by the First Amendment.
Dual role of parochial schools: perform both religios and secular functions
In Tilton: the Court decides that the Federal Government may finance
the separate function of secular education carried on in a parochial
setting. It reaches this result although sectarian institutions
undeniably will obtain substantial benefit from federal aid; without
federal funding to provide adequate facilities for secular education,
the student bodies of those institutions might remain stationary, or
even decrease in size, and the institutions might ultimately have to
close their doors.
It is enough that the States and the Federal Government are financing a
separable secular function of overriding importance in order to
sustain the legislation here challenged. That religion and private
interests other than education may substantially benefit does not
convert these laws into impermissible establishments of religion.
The goal of the statutes is not to aid religion but to implement secular goals.
TILTON v. RICHARDSON (ARIELLE) Appellants are citizens and taxpayers and residents of Connecticut and
June 28, 1971| Harlan, CJ. | Freedom of Religion brought the suit against the officials who administer the Act. Four
church-
PETITIONER: Tilton
RESPONDENTS: Richardson

SUMMARY: The Higher Education Facilities Act was passed in 1963 to


expbut and college and university facilities to meet the rising the number
of young people demanding higher education. The Act authorizes grants
to institutions but it excludes “any facility used or to be used for sectarian
instruction or as a place for religious worship.” Four church-related
universities received grants under the Act and were used for five projects
in said schools. Appellants, as taxpayers, brought the suit against the
officials who administer the Act. Appellants tried to show that the recipient
universities were “sectarian” and complained that the taxes they pay
partly finance such grants. The District Court ruled that Title I authorized
grants to church-related universities, finding that it had neither the
purpose nor the effect of promoting religion.

On appeal, the SC held that Congress intended the Act to include all colleges
and universities regardless of any affiliation with or sponsorship by a religious
body.

DOCTRINE: The Act prohibits use of the facilities for religious purposes,
but the Act makes no reference to religious affiliation or non-affiliation.
Under these circumstances, “institutions of higher education” must be
taken to include church-related colleges and universities.

FACTS:
The Higher Education Facilities Act was passed in 1963 in response to a
strong nationwide demand for the expansion of college and
university facilities to meet the rising numbere of young people
demanding higher education.
The Act authorizes grants and loans to institutions of higher education,
but one of the provisions of the Act expressly excludes “any facility
used or to be used for sectarian instruction or as a place for religious
worship,” or any facility which is used or to be used primarily in
connection with any part of the program of a school or department of
divinity.
The United States Commissioner of Education advises colleges and
universities applying for funds that under the said Act, no part of the
project may be used for sectarian instruction, religious worship, or
the programs of a divinity school. The United States is entitled to
recover an amount equal bto the proportion of the facility’s equal
value if the recipient violates the statutory conditions.
related universities in Connecticut receiving grants under Title I a legitimate secular objective entirely appropriate for governmental
where funds were used for five projects at these four institutions: 1) a action.
library building at Sacred Heart University; 2) a music, drama, and The argument that every form of financial aid to church-sponsored
arts building at Annhurst College; 3) a science building at Fairfield
University; 4) a library building at Fairfield University; 5) a language
laboratory at Albertus Magnus College. Appellants tried to show that
the four recipient institutions were “sectarian” by introducing
evidence of their relations with religious authorities, the content of
their curricula.
The colleges testified that they had fully complied with the conditions and
that their religious affiliation in no way interfered with the
performance of their secular educational functions.
The District Court ruled that Title I authorized grants to church-related
universities and sustained the constitutionality of the Act, finding that
it had neither the purpose nor the effect of promoting religion.

ISSUE/s: WoN the Act inhibits the free exercise of religion – NO

RULING: We conclude that the Act does not violate the Religion Clauses
except that part of 754 (b) (2) providing a 20-year limitation. We remand to
the District Court with directions to enter a judgment consistent with this
opinion.

RATIO:
The Court is satisfied that Congress intended the Act to include all
colleges and universities regardless of any affiliation with or
sponsorship by a religious body. The Act prohibits use of the facilities
for religious purposes, but the Act makes no reference to religious
affiliation or non-affiliation. Under these circumstances, “institutions of
higher education” must be taken to include church-related colleges and
universities.
In other decisions, the Court treated the three main concerns the
Establishment Clause sought to protect: sponsorship, financial
support, and active involvement of the sovereign in religious activity.
Every analysis must begin with the acknowledgment that there is no single
constitutional caliper that can be used to measure the precise degree to
which these three factors are present or absent. Instead, the analysis
must begin with the consideration of the cumulative criteria developed
over many years.
The questions to be considered in analyzing such acts are: 1) Does the
Act reflect a secular legislative purpose? 2) Is the primary effect of
the Act to advance or inhibit religion? 3) Does the administration of
the Act foster an excessive government entanglement with religion?
4) Does the implementation of the Act inhibit the free exercise of
religion?
The stated legislative purpose stated by the Congress provides that “the
security and welfare of the US require that this and future
generations of American youth be assured ample opportunity for the
fullest development of their intellectual capacities…” This expresses
activity violates the Religion Clauses was rejected long ago. In the Congress did not base the 20-year provision on any contrary
case at bar, the Act is challenged on the ground that its primary conclusion. If, at the end of 20 years, the building is, for example,
effect is to aid the religious purposes of church-related colleges. converted into a chapel or used to promote religious interests,
Construction grants aid these institutions yet all of these governmental the original federal grant will in part have the effect of advancing
assistance have been upheld. religion.
The Act was drafted to ensure that the subsidized facilities would be The restrictive obligations of a recipient institution cannot expire while the
devoted to the secular and not the religious function of the recipient
institutions. It authorizes grants only for academic facilities and
expressly prohibits their use for religious instruction, training, or
worship.
The records fully support the findings of the District Court that none of
the four church-related institutions has violated the conditions. The
institutions presented evidence that there had been no religious
services or worship in the federally financed facilities, no religious
symbols in or on them, and that they have bbeen used solely for
non-religious purposes.
The appellants rely on the argument that government may not subsidize
any activities of an institution of higher learning that in some of its
programs teaches religious doctrines. This argument is based on
Everson where the majority stated that the Establishment Clause
barred any “tax levied to support any religious institutions whatever
form they may adopt to teach or practice religion”
Two of the five federally financed buildings involved in the case are
libraries. The District Court found that no classes had been
conducted in either of these facilities.
There is no evidence that religion seeps into the use of any of these
facilities. The parties stipulated that courses at these institutions
are taught according to the academic requirements intrinsic to the
subject matter. Although appellants introduced several documents that
stated certain religious restrictions on what could be taught, other
evidence showed that the restrictions were not in fact enforced and that
the schools were characterized by an atmosphere of academic freedom
rather than religious indoctrination.
Appellants also seek to shift the attention to a “composite profile,” where
such “composite” institution imposes religious restrictions on admissions,
requires attendance at religious activities, compels obedience to the
docteines of the faith, etc. Some colleges fall within this category, but the
Court cannot strike down an Act of Congress on the basis of a
hypothetical “profile.”
If a recipient institution violates any of the statutory restrictions on the use of
a federally financed facility, the Act permits the Government to recover
an amount equal to the proportion of the facility’s present value. This
remedy is available to the Government only if the conditions are violated
“within twenty years after completion of construction.” Under the
provision, therefore, a recipient institution’s obligation not to use the
facility for sectarian instruction would appear to expire at the end of 20
years.
building has substantial value. The Court has found nothing in the stattue The invalidation of the 20-year limitation clause cannot cure the
intimating that Congress considered the 20-year provision essential to infirmities of the statute as a whole. The Government is giving
the statute as a whole. There is no basis for assuming that the Act would religious
have failed of passage without the provision.
In DiCenso, the District Court found that the parochial schools in Rhode Island
were “an integral part of the religious mission of the Catholic Church.” There,
the record supported the conclusion that the inculcation of religious values
was a substantial if not the dominant purpose of the institutions.
In the case at bar, appellants’ complaint contains similar allegations. Although the
District Court made no findings with respect to the religious character of the
four institutions of higher learing, the Court is not required to accept the
allegations as true under these circumstances, particularly when the
appellants do not contend that these four institutions are “sectarian.”
All four schools are governed by Catholic religious organizations, and the
faculties and students at each are predominantly Catholic. Neverthelss,
evidence shows that non-Catholics were admitted as students and
faculty. Not one of the institutions requires its students to attend religious
services, although they require their students to take theology courses.
The parties stipulated that the courses covered a range of human religious
experiences and are not limited to courses about the Roman Catholic
religion. The four schools subscribe to a well-established set of principles
of academic freedom.
In the Lemon and DiCenso cases, the state programs subsidized teachers. Since
teachers are not necessarily religiously neutral, greater governmental
surveillance would be required to guarantee that state salary would not
subsidize religious instruction. In the case at bar, the Government provides
facilities that are themselves religiously neutral. The risks of Government aid
to religion and the corresponding need for surveillance are therefore
reduced.
The government entanglements with religion are reduced by the
circumstance that the Government aid here is a one-time, single
purpose construction grant. There are no continuing financial relationships
or dependencies. The relationship therefore has less potential for realizing
the substantive evils against which the Religion Clauses were intended to
protect.
Appellants claim that the Free Exercise Clause is violated because they
were compelled to pay taxes, the proceeds of which in part finance
grants under the Act. However, they are unable to identify any
coercion directed at the practice or exercise of their religious
beliefs.

Douglas, Dissenting:
Academic facilities are “structures suitable for use as classrooms,
laboratoris, libraries…” Specifically excluded are facilities “used or to
be used for sectarian instruction.” The public purpose in secular
education is furthered by the program. Yet the sectarian purpose is
aided by making the parochial school system viable.
schools a block grant to build certain facilities. The fact that money is
given once at the beginning is without constitutional significance.
The facilities financed by taxpayers’ funds are not to be used for
“sectarian” purposes. Religious teachings and secular teaching are
so enmeshed in parochial schools that only the strictest supervision
and surveillance would insure compliance with the condition. A
parochial school operates on one budget. Money not spent for one
purpose becomes available for other purposes. Thus the fact that
there are no religious observances in federally financed facilities is
not controlling because required religious observances will take
place in other buildings.
In other words, surveillance creates an entanglement of government and
religion which the First Amendment was designed to avoid. Yet after
today's decision there will be a requirement of surveillance which
will last for the useful life of the building. The price of the subsidy
under the Act is violation of the Free Exercise Clause. Both the Free
Exercise Clause and academic freedom are violated when the
Government agent must be present to determine whether the
course content is satisfactory.
ALLEGHENY COUNTY v. GREATER PITTSBURGH ACLU (IYA) Respondents, the Greater Pittsburgh Chapter of the American Civil Liberties
July 3, 1989 | Blackmun, J. | Freedom of Religion Union and seven local residents, filed suit seeking permanently to enjoin
the county from displaying the creche and the city from displaying the
menorah
PETITIONER: Allegheny County
RESPONDENTS: Greater Pittsburgh Chapter of the American Civil
Liberties
Union

SUMMARY: Greater Pittsburgh questions the display of a creche


(Christian Nativity scene) and menorah (Jewish candelabra) at the front
of government offices during the winter season. They argue that such
display shows favor to the Christian and Jewish faith and is violative of
the Establishment Clause.

The Supreme Court held the display of the creche as violative of the
Clause for it gives the idea of endorsing a specific religious belief.
However, it upholds the validity of the menorah display for it stands
beside a giant Christmas tree which has attained a secular symbol. The
tree merely representing the holiday and not a specific religion.

DOCTRINE: It has been noted that the prohibition against governmental


endorsement of religion "precludes government from conveying or
attempting to convey a message that religion or a particular religious
belief is favored or preferred.” The government's use of religious
symbolism is unconstitutional if it has the effect of endorsing religious
beliefs, and the effect of the government's use of religious symbolism
depends upon its context.

FACTS:
This litigation concerns the constitutionality of two recurring holiday
displays located on public property in downtown Pittsburgh.
The first, a creche depicting the Christian Nativity scene, was placed on
the Grand Staircase of the Allegheny County Courthouse, which is
the "main," "most beautiful," and "most public" part of the courthouse.
The creche was donated by the Holy Name Society, a Roman Catholic
group, and bore a sign to that effect.
Its manager had at its crest an angel bearing a banner proclaiming
"Gloria in Excelsis Deo,” meaning "Glory to God in the Highest."
The second of the holiday displays in question was an 18-foot Chanukah
(Hanukkah) menorah or candelabrum, which was placed just outside
the City-County Building next to the city’s 45-foot decorated
Christmas tree.
At the foot of the tree was a sign bearing the mayor's name and
containing text declaring the city's "salute to liberty."
The menorah is owned by Chabad, a Jewish group, but is stored,
erected, and removed each year by the city.
on the ground that the displays violated the Establishment Clause of Under the Lemon analysis:
the First Amendment. a. A statute or practice which touches upon religion, if it is to be
The District Court denied relief, relying on Lynch v. Donnelly, which held
that:
A city's inclusion of a creche in its annual Christmas display in a
private park did not violate the Establishment Clause.
The Court of Appeals reversed, distinguishing Lynch v. Donnelly, and
holding that the creche and the menorah in the present case must be
understood as an impermissible governmental endorsement of
Christianity
and Judaism under Lemon v. Kurtzman.
ISSUE/s:
WON the two displays violated the Constitutional mandate of freedom of
religion
Creche (Nativity Scene) - YES
Menorah (Candelabra) - NO

RULING: The judgment of the Court of Appeals is affirmed in part and


reversed in part, and the cases are remanded for further proceedings.

RATIO:
It is settled law that no government official in this Nation may violate
these fundamental constitutional rights regarding matters of
conscience.
This Court has come to understand the Establishment Clause to mean
that government may not promote or affiliate itself with any
religious doctrine or organization, may not discriminate among
persons on the basis of their religious beliefs and practices,
may not delegate a governmental power to a religious
institution, and may not involve itself too deeply in such an
institution's affairs.
The Court gave this often-repeated summary on the Establishment
Clause: Neither a state nor the Federal Government can set up a
church. Neither can pass laws which aid one religion, aid all
religions, or prefer one religion over another. Neither can force
nor influence a person to go to or remain away from church
against his will or force him to profess a belief or disbelief in any
religion. No person can be punished for entertaining or
professing religious beliefs or disbeliefs, for church attendance or
non-attendance. No tax in any amount, large or small, can be
levied to support any religious activities or institutions, whatever
they may be called, or whatever form they may adopt to teach or
practice religion. Neither a state nor the Federal Government
can, openly or secretly, participate in the affairs of any religious
organizations or groups and vice versa.”
In Lemon v. Kurtzman, supra, the Court sought to refine these
principles by focusing on three "tests" for determining whether a
government practice violates the Establishment Clause.
permissible under the Establishment Clause, must have a secular
purpose;
It must neither advance nor inhibit religion in its principal or
primary effect; and
It must not foster an excessive entanglement with religion.
The Court pays close attention to whether the challenged governmental
practice either has the purpose or effect of “endorsing” religion.
It has been noted that the prohibition against governmental
endorsement of religion "precludes government from conveying
or attempting to convey a message that religion or a particular
religious belief is favored or preferred.”
In Lynch v Donnelly: the government's use of religious symbolism
is unconstitutional if it has the effect of endorsing religious
beliefs, and the effect of the government's use of religious
symbolism depends upon its context.
The display of the creche endorses Christian doctrine. The nativity scene
portrays a religious message favoring the Christian faith, it’s location
at the Grand Staircase also imposes the idea among the viewers the
government gives its support and approval. The Supreme Court
believes it to be violative of the Establishment Clause.
The menorah, however, stands beside a 45ft tall Christmas tree and a
sign that salutes liberty. The display must be viewed as a whole to
determine its intent to the viewing public.
The widely accepted view of the Christmas tree as the preeminent
secular symbol of the Christmas holiday season serves to
emphasize the secular component of the message communicated by
other elements of an accompanying holiday display, including the
Chanukah menorah. The tree, moreover, is clearly the predominant
element in the city’s display.
In the shadow of the tree, the menorah is readily understood as simply a
recognition that Christmas is not the only traditional way of observing
the winter-holiday season.
In these circumstances, then, the combination of the tree and the menorah
communicates, not a simultaneous endorsement of both the Christian
and Jewish faiths, but instead, a secular celebration of Christmas
coupled with an acknowledgment of Chanukah as a contemporaneous
alternative tradition.
The conclusion here that, in this particular context, the menorah's display
does not have an effect of endorsing religious faith does not
foreclose the possibility that the display of the menorah might violate
either the "purpose" or "entanglement" prong of the Lemon analysis.
ZOBREST v. CATALINA (ELIEL) Zobrest’s request for preliminary injunction, finding that the provision
June 18, 1993 | Rehnquist, J. | Freedom of Religion of an interpreter at Salpointe would likely offend the Establishment
Clause.
PETITIONER: James Zobrest and Larry Zobrest The court thereafter granted Catalina summary judgment, on the ground
that
RESPONDENTS: Catalina Foothills School District

SUMMARY: James Zobrest was born deaf. His parents, for religious
purposes, enrolled him in Salpointe. They request form Catalina Foothills,
from where he attended grades 6-8 a sign language interpreter. The
District denied his request for reasons that furnishing him a sign language
interpreter would violate the constitution, because the State would
symbolically be promoting a sectarian nonsectarian, and public nonpublic
faction. Zobrest invokes the IDEA and free exercise clause. Hence this
petition.

The SC held that the IDEA wherein the government will provide aid to
handicapped people is for individual benefits and not for sectarian
nonsectarian institutions. The benefits that those institutions receive are
merely incidental. It is a neutral government program, and it does not
challenge the establishment clause. The sign language interpreter does
not further another religious interest but only to translate as a whole
everything to Zobrest.

DOCTRINE: We have consistently held that government programs that


neutrally provide benefits to a broad class of citizens defined without
reference to religion are not readily subject to an Establishment Clause
challenge just because sectarian institutions may also receive an
attenuated financial benefit.

FACTS:
James Zobrest attended grades one through 5 in a school for the deaf,
and grades 6-8 in a public school operated by Catalina. While he
attended public school, Catalina furnished him with a sign language
interpreter.
For religious reasons, James’ parents enrolled him for the ninth grade in
Salpointe Catholic High School, a sectarian institution.
When Zobrest requested that Catalina supply Zobrest with an interpreter
at Salpointe, Catalina referred the matter to the County Attorney, who
concluded that providing an interpreter on the school’s premises
would violate the United States Constitution. The question next was
referred to the Arizon Attorney General, who concurred in the County
Attornery’s opinion. Catalina, accordingly declined to provide the
requested interpreter.
Zobrest asserted that the IDEA nad the Free Exercise Clause of the first
amendment require Catalina to provide him with an interpreter at
Salpointe, and that the Estblishment Clause does not bar such relief.
The complaint sought a preliminary injunction and “such ther and further
relief as the Court deems just and proper.” The District Court denied
the interpreter would act a conduit for the religious inculcation of Zobrest all from the IDEA, they are only incidental beneficiaries. Thus, the
– thereby, promoting Zobrest’s religious development at government function of the IDEA is hardly to privde desired financial support for
expense.” nonpublic, sectarian institutions
The CA applied the three part test announced in Lemon v. KurtzmanIt is
first found that the IDEA has a clear secular purpose: “to assist
States and Localities to provide for the education of all handicapped
children.”
The second is if applied as Zobrest proposed, would have the primary
effect of advancing religion and thus would run afoul of the
Establishment Clause. “By placing its employee in the sectarian
school,” the CA reasoned, “the government would create the
appearance that it was a ‘joint sponsor’ of the school’s activities.”
This the court held, would create the “symbolic union of government
and religion” found impermissible.
Hence this petition.

ISSUE/s:
WoN providing an interpreter violates the US Constitution – NO

RULING: Reversed CA decision.

RATIO:
We have consistently held that government programs that neutrally
provide benefits to a broad class of citizens defined without
reference to religion are not readily subject to an Establishment
Caluse challenge just because sectarian institutionas may also
receive an attenuated financial benefit.
The service at issue in this case is part of a general government program
that distributes benefits neutrally to any child qualifying as
“handicapped” under the IDEA, without regard to the “sectarian
nonsectarian, or public nonpublic nature” of the school the child
attends.
In other words, because the IDEA creates no financial incentive for
parents to choose a sectarian school, an interpreter’s presence there
cannot be attributed to state decision-making.
When the government offers a neutral service on the premises of a
sectarian school as part of a general program that “is in no way
skewed towards religion,” it follows under our prior decisions
that provision of that service does not offend the Establishment
Clause.
The State may not grant aid to a religious school, whether cash or
in kind, where the effect of the ai is ‘that of a direct subsidy to
the religious school’ form the State
Substantial aid to the educational function of such schools, we
explained, necessarily results in aid to the sectarian school
enterprise as a whole, and therefore brings about the direct and
substantial advancement of religious activity
Handicapped children, not sectarian schools, are the primary
beneficiaries of the IDEA; to the extent sectarian schools benefit at
Second, the task of a sign language interpreter seems to us quite
different from that of a teacher or guidance counselor. The
Establishment Clause layds down no absolute bar to the placing of a
public employee in a sectarian school.
Nothing in this record suggests that a sign language interpreter would do
more than accurately interpret whatever material is presented to the
class as a whole. In fact, ethical guidelines require interepreters to
“transmit everything that is said in exactly the same ay it was
intended.”
Zobrest’s parents have chosen of their own free will to place him in
a pervasively sectarian environment. The sign language
interpreter they have requested will neither add to nor subtract
from the environment, and hence the provision of such
assistance is not barred by the Establishment Clause.
The IDEA creates a neutral government program dispensing aid not
to schools but to individual handicapped children. If a
handicapped child chooses to enroll in a sectarian school, we
hold that the Establihsment Clause does not prevent the school
district from furnishing him with a sign language interpreter
there in order to facilitate his eduation.
AGOSTINI v. FELTON (JP) In 1965, Congress enacted Title I of the Elementary and
June 23, 1997 | O’Connor, J. | Freedom of Religion Secondary Education Act of 1965 to provide full educational
opportunity to every child regardless of economic
PETITIONER: Rachel Agostini, et al. background.
RESPONDENTS: Betty Louise Felton, et al.

SUMMARY: Petitioners (Board of Education of NY and parents of students


under Title 1 services) are filing to abandon the previous Aguilar v. Felton
(1985) doctrine (12 years ago). According to them, the recent changes in the
Establishment Clause law warranted the growingly disadvantageous effects
of maintaining the Aguilar doctrine such as rising costs. This previous Aguilar
case enjoined the local educational agencies (LEA’s) from conducting Title 1
services in the premises of private schools as they claimed these public
employees which conducted the classes will be influenced by the sectarian
environment of these private schools. And the law specifically prohibits
religious indoctrination.

The SC agrees that recent Establishment Clause cases such as Zobrest,


warrants the abandonment of the Aguilar doctrine. New York City's Title I
program does not run afoul of any of three primary criteria we currently
use to evaluate whether government aid has the effect of advancing
religion: it does not result in governmental indoctrination; define its
recipients by reference to religion; or
create an excessive entanglement. The public employee teachers should
not be presumed to be influenced by the sectarian environment of private
schools.

The SC therefore held that Title 1 services which is a federally funded


program providing supplemental, remedial instruction to disadvantaged
children on a neutral basis is not invalid under the Establishment
Clause when such instructionis given on the premises of sectarian
schools by government employees pursuant to a program containing
safeguards such as those present here. The Aguilar doctrine is
abandoned.

DOCTRINE: Three part test of whether a statues advances religion set


forth in Lemon v. Kurtzman: (a) The statute must have a secular
legislative prupose (b) Its principal or primary effect must be one that
neither advances nor inhibits religion (c) The statue must not foster an
excessive government entanglement with religion.

FACTS:
(Fair Warning: very long and difficult case)
Toward that end, Title I channels federal funds through the States, to local
educational agencies (LEA’s). The LEA’s spend these funds to provide
remedial education, guidance, and job counseling to eligible students.
An eligible student is one
Who resides within the boundaries of a public school located in a low
income area
Who is failing, or at risk of failing, the State’s student performance
standards
Title I must be available to all students whether they attend public or private
schools.
Title I servives themselves must be provided through public employees or other
persons independent of the private school and any religious institution.
Title I services must be secular, neutral and nonideological.
Peittioner Board of Education of the City of New York (Board), a LEA, grappled
ever since with how to provide Title I services to the private school students
within its jurisdiction.
Recognizing that more than 90% of the private schools within the Board’s
jurisdiction are sectarian (religious or believing in some sect), the Board
initially arranged to transport children to public schools for after school Title I
instruction.
But this was largely unsuccessful. Attendance was poor, teachers and children
were tired and parents were concerned for the safety of their children.
So the Board then developed another plan. This was the plan evaluated in
Aguilar v. Felton (1985). The plan called for the provision of Title 1 services
on private school premises during school hours.
Six federal taxpayers, respondents here, sued the Board. Respondents sought
declaratory relief claiming that the Board’s Title 1 program violates the
Establishment Clause.
The District Court that time permanently enjoined the Board from using public
funds for any plan or prgam under Title to the extent that it permits public
school teachers and guidance counselors to provide teaching and counseling
services on the premises of sectarian schools within New York City.
The Board then, modified its Title 1 program according to this decision. Rather
offer Title 1 instruction to parochial school students at their schools, The
Board reverted to its prior practice of providing instruction at public school
sites, in mobile instructional units and even resorted t computer aided
instruction. All of which did not require public employees to be physically
present on the premises of a religious school.
These resulted to additional costs. Complying with the Aguilar mandate made the
Board spend an additional $100 million.
These Aguilar costs made LEA’s cut back on the number of students who receive
Title 1 benefits due to lack of funds.
Petitioners, the Board and a new group of parents of parochial school students
entitled to Title 1 services, filed motions in the Distruct Court seeking relief
under Federal Rule of Civil Procedure 60(b) on remand from our present in Ball: The Board had "adopted a system for monitoring the
decision in Aguilar. religious content of publicly funded Title I classes in the religious
Rule 60(b): "On motion and upon such terms as are just, the court may schools."
relieve a party . . . from a final judgment [or] order . . . [when] it is no Even though this monitoring system might prevent the Title I program
longer equitable that the judgment should have prospective from being used to inculcate religion, the Court concluded in the
application." previous decision, as it had in Lemon and Meek, that the level of
The District Court denied the Rule 60b motion on the merits because monitoring necessary to be "certain" that the program had an
Aguilar’s demise had not yet occurred.
exclusively secular effect would "inevitably result in the
The CA affirmed. Hence this appeal.
excessive entanglement of church andstate," thereby running
ISSUE/s: afoul of Lemon's third prong (See Ratio 4 for the third
WoN the Establishment Clause law or jurisprudence has significantly prong/letter c).
changed since the SC decided Aguilar - YES The Court noted two further forms of entanglement inherent in New York
WoN the petitioners are entitled to relief from the District Court’s City's Title I program: the "administrative cooperation" required to
permanent injunction under Rule 60(b) – YES implement Title I services and the "dangers of political divisiveness"
RULING: The judgment of the CA is reversed and the case is remanded to that might grow out of the day to day decisions public officials would
the District Court with instructions to vacate its previous order. have to make in order to provide Title I services.
However, our more recent cases have undermined the assumptions
RATIO: upon which Ball and Aguilar relied. To be sure, the general principles
Petitioners point to three changes in the factual and legal landscape that we use to evaluate whether government aid violates the Establishment
they believe justify their claim for relief under Rule 60(b)(5). Clause have not changed since Aguilar was decided. For example, we
0 Exorbitant costs of complying with the injunction continue to ask whether the government acted with the purpose of
1 A majority of Justices have expressed their views that advancing or inhibiting religion, and the nature of that inquiry has
Aguilar should be overruled remained largely unchanged.
2 Aguilar has been undermined by subequent Establishment Likewise, we continue to explore whether the aid has the "effect" of
Clause decisions advancing or inhibiting religion.
For the first change, the courts already knew of the costs when the What has changed since we decided Ball and Aguilar is our
decision was made. For the second change, most justices did not understanding of the criteria used to assess whether aid to
tackle the question Aguilar’s propriety per se. religion has an impermissible effect.
Hence, petitioners' ability to satisfy the prerequisites of Rule 60(b)(5) As we have repeatedly recognized, government inculcation of
hinges on the third change, whether our later Establishment Clause religious beliefs has the impermissible effect of advancing
cases have so undermined Aguilar that it is no longer good law. The religion.
SC agreed with petitioner that the recent cases has indeed Our cases subsequent to Aguilar have, however, modified in two
undermined Aguilar. significant respects the approach we use to assess
Three part test of whether a statues advances religion set forth in indoctrination.
Lemon v. Kurtzman: First, we have abandoned the presumption erected in Meek and Ball that
0 The statute must have a secular legislative prupose the placement of public employees on parochial school grounds
1 Its principal or primary effect must be one that neither
inevitably results in the impermissible effect of state sponsored
advances nor inhibits religion
2 The statue must not foster an excessive government indoctrination or constitutes a symbolic union between government
entanglement with religion. and religion.
The New York City Title I program challenged in Aguilar closely resembled Zobrest v. Catalina Foothills School Dist. expressly rejected the notion--
the Shared Time program struck down in School Dist. Of Grand Rapids relied on in Ball and Aguilar--that, solely because of her presence on
v. Ball, but the Court found fault with an aspect of the Title I program not
private school property, a public employee will be presumed to inculcate
religion in the students.
Zobrest also implicitly repudiated another assumption on which Ball and Aguilar
turned: that thepresence of a public employee on private school property
creates an impermissible "symbolic link" between government and
religion. Title I funds are instead distributed to a public agency (an LEA) that
Second, we have departed from the rule relied on in Ball that all government dispenses services directly to the eligible students within its
aid that directly aids the educational function of religious schools is boundaries, no matter where they choose to attend school.
invalid. What is most fatal to the argument that New York City's Title I
In Witters v. Washington Dept. of Servs. for Blind, we held that the program directly subsidizes religion is that it applies with equal
Establishment Clause did not bar a State from issuing a vocational force when those
tuition grant to a blind person who wished to use the grant to attend
a Christian college and become a pastor, missionary, or youth
director.
Even though the grant recipient clearly would use the money to obtain
religious education, we observed that the tuition grants were made
available generally without regard to the sectarian nonsectarian, or
public nonpublic nature of the institution benefited.
In the said situation, any money that ultimately went to religious
institutions did so "only as a result of the genuinely independent and
private choices of" individuals.
Zobrest and Witters make clear that, under current law, New York City's Title
I program in Aguilar will not, as a matter of law, be deemed to
have the effect of advancing religion through indoctrination.
There is no reason to presume that, simply because she enters a
parochial school classroom, a full time public employee such as a
Title I teacher will depart from her assigned duties and instructions
and embark on religious indoctrination.
Thus, both our precedent and our experience require us to reject
respondents' remarkable argument that we must presume Title I
instructors to be "uncontrollable and sometimes very
unprofessional."
Zobrest also repudiates Ball's assumption that the presence of Title I
teachers in parochial school classrooms will, without more, create
the impression of a "symbolic union" between church and state.
We (The SC) do not see any perceptible difference in the degree of
symbolic union between a student receiving remedial instruction in a
classroom on his sectarian school's campus and one receiving
instruction in a van parked just at the school's curbside. To draw this
line based solely on the location of the public employee is neither
"sensible" nor "sound."
Nor under current law can we conclude that a program placing full
time public employees on parochial campuses to provide Title I
instruction would impermissibly finance religious
indoctrination.
services are provided off campus, and Aguilar implied that providing Souter, Dissenting: He believes Aguilar 1985 was a correct and
the services off campus is entirely consistent with the Establishment sensible decision, and his only reservation about its opinion is that
Clause. the emphasis on
Because the incentive is the same either way, we find no logical basis upon
which to conclude that Title I services are an impermissible subsidy of
religion when offered on campus, but allowable when offered off campus.
Accordingly, contrary to our conclusion in Aguilar, placing full time
employees on parochial school campuses does not as a matter of law
have the impermissible effect of advancing religion through
indoctrination.
Applying this reasoning to New York City's Title I program, it is clear that
Title I services are allocated on the basis of criteria that neither
favor nor disfavor religion.
The services are available to all children who meet the Act's eligibility
requirements, no matter what their religious beliefs or where they
go to school.
The Board's program does not, therefore, give aid recipients any
incentive to modify their religious beliefs or practices in order to
obtain those services.
On excessive entanglement: the factors we use to assess whether an
entanglement is "excessive" are similar to the factors we use to examine
"effect." That is, to assess entanglement, we have looked to:
the character and purposes of the institutions that are benefited
the nature of the aid that the State provides
the resulting relationship between the government and religious
authority
As discussed previously, the Court's finding of "excessive" entanglement in
Aguilar rested on three grounds:
(i) the program would require "pervasive monitoring by public
authorities" to ensure that Title I employees did not inculcate
religion
(ii) the program required "administrative cooperation" between the
Board and parochial schools
(iii) the program might increase the dangers of "political divisiveness
The last two consideration are insufficient to create an excessive
entanglement. They are present no matter where Title 1 services are
offered.
The first consideration also has been undermined. After Zobrest we no longer
presume that public employees will inculcate religion simply because
they happen to be in a sectarian environment.
the excessive entanglement produced by monitoring religious instructional
content obscured those facts that independently called for the application of
two central tenets of Establishment Clause jurisprudence.
The Court today relies solely on Zobrest to support its contention that we
have "abandoned the presumption erected in Meek and Ball. The
Court tries to press Zobrest into performing another service beyond
its reach.
There is the issue of precedent. Stare decisis is no barrier in the Court's eyes
because it reads Aguilar and Ball for exaggerated propositions that
Witters and Zobrest are supposed to have limited to the point of
abandoned doctrine.
Ginsburg, dissenting: Subsequent decisions, the majority says, have
undermined Aguilar and justify our immediate reconsideration. This
Court's Rules do not countenance the rehearing here granted. For
good reason, a proper application of those rules and the Federal
Rules of Civil Procedure would lead us to defer reconsideration of
Aguilar until we are presented with the issue in another case.
MITCHELL v. HELMS (HENRY) (LEA)”.
June 28, 2000 | Thomas, J. | Freedom of Religion Several restrictions apply to aid in private schools. Most significant is that
of the “services, materials, and equipment” that have to be “secular,
PETITIONER: Guy Mitchell, et al. neutral, and nonideological”. Further, private schools may not acquire
RESPONDENTS: Mary L. Helms, et al. control of Chapter 2 funds.
In Jefferson parish, private schools have primarily used their allocations
SUMMARY: The assailed law, Chapter 2 of the Education consolidation for nonrecurring expenses, usually materials and equipment. Among
and Improvement Act of 1981 channels federal funds to local education materials were library books, computers, projectors, etc. It appears
agencies which are usually public school districts, via state education that 30% of Chapter 2 funds spent in Jefferson Parish are allocated
agencies, to implement programs to assist children in elementary and for private schools.
secondary schools. Participating schools receive aid based on the December 1985, Helms et al filed a suit alleging that Chapter 2, as
number of students enrolled, and such funds are to be used to procure applied in Jefferson Parish, violated the Establishment Clause of the
books, materials, and equipment, that are “secular, neutral, and First Amendment of the Federal Constitution.
nonideological”. Respodnents filed a case assailing Chapter 2’s validity District court (Chief Judge Heebe) granted summary judgment in favor of
as it funded private schools in Jefferson Parish, which are, said to be the respondents, and held that Chapter 2 violated the Establishment
religious schools. Clause because under the second part of the three-part test in
Lemon v. Kurtzman, the program had the primary effect of advancing
SC used the standards in Agostini v. Felton to determine if the funding religion.
provided in Chapter 2 has an effect of advancing religion. Afterwhich, the SC 2 years later, Heebee died and J. Livaudais received the case (still in the
found that it does not implicate any of the factors. Chapter 2 therefore does same district court), to which he reversed the decision of Heebe and
not affect the establishment of religion because government action could not upheld Chapter 2.
be considered to be the source of any religious indoctrination that occurs in Respondents then appealed to the CA. And while the particular appeal
the schools. The federal funding is granted to a broad range of recipients was pending, the SC ruled on the case of Agostini. 6 CA ruled in favor
without regard to their religion. It does not offer a financial incentive to of respondents, and hence this petition.
undertake religious indoctrination.
ISSUE/s:
DOCTRINE: In distinguishing between indoctrination that is attributable WoN the law is prohibiting the right to the free exercise of religion – NO
to the State and indoctrination that is not, the SC consistently turned to
the principle of neutrality, upholding aid that is offered to a broad range of RULING: SC denied the petition, Chapter 2 is not a law respecting an
groups or persons without regard to their religion. establishment of religion.

RATIO:
FACTS:
The Establishment Clause of the First Amendment dictates that
Chapter 2 of the Education Consolidation and Improvement Act of 1981 “Congress shall make no law respecting an establishment of
channels federal funds to local educational agencies (LEAs), which religion”. The SC has consistently struggled to apply these simple
are usually public school districts, via state educational agencies words in the context of governmental aid to religious schools.
(SEAs), to implement programs to assist children in elementary and In Agostini, however, SC brought clarity to US jurisprudence insofar as
secondary schools. It stated, “for the acquisition and use of the subject matter is concerned, by overruling 2 anomalous
instructional and educational materials, including library services and precedengs and by consolidating some of their disparate
materials (including media materials), assessments, reference considerations under a revised test. The SC pointed out in this case
materials, computer software and hardware for instructional use, and that the Lemon test was modified.
other curricular materials” The SC provided the ff test in the Lemon case, which considered whether
LEAs and SEAs must offer assistance to both public and private (non-profit) a statute:
schools. Participating private schools receive aid based on the number Has a secular purpose
of students enrolled, and allocations for Chapter 2 funds for those Has a primary effect of advancing or inhibiting religion; or
schools must generally be “equal (consistent with the number of children Creates an excessive entanglement between government in religion.
to be served) to expenditures for programs… for children enrolled in the In Agostini, Lemon was modified for purposes of evaluating aid to
public schools of the schools
SC approved a program that under Title 1 of the ESEA, provided public employees to teach
remedial classes at private schools, including religious schools.
and examined only by the first and second factors. In Agostini, the Hence, Chapter 2 satisfies both the first and second criteria of Agostini. It
third factor in the Lemon test is no relevant when considering therefore does not have the effect of advancing religion. For the
distributing aid to religious schools. same reason, Chapter 2 also “cannot reasonably be viewed as an
With reference again to the Agostini case, the SC ruled that the question endorsement of religion”.
whether governmental aid to religious schools result in governmental
indoctrination is ultimately a question whether any religious
indoctrination that occurs in those schools could reasonably be
attributed to governmental action.
In distinguishing between indoctrination that is attributable to the State
and indoctrination that is not, the SC consistently turned to the
principle of neutrality, upholding aid that is offered to a broad range
of groups or persons without regard to their religion.
As a way of assuring neutrality, the SC repeatedly considered whether
any governmental aid that goes to a religious institution does so
“only as a result of the genuinely independent and private choices of
individuals”
Respondents’ claims are inconsistent with the recent case law of the SC,
and hence the court rejects them.
Applying the two relevant Agostini criteria (ratio #3), the SC sees no
basis for concluding that Jefferson Parish’s Chapter 2 program “has
the effect of advancing religion. Chapter 2 does not result in
governmental indoctrination, because it determines eligibility for aid
neutrally, allocates that aid based on the private choices of the
parents of schoolchildren, and does not provide aid that has an
impermissible content. Nor does Chapter 2 define its recipients by
reference to religion.
It is clear that Chapter 2 aid “is allocated on the basis of neutral, secular
criteria that neither favor nor disfavor religion, and is made available
to both religious and secular beneficiaries on a nondiscriminatory
basis”. The aid is allocated based on enrollment (fact #2). The
allocation criteria therefore create no improper incentive. Chapter 2
does, by statute, deviate from a pure per capita basis for allocating
aid to LEAs, increasing the per-pupil allocation based on the number
of children within an LEA who are from poor families, reside in poor
areas, or reside in rural areas.
It is clear as well that Chapter 2 aid makes a broad array of schools
eligible for the aid without regard to their religious affiliations or lack
thereof. The SC hence has no difficulty in concluding that Chapter 2
is neutral with regard to religion. The aid as well reaches participating
schools only as a consequence of private decisionmaking. It is the
students and paretns—not the government—who, through their
choice of school, determine who receives Chapter 2 funds. The aid
follows the child.
Chapter 2 aid does not provide to religious schools the aid that has an
impermissible content. The statute explicitly bars anything of the sort,
providing that all Chapter 2 aid for the benefit of children in private
schools shall be “secular, neutral, and nonideological” (fact #3)
It is not a law respecting an establishment of religion. Jefferson Parish
need not exclude religious schools from its Chapter 2 program.
O’Connor with Breyer, concurring: The standard created by the majority is
overly broad as it allows any type of federal funding for religious schools
that is secular in content and offered on a neutral basis. The majority
should also have presented the distinction between direct and indirect
funding.
Souter with Stevens and Ginsburg, dissenting: The Establishment Cause
is violated when government funds may be diverted to serve religious
purposes. The establishment prohibition of government religious funding
serves more than one end. It si meant to guarantee the right of individual
conscience against compulsion, to protect the integrity of religion agaist
the corrosion of secular support, and to preserve the unity of political
society against the implied exclusion of the less favored and the
antagonism of controversy over public support for religious cases.
ZELMAN v. SIMON-HARRIS ET AL. (DANNAH) Cleveland’s district had failed to meet any of the 18 state
June 27, 2002 | Rehnquist, C.J. | Establishment Clause standards for minimal acceptable performance. Only 1 in 10
ninth graders could pass a
PETITIONER: Zelman, Superintendent of Public Instruction of Ohio et al.
RESPONDENTS: Simmons-Harris et al.

SUMMARY: Because of the status of Cleveland’s public schools (worst


performing), Ohio enacted its Pilot Project Scholarship Program. The
program provides financial assistance to families in any Ohio school
district that is or has been “under federal court order requiring supervision
and operational management of the district by the state superintendent. It
offers either tuition or tutorial aid. 96% of the students/parents opted to
enroll in religiously affiliated schools. This was the reason why
respondent Ohio taxpayers challenged said program. The Ohio Supreme
Court rejected respondents’ federal claims, but held that the enactment of
the program violated certain procedural requirements of the Ohio
Constitution. The State Legislature immediately cured this defect, leaving
the basic provisions discussed above intact.

In 1999, respondent taxpayers filed this action with the US District Court.
It ruled against the program, and the CA affirmed the decision, stating
that the program had the “primary effect” of advancing religion in violation
of the Establishment Clause. The SC however, reversed, citing Mueller,
Witters, and Zobrest. The Court in these cases emphasized the principle
of private choice. If religious schools or institutions benefit from
programs or laws by the State, but only because of private choices of
individuals, then the programs or laws cannot be said to violate the
Establishment Clause.

DOCTRINE: Where a government aid program is neutral with respect to


religion, and provides assistance directly to a broad class of citizens who,
in turn, direct government aid to religious schools wholly as a result of
their own genuine and independent private choice, the program is not
readily subject to challenge under the Establishment Clause.

The court has repeatedly recognized that no reasonable observer would


think a neutral program of private choice, where state aid reaches
religious schools solely as a result of the numerous independent
decisions of private individuals, carries with it the imprimatur of
government endorsement.

FACTS:
For more than a generation, Cleveland’s public schools have been
among the worst performing public schools in the Nation; a “crisis of
magnitude” was even declared by a Federal District Court in 1995
and places the entire Cleveland school district under state control.
basic proficiency examination, and students at all levels performed at a
dismal rate compared with students in other Ohio public schools.
More than two-thirds of high school students either dropped or failed out before
graduation. Of those students who managed to reach their senior year, one
of every four still failed to graduate. Of those students who did graduate, few
could read, write, or compute at levels comparable to their counterparts in
other cities.
This is the reason why Ohio enacted its Pilot Project Scholarship Program. The
program provides financial assistance to families in any Ohio school district that
is or has been “under federal court order requiring supervision and operational
management of the district by the state superintendent.
The program provides two basic kinds of assistance:
The program provides tuition aid for students in kindergarten through
third grade, expanding each year through eigth grade, to attend a
participating public or private school of their parent’s choosing (more
relevant to the issue)
The program provides tutorial aid for students who choose to remain
enrollend in public school
The tuition program is designed to provide educational choices to parents who
reside in a covered district. Any private school, whether religious or
nonreligious, may participate in the program and accept program students
so long as the school is located within the boundaries of a covered district
and meets statewide educational standards.
Any public school located in a school district adjacent to the covered district may
also participate in the program. Adjacent public schools are eligible to receive
a $2,250 tuition grant for each program student accepted in addition to the
full amount of per-pupil state funding attributable to each additional student.
The distribution of tuition aid depends on the financial need of parents (not
relevant, see original for distribution).
For the tutorial aid, parents arrange for registered tutors and get reimbursement
(up to $360). The number of tutorial assistance grants offered to students in
a covered district must equal the number of tuition aid scholarships provided
to students enrolled at participating private or adjacent public schools.
The program has been in operation with the Cleveland City School District since
the 96-97 school year.
More than 3,700 students participated, most of whom (96%) enrolled in
religiously affiliated schools.
In 1996, respondents, a group of Ohio taxpayers, challenged the Ohio program in
state court on state and federal grounds. The Ohio Supreme Court rejected
respondents’ federal claims, but held that the enactment of the program violated
certain procedural requirements of the Ohio Constitution.
The State Legislature immediately cured this defect, leaving the genuinely independent and private choices of aid recipients.”
basic provisions discussed above intact. As in Mueller, “the program is made available generally without regard
In July 1999, respondent taxpayers filed this action in the US District to the sectarian-nonsectarian, or public-nonpublic nature of the
institution benefited.”
Court, seeking to enjoin the reenacted program on the ground that it
The holding rested not on whether few or many recipients chose to
violated the Establishment Clause of the United States Constitution.
expend government aid at a religious school but, rather, on whether
In August, the District Court issued a preliminary injunction barring further recipients generally were empowered to direct the aid to schools or
implementation of the program. In December, it granted summary institutions of their own choosing.
judgment for respondents. The CA affirmed the judgment, finding that
In Zobrest v. Catalina Foothills School Dist.:
the program had the “primary effect” of advancing religion in violation
The Court applied both Mueller and Witters, and it rejected an
of the Establishment Clause. Establishment Clause challenge to a federal program that permitted
sign-language interpreters to assist deaf children enrolled in
ISSUE/s: religious schools
WoN the pilot program offends the Establishment Clause of the US “Government programs that neutrally provide benefits to a broad class
Constitution – NO of citizens defined without reference to religion are not readily
WoN the Ohio program has the forbidden “effect” of advancing or subject to an Establishment Clause challenge.”
inhibiting religion – NO Again looking at the challenged program as a whole, the Court observed
that the program “distributes benefits neutrally to any child
RULING: Judgment of CA reversed. qualifying as ‘disabled’”. Its primary beneficiaries, were disabled
children, not sectarian schools.
RATIO: By according parents freedom to select a school of their choice, the
The Establishment Clause prevents a State from enacting laws that statute ensures that a government-paid interpreter will be present in
have the “purpose” or “effect” of advancing or inhibiting religion. a sectarian school only as a result of the private decision of
individual parents.
The Court cited three similar cases to support its decision of upholding
the constitutionality of the program. The three cases make clear that where a government aid program is
In Mueller v. Allen: neutral with respect to religion, and provides assistance directly to a
A Minnesota program authorizing tax deductions for various educational broad class of citizens who, in turn, direct government aid to religious
expenses, including private school tuition costs, even though the schools wholly as a result of their own genuine and independent
majority (96%) of the beneficiaries were parents of children in
religious schools was upheld. private choice, the program is not readily subject to challenge under
The class included “all parents,” including parents with “children who the Establishment Clause.
attend nonsectarian private schools or sectarian private schools” If numerous private choices, rather than the single choice of a
The program was not readily subject to challenge under the
government, determine the distribution of aid, pursuant to neutral
Establishment Clause
Viewing the program as a whole, the court emphasized the principle of eligibility criteria, then a government cannot, or at least cannot easily,
private choice, noting that public funds were made available to grant special favors that might lead to a religious establishment.
religious schools “only as a result of numerous, private choices of In the case at hand, the program challenged is a program of true private
individual parents of school-age children.” choice, constistent with Mueller, Witters and Zobrest, and thus
No imprimatur of state approval can be deemed to have been conferred constitutional.
on any particular religion, or on religion generally It is neutral in all respects toward religion. It confers educational
The program was one of true private choice, with no evidence that the State assistance directly to a broad class of individuals defined without
deliberately skewed incentives towards religious schools, was sufficient reference to religion.
for the program to survive scrutiny under the Establishment Clause. There are no financial incentives that skew the program toward religious
In Witters v. Washington Dept. of Servs. For Blind: schools.
A vocational scholarship program that provided tuition aid to a student It in fact creates financial disincentives for religious schools, with private
studying at a religious institution to become a pastor was upheld
schools receiving only half the government assistance given to
Looking at the program as a whole, the Court observed that “any aid … that
community schools and one-third the assistance given to magnet
schools.
Adjacent public schools, should any choose to accept program students, are also Families too have a financial disincentive to choose a private religious school
eligible to receive two to three times the state funding of a private religious
school.
ultimately flows to religious institutions does so only as a result of the
over other schools. Parents that choose to participate in the the program here, offer aid directly to a broad class of individual
scholarship program and then to enroll their children in a private recipients defined without regard to religion.
school (religious or nonreligious) must copay a portion of the school’s In sum, the Ohio program is entirely neutral with respect to religion. It
tuition. Families that choose a community school, magnet school, or permits individuals to exercise genuine choice among options public
and private, secular and religious.
traditional public school pay nothing.
The program is therefore a progam of true private choice. It does not
Respondent taxpaers suggest that even without a financial incentive for offend the Establishment clause.
parents to choose a religious school, the program creates a “public
perception that the State is endorsing religious practices and beliefs.”
But the court has repeatedly recognized that no reasonable observer
would think a neutral program of private choice, where state aid
reaches religious schools solely as a result of the numerous
independent decisions of private individuals, carries with it the
imprimatur of government endorsement.
Any objective observer familiar with the full history and context of the
Ohio program would reasonably view it as one aspect of a broader
undertaking to assist poor children in failed schools, not as an
endorsement of religious schooling in general.
Cleveland’s preponderance of religiously affiliated private schools
certainly did not arise as a result of the program; it is a phenomenon
common to many American cities.
It is true that 82% of Cleveland’s participating private schools are
religious schools, but it is also true that 81% of private schools in
Ohio are religious schools.
Respondent taxpayers and Justice Souter claim that the Court should
attach constitutional significance to the fact that 96% of scholarship
recipients have enrolled in religious schools. They claim that this
alone proves parents lack genuine choice, even if no parent has ever
said so.
The Court rejects this argument; it was flatly rejected in Mueller, where
the Court found it irrelevant that 96% of parents taking deductions for
tuition expenses paid tuition at religious schools.
The constitutionality of a neutral educational aid program simply does not
turn on whether and why, in a particular area, at a particular time,
most private schools are run by religious organizations, or most
recipients choose to use the aid at a religious school.
Respondent taxpayers also claim that the Court should look to the case
of Nyquist, bu the Court disagrees for two reasons:
The program there involved an NY program that gave a package of
benefits exclusively to private schools and the parents of the
enrollees. The function was unmistakably to provide desired
financial support for nonpublic, sectarian instutions. The program is
different from Ohio’s. It flatly prohibited the participation of any
public school, or parent of any public school enrollee.
Nyquist does not govern neutral educational assistance programs that, like
CAPITOL SQUARE REVIEW BOARD v. PINETTE & KKK seeking an injunction requiring the Board to issue the requested
(ARIELLE) permit.
June 29, 1995 | Scalia, J.| Establishment Clause

PETITIONER: Capitol Square Review Board


RESPONDENTS: Pinette & KKK

SUMMARY: The Ku Klux Klan filed an application with the Capitol


Square Review and Advisory Board to put up a cross on the Capitol
Square, which is a public forum. The Board denied the application on the
ground that the permit would violate the Establishment Clause. The
District Court determined that Capitol Square was a traditional public
forum open to all without any policy against free standing displays; that
the Klan’s cross was entirely private expression entitled to full First
Amendment protection; and that the Board had failed to show that the
display of the cross could be construed as endorsement of Christianity by
the State. The CA affirmed the District Court’s decision. The SC likewise
affirmed the CA, and held that the Establishment Clause tempers
religious speech, but it applies only to the words and acts of government.
It was never meant to serve as an impediment to purely private religious
speech connected to the State only through its occurrence in a public
forum.

DOCTRINE: The State may not, on the claim of misperception of official


endorsement, ban all private religious speech from the public square, or
discriminate against it by requiring religious speech alone to disclaim
public sponsorship.

FACTS:
Capitol Square is a 10-acre state owned plaza used for public speeches,
gatherings, and festivals advocating a variety of causes, both secular
and religious. The Ohio Admin. Code makes the square available for
use by the public for free discussion of public questions or for
activities of a broad public purpose.
The Ohio Rev. Code gives the Capitol Square Review and Advisory
Board responsibility for regulating public access. To use the square,
a group must simply fill out an official application form and meet
several criteria, which concern safety, sanitation and non-
interference and which are neutral as to the speech content of the
proposed event.
Diverse groups such as homosexual rights organizations, the Ku Klux
Klan and the United Way held rallies. Respondent Donnie Carr, an
officer of the Ohio Ku Klux Klan, filed an application to place a cross
on the square, but this was denied by the Board.
The Ohio Klan, through its leader Vincent Pinette, filed the present suit in
the United States District Court for the Southern District of Ohio,
The Board defended on the ground that the permit would violate the concede that they rejected the display precisely because its content
Establishment Clause. The District Court determined that Capitol was religious. The Board’s justification of such denial was based on
Square was a traditional public forum open to all without any policy the State’s interest in
against free standing displays; that the Klan’s cross was entirely
private expression entitled to full First Amendment protection.
The District Court issued the injunction, and the Board permitted the Klan
to erect its cross. The Board then received and granted several
additional applications to erect crosses on Capitol Square during
December 1993 and January 1994.
On appeal by the Board, the US CA affirmed the District Court’s
judgment.
Respondent KKK contend that the Court should treat the case as a case
in which freedom of speech (the Klan’s right to present the message
of the cross display) was denied because of the State’s
disagreement with that message’s political content, rather than
because of the State’s desire to distance itself from sectarian
religion.

ISSUE/s: WoN the CA erred in affirming the District Court’s decision in


upholding KKK’s right to place the cross on the square - NO

RULING: The judgment of the Court of Appeals is AFFIRMED.

RATIO:
KKK’s religious display was private expression. Precedents establish that
private religious speech, far from being a First Amendment orphan, is
as fully protected under the Free Speech Clause as secular private
expression.
In Anglo American history, government suppression of speech has so
commonly been directed precisely at religious speech that a free
speech without religion would be Hamlet without the prince.
The Court has not excluded from free speech protections religious
preaching, or even acts of worship.
The Board contends that the constitutional protection does not extend to
the length of permitting that expression to be made on Capitol
Square.
However, speech which is constitutionally protected against state
suppression is not accorded a guaranteed forum on all property
owned by the State.
The right to use government property for one’s private expression depends
upon whether the property has by law or tradition been given the status
of a public forum, or has been reserved for specific official uses. If the
former, a State’s right to limit protected expressive activity is sharply
circumscribed: it may impose reasonable, content-neutral time, place
and manner restrictions, but it may regulate expressive content ONLY
if such a restriction is necessary, and narrowly drawn, to serve a
compelling state interest.
The Board does not claim that their denial of KKK’s application was
based upon a content-neutral time, place or manner restriction. They
avoiding official endorsement of Christianity, as required by the Establishment Clause forbids, and private speech endorsing
Establishment Clause. religion, which the free speech and free exercise clauses protect.
There is no doubt that compliance with the Establishment Clause is a state The Board asserts that the distinction disappears when the private
interest sufficiently compelling to justify content-based restrictions on speech is
speech. Whether that interest is implicated in this case is a different
question. The Court has twice addressed the combination of private
religious expression, a forum available for public use, content-based
regulation, and a State’s interest in complying with the Establishment
Clause. Both times, the Court sruck down the restriction on religious
content.
Lamb’s Chapel case: Even if school property during off hours was not a
public forum, the school district violated an applicant’s free speech rights
by denying it use of the facilities solely because of the religious
viewpoint of the program it wished to present. The Lamb’s Chapel
reasoning applies in this case where the property at issue is not a school
but a full fledged public forum.
Widmar case: Compelling state interest in complying with the
Establishment Clause justified the content-based restriction was
rejected because the forum created by the State was open to a
broad spectrum of groups and would provide only incidental benefit
to religion. An open forum in a public university does not confer any
imprimatur of state approval on religious sects or practices.
In both cases (Widmar and Lamb’s Chapel), the State did not sponsor
the respondent’s expression, the expression was made on
government property that had been opened to the public for speech.
The Board claims that one feature of the present case is different from
Lamb’s Chapel and Widmar: the forum’s proximity to the seat of
government, which, they contend, may produce the perception that
the cross bears the State’s approval. The Board urges the Court to
apply the “endorsement test,” where the subject of the test was
either expression by the government itself, or else government action
alleged to discriminate in favor of private religious expression or
activity.
“Endorsement” connotes an expression or demonstration of approval or
support. As far as the Establishment Clause jurisprudence is
concerned, the Court has consistently held that it is no violation for
government to enact neutral policies that happen to benefit
religion.
Allegheny County case: The display of a privately sponsored crèche on
the Grand Staircase of the County Courthouse violated the
Establishment Clause. The staircase was not open to all on an
bequal basis, so the County was favoring sectarian religious
expression.
Once the Court determines that the benefit to religious groups from the
public forum was incidental and shared by other groups, the State’s
Establishment Clause defense will be rejected.
There is a difference between government speech and private speech
because government speech endorsing religion, which the
conducted too close to the symbols of government. But that must be
merely a subpart of a more general principle: that the distinction
disappears whenever private speech can be mistaken for government
speech.
Of course, giving sectarian religious speech preferential access to a forum
close to the seat of government would violate the Establishment Clause.
But this favoritism does not exist in the present case. Capitol Square is a
genuinely public forum for many years. Private religious speech cannot b
subject to veto by those who see favoritism where there is none.
J. Stevens, J. Souter, and J. O’Connor’s view: Private religious speech is
exiled to a realm of less protected expression inhabited only by sexually
explicit displays and commercial speech.
The Establishment Clause tempers religious speech, but it applies only
to the words and acts of government. It was never meant to serve
as an impediment to purely private religious speech connected to
the State only through its occurrence in a public forum.
To require access by a religious group, it was sufficient that the group’s activity
was not government sponsored, that the event was open to the public, and
that the benefit of the facilities was shared by various organizations.
In effect, every proposed act of private, religious expression in a public forum
would force officials to weigh a host of imponderables. How close to
government is too close? What kind of building, and in what context,
symbolizes state authority? If the State guessed wrong in one direction, it
would violate the Establishment Clause; if in the other, it would be liable
for suppressing free speech.
The “transferred endorsement” test would disrupt the settled principle that
policies providing incidental benefits to religion do not contravene the
Establishment Clause.
The State may not, on the claim of misperception of official
endorsement, ban all private religious speech from the public
square, or discriminate against it by requiring religious speech
alone to disclaim public sponsorship.
ISLAMIC DA’WAH v. EXECUTIVE SECRETARY (IYA) YES
July 9, 2003 | Corona, J. | Freedom of Religion

PETITIONER: Islamic Da’wah Council of the Philippines


RESPONDENTS: Office of the Executive Secretary, Office on Muslim
Affairs

SUMMARY: IDCP questions the validity of EO 46 which grants to OMA


the exclusive power to determine which food products are halal or
allowable for consumption under Moslem practice.

The SC declared the EO void for enroaching upon the religious freedom
of Muslim corporations. The classifying of a food product as halal is
deemed a religious function only the Moslem people can perform.

The State has not proven that there is valid justification for the
deprivation of muslim organization to issue halal certifications.

DOCTRINE: Only the prevention of an immediate and grave danger or


the security and welfare of the community can justify the infringement of
religious freedom.

FACTS:
Islamic Da’wah Council of the Philippines (IDCP) is a non-governmental
organization hat extends voluntary services to the Muslim Filipino
communities.
Among the functions of IDCP is to conduct seminars, orient
manufactures on halal food and issue halal certifications to qualified
products and manufacturers.
On October 26, 2001, the Office of the Executive Secretary issued EO 46
creating the Philippine Halal Certification Scheme and designating
the Office on Muslim Affairs (OMA) to oversee its implementation.
Under the EO, OMA has the exclusive authority issue halal certificates.
IDCP contends that the EO violates the constitutional provision on the
separation of Church and State.
It is unconstitutional for the government to formulate policies and
guidelines not he halal certification scheme because said scheme is
a function only religious organizations, entity or scholars can lawfully
and validly perform for the Muslims.
IDCP filed before the SC a petition for prohibition for the declaration of
nullity of EO 46 and prohibition of OMA form implementing subject
EO.
ISSUE/s:
WON the EO is violative of the principle of separation of church and state
-
RULING: Petition is GRANTED. EO 46 is declared NULL and VOID.

RATIO:
Classifying a food product as halal is a religious function because the
standards used are drawn from he Qur’an and Islamic beliefs.
By giving OMA the exclusive power to classify food products as halal,
EO 46 enroached on the religious freedom of Muslim organizations
like IDCP.
The State, in effect, forced Muslims to accept its own interpretation of the
Qur’an and Sunnah on halal food.
Only the prevention of an immediate and grave danger or the
security and welfare of the community can justify the
infringement of religious freedom.
If the government fails to show the seriousness and immediacy of
the threat, State intrusion is constitutionally unacceptable.
In a society with a democratic framework like ours, the State must
minimize its interference with the affairs of its citizens and instead
allow them to exercise reasonable freedom of personal and religious
activity.
In the case at hand, the Court finds no compelling justification for the
government to deprive muslim organizations the power to issue halal
certifications.
TARUC v. DE LA CRUZ (ELIEL) Bishop, however, failed to stop Taruc from carrying out his plans. After
March 10, 2005 | Corona, J. | Freedom of Religion which, declared Taruc expelled/excommunicated from the PIC for
disobedience, inciting dissension, and for thretenign to forcibly
PETITIONER: Dominador Taruc, Wilberto Dacera et al. occupy the Parish Church.
Bishop De La Cruz was reassigned, and his predecessor likewise did not
RESPONDENTS: Bishop Porfirio De La Cruz, Fr. Rustom Florano
find any valid reason to transfer Fr. Florano to another parish.
SUMMARY: Taruc et al were members of the Philippine Independent Meanwhile, Taruc contends that his excommunication and expulsion
Church (PIC). Apparently, the wife of Fr. Florano belonged to a political from the PIC was done without trial thus violating their right to due
party opposed to Taruc, which is why Taruc wanted the transfer of Fr. process of law. Hence this petition.
Florano. However, the Bishop denied because it was not a valid reason
ISSUE/s:
to order such. Because of the animosity, Taruc laid down his plans to the
Bishop to organize a mass held by Fr. Ambong. The Bishop pleaded WoN the expulsion was in violation of their rights– NO
Taruc to refrain from doing so because the credentials of Fr. Ambong
RULING: Denied.
were in doubt. When Taruc continued, the Bishop expelled and
excommunicated them. Hence the Petition.
RATIO:
The SC held that it had no jurisidicition to try the case. Because of the Sec 5, Art III: “No law shall be made respecting an establishment of
separation of the church and state, it is best left to the church the matters religion or prohibiting the free exercise thereof. The free
of official and disciplinary functions. The Court cannot delve into doctrinal exercise and enjoyment of religios profession and worship,
and discretionary differences of the Church. In terms of due process, the without discrimination or preference, shall forever be allowed.
Bishop pleaded to Taruc and afforded him opportunity but the latter No religious test shall be required for the exercise of civil or
political rights.”
ignored.
In our jurisidiction, we hold the Church and the State to be separate and
DOCTRINE: It is not for the courts to exercise control over church distinct from each other.
authorities in the performance of their discretionary and official functions. We agree with the CA that the expulsion/excommunication of members
Rather, it is for the members of religious institutions/organizations to of a religious institution/organization is a matter best left to the
conform to just church regulations. discretion of the officials, and the laws and canons, of said
institution/organization.
It is not for the courts to exercise control over church authorities in
FACTS: the performance of their discretionary and official functions.
Petitioners were lay members of the Philippine Independent Church Rather, it is for the members of religious
(PIC) in Socorro, Surigao del Norte. Respondents Porfirio De La institutions/organizations to conform to just church regulations.
Cruz (Bishop) and Rustom Florana were the bishop and parish Fonacier v. CA, we enunciated the doctrine that in disputes
priest, respectively, of the same church in that locality. involving religious institutions or organizations, there is one
Taruc (petitioners), clamaored for the transfer of Fr. Florano but Bishop area which the Court should not touch: doctrinal and
denied their request. It appears from records that the family of Fr. disciplinary differences.
Florano’s wife belonged to a political party opposed to Taruc’s, thus On due process: The records show that Bishop De La Cruz pleaded with
the animosity between the two factions with Fr. Florano being petitioners several times not to commit acts inimical to the best
identified with his wife’s political camp. Bishop, however, found this interests of PIC. They were also warned of the consequences of their
too flimsy a reason for transferring Fr. Florano to another parish. actions, among them their expulsion/excommunication from PIC. Yet,
Hostitlity among the memebrs of the PIC worsened when Taruc tried to these pleas and warnigns fell on deaf ears and petitioners went
organize an open mass to be celebrated by a certain Fr. Ambong ahead with their plans to defy their Bishop and foment hostility and
during the twon fiesta. disunity among the members of PIC in Socorro, Surigao del Norte.
When Taruc informed Bishp of his plan, the Bishop tried to dissuade him They should now take full responsibility for the chaos and dissension
from pushing through with it because Fr. Ambong was not a member they caused.
of the clergy of the diocese of Surigao an dhis credentials as a parish
priest were in doubt.
The Bishop also appealed to Taruc to refrain from committing acts
inimical and prejudicial to the best interests of the PIC.
VICTORIANO v. ELIZALDE (JP) organization is the representative of the employees.
September 12, 1974 | Zaldivar, J. | Freedom of Religion On June 18, 1961, however, Republic Act No. 3350 was enacted,
introducing an amendment to paragraph (4) subsection (a) of section
PETITIONER: Benjamin Victoriano 4 of Republic Act No. 875, as follows:
RESPONDENTS: Elizalde Rope Workers’ Union and Elizalde Rope “But such agreement shall not cover members of any
Factory, religious sects which prohibit affiliation of their members in
Inc. any such labor organization.”
SUMMARY: Petitioner Victoriano is a worker of Elizalde Roper Factory Inc., Being a member of a religious sect that prohibits the affiliation of its
nad thus was a member of the Company’s Labor Union. He was by default a members with any labor organization, Petitioner Victoriano presented
member because of RA 875 which allowd companies to require their his resignation to appellant Union.
employees to be part of their union upon employment. RA 875 was amended The management of the Company in turn notified Appellee and his
by RA 3350 however which exempts employees with religions that prohibit counsel that unless the Appellee could achieve a satisfactory
them from joining labor unions from the requirement of companies. arrangement with the Union, the Company would be constrained to
Victoriano, being a member of Iglesia ni Cristo which bans membership in
dismiss him from the service.
unions, files for resignation from the Union. The Union however declined and
questioned the constitutionality of the statute. The lower court ruled in favor In its answer, the Union invoked the “union security clause” of the
of petitioner Victoriano. collective bargaining agreement and assailed the constitutionality of
Republic Act No. 3350.
SC affirmed the CFI decision and ruled that RA 3350 passed the two The lower court decided that defendant Company is enjoined from
requirements to withstand the constitutional prohibition on the no-
dismissing plaintiff Victoriano from his present employment. Hence this
establishment of religion clause of the Constitution. RA 3350 has a secular
purpose of finding gainful employment which does not go against their appeal.
religious beliefs. The primary effect is also the protection of the laborer from
menacing collective bargaining agreements. The effect of favoring laborers of ISSUE/s:
one religion is only incidental. WoN Republic Act No. 3350 violates the establishment clause of the
Constitution – NO
DOCTRINE: A statute, in order to withstand the strictures of
RULING: Instant appeal is dismissed. CFI decision is affirmed.
constitutional prohibition (on the no-esthablishment of religion clause
of the cosntitution), must have: (a) a secular purpose and (b) a primary RATIO:
effect that neither advances nor inhibits religion. On freedom of association: Republic Act No. 3350 merely excludes ipso
jure from the application and coverage of the closed shop agreement
the employees belonging to any religious sects which prohibit
FACTS:
affiliation of their members with any labor organization.
Petitioner Benjamin Victoriano is a member of the religious sect known
What the exception provides, therefore, is that members of said religious
as the Iglesia ni Cristo. He had been in the employ of respondent
Elizalde Rope Facotry Inc. (Company). sects cannot be compelled or coerced to join labor unions even when
As such employee he was a member of the Elizalde Rope Workers’ said unions have closed shop agreements with the employers; that in
Union (Union) which had with the Company a collective bargaining spite of any closed shop agreement, members of said religious sects
agreement which reads: cannot be refused employment or dismissed from their jobs on the
“Membership in the Union shall be required as a condition of sole ground that they are not members of the collective bargaining
employment for all permanent employees workers covered union.
by this Agreement.” It is clear, therefore, that the assailed Act, far from infringing the constitutional
Under Republic Act No. 875, prior to its amendment by Republic Act provision on freedom of association, upholds and reinforces it.
No. 3350, the employer was not precluded (prevented) from: It does not prohibit the members of said religious sects from affiliating
Making an agreement with a labor organization to require as with labor unions. It still leaves to said members the liberty and the
a condition of employment membership therein, if such labor power to affiliate, or not to affiliate, with labor unions.
If the members of said religious sects prefer to sign up with the labor
union,
they can do so. If in deference and fealty to their religious faith, they It was intended to serve the secular purpose of advancing the
refuse to sign up, they can do so; the law does not coerce them to constitutional right to the free exercise of religion, by averting that
join; neither does the law prohibit them from joining; and neither may certain persons be refused work, or be dismissed from work, or be
the employer or labor union compel them to join. dispossessed of their right to work and of being impeded to pursue a
Republic Act No. 3350, therefore, does not violate the constitutional modest means of livelihood, by reason
provision on freedom of association.
On contract rights: the free exercise of religious profession or belief is
superior to contract rights. In case of conflict, the latter must,
therefore, yield to the former.
The Supreme Court of the United States has also declared on several
occasions that the rights in the First Amendment, which include
freedom of religion, enjoy a preferred position in the constitutional
system.
Religious freedom although not unlimited, is a fundamental personal
right and liberty, and has a preferred position in the hierarchy of
values.
It is only where unavoidable necessary to prevent an immediate and
grave danger to the security and welfare of the community that
infringement of religious freedom may be justified and only to
the smallest extent necessary to avoid the danger.
On violation of Sec. 1(7) of Article 3 of the 1935 Constitution (which
is Sec. 5 of Art. 3 of our 1987 Constitution): Any legislation
whose effect or purpose is to impede the observance of one or
all religions, or to discriminate invidiously between the
religions, is invalid, even though the burden may be
characterized as being only indirect.
But if the stage regulates conduct by enacting, within its power, a general
law which has for its purpose and effect to advance the state’s
secular goals, the statute is valid despite its indirect burden on
religious observance, unless the state can accomplish its purpose
without imposing such burden.
The statute, in order to withstand the strictures of constitutional
prohibition, must have:
a secular (non-religious) legislative purpose
a primary effect that neither advances nor inhibits religion.
Assessed by these criteria, Republic Act No. 3350 cannot be said to
violate the constitutional inhibition of the “no-establishment”
(of religion) clause of the constitution.
On the first requirement: The purpose of Republic Act No. 3350 is
secular, worldly, and temporal, not spiritual or religious or holy and
eternal.
of union security agreements.
To help its citizens to find gainful employment whereby they can make a living to
support themselves and their families is a valid objective of the state.
On the second requirement: The primary effects of the exemption from
closed shop agreements in favor of members of religious sects that
prohibit their members from affiliating with a labor organization, is the
protection of said employees against the aggregate force of the collective
bargaining agreement, and relieving certain citizens of a burden on their
religious beliefs; and by eliminating to a certain extent economic
insecurity due to unemployment, which is a serious menace to the
health, morals, and welfare of the people of the State, the Act also
promotes the well-being of society.
It is our view that the exemption from the effects of closed shop agreement does
not directly advance, or diminish, the interests of any particular religion.
Although the exemption may benefit those who are members of religious
sects that prohibit their members from joining labor unions, the benefit
upon the religious sects is merely incidental and indirect.
On the violation of the constitutional prohibition against requiring a religious
test fort eh exercise of a civil right: The Act does not require as a
qualification, or condition, for joining any lawful association membership
in any particular religion or in any religious sect.
Neither does the Act require affiliation with a religious sect that prohibits its
members from joining a labor union as a condition or qualification for
withdrawing from a labor union.
Joining or withdrawing from a labor union requires a positive act. Republic
Act No. 3350 only exempts members with such religious affiliation from
the coverage of closed shop agreements.
A conscientious religious objector need not perform a positive act or exercise the
right of resigning from the labor union·he is exempted from the coverage of
any closed shop agreement that a labor union may have entered into.
Fernando, Concurring: Fernando concurrs fully and entirely with the
Ponencia. Between the freedom of belief and the exercise of said belief,
there is quite a stretch of road to travel. If the exercise of said religious belief
clashes with the established institutions of society and with the law, then the
former must yield, and give way to the latter. The Government steps in and
either restrains said exercise or even prosecutes the one exercising it.
CANTWELL v. CONNECTICUT (HENRY) contribution was received, a pamphlet was delivered upon condition
May 20, 1940 | Roberts, J. | Freedom of Religion that it would be read.
Cassius street is in a thickly populated neighborhood, where about 90%
PETITIONER: Newton Cantwell, et al. of
RESPONDENTS: State of Connecticut

SUMMARY: Petitioner Cantwell and his two sons Jesse and Russel, are
members of a group known as Jehovah’s witnesses, and are claimint to be
ordained ministers. They were arrested in New Haven, Connecticut, and
each was charged by information in five counts, with statutory and common
law offenses.

After trial, each of them was convicted on the third count, which charged
a violation of 6294 of the General Statutes of Connecticut, and on the
fifth count, which charged commission of the common law offense of
inciting a breach of the peace. On appeal to the State SC, the conviction
of all three on the third count was affirmed. Jesse’s conviction on the fifth
count, was likewise affirmed.

Upon ruling of the US SC, the judgment is reversed, they held that such
acts of the petitioners are very well protected by the First and the
Fourteenth Amendments, that such acts do not pose as that of a clear
and present menace to public peace and order.

DOCTRINE: To condition the solicitation of aid for a perpetuation of


religious views or systems upon a license, the grant of which rests in the
exercise of a determination by state authority as to what is a religious
cause, is to lay a forbidden burden upon the exercise of liberty protected
by the Constitution.

FACTS:
6294 of the General Statutes of Connecticut provides, that no person
shall solicit money, services, subscriptions, or any valuable thing for
any alleged religious, charitable, or philantrophic cause, from other
than a member of the organization whose benefit such person is
oliciting or within the country in which such person or organization is
located unless such cause shall have been approved by the
secretary of the public welfare council.
Petitioners were engaged in going singly from house to house on
Cassius Street in New Haven. They were individually equipped with
a bag containing books and pamphlets on religious subjects, a
portable phonograph, and a set of records, each of which, when
played, introduced, and was a description of, one of the books.
Petitioner Cantwell and each of his sons asked the person who
responded to his call for permission to play one of the records. If
permissioin was granted, he asked the person to buy the book
described and, upon refusal, solicited such contributioin towards the
publication of the pamphlets as the listener was willing to make. If a
residents are Roman Catholics. A phonograph record, describing a The freedom to act msut have appropriate definition to preserve the
book entitled “enemies”, included an attack on the Catholic religion. enforcement of that protection. In every case the power to regulate
Petitioners are claiming that their activities were not within the statute but must be so exercised as not, in attaining a permissible end, unduly to
consisted only of distribution of books, pamphlets, and periodicals. infringe the protected freedom.
They are likewise assailing the statute’s validity as it constraints the
freedom to exercise religion.
State Supreme Court construed the finding of the trial court to be that “in
addition to the sale of the books and the distribution of the
pamphlets, the petitioners were also soliciting distributions or
donations of money for an alleged religious cause, and thereby came
within the purview of the statute.
State SC declared the statute constitutional as an effort by the State to
protect the public against fraud and imposition in the solicitation of
funds for what purported to be religious, charitable, or philanthropic
causes.
As far as the fifth count is observerd (the one where Jesse was
convicted), it was found that Jesse stopped two men in the street,
asked, and received, permission to play a phonograph record, and
played the record “enemies”, which attacked the religion and church
of the two men, who were Catholics.
Aggrieved, the petitioners filed the present motion.

ISSUE/s:
WoN the petitioner’s action was protected by the First and the
Fourteenth Amendments – YES

RULING: SC reversed the state SC ruling.

RATIO:
The statute, as construed and applied to the petitioners, deprives them of
their liberty without due process of law in contravention of the
Fourteenth Amendment. The fundamental concept of liberty
embodied in that Amendment emraces the liberties guaranteed by
the First Amendment.
The First Amendment declares that Congress shall make no law
respecting an establishment f religion or prohibiting the free exercise
thereof.
The Fourteenth Amendment has rendered legislatures of the statutes as
incompetent as Congress to enact such laws. The Constitutional
inhibition of legislation on the subject of religion has a double aspect.
On one hand, it forestalls compulsion by law of the acceptance of any
creed or practice of any form of worship. Freedom of conscience and
freedom to adhere to such religious organization or form of worship
as the individual may choose cannot be restricted by law. On the
other hand, it safegouards the free exercise of the chosen form of
religion.
The Amendment embraces two concepts: freedom to believe and
freedom to act. The first is absolute but, in the nature of things, the
second cannot be.
It is equally clear that a state may by general and non-discriminatory In the absence of a statute narrowly drawn to define and
legislation regulate the times, the places, and the manner of soliciting punish specific conduct as constituting a clear and
upon its streets, and of holding meetings thereon; and may in other present danger to a substantial interest of the State, the
respects safeguard the peace, good roder, and comfort of the petitioner’s communication, considered in the light of the
community,w ithotu unconstitutionally invading the liberties protected
by the Fourteenth Amendment.
The petitioners are right in their insistence that the Act in question is not
such a regulation. If a certificate is procured, solicitation is permitted
without restrained but, int eh absence of such certificate, solicitation
is altogether prohibited.
Petitioners claim that to require them to obtain a certificate as a condition
of soliciting support for their views amounts to a prior restraint on the
exercise of their religion within the meaning of the Constitution.
Penal laws are available to punish such conduct that may commit, with
impunity, frauds upon the public. Even the exercise of religion may
be at some slight inconvenience in order that the state may protect
its citizens from injury. Without a doubt a state may protect its
citizens from fraudulent solicitation by requiring as tranger in the
community, before permitting him publicly to solicit funds for any
purpose, to establish his identity and his authority to act for the
cause which he purports to represent.
The state is likewise free to regulate the time and manner of solicitation
generally, in the interest of public safety, peace, comfort, or
convenience.
But to condition the solicitation of aid for the perpetuation of
religious views or systems upon a license, the grant of which
rests in the exercise of a determinationby state authority as to
what is a religious cause, is to lay a forbidden burden upon the
exercise of liberty protected by the Constitution.
The SC held that the conviction of Jesse Cantwell on the fifth count must be
set aside. The fundamental law declares that interest of the US that the
free exercise of religion be not prohibited and that freedom to
communicate information and opinion be not abridged. Connecticut has
an obvious interest in the preservation and protection of peace and good
order within her borders.
The conviction on the fifth count was not pursuant to a statute revealing a
legislative judgment that street discussioin of religious affairs,
because of its tendency to provoke disorder, should be regulated, or
a judgment that the playing of a phonograph on the streets should in
the interest of comfort or privacy be limited or prevented.
SC noted that Jesse Cantwell, was upon a public street, where he had a
right to be, and where he had a right peacefully to impart his views to
others. There is no showing that his deportment was noisy, truculent,
overbearing, or offensive. He requested two pedestriants to listen,
which was granted. It is plain that he wished only to interest him with
his propaganda, hence he invaded no right or interest of the public or
of the men accosted.
constitutional guarantees, raised no clear and present menace to public
peace and order as to render him liable to conviction of the common law
offense in question.
US v. BALLARD (DANNAH) The Ballards moved to quash and they asserted that the indictment
April 24, 1944 | Douglas, J. | Good Faith in Believing attacked their religious beliefs and sought to restrict the free exercise
of religion in violation of the Constitution.
PETITIONER: United States Their motions were denied by the District Court.
RESPONDENTS: Guy Ballard, Edna and Donald Ballard It stated that “As far as this Court sees the issue, it is immaterial what
these defendants preached or wrote or taught in their classes….
SUMMARY: Upon an indictment charging use of the mails to defraud,
Therefore, the religious beliefs of these defendants cannot be an
and conspiracy so to do, the Ballards were convicted in the District Court.
issue in this court. The issue is: Did they honestly and in good faith
The indictment charged a scheme to defraud through representations—
believe those things? If they did, they should be acquitted… If not,
involving respondents' religious doctrines or beliefs—which were alleged
the jury should find them guilty. Therefore, religion cannot come into
to be false and known by the respondents to be false. this case”
Holding that the District Court had restricted the jury to the issue of The Circuit CA reversed the judgment of conviction and granted a new
respondents' good faith and that this was error, the Circuit Court of trial. In its view, the restriction of the issue in question to that of good
Appeals reversed and granted a new trial. The Supreme Court ruled that faith was error.
the action of the District Court was valid and correct, because questions Its reason was that the scheme to defraud contained allegations of false
regarding the truth and falsity of the Ballards’ religious beliefs or doctrines representations, and that it was necessary to prove that they
is protected under the First Amendment’s guarantee of religious freedom. schemed to make some of the representations, and that some were
false.
DOCTRINE: The Amendment embraces two concepts: freedom to Hence, this petition for certiorari which the Court granted due to
believe and freedom to act. The first is absolute but, in the nature of importance of questions presented.
things, the second cannot be.
ISSUE/s:
WoN the District Court was correct in confining the issue to whether or
FACTS: not the Ballards believed their representations to be true – YES
The Ballards were indicted and convicted for using, and conspiring to
use, the mails to defraud. RULING: The judgment is reversed and the cause is remanded to the Circuit
They created the I Am movement, which covered their religious beliefs. Court of Appeals for further proceedings in conformity to this opinion.
Some of the charges stated that:
Guy Ballard was selected and designated by “ascertained masters.” And RATIO:
St. Germain’s words would be transmitted to mankind through the The US contends that the District Court withdrew from the jury's
medium said Guy Ballard consideration only the truth or falsity of those representations which
That Guy Ballard, with Edna and Donald Ballard, by reason of their related to religious concepts or beliefs and that there were
alleged high spiritual attainments and righteous conduct, had been representations charged in the indictment which fell within a different
selected as divine messengers through which the words of the category.
alleged “ascended masters” would be communicated to mankind Hall v. US states
through the I Am movement Where an indictment contained 'all the necessary averments to
That they said they had the power to heal persons of ailments and constitute an offense created by the statute', a conviction would not
deceases
be set aside because a 'totally immaterial fact' was averred but not
That the Ballards well knew their representations were false proved.
The charges stated that the Ballards made the representations with the The Court is of the view that all of the representations charged in the
intention to cheat, wrong and defraud persons, to obtain from those indictment which related at least in part to the religious doctrines or
defrauded money, property and things of value beliefs of the Ballards were withheld from the jury.
The indictment contained 12 counts, one of which charged a conspiracy
to defraud.
The trial judge did not differentiate them. He referred in the charge to the
'religious beliefs' and 'doctrines taught by the defendants' as matters
withheld from the jury.
And in stating that the issue of good faith was the 'cardinal question' in
the
case he charged, as already noted, that 'The jury will be called upon to trial before a jury charged with finding their truth or falsity, then
to pass on the question of whether or not the Ballards honestly and the same can be done with the religious beliefs of any sect.
in good faith believed the representations which are set forth in the When the triers of fact undertake that task, they enter a forbidden
indictment domain. The First Amendment does not select any one group or
A careful reading of the whole charge leads us to agree with the Circuit any one type of religion for preferred treatment. It puts them all in
Court of Appeals on this phase of the case that the only issue that position.
submitted to the jury was the question as stated by the District Court,
of Ballards' belief in their representations and promises.’
The Amendment embraces two concepts: freedom to believe and
freedom to act. The first is absolute but, in the nature of things, the
second cannot be.
Freedom of thought, which includes freedom of religious belief, is basic
in a society of free men. It embraces the right to maintain theories of
life and of death and of the hereafter, which are rank heresy to
followers of the orthodox faiths.
Heresy trials are foreign to our Constitution. Men may believe what they
cannot prove. They may not be put to the proof of their religious
doctrines or beliefs. Religious experiences which are as real as life to
some may be incomprehensible to others.
Yet the fact that they may be beyond the ken of mortals does not
mean that they can be made suspect before the law. Many take
their gospel from the New Testament. But it would hardly be
supposed that they could be tried before a jury charged with the duty
of determining whether those teachings contained false
representations.
The miracles of the New Testament, the Divinity of Christ, life after death,
the power of prayer are deep in the religious convictions of many. If
one could be sent to jail because a jury in a hostile environment
found those teachings false, little indeed would be left of religious
freedom.
The Fathers of the Constitution were not unaware of the varied and
extreme views of religious sects, of the violence of disagreement
among them, and of the lack of any one religious creed on which all
men would agree.
They fashioned a charter of government which envisaged the widest
possible toleration of conflicting views. Man's relation to his God was
made no concern of the state. He was granted the right to worship as
he pleased and to answer to no man for the verity of his religious
views.
The religious views espoused by the Ballards might seem incredible, if
not preposterous, to most people. But if those doctrines are subject
So the Court concludes that the District Court ruled properly when it withheld
from the jury all questions concerning the truth or falsity of the religious
beliefs or doctrines of the Ballards.
Stone, Dissenting:
Not prepared to say that the constitutional guaranty of freedom of religion affords
immunity from criminal prosecution for the fraudulent procurement of money
by false statements as to one's religious experiences.
The state of one’s mind is a fact as capable of fraudulent misrepresentation
as is one's physical condition or the state of his bodily health. Since the
indictment and the evidence support the conviction, it is irrelevant
whether the religious experiences alleged did or did not in fact occur or
whether that issue could or could not, for constitutional reasons, have
been rightly submitted to the jury.
Certainly none of the Ballards’ constitutional rights are violated if they are
prosecuted for the fraudulent procurement of money by false
representations as to their beliefs, religious or otherwise.
Jackson, Dissenting:
Sees that the Ballard’s teachings were untained by any trace of truth. But
that does not dispose of the constitutional question whether
misrepresentation of religious experience or belief is prosecutable; it
rather emphasizes the danger of such prosecutions.
Difficulty to reconcile conclusion with traditional religious freedoms because
first, as a matter of either practice of philosophy, he does not see how
the issue can be separated as to what is believed from considerations
as to what is believable and second, any inquiry into intellectual honesty
in religion raises profound psychological problems
The wrong of these things is not the money the victims part with half so
much as in the mental and spiritual poison they get. But that is precisely
the thing the Constitution put beyond the reach of the prosecutor, for the
price of freedom of religion or of speech or of the press is that we must
put up with, and even pay for, a good deal of rubbish.
It is an impossible task for juries to separate fancied experiences from real
ones. Prosecutions of this character easily could degenerate into
religious persecution.
Would dismiss indictment and have done with this business of judicially
examining other people’s faiths.
AMERICAN BIBLE SOCIETY v . CITY OF MANILA (ARIELLE) business, American Bible Society paid under protest, but notifying the
April 30, 1957 | Felix, J. | Freedom of Religion City Treasurer that suild would be taken in court to question the legality
of the ordinances
PETITIONER: Federal Communications Commission
RESPONDENTS: Pacifica Foundation

SUMMARY: American Bible Society was being taxed by the City of


Manila, pursuant to various ordinances, for selling and distributing bibles
and other religious literature. American Bible Society claims that it cannot
be taxed by the city and to do so would be violative of their right to the
free exercise and enjoyment of religious profession. The lower court
dismissed the case for lack of merit. The CA certified the case to the SC
for the reason that errors assigned to the lower court involved only
questions of law.

The SC reversed the decision appealed from, and ordered the City of
Manila to return the amount paid by the American Bible Society. The
Court held that while the price asked for the bibles was in some instances
a little bit higher than the actual cost of the same, it still cannot mean that
American Bible Society was engaged in the business of selling for profit.
For this reason, the Court holds that the provisions of Ordinance No.
2529 cannot be applied to American Bible Society, for in doing so it would
impair its free exercise and enjoyment of its religious profession and
worship as swell as its rights of dissemination of religious beliefs.

DOCTRINE: The constitutional guarantee of the free exercise and


enjoyment of religious profession and worship carries with it the right to
disseminate religious information. Any restraint of such right can only be
justified like other restraints of freedom of expression on the grounds that
there is a clear and present danger of any substantive evil which the
State has the right to prevent.

FACTS:
Plaintiff American Bible Society is a foreign, non-stock, non-profit,
religious, missionary corporation duly registered and doing business
in the Philippines through its Philippine agency.
American Bible Society has been distributing and selling bibles and/or
gospel portions (except during the Japanese occupation) throughout
the Philippines and translating the same into several Philippine
dialects.
The acting City Treasurer of Manila informed American Bible Society that
it was conducting business without the necessary Mayor’s permit
and license, in violation of Ordinance No. 3000, and required plaintiff
to secure, within three days, the corresponding permit and license
fees.
American Bible Society protested against the requirement, but the Treasurer
demanded that plaintiff deposit and pay. To avoid the closing of its
under which the said fees were being collected. No. 79, which reads: all other businesses, trades or occupations not
In its complaint, American Bible Society prays that judgment be rendered mentioned in this Ordinance, except those upon which the City is not
declaring the said Municipal Ordinance illegal and unconstitutional, empowered to license or
and that the City Treasurer be ordered to refund what it paid.
Defendant City Treasurer answered the complaint, maintaining that said
ordinances were enacted by the Municipal Board of the City of
Manila by virtue of the power granted to it by the Revised
Administrative Code.
When the case was set for hearing, American Bible Society provided that
it has been in existence since 1899, and that its parent society is in
New York, and that its was never required to pay any municipal
license fee before the war, nor does the American Bible Society in
the US pay any license fee for the sale of bible.
American Bible Society also claimed that it never made any profit from
the sale of its bibles, and to maintain its operating cost, it obtains
substantial remittances from its New York office and voluntary
contributions and gifts from certain churches both in the US and in
the PH.
The Court rendered judgment which dismissed the case for lack of merit.
The American Bible Society appealed the matter to the CA which
certified the case to the SC for the reason that the errors assigned to
the lower court involved only questions of law.

ISSUE/s:
WoN the ordinances are applicable to plaintiff American Bible Society – NO

RULING: We hereby REVERSE the decision appealed from, sentencing


defendant City of Manila to return to plaintiff American Bible Society the sum
of P5,391.45 unduly collected from it.

RATIO:
American Bible Society anchors its argument on Sec. 1, subsection (7) of
Art. III of the Constitution which provides, “No law shall be made
respecting an establishment of religion, or prohibiting the free
exercise thereof, and the free exercise and enjoyment of religious
profession and worship, without discrimination or preference, shall
forever be allowed…”
Plaintiff contends that the ordinance is unconstitutional because they
provide for religious censorship and restrain the free exercise and
enjoyment of its religious profession, which is the distribution and
sale of bibles.
The ordinance in question is of general application and is not directed
against institutions like the American Bible Society, and it does not
contain any provisions prescribing religious censorship nor
restraining the free exercise and enjoyment of any religious
profession.
The business of American Bible Society involved in this case is not
mentioned in Setion 3 of the Ordinance, and the record does not show
that a permit is required. However, Sec. 3 of the ordinance contains item
to tax…
Therefore, the necessity of the permit is made to depend upon the power
of the City to license or tax said business. The license fees required
to be paid are not imposed directly upon any religious institution but
upon those engaged in any of the business or occupations
enumerated such as retail “dealers in general merchandise” which, it
is alleged, cover the business or occupation of selling bibles.
American Bible Society contends that Ordinances Nos. 2779, 2821, and
3028, are unconstitutional and illegal because it restrains the free
exercise and enjoyment of the religious profession and worship of
appellant.
Art. III Section 1, clause (7) of the Constitution guarantees the freedom of
religious profession and worship. “Religion has been spoken of as a
profession of faith to an active power that binds and elevates man to
its Creator.”
The constitutional guarantee of the free exercise and enjoyment of
religious profession and worship carries with it the right to
disseminate religious information. Any restraint of such right can
only be justified like other restraints of freedom of expression on the
grounds that there is a clear and present danger of any substantive
evil which the State has the right to prevent.
In the case at bar, the license fee involved is imposed upon American Bible
Society for its distribution and sale of bibles and other religious literature.
In Murdock v. Pennsylvania, the Court held that an ordinance requiring
a license be obtained before a person could canvass orders for goods,
paintings, etc. cannot be made to apply to members of Jehovah’s
Witnesses who went around door to door distributing literature published
by the Watch Tower Bible & Tract Society. The Court also held in that
case that religious groups and the press are not absolutely free from all
financial burdens of government. It is one thing to impose a tax on the
income or property of a preacher, and another to impose tax from him for
the privilege of delivering a sermon.
While the price vasked for the bibles was in some instances a little bit higher
than the actual cost of the same, it still cannot mean that American Bible
Society was engaged in the business of selling for profit. For this reason,
the
Court holds that the provisions of Ordinance No. 2529 cannot
be applied to American Bible Society, for in doing so it would
impair its free exercise and enjoyment of its religious
profession and worship as swell as its rights of dissemination
of religious beliefs.
However, the ordinance cannot be considered unconstitutional even if
applied to plaintiff, but as it is not applicahble to plaintiff and the City
of Manila is powerless to tax the business of American Bible Society,
the Court finds that ordinance No. 3000 is also inapplicable to said
business of American Bible Society.
ANG LADLAD v. COMELEC (IYA) COMELEC reiterated that Ang Ladlad does not have a concrete and
April 8, 2010 | Del Castillo, J. | Freedom of Religion genuine national political agenda to benefit the nation and that the
petition was validly dismissed on moral grounds

PETITIONER: Ang Ladlad LGBT Party


RESPONDENTS: Commission on Elections

SUMMARY: Ang Ladlad, an LGBT organization, seeks accreditation with


the COMELEC as a party list group.The COMELEC denies their petitions
on moral grounds saying that having the LGBT community represented
would contravene moral principles of Muslims and Christians.

The Supreme Court ruled in favor of Ang Ladlad, saying the COMELEC
erred in using moral disapproval as sufficient interest to justify the denial of
accreditation. The Constitution calls for governmental neutrality in religious
matters.

DOCTRINE : What the non-establishment clause calls for is “government


neutrality” in religious matters. It was grave violation of the non-
establishment clause for the COMELEC to utilize the Bible and the
Koran to justify the exclusion of Ang Ladlad.

FACTS:
Ang Ladlad is an organization composed of men and women who identify
themselves as lesbians, gays, bisexuals, or transgendered
individuals.
In 206, they first applied for registration with the COMELEC, but was denied
on the ground that the organization has no substantial membership
base.
In 2009, they filed again for a petition for registration. COMELEC
dismissed the petition on moral grounds stating that the LGBT sector
tolerates immorality which offends religious beliefs.
On MR, the COMELEC Chairman issued a Resolution which upheld the
dismissal arguing that Ladlad’s expressed sexual orientations would
benefit the nation as a whole and that it would contravene moral
principles of the Muslim and Christian faith.
On 2010, Ang Ladlad filed this present Petition praying that the Court
annul the Resolutions and direct the COMELEC to grant And
Ladlad’s application for accreditation.
The SC issued a TRO during the pendency of the case.
The Commission on Human Rights filed a Motion to Intervene and
opined that the denial of And Ladlad’s petition on moral grounds
violated the standard and principles f the Constitution, the UDHR,
and the ICCPR.
Ang Ladlad argued that the denial of accreditation insofar as it justified
the exclusion by using religious dogma, violated the constitutional
guarantees against the establishment of religion.
(not relevant but just in case) COMELEC also argued for the first time that Freedom of expression constitutes one of the essential foundations of a
the LGBT sector is not among the sectors enumerated by the democratic society, and this freedom applies not only to those that
Constitution and RA 7941, and that Ang Ladlad mad untruthful are favorably received but also to those that offend, shock, or disturb.
statements in its petition when it alleged its national existence contrary Any restriction imposed in this sphere must be proportionate to the legitimate
to a actual verification reports.

ISSUE/s:
WON the denial of accreditation violated constitutional guarantees
against establishment of religion - YES

RULING: Petition is GRANTED. The Commission is directed to GRANT Ang


Ladlad’s application for party-list accreditation.

RATIO:
The Court finds that Ang Ladlad has sufficient demonstrated its
compliance wit the legal requirements for accreditation.
Aside from COMELEC’s moral objection and the belated allegation of
non-existence, nowhere in the records has COMELEC ever
found/ruled that Ang Landlad is not qualified to register as a party-list
organization under any of the requisites under RA 7941 or the
guidelines in And Bagong Bayani.
Article III Sec. 5 of the Constitution provides that “no law shall be made
respecting an establishment of religion, or prohibiting the free
exercise thereof.”
What the non-establishment clause calls for is “government
neutrality” in religious matters.
It was grave violation of the non-establishment clause for the
COMELEC to utilize the Bible and the Koran to justify the
exclusion of Ang Ladlad.
COMELEC suggests that although the moral condemnation of
homosexuality and homosexual conduct may be religion-based, it
has long been transplanted into generally accepted public morals.
The Court disagrees with such opinion, the Philippines has not seen fir to
criminalize homosexual conduct. Evidently, these “generally accepted
public morals” have not been convincingly transplanted into the realm of
law.
The court holds that moral disapproval is not sufficient governmental
interest to justify exclusion of homosexuals form participation in the
party-list system.
The denial of Ang Ladlad’s registration on purely moral grounds
amounts more to a statement of dislike and disapproval of
homosexuals rather than a tool to further any substantial public
interest.
Under our system of laws, every group has the right to promote its
agenda and attempt to persuade society of the validity of its position
through normal democratic means.
aim pursued.
Absent any compelling state interest, it is not for the COMELEC or
this Court to impose its views on the populace.
Both expressions concerning one’s homosexuality and the activity of forming
a political association that supports LGBT individuals are protected as
well.
The present decision is fully in accord with the State’s international
obligations to protect and promote human rights. In particular, we
explicitly recognize the principle of non-discrimination as it relates to
the right to electoral participation, enunciated in the UDHR and the
ICCPR.
Puno, separate concurring: The assailed resolutions of the COMELEC
are violative of the constitutional directive that no religious test shall
be required for the exercise of civil or political rights. Ang Ladlad’s
right of political participation was unduly infringed when the
COMELEC, swayed by the private biases and personal prejudices of
its constituent members, arrogated unto itself the role of a religious
court or worse, a morality police.
Corona, dissenting: The enumeration of sectors considered as
marginalized and underrepresented in the fundamental law and in
the implementing law (RA 7941) cannot be without significance. To
ignore them is to disregard the texts of the Constitution and of RA
7941. As Ang Ladlad does not fall under the categories listed, the
party not being a marginalized sector, it should not be granted
accreditation.
Abad, separate opinion: Justice Abad declares that he fully agrees that
the COMELEC erred and discriminated when it denied Ang Ladlad’s
petition for sectoral party accreditation on religious and moral
grounds. The COMELEC has never applied these tests on regular
candidates for Congress. There is no reason for it to apply them on
Ang Ladlad.
EBRALINAG v. DIVISION SUPERINTENDENT (ELIEL) power and invades the sphere of the intellect and spirit which the
March 1, 1993 | Grino-Aquino, J. | Freedom of Religion Constitution protects against

PETITIONER: Roel Ebralinag et al.


RESPONDENTS: The Division Superintendent of Schools of Cebu

SUMMARY: The Petitioners in this case are school children under the
religious sect known as Jehovah’s Witnesses. Apparently, Jehovah’s
Witnesses teach their believers to refrain from saluting the Philippine flag,
sing the national anthem and recite the patriotic pledge. This is in line of
their belief not to embrace idols. The Divison superintendent then
expelled them in accordance to the law. Hence this petition.

The SC held that to compel them to salute, sing, and recite the pledge
would be in violation of their constitutional rights to freedom of religion.
The sole justification for prior restraint is when there is a presence of
grave danger. In this case, there is no danger; rather, the restraint would
only create the danger being avoided. However, they have no right to
cause any disruptions but must show respect instead.

DOCTRINE : “The right to religious profession and worship has a wofold


aspect, vis., freedom to believe and freedom to act on one’s belief. The
first is absolute as long as the belief is confined within the realm of
thought. The second is subject to regulation where the belief is transalted
into external acts that affect the public welfare.”

FACTS:
All the petitioners in these two cases were expelled from their classes by
the public school authorities in Cebu for refusing to salute the flagg,
sing the national anthem and recite the patriotic pledge as required
by RA 1265 of July 11, 1955, and by Department Order No. 8 dated
July 21, 1955 of the Deparment of Education, Culture and Sports
(DECS) making the flag ceremony compulsory in all educational
institutions.
RA 1265: “In case of failure to observe for the second time the flag
ceremony provided by this Act, the Secretary of Education, after
proper notice and hearing, shall cause the cancellation of the
recognition or permit of the private education institution responsible
for such failure.”
Jehovah’s Witnesses admittedly teach their children not to salute the
flag, sing the national anthem, and recit the patriotic pledge for they
believe that hose are “acts of worship” or “religious devotions” which
they “cannot conscientiously give xxx to anyone or anything except
God”
They feel bound by the Bible’s command to “guard ourselves fmor idols – 1
John 5:21.” They consider the falg as an image or idol representing the
State. They think the action of the local authorities in compelling the flag
salute and pledge transcends constitutional limitations on the State’s
official control. with reverence for the flag, patriotism, love of country and admiration for
Gerona case: “The flag is not an image but a symbol of the Republic of national heroes”
the Philippines, an emblem of national sovereignty, of national unity
and cohesion and of freedom and liberty which it and the
Constitution guarantee and protect. Under a system of complete
separation of church and state in the government, the flag is utterly
devoid of any religious significance. Saluting the flag does not
involve any religious ceremony. The flag salute is no more a religious
ceremony than the taking of an oath of office by a public official or by
a candidate for admission to the bar.”
The Court issued a temporary restraining order and a writ of preliminary
mandatory injunction commanding the respondents to immediately
readmit the petitioners to their respective classes until further orders
from this Court.

ISSUE/s:
WoN Jehovah’s witnesses may be expelled for not saluting the flag – NO

RULING: Prohibition is granted.

RATIO:
The idea that one may be compelled to salute the flag, sing the national
anthem, and recite the patriotic pledge, during a flag ceremony on
pain of being dismissed from one’s job or of being expelled from
school, is alien to the conscience of the present generation of
Filipinos who cut their teeth on the Bill of Rights which guarantees
their rights to free speech and the free exercise of religious
profession and worship.
Religious freedom is a fundamental right which is entitled to the
highest priority and the amplest protection among human
rights, for it involves the relationship of man to his Creator.
“The right to religious profession and worshiop has a wofold
aspect, vis., freedom to believe and freedom to act on one’s
belief. The first is absolute as long as the belief is confined
within the realm of thought. The second is subject to regulation
where the belief is transalted into external acts that affect the
public welfare.”
The sole justification for a prior restratint or limitation on the
exercise of religious freedom is the existence of a grave and
present danger of a character both grave and imminent, of a
serious evil to public safety, public morals, public health or any
other legitimate public interest, that the State has a right (and
duty) to prevent.” Absent such a threat to public safety, the
expulsion of the petitioners from the schools is not justified.
We are not persuaded that by exempting the Jehovah’s Witnesses from
saluting the flag, singing the national anthem and reciting the patriotic
pledge, this religious group which admittedly comprises a “small portion
of the school population” will shake up our part of the globe and
suddenly produce a nation” untaught and uninculcated in and unimbued
After all, what the petitioners seek only is exemption from the falg
ceremony, not exclusion from the public schools where they
may study the Constitution, the democratic way of life and form
of government, and learn not only the arts, sciences, Philippine
history and culture but also receive training for a vocation or
profession and be taught the virtues of “patriotism, respect for
human rights, appreciation for national heroes, the rights and
duties of citizenship, and moral and spiritual values as part of
the curricula.”
Expelling or banning the petitioners from Philippine schools will bring
about the very situation that this court had feared in Gerona. Forcing
a small religious group, through the iron hand of the law, to
participate in a ceremony that violates their religious beliefs, will
hardly be conducive to love of country or respect for duly constituted
authorities.
Moreover, the expulsion of members of Jehovah’s Witnesses from
the schools where they are enrolled will violate their right as
Philippine citizens, under the 1987 Constitution, to receive free
education, for it is the duty of the State to “protect and promote
the right of all citizens to quality education xxx and to make
such education accessible to all.”
Nevertheless, their right not to participate in the flag ceremony does not
give them a right to disrupt such patriotic exercises.
While the highest regard must be afforeded their right to the free
exercise of their religion, “this should not be taken to mean that
school authorities are powerless to discipline them” if they
should commit breaches of the peace by actions that offend th
sensisbilites, both religious and patriotic, of other persons.
Cruz, J. Concurring: It seems to me that every individual is entitled to
choose for himself whom or what to worship or whether to worship at
all. This is a personal decision he alone can make. The individual
may worship a spirit or a person or a beast or a tree and the State
cannot prevent him from doing so. For that matter, neither can it
compel him to do so. As long as his beliefs are not externalized in
acts that offend the public interest, he cannot be prohibited from
harboring them or punishe for doing so.
The State cannot interpret the Bible for them; only they can read it as
they see fit. Right or wrong, the meaning they derive from it cannot
be revised or reversed except perhaps by their own acknowledge
superiors. But certainly not the State. It has no competence in this
matter. Religion is forbidden territory that the State, for all its power
and authority, cannot invade.
As a valid form of expression, it cannot be compelled any more than it
can be prohibited in the face of valid religious objections like those
raised in this petition. To impose it on the petitioners is to deny them
the right not to speak when their religion bids them to be silent.
Padilla, J. Separate: For a select few to be exempt from the flag
ceremony and all that it represents even if the exemption is
predicated on respect for religious scruples, could be divisive in its
impact on the school population or community.
MASIKIP v. PASIG (DANNAH) of her lot is neither sufficient nor suitable to “provide land
June 23, 2006 | Sandoval-Gutierrez, J. | Eminent Domain opportunities to deserving poor sectors of our community.”

PETITIONER: Lourdes de la Paz Masikip


RESPONDENTS: City of Pasig, Hon Legaspi, Court of Appeals

SUMMARY: Respondent City of Pasig, pursuant to Ordinance No. 42


wrote a letter to Masikip, notifying her of its intention to expropriate a 1.5k
sqm portion of her property used for the “sports development and
recreational activities”. Masikip sent a reply stating that the intended
expropriation is unconstitutional.

Pasig filed a complaint for expropriation, and Masikip filed a Motion to


Dismiss. The RTC issued an Order denying the Motion to Dismiss,
stating that there was a genuine necessity to expropriate the property for
the sports and recreational activities of the residents of Pasig. The CA
affirmed.

The SC ruled that, especially since records show that there is already an
existing established sports and recreational facility. Moreover, the
intended beneficiary was not the residents of Caniogan, but the
Melendres Compound Homeowners Association. Hence, indeed, there is
no genuine necessity to expropriate Masikip’s property.

DOCTRINE: The very foundation of the right to exercise eminent domain


is a genuine necessity, and that necessity must be of a public character.
Moreover, the ascertainment of the necessity must precede or
accompany and not follow, the taking of the land.

FACTS:
Petitioner Masikip is the registered owner of a parcel of land with an area
of 4k+ sqm located in Caniogan, Pasig.
In a letter dated January 6, 1994, the then Municipality of Pasig, now City of
Pasig, respondent, notified petitioner of its intention to expropriate a
1,500 square meter portion of her property to be used for the “sports
development and recreational activities” of the residents of Barangay
Caniogan.
This was pursuant to Ordinance No. 42, Seriss of 1993 enacted by the
then Sangguinang Bayan of Pasig.
Again, on March 1994, respondent Pasig wrote another letter to Masikip,
but this time the purpose was allegedly “in line with the program of
the Municipal Government to provide land opportunities to deserving
poor sectors of our community.”
On May 1994, Masikip sent a reply to Pasig stating that the intended
expropriation is unconstitutional, invalid and oppressive as the area
In its December 1994 letter, Pasig reiterated that the purpose was to petitioner filed her motion to dismiss should govern. The new
“provide sports and recreational facilities to its poor residents.” provision cannot be applied retroactively to her prejudice
Subsequently, Pasig on February 1995 filed with the trial court a
complaint for expropriation, praying that the trial court issues an
order for the condemnation of the property; that commissioners be
appointed for the purpose of determining the just compensation; and
that judgment be rendered based on the report of the
commissioners.
Masikip filed a Motion to Dismiss, averring that Pasig has no cause of
action for the exercise of eminent domain (no genuine necessity for
the taking), and that the complaint is defective in form and
substance.
The trial court issued an Order denying the Motion to Dismiss, on the
ground that there is a genuine necessity to expropriate the
property for the sports and recreational acitivities of the
residents of Pasig.
Masikip’s MR was also denied, and the CA denied her Motion for
Reconsideration, hence this petition.

ISSUE/s:
WoN there is a genuine interest requiring the expropriation of Masikip’s
land in question – NO

RULING: Court of Appeals decision affirmed.

RATIO:
Procedural issue: the motion to dismiss clearly constitutes the
responsive pleading which takes the place of an answer to the
complaint for expropriation. Such motion puts in issue the right of
Pasig to expropriate Masikip’s property for the use specified.
The CA erred in holding that the motion to dismiss filed by Masikip
hypothetically admitted the truth of the facts alleged in the complaint,
“specifically that there is a genuine necessiry to expropriate her
property for public use”
Pursuant to Rule 67 of the Revised Rules of Court, the motion is a
responsive pleading joining the issues.
What the trial court should have done was to set the case for the
reception of evidence to determine whether there is indeed a
genuine necessity for the taking of the property, instead of summarily
making a finding that the taking is for public use and appointing
commissioners to fix just compensation.
The fact that the CA rendered it Decision after the 1997 Rules of Civil
Procedure is of no moment. It is only fair that the Rule at the time
Substantive: In US v. Toribio, the Court defined the power of eminent Applying this standard, the Court holds that respondent Pasig has failed
domain as “the right of a government to take and appropriate private to
property to public use, whenever the public exigency requires it, which
can be done only on condition of providing a reasonable compensation
therefor”
It has also been described as the power of the State or its
instrumentalities to take private property for public use and is
inseparable from sovereignty and inherent in government.
This power is lodged in the legislative branch of the government. It
delegates the exercise thereof to LGUS, other public entities and
public utility corporations, subject only to Constitutional limitations.
LGUs have no inherent power of eminent domain and may exercise it
only when expressly authorized by statute.
Section 19 of the Local Government Code prescribes the delegation by
Congress of the power of eminent domain to local government units
and lays down the parameters for its exercise.
Judicial review of the exercise of eminent domain is limited to the
following areas of concern:
The adequacy of compensation
The necessity of the taking
The public use character of the purpose of the taking
In the case at hand, Masikip contends that respondent Pasig failed to
establish a genuine necessity which justifies the condemnation of her
property.
While she does not dispute the intended public purpose, nonetheless,
she insists that there must be a genuine necessity for the proposed
use and purposes.
According to Masikip, there is already an established sports development
and recreational activity center at Rainforest Park in Pasig City, fully
operational and being utilized by its residents, including those from
Barangay Caniogan.
Respondent Pasig does not dispute this. Evidently, there is no “genuine
necessity” to justify the expropriation.
City of Manila v. Chinese: The very foundation of the right to exercise
eminent domain is a genuine necessity, and that necessity must be
of a public character. Moreover, the ascertainment of the necessity
must precede or accompany and not follow, the taking of the land.
City of Manila v. Arellano Law College: The necessity within the rule
that the particular property to be expropriated must be necessary,
does not mean an absolute but only a reasonable or practical
necessity, such as would combine the greatest benefit to the public
with the least inconvenience and expense to the condemning party
and the property owner consistent with such benefit.
establish that there is a genuine necessity to expropriate Masikip’s property.
The scrutiny of the records shows that the Certification issued by the
Caniogan Barangay Council, the basis for the passage of Ordinance 42
authorizing the expropriation, indicates that the intended beneficiary is
the Melendres Compound Homeowners Association.
It is a private, nonprofit organization. The beneficiary then is not the residents
of Caniogan.
It can be gleaned that members of said Association are desirous of having
their own private playground and recreational facility.
Masikip’s lot is the nearest vacant space available. The purpose is,
therefore, not clearly and categorically public.
The necessity has not been shown, especially since there exists an
alternative facility for sports development.
The right to own and possess property is one of the most cherished rights of
men. Unless the requisite of genuine necessity for the expropriation of
one’s property is clearly established, it shall be the duty of the courts to
protect the rights of individuals to their private property.
Important as the power of eminent domain may be, the inviolable sanctity
which the Constitution attaches to the property of the individual requires
not only that the purpose for the taking of private property be specified.
The genuine necessity for the taking, which must be of a public
character, must also be shown to exist.
MCIA v. LOZADA (ARIELLE) The trial court rendered a judgment in favor of the Republic and ordered
February 25, 2010 | Nachura, J. | Eminent Domain; Repurchase the latter to pay Lozada the fair market value at P3.00 per square
meter, with
PETITIONER: Mactan-Cebu International Airport Authority
RESPONDENTS: Bernardo L. Lozada, Sr.

SUMMARY: The subject lots were originally owned by Anastacio, but was
transferred to Lozada. It was occupied by the US Army and the Government,
for purposes of the expansion and improvement of the Lahug Airport, filed an
expropriation proceeding. The trial court rendered a judgment in favor of the
Republic, and ordered the latter to pay Lozada the fair market value at P3.00
per square meter. The affected landowners appealed. The Air Transporation
Office (ATO) formerly CAA, proposed a compromise settlement whereby the
owners affected by the expropriation would either not appeal or withdraw
their appeals in consideration of a commitment that the expropriated lots
would be resold at the price they were expropriated in the event that the ATO
would abandon the Lahug Airport. Because of this, Lozada did not pursue his
appeal. Thereafter, the lot was transferred and registered in the name of the
Republic. The improvement and expansion plan of the Lahug Airport,
however, was not pursued. In turn, Lozada wanted to repurchase the
property back. The issue is WoN the Republic is obliged to return the lot by
reason of the abandonment of the project, to Lozada.

The SC held that the condition not having materialized because the
airport had been abandoned, the former owner should then be allowed to
reacquire the expropriated property.

DOCTRINE: The taking of private property, is always subject to the


condition that the property be devoted to the specific public purpose
for which it was taken. If this particular purpose or intent is not initiated or
pursued,
then the former owners, if they desire, may seek the reversion of the
property, subject to the return of the amount of just compensation
received.

FACTS:
The disputed land located in Lahug, Cebu City, is owned by Anastacio
Deiparine, when the same was subject to expropriation proceedings
initiated by the Republic, for the expansion and improvement of the
Lahug Airport.
As early as 1947, the lots were already occupied by the US Army, and
was turned over to the Surplus Property Commission, the Bureau of
Aeronautics, the National Airport Corporation and then to the Civil
Aeronautics Administration (CAA).
During the pendency of the expropriation proceedings, respondent
Bernardo L. Lozada acquired the lot from Deiparine, and a TCT was
issued in Lozada’s name.
legal interest from November 16, 1947 – the time when the lot was Respondents are ordered to pay petitioners the necessary expenses the
first occupied by the airport. latter incurred in maintaining the lot.
The affected landowners appealed. The Air Transporation Office (ATO)
formerly CAA, proposed a compromise settlement whereby the
owners affected by the expropriation would either not appeal or
withdraw their appeals in consideration of a commitment that the
expropriated lots would be resold at the price they were expropriated
in the event that the ATO would abandon the Lahug Airport.
Because of this, Lozada did not pursue his appeal. Thereafter, the lot
was transferred and registered in the name of the Republic. The
improvement and expansion plan of the Lahug Airport, however, was
not pursued.
Lozada and the other landowners then contacted CAA Director Rivera
requesting to repurchase the lot, as per previous agreement. The
CAA said that there might still be a need for the Lahug Airport to be
used as an emergency DC-3 airport. It reiterated, however, the
assurance that should the office dispose and resell the properties,
the priority of the former owners would be subject to the approvial of
the President.
President Cory Aquino issued a Memorandum to the Dept. of
Transportation, directing the transfer of general aviation operations of
the Lahug Airport to the Mactan International Airport and upon such
transfer, the closure of the Lahug Airport. The Congress enacted RA
6958, entitled “An Act Creating the Mactan-Cebu International Airport
Authority, Transferring Existing Assets of the MCIA….”
From the date of the institution of the expropriation proceedings up to the
present, the public purpose of the said expropriation was never
initiated, realized, or implemented. Instead, the old airport was
converted into a commercial complex, and one of the lots became a
jail complex. The old airport was converted into what is now known
as the Ayala IT Park.
Lozada and the others instituted a complaint for the recovery of
possession and reconveyance of ownership. Petitioners asked for
the dismissal of the complaint and denied that the Government had
made assurances to reconvey the lots should the property not be
used for airport operations. They insisted that the judgment of
condemnation was unconditional and respondents were not entitled
to recover the expropriated property despite abandonment.
The trial court rendered judgment in favor of Lozada and ordered MCIAA
and ATO to restore to plaintiffs the possession and ownership of the
lands upon payment of the expropriation price.
Petitioners appealed to the CA, which affirmed the lower court’s ruling.
Hence this appeal.

ISSUE/s:
WoN the expropriated lot should be returned to Lozada – YES

RULING: Petition denied. Respondents are ordered to return to petitioners


the just compensation they received for the expropriation of the lot.
Petitioners are entitled to keep whatever fruits and income they may have The condition not having materialized because the airport had been
obtained from the lot. Respondents are entitled to keep whatever interests abandoned, the former owner should then be allowed to reacquire
they received as just compensation. The case is REMANDED to the RTC for the expropriated property.
the purpose of receiving evidence on the amounts that respondents will have Court revisited their ruling in Fery, which involved an expropriation suit
to pay petitioners. against parcels of land to be used as a site for a public market.
Instead of putting up a public market, Cabanatuan constructed
RATIO: residential houses for lease on the area.
Republic anchors their claim to the controverted property on the In a line of similar cases, the Court uniformly held that the transfer to a
supposition that the Decision in the expropriation proceedings did not third party of the expropriated property is not a ground for the
provide for the condition that should the intended use of the land be recovery of the same by its previous owner. Obviously, Fery was not
abandoned or aborted, the property would revert to respondents. decided pursuant to the now sacredly held constitutional right
In Fery v. Municipality of Cabanatuan, the Court declared that that private property shall not be taken for public use without
Government acquires only such rights in expropriated parcels of land just compensation.
as may be allowed by the character of its title over the properties. It is well-settled that the taking of private property by the Government’s
When land has been acquired for public use in fee simple, power of eminent domain is subject to to mandatory requirements:
unconditionally, either by the exercise of eminent domain or by That it is for a particular public purpose;
purchase, the former owner retains no right in the land, and the That just compensation be paid to the property owner.
public use may be abandoned or the land may be devoted to a With respect to the element of public use, the expropriator should commit
different use, without any impairment of the estate or any reversion to use the property pursuant to the purpose stated in the petition,
to the former owner. failing which, it should file another petition for the new purpose. If not,
Contrary to the stance of petitinoers, the Court has ruled otherwise in it is incumblent upon the expropriator to return the property to its
Heirs of Timoteo Moreno v. Mactan-Cebu International Airport. private owner, if the latter desires to reacquire the same.
Although Mactan Airport is being constructed, it does not take away Otherwise, the judgment of expropriation suffers an intrinsic flaw, as it
the actual usefulness and importance of the Lahug Airport: it is would lack one indispensable element for the proper exercise of the
handling the air traffic both civilian and military. No evidence was power of eminent domain, which is the particular public purpose for
adduced to show how soon is the Mactan Airport to be placed in which the property will be devoted.
operation and whether the Lahug Airport will be closed immediately In light of these premises, the Court holds that the taking of private
thereafter. property, is always subject to the condition that the property be
It is up to the other departments of the Government to determine said devoted to the specific public purpose for which it was taken. If
matters. The Court cannot substitute its judgment for those of the this particular purpose or intent is not initiated or pursued, then the
said departments. In the absence of such showing, the Court will former owners, if they desire, may seek the reversion of the property,
presume that the Lahug Airport will continue to be in operation. subject to the return of the amount of just compensation received.
In conclusion, the expropriated properties would remain to be so until it On the existence of an oral compromise agreement that entitled Lozada to
can be confirmed that Lahug Airport was no longer in operation. This repurchase the lot, the Court rules in the affirmative. Lozada was firm
inference implies two things: that he was promised that the lot would be reverted to him once the
After the Lahug Airport ceased its undertaking and the expropriated lots were public use of the lot ceases. He made it clear that the verbal promise
not being used for any airport expansion project, the rights vis-à-vis the was made in Lahug with other lot owners. It was just a verbal promise;
expropriated lots must be equitably adjusted; nevertheless, it is binding.
The foregoing declarations in the Decision should merge with and become
an intrinsic part of the fallo which is clearly inadequate since the
dispositive portion is not in accord with the findings as contained in the
body.
The Decision in Civil Case No. R-1881 should be read in its entirety, wherein
it is apparent that the acquisition by the republic of the expropriated lots
was subject to the condition that the Lahug Airport would continue its
operation.
DIDIPIO EARTH SAVERS v. GOZUN (IYA) On 7 September 2001, counsels for petitioners DESAMA filed a demand
March 30, 2006 | Chico-Nazario, J. | Eminent Domain letter addressed to then DENR Secretary Heherson Alvarez, for the
cancellation of the CAMC FTAA for the primary reason that Rep. Act
No. 7942 and its Implementing Rules and Regulations DAO 96-40
PETITIONER: Didipio Earth-Savers’ Multi Purpose Association are unconstitutional.
Incorporated (DESAMA), et. al. Demands of Didipio were rejected, hence this present petition.
RESPONDENTS: Elisea Gozun in her capacity as Secretary of the
Department of Environment and Natural Resources, et al. ISSUE/s:
SUMMARY: Dipidio Earth Savers’ question he constitutionality of the WON the questioned law, IRR, and FTAA are constitutional - YES
Philippine Mning Act, its IRR, and the Financial Technical Assistance
Agreement. They aver that its provisions constitute taking of private RULING: WHEREFORE, the instant petition for prohibition and mandamus is
property, which deprive them of their property rights. DISMISSED. Questioned law, IRR and FTAA are NOT
UNCONSTITUTIONAL.
The SC ruled that although the questioned laws are taking provisions,
they are unconstitutional. If the taking is founded on public purpose, the RATIO:
State may regulate such property and the Courts may fix just In seeking to nullify Rep. Act No. 7942 and its implementing rules DAO
compensation. 96-40 as unconstitutional, petitioners set their sight on Section 76 of
Rep. Act No. 7942 and Section 107 of DAO 96-40 which they claim
DOCTRINE: The taking to be valid must be for public use. Public use as allow the unlawful and unjust taking of private property for
a requirement for the valid exercise of the power of eminent domain is private purpose in contradiction with Section 9, Article III of
now synonymous with public interest, public benefit, public welfare and theConstitution mandating that private property shall not be taken
public convenience. except for public use and the corresponding payment of just
compensation.
They assert that public respondent DENR, through the Mining Act and its
FACTS:
Implementing Rules and Regulations, cannot, on its own, permit
This petition for prohibition and mandamus assails the constitutionality of
entry into a private property and allow taking of land without payment
Republic Act No. 7942 otherwise known as the Philippine Mining Act of
of just compensation.
1995, together with the Implementing Rules and Regulations issued
Republic v. Vda. de Castellvi defines “taking” under the concept of
pursuant thereto, Department of Environment and Natural Resources
eminent domain as entering upon private property for more than a
(DENR) Administrative Order No. 96- 40, s. 1996 (DAO 96-40) and of
momentary period, and, under the warrant or color of legal authority,
the Financial and Technical Assistance Agreement (FTAA) entered into
devoting it to a public use, or otherwise informally appropriating or
on 20 June 1994 by the Republic of the Philippines and Arimco Mining
injuriously affecting it in such a way as to substantially oust the owner
Corporation (AMC).
and deprive him of all beneficial enjoyment thereof.
Arimco is a foreign corporation, 99% of its stockholders are Australian. Dipidio, et al. claim that the entry into a private property by CAMC,
On 25 July 1987, then President Corazon C. Aquino promulgated pursuant to its FTAA, is for more than a momentary period, i.e., for
Executive Order No. 279 which authorized the DENR Secretary to 25 years, and renewable for another 25 years.
accept, consider and evaluate proposals from foreign-owned
that the entry into the property is under the warrant or color of
corporations or foreign investors for contracts of agreements
legal authority pursuant to the FTAA executed between the
involving either technical or financial assistance for large-scale
government and CAMC
exploration, development, and utilization of minerals, which, upon
that the entry substantially ousts the owner or possessor and
appropriate recommendation of the Secretary, the President may
deprives him of all beneficial enjoyment of the property.
execute with the foreign proponent.
These facts, according to the Dipidio, amount to taking. As such,
On March 1995, President FVR signed the Philippine Mining Act of 1995.
petitioners question the exercise of the power of eminent domain as
On August 15, 1995, DENR Secretary Ramos issued AO No. 23
unwarranted because respondents failed to prove that the entry into
containing the implementing guildlines of the RA, which was later
private property is devoted for public use.
superseded by the questions DAO No. 96-40.
Gozun, et al. argue that Section 76 is not a taking provision but a valid
Previously, however, President Ramos executed an FTAA with AMC over
exercise of the police power and by virtue of which, the state may
a total land area of 37,000 hectares covering the provinces of Nueva
Vizcaya and Quirino.
prescribe regulations to promote the health, morals, peace,
education, good order,
safety and general welfare of the people. the expropriator must enter a private property;
This government regulation involves the adjustment of rights for the entry must be for more than a momentary period;
the public good and that this adjustment curtails some the entry must be under warrant or color of legal authority;
potential for the use or economic exploitation of private
property.
The power of eminent domain is the inherent right of the state (and
of those entities to which the power has been lawfully
delegated) to condemn private property to public use upon
payment of just compensation.
Police power is the power of the state to promote public welfare by
restraining and regulating the use of liberty and property.
Although both police power and the power of eminent domain have the
general welfare for their object, and recent trends show a mingling of
the two with the latter being used as an implement of the former,
there are still traditional distinctions between the two.
Property condemned under police power is usually noxious or
intended for a noxious purpose; hence, no compensation
shall be paid.
In the exercise of police power, property rights of private
individuals are subjected to restraints and burdens in order
to secure the general comfort, health, and prosperity of the
state.
Where a property interest is merely restricted because the
continued use thereof would be injurious to public welfare, or
where property is destroyed because its continued existence
would be injurious to public interest, there is no
compensable taking.
When a property interest is appropriated and applied to some
public purpose, there is compensable taking.
According to noted constitutionalist, Fr. Joaquin Bernas, SJ, in the
exercise of its police power regulation, the state restricts the use of
private property, but none of the property interests in the bundle of
rights which constitute ownership is appropriated for use by or for the
benefit of the public.
If, however, in the regulation of the use of the property, somebody else
acquires the use or interest thereof, such restriction constitutes
compensable taking.
While the power of eminent domain often results in the appropriation of title
to or possession of property, it need not always be the case. Taking may
include trespass without actual eviction of the owner, material
impairment of the value of the property or prevention of the ordinary
uses for which the property was intended such as the establishment of
an easement.
In Ayala de Roxas v. City of Manila, it was held that the imposition of
burden over a private property through easement was considered
taking; hence, payment of just compensation is required.
In Republic v. Castellvi, this Court had the occasion to spell out the
requisites of taking in eminent domain:
the property must be devoted to public use or otherwise informally
appropriated or injuriously affected;
the utilization of the property for public use must be in such a way as
to oust the owner and deprive him of beneficial enjoyment of the
property.
As shown by the foregoing jurisprudence, a regulation which substantially
deprives the owner of his proprietary rights and restricts the beneficial
use and enjoyment for public use amounts to compensable taking.
The entry referred to in Section 76 and the easement rights under Section 75
of Rep. Act No. 7942 as well as the various rights to CAMC under its
FTAA are no different from the deprivation of proprietary rights in the
cases discussed which this Court considered as taking.
While this Court declares that the assailed provision is a taking provision, this
does not mean that it is unconstitutional on the ground that it allows
taking of private property without the determination of public use and the
payment of just compensation.
The taking to be valid must be for public use. Public use as a requirement for
the valid exercise of the power of eminent domain is now synonymous
with public interest, public benefit, public welfare and public convenience.
It includes the broader notion of indirect public benefit or advantage.
The FTAA provision under attack merely facilitates the implementation of the
FTAA given to CAMC and shields it from violating the Anti-Dummy Law.
The question on the judicial determination of just compensation has been
settled in the case of Export Processing Zone Authority v. Dulay wherein
the court declared that the determination of just compensation in eminent
domain cases is a judicial function.
Indeed, there is nothing in the provisions of the assailed law and its
implementing rules and regulations that exclude the courts from their
jurisdiction to determine just compensation in expropriation proceedings
involving mining operations.
Although Section 105 confers upon the Panel of Arbitrators the authority to
decide cases where surface owners, occupants, concessionaires refuse
permit holders entry, thus, necessitating involuntary taking, this does not
mean that the determination of the just compensation by the Panel of
Arbitrators or the Mines Adjudication Board is final and conclusive. The
determination is only preliminary unless accepted by all parties
concerned.
CITY OF MANILA v. ESTRADA (ELIEL) tenants and te later by Estrada and his family. The remaning
September 9, 1913 | Trent, J. | Expropriation improvements consisted

PETITIONER: City of Manila


RESPONDENTS: Balbina Estrada

SUMMARY: The City of Manila sought to expropriate an entire parcel of


land with its improvement. Initially, Estrada, the owner of said property
requested the compensation to be at P25/sqm, the majority report
decided at P20/sqm, and was fixed at P15/sqm. When the matter
however, was brought to this court, it decided based on preponderance of
evidence that the just compensation Estrada was entitled to after
assessment of commissioners is only P10/sqm, accordingly. Estrada then
contends that the value should be at P25/sqm, and not below P20, as
fixed by the majority report.

The SC held that P10 is just compensation. Based on preponderance of


evidence by the commissioners, the value of the land as assessed is only
P10/sqm, and can not go beyond or less, otherwise it would be unjust.
This, as decided by the commissioners is not grossly excessive or
inadequate. As for the contention that the report is final, the courts
decided that it is only persuasive, and it is still subject to the courts
discretion on whether or not it should accept the value for just
compensation.

DOCTRINE: The value and damages awarded must be a just


compensation and no more and no less. But in fixing these amounts, the
commissioners are not to act ad libitum.

FACTS:
The city of Manila sought to exprorpriate an entire parcel of land with its
improvement for use in connection with a new market at that time
beign erected in the district of Paco.
The court duly rendered its decision confirming the majoriy report as to
the improvements, but reducing the price of the land from P20 per
sqm, as fixed by the majority report, to P15, per sqm.
The record was therefore eleveated to this Court for a review of the evidence
and assigned errors of the parties. This Court held that P10 per sqm was
just compensation for the land, and rendered its decision accordingly.
The court justifies such action, first, upon the gorund that the great
preponderance of the evidence submitted to the commissions
showed that P10 per sqm was just compensation for the land taken,
and second, upon the power of the court to revise the report of the
commissioners when the amount awarded is grossly inadequate or
grossly excessive.
From the record it appears that the improvements on the land consisted of a
camarin in fairly good condition, appraised at P4,500; a dwelling house
in very bad condition, appraised at P1,500l the former being occupied by
of a stone wall surrounding the lot, appraised at P1,020 and some
trees, appraised at P150.

ISSUE/s:
WoN P10 per sqm is just compensation – YES

RULING: Holding on this branch of the case is supported not only be reason
but by the interpretation of similar provisions of law in other jurisdictions.

RATIO:
There is a considerable difference between this valuation and P25 per
sqm, as fixed by Estrada, or of the price fixed by the majority report
of the commissioners of P20 per sqm.
It is to be noted that no witnesses other than Estrada were called who
could confirm the higher valuation or even testify to an intermediate
price. The price of P10 per sqm is 66 percent greater than that
obtained for land on the opposite side of the estero, and this
difference would seem amploy sufficient to compensate for the more
favored location of the condemned land. That P10 per sqm is a just
compensation is shown by a great preponderance of the evidence.
Compensation means an equivalent for the value of the land taken.
Anything beyond that is more and anything short of that is less
than compensation. To compensate is to render something
which is equal to that taken or received.
The word “just” is used to intensify the meaning of the word
“compensation;” to convey the idea that the equivalent to be
rendered for the property taken shall be real, substantial, full,
ample.
“Just compensation,” therefore, as used in sec 246 of the Code of Civil
Procedure, means a fair and full equivalent for the loss sustained.
The compensation must be just to the public as well as to the
owners. Sec 244 “The commissioners shall assess the value of
property taken and used, and shall also assess the
consequential damages to the property not taken and deduct
from such consequential damages the consequential benefits to
be derived by the owners.
“To assess” is to perform a judicial act. The commissioners’ power is
limited to assessing the value and to determining the amount of the
damages. There it stops; they can go no further.
The value and damages awarded must be a just compensation and
no more and no less. But in fixing these amounts, the
commissioners are not to act ad libitum.
They are to discharge the trust reposed in them according to well
established rules and form their judgment upon correct legal
principles. To deny this is to place them where no one else in this
country is placed: Above the law and beyond accountability.
This court, after an examination of the evidence, found that the awards
as fixed by the majority of the commissioers and the trial court were
grossly
excessive; that a just compensation for the land taken was P10 per sqm, and
in a short opinion, rendered judgment accordingly.
The judgment of the court is necessary to give effect to the
proceedings. Nor is the report of the commissioners
conclusive, under any circumstances, so that the judgment of
the court is a mere detail or formality requisite to the
proceedings. The judgement of the court is rendered after a
consideration of the commissioners’ report and the excpetions
thereto submitted upon the hearing of the report.
If individual items which make up the total amount of the award in the
commissioners’ report could only be accepted or rejected in their
entirety, it would be necessary to return the case, so far as the
rejected portions of the report were concerned, for further
consideration before the same or new commissioners, and the court
could nto make a “final order and judgment” in the cause until the
rejected portions of the report had been re-report to it.
So it is clear from these provisisons that this court, in those cases
where the right of eminent domain has been exercised and
where the provisions of the above section have been complied
with, may exaine the testimony and decide the case by
preponderance of the evidence; or in other words, retry the
case upon the merits and render such order or judgment as
justice and equity may require.
The result is that, in our opinion, there is ample authority in the statute to
authorize the courts to change or modify the report of the
commissioners by increasing or decreasing the amount of the award,
if the facts of the case will justify such change or modification.
When the only error of the commissioners is that they have applied illegal
principles to the evidence submitted to them; or that they have
disregarded a clear preponderance of the evidence; or that they
have used an improper rule of assessment in arriving at the amount
of the award, then, in such a case, if the ‘evidence be clear and
convincing, the court should be able, by the use of those correct
legal principles which govern the case, to determine upon the
amount which should be awarded without remanding the cause.”
DAET v. CA (JP) Daet cannot exercise eminent domain as to has no funds to pay the
October 18, 1979 | Guerrero, J. | Eminent Domain reasonable value

PETITIONER: Municipality of Daet


RESPONDENTS: Court of Appeals, Liseng Giap and Co. Inc.

SUMMARY: The Municipality of Daet instituted condemnation


proceedings against respondent Li Seng for the public purpose of
converting the respondent’s land to a public park. Respondent filed to
dismiss such. The RTC granted such on the ground that petitioner had no
funds to pay the just compensation needed by the law. On the issue of on
the value of such land and improvements thereon, petitioner Municipality
motioned to follow the value of the commissioners constituted, while
respondents introduced expert testimony of a licensed broker. The CA
sided with the broker and held that the land should be appraised in 1969,
the time of the judgment.

The SC affirmed the CA in toto. The SC held that although as a general


rule the property should be appraised in the expropriation proceedings,
such is not applicable in the case at bar because there was no actual
taking during the expropriation proceedings. Plaintiff Daet should not be
allowed to stall the proceedings at the expense of respondent owner. The
land shold be appraised from the demolition by the Municipal Mayor of
the building on the land of respondent Li Seng as this constituted the
actual taking required by law.

DOCTRINE: When plaintiff takes possession before the institution of the


condemnation proceedings, the value should be fixed as of the time of
the taking of the said possession, not the filing of the complaint, and the
latter should be the basis for the determination of the value, when the
taking of the property involved coincides with or is subsequent to, the
commencement of the proceedings.

FACTS:
The Municipality of Daet instituted condemnation proceedings against
private respondent Li Seng Giap & Co., Inc. before the CFI of
Camarines Norte for the purpose of acquiring and subsequently
converting the property owned by private respondent as a public
park.
Li Seng Giap filed a motion to dismiss on the grounds that the proposed
expropriation has not been duly authorized by law, there is no
genuine necessity for th expropriation, the park could be put up in a
different place, the plaintiff Daet does not have that sufficient funds to
push through its project of constructing a park and to allow plaintiff
Daet to expropriate defendant’s property this time would be
only to needlessly deprive Li Seng of the use if its property.
The trial court dismissed the expropriation proceedings mainly on the
grounds that there is no genuine need for petitioner Daet to convert the
lot into a park. Or even if there is genuine necessity, still petitioner
of the building. comply with the order of the Court requiring the municipality to
The CA reversed the RTC recision and hled that Daet has a lawful right make the necessary deposit of the provisional value as fixed by the
to take the property for a public use, upon payment of just Court in its Order of April
compensation to be determined as of the date of the filing of the
complaint. The case was remanded to the RTC.
A board of commissioners was then appointed to be the committee on
appraisal. They held that the value of the property should be at P60
per square meter and the structure remaining thereon at P15,000.
Respondent Li Seng opposed such and provided additional evidence of
the expert testimony of a duly licensed broker, Engineer Aquino, who
appraised the land at P200 per square meter and the improvement
at P36,500 in 1969.
The RTC rendered a decision disregarding such valuation made by
the commissioners and used the appraisal of Engineer Auino in
1969 as the basis in determining the value of the land in 1962.
(The RTC used P117 per square meter though, not P200 as
declared by Engineer).
Both petitioner Daet and respondent Li Seng appealed. Petitioner to give
due course to the commissioner’s report, while respondent to insist
the P200 per square meter by the expert.
The CA sustained the valuation of the property in 1969 (as by the
Engineer Aquino), declaring the municipality to have a lawful right to
expropriate. Hence, this appeal.

ISSUE/s:
WoN the CA erred in its valuation of the property in question, giving
credence to the appraiser employed by private respondent – NO

RULING: Judgement of the CA is affirmed in toto. The SC finds substantial


basis for the the court to fix the value of the land at P200 per square meter
and the building at P36,500 as testified by the broker.

RATIO:
When plaintiff takes possession before the institution of the condemnation
proceedings, the value should be fixed as of the time of the taking of the
said possession, not the filing of the complaint, and the latter should be
the basis for the determination of the value, when the taking of the
property involved coincides with or is subsequent to, the commencement
of the proceedings.
It is a well-settled rule that just compensation means the equivalent
for the value of the property at the time of its taking.
In the case at bar, it is a fact that there has been no taking of the
property prior to the institution of the condemnation
proceedings.
And it cannot even be said that the filing of the complaint coincided with the
taking of the property by the plaintiff Daet because the latter did not
enter into possession of the property since it failed or did not
15, 1969. property because firstly, at the time judgment was rendered on
Plaintiff Daet continually appealed the value of the property from the CA December 2, 1969, petitioner had not actually taken possession
until the SC and in the meantime took no step to take possession of of the property sought to be expropriated and secondly, the SC
the land. finds the valuation determined by the Court of Appeals to be
While petitioner submitted a Manifestation on to this Court invoking just, fair and reasonable.
Presidential Decree No. 42 dated November 9, 1972 and manifesting
that it had made a deposit to the Philippine National Bank in the
amount of P54,370.00 as per PNB Certificate No. 9381, the SC
holds that petitioner has not made the correct and proper
deposit of the provisional value as fixed by the trial court.
It is elementary that Presidential Decree No. 42 of November 9, 1972
which grants the right to take or enter upon the possession of the
property sought to be expropriated if he deposits with the Philippine
National Bank an amount equivalent to the assessed value of the
property for purposes of taxation has no application to the case at
bar.
This is because in the case at bar, the Court of Appeals had already fixed
the value of the property at P200.00 per square meter and
P36,500.00 for the improvement in its decision promulgated on
October 18, 1972 about three weeks earlier than the issuance of the
Presidential Decree No. 42.
By not complying with the orders of the trial court and the appellate court,
petitioner Daet would benefit by its noncompliance and dilly-dallying
in taking possession of the property.
The SC will not sanction or allow to the prejudice of the private
respondent landowner Li Seng who should not be penalized by the
protracted delay of petitioner Daet in taking over the property over
a period of seventeen (17) years during which time private
respondent was deprived of the beneficial use of the land and the
improvement thereon.
Petitioner Daet had the duty to make the deposit in the amount
ascertained by the CA, fixed by court. This deposit serves the double
purpose of pre-payment of the property if the same is finally
expropriated and of an indemnity for damages if the proceedings are
dismissed.
The antecedent facts and circumstances of this case are unusual, unique
and abnormal such that by reason thereof, the SC agrees with the
judgment of the CA fixing the fair market value of the property sought to
be expropriated at P200 per sq. meter and the value of the improement
to be P36,500.
This is is in conformity with doctrinal rulings hereinabove cited that the value
should be fixed as of the time of the taking of the possession of the
On the weight of the opinions of the commissioners: in fixing these amounts,
the commissioners are not to act ad libitum. They are to discharge the trust
reposed in them according to well-established rules and form their judgment
upon correct legal principles. To deny this is to place them where no one else
in this country is placed, above the law and beyond accountability.
Corollary to this limitation, it has been held that reports submitted by
commissioners of appraisals in condemnation proceedings are not
binding, but merely advisory in character, as far as the court is
concerned.
On the expert opinion: Also, the value of a parcel of land taken by eminent
domain is always a matter of opinion, the same may be proved by
opinion evidence of the real estate appraiser.
Civil Engineer Aquino has in fact been an appraiser of long standing, being
one of the incorporators of CM Hoskins and Co., Inc., a corporation
engaged in real estate brokerage. He was competent enough to make
the fair market value appraisal.
The SC also held that the demolition of the building of Li Seng standing
on the land by the Municipal mayor constituted the actual laking of
possession of the property sought to be expropriated.
On PD No. 42: Presidential Decree No. 42 issued on November 9, 1972
does not limit the just compensation in expropriation proceedings
to the assessed value of the value sought to be condemned.
Said decree fixes only the provisional value of the property. As a provisional
value, it does not necessarily represent the true and correct value of the
land.
The assessed value of a property constitutes only a percentage of its
current fair market value. It cannot, thus, be the direct basis of just
compensation in expropriation proceedings.
But more importantly, this assignment of error is bereft of merit because
Presidential Decree No. 42 is inapplicable in the case at bar. As pointed
out by private respondent, it is a cardinal rule of statutory construction
that laws shall have only prospective effect.
NPC v. CA (HENRY) It must be noted, that accdg to the RTC, Php 3.75 per sqm is and
June 22, 1984 | Melencio-Herrera, J. | Just compensation represents the fair market value of the property sought by
NPC, which is based on the
PETITIONER: National Power Corporation (NPC)
RESPONDENTS: CA and B.E. San Diego, Inc.

SUMMARY: NPC and Sps. Sadang engaged in negotiations for the


purchase of a portion of the latter’s property for the construction of an
access road, a project spearheaded by the NPC. CFI ruled for the value
to be 3.75 per sqm, while CA ruled for the value to be 7.00.

SC held in this case reinstated the CFI ruling, that the value be 3.75, in
accordance with the value of the land at the time it was taken from the
owner and appropriated by the Govt (in this case, the NPC).

DOCTRINE: To determine due compensation for lands appropriated by


the Government, the basis should be the price or value at the time it was
taken from the owner and appropriated by the Government.

FACTS:
1961, petitioner NPC commenced negotiations with Sps. Sadang for the
purchase a portion of their land (8,746 sqm of the 62,285 sqm) in
San Mateo, Bulacan, for the purpose of constructing an access road
to its Angat River Hydroelectric Project.
Even when negotiations were not yet concluded, NPC obtainted
permission from spouses to begin construction of the road, which
happened Nov 1961.
December 1962, respondent BE San Diego, a realty firm acquired the
parcel of land at a public auction sale and was issued a title.
Feb 1963, NPC instituted proceedings for eminent domain against Sps.
Sadang in the CFI, later amended, with leave of court, to implead
respondent San Diego, which declared NPC to have the legal right to
acquire by eminent domain the absolute ownership of the land in
dispute (with just compensation at Php 3.75 per sqm).
An appeal was brought up to the CA by both parties, to which the
appellate court modified the RTC decision to raise the compensation
to Php 7.00 per sqm, and hence a petition by NPC to the SC was
made.

ISSUE/s:
WoN CA erred in fixing the amount of Php 7.00 as just compensation for
the portion of land sought – YES

RULING: SC set aside CA decision and reinstated the CFI decision.

RATIO:
characteristic of such land, being agricultural. On the other hand, CA said
that what is to be considered is that, in determining the value of the land
appropriated, the same conditions are to be regarded as in a sale of property
between private parties.
The inquiry must always be not what the property is worth in the market, but with
reference to the use to hwihc it is plainly adopted.
Hence the CA rued the price to be Php 7.00 per sqm.
Considering the two rulings, the SC found more merit on the trial court’s findings
(FMV = Php 3.75), for the following reasons:
Both documentary and oral evidence indicate that the land in question, at
the time of taking by NPC in 1961, was agricultural in use as well as
for taxation purposes
San Diego’s contention that the location and direction of the acess road
is burdensome is not borne out by the evidence. The Report of the
Commissioner of the Court revealed that NPC merely improved a
pre-existing mining road on the premises, which was only accessible
by carabao-drawn sledge during the rainy season.
The finding of the Trial Court that there is negligible, if any, consequential
damage to speak of, thus becomes readily tenable. San Diego was
not, as was the relief by the CA, prevented from carrying out the plan
of converting the property into a housing subdivision. On the
contrary, the Trial Court observed that the thoroughfare should
provide a marked improvement to the flourishing housing subdivision
managed by the respondent
Appraisal by a realty firm of Php 20.00 per sqm the price that San Diego
stresses the property should command, is not a fair market value.
Sps. Sadang offered to part with the property at Php 4.00 per sqm.
San Diego purchased the entire property (all 62.2k sqm) at a public
oction for Php 10,000.00, or at Php .16 per sqm. Previous to that,
property was even mortgaged to DBP for PHp 20,000.00
In the case at bar, the taking by NPC occurred in Nov 1961, whenit constructed
access road on the expropriated property when it was still owned by the
spouses. The convertibility of the property into a subdivision is not
controlling.
Since San Diego bought the land in question in the interim and was issued the
title December 1962, the taking as to it should commence only from the
said date.
As for the legal interest, it should accrue from the time of San Diego’s taking, at
6% per annum, up to the time payment is made by NPC.
EPZA v. DULAY (DANNAH) EPZA therefore offered to purchase said parcels of land from San
April 29, 1987 | Gutierrez, Jr., J. | Eminent Domain Antonio,

PETITIONER: Export Processing Zone Authority


RESPONDENTS: Hon. Ceferino Dulay, San Antonio Dev’t Corp.

SUMMARY: PD 1811 reserved a certain parcel of land of the public


domain for the establishment of an export processing zone by petitioner
Export Processing Zone Authority (EPZA). However, not all the reserved
areas was public land. A portion was owned by San Antonio
Development Corp. Hence, EPZA offered to purchases said parcels of
land. The parties however did not reach an agreement.

EPZA then filed with the CFI of Lapu Lapu a complaint for expropriation due
to Proclamation No. 1811. Hon. Dulay then issued a writ of possession,
authorizing EPZA to take immediate possession of the properties. Hon. Dulay
issued a second order, appointing certain persons as commissioners to
ascertain and report to the court the just compensation for the properties
sought to be expropriated. The assigned commissiners valued the land at
P15/sqm. EPZA filed an MR because under PD 1533, it is the owners or the
assessors which dictate the fair and current market value. The MR was
denied, hence the present petition.

The SC affirmed the ruling of the trial court, stating that PD 1533 is
unconstitutional for encroaching judicial prerogatives. The determination
of “just compensation” in eminent domain cases is a judicial function. The
executive department or the legislature may make the initial
determinations but when a party claims a violation of the guarantee in the
Bill of Rights that private property may not be taken for public use without
just compensation, no statute, decree, or executive order can mandate
that its own determination shall prevail over the court's findings.

DOCTRINE: Just compensation means the value of the property at the


time of taking. It means a fair and full equivalent for the loss sustained. All
the facts as to the condition of the property and its surroundings, its
improvements and capabilities, should be considered.

FACTS:
On January 15, 1979, the President issued Proclamation No. 1811,
reserving a certain parcel of land of the public domain, situated in
Lapu-Lapu Cebu, and covering a total of 1.1M+ sqm, for the
establishment of an export processing zone by petitioner Export
Processing Zone Authority (EPZA).
No all the reserved area, however, was public land. The proclamation
included, among others, 4 parcels of land with an aggregate area of
22k+ sqm owned and registered in the name of private respondent
San Antonio Dev’t Corp.
in accordance with the valuation set forth in Section 92, PD 464, as amended.
The parties failed to reach an agreement.
EPZA filed with the CFI of Lapu Lapu a complaint for expropriation with a prayer
for the issuance of a writ of possession against San Antonio Dev’t Corp, to
expropriate said lands pursuant to PD 66 as amended, in relation to
Proclamation No 1811, for the purpose of establishing the Mactan Export
Processing Zone.
On October 1980, respondent Hon. Dulay issued a writ of possession authorizing
EPZA to take immediate possession of the premises.
Hon. Dulay issued an order stating that the parties have agreed that the only
issue to be resolved is the just compensation for the properties.
On February 1981, Hon. Dulay issued the order of condemnation declaring EPZA
as having the lawful right to take the properties sought to be condemned,
upon payment of just compensation to be determined.
Hon. Dulay issued a second order, the subject of this petition, appointing certain
persons as commissioners to ascertain and report to the court the just
compensation for the properties sought to be expropriated.
On June 1981, three commissioners submitted their consolidated report
recommending the amount of P15.00 per square meter as the fair and
reasonable value of just compensation f or the properties.
On July 1981, EPZA filed an MR on the grounds that PD 1533 has superseded
Sections 5 to 8 of Rule 67 of the Rules of Court on the ascertainment of just
compensation through commissinoers.
Moreover, the amount of compensation as averred, must not exceed the
maximum amount set by PD 1533.
On November 1981, the trial court denied EPZA’s MR.
On February 1982, EPZA filed this present petition for certiorari and mandamus
with preliminary restraining order, enjoining the trial court from enforcing the
February 1981 order and from further proceeding with the hearing of the
expropriation case.
EPZA avers that under PD 1533, which is the applicable law, the basis of just
compensation shall be the fair and current market value declared by the
owner of the property sought to be expropriated or such market value as
determined by the assessor, whichever is lower.
Hence, there is no more need to appoint commissioners, and for said
commissioners to consider other highly variable factors in order to determine
just compensation.
EPZA further maintains that PD 1533 has vested on the assessors and property
owners themselves the power or duty to fix the market value of the
properties and that said property owners are given full opportunity to be
heard before the Local Board of Assessment Appeals and Central PD 1533: ^Repeeated + pursuant to the Real Property Tax Code,
Board of Assessment Appeals. whichever value is lower, prior to the recommendation or
Hence, vesting the assessor/property owner of the right to determine just decision of the appropriate
compensation is valid and constitutional.

ISSUE/s:
WoN Sections 5 to 8, Rule 67 of the RRC had been repeald/deemed
amended by PD 1533 insofar as the appointment of commissioners
to determine the just compensation concerned – NO
Stated otherwise, WoN the exclusive and mandatory mode of
determining just compensation in PD 1533 valid and constitutional –
NO

RULING: Petition dismissed. PD 1533 unconstitutional and void.

RATIO:
Prior to the promulgation of PDs 76, 464, 794 and 1533, the Court has
interpreted the eminent domain provisions of the Constitution and
established the meaning, under the fundamental law, of just
compensation and who has the power to determine it.
Municipality of Daet v. CA & Jm Tuason v. Land Tenure Admin: The
well settled rule is that just compensation means the equivalent for
the value of the property at the time of its taking. Anything beyond
that is more and anything short of that is less, than just
compensation. It means a fair and full equivalent for the loss
sustained, which is the measure of the indemnity, not whatever gain
would accrue to the expropriating entity.
Garcia v. CA: All the facts as to the condition of the property and its
surroundings, its improvements and capabilities may be shown and
considered in estimating its value.
Republic v. Santos: According to Section 8, Rule 67, the court is not
bound by the commissioners’ report. The Court may substitute its
own estimate of the value as gathered from the records.
Hoewver, the promulgation of the aforementioned decrees practically set
aside the above and many other precedents hammered out in the
course of evidence-laden, well argued, fully heard, studiously
deliberated, and judiciously considered court proceedings.
The decrees categorically and peremptorily limited the definition of just
compensation.
PD 76: The basis shall be the current and fair market value declared by
the owner or administrator, or such market value as determined by
the Assessor, whichever is lower. PD 464: Section 7 repeated this.
Government office to acquire the property.
The Court is constrained to declare the provisions of the Decrees on just
compensation unconstitutional and void and accordingly dismiss the instant
petition for lack of merit.
The method under the aforecited decrees constitutes impermissible
encroachment on judicial prerogatives.
Although in expropriation proceedings the court technically would still have the
power to determine the just compensation for the property, following the
applicable decrees, its task would be relegated to simply stating the lower
value of the property as declared either by the owner or the assessor.
As a necessary consequence, it would be useless for the court to appoint
commissioners under Rule 67 of the Rules of Court.
Moreover, the need to satisfy the due process clause in the taking of private
property is seemingly fulfilled since it cannot be said that a judicial
proceeding was not had before the actual taking.
However, the strict application of the decrees during the proceedings would be
nothing short of a mere formality or charade as the court has only to choose
between the valuation of the owner and that of the assessor, and its choice is
always limited to the lower of the two.
The court cannot exercise its discretion or independence in determining what is just
or fair. Even a grade school pupil could substitute for the judge insofar as the
determination of constitutional just compensation is concerned. -- LOL
National Housing Authority v. Reyes: Court upheld PD 464 as amended.
NHA contended that the owner’s declaration at P1.4k, which happened to be
lower than the assessor’s assessment, is the just compensation
Private respondent stressed that while there may be basis for the allegation that
the judge did not follow the decree, the matter is still subject to his final
disposition, he having been vested with the original and competetnt authority
to exercise his judicial discretion in the light of the constitutional clauses on
due process and equal protection.
The Court ruled that there should be a recognition that the law as it stands must
be applied; that the decree having spoken so clearly and unequivocably
calls for obedience; and that on a matter where the applicable law speaks in
no uncertain language, the Court has no choice except to yield to its
command.
While the Court yielded to executive prerogative exercised in the form of
absolute law-making power, its members, nonetheless, remained
uncomfortable with the implications of the decision and the abuse and
unfairness, which might follow in its wake.
In the case at hand, court is again confronted with the same question (issue).
This time, the Court states that courts have the power to determine just
compensation.
The trial court correctly stated that the valuation in the decree may only It is violative of due process to deny to the owner the opportunity to
serve as a guiding principle or one of the factors in determining just prove that the valuation in the tax documents is unfair or wrong.
compensation but it may not substitute the court's own judgment as
to what amount should be awarded and how to arrive at such
amount.
A returned to the earlier well-established doctrine is more in keeping with
the principle that the judiciary should live up to its mission "by
vitalizing and not denigrating constitutional rights."
The doctrine enunciated in NHA v. Reyes must necessarily be
abandoned. The basic unfairness of the decrees is readily apparent.
Just compensation means the value of the property at the time of taking.
It means a fair and full equivalent for the loss sustained. All the
facts as to the condition of the property and its surroundings, its
improvements and capabilities, should be considered.
In the case at hand, the tax declarations presented by the petitioner as
basis for just compensation were made by the Lapu-Lapu municipal,
later city assessor long before martial law, when land was not only
much cheaper but when assessed values of properties were stated
in figures constituting only a fraction of their true market value.
San Antonio was not even the owner of the properties at the time. It
purchased the lost for development purposes. To peg the value of
the lots on the basis of documents which are out of date and at
prices below the acquisition cost of present owners would be
arbitrary and confiscatory.
Various factors can come into play in valuation:
Values given by provincial assessors are usually uniform for wide
areas covering several barrios
Individual differences are never taken into account
The value of land is based on such generalities as its possible
cultivation for rice, corn, coconuts or other crops
Very often land described as “cogonal” has been cultivated for
generations
Buildings are described in terms of only 2/3 classes of materials
and estimates of areas are more often inaccurate
Tax values can serve as guides but cannot be absolute
substitutes for compensation
To say that the owners are estopped to question the valuations made by
assessors since they had the opportunity to protest is illusory. The
overwhelming mass of land owners accept unquestioningly what is
found in the tax declarations prepared by local assessors or municipal
clerks for them.
The Court returns to older and more sound precedents. The Court has the
duty to formulate guiding and controlling constitutional principles,
precepts, doctrines, or rules.
The determination of “just compensation” in eminent domain cases is a
judicial function.
The executive department or the legislature may make the initial
determinations but when a party claims a violation of the guarantee in the
Bill of Rights that private property may not be taken for public use without
just compensation, no statute, decree, or executive order can mandate
that its own determination shall prevail over the court's findings.
Much less can the courts be precluded from looking into the "just-ness" of the
decreed compensation.
MADDUMBA v. GSIS (ARIELLE) Respondent GSIS conducted a public bidding of several foreclosed
February 15, 1990 | Regalado, J. | Just Compensation properties. Included in the properties offered to the public was a
house and lot in Sta. Ana, Manila.
PETITIONER: Domingo B. Maddumba and Anita C. Maddumba Petitioner Domingo Maddumba participated in the public bidding and bid
in
RESPONDENTS: Government Service Insurance System

SUMMARY: Respondent GSIS conducted a public bidding of several


foreclosed properties. Petitioner Domingo Maddumba participated in the
public bidding and bid in the amount of P98,000.00. The bid was subject to
the condition that there should be a down payment of 35% of the amount, the
10% constituting the proposal bond with the remaining 25% to be paid after
the receipt of the notice of award/acceptance. Upon the receipt of award,
Maddumba offered to pay the additional 25% in Land Bank bonds at their
face value. Maddumba then offered to pay in cash the remaining 25% down
payment “and all future installments.” When the second monthly installment
became due, Maddumba told GSIS that he be allowed to pay the monthly
amortizations with his Land Bank bonds. GSIS denied Maddumba’s offer.
Maddumba relied on Sec. 85 of RA 3844, and supported his position with the
contention that the policy of the GSIS contravenes the ruling in Gonzales et
al. v. GSIS. Maddumba also submitted the opinion of the Ministry of Agrarian
Reform, which says that if the GSIS accepts the Land Bank bonds as
payment, it must accept the same at par or face value.

The SC held that Sec. 85 provides that the bonds issued by the Bank
may be used and shall be accepted for the purchase of shares of stocks
or assets of GOCCs. It is not disputed that under Sec. 85, a GOCC like
the GSIS, is compelled to accept Land Bank bonds as payment for the
purchase of its assets. Sec. 85 are primarily designed to cushion the
impact of dispossession. Acceptance of Land Bank bonds, instead of
money, involves a certain degree of sacrifice for the landowner. This is in
addition to the fact that, in case of expropriation of land covered by land
reform, the landowner will seldom get the compensation he desires.

DOCTRINE: The nature of a Land Bank bond itself fortifies the view that the
respondent may be compelled to accept those bonds at their face value.
Land Bank bonds are certificates of indebtedness, approved by the Monetary
Board of the
Central Bank. These bonds are deemed contracts and the obligations
resulting therefrom fall within the purview of the non-impairment
clause of the Constitution, and any impairment thereof may take any
encroachment in any respect upon the obligation and cannot be
permitted.

FACTS:
the amount of P98,000.00. The bid was subject to the condition that impact of dispossession. Acceptance of Land Bank bonds, instead of
there should be a down payment of 35% of the amount, the 10% money, involves a certain degree of sacrifice for the landowner. This
constituting the proposal bond with the remaining 25% to be paid is in addition to the fact
after the receipt of the notice of award/acceptance.
Upon the receipt of award, Maddumba offered to pay the additional 25% in
Land Bank bonds at their face value. These bonds were issued to
Maddumba as payment for his Riceland acquired by the Government
from him. However, the GSIS rejected the offer, and was withdrawn by
Maddumba.
Maddumba then offered to pay in cash the remaining 25% down
payment “and all future installments.” Subsequently, a Deed of
Conditional Sale was executed by the parties, where Maddumba as
vendee agreed to pay GSIS the balance of the purchase price of
P63,705.50 in 60 monthly installments.
The first installment was paid. When the second monthly installment
became due, Maddumba told GSIS that he be allowed to pay the
monthly amortizations with his Land Bank bonds.
GSIS denied Maddumba’s offer. Maddumba relied on Sec. 85 of RA
3844, and supported his position with the contention that the policy
of the GSIS contravenes the ruling in Gonzales et al. v. GSIS.
Maddumba also submitted the opinion of the Ministry of Agrarian
Reform, which says that if the GSIS accepts the Land Bank bonds as
payment, it must accept the same at par or face value.
Maddumba was advised by GSIS that Resolution 415 was adopted by
the Board of Trustees denying his request. Hence, this petition.

ISSUE/s: WoN under the provisions of Sec. 85 of RA 3844, the GSIS may be
compelled to accept Land Bank bonds at their face value in payment for a
residential house and lot purchased from the GSIS – YES

RULING:
Sec. 85 provides that the bonds issued by the Bank may be used and
shall be accepted for the purchase of shares of stocks or assets of
GOCCs. It is not disputed that under Sec. 85, a GOCC like the GSIS,
is compelled to accept Land Bank bonds as payment for the
purchase of its assets.
The bidder who offers to pay in bonds of the Land Bank is entitled to
preference. What GSIS is resisting, however, is its being compelled
to accept the bonds at face value.
GSIS claims that PD 251 has amended Sec. 85 by deleting and
eliminating the original provision that Land Bank bonds shall be
accepted “in the amount of their face value;” and to accept said
bonds at their face value will impair the actuarial solvency of the
GSIS.
The Court cannot agree with GSIS. GSIS’ arguments disregard the fact
that the provisions of Sec. 85 are primarily designed to cushion the
that, in case of expropriation of land covered by land reform, the landowner
will seldom get the compensation he desires.
Thus, discounting the Land Bank bonds, and thereby reducing their
effective value, entails and imposes an additional burden on his part.
It is in consideration of this sacrifice that the Court extends the rule
on liberality in the interpretation of the provisions of RA 3844, then
known as the Agricultural Land Reform Code.
The nature of a Land Bank bond itself fortifies the view that the
respondent may be compelled to accept those bonds at their face
value. Land Bank bonds are certificates of indebtedness, approved
by the Monetary Board of the Central Bank. These bonds are
deemed contracts and the obligations resulting therefrom fall
within the purview of the non-impairment clause of the
Constitution, and any impairment thereof may take any
encroachment in any respect upon the obligation and cannot be
permitted.
Thus, the value of these bonds cannot be diminished by any direct or indirect
act, particularly, since said bonds are fully guaranteed by the
Government.
It would be hermeneutically unjustified to adopt a flimsy theory which
would subject the parity of Land Bank bonds to qualifications and
distinctions when the law itself does not so provide.
The deed of conditional sale executed by the parties is subject to the
obligation of and guaranteed by the Government under said bonds.
Their agreement for the payment of installments cannot in any way
be construed as an alteration, nor should it detract from the essence
and compulsion of said obligation.
Even if Maddumba offered to pay his future installments in cash, that
offer was not voluntarily made because it was exacted from him
when GSIS refused to accept the Land Bank bonds. That incident
should not prevent Maddumba from making, and allow respondent to
refuse, an alternative mode of payment authorized by law.
GSIS cannot rely on the deletion by PD 251 of the provision in Sec. 85
that the bonds shall be accepted in the amount of their face value.
Implied repeals are not favored. In the present case, no such
intention to effect changes in the law exists nor is it even apparent.
Neither can GSIS complain that the acceptance of the bonds will impair
its actuarial solvency. The Court held in Gonzales that whatever
unfavorable results the acceptance may have on its finances, the
effects must be deemed to have been intended by PD 251, when it
provided for the payment in bonds to government lending institutions.
MERALCO v. PINEDA (IYA) the questioned Orders dated December 4, 1981 granting the motion
February 13, 1992 | Medialdea, J. | Eminent for payment of private respondents.
Domain PETITIONER: Manila Electric Company On December 15, 1981, private respondents filed an Omnibus Motion
praying that they be allowed to withdraw an additional sum of
RESPONDENTS: Hon. Gregorio Pineda, Teofilo Arayon Sr., Gil de
P90,125.50 from MERALCO’s deposit account with the Philippine
Guzman, Lucito Santiago, and Teresa Bautista
National Bank. Such was granted.
Private respondents filed another motion dated January 8, 1982 praying
SUMMARY: MERALCO through a complaint for eminent domain seeks
that petitioner be ordered to pay the sum of P169,200.00.
to construct power lines on properties owned by private respondents.
In response to private respondents’ motion for payment dated January 8,
The request was granted. Private respondents, however, filed motions
1982, petitioner filed an opposition alleging that private respondents
for payment for the property used by MERALCO. are not entitled to payment of just compensation at this stage of the
proceeding because there is still no appraisal and valuation of the
RTC Judge Pineda granted the motion for payment without a trial before
property. The RTC denied this MR.
the Commissioners in determining just compensation for the land in
Hence, this petition.
question. Decision of Judge Pineda is questioned by MERALCO. The
SC ruled that without prior trial with the Commissioners, judges cannot ISSUE/s:
proceed in establishing the amount for the property.
WON the respondent court can dispense with the assistance of a Board
DOCTRINE: In an expropriation case such as this one where the of
Commissioners in an expropriation proceeding and determine for
principal issue is the determination of just compensation, a trial before
itself the just compensation - NO
the Commissioners is indispensable to allow the parties to present
RULING: ACCORDINGLY, the petition is
evidence on the issue of just compensation
GRANTED RATIO:
MERALCO strongly maintains that the respondent courts act of
FACTS: determining and ordering the payment of just compensation to
On October 29, 1974, a complaint for eminent domain was filed by private respondents without formal presentation of evidence by the
petitioner MERALCO against forty-two (42) defendants with the RTC. parties on the reasonable value of the property constitutes a flagrant
The complaint alleges that for the purpose of constructing a 230 KV violation of their constitutional right to due process.
Transmission line from Barrio Malaya to Tower No. 220 at Pililla, Respondent judge, Hon. Pineda, arrived at the amount of just
Rizal, petitioner needs portions of the land of the private respondents compensation on its own, without the proper reception of evidence
consisting of an aggregate area of 237,321 square meters. before the Board of Commissioners.
Private respondents (Arayon, de Guzman, Santiago, Bautista) question, In an expropriation case such as this one where the principal issue is the
in their motion to dismiss, the area sought to be expropriated as too determination of just compensation, a trial before the Commissioners
excessive. is indispensable to allow the parties to present evidence on the issue
Despite the opposition of the private respondents, the court issued an of just compensation.
Order dated January 13, 1975 authorizing MERALCO to take or While it is true that the findings of commissioners may be disregarded
enter upon the possession of the property sought to be expropriated. and the court may substitute its own estimate of the value, the latter
Private respondents seek to have the amount of P272,000 be paid to may only do so for valid reasons, i.e., where the Commissioners
them as compensation. RTC denied such motion. have applied illegal principles to the evidence submitted to them or
Pursuant to a government policy, Meralco on October 30, 1979 sold to where they have disregarded a clear preponderance of evidence, or
the National Power Corporation (Napocor) the power plants and where the amount allowed is either grossly inadequate or excessive.
transmission lines, including the transmission lines traversing private Thus, trial with the aid of the commissioners is a substantial right that
respondents’ property. may not be done away with capriciously or for no reason at all.
MERALCO filed a motion to dismiss the complaint on the ground that it All premises considered, this Court is convinced that the Hon. Pineda’s
has lost all its interests over the transmission lines and properties act of determining and ordering the payment of just compensation
under expropriation because of their sale to the Napocor. without the assistance of a Board of Commissioners is a flagrant
On June 9, 1981, private respondents filed another motion for payment. But violation of MERALCO’s constitutional right to due process and is a
despite the opposition of MERALCO, the respondent court issued the gross violation of the mandated rule established by the Revised
first of Rules of Court.
SAN ROQUE v. REPUBLIC (ELIEL) WoN there was consummated compensation by the Republic– NO
September 7, 2007 | Nachura, J. | Expropriation

PETITIONER: San Roque Realty and Development Corporation


RESPONDENTS: Republic of the Philippines

SUMMARY: SRRDC is assailing the claim of the Republic over the two
parcels of land originanly owned by Ismael Rosales. According to the
Republic, the subject lands were expropriated for military purposes.
SRRDC contends that it bought the said properties in good faith and that
there was no valid expropriation. Hence the petition.

The SC held that there was incomplete expropriation because the Republic
failed to prove the payment of just compensation in full. Also, under the
Torrens sytem, the title of the land was registered under SRRDC. Republic
failed to explain why SRRDC had continuous possession of property for over
56 years. Thus, the State is barred by estoppel for inaction to cancel the
registration of title.

DOCTRINE: Time and again, we have declared that eminent domain


cases are to be strictly construed against the expropriator. The payment
of just compaensation for private property taken for public use is an
indispensable requiste for the exercise of the State’s sovereign power of
eminent domain.

FACTS:
The subject parcles of land are located at Lahug, Cebu City. It was originally
owned by Ismael Rosales, Pantaleon Cabrera and Francisco Racaza.
Subject parcels of land, together with 17 others, were the subject of an
expropriation proceeding intitiated by the then Commonwealth of the
Philippines for military purposes of the AFP.
Judge Martinez ordered the initial deposit of P9,500 as pre condition for
the entry on the lands sought to be expropriated. A decision as
rendered, condemning the parcels of land. However, the title of the
subject parcel of land was not transferred to the government.
The land was subdivided and two lots were acquired by San Roque
Realty and Development Corporation (SRRDC) and begun
construction of townhouses in the subject parcels of land.
Republic claimed that the purchase of SRRDC was null and void and that
it was the lawful owner of the lots. SRRDC defends, however, that it
was a buyer in good faith and that there was no valid expropriation
because it was initiated by the executed branch without legislative
approval. And expropriation was never consummated because the
government did not actually enter the land nor were the owners paid
any compensation.

ISSUE/s:
RULING: Granted. SRRDC is the lawful owner of the said parcel of land. The general rule it that the State cannot be put in estoppel or laches
by the mistakes or errors of its officials or agents. This rule,
RATIO: however, admits of exceptions. One exception is when the strict
The CA failed to resolve the issue of the Republic’s failure to application of the rule will defeat the effectiveness of a policy
register the property in its name, it also did not give any adopted to protect the public such as the Torrens system.
explanation as to why titel and continuous possession of the
property remained with SRRDC and its predecessors in interest
for 56 years.
The Republic’s bare contention and assumption cannot defeat SRRDC’s
apparent ownership over the subject properties.
Republic v. Lim: To retain ownership over said lots, and uphold the
principle that title to the expropriated property shall pass from
the owner to the expropriator only upon full payment of just
compensation.
Without full payment of just compensation, there can be no transfer
of title from the landowner to the exprorpriator. Thus, we ruled
that the Republic’s failure to pay just compensation precluded
the perfection of its title over Lot. 932.
Time and again, we have declared that eminent domain cases are to
be strictly construed against the expropriator. The payment of
just compaensation for private property taken for public use is
an indispensable requiste for the exercise of the State’s
sovereign power of eminent domain.
Failure to observe the requirement renders the taking ineffectual,
notwithstanding the avowed public purose. To disregard this
limitation on the exercise of governmental power to
exprorpriate is to ride roughshod over private rights.
From the records of this case and our previous findings in the related cases,
the Repubic manifestly failed to present clear and convincing evidence
of full payment of just compensation and receipt therof by the property
owners.
More importantly, if the Republic had actually made full payment of
just compensation, in the ordinary course of things, it would
have led to the cancellation of title, or at least, the annotation of
the lien in favor of the government on the certificate of title
covering lot no. 933.
From the foregoing, it is clear that it was incumbent upon the Republic to
cause the registration of the subject propeorties in its name or record
the decree of expropriation on the title. Yet, not only di dthe Republic
fail to register the subject properties in its name, it failed to do so for
56 years.
Laches is the failure or neglect, for an ureasonable and unexplained
length of time, to do that which by exercising due diligence could
or should have been done earlier; it is negligence oromission to
assert a right within a reasonable time, warranting a presumption
that the party entitled to assert it either has abandoned it or
declined to assert it.
Republic v. CA: We ruled that the immunity of government from laches
and estoppel is not absolute, and the government’s silence or
inaction for nearly 20 years to correct and recover the alleged
increase in the land area of St. Jude was tantatmount to laches.
In the case at bench, the Republic failed to register the subject
proeprties in its name and incurred in laches spanning more
than five and a half decades.
In the instant case, the Republic’s aderse claim of ownership over the
subject properties may have given SRRDC’s predecessors in interst,
the sellers, voidable title to the subject properties.
However, we stress that prior to the acquisition of the subject
properties, Lot 933 had already been subdivided and covered by
separate titles of the subsequent transferees. Thesse titles,
including the titles to the subject properties, had not been
voided at the time of the sale to SRRDC. As such, SRRDC
acquired good title to the subject proeprties, having purchased
them in good faith, for value, and without notice of the seller’s
defect of title, if any.
LBP v. SPOUSES ORILLA (JP) Unsatisfied, petitioner spouses filed an action for the determination fo just
June 27, 2008 | Nachura, J. | Eminent Domain compensation before the RTC as a Special Agrarian Court (SAC) of
Tagbilaran City.
PETITIONER: Land Bank of the Philippines
RESPONDENTS: Spouses Placido Orilla nd Clara dy Orilla

SUMMARY: Spouses Orilla (respondents) are owners of Lot No. 1 in


Bohol. The spouses then received a Notice of Land Valuation and
Acquisition where they were informed that pursuant to the
Comprehensive Agrarian Reform Law, their land would compulsorily
acquired by the government for P371k as valued by petitioner Land Bank
of the Phillipines. The Special Agrarian Court held that such value was
too low and increased the just compensation to P7 per square meter.
Petitioner Bank appealed.

The SC affirmed the SAC. The SC held that due to the advanced age of
the respondents and the fact that their lands are being used by tenant-
farmers who are not paying them, they are being denied “promt
payment.” The SC also held that the respondents’ motion for execution of
the SAC judgment despite the appeal of petitioner Bank is valid because
of the good reasons present in the case at bar which is the revolutionary
kind of appropriation by the Comprehensive Agrarian Reform Law and
the aforesaid denial of the respondents to promt payment as required by
jurisprudence.

DOCTRINE: Without prompt payment, compensation cannot be


considered “just” inasmuch as the property owner is made to suffer the
consequences of being immediately deprived of his land while being
made to wait for a decade or more before actually receiving the amount
necessary to cope with his loss.

FACTS:
Spouses Placido and Clara Orilla (respondents) were the owners of Lot
No. 1 in Bohol.
In the latter part of November 1996, the Department of Agrarian Reform
Provincial Agrarian Reform Office (DAR-PARO) of Bohol sent
respondents a Notice of Land Valuation and Acquisition dated
November 15, 1996 informing them of the compulsory acquisition of
21.1289 hectares of their landholdings.
This is pursuant to the Comprehensive Agrarian Reform Law (Republic
Act [RA] 6657) for P371,154.99 as compensation based on the
valuation made by the Land Bank of the Philippines (petitioner).
Petitioners Orilla Spouses rejected the said valuation. The provincial
Department of Agrarian Reform Adjudication Board affirmed the
valuation made by petitioner Bank.
The SAC held that such valuation by petitioner Bank is too low nd fixed exception to the rule that only a final judgment may be executed, it
the just compensation amount to P7.00 per square meter. must be strictly
Petitioner Bank appealed. Respondents however, filed a Motion for
Execution Pening Appeal pursuant to Rule 39 of the 1997 Rules of
Civil Procedure and the consolidated cases of LBP v. CA, et al. and
DAR v. CA, et al.
Respondents Spouses claimed that the total amount of P1,479,023
(equivalent to P7.00 per square meter of the 21 hectare land),
adjugjed by the SAC as just compensation, could then be withdrawn
by them.
The SAC granted such motion. Respondent bank was ordered to
immediately deposite the total amount due to the spouses.
The Bank appealed to the CA. The CA ruled affirmed the SAC and
granted the execution pending appeal and held that such is
consistent with justice, fairness, and equity, as respondents had
been deprived of the use and possession of their property
pursuant to RA 6657 and are entitled to be immediately
compensated with the amount as determined by the SAC under
the principle of “prompt payment” of just compensation.
Hence, this appeal.

ISSUE/s:
WoN the SAC decision should be upheld – YES
WoN respondents are entitled to execution pending appeal of the
compensation fixed by the sac based on the principle of prompt
payment of just compensation - YES

RULING: Court of Appeals decision is affirmed.

RATIO:
Petitioner Bank asserts that, according to our ruling in Land Bank of the
Philippines v. Court of Appeals, the principle of “prompt payment” of
just compensation is already satisfied by the concurrence of two (2)
conditions:
the deposits made by petitioner in any accessible bank,
equivalent to the DAR/LBP valuation of the expropriated
property as provisional compensation, must be in cash and
bonds as expressly provided for by Section 16(e) of RA
6657, not merely earmarked or reserved in trust
the deposits must be immediately released to the landowner,
even pending the final judicia determination of just
compensation.
The SC denies such appeal.
As provided in Sec. 2(a) of Rule 39 of the Rules of Court, execution of
the judgment or final order pending appeal is discretionary. As an
construed. Just compensation is defined as the full and fair equivalent of the
Thus, execution pending appeal should not be granted routinely but only property taken from its owner by the expropriator. It has been
in extraordinary circumstances. repeatedly stressed by this Court that the true measure is not the
The Rules of Court does not enumerate the circumstances which would taker’s gain but the owner’s loss.
justify the execution of the judgment or decision pending appeal.
However, the SC has held that “good reasons” consist of
compelling or superior circumstances demanding urgency
which will outweigh the injury or damages suffered should the
losing party secure a reversal of the judgment or final order.
In this case, do good reasons exist to justify the grant by the SAC of
the motion for execution pending appeal? The answer is a
resounding yes.
The expropriation of private property under RA 6657 is a
revolutionary kind of expropriation, being a means to obtain
social justice by distributing land to the farmers, envisioning
freedom from the bondage to the land they actually till.
As an exercise of police power, it puts the landowner, not the
government, in a situation where the odds are practically against
him. He cannot resist it.
His only consolation is that he can negotiate for the amount of
compensation to be paid for the property taken by the government.
As expected, the landowner will exercise this right to the hilt, subject to
the limitation that he can only be entitled to “just compensation.”
Clearly therefore, by rejecting and disputing the valuation of the
DAR, the landowner is merely exercising his right to seek just
compensation.
The SAC found that the valuation made by petitioner Bank, and affirmed
by the DAR, was unjustly way below the fair valuation of the
landholding at the time of its taking by the DAR.
The SAC, mindful also of the advanced age of respondents
(husband is 71 years old) at the time of the presentation of
evidence for the determination of just compensation and the
fact that all along, the land has been under the enjoyment of
farmer-beneficiaries without the respondents yet being paid
therefor, deemed it proper to grant their motion for execution
pending appeal with the objective of ensuring “prompt payment” of
just compensation.
Contrary to the view of petitioner, “prompt payment” of just compensation is
not satisfied by the mere deposit with any accessible bank of the
provisional compensation determined by it or by the DAR, and its
subsequent release to the landowner after compliance with the legal
requirements set by RA 6657.
Without prompt payment, compensation cannot be considered “just”
inasmuch as the property owner is made to suffer the
consequences of being immediately deprived of his land while
being made to wait for a decade or more before actually receiving
the amount necessary to cope with his loss.
Put differently, while prompt payment of just compensation requires the
immediate deposit and release to the landowner of the provisional
compensation as determined by the DAR, it does not end there.
Verily, it also encompasses the payment in full of the just compensation
to the landholders as finally determined by the courts.
Thus, it cannot be said that there is already prompt payment of just
compensation when there is only a partial payment thereof, as in this
case.
As correctly pointed out by respondents, the reversal of the November 20,
2000 SAC Decision, in the sense that petitioner will pay nothing at all to
respondents, is an impossibility, considering the constitutional mandate
that just compensation be paid for expropriated property.
DE KNECHT v. BAUTISTA (HENRY) extension of EDSA through Cuneta Ave instead of the new plan
October 30, 1980 | Fernandez, J. | Expropriation going through Fernando Rein and Del Pan Streets.
President Marcos referred the matter to the Human Settlements
PETITIONER: Cristina De Knecht Commission
RESPONDENTS: Hon. Pedro JL. Bautista, Presiding Judge of CFI
Pasay Branch 3, and the Republic of the Philippines

SUMMARY: MPH prepared a plan to extend EDSA to Roxas Boulevard


by passing through Cuneta Ave (A1) but then shortly after, plans were
changed and the extension was planned to pass through the streets of
Fernando Rein and Del Pan streets (A2). With the CFI ruling in favor of
the respondents, that they may push through the extension using the A2
route (despite recommendations by the President, through the Human
Settlements Commission).

Upon appeal to the SC, the claims of the petitioners were upheld. That
indeed, in consideration of the functionality, cost, and social impact, the
Republic shall push through with the original plan, that is the extension of
EDSA in compliance with the A1 plan.

DOCTRINE: There is no question as to the right of the Republic of the


Philippines to take private property for public use upon the payment of
just compensation. It is recognized, however, that the government may
not capriciously or arbitrarily choose what private property should be
taken.

FACTS:
It is alleged by the petitioners that more than 10 years ago, the
government, through Department of Public Works and
Communications (now Ministry of Public Highways or MPH),
prepared a plan to extend EDSA to Roxas Boulevard that the
proposed extension, an adjuct of another road-building program, the
Manila-Cavite Costal Road Project, would pass through the Cuneta
Avenue up to Roxas Boulevard; that this route would be a straight
one, taking int account the direction of EDSA.
That preparatory to the implementation of the said plan, the Secretary
Baltazar Aquino of the MPH directed the City Engineer of Pasay City
not to issue temporary or permanent permits for the construction
and/or improvement of buildings and other structures located within
the proposed extension through Cuneta Avenue (Alignment 1).
Shortly after the MPH decided to make the proposed extension through
Fernando Rein and Del Pan Streets (Alignment 2) which are lined
with old substantial houses, that upon learning o the changed plan,
the owners of the residential houses would be affected (aka the
petitioner, being one of them).
A formal petition to former President Marcos was filed asking him to
order the MPH to adopt the original plan of making the public
(HSC) for investigation and recommendation, who actually said that made studies before deciding on Cuneta, which is why it is odd why all of
the original plan should push through instead of the new one. a sudden the extension plan was instead changed to pass through
Regardless of such recommendation by the HSC, MPH insisted on Fernando Rein and
implementing the plan to make the extension through Fernando Rein
and Del Pan.
A complaint for expropriation was filed in the CFI, against the owners of
houses standing along Fernando Rein and Del Pan streets, among
them that of the petitioner’s.
CFI ruled in favor of the government and issued a writ of possession
authorizing the Republic to take and enter upon the possession of
the properties sought to be taken. Hence, a petition is made in the
SC.

ISSUE/s:
WoN alignment 2 must be used instead of alignment 1 – NO

RULING: SC granted the petition.

RATIO:
The petitioner in this case basically claims that the choice of property to
be expropriated cannot be without rhyme or reason. The condemnor
may not choose just any property. Where the legislature has
delegated a power of eminent domain, the question of the necessity
for taking a particular line for the intended improvement rests in the
discretion of the grantee power subject to review by the courts in
case.
Further, the petitioner claims that the choice of property must be
examined for bad faith, arbitrariness, or capriciousness and due
process, and that it requires determination as to whether or not the
proposed location was proper in terms of public interests.
Respondent Republic maintain that the ruling of the respondent court is
valid insofar as the compliance with all the statutory requirements
which entitled it to have immediate possession of the parties
involved.
Further they claim that there was no sudden change of plan in the
selection of the site, in fact, the residents were even notified of such
proposed plan.
There is no question that as to the right of the Republic of the Philippines
to take private property for public use upon ayment of just
compensation. However, it is recognized that the government may
not capriciously or arbitrarily hcoose what private property should be
taken.
In the case of JM Tuazon & Co., Inc v. Land Tenure Administration, the
SC said that a landowner is covered by the mantle of protection due
process affords. It is a mandate of reason. It frowns on arbitrariness;
it is the antithesis of any governmental act that smacks of whim or
caprice. It negates state power to act in an oppressive manner.
In the present case, it is a fact that the MPH originally established the
extension of EDSA along Cuneta Avenue. It is presumed that the MPH
De Pan instead.
HSC recommended based on the criteria of functionality, social impact,
and cost:
Functionality – Alignment 1 (Cuneta) is straighter than Alignment
2 (Del Pan-Fernando Rein). A1 is definitely the countour
conforming to EDSA and whereas A2 affords a greater
radious of unnatural curvature as it hooks slightly northward
before joining Roxas boulevard
Social impact – social impact is greater on the residents of A
Cost – despite A1 being slightly more expensive, that’s just about
it, SLIGHTLY more expensive. Hence cost should not be a
factor, but the functionality and social impact
It is to be noted that, the facts of record and the recommendations of the
Human Settlements Commission, it is evident that the changed plan
to extend EDSA through Fernando Rein and Del Pan is indeed
arbitrary and should not receive judicial approval. The
respondent judge indeed committed grave abuse of discretion
allowing the Republic to take immediate possession of the properties
sought to be expropriated.
MANOTOC v. NHA (DANNAH) program (ZIP) because the findings of the representative of the
May 21, 1987 | Gutierrez, Jr., J. | Eminent Domain: Due Process City of Manila and the

PETITIONER: Elisa Manotok et al.


RESPONDENTS: National Housing Authority, Republic of the Philippines

SUMMARY: PD 1669 and 1670 were issued by the President declaring


Tambunting Estate and Sunog-Apog as expropriated. This was pursuant
to the Letter of Instruction 555 instituting a nationwide slum improvement
and resettlement program. Elisa Manotoc, one of the property owners
whose land was “automatically expropriated” filed the present petition
questioning the PDs as violative of the Constitution. She questions the
manner at which just compensation was arrived at.

Owners of Tambunting Estate and Sunog-Apo followed suit. They


maintain that their right to due process and equal protection was violated.
1. because by the mere passage of the said decrees their properties
were automatically expropriated and they were immediately deprived of
the ownership and possession thereof without being given the chance to
oppose such expropriation or to contest the just compensation to which
they are entitled.

The SC ruled in favor of Manotoc et al., stating that there was no valid
basis for picking the areas to be expropriated, as these areas did not
have squatters and were actually developed areas. Moreover it ruled that
the market value stated by the city assessor alone cannot substitute for
the court's judgment in expropriation proceedings. It is violative of the
due process and the eminent domain provisions of the Constitution to
deny to a property owner the opportunity to prove that the valuation
made by a local assessor is wrong or prejudiced.

DOCTRINE: In other words, although due process does not always


necessarily demand that a proceeding be had before a court of law, it still
mandates some form of proceeding wherein notice and reasonable
opportunity to be heard are given to the owner to protect his property
rights.

FACTS:
On June 11, the President issued Letter of Instruction 555 instituting a
nationwide slum improvement and resettlement program (SIR). On
the same date, she also issued LOI 557, adopting slum improvement
as a national housing policy.
In compliance with LOI 555, the Governor of Metro Manila issued an
Executive Order adopting the Metroplitan Manila Zonal Improvement
Program which included the properties known as the Tambunting Estate
and the Sunog-Apog area in its priority list for a zonal improvement
National Housing Authority (NHA) described these as blighted communities.
On March 1978, a fire razed almost the entire Tambunting Estate. Following this
calamity, the President and the Metro Manila Governor made public
announcement that the national government would acquire the property for
the fire victims.
The President also designated the NHA to negotiate with the owners of the
property for the acquisition of the same. This, however, did not materialize as
the negotiations for the purchase of the property failed.
On December 1978, the President issued Proclamation No. 1810 declaring all sites
identified by the Metro Manila local governmetns and approved by the Ministry of
Human Settlements to be included in the ZIP upon eproclamation.
The estates aforementioned were among sites included.
On June 1980, the President issued the challenged PDs 1669 and 1670 which
respectively declared said sites expropriated.
On April 1980, the NHA, through its General Manager wrote the Register of
Deeds of Manila, furnishing copies of the PDs with the request that the
certificates of title covering the properties in question be cancelled and new
certificates of title be issued in the name of the Republic of the Philippines.
However, the Register of Deeds requested the submission of the owner's copy of
the certificates of title of the properties in question to enable her to implement
the aforementioned decrees.
Subsquently, petitioner Manotok, one of the owners of the properties to be
expropriated, received from the NHA a letter informing her that NHa had
deposited with the PNB the total amount of P5M which included the amount
of P3.4M representing the first annual installment for the Tambunting estate
pursuant to PD 1669.
Also another P5M which included the amount of P1.6M representing the first
annual installment for Sunog-Apog area under PD 1670.
Manotok was also informed that she was free to withdraw her share in the
properties upon surrender by her of the titles pertaining to said properties.
Moreover, if she failed to avail herself the offer, NHA would take necessary legal
steps to implement the decrees.
On August 1980, Manotok wrote to NHA alleging that the amounts as fixed in the
decrees do not constitute the “just compensation” envisioned in the
Constitution.
She expressed doubts about the constitutionality of the decrees and informed
NHA that she did not believe that she was obliged to withdraw P5M or
surrender her titles over the properties.
Meanwhile, some officials of the NHA circulated instructions to the tenants-
occupants of the properties in dispute not to pay their rentals to the Manotok
et al. for their lease-occupancy of the properties in view of the fixed by decree, but the decisions, rulings, orders, or
passage of P.D. Nos. 1669 and 1670. resolutions of the NHA are
Hence, the owners of Tambunting Estate filed a petition to declare
unconstitutional PD 1669, and the owners of Sunog-Apog followed
suit with regard to PD 1670.
Manotok et al maintain that the PDs violate their constitutional right to
due process and equal protection of the law because by the mere
passage of the said decrees their properties were automatically
expropriated and they were immediately deprived of the ownership
and possession thereof without being given the chance to oppose
such expropriation or to contest the just compensation to which they
are entitled.

ISSUE/s:
WoN the PDs are unconstitutional for not following Rule 67 of the
Revised Rules of Court – YES

RULING: Petitions granted. PDs 1669 and 1670 declared unconstitutional.

RATIO:
The power of eminent domain is inherent in every state and the
provisions in the Constitution pertaining to such power only serve to
limit its exercise in order to protect the individual against whose
property the power is sought to be enforced.
Republic v. Juan: The right of eminent domain is inherent in the
Republic of the Philippines as a body sovereign. It is not subject to
any other limitation other than those imposed the by the Constitution
which are:
The taking must be for a public use
The payment of just compensation must be made
Due process must be observed in the taking
The challenged decrees are unfair in the procedures adopted and the
powers given to NHA.
The Tambunting area was proclaimed to be blighted and directly
expropriated without the slightest semblance of a hearing or any
proceeding whatsoever. The expropriation is instant and automatic to
take effect upon the signing of the decree.
The P3.4M appropriated is not a deposit but constitutes an installment
payment, the maximum price of which is fixed so as not to exceed
P17M.
There is no provision for any interests to be paid on the unpaid
installments spread out over a period of five years.
Not only are the owners given absolutely no opportunity to contest the
expropriation, plead their side, or question the amount of payments
expressly declared as beyond the reach of judicial review.
An appeal may be made to the Office of the President but the courts are
completely enjoined from any inquiry or participation whatsoever in the
expropriation of the subdivision or its incidents.
Haguisan v. Emilia: Court sustained the contention that prior hearing is no
longer necessary under PD 42 in ascertaining the value of property to be
expropriated and before government may take possession.
NHA v. Reyes: Court upheld decrees which state that the basis for just
compensation shall be the market value declared by the owner for tax
purposes or such market value as determined by the government assessor,
whichever is lower.
Subsequent developments have shown that a disregard for basic liberties and
the shortcut methods embodied in the decrees on expropriation do not
achieve the desired results.
The due process clause cannot be rendered nugatory everytime a specific
decree or law orders the expropriation of somebody's property and provides
its own peculiar manner of taking the same.
Neither should the courts adopt a hands-off policy just because the public use
has been ordained as existing by the decree or the just compensation has
been fixed and determined bef orehand by a statute.
Dohany v. Rogers: Underscores the extend by which the due process clause
guarantees protection from arbitrary exercise of the power of eminenet
domain.
In other words, although due process does not always necessarily demand that a
proceeding be had before a court of law, it still mandates some form of
proceeding wherein notice and reasonable opportunity to be heard are given
to the owner to protect his property rights.
Although there are exceptional situations when, in the exercise of the power of
eminent domain, the requirement of due process may not necessarily entail
judicial process, the courts will have to step in and probe into such an
alleged violation where it is alleged that a person’s right to due process has
been violated in the taking of his property.
De Knecht v. Bautista: Although the Republic may exercise its power of
eminent domain, it is recognized that the government may not capriciously or
arbitrarily choose what private property should be taken.
Same case: With due recognition then of the power of Congress to designate the
particular property to be taken and how much thereof may be condemned in the
exercise of the power of expropriation, it is still a judicial question whether in the
exercise of such competence, the party adversely affected is the victim of
partiality and prejudice. That the equal protection clause will
not allow. The provisions of the decree on relocation of squatter families and on
City of Manila v. Chinese Community: The foundation of the right to the re-blocking of existing structures to allow introduction of basic
exercise eminent domain is a genuine necessity and that necessity facilities and services have no basis.
must be of a public character.
City of Manila v. Arellano Law: A necessity must exist for the taking of
private property for the proposed uses and purposes but accepted
the fact that modern decisions do not call for absolute necessity.
In the petitions at hand, there is no showing whatsoever as to why the
properties involved were singled out for expropriation through
decrees or what necessity impelled the particular choices or
selections.
In expropriations through legislation, there are, at least, debates in
Congress open to the public, scrutiny by individual members of the
legislature, and very often, public hearings before the statute is
enacted. Congressional records can be examined.
In the case at hand, the decrees show no reasons whatsoever for the
choice of the properties as housing projects.
The Tambunting Estate is valuable commercial property. It is a foregone
conclusion that the favored squatters allowed to buy these choice
lots would lose no time, once it is possible to do so, to either lease
out or sell their lots to wealthy merchants even as they seek other
places where they can set up new squatter colonies. The public use
and social justice ends stated in the whereas clauses of P.D. 1669
and P.D. 1670 would not be served thereby.
The provision of PD 1669 allowing NHA, at its sole option, to put portions of
the expropriated area to commercial use in order to defray the
development costs of its housing projects cannot stand constitutional
scrutiny.
The legislature, according to the Guido case, may not take the property
of one citizen and transfer it to another, even for a full compensation,
when the public interest is not thereby promoted.
The Government still has to prove that expropriation of commercial
properties in order to lease them out also for commercial purposes
would be "public use" under the Constitution.
PD 1670 suffers from a similar infirmity. There is no showing how the
President arrived at the conclusion that the Sunog-Apog area is a
blighted community.
The many pictures submitted as exhibits by the petitioners show a well-
developed area subdivided into residential lots with either middle-
income or upper class homes. There are no squatters.
The area is well-developed with roads, drainage and sewer facilities, water
connections to the Metropolitan Waterworks and Sewerage System,
electric connections to Manila Electric Company, and telephone
connections to the Philippine Long Distance Telephone Company.
There are many squatter colonies in Metro Manila in need of upgrading. The
Government should have attended to them first. There is no showing for
a need to demolish the existing valuable improvements in order to
upgrade Sunog-Apog.
The Court finds both PDs violative of Manotoc et al’s right to due process of
law and therefore, must fail the test of constitutionality.
The decrees do not by themselves, provide for any form of hearing or
procedure by which the petitioners can question the propriety of the
expropriation of their properties or the reasonableness of the just
compensation.
Having failed to provide for a hearing, the Government should have filed an
expropriation case under Rule 67 of the Revised Rules of Court but it did
not do so.
In effect, the properties, under the decrees were "automatically expropriated."
This became more evident when the NHA wrote the Register of Deeds and
requested her to cancel the certificate of titles of the petitioners, furnishing
said Register of Deeds only with copies of the decrees to support its request.
This is hardly the due process of law which the state is expected to observe
when it exercises the power of eminent domain.
The government states that there is no arbitrary determination of the fair
market value of the property by the government assessors because if the
owner is not satisfied with the assessor's action.
He may within sixty (60) days appeal to the Board of Assessment Appeals of
the province or city as the case may be and if said owner is still
unsatisfied, he may appeal further to the Central Board of Assessment
Appeals pursuant to P.D. No. 464.
The Government argues that with this procedure, the due process
requirement is fulfilled.
The Court cannot sustain the argument.
PD 464, as amended, otherwise known as the Real Property Tax Code,
provides for the procedure on how to contest assessments but does not
deal with questions as to the propriety of the expropriation and the
manner of payment of just compensation in the exercise of the power of
eminent domain.
We find this wholly unsatisfactory. It cannot in anyway substitute for the
expropriation proceeding under Rule 67 of the Revised Rules of Court.
Another infirmity is the determination of just compensation. case, should not be binding on the property owners for, as stated in
Pursuant to PD 1533, the basis of the just compensation is the market the above cases,
value of the property "prior to the recommendation or decision of the
appropriate Government Office to acquire the property.”
In these petitions, a maximum amount of compensation was imposed by
the decrees and these amounts were only a little more than the
assessed value of the properties in 1978 when, according to the
government, it decided to acquire said properties.
The fixing of the maximum amounts of compensation and the bases
thereof which are the assessed values of the properties in 1978
deprive the petitioner of the opportunity to prove a higher value
because, the actual or symbolic taking of such properties occurred
only in 1980 when the questioned decrees were promulgated.
A television or news announcement or the mere fact of the property's
inclusion in the Zonal Improvement Program (ZIP) cannot suffice as
formal notice because for the compensation to be just, it must
approximate the value of the property at the time of its taking and the
government can be said to have decided to acquire or take the
property only after it has, at the least, commenced a proceeding,
judicial or otherwise, for this purpose.
Municipality of Daet v. CA: Court upheld the determination of just
compensation and the rationale behind it either at the time of the
actual taking of the government or at the time of the judgment by the
court, whichever came first
Furthermore, the so-called "conditions" of the properties should not be
determined through a decree but must be shown in an appropriate
proceeding in order to arrive at a just valuation of the property.
Garcia v. CA: …All the facts as to the condition of the property and its
surroundings, its improvements and capabilities may be shown and
considered in estimating its value.
In other cases involving expropriations under certain PDs, the Court decided
to invalidate the mode of fixing just compensation under said decrees.
With more reason should the method in PDs 1669 and 1670 be declared
infirm.
The market value stated by the city assessor alone cannot substitute for
the court's judgment in expropriation proceedings. It is violative of the
due process and the eminent domain provisions of the Constitution
to deny to a property owner the opportunity to prove that the
valuation made by a local assessor is wrong or prejudiced.
The maximum amounts, therefore, which were provided for in the questioned
decrees cannot adequately reflect the value of the property and, in any
there are other factors to be taken into consideration.
Teehankee, Concurring: The judgment at hand overturns the majority ruling
in Tuason.
“The power of Congress to designate the particular property to be taken and
how much thereof may be condemned in the exercise of the power of
"expropriation" must be duly recognized, leaving only as "a judicial
question whether in the exercise of such competence, the party
adversely affected is the victim of partiality and prejudice. That the equal
protection clause will not allow."
The Court now clearly rules that such singling out of properties to be
expropriated by Presidential Decree as in the case at bar, or by act of the
legislature as in Tuason, does not foreclose judicial scrutiny and
determination as to whether such expropriation by legislative act
transgresses the due process and equal protection, and just
compensation guarantees of the Constitution.
As in all eminent domain proceedings, the State may not capriciously or
arbitrarily single out specific property for condemnation and must show
the necessity of the taking for public use.
REPUBLIC THE PHILIPPINES v. DE KNECHT (ARIELLE) On July 16, 1979 (first case), De Knecht filed with the SC a petition for
February 12, 1990 | Gancayco, J. | Expropriation certiorari, praying that the respondent be commanded to desist from
further
PETITIONER: Republic of the Philippines
RESPONDENTS: Cristina De Knecht and Court of Appeals

SUMMARY: The Republic filed an expropriation proceeding against the


owners of the houses along Fernando Rein-Del Pan streets. De Knecht
filed a motion to dismiss. Subsequently, De Knecht filed for a TRO to
restrain the Republic from proceeding with the taking of the property. The
Republic already deposited the 10% of the amount of compensation. The
lower court authorized the Republic to enter and take possession of the
property. Subsequently, BP 340 was enacted, expropriating the same
properties for the same purpose. De Knecht appealed to the CA, which
reversed the trial court’s decision. In the first case, the SC held that the
choice of streets was arbitrary and should not receive judicial approval.

The Republic appealed again, and the SC held that, due to supervening
events (De Knecht’s property was the only obstacle in the way that is
preventing the Government from completing its project), the Court is
constrained to reverse its previous decision regarding the same case.

DOCTRINE: While it is true that the final judgment of the SC on the


subject becomes the law of the case between the parties, it is equally
true that the right of the Republic to take private properties for public use
upon the payment of the just compensation is provided in the
Constitution and our laws.

FACTS:
The Republic of the Philippines filed in the CFI an expropriation
proceeding against the owners of the houses along Fernando Rein-
Del Pan streets, among them was Cristina De Knecht together with
Concepcion Cabarrus, and some other 15 defendants.
De Knecht filed a motion to dismiss alleging lack of jurisdiction, pendency
of appeal with the President of the Philippines, prematureness of
complaint, and arbitrary and erroneous valuation of the properties.
Subsequently, De Knecht filed for a TRO to restrain the Republic from
proceeding with the taking of immediate possession and control of
the property to be condemned.
Republic filed for an issuance of a writ of possession of the property on
the ground that it had made the required deposit with the PNB of
10% of the amount of compensation stated in the complaint.
The lower court issued a writ of possession authorizing the Republic to
enter into and take possession of the properties, and created a
Committee of three to determine the just compensation for the lands.
proceeding in the expropriation action. The SC granted the petition, Subsequently, BP 340 was enacted. As its basis, Republic filed a motion
setting aside the order authorizing the Republic to take or enter upon to dismiss the case before the trial court and was granted.
the possession of the properties.
Defendants de Elizalde and Roxas moved to dismiss the expropriation
action in compliance with the dispositve portion of the aforesaid
decision of the SC which had become final.
The Republic filed a motion to dismiss said case due to the enactment of
BP 340 expropriating the same properties and for the same purpose.
The lower court dismissed the case by reason of the enactment of
said law. MR was denied.
De Knect appealed to the CA, which set aside the order of the lower
court. Hence this petition.

ISSUE/s: WoN an expropriation proceeding that was determined by a final


judgment of the Court may be the subject of a subsequent legislation for
expropriation – YES.

RULING: Petition is GRANTED and the decision of the CA is REVERSED


and SET ASIDE.

RATIO:
There is no question that as early as 1977, the national government,
through DPWH, began work on what was to be the westward
extension of EDSA. The projects were aimed at easing traffic
congestion in Baclaran, controlling flood, and completing the Manila
Food and Control Drainage Project.
The Republic acquired the properties through negotiated purchase about
80-85% of the lands involved in the project whose owners did not
raise any objection as to arbitrariness on the choice of the project
and its route.
It is only with respect to the remaining 10-15% that the Republic cannot
negotiate through a sales agreement with a few land owners,
including De Knecht whose holding is hardly 5% of the whole route
area. Thus, the Republic filed the expropriation proceeding.
In the decision of the Court in De Knecht v. Bautista, the Court held that
the choice of the streets as the line through which ESDA should be
extended to Roxas Boulevard is arbitrary and should not receive
judicial approval. It is based on the recommendation of the Human
Settlements Commissino that Cuneta street will minimize the social
impact as the buildings therein are mostly motels.
In view of the finding, this Court set aside the order of the trial court
authorizing the Republic to take possession of the properties. Said
decision became final as no action was taken by the lower court on
the directive.
On appeal, the CA held that the decision of the SC having became final,
the Republic’s right as determined should no longer be disturbed.
Thus, CA set aside the questioned order of the trial court and issued
another order dismissing the expropriation proceedings before the
lower court.
While it is true that the final judgment of the SC on the subject
becomes the law of the case between the parties, it is equally
true that the right of the Republic to take private properties for
public use upon the payment of the just compensation is
provided in the Constitution and our laws. Such expropriation
proceedings may be undertaken by the Republic not only by
voluntary negotiation with the landowners, but also by taking
appropriate court action or legislation.
When BP 340 was passed expropriating the very properties of the
present proceedings, it appears that it was based on supervening
events that occurred after the decision of the SC was rendered in De
Knecht, justifying the expropriation through the Fernando-Del Pan
streets. The social impact factor which persuaded the SC to consider
this extension to be arbitrary had disappeared. All residents in the
area have been relocated and duly compensated.
80% of EDSA outfall and 30% of EDSA extension had been completed.
Only private respondent (De Knecht) remains as the solitary obstacle
to the project. The instant case stands in the way of the final solution
of the problems sought to be remedied by the EDSA extension
because the single piece of property ‘occupied’ by De Knecht,
although already expropriated umder BP 340, is the only parcel of
land where the Government engineers could not enter due to the
‘armed’ resistance offered by De Knecht.
The Court finds justification in proceeding with the said
expropriation proceedings due to the supervening events after
the rendition of the Decision of this Court in De Knecht.
BP 340 therefore effectively superseded the final and executory decision
of the Court.
REPUBLIC v. BPI (IYA) A MR was filed by BPI but was denied.
September 11, 2013 | Carpio, J. | Eminent Domain A new trial before the Board of Commissioners ensued for the
determination of just compensation for the building.
PETITIONER: Republic of the Philippines, Department of Public Works and
Highways
RESPONDENTS: Bank of the Philippine Islands

SUMMARY: DPWH filed a case for expropriation against BPI. Such was
granted by the courts. BPI, however, seeks to have the building erected
on such land paid as well. DPWH contends that the original plan
affecting building was not implemented, therefore should not be made to
pay for such.

The RTC ordered DPWH to pay BPI for consequential damages as a


result of the exporpriation in the amount of P1.9M. DWPH appealed the
RTC order but was denied. The SC ruled in favor of BPI, saying that the
rules on expropriation clearly provide a legal basis for the award of
consequential damages.

DOCTRINE: Consequential damages are awarded if as a result of the


expropriation, the remaining property of the owner suffers from an
impairment or decrease in value.

FACTS:
On 12 February 1998, Department the of Public Works and Highways
(DPWII) filed with the Regional Trial Court a case for expropriation
against portions of the properties of Bank of the Philippine Islands
(BPI) and of Bayani Villanueva (Villanueva) situated in Pamplona,
Las Piñas City.
Neither BPI nor Villanueva objected to the propriety of the expropriation;
hence, the trial court constituted a Board of Commissioners to
determine the just compensation.
The Board of Commissioners recommended the amount of P40,000.00
per square meter as the fair market value. On 25 November 1998,
the trial court in its Decision set the fair market value at P40,000.00
per square meter.
Meanwhile, BPI filed on 16 December 1998 a Motion for Partial New Trial
to determine the just compensation of its building, which was not
included in the Decision dated 25 November that fixed the just
compensation for the parcels of land. The trial court granted partial
new trial.
BPI was allowed to continue with the trial ex parte, due to the absence of
response from DPWH.
The government through a trial court order is made to pay BPI the
amount of P2,633,000.00 for the building.
DPWH moved for reconsideration on the ground that the proceeding
fixing the just compensation of the building is null and void for not
complying with the mandatory procedure. This was granted.
DPWH filed a motion before the RTC objecting to the propriety of paying
just compensation for BPI’s building and praying that BPI’s claim for
additional just compensation be denied because the original plan
affecting the subject property was not implemented.
BPI contends that it was not informed that the original plan was not
implemented.
The RTC denied the DWPH motion and ruled that the demolition of the
old building of BPI can be construed as a consequential damage
sugared by the BPI as a result of the expropriation.
DWPH disordered to submit a nominee to the Board of Commissioners.
Commissioner for BPI Savellano recommended the amount of
P2,633,000.00, which was based on the appraisal conducted by an
independent professional business and property consultant.
On the other hand, Commissioner for petitioner Gervacio recommended
the amount of P1,905,600.00.
The RTC issued the Order dated 03 February 2003, adopting the
recommendation of Gervacio of P1,905,600.00.
DPWH filed an appeal with the CA. The CA dismissed the appeal and
affirmed the order of the RTC.
Hence, this petition.

ISSUE/s:
WON the award of additional just compensation for BPI’s building in the
amount fixed therefor is unfounded and without legal basis - NO

RULING: WHEREFORE, we DENY the petition. We AFFIRM the Court of


Appeals’ Decision.

RATIO:
Eminent domain is the authority and right of the State, as sovereign, to
take private property for public use upon observance of due process
of law and payment of just compensation.
The State’s power of eminent domain is limited by the constitutional
mandate that private property shall not be taken for public use
without just compensation.
Just compensation is the full and fair equivalent of the property sought to
be expropriated
The general rule is that the just compensation to which the owner of the
condemned property is entitled to is the market value. The general
rule, however, is modified where only a part of a certain property is
expropriated.
In such a case, the owner is not restricted to compensation for the
portion actually taken, he is also entitled to recover the consequential
damage, if any, to the remaining part of the property.
DPWH contends that BPI’s building was “never taken” by the
government, and that to award consequential damages for the
building was unfounded and without legal basis.
No actual taking of the building is necessary to grant consequential damages.
Consequential damages are awarded if as a result of the expropriation,
the remaining property of the owner suffers from an impairment or
decrease in value.
The rules on expropriation clearly provide a legal basis for the award of
consequential damages. Section 6 of Rule 67 of the Rules of Court.
NPC v. MANALASTAS (ELIEL) However, NPC assailed that the devaluation of the peso should not have
January 27, 2016 | Peralta, J. | Expropriation been put into consideration. Hence this petition.

PETITIONER: National Power Corporation


RESPONDENTS: Elizabeth Manalastas and Bea Castillo

SUMMARY: NPC constructed transmission lines on the parcel of land of


Manalastas without consent and expropriation. Manalastas contends that
it should removed the transmission lines and pay damages or
alternatively pay them with just compensation of P800 per sqm. NPC
assails the decision ordering them to pay P800 per sqm, because it
included the inflation rate wherein the market value at the time of taking
was only P170 per sqm.

The SC held that in determining just compensation the inflation rate


should not be included in the computation because it is already
accounted for in the payment of interest and other damages. Doing so
would be incongruous and unfair to the party expropriating. Thus, just
compensation should only be the market value at the time of taking.

DOCTRINE: To recapitualate, the formula for determination of just


compensation to landowners does not include the factor for inflation rate,
as inflation is properly accounted for throught payment of interest on the
amount due to the landowner, and through the award of exemplary
damages and attorney’s fees in cases where there was irregularity in the
taking of property.

FACTS:
National Power Corporation (NPC), a GOCC involved in the development
of hydro-electric generation of power and production of electricity,
and the construction, operation and maintenance of powr plants,
trnamission lines, power stations and substations, among others,
constructed a 230 KV transmission line for the Naga-Tiwi line and a
69 KV transmission line for the Naga-Tinambac line on Manalastas’
parcel of land affecting an area of 26,919 sqm.
NPC entered said land without the knowledge or consent of Manalasta,
without properly intitiation expropriation proceedings, and without
any compensation to Manalastas.
Because of said transmission lines, Manalastas alleged that they could
no longer use their land as part of a subdivision project as originally
intended, which ultimately caused financial loss to their family.
Thus, Manalastas filed a complaint against NPC and its officers with the
RTC. Demanded the removal of the power lines and its accessories
and payment of damages, or in the alternative, payment of the fair
market value of the affected areas totaling 26,000 sqm. at P800 per
sqm.
ISSUE/s:
WoN inflation should be included in determining just compensation – NO

RULING: Granted. Only liable to pay just compensation of P170, value at the
time of taking.

RATIO:
None of the parties contest the finding that the fair market value of the
property at the time of taking was P170 per sqm.
Sec. of Public Works and Highways v. Sps. Tecson: The Court
stressed that “just compensation is the value of the property at
the time of taking that is controlling for purposes of
compensation.”
In a motion for reconsideration of the Decision in said case, the
landowners argued that it would be unjust if the amount that
will be awarded to them today will be base on the value of the
property at the time of actual taking.
The foregoing clearly dictates that valuation of the land for
purposes of determingin just compensation should not include
the inflation rate of the PHP because the delay in payment of the
price of expropriated land is sufficiently recompensed through
payment of interest on the market value of the land as of the
time of taking from the landowner.
Moreover, the fact that it was NPC’s own counsel below that
recommended the inclusion of the inflation rate in the determination
of just compensation should not be taken against NPC.
It is incongruous for the court below to uphold a proposition merely
because it was recommended by a party, despite the same being
erroneous.
Lastly, in addition to the award for interest, Art 2229 of the Civil Code
provides that “exemplary or corrective damages are imposed by way
of example or correction for the public good” and Art 2208 of the
same code states that attorney’s fees may be awarded by the court
in cases where such would be just and equitable.
Indeed, government agencies should be admonished and made to
realize that its negligence and inaction in failing to commence the
proper expropriation procedings before taking private property, as
provided for by law, cannot be countenanced by the Court.
To recapitualate, the formula for determination of just
compensation to landowners does not include the factor for
inflation rate, as inflation is properly accounted for through
payment of interest on the amount due to the landowner, and
through the award of exemplary damages and attorney’s fees in
cases where there was irregularity in the taking of property.
HOME BUILDING LOAN ASSN. v. BLAISDELL (JP) may obtain an extension for such time as the court may deem just
January 8, 1934 | Hughes, C. J. | Obligation of Contracts and equitable

PETITIONER: Home Building Loan Association


RESPONDENTS: Blaisdell

SUMMARY: The Sec. 4 of the Minnesota Mortgage Moratorium Law


allowed the temporary extension of the redemption of real property from
foreclosures if such public emergency exists and justifies such. During
that time there existed a sever financial and economic depression.
Appellees herein, spouses Blaisdell (the case did not mention names),
filed for the extension to redeem their property which was foreclosed
citing such depression. Appellant Home Building Assn. (again, not sure of
who the parties are) who bought such property held that such law impairs
obligations and contracts and should thus be void. District Court granted
appellant’s motion to dismiss. SC of Minnesota reversed.

The SC held that an emergency does exist and the extension of the
period for redemption is justified to protect both parties in a mortgage,
and due to such operates for a legitimate end. It also held that since the
mortgagor (appellant Home Building) was still being paid even during the
extended period with rents by the mortgagee (appellee spouses), he is
not prejudiced. The law also operates temporarily or only until the
emergency has ceased. The extension is affirmed.

DOCTRINE: Economic or emergency conditions may arise in which a


temporary restraint of enforcement of contracts will be consistent with the
spirit and purpose of the contract clause, and thus be within the range of
the reserved power of the State to protect the vital interests of the
community.

FACTS:
195. A Minnesota statute, called the Minnesota Mortgage Moratorium
Law, declares the existence of an emergency demanding an
exercise of the police power for the protection of the public and to
promote the general welfare of the people, by temporarily
extending the time allowed by existing law for redeeming real
property from foreclosure and sale under existing mortgages.
A severe financial and economic depression has existed for
several years, resulting in almost complete lack of credit for
farmers, business men and property owners, and in extreme
stagnation of business, agriculture and industry.
That, under the existing conditions, foreclosure of many real
estate mortgages by advertisement would prevent fair, open
and competitive bidding in the manner contemplated by law.
The Act then provides, inter alia, as to foreclosure sales, that, where the
period for redemption has not already expired, the mortgagor or owner
in possession, by applying to a state court before its expiration,
(Section 4), but in no case beyond May 1, 1935. people, and is paramount to any rights under contracts between
The court is empowered to alter the terms of extensions as change of individuals.
conditions may require. The Act automatically extends, to 30 days
from its date, redemption periods which otherwise would expire
within that time. It is to remain in effect only during the emergency,
and in no event beyond May 1, 1935.
The appellees (spouses Blaisdell) applied to the District Court of
Hennepin for an order extending the period of redemption from a
foreclosure sale. Their petition stated that they owned a lot in
Minneapolis which they had mortgaged to appellant Home Building
Loan Assn. (Sidenote: the case was confusing on who are the
parties, you are better of saying appellant/appellee
than the names I’ve provided. I only inferred that these were the
parties)
199. By reason of their default, the mortgage had been foreclosed and sold
to appellant Home Building. that, because of the economic depression
appellees Blaisdell had been unable to obtain a new loan or to redeem,
and that, unless the period of redemption were extended, the property
would be irretrievably lost, and that the reasonable value of the property
greatly exceeded the amount due on the mortgage, including all liens,
costs and expenses.
Appellant Home Building objected to the introduction of evidence upon
the ground that the statute was invalid under the federal and state
constitutions, and moved that the petition be dismissed on the
ground that it violates the non-impairment of contracts.
District court granted the motion to dismiss. Supreme Court of the State
reversed. The latter courted extended such redemption subject to
conditions that appellees should still pay $40 a month through the
extended period and additional amounts which go to the payment of
taxes, insurance, interest, and mortgage indebtedness.
This decision is here under review.

ISSUE/s:
WoN such public economic emergency exists in the case at bar - YES
WoN such public economic emergency justified the impairment of the
obligation in question – YES

RULING: Judgment in question is affirmed.

RATIO:
The interdiction of statutes impairing the obligation of contracts does not
prevent the State from exercising such powers as are vested in it for
the promotion of the common weal, or are necessary for the general
good of the public, though contracts previously entered into between
individuals may thereby be affected.
This power, which in its various ramifications is known as the police
power, is an exercise of the sovereign right of the Government to
protect the lives, health, morals, comfort and general welfare of the
The question is not whether the legislative action affects contracts could not validly outlast the emergency or be so extended as
incidentally, or directly, or indirectly, but whether the legislation is virtually to destroy the contracts.
addressed to a legitimate end and the measures taken are
reasonable and appropriate to that end.
Another argument, which comes more closely to the point, is that the
state power may be addressed directly to the prevention of the
enforcement of contracts only when these are of a sort which the
legislature in its discretion may denounce as being in themselves
hostile to public morals, or public health, safety or welfare, or where
the prohibition is merely of injurious practices.
It cannot be maintained that the constitutional prohibition should be so
construed as to prevent limited and temporary interpositions with
respect to the enforcement of contracts if made necessary by a great
public calamity
such as fire, flood, or earthquake.
260. An emergency did exist in Minnesota which furnished a proper
occasion for the exercise of the reserved power of the State to
protect the vital interests of the community.
261. As the Supreme Court of Minnesota said, the economic emergency
which threatened "the loss of homes and lands which furnish those in
possession the necessary shelter and means of subsistence" was a
"potent cause" for the enactment of the statute.
The legislation was addressed to a legitimate end, that is, the legislation
was not for the mere advantage of particular individuals, but for the
protection of a basic interest of society.
The conditions upon which the period of redemption is extended do not
appear to be unreasonable.
The initial extension of the time of redemption for thirty days from the
approval of the Act was obviously to give a reasonable opportunity
for the authorized application to the court.
The integrity of the mortgage indebtedness is not impaired; interest
continues to run; the validity of the sale and the right of a mortgagee-
purchaser to title or to obtain a deficiency judgment if the mortgagor
fails to redeem within the extended period are maintained, and the
conditions of redemption, if redemption there be, stand as they were
under the prior law.
The mortgagor, during the extended period, is not ousted from
possession, but he must pay the rental value of the premises as
ascertained in judicial proceedings, and this amount is applied to the
carrying of the property and to interest upon the indebtedness.
The mortgagee-purchaser, during the time that he cannot obtain
possession, thus is not left without compensation for the withholding
of possession.
The legislation is temporary in operation. It is limited to the exigency
which called it forth. While the postponement of the period of
redemption from the foreclosure sale is to May 1, 1935, that period
may be reduced by the order of the court under the statute, in case
of a change in circumstances, and the operation of the statute itself
Devanter, McReynolds, Butler Concurring: The statute, is not merely a
modification of the remedy; it effects a material and injurious change in
the obligation. The legally enforceable right of the creditor when the
statute was passed was, at once upon default of redemption, to become
the fee simple owner of the property. Extension of the time for
redemption for two years, whatever compensation be given in its place,
destroys that specific right and the correlative obligation.
It cannot be foreseen what will happen to the property during that long period
of time. The buildings may deteriorate in quality; the value of the property
may fall to a sum far below the purchase price; the financial needs of
appellant may become so pressing as to render it urgently necessary
that the property shall be sold for whatever it may bring.
However, the dissenters quite agree with the opinion of the court that
whether the legislation under review is wise or unwise is a matter with
which they have nothing to do. Whether it is likely to work well or work ill
presents a question entirely irrelevant to the issue. Being unable to reach
any other conclusion than that the Minnesota statute infringes the
constitutional restriction under review, they have no choice but to say so.
RUTTER v. ESTEBAN (HENRY) Sec 2 of RA 342, payment of his obligation cannot be enforced until after
May 18, 1953 | Bautista, J. | Contracts the lapse of 8 yrs form settlement of his claim by PH War Damage
Commission, and this period has not expired
PETITIONER: Royal Rutter
RESPONDENTS: Placido Esteban

SUMMARY: Rutter sold to Esteban two parcels of land in Manila, but


there was a balance that was not paid for by Esteban. Rutter filed a case
against Esteban in the CFI, and in defense, Esteban used RA 342
(Moratorium Law), claiming that his obligation shall not arise yet as the
period of 8 years as indicated in the statute has not yet lapsed. CFI ruled
in favor of Esteban, hence a file to the SC by Rutter was made.

SC held that RA 342 shall be struck down for it being oppressive and
unconventional to the part of the creditor. Although moratorium laws, as
backed up by jurisprudence, may be declared as constitutional, it was
because the circumstances purported for such statutes to be reasonable.
In application to the present case, a Moratorium Law is no longer
necessary, as the country has recovered from the war and the financial
state of the people have been restored to its normal state. Prolonging the
validity of such statute will be an invalid exercise of police power, as
contracts will be impaired insofar as equity between the debtor and the
creditor is concerned.

DOCTRINE: The true test of the constitutionality of a moratorium


therefore lies in the determination of the period of suspension of the
remedy. It is required that such suspension be definite and reasonable,
otherwise it would be violative of the constitution.

FACTS:
August 1941, petitioner Rutter sold to respondent Esteban 2 parcels of
land in Manila for the sum of Php 9,600, of which Php 4,800 were
paid outright, and the balance (other half) was made payable as
follows:
Php 2,400 on or before Aug 1942
Php 2,400 on or before Aug 1943, with interest rate at 7% per
annum
To secure payment of said balance of 4.8k, a first mortgage over the
same parcels of land was constituted. Esteban failed to pay the 2
installments as agreed, as well as the interest that had accrued.
An action was filed against him by Rutter in the CFI, which contains a
prayer for the sale of the properties mortgaged in accordance with
law.
Esteban admitted theat he was unable to pay, but set up as a defense the
moratorium clause embodied in RA 342. He claims that this is a
prewar obligation contract (dated Aug 1941), that he is a war sufferer,
having filed his laim with the PH War Damage Commission for the
losses he had suffered as consequence of the last war, and that under
yet.
CFI ruled that the obligation is not yet demandable under the moratoroim
law. MR denied, hence this appeal.

ISSUE/s:
WoN RA 342 (Moratorium Law) is constitutional – NO

RULING: SC reversed the CFI ruling.

RATIO:
Section 2, RA 342: “All debts and other monetary obligations contracted
before Dec 8, 1941, any provision in the contract creating the same
or in any subsequent agreement affecting such obligation to the
contrary notwithstanding, shall not be due and demandable for a
period of 8 years from and after settlement of the war damage claim
of the debtor by the PH War Damage Commission.
It clearly appears in the said act that the nullificatioin of its provisions will
have the effect of reviving the previous moratorium orders issued by
the President of the PH.
Statues declaring a moratorium on the enforcement of monetary
obligations are not of recent enactment. These moratorium laws are
not new.
The moratorium is a postponement of fulfillment of obligations decreed
by the state through the medium of the courts or the legislature.
These laws were declared constitutional. However, some courts have
also declared that “such statutes are void as to contracts made
before their passage where the suspension of remedies prescribed is
indefinite or is unreasonable in duration”.
The true test of the constitutionality of a moratorium therefore lies in the
determination of the period of suspension of the remedy. It is
required that such suspension be definite and reasonable, otherwise
it would be violative of the constitution.
One of the arguments advanced assailing the moratorium law is that it
impairs the obligation of contract which is prohibited by the
Constitution. This argument does not hold water. While it may be
conceded, it is however justified as a valid exercise of police power.
In the case of Home Bldg and Loan Assoc v. Blaisdell, the validity of Charter
339 of the laws called the Minnesota Mortgage Moratorium Law is being
assailed as being repugnant to the contract clause of the Federal
Constitution. SC held that, although conceding that the obligations of the
mortgage contract were impaired, the court decided that what is thus
described as an impairment was, notwithstanding the contract clause of
the Federal Constitution, within the police power of the State as that
power was called into exercise by the public economic emergency which
the legislature has found to exist.
Moreover, the SC held that not only was the constitutional provision qualified
by the measure of control which the State retains over remedial
processes, but theState also continues to possess authority to safeguard
the vital interest
of its people. The policy of protecting contracts against impairment First Trust Co. of Lincoln v. Smith – Nebraska Moratorium
presupposes the maintenance of a government by virtue of which Law extending the benefit of the remedy to a period of 6
contractual relations are worth while, a government which retains years, was held unconstitutional
adequate authority to secure the peace and war good order of Milkint v. McNeely – Certain acts of legislature extending period of
society.
The SC noted that RA 342 only extends relief to debtors of prewar
obligations who suffered from the ravages of the last war and who
filed a claim for their losses with the PH War Damage Commission.
It is therein provided that the said obligation shall not be due and
demandable for a period of 8 yrs from and after settlement of the
claim filed by the debtor with said Commission.
The purpose of the law is to afford prewar debtors an opportunity to
rehabilitate themselves by giving them a reasonable time within
which to pay their prewar debts so as to prevent them from
being victimized by their creditors.
But the SC should not lose sight of the fact that these obligations had
been pending since 1945 as a result of the issuance of EO 25 and
32 and at present their enforcementis still inhibited because of RA
342 and would continue to be unencofceable during the 8-year
period granted to prewqar debtors, which essentially means creditors
will have to wait for 12 years before they will be paid.
This period seems unreasonable, if not oppressive. While the
purpose of Congress is plausible, the relief accorded works injustice
to creditors who are practically left at the mercy of the debtors. Their
hope to effect collection becomes extremely remote, more so if the
credits are unsecured.
In the ff cases, the US declared moratorium laws as violative of the
contract clause of the Constitution, because the period granted to
debtorws was a relief unwarranted by the contemplated emergency:
WB Worthen Co. v. Thomas
WB Worthen v. Kavanaugh
Luisville Joint Stock Land Bank v. Radford
Moreover, the ff cases are cited by the SC which practically involve the
same ruling and which reflect the tendency of courts towards
legislation involving modification of mortgage or monetary contracts
which contain provisions that are deemed unreasonable or
oppressive:
Pouquette v. O’Brien – the statute authorizing courts to extend
for a period of not longer than 2 years all actions of
foreclosures of real estate mortgages and another statute
authorizing to extend foreclosure proceedings not later than
March 1941, was held unconstitutional
First Trust Joint Stock Bank of Chicago v. Adolph Arp et al. –
Moratorium Act 1933, 1935, and 19357 that covers the 1933
Moratorium act extension for 6years, was held
unconstitutional
redemption 3 years beyond the 1-year period then allowed by
law, was declared unconstitutional
Haynes v. Treadway – statute which extends right of redemption rom
6 months to 1 year was held unconstitutional
Swinburne v. Mills – a statute that provides, ona decree for
foreclosure of a mortgage executed before the act was passed,
debtor shall be entitled to have the order of sale stayed for 1
year, was held unconstitutional
These cases apply with added force in this jurisdiction considering the
conditions now prevailing in the PH.
We do not need to go far to appreciate such situation. All over the country,
we already experience a wave of reconstruction and rehabilitation,
thanks to US Aid and the innate progressive spirit of our people.
This aid and spirit worked wonders in sho short a time that it can now be
safely stated that in the main the financial condition of our country and
our people, individually and collectively, has returned to normal
notwithstanding occasional reverses caused by local dissidence
and the sporadic disturbance of peace and order in our midst.
In the face of the foregoing observations, the SC held that the ony way to
open us under the present circumstances is to declare that the continued
operation and enforcement of RA 342 at the present time is
unreasonable and oppressive, and should not be prolonged a minute
longer, and shall be declared null and void.
ABELLA v. NLRC (DANNAH) On Novemrber 1981, Quitco and Dionele filed a complaint against
July 20, 1987 | Paras, J. | Non-Impairment Clause Rosalina with the Ministry of Labor and Employment in Bacolod for
overtime pay,
PETITIONER: Rosalina Perez Abella et al
RESPONDENTS: Hon. National Labor Relations Commission, Quitco,
Dionele

SUMMARY: Rosalina leased Hacienda Canao-Ramona for a period of 10


years, renewable at her option for another 10 years. In 1970 she decided
to renew her lease. During the existence of her lease she employed
Quitco and Dionele. However, when her leasehold rights expired, she
dismissed Quitco and Dionele and turned over the Hacienda to the
owners.

Quitco and Dionele filed with the Minitrsy of Labor and Employment a
complaint for overtime pay, illegal dismissal and reinstatement of backwages.
Labor Arbiter Lucas ruled that the dismissal was warranted but granted
separation pay, which the National Labor Relations Commission affirmed.
Hence this petition.

Rosalina avers that BP 130 granting separation pay violates the


constitutional guarantee against impairment of obligations and contracts.
However, the Court ruled citing Anusencion v. National Labor Union that
the legislation is sustained because in the implementation and
interpretation of the provisions of the Labor Code and its implementing
regulations, the working man's welfare should be the primordial and
paramount consideration.

DOCTRINE: Legislation impairing the obligation of contracts can be


sustained when it is enacted for the promotion of the general good of the
people, and when the means adopted must be legitimate, i.e. within the
scope of the reserved power of the state construed in harmony with the
constitutional limitation of that power.

FACTS:
On June 1960, petitioner Rosalina leased a farm land in Negros
Occidental (Hacienda Canao-Ramona) for a period of 10 years
renewable at her option for another 10 years.
On August 1970 she opted to extend the lease for 10 years.
During the existence of the lease she employed private respondents
Quitco and Dionele.
Dionele has been a regular farm worker and was promoted to Cabo in
1963. Quitco started as a regular employee in 1968 and was
promoted to Cabo in November the same year.
Upon expiration of her leasehold rights, Rosalina dismissed Quitco and
Dionele and turned over the hacienda to the owners who continued
to manage and cultivate and operate the farm.
illegal dismissal and reinstatement with backwages.
Labor Arbiter Lucas ruled that the dismissal is warranted by cessation of
business but granted separation pay.
On appeal, the National Labor Relations Commission (NLRC) affirmed.
Rosalina claims that since her lease agreement had expired, she is not
liable for paying separation pay. Neither could she reinstate them as
a complete cessation or closure of a business operation isa just
cause for employment termination under Art 272 of the Labor Code.
On the other hand, the Labor Arbiter’s legal basis in granting separation
pay is BP 130, amending the Labor Code which states that “In case
of retrenchment… the separation pay shall be equivalent to one
month pay or at least ½ month pay for every year of service,
whichever is higher. A fraction of at least 6 months shall be
considered 1 whole year”

ISSUE/s:
WoN Quitco and Dionele are entitled to separation pay – YES

RULING: Decision of Labor Arbiter and Ministry of Labor and Employment


affirmed

RATIO:
There is no question that Art 284 of the Labor code as amended by BP
130 is the applicable law. Article 272 of the same Code invoked by
Rosalina pertains to the just causes of termination.
The Labor Arbiter does not argue the justification of the termination of
employment but applied Article 284 as amended, which provides for
the rights of the employees under the circumstances of termination.
Rosalina contends that the aforequoted provision violates the
constitutional guarantee against impairment of obligations and
contracts.
This is because when she leased the Hacienda, neither she nor the
lessor contemplated the creation of the obligation to pay separation
pay to workers at the end of the lease.
Such contention is untenable. Anusencion v. National Labor Union:
The prohibition to impair the obligation of contracts is not absolute and
unqualified
The prohibition is not to read with literal exactness like a mathematical
formula for it prohibits unreasonable impairment only.
Legislation appropriate to safeguard said interest may modify or
abrogate contracts already in effect. For not only are existing laws
read into contracts in order to fix the obligations as between the
parties but the reservation of essential attributes of sovereign
power is also read into contracts as a postulate of the legal order.
In order to determine whether legislation unconstitutionally impairs
contract of obligations, no unchanging yardstick, applicable at all times and
under all circumstances, by which the validity of each statute may be
measured or determined, has been fashioned, but every case must be
determined upon its own circumstances.
Legislation impairing the obligation of contracts can be sustained when
it is enacted for the promotion of the general good of the people,
and when the means adopted must be legitimate, i.e. within the
scope of the reserved power of the state construed in harmony with
the constitutional limitation of that power.
The purpose of Article 284 as amended is obvious—the protection of the
workers whose employment is terminated because of the closure of
establishment and reduction of personnel.
Without said law, employees like Quitco and Dionele in the case at hand
will lose the benefits to which they are entitled for the thirty three
years of service in the case of Dionele and fourteen years in the
case of Quitco.
Moreover, to come under the constitutional prohibition, the law must
effect a change in the rights of the parties with reference to each
other and not with reference to non- parties.
Article 284 as amended refers to employment benefits to farm hands
who were not parties to petitioner's lease contract with the owner of
Hacienda Danao-Ramona. That contract cannot have the effect of
annulling subsequent legislation designed to protect the interest of
the working class.
In any event, it is well-settled that in the implementation and
interpretation of the provisions of the Labor Code and its
implementing regulations, the working man's welfare should be the
primordial and paramount consideration.
The policy is to extend the applicability of the decree to a greater number
of employees who can avail of the benefits under the law, which is in
consonance with the avowed policy of the State to give maximum aid
and protection to labor.
PRESLEY v. BEL-AIR VILLAGE ASSOCIATION, INC. (ARIELLE) Under the Deed of Restrictions, Bel-Air is classified as a purely
August 21, 1991 | Gutierrez Jr., J. | Impairment of obligation of residential area, particularly Jupiter Road, which is owned by and
contracts registered in the name of BAVA.
It was also established that the Almendrases had not paid the BAVA
PETITIONER: Enedina Presley
RESPONDENTS: Bel-Air Village Association, Inc., and the Hon. Court of
Appeals

SUMMARY: Almendras owns a property in Bel-Air, which was leased to


Presley. Presley put up a pandesal store in the leased property, which
was against the Deed of Restrictions, which provides that the house and
lot shall be used only for residential and not for commercial purposes.
When BAVA knew about this, it sent a letter to Presley, asking her to
cease and desist from operating the store. Under the Deed of
Restrictions, Bel-Air was purely classified as a residential area, including
Jupiter Street.

The trial court rendered judgment in favor of BAVA, which was affirmed by the
CA. The SC held that the issues presented in the case at hand have been
dealt with in the Sangalang case. Moreover, Jupiter Street has been re-
classified into a high density commercial zone by MMC. Jupiter Street has
been highly commercialized since the passage of Ordinance No. 81-01. The
records indicate that commercial buildings, offices, restaurants, and stores
have already sprouted in the area.

DOCTRINE: Contractual stipulations on the use of the land can be


impaired if necessary to reconcile with the legitimate exercise of police
power.

FACTS:
A complaint for specific performance and damages was filed by plaintiff
Bel-Air Village Association, Inc. (BAVA) against Teofilo Almendras
and Rosario Almendras (now both deceased, substituted by
defendant Enedina Presley) for violation of the Deed of Restrictions
of BAVA.
According to the Deed of Restrictions, the subject house and lot shall be
used only for residential and not for commercial purposes.
The Almendrases were registered owners of a house in Bel Air, and as
such, were members of the BAVA pursuant to the Deed of
Restrictions annotated in their TCT. Presley, as lessee of the
property, is the owner and operator of a “Hot Pan de Sal Store” in the
same address.
When the Almendrases bought their property, the Deed of Restrictions
was already annotated in their title, providing that the lot must be
used only for residential purpose. When BAVA came to know of the
pandesal store, it sent a letter to Presley, asking them to desist from
operating the store.
membership dues. They contended that there was no written and the numerous pleadings filed by different contending parties,
contract between them and BAVA. Only a consensual contract the Court was misled and erred in concluding that Jupiter Street
existed whereby Almendras regularly pays his dues to BAVA for was reclassified as a high density commercial zone when in fact,
services such as security, garbage collection and maintenance and it is still considered as a residential zone.
repair of Jupiter Street. When the services were withdrawn by BAVA,
If BAVA’s observations were accurate, the Court will not hesitate to
there was no more reason for BAVA to demand payment of such
dues. correct the situation. However, after examining the pleadings, the
The trial court rendered judgment in favor of BAVA, which was affirmed by Court
the CA. The CA denied the Motion for Reconsideration. Hence, the
petition.
ISSUE/s:
WoN the ruling of CA adjudging Presley solidarily liable to pay the
alleged unpaid association dues is incorrect – MOOT AND
ACADEMIC (Presley purchased the property and settled all
association dues)
WoN the ruling of the CA awarding attorney’s fees is without any factual
or legal basis – YES
WoN the ruling of the CA is not in accordance with the recent
consolidated decision of the SC in Re Sangalang, Bel-Air Village v.
IAC – YES

RULING: The petition is hereby GRANTED. The decision of the respondent


court is REVERSED and SET ASIDE. The complaint of private respondent is
DISMISSED.

RATIO:
The issues raised in the petition have already been dealt with in the
consolidated cases decided in Sangalang. When CA promulgated
the questioned decision, the Sangalang case had not yet been
decided. It was however, aware of the pending case as it made
mention of the several cases brought to court by BAVA against the
commercial establishments.
Presley is similarly situated as the private respondents in Sangalang
who converted their residential homes to commercial
establishments.
In Sangalang, the Court held that the Ayala Corporation is not liable
for the opening of Jupiter Street to the general public, not only
because Jupiter is not covered by the restrictive easements
based on the ‘deed of restrictions’ but chiefly bcause the
National Government itself, through the Metro Manila
Commission (MMC), had reclassified Jupiter Street into a high
density commercial zone.
In the instant petition, BAVA assails the Court’s decision in the
Sangalang case and that due to the multitude of issues raised
found no reason to reconsider the Sangalang doctrine. BAVA failed to
present any proofs or convincing arguments to substantiate its claim that
Jupiter is still classified as a residential zone.
No new zoning reclassification, ordinance, certification to the effect
or jurisprudence for that matter was brought to the attention of
the Court which would necessarily compel the Court to take a
second look at the Sangalang case.
The Court cannot reverse a precedent and rule favorably on the
strength of mere inferences. The CA was not at all entirely wrong
in upholding the Deed of Restrictions annotated in the title. The
CA held that the provisions of the deed are in the nature of
contractual obligations freely entered into by the parties.
Undoubtedly, they are valid and can be enforced against
Presley.
However, these contractual stipulations on the use of the land
can be impaired if necessary to reconcile with the legitimate
exercise of police power.
The Court reiterates its pronouncements in the Sangalang case
which held that as far as the Bel-Air subdivision is concerned,
certainly, the restrictive easements are valid and enforceable.
But they are subject to the overriding demands and interests of
the greater number as the State may determine in the legitimate
exercise of police power.
Jupiter Street has been highly commercialized since the passage of
Ordinance No. 81-01. The records indicate that commercial
buildings, offices, restaurants, and stores have already sprouted
in the area.
Therefore, there is no reason why Presley should be singled out and
prohibited from putting up her hot pan de sal store. Thus, the
CA’s decision has to be reversed.
MINERS ASSOCIATION v. FACTORAN (IYA) scale mining leases and those pertaining to sand and gravel and quarry
January 16, 1995 | Romero, J. | Non-Impairment of Contracts resources

PETITIONER: Miners Association of the Philippines, Inc.


RESPONDENTS: Hon. Fulgencio, S. Facturan, Jr., Secretary of DENR
and Joel D. Muyco, Director of Mines and Geoscineces Bureau

SUMMARY: DENR issued AOs 57 and 82, provisions of which provide


for the termination of mining claims granted after the effectivity of the
1987 Constitution should they fail to comply with the terms set therein.
Miners Association assails the validity and constitutionality of the said
AOs for contravening the non-impairment clause.

The SC ruled that the AOs pursuant to EO 211 and 279 are valid
exercise of police power and cannot be limited by the non-impairment
clause. The exploration, development, and utilization of natural
resources are considered of paramount public interest which the State
may regulate.

DOCTRINE: The State, in the exercise of its police power in this regard,
may not be precluded by the constitutional restriction on non-impairment
of contract from altering, modifying and amending the mining leases or
agreements granted

FACTS:
The controversy of the present case relates to the change introduced by
Article XII, Section 2 of the 1987 constitution not he system of
exploration, development and utilization of the country's natural
resources.
The adoption of the concept of jura regalia that all natural resources are
owned by the State embodied in the 1935, 1973 and 1987
Constitutions, ushered in the adoption of the constitutional policy of
“full control and supervision by the State” in the exploration,
development and utilization of the country’s national resources.
Pursuant to Article XII, Section 2 of the Constiution, President Cory
Aquino, exercising her legislative powers, issued EO 211 prescribing
the interim procedures in the processing and approval of application
for exploration, development, and utilization of minerals.
EO 279 was likewise promulgated authorizing the DENR secretary to
negotiate and conclude joint benture, agreements for the exploration
of mineral resources and prescribing guiltiness for such agreements.
To implement said legislative acts, the Secretary of the Department of
Environment and Natural Resources (DENR) in turn promulgated
Administrative Order Nos. 57 and 82, the validity and constitutionality
of which are being challenged in this petition.
Article 9 of AO 57 said that all existing mining leases or agreements which
were granted after the effectivity of the 1987 Constitution, except small
covering an area of twenty (20) hectares or less, shall be converted
into production-sharing agreements within one (1) year from the
effectivity of these guidelines.
AO 82 played down the “Procedural Guidelines on the Award of Mineral
Production Sharing Agreement through Negotiation.”
Section 3 enumerates the persons required to submit Letters of Intent
and mineral Production Sharing Agreements within 2 years form the
effectivity of AO 57. Failure do to so shall cause the abandonment of
mining claims.
Hence, the intent petition by Miners Association of the Philippines, Inc.
assailing the validity and constitutionality of the two AOs.
Miners Association contends that both orders violate the non-impairment
of contract provision under Article III, Section 10 of the 1987
Constitution on the ground that the AOs unduly pre-terminate
existing mining leases and other mining agreements.

ISSUE/s:
WON AO 57 and 82 violate the non-impairment of contracts clause - NO

RULING: The validity and constitutionality of AOs 57 and 82 are sustained


and their force and effect upheld.

RATIO:
Article XII, Section 2 of the 1987 Constitution does not apply retroactively
to “license, concession or lease” granted by the government under
the 1973 Constitution or before the effectivity of the 1987
Constitution on February 2, 1987.Administrative Order No. 57
applies only to all existingmining leases or agreements which were
granted after theeffectivity of the 1987 Constitution pursuant to
Executive Order No. 211
It bears mention that under the text of Executive Order No. 211, there is
a reservation clause which provides that the privileges as well as the
terms and conditions of all existing mining leases or agreements
granted after the effectivity of the 1987 Constitution, pursuant to
Executive Order No. 211, shall be subject to any and all
modifications or alterations which Congress may adopt.
Hence, the strictures of the non-impairment of contract clause do not
apply to the aforesaid mining leases and agreements after the
effectivity of the 1987 Constitution.
They can be amended, modified, or altered by a statue passed by
Congress to achieve the purpose of Article XII, Section 2 of the
Constitution. arbitrary or oppressive.
The economic policy on the exploration, development and utilization of
the country’s natural resources under Article XII, Section 2 of the
1987 Constitution could not be any clearer.
The exploration, development and utilization of the country’s natural
resources are matters vital to the public interest and the general
welfare of the people.
Accordingly, the State, in the exercise of its police power in this regard,
may not be precluded by the constitutional restriction on non-
impairment of contract from altering, modifying and amending the
mining leases or agreements granted under Presidential Decree No.
463, as amended, pursuant to Executive Order No. 211.
Police power, being co-extensive with the necessities of the case and the
demands of public interest, extends to all the vital public needs.
The Court, therefore, rules that the questioned administrative orders are
reasonably directed to the accomplishment of the purposes of the
law under which they were issued and were intended to secure the
paramount interest of the public, their economic growth and welfare.
ORTIGAS v. FEATI BANK (ELIEL) sanitary installations connected either to the public sewer or to
December 14, 1979 | Santos, J. | Non-impairment Clause an approved septic tank, and (c) shall not be a distance of less
than two (2) meters
PETITIONER: Ortigas & Co., Limited Partnership
RESPONDENTS: Feati Bank and Trust Co.

SUMMARY: Lots 5 and 6 are subject to the restriction that the parcels of
land contained therein shall be used exclusively for residential purposes.
These lots were acquired by Feati Bank and Trust Co., and upon said
lots they acquired building and planning permits for the construction of a
bank, which is for commercial purposes. When Feati Bank were laying
foundation for the construction, Ortigas complained bringing up that the
lots were only supposed to be for residential purposes and not
industrial/commercial purposes. Hence this petition.

The SC held that the Municipality of Mandaluyong, under the Local


Autonomy Act, valid exercised police power in Resolution 27 converting
the area into a commercial/industrial zone. Although there is a non-
impairment clause guaranteed in the constitution, it is not absolute. The
contractual obligations, right to property, may be interfered by the State in
order to promote general welfare. Such is in thise case, proving that the
area is unprofitable, it may be a hazard to the health and comfort of the
people.

DOCTRINE: The motives behind the passage of the questioned


resolution being reasonable, and it being a “legitimate response to a felt
public need,” not whimsical or oppressive, the non-impairment of
contracts clause of the Constitution will not bar the municipality’s proper
exercise of the power

FACTS:
Augusto Padilla and Natividad Angeles entered into separate
agreemetns of sale on installments over two parcels of land, known
as Lots. No. 5 and 6 of the Highway Hills Subdivision, situated at
Mandaluyong, Rizal.
Augusto and Natividad transferred their rights and interest over the aforesaid
lots in favor of one Emma Chavez. Upon completion of payment of the
purchase price, Augusto and Natividad excuted the corresponding deds
of sale in favor of Emma Chavez. Both the agreements (of sale on
installment) and the deeds of sale contained the stipulations or
restrictions that:
The parcel of land subject of this deed of sale shall be used by
the Buyer exclusively for residential purposes, and she
shall not be entitled to take or remove soil, stones or gravel
from it or any other lots belonging to the Seller.
All buildings and other improvements (except the fence) which may
be constructed at any time in said lot must be, (a) of strong
materials and properly painted, (b) provided with modern
from its boundary lines.
Eventually, Feati Bank acquired Lots 5 and 6. Feati bought Lot 5 directly
from Emma Chavez, “free from all liens and encumbrances,” while
Lot 6 was acquired from Republic Flour Mills through a “Deed of
Exchange.”
Ortigas claims that the restrictions annotated with the lots were imposed
as part of its general building scheme designed for the beautification
and development of the Highway Hills Subdivision, which forms part
of the big landed estate of Ortigas where commercial and industrial
sites are also designated or established.
Feati Bank, upon the other hand, maintains that the area along the
western part of EDSA from Shaw Blvd. to Pasig river, has been
declared a commercial and industrial zone, per Resolution No. 27.
It alleges that Ortigas ‘completely sold and transferred to third
persons all lots in said subdivision facing EDSA” and the
subject lots thereunder were acquired by it “only on July 23,
1962 or more than two years after the area had been
declared a commercial and industrial zone.
Feati Bank began laying the foundation and commenced the construction of
a building on Lots 5 and 6, to be devoted to banking purposes, but which
Feati claims could also be devoted to, and used exclusively for,
residential purposes
The following day, Ortigas demanded in writing that Feati stop the
construction of the commercial building on the said lots. The latter
refused to comply with the demand, contending that the building was
being constructed in accordance with the zoning regulations, having
filed building and planning permit applications with the Municipality of
Mandaluyong, and it had accordingly obtained building and planning
permits to proceed with the construction.
Hence this petition

ISSUE/s:
WoN Resolution 27 is a valid exercise of police power –YES

RULING: Affirmed from lower courts that Resolution 27 is a valid exercise of


police power.

RATIO:
Sec 3 of RA 2264, otherwise known as the Local Autonomy Act,
“empowers a Municipal Council to adopt zoning and subdivision
ordinances or regulations for the municipality.”
Clearly, the law does not restrict the exercise of the power through an
ordinance. Therefore, granting that Resolution 27 is not an
ordinance, it certainly is a regulatory measure within the intendment
or ambit of the word “regulation” under the provision. As a matter of
fact the same section declares tha tht epower exists “Any provision
of law to the contrary notwithstanding.”
The same section further mandates that the general welfare clause be
liberally
interpreted in case of doubt, so as to give more power to local need,” not whimsical or oppressive, the non-impairment of
governments in promoting the economic conditions, social welfare contracts clause of the Constitution will not bar the
and material progress of the people in the community. municipality’s proper exercise of the power.
The only exceptions under Section 12 are existing vested rights arising
out of a contract between “a province, city or municipality on one
hadn and a third party on the other,” in which case the original terms
and provisions of the contract should govern.
With regard to the contention that said resolution cannot nullify the
contractual obligations assumed by Feati Bank – referring to the
restrictions incorporated in the deeds of sale and later in the
corresponding Transfer Certificate Titles – it should be stressed,
that while non-impairment of contracts is constitutionally
guaranteed, the rule is not absolute, since it has to be reconciled
with the legitimate exercise of police power.
PLDT v. Davao, Police power “is elastic and must be responsive to
various social conditions; it is not, confined within arrow
circumscriptions of precedents resting on past conditions; it
must follow the legal progress of a democratic way of life.”
Vda. De Genuino v. Court of Agrarian Reforms, we do not see why
public welfare when clashing with the individual right to property
should not be made to prevail through the state’s exercise of its
police power
Resolution No. 27 declaring the western part of EDSA from Shaw Blvd. to
Pasig River as an industrial and commercial zone, was obviously passed
by the Municipal Council of Mandaluyong, Rizal in the exercise of police
power to safeguard or promote the health, safety, peace, good order and
general welfare of the people in the locality, Judicial notice may be taken
of the conditions prevailing in the area, especially where lots 5 and 6 are
located.
The lots themselves not only front the highway; industrial and
commercial complexes have flourished about the place. EDSA, a
main traffic artery which runs through several cities and
municipalities in the Metro Manila area, supports an endless stream
of traffic and the resulting activity, noise and pollution are hardly
conducive to the health, safety or welfare of the residents in its route.
Having been expressly granted the power to adopt zoning and
subdivision ordinances or regulations, the municipality of
Mandaluyong, through its Municipal council, was reasonably, if not
perfectly, justified under the circumstances in passing the subject
resolution
Thus, the state, in order to promote the general welfare, may
interfere with personal liberty, with property, and with business
and occupations. Persons may be subjected to all kinds of
restrains and burdens, in order to secure the general comfort
health and prosperity of the stae and to this fundamental aim of
our Government, the rights of the individual are subordinated.
The motives behind the passage of the questioned resolution being
reasonable, and it being a “legitimate response to a felt public
Applying the principle just stated to the present controversy, we can say
that sincie it is now unprofitable, it may be a hazard to the health and
comfort, to use Lots 5 and 6 for strictly residential purposes, Feati Bank
should be permitted, on the strength of the resolution promulgated under the
police power of the municipality to se the same for commercial purposes.
Barredo, J. Concurring: I hold it as a matter of public knowledge that the
place in question is commercial. It would be worse if the same were to be
left as residential and all around are already commercial.
Fernando, J. Concurring: The only point I would wish to add is that in the
process of such balancing and adjustment, the present Constitution, the
Philippine American Life Insurance Co. decision having been
promulgated under the 1935 Charter, leaves no doubt that the claim to
property rights based on the non-impairment clause has a lesser weight.
For as explicitly provided by our present fundamental law: "The State
shall promote social Justice to ensure the dignity, welfare, and security of
all the people. Towards this end, the State shall regulate the acquisition,
ownership, use, enjoyment, and disposition of private property, and
equitably diffuse property ownership and profits
Abad Santos, J. Dissenting: But Resolution No. 27, cannot be described as
promotive of the health, morals, peace, education, good order or safety
and general welfare of the people of Mandaluyong. On the contrary, its
effect is the opposite. For the serenity, peace and quite of a residential
section would by the resolution be replaced by the chaos, turmoil and
frenzy of commerce and industry. Where there would be no industrial and
noise pollution these bane of so-called progress would now pervade and
suffocate the environment to the detriment of the ecology. To
characterize the ordinance as an exercise of police power would be
retrogressive. It will set back all the efforts of the Ministry of Human
Settlements to improve the quality of life especially in Metro Manila. It will
make Metro Manila, not the city of man as envisioned by its Governor but
a city of commerce and industry.
JUAREZ v. CA (JP) BP 877 Sec. 5: Grounds for Judicial Ejectment.·Ejectment shall be
Ocobter 7, 1992 | Cruz, J. | Obligations of Contracts allowed
on the following grounds: (a) Assignment of lease or subleasing of
PETITIONER: Virginia Ocampo Juarez residential units in whole or in part, including the acceptance of
boarders or bedspacers, without the written consent of the
RESPONDENTS: Court of Appeals, Cetus Development
owner/lessor.
SUMMARY: A lot located in Quezon Boulevard was leased to Sevillano
Ocampo who later on passed such with a house to his sister Angela.
Petitioner Virginia took over in view of her mother Angela’s advanced
age. The landlord of such lot, respondent Cetus filed an ejectment suit
against Virginia for violating BP 877 which prohibits subleasing of lots
without the consent of the original landlord. The RTC dismissed but CA
reversed.

The SC affirmed the CA. The SC held that because the contract did not
specify a specific term, such is presumed to renew monthly, because
such is being paid monthly. Hence, when the contract renewed the next
month, it now falls within the ambit of BP 877. Assuming arguendo that
contract does not renew monthly, such is still prohibited because BP 877
actually covers current contracts. With regard to the impairment clause,
the SC held that public interest should override such. The public interest
herein is that landlords shall be protected from lessees who abuse their
statutory rights. Virginia herein is charging ten times her own rental to the
sublessee. This prejudices Virginia’s original landlord, Cetus.

DOCTRINE: As long as the contract affects the public welfare one way or
another so as to require the interference of the State, then must the
police power be asserted, and prevail, over the impairment clause.

In allowing the ejectment of the lessee who has subleased the leased
premises without the written consent of the lessor, the law has taken note of
one of the unconscionable practices of lessees that have caused much
prejudice to the lessor.

FACTS:
A lot located in Quezon Boulevard was leased to Sevillano Ocampo who
lived with his parents and his sister Angela. Angela took over the
lease. Angela lived with her children, one of whom is petitioner
Virginia.
Because of her advanced age, she moved to her daughter Virginia’s
house in Pasay. Angela then leased the house to Roberto
Capuchino.
Meanwhile, the lot (where the house stood) was sold to Susanna Realty,
Inc. whin in turn sold it to respondent Cetus Development
Corporation.
Cetus then filed for a complaint for ejectment against petitioner Virginia
on the ground that the lessee (Virginia) had subleased the property
without the consent of the landlord (Cetus) in violation of BP 877.
The MTC of Manila dismissed such because the sublease was made
priot to the effectivity of such law. RTC affirmed. The latter also
added that the real property in interest was not Virginia but Angela
(her mother).
CA reversed. I tuled that BP 877 was applicable because the contract of
lease did not specify a fixed term and payment of rental was made
on a monthly bases. The contract then was deemed month to month.
Hence, when the contract renewed the following month, it became
subject to BP 877. Hence this appeal.

ISSUE/s:
WoN the landlord may eject the petitioner who is subleasing and
profiteering from the lot without the consent of such landlord – YES

RULING: Petition is denied.

RATIO:
The SC rules with Cetus.
Acticle 1687 of the Civil Code provides: If the period for the lease has not
been fixed, it is understood to be from year to year, if the rent agreed
upon is annual; from month to month, if it is monthly.
Like the principal contract, the sublease was also terminable at the end
of each month because no specific period has been prescribed and
the rentals were also payable monthly.
When the sublease was renewed in July 1985, it had become invalid
under the provisions of BP 877, which had already become effective.
The law was operating prospectively upon the new, or renewed,
contract of sublease which now could no longer be considered valid
without the written consent of the lessor.
But even if the sublease were not terminable when the statute became
effective on June 12, 1985, and had a fixed period going beyond that
date, that period could still be cut short by operation of BP 877. This
time the law would operate retroactively, to affect existing contracts
of sub- lease concluded before the law came into effect.
Thus, BP 877 provides as follows: All residential units the total monthly
rental of which does not exceed four hundred eighty pesos (P480.00)
as of the effectivity of this Act shall be covered by this Act.
The petitioner Virginia complains that the retroactive application of the
law would violate the impairment clause. The argument does not
impress. The impairment clause is now no longer inviolate.
More and more, the interests of the public have become involved in what
are supposed to be still private agreements, which have as a result
been removed from the protection of the impairment clause. These
agreements have come within the embrace of the police power, that
obtrusive protector of the public interest.
As long as the contract affects the public welfare one way or another so
as to require the interference of the State, then must the police
power be asserted, and prevail, over the impairment clause.
Housing is one of the most serious social problems of the country. The
regulation of rentals, especially in the urban areas, has long been the
continuing concern of the government, as manifested by the
succession of laws on the subject, beginning with Rep. Act No. 6359,
the original House Rentals Law.
In allowing the ejectment of the lessee who has subleased the
leased premises without the written consent of the lessor, the
law has taken note of one of the unconscionable practices of
lessees that have caused much prejudice to the lessor.
The subject lease is a case in point. It is not denied that the rental on the lot is
only P69.70 per month. On the other hand, the petitioner has declared that
she charges Capuchino a monthly rental of P400.00 under the sublease
contract, or almost ten times her own rental.
While it is true that this covers both the lot and the building, the
point is that she is not paying the lessor enough for the use of
the lot in light of the total rental she is charging Capuchino for
the use of the house and lot.
The purpose of the law in interfering with such contracts is to protect both
the landlord and the tenant from their mutual impositions that can
only cause detriment to society as a whole. The practices above
discussed have to be the subject of government regulation and even
prohibition and cannot seek legitimate refuge in the impairment
clause.
The petitioner’s (Virginia) contention that BP 877 is an ex post facto law
must also be rejected. It is not penal in nature and the mere fact that
it contains penal provisions does not make it so. At any rate, she is
not being prosecuted under the said penal provisions. She was sued
in the municipal court in a civil complaint to eject her from the lot on
the ground that she had unlawfully subleased it.
The petitionerE’s final contention is that she is not the proper party
defendant in the ejectment case but her mother Angela, who
inherited the leasehold right from Servillano Ocampo. That claim is
also unacceptable. It is obvious that the petitioner has been acting
on behalf of her mother, who is now 92 years of age and living in her
care.
HACIENDA LUISITA v. PARC (HENRY) the contract and its implementation cannot successdully be resisted by
July 5, 2011 | Velasco, Jr., J. | Impairment of contractual rights and force of the non-impairment guarantee.
obligations

PETITIONER: Hacienda Luicita, Inc. Luisita Industrial Park, and RCBC


RESPONDENTS: Presidential Agrarian Reform Council (PARC),
Secretary Nasser Pangandaman of the DAR, Alyansa ng mga
Manggagawang Bukid ng Hacienda Luisita, Rene Galang, Noel Mallari,
and Julio Suniga and his Supervisory Group of the Hacienda Luisita, Inc.,
and Windsor Andaya

SUMMARY: Hacienda Luisita is sold by its Spanish owners to Tadeco.


Apparently, during the martial law administration, the trial court ordered
them to surrender the said property to the government. Upon appeal to
the CA, it was dismissed, and that Tadeco must obtain from PARC an
approval of a stock distribution plan, in order for beneficiaries (who are
farmers) to be able to get their rightful share, in the light of EO 229 Sec
10 (see fact#11). RA 6657, specifically provides the Stock Distribution
Plan. In line with this, DAO 10 provides the guidelines and procedures for
corporate landowners desiring to avail of the stock distribution plan.

Tadeco was amenable to the SDP, and even established HLI to facilitate the
said distribution. Eventually, the SDOA entered into by the beneficiaries with
the HLI was approved by the PARC, and that stock distribution was going
along fine, until respondent farmers filed a complaint claiming that their lives
have not improved and that HLI committed violations of the approved SDOA.
They were seeking for the revocation of the SDOA. DAR made a special task
force to look into the manner and eventually discovered that indeed, HLI has
not been compliant to the SDOA. Hence, they recommended for the
revocation of the SDOA.

HLI claims as defense that the revocation of the SDOA through DAR’s
issuance of the resolution subjecting the property to compulsory land
distribution is a violation of Art 3 Sec 10 of the Constitution, as it impairs
the obligatioin of the contract (SDOA) between the parties.

SC denied the petition of HLI and ruled that such resolution is valid as it
is compliant with a law that already existed vis-à-vis the creation of the
contract (SDOA) between HLI and the beneficiaries. Moreso, such law is
not only existing, but also appropriate, and hence its legal effect shall be
recognized. To emphasize, the laws being talked about in the
constitutional provision are those that are about to be passed. Not
those that are already existing.

DOCTRINE: A law authorizing interference, when appropriate, in the


contractual relations between or among parties is deemed to read into
FACTS: RA 6657, similar to EO 229, specifically in Sec 31, provides the
Hacienda Luisita de Tarlac (Hacienda Luisita) is once a 6,443-hectare Stock Distribution Plan.
mixed agricultural-industrial-residential property passing through Vis-à-vis the stock distribution aspect of Sec 31, DAR issued Admin
several municipalities of Tarlac and owned by Compania General Order No. 10 (DAO 10) for the guidelines and procedures for
de Tobacos de Filipinas (Tabacalera). Corporate
1957, Spanish owners offered to sell Hacienda Luisita as well as their
controlling interest in the sugar mill within the hacienda, the Central
Azucarera de Tarlac (CAT), as an indivisible transaction.
The Tarlac Development Corp (Tadeco) then owned and/or controlled
by Jose Cojuangco, Sr. Group, was willing to buy. As agreed,
Tadeco undertook to pay the purchase price for Hacienda Luisita in
pesos, while that for the controlling interest in CAT in US Dollars.
To facilitate the adverted sale-and-purchase package, the Govt,
through the then Central Bank, assisted the Tadeco to obtain a
dollar loan from a US bank. Also, GSIS Board of Trustees
extended on Nov 1957 a loan of Php 5.911M in favor of Tadeco to
pay the peso price component of the sale.
One of the conditions of the contained in the approving GSIS Reso No.
3203, as amended by Reso. 356, which stated that, “the lots
comprising the Hacienda Luisita shall be subdivided by the
applicant-corporation and sold at cost to the tenants, should there
be any, and whenever conditions should exist, warranting such
action under the provisions of the Land Tenure Act.”
By March 1958, Tadeco fully paid the purchase price of both the
property and the interest in CAT.
May 1980, the martial law administration filed a suit before the RTC
against Tadeco, et al., for them to surrender Hacienda Luisita to
the then Ministry of Agrarian Reform (MAR, now the Department of
Agriarian Reform [DAR]) so that the land can be distributed to
farmers at cost.
Tadeco alleged that Hacienda Luisita does not have tenants, besides
which sugar lands are not covered by the existing agrarian reform
legislations. As perceived then, the government commenced the
case against Tadeco as a political message to the family of the late
Benigno Aquino, Jr.
RTC ordered Tadeco to surrender Hacienda Luisita to the MAR.
March 1988, OSG moved to withdraw the government’s case against
Tadeco. By Resolution, CA dismissed the case and the Marcos
government initially instituted and won against Tadeco. The
dismissal, however, was made subject to the obtention by Tadeco
of PARC’s approval of a stock distribution plan (SDP) that must
initially be implemented after such approval shall have secured.
Sec 10 of EO 229 allows corporate landowners, as an alternative to the
actual land transfer scheme of Comprehensive Agrarian Reform
Program (CARP), to give qualified beneficiaries the right to purchase
shares of stocks of the corporation under a stock ownership
arrangement and/or land-to-share ratio.
Landowners desiring to avail themselves of the Stock During the SDP approval, HLI had a pool of farmworkers (6,296) permanent,
Distributioin Plan under Sec 31 of RA 6657. seasonal, and casual mastterlist/payroll and non-masterlist members.
To begin with, the SDP seemed to be Tadeco’s preferred option, it even 1989-2005, HLI claimed to have extended the ff benefits to FWBs:
organized a spin-off corporation, HLI, as vehicle to facilitate the Php 3B worth salaries, wages, fringe benefits
farmworkers’ stock acquisition. 59M shares of stock distributed for free
Tadeco assigned and conveyed to HLI the agricultural land portion (4,915 Php 150M representing 3% of the gross produce
hectares) and other farm-related properties of Hacienda Luisita in Php 37.5M, representing 3% from the sale of 80 hectares at 80M for
exchange for HLI shares of stock. SCTEX
Pedro Cojuangco, Josephine Reyes, Teresita Lopa, Jose Cojuangco, and 240-sqm homelots distributed for free
Paz Teopaco, were the incorporators of HLI. 2.4M representing 3% from sale of 80 hectares at PHp 80M for
To accommodate the asset transfer from Tadeco to HLI, the latter, with SCTEX
SEC’s approval, increased its capital stock from Php 1.5M divided Social Service benefits, such as but not limited to free
into 1.5M shares with a par value of 1php/share to Php 400M divided hospitalization/medical/maternity services, old age/death
into 400M shares with also the same par value. 150M of which were benefits and no interest bearing salary/educational loans and
to be issued only to qualified and registered benificiaries of the rice sugar
CARP, and the remaining 250M to any stockholder of the Aug 1995, HLI applied for the conversion of 500 hectares of land from
corporation. agricultural to industrial use. The application accdg to HLI had the
As appearing in the proposed SDP, Tadeco contributed a total of Php backing of 5,000 or so FWBs, including respondent Galang et al.
355,531,462 in the capital stock of HLI. This was approved eventually by the DAR through Ernesto Garilao,
May 1989, some 93% of the then farmerworker-beneficiaries (FWBs) subject to payment of 3% of the gross selling price to the FWBs and
complement of Hacienda Luisita signified in a referendum their to HLI’s continued compliance with its undertakings under the SDP,
acceptance of the HLI’s proposed Stock Development Distribution among other conditions.
Option Plan (SDOP). The same month, the Stock Distribution Option December 1996, HLI ceded 300 hectares of the converted area to
Agreement (SDOA) as a MOA, was entered into by Tadeco, HLI, and Centennary Holdings, Inc (Centennary) in exchange for subscription
5,848 qualified FWBs an is attested by DAR Secretary Juico. of 12M shares of stocks.
As seen from the SDOA, included as part of the SDP are the ff: HLI transferred the remaining 200 hectares to Luisita Realty Corporation
Production-sharing equivalent to 3% of gross sales form the (LRC) in two separate transactions in 1997 and 1998, both uniformly
production of the agricultural and payable to the FWBs in involving 100 hectares for Php 250M each.
cash dividends or incentive bonus Centennary sold the 300 hectares to Luisita Industrial Park Corporation
Distribution of free homelots of not more than 240 sqm each to (LIPCO) for Php 750M. LIPCO’s goal was to develop an industrial
family beneficiaries. complex.
The production sharing, as the SDP indicated, is payable irrespective of Later on, in a Deed of Absolute Assignment, LIPCO transferred the
whether HLI makes money or not, implying that the benefits do not parcels to RCBC by way of dacion en pago, of Lipco’s Php 431.7
partake the nature of dividends, as the termis ordinarily understood loan obligations.
under corporation law. Apart from the 500 hectares alluded to, another 80.51 hectares were
The assigned value of the agricultural land in the hacienda was Php later detached from the area coverage of Hacienda Luisita which had
196.63M, while the total assets of HLI was Php 590.55M with net been acquired by the government as part of the SCTEX.
assets of Php 355.53m, Tadeco/HLI would admit that the ratio of the 4.3 of the original 4.9 hectares remained which Tadeco ceded to HLI.
land-to-shares of stock corresponds to 33.3% of the outstanding In short, there was a state of things when 2 petitions reached the DAR in
capital stock of the HLI equivalent to 118.3M shares with par value of 2003. First was a protest by respondents and 60 other supervisors to
Php 1/share. revoke the SDOA, claiming that the HLI failed to give their dividends
HLI submitted its SDP to DAR. Notably, in the follow up referendum the at 1% share in gross sales, as well as the 33% share in the proceeds
DAR conducted, 5,117 FWBs out of the 5,315 who participated opted of the sale of te converted 500 hectares of land.
to receive shares in HLI. 132 chose actual land distribution. They further claimed that their lives have not improved contrary to the
DAR proposed revisions but Tadeco/HLI responded that such proposals promise and rationale for the adoption of the SDOA. Moreso, that HLI
are already embodied in the SDP and the MOA. Following such violated SDOA’s terms, hence they pray for a negotiation of the
exchange, respondent PARC, through Sec Miriam Defensor- SDOA, or the alternative, its revocation, which is what was claimed in
Santiago, by Reso. 89-12-2, approved of the SDP of Tadeco/HLI. the second petition.
Meanwhile, DAR made a Special Task Force to review the terms and
conditions of the SDOA and the PARC Resolution relative to HLI’s Serrano v. Gallant: the prohibition against impairment of the obligation of
SDP, among others. The Task Fofce found that HLI has not complied contracts is aligned with the general principles that laws newly enacted
with RA 6657 despite the implementation of the SDP. The report was have only a prospective operation, and cannot affect acts or contracts
adopted by then Sec. Nasser Pangandaman. already perfected. Thus, the non-impairment clause under Sec 10 Art 3
Sec. Pangandaman recommended to the PARC Exec Committee that: of the Constitution is limited in application to laws about to be enacted
Recall/revocation of PARC Reso No. 89-12-2 approving HLI’s SDP that would in any way derogate from existing acts or contracts by
Acquisition of Hacienda Luisita through compulsory acquisition enlarging, abridging, or in any manner changing the intention of the
scheme parties thereto.
December 2005, PARC issued the assailed Reso No. 2005-32-01 and Resolution 2005-32-01 is not the kind of issuance within the ambit
such was served on HLI, who sought reconsideration, but denied. of Art 3 Sec 10 of the constitution, “No law impairing the
Hence, this appeal. obligations of contracts shall be passed”. Because, to stress, RA
Most important in the claims of HLI is its claim that subjecting its 6657 in relation to DAO 10 is already existing.
landholdings to compulsory distribution after its approved SDP HLI tags the SDOA as an ordinary civil law contract and a breach of its
has been implemented would impair the judicial obligations terms is not a PARC administrative matter, but one that gives rise to
created under the SDOA. (Why ito yung relevant? Sec 10 is about a cause of action cognizable by regular courts.
laws not impairing obligations of contracts. And HLI claims that the This contention has little to stand on its own. SDOA is a special contract
PARC Resolution simply does that, by revoking the effect of their imbued with public interest, entered into and crafted pursuant to the
contract, SDOA) provisions of RA 6657. It embodies the SDP, which requires for its
validity, or at least enforceability, PARC’s approval.
ISSUE/s: The fact that the certificate of compliance is revocable by th same issuing
WoN PARC Resolution 2005-32-01 (in compliance to RA 6657 and DAO authority supports the idea that everything about the implementation
10) impairs the obligation of the contract (SDOA) - NO of the SDP is, at the first instance, subject to administrative
adjudication.
RULING: SC denied the petition. CORONA, dissenting: By its plain language, it requires that the law
implementing the agrarian reform program envisioned by the
RATIO: Constitution should employ a land redistribution mechanism. Subject
The pressing argument of HLI is that subjecting its landholdings to only to retention limits as may be prescribed bt Congress and to
compulsory distribution after its approved SDP has been payment of just compensation, ownership of all agricultural lands are
implemented would impair the contractual obligations created under to be distributed and transferred to the farmers and farmworkers who
the SDOA. till the land. There is absolutely no doubt in his mind that the
The broad sweep of HLI’s argument ignores certain established legal Constitution has ordained land distribution as the mechanism of
percepts and must be rejected. agrarian reform.
A law authorizing interference, when appropriate, in the contractual The Constitution has vested the SC with the power and duty to determine
relations between or among parties is deemed to read into the and declare whether the scales of constitutionality have been kept in
contract and its implementation cannot successdully be resisted by balance or unduly tipped, whether an official action is constitutional or
force of the non-impairment guarantee. not. As the fundamental and supreme law of the land, the Constitution
There is, in that instance, no effect of the impairment clause, the non- also serves as the counterweight against which the validity of all actions
impairment protection being applicable only to laws that derogate of the government is weighed. With it, the SC ascertains whether the
prior acts or contracts by enlarging, abridging, or in any manner action of a department, agency, or public officer preserves a
changing the intention of the parties. constitutional equilibrium or disturbs it.
Impairment, in fine, obtains if a subsequent law changes the terms The river cannot rise higher than its source. An unconstitutional provision
of a contract between the parties, imposes new conditions, cannot be the basis of a constitutiobnal act. As the SDP of petitioner
dispenses with those agreed upon, or withdraws existing HLI is based on Sec 31 of RA 6657 which is unconstitutional, the
remedies for the enforcement of the rights of the parties. stock distribution plan must perforce also be unconstitutional.
In application with the case at hand, it is necessary that the constitutional Brion, concurring and dissenting: Since the land is subject to compulsory
prescription would not apply to laws ALREADY in effect at the time of coverage under the CARL, HLI is entitled to just compensation. For
the contract execution. RA 6657 in relation to DAO 10 is NOT A purposes of just compensation, the taking should be reckoned not from
SUBSEQUENT LAW. the Court or the PARC’s declaration of nullity of the SDP, but from May
11, 1989—when the invalid SDOA/SDP was executed for purposes of
compliance with the CARL’s requirements. To repeat, May 11, 1989 is the point in
time when
HLI compiled with its obligation under the CARL as a corporate landowner,
through the stock distribution mode of compliance. This is the point, too,
when the parties themselves determined—albeit under a contract that is null
and void, but within the period of coverage that the CARL required and
pursuant to the terms of what this law allowed—that compliance with the
CARL should take place. From the eminent domain perspective, this is the
point when the deemed “taking” of the land, for agrarian reform purposes,
should have taken place if the compulsory coverage and direct distribution of
lands had been the compliance route taken. As the chosen mode of
compliance was declared a nullity, the alternative compulsory coverage (that
the SDOA was intended to replace) and the accompanying “taking” should
thus be reckoned from May 11, 1989.
Mendoza, separate: Land reform entails land distribution to those who
till the land. If there is no actual land distribution, there is no land
reform. Indeed, the distribution of shares of stock, not land, cannot
be considered as compliance with the constitutional provision on
agrarian reform. Section 31 of RA 6657, which allows stock
distribution, directly and explicitly contravenes Sec 4, Art 13 of the
Constitution. Doubtless, the SDP of petitioner HLI which has its basis
Sec 31 of RA 6657, is unconstitutional.
Sereno, dissenting: Petitioner HLI’s insistence on the non-impairment
clause is misplaced, as it deals with a fundamental right against the
exercise of legislative power, and not of judicial or quasi-judicial
power. In Lim v. Sec of Agriculture, the Court explained the scope of
the non-impairment clause thus: For it is well-settled that a law within
the meaning of this constitutional provision has reference primarily to
statutes and ordinances of municipal corporations. Executive orders
issued by the President whether derived from his constitutional
power or valid statutes may likewise be considered as such. It does
not cover, therefore, the exercise of the quasi-judicial power of a
department head even if affirmed by the President. The
administrative process in such a case partakes more of an
adjudicatory character. It is bereft of any legislative significance. It
falls outside the scope of the non-impairment clause.
In exchange, petitioner HLI as the previous landowners is entitled to the
payment of just compensation of the value of the land at the time of
the taking. Since the award of direct land transfer is being settled by
the Court only now, then the value of the property should be similarly
pegged at this point. The constituted limitation of “just compensation”
is considered to the sum equivalent to the market value of the
property, broadly described to be the price fixed by the seller in open
market in the usual and ordinary course of legal action and
competition; or the fair value of the property as between one who
receives ad one who desires to sell, if fixed at the time of the actual
taking by the government.
GOLDENWAY MERCHANDISING v. EQUITABLE PCI (DANNAH) In March 8, 2001 the counsel of GMC wrote seeking to redeem the
March 13, 2013 | Villarama Jr., J. | Non-Impairment Clause foreclosed properties by tendering a check in the amount of P3.5M. The
same was
PETITIONER: Goldenway Merchandising Corporation
RESPONDENTS: Equitable PCI Bank

SUMMARY: Goldenway Merchandising failed to pay its loan obligation to


Equitable PCI Bank, which lead to the foreclosure of his mortgaged
property. GMC wanted to redeem said property, but PCI refuses to grant
such requisition on the ground that the right to redemption has lapsed
because the property was already registered. GMC assails Section 47 of
RA 8791 as violative of the non-impairment clause.

Lower courts and SC all agree that the questioned provision is a valid
exercise of police power. The freedom to contract is not absolute; all
contracts and all rights are subject to the police power of the State and
not only may regulations which affect them be established by the State,
but all such regulations must be subject to change from time to time, as
the general well-being of the community may require, or as the
circumstances may change, or as experience may demonstrate the
necessity.

The General Banking Law of 2000 was crafted in the aftermoath of the
1997 Southeast Asian financial crisis, and it sought to reform the General
Banking Act by fashioning a legal framework for maintaining a safe and
sound banking system. In this context, the amendment introduced by
Section 47 embodied one of such safe and sound practices aimed at
ensuring the solvency and liquidity of our banks. This legitimate public
interest pursued by the legislature further enfeebles GMC’s impairment of
contract theory. 14. As with other individual rights to contract and to
property, the right of redemption has to give way to police power
exercised for public welfare.

DOCTRINE: Settled is the rule that the non-impairment clause of the


Constitution must yield to the loftier purposes targeted by the
Government. The right granted by this provision must submit to the
demands and necessities of the State’s power of regulation.

FACTS:
Goldenway Merchandising Corporation (GMC) executed a Real Estate
Mortgage in favor of Equitable PCI Bank over its real properties situated
in Bulacan. The Mortgage secured the P2M loan granted by PCI Bank to
GMC.
GMC failed to settle its loan obligation, which lead to PCI Bank
extrajudicially foreclosing the mortgaged property.
During public auction, the property was sold to PCI Bank for P3.5M. The
Certificate of Sale was registered and inscribed on February 16,
2001.
reiterated to the counsel of PCI Bank in March 12.
Counsel of PCI Bank informed GMC that such redemption is no longer
possible because the certificate of sale had already been registered.
On December 7, 2001, PCI filed a complaint for specific performance
and damages against PCI asserting that it is the one-year period of
redemption under Act No. 3135 which should apply and not the
shorter redemption period provided in Section 47 of RA 8791.
GMC argues that applying Section 47 would result in the impairment of
obligation of contracts and violation of the equal protection clause.
RTC rendered a decision dismissing the complaint. It noted that the
issue of constitutionality was never raised by petitioner during pre-
trial and trial. The attempt of GMC to redeem was already late and
there was no valid redemption made because the counsel did not
have authority to transact on behalf of the company.
GMC appealed to the CA. The CA affirmed the RTC decision saying that
GMC failed to justify why Section 47 of RA 8791 should be declared
unconstitutional. The MR was also denied.
Hence, the present petition.

ISSUE/s:
WoN Section 47 of RA 8791 is violative of the proscription against
impairment of obligations of contract and equal protection clause. -
NO

RULING: Petition denied.

RATIO:
For a law to be nullified, it must be shown that there is a clear and
unequivocal breach of the Constitution.
GMC’s contention that Section 47 of R.A. 8791 violates the constitutional
proscription against impairment of the obligation of contract has no
basis.
The purpose of the non-impairment clause is to safeguard the integrity of
contracts against unwarranted interference by the State.
As a rule, contracts should not be tampered with by subsequent laws
that would change or modify the rights and obligations of the parties.
Impairment is anything that diminishes the efficacy of the contract.
There is an impairment if a subsequent law changes the terms of a
contract between the parties, imposes new conditions, dispenses
with those agreed upon or withdraws remedies for the enforcement
of the rights of the parties.
Under RA 8791, juridical persons are allowed to exercise the right of
redemption only until, but not after, the registration of the certificate
of foreclosure sale and in no case more than three months after
foreclosure, whichever comes first.
Section 47 did not divest juridical persons of the right to redeem their
foreclosed properties but only modified the time for exercise of such
right by reducing the one year period originally provided in Act 3135.
The contention that Section 47 is violative of the equal protection clause as it
discriminated mortgagors/property owners who are juridical persons does not
have merit.
Equal protection permits reasonable classification. Classes may be
treated differently form another where groupings e based on
reasonable and real distinctions.
The legislature clearly intended to shorten the period of redemption for
juridical persons. The difference in treatment of juridical persons and
natural persons was based on the nature of the properties
foreclosed.
Residences are allowed the more liberal one year redemption period
Industrial or commercial property is reduced to a shorter redemption
term to reduce the period of uncertainty in ownership of property
and enable mortgage-banks to dispose sooner of the acquired
assets. This is to ensure the solvency and liquidity of banks as
provided in the General Banking Law of 2000.
The General Banking Law of 2000 was crafted in the aftermoath of the
1997 Southeast Asian financial crisis, and it sought to reform the
General Banking Act by fashioning a legal framework for maintaining
a safe and sound banking system. In this context, the amendment
introduced by Section 47 embodied one of such safe and sound
practices aimed at ensuring the solvency and liquidity of our banks
This legitimate public interest pursued by the legislature further enfeebles
GMC’s impairment of contract theory.
As with other individual rights to contract and to property, the right of
redemption has to give way to police power exercised for public
welfare.
The freedom to contract is not absolute; all contracts and all rights are
subject to the police power of the State and not only may regulations
which affect them be established by the State, but all such
regulations must be subject to change from time to time, as the
general well-being of the community may require, or as the
circumstances may change, or as experience may demonstrate the
necessity.
Settled is the rule that the non-impairment clause of the Constitution
must yield to the loftier purposes targeted by the Government. The
right granted by this provision must submit to the demands and
necessities of the State’s power of regulation.
Such authority to regulate businesses extends to the banking industry
which, as this Court has time and again emphasized, is undeniably
imbued with public interest.
PRYCE CORPORATION v. COURT OF APPEALS (ARIELLE) Pricing of the assets for dacion shall be based on the average of two
valuation appraisals from independent third-party appraisers accredited
February 4, 2008 | Sandoval-Gutierrez, J. | Impairment of obligation of with BSP;
contracts

PETITIONER: Pryce Corporation


RESPONDENTS: The Court of Appeals and China Banking Corporation

SUMMARY: Due to the Asian financial crisis, Pryce Corporation’s


operatios was heavily affected. Thus, it filed a petition before the RTC for
rehabilitation and the appointment of a Rehabilitation Receiver. The RTC
granted the petition and approved of Pryce’s Rehabilitation Plan, which
provided that its creditors will be paid through dacion en pago of assets
already mortgaged to them.

China Banking Corporation alleged that Pryce was solvent and it filed the
petition to force its creditors to accept dacion payments. The CA reversed
the RTC’s decision.

The SC held that Sec. 6 of the Interim Rules of Procedure on Corporate


Rehabilitation provides that the petition must be “sufficient in form and
substance.” The petition for rehabilitation does not allege that there is a
clear and imminent danger that Pryce will lose its corporate assets if a
receiver is not appointed. The “serious situation test” laid down by Rizal
Commercial Banking Corporation has not been met.

DOCTRINE: The purpose of the law in directing the appointment of


receivers is to protect the interests of the corporate investors and
creditors.

FACTS:
Petitioner Pryce Corporation was incorporated under Philippine laws and
its primary purpose was to develop real estate in Mindanao. It
engaged in the development of memorial parks, operated a major
hotel in CDO, and produced industrial gases.
During the 1997 Asian financial crisis, Pryce’s operations was severely
affected, resulting in heavy losses. Thus, Pryce filed with the RTC a
petition for rehabilitation and prayed for the appointment of a
Rehabilitation Receiver from among the nominees named and the
staying of the enforcement of all claims, monetary or otherwise
against it.
The features of the Rehabilitation Plan included the following:
The bank creditors will be paid through dacion en pago of assets already
mortgaged to them;
In case the value of the mortgaged assets is less than the amount of the
obligation to be paid, the deficiency shall be settled by way of dacion of
memorial park lots owned by Pryce;
All penalties shall be waived by the creditors; Sec. 6 of the Interim Rules of Procedure on Corporate Rehabilitation
The RTC issued a “Stay Order” directing that all claims against Pryce be provides that the petition must be “sufficient in form and
deferred. The RTC then appointed Gener T. Mendoza as substance.”
Rehabilitation Receiver. The petition was opposed by Pryce’s bank- In Rizal Commercial Banking Corporation v. IAC, the Court held that
creditors. The BPI claimed that the petition and the proposed
Rehabilitation Plan are coercive and violative of the contract. The
Land Bank of the Philippines contended that the petition is
unacceptable because of the urealistic valuation of the properties
subject of the dacion en pago.
The China Banking Corporation alleged that Pryce is solvent and that it filed
the petition to force its creditors to accept dacion payments. In effect,
Pryce passed on to the creditors the burden of marketing and financing
unwanted memorial lots, while exempting itself from paying interests and
penalties.
The RTC issued an Order, referring the Rehabilitation Plan to Gener
Mendoza, the Rehabilitation Receiver, for evaluation and
recommendation.
The Rehabilitation Receiver, submitted a Rehabilitation Plan,
recommending the following:
Payment of all bank loans and long-term commercial papers through dacion
en pago;
Payment of all non-bank and other payables amounting to at least 500k
through a dacion of memorial park lots;
Payment in cash over a three-year period, without interest, of all non-bank,
trade and other payables amounting to less than 500k each.
The Rehabilitation Reciever further proposed amendments, providing
that the asset base from which the creditors may choose to be paid
has been broadened and will no longer be limited to assets already
mortgaged.
The RTC approved the Amended Rehabilitation Plan, finding Pryce
eligible to be placed in a state of corporate rehabilitation.
Respondent China Banking Corporation filed with the CA a petition for
review, alleging that in approving the Amended Rehabilitation Plan,
the RTC impaired the obligations of contracts.
The CA granted China Banking’s petition, and reversed the assailed
Orders of the RTC, hence the petition.

ISSUE/s:
WoN the CA erred in denying the petition for rehabilitation of petitioner
Pryce Corporation – NO

RULING:
Petition is DENIED. The assailed Decision of the CA is AFFIRMED. Let
the records of the case be REMANDED to the RTC.

RATIO:
under Sec 6 of PD No. 902-A, receivers may be appointed whenever
necessary in order to preserve the rights of the parties-litigants; and/or to
protect the interest of the investing public and creditors.
The situations contemplated in these instances are serious in nature.
There must exist a clear and imminent danger of losing the
corporate assets if a receiver is not appointed.
Absent such danger, the need for appointing a receiver does not
exist. Simply put, the purpose of the law in directing the
appointment of receivers is to protect the interests of the
corporate investors and creditors.
The Court agrees with the CA that the petition for rehabilitation does not
allege that there is a clear and imminent danger that Pryce will lose
its corporate assets if a receiver is not appointed. The “serious
situation test” laid down by Rizal Commercial Banking Corporation
has not been met.
The Stay Order issued by the RTC does not state any serious
situation affecting Pryce’s corporate assets. In appointing Gener
Mendoza as Rehabilitation Receiver, the only basis of the lower
court was its finding that “the petition is sufficient in form and
substance.”
However, the lower court did not specify any reason or ground to
sustain such finding. Clearly, the petition failed to comply with the
serious situation test.
In determining whether Pryce’s financial situation is serious and
whether there is a clear and imminent danger that it will lose its
corporate assets, the RTC, acting as commercial court, should
conduct a hearing where both parties can present their
respective evidence.
PRYCE CORPORATION v. CHINA BANKING (IYA) Corporation’s properties without Chinabank’s consent not only
February 18, 214 | Leonen, J. | Non-Impairment of Contracts violated the mutuality of contracts and due process, but was also
antithetical to the avowed policies of the State to maintain a
competitive financial institution.
PETITIONER: Pryce Corporation The Bank of the Philippine Islands (BPI), another creditor of petitioner
RESPONDENTS: China Banking Corporation Pryce Corporation, filed a separate petition with the Court of Appeals
assailing the same order by the rehabilitation court.
SUMMARY: The SC in its previous ruling allowed China Banking Corp BPI called the attention of the court to the non-impairment clause and the
and Pryce Corporation to settle their disputes through a mutual mutuality of contracts purportedly violated by the approved
agreement. They were given a period of 2 months, However, since no rehabilitation plan.
agreement was settled, the SC now proceeds to decide not he MR filed CA set aside the rehabilitation court’s order favoring Chinabank.
by Pryce Corporation. With regard to BPI’s separate appeal, the CA dismissed the petition,
which was brought to the SC. The SC which denied the claims of BPI
Pryce contends that the decision in BPI has the effect of res judicata with finality, favoring Pryce Corporation.
withe regard to the case with Chinabank. Chinabank argues that the Pryce Corporation appealed to the SC the decision of the CA granting
Interim Rules allowing rehabiliation court to interfere with creditor-debtor Chinabank’s petition.
relationships of corporations violates the non-impairment clause. The SC allowed Chinabank and Pryce to enter into a mutually accepted
agreement for a period of 2 months, however no agreement was
The Court ruled with finality that the inherent power of the State of police settled.
power cannot be limited by the non-impairment clause. That corporate Hence, the present motion.
rehabilitation is vested with public interest which the State must protect.
ISSUE/s:
DOCTRINE: Nevertheless, this court has brushed aside invocations of
the non-impairment clause to give way to a valid exercise of police WON the BPI ruling has the effect of res judicata - YES
power and afford protection to labor. WON the Interim Rules violate the non-impairment clause - NO (proceed
to Ratio #14 onward)

FACTS: RULING: WHEREFORE, the motion of Pryce Corporation is


This case resolves conflicting decisions between two divisions. Only one GRANTED.
may serve as res judicata or a bar for the other to proceed.
The present case originated from a petition for corporate rehabilitation RATIO:
field by Pryce Corporation before the RTC. Pryce Corporation argues that the issue on the validity of the
The RTC appointed Gener Mendoza as rehabilitation receiver. rehabilitation court is now res judicata by citric of the decision of the
He did not approve of the plan submitted before him nd instead Supreme Court in the case with BPI (Fact #8).
amended such which the rehabilitation court approved. It Pryce Corp also contends that the Interim Rules of Procedure on
found Pryce Corporation “eligible to be placed in a state of Corporate Rehabilitation does not require the rehabilitation court to
corporate rehabilitation.” hold hearing before issuing stay order.
China Banking Corporation elevated the case tot he CA questioning the According to the doctrine of res judicata, “a final judgment or decree on
order under the approved Rehabilitation plan which states: the merits by a court of competent jurisdiction is conclusive of the
The bank creditors will be paid through dacion en pago of assets already rights of the parties or their privies in all later suits on all points and
mortgaged to them; matters determined in the former suit.”
In case the value of the mortgaged assets is less than the amount of the The elements for res judicata to apply are as follows:
obligation to be paid, the deficiency shall be settled by way of dacion the former judgment was final;
of memorial park lots owned by Pryce; the court that rendered it had jurisdiction over the subject matter
Pricing of the assets for dacion shall be based on the average of two
and the parties;
valuation appraisals from independent third-party appraisers accredited
the judgment was based on the merits; and
with BSP;
between the first and section actions, there was an identity of
All penalties shall be waived by the creditors; parties, subject matters, and cause of action
Chinabank avers that the plan’s approval authorizing deacon en pago of
Res judicata embraces two concepts:
Pryce
bar by prior judgment
conclusiveness of judgment Rather than leave it to the strongest or most resourceful
Bar by prior judgment exists when, as between the first case where the amongst all of them, the state steps in to equitably distribute
judgment was rendered and the second case that is sought to be the corporation’s limited
barred, there is identity of parties, subject matter, and causes of
action.
The concept of conclusiveness of judgment finds application when a fact or
question has been squarely put in issue, judicially passed upon, and
adjudged in a former suit by a court of competent jurisdiction. This
principle only needs identity of parties and issues to apply.
The elements of res judicata through bar by prior judgment are present in
this case.
In the present case, respondent China Banking Corporation and BPI are
creditors of petitioner Pryce Corporation and are both questioning
the rehabilitation court’s approval of the amended rehabilitation plan.
Thus, there is substantial identity of parties since they are litigating for
the same matter and in the same capacity as creditors of petitioner
Pryce Corporation.
There is no question that both cases deal with the subject matter of
petitioner Pryce Corporation’s rehabilitation. The element of identity
of causes of action also exists.
Since the order approving the amended rehabilitation plan was affirmed and
made final in the BPI case, this plan binds all creditors including
Chinabank.
The rehabilitation court complied with the Interim Rules in its order on the
issuance of a stay order and appointment of Gener T. Mendoza as
rehabilitation receiver.
Chinabank mainly argues the violation of the constitutional proscription
against impairment of contractual obligations, that the Interim Rules
does not empower commercial courts to render without force and
effect valid contractual stipulations.
The non-impairment clause first appeared int eh United States
Consitution as States Constitution as a safeguard against the
issuance of worthless paper money that disturbed economic stability
after the American Revolution.
This clause first became operative in the Philippines through the
Philippine Bill of 1902, the fifth paragraph of Section 5 which states
that no law impairing the obligation of contracts shall be enacted.
Such was consistently adopted in the PH Constitutions.
Nevertheless, this court has brushed aside invocations of the non-
impairment clause to give way to a valid exercise of police
power and afford protection to labor.
Corporate rehabilitation is one of many statutorily provided
remedies for businesses that experience a downturn. Rather
than leave the various creditors unprotected, legislation now
provides for an orderly procedure of equitably and fairly
addressing their concerns.
Necessarily, a business in the red and about to incur tremendous
losses may not be able to pay all its creditors.
resources.
The cram-down principle adopted by the Interim Rules does, in effect,
dilute contracts.
Rather than let struggling corporations slip and vanish, the better
option is to allow commercial courts to come in and apply the
process for corporate rehabilitation.
This option is preferred so as to avoid what Garrett Hardin called the Tragedy
of Commons. Here, Hardin submits that coercive government regulation
is necessary to prevent the degradation of common-pool resources since
individual resource appropriators receive the full benefit of their use and
bear only a share of their cost.
RE: LETTER DATED APRIL 18, 2011 OF CHIEF PUBLIC instituting an action in court,” are not exempted from the payment of
ATTORNEY PERSIDA RUEDA-ACOSTA REQUESTING sheriff’s expenses.
EXEMPTION FROM THE PAYMENT OF SHERIFF’S EXPENSES The Office of the Court Administrator (OCA) explained that sheriff’s
(ELIEL)
July 30, 2013 | Reyes, J. | Free access to courts and legal
assistance

PETITIONER: Attorney Persida Rueda-Acosta


RESPONDENTS: Office of the Court Administrator

SUMMARY: Atty Acosta sought for a clarification from the Office of the
Court Administrator for the exemption regarding the payment of Sheriff’s
fees. Atty. Acosta contends that under RA 9406, PAO’s clients are
exempted from paying legal fees and anything incidental for the institution
of an action in court. The OCA explained, that sheriff’s fees are not those
that are exempted from the fees under RA 9406. Hence, this petition by
Atty. Acosta.

The SC held that sheriff’s fees are not among those that are exempted
from legal fees of PAO’s clients. Sheriff’s fees are those that are
deposited to the Clerk of Court to defray the actual travel expenses of
such officers. However, as a principle and guarantee laid down in the
Constitution, the free acess to the courts and legal access to the under
privileged shall be recognized. The SC then held that the Public
Attorneys Office are authorized to serve summons, subpoenas and other
processes in behalf of their clients. And in turn, could be compensated
instead in their costs of suit, attorney’s fees or contigents fees.

DOCTRINE: The Constitution affords litigants –moneyed or poor – equal


access to the courts; moreover, it specifically provides that poverty shall
not bar any person from having access to the courts. Accodingly, laws
and rules must be formulated, interpreted, and implement pursuant to the
intent and spirit of this constitutional provision

FACTS:
Atty. Acosta sought a clarification as to the exemption of Public Attorney’s
Office (PAO) clients from the payment of sheriff’s expenses, alleging
that PAO’s clients in its Regional Office in Region VII are being
charged with the payment of sheriff’s expenses in the amount of
P1,000.00 upon the filing of a civil action in court.
She claimed that sheriff’s expenses should not be exacted from PAO’s
clients since Section 6 of RA 9406 specifically exempts them from
the payment of docket and other fees incidental to instituting an
action in court and the other quasi-judicial bodies.
OCA clarified that PAO’s clients, notwithstanding their exemption under Sec
6 of RA 9406 from payment of “docket and other fees incidental to
expenses, strictly speaking, are not considered as “legal fees” under Indeed, requiring PAO’s clients to pay sheriff’s expenses, despite their
Rule 141 of the Rules of Court since they are not payable to the exemption from the payment of docket and other legal fees, would
government; they are payable to the sheriff/process server to defray effectly fetter their free access to the courts thereby negating the
his travel expenses in serving court processes in relation to the laudable intent of
litigant’s case.
Atty. Acosta maintained that, while sheriff’s expenses may not be strictly
considered as a legal fee, they are nevertheless considered as a fee
which is incidental to the filing of an action in court and, hence,
should not be exacted from PAO’s clients.
The Court en banc issued Resolution dated November 22, 2011 which
denied Atty. Acosta’s request for exemption from the payment of sheriff’s
expenses.
The OCA averred that the exemption of PAO’s clients from payment of
legal fees is not an absolute rule and that the Court is not precluded
from providing limitations thereto.
Hence this petition

ISSUE/s:
WoN PAO’s clients are exempted from paying sheriff’s fees – NO

RULING: Guilty of the crime of illegal possession of firearms.

RATIO:
Contrary to Atty. Acosta’s claim a plain reading of the said provision clearly
shows that the exemption gratned to PAO’s clients cannot be extended
to the payment of sheriff’s expenses; the exemption is specifically limited
to the payment of fees, i.e., docket and other fees incidental to instituting
an action
The term “fees” is defined as a charged fixed by law or by an institution
for certain privileges or services.
Sheriff’s expenses, however, cannot be classified as a “fee” within the
purview of the exemption granted to PAO’s clients under Sec 6 of RA
9406.
Sheriff’s expenses are not exacted for any service rendered by the
court; they are the amount deposited to the Clerk of Court upon
filing of the complaint to defray the actual travel expenses of
the sheriff, process server or other court-authorized persons in
the service of summons, subpoena and other court processes
that would be issued relative to the trail of the case.
It is not the same as sheriff’s fees under Sec 10, Rule 141 of the Rules of
Court, which refers to those imposed by the court for services
rendered to a party incident to the proceedings before it.
The Court, however, is not unmindful of the predicament of PAO’s
clients. In exempting PAI’s clients from paying docket and other
legal fees, RA 9406 intended to ensure that the indigents and
the less privileged, who do not have the means to pay the said
fees, would nto be denied access to courts by reason of poverty
Congress in enacting RA 9406
Free access to the courts and adequate legal assistance are among
the fundamental rights which the Constitution extends to the
less privileged. Thus, section 11, Art 3 of the 1987 Constitution
mandates that “free access to the courts and quasi-judicial
bodies and adequate legal assistance shall not be denied to any
person by reason of poverty.”
The Constitution affords litigants –moneyed or poor – equal access
to the courts; moreover, it specifically provides that poverty
shall not bar any person from having access to the courts.
Accodingly, laws and rules must be formulated, interpreted, and
implement pursuant to the intent and spirit of this constitutional
provision
Access to justice by all, especially by the poor, is not simply an
ideal in our society. Its existence is essential in a democracy
nad in the rule of law.
Without doubt, one of the most precious rights which must be
shielded and secured is the unhampered access to the justice
system by the poor, the underprivileged and the marginalized.
Having the foregoing principles in mind, the Court, heeding the
constitutional mandate of ensuring free access to the courts and
adequate legal assistance to the marginalized and less privileged,
hereby authorizes the officials and employees of PAO to serve
summons, subpoena and other court processes pursuant to Sec 3,
Rule 14 of the Rules of Court. The authority given herein by the
Court to the officials and employees of PAO shall be limited only to
cases involving their client.
Authorizing the officials and employees of PAO to serve the
summons, subpoenas and other court processes in behalf of
their clients would relieve the latter from the burden of paying
for the sheriff’s expenses despite their non-exemption from the
payment thereof under Sec 6 of RA 9406.
The amount to be defrayed in the service of summons, subpoena
and other court processes in behalf of its clients would
consequently have to be taken from the operating expenses of
PAO.
In turn, the amount advanced by PAO as actual travel expenses may
be taken from the amount recovered from the adversaris of
PAI’s cleints as costs of suit, attorney’s fees or contingent fees
prior to the deposit thereof in the Nationa Treasury.
MIRANDA v. ARIZONA (JP) In all four cases, the questioning elicited oral admissions, and,
June 13, 1966 | Warren, C. J. | Right to Remain Silent, Counsel, Waiver in three of them, signed statements as well, which were
admitted at their trials. All defendants were convicted, and all
PETITIONER: Ernesto Miranda convictions, except in No. 584, were
RESPONDENTS: State of Arizona

SUMMARY: Four cases are presented wherein in each of these cases,


the defendant, while in police custody, was questioned by authorities in
intimdating situations. None of these defendants was given full and
effective warning of their right to remain silent and right to counsel. Most
were alleged to have waived such rights eventhough they in fact did not.

The SC reverses the lower SC decisions on the first three cases. The SC
affirmed the last case however. In sum, the SC reiterated that at the start
of the custodial investigation or detention, the defendants should have
been informed of their rights. The SC also held that waiver of such
fundamental rights cannot be presumed. In the absence of an effective
waiver, it would be deemed that such confessions presented are
inadmissible for being forced out of the defendants by the police
authorities.

DOCTRINE: When an individual is taken into custody or otherwise


deprived of his freedom by the authorities in any significant way and is
subjected to questioning, the privilege against self-incrimination is
jeopardized. Procedural safegards must be employed to protect the
privilege.

He must be warned prior to any questioning that he has the right to


remain silent, that anything he says can be used against him in a court of
law, that he has the right to the presence of an attorney, and that, if he
cannot afford an attorney one will be appointed for him prior to any
questioning if he so desires.

After such warnings have been given, and such opportunity afforded him,
the individual may knowingly and intelligently waive these rights and
agree to answer questions or make a statement. But unless and until
such warnings and waiver are demonstrated by the prosecution at trial,
no evidence obtained as a result of interrogation can be used against
him.

FACTS:
(Note: Landmark case. Miranda Rights. Read the origs.)
In each of these cases, the defendant, while in police custody, was
questioned by police officers, detectives, or a prosecuting attorney in
a room in which he was cut off from the outside world.
None of the defendants was given a full and effective warning of his
rights at the outset of the interrogation process.
affirmed on appeal.
No. 759. Miranda v. Arizona:
Petitioner, Ernesto Miranda (inidigent Mexican with pronounced sexual
fantasies), was arrested at his home and taken in custody to a
Phoenix police station. He was there identified by the complaining
witness.
The police then took him to "Interrogation Room No. 2" of the detective
bureau. There he was questioned by two police officers.
The officers admitted at trial that Miranda was not advised that he had a
right to have an attorney present.
Two hours later, the officers emerged from the interrogation room with a
written confession signed by Miranda.
At the top of the statement was a typed paragraph stating that the
confession was made voluntarily, without threats or promises of
immunity and "with full knowledge of my legal rights, understanding
any statement I make may be used against me."
Miranda was found guilty of kidnapping and rape. He was sentenced to
20 to 30 years' imprisonment on each count, the sentences to run
concurrently.
On appeal, the Supreme Court of Arizona affirmed the conviction.
In reaching its decision, the court emphasized heavily the fact that
Miranda did not specifically request counsel.
No. 760. Vignera v. New York
Petitioner, Michael Vignera, was picked up by New York police in
connection with the robbery three days earlier of a Brooklyn dress
shop.
He was taken to the 66th Detective Squad. There a detective questioned
Vignera with respect to the robbery. Vignera orally admitted the
robbery to the detective.
The detective was asked on cross-examination at trial by defense counsel
whether Vignera was warned of his right to counsel before being
interrogated. The prosecution objected to the question, and the trial
judge sustained the objection. Thus, the defense was precluded from
making any showing that warnings had not been given.
Vignera was questioned by an assistant district attorney in the presence
of a hearing reporter, who transcribed the questions and Vignera's
answers. This verbatim account of these proceedings contains no
statement of any warnings given by the assistant district attorney.
The trial judge charged the jury: The law doesn't say that the confession
is void or invalidated because the police officer didn't advise the
defendant as to his rights.
Vignera was found guilty of first degree robbery. He was adjudged a third-
felony offender and sentenced to 30-60 years’ imprisonment.
In argument to the Court of Appeals who affirmed, the State contended
that Vignera had no constitutional right to be advised of
his right to counsel or his privilege against self-incrimination. crime, the police then released the other four persons arrested
No. 761. Westover v. United States with him.
Petitioner, Carl Calvin Westover, was arrested by local police in Stewart was charged with kidnapping to commit robbery, rape, and
Kansas City as a suspect in two Kansas City robberies. murder. The jury found Stewart guilty of robbery and first degree
Kansas City police interrogated Westover on the night of his
arrest. He denied any knowledge of criminal activities. The
next day, local officers interrogated him again throughout the
morning.
Shortly before noon, they informed the FBI that they were
through interrogating Westover and that the FBI could
proceed to interrogate him.
There is nothing in the record to indicate that Westover was ever
given any warning as to his rights by local police.
Special agents of the FBI continued the interrogation in a private
interview room of the Kansas City Police Department, this
time with respect to the robbery of a savings and loan
association and bank in Sacramento, California.
After two or two and one-half hours, Westover signed separate
confessions to each of these two robberies which had been
prepared by one of the agents during the interrogation.
At trial, one of the agents testified, and a paragraph on each of
the statements states, that the agents advised Westover that
he did not have to make a statement, that any statement he
made could be used against him, and that he had the right to
see an attorney.
He was sentenced to 15 years imprisonment on each count, the
sentences to run consecutively. On appeal, the conviction
was affirmed by the Court of Appeals for the Ninth Circuit.
No. 584. California v. Stewart
In the course of investigating a series of purse-snatch robberies
in which one of the victims had died of injuries inflicted by
her assailant, respondent, Roy Allen Stewart (indigent Negro
who had dropped out of school in sixth grade), was pointed
out to Los Angeles police as the endorser of dividend checks
taken in one of the robberies.
One of the officers asked Stewart if they could search the house,
to which he replied, "Go ahead." The search turned up
various items taken from the five robbery victims.
Stewart was taken to the University Station of the Los Angeles
Police Department, where he was placed in a cell. During
the next five days, police interrogated Stewart on nine
different occasions.
During the ninth interrogation session, Stewart admitted that he had
robbed the deceased and stated that he had not meant to hurt
her.
Police then brought Stewart before a magistrate for the first time.
Since there was no evidence to connect them with any
murder, and fixed the penalty as death. "legal rights" does not approach the knowing and intelligent
The SC of California however, reversed. It held that, under waiver required to relinquish constitutional rights.
this Court's decision in Escobedo, Stewart should have For Vignera v. New York
been advised of his right to remain silent and of his right The SC reverses. The foregoing indicates that Vignera was not
to counsel, and that it would not presume in the face of a
silent record that the police advised Stewart of his rights.

ISSUE/s:
WoN Fifth Amendment privilege of the defendants in the four cases
were violated in the conduct of the police officers handling them
during custodial investigations – YES
RULING: The judgments of the Supreme Court Of Arizona in No. 759,
of the New York Court of Appeals in No. 760, and of the Court of
Appeals for the Ninth Circuit in No. 761, are reversed. The judgment of
the Supreme Court of California in No.
584 is affirmed.

RATIO:
272. When an individual is taken into custody or otherwise deprived
of his freedom by the authorities in any significant way and is
subjected to questioning, the privilege against self-incrimination
is jeopardized. Procedural safegards must be employed to
protect the privilege.
273. He must be warned prior to any questioning that he has the
right to remain silent, that anything he says can be used against
him in a court of law, that he has the right to the presence of an
attorney, and that, if he cannot afford an attorney one will be
appointed for him prior to any questioning if he so desires.
After such warnings have been given, and such opportunity afforded
him, the individual may knowingly and intelligently waive these
rights and agree to answer questions or make a statement. But
unless and until such warnings and waiver are demonstrated by
the prosecution at trial, no evidence obtained as a result of
interrogation can be used against him.
For Miranda v. Arizona:
The SC reverses the lower court decision. From the
testimony of the officers and by the admission of
respondent, it is clear that Miranda was not in any way
apprised of his right to consult with an attorney and to
have one present during the interrogation, nor was his
right not to be compelled to incriminate himself
effectively protected in any other manner. Without these
warnings, the statements were inadmissible.
The mere fact that he signed a statement which contained a
typed-in clause stating that he had "full knowledge" of his
warned of any of his rights before the questioning by the detective
and by the assistant district attorney.
No other steps were taken to protect these rights. Thus, he was
not effectively apprised of his Fifth Amendment privilege or of
his right to have counsel present, and his statements are
inadmissible.
For Westover v. United States
The SC reverses. Fromt he the facts of this case, the SC cannot
find that Westover knowingly and intelligently waived his
right to remain silent and his right to consult with counsel
prior to the time he made the statement.
The FBI interrogation began immediately upon the conclusion of
the interrogation by Kansas City police, and was conducted
in local police headquarters.
Although the two law enforcement authorities are legally distinct,
and the crimes for which they interrogated Westover were
different, the impact on him was that of a continuous period of
questioning.
There is no evidence of any warning given prior to the FBI
interrogation, nor is there any evidence of an articulated
waiver of rights after the FBI commenced its interrogation.
Despite the fact that the FBI agents gave warnings at the outset
of their interview, from Westover's point of view, the warnings
came at the end of the interrogation process.
The FBI interrogation was conducted immediately following the
state interrogation in the same police station, in the same
compelling surroundings.
For California v. Stewart:
The SC affirms. In dealing with custodial interrogation, the SC
will not presume that a defendant has been effectively
apprised of his rights and that his privilege against self-
incrimination has been adequately safeguarded on a record
that does not show that any warnings have been given or
that any effective alternative has been employed.
Nor can a knowing and intelligent waiver of these rights be
assumed on a silent record.
Furthermore, Stewart's steadfast denial of the alleged offenses
through eight of the nine interrogations over a period of five
days is subject to no other construction than that he was
compelled by persistent interrogation to forgo his Fifth
Amendment privilege.
MAGTOTO v. MANGUERA (HENRY) against him. Any confession obtained in violation of this section
March 3, 1975 | Fernandez, J. | Rights of the accused shall be inadmissible

PETITIONER: Clemente Magtoto


RESPONDENTS: Hon. Miguel M. Manguera, Judge of CFI Occidental
Mindoro (Br. 2), People of the Philippines, Ignacio Calara, Jr., and Lourdes
Calara

SUMMARY: The consolidated cases at bar (all 3 of them) actually involve


the question with regard the interpretation of Sec 20, Art IV of the 1973
Constitution, which basically includes among others the right to counsel,
and the right to be informed of such right. Such provision is being asked
to be construed because in the CFI, the three cases involved were
judged in the manner that such confessions obtained from the detained
person are admissible. Considering that such confessions were made
before the effectivity of the 1973 Constitution, this CFI decision was
grounded on the fact that there is no retroactive effect as to the said
constitutional provision.

What the petitioners claim in this case is that, they are denied of the right
to counsel and to be informed of such right, if the CFI judgment will be
binding. They said that it should have a retroactive effect because they
claim that that was the intent of the Consti Comm.

SC basically sustained the CFI ruling and backed it up with the history of
the said provision. Moreso, it was emphasized by jurisprudence both in
the PH and the US, that such rule is to have only a prospective effect.
Such provision, if given retroactive effect might be a problematic stint
insofar as the administration of justice is concerned. Therefore, it was
held that the confessions are admissible, since there was no existence of
such rights during the time the confessions were made.

DOCTRINE: To give a retroactive effect to this constitutional guarantee to


counsel would have a great unsettling effect on the administration of
justice in this country. It may lead to the acquittal of guilty individual sand
thus cause injustice to the People and the offended parties in many
criminal cases where confessions were obtained before the effectivity of
the 1973 Constitution and in accordance with the rules then in force
although without assistance of counsel.

FACTS:
This case basically revolves around the interpretation of Sec 20, Art IV of
the Constitution, which reads:
“No person shall be compelled to be a witness against himself. Any
person under investigation for the commission of an offense
shall have the right to remain silent and to counsel, and to be
informed of such right. No force, violence, threat, intimidation,
or any other means which vitiates the free will shall be used
in evidence” in the other case, whichheld the confessions inadmissible, although
Consequently, a confession obtained from a person under investigation they have not been
for the commission of an offense, who has not been informed of his
right to counsel, is inadmissible in evidence if the same had been
obtained after the effectivity of the 1973 Constitution, even if
presented after January 17, 1973 (effectivity of 1973 Consti), and
even if he had not been informed of his right to counsel, since no law
gave the accused the right to be so informed before that date.
In context, what happened in this case was, Sec 20 of Art IV of the 1973
Constitution (eto yung Sec 12 ng Art 3 ng 1987 Consti) was granted
for the first time, to a person under investigation for the commission
of an offense. And, the last sentence of Sec 20, which means that
any confession obtained in violation of this right shall be inadmissible
in evidence, and should be given effect only when the right already
existed and had been violated.
The confessions of the accused in the 3 consolidated cases (let’s call them
case A B and C) were taken before the effectivity of the 1973
Constitution in accordance with the rues then in force, no right had been
violated as to render them inadmissible in evidence although they were
not informed of their “right to remain silent and to counsel” and “to be
informed of such right”. To emphasize, no right existed at that time.
Aggrieved, petitioners (Magtoto, in case A) filed the present petition
because the CFI ruled that such evidence are admissible in evidence.
Petitioners basically claim that it should be inadmissible, there being no
counsel and that they were not informed of such right. (nagsimula lang
talaga yung case na sinabing nag decide na yung CFI regarding the 3
cases, so what you see is what you get sa facts 1-4)

ISSUE/s:
WoN the confessions by the accused are admissible in evidence – YES

RULING: SC sustained the CFI ruling.

RATIO:
An argument was raised, that Art 125 of the RPIC which was added by
RA 1083 enacted in 1954, stated that “In every case, the person
detained shall be informed of the cause of his detention and shall be
allowed, upon his request to communicate and confer at any time
with his attorney or counsel”, to which the SC held untenable. This
provision only granted the right to be informed of the cause of
detention.
The SC went back to the discussion of the Congress of RA 1083 back
when it was just a bill (Senate Bill No 50), where the right to counsel
and being informed of such right was just a mere personal opinion of
Senator Quenco. Consequently, the Congress did not word it in such
a manner that such right be granted to the accused/ detainee.
The SC sustains the CFI rulings declaring admissible the confessions of
the accused in the two (consolidated) cases, and set aside the order
informed of their right to remain silent and to counsel before they The authors of the dissenting opinions of this case ignore the historical
gave confessions, because they were given before the effectivity of fact
the New Constitution.
The SC acknowledged the history of the new right granted to a detained
person by Sec 20, and that it purports to be a right to be accorded
with a prospective, not a retroactive effect, so as to cover only
confessions taken after the effectivity of the New Constitution.
A confession is defined in Sec 29 Rule 130 of the Rules of Court as “the
declaration of an accused expressly acknowledging his guilt of the
offense charged” which, according to the same provision, may be
given in evidence against him.
Further, according to Sec 3 Rule 133 of the Rules of Court, “An
extrajudicial confession made by an accused, shall not be sufficient
gorund for convictioin, unless corroborated by evidence of corpus
delicti”.
US v. Delos Santos is a case where the SC held that confessions
constitute an evidence of a higher order since it is supported by the
strong presumption that no person of normal mind would deliberately
and knowingly confess to a crime unless prompted by truth and
conscience. The fundamental rule is that a confession must be
voluntary to be admissible. Which must be shown to the
satisfaction of the Court that the same was freely and voluntarily
made.
But that was before. The said provision was repealed and now, a
confession is admissible in evidence without previous proof of
its voluntariness on the theory that it is presumed to be
voluntary until the contrary is proved (People v. Dorado)
Several cases in the Philippines have held rulings with regard to the
admissibility of confessions of the accused, but what is binding now
is that of People v. Urro, where involuntary or coerced confessions
obtained by force or intimidation are null and void and are abhorred
by law which proscribes the use of usch cruel and inhuman methods
to secure a confession”
In the light of US jurisprudence, Miranda v. Arizona, it was held that, an
individual taken into custody must be warned prior to any questioning
that he has the right to remain silent, that anything he says can be
used against him in a court of law, that he has the right to the
presence of an attorney, and that if he cannot afford an attorney one
will be appointed for him prior to any question ing if he so desires.
However, in People v. Jose and People v. Paras, the SC rejected the
Miranda rule, on the inadmissibility of extrajudicial stateements given
without the assistance of counsel
The SC found every reason to emphasize the historical background of
Sec 20 of the 1973 Constitution, which clearly shows that the new
right granted to a detained person to counsel and to be informed of
such right under pain of his confession being declared inadmissible
in evidence, has and should be given a prospective and not a
retroactive effect.
that the constitutional and legal guarantees as well as the legal
precedents that insure that the confession be voluntary, underwent a
slow and tedious development. The constitutional guarantee in question
might indeed have come late in the progress of the law on the matter, but
it is only now that it had come under Sec 20 Art IV of the 1973
Constitution, that our duty and power ordain us to proclaim; the SC
cannot properly do more.
To give a retroactive effect to this constitutional guarantee to counsel
would have a great unsettling effect on the administration of justice
in this country. It may lead to the acquittal of guilty individual sand
thus cause injustice to the People and the offended parties in many
criminal cases where confessions were obtained before the
effectivity of the 1973 Constitution and in accordance with the rules
then in force although without assistance of counsel.
The Consti Convention could not have intended such a disastrous
conseqwnce in the administration of justice. For if the cause of justice
suffers when an innocent person is convicted, it equally suffers when a
guilty one is acquitted.
Castro, dissenting: Justice Castro is in the firm view that the second
paragraph of Article 125 of the RPC makes it an obligation on the part of
any detaining officer to inform the person detained of his right to coiunsel
before the very inception of custodial inquest, and that this obligation was
made a statutory one as early as in the year 1954. Hence, it is an error to
say that Sec 20 of the 1973 Constitution granted, for the first time, the
right to counsel to a person under custodial interrogation.
Teehankee, dissenting: The Constitution now expressly protects “a person
under investigation for the commission of an offense from the
overwhelming power of the State and from official abuse and
lawlessness and guarantees that “he shall have the right to remain silent
and to counsel and to be informed of such right.” In order to give force
and meaning to the constitutional guarantee, it flatly outlaws the
admission of any confessioin obtained from a person under investigation
who has not been afforded his right to silence and counsel and to be
informed of such right.
Outlawing of all such confessions is plain, unqualified, and without distinction
whether the invalid confession be obtained before or after the effectivity
of the Constitution. The Court is called upon to enforce the plain mandate
of the Constitution outlawing the admission of such invalid confessions.
Fernando, dissenting: a judge is bereft of the competence, even if he were
so open minded, to impress with admissibility any confession unless the
person under investigation was informed of his right to remain silent and
his right to counsel. Absent such a showing, whatever statement or
admission was obtained during such stage of custodial interrogation is a
worthless piece of paper. So the Constitution demands.
It speaks of no uncertain terms from and after January 17, 1973 when it became
effective. The crucial date is not when the confession was obtained, but
when it was sought to be offered in evidence. Parenthetically, such a mode
of viewing the issue would indicate the irrelevancy of the question of
prospectivity. To repeat, there is no imprecision in the terminology of the
fundamental law. It is quite emphatic in its choice of the phrase, “inadmissible
in evidence” This then is, for Justice Fernando, one of those cases where its
first and fundamental duty to apply the law with the Constitution at the top
ring in the hierarchy of legal nounts. Interpretation therefore comes in only
after it has been demonstrated that application is impossible or inadequate
without its aid.
Antonio, concurring: The constant doctrine of this Court has always
been infavor of the admissibility of statements obtained from a
defendant under police custodial interrogation where the same has
been obtained freely and voluntarily. The Court has always held that
it will suffice for the admission of an extrajudicial confession of an
accused that it appears to have been given under conditions which
accredit prima facie its admissibility, leaving the accused at liberty to
show it was not voluntarily given or was obtained by undue pressure,
thus destroying its weight, and that a presumption of law favors the
spontaneity and voluntariness of a statement given by the defendant
in a criminal case and the burden is upon him to destroy that
presumption.
The purpose of requiring the presence of counsel in police custodial
investigation in Sec 20 of Art IV of the Constitution is to serve as an
effective deterrent to lawless police action. The SC cannot say that this
purpose would be advanced by making the requirement retrospective. If
any misconduct had been committed by the police in connection with the
taking of statements of suspects during custodial interrogation prior to
the effectivity of the New Constitution, it will not be making this
prescription retroactive.
Constitutional provisions should not be given a retrospective
operation, unless that is the unmistakable intention of the words
used or the obvious design of the authors. The rule is prospective.
There is no indication in the language in the Bill of Rights intending it
to operate retrospectively.
PEOPLE v. MAHINAY (DANNAH) When found by the Valenzuela Police, she was wearing a printed blouse,
February 1, 1999 | Per Curiam | Miranda Doctrine no

PETITIONER: People of the Philippines


RESPONDENTS: Larry Mahinay

SUMMARY: Mahinay was the houseboy of Maria Isip. One day, after his
drinking spree with Gregorio Rivera, he asked permission from Isip to go
out with his friends. That same day, Elvira Chan noticed that her daughter
was missing. (Chan and Isip are neighbors). Victoria Chan’s (12 y/o) body
was found in the septic tank by a certain Boy. Upon investigation by the
police, Chan had bruises on her face and was not wearing an underwear.
When they checked the second floor of Isip’s unfinished house, they
found Victoria’s clothes, and some items which belonged to Mahinay.

Mahinay got caught in Ibaan, Batangas. He gave an extrajudicial


confession in the presence of Atty. Viernes (lawyer provided by PAO)
about how he raped and killed Victoria. However, during trial, he claimed
that Victoria was brought to him by Zaldy and Boyet already dead; that
they forced him to rape her and dispose the body. The Court affirmed the
trial court’s conviction based on circumstantial evidence. Through Atty.
Viernes’ testimony it was also ruled that Mahinay was informed of his
constitutional rights prior to execution of his extrajudicial confession.

DOCTRINE: If a suspect is informed properly informed of his


constitutional rights prior to executing an extrajudicial confession, the
same is considered admissible as evidence.

FACTS:
Mahinay, who was a houseboy of Maria Isip, was charged with the rape
and homicide of Ma. Victoria Chan, 12 years old. Victoria was Isip’s
neighbor.
On June 25, 1995, Mahinay joined Rivera in a drinking spree. 10am they
were already drunk. After this, Mahinay asked permission from Isip to
go out with friends.
Same day, Elvira Chan noticed her daughter was missing. She last saw
her daughter wearing a pair of white shorts, brown belt, a yellow hair
ribbon, printed blue blouse, dirty white panty, white lady sando and
blue rubber slippers
Mahinay failed to show up for supper that night.
The next day Mahinay boarded a jeepney driving by Trinidad at the
talipapa and alighted at the top of the bridge of NLEX, and then
disappeared.
That same morning, a certain Boy found the dead body of Ma. Victoria
inside the septic tank; he immediately reported this to Victoria’s
parents.
underwear and her face bore bruises. Sgt. Roberto Suni who was on his way to his in-law’s house met
The Police were informed about Mahinay’s disappearance, and that it Mahinay walking on the road leading to his in-law’s residence which
was unlikely for him to not return within the same day of leaving. is about 50-75
At the scene of the crime, which was the second floor of Isip’s house
under construction, they retrieved a pair of dirty white shots, brown
belt, yellow hair ribbon which was identified by Elvira Chan to belong
to her daughter.
They also found a pair of blue slippers which Isip identified as that of
Mahinay’s. Also, they found close to the septic tank an underwear,
leather wallet, pair of diry long pants and pliers positively identified
by Isip as Mahinay’s belongings.
Mahinay was finally arrested in Ibaan, Batangas. He was brought to the
Police Station and with the assistance of Atty. Viernes, he executed an
extra-judicial confession wherein he narrated in detail how he raped and
killed Victoria.
Also, when Mahinay came face to face with the Victoria’s mother and
aunt, he confided to them that he was not alone in raping and killing
the victim. He pointed to Zaldy and Boyet as his co-conspirators
The lower convicted him and sentenced him to death. Hence, this case
pursuant to automatic review.
Mahinay, in his testimony claims that after drinking, he was awakened by
Zaldy and Boyet who had the corpse of Victoria. They threatened
him with a knife and asked him to rape her. Then, they asked help to
bring her to the septic tank.

ISSUE/s:
WoN Mahinay was informed of his constitutional rights prior to his
execution of an extrajudicial confession – YES

RULING: Conviction affirmed.

RATIO:
Requisites for conviction based on circumstantial evidence:
There is more than one circumstance
The facts from which the inferences are derived are proven
The combination of all the circumstances is such as to produce a
conviction beyond reasonable doubt.
Circumstantial evidence present:
Prosecution witness Norgina Rivera, sister-in-law-of isip, testified that at
9pm in her store Mahinay tried to buy lugaw. His hair was
disarranged, he was drunk and walking in a zigzagging manner. He
appeared uneasy and seemed to be thinking deeply
meters away from the unfinished house of Isip. He also sawa the Also, if they just wanted to dump the body in the septic tank, why
victim Victoria standing at the gate of the unfinished house the would they proceed to the second floor where Mahinay was, just to
same evening. go down again to the septic tank?)
Maria Isip testified that Mahinay was her houseboy, and that he asked
MIRANDA RIGHTS:
permission to leave that morning. It was customary for Mahinay to
The person arrested, detained, invited or under custodial investigation
return the same day or next morning, but he failed to until he was
must be informed in a language known to and understood by him of
arrested in batangas
the reason for the arrest and he must be shown the warrant of
Fernando Trinidad, a jeepney driver pinpointed Mahinay as one of the
arrest, if any. Every other warnings, information or communication
passengers who boarded his jeep and alighted at NLEX
must be in a language known to and understood by said person
Personal belongings of Victoria were found in the unfinished big house
He must be warned that he has a right to remain silent and that any
where Mahinay slept on the night of the incident. This is a clear
statement he makes may be used as evidence against him
indication that she was raped and killed in the same area.
He must be informed that he has the right to be assisted at all times and
Mahinay, during custodial investigation and after having been informed
have the presence of an independent and competent lawyer,
of his constitutional rights with the assistance of Atty. Restituto
preferably of his own choice
Viernes of the Public Attorney’s Office voluntarily gave his
He must be informed that if he has no lawyer or cannot afford the
statement admitting the commission of the crime. Said confession
services of a lawyer, one will be provided for him; and that a lawyer
is believed to have been freely and voluntary given. That Mahinay
may also be engaged by any person in his behalf, or may be
did not complain to the proper authorities of any maltreatment on
appointed by the court upon petition of the person arrested or one
his person.
acting in his behalf
Mahinay testified that he was not able to enter the apartment where he
That whether or not the person arrested has a lawyer, he must be
usually sleeps because it was already closed, hence he proceeded
informed that no custodial investigation in any form shall be
to the second floor of the unfinished house and slept.
conducted except in the presence of his counsel or after a valid
If he did not commit the crime and was just forced to dispose Victoria’s
waiver has been made
body, he could have told this to the lady reporter who interviewed
The person arrested must be informed that, at any time, he has the right
him. His failure and omission to reveal the same is unnatural
to communicate or confer by the most expedient means·telephone,
The circumstance of flight strongly indicates his consciousness of guilt
radio, letter or messenger·with his lawyer (either retained or
Based on the evidence on record, sexual intercourse with Victoria was
appointed), any member of his immediate family, or any medical
adequately proven by the post mortem examination. doctor, priest or minister chosen by him or by any one from his
As may be gleaned from his extrajudicial confession, Mahinay himself immediate family or by his counsel, or be visited by/confer with duly
admitted that he had sexual congress with the unconscious child. accredited national or international non- government organization. It
Another thing that militates against Mahinay is his extra-judicial shall be the responsibility of the officer to ensure that this is
confession, which he, however, claims was executed in violation of accomplished
his constitutional right to counsel. He must be informed that he has the right to waive any of said rights
But his contention is belied by the records as well as the testimony of the provided it is made voluntarily, knowingly and intelligently and
lawyer who assisted, warned and explained to him his ensure that he understood the same
In addition, if the person arrested waives his right to a lawyer, he must
constitutionally guaranteed pre-interrogatory and custodial rights (as
be informed that it must be done in writing and in the presence of
seen in the interrogation during trial)
counsel, otherwise, he must be warned that the waiver is void
Atty. Viernes explained to him his rights under the constitution
even if he insist on his waiver and chooses to speak
It was also explained one by one by Police Officer Alabastro
That the person arrested must be informed that he may indicate in any
The rights were reduced into writing
manner at any time or stage of the process that he does not wish to
Atty. Viernes, after reducing into writing his rights, asked him to sign to
be questioned with warning that once he makes such indication, the
acknowledge and these were signed in Atty. Viernes’ presence
police may not interrogate him if the same had not yet commenced,
The rights were explained in Filipino
or the interrogation must cease if it has already begun
Larry Mahinay, after being apprised of his rights, continued to proceed
The person arrested must be informed that his initial waiver of his right
with his statement
to remain silent, the right to counsel or any of his rights does not bar
(His defense that Zaldy and Boyet forced him to rape the child is incredible.
him from invoking it at any time during the process, regardless of
whether he may have answered some questions or volunteered some
statements
He must also be informed that any statement or evidence, as the case
may be, obtained in violation of any of the foregoing, whether
inculpatory or exculpatory, in whole or in part, shall be inadmissible
in evidence.
Four members of the Court·although maintaining their adherence to the
separate opinions expressed in People v. Echegaray that R.A. No.
7659, insofar as it prescribes the death penalty, is
unconstitutional·nevertheless submit to the ruling of the Court, by a
majority vote, that the law is constitutional and that the death penalty
should accordingly be imposed.
PEOPLE v. CAMAT (ARIELLE) Camat and Del Rosario claim that they did not know each other prior to
April 2, 1996 | Regalado, J. | Origins and Rationale the date of the commission of the crime and that they met each other
only after they were arrested and brought to the police precinct.
PETITIONER: People of the Philippines Patrolman Odeo Cariño, to whom the case was assigned for
RESPONDENTS: Armando Camat and Wilfredo Del Rosario investigation, stated that Camat orally admitted to him his
participation in the killing of the soldier during interrogation.
SUMMARY: Camat and Del Rosario robbed Sinoy and Penalver, and in In addition, Camat also allegedly gave the names of Del Rosario and
the occasion thereof, killed Sinoy but Penalver survived. They were alluded to him as the one who actually stabbed Sinoy.
convicted by the lower court with the so-called special complex crime of With this information, Carino and another policeman traced the whereabouts
robbery with homicide and frustrated homicide. Del Rosario allegedly of Del Rosario and when they found him, invited him for questioning.
confessed to the police his involvement in the crime and informed the Del Rosario allegedly confessed to Carino his involvement in the crime
latter that the electric tester stolen from the victims could be recovered and informed the latter that the electric tester could be recovered
from his relatives. Camat and Del Rosario insist that the lower court from his relatives.
cannot rely on the extrajudicial confession of Del Rosario as a basis for The investigation of the case centred upon Camat only after the latter
their conviction because such confession was obtained during custodial was pointed to by a vendor who allegedly saw what happened.
investigation in violation of their constitutional rights. Camat was arrested by policemen for acts of lasciviousness, upon
the complaint of his sister-in-law.
The SC held that the lower court cannot rely on the extrajudicial Since Camat fitted the description given by the eyewitness, Carino
confession made as basis for the appellants’ conviction because there fetched the vendor to verift Camat’s identity. Said witness recognized
was no evidence that it was made with th assistance of a counsel. and identified Camat as the one who killed Sinoy. Carino did not give
However, the judgment of conviction stands because the prosecution has the identity of the vendor-witness who was afraid of the accused.
The trial court convicted Camat and Del Rosario with the so-called
satisfactorily proved the guilt of both Camat and Del Rosario beyond
special complex crime of robbery with homicide and frustrated
reasonable doubt.
homicide.
DOCTRINE: Even if the confession is gospel truth, if it was made without Camat and Del Rosario insist that the trial court cannot rely on the
extrajudicial confession of Camat as a basis for their conviction
the assistance of counsel, it is inadmissible in evidence regardless of the
because such confession was obtained during custodial investigation
absence of coercion or even if it had been voluntarily given. in violation of their constitutional rights.
Moreover, appellants aver that the lower court also erred in making an
FACTS: inference of guilt from the extrajudicial confession of Del Rosario
Sinoy and Penalver both membres of the Philippine Marines were wherein he supposedly gave to the investigating policeman the name
walking along Quirino Avenue. They came back from a birthday party os his relarive in possession of the electric tester.
and were wearing civilian clothes.
While walking, they noticed two persons trailing them closely. The place was ISSUE/s:
well-lit. Penalver was carrying a clutch bag containing a Sanwa electric WoN Camat and Del Rosario’s extrajudicial confession is admissible –
tester. NO
Del Rosario rushed to Sinoy and kicked the latter. Camat followed Del
Rosario and pulled out a knife and stabbed Sinoy. Penalver kicked RULING: The judgment appealed from is AFFIRMED, with the
Camat who in turn stabbed him.
MODIFICATIONS that appellants are guilty of the crime of robbery with
Realizing they were at the losing end, Sinoy and Penalver ran away. With
the aid of a policeman, they were brought to the hospital. Sinoy died homicide.
at the hospital. Penalver was transferred to the AFP Medical Center
and was discharged days after. RATIO:
Camat and Del Rosario interposed the defense of alibi and denied any The rights invoked by Camat and Del Rosario are premised upon Sec 20
participation in the commission of the felony. Camat claimed that he Art IV of the 1973 Constitution (now sec 12 Art 3). The provision was
was already in his house and preparing to sleep. interpreted in the case of Morales v. Enrile wherein the Court laid
Del Rosario contended that he and his wife were at their stall selling
down the procedure to be followed in custodial investigations:
vegetables along a sidewalk of Quirino Avenue in Baclaran and went
home at 7PM.
At the time a person is arrested, it shall be the duty of the arresting officer to
inform him of the reason for the arrest and he must be shown the
warrant of
arrest, if any. prosecution has satisfactorily proved the guilt of both accused
He shall be informed of his constitutional rights to remain silent and to beyond reasonable doubt.
counsel, and that any statement he might make could be used against It bears repeating that findings of the trial court pertaining to the
him.
credibility of witnesses deserve great respect since it had the
The person arrested shall have the right to communicate with his lawyer,
relatie, or anyone he chooses. opportunity to hear and observe their demeanor as they testified
No custodial investigation shall be conducted unless it be in the presence of on the witness stand.
counsel engaged by the person arrested or appointed by the court.
The right to counsel may be waived but the waiver shall not be valid unless
made with the assistance of counsel.
These rights begin to be available where the investigation is no
longer a general inquiry into an unsolved crime but has began
to focus on a particular suspect, the suspect has been taken into
police custody, and the police carry out a process of interrogation
that lends itself to eliciting incriminating statements.
A reading of the challenged decision shows that the trial court relied upon
appellants’ confessions to disaffirm their credibility and to impugn
their denial of complicity in the commission of the felony.
This the lower court cannot do because absent any showing that Camat and
Del Rosario were duly advised of the mandatory guarantees
under the Bill of Rights, their confessions made befor Carino
are inadmissible against them and cannot be used in support of
their conviction.
It is now incumbent upon the prosecution to prove during the trial that
prior to questioning, the confessant was warned of his
constitutionally protected rights because the presumption of
regularity of official acts does not apply during in-custody
investigation.
Even if the confession is gospel truth, if it was made without the
assistance of counsel, it is inadmissible in evidence regardless
of the absence of coercion or even if it had been voluntarily
given.
As to the implication of Del Rosario in the extrajudicial confession of
Camat, no reliance can be placed on the imputation because it
violates the rule on res inter alios acta and does not fall under its
exceptions since it was made after the supposed homicidal
conspiracy.
An extrajudicial confession is binding only upon the confessant and
is not admissible against his co-accused.
Even disregarding the extrajudicial confessions of appellants, the
judgment of conviction stands. Although there is only one eyewitness
presented by the prosecution, the Court is of the opinion that the
Additionally, there was no ulterior motive on the part of Penalver that might
have led him to give fabricated testimony against appellants. His positive
testimony is sufficient for conviction.
Camat and Del Rosario also contend that the failure of the prosecution to present
in court the police informer deprived them of their constitutional right of
confrontation. The Court reiterates that an accused’s constitutional right to
meet the witnesses face to face is limited to proceedings before the trial
court.
Camat and Del Rosario’s reliance on this constitutional right is misplaced as
the same is available to him at the trial and not during a custodial
investigation.
As to their alibi, it must be noted that alibi becomes less plausible as a
defense when it is established by the accused himself and his immediate
rlatives who, as in tthis case, are appellants’ mother and mother-in-law
because they would naturally be expected to make statements in his
favor.
Appellants should have been indicted only for the special complex crime of
robbery with homicide under Art 294 of the RPC. There is no crime of
robbery with homicide and frustrated homicide.
PEOPLE v. TAYLARAN (IYA) (SUPER CREATIVE PWEDE NA MAGING TELENOVELLA SCENE
October 23, 1981 | De Castro, J. | Right to Silence and to Counsel Evidence for the Defense until Fact #12) After Gregorio tended to
his carabao, on his way home he was bitten by a snake.
PETITIONER: People of the Philippines
RESPONDENTS: Gregorio Taylaran

SUMMARY: Gregorio was charged with the murder of his grandmother-in-


law, Ofremia Atup. While Ofremia was healing his snake-bite, Gregorio
suddenly stabbed her, which resulted to her death. As defense, Gregorio
presents a very improbable and creative story as to the events that have
transpired.

The trail court found him guilty of murder giving credence to the
testimonies of Patrolman Basilad and Juanita Basalla. Gregorio admitted
upon surrender to Pat. Basilad that he killed Ofremia because she
allegedly promised to kill him through witchcraft.

The Supreme Court ruled that the trial court ruling is not in violation of
Sec 12 Art. III for what is testified is only what Gregorio told the police
why he is surrendering to them.

DOCTRINE: The applicability of the foregoing provision does not seem


to contemplate cases like the present where no written confession was
sought to be presented in evidence as a result of formal custodial
investigation.

FACTS:
(Evidence for the Prosecution until Fact #6) Gregorio Taylaran,
accused, called at thehouse of deceased Ofremia Atup for the
purpose of submitting himself to thelatter for treatment of his
snakebite located at this left foot.
Ofremia, local quack doctor, opened the door for him. Once inside, the
deceased took her “medicine paraphernalia” and started treating
Gregorio.
Then all of a sudden the Gregorio drew his small bolo and stabbed the
deceased several times causing her to fall on the floor dead.
After killing the Ofremia, Gregorio proceeded to the house of the son of
the deceased for the purpose of killing him and his wife but Gregorio
did not accomplish his purpose because the Ofremia’s son refused to
let him enter his house.
After that the Gregorio surrendered himself with his bolo to policeman
Demetrio Basilad who was then on guard at the municipal hall of
Ubay, Bohol.
When asked why he killed Ofremia who was also his grandmother-in-law,
Gregorio answered, ‘because she promised to kill me with a ‘barang’,
hence killed her first.
He then proceeded to the house of his grandmother-in-law, Ofremia who
was a known quack doctor known to cure snake-bites.
Ofremia treated to his wounds. She instructed him to open the wound
using a bolo so that the venom can be drained out.
While opening his snake-bite, he accidentally put out he light of the
kerosene lamp.
This prompted Ofremia to relight the lamp, when she bended down to
reach for the light, Gregorio simultaneously lifted his right hand
(which was holding the bolo) and accidentally hit Ofremia’s right
chest.
Realizing that Ofremia was fatally wouded, he asked for her forgiveness
and after that ran away.
The trial court found Gregorio guilty of the crime charged. Hence, the
present appeal.

ISSUE/s:
WON the trial court erred in convicting him of the crime of murder - NO
WON Gregorio was deprived of his constitutional right under Sec 12 Art
III - NO

RULING: SC affirmed the lower courts decision.

RATIO:
It is extremely difficult to accept here accident version of Gregorio, which
he purveyed without corroboration.
The wounding was with intent to kill is reflected by Gregorio’s statement
that he killed the old woman because she had allegedly promised to
kill him by “barang” or witchcraft, which he gave upon surrendering to
Patrolman Demetrio Basilad.
For Pat. Basilad to testify on what appellant said on this score is thus
perfectly proper, and full credence must be accorded to him, being
obviously an impartial witness.
Gregorio of course denies having made the admission, but in the light of
the other evidence of the prosecution, his denial is not convincing.
As demonstrated earlier, his accident theory of the killing merits not
much credibility from the mere fact that more than one wound was
inflicted which could not have resulted from just one blow.
Repeated blows easily negates any claim of wounding by mere accident.
The fact that he was not allowed to enter the house of Juanita Busalla,
daughter of the deceased, when he went there directly from the old
woman’s house, would show that he appeared, by his behavior or
words, that he was dangerously in an angry mood, which is
indicative of being a deliberate killer rather than a sorrowful and
harmless penitent for a killing he has committed only by accident.
As Juanita also testified, when appellant was already in jail, he told her
that he killed her mother because of witchcraft, corroborating Pat.
Basilad’s testimony.
It is hard to see why the aforementioned witnesses testified on the admission
of Gregorio the way they did unless they were prompted only by the
truth.
If Gregorio had surrendered with an admission of killing the old woman
by accident, as he must have tried to impress upon the authorities if
such was the truth, Pat. Basilad had no reason to give the killing the
graver character than what it really was.
As far as he is concerned, he had no more problem relative to the
solution of the crime, which is the usual cause for police twisting the
truth or other form of excesses when conducting investigations the
desire to solve a crime by all means.
Appellant has also invoked the provision of Article IV, Section 20 of the
Constitution (NOW SECTION 12, ART III) in trying to block the
admission of his declaration to Pat. Basilad that he killed Ofremia
Atup because of her alleged vow to kill him by witchcraft, contending
that the safeguards therefor have not been made available to him.
The applicability of the foregoing provision does not seem to contemplate
cases like the present where no written confession was sought to be
presented in evidence as a result of formal custodial investigation.
What was testified to is only what Gregorio told the police why he is
surrendering to them.
At any rate, even without the admission, the accident version of appellant
is inherently incredible.
PEOPLE v. MARCOS (ELIEL) cousin and resides just acorss the street from the Gonzales’
January 9, 1987 | Alampay, J. | Right to counsel residence

PETITIONER: People of the Philippines


RESPONDENTS: Moises Marcos y De La Rosa

SUMMARY: A ransom note was given to Benito for the amount of


P200,000, in exchange for the liberty of his son Benedict, nine year old
who was kidnapped by 3 men under the pretext of Benito being in the
hospital. Benito sought for help with the NBI but Marcos told him that he
would help recover Benedict and reduce the amount to P20,000. After
they were able to rescue Benedict, Benito informed the NBI to which they
were suspicious as to how they were able to resuce the kid. When Marcos
was invited for questioning, he wrote a sworn affidavit admitting his
actions. He now contends that the sworn affidavit is inadmissible because
it was done without counsel. Hence this petition.

The SC held that the sworn affidavit of Marcos is admissible as evidence


and extrajudicial confession of the crime he committed. The right to
counsel is unavailing in this case, because Marcos was not under police
custody, he was only invited for questioning. Furthermore, his sworn
affidavit was done with an express waiver after being informed of his right.
Other than the affidavit presented as evidence against him, testmonies
were also given credence for his conviction. However, the SC held that
death penalty is wrongly imposed, since he already regretted his actions
and thus, life imprisonmen is proper.

DOCTRINE: The Court in this regard, finds that Marcos’ protestations do


not warrant reversal of the appealed judgment. When Marcos gave his
sworn statement before the NBI agent he was not then under police
custody.

It is significant to consider that Marcos was duly informed of his right to


remain silent. He was warned that any statement he makes may be used
against him and that he is entitled to be assisted by a lawyer of his choice.

FACTS:
Benedict Gonzales, a 9-year old pupil of the St. Martin de Porres
Catholic School in Paombong, Bulacan, while on his way home form
school, was approached by 3 men on board an owner-type jeep. On
the pretext that the boy’s father, Benito Gonzales met an accident,
the men asked Benedict to go with them to the hospital
The unsuspecting Benedict went with the 3 men who brought him to an
isolated hut situated at Baesa, Caloocan City, about 1 km from the
North Diversion Road. At said hut, Benedict was detained for 2
nights and 1 day, guarded during the day by two of the men
After he received a note asking for P200,000 cash as ransom, Benito,
Benedict’s father, sought the help of Marcos. Marcos is Benito’s first
From the NBI, Benito proceeded to the office of Engr. Cesar Gonzales, It is significant to consider that Marcos was duly informed of his right
where Marcos advised Benito to disregard the NBI original plan. He to remain silent. He was warned that any statement he makes may
volunteered to talk with the kidnappers to reduce the ransom money be used
to P20,000 and proposed to raise the amount through a loan from a
friend. Benito fearful for the life of his son and having no ready cash
at the time, readily agreed to Marcos’ proposal
From there, Marcos and Benito proceeded to the store of Romeo Castro
purportedly to secure loan of P20,000. Marcos alone talked to Castro
inside the latter’s store and, about 3 minutes later, invited Benito.
After the usual introductions, Benito believing that Marcos was able
to secure a loan from Castro, promised to pay the same within the
week.
Benito Gonzales reported to the NBI that Marcos was able to get
Benedict for the amount of P20,000. Noting some suspicious
circumstances in Marcos’ story, the NBI conducted further
investigation. The NBI agents questioned not only Benito and
Benedict Gonzales btu also Marcos and Romeo Castro, who
allegedly loaned the amount of P20,000 ransom money
In his sworn statement, voluntaryily given before BI Agent Libit, Marcos
admitted that he, together with Danilo Castro, “Jun” and alias “Peter
Doe,” planned and executed the kidnapping of Benedict
Marcos, however alleges, that he only wrote the ransom note out of fear
that his children will be hurt. He also alleges that the sworn
statement was done without assistance by counsel and is
admissible. Hence this petition.

ISSUE/s:
WoN the sworn affidavit is admissible – YES

RULING: Dismissed and remanded to CFI Manila for further proceedings

RATIO:
Marcos maintains that his extrajudicial confession is inadmissible as
evidence against him because the same was obtained from him
without the assistance of a counsel. He avers that although he
waived his right to counsel, this waiver is without legal effect as such
was made without the assistance of a lawyer, a requisite which
should have been complied.
The Court in this regard, finds that Marcos’ protestations do not
warrant reversal of the appealed judgment. When Marcos gave
his sworn statement before the NBI agent he was not then
under police custody.
He was merely invited for questioning so he can shed light on the
kidnapping of Benedict. He was even allowed to go home after
the investigation. Marcos who is a retired first lieutenant in the
Philippine Constabulary and who had studied up to third year in
Mechanical engineering, admitted having voluntarily given his
sworn statement to the NBI
against him and that he is entitled to be assisted by a lawyer of his choice. imprisonment
Furthermore, in the sworn statement of Marcos, his waiver of his
right to counsel is clearly expressed. Testifying before the trial
court, Marcos expressly acknowledged that he voluntarily
signed his sworn statement
Considering all the foregoing circumstances, the Court is of the view that
Marcos’ admissions, voluntarily made and confirmed by him in open
court during his trial, render worthless the challenge now interporsed
by him to the admissibility of Marcos’ sworn statement
Apart from the extrajudicial statements of Marcos, the other evidence
submitted by the prosecution include the testimony of Benedict
Gonzales, a 9 year old kidnap victim who attested to and confirmed
the fact that it was Marcos, as his “Lolo Nito” who had taken him
from the plaee of detention in Baesa and brought him back home to
his parents.
All the above mentioned evidence, separate and independent from the
extrajudicial statement, executed by Marcos, are likewise proof
establishing beyond reasonable doubt his guilt
Marcos claims that his cooperation was not given voluntarily and
therefore he should not be regarded as a principla for even assuming
that he participated in the commission of the crime, his participation
was merely that of an accomplice
We cannot give credence to the claim of Marcos that he was coerced into
writing the ransomnote. There is no averment nor is there even
mention of any such alleged coercion exerted on him when marcos
executed his sworn statement, wherein he described the role he
played in the kidnapping of Benedict Gonzales
The Court agrees with the conclusion arrived at by the trial court that
Marcos’ participation in the said felony was as a principal. It was
Marcos who informed his co-accused that Benito Gonzales, father of
Benedict, is among the richest residents of Bulacan.
It was undisputed that Marcos received from Benito Gonzales the
amount of P20,000 which was intended to serve as reimbursement
for the money allegedly to serve as reimbursement for the money
allegedly borrowed and which Benito was made to believe was paid
for the release of Benedict
Finally, Marcos contends that the death penalty should not have been
imposed on him because the amount of the ransom money involved
is small. He maintains that the death penalty constitutes a cruel and
unusual punishment, disallowed by Sec 21, Art 4 of the 1973
Constitution
The Court’s impressions it that when marcos began to realize the extent
of the grief and torment being undergone by the family of Benedict,
he must have even at that tiem, already regretted his felonious act
and thus he voluntarily returned the kindapped boy to his parents
although full payment of the P200,000, being demanded in the
ransom note was not given by the boy’s family
Thus, we find the extreme penalty of death imposed on Marcos to be
inappropriate. In the view of the Court, udner the given
circumstances in the case, the penalty that should be imposed
should be reduced to life
PEOPLE v. RAPEZA (JP) Accused Rapeza was expressly advised that he was being investigated
for the death of Libas and Ganzon.
April 4, 2007 | Tinga, J. | Right to Counsel, etc.

PETITIONER: People of the Philippines


RESPONDENTS: Jerry Rapeza y Francisco

SUMMARY: Upon a report the police investigated the murder of a woman


and a man inside one house. They were eventually led to accused herin
Rapeza. Rapeza then made an extrajudicial confession of the crime and
implicated his co-accused Mike. They were convitcted of two crimes of
murder. CA affirmed.

The SC held that such extrajudicial confession was inadmissible. Rapeza


was not informed of his rights at the start of the custodial investigation.
Although it was recited to him, he did not understand it. He was likewise
not assisted by a counsel of his choice. His counsel also merely notarized
statements and did not assist him in the proceedings. He is acquitted.

DOCTRINE: The Court has consistently held that an extrajudicial


confession, to be admissible, must conform to the following requisites: 1)
the confession must be voluntary; 2) the confession must be made with
the assistance of a competent and independent counsel, preferably of
the confessant’s choice; 3) the confession must be express; and 4) the
confession must be in writing.

FACTS:
An unidentified woman went to the Culion Municipal Station and reported
a killing that had taken place in Sitio Cawa-cawa, Palawan.
The officer in charge sent to the victim’s house an investigating team.
There they saw two bloodied bodies, that of a woman lying on the
floor of the sala (Priscilla Libas) and that of a man inside the
bedroom (Cesar Ganzon).
Upon information supplied by a certain Mr. Dela Cruz that accused
Rapeza wanted to confess the crimes, the police officers set out to
look for accused Rapeza.
Rapeza was found fishing in Asinan Island and he was invited for questioning
by the police officers. Accused was then brought to the police station.
SPO2 Gapas (one of the police officers) requested Kagawad Arnel
Alcantara to provide accused with a lawyer.
The following day, accused Rapeza was brought to the house of Atty.
Roberto Reyes, the only available lawyer in the municipality.
The typewriter at the police station was out of order at that time and Atty.
Reyes could not go to the police station as he was suffering from
rheumatism.
At the house of Atty. Reyes, SPO2 Cuizon and an interpreter, SPO2
Gapas proceeded with the custodial investigation of appellant who
was assisted by Atty. Reyes.
Per Sinumpaang Salaysay that the accused Rapeza executed, he was the trial court.
informed of his constitutional rights (right to be silent, counsel, etc.). ISSUE/s:
Accused Rapeza then narrated the facts of the crime: WoN Rapeza’s extrajudicial confession is admissible in evidence to
warrant the verdict of guilt – NO
He was drinking with Mike Regino (co-accused but at large at
the moment of the case) and Mike said that he had illwill
against the two victims, Libas and Ganzon. Mike wanted to
kill them.

Rapeza said he did not want to partake in the crime but Mike
threw a fit and said “pamangkin mo pa naman ako.”

4pm that day, they went to the house of the victims. Mike slit
the neck of Libas and Rapeza went upstairs and stabbed
Ganzon.
An interpreter was provided accused Rapeza as he was not well versed
in Tagalog being a native of Samar. As he is illiterate, appellant
Rapeza affixed only his thumbmark on the statement above his
printed name.
Bonifacio Abad, the interpreter, and Atty. Reyes, as the assisting counsel,
also signed the statement. Atty. Reyes signed again as the notary
public who notarized the statement.
Thereafter, a complaint for multiple murder was filed against appellant
Regino, and Regino was likewise arrested.
Testifying in his defense, accused Rapeza presented a different story
during the trial. Rapeza testified that he did not know the victims and
that he had nothing to do with their deaths. He was a native of
Samar and he did not know how to read or write as he never
attended school. He claims that he was mauled by the police while
he was helping load the bodies of the victims in a banca.
Rapeza mainly contends that the extrajudicial confession upon which the
trial court placed heavy emphasis to find him guilty suffers from
constitutional infirmity as it was extracted in violation of the due
process guidelines. Specifically, he claims that he affixed his
thumbmark through violence and intimidation.
He stresses that he was not informed of his rights during the time of his
detention when he was already considered a suspect as the police
had already received information of his alleged involvement in the
crimes.
Neither did a competent and independent counsel assist him from the
time he was detained until trial began. Assuming Atty. Reyes was
indeed designated as counsel to assist appellant for purposes of the
custodial investigation, said lawyer, however was not his personal
choice.
On the basis of accused Rapeza’s extrajudicial confession (Ratio 11), the
RTC found him guilty of two crimes of murder. The Court of Appeals
upheld
communicated to him, as well as the contents of his
RULING: Accused Rapeza is acquitted. alleged confession.
The prosecution underscores the presence of an interpreter in
RATIO: the person of Abad to buttress its claim that appellant was
A confession is admissible in evidence if it is satisfactorily shown to have informed of his rights in the dialect known to him.
been obtained within the limits imposed by the 1987 Constitution However, the presence of an interpreter during the interrogation
(Sec. 12, Art. 3). was not sufficiently established. Although the confession
The Court has consistently held that an extrajudicial confession, to be bears the signature of Abad, it is uncertain whether he was
admissible, must conform to the following requisites: 1) the indeed present to assist appellant in making the alleged
confession must be voluntary; 2) the confession must be made with confession.
the assistance of a competent and independent counsel, preferably SPO2 Gapas’ testimony as to what was translated to appellant
of the confessant’s choice; 3) the confession must be express; and Rapeza and the latter’s responses thereto were not of his
4) the confession must be in writing. personal knowledge. Therefore, without the testimony of
If all the foregoing requisites are met, the confession constitutes Abad (the interpreter), it cannot be said with certainty that
evidence of a high order because it is presumed that no person of appellant was informed of his rights and that he understood
normal mind will knowingly and deliberately confess to a crime them.
unless prompted by truth and conscience. Confession was not made with the assistance of competent and
Appellant Rapeza was not informed of his constitutional rights in independent counsel of appellant’s choice.
custodial investigation. Settled is the rule that the moment a police officer tries to elicit
Appellant Rapeza did not voluntarily surrender to the police but admissions or confessions or even plain information from a
was “invited” by SPO2 Gapas to the police station. There he suspect, the latter should, at that juncture, be assisted by
was detained from 11 o’clock in the morning of 22 October counsel, unless he waives this right in writing and in the
1995 up to the morning of 23 October 1995 before his presence of counsel. Appellant Rapeza did not make any
extrajudicial statement was allegedly taken. such waiver.
At this juncture, appellant Rapeza should have been Assuming that Atty. Reyes did assist appellant, still there would
informed of his constitutional rights as he was already be grave doubts as to his competence and independence as
considered a suspect, contrary to the finding of the trial appellant Rapeza’s counsel for purposes of the custodial
court that the mandatory constitutional guidelines only investigation.
attached when the investigators started to propound 0 Competent counsel standard: The competent or
questions to appellant on 23 October 1995 in the house of independent lawyer so engaged should be present
Atty. Reyes. from the beginning to end, i.e., at all stages of the
Even supposing that the custodial investigation started only on interview, counseling or advising caution reasonably
23 October 1995, a review of the records reveals that the at every turn of the investigation, and stopping the
taking of appellant Rapeza’s confession was flawed interrogation once in a while either to give advice to
nonetheless. the accused that he may either continue, choose to
It is stated in the alleged confession that prior to questioning SPO2 remain silent or terminate the interview.
Gapas had informed appellant Rapeza in Tagalog of his rights. Although Atty. Reyes signed the confession as Rapeza;s counsel
However these is no showing that had actually understood his and he himself notarized the statement, there is no evidence
rights. He was not even informed that he may waive such rights on how he assisted appellant. The confession itself and the
only in writing and in the presence of counsel. testimonies of SPO2 Gapas and SPO2 Cuizon bear no
Since comprehension is the objective, the degree of explanation indication that Atty. Reyes had explained to appellant Rapeza
required will necessarily depend on the education, his rights.
intelligence, and other relevant personal circumstances of It appears that his, Atty. Reyes’, participation in the proceeding was
the person undergoing investigation. confined to the notarization of appellant Rapeza’s confession.
In this case, it was established that at the time of the investigation Such participation is not the kind of legal assistance that should
appellant was illiterate and was not well versed in Tagalog. This be accorded to appellant in legal contemplation.
fact should engender a higher degree of scrutiny in determining Furthermore, Atty. Reyes was not appellant Rapeza’s counsel of choice
whether he understood his rights as allegedly but was picked out by the police officers allegedly through the
barangay officials. Appellant’s failure to interpose any objection to having
Atty. Reyes as his counsel cannot be taken as consent under the prevailing
circumstances.
PEOPLE v. JUDGE AYSON (HENRY) Although he planned on paying back, he was prevented by
July 7, 1989 | Naravasa, J. | Rights of a person in a custodial shame
interrogation He was still willing to settle the obligation and to pay on a staggered

PETITIONER: People of the Philippines


RESPONDENTS: Hon. Judge Ruben Ayson, RTC Br 6 (Baguio), and
Felipe
Ramos

SUMMARY: This is a case involving respondent Ramos who was an


employee of PAL ticketing office in Baguio. He was charged with estafa.
But prior to that, an investigation by PAL was already made, to which
Ramos has confessed and admitted his failure to remit the ticket sales
amounting to 76k. 2 months after the investigation a case was filed
against him for estafa, to which the prosecutors offered the evidence
obtained during the PAL investigation with regards his confession
(Exhibits A and K), to which the respondent RTC Judge Ayson rendered
as inadmissible in evidence, as it is within the ambit of the constitutional
provision, protecting the right of an accused. He claims that the accused
Ramos was not in the presence of counsel when he confessed, and
therefore such evidence is inadmissible. Such ruling did not satisfy the
prosecution and thus the case was appealed to the SC.

The SC ruled that Exhibits A and K are admissible in evidence, they


being not within the ambit of the constitutional right in custodial
interrogation. Clearly, the confession was made in an administrative
investigation, and not in custody. Hence, his confession, even if without
counsel, is rendered admissible. There is no violation of his rights as
indicated in the constitution insofar as custodial interrogations are
concerned.

DOCTRINE: Not every statement made to the police by a person


involved in some crime is within the scope of the constitutional protection.
If not made under custodial interrogation or under investigation of the
commission of an offense, the statement is not protected.

FACTS:
Resondent Felipe was a ticket freight clerk of PAL, assigned at its Baguio
City Station.
Being involved in irregularities in the sale of tickets, he was investigated
and prior to such investigation, he was given a note that read
something about him being willing to settle the irregularities charged
against him in the amount of Php 76,000.00
The investigation pushed through, which had the following findings (from
Ramos’ answers to the questions)
He did not make disclosure of the tickets in the Audit Team’s
findings
Proceeds were misused by him
basis (amount to be known in next investigation)
2 months later, an information was filed against respondent Ramos,
charging him with estafa.
In the information it was alleged that he did not remit the ticket sales
amounting to 76k despite repeated demands.
On arraignment he pleaded “not guilty”.
The prosecutors of the case made a written offer of evidence which
included the statement of Ramos taken at the PAL Baguio Ticket
Office investigation (the stuff he said sa fact#3), labeled as Exhibit A,
and his handwritten admission, marked as Exhibit K.
Respondent Ramos’ counsel filed an objection regarding Exhibit A,
claiming that it was taken without Ramos being represented by a
lawyer and therefore inadmissible in evidence.
Respondent Judge Ayson rejected Exhibit A and K, and declared it
inadmissible in evidence, and held that it was taken in violation of the
constitutional right to be reminded of the right to remain silent and to
have counsel.
Prosecutors filed MR, which was denied. The RTC used as basis the case of
Morales v. Enrile, to the effect that “in custodial investigations the right to
counsel may be waived but the waiver shall not be valid unless made
with the assistance of counsel” and that such right may only be waived
in writing and in the presence of counsel. Judge Ayson even justified
further, indicating that even if Ramos was not detained at the time of the
administrative investigation, is not an exception that will render the
confession admissible
Hence, a petition by the prosecutors, to which the SC issued a TRO
enjoining respondents from further rproceeding with the trial and hearing
of the case.

ISSUE/s:
WoN Exhibits A and K are admissible in evidence – YES

RULING: SC granted the petition.

RATIO:
Sol Gen was ordered to coment, to which it expressed its stand that
Exhibits A and K be admissible in evidence, siding obviously with the
prosecution.
At the core of the controversy is Sec 20, Art IV of the 1973 Constitution,
which respondent Judge Ayuson has given a construction which is
disputed by the prosecution. Such provision reads:
“No person shall be compelled to be a witness against himself.
Any person under investigation for the commission of an
offense shall have the right to remain silent and to counsel,
and to be informed of such right. No force, violence, threat,
intimidation, or any other meands which vitiates the free will
shall be used against him. Any confession obtained in the
violation of this section shal be inadmissible in evidence”
Basically there are 2 rights being dealt with in this section:
Right against self incrimination – right of a person not to be
compelled to be a witness against himself (elaborated in Sec
17, Art III of 1987 Constitution)
Right of a person in custodial interrogation – right of every
suspect under investigation for the commission of an offense
(elaborated in Sec 12, Art III of 1987 Constitution)
Insofar as the right against self-incrimination is concerned, the SC
mentioned that this right is accorded to every person who gives
evidence, whether voluntary or under compulsion of subpoena, in
any civil, criminal, or administrative proceeding. The right basically is,
the right NOT to be compelled to be a witness against oneself. This
right is not self-executing or automatically operational. It must be
calaimed. Otherwise, the protection does not come into play.
On the right in custodial interrogation (relevant topic AF): this
applies to persons under investigation for the commission of an
uffence.
This provision granting explicit rights to persons under investigation for
an offense was not in the 1935 Constitution. It is derived pa from the
case of Miranda v. Arizona, which is described as an earthquake in
the world of law enforcement (extra nalang tong earthquake thing
haha sasabihin ko to!!!)
Sec 20 (1973 consti) states that any person under investigation for the
commission of an offense:
Shall have the right to remain silent and to counsel, and to be
informed of such right
No force, violence, threat, intimidation, or any other means which
vitiates the free will shall be used against him; and
Any confession obtained in violation of these rights shall be
inadmissible in evidence
In the Miranda case, the procedural safeguards for a person in custody
and the interrogation conducted were summarizerd. Such had an
objective to prohibit incommunicado interrogation of individuals in a
police-dominated atmosphere, resulting in self-incriminating
statement without full warnings of constitutional rights.
The rights specified only exist in custodial interrogations, or in-custody
interrogation of accused persons. And as defined by jurisprudence,
custodial interrogation is the questioning initiated by law
enforcement officers after a person has been taken into custody
or otherwise deprived of his freedom of action in any significant
way.
Not every statement made to the police by a person involved in some
crime is within the scope of the constitutional protection. If not made
under custodial interrogation or under investigation of the
commission of an offense, the statement is not protected.
Thus, in one case, where a person went to a police precinct and before
any sort of investigation he declared that he was giving himself up for
the crime he committed, the SC ruled that such statement was
admissible, compliance with the constitutional procedure on custodial
interrogation does not cover such circumstance. (Peo v. Taylaran)
PEOPLE v. MARRA (DANNAH) the same was returned unserved, hence trial proceeded with
September 20, 1994 | Paras, J. | Custodial Investigation regard to herein accused-appellan Samuel Marra alone.

PETITIONER: People of the Philippines


RESPONDENTS: Samuel Marra, Allan Tan, Peter Doe, Paul Doe et al.

SUMMARY: Tandoc and Din were conversing in front of Lucky Hotel


when a man passed by and gave them the bad sign (finuckyou sila,
ganun). They approached the guy and asked for an explanation. Two
other men approached them and Tandoc told them they just wanted an
explanation. Din was surprised when Tandoc suddenly slapped one of the
two men. A brawl then ensued, and the three men ran away.

While walking back to the hotel they saw a couple of men running
towards them, so they went inside sensing danger. When they thought
there was no more danger, Tandoc went out because he was supposed
to head home. However, Din saw a person in a security guard’s uniform
shoot Tandoc. Tandoc was brought to the hospital but was pronounced
dead after an hour.

SPO3 De Vera received a report of a shooting which he investigated. Din


told him that the suspect was wearing a security guard’s uniform. They
found that suspect to be Marra, security guard of Linda’s Ihaw Ihaw.
Marra told them the firearm issued to him was in his house. Upon arrival,
De Vera saw that it had 4 live bullets and one empty shell, and De Vera
smelled gunpowder. De Vera, asked him point blank why he shot Tandoc.
Initially Marra denied but confessed afterwards. The trial court convicted
him of murder.

The Court ruled that Marra’s confession was admissible as evidence, as it


was done prior to custodial investigation. There was no coercion whatsoever
to compel him to make such a statement. Indeed, he could have refused to
answer questions from the very start when the policemen requested that they
all go to his residence.

DOCTRINE: The declaration of an accused acknowledging his guilt of the


offense charged, or of any offense necessarily included therein may be
given in evidence against him and, in certain circumstances, this
admission may be considered as part of the res gestae.

FACTS:
An information nwas filed with the RTC of Dagupan where Samuel
Marra, John Doe, Peter doe et al. were charged with the crime of
murder for the fatal shooting of Nelson Tandoc.
An amended information was filed wherein Allan Tan was indicted as an
accused instead of John Doe. A warrant was issued against Tan but
Jimmy Din positively identified Marra as the triggerman in the killing of Nelson
Tandoc.
Din recounted that at around 2AM on March 7 1992, he and his friend Tandoc were
conversing in front of Lucky Hotel which was owned by Don’s father.
A man passed by on the opposite side of the street who made a dirty sign with
his finger and Din informed Tandoc thereof.
Said man repeated the act and called them by waving his hands.
Infuriated, they followed the man until he stopped in front of Dunkin Donuts, and
they demanded an explanation.
Two men then arrived and one of them asked what was going on. Tandoc told
them they just wanted an explanation.
Din was surprised when Tandoc unexpectedly slapped one of the two men.
A brawl then ensued. After the fisticuffs, the three opponents ran away.
Tandoc and Din decided to walk back to the hotel. When they were about to
enter, they noticed the men with whom they had a brawl with running towards
them.
Sensing danger, they ran inside the annex building of the hote, entered a room
and waited until they felt that the situation had normalized
After 10-15 mins, they left the room. Having decided to go home, Tandoc opened
the sliding door
All of a sudden, Din saw Marra (wearing a security guard’s uniform) shoot
Tandoc with a revolver.
Tandoc was brought to the Villaflor Hospital and died one hour later.
SPO3 de Vera of the Dagupan Police Station received a report about a shooting
incident. Din informed them that he could recognized who shot Tandoc and
that he was wearing a security guard uniform.
The security guard of a bus station told them that he saw the guard of Linda’s
Ihaw-Ihaw and a couple of men chasing two persons
They approached the security guard who was then eating in a carinderia.
After a series of questions, they learned that he was Samuel Marra, that his tour
of duty was from 7:00 P.M. of a preceding day to 6:00 A.M. the following day,
that he was still on duty at around 2:30 in the morning of March 7, 1992, and
that the firearm issued to him was in his house.
Upon their request to see the firearm, they proceeded to Marra’s residence at
Interior Nueva Street.
Marra showed his .38 caliber and De Vera found 4 live bullets and one spent
shell. Smelling gunpowder, De Vera inquired when Marra last fired.
Abruptly, De Vera asked him point-blank why he shot Tandoc. Marra at first denied
the accusation but when informed that someone saw him do it, he said that he
did so in self-defense, firing at the victim only of the incident.
However, persistent efforts on the part of the policemen to thereafter On the admissibility of Marra’s confession:
locate said bladed weapon proved futile. Marra also admitted that
prior to the incident, he chased the victim and Din. The officers then
took Marra to the police station where he was detained.
Marra’s version was that at 5AM he was approached by policemen and
that when he showed his gun at his residence, there were 5 live
bullets, not four and one empty shell. He also claimed he had never
met Din.
The argument of the defense was that it was not possible for Din to see
Marra as his vision was obstructed by the door, and he was 45
meters away from the shooting.

ISSUE/s:
WoN Marra’s confession to De Vera is admissible as evidence – YES
WoN Marra was under custodial investigation when he confessed – NO

RULING: Marra guilty. Judgment affirmed.

RATIO:
Din was indeed in a position to know the identity of the assailant. Firstly, Din
knew for a fact that the persons he and Tandoc fought with near the
Dunkin Donuts store were the same men who chased them while they
were on their way back to the hotel because he was able to take a good
look at them.
During the chase, he naturally turned around to look at the men who
were running after them and who were at that time in front of the
Balingit Trading store which was well-lighted.
Secondly, we do not agree with appellant that the door blocked the view
of Din. Said door, partly made of plywood, had a spring hinge which
makes it possible for the door to close by itself.
However, at that time the spring hinge had been weakened by long and
constant use such that it would take some time for it to close the
door, thereby allowing Din sufficient opportunity to have an
unobstructed view of the scene outside.
Thirdly, Din was quite near the Tandoc and Marra, which proximity
enabled him to clearly see what really happened. He thus readily
perceived the actual shooting at the time when Tandoc pushed the
door open.
Lastly, the place was brightly illuminated by a 20-watt fluorescent bulb
installed on the outside wall in front of the hotel. Marra was only
about three meters away therefrom. Such physical conditions would
undeniably afford a clear view from inside the hotel of the immediate
area outside and in front of the same where the incident took place.
Article III Sec 12:
“Any person under investigation for the commission of an offense shall have the
right to be informed of his right to remain silent and to have competent and
independent counsel preferably of his own choice.”
Custodial investigation involves any questioning initiated by law enforcement
officers after a person has been taken into custody or otherwise deprived of
his freedom of action in any significant way.
It is only after the investigation ceases to be a general inquiry into an unsolved crime
and begins to focus on a particular suspect, the suspect is taken into custody,
and the police carries out a process of interrogations that lends itself to eliciting
incriminating statements that the rule begins to operate.
In the case at hand, Marra was not under custodial investigation when he made
the admission. There was no coercion whatsoever to compel him to make
such a statement. Indeed, he could have refused to answer questions from
the very start when the policemen requested that they all go to his residence.
The police inquiry had not yet reached a level wherein they considered him as a
particular suspect. They were just probing into a number of possibilities,
having been merely informed that the suspect was wearing what could be a
security guard’s uniform.
The declaration of an accused acknowledging his guilt of the offense charged, or
of any offense necessarily included therein may be given in evidence against
him and, in certain circumstances, this admission may be considered as part
of the res gestae.
The rule is that, any person, otherwise competent as a witness, who heard the
confession, is competent to testify as to the substance of what he heard if he
heard and understood all of it. An oral confession need not be repeated
verbatim, but in such a case it must be given in substance.
PEOPLE v. MAQUEDA (ARIELLE) However, in the early morning of August 27, 1991, in the sanctity of
March 22, 1995 | Davide Jr., J. | When the Rights Become Available their

PETITIONER: People of the Philippines


RESPONDENTS: Hector Maqueda and Rene Salvamante

SUMMARY: Salvamante and Maqueda allegedly killed William Barker


and inflicted serious physical injuries upon Teresita Barker, in the course
of robbing the spouses’ house in Benguet. The two accused hit both the
spouses with a lead pipe. Since Salvamante remained at large, only
Maqueda was arrested. Maqueda filed an application for bail, and stated
that he was willing and volunteering to be a State witness, him appearing
to be the least guilty in the case. Maqueda signed a Sinumpaang
Salaysay where he narrated his participation in the crime at the Barker
house.

The trial court found Maqueda guilty beyond reasonable doubt of the
crime of robbery with homicide and serious physical injuries. The trial
court held that the admissibility of the Sinumpaang Salaysay should not
be tested under Sec 12 Art III, but on the voluntariness of its execution.
Since voluntariness is presumed, Maqueda had the burden of proving
otherwise, which he failed to do. Hence, the Sinumpaang Salaysay was
admissible against him.

The SC ruled otherwise. Maqueda signed the Sinumpaang Salaysay


without the assistance of a counsel, thus inadmissible. The Court did not
agree with the trial court with its sweeping view that after filing a criminal
complaint, an accused no longer has the right to remain silent and to
counsel, but has the right to refuse to be a witness and not to have any
prejudice to him by such refusal. If that were so, then there would be a
hiatus in the criminal justice process where an accused is deprived of his
constitutional rights to remain silent and to counsel and to be informed of
such rights. It was, therefore, wrong for the trial court to hold that Section
12(1) is strictly limited to custodial investigation and that it does not apply
to a person against whom a criminal complaint or information has already
been filed.

DOCTRINE: The right to remain silent and to counsel and to be informed


are available to a person at any time before arraignment whenever he is
investigated for the commission of an offense.

FACTS:
Britisher Horace William Barker, a consultant of the World Bank, and his
Filipino wife, Teresita Mendoza, chose to live in Benguet, where it
was much peaceful.
own home, Horace was brutally slain and Teresita badly battered with lead
pipes on the occasion of robbery.
Sufficient prima facie evidence pointed to Rene Salvamante, the victims’ former
houseboy, as one of the perpetrators.
As to Rene’s co-conspirator, the prosecution initially included one Richard Malig y
Severino in the information for robbery with homicide and serious physical
injuries.
Only Malig was arrested. Prior to his arraignment, the prosecution filed a motion
to amend the information to implead as co-accused Hector Maqueda alias
Putol (loool heeheh) because the evaluation of the evidence subsequently
submitted established his complicity in the crime.
The motion to drop Malig was granted and warrants for the arrest of accused
Salvamante and Maqueda were issued. Maqueda was subsequently arrested
and then he filed an application for bail.
He stated that he is willing and volunteering to be a State witness in the case, it
appearing that he is the least guilty. The prosecution filed an Amended
Information with only Salvamante and Maqueda as the accused.
Since Salvamante continues to elude arrest and has remained at large, trial
proceeded against Maqueda only after he entered a plea of not guilty. The trial
court found Maqueda guilty beyond reasonable doubt of the crime of robbery
with homicide and serious physical injuries.
The prosecution detailed the summary of the event. They testified that the
spouses went to their bedroom after Teresita had checked the main doors of
their house to see if they had been locked.
6am the following day, the Norie, the househelp of the Barkers got up, opened the
door to the garage and then to the bathroom to wash her face. When she
opened the door to the toiled, she saw Rene Salvamante.
She knew Salvamante very well because he and his sister were former
househelps of the Barkers whom she and Julieta had replaced.
Salvamante suddenly strangled her. while she was fighting back, Norie happened
to see a fair-complexioned, tall man with a high-bridged nose at Salvamante’s
side, whom she identified as Maqueda.
After Norie broke free from Salvamante, she shouted for help but Salvamante
chased her back inside the house. Julieta was awakened by the shouts and
got out of bed and upon opening the door of her room, saw a man with his
right hand brandishing a lead pipe standing two meters from her. At trial, she
pointed to Maqueda as the man she saw.
Teresita Mendoza was awakened and went down to the dining room, where she
saw Salvamante and a companion. Suddenly, the two rushed towards her and
beat her up with lead pipes. Salvamante also hit Norie with the lead pipe on
her back and she fell to the floor.
That same day, Mike Tabayan and Mark Pacio were resting in a waiting shed which is
a kilometer away from the house of the Barkers. When they saw two men reached
the shed, he and Mark noticed that one of the men had an amputated left hand
and a right hand with a missing thumb and index finger.
The taller man asked Mike and Mark whether the road they were following
would lead to Naguilian, La Union. Mike replied that it did not. The Maqueda of his constitutional rights beofore the latter was investigated and
two men boarded a jeep bound for Baguio. that he voluntarily and freely gave his Sinumpaang Salaysay (referred to as
Mike identified through a picture the shorter man as Salvamante and extrajudicial
pointed to Maqueda as the taller man.
Julieta and Norie saw the Barkers bathed in their own blood and rushed
to the place of Janet Albon to seek help. They then called the police.
Security guards of the Baguio College Foundation arrived. The team
conducted an initial investigation only because it found out that the
scene of the crime was within the jurisdiction of the Tuba Police
Station which was difficult to get in touch with that time.
BCF Security Officer Glen Enriquez conducted his own investigation. He
went to the master’s bedroom and saw several pieces of jewelry
scattered on the floor and an empty cabinet. He noticed footprints at
the back of the house, and observed that the grass below it was
parted as if someone had passed through and created a trail toward
the Asin road of Tuba.
The police team came to the hospital bed of Mrs. Barker, showed her
pictures of several persons, and asked her to identify the persons who
had assaulted her. She pointed to a person who turned out to be Richard
Malig. The doctor told the team that it was improper for them to conduct
it without first consulting him since Mrs. Barker had not yet fully
recovered consciousness.
Ray Dean Salvosa, Exec VP of BCF, ordered Enriquez to go to
Guinyangan, Quezon, to coordinate with the police in determining the
whereabouts of Salvamante. He was able to obtain information from
the barangay captain that he saw Salvamante together with a certain
Putol, but they already left the house.
Requeron’s daughter called Enriquez to inform im that Putol, or
Maqueda, has been arrested in Guinyangan and was brought to the
provincial jail.
SPO3 Armando Molleno got Maqueda’s statement and he did so.
According to him, he informed Maqueda of his rights under the
Constitution and Maqueda signed a Sinumpaang Salaysay where he
narrated his participation in the crime at the Barker house.
Maqueda filed a Motion to Grant Bail and stated that he is willing and
volunteering to be a State witness because he is the least guilty.
Prosecutor Zarate talked to Maqueda regarding the statement, and
asked him if he was in the company of Salvamante in entering the
house of Barkers.
Zarate told Maqueda that he would oppose the motion for bail since he
was the only accused in the trial. Maqueda narrated to Salvosa that
Salvamante brought him to Baguio to find a job as a peanut vendor
and Salvamante brought him to the Barker house. Salvamante
revealed to him that his real purpose was to rob the Barkers.
The prosecution rebitted Maqueda’s testimony by presenting Fredesminda
Castrence and SPO3 Armando Molleno. They testified that it was
impossible for Castrence to hire Maqueda on July 5 1991 when her
business started only on August 30. MOlleno declared that he informed
confession). party as to a relevant fact may be given in evidence against him. A
Although the trial court had doubts on the identification of Maqueda confession is the declaration of an accused acknowledging his guilt
by prosecution witnesses, it decreed a conviction based on the of the offense charged.
confession and the proof of corpus delicti as well as
circumstantial evidence.
Circumstances shown by the prosecution which tend to show the
guilt of the
accused according to the trial court:
Counsel did not offer any objection regarding the fact that despite being
handicapped, Maqueda could still grip a lead pipe;
Maqueda’s presence in the vicinity of the crime right after the incident in
the company of Salvamante;
He is familiar with Salvamante as they come from the same town;
His Motion to Grant Bail contains the statement that he is willing to be a
State Witness…
Maqueda’s brief only consisted of an alibi which stated that he was
not in Benguet but in Muntinlupa when the crime was
committed. He also pointed the failure of the witnesses for the
prosecution to identify him. Maqueda appealed his conviction to
the SC.

ISSUE/s:
WoN Maqueda is guilty beyond reasonable doubt of the crime charged
– YES
WoN the rights under Section 12 Art III of the Constitution may still
be invoked even after the filing of a criminal complaint – YES

RULING: The instant appeal is DISMISSED and the appealed decision


of the RTC is AFFIRMED in toto.

RATIO:
There is no merit in the appeal. The defense of alibi is
unconvincing. The accused’s arguments which stress the
incredibility of the testimonies of Mrs. Barker and the
househelps identifying Maqueda are misdirected and misplaced.
The trial court ruled that Mrs. Barker and the housemaids were not
able to positively identify Maqueda. The trial court based his
conviction on his extrajudicial confession and the proof of
corpus delicti, as well as on circumstantial evidence.
The trial court made a distinction between an extrajudicial
confession – the Sinumpaang Salaysay – and an extrajudicial
admission – the verbal admissions to Prosecutor Zarate and
Ray Dean Salvosa.
A perusal of the Sinumpaan Salaysay fails to convince the Court
that it is an extrajudicial confession. It is only an extrajudicial
admission. An admission is the act, declaration or omission of
In a confession there is an acknowledgment of guilt. The term admission Custodial interrogation means questioning initiated by law
is usually applied in criminal cases to statements of fact by the enforcement officers after a person has been taken into
accused which do not directly involve an acknowledgment of his guilt custody or otherwise deprived
or of the criminal intent to commit the offense.
Under Sec 3 of Rule 133, an extrajudicial confession made by the
accused is not sufficient for conviction unless corroborated by
evidence of corpus delicti.
The trial court admitted the Sinumpaang Salaysay of Maqueda
although it was taken without the assistance of counsel
because it was of the opinion that since an inormation had
already been filed in court against him and was arrested, the
Sinumpaang Salaysay was not taken during custodial
investigation.
The trial court held that the admissibility of the Sinumpaang
Salaysay should not be tested under Sec 12 Art III, but on the
voluntariness of its execution. Since voluntariness is presumed,
Maqueda had the burden of proving otherwise, which he failed
to do. Hence, the Sinumpaang Salaysay was admissible against
him.
As to the admission made by Maqueda to Prosecutor Zarate, the trial
court admitted their testimony only to prove the tenor of their
conversation but not to prove the truth of the admission because
such testimony was objected to as hearsay.
We cannot agree with the trial court with its sweeping view that after
such filing, an accused no longer has the right to remain silent and to
counsel, but has the right to refuse to be a witness and not to have
any prejudice to him by such refusal.
If that were so, then there would be a hiatus in the criminal justice
process where an accused is deprived of his constitutional rights to
remain silent and to counsel and to be informed of such rights.
The exercise of the rights to remain silent and to counsel and to be
informed under Section 12(1) are not confined to that period
prior to the filing of a criminal complaint or information but are
available at that state when a person is under investigation for
the commission of an offense.
In Miranda v. Arizona, the Court explicithly stated that the prosecution
may not use statements, whether exculpatory or inculpatory,
stemming from custodial interrogation of the defendant unless it
demonstrates the use of procedural safeguards effective to secure
the privilege against self-incrimination.
of his freedom of action in any significant way.
In Galman v. Pamaran, the Court held that the fact that the framers of our
Constitution did not choose to use the term “custodial” by having it
inserted between the words “under” and “investigation,” goes to prove that
they did not adopt in toto the entire fabric of the Miranda doctrine.
Clearly, the second paragraph of Section 20 has even broadened the
application of Miranda by making it applicable to the investigation for the
commission of an offense of a person not in custody.
The second paragraph of Section 20 changed the rule that the rights of the
accused only begin upon arraignment. In Morales v. Enrile, the Court held
that at the time a person is arrested, it shall be the duty of the
arresting officer to inform him of the reason for the arrest and he
must be shown the warrant of arrest, if any.
Moreover, he must be informed of his constitutional rights to remain silent
and to counsel, and that any statement he might make could be used
against him. The person arrest shall also have the right to communicate
with his lawyer, a relative, or anyone he choose by the most expedient
means.
No custodial investigation shall be conducted unless it be in the
presence of counsel engaged by the person arrested, by any person
on his behalf, or appointed by the court upon petition.
The arresting officer is required to inform the person to be arrested and
show him the warrant of arrest. This means that a case had been filed
against him in a court of either preliminary or original jurisdiction.
From the foregoing, it is clear that the right to remain silent and to
counsel and to be informed are available to a person at any time
before arraignment whenever he is investigated for the commission
of an offense.
Thus, Section 12(2) provides that in all criminal prosecutions, the accused shall
enjoy the right to be heard by himself and counsel. In People v. Holgado, the
Court held that even the most intelligent man may have no skill in the science
of th3e law, and he may be convicted not because he is guilty but because he
does not know how to establish his innocence. It is for that reason that the
right to be aassisted by counsel is deemed so important. It is essential for the
Court to assign a counsel de oficio if the accused does not have one.
It was, therefore, wrong for the trial court to hold that Section 12(1) is strictly
limited to custodial investigation and that it does not apply to a person against
whom a criminal complaint or information has already been filed.
Once a criminal complaint or information is filed in court, he must be delivered to
the nearest police station and since the court has already acquired jurisdiction
over his person, it would be improper for any public officer to
investigate him in conection with the commission of the offese for which he
is charged.
The Sinumpaang Salaysay of Maqueda after his arrest was taken in
palpable violation of his rights under Section 12(1). Maqueda was not
even told of any of his constitutional rights under the said section.
The statement was also taken in the absence of counsel. Such
uncounseled Sinumpaang Salaysay is wholly inadmissible pursuant
to paragraph 3 of Section 12.
However, the extrajudicial admissions of Maqueda to Prosecutor Zarate
stand on a different footing. These are not governmed by the
exclusionary rules under the Bill of Rights.
Maqueda voluntarily and freely made them to Prosecutor Zarate not in
the course of an investigation, but in connection with Maqueda’s plea
to be utilized as a state witness.
The provisions of the Bill of Rights are primarily limitations on government,
declaring the rights that exist without governmental grant, that may not
be taken away by government and that government has the duty to
protect. The Constitution does not govern the relationships between
individuals.
Accordinly Maqueda’s admissions to Ray Dean Salvosa, a private party,
are admissible in evidence under Section 26 Rule 130 of the Rules of
Court. In Aballe v. People, the Court held that the declaration of an
accused expressly acknowledging his guilt of the offense may be
given in evidence against him and any person.
To be added to Maqueda’s extrajudicial admission is his Urgent Motion for
Bail wherein he explicitly stated that he is willing to and volunteering to
be a state witness. In the light of his admissions to Prosecutor Zarate an
Ray Dean Salvosa and his willingness to be a state witness, Maqueda’s
participation in the commission of the crime was established beyond
moral certainty.
PEOPLE v. BALISTEROS (IYA) Then, she suddenly saw appellants coming out of the grove, Balisteros,
October 7, 1994 | Regalado, J. | Extrajudicial Confessions

PETITIONER: People of the Philippines


RESPONDENTS: Salvador Balisteros, Nilo Avestro, (BOTH REFFERED
TO AS THE ACCUSED) Ernesto Galvante

SUMMARY: Information against Balisteros, Avestro, and Galvante was


filed for the murder of a Romero Abad. The accused pleaded not guilty,
the prosecution hen moved for the exclusion of accused Ernesto
Galvante form the information so that he could become a witness for the
Government.

Galvante gave an affidavit and later on testified in court implicating


Balisteros and Avestro. The RTC ruled in favor of the prosecution. On
appeal, the accused interpose that the extrajudicial confession done by
Galvante is inadmissible as evidence invoking Section 12 of Article III.

The Supreme Court affirmed the lower court decision stating that the
objection can be raised only by the confessant whose rights have been
violated as such right is personal in nature. There is also no need for
counsel for the confession to admissible for it was done in open court.

DOCTRINE: By repeating his confession in court, Galvante converted it


into a judicial confession which, having been allowed by the trial court,
eliminated the need for assistance of counsel which is required in
extrajudicial confessions.

FACTS:
The prosecution presents testimonies of a Eliza Merceda and of Ernesto
Galvante. Erneste Galvante was originally an accused, however he
was removed from he information for being placed as a state
witness.
The present case is about the killing of one Romero Abad where the
herein accused acted in conspiracy and committed the felony with
treachery and evident premeditation.
The facts admitted in the trial court as presented by the prosecution is as
follows:
On August 16, 1991, at around 8:00 a.m., appellants were having a
drinking spree in the hut of appellant Balisteros which is located
inside the victim’s ricefield at Siling Matanda, Pandi, Bulacan.
The victim, Romeo Abad, dropped by the house of appellant Balisteros,
then proceeded to the ricefield.
Upon arriving at his ricefield at past 12:00 noon, Abad left again to graze
his horse near the bamboo grove
The helper, Eliza Merceada, on her way home, heard two shouts “huwag,
huwag” coming from the bamboo grove.
carrying a bolo and walking fast and Avestroz and Galvante, both None of these witnesses had any motive for testifying falsely against
barefooted. appellants.
When Merceada found out that Abad was not home, nor was he at the When there is no evidence and nothing to indicate that the principal
ricefield, she went near the bamboo grove where she heard the witness
shouts and found the lifeless body of Abad
At past one 1:00 o’clock that day, appellants took a ride in a passenger
jeepney driven by Florencio de la Cruz along the road at Siling
Matanda, then alighted at Pandi in a corner going to Balagtas and
Sta. Maria, Bulacan.
Liberato Antonio, a store vendor, also saw appellant Balisteros and
Avestroz and Galvante, at his store at around 2:00 p.m. that day, the
latter two, both barefooted.
They bought pairs of slippers from him and left thereafter.
The policemen who chased appellants eventually caught them at Plaza
Lawton, about to take a bus ride going to Mindoro.
Galvante confessed to appellant’s plan to kill the victim, who wanted to
eject appellant Balisteros from his house inside said victim’s ricefield.
He also claimed that appellants armed themselves with a “taris” and a
small knife; then, they all proceeded to the ricefield near the bamboo
grove and waited until Abad came, asking for his horse from Nilo,
who replied that it was tied.
All of a sudden, when Abad turned his back, appellant Balisteros held the
victim by the neck and when the latter turned around, Balisteros
stabbed him three times on the stomach.
Appellants denied all allegations and posited the defense alibis. The
lower court gave full faith and credit to the prosecution’s version and
concluded that the defense of denial and alibi has no weight after
Ernesto Galvante, an eyewitness, positively identified and pointed to
Balisteros and Avestroz as the killers of Abad.
Hence, this appeal to the SC .

ISSUE/s:
WON the accused are guilty of the murder of Romero Abad - YES

RULING: WHEREFORE, the assailed judgment is herebyAFFIRMED.

RATIO:
Proceed to Ratio 14 for explanation on Section 12
The appeal hinges on the issue of credibility and they have utterly failed
to present convincing reasons why the respect which appellate
courts give to findings of trial courts should not be maintained in this
case.
With reasoned consistency, the SC have reiterated that much leeway is
given to the trial court in assessing the credibility of the person testifying
before it.
The SC agrees with the court below that the prosecution’s witnesses are
more credible.
for the prosecution was actuated by improper motives, the presumption It should also be noted that that Galvante repeated in court what he had
is that he was not so actuated and his testimony is entitled to full faith stated in his affidavit and, although he was likewise subjected to a
and credit. thorough cross-
Appellants likewise endeavored to impeach the credibility of Police
Officer Alejandro Sagala who conducted the investigation of that
gruesome killing.
Appellants argued that said police officer “appeared to be a very ideal
and devoted investigator in his testimonies casting doubt on the
same” and that “the defense is just puzzled on the zealousness
shown by Sagala in handling the investigation of the case.”
The laudable efforts of police officer Sagala in investigating and
eventually solving the case should even be commended and
emulated, rather than condemned or doubted.
In fact, Sagala’s demonstrated dedication to his duties is indicative of the
strength of his character and fortifies his credibility as a witness.
It is axiomatic that a person’s guilt may be established not only by direct
evidence but also by circumstantial evidence, which is sufficient to
convict as long as:
there is more than one circumstance;
the facts from which the inferences are derived are proven; and
the combination of all the circumstances is such as to produce a
conviction beyond reasonable doubt
It is said that circumstantial evidence consists of such incidental facts
surrounding the main fact in issue and are compared to the strands
in a rope, where no one of them may be strong enough to establish
the guilt of the accused beyond reasonable doubt.
The entire conduct of the accused may be looked to for corroborative
circumstances, and if therefrom his connection with the commission
of the crime may be fairly inferred, the corroboration is legally
sufficient.
Another contention of appellants is the alleged violation of the
Constitution, particularly Section 12 of Article III, when in accordance
with the present rule for the qualification of a state witness, Galvante
executed a sworn statement wherein he categorically admitted his
guilt but without the assistance of a counsel.
Appellants also claim that Galvante “made his sworn statement in the
presence of Atty. Alejandro, the counsel of the plaintiff, Mr. Matic, the
two brothers of the deceased, the Mayor of Pandi and another
person whom he did not know.
With this battery of persons surrounding him, we could imagine the
pressure exerted on Galvante.
What is provided by the modified formulation in the 1987 Constitution is
that a confession taken in violation of said Section 12 and Section 17
of the same Article “shall be inadmissible in evidence against him”
meaning the confessant.
This objection can be raised only by the confessant whose rights have
been violated as such right is personal in nature.
examination, he stood fast on his confession and the revelations therein.
Also, by repeating his confession in court, Galvante converted it into a
judicial confession which, having been allowed by the trial court,
eliminated the need for assistance of counsel which is required in
extrajudicial confessions.
Furthermore, even in extrajudicial confessions which under jurisprudential
doctrines have been held to be generally binding upon the confessant
and not against his co-accused, Galvante’s confession would readily fall
into the exceptions to that rule since appellants are charged as co-
conspirators and said confession is used only as a corroborating
evidence, or as circumstantial evidence to show the probability of
participation by the co-conspirator, or is corroborated by other evidence
of record.
GAMBOA v. CRUZ (ELIEL)
June 27, 1988 | Padilla, J. | Right to counsel ISSUE/s:
WoN Gamboa’s constitutional rights were violated – NO
PETITIONER: Christopher Gamboa
RESPONDENTS: Hon. Alfredo Cruz

SUMMARY: Gamboa was arrested for vagrancy and was detained in the
Manila Precinct. During the police-line up, Gamboa was identified by
Complainant Bernal as a companion in the crime of Robbery. After which,
he was being charged for such crime. In open court, instead of presenting
evidence for the defense, he filed a Motion to Acquit or Demurrer to
Evidence allegeing that his constitutional rights to counsel were violated.
Hence this petition.

The SC held that the right to counsel is only available during the start of
the investigation. It was observed that the police line-up was not a part of
the custodial inquest, and thus, not yet entitled to such right. As to due
process, the court held that he was afforded the right to answer, since
that it the essence of due process. As to the Motion to Acquit, the SC
held that it was not the proper remedy, and thus, he has to wait for the
judgment before he could appeal.

DOCTRINE: The right to counsel attaches upon the start of an


investigation, i.e. when the investigation officer starts to ask questions to
elicit information and/or confessions or admissions from the
respondent/accused

FACTS:
Gamboa was arrested for vagrancy, without a warrant of arrest, by
Patrolman Arturo Palencia. Thereafter, he was brought to Precinct 2,
Manila, where he was booked for vagrancy and then detained
therein together with several others
During the lineup of 5 detainees, including Gamboa, Erlinda Bernal
pointed to Gamboa and said, “that one is a companion.” After the
identification, the other detainees were brought back to their cell but
Gamboa was ordered to stay on. While the complainant Bernal was
being interrogated by the police investigator, Gamboa was told to sit
down in front of her
An information for robbery was filed against Gamboa. He was arraigned.
Thereafter, heariings were held. The prosecution formally offered its
evidence and rested its case.
Gamboa, by counsel, instead of presenting his defense, manifested in
open court that he was filing a Motion to Acquit or Demurrer to
Evidence. Gamboa filed said Motion predicated on the ground that
the conduct of the line-up, without notice to, and in the absence of,
his counsel violated his constitutional rights to counsel and to due
process. Hence this petition
RULING: Dismissed and remanded to CFI Manila for further proceedings As may be observed, the 1973 and 1987 Philippine Constitutions go
farther and beyond the guarantee of the right to counsel under the
RATIO: Sixth and Foureenth Amendments to the US Constitution.
The rights to counsel and to due process of law are indeed 2 of the For while, under the latter, the right to counsel “attaches only at or after
fundamental rights guaranteed by the Constitution, whether it
be the 1973 or 1987 Constitution. In a democratic society, like
ours, every person is entitled to the full enjoyment of the rigths
guaranteed by the Constitution
The right to counsel attaches upon the start of an investigation, i.e.
when the investigation officer starts to ask questions to elicit
information and/or confessions or admissions from the
respondent/accused
At such point or stage, the person being interrogated must be
assisted by counsel to avoid the pernicious practice of
extorting false or coerced admissions or confessions from the
lips of the person undergoing interrogation, for the commission
of an offense
Any person under investigation must, among other things, be assisted by
counsel. The Constitution in Sec 12, Art III is cler. They leave no
room for equivocation.
Accordingly, in several cases, this Court has consistently held that
no custodial investigation shall be conducted unless it be in the
presence of counsel, engaged by the person arrested, or by any
person in his behalf, or appointed by the court upon petition
either of the detainee himself, or by anyone in his behalf, and
that, while the right may be waived, the waiver shall not be valid
unless made in writing and in the presence of counsel
As aptly observed, however, by the Solicitor General, the police
line-up (at lease, in this case) was not part of the custodial
inquest, hence, Gamboa was not yet entitled, at such stage, to
counsel
Even under the constitutional guarantees obtaining in the United States,
Gamboa would have no cause for claiming a violation of his rights to
counsel and due process.
Kirby v. Illinois, it was held that the constitutional right to counsel did not
attach until judicial criminal proceedings were inititated, and that the
exclusionary rule relating to out-of-court identifications in the
absence of counsel did not apply to identification testimony based
upon a police station showup which took place before the accused
had been indicted or otherwise formally charged with any criminal
offense
Justice Steward, In a line of constitutional cases in this cOurt stemming
back to the Court’s landmark opinion in Powell v. Alabama, it has
been firmly established that a person’s Sixth and Fourteenth
Amendment right to counsel attaches only at or after the time that
adversary judicial proceedings have been initiated agaistn him
the time that adversary judicial proceedings have been initiated which are groudns for a motion to quash, except where the complaint
against him (the accused),” under the 1973 and 1987 Philippine or information does not charge an offense, or the court is without
Constitutions, the right to counsel attaches at the start of jurisdiction of the same
investigaton against a respondent and, therefore, even before YAP, C.J. Dissenting: In my opinion, after the police line-up with other
adversary judicial proceedings against the accused have begun detainees in which the accsued was pointed out by the complainant
Given the clear constitutional intent in the 1973 and 1987 Bernal as one of the “companions” of those who allegedly committed
Constitutions, to extend to those under police investigation the the crime of robbery, the investigatory part of the proceedings started
right to counsel, this occasion may be better than any to remind when the accsued was signaled out and “ordered to sit down in front
police investigators that, while the Court finds no real need to of the complainant” while the latter gave her statement which led to
afford a suspect the services of counsel during a police line-up, the filing of the information
the moment there is a move or even an urge of said I do not agree with the view that since the accused was not asked any
investigators to elicit admissions or confessison or even plain question, he was not “under investigation.” The investigation
information which may appear innocent or innocuous at the commenced the moment he was taken from the police line-up and
time, from said suspect, he should then and there be assisted made to sit in front of the complainant, while the latter made her
by counsel, unless he waives the right, but the waiver shall be statement to the police.
made in writing and in the presence of counsel Neither do I agree with the view of the Solicitor general, which is
On the right to due process, the Court finds that Gamboa was not, in any sustained by the majority opinion, that the accused at that point was
way, deprived of this substantive and constitutional right, as he was not entitled to be informed of his right to counsel, because the “police
duly represented by a member of the Bar did not, at that stage, exact a confession to be used agaist him.”
He was accorded all the opportunities to be heard and to present The right to counsel must be afforded to the accused the moment he is
evidence to substantitate his defense; only that he chose not to, and under custodial investigation, and not only when a confession is
instead opted to file a Motion to Acquit after the prosecution had being exacted from him.
rested its case. What due process abhors is the absolute lack of Cruz, J. Separate: I concur because it does not appear from the
opportunity to be heard. The case at bar is far from this situation narration of the facts in this case that improper suggestions were
In any event, certiorari and prohibition are not the proper remedies made by the police to influence the witnesses in the identification of
against an order denying a Motion to Acquit. Sec 1, Rule 117 of the the accused
Rules of Cort provides that, upon arraignment, the defendant shall Sarmiento, J. Dissenting: It is the view of the majority that “the police
immediately either move to quash the complaint or information or line-up (at lease, in this case) was not part of the custodial inquest,
plead thereto, or do both and that, if the defendant moves to quash, hence, petitioner was not yet entitled, at such stage, to counsel.” It is
without pleading, and the motion is withdrawn or overruled, he my own view, however, that given the particular circumstances of this
should immediately plead, which means that the trial must proceed. case, he was entitled to counsel pursuant to the provisison of Sec 12,
If, after trial on the merits, judgment is rendered adversely to the movant of Art 3.
(in the motion to quash), he can appeal the judgment and raise the It is noteworthy that the accused was already in custody at the time. And
same defenses or objections (earlier raised in his motion to quash) although he was detained for some other cause (vagrancy), it left him
which would then be subject to review by the appellate court little or no choice other than to face his accuser.
An order denying a Motion to Acquit (like an order denying a motion to While I am not prepared to hold that a police line-up per se amounts to a
quash) is interlocutory and not a final order. It is, therefore, not critical stage of the investigation, for in most cases, it merely forms
appealable. Neither can it be the subject of a petition for certiorari. part of the evidence-gathering process, the fact that the acccsued
Such order of denial may only be reviewed, in the ordinary course of stood charged for an offense and has been detained therefor should
law, by an appeal from the judgment, after trial make this case different
Conformably with the above rulings, whether or not Gamboa was,
afforded his rights to counsel and to due process is a question
which he could raise, as a defense or objection, upon the trial
on the merits, and, if that defense or objection should fail, he
could still raise the same on appeal.
On the other hadn, if a defendant does not move t quash the complaint or
information before he pleads, he shall be taken to have waived all
objections
PEOPLE v. LOVERIA (JP) Loveria assails the manner in which he was identified by Driver
July 2, 1990 | Cortes, J. | Right to Counsel Manzanero at the headquarters of the Phil. Constabulary, claiming
violation of his
PETITIONER: People of the Philippines
RESPONDENTS: David Loveria y Santos

SUMMARY: Manzanero was driving a packed jeepney when a hold up


was initiated by accused Loveria. Loveria and his three companions then
divested the jeep passengers of their belongings. Manzanero, his
conductor and a passenger was stabbed. The latter died. RTC found
Loveria guilty of the complex crim of Robbeyr with Homicide and
Frustrated Homicide, in a band. Loveria claimed he was not assisted by
counsel during the proceedings.

The SC held that Loveria cannot invoke the constitutional guarantee


because it can only be invoked while he is under custodial investigation.
In this case, there was already a case pending in court. Loveria also
cannot invoke the right against Manzanero while the latter was pointing
at him to be the culprit. It was Manzanero being questioned. Loveria was
not under custodial investigation at that time. Loveria is guilty.

DOCTRINE: The so-called Miranda rights contained in the constitutional


provisions (Sec. 12) may be invoked by a person only while he is under
custodial investigation. It also cannot be invoked when it was the
complainant being questioned.

FACTS:
Cerilo Manzanero was driving a jeepney fully loaded with passengers.
While crossing the bridge of Barangay Baranca, accused Loveria
shouted “hold-up” and Manzanero stopped his jeep.
Accused Loveria who was seated right behind Manzanero, poked a knife
on the latter’s side of his nape. Manzanero was later on stabbed with
knives at the front and back of his body.
Accused Loveria had three companions seated at the different parts of
the jeep. They divested the passengers of their jewelries, watches,
rings and necklaces.
Manzanero’s conductor was likewise stabbed but he sustained only slight
injury on his finger. His watch was taken.
Passenger Ricardo Yamson who was seated in between Manzanero and
his conductor was also stabbed. He died that night.
The RTC found Loveria guilty of the complex crime of Robbery with
Homicide and Frustrated Homicide, with the aggravating
circumstance of being committed in band. Hence this appeal.
Loveria assails the credibility of the witnesses, the driver himself and the
conductor and a passenger who saw the other companions of the
accused.
constitutional right to counsel (Sec. 12, Art. 3).

ISSUE/s:
WoN the accused can invoke his constitutional rights to counsel already
pending in court – NO

RULING: Judgment in question is affirmed.

RATIO:
The court must emphasize that the so-called Miranda rights contained in
the constitutional provisions (Sec. 12) may be invoked by a person
only while he is under custodial investigation which has been defined
as the “questioning initiated by law enforcement officers after a
person has been taken into custody or otherwise deprived of his
freedom of action in any significant way.”
These constitutional rights may no longer be claimed by a defendant in a
criminal case already pending in court.
The Court held in Gamboa v. Cruz that in the police line-up conducted, it was
the complainant who was being investigated and who gave a
statement to the police while the accused was not questioned at all.
Thus, the Court concluded that the latter could not, during the line-
up, invoke his right to counsel because he was not under custodial
interrogation.
Since, as in the Gamboa case, the Loveria was not investigated
when Manzanero was in the process of identifying him, he
cannot claim that his right to counsel was violated because at
that stage, he was not entitled to the constitutional guarantee
invoked. It was Manzanero who was being questioned.
But even assuming that the process of identification of the appellant by
Manzanero at the PC headquarters was attended by constitutional
infirmities, only Manzanero’s sworn statement where he identified
appellant and which was taken by Pat. Ayun, would be excluded for
being inadmissible in evidence.
The positive identification testimonies of the other witnesses, taken
together with the other evidence on record, would be sufficient to
sustain the trial court’s judgment of conviction.
PEOPLE v. HATTON (HENRY) guy tapped the shoulder of Algarme and stabbed him with his right hand.
June 16, 1992 | Medialdea, J. | Custodial Investigation rights Algarme

PETITIONER: People of the Philippines


RESPONDENTS: Glenn Hatton

SUMMARY: This case primarily involes the murder of Algarme, during the
night of a fiesta celebration in Northern Samar. He was stabbed to his
death, but his friends claim that they recognized the features of the killer.
The next day of the incident, Algarme’s friends informed the police that
the can identify the killer if they are able to present people as suspects.
The following day, there was a line-up, to which the Ongue (friend of
Algarme) pointed out to Hatton as the killer. Trial commenced and Hatton
was found guilty beyond reasonable doubt by the trial court, to which
Hatton sought appeal, because he claims that he was not afforded right
to counsel.

SC held in sum that the policemen did not violate his constitutional right in
custodial investigations insofar as affording him counsel is concerned,
because it must be noted that line-ups are not yet part of the investigation
per se that affords the constitutional right to counsel, etc. to the accused.
However the SC held that despite no right violated, the Hatton is acquitted.
Besides the ambiguity in the identification of suspect (because the injury was
purportedly done by a right-handed person, and Hatton is left-handed), the
SC stated that Hatton was not positively identified in court. Although his
name was referred to by both Basierto and Ongue (friends of the victim), he
was not identified in court. Failure of the prosecution witness to positively
identify the assailant (malefactor) in court is fatal to the prosecution’s cause.
Pre-trial identification is not sufficient.

DOCTRINE: It is ruled that the right to counsel attaches upon the start of
an investigation, or when the investigating officer starts to ask questions
to elicit information and/or confessions or admissions from the
respondent/accused.

FACTS:
Respondent Hatton was charged with murder for the death of Faustino
Algarme.
Upon arraignment, he pleaded not guilty.
According to the principal witnesses (Basierto and Ongue), here’s what
happened: There was a fiesta being celebrated in Northen Samar.
Algarme, the victim, and 3 of his friends (Jesus Aboda, Romeo
Basierto, and Edgardo Ongue), were on their way to the house of
Engr. Corbillo after drinking 2 bottles of beer at Aileen’s resto.
The group walked side by side Algarme, who was in the extreme left at
the edge of Bonifacio Street.
7pm the neared the CLAO office (di sinabi ano yung CLAO), and they
noticed 2 men approaching. One was short, the other was tall. The tall
shouted for help and called Romy.
At that moment, Ongue gazed at the tall man who tried to pull back the
knife from the Algarme’s back. He didn’t recognize the guy but took
notice of the features.
The tall guy didn’t get the knife and ran towards the corner of a house
owned by Nonong Hatton.
Basierto testified that as soon as the tall guy stabbed Algarme, he ran
and Basierto chased him. Basierto recognized the guy when the
latter turned his face towards him. He was recognized because both
were residents of Catarman.
Basierto reported the incident to the police headquarters. 3 policemen,
one of whom testified as Titing Valera, accompanied him to the
scene where they beamed flashlights towards the house of Nonong
Hatton where the tall man was believed to have entered.
According to Ongue, a day after the incident, he narrated to the police
officers what really transpired, and told them that the features of the
tall guy was still in his memory and he can confirm the person if the
police is able to present any person, to which the police was not able
to do so
Second day after the incident, Ongue was picked up from his office and
made to identify their suspect, who turned out to be the tall guy. (so,
tall guy = Hatton)
Hatton interposed the defense of alibi, and said that he was in the house
of his friend, Eddie Laguitan. They were joined by their friends Eric,
Edgar, Marlon, Jaime, and a girl named Imelda. That it was the
baptism of theniece of Eddie that’s why they were there for drinks.
The lower court found Jatton guilty beyond reasonable doubt for the
crime charged. Hence, this present appeal was filed in the SC.

ISSUE/s:
WoN Hatton’s custodial rights were violated when he was found guilty –
NO. he was not in custodial inquest yet, no rights are available

RULING: SC acquitted Hatton.

RATIO:
Hatton is claiming that the judge in the lower court manifested bias and
partiality in hearing the case. The judge allegedly asked leading
questions and various points not asked by the fiscal when the
prosecution presented its first witness.
The judge practically took over the task of conducting the direct
examination. Basically, the judge even asked more questions than
the fiscal during the direct examination, the cross examination, and
the trial.
SC confirmed such behavior of the judge, as provided for in the
Transcript of Stenographic notes. Indeed, he asked more questions
than did the counsel of the accused or the fiscal.
However, the SC conceded that the trial judge did not manifest bias in
favor
of the prosecutioin in asking the witnesses for the prosecution nor The Solicitor General even observed that a police line-up was not
any hostility or malice against the defense witnesses. part of the custodial inquest, hence, there is still no entitlement to
The SC used as basis People v. Ibasan, where they held that it is not counsel. In the line-up stage, Hatton is not yet held to answer for
denied that the court had certain points conducted its own the criminal offense.
questioning during the proceedings. The records, however show that
the court’s questions did not amount to interference as to make the
case for the prosecution. It was merely made to calrify certain points
and confirm certain statements.
With regard the claim that the judge rendered the decision a day before
the last day of the trial, it was ruled that such was only a
typographical error and that the decision reflected that the trial judge
took into consideration the defense o the accused and the testimony
of his witnesses in arriving at his decision.
After the careful study of the case at bar, the SC held that the
judgment of conviction must be reversed. Yes naman bakit kaya?
Abangan…
The witness for the prosecution stated that the knife was stabbed using
the right hand. Hatton is left handed, and the SC holds that a left-
handed person cannot be expected to deliver a thrust with the same
intensity using his right hand.
That, it is erroneous for the trial judge to concluded that there was no
direct proof that from the location of the wound, the relative positiosn
of the people assailing and the victim and the trajectory of the blow
considering the internal organ pierced, the injury could not have
been inflicted by a right-handed person. It is unnatural for a left
handed person to use his right hand in accomplishing a tough act as
stabbing another.
Second, the identification made by Ongue of Hatton as the person who
stabbed was irregular. From his testimony, it si evident that he did not
positively identify the accused. At the time of the incident, he made a
very fleeting glance of the victim.
Also, seeking shelter under the US v. Wade case, Hatton argues that
when he was presented in a line-up of sorts he was not represented
by a counsel. With this he claims a violation of his constitutional right
to counsel during a custodial investigation.
SC found no merit in this argument. US v. Wade ruled that the presence
of counsel is indispensable in a post-indictment line-up. In that case,
the accused had already been arrested and a lawyer already
appointed to represent him. It was 15 days after his arrest and
appointment of a lawyer to represent him when he was presented in
a line-up.
In the present case, Hatton was brought to the police station only to be
identified by a witness to the killing of Algarme. He was not yet under
custodial investigation.
It is ruled that the right to counsel attaches upon the start of an investigation,
or when the investigating officer starts to ask questions to elicit
information and/or confessions or admissions from the
respondent/accused.
A case was previously ruled by the SC, whereit was held that the police could
not have violated petitioner’s right to counsel an due rocess as the
confrontation betweent he State and him had not begun. He did not give
any statement to the police during the line-up. He was not even
interrogated at all. In fact, the police did not exact a confession to be
userd against him. He was just ordered to sit down in front of the
complainant. (Gamboa v. Cruz)
PEOPLE v. FRAGO (DANNAH) rape by reason of causes other than the spontaneous
May 31, 1994 | Bellosillo, J. | Right to Counsel desistance of Frago; Ronalyn was awakened and shouted for
help, forcing Frago to release her and ran away
PETITIONER: People of the Philippines
RESPONDENTS: Orlando Frago

SUMMARY: Frago was charged with attempted rape, for trying to have
carnal knowledge with Ronalyn Pastera, and rape, for having carnal
knowledge with Jicelyn Lansap (both allegedly happened on the same
day). He was convicted by the trial court for rape, but acquitted on the
basis of reasonable doubt for the attempted rape. Frago allegedly carried
a sleeping Jicelyn to the house of a certain Andor, and raped her while
she was deeply asleep. Jicelyn awoke to body pains and was surprised
that she was in said house. She then went home, told her cousins and
her mom about what happened. Both Jicelyn and Ronalyn’s father
reported the incidents on the same day.

Fargo imputed error to the trial court in convicting him on the basis of an
identification which was made without the assistance of counsel and
according credence to the story of Jicelyn, which he considers fantastic,
thereby denying his constitutional right to be presumed innocent until proved
guilty beyond reasonable doubt. He cited the case of People v. Hassan
where Hassan was acquitted because he had no counsel when he was
identified. The Court however ruled that the Hassan doctrine is inapplicable,
because when Hassan was identified he was alone. Fargo, on the other
hand, was identified alongside 10 men.

The Court however acquitted Fargo because of doubts regarding


Jicelyn’s identification of him. From her testimony, there was no way she
would be able to remember how he looked like, and she could not even
describe him properly. The Court believed that Jicelyn and the Pastera
sisters discussed Frago’s features which led to his positive identification.

DOCTRINE: The right to counsel attaches upon the start of an


investigation, i.e., when the investigating officer starts to ask questions to
elicit information and/or confessions or admissions from the respondent/
accused.

FACTS:
From the Information: in the morning of September 26 1990, while
Jicelyn Lansap was sleeping, Frago carried her to a nearby house
belonging to Andor and there had carnal knowledge with her, ho was
deeply asleep.
In the morning of September 26 1990, Frago entered the room of
Ronalyn Pastera, a 9 year old girl, he held her head and tried to
carry her. However, he was unable to perform all the overt acts of
The trial court found Fargo guilty of rape, but was acquitted for attempted rape
on “reasonable doubt occasioned by lack of clear and convincing evidence
that the accused Orlando Frago indeed performed against Ronalyn Pastera
overt acts constituting commencement of the commission of the crime of
rape”
In holding Fargo guilty of rape, the trial court based its decision on finding that he
was positively identified by Jicelyn and that there was no ill motive on her
part to testify against him.
Jicelyn was a 15 y/o highschool student and was a boarder in Fausto Morales’
house, together with her cousins Susan and Adea.
The version of the prosecution is that before proceeding to the boarding house of
Jicelyn Lansap, the accused, an ice cream vendor, first went to the residence
of Ronalyn Pastera at around three forty-five in the morning and
surreptitiously entered Ronalyn’s bedroom where she was sleeping.
He fanned her face with his handkerchief and then lifted her bodily from the floor.
He was about to take her out of the room when she suddenly woke up and
screamed for help thus prompting her father to respond immediately by
switching on the lights.
As a consequence, the Fargo had to drop Ronalyn on the floor and run out of the
house. The prosecution would seem to infer that from the house of Ronalyn
where he failed in his alleged attempt to defile her, the accused next went to
the boarding house of Jicelyn some fifty meters away.
According to Jicelyn, she and her cousins went to bed at about eight o’clock in
the evening of 25 September 1990. Then at around five-thirty the following
morning, she was awakened by Fargo who was already strangling her. She
shouted for help so that he immediately ran away.
She felt pain all over her body, more particularly in her private part, and
discovered that she was no longer wearing her skirt and underwear. To her
consternation, she found herself in the vacant house of a certain Dado
Andor. So she lost no time looking for her way home.
She relayed the incident to her cousins who related it to her mother. When
brought to the doctor, it was found that there was spermatozoa and that she
had lost her virginity.
On the same day, Ronalyn’s father and Jicelyn reported the incidents to the
police authorities. Both Ronalyn and Jicelyn identified the accused as their
attacker in the police line-up on 28 September and 8 October 1990,
respectively, and then filed their formal complaints against him.
Fargo seeks sanctuary in the alcove of denial and alibi. He claims that at nine o’clock
in the evening of 25 September 1990, he was already asleep with his wife and
children. He woke up at six o’clock the following morning. He was
very tired that night because he was vending ice cream in the the whole At such point or stage, the person being interrogated must be assisted
day. by counsel to avoid the pernicious practice of extorting false or
Fargo imputed error to the trial court in convicting him on the basis of an coerced admissions or confessions from the lips of the person
undergoing interrogation, for the commission of an offense.
identification which was made without the assistance of counsel and
As aptly observed by the SolGen, the police line-up (at least, in this
according credence to the story of Jicelyn, which he considers
case) was not part of the custodial inquest, hence, petitioner was
fantastic, thereby denying his constitutional right to be presumed not yet entitled, at such stage, to counsel.
innocent until proved guilty beyond reasonable doubt. Since petitioner in the course of his identification in the police line-up
He argues that it was during his detention, when he was not assisted by had not yet been held to answer for a criminal offense, he was,
counsel, that he was identified by Jicelyn. therefore, not deprived of his right to be assisted by counsel
He invoked People v. Hassan where the Court affirmed the right of an because the accusatory process had not yet set in. The police could
accused to counsel at all stages of the proceedings, the most crucial not have violated petitioner’s right to counsel and due process as
of which is his identification, and denial thereof entitles him to the confrontation between the State and him had not begun. In fact,
when he was identified in the police line-up by complainant he did
acquittal.
not give any statement to the police.
There is nothing in the records which shows that in the course of the
ISSUE/s:
identification from the police line-up the police investigator sought to
WoN the identification of Fargo without assistance of counsel violated his
constitutional right – NO extract any admission or confession from Fargo.
The Court however sustain’s the defense on the insufficiency of
RULING: Judgment reversed. Fargo acquitted. identification. Jicelyn has no reliable basis for pointing to Frago as
the person who raped her.
RATIO: She says that his face was covered; that he had long hair; that while the
People v. Hassan: person who raped her had high nose (matangos) his nose is just
The manner by which Jose Samson, Jr., was made to confront and “katamtaman”; that she did not have the opportunity to observe the
identify the accused alone at the funeral parlor, without being
height of the rapist; and, that the only evidence of sexual intercourse
placed in a police line-up, was pointedly suggestive, generated
is the result of the medical examination.
confidence where there was none, activated visual imagination,
and, all told, subverted his reliability as eyewitness. According to Frago, Jicelyn had a suitor for whom she had “feelings of
This unusual, coarse and highly singular method of identification, which affection,” thus concluding that she may have submitted herself to
revolts against the accepted principles of scientific crime detection, him.
alienates the esteem of every just man, and commands neither our On the part of the Pastera sisters, they may have recognized Fargo
respect nor acceptance positively. They identified him on Sep. 28, Jicelyn pointed him out only on
Moreover, the confrontation arranged by the police investigator between the October 8.
self-proclaimed eyewitness and the accused did violence to the right of Under the circumstances, the conclusion is that Jicelyn’s identification of
the latter to counsel in all stages of the investigation into the
Orlando Frago was merely patterned after the identification made by
commission of a crime especially at its most crucial stage·the
the Pastera sisters. This is then a derivative, not positive,
identification of the accused.
identification.
While the infringement of the constitutional protection to the rights of the
Besides, it appears highly incredible that Jicelyn could be bodily lifted
accused should result in the acquittal of the accused in proper cases
from her room in her boarding house and taken some-three hundred
as a matter of course, Fargo’s reliance on Hassan is misplaced and
shows a deficient comprehension of our rationalization therein. meters away to the vacant house of Dado Andor where she was
In that case, there was no line-up of suspects. Only the accused. In contrast, supposedly abused without awakening her and her two cousins who
Fargo was singled out by Jicelyn in a police line-up composed of ten were all sleeping side by side with her.
persons.
Gamboa v. Cruz:
The right to counsel attaches upon the start of an investigation, i.e.,
when the investigating officer starts to ask questions to elicit
information and/or confessions or admissions from the respondent/
accused.
PEOPLE v. GAMBOA (ARIELLE) Gamboa was still holding the shotgun at this point. Soledad together
February 25, 1995 | Gancayco, J. | Paraffin Test with Acre, Gascon, and Maj. Impas, boarded the latter’s car and
brought Rene to the hospital.
PETITIONER: People of the Philippines
ISSUE/s:
RESPONDENTS: John Gabriel Gamboa
WoN the paraffin test conducted violated Gamboa’s constitutional right
SUMMARY: Gamboa, together with Celdran, allegedly killed Rene Impas, against self-incrimination – NO
Soledad’s common-law husband. Gamboa and Celdran were charged
with murder, but Celdran’s case was dismissed. The RTC found Gamboa RULING: The decision appealed from is AFFIRMED.
guilty beyond reasonable doubt. Gamboa, in his appeal, contends that
the lower court erred in not rejecting the paraffin test as inadmissible in RATIO: (Discussion related to our topic starts at Ratio #23)
evidence, and that it violated his right against self-incrimination.
Gamboa raises the issue relating to the credibility of the prosecution
The SC held that this constitutional right extends only to testimonial witnesses in that their testimonies are full of inconsistencies which
compulsion and not when the body of the accused is proposed to be elicit doubt as to their truthfulness.
examined as in this case. Gamboa’s right against self-incrimination was Soledad allegedly testified that Gabriel shot the victim twice, while the
not violated by the taking of paraffin tests of his hands. victim’s father said that he heard three successive shots. There is no
inconsistency here.
DOCTRINE: The constitutional right against self-incrimination applies Gamboa states that while Acre testified that at the time of the shooting,
only to testimonial compulsion, and not when the body of the accused is he could clearly be seen from the victim’s room, Soledad never
proposed to be examined by way of paraffin tests. mentioned having seen Acre near the door of the victim’s room.
Suffice it to state that at the time of the shooting, Gamboa and Celdran
FACTS: were standing at the door, effectively blocking the view outside the
Defendant-appellant John Gabriel Gamboa was charged with the crime room, hence Soledad did not see Acre. And Soledad was focused on
of murder together with Migel Celdran. After arraignment but during the victim, duh.
trial, the case against Celdran was dismissed.
Gamboa makes much capital of the fact that Acre did not reveal the
Thereafter, a decision was rendered, finding Gamboa guilty of the crime
of murder as penalized under Art 248 of the RPC. Aside from the identity of Gamboa to the victim’s father when they were together in
penalty of reclusion perpetua, he was also ordered to indemnigy the the car. This is understandable considering that Soledad had already
heirs of the deceased in the amount of P30,000. The weapon, a revealed Gamboa’s identity to Major Impas when they boarded the
shotgun, was ordered forfeited in favor of the government. car.
Gamboa interposed an appeal, alleging that the trial court erred in: On cross-examination, Gascon stated that after hearing the gunshots, he
Giving credence to the testimonies of the prosecution witnesses ran out of his house to ascertain where the shots came from. Another
In holding that he had been positively identified as the assailant
In holding that he confessed to the killing alleged discrepancy is between the version of Major Impas that
In not rejecting the alleged murder weapon as inadmissible evidence Gamboa and his companion were running towards the car and that of
In holding that the weapon was the actual murder weapon Gascon’s testimony that Gamboa was backtracking towards tabriel
Not rejecting the paraffin test results as inadmissible evidence
fleeing from the scene of the crime to their get away car.
At around 1am in the morning of August 21, 1988, Cristina Soledad,
common-law wife of Rene Impas, was conversing with the latter The alleged contradiction between Gascon’s affidavit, wherein he
inside a bedroom in Rene’s house. Suddenly, someone kicked open mentioned that he saw the appellant and another person
the door and Soledad saw Gabriel and Celdran. running towards the car, and his testimony on cross-
Gamboa fired his shotgun at Rene and was hit on the right side of the examination that he only saw Gamboa, is of no material
chest. Gamboa then fired another shot hitting the victim on the consequent because Gamboa has been positively identified as
abdomen. Rene died immediately. the assailant.
Rico Acre, a tenant in the house, came to help and loaded Rene in the
car.
The Court is not persuaded that Gamboa’s claim of contradictions and
inconsistencies on the part of the prosecution witnesses puts into
serious doubt their credibility.
Different persons who witnessed an incident from different angles and Nevertheless, the Court is not persuaded that the police investigators
situations could not be expected to give uniform details of what they would
saw and heard. Such minor discrepancies are to be expected
because of the human differences in perception.
Under the second assigned error (positively identified as assailant),
Gamboa alleges that his identification by the prosecution cannot be
relied upon because they did not immediately inform the police
investigators of his identity upon their arrival.
Although it may be true that the eyewitnesses did not immediately
identify Gamboa as the assailant to the policemen, it is also a matter
of fact that Major Impas informed Cpl. Petallar that the assailant is
Gamboa. The second instance was when Soledad told Cpl. Petallar
that Gamboa was the one who shot Rene.
It is quite understandable when the witnesses do not immediately report
the identity of the offender after a startling occurrence especially
when they are related to the victim. More so as in the case of Major
Impas who is the victim’s father.
The inadmissibility of the alleged verbal confession of Gamboa is raised on
the ground that he was maltreated as a result of which he suffered 27
injuries.
It does not appear, however, that the prosecution proposed to rely on this
alleged confession of Gamboa, or that the trial court considered the
same at all in the resolution of the case.
If it were to be considered at all, it would be worthless because of the
undeniable fact that Gamboa was not only arrested without a warrant
and entry to his house was effected without one, but worse, he was
maltreated since his arrest so much so that he suffered multiple
injuries.
The police investigators responsible for this mandhandling should be
investigated and held to account. Such involuntary confession
cannot help the case of the prosecution.
Under the fifth assigned error, Gamboa questions the admissibility of the
shotgun as the alleged murder weapon. He says it was not found in
his possession but his house was searched and the shotgun was
confiscated without a search warrant.
He also alleges that the three empty shells that were submitted for the
ballistics examination were not recovered from the scene of the
crime and their production is a frameup by the police.
Again, the Court observes that the police confiscated the shotgun from
the premises of the residence without a search warrant. Such
violation of the constitutional rights of a person should be
investigated.
willingly allow themselves to be instruments to frame Gamboa for so
serious a crime as murder. It appears that the three empty shells were
actually recovered from the vicinity of the scene of the crime.
Even if the Court disregards the shotgun as having been illegally secured,
still there is adequate evidence to justify a verdict of conviction.
Indeed, the Court did not even consider it necessary to inquire into the
motive of Gamboa in the light of his positive identification by the
proseucution witnesses.
As to the paraffin test to which Gamboa was subjected to, he raises the
question that it was not conducted in the presence of his lawyer.
This right is afforded to any person under investigation for the commission of
an offense whose confession or admission may not be taken unless he is
informed of his right to remain silent and to have competent and
independent counsel of his own choice.
His right against self-incrimination is not violated by the taking of the
paraffin test of his hands. This constitutional right extends only to
testimonial compulsion and not when the body of the accused is
proposed to be examined as in this case.
Indeed, the paraffin test proved positively that he just recently fired a gun.
PEOPLE v. LINSANGAN (IYA) They signaled the suspect who was 6 meters away from them to
April 16, 1991 | Griño-Aquino, J. | Right against self incrimination approach. They, with the suspect, walked toward a wooden house.
They then told him
PETITIONER: People of the Philippines
RESPONDENTS: Carlito Linsangan

SUMMARY: Police operatives received intel that there was rampant drug
use in Tondo, Manila. They organized a buy-bust team. With the help of a
person caught in violation of RA 6425, they were able to identify a seller of
illegal drugs.

The buy-bust operation they conducted was a success, they were able to
arrest Carlito Linsangan while in the act of dealing illegal drugs. He was
then asked to sign the marked bills received by him during the bust. The
RTC found him guilty of violating the Dangerous Drugs Act.

On appeal, Linsangan alleges that when he signed the bills he was not
assisted by counsel, therefore the bills are inadmissible as evidence.
The Supreme Court clarified that his conviction was not based on the
P10 bills alone but on the actual act of selling marijuana.

DOCTRINE: Although he was not assisted by counsel when counsel


when he initialed the P10-bills that the police found tucked in his waist,
his right against self incrimination was not violated for his possession of
the marked bills did not constitute a crime; the subject of the prosecution
was his act of selling marijuana cigarettes.

FACTS:
Police operatives of the Drug Enforcement Unit were informed that there
was rampant drug using and pushing in Tondo, Manila.
The pusher was described to them as a boy of 20 years, 5’5” in height,
and of ordinary build.
He allegedly sold marijuana to anybody, regardless of age.
Police Lieutenant Manuel Caeg and the other members of the unit
organized a buy-bust operation. Patrolman Ruiz was designated as
team leader.
5 days before the appointed date, the police operatives conducted a test-
buy operation.
They arrested a person for violation of Section 8 of RA 6425 (Possession
or Use of Prohibited Drug).
The person arrested informed them that he bought marijuana at
Dinalupihan Street in Tondo.
Before the group left the office for the area of operation, two 10 peso bills
were given to Patrolman Tomas Corpuz who had them marked with
his initials T.C.
that they would be buying marijuana for P20.
After handing the bill to the suspect, he proceeded inside the house.
Shortly after, the suspect emerged from the house and handed the
handrolled marijuana.
Pat. Corpuz took them and at the same time grabbed the suspects’ hand
introducing himself as police and arresting him.
The suspect with the other police boarded a jeep and returned to the
police station. His identity was discovered to be Carlito Linsangan.
He was then asked to sign his name on the P10 bills earlier handed to
him as evidence. The suspected marijuana was sent to the NBI for
examination.
As soon as Pat. Ruiz received the NBI report on the examination, he filed
a report for violation of the Dangerous Drugs Law with the fiscal’s
office.
Linsangan denied the charge. He alleged that the police officers
fabricated the charge against him for in the last week of September,
Corpuz and Ruiz tried to board his tricycle to arrest someone but he
refused for fear of being involved in a case.
The trial court found him guilty. Hence, the appeal to this Court.

ISSUE/s:
WON Linsangan is guilty of the Dangerous Drugs Act - YES

RULING: Decision of the lower court is affirmed

RATIO:
The court’s assessment of the credibility of the prosecution’s witnesses is
entitled to great respect unless and until they are clearly shown to be
arbitrary, which the defense failed to do.
The alleged motive of the policemen for fabricating the charge against
him and planting marked money on his person is not credible.
The Court is unable to imagine that a lowly tricycle driver would have the
temerity to defy a pair of armed policemen by refusing to give them a
ride in his tricycle to pursue a law violator.
Linsangan was likewise not denied due process during the custodial
investigation.
Although he was not assisted by counsel when counsel when he initialed
the P10-bills that the police found tucked in his waist, his right
against self incrimination was not violated for his possession of the
marked bills did not constitute a crime; the subject of the prosecution
was his act of selling marijuana cigarettes.
His conviction was not based on the presence of his initials on the P10-
bills, but on the fact that the trial court believed the testimony of the
policemen that they arrested him while he was actually engaged in
selling marijuana cigarettes to a member of the arresting party.
PEOPLE v. DE LAS MARINAS (ELIEL) later knew to be Orlando de las Marinas and Angel Torres,
April 30, 1991 | Paras, J. | Right to remain silent respectively.
They brought the two accused to the 13 th Narcotics Unit for further
PETITIONER: People of the Philippines investigation. They also executed an affidavit of arrest. They also
prepared the Receipt for Property Seized and asked Lando and Boy
RESPONDENTS: Orlando de las Marinas y Ibboos to sign it which
SUMMARY: An entrapment operation was set up after a confidential
informant gave a tip regarding the active selling of shabu in Cabrera
Street by Lando and Boy. When the entrapment was executed, Lando
and Boy were arrested and brought into the 13 th Narcotics Unit for further
investigation. It is also there that they were made to sign the RECEIPT
FOR PROPERTY SEIZED. Lando contends that the receipt, which they
had signed was inadmissible as evidence against them for violation of the
constitution.

The SC held that the RECEIPT FOR PROPERTY SEIZED was indeed a
violation of their constitutional right to remain silent. They were made to
admit that thay own the “shabu” allegedly confiscated from them, in turn
resulting to extrajudicial confession. However, the court still found them guilty
because of the testimonies.

DOCTRINE: No doubt this is a violation of the constitutional right of


Lando to remain silent. He was made to admit the commission of the
offense without informnign him of his constitutional rights. Such a
confession obtained in violation of the Constitution is inadmissible in
evidence

FACTS:
The 13th Narcotics Unit, thru T/Sgt. Jamie Raposas, received an
information from a confidential informant who arrived at their office,
that a certain Lando and one Boy of Cabrera Street, were actively
engaged in selling shabu to prospective buyers at the said place.
A team composed of T/Sgt. Jame Raposas, Sgt. Armando Isidro, Sgt.
Vicente Jimenez, Pat. Deogracias Gorgonio and himself was formed to
conduct a buy-bust operation. Arcoy was instructed by team Leader
Raposas to act as the poseur buyer. He was given marked money
consisting of P100 and P50.
The confidential informant introduced Arcoy to Lando and Boy who were at
the time in front of a Variety Store at Cabrera Street. Lando asked him
how much shabu did he need and he replied that he needed shabu
worth P150.
Lando asked for the money, gavi it to Boy who left and returned after
about 5 to 10 minutes and handed a small aluminum foil to Lando,
who in turn, gave it to him.
After opening the foil and finding out that it contained shabu, he made
the pre-arranged signal by scratching his head. At this juncture, his
teammates arrived and they arrested Lando and Boy whome they
they did.
Lando contends that he signed an affidavit admitting that the shabu
came from him because of torture and threats. Hence this appeal.

ISSUE/s:
WoN the Receipt for Property Seized are admissible – NO
WoN the testimonies should be given credence - YES

RULING: Guilty beyond reasonable doubt

RATIO:
Under the first assigned error, Lando claims that Exhibits F and F-1
which refer to a RECEIPT FOR PROPERTY SEIZED, signed by
Lando and Boy acknowledging that they are the owners of the
seized listed properties, is inadmissible in evidence for having
been taken in violation of the Constitution
Obviously, Lando was the victim of a clever ruese to make him sign
the RECEIPT FOR PROPERTY SEIZED which, in effect, is an
extra-judicial confession of the commission of the offense
Indeed, it is unusual for Lando to be made to sign said receipt for
what were allegedly taken frm him. It is the police officers who
confiscated the same who should have signed said receipt
No doubt this is a violation of the constitutional right of Lando to
remain silent. He was made to admit the commission of the
offense without informnign him of his constitutional rights.
Such a confession obtained in violation of the Constitution is
inadmissible in evidence
However, the trail court relied not merely on the strength of the property
seized in its findings of guit, but more importantly on the testimonial
and documentary evidence of the prosecution
The second and third assigned errors are factual in nature wherein
Lando raises the question of credibility of the witnesses and
reiterates his defense denying that he had been appreheneded in
the act of selling shabu
It is well established that the evaluation of testimony is the primary task of
the trial court where there are conflicting versions and that the findings of
the trial court on the credibility of witnesses are generally accorded great
respect
Be it noted that while Lando attempts to discredit the testimony of said
police officers, he has not shown any improper motive why they
should testify in the manner that they did. Thus, their testimonies are
entitled to full faith and credence. As a matter of fact, courts give
credence to entrapping police officers, as in this case, as they are
presumed to be in the regular performance of official duties
PEOPLE v. ENRIQUE (JP) RULING: Judgment in question is affirmed.
December 9, 1991 | Medialdea, J. | Right to Counsel
RATIO:
PETITIONER: People of the Philippines The SC affirmed the RTC. Appellant Enrique completely misses the
RESPONDENTS: Antonio Enrique, Jr. whole point of his prosecution and ultimate conviction under RA
6425. He was caught in flagrante delicto selling marijuana cigarettes
SUMMARY: Accused Enrique was reported to be selling marijuana to a poseur-buyer in exchange for money. The serial number of the
cigarettes. He was arrested in flragrante delicto through an entrapment marked money need not even be stated in the information.
operation where he sold for a marked P10 bill cigarettes to poseur-buyer The marked money and the marijuana cigarettes were immediately
Pat. Navidad. He claimed that evidence against him is inadmissible as seized by the arresting officers contrary to Accused Enrique’s
they were taken during custodial investigation. It was shown that he allegation that the same were taken during the custodial
admitted the crime. RTC convicted him of RA 6425. investigation. In fact, there was a receipt of the property seized from
the accused-appellant signed by the arresting officers and a copy
The SC affirmed the conviction. Although as for the admission of the thereof was furnished him.
accused, the SC sided with him that such is inadmissible. He was not Apart from the seized items, the NARCOM officer, in a clear and categorical
assisted by counsel and he was not informed of his rights during the manner, testified to the circumstances of the appellant Enrique’s
entrapment and arrest which was corroborated by the testimony of his
custodial investigation.
companion officer
The SC notes however that, during the investigation, the accused Enrique,
DOCTRINE: Admission obtained in violation of right to counsel
unassisted by counsel, wrote his name on the rolled marijuana cigarettes.
inadmissible.
Accused must also be informed of his rights during custodial investigation. Pat. Quebalayan testified that the accused affixed his name thereon
voluntarily and that this gesture was intended as a security against
substitution.
FACTS: Yet, he (Quebalayan) and the other arresting officers exerted no efforts
Civilian informer whose identity cannot be divulged reported that one to inform the accused-appellant Enrique of his rights under
alias Bong (turned out to be accused Enrique) was engaged in custodial investigation. In effect, appellant’s act amounted to an
selling marijuana cigarettes. admission of his culpability in violation of the constitution. Such
A team composed of Pat. Alejandro and Danilo Natividad of the 2 nd admission is therefore inadmissible as evidence.
Narcotic Unit of INP Aparri was organized. Notwithstanding this setback for the prosecution, appellant Enriqe
Pat. Navidad acted as a poseur buyer. The suspect Alias Bong was cannot be acquitted because his conviction did not rest solely
approached by Pat. Navidad who in turn handed the marked P10 bill. on his admission. He has completely failed to rebut by contrary
Pat. Navidad than gave the pre-arranged signal to the team to close evidence the testimonies and documentary evidence presented by
in and effect the arrest of Alias Bong. the prosecution.
Accused Enrique vigorously insists his innoncence. He claims that all the
exhibits presented in court (Exhibit A was the ten peso bill, B was the
cigarette pack, D is the siganutre of the officers, E was the laboratory
test confiriming the drug) are inadmissible because they were taken
during custodial onvestigation and therefore, violative of the
constitution. He claims no receipts were presented.
Note: Ratio explains that he later on admitted the crime but with
procedural setbacks.
The RTC convicted Enrique of violation of Sec. 4, Art. 2 in relation to Sec.
2, Art. 1 of RA No. 6425 or otherwise known as Dangerous Drugs Act
of 1972.

ISSUE/s:
WoN the admission of the accused is admissible as evidence – NO (but
he is not acquitted)
PEOPLE v. BANDIN (HENRY) Bandin then, after the trial court decided on his case, filed an appeal to
June 16, 1992 | Medialdea, J. | Right to counsel in a custodial the
investigation

PETITIONER: People of the Philippines


RESPONDENTS: Paul Bandin Y Narciso @ “Abling”

SUMMARY: This case involves the finding of the RTC of Bandin guilty of
drug trafficking in Albay. He was caught after an informant reported to the
NARCOM of his actions, and then the latter conducted a buy-bust
operation. Having caught in flagrante delicto, he is evidently lead to the
office where he was asked to sign documents, without counsel: Receipt
of Property Siezed, Booking Sheet, Arrest Report. He now claims in his
appeal to the SC that these signatures be inadmissible in evidence
because they are made without counsel and therefore in violation of his
constitutional right.

The SC ruled that the signature in the Booking Sheet and Arrest report
are not admissions of the commission of the crime and therefore not
within the ambit of the Sec 12 protected rights. However, the signature in
the Receipt of Property Seized is. BUT, despite such findings, the SC
affirmed the RTC decision because again, Bandin was caught in
flagrante delicto, and so despite inadmissibility of one signature in one
document, he is still guilty beyond reasonable doubt for drug trafficking.

DOCTRINE: When an arrested person signs a Booking Sheet and Arrest


Report at a police station, he does not admit the commission of an
offense nor confess to any incrmiminating circumstance.

The signature of the accused-appellant on the Receipt of Property


Seized is a declaration against his interest and a tacit admission of the
crime charged, for mere unexplained possession of prohibited drugs is
punished by law.

FACTS:
This case basically revolves on the buy-bust operation of NARCOM in
Albay, where respondent Bandin was charged for drug pushing (RA
6425, Art II Sec 4 violation), allegedly committed at 3:20pm
The RTC rendered Bandin guilty beyond reasonable doubt for such
offense, for transporting marijuana in tea bags, and cigarette form.
So in the afternoon of June 24 1991, an informer reported to the office of
NARCOM in Legazpi City that Bandin was trafficking drugs at
Bagtang Terminal in Albay.
NARCOM organized a buy-bust operation, to which was a success as
they caught Bandin red handed. He was brought to the NARCOM
office for investigation, where he signed a document (Recepit of
Property Seized) and Booking Sheet as well as the Arrest Report. He
signed these documents unassisted by counsel.
SC assailing him not being afforded of his constitutional right to
counsel.

ISSUE/s:
WoN the NARCOM violated his constitutional right to counsel in a
custodial investigation – NO

RULING: SC affirmed the trial court ruling

RATIO:
The SC reiterated the ruling of the SC in People v. Rualo, that when an
arrested person signs a Booking Sheet and Arrest Report at a police
station, he does not admit the commission of an offense nor confess
to any incrmiminating circumstance.
The booking Sheet is merely a statement of the accused’s being booked
and of the date which accompanies the fact of an arrest.
It is a police report and may be useful in charges of arbitrary detention
against the police themselves. it is not an extra-judicial statement
and cannot be the basis of a judgment of conviction.
As far as the signature in the Receipt of Propertuy Seized, Bandin claims
that his signature is inadmissible as evidence because it was given
without assistance of counsel, to which the SC agrees.
SC cited the case of People v. Mauyao and People v. Turla, where the
SC held that the signature of the accused-appellant on the Receipt of
Property Seized is a declaration against his interest and a tacit
admission of the crime charged, for mere unexplained possession of
prohibited drugs is punished by law. The signature of the accused-
appellant on the receipt is tantamountto an uncounseled extra-
judicial confession outlawed by the Bill of Rights. It is, therefore,
inadmissible as evidence for any admission wrung from the accused
in violation of his constitutional rights is inadmissible against him.
Nevertheless, despite the exclusion of the Receipt of Property Seized,
the guilt of Bandin has been established beyond reasonable doubt
by other evidence in record.
The clear and convincing testimonies of the officers, among others, is
one of those. Specifically, because they are coming from the vantage
point of a buy-bust operation, where Bandin was caught red-handed.
Bandin was cought in flagrante delicto for he was carrying marijuana,
hence committing a crime, at the time of his arrest. The warrantless
search which was conducted following a lawful arrest, was valid.
PEOPLE v. ESCORDIAL (DANNAH) told by a man sitting inside said jeepney to go home lest they
January 16, 2002 | Mendoza, J. | Line-up would meet an accident. He was later identified by Joniega
and Esmeralda as the accused.
PETITIONER: People of the Philippines In front of the jeepney in a boarding house was where Michelle, Erma
RESPONDENTS: Anthony Escorial and

SUMMARY: Michelle and Erma, who lived in a boarding house with Ma.
Teresa were robbed by a man. Michelle was also raped by the man twice. All
these happened while the girls were blindfolded. The man spoke to them
after and told Michelle that he would always see her and catcall her, but she
never responded.

Because of this, police officer Tancinco investigated the matter. A kid


named Esmeralda allegedly saw the man from his room when this
happened, and Michelle could identify the man by his voice and his
keloid at his nape. Their descriptions led them to believe the crime was
done by Anthony Escordial, who worked in Coffee Break Corner.
Escordial was found in his hometown playing basketball when they
invited him for interrogation.

According to Escordial, he was tortured and forced to admit to the crime,


which he never did. The Court discussed the rule regarding admissibility
of evidence. Here, Escordial was identified by Michelle Darunda in a
show-up on January 3, 1997 and by Erma Blanca, Ma. Teresa Gellaver,
Jason Joniega, and Mark Esmeralda in a police line-up on various dates
after his arrest. Having been made when accused-appellant did not have
the assistance of counsel, these out-of-court identifications are
inadmissible in evidence against him. (However since the defense failed
to bring the matter up it was deemed waived).

The Court however acquitted Escordial because his guilty was not
proven beyond reasonable doubt. In dismissing the disparity between
Escordial’s appearance and Michelle’s description of her attacker, the
trial court dwelt on the apparent roughness of Escordial’s skin and the
probability that Michelle might have felt only the arch of the spinal chord
of her assailant. However, mere speculations and probabilities cannot
take the place of proof beyond reasonable doubt required by law to be
established by the prosecution.

DOCTRINE: The court has thus ruled that any identification of an


uncounseled accused made in a police line-up, or in a show-up for that
matter, after the start of the custodial investigation is inadmissible as
evidence against him.

FACTS:
Joniega and Esmeralda testified that at 8pm of December 1996, they and
Mark Lucena were playing inside a jeepney. However, the boys were
Ma. Teresa lived. At 9pm on the same day while they were asleep, Erma was
awakend by a man.
His face was covered by a shirt and had a knife 4 inches long. He asked for
money. Erma pointed to her bag, gave him P300 but when the man insisted
gave all her money.
Michelle gave her P3,100. Teresa claimed her money was in the next room but
he did not allow her to leave the room.
The man gave them shirts to blindfold each other, but he was the one who
blindfolded Michelle. He then raped her.
Michelle said that although she was blindfolded and could not see, she could feel
that the man had no cover on his face when he was raping her. She felt that
his chest was rough and had some scars. When he placed her hands on his
nape, she felt that it was also rough.
On the other hand, Erma claimed she was able to see through her blindfold and
that she saw the man’s face because of the light coming from the lamp post
outside the boarding house. Their bedroom window had panes through which
the light filtered in.
After raping Michelle, he talked to the three women and told Michelle that he
used to catcall her and called her a beautiful girl whenever she passed by his
place but Michelle had ignored him.
The man again raped Michelle by inserting his penis in her anus. After which he
returned to Michelle a pair of earrings which he had taken from her. He then
left, but not before warning the women not to report the matter to anyone or
he would kill them.
Mark Esmeralda testified that he was in his bedroom on the second floor of their
house, toying with a flashlight, when he saw from his bedroom window a
man wearing denim shorts coming out of the boarding house.
He went downstairs and told his parents what he had seen. Mark identified
Escorial as the man he saw that night.
The three women asked help from their neighbor Tiyo Anong. The matter was
reported to the police. When the policemen arrived, they asked Michelle to
describe the assailant, but she told them that she could only identify his voice
and his eyes. Accompanied by the police, the three women looked for the man
around the Libertad area, but they did not find him.
Upon returning home, Michelle confessed to her aunt and uncle about the rape
(di niya pinareport muna sa pulis the first time). They took her to the
Women’s Desk of the police station to report the rape
PO3 Tancinco said that assailant was described to him as wearing long hair and
having a rough projection on the back of his neck, small eyes, a slim body,
and a brown complexion.
Tancinco went around Margarita Extension and learned about the children Dr. Jocson who examined Michelle testified that there was penetration. Also, her
playing on the street around the time the intruder entered the boarding hymen had lacerations which were caused by abrasions due to force or
house.
Tancinco was likewise informed by Esmeralda that the person he saw
inside the jeepney was the same person he saw coming out of the
boarding house later that night.
At around 8 o’clock that evening, Tancinco questioned a certain Tiyo
Anong and Ramie about the identity of the suspect. Ramie said that
the description of the suspect fitted that of a worker at a café called
Coffee Break Corner, about two houses away from the boarding
house.
Fidel Hinolan, owner of the café told them that Escordial was his helper
and that the latter had gone home on December 27, 1996 to
Barangay Miranda, Pontevedra, Negros Occidental.
Tancinco asked the help of the police in Pontevedra to locate Escordial.
They found him at the basketball court and “invited” him to go to the
police station for questioning.
Michelle remained at the Pontevedra police station. When Escordial was
brought there, he saw Michelle and blushed. Michelle looked at him
and recognized him as the man who had robbed and raped her on
December 27, 1996.
Escordial was asked to take off his t-shirt. Michelle said that she just kept
quiet while Escordial tried to talk to her.
However, according to Tancinco, Michelle confirmed to him that Escordial
was the man who had attacked her, identifying him through a rough
projection, or a keloid, on the back of his neck and his voice.
A photo of Escordial was taken at the Pontevedra police station.
At the Bacolod station, Erma, Teresa, Joniega and Esmeralda were
asked whether Escordial was the same person they saw on the night
of the incident. They were taken one by one to the jail cell and asked
to point to the person that they had seen that night. They picked
Escordial out of four people who were inside the jail cell.
Michelle executed an affidavit (Jan. 4) identifying Escordial as the person
who robbed and raped her.
She testified that she and her friends had gone to the Coffee Break
Corner sometime in September or October 1996. On the way home,
she was approached by Escordial.
He asked Michelle what her name was, and she gave it to him, albeit
reluctantly. She usually passed by the said café when going home
and Escordial would often whistle at her and call her a beautiful girl.
pressure applied on the vaginal area. independent counsel, he has not shown that, as a result of his
She could not remember either whether there was sperm in custodial interrogation, the police obtained any statement from him
Michelle’s vagina when she examined the latter. ·whether inculpatory or exculpatory·which was used in evidence
She testified that it could not be determined how many times against him.
Michelle had previously engaged in sexual intercourse because
this would depend on the elasticity of the her hymen. She
opined, however, that it would be less than 10 times.
Dr. Jocson stated it was possible Michelle agreed to have sexual
intercourse voluntarily based on the lack of marks of violence
although it was also possible that she was merely forced to have
sex because she was threatened.
Escordial testified that on the evening of December 26 he aksed
permission from Hinolan to go home to Pontevedra and stay
there until January. Also while in Pontevedra he took care of the
cocks of an Elias Sombito.
He was also helping his friend’s mother with the household chores.
These were corroborated.
The defense also presented other witnesses, one of which who
testified that Michelle could not even identify Escordial as her
attacker.
Escordial testified that he was beaten up and tortured. He was forced
to admit to the crime several times, and tortured everytime he
denied.
The witnesses of the defense also testified that they found Escordial
tied to a chair, beaten up. They brought him to the hospital where
they found that he had sustained injuries, including a broken
ankle.
The trial court still convicted Escordial of robbery with rape despite
the witnesses presented by the defense.

ISSUE/s:
WoN Escordial’s identification is admissible as evidence – NO (pero
na-waive)

RULING: Judgment reversed. Escordial acquitted.

RATIO:
On the legality of his arrest: Although he was arrested without a
warrant and just a mission order, when Escordial pleaded not
guilty, he waived objection to the legality of his arrest.
On admissibility of evidence from his interrogation: While it cannot be
be denied that Escordial was deprived of his right to be informed
of his rights to remain silent and to have competent and
In fact, Escordial testified that at no point, even when subjected to Having been made when accused-appellant did not have the
physical torture, did he ever admit committing the crime with which assistance of counsel, these out-of-court identifications are
he was charged. In other words, no uncounseled statement was inadmissible in evidence against him.
obtained from Escordial which should have been excluded as Be that as it may, as the defense failed to object immediately when
evidence against him. these witnesses were presented by the prosecution or when
On assistance of counsel: He was never assisted by counsel, whether specific questions
of his own choice or provided by the police officers, from the time of
his arrest in Pontevedra, Negros Occidental to the time of his
continued detention at the Bacolod police station.
Although Escordial made no statement during this time, this fact remains
important insofar as it affects the admissibility of the out-of-court
identification of accused-appellant by the prosecution witnesses,
namely, Michelle Darunday, Erma Blanca, Ma. Teresa Gellaver, Mark
Esmeralda, and Jason Joniega.
As a rule, an accused is not entitled to the assistance of counsel in a
police line-up considering that such is usually not a part of the
custodial inquest.
However, the cases at bar are different inasmuch as Escordial, having
been the focus of attention by the police after he had been pointed to
by a certain Ramie as the possible perpetrator of the crime, was
already under custodial investigation when these out-of-court
identifications were conducted by the police.
An out-of-court identification of an accused can be made in various ways:
In a show-up: The the accused alone is brought face to face with the
witness for identification,
In a police line-up: The suspect is identified by a witness from a group
of persons gathered for that purpose.
During custodial investigation, these types of identification have been
recognized as “critical confrontations of the accused by the
prosecution” which necessitate the presence of counsel for the
accused.
This is because the results of these pre-trial proceedings “might well
settle the accused’s fate and reduce the trial itself to a mere
formality.”
The court has thus ruled that any identification of an uncounseled
accused made in a police line-up, or in a show-up for that matter,
after the start of the custodial investigation is inadmissible as
evidence against him.
Here, Escordial was identified by Michelle Darunda in a show-up on
January 3, 1997 and by Erma Blanca, Ma. Teresa Gellaver, Jason
Joniega, and Mark Esmeralda in a police line-up on various dates
after his arrest.
regarding this matter were asked of them, as required by Rule 132, §36
of the Rules on Evidence, Escordial must be deemed to have waived his
right to object to the admissibility of these testimonies.
Furthermore, the inadmissibility of these out-of-court identifications does not
render the in-court identification of accused-appellant inadmissible for
being the “fruits of the poisonous tree”.
This in-court identification was what formed the basis of the trial court’s
conviction of Escordial.
As it was not derived or drawn from the illegal arrest of Escordial or as a
consequence thereof, it is admissible as evidence against him. However,
whether or not such prosecution evidence satisfies the requirement of
proof beyond reasonable doubt is another matter altogether.
However, the Court acquitted Escordial as prosecution was not able to prove
his guilt beyond reasonable doubt.
This is due to the fact that the description of Escordial was different, and that
it was impossible for Michelle to actually see Escordial as she was
blindfolded.
“In dismissing the disparity between Escordial’s appearance and Michelle’s
description of her attacker, the trial court dwelt on the apparent
roughness of Escordial’s skin and the probability that Michelle might
have felt only the arch of the spinal chord of her assailant. However,
mere speculations and probabilities cannot take the place of proof
beyond reasonable doubt required by law to be established by the
prosecution.”
Also daw kasi si Erma yung nakakita kay Escordial bakit hindi siya yung
dinala initially to identify Escordial. And bakit one week late yung affidavit
ni Erma—it belies her claim na she saw Escordial even when she was
blindfolded.
PEOPLE v. ANG CHUN KIT (ARIELLE) After ascertaining that the bag contained more or less 1 kilo of shabu,
December 29, 1995 | Bellosillo, J. | Signature SPO2 handed the boodle money to Ang Chun Kit. Then, SPO2
casually lit a
PETITIONER: People of the Philippines
RESPONDENTS: Ang Chun Kit

SUMMARY: Ang Chun Kit, a member of a Hong Kong-based drug


syndicate, was arrested in a buy-bust operation in Manila. A NARCOM
agent posed as a buyer, and the fictitious sale took place in Cardinal
Santos Medical Center, where Ang Chun Kit sold 1 kilo of shabu for
P400,000. After the transaction, Ang Chun Kit was arrested right then
and there. He was found guilty beyond reasonable doubt for violation of
RA 6425 by the trial court.

On appeal, Ang Chun Kit contends that his signature in the receipt of the
items confiscated from him was inadmissible in evidence as he was not
assisted by a counsel. The SC agreed with Ang Chun Kit that such was
inadmissible in evidence, but still convicted Ang Chun Kit with the crime
because conformity to the questioned documents has not been a factor in
his conviction since his guilt has been adequately established by the
detailed testimonies of the officers who apprehended him.

DOCTRINE: A signature on the receipt or lists of items confiscated from


an accused is inadmissible in evidence if there is no showing that he was
then assisted by counsel.

FACTS:
Ang Chun Kit, a Chinese national reputed to be a member of a Hong
Kong-based drug syndicate operating in Metro Manila, was arrested
by NARCOM operatives in a buy-bust operation after he sold to an
agent for 400k a kilo of shabu.
On November 8, 1991, a Confidential Informer (CI) arranged a
transaction with a drug dealer interested in selling a kilo of shabu and
agreed to consummate the sale later that evening.
Chief Investigator Razon immediately organized a buy-bust team. Forty
bundles of genuine and counterfeit P100-bills were prepared with
each bundle supposed to contain P10,000.
In the afternoon, the team went to Cardinal Santos Medical Center. The
CI and SPO2 Jacobo who was carrying the bag of money proceeded
to the lobby of the hospital while others moved around to avoid
detection.
Ang Chun Kit arrived in a gray Toyota. He was met at the lobby by the CI
who introduced SPO2 Jacobo to him as the person interested to buy
shabu. After taking a look at the bundles of money, SPO2 and CI
followed Ang Chun Kit to the parking lot where he took out a blue SM
Shoemart plastic bag and handed it to them.
cigarette to signal the other NARCOM operatives to move in and
effect the arrest.
The team closed in and placed Ang Chun Kit under arrest and seized
the money from him. They also searched his vehicle and found on
the dashboard of his car 3 packets more of shabu.
SPO2 San Jose brought the shabu to the PC Crime Laboratory where
the chemist confirmed the substance found in the bag. Ang Chun Kit
refuted the charges and explained that his presence at the hospital
was because he was preparing to have dinner with friends in
Greenhills, when one of his friends asked him if he could ride with
him to visit a sick friend at the Cardinal Santos Medical Center.
Ang Chun Kit dropped his friends at the lobby, and he proceeded to the
parking lot to answer the call of nature. Then, he lit a cigarette when
some 30 minutes lateer, plainclothsmen with guns drawn suddenly
appeared from nowhere and arrested him and his friend without
informing them the reason for their arrest.
The security guard on duty testified that from the time he took his post
that night until he left there was no untoward incident at the hospital
lobby. His statements were corroborated by his supervisor.
Ang Chun Kit concludes that if there was an unusual incident at the
lobby, then the security guard would have noted it in their logbook.
The RTC, giving credence to the testimonies of the prosecution
witnesses, found Ang Chun Kit guilty of selling shabu in violation of
RA 6425.
Ang Chun Kit maintains his innocence and faults the RTC in not holding
that the crime could not have been committed under the
circumstances narrated by the arresting officers and that the alleged
buy-bust operation was a frame-up and the evidence merely planted.
Moreover, he argues that the prosecution was not able to prove his guilt
beyond reasonable doubt since every piece of evidence presented
against him is tainted with constitutional infirmities.

ISSUE/s:
WoN Ang Chun Kit’s signature on the receipt of items confiscated is
inadmissible in evidence for not being assisted by a counsel – YES;
but still guilty of the crime charged.

RULING: The decision of the trial court finding Ang Chun Kit guilty beyond
reasonable doubt of selling shabu is AFFIRMED.

RATIO: (Points relevant to our topic starts at Ratio #22)


The crux of the appeal hinges on the credibility of witnesses. In People v.
Co, the Court held that it is doctrinally entrenched that the evaluation
of the testimonies by the trial court is received on appeal with the
highest respect because such court has the direct opportunity to
observe the witnesses on stand.
When the issue is one of credibility of witnesses, the appellate courts will drug pushing, what is involved in this case is not a measly sum
generally not disturb the trial court’s findings. In the case at bar, the of money and a small
Court does not see any fact of substance and value which may have
been overlooked by the trial court.
Ang Chun Kit underscores what he perceived to be a flipflopping stance
of poseur-buyer SPO2 Jacobo. In one instance, Jacobo said that he
saw the shabu at the hospital lobby contrary to the version of the
prosecution witnesses that the prohibited drugs were taken from the
trunk of the car.
The Court does not consider the supposes inconsistencies substantial or
of such nature as to cast serious doubt on the credibility of the
prosecution witnesses. They only appear to be honest lapses which
do not impair the intrinsic credibility of their testimonies.
For sure the alleged inconsistencies do not detract from the established
fact that Ang Chun Kit was caught in flagrante delicto as a result of a
buy-bust operation.
In every prosecution for illegal sale of dangerous drugs what is material
and indispensable is the submission of proof that the sale of illicit
drug took place between the seller and the poseur buyer.
Ang Chun Kit submits that it is beyond human comprehension how such
a big transaction, illegal at that, could be perfect in front of the
watchful eyes of so many people. The Court can comprehend.
From the testimony of the prosecution witnesses, the exchange was casual
and swift: Ang Chun Kit was introduced to the poseur-buyer by the CI;
he was shown the money; he passed on the prohibited drug to the
poseur-buyer.
The selling of regulated or prohibited drugs to complete strangers, openly
and in public places, has become a common occurrence, a sad fact
which the Court has taken notice of and attributed to the growing
casualness of drug pushers in the pursuit of their clandestine activity.
Ang Chun Kit then harps on the testimonies of his witnesses, the security
guards on duty who said that there was nothing untoward that
happened at the hospital lobby or premises.
If the Court were to believe these security guards in their version then all
the more should the Court discredit Ang Chun Kit himself who
narrated that while he was at the parking lot several armed men and
women suddenly swooped down on him and then arrested him for no
apparent reason.
Certainly, if these security guards were conscientious in the performance
of their duties, as how Ang Chun Kit would like them to appear, then
they should have noticed and noted in their logbook the arrest of Ang
Chun Kit in the hospital.
The defense also asks the Court to reject the story of the prosecution
that the shabu was contained in one plastic bag instead of several
small plastic bags as how drug dealers would normally pack the
prohibited drug for easy concealment.
The Court cannot yield to such contention. While the swiftness with
which the transaction was undertaken is reminiscent of small-time
quantity of drugs that could be packed in tea bags. established by the detailed testimonies of the officers who
The defense also faults the prosecution for its failure to present the marked apprehended him.
money and urges the Court to apply People v. Distrito. The reliance in
the case is misplaced.
The Court has often ruled that the absence of marked money used in buy-
bust operations does not create a hiatus in the evidence for the
prosecution.
Ang Chun Kit also takes to task the absence of a blotter report before the
buy-bust operation and the supposed failure of the apprehending officers
to seal the plastic bag of shabu upon its seizure.
There are trivialities which do not abate the fact that Ang Chun Kit was
arrested after he unlawfully sold the shabu. A prior blotter report and
the sealing of the plastic bag of shabu are not indispensable nor
required in buy-bust operations.
The defense also argues that the shabu found in the car is inadmissible in
evidence as it was procured through an illegal search and seizure, the
same having been found inside the car and not in the person of the
accused who was outside the car.
But the search inside the car was an incident of a lawful arrest. It must be
remembered that the accused was with a driver who was inside the car.
Upon the arrest of the accused, the arresting agents also had to
neutralize the driver inside the car who could be presumed at that
instance to be acting together and in conspiracy with the accused.
In People v. Figueroa, the Court held that the warrantless search and
seizure, as an incident to a suspect’s lawful arrest, may extend beyond
the person of the one arrested to include the premises or surroundings
under his immediate control.
The Court however agrees with Ang Chun Kit that his signature on the
receipt or lists of items confiscated from him is inadmissible in
evidence as there is no showing that he was then assisted by
counsel.
In People v. Mauyao, the Court held that the conformance to these
documents are declarations against interest and tacit admissions of
the crime charged, since merely unexplained possession of
prohibited drugs is punished by law. They have been obtained in
violation of his right as a person under custodial investigation for the
commission of an offense, there being nothing in the records to show
that he was assisted by counsel.
With regard to the Booking Sheet and Arrest Report, the Court held in People v.
Morico that when an arrested person signs a Booking Sheet and Arrest
Report at a police station, he does not admit the commission of an
offense nor confess to any incriminating circumstance.
The Booking Sheet is merely a statement of the accused’s being booked and
of the date which accompanies the fact of an arrest. It is a police report
and may be useful in charges of arbitrary detention. It is not an extra-
judicial statement and cannot be the basis of judgment of convictions.
Ang Chun Kit’s conformity to the questioned documents has not been a
factor in his conviction since his guilt has been adequately
Hence, even disregarding the questioned documents, the Court still finds
Ang Chun Kit guilty beyond reasonable doubt of the crime charged.
PEOPLE v. GALLARDO (IYA) Upon notice that Editha was gone, a report was made with the
February 17, 2000 | Davide Jr., C.J. | Right of Accused baranggay and a search for the child ensued.
As Gallarde was the last person seen with Editha, the searching team
proceeded to his residence.
PETITIONER: People of the Philippines
Subsequently Gallarde was seen wearing shorts in his own toilet.
RESPONDENTS: Radel Gallarde At past 10:00 in the evening during an intensive search for the then
missing Editha, her lifeless body was found in a shallow grave
SUMMARY: Gallarde was drinking at the house of the Talan’s with situated some distance behind Gallarde’s residence.
several of their neighbors. That night, he was last person seen Before Editha’s body was discovered, a searcher found a girl’s slipper,
conversing with he minor Editha Talan. among thickets seven meters away from Gallarde’s house. Another
searcher saw a second slipper of the same color and size as the first
When Editha was noticed to be missing, a report was filed with the one. Both slippers were Editha’s.
baranggay, to which a search for the child ensued. As there was a A third rubber slipper was thereafter found in the field, near it was an old
testimony that Gallarde was the last person seen with Editha, the slipper, 8-9 inches long and with a hole at the rear end.
searchers proceeded to his house. At first Gallarde was not in his house Gallarde was not at home when searchers went to look for him there,
but was later on found inside his toilet with muddied shorts. after Cabinta told them that Editha was last seen with Gallarde.
When Gallarde was discovered squatting in the dark toilet behind his
The body of Editha was found late that night. She was no longer wearing
house and beside the thickets, his shorts were up and on. His hands
clothes and was buried in mud. Gallarde was arrested as suspect and and knees were soiled.
was delivered to the PNP Camp. At the toilet he was asked the innocent question of where Editha was and
he answered revealingly, thus: “I did not do anything to her” and “I let
Two informations were filed against him for murder and for rape with
her go and brought her back to the dike and let her go home.”
homicide.The RTC only convicted him of murder. On appeal to the SC, the
When asked where he had been, as the toilet was first seen empty,
crime was changed to homicide for there was no qualifying circumstance
Gallarde said he was with Kiko and he slept at the latter’s house,
alleged. The SC also ruled that the photographs taken of the offender while
which answer Mario Bado promptly refuted saying, (LOL NI
already in the custody of the police despite without counsel present are
TRANSLATE NILA YUNG “putang in a mo” to english) “Vulva of your
admissible in evidence. mother . . . Kiko was with me drinking.”
Yanked out of the dark toilet near his own house, Gallarde joined Kgd.
DOCTRINE: Taking pictures of an accused even without the assistance
Mario Fernandez sans protest.
of counsel, being purely mechanical act, is not a violation of his On their way, they met policemen on board a vehicle to which Gallarde
constitutional right against self-incrimination. against self-incrimination. was turned over.
Two informations were filed against him, one for murder and another for
FACTS: the special complex crime of rape with homicide.
Gallarde, 18, and Editha, 10, were neighbors and friends, even as she At Camp Narciso Ramos of the PNP, the police informed him that he was
used to frequent his place. a suspect int he rape and killing of Edith Talan and he told them that
Both were at the Talan residence on the night of May 6, 1997 while he did not commit the crime.
neighbors indulged themselves in beer. The RTC found him guilty of murder, but not of rape for lack of proof of
Among said neighbors Cabinta saw them hand in hand by the toilet carnal knowledge. Hence, this appeal.
situated five (5) meters east of the Talan kitchen.
After Cabinta whistled he saw Gallarde run home towards north after ISSUE/s:
letting go of Editha’s hands. WON Gallarde is guilty of killing Editha Talan - YES
Neighbor Clemente also noticed that Gallarde disappeared, and that
Editha returned to the kitchen. RULING: Radel Gallarde is hereby found guilty beyond reasonable doubt of
Cabinta followed Editha back to the kitchen, and saw her holding a the crime of homicide.
kerosene lamp. She told him that she was going to look for “Dalpac”
and off she went in the same direction Gallarde took.
Gallarde wore short pants and rubber slippers at the drinking place.
RATIO:
The Supreme Court does not agree with the conviction of murder, only
homicide was committed for there was a lack of allegation of any
qualifying circumstance.
The Supreme Court nevertheless agrees with the trial court that the
evidence for the prosecution, although circumstantial, was sufficient
to establish beyond reasonable doubt the guilt of Gallarde for the
death of Editha.
The rules on evidence and precedents sustain the conviction of an
accused through circumstantial evidence, as long as the following
requisites are present:
there must be more than one circumstance;
the inference must be based on proven facts; and
the combination of all circumstances produces a conviction
beyond doubt of the guilt of the accused
DI KO ALAM SAN TO NANGGALING BIGLA PERO ETO YUNG
RELEVANT SA TOPIC The Supreme Court cannot agree with the
trial court’s rejection of the photographs taken of Gallarde
immediately after the incident on the ground that “the same were
taken while Gallarde was already under the mercy of the police”
Taking pictures of an accused even without the assistance of counsel,
being purely mechanical act, is not a violation of his constitutional
right against self-incrimination. against self-incrimination.
The constitutional right of an accused against self-incrimination
proscribes the use of physical or moral compulsion to extort
communications from the accused and not the inclusion of his body
in evidence when it may be material.
Purely mechanical acts are not included in the prohibition as the accused
does not thereby speak his guilt, hence the assistance and guiding
hand of counsel is not required.
The essence of the right against self-incrimination is testimonial compulsion,
that is, the giving of evidence against himself through a testimonial act.
ESTACIO v. SANDIGANBAYAN (ELIEL) After having opened three current accounts with three different banks,
March 6, 1990 | Paras, J. | Right to counsel Villasanta and the syndicate began their insidious operations.
Villasanta deposited three banks with his current account at the
PETITIONER: Jesus Estacio Y Estrella Traders Royal Bank, Pasig branch totaling P653,485.70
Two of these checks were drawn against his Current account with the
RESPONDENTS: Honorable Sandiganbayan
LUCENA branch of Solid Bank. It bears emphasizing that at the time
SUMMARY: Basicaly, Villasanta, Estacio and Valentino are in conspiracy these checks were issued, Villasanta had a mere P1,500 deposited
of defrauding the Central Bank and Solid Bank in the amount of with his Current Account with the Solid Bank, LUCENA branch.
P648,564.70. They start their scheme by opening seveal bank accounts The deposited checks were forwarded to the Clearing House of the
from different branches, and then issue checks. After the issuance of Central Bank of rht standard clearing thereof. At the Central Bank
checks, an insider in the clearing house will falsify the documents and the Clearing House were accused Manuel Valentino (who later on
Central Bank Manifest to provide a demand letter to the bank ordering the became a state witness) a bookkeeper detailed at the Clearing
credit of such amount. Estacio gave an extrajudicial confession regarding Office, Central Bank of the Philippines and Jesus Estacio, janitor-
his participation in the crime and contended that the same was not mesenger of the Central Bank of the Philippines who were tehn
admissible. He alleges that it was obtained through force and intimidation waiting for the demand envelope containing the checks deposited by
and that it was done without the assistance of counsel. Villasanta
As soon as the demand envelope arrived, Estacio got the same, placed it
inside his push cart and brought the envelope inside the comfort
The SC held that there was no evidence of force or intimidation in
obtaining the confession, rather it was given freely and voluntarily with room at the 4th floor of the Central Bank Building. Accused Estacio
the assistance of counsel. Although the counsel was not present during then waited for Valentino and Villasanta.
the whole investigation, it was cured when at the closing stage, the When Valentino and Villasanta arrived, the former took the demand
lawyer read the statement and talked to Estacio regarding the envelope and pulled out the checks in question and thereafter gave
consequences of the confession and thereafter signed it. Regardless of the same to Villasanta. Valentino got hold of the attached bank
the evidence, the testimonies show that Estacio is indeed guilty beyond clearing statement of Solid Bank, Lucena Branch.
The amount of P628,564 under the column ‘Amount Received’ opposite
reasonable doubt for the crime of falsification and his voluntary
the words ‘Traders Royal Bank’, was thereafter crossed out to make
surrended is not appreciated by the court.
it appear that the same was not received while the total amount of
DOCTRINE: But while it is true that Estacios’ waiver of his right to remain ‘P992,723.99’ appearing in the clearing statement was likewise
silent and to assistance by counsel was not made in the presence of crossed out and replaced by the figures ‘P344,238.29’
counsel, the defect was cured and the requirement laid down in the Galit After Estacio had altered the bank clearing statement, Valentino, the CB
case was substantially complied with when Estacio’s lawyer, Atty. bookkeeper, prepared a Central Bank Manifest and in the column
Madarietta, arrived at the closing stage of the interrogation, read the ‘Amount’ he placed the correct figures P992,723.99 in the line
opposite the Consolidated Bank and Trust Company
statement and talked to Estacio before the latter signed it.
As soon as this Manifest was assigned by prosecution witness Magsalin,
CB Chief of Divison, Valentino superimposed thereon the amount of
FACTS: P344,238.29 thereby making it appear that such was the only
Accused Romereo Villasanta (still at large) opened a Current Account amount received for Solid Bank, Lucena Branch
with the Lucena City Branch of the Solid Bank. In his information The demand envelope for Solid Bank, Lucena together with the altered
sheet, Villasanta represented himself as engaged in the construction Central Bank Manifest and the altered bank clearing statement, were
business. Villasanta deposited an initial amount of P1,500 thereafter sent to the Regional Central Bank Clearing House at
As part of the sinister plot, Villasanta opened another current account Lucena City.
with the Cubao Branch of the Solid Bank. Villasanta represented The Regional Central Bank Clearing House official, Valenzena, testified
himself anew as a businessman and owner of the Romero Villasanta that he noticed nothing irregular when he received the demand
Construction in QC. Villasanta deposited initially a check for P10,000 envelope for Solid Bank, Lucena, including the banking clearing
and cash in the amount of P10,020. statement and the Central Bank Manifest.
Evidence further shows that on August 25,1981, Villasanta opened a The current account in Solid Bank Lucena, in the name of Villasanta was
current account with the Traders Royal Bank, Pasig Branch. He accordingly credited witht eh amount of P653,485.70. This
presented himself as engaged in the construction business. He representes the amount of the 2 checks in the total sum of
made an initial deposit in the amount of P2,000. P648,564.70, while the P500,000
represented the third check deposited by Villasanta with Traders Royal But while it is true that Estacios’ waiver of his right to remain
Bank. silent and to assistance by counsel was not made in the
After the foregoing withdrawals and/or encashments, the balance of his presence of counsel, the defect was cured and the
account is P1,012.04 All told, the syndicate was able to defraud the requirement laid down in the Galit case was
Solid Bank, Lucea Branch and/or the Centrail in the total amount of
P648,564.70. This syndicate was ableto achieve by falsigying the
Central Bank Manifest and the celaring statement for Solid Bank,
Lucena.

ISSUE/s:
WoN the extrajudicial confession of Estacio is admissible – YES

RULING: MR denied. Guilty of conspiracy for Estafa through Falsification of


Public/Commercial documents

RATIO:
“We are informing you that you are under investigation in connection with
the complaint of the Consolidated Bank & Trust Corporation an the
Central Bank for alleged Estate Falsification committed at the Central
Bank of the Philippines. But before we ask you any question, you
must understand your legal rights. You have the right to remain
silent. You have the right not to give any statement if you do not
wish to. Anything you say may be used as evidence agasint you
in any proceedings. You are entitled to the assistance of
counsel of your own choice. If you cannot afford a lawyer and
you want one, a lawyer will be appointed for you before we ask
you any question. Now, after having been informed of your
rights, are you still willing to give a free and voluntary statement
and swear to tell the truth in this investigation.”
The foregoing clearly shows that the investigator had advised Estacio of
his constitutional rights.
People v. Galit, At the time the person is arrested, it shall be the duty of
the arresting officer to inform him of the reason for the arrest and
he must show the warrant of arrest, if any. He shall be informed of
his constitutional rights to remain silent and to counsel, and that
ny statement he might make could be used against him. The
person arrested shall have the right to communicate with his
lawyer, a relative or any one who chooses by the most expedient
means by telephone if possible ro by letter or messenger. It shall
be the responsibility of the arresting officer to see to it that this is
accomplished. No custodial investigation shall be conducted
unless it be in the presence of counsel engaged by the person
arrest by any person in his behalf, or appointed by the court upon
pettiton either of the detainee himself or by any one on his behalf.
The right to counsel may be waived but the waiver shall not be
valid unless made with the assistance of counsel, whether
exculpatory or inculpatory, in whole or in part shall be inadmissible
in evidence
substantially complied with when Estacio’s lawyer, Atty. Madarietta,
arrived at the closing stage of the interrogation, read the statement and
talked to Estacio before the latter signed it.
As the Sandiganbayan aptly ruled, if Estacio had not voluntarily waived his
constitutional rights prior to the investigation or had wanted to change his
mind by availing of his right to remain silent after his counsel arrived and
read the statement before he signed it, he could easily have refused to sign
the same and demand possession of the unsigned statement
The repudiation of Estacio of his confession on the ground that it was extracted
through force and intimidation is negated by the fact that the confession
contained details which only Estacio could have known.
Be that as it may, the point sought to be made by Estacio is of no moment. Even
without the extra-judicial confession, the evidence on record is sufficient to
sustain Estacio’s culpability as a co-conspiritor in the defraudation of the
bank.
Valentino, Estacio’s co-accused turned state witness, positively and categorically
testified on the knowledge and participation of Estacio in the felonious act
Anent estacio’s claim that Sandiganbayan erred when it held that Estacio was
part of the conspiracy, suffice it to say tat Estacio’s culpability as a co-
conspirator in the defraudation of the bank was amply demonstrated and
established by the evidence on record
PEOPLE v. DE JESUS (JP) Pat. Lorbes fetched a lawyer from CLAO, Atty. Oscar Saldivar, and inthe
September 2, 1992 | Nocon, J. | Right to Counsel latter’s presence, reduced the statements of the two accused to
writing.
PETITIONER: People of the Philippines Appellant Tupaz defended he was actually mauled, detained, and forced
by the police officers to admit the crime.
RESPONDENTS: Eduardo de Jesus y Liamsy, Carlos Tupaz y Pamintuan

SUMMARY: Accused de Jesus et al. were arrested for flagging down the
tricycle of Leonardo Garcia, stabbing the victim and stealing his measly
daily income. Eddie and Tupaz surrendered and came with the officers to
the police headquarters. The two accused were then interrogated without
counsel. Pat. Lorbes then subsequently fetched a lawyer to reduce such
statements into writing. RTC convicted the accused.

The SC acquitted the accused. The right to counsel attached on the start
on the investigation. A subsequent writing of such statements initially said
without counsel is still inadmissible. The initial interview by Pat. Lorves
was already a custodial investigation. Accused’s right to counsel was
violated.
DOCTRINE: The right to counsel attaches upon the start of an
investigation, i.e., when the investigating officer starts to ask questions to
elicit information or confession or admission from the accused.

FACTS:
Three unidentified men flagged down the tricycle of Leonardo Garcia.
After a few minute’s ride, Leonardo was stabbed 17 times on the
neck, trunk and on the extremities, causing his death. His meager
earning that day of P40 was taken from him.
A certain Violeta de Vera reported the incident to the Pasic Police
Station.
Pat. Lorbes went to the scene of the crime. However, he failed to gather
any information on the commission of the crime or identities of the
assailants since no one among those he interviewed witness the
incident.
Cpl. Limpo and Pfc. Zapat were dispatched then to a particular stall
along the public market to look for a certain “Eddie” and another
named Tupaz in connection with the crime.
Upon finding the two, the police officers asked them whether they had
knowledge of the stabbing incident, to which they allegedly answered
in the affirmative.
Accused Edgardo de Jesus, “Eddie,” then surrendered to the officers two
bladed weapons: one dagger and one stainless knif.
Both Eddie and Tupaz were brought by the officers to police
headquarters.
The two were then interrogated by Pat. Lorbes without the assistance of
counsel.
RTC of Pasig found accused Carlos Tupaz guilty of the crime of Robbery
with Homicide.

ISSUE/s:
WoN admissions obtained during custodial interrogations without the
benefit of Miranda warnings and in the absence of counsel are
admissible in evidence when the same are later reduced into writing
and signed in the presence of counsel – NO

RULING: Accued Tupaz is acquitted.

RATIO:
In this instant case, the investigating officer Pfc. Rogelio Lorbes admitted that
the two accused, Tupaz and de Jesus were turned over to him for
investigation on September 9, 1987 by the CID, interviewed them and
solicited from them facts and information surrounding the robbery hold-up
with homicide without the assistance of a lawyer.
The facts and information were later reduced to writing on September 10, 1987
in the presence of a CLAO lawyer, a certain Atty. Saldivar.
Considering such circumstances, there was an apparent violation
of the accused right to counsel.
The right to counsel attaches upon the start of an investigation, i.e.,
when the investigating officer starts to ask questions to elicit
information or confession or admission from the accused.
Appellant in this case was already arrested as one of the principal suspects
in the killing of Garcia when he was turned over to Pfc. Lorbes for
investigation.
Consequently, the “interview” conducted by Pfc. Lorbes cannot be
considered merely as a general inquiry but rather a custodial
investigation.
Custodial investigation is the stage where the police investigation is no
longer a general inquiry into an unsolved crime but has began to focus
on a particular suspect who had been taken into custody by the police
who carry out a process of interrogation that lends itself to elicit
incriminating statements.
It is when questions are initiated by law enforcement officers after a
person has been taken into custody or otherwise deprived of his
freedom of action in any signifact way.
As to the other constitutional pre-condition, particularly that of the
Miranda warning, the trial court was convinced that this was met in
as much as the sworn statements show tht this was compiled with.
The SC disagreed.
It is possible that the accused-appellant really took part in the robbery
hold-up, but the Court is not persuaded to the point of moral
certainty, which is the high standard of proof required for conviction.
If the SC disregards the extrajudicial confession, the SC finds that the
prosecution has no other evidence to sustain a finding of guilty
beyond reasonable doubt.
PEOPLE v. BOLANOS (HENRY) Through this admission the RTC found Bolanos guilty beyond reasonable
July 3, 1992 | Paras, J. | Rights of those under custodial investigatioin doubt of the crime of murder. Hence this appeal

PETITIONER: People of the Philippines


RESPONDENTS: Ramon Bolanos

SUMMARY: This case involves a case of the murder of Oscar Pagdalian.


It was alleged that one of his companions killed him, to which were
identified as Magtibay and Bolanos. When both were apprehended and
were taken in the police vehicle, Bolanos confessed that he was the one
who killed Pagdalian. This was put in evidence, which basically
determined the course of the RTC’s decision to convict him.

Upon appeal to the SC, it was held that he was not afforded his
Constitutional rights under Sec 12, and he confessed without assistance
of counsel, which renders his entire confession inadmissible in evidence.
With this being the sole ground of his conviction, the SC found it proper
to acquit him.

DOCTRINE: Being on board the police vehicle on the way to the Police
Station is already part of the custodial investigation. Consequently, the
right to be informed of the accused’s Constitutional right insofar as
custodial investigation is concerned is essential to render any confession
obtained during the ride admissible in evidence.

FACTS:
Patrolmen Rolando Alcantara and Francisco Dayao testified that they
proceeded to the scene of the crime of Marble Supply, Bulacan, and
upon arrival they saw the deceased Oscar Pagdalian lying on an
improvised bed full of blood withs tab wounds.
They inquired about the circumstances of the incident and were informed
that the deceased was with 2 companions, on the previous night, 1
whom was the accused had a drinking spree with the deceased and
the other (Claudio Magtibay) till the wee hours of the following
morning.
When they apprehended the accused Bolanos, they found the firearm of
the deceased on the chair where the accused was allegedly seated;
that they boarded him with Claudio on the police vehicle and brought
them to the police station.
In the vehicle, Bolanos admitted that he killed the deceased because he
was abusive.
During trial, it was clearly established that the oral admission of the
appellant was given without the assistance of counsel as it was
made while on board the police vehicle on their way to the police
station.
ISSUE/s:
WoN Bolanos’ constitutional right in custodial investigatioin is violated –
YES

RULING: SC acquitted Bolanos.

RATIO:
A Manifestation was filed by the Sol Gen’s Office, with the position that
the lower court erred in admitting in evidence the extrajudicial
confession of appellant while on board the police patrol jeep. They
said, “Assuming that it was given, it was done in violation of the
appellant (Bolanos’) Constitutional right to be informed, to remain
silent, and to have a counsel of his choice, while already under
police custody”
The SC ruled that being already under custodial investigation while
on board the police patrol jeep on the way to the Police Station
where formal investigation may have been conducted, the
appellant should have been informed of his Constitutional
rights under Art 3 Sec 12 of the 1987 Constitution.
Considering the clear requirements of the Constitution with respect to the
manner by which confession can be admissible in evidence, and the
glaring fact that the alleged confession obtained while on board the
police vehicle was the only reason for the conviction,b esides
appellant’s conviction was not proved beyond reasonable doubt, this
Court has no response but t reverse the subject judgment under
review.
PEOPLE v. LUCERO (DANNAH) Three men alighted from the car, entered the Benz, and announced a
May 29, 1995 | Puno, J. | Right to Counsel hold up

PETITIONER: People of the Philippines


RESPONDENTS: Alejandro Lucero

SUMMARY: The car of Dr. Madrid, while on the way home with his driver,
was blocked by another car. Three men then entered Dr. Madrid’s car and
announced a hold-up. They got his rolex, rings, wallet and etc. They drove
around for a few hours. When the malefactors were about to alight, the
person beside the driver shot him. Dr. Madrid and his driver were brought to
a hospital, where his driver died.

Dr. Madrid reported the incident to the QC Police but they did not act on
the complaint, which prompted him to report to the Special Operations
Group of the Central Intelligence Service (CIS). Two months later, the
CIS’ efforst paid off. They interrogated the Echavez brothers and Lucero.
Pfc. Pursal, who was assigned to conduct investigation stated that prior
to interrogation, Lucero had already admitted that he participated in the
crime.

Atty. Peralta arrived on Lucero’s second day of detention and informed


Lucero of his constitutional rights. Lucero gave no reaction, but Atty.
Peralta believed Lucero understood. Atty. Peralta left for his friend’s wake
when interrogation started. The next day, 2 CIS agents with Lucero went
to Atty. Peralta’s residence and showed him Lucero’s extrajudicial
confession. Atty. Peralta examined and signed the extrajudicial
confession after Lucero affirmed that he knew of its legal implications.

The trial court acquitted the Echavez brothers but convicted Lucero of
robbery with homicide. The SC reversed, stating that there was no
substantial compliance with Lucero’s right to counsel. Atty. Peralta was not
present during the investigation, and it is most likely that Lucero gave his
confession under duress.

DOCTRINE: When the Constitution requires the right to counsel, it did


not mean any kind of counsel but effective and vigilant counsel.

FACTS:
On May 1988, private complainant Dr. Madrid spent the night at his
boarding house in Bago-Bantay, QC. He wanted to return to his
residence at Project 6 but his driver, Lorenzo Bernales, advised him
not to leave that night for Bernales overheard that the group of
Balbino and Bienvenido Echavez would rob him on his way home.
The next day at 7AM Dr. Madrid requested his driver to bring him home.
While en rout, a car overtook the Mercedes Benz they were riding
and blocked their way
The man who sat beside Dr. Madrid got his rolex, diamond rings, wallet,
necklace and bracelet.
After driving around a couple of hours, the malefactors stopped the car and
alighted. However, the man beside Dr. Madrid’s driver shot the driver at the
chest before fleeing.
Dr. Madrid and his driver Bernales were rushed to the hospital, but Bernales died
due to hemorrhage.
Dr. Madrid reported the incident to the QC Police but when no action was taken,
he filed a complaint with the Special Operations Group of the Central
Intelligence Service (CIS).
Two months later, the CIS’ efforts paid off. Led by Capt. Boac, they interrogated
Bienvenido Echavez.
Two days later, they apprehended Balbino Echavez and Alejandro Lucero.
Pfc. Pursal was assigned to conduct investigations. He declared that even before
the investigation started, Lucero verbally admitted his participation in the
crime and that8he was the one who shot Bernales, the driver of Dr. Madrid.
Pfc. Pursal informed Lucero of his constitutional rights to remain silent and to
counsel. Lucero needed a lawyer so Atty. Peralta appeared at the
investigator’s office.
He identified himself as the lawyer who was requested to assist Lucero and
inquired about the latter’s whereabouts. He was then directed to where
Lucero was.
Atty. Peralta conferred with Lucero. He also apprised Lucero of his constitutional
rights.
He explained to Lucero that he has the right to remain silent, that he is not
obliged to give any statement to the investigators, and that even if he has
already given a statement, he may refuse to sign it. He observed no reaction
from Lucero. Nonetheless, Atty. Peralta gathered the impression that Lucero
understood his advice.
When the investigator started asking the preliminary questions, Atty. Peralta left to
attend the wake of his friend, Capt. Emilio Dacanay, at Fort Bonifacio. He gave
word that in case of need, he could be reached at his residence.
The next morning, Lucero was accompanied by 2 CIS agents to Atty. Peralta’s
house. The extrajudicial statements of Lucero was presented to Atty. Peralta.
It was already signed by Lucero.
In the presence of the 2 agents, Atty. Peralta examined said statement and
explained to Lucero its legal implications. He asked Lucero whether he gave
the statements voluntarily. Lucero replied in the affirmative. Atty. Peralta then
signed the statement.
The three accused (Echavez, Echavez and Lucero) denied complicity in The right to counsel is constitutionalized because of the hostility against the
the crime charged. use of duress and other undue influence in extracting confessions from a
Lucero’s defense was an alibi. He testified that he was repairing suspect. Force and fraud tarnish confessions and render them
upholstery at home, and was with his cousin and wife. inadmissible.
He was apprehended more than 2 months after the commission of the The trial court did not display the required sensitivity to Lucero’s right to
crime. He was surprised when several un identified men accosted counsel. Indeed, it did not impose a rigorous respect for the right. It
him while he was walking towards his house. was satisfied that there was “substantial” compliance with the
They chased him, handcuffed and blindfolded him and pushed him into a requirements of right to counsel.
jeep. He was blindfolded the whole night and did not know where he However, Atty. Peralta arrived only on Lucero’s second night of detention.
was taken. The men turned out to be police officers. Later, he He admitted that he received no reaction from Lucero when he
identified one of the men to be Capt. Boak, head of the CIS Special explained Lucero’s rights. His impression was that Lucero
Operations Group. understood him.
He claimed that he was tortured. He was not informed of the offense for Worse, Atty. Peralta left Lucero in the custody of the CIS agents when his
which he was being investigated. Neither did they reveal the identity real interrogation started.
of the complainant. His attitude did not speak well of the importance he gave to his role as
He also denied knowing Dr. Madrid, the Echavez brothers and the other counsel to a person under custodial interrogation for the commission
accused in this case. He said he only met Dr. Madrid at the CIS of a very serious offense. It was during his absence that Lucero gave
Office during the police line-up. He was made to line-up four (4) an uncounselled confession.
times before Dr. Madrid finally identified him on the fourth time. In the presence of the 2 CIS agents, Atty. Peralta asked Lucero if he
Lucero also claimed he signed the extrajudicial confession under duress. understood the statements he gave and if he signed it voluntarily.
He likewise confirmed that Atty. Peralta was not present during his Lucero, of course, affirmed the voluntariness of the execution of the
actual custodial interrogation. confession.
The trial court acquitted the Echavez brothers but convicted Lucero. Atty. Peralta was satisfied and the trial court ruled that Lucero’s right to
counsel was not infringed. The Court disagrees.
ISSUE/s: When the Constitution requires the right to counsel, it did not mean any
WoN Lucero’s extra-judicial confession is admissible – NO kind of counsel but effective and vigilant counsel.
People v. De Guzman: in custodial investigation, the right to counsel
RULING: Judgment reversed. Lucero acquitted.
attaches from the moment the investigation starts, i.e., when the
investigating officer starts to ask questions to elicit information and
RATIO:
confessions or admissions from the accused.
On positive identification: The credibility of identification is open to doubt.
In the case at hand, at the crucial point when the interrogation was just
He had to participate in the police line-up 4 times before Dr. Madrid
starting, Atty. Peralta left Lucero to attend the wake of a friend. At that
was able to identify him.
Dr. Madrid was robbed in broad daylight, they drove around for 3 hours. critical stage, Lucero gave his uncounselled extra- judicial
The robbers did not wear masks. It was unlikely Dr. Madrid would not confession.
remember his face. Surely, such a confession where appellant was unprotected from
Moreover, initially Dr. Madrid claimed he could identify one of them, then mischief cannot convict.
took it back and said he could identify two. Then he swore he could
identify three.
On Lucero’s extra-judicial confession: The Constitution requires that a
person under investigation for the commission of a crime should be
provided with counsel.
PEOPLE v. PAROJINOG (ARIELLE) He also stated that the ambush was planned by Bert Dingding and Ronie
November 18, 1991 | Medialdea, J. | Right to Counsel Aljo;

PETITIONER: People of the Philippines


RESPONDENTS: Renee Parojinog

SUMMARY: Parojinog, together with the other co-accused, were charged


and convicted by the lower court with the murder of two policemen and
one PC soldier in the course of an ambush. During the investigation,
Parojinog was informed of his right to counsel. Atty. Fuentes was
assigned to help him with the case. Thereafter, Parojinog confessed that
he surrendered to Governor Sagrado in connection with the ambush of
the group of policemen and PC soldiers. In his extrajudicial confession,
Parojinog admitted that he was a member of the CPP-NPA.

At the trial, the prosecution presented the affidavit executed by Parojinog.


On appeal, Parojinog alleges that although he was given a lawyer before
the investigation, he had none during the investigation because Atty.
Fuentes allegedly went out and claimed that Atty. Fuentes was not his
choice for counsel, but the policemen’s choice.

The SC held that under Sec 12 (1) of the Constitution, a person under
investigation for the commission of an offense may choose his own counsel
but if he cannot afford the services of counsel, he must be provided with one.
However, in the case at bar, when Parojinog was assigned to Atty. Fuentes,
the former did not interpose any objection to it. Thus, Parojinog acquiesced
to the choice of the investigators.

DOCTRINE: A person under investigation for the commission of an


offense may choose his own counsel but if he cannot afford the services
of counsel, he must be provided with one.

FACTS:
Accused-appellant, Renee Parojinog was charged before the RTC,
together with his co-accused Ronie Aljo, Bert Dingding, Fe Mulo,
Miriam Mulo and 18 John Does for the crime of triple murder.
Corporal Godofredo Gallardo, then the designated Chief of the Integrated
National Police of Ozamiz, went to Barangay Sangay Daku. Along
their way, they were ambushed by the members of the CPP-NPA.
Parojinog was investigated by Pcpl. Benjamin de los Santos. During the
investigation, Parojinog confesse that he surrendered to Governor
Fortunato Sagrado in connection with the ambus of a group of police
and PC soldiers which resulted in the death of two policemen and a
PC soldier.
In Parojinog’s extra-judicial confession, he admitted that he was a
member of the NPA, and that while member of the NPA, he was
assigned to collect rice from people in the Darrio.
that the men carried armalite or garand while the women earned .38
caliber guns.
The records show that before the investigation, Pcpl. Santos apprised
Parojinog of his constitutional rights to counsel of his own choice and
if he did not have one, a certain Atty. Fuentes will be his lawyer.
Thereafter he was informed of his right to remain silent or not to
answer any questions if he did not like to answer. Finally, he was
warned that his statement may be used as evidence against him.
Atty. Fuentes assisted Parojinog during the entire investigation which
lasted for an hour. After the investigation, Parojinog signed his extra-
judicial investigation, which Atty. Fuentes also signed.
Two policemen escored Parojinog to the Office of the City Fiscal for him
to swear to his written confession. Before swearing in Parojinog, the
fiscal verified whether the statements were indeed his.
At the trial, the prosecution presented witnesses and offered the affidavit
executed by Parojinog (Exhibit A). The defense consisted only of the
testimony of Parojinog who said that he was in his house and worked
on his cornfield the whole day when the crime was committed.
Parojinog alleged that although he was given a lawyer before the
investigation, he had none during the investigation because Atty.
Fuentes went out and claimed that Atty. Fuentes was not his choice
but the policemen’s choice.

ISSUE/s:
WoN the CA erred in affirming the decision of the lower court and in
admitting and giving credence to the extrajudicial confession – NO

RULING: The judgment of conviction against Parojinog is AFFIRMED with


modiciations that he is sentenced to suffer three penalties of reclusion
perpetua.

RATIO:
It is evident that the fate of Parojinog in this case lies with the
admissibility or non-admissibility of his extrajudicial confession.
Parojinog imputes involuntariness to said confession because he was
allegedly denied his constitutional right to counsel during his
custodial investigation by the police.
He claims that the lawyer did not actually attend the investigation as the
latter went out and that the lawyer was not his choice.
The records of the case belie the claim. Atty. Fuentes, who was
presented by the prosecution, testified that Parojinog agreed to be
assisted by him during his custodial investigation.
The fiscal declared in court that she first informed Parojinog of his
constitutional right to be assisted by counsel to which Parojinog
replied that he was assisted by Atty. Fuentes during the investigation.
Despite Parojinog’s claim that Atty. Fuentes was not his choice, Section
12
of Art. III of the Constitution provides that a person under
investigation for the commission of an offense may choose his own
counsel but if he cannot afford the services of counsel, he must be
provided with one.
While the initial choice of the lawyer in the latter case is naturally
lodged in the police investigators, the accused really has the
final choice as he may reject that counsel chosen for him and
ask for another one.
In the case at bar, the records show that no objection was voiced by
Parojinog throughout the entire proceedings of the investigation and
afterwards when he subscribed to its veracity before the City
Prosecutor.
Thus, Parojinog apparently acquiesced to the choice of investigators. He
complained for the first time that Atty. Fuentes was not his choice
only during trial. Thus, it was too late.
Worthy to note is the fact that Parojinog did not categorically claim that
he was intimidated or forced to give his extrajudicial confession.
Neither did he claim that he was promised any reward or leniency.
The settled rule is that a confession is admissible until the accused
successfully proves that it was given as a result of violence,
intimidation, threat or promise of reward or leniency.
For Parojinog’s failure to successfully assail the voluntariness of his
confession, the Court cannot set it aside. Besides, his
uncorroborated alibi could not prevail over his own admission that he
was one of those who ambushed the group of police and PC
soldiers.
Alibi is a weak defense in the light further of the fact that Parojinog
himself surrendered to Governor Sagrado in connection with the
ambush. Parojinog did not assail this fact nor explain that he came
into the fold of the law in some other way. This circumstance serves
to underscore the veracity of the voluntary confession.
The alibi apparently is due to the change of heart of Parojinog upon
learning of the afflictive penalty for such grievous crime of murder.
PEOPLE v. PAMON (IYA) On March 14, 1987, Fortunato Pamon was arrested by virtue of a arrest
January 25, 1993 | Campos Jr., J. | Extrajudicial Confession for a murder charge against him in the RTC.
Fortunato Pamon, in the presence of Atty. Rubencio Ligorio of the
Citizens Legal Assistance Office (CLAO), executed before Pfc.
PETITIONER: People of the Philippines Roland Salatandre of the CIS a Confession marked as Exhibit “A".
RESPONDENTS: Fortunato Pamon, Gerson Dulang, and John Doe He admitted that he shot and killed Robert Te.
Furthermore, he implicated John Doe, alias "Dodo", Gerson
SUMMARY: While driving his truck, Robert Te (with his employees) Dulang, and Inocencio Feras.
encounter road problems. While he was attempting to get their vehicle He also narrated the circumstances leading to his being hired as
moving again, Te was suddenly shot causing his instantaneous death. a gunman, his meeting with "Dodo" for the first time in the
Pamon and Dulang were charged for the murder of Robert Te. house of Inocencio Feras, and how they got to Lipakan in the
morning of July 26, 1985. Furthermore, he said that he was
Pamon made extrajudicial statements admitting that he killed Te and promised P15,000.00 by Inocencio Feras as payment for the
implicating Dulang as the person who paid him to do the act. The trial job and that alias "Dodo" would receive the same amount
court admitted the extrajudicial statement as evidence and ruled that from Gerson Dulang.
Pamon and Dulang were guilty of murder. This extrajudicial confession was subscribed and sworn to before Judge
Vicente Aseniero on March 20, 1987. Fortunato Pamon reaffirmed his
On appeal, the question he admissibility of the extrajudicial confession. Confession during the preliminary investigation of the case.
The Supreme Court ruled that the Confession was admissible as to An information for murder was filed against Fortunato Pamon, as
Pamon only. Dulang is acquitted of the crime. principal by direct participation, Inocencio Feras and Gerson Dulang
as principals by inducemet, and John Doe, alias "Dodo" as
DOCTRINE: The rights of a party cannot be prejudiced by an act, accomplice.
declaration, or omission of another.An extrajudicial confession is binding Testimonies from several persons were presented during trial.
only upon the confessant and is not admissible against his co-accused. Evangeline Te (widow of Robert) pointing to Dulang’s intent to
have Robert killed.
FACTS: Rolando Salatandre testified that the extrajudicial Confession
Robert Te drove his 3/4-ton cargo truck from his residence in Sta. was voluntary and in accordance with the constitutional
Filomena to Sindutan, Roxas, Zamboanga del Norte to buy copra. mandate. This was reaffirmed by Judge Aseniero.
With him were Hipolito Andig, Victorino Jauculan, Orlando Tapia, and two Victoriano Jauculan, an employee of Robert Te, pointed to
other laborers. Fortunato Pamon as the gunman. Hipolito Andig likewise
While they were negotiating a road in Lipakan, the truck got stuck in the identified Fortunato Pamon as the killer.
mud. As a result, the trucks of Lily Wong and Gerson Dulang which The defense on the other hand presented the following testimonies:
were following his truck were blocked and could not proceed. Gerson Dulang who professed ignorance of the crime;
In order to pull the truck from the mud, Robert Te ordered his Raul Curativo, a neighbor of Fortunato Pamon, who described the
companions to tie the wrench of the truck to a coconut tree with a killer as "short, dark in complexion, with curly hair and was
cable. Robert Te remained behind the wheel to maneuver the truck. bearded and that at the time of the killing Pamon was plowing
While in that position, a man approached Robert Te and shot him on the his field.
bridge of his nose. The latter died instantly. Fortunato Pamon who denied the extrajudicial confession.
Another shot was fired and Cesar Siga was hit. Thereafter, the truck was The RTC convicted Pamon, Dulang, and John Doe of the murder of
burned by another man. Robert Te. Hence, the present appeal.
The gunman escaped and boarded the last truck which was the one Fortunato Pamon avers that the trial court erred in upholding the validity
owned by Gerson Dulang. of his arrest and the voluntariness and admissibility of his
Initial investigations by the police and the NBI pointed to the New extrajudicial Confession, and in not considering the testimony of a
People's Army (NPA) as the killers. witness, Raul Curativo, that Fortunato Pamon was not the killer.
However, subsequent investigations by the Criminal Investigation Service Both Pamon and Dulang assail the admissibility of the extrajudicial
(CIS) yielded Fortunato Pamon as the one responsible for Robert Confession.
Te's death.
ISSUE/s:
WON the extrajudicial confession can be sustained as evidence - YES
The decision in People vs. Alvarez is also relevant to the case at
RULING: WHEREFORE, the decision of the trial court is MODIFIED. bar. We said therein that "while it may be that a lawyer was
Conviction of Pamon is affirmed, while that of Dulang is REVERSED and provided by the police, Alvarez never signified to have a
lawyer of his choice."
SET ASIDE.
Thus, the trial court's findings that Fortunato Pamon was assisted
by a counsel of his choice is hereby sustained.
RATIO:
The SC is constrained to uphold the admissibility of his extrajudicial
Confession.
A confession constitutes an evidence of high order since it is supported by
the strong presumption that no person of normal mind would deliberately
and knowingly confess to a crime unless prompted by truth and his
conscience.
This presumption of spontaneity and voluntariness stands unless the
defense proves otherwise.
A confession is admissible until the accused successfully proves that it
was given as a result of violence, intimidation, threat, or promise of
reward of leniency.
The SC believes that Fortunato Pamon has not presented enough proof to
overcome this presumption. Apart from his testimony that he was
maltreated, Fortunato Pamon presented no other substantial proof to
buttress his claim
People v Damaso: "that where the defendants did not present evidence
of compulsion, or duress nor violence on their person; where they
failed to complain to the officer who administered their oaths; where
they did not institute any criminal or administrative action against
their alleged intimidators for maltreatment; where there appeared to
be no marks of violence on their bodies; and where they did not have
themselves examined by a reputable physician to buttress their
claim, all these were considered by this Court as factors indicating
voluntariness.”
Aside from holding that the extrajudicial Confession of Fortunato Pamon
had been voluntarily given, We also hold that it was given in the
presence and with the assistance of counsel.
The SC is well aware of the constitutional mandate that he counsel
present must not be just any counsel, but one who has been chosen
by the accused.
In a recent case, the SC affirmed the rule that ". . . no in-custody
investigation shall be conducted unless it be in the presence of counsel
engaged by the person arrested, by any person in his behalf or
appointed by the court upon petition either of the detainee himself or by
someone in his behalf".
Where counsel is provided for by investigators, the confession taken in
the presence of such counsel is inadmissible as evidence because it
fails to satisfy the constitutional guarantee.
But this doctrine recognizes certain exceptions. Where the counsel has
been appointed by the investigators with the conformity of the
confessant, the latter's confession is considered as valid and binding
upon him.
However, the SC cannot sustain the trial court's reasoning that if the confession
is not admissible against the accused, it will not also be admissible against
those who had been implicated therein.
But, if it is admissible against the former, then it will also be admissible against
the latter. This simply ignores the doctrine: RES INTER ALIOS ACTA ALTERI
NOCERI NON DEBET.
The rights of a party cannot be prejudiced by an act, declaration, or
omission of another.
An extrajudicial confession is binding only upon the confessant and is not admissible
against his co-accused.
This is so because the co-accused has no opportunity to cross-examine the
confessant and thus, as against him, the confession is hearsay.
PEOPLE v. BAELLO (ELIEL) The police were able to recover the missing television set from the house
July 1, 1993 | Padilla, J. | Right to counsel of Eugenio Tagifa the husband of Baello’s sister. Tagifa was brought
to the police station for questioning. Tagifa executed a “Sinumpaang
PETITIONER: People of the Philippines Salaysay”
RESPONDENTS: John Amet Baello y Guintivano @ “TOTONG”

SUMMARY: The house of Eustaquio Borja was robbed and their daughter
was killed during the incident. When the police managed to recover the stolen
proeprties from Borja in the house of Tagifa, the latter admitted that Baello
was the one liable for the crime. Baello was caught and invited for
questioning, he was stated his rights, and when he was told if he could afford
a lawyer, he replied in the negative. As such, he was assigned a lawyer, Atty.
Generoso. He confided with the Atty. Generoso, and confessed to the crime
of robbery but not the killing. After which, he contends that the extrajudicial
confession is inadmissible because he was not given fully and duly the
assistance of counsel. Hence this petition.

The SC held that the sworn affidavit of his confession is not constitutitonally
infirm. He was assisted by counsel, and as stated therein, he voluntarily
confessed to the crime and admitted that he was told of his rights and the
consequences of his confession. Thus, the document is admissible as
evidence. There is also a presumption of regularity of the performance of
Atty. Generoso. Baello was not able to overthrow the presumption that he is
remiss of his duty as counsel. And in relation to the crime of killing, the
conspiracy was proven from the actions.

DOCTRINE: Any person under investigation for the commission of an


offense shall have the right to be informed of his right to remain silent and
to have copetent and independent counsel preferably of his own choice.
If the person cannot afford the services of counsel, he must be provided
with one. These rights cannot be waived exept in writing and in the
presence of counsel.

FACTS:
Brgy. Captain Eustaquio R. Borja awoke to find out that the front door of
his residence at Pasig, Manila, was open and that their television set
in the sala was missing. Eustaquio told his wife about what he saw
and together they proceeded upstairs to the 2 nd floor to check on
their 22 year old daughter, Veronica Borja. They noticed that the door
to her room was open.
Upon entering the room, they were shocked to find the bloodied corpse
of their daughter lying bed. The window of her room was open.
Eustaquio instructed his wife not to touch the body while he
summoned the authorities. He proceeded to the Brgy. Hall from
where called the police.
The couple later discovered that a cassette player, a camera, and
various pieces of jewelry in their daughter’s cabinet, all worth about
P50,000.00, were likewise missing
wherein he pointed to Baella as the person who had plaed the People v. Parojinog, It is very clear from the aforequoted provision
television set under the stairs of his house. that
Baello was captured in Bankla, Makati by elements of the Intelligence
and Special Operations Unit (ISOU) of the Pasig Police and brought
to the police station. He made an oral admission of his participation
in the commission of the crime.
Baello was asked if he could understand, read, and write Tagalog, and
he replied that he could. Baello was likewise asked if he could afford
the services of counsel; he answered that he could not.
Upon beings asked if he was willing to avail of the services of Atty.
Generoso of the Public Attorney’s Office, Baello replied in the
affirmative. Atty. Generoso tehn brought Baello away from the police
investigators so that the two of them could talk privately.
Baello then told Atty. Generoso that he was the one who took the
television set but denied having killed Veronica Borja. Afterwards, the
police started the formal investigation of Baello I the presence of Atty.
Generoso.
Baello have his statement before the police ad this was reduced into
writing and marked at the trial. Atty. Generoso read the statement to
Baello and then let the accused read it himself. Baello signed the
statement, after which Att. Generoso also signed the same.
Baello contends that the confession is inadmissible in evidence, because
he was not fully and duly assisted by counsel. Hence this petition.

ISSUE/s:
WoN the extra-judicial admission of Baello is admissible – YES

RULING: Guilty of the crimes of Robbery with homicide. Confession is


admissible.

RATIO:
In his first assigned error, Baello maintains that he was not “fully and duly
assisted by a counsel engaged by him.” Hence, his extra judicial
confession is constitutionally infirm and inadmissible in evidence.
The records of the case, however, clearly belie this allegation of Baella.
While it is true that Atty. Generoso was not initially his counsel of
choce, the fact remains that after Baello was asked if he could
afford the services of counsel and he answered in the negative, he
was informed that he would be provided with one to assist him
during the investigation.
He then voluntarily accepted the services of Atty. Generoso. This was in
compliance with paragraph 1, Sec 12, Art III of the Constitution:
Any person under investigation for the commission of an
offense shall have the right to be informed of his right to
remain silent and to have copetent and independent
counsel preferably of his own choice. If the person cannot
afford the services of counsel, he must be provided with
one. These rights cannot be waived exept in writing and in
the presence of counsel.
a person under investigation for the commission of an offense may choose
his own counsel but if he cannot afford the services of counsel, he must
be provided with one. While the initial choice of the lawyer in the latter
case is naturally lodged in the police investigaros, the accused really has
the final choice as he may reject the counsel for him and ask for another
one.
The sworn statement completely belies the allegation that Baello was not
fully assisted by Atty. Generoso during the investigation. Said
document discloses that Baello was informed of his constitutional
rights by Atty. Generoso in extensor.
Atty. Generoso conferred with Baello, warned the latter of the
ocnsequences of his confession and even advised him not to
make any; however, Baello insisted on going ahead with his
confession, although he only confessed to the robbery
It was only after the said conference that Baello gave a statement. After it
was complete, Atty. Generoso again explained to him the cotents and
the adverse effects of his confession, but Baello found himself at
ease with his conscience by voluntarily affixing his signature therein.
Atty. Generoso, as an officer of the PAO, would not have affixed his
signature in the extrajudicial confession as counsel for the accused
had he known of any infirmity in its execution. If he did so he would
have been remiss in the performance of his duty and unfaithful to his
office. But there must be convincing proof of that for he has in his
favor the presumption of regularty in the performance of his duty.
People v. Layuso, This court denounces in the strongest terms
possible the widespread misconception that the presence of a
lawyer under the ‘right to counsel’ provision of the Constitution
is intended to stop an accused form saying anything which
might incriminate him. The right to counsel is intended to
preclude the slightest coercion as would lead the accused to
admit something false. The lawyer, however, should never
prevent an accused from freely and voluntarily telling the truth.
Whether it is an extra-judicial statement or testimony in open
court, the purpose is always the ascertainment of truth.
Indeed, Baello delibereately and carefully confined his participation
to the robbery, and the prosecution had no direct evidence that
Baello took part in the killing of Veronica. But Baello is not
thereby absolved from any liability for her death.
Once conspiracy is established between Baello and Jerry in the
commission of the crime of robbery, Baello would be equally
culpable for homicide committed by Jerry on the occasion of
the robbery, unless the former proved that he endeavored to
prevent Jerry from committing homicide.
PEOPLE v. AGUSTIN (JP) In the morning of 10 February 1987, “Jimmy” who turned out to be
January 25, 1995 | Davide, Jr., J. | Right to Counsel appellant Jaime Agustin, was picked up in Sto. Tomas, Pangasinan,
by military personnel. He narrated in the office of the City Fiscal his
PETITIONER: People of the Philippines knowledge of the shooting.
RESPONDENTS: Jaime Agustin, Wilfredo Quiaño et al. The defense presented the appellant Agustin and his wife, Elizabeth Agustin.
Appellant Agustin, who is a farmer and whose highest educational
SUMMARY: Accused Agustin was convicted by the RTC of two counts of attainment was grade four, impugned the validity of his extrajudicial
murder for being in conspiracy with the shooting of the famiy of Dr. statement.
Napoleon Bayquen, a dentist. He claimed that his extrajudicial confession Agustin claimed that while he was buying fertilizer for his farm, he was
should inadmissible because such was obtain by making him kneel on met by two armed men who took him to their car. He was made to
gunpoing and forced to admit such. He also claimed that the counsel kneel at gunpoint in order to force him to admis his involvement in
given to him was not his choice. the shooting, which he finally did out of fear.
While he was giving his statement at the fiscal’s office, the armed men
The SC ruled that such extrajudicial confession is indeed inadmissible. stayed with him and their presence deterred him from telling the
Accused only finished fourth grade and was a farmer. Although the rights investigating fiscal that he was being threatened.
were recited to him, there is no evidence that such is understood by him. He further declared that although he was given a lawyer, Atty. Reynaldo
The counsel given to him likewise was ineffective. The counsel did not Cajucom, to assist him, he, nevertheless, asked for his uncle who is
explain the accused’s rights in the dialect of the accused which is Ilocano. a lawyer, Atty. Oliver Tabin, and that Atty. Cajucom interviewed him
The counsel was also the firm partner of the opposing private prosecutor. for only two minutes in English and Tagalog but not in Ilocano, the
The counsel also should have known that the accused’s unwarranted dialect he understands. Worse, he explained that the lawyer who
arrest is illegal for the crime was consummated five months ago. Agustin assisted him was a law partner of the opposing private prosecutor.
is acquitted. Finally, he asserted that he was promised by his captors that he would be
discharged as a state witness if he cooperates, but the plan did not
DOCTRINE: It is not enough for the investigator to merely repeat to the push through because his co-accused, Quiaño, escaped.
person under investigation the provisions of Section 20, Article IV of the The RTC gave scant consideration to the alleged force and intimidation.
1973 Constitution or Section 12, Article III of the present Constitution; the Agustin was convicted of two counts of murder. The other charges
former must also explain the effects of such provision in practical terms, were dropped for insufficiency of evidence. Hence, this appeal.
e.g., what the person under investigation may or may not do, and in a ISSUE/s:
language the subject fairly understands. WoN Agustin’s extrajudicial statement is inadmissible as evidence for
being obtained in violation of Sec. 12, Art. 3 – YES
FACTS: RULING: Accused Agustin is acquitted.
Five separate informations. Accused Agustin et al were charged with
murder, with frustrated murder, and with attempted murder. RATIO:
Dr. Napoleon Bayquen, a dentist, together with his son, his son’s The extrajudicial admission. not extrajudicial confession (to be explained
girlfriend and his daughter, and Danny Ancheta, a family friend, were below), of the appellant Agustin, which is the only evidence of the
on their way aboard their Brasilia (car) to the doctor’s residence. prosecution linking him to the commission of the crime charged, is
While they were cruising along Malvar Street, a man came out from the
wholly inadmissible because it was taken in violation of Sec. 12, Art.
right sie of a car parked about two meters to the church.
3 of the Constitution.
The man approached the Brasilia, aimed his armalite rifle through its
Before continuing it is wise to differentiate between extrajudicial confession
window, and fired at the passengers.
and admission. Acc. to Secs. 26 and 33 , Rule 30 of the Rules of Court,
The gunman then sped away.
in a confession there is an acknowledgment of guilt. Admission on the
All those in the car were hit and Dr. Bayquen and his son’s girlfriend died other hand, is usually applied in criminal cses to statement of facts by
on the spot. Dr. Bayquen’s head was blown off. the accused which do not directly involve acknowledgment of guilt or his
Accussed Quiaño, an alleged former military agent confessed during the criminal intent.
investigation that he was the triggerman in the fatal shooting. He Nevertheless, when what is involved is the issue of admissibility in
implicated his fellow military agents as the bagman. A certain Jimmy evidence under Section 12, Article III of the Constitution, the
was the provider of the armalite. distinction is irrelevant because Paragraph 3 thereof expressly refers
to both confession and
admission. Atty. Cajucom was in fact foisted upon the appellant, for as shown in
the above-quoted portion of Exhibit “C,” the City Fiscal
In Morales vs. Enrile, this Court, applying Section 20, Article IV of the
immediately suggested
1973 Constitution, laid down the duties of an investigator during
custodial investigation and ruled that the waiver of the right to
counsel would not be valid unless made with the assistance of
counsel.
It is not enough for the investigator to merely repeat to the person under
investigation the provisions of Section 20, Article IV of the 1973
Constitution or Section 12, Article III of the present Constitution; the
former must also explain the effects of such provision in practical
terms, e.g., what the person under investigation may or may not do,
and in a language the subject fairly understands.
In further ensuring the right to counsel, it is not enough that the subject is
informed of such right; he should also be asked if he wants to avail of
the same and should be told that he can ask for counsel if he so
desires or that one will be provided him at his request.
If he decides not to retain counsel of his choice or avail of one to be provided
for him and, therefore, chooses to waive his right to counsel, such waiver,
to be valid, must be made with the assistance of counsel.
The extrajudicial admission of the appellant, contained in twenty-two
pages of yellow pad, does, indeed, appear to be signed by him and
Atty. Reynaldo Cajucom. What we find in these yellow pads are
stenographic notes.
“Since we (the SC) cannot even read or decipher the stenographic
notes in the yellow pads, we cannot expect the appellant
Agustin, who is a farmer and who reached only the fourth
grade, to read or decipher its contents.”
We (the SC) have to rely solely on the transcript and presume its
accuracy. A perusal of the transcript convinces us that the appellant
Agustin was not given a fair deal and was deprived of his rights
under Section 12(1), Article III of the Constitution.
Agustin was not explicity told of his right to have a competent and
idnependent counsel of his choice, specifically asked if he had
in mind any such counsel and, if so, whether he could afford to
hire his services, and, if he could not, whether he would agree
to be assisted by one to be provided for him.
He was not categorically informed that he could waive his rights to
remain silent and to counsel and that this waiver must be in
writing and in the presence of his counsel.
He had, in fact, waived his right to remain silent by agreeing to be
investigated. Yet, no written waiver of such right appears in the
transcript and no other independent evidence was offered to prove
its existence.
Secondly, Atty. Cajucom can hardly be said to have been voluntarily and
intelligently “accepted” by the appellant as his counsel to assist him in the
investigation. Atty. Cajucom’s presence in the Office of the City Fiscal at the
time the appellant was brought there for investigation is unclear to us.
the availability of Atty. Cajucom without first distinctly asking the appellant
if he had a counsel of his own choice and if he had one, whether he
could hire such counsel.
This left Agustin with no freedom to intelligently and freely pick his
counsel. For as earlier stated, he was not even asked if he had a
lawyer of his own choice and whether he could afford to hire such
lawyer; on the other hand, the City Fiscal clearly suggested the
availability of Atty. Cajucom.
Then too, present at that time were Capt. Antonio Ayat and Sgt. Roberto Rambac,
military officers. The presence of the military officers and the continuing fear
that if he did not cooperate, something would happen to him, was like a
Damocles sword which vitiated his free will.
His counsel, was likewise not an independent counsel. There are special
circumstances in these case which convince us that he was unable to
assist the appellant in a satisfactory manner.
For one, he admitted on cross-examination that at that time, and
even until the time he took the witness stand, he was an
associate of the private prosecutor, Atty. Arthur Galace, in
these and the companion cases.
Then we have misgivings on whether Atty. Cajucom was in fact
understood by the appellant when the former informed the
appellant of his constitutional rights in English and Tagalog
considering that the appellant, a fourth grader and a farmer,
could only understand Ilocano.
It appears to us that Atty. Cajucom did not actually impress upon the
appellant that he was one of the accused; rather, Atty. Cajucom made
the appellant believe that he was only a witness.
Atty. Cajucom knew or ought to have known that the arrest was unlawful.
The crim was committed five months ago. Such cannot be a valid
warrantless arrest for such is not “had just been comitted.” If he were
then truly moved by his duty to fully assist the appellant, he should
have forthwith taken the appropriate measures for the immediate
release of the appellant instead of allowing the City Fiscal to
investigate him.
PEOPLE v. GUILLERMO (HENRY) Campos eventually heard loud noise coming from the area of Keyser
January 20, 2004 | Quisimbing, J. | Custodial Investigation Rights Plastics, to which he thought was only made by the machines that made
the plastics.
PETITIONER: People of the Philippines At noon, Campos was interrupted in the performance of his duties when
RESPONDENTS: Eric Guillermo y Garcia he saw Guillermo look at one of the holes in the wall. According to
Campos, Guilllermo told him that he killed Victor Keyser and needed
SUMMARY: This case basically revolved around the killing of Victor his assistance to help him carry the corpse to the garbage dump
Keyser by Guillermo by means of chopping him into pieces by a saw where he could burn it.
after hitting him with a piece of wood, which caused his death. Shocked, Campos called the police who told him to immediately secure
the premises and not let Guillermo escape. 10 mins, police came,
He admitted his commission of the crime to Campos (security guard of with a team with SPO4 Bautista, SPO1 Reyes, and Police Aide
Greatmore), to the police (by his confession to the police during the Dizon, Jr. plus a police photographer, Felix Marcelo.
custodial investigation, without being assisted by counsel, and without The police tried to enter the premises of Keyser but found the gates
being informed of his right as he was only asked to read the same in a securely locked. They talked to Guillermo who later gave them keys
wall), and to the media (he was interviewed by GMA and ABS-CBN that opened the gate.
because the death of Keyser apparently shocked the nation. Once inside SPO4 Bautista and Reyes immediately approached
Guillermo, who said, “Sir, hindi ako lalaban, susuko ako, haharapin
RTC found him guilty of the murder of Keyser, and was to be afforded the koi to” (YEEESSSSSS!!!!!!!)
death penalty by lethal injection. Upon appeal to the SC, to which Guillermo pointed where the body was, after the police asked him. The
Guillermo contends that he is deprived of his constitutional rights during police then saw the body and saw that it was chopped into pieces,
custodial investigations, the SC held that his confession made to the with the head stuffed inside a cement bag.
police is rendered inadmissible, as it was not in assistance with counsel. When the police asked how he did it, according to the prosecution
However, this does not entail that Guillermo is acquitted. The SC noted witness, Guillermo said that he bashed the head of the victim with a
that even if the confession to the police is inadmissible, there is evidence piece of wood, then chopped the body with a carpenter’s saw (7 pcs).
presented by the prosecution that is strong and in support of the As to his motive, Guillermo replied that Keyser had been maltreating him
conviction of Guillermo (admission to Campos, as well as to the media, and his co-employees and so he did not regret his act of killing.
among others). Police brought Guillermo to the Antipolo PNP Station for further
investigation, where SPO1 Carlos conducted the investigation,
DOCTRINE: The right of a person under interrogation “to be informed” without apprising Guillermo of his constitutional rights and without
implies a correlative obligatioin on the part of the police investigator to providing him with the services of counsel.
explain and contemplates an effective communication that results in an Keyser’s death shocked the nation and he was interviewed by ABS and
understanding of what is conveyed. Absent that understanding, there is a GMA, to which he responded that he felt no remorse and that he did
denial of the right to be informed, as it cannot be said that the person has it because he was not paid for years, and was treated “like an
been truly informed of his rights. Ceremonial shortcuts in the animal”
communication of abstract constitutional principles ought not be allowed In the trial, Guillermo denied that he killed. He claims he was just a victim
for it diminishes the liberty of the person facing custodial investigation. of a frame up by the police, that they gave him the script, and that he
did not kill Keyser at all because the police did it. RTC did not believe
Guilllermo, he was found guilty and was sentenced to lethal injection.
Hence a petition for automatic review of the RTC decision was filed with
FACTS:
the SC.
Victor was the owner and manager of Keyser Plastic Manufacturing
Court, in Antipolo. He shared his building with Greatmore ISSUE/s:
Corporation, a manufacturer of faucets. A wall separated the two
WoN the constitutional rights of Guillermo as regards custodial
firms, made of hollow blocks, while the upper portion was made of
investigation was violated – YES
lawanit boards, with two large holes which allows a person to see
what’s on the other side. RULING: SC affirmed the CA rulign
March 22 1998, witness Campos, Greatmore’s security guard, saw
Guillermo enter Keyser Plastics. Campos ignored him as he new him RATIO:
to be one of Keyser’s trusted employees.
Guillermo contends that his conviction was based on inadmissible
evidence.
He claims that there is no clear showing that he was informed of his As provided for by People v. Dano, even if the admission or
constitutional rights nor was he made to understand the same by the confession of an accused is gospel truth, if it was made without
police investigators. assistance of counsel, it is inadmissible in evidence regardless of
E says he was only made to read said rights in printed form posed on the the absence of coercion or even if it had been voluntarily given.
wall at the police precinct. He was not provided with the services of
counsel during the custodial investigation, as admitted by SPO1
Reyes.
I view of no showing on record that he had waived his constitutional
rights, Guillermo argues that any evidence gathered from him,
including his alleged confession, must be deemed inadmissible.
The OSG counters that the evidence clearly shows that Guillermo
committed the crime. He admitted he killed his employer to Campos,
he admitted the same to the media, and Guillermo voluntarily
confessed to the killing even before the police could enter the
premises and even before any question could be posed to him.
OSG contends that not every statement made to the police by a suspect
in a crime falls within the ambit of constitutional protection. Hence, if
not made under “custodial investigation” or “under investigation for
the commission of an offense”, the statement is not protected by the
Bill of Rights.
However, in the SC’s perspective, the confession Guillermo made while
he was under investigation by SPO1 Reyes falls short of the
protective standards aid down by the Constitution.
Under Art 3, a confession to be admissible must satisfy the following
requisites:
Must be voluntary
Must be made with the assistance of competent and independent
counsel
Must be express
Must be in writing
In the present case, the testimony of SPO1 Reyes on cross-examination
cleary shows the cavalier treatment by the police of said constitutional
guarantees. This can only be inferred from the transcript of his testimony
that they in fact only assumed that Guillermo was aware of his
constitutional rights by simply basing it from the line Guillermo said,
“wala akong dapat pagsisihan”
Per SC, Guillermo’s confession at the police station lacks the safeguards
required by the Bill of Rights. The investigating officer made no serious
effort tto make Guillermo aware of his basic rights under custodial
investigation.
While the investigating officer was aware of Guillermo’s right to be
represented by counsel, the officer exerted no effort to provide him
with one on the flimsy excuse that it was a Sunday (wtf).
Despite absence of counsel, the officer proceeded with said
investigation. Moreover, the record is bare of any showing that
appellant had waived his constitutional rights in writing and in the
presence of counsel.
The right of a person under interrogation “to be informed” implies a
correlative obligatioin on the part of the police investigator to explain
and contemplates an effective communication that results in an
understanding of what is conveyed. Absent that understanding, there is
a denial of the right to be informed, as it cannot be said that the person
has been truly informed of his rights. Ceremonial shortcuts in the
communication of abstract constitutional principles ought not be
allowed for it diminishes the liberty of the person facing custodial
investigation.
However, this doesn’t necessarily mean Guillermo is acquitted just because
the evidence of his confession is inadmissible. For, constitutional
safeguards on custodial investigation (Miranda Principles) do not
apply to spontaneous statements, or those not elicited through
questioning by law enforcement authorities but given an ordinary
manner whereby the appellant verbally admits to having committed
the offense.
The rights enumerated in Art 3 Sec 12 are meant to preclude the slightest
use of the State’s coercive power as would lead an accused to admit
something false. But it is not intended to prevent him from freely and
voluntarily admitting the truth outside the sphere of such power.
The facts of this present case clearly show that Guillermo admitted the
commission of the crime not just to police people but also to private
individuals (even the media).
This leads the SC to hold that despite the inadmissibility of Guillermo’s
confession to the police, the prosecution has amply proven his guilt in
killing Victor Keyser.
PEOPLE v. MOJELLO (DANNAH) Since he was used to seeing them together, he did not find anything
March 9, 2004 | Ynares-Santiago J. | Extrajudicial Confession strange about this.

PETITIONER: People of the Philippines


RESPONDENTS: Dindo “Bobet” Mojello

SUMMARY: Rayco was having drinks with his friends. On his way home,
he saw his niece Lenlen with Mojello. They were always together so he
did not find anything strange about it. He proceeded home and on the
next morning they were informed that Lenlen’s corpse has been found.
She was naked and covered in bruises.

Mojello was then arrested while trying to board a motor launch. On an


investigation conducted by SPO2 Giducos, he admitted he was the
perpertraror of the deed. Mojello was assisted by Atty. Giduquio during
his custodial interrogation. During trial however he claims that his
confession is inadmissible as evidence because he did not voluntarily
give it, and he invoked the fruit of the poisonous tree doctrine.

The Court, although conceding that Mojello was initially not assisted by
counsel, claims this was cured by the assistance of Atty. Giduquio during
the latter part of the investigation. “Even though improper interrogation
methods were used at the outset, there is still a possibility of obtaining a
legally valid confession later on by properly interrogating the subject
under different conditions and circumstances thant those which prevailed
originally.”

The Court affirmed the trial court’s conviction however modified it, as the
homicide was not confessed to nor was there substantial evidence to
suppor the charge that it was also Mojello who killed Lenlen. (Sa rape
lang siya umamin)

PLS READ DISSENT. I SUPER AGREE W/ QUISUMBING.

DOCTRINE: The confessant bears the burden of proof that his


confession is tainted with duress, compulsion or coercion by
substantiating his claim with independent evidence other than his own
self-serving claims that the admissions in his affidavit are untrue and
unwillingly executed. Bare assertions will certainly not suffice to overturn
the presumption.

FACTS:
On December 15, 1996, Rayco was having drinks which included
Capacito and his wife, and the spouses Ilustrimo at the Capacito
Residence.
Rayco left the group to go home and on his way home, he saw his niece
Lenlen with appellant Mojello walking together.
On December 16, the Rayco family was informed that Lenlen’s body was
found at the seashore. Rayco saw the naked and bruised body of his
niece. His conscience at hime and he almost committed suicide.
Mojello was arrested while attempting to board a motor launch bound for
Cadiz.
On an investigation conducted by SPO2 Giducos, he admitted he was
the perpertraror of the deed. Mojello was assisted by Atty. Giduquio
during his custodial interrogation.
His confession was witnessed by the 2 barangay captains.
Bgy. Cap. Batobalanos testified that after the confession was executed,
the contents of the document were read to Mojello who voluntarily
signed it.
Dr. Sator who conducted the autopsy testified that Lenlen was indeed
raped, and that she died of asphyxiation.
Mojello’s defense: the confession which he executed was not freely,
intelligently and voluntarily entered into.
He also claims he was not knowingly and intelligently apprised of his
constitutional rights before the confession was taken from him.
He raises the fruit of the poisonous tree doctrine.

ISSUE/s:
WoN Mojello’s extrajudicial confession is admissible – YES
WoN Mojello is guilty of rape w/ homicide – Rape only

RULING: Judgment modified. Guilty of rape but acquitted of homicide.

RATIO:
Article III Section 12:
“Any person under investigation for the commission of an offense shall
have the right to be informed of his right to remain silent and to
have competent and independent counsel preferably of his own
choice. If the person cannot afford the services of counsel, he must
be provided with one. These rights cannot be waived except in
writing and in the presence of counsel.”
The Miranda Doctrine requires that:
Any person under custodial investigation has the right to remain silent
Anything he says can and will be used against him in a court of law
He has the right to talk to an attorney before being questioned and to
have his counsel present when being questioned
If he cannot afford an attorney, one will be provided before any
questioning if he so desires.
It may be observed that the Philippine law on custodial investigation has
evolved to provide for more stringent standards than what was originally
laid
out in Miranda v. Arizona improper waiver of the right to counsel as it was not made in
The extrajudicial confession executed by Mojello on December 23, 1996, writing and in the presence of counsel.
applying Art. III, Sec. 12, par. 1 of the Constitution in relation to Rep. However, the December 23, 1996 custodial investigation which elicited
Act No. 7438, Sec. 2 complies with the strict constitutional the Mojello’s confession should nevertheless be upheld for having
requirements on the right to counsel. In other words, the extrajudicial complied with
confession of the appellant is valid and therefore admissible in
evidence.
Mojello was undoubtedly apprised of his Miranda rights under the
Constitution.
The confession itself expressly stated that the investigating officers
informed him of such rights.
Atty. Giduquio testified that while he was attending a Sangguniang Bayan
session, he was requested by the Chief of Police of Sta. Fe to assist
Mojello.
Mojello manifested on record his desire to have Atty. Giduquio as his
counsel, with the latter categorically stating that before the
investigation was conducted and Mojello’s statement taken, he
advised Mojello of his constitutional rights.
Atty. Giduquio even told Mojello to answer only the questions he
understood freely and not to do so if he was not sure of his answer.
Atty. represented Mojello during the initial stages of the trial of the
present case.
The phrase preferably of his own choice does not convey the message
that the choice of a lawyer by a person under investigation is
exclusive as to preclude other equally competent and independent
attorneys from handling the defense; otherwise the tempo of
custodial investigation will be solely in the hands of the accused who
can impede, nay, obstruct the progress of the interrogation by simply
selecting a lawyer who, for one reason or another, is not available to
protect his interest.
People v. Continente: While of a lawyer in cases where the person
under custodial interrogation cannot afford the services of counsel·or
where the preferred lawyer is not available·is naturally lodged in the
police investigators, the suspect has the final choice as he may
reject the counsel chosen for him and ask for another one.
The right to counsel at all times is intended to preclude the slightest coercion
as would lead the accused to admit something false. The lawyer,
however, should never prevent an accused from freely and voluntarily
telling the truth.
Concededly, the December 17, 1996 custodial investigation upon
Mojello’s apprehension by the police authorities violated the Miranda
doctrine on two grounds: (1) no counsel was present; and (2)
Art. III, Sec. 12, par. 1.
Even though improper interrogation methods were used at the outset, there
is still a possibility of obtaining a legally valid confession later on by
properly interrogating the subject under different conditions and
circumstances thant those which prevailed originally.
The records of this case clearly reflect that Mojello freely, voluntarily and
intelligently entered into the extrajudicial confession in full compliance
with the Miranda doctrine.
Judge Jaca of the MCTC declared that he explained to Mojello the contents
of the extrajudicial confession and asked if he understood it.
On cross-examination, appellant Mojello claimed his life was threatened,
thereby inducing him to execute an extrajudicial confession, yet he
neither filed any case against the person who threatened him, nor did he
report this to his counsel.
He further claimed that he did not understand the contents of the confession
which was read in the Visayan dialect, yet he admits that he uses the
Visayan dialect in his daily discourse.
People v. Pia: Where appellants did not present evidence of compulsion or
duress or violence on their persons; where they failed to complain to officers
who administered the oaths; where they did not institute any criminal or
administrative action against their alleged maltreatment; where there
appears no marks of violence on their bodies and where they did not have
themselves examined by a reputable physician to buttress their claim, all
these should be considered as factors indicating voluntariness of
confessions.
The failure of Mojello to complain to the swearing officer or to file charges against
the persons who allegedly maltreated him, although he had all the chances
to do so, manifests voluntariness in the execution of his confessions.
People v. Enanoria: Another indicum of voluntariness is the disclosure of details
in the confession which could have been known only to the declarant.
The confessant bears the burden of proof that his confession is tainted with
duress, compulsion or coercion by substantiating his claim with
independent evidence other than his own self-serving claims that the
admissions in his affidavit are untrue and unwillingly executed. Bare
assertions will certainly not suffice to overturn the presumption.
The test for determining if a confession is voluntary is whether the
defendant’s will was overborne at the time he confessed.
The confession, having strictly complied with the constitutional requirements
under Art. III, Sec. 12, par. 1, is deemed admissible in evidence against
Mojello.
It follows that the admission of culpability made therein is admissible. It is
therefore not “fruit of the poisonous tree” since the tree itself is not Counsel provided by the police later was neither independent
poisonous. nor competent, but was biased against Mojello’s interest.
The categorical admission of Mojello to the crime of rape, coupled with The right to counsel means the right to competent and independent
the corpus delicti as established by the Medico-Legal Report and the counsel
testimony of Rogelio Rayco, leads us to no other conclusion than
that of Mojello’s guilt for the rape of Lenlen
However, on homicide: records do not show that Mojello admitted to the
killing. Neither is there sufficient circumstatial evidence to support
this.
Quisumbing, Dissenting: Must apply the strictest standard of review.
The issue is whether the extrajudicial confession of Mojello taken by the
police authorities could be validly admitted in evidence to convict
Mojello of a capital offense.
Numerous decisions of this Court hold that for an extrajudicial confession
to be admissible, it must be:
Voluntary;
Made with the assistance of competent and independent counsel;
Express;
In writing
During initial custodial investigation, Mojello had no counsel, nor
was he informed of his right to counsel.
The majority opinion concedes that the first custodial interrogation was
constitutionally infirm because of the absence of counsel. But the
ponencia considers the succeeding interrogation lawful, without any
infirmity, despite Mojello’s claim that he was boxed and mauled by
the police.
The ponencia holds that the police complied with the Miranda
requirements, given the fact that the presence of a counsel was
finally secured, and that the content of the extrajudicial confession
which Mojello signed was translated into Visayan dialect, which he
understood, before he affixed his signature thereon. On this point, we
cannot agree.
The facts unmistakably show a violation by the police of Section 17 of the
Bill of Rights that, “no person shall be compelled to be a witness
against himself.”
What happened was contrary to the prevailing doctrine that the accused
under custodial interrogation must continuously have a counsel
assisting him from the very start thereof.
People v. Delmo: Established is the rule that the moment the police try
to elicit admissions or confessions or even plain information from a
suspect, he should, at that juncture, be assisted by counsel, unless
he waives this right in writing and in the presence of counsel.
preferably of his own choice.
What emerges from a perusal of Atty. Giduquio’s testimony in the present
case is that the lawyer was merely picked out from a Sangguniang
Bayan meeting by the law enforcers themselves, for the sole purpose of
making it appear that Mojello was assisted by a counsel in making his
confession.
It can be seen that the close ties between prosecution witness Batobalonos
and Atty. Giduquio fortifies the conclusion that he was the chosen
counsel by the prosecution and the police for their own purposes,
contrary to the interest of Mojello, and to his great prejudice.
The confession of Mojello was not voluntary but extorted, a “fruit of the
poisonous tree.”
The mantle of protection afforded by the aforecited constitutional provision,
otherwise known as the “exclusionary rule,” is premised on the
presumption that the accused is thrust into an unfamiliar atmosphere
running through menacing police interrogation procedures where the
potentiality for compulsion, physical or psychological, is forcefully
apparent.
People v. Januario: Evidence gathered by virtue of an illegally obtained
confession is inadmissible. In that case, appellants first orally admitted
participation in the commission of the crime in Naga City and later
executed written confessions in Manila. This Court held that inasmuch as
their initial confessions are unconstitutional for being uncounselled, the
latter one becomes inadmissible as well for it was a product of the first.
In the case at hand, appellant Mojello’s first signed statement on December
17, 1996, taken illegally and involuntarily, without the presence of
counsel could not be admitted in evidence.
Neither should his flawed second statement taken without assistance of his
chosen counsel on December 23, 1996, be used against him. For the
alleged confession of Mojello is the undeniably unlawful consequence of
the first instance of custodial interrogation without counsel, a classic case
of a fruit of the poisonous tree.
PEOPLE v. SAYABOC (ARIELLE) PROSECUTION:
January 15, 2004 | Davide, Jr., J. | Right to Counsel

PETITIONER: People of the Philippines


RESPONDENTS: Benjamin Sayaboc, Patricio Escorpiso, Marlon Buenviaje,
Miguel Buenviaje

SUMMARY: Sayaboc, together with Buenviaje and Escorpiso, were


charged and convicted by the trial court for murder and homicide, in
relation to the killing of Joseph Galam. During custodial investigation,
Sayaboc informed Chief Barangay Tanod Carungao of his desire to be
assisted by his own counsel. However, Sayaboc could not name one,
therefore, Carungao assigned Atty. Cornejo as his counsel. During the
investigation, Carungao would stop questioning Sayaboc whenever Atty.
Cornejo would leave to go to the comfort room. That night, Sayaboc
executed an extrajudicial confession in Ilocano.

Sayaboc was then convicted of the crime charged based on his


extrajudicial confession. On appeal, Sayaboc claims that he was not
assisted by a competent and independent counsel, because Atty. Cornejo
remained silent during the investigation. The SC held that Sayaboc was
not afforded his constitutional
right to a competent counsel. While the Court was unable to rule on the
unsubstantiated claim that Atty. Cornejo was partial to the police, the facts
show that Atty. Cornejo remained silent throughout the custodial
investigation. Trial court attributed Atty. Cornejo’s silence to Sayaboc’s
garrulous nature and intelligence, as the latter wanted to be a central
figure in a drama, albeit tragic. The Court finds this explanation
unacceptable. It is immaterial. The waiver of a right is within the rights of
a suspect. What is lacking is a showing, to the satisfaction of the Court, of
faithful attempt at each stage of the investigation to make Sayaboc aware
of the consequences of his actions.

DOCTRINE: The right to a competent and independent counsel means


that the counsel should satisfy himself, during the conduct of the
investigation, that the suspects understands the import and
consequences of answering the questions propounded.

FACTS:
Appellant Benjamin Sayaboc was found guilty beyond reasonable doubt
by the lower court of the crime of murder. Appellant Marlon Buenviaje
was guilty as principal, and appellants Miguel Buenviage and Patricio
Escorpiso guilty as accomplices in the crime of homicide.
At their arraignment, the appellants pleaded not guilty to the charge of
murder.
While prosecution witness Abel Ramos was at a vulcanizing shop, he heard
Tessie Pawid screaming from across the road: “Enough, enough, enough!” In
front of her was Marlon Buenviaje and Joseph Galam, who were engaged in
a fisticuff.
By the time Pawid was able to subdue the two men by standing between them
and embracing Galam, Buenviaje’s face was already bloodied and Galam’s
shirt torn. As Buenviaje was about to leave, he turned to Galam, and with his
right index finger making a slicing motion across his throat, shouted,
“Putang-ina mo Joseph, may araw ka rin, papatayin kita.” Galam replied with,
“Gago, traydor, gold digger, halika.”
Three months later, Galam was shot to death at the Rooftop Disco owned by
him. According to a waitress, 3pm that day, a man whom she later identified
as Sayaboc, rang the doorbell and asked whether a woman wearing a green
shirt checked in. She answered in the negative.
Sayaboc then asked the waitress what time their boss arrives, and she answered
that she did not know.
Then, Tessi Pilar, the caretaker of the lodging house, stated that between 5:30
and 5:45 pm, Sayaboc, ordered a bottle of beer. When she gave the beer to
Sayaboc, the latter was angry and asked why it took her so long to bring the
beer. Subsequently, Galam’s vehicle arrived.
Shortly thereafter, they heard four gunbursts emanating from the ground floor.
When Jaramillo (waitress) looked down, she saw Sayaboc shooting Galam.
Meanwhile, that day, Joselito Parungao, Chief Barangay Tanod, was on his way
to a restaurant and saw Marlon Buenviaje with his father and Patricio
Escorpiso. The three were aboard a tricycle parked in a vacant lot between
the Rooftop Disco and Diego Theater.
While Parungao was waiting for his food, he noticed that the tricycle was still
parked. After getting his food, Parungao heard four gunshots from behind the
Rooftop building. He later saw a person, whom he later came to know as
Sayaboc, walking briskly toward the tricycle.
SPO4 Roberto Carungao assigned investigators to go to the scene of the crime
to gather evidence.
Pilar and Jaramillo identified Benjamin Sayaboc at the PNP Provincial
Headquarters as the gunman who shot Galam. Carungao was called to take
the statement of Sayaboc.
When he arrived, he saw Sayaboc being interviewed by reporters inside the
inbestigation room. He brought Sayaboc to the inner part of the room. Before
taking Sayaboc’s statement, Carungao informed him of his constitutional
rights.
Sayaboc told Carungao that he wanted to have a counsel of his own choice. But
since Sayaboc could not name one, Carungao asked the police officers to
get a lawyer. The police officers brought Atty. Cornejo of the PAO. Carungao
heard Sayaboc say, “okay” and continued his investigation, during which Atty.
Cornejo remained silent the entire time.
However, Carungao would stop questioning Sayaboc whenever Atty.
Cornejo would leave to go to the comfort room. That night, Sayaboc Jurisprudence provides that extrajudicial confessions are
executed an extrajudicial confession in Ilocano. presumed to be voluntary. The condition for this
He confessed to killing Galam at the behest of Marlon Buenviaje, and
presumption, howerver, is that the
implicated Miguel Buenviaje and Patricio Escorpiso. The confession
was
signed by Atty. Cornejo and attested to by a fiscal.
DEFENSE
Sayaboc interposed an alibi, and denied having met Atty. Cornejo or
having been informed of his rights. He testified to having been
beaten by police officers in the investigating room, who coerced him
to confess to having killed Galam.
Aside from his testimony, he submitted a handwritten statement and an
affidavit to support his claim of police brutality and retraction of his
confession.
The trial court found Benjamin Sayaboc guilty of the crime of murder with
treachery as the qualifying circumstance and craft and price as
aggravating circumstances. As for Buenviaje and Escorpiso, the
court held that the treachery employed by Sayaboc could not be
taken against them and therefore, declared them guilty of homicide
only.

ISSUE/s:
WoN the trial court erred in admitting in evidence the extrajudicial
confession of Sayaboc when it was taken without the assistance of a
competent and independent counsel – YES

RULING: The decision of the RTC is MODIFIED. Appellants Sayoboc and


Buenviaje are found guilty beyond reasonable doubt of the crime of
homicide.

RATIO: I didn’t include na discussion on treachery~


Appellants argue that the extrajudicial confession of Sayaboc may not be
admitted in evidence against him because Atty. Cornejo was not a
competent, independent, vigilant and effective counsel.
According to them, Atty. Cornejo was ineffective because he remained
silent during the entire proceedings. He was not independent, as he
was formerly a judge in the National Police Commission, which was
holding court inside the PNP Command of Bayombong.
No evidence was adduced during trial to substantiate the claim that Atty.
Cornejo used to be connected with the NAPOLCOM. Moreover, the
claim was made for the first time in the appeal, and was based
merely on an information furnished by defense counsel Atty. Castro
to Sayaboc’s counsel in the appeal.
prosecution is able to show that the constitutional requirements
safeguarding an accused’s rights during custodial investigation have
been strictly complied with, especially when the extrajudicial confession
has been denounced.
The rationale for this requirement is to allay any fear that the person being
investigated would succumb to coercion while in the unfamiliar or intimidating
environment that is inherent in custodial investigations.
Therefore, even if the confession may appear to have been given voluntarily
since the confessant did not file charges against his alleged
intimidators for maltreatment, the failure to properly inform a suspect
of his rights during a custodial investigation renders the confession
valueless and inadmissible.
In the case at bar, contrary to SPO4 Carungao’s claim that he conferred with
Sayaboc for half an hour informing him about his constitutional rights, the
extrajudicial confession provides only apart of an absence of an express
waiver of his rights, and the passing of information of the kind held to be in
violation of the right to be informed under Sec 12 Art III.
In People v. Jara, the Court explained that the stereotyped “advice” appearing in
practically all extrajudicial confessions which are later repudiated has
assumed the nature of a “legal form”.
The right to be informed requires the “transmission of meaningful
information rather than just the ceremonial and perfunctory recitation
of an abstract constitutional principle. It should allow the suspect to
consider the effects and consequences of any waiver he might make of these
rights. More so when the suspect is one like Sayaboc, who has an
educational attainment of Grade 4, is a stranger in Nueva Vizcaya.
The Court rules that Sayaboc was not afforded his constitutional right to a
competent counsel. While the Court was unable to rule on the
unsubstantiated claim that Atty. Cornejo was partial to the police, the facts
show that Atty. Cornejo remained silent throughout the custodial
investigation.
Trial court attributed Atty. Cornejo’s silence to Sayaboc’s garrulous nature and
intelligence, as the latter wanted to be a central figure in a drama, albeit
tragic. The Court finds this explanation unacceptable. It is immaterial.
The waiver of a right is within the rights of a suspect. What is lacking is a
showing, to the satisfaction of the Court, of faithful attempt at each stage of
the investigation to make Sayaboc aware of the consequences of his actions.
If anything, it appears that Sayaboc’s counsel was ineffectual for having been
cowed by his client’s enthusiasm to speak.
The right to a competent and independent counsel means that the counsel
should satisfy himself, during the conduct of the investigation, that the
suspects understands the import and consequences of answering the
questions propounded.
In People v. Deniega, the Court said that the desired role of a counsel
in the process of custodial investigation is rendered
meaningless if the lawyer merely gives perfunctory advice as
opposed to a meaningful advocacy of the rights of the person
undergoing questioning.
This is not to say that a counsel should try to prevent an accused from
making a confession. Indeed, as an officer of the court, it is an
attorney’s duty to seek the truth. However, counsel should be able to
explain the nature of the questions by conferring with his client and
halting the investigation should the need arise.
The duty of a lawyer includes ensuring that the suspect under custodial
investigation is aware that the right of an accused to remain silent
may be invoked at any time. Even the hardest of criminals have
rights that cannot be interfered with.
For these reasons, the extrajudicial confession of Sayaboc cannot be
used in evidence against him. However, the prosecution has
discharged its burden of proving his guilt for the crime of homicide. à
because of positive identification of witnesses juxtaposed with
Sayaboc’s bare denial and weak alibi.
PEOPLE v. OLERMO (IYA) They subsequently inquired with the Philippine overseas Employment
July 17, 2003 | Azcuna, J. | Right to Counsel Agency which imformed them that Olermo is not a licensed recruiter.
CASE 3: Complainant Alfred Bryant Berador testified that he was introduced

PETITIONER: People of the Philippines


RESPONDENTS: Marlene Olermo alias Marlene Tolentino

SUMMARY: Olermo was accused of illegal recruitment in large scale


and five counts of estafa.The RTC found her guilty of all charges. On
appeal to the SC Olermo alleges that during the hearing before the lower
court, she was given incompetent counsel for different persons
represented her in different hearing dates. She contends that because of
the change in counsels, her interests were not properly represented.

The SC ruled that the right to counsel is intended to preclude the


slightest coercion as would lead the appellant to admit something false.
It does not mean that the courts cannot appoint a lawyer for the counsel
should the need arise. If the appellant was to be given choice in whoever
they wish to represent them, the judicial process would be subject to
abuse by delay.

DOCTRINE: The words “preferably of his own choice” do not mean that
the choice of a lawyer by appellant is exclusive as to preclude other
equally competent and independent attorneys from handling the
defense.

FACTS:
Marlene Olermo aka Marlene Tolentino was accused of illegal
recruitment in large scale and five counts of estafa.
CASE 1: Complainant Napoleon C. Aparicio, jobless, testified that he
came to know appellant through his sister in February 1993.
Olermo informed Aparicio that he needed to pay her 40,000 for a work
permit and a plane ticket to Saipan where he is allegedly to be
employed.
Aparicio delivered to her the said amounts in two installments.
He was however not able to leave on the planned dates for it was moved
several times.
Suspicious he reported the matter to the NBI.
Olermo then pretended to refund the amounts paid by issuing a check,
which bounced upon presentation to drawee bank.
CASE 2: Complainant Ariston Villanueva and Mary Jane Aquino-
Villanueva testified that he read the advertisement of Olermo in a
newspaper in April 1993 offering assistance to those who would like
to work overseas. They paid P35,000 each.
Their departure however, kept on being postponed. They asked for a
refund, but was issued a check which bounced. Only P19,000 was
returned to them.
to Olermo bone of her partners in the agency. Because of these instances, Olermo claims that she was deprived of her
He was asked to pay a total of P24,000 as placement and processing right to competent counsel because the lawyers who represented her
fees for his employment in Japan. He was issued a receipt for each in the
payment made.
He was however not allowed to leave for Japan immediatel for he was
required to undergo a seminar to learn Nippongo for one week.
On the 4th day of the seminar, Olermo was arrested by the authorities.
Berador there learned that Olermo did not have a license to recruit
workers for overseas employment.
CASE 4: Frennie Majarucon was introduced to Olermo by her kumadre
named Elvie.
Olermo informed her that she had an available job in Hong Kong but would
need P45,000 for placement and processing fees and P2,000 for her
passposrt.
Majarucon was only able to give P22,000, which was evidenced by
receipts
Majarucon never left for Hong Kong. She inquired if she could be
refunded but was informed that Olermo was already in jail for illegal
recruitment.
DEFENSE: Olermo denied all the charges against her.
She alleged that she was engaged only in visa assistance.
She denied ever having represented herself as possessing authority to
deploy workers for overseas employment. She thus explained that
she only offered complainants Villanueva, Aquino-Villanueva,
Aparicio and Majarucon assistance in processing their tourist visas.
With respect to the accusation of complainant Berador, appellant alleged
that she was only helping him process his trainee’s visa.
The RTC found Olermo guilty beyond reasonable doubt for all 5 cases
abovementioned.
Hence, the appeal to the SC. One of the assigned errors by Olermo was
that the lower court erred in disregarding her right to have a
competent and independent counsel.

ISSUE/s:
WON Olermo was deprived of her right to counsel – NO

RULING: SC denied the claim of Olermo with regard to incompetent counsel

RATIO:
Olermo notes that during the presentation of the prosecution’s first
witness on August 11, 1993, she was represented by Atty. Hortensio
Domingo, who was not her retained counsel for the case.
During the hearing, Atty. Domingo manifested that Ilermo herself
requested him to represent her in that day’s hearing since her
counsel, Atty. Yuseco, was still in Cagayan.
During the second, third, fourth and fifth hearings, Olermo was
represented by another counsel, a de oficio one, a certain Atty.
Ricardo Perez, again because counsel for Olermo was not around.
abovementioned hearings were not familiar with her case and, hence, were
not able adequately to protect her interests.
Article III, Section 12, paragraph (1) of the Constitution provides:
“Any person under investigation for the commission of an offense
shall have the right x x x to have competent and independent
counsel preferably of his own choice, x x x.”
The right to counsel is intended to preclude the slightest coercion as
would lead the appellant to admit something false.
Moreover, the words “preferably of his own choice” do not mean that the
choice of a lawyer by appellant is exclusive as to preclude other
equally competent and independent attorneys from handling the
defense.
If this were so, the tempo of justice would be solely within the control of
appellant who could choose to impede the judicial process by simply
selecting a lawyer who, for one reason or another, is not available to
defend her.
TANENGGEE v. PEOPLE (ELIEL) Once said documents were forged and falsified, Tanenggee released and
June 6, 2013 | Del Castillo, J. | Right to counsel obtained from Metrobank the proceeds of the alleged loan and

PETITIONER: Carlos Tanenggee


RESPONDENTS: People of the Philippines

SUMMARY: Tanenggee, Metrobank Commercio Branch, was charged


with estafa through falsification. He prepared promissory notes and
cashier’s check under the name of Romeo Tan, a valued client in their
bank in the amount of P40,000,000. It was allegedly supposed to be a
loan for Tan, but it was appropriated by him personally. When Metrobank
conducted an internal audit, they found the anomaly and asked for
Tanenggee’s statement, and he gave an extrajudicial confession
confirming such action. He contends that the statement was obtained
without counsel and through force and intimidation. Hence this petition.

The SC held that evidence against him for the extrajudicial confession is
admissible. The right to counsel guaranteed by the constitution only
applies in custodial interrogation, wherein it was intiated by law
enforcement authorities. However, in this case, the confession was given
to administrative authorities, and thus it is valid. It was also declared
voluntary, as there was no evidence overthrowing the presumption that
the statement was obtained through force and intimidation. In summary,
Tanenggee was guilty of Estafa through Falsification.

DOCTRINE: The constitutional proscription against the admissibility of


admission or confession of guilt obtained in violation of Sec 12, Art 3 o
the Constitution, as correctly observed by the CA and the OSG, is
applicable only in custodial interrogation.

FACTS:
Five separate informations for estafa through falsification of commercial
documetns were filed against Tanenggee. The said informations portray
the same mode of omission of the crime but differ with respect to the
numbers of the checks and promissory notes involved and the dates and
amounts thereof
The RTC entered a plea of not guilty for Tanenggee after he refused to
enter a plea. The cases were tehn consolidated and jointly tried.
The prosecution alleged that on different occasions, Tanenngee caused
to be prepared promissory notes and cashier’s checks in the name of
Romeo Tan, a valued client of the bank since he has substantial
deposits in his account, in connection with the purported loans
obtained by the latter from the bank.
Tanenggee affixed, forged or caused to be signed the signature of Tan as
endorser and payee of the proceeds fo the checks at the back of the
same to show that the latter had ineeded endorsed the same for
payment. He handed the checks to the Loans clerk, Maria Miranda,
for encashment
misappropriated the same to his use and benefit.
After the discovery of the irregular loans, an internal audit was conducted
and an administrative investigation was held in the Head Office of
Metrobank, during which Tanenggee signed a written statement in
the form of questions and answers
For his defense, Tanenggee testified that he is a holder of a Masters
degree from the Asian Institute of Management, and was the branch
manager of Metrobank Commercio Branch from 1994 until he was
charged in 1998 with the above offense.
Tanenggee claimed that he was able to solicit Romeo Tan as a client-
depositor when he was the branch manager of Metrobank
Commercio. As a valued client, Romeo Tan was granted a credit line
for P40,000,000 by Metrobank.
Tan was also allowed to open a fictitious account for his personal use
and was assisted personally by Tanenggee in his dealings with the
bank. In the middle of 1997, Tan allegedly opened a fictititous
account and used the name Jose Tan.
Such practice for valued clients was allowed by and known to the bank to
hide their finances due to rampant kidnappings or from the BIR or
from their spouses.
Tanenggee claimed that Elevado, after the internal audit, asked him to sign a
paper in connection with the audit investigation; that he inquired what he
was made to sign but was not offered any explanation; that he was
intimidated to sign and was threatened by the police that he will be
brought to the precinct if he will not sign; that he was not able to consult
a lawyer since he was not apprised of the purpose of the meeting; and
that “just to get it over with” he signed the paper which turned out to be a
confession. Hence this petition.

ISSUE/s:
WoN the extrajudicial confession of Tanenggee is admissible – YES

RULING: Petition denied. Guilty of Estafa through Falsification.

RATIO:
The constitutional proscription against the admissibility of
admission or confession of guilt obtained in violation of Sec 12,
Art 3 o the Constitution, as correctly observed by the CA and
the OSG, is applicable only in custodial interrogation.
Custodial interrogation means any questioning initiated by law
enforcement authorities after a person is taken into custody or
otherwise deprived of his freedom of action in any significant
manner.
Indeed, a person under custodial investigation is guaranteed
certain rights which attach upon the commencement thereof,
viz: (1) to remain silent, (2) to have competent and independent
counsel preferably of his own choice, and (3) to be informed of
the two other rights above
In the present case, while it is undisputed that Tanenggee gave an
uncounselled written statement regarding an anomaly discovered in the
branch he managed, the following are clea: (1) the questioning was not
intiated by a law enforcement authority but merely by an internal affairs
manager of the bank; and (2) Tanenggee was neither arrested nor
restrained of his liberty in any significant manner during the
questioning.
Clearly, Tanenggee cannot be said to be under custodial investigation
and to have been deprived of the constitutional prerogative during
the taking of his written statement.
Remolona v. CSC, we declared that the right to counsel “applies only to
admissions made in a criminal investigation but not to those made in
an administrative investigation.”
Here, Tanenggee’s written statement was given during an administrative
inquiry conducted by his employer in connection with an
anomaly/irregularity he allegedly committed in the course of his
employment.
No error can therefore be attributed to the courts below in admitting in
evidence and in giving due consideration to his written statement as
there is no constitutional impediment to its admissibility
Moreover, it is settled that a confession is presumed voluntary until
the contrary is proved and the confessant bears the burden of
proving the contrary. Tanenggee failed to overcome this
presumption.
On the contrary, his written statement was found to have eben
executed freely and consciously. The pertinent details he narrated
in his statement were of such nature and quality that only a
perpetrator of the crime could furnish. The details contained
therein attest to its voluntariness
People v. Muit, it was held that “one of the indicia of voluntariness
in tehe execution of petitioners extrajudicial statmenet is that it
contains many details and facts which the investigating officers
could not have known and could not have supplied without the
knowledge and information given by him
It is settled rule that where the Tanenggee did not present evidence of
compulsion, where he did not institute any criminal or administrative
action against his supposed intimidators, where no physical evidence
of violence was presented, his extrajudicial statement shall be
considered as having been voluntarily executed.
PEOPLE v. SUNGA (JP) They reached Irawan, Puerto princesa wherein Jocelyn was met by
March 27, 2003 | Carpio-Morales, J. | Right to Counsel Sunga who held her and by Ramil Lansang who wrapped his arm
around her waist as they dragged to a nearby “buho” clumps.
PETITIONER: People of the Philippines
RESPONDENTS: Rey Sunga, Ramil Lansang, et al.

SUMMARY: State witness Locil implicated the accused herein, Sunga et


al. for the crime of rape with homicide for the death of a minor, Jocelyn. It
was alleged that Jocelyn was picked up from Mendoza park and was
raped by each of the accused in a secluded place. Locil witnessed the
crime and was threatened not to report. The accused were convicted of
the crime charged.

The SC upon automatic review reversed. Locil’s testimony, although


indispensible as she was the only witness to the crime, should be
corroborated enough to establish truston such. The two extra judicial
confessions of co-accused Sunga should have corroborated such.
However, the two extra-judicial confessions were taken without the
assistance of counsel. Although a counsel was requested by Sunga, such
counsel was the City Prosecutor of the place. Such cannot be a valid
counsel for conflict of interest against the accused. It was also proven
that such counsel did not effectively help in the court proceedings. The
accused are acquitted.

DOCTRINE: The independent counsel for the accused in custodial


investigations cannot be a special counsel, public or private prosecutor,
counsel of the police, or a municipal attorney whose interest is admittedly
adverse to the accused.

The right to counsel involves more than just the presence of a lawyer in
the courtroom or the mere propounding of standard questions and
objections; rather it means an efficient and decisive legal assistance and
not a simple perfunctory representation.

FACTS:
Locil, then 14 years old and an elementary school dropout, applied to be
a state witness for a heinous crime of rape with homicide for the
death of Jocelyn Tan, a minor and a highschool student of Palawan
Integrated National School (PINS), whose mutilated body was
founda at coffee plantation.
The prosection via Locil presented that: Locil boarded a tricyle bearing
the marking Ryan-Ryan. On board the tricycle was a lesbian who
invited Locil for a joy ride.
The tricycle repaired to Mendoza Park wherein the Lesbian talked and
picked up the victim, Joceyln Tan. Joceyln joined them. The tricycle was
drive ben Pascua in place of the previous driver, Sunga, who had in the
meantime left.
There, Jocelyn was made to lie down. Lansang stripped and palced
himself on top of Jocelyn and started pumping. Sunga then followed
and also had sexual intercourse with the victim. Pascua kept Jocelyn
pinned down. And he also had sexual intercourse with her.
After, Sunga then with a sharp laded weapon, stabbed the abdomen of
the motionless Jocelyn. Lansang then smashed Jocelyn’s gead with
an irregularly shaped stone.
Locil who witnessed verything was then pulled by the lesbian and led
back into the tricycle. She alighted from the tricycle when they
reached Mendoza park and was warned to keep mum about the
incident, lest something bad would happen to her.
Following the discovery of Jocelyn’s corpse, Locil was arrested. This is
where she filed as a state witness.
The trial court convicted Sunga and Lansang as principals of the crime of
Rape with Homicide and Pascua as principal for the crime of Rape.
The other accused were acquitted.
Hence, this automatic review of the case pursuant to Artucle 47 of the
RPC.

ISSUE/s:
WoN the testimony of the state witness herein, corroborated only by two
extrajudicial confessions taken without counsel, can be validly used
by the court – NO

RULING: Reversed. The accused are acquitted.

RATIO:
It appears that the conditions for Locil’s discharge under Sec. 9, Rule 119
of the Revised Rules of Court were satisfied:
The discharge must be with the consent of the accused sought
to be a state witness
His testimony is absolutely necessary
No other direct evidence is available
His testimony can be substantially corroborated in its material points
He does not appeal to be the most guilty
He has not been convicted of any offense involving moral turpitude.
Based on Locil’s sworn statement, she was the only person who saw
what happened to Jocelyn. Her testimony was thus indispensable.
Yet like any other testimony, this Court may not readily accept LocilÊs
statements hook, line and sinker because in the assessment of the
testimony of a co-accused-turned state witness, the same must be
received with great caution and must be carefully scrutinized.
The rule in this jurisdiction is that the testimony of a self-confessed
accomplice or co-conspirator imputing the blame to or
implicating his co-accused cannot, by itself and without
corroboration, be regarded as proof to a moral certainty that the
latter committted or participated in the commission of the
crime.
The tesimony must be susbtantially corroborated in tis material points face, it was executed not in the presence of counsel,
by unimpeachable testimony and strong circumstances and must contrary to the express requirement of the Constitution.
be such an extent that its trustworthiness becomes manifest.
Appellant Sunga’s two extrajudicial confessions, which strictly speaking were
admissions for they referred to statements of fact which did not directly
involve an acknowledgement of guilt or of the criminal intent to commit the
offense with which he was charged, could have lent corroborative support
to Locil’s testimony.
However, to the trial court’s ruling, this Court finds Sunga’s admissions to
be inadmissible in evidence not only against him but also against his
co-accused appellants.
The right to counsel was denied Sunga during his execution of Exhibit “A”
admission before the police on the ground that the counsel who assisted
him, Atty. Agustin Rocamora, was the City Legal Officer of Puerto
Princesa.
In People v. Bandula, this Court made it sufficiently clear that the independent
counsel for the accused in custodial investigations cannot be a special
counsel, public or private prosecutor, counsel of the police, or a municipal
attorney whose interest is admittedly adverse to the accused.
That Sunga chose him to be his counsel, even if trye, did not render his
(Sunga’s) admission admissible. Being of a very low educational
attainment, Sunga could not have possibly known the ramifications of his
choice of a city legal officer to be his counsel. The duty of law enforcers
to inform him of his Constitutional rights during custodial interrogations
to their full, proper and precise extent does not appear to have been
discharged.
Nothing in the records shows that Atty. Rocamora exerted efforts to
safeguard Sunga’s rights and interests, especially that of his right not
to be a witness against himself.
In fact, glaringly, Atty. Rocamora was not even made to testify so he
could have related the extent of legal assistance he extended to
Sunga at the police station.
Moreover, that Sunga was first questioned by SPO4 Pantollano and
Patrolman Bolos before he was investigated by SPO2 Janoras does
not escape the attention of this Court.
He did not have any lawyer by his side at the time these two policemen
started asking him questions about Jocelyn’s death. At that point,
Sunga was already under custodial investigation without the
assistance of counsel.
Like Exhibit “A,” Sunga’s second extrajudicial admission, Exhibit “I” is
inadmissible, due to the absence of counsel to assist him when he
executed it. To this Court, conditions therein are constitutive of an
atmosphere pervading that of a custodial investigation and
necessitating the assistance of a competent and independent
counsel of Sunga’s choice as a matter of right but which he had
none.
The waiver by Sunga of his right to counsel as contained in his
sworn statement-Exhibit “I” was not a valid waiver for, on its
Atty. Rocamora was appointed Sunga’s counsel de officio but just like the
assistance he extended during the execution of Exhibit “A,” Atty.
Rocamora utterly did nothing in defense of Sunga’s cause.
While Sunga was being asked by the judge a barrage of questions calling for
answers which could and did incriminate him, Atty. Rocamora did not
offer the slightest objection to shield his client from the damning nature
thereof.
In the case at bar, Sunga was thrust into the preliminary investigation and
while he did have a counsel, for the latter’s lack of vigilance and
commitment to Sunga’s rights, he was virtually denied his right to
counsel.
The right to counsel involves more than just the presence of a lawyer in the
courtroom or the mere propounding of standard questions and
objections; rather it means an efficient and decisive legal assistance
and not a simple perfunctory representation.
As for the rest of the prosecution evidence, it fails to corroborate Locil’s
testimony. The declarations of witnesses Tan, Devilleres and Gabinete
can in no way enhance the veracity of the essential, material aspects of
Locil’s account for they relate not to the crime itself but to events
thereafter.
Standing uncorroborated, can Locil’s testimoney serve as a basis for
appellant’s (Sunga) conviction? Her testimony may, even if
uncorroborated, be sufficient as when it is shown to be sincere in itself
because it is given unhesitatingly and in a straightforward manner and
full of details which, by their nature, could not have been the result of
deliberate afterthought.
An exhaustive review of the transcript of stenographic notes of Locil’s
testimony reveals, however, that the manner by which she related it
was punctuated with marks of tentativeness, uncertainty and
indecisiveness which the trial court unfortunately failed to take note
of in its decision on review.
PEOPLE v. IBAÑEZ (HENRY) door because his hands were tied. True enough, he was handcuffed to
June 10, 2013 | Brion, J. | Custodial Investigation the vault.

PETITIONER: People of the Philippines


RESPONDENTS: Jose Armando Cervantes Cachuela, Benhamin Cruz
Ibañez

SUMMARY: This case is plainly about the robbery in Weapons Systems


Corp, that came together with the death of one employee (gunsmith). So
when Henessy, the sales rep, one morning went to the office and unable
to enter because of the locked door, she asked her officemate at the back
of the office to open the door for her but apparently he is unable to as his
hands were tied. This prompted her to call the police for help and upon
entering, they saw in the firing range, the body of their officemate Rex.

NBI conducted an entrapment operation through the aid of their asset in


order to arrest the people responsible. And it was later figured that
Cachuela and his group together with Ibañez were the ones responsible,
together with Nabilgas, who appeared to give an extrajudicial confession,
without assistance of a consel of his choice.

RTC eventually found them guilty, to which the CA affirmed. Upon appeal
to the SC, the decision of the CA was also affirmed. However, insofar as
the admissibility of the extrajudicial confession of Nabilgas was
concerned, the SC held that it is inadmissible. It actually was emphasized
by the Court by reasons of the fact that apparently, Nabilgas already had
a lawyer (Atty. Paglinawan) but the NBI ignored it and had him assisted
by one Atty. Go. This being tantamount to a violation of his custodial right
as accorded by the constitution was the basis of the SC’s rendering of
such evidence as inadmissible.

DOCTRINE: The court consistently held that for a confession to be


admissible, it must adhere to the ff requirements: (b) Must be made with
assistance of a competent and independent counsel (preferably
confessant’s choice)

FACTS:
On July 2004, Ibañez went to Weapons System Corporation (WSC) on
board an old car, and told Henessy Auron, WSC’s Secretary and
Sales Rep, that he was the one who bought a gun barrel at the
company’s gun show in Megamall.
Ibañez inquired from Henessya bout the schedule and the rates of
WSC’s firing range and the fee for the gun club, and also asked
when there are many people in the range, and whether Henessy was
the only female employee (relevant kaya ito? Abangan…)
9am of July 26, Henessy arrived at WSC and rang the doorbell, but no one
opened the oor. She went at the back of the office where the firing range
was and saw her officemate Zaldy, who signaled that he cannot open the
When Henessy was finally able to enter, after calling Raymundo, their 3 Level of certainty demonstrated by the witness at the
ops manager, as well as the police who broke the door open, Zaldy
told them that Rex, their gunsmith, was in the firing range. What they
saw when they went outside was his lifeless body, which upon
autopsy was reported to die of several gunshots all over his body.
NBI receive an information from an asset that the group of Cachuela was
involved in the robbery and the killing in WSC, and that the same
group is looking for prospective buyers of firearms. NBI planned to
entrap them, to which they were successful. One Nabilgas was
arrested, who apparently surrendered and gave the names of other
people involved in the crime. Cachulela was arrested, too.
NBI likewise made a follow-up operation on Ibañez whom the asset also
contacted, who directed the asset to bring the prospective buyers to
his residence in Cavite. When asked whether the guns had papers,
Ibañez was arrested when the NBI sensed he was becoming
suspicious.
In a police line-up in the NBI office Zaldy pointed to Ibañez, Nabilgas,
Cachuela, as the persons responsible for the robbery and killing in
WSC. Nabilgas also executed a handwritten confession implicating
Ibanez et al in the crime.
RTC found Nabilgas, Zaldy, and Ibañez guilty of the special complex
rime of robbery with homicide, to which they sought an appeal to the
CA, which affirmed the RTC decision, basing on circumstancial
evidence. (irrelevant to topic so read origs for the details for this one.
Page 169) Hence, this appeal.

ISSUE/s:
WoN the extrajudicial confession is admissible in evidence – NO

RULING: SC affirmed the CA decision.

RATIO:
With regard the admissibility of the out-of-court identification and the
extrajudicial confession, the SC noted that Zaldy identified the
respondents as the persons involved in the robbery and the killing.
However the SC recognized that zaldy did not testify in court since
he was brought to the National Center for Mental Health, where he
subsequently died in the course of the trial. Lino’s testimony (during
the trial, nag testify si Lino na si Zaldy daw nagsabi na sila
respondents ang responsible sa robbery) is therefore to be examined
in greater scrutiny.
In People v. Algarme, the procedure of out-of-court identification and the
test to determine admissibility is:
a. Totality of circumstances test – considering the ff factors:
0 Witness’ opportunity to view the criminal at te time of
the crime
1 Witness’ degree of attention at that time
2 Accuracy of any prior description, given by the
witness
identification understood the nature and the consequence of his extrajudicial
4 Length of time between the crime and the indentification confession and its impact on his
5 Suggestiveness of the identification procedure
In application to the present case, Lino merely stated that Zaldy, during a
police line-up, identified the respondents as the persons responsible.
Lino didn’t state when the line-up took pace, how it was conducted,
who the persons in the line-up were (if there were others), and
whether the lineup was confined to persons with the same built as
respondents. Lino did not indicate who accompanied Zaldy prior and
during the line-up
SC therefore ruled that Lino’s failure to state relevant details surrounding
the police line-up is a glaring omission that renders unreliable Zaldy’s
out-of-court identification.
Records likewise bear that Nabilgas executed an extrajudicial confession
at the NBI office, where he implicated the respondents and Zaldy in
the crime charged. During trial, he repudiated his confession and
claimed he was tortured by NBI agents who forced him to copy a
previously prepared statement.
The SC held that the extrajudicial confessioin is inadmissible in evidence.
The court consistently held that for a confession to be admissible, it must
adhere to the ff requirements:
Confession must be voluntary
Must be made with assistance of a competent and independent
counsel (preferably confessant’s choice)
Must be express
Must be in writing
Nabligas was already under custodial investigation by authorities when
he executed the alleged written confession. Again, a custodial
investigation is understood as any questioning initiated by law
enforcement authorities after a person is taken into custody or
otherwise deprived of his freedom of action in any significant
manner. It begins when there is no longera general inquiry into an
unresolved crime and the investigation started to focus on a
particular person as a suspect
In people v. Rapeza, the SC explained that the lawyer called to be
present during custodial investigations should be the choice fo the
individual being investigated. If the lawyer is furnished by the police,
it is important that the lawyer should be competent, independent, and
prepared to fully safeguard the constitutional rights of the accused
The SC ruled that Nabligas’ confession was not made with assistance of
a competent and independent counsel. The services of Atty. Go who
acted in Nabligas’ behalf, were provided by NBI, who was assigned
despite Nabligas’ open declaration that he has a lawyer, Atty.
PAglinawan.
Moreover, Atty. Go did not disclose that she was a lawyer when she
assisted him, and presented herself to be a mere witness to the
confession. Nothing in the records show that Atty. Go ascertained
whether the confession of Nabligas was voluntary, and whether he
constitutional rights.
THIS is not the kind of assistance required of lawyers in a custodial
investigation.
An effective and vigilant counsel necessarily and logically requires that the
lawyer be present and be able to advise and assist his client from the
time the confessant anwers the first question asked by the investigating
officer until the signing of the extrajudicial confession
PEOPLE v. MATIGNAS (DANNAH) as the killer because he looked like the man who was tailing
March 12, 2002 | Panganiban, J. | Cherry before her

PETITIONER: People of the Philippines


RESPONDENTS: Jemreich Matignas, Noel De Guzman et al.

SUMMARY: Matignas et al were charged with rape and murder.


Witnesses were able to testify that they were the people last seen near
Cherry, the victim. Some also testified that they saw Matignas et al. grab
Cherry. After that night, her body was found. One of appellant De
Guzman’s assigned errors was that the trial court did considered his
extrajudicial confession as admissible despite being executed without the
presence of counsel.

Instead of giving Appellant De Guzman the option to choose his own


counsel, NBI Regional Director Salvador Ranin instead summoned Atty.
Florante Dizon to assist Appellant De Guzman. Moreover, the testimony
of Atty. Dizon shows that the custodial investigation started without his
presence.

In the end, however, the Court modified the judgment, ruling that rape
was not proven beyond reasonable doubt by circumstantial evidence,
and that instead of homicide, it’s murder because of the presence of
qualified aggravating circumstances.

DOCTRINE: The belated arrival of a lawyer the following day, though


prior to the actual signing of the uncounseled confession, did not cure the
defect, for the investigators had already extracted incriminatory
statements from the accused.

FACTS:
Version of the prosecution: Herminia Olaez woke up so she could fetch
her daughter Cherry who would be coming home from her work at
Cravings Katipunan.
Before 3AM they Herminia and Yolanda proceeded to the waiting shed
near their home, near Smokey’s Fastfood.
They waited until 4AM but since Cherry did not arrive, went home
because Herminia had to cook foor for Yolanda.
Tita Bautista called Herminia because she found some clothes and and
ID belonging to Cherry by the alley.
Josephine and Roberto, Cherry’s siblings went out to verify and saw
Cherry’s dead body.
SPO2 Santos was informed of the incident, proceeded to the area and
saw Cherry’s pants, underewear and deatched jewelry.
The Montalban Police charged a certain Jablo for the crime after he was
singled out in a police line-up by Nelita dela Cruz who pointed to him
death.
The case however was dismissed for insufficiency of evidence.
New witnesses came out: Hernandez and Fernandez who claimed they saw
appellants Matignas and de Guzman tail and grab Cherry beside the E.
Rodriguez Elementary School (ERES) gate past 2AM.
Hernandez went to the NBI and in a lineup positively identified both appellants
Matignas and de Guzman.
Nelita also appeared and identified de Guzman as the person whom she saw
following the victim that early morning of January 10, 1994. This time, Nelita
de la Cruz was certain. She explained that she had earlier mistaken Jablo for
de Guzman because of similarities in their features, build and the manner in
which they walk.
Version of the defense: Claims that Yolanda’s testimony was not credible as she
initially stated it was 12:30 am when they went to fetch Cherry.
She also claimed having seen Matignas and his wife Jenny Serrano at about that
time who even told her (Yolanda) that the Matignas couple would buy goto at
the corner of the plaza.
Nelita described the man following Cherry to be wearing camo long pants, a shit
and a cap. The man was walking with his heel almost pointing to each other
and whose feet were moving sidewise at 90 degrees to the left and 90
degrees to the right.
Nelita initially identified Cesar Hablo but 7 months later told NBI that it was not
Cesar but de Guzman.
Noel De Guzman cited testimonies:
Virginia de Guzman: Mother of Noel de Guzman. Did not see witnesses
Hernandez and Fernandez around the area where Cherry lived. Her son
was asleep then
Noel: He went to bed early. While he was in their house 6 persons suddenly barged
into their house and was ordered to lie down; he was handcuffed. No warrant of
arrest. He was not invited for investigation. He was mauled, asked to confess but
he refused. He was also tortured some more.
Oliver Yalong: One of the security guards at ERES. Nothing unusual happened
on the night of the murder.
Dr. Brion: Medico-legal. Disagrees with the conlusions made by Dr. Baltazar
more particularly with respect to the latterÊs claim that even if a woman has
been sexually abused by around four young men it is possible that the
vaginal canal would still be negative of spermatozoa. According to him,
young men easily ejaculate and, therefore, the absence of spermatozoa in
the vaginal canal negates the commission of rape
RTC ruled in favor of the prosecution, convicted Matignas et al.
ISSUE/s: The confession must involve an express and categorical
WoN the trial court erred in finding the prosecution witnesses credible – NO acknowledgment of the guilt
WoN the circumstantial evidence was sufficient to warrant conviction – YES The facts admitted must constitute a criminal offense
WoN RTC erred in not rejecting de Guzman’s statement given to the The confession must have been given voluntarily
NBI – YES The confession must have been intelligently made by the accused while
realizing the importance of such act
RULING: Judgment modified, each found guilty of murder. True, Appellant De Guzman was assisted by a lawyer when he reduced his
extrajudicial confession into writing and signed it. However, this fact did
RATIO: not satisfy the requirement that an “independent counsel” be made
Credibility of Prosecution Witnesses: The testimonies were clear and available to a person under custodial investigation as guaranteed by
straightforward in depicting how Matignas et al. had followed and Section 12(1), Article
later grabbed the victim near the vicinity of the locus criminis a few III
hours before her body was found Instead of giving Appellant De Guzman the option to choose his own
Circumstances from unbroken chain: counsel, NBI Regional Director Salvador Ranin instead summoned Atty.
Testimony of Hernandez who just came from a jueteng lottery saw a
Florante Dizon to assist Appellant De Guzman. Moreover, the testimony
man following a girl and upon reaching A. Boni Street, another man
of Atty. Dizon shows that the custodial investigation started without his
appeared and likewise tailed her; upon reaching ERES gate, both
men suddenly embraced, pulled and grabbed the girl presence.
Fernandez, who was with Hernandez at that time because he also People v. Compil: The belated arrival of a lawyer the following day,
participated in the said lottery, gave a similar testimony. though prior to the actual signing of the uncounseled confession, did
Dela Cruz narrated that she saw Cherry in the wee hours of the morning not cure the defect, for the investigators had already extracted
and that she was being followed by De Guzman incriminatory statements from the accused.
Perez said he also observed Matignas near the gambling place Since De Guzman’s counsel in the present case arrived after the
The finding of the body of Cherry at a vacant lot near EREs school
custodial investigation had already been started by the NBI, the
The finding of Matignas’ bullcap near the place where the body of
Cherry was found
assailed extrajudicial confession is deemed constitutionally flawed
The admission of Matignas that he was out prowling his neighborhood in and therefore inadmissible in evidence.
the early hours of January 10, 1994
Matignas et al. were the last persons seen with Cherry before her
corpse was found
Hernandez, Fernandez, Dela Cruz and Perez had no ill motive to testify
against Matignas et al.
It is undisputed that Matignas et al. were physically present at the scene of
the crime and its immediate vicinity, and that several eyewitnesses
positively identified them as the same persons who tailed, embraced and
pulled the victim in front of the gate of the ERES School along A.
Bonifacio Street before her body was discovered at a vacant lot near the
said school.
De Guzman assails his extrajudicial confession for having allegedly been
extracted through torture. He adds that no warrant of arrest was
served upon him when he was “invited for questioning,” and that he
was not assisted by counsel during custodial investigation. The Court
agrees.
For an an extrajudicial confession to be admissible, the following
requisites must be met:
MANILA WATEER COMPANY v. DEL ROSARIO (ARIELLE) During the formal investigation Del Rosario was found responsible for the
January 29, 2014 | Perez, J. | Right to Counsel loss of the water meters and therefore liable for violating Section 11.1
of the Company’s Code of Conduct. Del Rosario was dismissed.
PETITIONER: Manila Water Company
RESPONDENTS: Carlito Del Rosario

SUMMARY: Carlito Del Rosario worked for MWSS as an Instrument


Technician. He allegedly stole some of the company’s water meters and
sold it to the latter’s contractors. During investigation, Del Rosario
admitted that he was indeed involved in the taking of the water meters.
He was then dismissed from service pursuant to Sec 11.1 of the
Company’s Code of Conduct. The Labor Arbiter dismissed Del Rosario’s
complaint for illegal dismissal, but awarded him separation pay for his
length of service. On appeal, Del Rosario contends that his admission
cannot be the basis of his dismissal because such is inadmissible as he
was allegedly coerced by the company to make the admission.

The SC held that the constitutional right to counsel is available only


during custodial investigation. If the investigation is merely administrative
conducted by the employer and not a criminal investigation, the
admission made during such investigation may be used as evidence to
justify dismissal.

DOCTRINE: The constitutional right to counsel is available only during


criminal investigations, and not administrative investigations.

FACTS:
Carlito Del Rosario was employed as Instrument Technician by
Metropolitan Waterworks and Sewerage System (MWSS).
Subsequently, MWSS was reorganized pursuant to RA 8041, orbthe
National Crisis Act of 1995 and its implementing guidelines – EO
286.
Because of the reorganization, Manila Water absorbed some employees
of MWSS including Del Rosario. He officially became an employee of
Manila Water.
In 2000, Manila Water discovered that 24 water meters were missing in
its stockroom. Upon initial investigation, it appeared that Del Rosario
and his co-employee, Manguera, were involved in the pilferage and
the sale of water meters to the company’s contractor.
Manila Water issued a Memorandum, directing Del Rosario to explain in
writing within 72 hours why he should not be dealt with
administratively for the loss of the water meters.
In his letter-explanation, Del Rosario confessed his involvement in the
act charged and pleaded for forgiveness, promising not to commit
similar acts in the future. Manila Water conducted a hearing to afford
Del Rosario the opportunity to personally defend himself.
This prompted Del Rosario to file an action for illegal dismissal claiming CA erred in awarding separation pay without taking into consideration
that his severance from employment is without just cause. He stated that the transgression Del Rosario committed constitutes a serious
that his admission to the misconduct charged was not voluntary, but
offense.
was coerced by the company. Such admission therefore, made
without the assistance of a counsel, could not be made basis in
terminating his employment.
Manila Water refuted Del Rosario’s claims and pointed out that he was
indeed involved in the taking of the water meters. Manila Water also
invoked Section 11.1 of the Company’s Code of Conduct and said
that stealing the company’s property is punishable by dismissal.
Manila Water invited the attention of the Court to the fact that Del
Rosario himself confessed his involvement to the loss of the water
meters not only in his letter-explanation, but also during the formal
investigation.
The Labor Arbiter rendered its decision which dismissed the complaint
filed by Del Rosario, but awarded him separation pay for his length of
service (21 years).
NLRC dismissed the appeal interposed by Manila Water. The CA then
reversed the NLRC and held that it committed grave abuse of
discretion when it dismissed Manila Water’s appeal on mere
technicality. However, the CA affirmed the decision of the Labor
Arbiter. Hence this appeal.

ISSUE/s:
WoN Del Rosario’s admission during the investigation can be used as
basis for his dismissal – YES

RULING: The assailed decision of the CA is REVERSED.

RATIO:
Del Rosario maintains that there is no legal ground to justify his termination
from employment. He insists that his admission pertaining to his
involvement in the loss of the water meters was merely coerced by the
company.
It must be stressed at the outset that the correctness of the Labor
Arbiter’s pronouncement is no longer an issue and is beyond
modification. While Manila Water timely appealed the ruling, Del
Rosario did not question rhe dismissal of his illegal termination case.
It may be mentioned that the constitutional right to counsel is
available only during custodial investigation. If the investigation
is merely administrative conducted by the employer and not a
criminal investigation, the admission made during such
investigation may be used as evidence to justify dismissal. –
this is the only discussion related to our topic. It discussed
mainly on labor and separation pay.
PEOPLE v. PINLAC (IYA) He then went to the Makati Police Station to report the incident. On
September 26, 1988 |Paras, J. | Right of Accused return, to the house with he police officers, he learned about the
death of Osamu.
Police Detective Mallari, with his team, acting on the report took photographs
PETITIONER: People of the Philippines
RESPONDENTS: Ronilo Pinlac

SUMMARY: Pinlac was accused and found guilty of the robbery and
robbery with homicide at the homes of two Japanese nationals, Sato and
Osamu.

Among the presented evidence in lower court by the prosecution was an


extrajudicial confession made by Pinlac. However, such was obtained
through force and violence, Pinlac was subjected to torture and was
forced to sign the prepared confession without it being explained to him
and without counsel.

The SC reversed the RTC ruling finding him guilty for the extrajudicial
confession was done in contravention of the Constitution.

DOCTRINE: The right of a person under interrogation “to be informed”


implies a correlative obligation on the part of the police investigator to
explain, and contemplates an effective communication that results in
understanding what is conveyed. Short of this, there is a denial of the
right, as it cannot truly be said that the person has been “informed” of his
rights.

FACTS:
Ronilo Pinlac was charged in two separate informations of robbery and
robbery with homicide.
He was found by the RTC guilty beyond reasonable doubt for both
charges. The facts as summarized are as follows:
Two Japanese nationals were neighbors in San Lorenzo Village, Makati.
Mr. Koji Sato, living alone, had a housemaid by the name of Irene
Jandayan and a cook names Delia Marcelino.
Delie was employed for almost a year when she went on maternity leave
since she was to deliver a child with her husband Pinlac, who
frequently visited her in Sato’s place.
A low concrete fence separate the house of Sato form Mr. Saeki Osamu.
Osamu lived with his wife Hiroko Saeki. Osamu was a staff member
of the Japan International Cooperation Agency.
On April 7th, Sato went out of his house, Jandayan also left for her day-
off. Jandayan before leaving saw to it that the windows and doors
were securely closed and locked.
Returning home on the evening of the same day, Sato noticed that the
front door was already unlocked and several belongings were
missing.
of the scene of the crime. The death weapon, a kitchen knife, was
also recovered in the living room of the house.
Osamu’s maid, Evelyn Salomea, revealed that she saw Pinlac enter the
house of Sato.
The police then went to the residence of Marcelino (wife of Pinlac),
finding Pinlac thereat, invited him to he police station.
The RTC, as earlier stated, convicted him of the crimes. Hence, this
appeal.
The accused Pinlac denied all allegations. He claimed that he never left
the house on the 7th of April.
He alleges that policemen came to his house and arrested him without
any warrant of arrest shown despite demand.
He was also brought to the house of the Japanese men where he was
forced to reenact according to what the police theorized how the
crime was commited. It was at that moment that the prints the
accused shoes were all over the premises of Osamu and Sato’s
houses.
He alleges that he was also tortured to admit the crimes. He succumbed
to the wishes of his torturers after 7 hours of cruelty. It was then
when he finally signed a prepared confession which he was not
allowed to read nor was it explained to him.
In assailing his conviction, the accused Pinlac contends that the trial
court erred in admitting in evidence his extra-judicial confession,
which was allegedly obtained thru force, torture, violence and
intimidation, without having been apprised of his constitutional rights
and without the assistance of counsel.

ISSUE/s:
WON the extrajudicial Confession is admissible - NO

RULING: The judgment of conviction is reversed and the accused is


acquitted of the crime charged.

RATIO:
The only evidence furnished by the police authorities were merely
circumstantial evidence regarding the fingerprints of the accused
found in the window stabs of the maid’s quarters and in the kitchen
cabinet in the house of Mr. Sato.
But this was satisfactorily explained by the accused to the effect that
aside from being a frequent visitor in the house of Mr. Sato where his
wife works as a cook wherein at those times he could have
unknowingly left his fingerprints, but most especially during the time
when he was arrested and ordered to reenact.
The only evidence presented by the prosecution which could have been
fatal, is the extra-judicial confession of the accused, which is now
being assailed as violative of the Constitution.
In People v. Galit: This Court reiterated the correct procedure for peace
officers to follow when making arrest and in conducting a custodial
investigation. At the time a person is arrested, it shall be the duty of the
arresting officer to inform him of the reason for the arrest and he must be
shown the warrant of arrest…
When the Constitution requires a person under investigation “to be informed”
of his right to remain silent and to counsel, it must be presumed to
contemplate the transmission of a meaningful information rather than
just the ceremonial and perfunctory recitation of an abstract
constitutional principle.
As a rule, therefore, it would not be sufficient for a police officer just to
repeat to the person under investigation the provisions of the
Constitution.
He is not only duty-bound to tell the person the rights to which the latter
is entitled; he must also explain their effects in practical terms.
In other words, the right of a person under interrogation “to be informed”
implies a correlative obligation on the part of the police investigator
to explain, and contemplates an effective communication that results
in understanding what is conveyed. Short of this, there is a denial of
the right, as it cannot truly be said that the person has been
“informed” of his rights.
Thus, in People vs. Ramos, supra, the Court ruled that the verbal
admission of the accused during custodial investigation was
inadmissible, although he had been apprised of his constitutional
rights to silence and to counsel, for the reason that the prosecution
failed to show that those rights were explained to him, such that it
could not be said that the apprisal was sufficiently manifested and
intelligently understood by the accused.
Going to the instant case, We find that the evidence for the prosecution
failed to prove compliance with these constitutional rights.
Furthermore, the accused was not assisted by counsel and his alleged
waiver was made without the assistance of counsel.
PEOPLE v. ROUS (ELIEL) 2 days thereafter, Pasahol died in the Lorma Hosital at La Union where
March 27, 1995 | Melo, J. | Right to counsel he was brought by Rolando Laygo, as a result of the inflicted gun
shot wounds, as evidenced by Pasahol’s pictures.
PETITIONER: People of the Philippines The only available person who has actually witnessed how and where the
crime was committed was Rolando Laygo who suffered no injury and
RESPONDENTS: Socrates Rous aka “Bobby”, Rolando Laygo, Primitivo
was
Garcio, Virgilio Pradis and Celestino Rabina

SUMMARY: Laygo and the victim Pasahol were on board the latter’s car
from Ilocos Sur bound for Bulacan.When they reached La Union,
however, two armed men approached the car and robbed Pasahol’s bag
from the compartment, which contained P600,000 worth of jewelry and
was shot. When Laygo, the only companion with Pasahol during the
commission of the crime, was brought for questioning, he broke down
and admitted his conspiracy and participation of the crime with Socrates
Rous. However, he contends that his extrajudicial confession was
inadmissible because Atty. Datlag was not present when the investigation
started and when it was about to end.

The SC held that the evidence against Laygo is admissible. He was


provided with counsel. Although Atty. Datlag was not present all
throughout, Laygo and Gaddi proceeded to Atty. Datlag’s office to confirm
and verify the confession, to which after they both signed. Thus, the
constitutional requirement is complied. This goes the same for the
extrajudicial confession of Rous when he was caught.

DOCTRINE: The fact that Atty. Datlag arrived shortly after the
investiagtion of Laygo had begun and left before the confession was
concluded does not negate the validity and admissibility of said
confession for the reason that after the confession was put down in
writing, Laygo and Gaddi proceeded to the office of Atty. Datlag and the
latter then read the confession, conferred with Laygo and then advised
Laygo to sign the confession.

FACTS:
Victim Pastor Pasahol boarded in his car driven by Laygo who was only
his companion, and left Ilocos Sur bound for Meycauayan, Bulacan.
When they reached La Union, 2 aremed men, who according to
Laygo, alighted from a red car which stopped besied them, and tehn
the two shot Pasahol.
Thereupon, one of the assailants took the clutch bag from the
compartment of Pasahol’s car which, according to Pasahol’s wife,
contained gold coins, earrings with diamond and refined gold worth
more than P600,000.
Thereafter, the hold-uppers took Pasahol’scar, a Mitsubishi Lancer, which
was found abandoned by the peace officers at La Union, about 2
kms away from the place where the crime.
the only companion of Pasahol in the car at the time of robbery and The very purpose of said constitutional requirement is to prevent
the shooting of victim. the use of coercion in extracting a confession form a suspect.
Sgt. Gaddi invited Laygo to the CIS Office and after questioning him for Any form of
30 minutes, Laygo broke down and he admitted that he and Socrates
Rous were parties to the conspiracy of the original plan to commit
Robbery and not to inflict harm on Pastor Pasahol.
With Rolando Laygo’s revelation, Sgt. Gaddi took his sworn statement
with the assistance of Atty. Abraham Datlag, which is a confession of
his participation in the commission of robbery and implicated
Socrated Rous in the commission of the crime
Laygo, on the same date, also executed a document entitled “Kusang
Loob na Kahilingan” also assisted by Atty. Datlag who assisted him in
the custodial investigation and witnessed by Sps. Fely and Tiburcio
Laygo.
Both documents bear the signatures of Atty. Datlag who assisted him in
the custodial investigation. Laygo also executed another
“Sinumpaang Salaysay” without the assistance of a lawyer.
Laygo contends that he gave a written statement and that his sworn
satement his inadmissible because he was forced to sign it. That the
taking thereof was started and finished without the assistance of
counsel. Hence this petition.

ISSUE/s:
WoN the sworn statement of Laygo is admissible – YES

RULING: Guilty of the crime of Highway Robbery (PD 532) with homicide

RATIO:
The record show that the investigating officer fully informed Laygo of his
right to counsel and categorically asked Laygo whether he wanted
the assistance of counsel, to which inquiry, Laygo expressed his
desire to be so assisted by counsel.
Thereupon, the investigating office, Sgt. Gaddi, brought him to the office
of Atty. Datlag. Laygo and Atty. Datlag conferred for a while;
thereafter, Sgt. Gaddi and Laygo returned to the CIS office of Sgt.
Gaddi and started the investigation.
Atty. Datlag arrived soon after the investigation started and left before the
lat three questions were asked, instructing them to follow him to his
office. After the extrajudicial statement of Laygo was finished, Gaddi
and Laygo went to the office of Atty. Datlag who read and examined
the statement, after which, Atty. Datlag conferred with Laygo and
then advised Laygo to sign. Laygo did so and Atty. Datlag thereupon
signed the statement.
From the above facts, we find that there was more than substantial
compliance with the constitutional requirement that a person
under investigation for the commission of a crime should be
provided with counsel. (Sec 12(1) Art III)
coercion, whether physical, mental, or emotional in extracting
confessions stamps the confession with the taint of inadmissibility.
Nowhere in the evidence is it shown that coercion was ever employed by
the investigating officer in obtaining the confession of Laygo. The
investigation was even witnessed by relatives of Layo.
The fact that Atty. Datlag arrived shortly after the ivnestiagtion of Laygo
had begun and left before the confession was concluded does not
negate the validity and admissibility of said confession for the
reason that after the confession was put down in writing, Laygo
and Gaddi proceeded to the office of Atty. Datlag and the latter then
read the confession, conferred with Laygo and then advised Laygo
to sign the confession.
It will readily be seen that the confession was voluntary and the
signing thereof by Laygo was done upon advice of cousnel. The
constitutional requriements were thus fully complied with.
A confession constitutes evidence of high order since it is
supported by the strong presumption that no person of normal
mind would deliberately and knowingly confess to a crime
unless prompted by truth and his conscience.
The same ruling applies to the extrajudicial confession of Rous.
Although Atty. Ferred, the counsel of Rous, was not present
when the confession was taken, after the confession was
prepared, Gaddi brought Rous to the office of Atty. Ferrer who
read the confession and fully explained it to Rous.
PEOPLE v. ALICANDO (JP) Khazie Mae under his house. He informed her parents and reported
December 12, 1995 | Puno, J. | Right to Counsel to the polic.
Rebada (witness) then suffered a change of heat and informed Romeo
PETITIONER: People of the Philippines (father) and his wife that it was accused Alicando who committed the
crime.
RESPONDENTS: Arnel Alicando

SUMMARY: Witness neighbor Rebada peeped through the window of


accused Alicando’s house and was shocked to find accused on top of
four year old victim, Khazie Mae. The latter’s body was found by another
neighbor. Rebada implicated accused Alicando. Accused Alicando the
made an uncounseled verbal confession, offering physical evidence. He
was found guilty and was sentenced to death by electrocution.

The SC upon automatic review, reversed. It held that uncounseled


confessions have long been held inadmissible. Accused’s right to due
process was also violated for having been apprised of the information
against him in English, a language not known to him. Since his
confession is inadmissible, by the doctrine of the fruit of the poisonous
tree, the physical evidence presented from such, is likewise inadmissible.
The case is remanded for lack of evidence.

DOCTRINE: It is now familiar learning that the Constitution has


stigmatized as inadmissible evidence uncounselled confession or
admission. Section 12 paragraphs (1) and (3) of Article III of the
Constitution provides so.

Fruit of the poisonous tree doctrine: once the primary source (the “tree”)
is shown t have been unlawfully obtained, any secondary or derivative
evidence (the “fruit”) derived from it is also inadmissible.

FACTS:
Romeo Penecilla, father of the four year old victim Khazie Mae, was
drinking liquor with his neighbos. Accused Alicando joined them but
left every now and then.
Luisa Rebada, a neighbor, saw the victim Khazie Mae at the window of
Accused Alicando’s house. She offered to buy Khazie yemas but
accused Alicando closed the window.
Soon, he sheard the victim Khazie crying. She approached the house
again and peeped through an opening between its floor and door.
The sight shocked her· accused Alicando was naked, on top of the
victim, his left hand choking her neck. She retreated to her house in
fright.
Romeo (father of victim) returned to his house and did not find Khazie.
Rebada (the witness) did not tell the parents of Khazie what she
knew.
Another neighbor, Leopoldo Santiago went down from his house to
answer the call of nature. He then discovered the lifeless body of
Forthwith, accused Alicando was arrested and interrogated by PO3
Danilo Tan. He verbally confessed his guilt without the assistance of
counsel.
On the basis of his uncounselled verbal confession and follow up
interrogations, the police came to know and recovered from
accused’s house, Khazie Mae’s green slippers, a pair of gold
earrings, a buri mat, a stained pillow and a stained T-shirt all of which
were presented as evidence for the prosecution.
The RTC found him guilty and sentenced him to death.
Hence, the automatic review by the SC.

ISSUE/s:
WoN uncounselled confession is admissible as evidence – NO

RULING: Lower decision is annulled. Case is remanded to the trial court for
further proceedings.

RATIO:
(Note: Apellant = Accused Alicando)
The SC finds that the Decision of the trial court sentencing the appellant
to death is shot full of errors, both substantive and procedural. The
conviction is based on an amalgam of inadmissible and incredible
evidence and supported by scoliotic logic.
First. The arraignment of the appellant is null and void. The trial judge
failed to follow section (1) (a) of Rule 116 on arraignment.
The reading of the complaint or information to the appellant in the
language or dialect known to him is a new requirement imposed by
the 1985 Rules on Criminal Procedure. It implements the
constitutional right of an appellant “x x x to be informed of the nature
and cause of the accusation against him.”
In the case at bar, the records do not reveal that the Information against
the appellant was read in the language or dialect known to him. The
Information against the appellant is written in the English language. It
is unbeknown whether the appellant knows the English language.
Second. The plea of guilt made by the appellant is likewise null and void.
The trial court violated section 3 of Rule 116 when it accepted the
plea of guilt of the appellant.
The questions of the trial court failed to show the voluntariness of the plea of
guilt of the appellant nor did the questions demonstrate appellant’s full
comprehension of the consequences of his plea. The records do not
reveal any information about the personality profile of the appellant
which can serve as a trustworthy index of his capacity to give a free and
informed plea of guilt.
The questions were framed in English yet there is no inkling that
appellant has a nodding acquaintance of English. It will be noted too
that the trial court did not bother to explain to the appellant the
essential elements of the crime of rape with homicide.
Third. Some prosecution evidence, offered independently of the plea of It matters not that in the course of the hearing, the appellant failed to
guilt of the appellant, were inadmissible, yet, were considered by the make a timely objection to the introduction of these
trial court in convicting the appellant. constitutionally proscribed
The physical evidence offered by the prosecution were inadmissible
for they were gathered by PO3 Tan as a result of custodial
interrogation where appellant Alicando verbally confessed to
the crime without benefit of counsel.
It is now familiar learning that the Constitution has stigmatized as
inadmissible evidence uncounselled confession or admission.
Section 12 paragraphs (1) and (3) of Article III of the Constitution
provides so.
In the case at bar, PO3 Tan did not even have the simple sense to reduce the all
important confession of the appellant in writing. Neither did he present any
writing showing that appellant waived his right to silence and to have
competent and independent counsel.
Despite the blatant violation of appellant’s constitutional right, the trial
court allowed his uncounselled confession to flow into the records
and illicitly used it in sentencing him to death.
It is not only the uncounselled confession that is condemned as
inadmissible, but also evidence derived therefrom. The pillow
and the T-shirt with the alleged bloodstains were evidence
derived from the uncounselled confession illegally extracted by
the police from the appellant.
We have not only constitutionalized the Miranda warnings in our
jurisdiction. We have also adopted the libertarian exclusionary rule
known as the “fruit of the poisonous tree,” a phrase minted by Mr.
Justice Felix Frankfurter in the celebrated case of Nardone v. United
States.
According to this rule, once the primary source (the “tree”) is
shown t have been unlawfully obtained, any secondary or
derivative evidence (the “fruit”) derived from it is also
inadmissible. Stated otherwise, illegally seized evidence is
obtained as a direct result of the illegal act, whereas the “fruit of
the poisonous tree” is the indirect result of the same illegal act.
But even assuming arguendo that the pillow and the t-shirt were
admissible evidence, still, the trial court erred in holding that they
“strongly corroborated the testimony of Luisa Rebada that the victim
was raped.”
For one, there was no basis for the trial court to conclude that the stains
on the pillow and t-shirt were human bloodstains. It must also be
noted that it is not unnatural for appellant to have bloodstains on his
shirt. He is a butcher by occupation.
Par. 1 of Section 12 of Article III of the Constitution provides only
one mode of waiver·the waiver must be in writing and in the
presence of counsel. In the case at bar, the records show that
the prosecution utterly failed to discharge this burden.
evidence. The lack of objection did not satisfy the heavy burden of proof
that rested on the prosecution.
In sum, the Court cannot send the appellant to die in the electric chair on the
basis of the procedural irregularities committed by, and the inadmissible
evidence considered by the trial court.
Since in the case at bar, the arraignment of the appellant is void, his
judgment of conviction is also void.
Kapunan, dissenting: It is well-settled in this jurisdiction that the testimony
of a lone witness, free from signs of impropriety or falsehood, is sufficient
to convict an accused even if uncorroborated. In this case, Rebada’s
testimony was positive and straightforward.
Convictions for the crime of rape have been sustained by this Court in an
overwhelming number of cases on uncorroborated evidence given
almost exclusively by the complainant alone.
Together with the direct testimony of the eyewitness, Rebada, there is
sufficient evidence corroborating and unmistakably pointing to the
appellant as the author of the crime. Khazi Mae was last seen in the
company of the appellant. Rebada testified that she saw appellant naked
on top of Khazi Mae. Recovered from the latterÊs house were Khazi
MaeÊs green slippers, pair of gold earrings, her dress, bloodied buri mat
and pillow.
In a long line of cases, courts have recognized that evidence derived from
information obtained illegally is not absolutely inadmissible under the fruit
of the poisonous tree doctrine where it is shown that such evidence
would have been inevitably gained even without the unlawful act. He
submits that secondary physical evidence would have been discovered
anyway.
PEOPLE v. DE GUZMAN (HENRY) Of course, De Guzman denied all these. He claims that he was dumped
March 4, 1991 | Cruz, J. | Custodial Investigation Rights in a fishpond and then in a “tarima” (di ko alam what is this) where
they left him
PETITIONER: People of the Philippines
RESPONDENTS: Benjamin Peralta De Guzman

SUMMARY: This is a case concerning the arrest of De Guzman and his


claim that his signature in the Receipt of Seized Property is inadmissible,
it being obtained through force without counsel, which is in violation of his
constitutional right insofar as custodial investigation is concerned.

De Guzman was caught in a buy bust operation with NARCOM agent


Querubin acting as the “buyer” of the drugs. It might be important to note,
that De Guzman had another pending case for the same (drugs lol yes).
When in the NARCOM headquarters, he purportedly signed the Receipt
of Seized Property upon confiscation of his drugs, which were tested and
are positive. Upon filing of a case in the RTC, he was held guilty for
selling drugs.

Upon appeal to the SC, he was acquitted. The SC recognized the weak
defense, but also pointed out the weak arguments of the prosecution. It
was stressed that, the prosecution’s arguments will matter and it will
determine the conviction. Moreover, the SC included that the signing of
the Receipt of Seized Property is already an admission, and therefore is
within the ambit of rights being protected by Section 12. Therefore the
evidence of such signature is inadmissible, making the prosecution’s
arguments even weaker.

DOCTRINE: The signing of the Receipt of Seized Property without counsel


and with force is definitely inadmissible in evidence as it is within the ambit of
the protection of Sec 12, Art 3 of the Constitution, on custodial investigation
rights.

FACTS:
Respondent De Guzman claims he was a chicharon vendor but he was
arrested for selling marijuana, and was charged with violation of the
Dangerous Drugs Act
In the afternoon of Dec 7, 1987, the NARCOM office received a tip from a
confidential informer named Bazar that De Guzman would be selling
marijuana at Virgen de los Flores from 6pm-12mn.
The NARCOM Chief organized a buy bust team with the confidential
informer himself.
Upon closing in to De Guzman, the NARCOM agents finally revealed
their identity and arrested him after catching him red handed.
Bazar added that De Guzman was known to him as being actively
engaged in the selling of marijuana and was in fact facing charges
for this same offence in another pending criminal case.
until the following morning, where he signed the Receipt of Seized
Property because he was afraid of further punishment.
He was detained at the PC Stockade for 3 days, during which he was
subjected to torture and third degree investigation until he was
transferred to the provincial jail. He further alims that a motorcycle
belonging to him had 3 months earlier been taken by Bazar as it was
parked in front ofa gambling den.
There were also two testimonies, which stated that they saw De Guzman
arrested by 3 men and was tied with wire before being taken away.
The RTC assessed the evidence, and gave more credence to Bazar’s
testimony. The RTC judge considered it significant that De Guzman
had not protested when he was arrested. He had not denied that the
marijuana had been confiscated from him and had in fact later
signed a receipt without objection.
He was convicted after trial by the RTC. In this appeal he asks for the
reversal of his conviction on the ground that the evidence against
him was insufficient to establish guilt beyond reasonable doubt.

ISSUE/s:
WoN De Guzman’s Constitutional right in custodial investigatioin was
violated – YES

RULING: SC acquitted De Guzman.

RATIO:
The SC is not inclined in believing De Guzman’s testimony. Even his
mere denial of the charges was not convincing, especially if it is
viewed in ht elight of the other criminal case against him for the
same offense of selling marijuana and of his notoriety as a drug
pusher.
The defense is basically weak and fails to convince that De Guzman is
not guilty of the charge that he sold Marijuana.
However, as weak as the defense may be, the prosecution is even
weaker. It cannot by itself sustain a conviction. The SC has said
times without number that in every criminal case, the conviction of
the accused must depend not on the weakness of the defense but on
the strength of the prosecution.
In close scrutiny, the testimony given by Bazar tells little of the supposed
sale of marijuana by De Guzman. From his vantage point of 10m
away, Bazar couldn’t have heard the supposed conversation
between De Guzman and the NARCOM agent in the buy bust ops.
All Bazar said he saw was the prearranged signal, but that alone proved
nothing. Surely, De Guzman’s conviction cannot be based solely on
the fact that NARCOM agents threw away a lighted cigarette (as
signal na nahuli na si De Guzman in the act of selling drugs).
The SC is puzzled as to why the prosecution did not present Querubin
(NARCOM agent) who allegedly bought the marijuana from De Guzman
and paid him the marked money. Querubin’s identity need not be
concealed like
that of the former (De Guzman).
It is true that each party has the discretion to plot its own strategy, as we
have said often enough in many cases, but under the circumtances
of this particular case the choice by the prosecution was less than
tactical.
The prosecution suggests that if the defense really felt that Querubin could
advance tis cause, ther was nothing to prevent it from calling him as
witness, as it had a right to do under the Bill of Rights and the Rules of
Court.
The trial judge regarded the signs of De Guzman’s guilt of his
submissiveness when he was arrested and when he sgned the “Receipt
of Seized Property” (Exhibit C). One can easily be cowed into silence by
men with drawn guns and ostensible authority and may even be
intimidated into involuntary sadmissions as De Guzman claims he was
when he signed the receipt.
That, the receipt was in reality an admission which De Guzman was
forced to make without the assistance of counsel and without being
informed of his constitutional rights insofar as custodial investigation
is concerned.
SC held that such signature is inadmissible under the Bill of rights, which
is consistent with the SC rulings ever since People v. Galit
PEOPLE v. MALIMIT (DANNAH) Condensed facts of the case:
November 14, 1996 | Francisco, J. | Admissibility of Evidence

PETITIONER: People of the Philippines


RESPONDENTS: Jose Encarnacion Malimit

SUMMARY: Malaki owned a store. He was minding the store while his
helper Batin was cooking. Farmer Rondon passed by the store to buy
chemicals for his rice farm. When Batin was about to ask Malaki if he was
to prepare supper, he saw Malimit with a Bolo and Malaki bathing in his
own blood. Rondon who just left the store saw Malaki leave the store as
well. Rondon and Batin reported the incident to Malaki’s brother-in-law
Beloy. When they went back to the scene of the crime Malaki was dead.
They noticed the store’s drawer was open and Malaki’s wallet was
missing from his pocket. (Di kwinento pano nahuli Malimit)

Malimit believs that the trial court should not have allowed the
presentation Malaki’s wallet as evidence as he pointed the location of
said wallet to police in violation of his constitutional right. The Court ruled
that the prohibition does not apply to the instant case where the evidence
sought to be excluded is not an incriminating statement but an object
evidence.

DOCTRINE: These are the so-called “Miranda rights” so oftenly


disregarded by our men in uniform. However, infractions thereof render
inadmissible only the extrajudicial confession or admission made during
custodial investigation.

The admissibility of other evidence, provided they are relevant to the


issue and is not otherwise excluded by law or rules, is not affected even if
obtained or taken in the course of custodial investigation

FACTS:
Appellant Malimit, charged with and convicted of the special complex
crime of robbery with homicide, was meted by the trial court the
penalty of reclusion perpetua.
212. He was also ordered to indemnify the heirs of Onofre Malaki P50k.
Appellan Malimit in this appeal asks for his acquittal alleging that the trial
court committed the following errors:
Erred in giving credence to unreliable testimonies
Erred in admitting as evidence the wallet and its contents although
the circumstances which led to its production was obtained in
violation of his constitutional rights
Erred in his conviction despite failure to prove guilt beyond reasonable
doubt
Onofre Malaki was a store owner, he was attending to his store.
He has a houseboy named Batin who, at the time, was busy cooking
chicken for supper
Florencio Rondon, a farmer, arrived at Malaki’s store
After Batin was finished cooking he was going to ask Malaki if he was to
serve supper but was surprised to see Malimit with a bolo
Batin saw Malaki bathing in his own blood, struggling for his life
Rondon was 5 meters away; was able to see Malimit’s face (while he
was rushing out) because of a petromax lamp
Batin and Rondon rushed to Malaki’s brother-in-law’s house (Beloy) and
informed Beloy of what happened
They went back to the store, saw Malaki’s lifeless body
Beloy noticed the store’s drawer was opened and ransacked; wallet of
Malaki was missing from his pocket

ISSUE/s: Relevant issue: WoN Malaki’s wallet and its contents is admissible
in evidence – YES

RULING: Appealed judgment affirmed

RATIO: (Fastforward to #8 if you wanna read the related ratio)


279. On credibility of prosecution witnesses: Malimit questions why there
is a delay in revealing what they knew about the incident. The crime
took place on April 1991, it was only on September 1992 when they
tagged him as the culprit.
The date (September) was merely the date when Rondon and Batin
executed their respective affidavits, narrating that they saw the
appellant on the night of April 15, 1991 carrying a bolo stained with
blood and rushing out of
Malaki’s store
281. As to the delay: suffice it to state that extant from the records are
ample testimonial evidence negating Malimit’s protestation, to wit:
After having discovered the commission of a crime, Batin and Rondon
immediately look for Beloy and informed him that Malimit was the
only person they saw running away from the crime scene
Beloy and Batin reported the crime with the CAFGU (di sinabi ano)
detachment in their barangay where Batin declared it was Malimit
who robbed Malaki
Batin again made a similar statement at the Silago Police Station
282. On non-presentation by the prosecution of the police blotter: It was
not necessary for the prosecution to present as evidence a copy of
the aforementioned police blotter.
Entries in the police blotter are merely corroborative evidence of the
uncontroverted testimony of Batin that he identified the appellant as
the
perpetrator of the crime before the Silago police. under Article III, Section 12 (di ko na ittype yung paragaphs 1
Besides, if Malimit believed he was not identified therein, then he should and 3 okay? Yun yung cinite)
have secured a copy thereof from the Silago Police Station and These are the so-called “Miranda rights” so oftenly disregarded by
utilized the same as controverting evidence to impeach Batin’s our men in uniform. However, infractions thereof render
credibility as witness. inadmissible only the
Even assuming arguendo that Rondon and Batin identified Malimit only
on September 15, 1991, or after the lapse of five months from
commission of the crime, this fact alone does not render their
testimony less credible. The
non-disclosure by the witness to the police officers of Malimit’s
identity immediately after the occurrence of the crime is not entirely
against human experience.
286. On admissibility as evidence of the wallet with its contents:
Malaki’s residence certificate
ID card
Bunch of keys
Malimit believes this violates his right against self-incrimination. He also
sought for the exclusion because during the custodial investigation,
wherein he pointed to the investigating policemen the place where
he hid Malaki’s wallet, he was not informed of his constitutional
rights.
The Court is not persuaded.
The right against self- incrimination guaranteed under our fundamental
law finds no application in this case.
Justice Holmes in Holt v. US: “ xxx is a prohibition of the use of physical
or moral compulsion, to extort communications from him”
It is simply a prohibition against legal process to extract from the
[accused]’s own lips, against his will, admission of guilt.
It does not apply to the instant case where the evidence sought to be
excluded is not an incriminating statement but an object evidence.
Wigmore (on treatise on evidence):
“If in other words (the rule) created inviolability not only for his [physical
control of his] own vocal utterances, but also for his physical control in
whatever form exercised, then, it would be possible for a guilty person
to shut himself up in his house, with all the tools and indicia of his
crime, and defy the authority of the law to employ in evidence anything
that might be obtained by forcibly overthrowing his possession and
compelling the surrender of the evidential articles—a clear reductio ad
absurdum.In other words, it is not merely compulsion that is the kernel
of the privilege, ***
but testimonial compulsion.”
Neither is the court prepared to order the exclusion of the questioned
pieces of evidence pursuant to the provision of the Constitution
extrajudicial confession or admission made during custodial
investigation.
The admissibility of other evidence, provided they are relevant to the issue
and is not otherwise excluded by law or rules, is not affected even if
obtained or taken in the course of custodial investigation.
Concededly, Malimit was not informed of his right to remain silent and to
have his own counsel by the investigating policemen during the custodial
investigation. Neither did he execute a written waiver of these rights in
accordance with the constitutional prescriptions. Nevertheless, these
constitutional short-cuts do not affect the admissibility of Malaki’s wallet,
identification card, residence certificate and keys for the purpose of
establishing other facts relevant to the crime.
Thus, the wallet is admissible to establish the fact that it was the very wallet
taken from Malaki on the night of the robbery. The identification card,
residence certificate and keys found inside the wallet, on the other hand,
are admissible to prove that the wallet really belongs to Malaki.
Even assuming arguendo that these pieces of evidence are inadmissible, the
same will not detract from Malimit’s culpability considering the existence
of other evidence and circumstances establishing Malimit’s identity and
guilt as perpetrator of the crime charged.
5 circumstances constituting an unbroken chain of events:
Malimit was seen by Rondon and Batin, whose credibilities were
untarnished, holding a bolo in his right hand and rushing out of Malaki’s
store seconds prior to their discovery of the crime
Malaki sustained multiple stab wounds and he died of “cardiac arrest,
secondary to severe external hemorrhage due to multiple stab wounds
Witness Elmer Ladica saw the Malimit on August 6, 1991, accompanied by
some policemen, retrieve Malaki’s wallet underneath a stone at the
seashore in Barangay Hingatungan
Malimit himself admitted in his testimony that on August 6, 1991, he
accompanied several policemen to the seashore where he hid Malaki’s
wallet
Malimit’s flight and his subsequent disappearance from Hingatungan
immediately after the incident.
Malimit interposed the defense of alibi, that he was with his wife on that night.
However they did not call her to the witness stand. Also, his house is only
80 meters away. Impossibility at scene of the crime is what is needed for
the defense of alibi to succeed.
Malimit’s insistence that he found Malaki’s wallet by chance while gathering
shells is incredible. Also his fear of being implicated in the crime for
which reason he hid the wallet underneath a stone, hardly inspires belief.
CARDINES v. ROSETE (ARIELLE) illegal recruitment was punishable with life imprisonment hence non-
March 22, 1995 | Bellosillo, J. | Right to Bail and Recognizance bailable.
Records show that PNP Chief Investigator Pajarillo filed a criminal complaint
PETITIONER: Joaquin Pellicena
RESPONDENTS: Leoncio Gonzalez

SUMMARY: Emilio Claro and Erlie Claro, upon the complaint of


Cardines, were charged with illegal recruitment, which is punished with
life imprisonment. The accused applied for bail, and was granted by
respondent Judge Rosete on the ground that the right to bail is granted
by the Constitution and should not be denied the accused except those
charged with a capital offense and sicne the maximum penalty that could
be imposed is life imprisonment, the penalty therefore is still bailable.

Cardines contends that illegal recruitment has a penalty of life


imprisonment, Claro should not have been granted bail. The Court
agrees with respondent Judge that the accused charged with an offense
penalized with life imprisonment should be granted bail as a matter of
right. The law existing at the time of the alleged illegal recruitment and
when the accused applied for bail was the 1985 Rules on Criminal
Procedure. It provides that all persons in custody shall before conviction,
be entitled to bail as a matter of right except those charged with a capital
offense or an offense which under the law at the time of its commission
and at the time of application for bail, is punishable by reclusion
perpetua.

In the above law, “life imprisonment” was not among the exception, which
leads the Court to the conclusion that persons accused of crimes
punishable with “life imprisonment” were entitled to bail as a matter of
right. Life imprisonment is not the same with reclusion perpetua.
Moreover, under Adm. Cir. No. 12-94, a person charged with an offense
punishable with death, reclusion perpetua or life imprisonment is no
longer entitled to bail as a matter of right when evidence of guilt is strong.

DOCTRINE: Under Adm. Cir. No. 12-94, a person charged with an


offense punishable with death, reclusion perpetua or life imprisonment is
no longer entitled to bail as a matter of right when evidence of guilt is
strong.

FACTS:
Respondent Judge Gregorio Rosete of the MCTC of Tarlac, is charged
with misconduct and ignorance of the law for allegedly grandting bail
to Erlie Claro and Emilio Claro in conspiracy with his Cleark of Court
and Criminal Docket Clerk, knowing fully well that the charge of
charging Erlie Claro, Emilio Claro, and Albert Reyes with illegal The Court agrees with respondent Judge. The complaint is devoid of merit.
recruitment. The law existing at the time of the alleged illegal recruitment, which was
The complaining witnesses, Archimedes Cardines, claimed that the sometime in May to July 1993, and when the accused applied for bail
accused, who had no license or authority to conduct recruitment and was the
placement of workers, promised to hire them for overseas
employment upon payment of a fee.
The complaint was filed with respondent Judge for preliminary
investigation. A second criminal complaint for illegal recruitment was
filed by PNP Senior Inspector Julian Saygo against the same
accused upon the complaint of Archimedes Cardines and Lea
Cardines.
Subsequently, the accused Erlie Claro and Emilio Claro were placed
under preventive detention, while Albert Reyes remained at large.
Upon application of Claro, respondent Judge Rosete granted them
provisional liberty on a bond of P20,000 each on the ground that the
right to bail was guaranteed by the Constitution and should not be
denied the accused except those charged with a capital offense and
since the maximum penalty that could be possibly imposed is life
imprisonment xxx the penalty therefore is still bailable.
After conducting preliminary investigation, Judge Rosete was convinced
that the crime charged has been committed and that the accused
were probably guilty thereof.
Judge Rosete cancelled the bail bonds and issued warrants for their
arrest.

ISSUE/s:
WoN respondent Judge erred in granting bail to the accused – NO

RULING: The complaint against respondent Judge Gregorio Rosete for


misconduct and gross ignorance of the law is DISMISSED for lack of merit.

RATIO:
Cardines contend that one of the offenses charged is illegal recruitment
in large scale constituting economic sabotage under Art. 38 of the
Labor Code as it was committed against three or more persons
individually or as a group. Since the penalty for economic sabotage
is life imprisonment and a fine of P100,000, Claro should not have
been granted bail.
Judge Rosete maintains that under Sec. 3 Rule 114 of the 1985 Rules on
Criminal Procedure, the accused charged with an offense penalized
with life imprisonment should be granted bail as a matter of right.
1985 Rules on Criminal Procedure. Thus, under Rule 114, when a person is charged with an offense
Sec 3 Rule 114 provides that all persons in custody shall, before final which is not capital, or one which is not punishable with
conviction, be entitled to bail as a matter of right, except those reclusion perpetua or life imprisonment, admission to bail is a
charged with matter of right.
capital offense or an offense which under the law at the time of its However, when the imposable penalty for the offense charged is
commission and at the time of the application for bail, is punishable death,
by reclusion perpetua.
Clearly, the rule at that time was that a person under detention shall
before conviction be granted bail as a matter of right. Two exceptions
were recognized:
When the person was charged with a capital offense, or
When the offense charged was punishable with reclusion perpetua
Interestingly, “life imprisonment” was not among the exceptions, which
leads the Court to the conclusion that persons accused of crimes
punishable with “life imprisonment” were entitled to bail as a matter
of right.
Cardines finds difficulty disassociating the concept of “life imprisonment”
from “reclusion perpetua.” As the Court repeated, these terms are
not synonymous.
“Life imprisonment” may appear to be the English translation of
“reclusion perpetua,” in law it goes deeper than that. Life
imprisonment is imposed for serious offenses penalized by special
laws, while reclusion perpetua is prescribed under the RPC.
Life imprisonment does not carry with it any accessory penalty, unlike
reclusion perpetua. Moreover, life imprisonment does not appear to have
any definite extent or duration, while reclusion perpetua entails
incarceration for at least 30 years after which the convict becomes
eligible for pardon.
There is no better proof of the non-inclusion of “life imprisonment” in
reclusion perpetua than Adm. Cir. No. 12-94 which amended Rule
114 of the 1985 Rules on Criminal Procedure.
Under Adm. Cir. No. 12-94, a person charged with an offense
punishable with death, reclusion perpetua or life imprisonment is
no longer entitled to bail as a matter of right when evidence of guilt
is strong.
If “life imprisonment” be considered embraced in reclusion perpetua, as
implied from Cardines’ arguments, there would have been no need
for its separate inclusion in Adm. Cir. No. 12-94.
Obviously, the amendments therein cannot be applied retroactively to
the present case since the application would be unfavorable to the
accused.
reclusion perpetua or life imprisonment, a person charged is no
longer entitled to bail as a matter of right for then admission to bail is
addressed to the sound discretion of the court depending on whether or
not the evidence of guilt is strong.
When bail is a matter of discretion and the detainee who is charged in a criminal
complaint with the municipal court for preliminary investigation files a motion
for bail, the prosecution must be given an opportunity to present all the
evidence it may desire to introduce before resolving the motion.
In the case of respondent Judge Rosete, even assuming arguendo that he did
not give the prosecution an opportunity to be heard and present evidence on
the guilt of the accused, he cannot be faulted because the crime charged
is punishable with life imprisonment which at the time of its
commission and the filing of the criminal complaint was bailable as a
matter of right.
PADERANGA v. CA (IYA) Galarion later escaped from prison.
August 28, 1995 | Regalado, J. | Right to Bail In a bizarre twist of events, one Felizardo Roxas was implicated.
In an amended information, he was charged as co-accused.

PETITIONER: Miguel Paderanga


RESPONDENTS: Court of Appeals and People of the Philippines

SUMMARY: Paderanga was imputed as mastermind to the killing of the


Bucag family. Before a warrant of arrest could be served on him, he filed
through his counsel a motion for admission to bail. State Prosecutor
Gongoyon was assigned to represent the prosecution.

He was not able to appear before the court during the trial of the motion
because at that time he was confined in a nearby hospital. The RTC
nonetheless proceeded with the case. Assistant Prosecutor Abejoy for
the prosecution as present and manifested that there were no objections
to the motion for admission to bail. The RTC then granted Paderanga
bail.

Months later, State Prosecutor Gingoyon questions the order granting


bail (CA proceedings). He avers that Paderanga was never “in custody of
the law” and therefore cannot request for bail. He also questions the
authority of Abejoy to represent the prosecution. CA rules in favor of
Gingoyon.

The SC reverses the decision, and reinstates the RTC order granting
bail. Read entire Ratio for explanation.

DOCTRINE: As bail is intended to obtain or secure one’s provisional


liberty, the same cannot be posted before custody over him has been
acquired by the judicial authorities, either by his lawful arrest or voluntary
surrender.

As the grant of bail becomes a matter of judicial discretion on the part of


the court under the exceptions to the rule, a hearing, mandatory in
nature and which should be summary or otherwise in the discretion of
the court, is required with the participation of both the defense and a duly
notified representative of the prosecution, this time to ascertain whether
or not the evidence of guilt is strong for the provisional liberty of the
applicant.

FACTS:
On January 28, 1990, petitioner was belatedly charged in an amended
information as a coconspirator in the crime of multiple murder for the
killing of members of the Bucag family.
Initially, there were 8 accused suspects. However, only one of the
accused, Fleipe Galarion, was apprehended and was eventually
convicted.
Miguel Paderanga was his (Roxas) former employer. Paderanga acted as
counsel for Roxas. Note that during the killing, Paderanga was the Mayor of
the town it occured.
In the course of the preliminary investigation, Roxas in a signed affidavit
implicated Paderanga as the supposed mastermind behind the massacre of
the Bucag family.
The Department of Justice, at the instance of the City Prosecutor of Cagayan,
designated a replacement, State Prosecutor Henrick F. Gingoyon, for
purposes of both the preliminary investigation and prosecution of Criminal
Case.
The trial of the case was all set to start with the issuance of an arrest warrant for
Paderanga’s apprehension but, before it could be served on him, Paderanga,
through counsel, filed on October 28, 1992 a motion for admission to bail
with the trial court which set the same for hearing on November 5, 1992.
Paderanga duly furnished copies of the motion to State Prosecutor Henrick F.
Gingoyon, the Regional State Prosecutor’s Office, and the private
prosecutor, Atty. Benjamin Guimong.
On November 5, 1992, the trial court proceeded to hear the application for bail.
Only Assistant Prosecutor Erlindo Abejo of the Regional State Prosecutor’s
Office appeared for the prosecution.
As Paderanga was then confined at the Cagayan Capitol College General
Hospital due to “acute costochondritis,” his counsel manifested that they
were submitting custody over the person of their client to the local chapter
president of the Integrated Bar of the Philippines and that, for purposes of
said hearing on his bail application, he be considered as being in the custody
of the law.
Prosecutor Abejo informed the trial court that in accordance with the directive of the
chief of their office, Regional State Prosecutor Jesus Zozobrado, the prosecution
was neither supporting nor opposing the application for bail and that they were
submitting the same to the sound discretion of the trial judge.
Prosecutor Abejo announced that he was waiving any further presentation of
evidence.
The trial curt admitted Paderanga to bail int he amount of P200,000.
A subsequent motion for reconsideration of said resolution was filed twenty
days later on November 26, 1992 by Prosecutor Gingoyon, who allegedly
received his copy of the petition for admission to bail on the day after the
hearing, was denied by the trial court in its omnibus order.
Prosecutor Gingoyon elevated the matter to the CA on the ground that the order
granting bail is tainted with grave abuse of discretion and therefore should be
annulled.
Court of Appeals observed in its decision that at the time of Paderanga’s
application for bail, he was not yet “in the custody of the law,” apparently
because he filed his motion for admission to bail before he was actually
arrested or had voluntarily surrendered.
It further noted that apart from the circumstance that Paderanga was Paderanga cites Santiago v. Vasquez in his argument stating that
charged with a crime punishable by reclusion perpetua, the evidence he had similar situations as to the petitioner in the said case:
of guilt was strong as borne out by the fact that no bail was Petitioner was at the time confined in a hospital
recommended by the prosecution, for which reasons it held that the recuperating from serious physical injuries which she
grant of bail was doubly improvident. sustained in a major vehicular
Lastly, the prosecution, according to the CA, was not afforded an
opportunity to oppose petitioner’s application for bail contrary to the
requirements of due process.
The review on the ground that the order granting bail is tainted with grave
abuse of discretion and therefore should be annulled.
Hence, this appeal.

ISSUE/s: ( Kaya two times siya nasa list sa syllabus ni Ma’am quasi two
important discussions were made. Please take time to read the entire ratio.)
WON Paderanga should be granted bail - YES (RATIO 1 TO 13)
WON due process was accorded to the prosecution - YES (RATIO 14 TO
26)

RULING: The order of the RTC granting bail o Paderanga is REINSTATED.

RATIO:
Section 1 of Rule 114, as amended, defines bail as the security given for
the release of a person in custody of the law, furnished by him or a
bondsman, conditioned upon his appearing before any court as
required under the conditions specified in said Rule.
Its main purpose, then, is to relieve an accused from the rigors of
imprisonment until his conviction and yet secure his appearance at
the trial.
As bail is intended to obtain or secure one’s provisional liberty, the
same cannot be posted before custody over him has been acquired
by the judicial authorities, either by his lawful arrest or voluntary
surrender.
The rationale behind the rule is that it discourages and prevents resort to the
former pernicious practice whereby an accused could just send another
in his stead to post his bail, without recognizing the jurisdiction of the
court by his personal appearance therein and compliance with the
requirements therefor.
Thus, in Feliciano vs. Pasicolan, where the petitioner who had been charged
with kidnapping with murder went into hiding without surrendering
himself, and shortly thereafter filed a motion asking the court to fix the
amount of the bail bond for his release pending trial, the Supreme Court
categorically pronounced that said petitioner was not eligible for
admission to bail.
As a paramount requisite then, only those persons who have either been
arrested, detained, or otherwise deprived of their freedom will ever
have occasion to seek the protective mantle extended by the right to
bail.
mishap.
Consequently, she expressly sought leave “that she be considered as
having placed herself under the jurisdiction of the Sandiganbayan for
purposes oft he required trial.
On the basis of said ex parte motion and the peculiar circumstances
obtained in that incident, the Sandiganbayan authorized petitioner
to post a cash bail bond for her provisional liberty without need of
her personal appearance in view of her physical incapacity and as
a matter of humane consideration.
In the case of Paderanga, it may be conceded that he had indeed filed his motion
for admission to bail before he was actually and physically placed under
arrest. He may, however, at that point and in the factual ambience thereof, be
considered as being constructively and legally under custody.
He should be deemed to have voluntarily submitted his person to the custody of
the law and, necessarily, to the jurisdiction of the trial court which thereafter
granted bail as prayed for.
The undeniable fact is that Paderanga was by then in the constructive custody of
the law.
Apparently, both the trial court and the prosecutors agreed on that point since
they never attempted to have him physically restrained.
Through his lawyers, he expressly submitted to physical and legal control over his
person, firstly, by filing the application for bail with the trial court; secondly, by
furnishing true information of his actual whereabouts; and, more importantly, by
unequivocally recognizing the jurisdiction of the said court.
Moreover, when it came to his knowledge that a warrant for his arrest had been
issued, petitioner never made any attempt or evinced any intent to evade the
clutches of the law or concealed his whereabouts from the authorities since
the day he was charged in court, up to the submission of his application for
bail, and until the day of the hearing thereof.
Section 13, Article III of the Constitution lays down the rule that before conviction,
all indictees shall be allowed bail, except only those charged with offenses
punishable by reclusion perpetua when the evidence of guilt is strong.
In pursuance thereof, Section 4 of Rule 114, as amended, now provides that all
persons in custody shall, before conviction by a regional trial court of an
offense not punishable by death, reclusion perpetua or life imprisonment, be
admitted to bail as a matter of right.
The right to bail, which may be waived considering its personal nature and which,
to repeat, arises from the time one is placed in the custody of the law, springs
from the presumption of innocence accorded every accused upon whom
should not be inflicted incarceration at the outset since after trial he would be
entitled to acquittal, unless his guilt be established beyond reasonable doubt.
Upon proper application for admission to bail, the court having custody of the
accused should, as a matter of course, grant the same after a hearing
conducted to specifically determine the conditions of the bail in
accordance with Section 6 (now, Section 2) of Rule 114.
As the grant of bail becomes a matter of judicial discretion on the
part of the court under the exceptions to the rule, a hearing,
mandatory in nature and which should be summary or
otherwise in the discretion of the court, is required with the
participation of both the defense and a duly notified
representative of the prosecution, this time to ascertain whether
or not the evidence of guilt is strong for the provisional liberty
of the applicant.
If the prosecution is denied this opportunity, there would be a denial of
procedural due process, as a consequence of which the court’s order
in respect of the motion or petition is void.
In this appeal, the prosecution assails what it considers to be a violation
of procedural due process when the court below allowed Assistant
Prosecutor Erlindo Abejo of the Regional State Prosecutor’s Office to
appear in behalf of the prosecution, instead of State Prosecutor
Henrick F. Gingoyon who is claimed to be the sole government
prosecutor expressly authorized to handle the case and who
received his copy of the motion only on the day after the hearing had
been conducted.
The SC is not impressed by this argument. Although it is now claimed
that Prosecutor Abejo was allegedly not familiar with the case, he
nonetheless was explicitly instructed about the position of the
Regional State Prosecutor’s Office on the matter.
Prosecutor Zozobrado, whose office received its copy of the motion on
the very day when it was sent, that is, October 28, 1992, duly
instructed Prosecutor Abejo to manifest to the court that the
prosecution was neither supporting nor opposing the application for
bail and that they were submitting the matter to its sound discretion.
(See: Fact #13)
It is further evident that the prosecution, on the instructions of Regional
State Prosecutor Zozobrado, had no intention at all to oppose the
motion for bail and this should be so notwithstanding the statement
that they were “neither supporting nor opposing” the motion.
In Re: First Indorsement case (among others cited) it was held that
where the prosecutor interposes no objection to the motion of the
accused, the trial court should nevertheless set the application for
hearing and from there diligently ascertain from the prosecution
whether the latter is really not contesting the bail application.
No irregularity, in the context of procedural due process, could therefore be
attributed to the trial court here as regards its order granting bail to
Paderanga.
A review of the transcript of the stenographic notes abundantly reveals
scrupulous adherence to procedural rules.
YAP v. CA (ELIEL) WoN the proposed bail of P5,500,000 was violative of right against
June 6, 2001 | Gonzaga-Reyes, J. | Excessive Bail excessive bail – YES

PETITIONER: Francisco Yap, Jr.


RESPONDENTS: CA and People of the Philippines

SUMMARY: Francisco Yap was found guilty beyond reasonable doubt of


the crime of Estafa by the RTC. On appeal, he filed for a motion to fix
bail, to which the OSG set at P5,500,000 – equivalent to the civil liability
of Yap. He contends that the amount fixed for bail is excessive, thus a
violation of his constitutional right against excessive bail.

The SC held that indeed the amount fixed for bail is excessive and would
render the right to bail nugatory. However, the SC stated that it has
options and the discretion to set the amount of bail at a certain amount
for justified reasons. In this case, Yap is a flight risk as admitted by him;
hence, the amount of P200,000 is reasonable enough to ensure the
appearance of Yap during trial.

DOCTRINE: The purpose for bail is to guarantee the appearance of the


accused at the trial, or whenever so required by the court. The amount
should be high enough to assure the presence of the accused when
required but no higher than is reasonably calculated to fulfill this purpose

FACTS:
For misappropriating amounts equivalent to P5,500,000.00, Yap was
convicted of estafa by the RTC of Pasig City and was sentenced to 4
years and 2 months of prision correcicional, as minimum, to 8 years
of prision mayor as maximum, in addition to 1 year for each
additional P10,000.00 in excess of P22,000.00 but in no case shall it
exceed 20 years.
He filed a notice of appeal, and moved to be allowed provisional liberty
under the cash bond he had filed earlier in the proceedings. The
motion was denied by the trial court.
After the records of the case were transmitted to the CA, Yap filed with
the said court a Motion to Fix Bail For the Provisional Liberty of Yap
Pending Appeal, invoking the last paragraph of Sec 5, Rule 114 of
the 1997 Revised Rules of Court
Asked to comment on this motion, the SolGen opined that Yap amy be
allowed to post bail in the amount of P5,500,000.00 and be required
to secura a certification/guaranty from the Mayor of the place of his
residence that he is a resident of the area and that he will remain to
be so until final judgment is rendered or in case he transfers
residence, it must be with prior notice to the court and private
complainant.

ISSUE/s:
Nature and circumstances of the offense;
RULING: Bail is fixed at P200,000

RATIO:
Sec 5, Rule 114 of the 1997 Rules of Court
SEC 5. Bail, when discretionary – Upon conviction by the
Regional Trial Court of an offense not punishbale by death,
reclusion perpetua or life imprisonment, the court, on
application, may admit the accused to bail
The court, in its discretion, may allow Yap to continue on provisional
liberty under the same bail bond during the period to appeal subject
to the consent of the bondsman
If the court imposed a penalty of imprisonment exceeding 6 years, but
not more than 20 years, accused shall be denied bail, or his bail
previously granted shall be cancelled, upon a showing by the
prosecution, with notice to the accused, of the following or other
similar circumstances:
That the accused is a recidivist, quasi-recidivist, or habitual
delinquent, or has committed the crime aggravated by the
circumstance of reiteration
That the accused is found to have previously escaped from legal
confinement, evaded sentence, or has violated the
conditions of his bail without valid justification;
That the accused committed the offense while on probation,
parole, or under conditional pardon;
That the circumstances of the accused or his case indicate the
probability of flight if released on bail; or
That there is undue risk that during the pendency of the appeal,
the accused may commit another crime
There is no question that in the present case the CA exercised its
discretion in favor of allowing bail to Yap on appeal. CA stated that it
was doing so for humanitarian reasons, and despite a perceived high
risk of flight, as by Yap’s admission he went out of the country
several times during the pendency of the case, for which reason the
court deemed it necessary to peg the amount of bail at
P5,500,000.00
The prohibition against requiring excessive bail is enshrined in the
Constitution. The obvious rationale, as decalred in the leading case
of De la Camara v. Enage, is that imposing bail in an excessive
amount could render meaningless the right to bail
Villaseor v. Abao, this Court made the pronouncement that it will not
hesitate to exercise its supervisory powers over lower courts should
the latter, after holding the accused entitled to bail, effectively deny
the same by imposing a prohibitory sum or exacting unreasonable
conditions
Sec 9, Rule 114 of the Revised Rules of Criminal Procedure advises
courts to consider the following factors in the setting of the amount of
bail
Financial ability of the accused to give bail;
Penalty for the offense charged; In the same vein, the Court has held that the discretion to extend bal
Character and reputation of the accused; during the course of the appeal should be exercised with grave
Age and health of the accused; caution and for strong reasons, considering that the accused had
Weight of the evidence against the accused; beein in fact convicted by the trial
Probability of the accused appearing at the trial;
Forfeiture of other bail;
The fact that the accused was a fugitive from justice when
arrested; and
Pendency of other cases where the accused is on bail
Thus, the court has wide latitude in fixing the amount of bail. Where
it fears that the accused may jump bail, it is certainly not
precluded from installing devices to ensure against the same.
Options may include increasing the bail bond to an appropriate
level, or requiring the person to report periodically to the court
and to make an accounting of his movements
In the present case, where Yap was found to have left the country
several times while the case was pending, the CA required the
confiscation of his passport and the issuance of a hold-departure
order against him
Under the circumstances of this case, we find that appropriate conditions
have been imposed in the bail bond to ensure agasint the risk of flight,
particularly, the combination of the hold-departure order and the
requirement that Yap inform the court of any change of residence and of
his whereabouts.
Although an increase in the amount of bail while the case is on appeal
may be meritorious, we find that the setting of the amount at
P5,500,000.00 is unreasonable, excessive, and constitutes an
effective denial of Yap’s right to bail.
The purpose for bail is to guarantee the appearance of the accused
at the trial, or whenever so required by the court. The amount
should be high enough to assure the presence of the accused
when required but no higher than is reasonably calculated to
fulfill this purpose
To fix bail at an amount equivalent to the civil liability of which Yap
is charged is to permit the impression that the amount paid as
bail is an exaction of the civil liability that Yap is charged of; this
we cannot allow because bail is not intended as a punishment,
nor as a satisfaction of civil liability which should necessarily
await the judgment of the appellate court
It militates emphasis that Yap is seeking bail on appeal. Sec 5, Rule
114 of the Revised Rules of Criminal Procedure is clear that
although the grant of bail on appeal in non-capital offenses is
discretionary, when the penalty iposed on the convicted
accused exceeds 6 years and circumstances exist that point to
the probability of flight if released on bail, then the accued
mustbe denied bail, or his bail previously granted should be
cancelled
court
Yap is seeking bail on appeal. He was in fact declared guilty beyond
reasonable doubt by the RTC, and due to the serious amount of fraud
involved, sentenced to imprisonment for 20 years – the maximum penalty
for estafa by false pretenses or fraudulent acts allowed by the RPC.
Although it cannot be controverted that the CA, despite the foregoing
considerations and the possibility of flight still wielded its
discretion to grant Yap bail, the setting of bail in the amount of
P5,500,000.00 is unjustified as having no legal nor factual basis.
Guided by the penalty imposed by the lower court and the weight of the
evidence against Yap, we believe that the amount of P200,000 is
more reasonable
NARCISO v. STA. ROMANA CRUZ (JP) Bail.
March 17, 2000 | Panganiban, J. | Right to Bail Such trial was postponed then it was postponed again. Not obtaining any
resolution on her Motion to lift Order Allowing Accused to Post Bail,
PETITIONER: Joselito V. Narciso private complainant filed this petition before the CA.
RESPONDENTS: Flor Marie Sta. Romana-Cruz The CA annulled the RTC order granting the bail. Hence this appeal.
Petitioner-Accused Narciso claims that the CA erred when it
SUMMARY: Accused Joselito Narciso was found guilty of the crime of reversed and set aside the order of the Regional Trial Court of
parricide for the death of his wife, Corazon. He filed for a motion to be Quezon City which granted the petitioner his constitutional right
allowed to post bail. The RTC granted such on the same since there was to bail, considering the absence of strong evidence or proof of
no objection from the prosecutor. The CA reversed. Petitioner Joselito his guilt, and more especially when the public prosecutors, who
appealed alleging that his evident of guilt is not strong and the have direct control of the proceedings and after assessment of
prosecutor’s silence was a recommendation for the grant of bail. the evidence, have themselves recommended the grant of bail.

The SC held that a hearing is required to determine whether an ISSUE/s:


accused’s guilt is is strong. Absence such, such granting of bail is void for WoN the grant of bail by the RTC was valid and the CA erred in annulling
having been issued with grave abuse of discretion. Here, the motion for such – NO
bail was approved in so little time that it is impossible that a hearing was
RULING: CA decision is affirmed.
indeed conducted. The SC affirmed the CA. The decision granting the
motion to bail must be annulled.
RATIO:
DOCTRINE: The determination of whether or not the evidence of guilt Section 13, Article III of the Constitution provides: "All persons, except
those charged with offenses punishable by reclusion perpetua when
is strong, being a matter of judicial discretion, remains with the judge.
evidence of guilt is strong, shall, before conviction, be bailable by
This discretion by the very nature of things, may rightly be exercised sufficient sureties, or be released on recognizance as may be
only after the evidence is submitted to the court at the hearing. provided by law.
Although petitioner was charged with parricide which is punishable with
Whether summary or otherwise in the discretion of the court, must reclusion perpetua, he argued before the CA that he was entitled to
actually be conducted to determine whether or not the evidence of guilt bail because the evidence of his guilt was not strong. He contended
against the accused is strong. that the prosecutor's conformity to his Motion for Bail was tantamount
to a finding that the prosecution evidence against him was not strong.
The Court of Appeals ruled, however, that there was no basis for
FACTS: such finding of his guilt, since no hearing had been conducted
on the application for bail -- summary or otherwise.
After conducting a preliminary investigation on the death of Corazon Sta.
The appellate court found that only ten minutes had elapsed
Romana-Narciso, wife of accused Joselito Narciso, the Asst. City
between the filing of the Motion by the accused and the Order
Prosecutor filed an information for parricide against Joselito.
granting bail, a lapse of time that could not be deemed sufficient
Accused filed an Urgent Ex-Parte to Allow Accused to Post Bail.
for the trial court to receive and evaluate any evidence. The SC
The RTC decided on the same day that there being no objection by
agrees with the CA.
the City Prosecutor Candido Rivera to the accused posting a
cashbond of P150k, the RTC Judge approves the Motion to The determination of whether or not the evidence of guilt is strong, being
Allow Accused Narciso to Post Bail. a matter of judicial discretion, remains with the judge. This discretion
The private prosecutor representing private complainant Flor Marie Sta. by the very nature of things, may rightly be exercised only after the
Romana-Cruz, a sister of accused's deceased wife, filed an "Urgent evidence is submitted to the court at the hearing.
Motion to Lift Order Allowing Accused To Post Bail'. Whether summary or otherwise in the discretion of the court, must
Accused Narciso objected to such motion. Complainant Flor Marie filed actually be conducted to determine whether or not the evidence of
her opoosition to the motion to expunge. guilt against the accused is strong.
Complainant Narciso moved for the postponement of the trials pending A summary hearing means such brief and speedy method of receiving
the resolution of their Urgent Motion to Lift Order Allowing Accused to and considering the evidence of guilt as is practicable and consistent
Post
with the purpose of hearing which is merely to determine the weight
of evidence for
the purposes of bail.
The absence of objection from the prosecution is never a basis for the
grant of bail in such cases, for the judge has no right to presume that
the prosecutor knows what he is doing on account of familiarity with
the case.
Said reasoning is tantamount to ceding to the prosecutor the duty of
exercising judicial discretion to determine whether the guilt of the
accused is strong.
Judicial discretion is the domain of the judge before whom the petition for
provisional liberty will be decided. The mandated duty to exercise
discretion has never been reposed upon the prosecutor.
In Basco v. Rapatalo, the SC enunciated the duties of the trial judge
in such petition:
Notify the prosecutor of the hearing for the application for
bail and requrie him to submit his recommendation.
Conduct the hearing regardless whether the prosecution
refuses to present evidence
Decide wehter the evidence of guilt is strong based on the
summary evidence of the prosecution
If the guilt is not strong, discharge the accused upon the
approval of the bailbond. Otherwise, petition is denied.
Additionally, the court's grant or refusal of bail must contain a summary of
the evidence for the prosecution, on the basis of which should be
formulated the judge's own conclusion on whether such evidence is
strong enough to indicate the guilt of the accused.
Clearly, the grant of bail by Executive Judge Santiago was laced with
grave abuse of discretion and the Court of Appeals was correct in
reversing him.
PEOPLE v. CABRAL (HENRY) evidence against him was strong, hence they filed 2 motions with the
February 18, 1999 | Romero, J. | Right to bail lower court, which was denied.
As recourse, the People filed a petition before the CA, which was
PETITIONER: People of the Philippines likewise denied. The Ca ruled that they examined in close detail the
RESPONDENTS: Hon. Alfredo Cabral (RTC Br 30 Camarines Sur records of the case and that they are not inclined to declare that
there was grave abuse in the RTC’s exercise of discretion in allowing
Presiding Judge), and Roderick Odiamar
Odiamar to obtain bail. Hence, this petition.
SUMMARY: This case basically revolves around the fact that the RTC
ISSUE/s:
approved the motion to bail of Odiamar. Insofar as the relevant facts are
concerned, we are talking about at a rape case by Odiamar, with WoN the CA erred in affirming the RTC ruling granting bail to Odiamar –
Odiamar as the accused. He raped her in the resort after making her YES
believe that she is in a joy ride, to which she consented (sa joyride
RULING: SC reversed the CA decision and ordered the rearrest of Odiamar if
consented, not sa rape). Petitioner People is assailing such grant to bail
but was denied both in the RTC and the CA. Hence, this present petition. his bail bond has been approved

RATIO:
SC reversed the ruling and held that although bail is a matter of discretion
of the court when evidence of guilt is strong, it is very evident that in the SC noted the lower court’s ruling that the evidence of guilt was nt strong,
case at bar, the RTC did not properly follow the guidelines and to which the OSG disagreed with.
procedures insofar as judicial discretion is concerned. Among others, the It was, notably, the OSG’s arguments back in the hearing for bail that has
blatant disregard of the RTC of expert testimonies and the psychological impressed the SC to find merit in the present petition.
effect to Cecilla of the incident was what made the RTC perceive of the Art 3 Sec 13 of the BoR provides, “All persons, except those charged
evidence as not that strong—which altogether led for them to grant bail. with offenses punishable by reclusion perpetua when evidence of
This in turn should then have the SC hold that it is but propert that the guilt is strong, shall before conviction, be bailable by sufficient
bial not be granted. Accordingly, the rearrest of Odiamar is ordered if in surities, or be released on recognizance as may be provided by law.
case the bail bond was granted. The right to bail shall not be impaired even when the privilege of the
writ of habeas corpus is suspended. Excessive bail shall not be
DOCTRINE: In cases where the accused is charged with a crime required.”
punishable by reclusion perpetua to death, bail is discretionary and not a In view of the above exception to the constitutional guarantee on bail and
matter of right. The grant or denial of an application for bail is dependent in accordance with its rule-making powers, SC in promulgating the
on whether the evidence of guilt is strong which the lower court should Rules of Court, adopted the ff provision, “No person charged with a
capital offense, or an offense punishable by reclusion perpetua or life
determine in a hearing called for the purpose.
imprisonment, when evidence of guilt is strong, shall be admitted to
bail regardless of the stage of the criminal prosecution.”
FACTS: In the present case, Odiamar was being charged with rape qualified by
Accused Roderick Odiamar was charged with rape upon complaint of the use of a deadly weapon punishable by reclusion perpetua to
Cecille Buenafe. death. As such, bail is discreationary and not a matter of right. The
Cecille rode a jeepney driven by Odiamar in the evening of July 20. She grant or denial of an application of bail is, therefore, dependent on
was aware that it was a “joy ride” and did not even protest when the whether the evidence of guilt is strong which the lower court should
jeep proceeded to Pilapil Beach Resort, instead of Sabang, where determine in a hearing called for that purpose.
she and Stephen Florece intended to go. By judicial discretion, the law mandates the determination of whether
Upon arrival in the resort, Cecille followed Odiamar. Here she was forced proof is evident or the presumption of guilt is strong
to drink alcohol until she was drunk and smoke and take drugs. Proof evident / evident proof – clear, strong evidence which leads
Afterwhich, she was taken in a cottage where she was raped. a well-guarded dispassionate judgment to the conclusion that
In a bid to secure temporary liberty, Odiamar filed a motion praying that the offense has been committed as charged, that accused is
he be released on bail which petitioner People / Cecille opposed by the guilty agent, and that he will probably be pnished
presenting real, documentary, and testimonial evidence. capitally if the law is administered
RTC granted the motion for bail. Presumption great – exists when circumstances testified to are
The prosecution (People),, believed that Odiamar is not entitled to bail as the such
that the inference of guilt naturally to be drawn therefrom is The applicable provisions of the Constitution and the statutes
strong, clear, and convincing to an unbiased judgment and – when evidence of guilt is strong, bail is not a matter of
excludes all reasonable probability of any other conclusion right
Other words, the test is not whetherthe evidence establishes guilt The rules which the SC may promulgate – sec 7, rule 14
beyond reasonable doubt but rather whetehr it shows evident
guilt or a great presumption of guilt.
As such, the court is ministerially bound to decide which circumstances
and factors are present which would show evident guilt or
presumption of guilt as defined.
SC observed that the RTC’s order failed to mention significant factors
and circumstances which to the SC’s mind are strong, clear, and
convincing:
It executed a testimony of one Dr. Belmonte as well as her
findings that Cecilla manifested psychotic signs and
symptoms such as unusual fear, sleeplessness, suicidal
thouhts, among others. This testimony should have
considered and included in the summary as it was given by
an expert witness
The unrebutted offer of compromise by Odiamar is an implied
admission of guilt which should have been noted as an offer
of a compromise which is generally considered as
admissible evidence against the party making it (in this case,
Odiamar).
Besides that, the RTC misapplied some doctrines in criminal law:
RTC intoned the following doctrine that “evidence to be believed
must not only proceed from the mouth of a credible witness
but it must be credible in itself in conformity with common
experience and observation of mankind”
According to the RTC, the credibility of the complaint is suspect because
she willingly went with Odiamar to the resort where she was
allegedly raped, among others. Because of these findings, the RTC
doubted the credibility of Cecilla and stated that the crime of rape is
not to be presumed and that the sexual acts were presumed to be
consensual.
SC disagrees to the above (ratio 11) as Cecille was just consenting to the
entrance to the resort as she was in the impression that it was a joy
ride. RTC’s conclusion is bereft of any legal or factual support.
The SC ruled that the facgt that vital prosecution evidence and
testimonies have been irregularly disregarded indicate that they have
not been considered at all in arriving at the decision to grant bail. The
SC can’t help but observe that RTC exerted efforts to show that the
evidence of guilt is not strong by its non sequitur justifications.
It is the view of the SC that the expert testimonies of the Doctors who
attended to Cecilla are indications of strength of the evidence of guilt
of Odiamar.
Moreover, the SC held that RTC did not strictly comply to the
jurisprudencial guidelines in the exercise of discretion guided by:
Those principles of equity and justice which are deemed to be part of
the laws of the land
Recently the SC laid down the following duties of a judge in case an
application for bail is filed:
Notify prosecutor of hearing of application for bail to require him to
submit recommendation
Conduct hearing for the said application regardless of whether or not
prosecution refuses to present evidence to show that guilt of
accused is strong for the purpose ofenabling the court to exercise
discretion
Decide whether the evidence of guilt of the accused is strong based
on the summary of evidence of the prosecution
If guilt is not strong, discharge the accucsed upon approval of
bailbond. Otherwise, deny petition
Based on the above procedure, after hearing, the court’s order granting or
refusing bail must contain a summary of evidence for the prosecution.
Which is essential because:
Summary of the evidence in the order is an extension of the hearing
proper, thus, a part of procedural due process wherein the
evidence presented during the prior hearing is formally
recognized as having been presented and most importantly,
considered
Summary of the evidence in the order is the basis for te judge’s
exercising of his judicial discretion
Vitug, dissenting: In an indictment for a capital offense, the accused is not
entitled to bail when the evidence of guilt is strong, and it is the duty of
the judge to hear the parties and to make an intelligent assessment of
the evidence presented. When the judge views the evidence of guilt in
such a capital offense not to be strong, the grant of bail becomes a
matter of sound discretion on his part. For the grant of bail to be rightfully
dismissed in this case, it must be in such a case that the judge
committed grave abuse of discretion. In this case such instance is absent
and therefore, the case should accordingly be dismissed instead of the
petition being granted.
GOV’T OF USA v. PURGANAN (DANNAH) appropriate action, pursuant to Section 5 of Presidential
September 24, 2002 | Davide, Jr., J. | Right to Bail: Extradition Decree (PD) No. 1069, also known as the Extradition Law.

PETITIONER: Gov’t of the USA rep. by Philippine DOJ


RESPONDENTS: Hon. Guillermo Purganan, Mark Jimenez

SUMMARY: On May 18, 2001, the Government of the USA, represented


by the Philippine Department of Justice, filed with the RTC, the Petition
for Extradition praying for the issuance of an order for Mark Jimenez’
“immediate arrest” pursuant to Sec. 6 of PD 1069 in order to prevent the
flight of Jimenez.

Before the RTC could act on the petition, Mr. Jimenez filed before it an
“Urgent Manifestation/Ex-Parte Motion” praying for his application for an
arrest warrant be set for hearing. After the hearing, as required by the
court, Mr. Jimenez submitted his Memorandum. Therein seeking an
alternative prayer that in case a warrant should issue, he be allowed to
post bail in the amount of P100,000. The court ordered the issuance of a
warrant for his arrest and fixing bail for his temporary liberty at P1M in
cash. After he had surrendered his passport and posted the required
cash bond, Jimenez was granted provisional liberty.

The DOJ questioned said order (will discuss the bail issue lang here).
The Court agreed with the DOJ that there is no provision in the Philippine
Constitution granting the right to bail to a person who is the subject of an
extradition request and arrest warrant. As suggested by the use of the
word “conviction,” in Section 13, the constitutional provision on bail
applies only when a person has been
arrested and detained for violation of Philippine criminal laws. It does not
apply to extradition proceedings, because extradition courts do not
render judgments of conviction or acquittal.

DOCTRINE: To stress, extradition proceedings are separate and distinct from the
trial for the offenses for which he is charged. [He] should apply for bail before the
courts trying the criminal cases against him, not before the extradition court.

FACTS:
Pursuant to the existing RP-US Extradition Treaty, the US Government
sent to the Philippine Government Note Verbals, accompanied by
duty authenticated documents requesting the extradition of Mark B.
Jimenez, also known as Mario Batacan Crespo.
Upon receipt of the Notes and documents, the secretary of foreign affairs
(SFA) transmitted them to the secretary of justice (SOJ) for
Upon learning of the request for his extradition, Jimenez sought and was granted
a Temporary Restraining Order (TRO) by the RTC of Manila, Branch 25
The TRO prohibited the Department of Justice (DOJ) from filing with the RTC a
petition for his extradition. The validity of the TRO was, however, assailed by
the SOJ in a Petition before this Court in the said G.R. No. 139465.
Initially, the Court—by a vote of 9-6—dismissed the Petition. The SOJ was
ordered to furnish Jimenez copies of the extradition request and its
supporting papers and to grant the latter a reasonable period within which to
file a comment and supporting evidence.
In the MR, by an identical vote of 9-6 after three justices changed their votes—it
reconsidered and reversed its earlier Decision.
It held that Jimenez was bereft of the right to notice and hearing during the
evaluation stage of the extradition process. This Resolution has become final
and executory.
Finding no more legal obstacle, the Government of the United States of America,
represented by the Philippine DOJ, filed with the RTC on May 18, 2001, the
appropriate Petition for Extradition
The Petition alleged that Jimenez was the subject of an arrest warrant issued by
the United States District Court for the Southern District of Florida on April
15, 1999.
Warrant was issued because of:
Conspiracy to defraud the United States and to commit certain offenses in
violation of Title 18 US Code Section 371
Tax evasion
Wire fraud
False statements
Illegal campaign contributions
In order to prevent the flight of Jimenez, the Petition prayed for the issuance of
an order for his “immediate arrest” pursuant to Section 6 PD 1069.
RTC granted the Motion of Jimenez and set the case (for the hearing of his arrest
warrant).
In the memorandum Jimenez submitted, he sought an alternate prayer: that in
case a warrant should issue, he be allowed to post bail in the amount of
P100,000.
The court then issued its questioned Order, directing the issuance of a warrant
for his arrest and fixing bail for his temporary liberty at one million pesos in
cash.
After he had surrendered his passport and posted the required cash bond,
Jimenez was granted provisional liberty via the challenged Order dated July
4, 2001 from these records adequate for him to make an initial determination
of whether the accused was someone who should immediately be
ISSUE/s: arrested in order to “best serve the ends of justice.”
WoN Jimenez is entitled to notice and hearing before a warrant for his He could have determined whether such facts and circumstances existed
arrest can be issued – NO as would lead a reasonably discreet and prudent person to believe
WoN he is entitled to bail and to provisional liberty while the extradition that the extradition request was prima facie meritorious.
proceedings are pending – NO The Court stresses that the prima facie existence of probable cause for
hearing the petition and, a priori, for issuing an arrest warrant was
RULING: Petition granted. Assailed RTC Order declared Null and Void. The already evident from the Petition itself and its supporting documents.
bail bond posted by Jimenez is CANCELLED. Hence, after having already determined therefrom that a prima facie
finding did exist, Judge Purganan gravely abused his discretion when
RATIO: (Substantive issues nalang ha ang haba nito) he set the matter for hearing upon motion of Jimenez.
Postulates of extradition: Moreover, the law specifies that the court sets a hearing upon receipt of the
Extradition is a major instrument for the suppression of crime
answer or upon failure of the accused to answer after receiving the
The requesting state will accord due process to the accused
The proceedings are sui generis summons.
Compliance shall be in good faith As argued by Petitioner DOJ, sending to persons sought to be extradited
There is an underlying risk of flight a notice of the request for their arrest and setting it for hearing at
Extradition hearings would not even begin, if only the accused were some future date would give them ample opportunity to prepare and
willing to submit to trial in the requesting country. execute an escape. Neither the Treaty nor the Law could have
Prior acts of Jimenez—leaving the requesting state right before the intended that consequence, for the very purpose of both would have
conclusion of his indictment proceedings there and remaining in the been defeated by the escape of the accused from the requested
requested state despite learning that the requesting state is seeking state.
his return and that the crimes he is charged with are bailable— Even Section 2 of Article III of our Constitution, which is invoked by
eloquently speak of his aversion to the processes in the requesting Jimenez, does not require a notice or a hearing before the issuance
state, as well as his predisposition to avoid them at all cost. of a warrant of arrest.
On notice and hearing: It is significant to note that Section 6 of PD 1069, Upon receipt of a petition for extradition and its supporting documents,
our Extradition Law, uses the word “immediate” to qualify the arrest the judge must study them and make, as soon as possible, a prima
of the accused. facie finding whether (a) they are sufficient in form and substance, (b)
The law could not have intended the word as a mere superfluity but, on they show compliance with the Extradition Treaty and Law, and (c)
the whole, as a means of imparting a sense of urgency and swiftness the person sought is extraditable.
in the determination of whether a warrant of arrest should be issued. If the presence of a prima facie case is determined, then the magistrate
By using the phrase “if it appears,” the law further conveys that accuracy must immediately issue a warrant for the arrest of the extraditee, who
is not as important as speed at such early stage. The trial court is not is at the same time summoned to answer the petition and to appear
expected to make an exhaustive determination to ferret out the true at scheduled summary hearings.
and actual situation, immediately upon the filing of the petition. Prior to the issuance of the warrant, the judge must not inform or notify the
From the knowledge and the material then available to it, the court is potential extraditee of the pendency of the petition, lest the latter be
expected merely to get a good first impression—a prima facie finding given the opportunity to escape and frustrate the proceedings. In our
—sufficient to make a speedy initial determination as regards the opinion, the foregoing procedure will “best serve the ends of justice” in
arrest and detention of the accused. extradition cases.
It is evident that Judge Purganan could have already gotten an impression On entitlement to Bail: Article III Section 13 was cited
Jimenez maintains that this constitutional provision secures the right to
bail of all persons, including those sought to be extradited.
He also alleges that the relevance to the present case of Section 4 Rule 114
of the Rules of Court which, insofar as practicable and consistent But because he left the jurisdiction of the requesting state before
with the summary nature of extradition proceedings, shall also apply those proceedings could be completed, it was hindered from
according to Section 9 of PD 1069. continuing with the
On the other hand, the DOJ claims that there is no provision in the
Philippine Constitution granting the right to bail to a person who is
the subject of an extradition request and arrest warrant.
The Court agrees with DOJ. As sugggested by the use of the word
“conviction,” the constitutional provision on bail quoted above, as
well as Section 4 of Rule 114 of the Rules of Court, applies only
when a person has been arrested and detained for violation of
Philippine criminal laws. It does not apply to extradition proceedings,
because extradition courts do not render judgments of conviction or
acquittal.
Moreover, the constitutional right to bail “flows from the presumption of
innocence in favor of every accused who should not be subjected to
the loss of freedom as thereafter he would be entitled to acquittal,
unless his guilt be proved beyond reasonable doubt.”
The provision in the Constitution stating that the “right to bail shall not be
impaired even when the privilege of the writ of habeas corpus is
suspended” does not detract from the rule that the constitutional right
to bail is available only in criminal proceedings.
The second sentence merely emphasizes the right to bail in criminal
proceedings for the aforementioned offenses. It cannot be taken to
mean that the right is available even in extradition proceedings that
are not criminal in nature.
That the offenses for which Jimenez is sought to be extradited are
bailable in the United States is not an argument to grant him one in
the present case.
To stress, extradition proceedings are separate and distinct from the trial
for the offenses for which he is charged. He should apply for bail
before the courts trying the criminal cases against him, not before
the extradition court.
On due process: Jimenez’ detention prior to the conclusion of the extradition
proceedings does not amount to a violation of his right to due process.
Where the circumstances—such as those present in an extradition case
—call for it, a subsequent opportunity to be heard is enough. The
Court finds no arbitrariness, either, in the immediate deprivation of
his liberty prior to his being heard.
It is also worth noting that before the US government requested the
extradition of respondent, proceedings had already been conducted
in that country.
due processes prescribed under its laws. His invocation of due process
now has thus become hollow.
Moreover, we cannot allow our country to be a haven for fugitives, cowards
and weaklings who, instead of facing the consequences of their actions,
choose to run and hide. Hence, it would not be good policy to increase
the risk of violating our treaty obligations if, through overprotection or
excessively liberal treatment, persons sought to be extradited are able to
evade arrest or escape from our custody.
In the absence of any provision—in the Constitution, the law or the treaty—
expressly guaranteeing the right to bail in extradition proceedings,
adopting the practice of not granting them bail, as a general rule, would
be a step towards deterring fugitives from coming to the Philippines to
hide from or evade their prosecutors.
The denial of bail as a matter of course in extradition cases falls into place
with and gives life to Article 14 of the Treaty, since this practice would
encourage the accused to voluntarily surrender to the requesting state to
cut short their detention here.
Likewise, their detention pending the resolution of extradition proceedings would
fall into place with the emphasis of the Extradition Law on the summary
nature of extradition cases and the need for their speedy disposition.
The rule is that bail is not a matter of right in extradition cases. However, the
judiciary has the constitutional duty to curb grave abuse of discretion and
tyranny, as well as the power to promulgate rules to protect and enforce
constitutional rights.
The right to due process is broad enough to induce the grant of basic
fairness to extraditees. Indeed, the right to due process extends to the
“life, liberty or property” of every person. It is “dynamic and resilient,
adaptable to every situation calling for its application.”
To best serve the ends of justice, after a potential extraditee has been
arrested or placed under the custody of the law, bail may be applied for
and granted as an exception, only upon a clear and convincing showing
That, once granted bail, the applicant will not be a flight risk or a danger to
the community
That there exist special, humanitarian and compelling circumstances
including, as a matter of reciprocity, those cited by the highest court in
the requesting state when it grants provisional liberty in extradition
cases therein.
Since this exception has no express or specific statutory basis, and since it is
derived essentially from general principles of justice and fairness, the
applicant bears the burden of proving the above two-tiered requirement
with clarity, precision and emphatic forcefulness.
Jimenez further claims that he is not a flight risk. To support this claim, he Bail may be set at huge amounts or passports cancelled and hold-
stresses that he learned of the extradition request in June 1999; yet, departure orders issued or border patrols heightened, in order
he has not fled the country. that the extraditee may not flee from our jurisdiction. In this
True, he has not actually fled during the preliminary stages of the request regard, while I agree that it is the
for his extradition. Yet, this fact cannot be taken to mean that he will
not flee as the process moves forward to its conclusion, as he hears
the footsteps of the requesting government inching closer and closer.
That he has not yet fled from the Philippines cannot be taken to mean
that he will stand his ground and still be within reach of our
government if and when it matters; that is, upon the resolution of the
Petition for Extradition.
In any event, it is settled that bail may be applied for and granted by the
trial court at anytime after the applicant has been taken into custody
and prior to judgment, even after bail has been previously denied.
In the present case, the extradition court may continue hearing evidence
on the application for bail, which may be granted in accordance with
the guidelines in this Decision.
Bellosillo, Separate: The Government maintains that an extradition
court has no power to authorize bail in the absence of any law
conferring such power; and that the 1987 Constitution, as well as the
Rules of Court, as amended, applies only to persons arrested and
detained for violation of Philippine Laws, but not to extradition
proceedings in which courts do not render judgments of conviction or
acquittal.
Both the extradition treaty between the United States and the Philippines,
and the Philippine Extradition Law (PD 1069) contain no provision
expressly withholding from the courts the power to grant bail.
Indeed, the treaty fails to even remotely suggest such judicial limitation
insisted upon by the Government. Truly, there is neither logic nor
persuasion to the suggestion that bail should only be allowed in criminal
cases, or that class of cases where courts must “render judgments of
conviction or acquittal.
Bail as a remedy is available where there is deprivation of liberty prior or
during trial. US v. Co Siaco: Involves a deportation proceeding. The
Court allowed the potential deportee to post bail although a
deportation proceeding is not criminal in nature and there was then
no law providing for bail in deportation cases.
The Court cannot curtail a citizen’s right to freedom on speculations and
fears where there exist reasonable mechanisms appropriate to address
them. To my mind, the risk of flight does not ipso facto call for denying
his right to bail..
extraditee’s burden to prove the least likelihood of flight, the extradition
court is also entitled to presume that the executive branch has done all it
can to forestall his sudden disappearance.
To unduly sacrifice the civil liberties of an individual by reason of an
unfounded fear of being unable to fulfill treaty obligations, would be to
render impotent the ideals of the dignity of the human person, thereby
destroying something of what is noble in our way of life.
Puno, Separate: While this Court is obliged to accord due respect to the
state’s interests to comply with its treaty obligations, it cannot also shirk
from its duty to protect the fundamental rights of its citizens.
A potential extradite can hinge his right to bail in our Constitution. The mere
silence of our extradition treaty with the US and our extradition law does
not negate the right to bail of a potential extraditee.
But while an extraditee may apply for bail, its grant depends on presentation
of clear and convincing evidence that the extraditee will not frustrate the
ends of justice by fleeing from our jurisdiction.
Like the privilege of the writ of habeas corpus, the right to bail gives flesh to
the guarantee to liberty, without which, the right to liberty can prove
meaningless, and due process will only be an empty slogan. However,
unlike the privilege of habeas corpus which is principally a remedy
against illegal restraint on liberty, the right to bail is available even when
the reason for the detention is lawful.
The purpose of bail is to relieve a person the rigors of prolonged
imprisonment until the main case against him is resolved, and at the
same time, insure his attendance when required by the authorities. It is
the prospect of prolonged detention, not the detention itself, which
offends the constitutional right to due process.
The right of an extraditee to apply for bail should be treated in light of our other
treaty obligations, especially those concerning the promotion and protection
of human rights. (Cited Vienna Convention on the Law of Treaties,
UDHR, ICCPR)
There is no customary rule of international law prohibiting bail in extradition
cases. At present, there is no customary norm prohibiting bail in
extradition cases. On the contrary, most countries, including Canada,
Australia, the United Kingdom, South Africa and Pakistan, among others,
allow a potential extraditee to be released on bail. Members of the
European Union have recently ratified the European Convention on
Extradition, which also provides a procedure for bail.
Even the US grants bail to an extradite, albeit in exceptional circumstances.
Wright v. Henckel recognized the authority of the circuit courts to
receive
application for and grant bail in certain exceptional case restricted in liberty of movement. Lawful arrest and
While an extraditee may apply for bail, its grant is discretionary confinement are germane to the purposes of the
depending on whether it will frustrate the ends of justice.
After the warrant of arrest is issued, the burden of proof on the right to be
admitted to bail shifts on the potential extraditee. In criminal cases,
the presumption lies in favor of granting bail.
This is so because of the constitutional presumption of innocence, which
is not overturned by the finding of probable cause upon which the
warrant of arrest against the accused was issued. However, the
presumption of innocence, from which the ordinary presumption in
favor of granting bail emanates, is inoperative in extradition cases.
The issuance of the warrant of arrest in extradition cases is not
based on the finding that the accused is probably guilty of the
offense for which he was charged in the requesting State.
Vitug, Separate: The edict in the Bill of Rights granting to all persons,
without distinction, the fundamental right to bail, is clear—no statute
or treaty can abrogate or discard its language and its intent.
The Eigth Amendment of the U.S. Federal Constitution does not
expressly provide for the grant of bail—it recognizes merely by
implication the right to bail by simply disallowing excessive bail.
Ynares-Santiago, Dissenting: We must consider the implications of a
ruling that in criminal proceedings, the constitutional rights of the
accused must be protected, but in a case neither criminal nor civil,
one which we call “sui generis,” basic freedoms become irrelevant
and non-available.
The majority opinion gives five (5) postulates of extradition. With all due
respect, I fail to see how compliance with these postulates should
result in a disregard for constitutional liberties.
Carpio, Concurring: Following the emerging trend in the United States, and
guided by our own experience in combating transnational crimes
including international terrorism, the Court should rule that our
extradition courts may, after the arrest of the extraditee, grant the
extraditee bail if he establishes that he does not pose a flight risk or a
danger to the community, and there is no other special circumstance
that would warrant denial of bail.
In case maam asks: inaavail ni kuya Jimenez yung defense ng
disenfranchisement kasi he was voted member of the HOR. But the
Court cited People v. Jalosjos: “election to the position of Congressman
is not a reasonable classification in criminal law enforcement. The
functions and duties of the office are not substantial distinctions which lift
him from the class of prisoners interrupted in their freedom and
law and apply to all those belonging to the same class”
SERAPIO v. SANDIGANBAYAN (ARIELLE) Subsequently, the Ombudsman filed with the Sandiganbayan several
January 28, 2003 | Callejo, Sr., J. | Right to Bail and Recognizance Informations against Erap, including Serapio, with plunder. No bail
was recommended for their provisional release.
PETITIONER: Atty. Edward Serapio
RESPONDENTS: Sandiganbayan

SUMMARY: Serapio, a Board of Trustee and counsel of the Erap Muslim


Youth Foundation, received P200 million from Chavit Singson (donation
daw). Subsequently, Chavit publicly accused Erap and his cohorts of
engaging in illegal activities, specifically in operating jueteng. Serapio
was included in the charge of plunder. He filed a petition for bail, but the
Sandiganbayan said that arraignment should come first before the
petition for bail be acted on, it deferred Serapio’s petition for bail.

The SC ruled the arraignment of an accused is not a prerequisite to the


conduct of hearings on his petition for bail. A person is allowed to petition
for bail as soon as he is deprived of his liberty by virtue of his arrest or
voluntary surrender. An accused need not wait for his arraignment before
filing a petition for bail. To condition that the grant of bail to an accused
on his arraignment would be to place him on a position where he has to
choose between 1) filing a motion to quash and thus delay his release on
bail because until his motion to quash can be resolved, his arraignment
cannot be held, and 2) foregoing the filing of a motion to quash so that he
can be arraigned at once and be released on bail.

DOCTRINE: It is not necessary that an accused be first arraigned before the


conduct of hearings on his application for bail. For when bail is a matter of
right, an accused may apply for and be granted bail even prior to
arraignment.

FACTS:
Petitioner Edward Serapio was a member of the Board of Trustees and
the Legal Counsel of the Erap Mulslim Youth Foundation, a non-
stock, nonprofit foundation established for the purpose of providing
educational opportunities for the poor but deserving Muslim youth
and students.
Serapio, as trustee of the Foundation, received on its behalf P200 million
from Chavit Singson through the latter’s assistant.
Seraptio received the donation and turned over said amount to the
Foundation’s treasurer who deposited it to the Equitable PCI Bank.
Singson publicly accused Erap and his cohorts of engaging in several
illegal activities, including its operation on the illegal numbers game
known as jueteng. This triggered the filing with the Ombudsman of
several criminal complaints against Erap and Serapio, among others.
Sandiganbayan found probable cause to justify the issuance of the We do not agree with petitioner. Under Section 6, Rule 110 of RRCP,
warrants of arrest. Serapio voluntarily surrendered to PNP Chief provides that acts or omission complained of must be alleged in such
Leandro Mendoza.
form as
Serapio’s co-accused, Jinggoy Estrada, filed a Very Urgent Omnibus
Motion alleging that he was entitled to bail as a matter of right.
During hearing on Serapio’s Urgent Petition for Bail, the prosecution
moved for the resetting of the arraignment of the accused earlier
than scheduled. This was denied by the Sandiganbayan and issued
an order declaring that the petition for bail can and should be heard
before petitioner’s arraignment and even before other co-accused
filed their respective petitions for bail.
The Ombudsman filed an urgent motion for early arraignment of Erap,
Jinggoy and Serapio and a motion for joint bail hearings of the three.
Serapio questioned the propriety of including Erap and Jinggoy in the
hearing of his petition for bail.
Sandiganbayan canceled the bail hearing due to pending incidents yet to
be resolved. The supposed subsequent bail hearing after the first
one was cancelled did not proceed as well because the amended
Information does not allege a combination or series of overt acts
indicative of an overall unlawful scheme or conspiracy.
Serapio prayed that the People be declared to have waived their right to
present evidence in opposition to his petition for bail and premised
on the failure of the People to adduce strong evidence of his guilt of
plunder, that he be granted provisional liberty on bail after due
proceedings.
Serapio filed with the SC a petition for certiorari, alleging that the
Sandibagan bayan acted without or in excess of jurisdiction or with
grave abuse of discretion in issuing its Resolution denying his motion
to quash, despite the fact that material inculpatory allegations of the
amended information against him do not constitute the crime of
plunder and that he is charged, under the amended information, for
more than one offense.
Jinggoy also filed a petition for the nullification of a resolution of the
Sandiganbayan denying his motion to fix bail.
Serapio avers that there is no allegation in the amended information of a
combination or series of overt or criminal acts constituting plunder.
Serapio avers that his single act of toleration or protection of illegal gambling
does not constitute the requisite “combination or series of acts” for
plunder.

ISSUE/s:
WoN Serapio is entitled to post bail – NO
WoN arraignment should be done first before conducting the hearing for bail
– NO

RULING: Petition is PARTIALLY GRANTED. The joint hearing for bail is SET
ASIDE; the arraignment of Serapio also SET ASIDE;

RATIO:
is sufficient to enable a person of common understanding to know A person is allowed to petition for bail as soon as he is deprived
what offense is intended to be charged. of his liberty by virtue of his arrest or voluntary surrender.
The information must allege clearly the elements of the crime charged. An accused need
The purpose of this is to inform the accused of the nature of the
accusation against him to prepare for his defense. The use of
derivatives or synonyms of basic facts constituting the offense
chaged is sufficient.
In this case, the amended information specifically alleges that all the
accused, including Serapio, connived and conspired with Erap to
commit plunder “through any or a combination or a series of overt or
criminal acts or similar schemes or means.”
Serapio and his co-accused are charged with receiving or collecting on
several instances money in the amount of 545 million. It is not
necessary to allege in the amended information a pattern of over or
criminal acts because Sec 3 of RA 7080 states that the same is
evidentiary and the general rule is that matters of evidence need not
be alleged in the information.
It is clear on the face of the amended information that Serapio and his
co-accused are charged only with one crime of plunder. It bears
stressing that the predicate acts merely constitute acts of plunder
and are not crime separate and independent of the crime plunder.
Serapio contends that he cannot be charged with plunder because the 200
million he received is not ill-gotten wealth and there is no evidence
linking him to the collection and receipt of jueteng money.
GR NO 148468 CASE: related na to bail~
Serapio contends that the Sandiganbayan erred when it deferred the
hearing of his petition for bail. He insists that the Rules on Criminal
Procedure does not require that he be arraigned first prior to the
conduct bail hearing since the latter can stand alone and must be
heard immediately.
The People insist that arraignment is necessary before bail hearings may
be commenced, because it is only upon arraignment that the issues
are joined. The People stress that it is only when an accused pleads
not guilty may he file a petition for bail.
The issue as to whether an arraignment is necessary before the conduct
of bail hearings in Serapio’s case has become moot, because the
Sandiganbayan already entered a plea of not guilty on his behalf, but
the Court will still discuss it to educate the bench and bar.
THE CONTENTION OF SERAPIO IS WELL-TAKEN. The arraignment
of an accused is not a prerequisite to the conduct of hearings
on his petition for bail.
not wait for his arraignment before filing a petition for bail.
In Lavides v. CA, the Court ruled that in cases where it is authorized, bail
should be granted before arraignment, otherwise, the accused may be
precluded from filing a motion to quash.
However, the foregoing pronouncement should not be taken to mean that the
hearing on a petition for bail should at all times precede arraignment,
because the rule is that a person deprived of his liberty by virtue of his
arrest or voluntary surrender may apply for bail as soon as he is deprived
of his liberty, even before a complaint or information is filed against him.
The Court’s pronouncement in Lavides should be understood in light the fact
that the accused in said case filed a petition for bail as well as a motion
to quash the information filed against him.
Hence, to condition that the grant of bail to an accused on his arraignment
would be to place him on a position where he has to choose between 1)
filing a motion to quash and thus delay his release on bail because until
his motion to quash can be resolved, his arraignment cannot be held,
and 2) foregoing the filing of a motion to quash so that he can be
arraigned at once and be released on bail.
It is therefore not necessary that an accused be first arraigned before
the conduct of hearings on his application for bail. For when bail is
a matter of right, an accused may apply for and be granted bail
even prior to arraignment.
The Lavides ruling also implies that an application for bail in a case involving
an offense punishable by reclusion perpetua to death may also be heard
even before an accused is arraigned.
Further, if the court finds that the accused is entitled to bail because the
evidence against him is not strong, he may be granted provisional liberty
even prior to arraignment; for in such situation, bail would be “authorized”
under the circumstances.
Sandiganbayan committed grave abuse of its discretion amounting to excess
of jurisdiction in ordering the arraignment of petitioner before proceeding
with the hearing of his petition for bail.
On the issue of w/n Serapio may file a motion to quash during
pendency of his petition for bail: The two are not inconsistent. Bail is
the security given for the release of a person in the custody of the law,
furnished by him or a bondsman, to guarantee his appearance before
any court as required under the Rules of Court.
Purpose of bail is to obtain the provisional liberty of a person charged
with an offense, until his conviction while at the same time securing
his appearance at the trial.
On the other hand, a motion to quash is the mode by which an accused If it is satisfactorily shown that the evidence of guilt is strong, it is the
assails the validity of a criminal complaint filed against him. As a court’s duty to deny the application for bail. However, when the
general rule, an accused may file a motion to quash before evidence of guilt is not strong, bail becomes a matter of right.
arraignment.
These two reliefs have objectives which are not necessarily antithetical to
each other. Certainly, the right of an accused to seek provisional
liberty when charged with an offense not punishable by death,
reclusion perpetua or life imprisonment, or when charged with an
offense punishable by such penalties but without strong evidence of
guilt, does not preclude his right to assail the validity of the
information charging him with such offense.
However, if a motion to quash a criminal complaint on the ground that the
same does not charge any offense is granted and the case is
dismissed, the petition for bail may become moot and academic.
On w/ n it is mandatory that the hearings on the petitions for bail of
Serapio and Jinggoy be heard jointly: The matter of whether or
not to conduct a joint hearing of two or more petitions for bail filed by
two different accused is addressed to the sound discretion of the trial
court, as there is no provision in the RRCP or the Rules of Procedure
of the Sandiganbayan governing the matter.
In Ocampo v. Bernabe, the Court held that in a petition for bail hearing,
the court is to conduct only a summary hearing. The court does not
try the merits or enter into any inquiry as to the weight that ought to
be given to the evidence against the accused.
It may confine itself to receiving such evidence as has reference to
substantial matters, avoiding unnecessary thoroughness in the
examination and cross-examination of witnesses.
A joint hearing of two separate petitions for bail by two accused will of
course avoid duplication of time and effort.
A person charged with a capital offense is not absolutely denied the
opportunity to obtain provisional liberty on bail pending
judgment of his case. However, as to such person, bail is not a
matter of right but is discretionary upon the court.
Under Sec 8 of Rule 114, there must be a showing that the evidence of
guilt against a person charged with a capital offense is not strong for
the court to grant him bail. Thus, upon application for bail by a
person charged with a capital offense, a hearing must be conducted,
where the prosecution must be accorded an opportunity to discharge
its burden of proving that the evidence of guilt against the accused is
strong.
In this case, Serapio is not entitled to bail as a matter of right. The
prosecution did not waive, expressly or impliedly, its right to
adduce evidence in opposition to the petition for bail of Serapio.
It must be noted that Sandiganbayan already scheduled the hearing for bail
but the same were reset due to pending incidents. It was re-scheduled
but it did not push through because of this petition. [They filed a lot of
petitions and urgent omnibus motions kasi, that’s why may delay.]
RODRIGUEZ v. JUDGE (IYA) RULING: Imelda Rodriguez is to be placed back in bail
February 27, 2006 | Quisimbing, J. | Right to Bail

PETITIONER: Eduardo Rodriguez and Imelda Rodriguez


RESPONDENTS: Hon. Judge of the RTC, Government of the United
States of America, represented by the Philippine DOJ and Director of
NBI

SUMMARY: Rodriguez’ have a pending extradition case before the US


Government. They were initially granted bail. This was however,
cancelled in compliance with an SC directive.

Petitioners question the cancellation for its was done without prior notice.
The SC ruled that a subsequent cancellation cannot be made without
first giving opportunity to the petitioner to be heard. Since petitioner is no
longer a flight risk there is no reason why they should be detained.

DOCTRINE: The SC believes that the benefits of continued temporary


liberty on bail should not be revoked and their grant of bail should not be
cancelled, without the copetitioner being given notice and without her
being heard why her temporary liberty should not be discontinued.

FACTS:
The case stemmed from the petition for extradition filed on March 12,
2001 by the Government of the United States of America (US
government) through the Department of Justice (DOJ) against the
petitioners.
After their arrest, petitioners applied for bail which the trial court granted
on September 25, 2001.
The bail was set for one million pesos for each. Petitioners then posted
cash bonds.
The US government moved for reconsideration of the grant of bail, but
the motion was denied by the trial court. Unsatisfied, the US
government filed a petition for certiorari with this Court.
Thereafter, SC directed the trial court to resolve the matter of bail which,
according to its November 28, 2008 Order, shall be subject to
whatever ruling that this Court may have in the similar case of Mark
Jimenez entitled Government of the United States of America v.
Purganan.
In compliance with the SC directive, the trial court without prior notice
and hearing cancelled the cash bond of the petitioners and issued a
warrant of arrest.
Hence, this case.

ISSUE/s:
WON petitioner Imelda Rodriguez may be granted bail - YES
RATIO: But this is for cases pending the issuance of a warrant of arrest, not in a
Petitioners assert that their bail cannot be cancelled without due process cancellation of a bail that had been issued after determination that
of law. By way of analogy, they point to Rule 114, Section 21 of the the
Rules of Court where the surety or bonding company is required to
be notified and allowed to show cause why the bail bond should not
be cancelled.
Respondents, for their part, argue that prior notice and hearing are not
required to cancel petitioners’ bail, and the issuance of a warrant of
arrest ex parte against an extraditee is not a violation of the due
process clause. They further maintain that prior notice and hearing
would defeat the purpose of the arrest warrant since it could give
warning that respondents would be arrested and even encourage
them to flee.
More significantly, petitioners claim that their bail should not have been
cancelled since their situation falls within the exception to the general
rule of no-bail.
They allege that their continuous offer for voluntary extradition is a
special circumstance that should be considered in determining that
their temporary liberty while on bail be allowed to continue.
They cite that petitioner Eduardo is in fact already in the United States
attending the trial.
They also have not taken flight as fugitives. Besides, according to
petitioners, the State is more than assured they would not flee
because their passports were already confiscated and there is an
existing hold-departure order against them.
Moreover, petitioners assert, they are not a danger to the community.
The issue of prior notice and hearing in extradition cases is not new. In
Secretary of Justice v. Lantion, by a vote of nine to six, we initially
ruled that notice and hearing should be afforded the extraditee even
when a possible extradition is still being evaluated. The Court,
deliberating on a motion for reconsideration also by a vote of nine to
six, qualified and declared that prospective extraditees are entitled to
notice and hearing only when the case is filed in court and not during
the process of evaluation.
In the later case of Purganan, eight justices concurred that a possible
extraditee is not entitled to notice and hearing before the issuance of
a warrant of arrest while six others dissented.
The SC is now confronted with the question of whether a prospective
extraditee is entitled to notice and hearing before the cancellation of
his or her bail.
The issue has become moot and academic insofar as petitioner Eduardo
Rodriguez is concerned. He is now in the USA facing the charges
against him. But copetitioner Imelda Gener Rodriguez is here and
stands on a different footing. SC agree that her bail should be
restored.
In Purganan, we said that a prospective extraditee is not entitled to
notice and hearing before the issuance of a warrant of arrest,
because notifying him before his arrest only tips him of his pending
arrest.
extraditee is a no-flight risk. The policy is that a prospective extraditee is
arrested and detained to avoid his flight from justice.
If after his arrest and if the trial court finds that he is no flight risk, it
grants him bail. The grant of the bail, presupposes that the
copetitioner has already presented evidence to prove her right to be
on bail, that she is no flight risk, and the trial court had already
exercised its sound discretion and had already determined that
under the Constitution and laws in force, copetitioner is entitled to
provisional release.
The SC believes that the benefits of continued temporary liberty on bail
should not be revoked and their grant of bail should not be cancelled,
without the copetitioner being given notice and without her being
heard why her temporary liberty should not be discontinued.
The SC emphasizes that bail may be granted to a possible extraditee
only upon a clear and convincing showing:
that he will not be a flight risk or a danger to the community, and
that there exist special, humanitarian and compelling circumstances.
LEVISTE v. CA (ELIEL)
March 17, 2010 | Corona, J. | Right to Bail RULING: Dismissed.

PETITIONER: Jose Antonio Leviste RATIO:


RESPONDENTS: CA and People of the Philippines It cannot be said that the CA issued the assailed resolution without or in
excess of its jurisdiction. One, pending appeal of a conviction by the
SUMMARY: Leviste was convicted in the RTC of the crime of murder of RTC of an offense not punishable by death, reclusion pepetua, or life
Rafael de las Alas. He appealed his conviction to the CA and filed for a imprisonment, admission to bail is expressly declared to be
motion to bail, because of advanced age and health. However, the CA discretionary
denied such because of jurisprudence citing that it is not a strong reason Two, the discretion to allow or disallow bail pending appeal in a case such as
or sufficient reason for bail. Leviste contends that he should have been this where the decision of the trial court convincing the accused changed
granted bail because he does not suffer from any of the circumstances the nature of the offense from non-bailable to bailable is exclusively
under the 3rd paragraph, Sec 5 of Rule 114 (See ratio 3). Hence this lodged by the rules with the appellate court. Thus, the CA had jurisdiction
petition. to hear and resolve Leviste’s urgent application for admission to bail
pending appeal
The SC held that absence of any of the circumstances of bail does not The 3rd paragraph of Sec 5, Rule 114 applies to two scenarios where the
automatically grant bail to an accused. It is still subject to the discretion of penalty imposed on Leviste applying for bail is imprisonment
the judge as to whether or not to allow bail. The history of the right to bail exceeding 6 years. The first scenario deals with the
establishes the stringent requirement of granting bail, otherwise, it would circumstances enumerated in the said paragraph (namely,
be deterrent to the criminal laws. Furthermore, after conviction, recidivism, quasi-recidivism, habitual delinquency or commission of
presumption of innocence terminates, thus, right to bail ceases. In the the crime aggravated by the circumstance of reiteration; previous
present case, Leviste was already convicted in the trial court, therefore, a escape from legal confinement, evasion of sentence or violation of
stringent approach of judicial discretion should be applied. the conditions of his bail without a valid justification; commission of
the offense while under probation, parole or conditional pardon;
DOCTRINE: Permitting bail too freely in spite of conviction invites circumstnaces indicating the probability of flight if released on bail;
frivolous and time-wasting appeals which will make a mockery of our undue risk of committing another crime during the pendency of the
appeal; or other similar circumstances) not present. The second
criminal justice system and court processes
scenario contemplates the existence of at least one of the said
circumtances.
FACTS: In the first situation, bail is a matter of sound judicial discretion. This means
Charged with the murder of Rafael de las Alas, Leviste was convicted by rd
that, if none of the circumstances mentieodn in the 3 paragraph of Sec
the RTC of Makati City for the lesser crime of homicide and 5, Rule 114 is present, the appellate court has the discretion to grant or
sentenced to suffer an indeterminate penalty of 6 years and 1 day of deny bail
prision mayor as minimum to 12 years and 1 day of reclusion An application for bail pending appeal may be denied even if the
temporal as maximum bail negating circumstances in the 3rd paragraph of Sec 5, Rule
He appled his conviction to the CA. Pending appeal, he filed an urgent 114 are absent. In other words, the appellate courst denial of
application for admission to bail, citing his advanced age and health bail pending appeal where none of the said circumstnaces
condition, and claiming the absence of any risk or possibility of flight exists does not, by and of itself, constitute abuse of discretion
on his part On the other hand, in the second situation, the appellate court
The CA denied Leviste’s application for bail. It invoked the bedrock exercises a more strigent discretion, that is, to carefully
principle in the matter of bail pending appeal, that the discretion to ascertain whether any of the enumerate circumstances in fact
extend bail during the course of appeal should be exercised with exists. If it so determines, it has no other option except to deny
grave caution and only for strong reasons. or revoke bail pending appeal. Conversly, if the appellate court
Citing well-established jurisprudence, it ruled that bail is not a sick pass grants bail pedning appeal, grave abuse of discretion will
for an ailing or aged detainee or a prisoner needing medical care thereby be committed.
outside the prison facility. Given these two distinct scenarios, therefore, any application for bail
pending appeal should be viewed from the perspective of two stages:
ISSUE/s: (1) the determination of discretion stage, where the appellate court
WoN Leviste should be granted bail –NO must determine whether any of the circumstances in the 3 rd
paragraph, sec 5,Rule 114 is present; this will establish whether or
not the appellate court will exercise sound jurisdiction or stringent
discretion in resolving the application for bail pending appeal
(2) The exercise of discretion stage where, assuming the appellants case exceeding 6 years, bail sought to be granted if none of the
falls within the first scenario allowing the exercise of sound discretion, listed bail-negating circumstances exists
the appellate court may consider all relevant circumstances, other than In our jurisdiction, the trend towards a strict attitude towards the
rd
those mentioned in the 3 paragraph of Sec 5, Rule 114, including the allowance of bail pending appeal is anchored on the
demands of equity and justice; on the basis thereof, it may either allow principle that
or disallow bail
On the other hand, if the appellant case falls within the second scenario,
the appellate courts stringent discretion requires that the exercise
thereof be primarily focused on the determination of the proof of the
presence of any of the circusmtances that are prejudicial to the
allowance of bail. This is so because the existence of any of those
circumstances is by itself sufficient to deny or revoke bail
Nonetheless, a finding that none of the circumstances is present
will not automatically result in the grant of bail. Such finding will
simply authorize the court to use the less stringent sound
discretion approach
The judicial discretion granted to the proper court to rule on applications
for bail pending appeal must necessarily involve the exercise of
judgement on the part of the court. The court must be allowed
reasonable latitude to express its own view of the case, its
appreciation of the facts and its understanding of the applicable law
on the matter
In other words, a threshold requirement for the grant of bail is a
showing that the appeal is not pro form and merely intended for
delay but presents a fairly debatable issue
Even more significantly, this comports with the very storng
presumption on appeal that the lower courts exercise of
discretionary power was sound, specially snce the rules on
criminal procedure require that no judgment shall be reversed
or modified by the CA except for substantial error
Moreover, to limit the bail-negating circumstances to the 5
situations mentioned in 3rd paragraph of Sec 5, Rule 114 is
wrong. By restricting the bail-negative circumstances to those
expressly mentioned, Leviste applies the expression unius est
exclusion alterius rule in statutory construction
However, the very language of the 3rd paragraph of Sec 5, Rule 114
contradicts the idea that the enumeration of the 5 sitautions therein
was meant to be exclusive
The provision categorically refers to the following or other similar
circumstances. Hence, under the rules, similarly relevant
situations other than those listed in the 3rd paragraph of Sec 5,
Rule 114 may be considered in the allowance, denial or
revocation of bail pending appeal
Finally, laws and rules should not be interpreted in such a way that
leads to unreasonable or senseless consequences. An absurd
situation will result from adopting Leviste’s interpretation that,
where the penalty imposed by the trial court is imprisonment
judicial discretion particularly with respect to extending bail should
be exercised not with laxity but with caution and only for strong
reasons
As a matter of fact, endorsing the reasoning quoted above and relying
thereon, the Court declared in Yap v. CA, that denial of bail pending
appeal is a matter of wise discretion
Sec 13, Article 2 of the Constitution:
SEC 13. All persons, except those charged with offenses
punishable by reclusion perpetua when evidence of guilt is
strong, shall, before conviction, be bailable by sufficient
sureties, or be released on recognizance as may be provided
by law
After conviction by the trial court, the presumption of innocence terminates and,
accordingly, the constitutiona right to bail ends. From then on, the grant of
bail is subject to judicial discretion. At the risk of being repetitious, such
discretion must be exercised with grave caution and only for strong reasons
Considering that the Leviste was in fact convicted by the trial court,
allowance of bail pending appeal should be guided by a stringent-
standards approach. This judicial disposition finds strong support
in the history and evolution of the rules on bail and the language of
Sec 5, Rule 114 of the Rules of Court. It is likewise consistent with
the trial courts initial determination that the accused should be in
prison.
Furthermore, letting Leviste out on bail despite his conviction may
destroy the deterrent effect of our criminal laws. This is especially
germane to bail pending appeal because long delays often separate
sentencing in the trial court and appellate review.
In addition, at the post-conviction stage, Leviste faces a certain prison
sentence and thus may be more likely to flee regardless of bail
bonds or other release conditions. Finally, permitting bail too freely
in spite of conviction invites frivolous and time-wasting appeals
which will make a mockery of our criminal justice system and court
processes.
GOVERNMENT OF HONG KONG v. OLALIA, JR. (JP) Munoz filed with the CA a petition questioning the validity of the Order of
April 19, 2007 | Sandoval-Gutierrez, J. | Bail Arrest. To which the CA declared as void.
The DOJ then field with the SC a petition for review on certiorari praying
that the decision of the CA be reversed. The SC granted the petition
PETITIONER: Government of Hong Kong Special Administrative Region
and upheld the validity of the Order of Arrest.
RESPONDENTS: Hon. Felixbierto Olalia, Jr. and Juan Antonio Munoz The Government of HK Special Administrative Region filed with the
Note: petitioners are represented by the PH DOJ during the trial RTC a petition for the extradition of Munoz. Munoz filed a
petition for bail, on the same case, which was opposed by Gov
SUMMARY: Muñoz is facing criminal charges with the Government of of HK.
Hong Kong. His arrest is requested by HK through the PH DOJ. When a RTC judge denied the petition for bail holding that there is no PH
petition for extradition was filed before the RTC, Munoz applied for bail. law granting bail in extradition cases and that Munoz is
Munoz was granted bail by the RTC. HK filed a motion to dismiss the considered a high “flight risk.”
grant but was denied. HK then raised the issue to the SC. Muñoz filed an MR, (the previous judge inhibited himself therefore was
assigned to a different judge) which was granted allowing him to post
The SC overturned its previous decision not allowing bail. Initially, the SC
bail.
explained that some jurisprudence dictate that bail applies solely to criminal
Gov of HK filed an urgent motion to vacate the order allowing to post bail
proceedings and since extradition proceedings are administrative and does
but was denied.
not render conviction or acquittal, such is not applicable. Due to international
Hence, this instant petition.
treaties and agreements however, the SC is bound to give importance to the
rights of the individual as extradition may entail a deprivation of liberty. Bail ISSUE/s:
was allowed in deportation proceedings which are of the same situation with WON the RTC committed grave abuse of discretion amounting to lack or
extradition proceedings. Likewise, Muñoz was already deprived of his liberty excess of jurisdiction in admitting private respondent to bail - NO
for two years. He is entitled to bail. The SC set a standard of “clear and
convincing evidence” for the courts to follow in granting bail for extradition RULING: The case is remanded to the trial court for further
cases. determination.
DOCTRINE: While extradition is not a criminal proceeding, it entails
deprivation of liberty and the means employed to attain the purpose of RATIO:
extradition is also “the machinery of criminal law”. Govt. of HK maintains that nothing in the Constitution or statutory law
providing that a potential extraditee has a right to bail, the right being
A potential extraditee may be subjected to arrest, to a prolonged restraint limited solely to criminal proceedings.
of liberty, and forced to transfer to the demanding state following the Muñoz claims that the right to bail is guaranteed under the Bill of Rights,
proceedings. "Temporary detention" may be a necessary step in the and it extends to prospective extraditees because extradition is a
process of extradition, but the length of time of the detention should be harsh process resulting in a prolonged deprivation of one’s liberty.
reasonable. Sec. 13. All persons, except those charged with offenses punishable by
reclusion perpetua when evidence of guilt is strong, shall, before
conviction, be bailable by sufficient sureties, or be released on
FACTS: recognizance as may be provided by law. Xxx
Juan Antonio Muñoz was charged before the Hong Kong Court with 3 As suggested by the use of the word "conviction," the constitutional
counts of the offense of “accepting an advantage was agent” in provision on bail quoted above, as well as Section 4, Rule 114 of the
violation of the Prevention of Bribery Ordinance of HK. Rules of Court, applies only when a person has been arrested and
He also faced 7 counts of the offense of conspiracy to defaud penalized detained for violation of Philippine criminal laws.
by the common law of HK. It does not apply to extradition proceedings because extradition courts do
Warrants of arrest were issued against him. The DOJ after receiving from not render judgments of conviction or acquittal.
the HK DOJ a request for provisional arrest of Munoz forwarded it to The constitutional provision on bail will not apply to a case like
the NBI. The NBI in turn filed with the RTC an application for the extradition, where the presumption of innocence is not at issue.
provisional arrest of Munoz. Previous jurisprudence (Gov of USA v Hon. Guillermo Purganan and
The RTC issued the Order and on the same day the NBI arrested and Mark Jimenez) held that the constitutional provision on bail does not
detained him. apply to extradition proceedings.
It must also be noted that the suspension of the privilege of the writ of
habeas
corpus based on the second sentence of the provision finds process of extradition, but the length of time of the detention
application "only to persons judicially charged for rebellion or should be reasonable.
offenses inherent in or directly connected with invasion" (Sec. 18,
Art. VIII, Constitution).
Hence, the second sentence in the constitutional provision on bail merely
emphasizes the right to bail in criminal proceedings for the
aforementioned offenses. It cannot be taken to mean that the right is
available even in extradition proceedings that are not criminal in
nature.
At first glance, such ruling applies to the instant case, but he SC however
cannot ignore the following trend of international law giving primacy
on the worth of the individual person and the sanctity of human
rights. It is also the duty of the SC to balance the rights of the
individual under the Constitution and the law on extradition.
As a signatory of the UN Declaration on Human Rights, we are bound to
adhere to it as it is now recognized as customarily biding upon the
members of the international community.
Due to the international treaties, a reexamination of the previous ruling of
this court not grating bail to extradition proceedings is in order.
The PH has allowed bail on administrative cases involving deportation
and quarantine. There would be no justification why this should not
be allowed din extradition cases.
Bail has been allowed in this jurisdiction to persons in detention during
the pendency of administrative proceedings, taking into cognizance
the obligation of the Philippines under international conventions to
uphold human rights.
If bail can be granted in deportation cases, the SC sees no justification
why it should not also be allowed in extradition cases.
Likewise, considering that the Universal Declaration of Human Rights
applies to deportation cases, there is no reason why it cannot be
invoked in extradition cases. After all, both are administrative
proceedings where the innocence or guilt of the person detained is
not in issue.
Section 2(a) of Presidential Decree (P.D.) No. 1069 (The Philippine
Extradition Law) defines "extradition" as "the removal of an accused
from the Philippines with the object of placing him at the disposal of
foreign authorities to enable the requesting state or government to
hold him in connection with any criminal investigation directed
against him or the execution of a penalty imposed on him under the
penal or criminal law of the requesting state or government."
While extradition is not a criminal proceeding, it entails deprivation of
liberty and the means employed to attain the purpose of extradition is
also “the machinery of criminal law”.
Obviously, an extradition proceeding, while ostensibly administrative,
bears all earmarks of a criminal process.
A potential extraditee may be subjected to arrest, to a prolonged restraint
of liberty, and forced to transfer to the demanding state following the
proceedings. "Temporary detention" may be a necessary step in the
Records show that Munoz was arrested and remained incarcerated for 2 over 2
years without having been convicted of any crime. Such an extended period
of detention is a serious deprivation of his fundamental right to liberty.
The time-honored principle of pacta sunt servanda demands that the Philippines
honor its obligations under the Extradition Treaty it entered into with the
Hong Kong Special Administrative Region. However, it does not necessarily
mean that in keeping with its treaty obligations, the Philippines should
diminish a potential extraditee's rights to life, liberty, and due process.
The standard of due process in the granting of bail for extradition cases, will
not be the same as that of criminal proceedings. The quantum of
evidence needed would be “clear and convincing evidence.”
In the present case, there is no showing that Munoz is a flight risk. The case
is therefore remanded to the trial court to determine whether Munoz may
be granted bail on the basis of “clear and convincing evidence.”
ENRILE v. SANDIGANBAYAN (HENRY) Although he was charged with plunder, penalty as to him would
August 18, 2015 | Bersamin, J. | Right to bail only be reclusion temporal not perpetua

PETITIONER: Juan Ponce Enrile


RESPONDENTS: Sandiganbayan, and People of the Philippines

SUMMARY: This case basically revolves on the charge on Enrile insofar


as the PDAF misappropriations are concerned. In this case, he
repeatedly forwarded his motions for bail but was perpetually denied of
by the Sandiganbayan and likewise the CA. Among others, he has
indicated that the evidence was not strong with regard his guilt, and that
he is not a flight risk due to his medical condition and his age.

Upon this petition for certiorari in the SC, it was held that he must be granted
the right to bail because of his health condition. It was mentioned by the SC
that the Sandiganbayan, in its denial of the motion to bail, ignored the core
purpose of bail, which is to secure the appearance of the accused. By
denying bail, Sandiganbayan ignores the health issues faced by Enrile which
could be the purpose of a worse condition that might even prevent him to be
present in the trial.

DOCTRINE: The purpose of bail is to guarantee the appearance of the


accused at the trial, or whenever so required by the trial court. The
amount of bail should be high enough to assure the presence of the
accused when so required, but it should be no higher than is reasonably
calculated to fulfill this purpose. Bail acts as a reconciling mechanism to
accommodate both the accused’s interest in his provisional liberty before
or during the trial, and the society’s interest in assuring the accused’s
presence at trial

FACTS:
Ombudsman charged Enrile and several others with plunder in the
Sandiganbayan on the basis of their involvement in the misuse of
appropriations under PDAF.
Enrile filed his Omnibus Motion and Supplemental Opposition, praying
that he be allowed to post bail should probable cause be found
against him. The motions were heard by the Sandiganbayan after
the Prosecution filed its Consolidated Position.
Sandiganbayan denied Enrile’s motion on bail as it is premature
considering that Enrile had not yet voluntarily surrendered or been
placed under custody of the law. Accordingly, Sandiganbayan
ordered the arrest of Enrile.
Upon issuance of his warrant of arrest, Enrile voluntarily surrendered and
was later confined at the PNP following his medical examination.
Enrile further then argued that he must be allowed to post bail because:
Prosecution had not yet established that the evidence of his guilt
was strong
He was not a flight risk and his age and physical condition must
further be considered
Sandiganbayan again denied the motion of Enrile as there has been no
finding that the evidence of guilt is not strong and hence the Court is
notyet duty-bound to fix the amount of his bail.
MR was filed and then denied, and hence this petition for certiorari.

ISSUE/s:
52. WoN Enrile shall be granted bail –

RULING: SC granted the petition wtf.

RATIO:
Enrile claims that:
before judgment of conviction, an accused is entitled to bail as
matter of right
that it is the duty and burden of the Prosecution to show clearly
and conclusively that Enrile comes under the exception and
cannot be excluded from enjoying the right to bail
that the prosecutioin failed to establish that Enrile’s conviction of
plunder is punishable by reclusion perpetua considering 2
mitigating circumstances (age and voluntary surrender)
prosecution has not come forward with proof showing that his
guilt for the crime of plunder is strong
he should not be considered a flight risk because he’s 90 and in
consideration of his medical condition and social standing
The Ombudsman then contends that Enrile’s right to bail is discretionary
as he is charged with a capital offense. That to be granted bail, it is
mandatory that a bail hearing be conducted to determine whether
there is strong evidence of guilt or lack of it.
SC mentioned that bail is not granted to prevent the accused from
committing additional crimes, rather, to guarantee the appearance of the
accused at the trial, or whenever so required bythe trial court. The
amount of bail should be high enough to assure presence of the
accused when so required, but should be no higher than is reasonably
calculated to fulfill this purpose.
Right to bail is expressly afforded in Sec 13 Art 3 of the Constitution, and
repeated in Sec 7 Rule 114 of the Rules of Court, which reads, “No
person charged with a capital offense, or an offense punishable by
reclusion perpetua or life imprisonment, shall be admitted to bil when
evidence of guilt is strong, regardless of the stage of the criminal
prosecution.
A capital offense in the context of the rule refers to an offense that, under
the law existing at the time of its commission and application for
admission to bail, may be punished with death.
The general rule is, hence, any person, before being convicted of any
criminal offense, shall be bailable, unless charged with a capital
offense one punishable by reclusion perpetua or life imprisonment,
and the evidence of
his guilt is strong. Court that, unless allowance of bail is forbidden by law in the
From the moment an accused is placed under arrest, or is detained or particular case, illness of the prisoner, independently of themerits of
restrained by officers of the law, he can claim guarantee of his the case, is a circumstance and the humanity of the law makes it a
provisional liberty under the Bill of Rights consideration which should, regardless of the charge and the stage
On the other hand, granting of bail is discretionary: of the proceeding, influence the court to exercise its discretion to
Upon conviction by RTC of an offense not punishable by death, admit the prisoner to bail.
reclusion perpetua, or life imprisonment; or Granting provisional liberty to Enrile will enable him to have his medical
If RTC imposed a penalty of imprisonment exceeding 6 years, condition properly addressed and better attended to by competent
among others physicians in the hospitals of his choice. This will not only aid in his
For purposes of admission to bail, determination of WoN evidence of guilt adequate preparation of his defense, more importantly, will guarantee
is strong in criminal cases involving capital offenses, or offenses his appearance in court for the trial.
punishable by reclusion perpetua or life imprisonment lies with the SC concluded by saying that Sandiganbayan ignored the objective of bail
discretion of the trial court. to ensure the appearance of the accused during trial and
Concerned Citizens v. Elma was a case where the SC ruled that, such unwarrantedly disregarded the clear showing of the fragile health and
discretion may be exercised only after the hearing called to ascertain advanced age of Enrile.
the degree of guilt of the accused for the purpose of whether or not Leonen, dissenting: This petition should not be granted. The action of
he should be granted provisional liberty. the Sandiganbayan in the Motion to Fix Bail was proper. Bail isnot a
Bail canot be allowed when its grant is a matter of discretion ont eh part matter of right in cases where the crime charged is plunder and the
of the trial court UNLESS there has been a hearing with notice to the imposable penalty is reclusion perpetua. Neither was there grave
prosecution. This hearing will determine whether or not the evidence abuse of discretion by the Saniganbayan when it failed to release
of guilt against the accused is strong. accused on bail for medical or humanitarian reasons. His release for
In resolving bail applications of an accused charged with a capital medical and humanitarian reasons was not the basis for his prayer in
offense, or offense punishable by reclusion perpetua or life his Motion to Fix Bail filed before the Sandiganbayan. Neither did he
imprisonment, trial judge is expected to comply with guidelines base his prayer for the grant of bail in his petition on his medical
outlined in Cortes v. Catral: condition. The majority’s opinion—other than the invocation of a
In all cases whether bail is a matter of right of discretion, notify general human rights principle—does not provide clear legal basis for
prosecutor of hearing of application for bail to require him to the grant of bail on humanitarian grounds.
submit his recommendation Sandiganbayan did not commit grave abuse of discretion when it failed to
Where bail is a matter of discretion, conduct a hearing fo the release petitioner on bail for medical or humanitarian reasons.
application for bail regardless of whether or not the Petitioner did not ask that bail be granted because of his medical
prosecution refuses to present evidence to show that the condition or for humanitarian reasons. Neither petitioner nor
guilt of the accused is storng for the purpose of enabling prosecution developed arguments on this point at the
court to exercise sound jurisdiction Sandiganbayan or this court to establish legal and factual basis for
Decide whether guilt of accused is strong based on summary of this special kind of bail case. Yet, this becomes the very basis of the
evidence grant of bail.
If guilt not strong, discharge accused upon approval of bailbond.
Otherwise, deny
SC reversed the CA ruling and granted bail on the basis of UDHR’s
provision that stated that the accused has the right to bail.
Also, because of Enrile’s medical condition, as stated by Dr.
Gonzales (from PGH). That, there is no question at all that Enrile’s
advanced age and ill health required special medical agttention.
Confinement at PNP was not even recommended by the internist
doctor because of limitations of medical support at that hospital.
The SC then said that, denying him bail despite imperiling his health and life
would not serve the true objective of preventive incarceration during trial.
Granting his bail is unprecedented. SC held in Dela Rama v. The People’s
PEOPLE v. JUDGE DONATO AND SALAS (DANNAH) and identities are still unknown, risen publicly and taken arms
June 5, 1991 | Davide, Jr., J. | Right to Bail: Waiver throughout the country against the Government of

PETITIONER: People of the Philippines


RESPONDENTS: Hon. Donato, Rodolfo Salas alias Commander Bilog

SUMMARY: Salas was charged with the crime of Rebellion. A day after the
filing of the Information, he and his co-accused initiated a petition for habeas
corpus. The agreement was that “Salas will remain in legal custody and will
face trial before the court having custody over his person” and the warrants
for the arrest of his co-accused are deemed recalled and they shall be
immediately released but shall submit themselves to the court having
jurisdiction over their person.

Judge Donato granted bail to Salas. The People opposed this, alleging
that although bail is a matter of right (at that time the penalty was prision
mayor), Salas poses a threat to security and circumstances show that he
might abuse ther right and might not be around during trial (see Fact
#16). The Supreme Court ruled that although at the time of the trial (at the
SC) level the punishment for Rebellion was reclusion perpetua, it cannot
apply to Salas as this is not in his favor. However, the Court agreed with
The People that Salas, in the petition for habeas corpus, (as can be
gleaned from the language used in the agreement) waived his right to
bail. And bail, although a Constitutional right, may be waived (the Court
listed others such as right to searches and seizures and Section 12
rights). In agreeing to remain in legal custody even during the pendency
of the trial of his criminal case, has expressly waived his right to bail

DOCTRINE: The Court hereby rules that the right to bail is another of the
constitutional rights which can be waived. It is a right which is personal to
the accused and whose waiver would not be contrary to law, public order,
public policy, morals, or good customs, or prejudicial to a third person
with a right recognized by law.

FACTS:
Private respondent Rodolfo Salas “Commander Bilog” and his co-
accused were charged for the crime of Rebellion under Art. 134 of
the RPC:
That in or about 1968 and for some time before said year and continuously
hereafter until the present time, in the City of Manila and elsewhere in
the Philippines, the Communist Party of the Philippines, its military arm,
the New People’s Army, its mass infiltration network, the National
Democratic Front with its other subordinate organizations and fronts,
have, under the direction and control of said organizations’ leaders,
among whom are the aforenamed accused, and with the aid,
participation or support of members and followers whose whereabouts
the Republic of the Philippines for the purpose of overthrowing the present
Government, the seat of which is in the City of Manila, or of removing from
the allegiance to that government and its laws, the country’s territory or part
of it
That from 1970 to present, the above-named accused in their capacities as
leaders of the aforenamed organizations, in conspiracy with, and in support
of the cause of, the organizations aforementioned, engaged themselves in
war against the forces of the government, destroying property or committing
serious violence, and other acts in the pursuit of their unlawful purpose, such
as x x x.”
At the time the Information was filed, Salas and his co-accused were in military
custody following their arrest at Philippine General Hospital in Taft Avenue,
Manila; he had earlier escaped from military detention and a cash reward of
P250k was offered for his capture
A day after the filing of the original information, a petition for habeas corpus for
Salas and his co-accused was filed with the Court (important; will be
discussed in ratio) on the basis of the agreement of the parties under which
Salas “will remain in legal custody and will face trial before the court having
custody over his person” and the warrants for the arrest of his co-accused
are deemed recalled and they shall be immediately released but shall submit
themselves to the court having jurisdiction over their person.
Salas filed a Motion to Quash the Information. The People (of the Philippines)
filed an Opposition citing, among other grounds, the fact that in the Joint
Manifestation and Motion (in GR 76009), Salas categorically conceded that
he will remain in legal custody and face trial before the court having
custody of his person.
Donato denied the motion to quash.
Instead of asking for a reconsideration of the Order, Salas filed a petition for bail,
in which The People opposed on the ground that since rebellion became a
capital offense under certain PDs which amended Art. 135 (imposing the
penalty of reclusion perpetua to death), Salas is no longer entitled to bail as
evidence of his guilt is strong.
The President issued EO 187 repealing said PDs and restoring the original
penalty for rebellion, which was prision mayor.
In his Order, Judge Donato, taking into consideration EO 187, granted Salas bail
fixed at P30k and imposed the additional condition that Salas should report
to the court once every 2 months within the first 10 days.
The contention of The People was that it would be dangerous to grant bail to
Salas considering his stature in the CPP-NPA hierarchy, and that his release
would allow his return to said organization.
To this Judge Donato replied that although there’s a clash between Salas’
constitutional right to bail in a non-capital offense and the existence An arrestee may be incarcerated until trial as he presents a risk of flight
of the government that bestows the right, the paramount interest of
the state, emphasis is placed on the dignity of man and the worth of
individual.
The conflict should be resolved in favor of the individual who, in the eyes
of the law, is alone in the assertion of his rights under the Bill of
Rights as against the State.
Anyway, the government is that powerful and strong, having the
resources, manpower and the wherewithals to fight those ‘who
oppose, threathen (sic) and destroy a just and orderly society and its
existing civil and political institutions.’
The fear of Salas to sow further disorders and anarchy cannot be reason
to deny him bail; for the law is very explicit that when it comes to
bailable offenses an accused is entitled as a matter of right to bail.
Dura est lex sed lex.
In an MR for the above order, The People asked the court to increase the
bail to P100k alleging that per DOJ Circular No. 10, the bail for the
provisional release of an accused should be in an amount computed at
P10,000.00 per year of imprisonment based on the medium penalty
imposable for the offense
And explaining that it is recommending 100k because Salas had
escaped in the past, and the offense he is charged with is rebellion
(as compared to murder, homicide) the rebellious acts are not
consummated until the well-organized plan to overthrow the
government through armed struggle and replace it with an alien
system based on a foreign ideology is attained.
The People filed a supplemental MR indirectly asking the court to deny
bail and allow it to present evidence in support thereof:
Salas evaded the authorities for 13 years and was an escapee from
detention when arrested
He was not arrested at his residence as he had no known address
He used the false name “Manuel Castro” at the time of his arrest;
presented a fake driver’s license
The address he gave in Cavite also turned out to be a false address
He and his companions were on board a private vehicle with a declared
owner whose identity and address were also found to be false
A reward for P250k was offered and paid for his arrest
According to The People these clearly indicate that Salas does not
entertain the slightest intention to appear in court for trial, if released.
Also that Salas as Chairman of the Communist Party and the NPA are
engaged in an open warfare with the country
The People further invokes precedents in the USA holding that:
There is no absolute constitutional barrier to detention of potentially
dangerous resident aliens pending deportation proceedings
Sustaining a detention prior to trial of arrestee charged with serious Therefore, before conviction bail is either a matter of right or of discretion.
felonies who are found after an adversary hearing to pose threat to It is a matter of right when the offense charged is punishable by any
the safety of individuals and to the community which no condition
penalty
of release can dispel
Judge Donato handed down the Order which did not deny Salas’
right to bail but instead increased the amount from P30k to P50k,
still with the imposed condition
The People argues that Salas is estopped from invoking his right to
bail, having expressly waived it in G.R. No. 76009 when he
agreed to “remain in legal custody and face trial before the court
having custody of his person”
The People also avers that the right to bail, even in non-capital
offenses, is not absolute when there is prima facie evidence that
the accused is a serious threat to the very existence of the State,
in which case the prosecution must be allowed to present
evidence for the denial of bail.

ISSUE/s:
WoN the right to bail may, under circumstances, be denied to a
person who is charged with an otherwise bailable offense – NO
WoN such right may be waived – YES!

RULING: Orders of Judge Salas nullified and set aside

RATIO:
The restored law was the governing law at the time the respondent
court resolved the petition for bail.
The Court agrees with Judge Donato that bail cannot be denied to
Salas for he is charged with Rebellion to which is attached the
penalty of prision mayor. It is therefore, a bailable offense
Section 13, Article III:
All persons, except those charged with offenses punishable by
reclusion perpetua when evidence of guilt is strong, shall,
before conviction, be bailable by sufficient sureties, or be
released on recognizance as may be prescribed by law. The
right to bail shall not be impaired even when the privilege of the
writ of habeas corpus is suspended. Excessive bail shall not be
required
Section 3, Rule 114 of the Rules of Court:
All persons in custody shall, before final conviction, be entitled to
bail as a matter of right, except those charged with a capital
offense or an offense which, under the law at the time of its
commission and at the time of the application for bail, is
punishable by reclusion perpetua, when evidence of guilt is
strong.
lower than reclusion perpetua. To that extent the right is absolute. no court or judge can properly refuse, in all cases not embraced in
People v. Hernandez (similar case for rebellion): Despite the fact that the exceptions.
the accused was already convicted, although erroneously, by the trial Under such provisions bail is a matter of right even in cases of
court for the complex crime of rebellion with multiple murders, arsons capital offenses, unless the proof of guilt is evident or the
presumption thereof is great!”
and robberies, and sentenced to life imprisonment, the Court granted
bail in the amount of P30,000.00 during the pendency of his appeal
from such conviction.
The stand of the People then was to deny bail because the security of
the State so requires, to which the Court responded:
Individual freedom is too basic, too transcendental and vital in a republican
state, like ours, to be derived upon mere general principles and abstract
consideration of public safety. Indeed, the preservation of liberty is such
a major preoccupation of our political system that, not satisfied with
guaranteeing its enjoyment in the very first paragraph of section (1) of
the Bill of Rights, the framers of our Constitution devoted paragraphs
(3), (4),
(5), (6), (7), (8), (11), (12), (13), (14), (15), (16), (17), (18), and (21)
of said section (1) to the protection of several aspects of freedom”
The 1987 Constitution strengthens further the right to bail by explicitly
providing that it shall not be impaired even when the privilege of the
writ of habeas corpus is suspended.
This overturns the Court’s ruling in Garcia-Padilla v. Enrile:
The suspension of the privilege of the writ of habeas corpus must, indeed,
carry with it the suspension of the right to bail, if the government’s
campaign to suppress the rebellion is to be enhanced and rendered
effective.
If the right to bail may be demanded during the continuance of the
rebellion, and those arrested, captured and detained in the course
thereof will be released, they would, without the least doubt, rejoin
their comrades in the field thereby jeopardizing the success of
government efforts to bring to an end the invasion, rebellion or
insurrection
Upon the other hand, if the offense charged is punishable by reclusion
perpetua bail becomes a matter of discretion. It shall be denied if the
evidence of guilt is strong. The court’s discretion is limited to
determining whether or not evidence of guilt is strong. But once it is
determined that the evidence of guilt is not strong, bail also becomes
a matter of right.
Teehankee v. Director of Prisons: The provision on bail in our Constitution
is patterned after similar provisions in the Constitution of the US
The Constitution of the United States and the constitution of the many states
provide that all persons shall be bailable by sufficient sureties, except
for capital offenses, where the proof is evident or the presumption of
guilt is great, and, under such provisions, bail is a matter of right which
Accordingly, the prosecution does not have the right to present evidence for
the denial of bail in the instances where bail is a matter of right.
However, in the cases where the grant of bail is discretionary, due process
requires that the prosecution must be given an opportunity to present,
within a reasonable time, all the evidence that it may desire to introduce
before the court should resolve the motion for bail
The Court however agrees that it was error for Judge Donato to fix the bond
at P30k then later at P50k without hearing the prosecution.
People v. Dacudao: Certain guidelines in the fixing of a bailbond call for the
presentation of evidence and reasonable opportunity for the prosecution
to refute it. Among them are the nature and circumstances of the crime,
character and reputation of the accused, the weight of the evidence
against him, the probability of the accused appearing at the trial, whether
or not the accused is a fugitive from justice, and whether or not the
accused is under bond in other case
RA 6968 amended Article 135 by increasing the penalty for rebellion to
reclusion perpetua. However, it cannot apply to Salas as it is not
favorable to him (so bawal retroactivity)
The Court agrees with The People that Salas has, however, waived his right
to bail in GR 76009.
It is the stand of The People that Salas, “in agreeing to remain in legal
custody even during the pendency of the trial of his criminal case, has
expressly waived his right to bail”
Upon the other hand, Salas asserts that this claim is totally devoid of factual
and legal basis, for in their petition for habeas corpus they precisely
questioned the legality of his and his co-accused’s arrest and continued
detention, which was not resolved by the Court.
Moreover, the matter of the right to bail was neither raised by either party nor
resolved by this Court, and the legal steps promptly taken by Salas after
the agreement was reached (like filing motion to quash and petition for
bail) were clear and positive assertions of his statutory and constitutional
rights to be granted not only provisional but final and permanent liberty.
Finally, Salas maintains that the term “legal custody” as used in the Joint
Manifestation and Motion simply means that he agreed to continue to be
in the custody of the law or in custodia legis and nothing else; it is not to
be interpreted as waiver.
Interestingly, he admits that: “Custody has been held to mean nothing less than
actual imprisonment. It is also defined as the detainer of a person by virtue of
a lawful authority, or the ‘care and possession of a thing or person.”
He further admits that, in the light of Section 1 of Rule 114 of the Rules of
Court and settled jurisprudence, the “constitutional right to bail is The principle is recognized that everyone has a right to waive, and
subject to the limitation that the person applying for admission to bail agree to waive, the advantage of a law or rule made solely for
should be in the custody of the law or otherwise deprived of his the benefit and protection of the individual in his private
liberty capacity, if it can be dispensed with and relinquished without
infringing on any public right, and without
Note should be made of the deliberate care of the parties in making a
fine distinction between legal custody and court having custody over
the person in respect to Rodolfo Salas and court having jurisdiction
over the persons of his co- accused.
Such a fine distinction was precisely intended to emphasize the agreement
that Rodolfo Salas will not be released, but should remain in custody.
Had the parties intended otherwise, or had this been unclear to private
respondent and his counsel, they should have insisted on the use of a
clearer language.
The purpose of bail is to relieve an accused from imprisonment until his
conviction and yet secure his appearance at trial.
It presupposes that the person applying for it should be in the custody of
the law or otherwise deprived of liberty.
Consequently, having agreed in G.R. No. 76009 to remain in legal
custody, private respondent had unequivocably waived his right to
bail.
Article 6 of the NCC provides that “Rights may be waived, unless the
waiver is contrary to law, public order, public policy, morals, or good
customs, or prejudicial to a third person with a right recognized by
law”
Waiver is defined as “a voluntary and intentional relinquishment or
abandonment of a known existing legal right, advantage, benefit,
claim or privilege, which except for such waiver the party would have
enjoyed; the voluntary abandonment or surrender, by a capable
person, of a right known by him to exist, with the intent that such
right shall be surrendered and such person forever deprived of its
benefit; or such conduct as warrants an inference of the
relinquishment of such right; or the intentional doing of an act
inconsistent with claiming it.
As to what rights and privileges may be waived:
The doctrine of waiver extends to rights and privileges of any character
Since the word “waiver” covers every conceivable right, it is the general
rule that a person may waive any matter which affects his property,
and any alienable right or privilege of which he is the owner or
which belongs to him or to which he is legally entitled, whether
secured by contract, conferred with statute, or guaranteed by
constitution, provided such rights and privileges rest in the
individual, are intended for his sole benefit, do not infringe on the
rights of others, and further provided the waiver of the right or
privilege is not forbidden by law
detriment to the community at large
Commonwealth v. Petrillo: Rights guaranteed to one accused of a crime
fall naturally into two classes:
Those in which the state, as well as the accused, is interested;
Those which are personal to the accused, which are in the nature of
personal privileges.
Those of the first class cannot be waived; those of the second may be
It is “competent for a person to waive a right guaranteed by the Constitution, and
to consent to action which would be invalid if taken against his will”
The Court has recognized waivers of constitutional rights such as, for
example, the right against unreasonable searches and seizures; the right
to counsel and to remain silent; and the right to be heard
Even the Constitution expressly recognizes waivers of Section 12 rights:
“These rights cannot be waived except in writing and in the presence of
counsel”
This provision merely particularizes the form and manner of the waiver; it,
nevertheless, clearly suggests that the other rights may be waived in
some other form or manner provided such waiver will not offend Article 6
of the Civil Code.
The Court hereby rules that the right to bail is another of the constitutional
rights which can be waived. It is a right which is personal to the accused
and whose waiver would not be contrary to law, public order, public
policy, morals, or good customs, or prejudicial to a third person with a
right recognized by law.
On Judge Donato: He clearly acted with grave abuse of discretion in granting
bail to Salas
DE LA CAMARA v. ENAGE (ARIELLE) multiple murder and P355,200 for the offense of multiple frustrated
September 17, 1971 | Fernando, J.| Excessive Bail murder.

PETITIONER: Ricardo De La Camara


RESPONDENTS: Hon. Manuel Lopez Enage

SUMMARY: Mayor De La Camara was arrested for his alleged participation


in the killing of 14 and the wounding of 12 other laborers of the Tirador
Logging Co. De La Camara filed a petition for bail. Respondent Judge
granted this petition, and fixed the amount of bail at the excessive amount of
P1,195,200, the sum of P840,000 for the information charging multiple
murder and P355,200 for the offense of multiple frustrated murder. A motion
for reconsideration to reduce the amount was filed. But respondent Judge
remained adamant, hence the petition.

The SC held that Before conviction, every person is bailable except if


charged with capital offenses when the evidence of guilt is strong.
Such right flows from the presumption of innocence in favor of every
accused who should not be subjected to the loss of freedom as he would
be entitled to acquittal, unless his guilt be proved beyond reasonable
doubt. Where, however, the right to bail exists, it should not be rendered
nugatory by requiring a sum that is excessive. If there were no
prohibition against excessive bail, the right to bail becomes
meaningless.

DOCTRINE: If the right to bail exists, it should not be rendered


meaningless by requiring such an excessive sum.

FACTS:
Petitioner Ricardo de la Camara, Municipal Mayor of Magsaysay,
Misamis Oriental, was arrested and detained for his alleged
participation in the killing of fourteen and the wounding of twelve
other laborers of the Tirador Logging Co.
The Provincial Fiscal filed with the CFI a case for multiple frustrated
murder and another for multiple murder against De La Camara and
his other co-accused.
De La Camara filed a petition for bail, premised on the assertion that
there was no evidence to link him with such fatal incident. He
likewise maintained his innocence.
The trial judge started the trial. As of the time of the filing of the petition,
the defense had not presented its evidence. Respondent Judge
issued an order granting De La Camara’s application for bail,
admitting that there was a failure on the part of the prosecution to
prove that De La Camara would flee even if he had the opportunity.
Respondent Jude fixed the amount of bail at the excessive amount of
P1,195,200, the sum of P840,000 for the information charging
Subsequently, an allegation that the Sec. of Justice, Vicente Abad him with a death sentence, an ever-present threat, temptation to flee
Santos, upon being informed of such order, sent a telegram to the jurisdiction would be too great to be resisted.
respondent Judge stating that the bond is “excessive” and
suggesting a P40,000 bond instead would be reasonable.
A motion for reconsideration to reduce the amount was filed. But
respondent Judge remained adamant, hence the petition.
Respondent Judge Enage filed an answer, which, to his mind, would
disprove any charge that he was guilty of grave abuse of discretion.
It stressed, moreover, that the challenged order would find support in
circulars of the DOJ.
In the hearing, there was no appearance for both petitioner and
respondents. Instead of a reply, Judge Enage submitted a
supplemental answer where he alleged that De La Camara escaped
from the provincial jail and had since then remained at large.

ISSUE/s:
WoN the bail is excessive – YES

RULING: Case is dismissed for being moot and academic.

RATIO:
The fact that the case is moot and academic should not preclude the
tribunal from setting forth in language clear and unmistakable, the
obligation of fidelity on the part of the lower court judges to the
unequivocal command of the Constitution that excessive bail shall
not be required.
Before conviction, every person is bailable except if charged with
capital offenses when the evidence of guilt is strong.
Such right flows from the presumption of innocence in favor of every
accused who should not be subjected to the loss of freedom as he
would be entitled to acquittal, unless his guilt be proved beyond
reasonable doubt.
It is not beyond the realm of probability, however, that a person charged
with a crime, especially so where his defense is weak, would just
simply make himself scarce and thus frustrate the hearing of his
case.
A bail is intended as a guarantee that such an intent would be thwarted.
It is a “mode short of confinement which would, with reasonable
certainty, insure the attendance of the accused” for the subsequent
trial.
Nor is there anything unreasonable in denying this right to one charged
with a capital offense when evidence of guilt is strong, as the
likelihood is, rather than await the outcome of the proceeding against
Where, however, the right to bail exists, it should not be rendered
nugatory by requiring a sum that is excessive. If there were no
prohibition against excessive bail, the right to bail becomes
meaningless.
According to American jurisprudence, “the sole permissible function of
money bail is to assure the accused’s presence at trial.”
Nothing can be clearer than that the challenged order of Judge Enage
fixing the amount of P1,195,200 as the bail that should be posted by
De La Camara, the sum of P840,000 for the information for muder,
and P355,200 for frustrated murder, is clearly violative of the
constitutional provision.
There is an attempt on the part of Judge Enage to justify his decision
baswd on his reliance on Villaseñor v. Albano. According to that
case, the guidelines in the fixing of bail are as follows:
The ability of the accused to give bail;
Nature of the offense;
Penalty for the offense charged;
Character and reputation of the accused;
Health of the accused;
Character and strength of evidence;
Probability of the accused appearing in trial;
Forfeiture of other bonds;
Whether the accused was a fugitive from justice when arrested;
If the accused is under bond for appearance at trial in other cases
However, Judge Enage ignored the decisive consideration appearing at the
end of the above opinion: “Discretion, indeed, is with the court called
upon to rule on the question of bail. However, where conditions
imposed upon
defendant seeking bail would amount to a refusal thereof and
render nugatory the constitutional right to bail, we will not
hesitate to exercise our supervisory powers to provide the
remedy required.”
No attempt at rationalization can therefore give a color of validity to the
challenged order. There is grim irony in an accused being told that he
has a right to bail but at the same time being required to post such
an exorbitant sum.
Nor is there any justification for imputing respondent Judge’s inability to fix
lesser amount by virtue of an alleged reliance on a decision of the
Court. There was a clear misreading of the Albano opinion when
such a meaning was ascribed to it.
With petitioner, however, having escaped from the provincial jail, no
ruling can be had on his plea to nullify the above order.
PEOPLE v. NITCHA (IYA) Not long thereafter and while the Sibayans were still on their way, appellant
Florestan Nitcha, brother of Doro, arrived at the sarisari store
January 19, 1995 | Melo, J. | Right to Bail
brandishing a gun and shouting in Tagalog, "Walanghiya kayo, putangina
ninyo, papatayin
PETITIONER: People of the Philippines
RESPONDENTS: Pat. Florestian Nitcha (APPELLANT)

SUMMARY: A fight ensued between the brother of Florentian and a


certain Jojo Belmonte. Several persons (Sibayan’s) tried to pacify the
fight. The brawlers were already separated when Florestian arrived and
fired his gun at the direction of the Sibayans. May Sibayan was hit at the
back of her head. He then aimed at Joselito Sibayan but missed. May
died from he incident.

The RTC found Florestian Nitcha guilty of homicide. On appeal, a


defense of Florestian was that his posting of bail bond is an obstacle to
his immediate incarceration.

The SC ruled that when the crime is punished by reclusion perpetua , no bail
may be issued. The SC, however ruled that the crime is only homicide
(punished by reclusion temporal) because of lack of any qualifying
circumstance.

DOCTRINE: An accused who is charged with a capital offense or an


offense punishable by reclusion perpetua, shall no longer be entitled to
bail as a matter of right even if he appeals the case to this Court since
his conviction clearly imports that the evidence of his guilt of the offense
charged is strong.

FACTS:
On October 1990, at around 7 o'clock in the evening, Jojo Belmonte went
out of his house located at Purok IV, Barangay Alac, San Quintin,
Pangasinan to buy some cigarettes in a nearby store.
Before Jojo Belmonte could buy the cigarettes, Doro Nitcha arrived,
uttered the words "You are one of them" ("Maysa ca met") in the
vernacular, thenstarted mauling him.
Unable to endure the pain, Jojo fought back. A few minutes had gone into
the fight when May Villarica (a.k.a. Lydia) Joselito, Agustin and
Marcelina (Nenet), all surnamed Sibayan, arrived.
May and Joselito tried to pacify the two protagonists, however, their
efforts proved futile as Doro Nitcha refused to be pacified.
The fighting stopped upon the arrival of Doro's sister Victoria Corpuz
(Baby) who, upon seeing the commotion, dragged Doro away from
the fight and brought him home.
Likewise, Marcelina (Nenet), Agustin, May and Joselito proceeded
towards their house located in front of the store where the incident
occurred.
ko kayong lahat!”
After uttering those words, appellant fired his gun in the direction of the
Sibayans, the bullet hitting May at the back of her head and existing
through the middle of her forehead.
Appellant then aimed his gun at Joselito but missed.
May was brought to the Eastern Pangasinan District Hospital in Tayug,
Pangasinan where she was given first aid treatment.
Upon the advice of a doctor, the victim was brought to a hospital in
Dagupan City.
May, however, expired on the way thereto. Shortly after the shooting
incident, appellant went back to his mother's house before
proceeding to the police station of San Quintin, Pangasinan where
he surrendered himself together with his service firearm.
Premised on the foregoing backdrop, an indictment for murder was filed
against herein accused-appellant who, after trial, was found guilty
beyond reasonable doubt on account of the positive identification
made by the People's witnesses, the defense of accused-appellant
anchored on denial being found unavailing.
Accused-appellant argues that procedural infirmities attended the trial
below, that there was no preliminary investigation conducted and that
his posting of bail is an obstacle to his incarceration

ISSUE/s:
WON accused-appellant is guilty of murder - NO
WON the posting of bail is an obstacle to the incarceration of accused -
NO

RULING: Imelda Rodriguez is to be placed back in bail

RATIO:
Even on the assumption that accused-appellant’s arrest was illegal for
want of preliminary investigation, such a hypothesis was nonetheless
negated by accused-appellant’s act of posting a bail bond thereafter,
apart from the fact that he entered a plea of not guilty which is
tantamount to foregoing the right to question the assumed
irregularity.
The subsistence of a bail bond is also no legal obstacle to accused-
appellant’s immediate incarceration after promulgation of a decision
involving a felony punishable by reclusion perpetua following the
principle enunciated in People vs Fortes:
. . . before conviction bail is either a matter of right or of
discretion. It is a matter of right when the offense charged is
punishable by any penalty lower than reclusion perpetua. To
that extent the right is absolute.
Upon the other hand, if the offense charged is punishable by
reclusion perpetua bail becomes a matter of discretion. It
shall be denied if the evidence of guilt is strong.
The court's discretion is limited to determining whether or not
evidence of guilt is strong.
But once it is determined that the evidence of guilt is not strong,
bail also becomes a matter of right. . . .
The clear implication therefore, is that if an accused who is
charged with a crime punishable by reclusion perpetua is
convicted by the trial court and sentenced to suffer such a
penalty, bail is neither a matter of right on the part of the
accused nor of discretion on the part of the court.
In such a situation, the court would not have only determined that
the evidence of guilt is strong — which would have been
sufficient to deny bail even before conviction — it would have
likewise ruled that the accused's guilt has been proven
beyond reasonable doubt.
Bail must not then be granted to the accused during the
pendency of his appeal from the judgment of conviction.
In People v Ricardo Cortez: An accused who is charged with a capital
offense or an offense punishable by reclusion perpetua, shall no
longer be entitled to bail as a matter of right even if he appeals the
case to this Court since his conviction clearly imports that the
evidence of his guilt of the offense charged is strong.
The Court found that there is no qualifying circumstance attendant in the
killing. The crime committed by accused- appellant is, therefore,
homicide and not murder.
The penalty for homicide, under Article 249 of the Revised Penal Code,
is reclusion temporal.

You might also like